Download as pdf or txt
Download as pdf or txt
You are on page 1of 1161

About Pearson

Pearson is the world’s learning company, with presence across 70 countries


worldwide. Our unique insights and world-class expertise comes from a long
history of working closely with renowned teachers, authors and thought leaders,
as a result of which, we have emerged as the preferred choice for millions of
teachers and learners across the world.
We believe learning opens up opportunities, creates fulfilling careers and hence
better lives. We hence collaborate with the best of minds to deliver you class-
leading products, spread across the Higher Education and K12 spectrum.
Superior learning experience and improved outcomes are at the heart of
everything we do. This product is the result of one such effort.
Your feedback plays a critical role in the evolution of our products and you can
contact us – reachus@pearson.com. We look forward to it.

A01_THERMAL ENGINEERING_SE_XXXX_A01.indd 1 6/20/2018 9:29:10 AM


Thispageisintentionallyleftblank
thermal
Engineering
Sadhu Singh
Former Professor and Head
Mechanical Engineering Department
and
Dean, Faculty of Engineering and Technology
Govind Ballabh Pant University of Agriculture and Technology, Uttarakhand
and
Former Director (Colleges), Punjab Technical University, Jalandhar, Punjab

Sukumar Pati
Assistant Professor
Department of Mechanical Engineering
National Institute of Technology Silchar, Assam

A01_THERMAL ENGINEERING_SE_XXXX_A01.indd 3 6/20/2018 9:29:14 AM


Copyright © 2018 Pearson India Education Services Pvt. Ltd

Published by Pearson India Education Services Pvt. Ltd, CIN: U72200TN2005PTC057128.

No part of this eBook may be used or reproduced in any manner whatsoever without the
publisher’s prior written consent.

This eBook may or may not include all assets that were part of the print version.
The publisher reserves the right to remove any material in this eBook at any time.

ISBN 978-93-528-6668-7
eISBN 9789353063320

Head Office: 15th Floor, Tower-B, World Trade Tower, Plot No. 1, Block-C, Sector 16,
Noida 201 301, Uttar Pradesh, India.
Registered Office: 4th Floor, Software Block, Elnet Software City, TS 140, Block 2 & 9,
Rajiv Gandhi Salai, Taramani, Chennai - 600 113, Tamil Nadu, India.
Fax: 080-30461003, Phone: 080-30461060
Website: in.pearson.com, Email: companysecretary.india@pearson.com
Dedicated to my Parents
—Sadhu Singh

Dedicated to my beloved Parents


Late Sakti Pada Pati and Usha Rani Pati
—Sukumar Pati

A01_THERMAL ENGINEERING_SE_XXXX_A01.indd 5 6/20/2018 9:29:14 AM


Thispageisintentionallyleftblank
Brief Contents

Preface xxvii
About the Authors xxix

Chapter 1 Fuels and Combustion 1

Chapter 2 Properties of Steam 51

Chapter 3 Steam Generators 87

Chapter 4 Steam Power Cycles 161

Chapter 5 Steam Engines 214

Chapter 6 Flow Through Steam Nozzles 253

Chapter 7 Steam Turbines 316

Chapter 8 Steam Condensers 386

Chapter 9 Gas Power Cycles 423

Chapter 10 Internal Combustion Engine Systems 485

Chapter 11 Performance of Internal Combustion Engines 622

Chapter 12 Reciprocating Air Compressors 690

Chapter 13 Rotary Air Compressors 735

Chapter 14 Centrifugal Air Compressors 744

Chapter 15 Axial Flow Air Compressors 789

Chapter 16 Gas Turbines 840

A01_THERMAL ENGINEERING_SE_XXXX_A01.indd 7 6/20/2018 2:44:41 PM


viii Contents
Brief

Chapter 17  Jet Propulsion 922

Chapter 18  Introduction to Refrigeration961

Chapter 19  Vapour Compression and Vapour Absorption Systems 999

Chapter 20  Air-Conditioning and Psychrometrics 1034

Appendix A 1091
Index1121

A01_THERMAL ENGINEERING_SE_XXXX_A01.indd 8 6/20/2018 9:29:14 AM


Contents

Preface xxvii
About the Authors xxix

Chapter 1 Fuels and Combustion 1


1.1 Introduction 1
1.2 Classification of Fuels 1
1.3 Solid Fuels 2
1.3.1 Primary Fuels 2
1.3.2 Secondary Fuels 3
1.3.3 Desirable Properties of Coal 3
1.3.4 Ranking of Coal 4
1.3.5 Grading of Coal 4
1.4 Liquid Fuels 4
1.4.1 Advantages and Disadvantages of Liquid Fuels Over
Solid Fuels 5
1.4.2 Calorific Value of Liquid Fuels 5
1.4.3 Desirable Properties of Liquid Fuels 5
1.5 Gaseous Fuels 6
1.5.1 Calorific Value of Gaseous Fuels 6
1.5.2 Advantages and Disadvantages of Gaseous Fuels 7
1.5.3 Important Properties of Gaseous Fuels 7
1.6 Liquefied Gases 7
1.6.1 Liquefied Petroleum Gas 8
1.6.2 Liquefied or Compressed Natural Gas 8
1.7 Biofuels 8
1.8 Analysis of Fuels 9
1.8.1 Proximate Analysis 9
1.8.2 Ultimate Analysis 9
1.9 Calorific Value of Fuels 9

A01_THERMAL ENGINEERING_SE_XXXX_A01.indd 9 6/20/2018 2:44:42 PM


x Contents

1.10  Combustion of Fuels  10


1.11  Combustion of Hydrocarbon Fuel  11
1.12  Minimum Air Required for Complete Combustion
  of Solid/Liquid Fuels  11
1.13  Conversion of Volumetric Analysis to Mass (or Gravimetric)
  Analysis and Vice-Versa  12
1.14  Determination of Air Supplied  13
1.14.1  Percentage of Carbon by Mass in Fuel and Volumetric
  Analysis is Known  13
1.14.2  Excess Air Supplied  14
1.15  Determination of Percentage of Carbon in Fuel Burning
  to CO and CO2  14
1.16  Determination of Minimum Quantity of Air Required
  for Complete Combustion of Gaseous Fuel  15
1.17  Determination of Excess Air Supplied for Gaseous Fuel  15
1.18  Flue Gas Analysis  16
1.18.1  Orsat Apparatus Construction  16
1.19  Bomb Calorimeter  17
1.19.1  Construction 17
1.19.2  Working 17
1.19.3  Cooling Correction  19
1.20  Boys Gas Calorimeter  19
1.20.1  Construction 19
1.20.1  Working 20
Summary for Quick Revision ·  Multiple-choice Questions ·  Review
Questions ·  Exercises ·  Answers to Multiple-choice Questions

Chapter 2  Properties of Steam                 51


2.1  Pure Substance  51
2.2  Constant Pressure Formation of Steam  51
2.3  Properties of Steam  52
2.4  Steam Tables  55
2.5  Temperature−Entropy Diagram for Water and Steam  55
2.6  Enthalpy−Entropy or Mollier Diagram of Steam  56
2.7  Various Processes for Steam  57
2.7.1  Constant Volume Process  57
2.7.2  Constant Pressure Process  58
2.7.3  Isothermal Process  59
2.7.4  Hyperbolic Process  59
2.7.5  Reversible Adiabatic or Isentropic Process  60
2.7.6  Polytropic Process  61
2.7.7  Throttling Process  62
2.8  Determination of Dryness Fraction of Steam  62

A01_THERMAL ENGINEERING_SE_XXXX_A01.indd 10 6/20/2018 9:29:14 AM


xi
Contents

2.8.1  Barrel Calorimeter  63


2.8.2  Separating Calorimeter  64
2.8.3  Throttling Calorimeter  65
2.8.4  Combined Separating and Throttling Calorimeter  66
Summary for Quick Revision ·  Multiple-choice Questions ·  Review
Questions ·  Exercises ·  Answers to Multiple-choice Questions

Chapter 3  Steam Generators                 87


  3.1  Introduction 87
 3.2  Classification of Steam Generators  87
 3.3  Comparison of Fire Tube and Water Tube Boilers  88
 3.4  Requirements of a Good Boiler  89
 3.5  Factors Affecting Boiler Selection  89
 3.6  Description of Boilers  89
3.6.1  Fire Tube Boilers  89
3.6.2  Water Tube Boilers  93
  3.7  High Pressure Boilers  94
3.7.1  Boiler Circulation  95
3.7.2  Advantages of Forced Circulation Boilers  95
3.7.3  LaMont Boiler  96
3.7.4  Benson Boiler  97
3.7.5  Loeffler Boiler  97
3.7.6  Schmidt-Hartmann Boiler  98
3.7.7  Velox Boiler  98
3.7.8  Once-through Boiler  99
 3.8  Circulation 99
 3.9  Steam Drum  100
3.9.1  Mechanism of Separation of Moisture in Drum  100
3.10  Fluidised Bed Boiler  102
3.10.1  Bubbling Fluidised Bed Boiler (BFBB)  102
3.10.2  Advantages of BFBB  103
3.11  Boiler Mountings  103
3.11.1  Water Level Indicator  104
3.11.2  Pressure Gauge  104
3.11.3  Steam Stop Valve  105
3.11.4  Feed Check Valve  105
3.11.5  Blow-Down Cock  106
3.11.6  Fusible Plug  106
3.11.7  Safety Valves  107
3.11.8  High Steam and Low Water Safety Valve  108
3.12  Boiler Accessories  109
3.12.1  Air Preheater  110
3.12.2  Economiser 111
3.12.3  Superheater 112

A01_THERMAL ENGINEERING_SE_XXXX_A01.indd 11 6/20/2018 9:29:14 AM


xii Contents

3.13  Steam Accumulators  113


3.13.1  Variable Pressure Accumulator  113
3.13.2  Constant Pressure Accumulator  114
3.14  Performance of Steam Generator  115
3.14.1  Evaporation Rate  115
3.14.2  Performance 115
3.14.3  Boiler Thermal Efficiency  116
3.14.4  Heat Losses in a Boiler Plant  116
3.14.5  Boiler Trial and Heat Balance Sheet  117
3.15  Steam Generator Control  130
3.16  Electrostatic Precipitator  131
3.17  Draught 136
3.17.1  Classification of Draught  136
3.17.2  Natural Draught  136
3.17.3  Height and Diameter of Chimney  137
3.17.4 Condition for Maximum Discharge
 Through Chimney 138
3.17.5  Efficiency of Chimney  139
3.17.6  Advantages and Disadvantages of Natural Draught  140
3.17.7  Draught Losses  140
3.17.8  Artificial Draught  140
3.17.9  Comparison of Forced and Induced Draughts  142
3.17.10  Comparison of Mechanical and Natural Draughts  142
3.17.11  Balanced Draught  142
3.17.12  Steam Jet Draught  143
Summary for Quick Revision ·  Multiple-choice Questions ·  Explanatory
Notes ·  Review Questions ·  Exercises ·  Answers to Multiple-choice
Questions

Chapter 4  Steam Power Cycles               161


4.1 Introduction 161
4.2  Carnot Vapour Cycle  161
4.2.1  Drawbacks of Carnot Cycle  163
4.3 Rankine Cycle 165
4.3.1  Analysis of Rankine Cycle  166
4.3.2  Effect of Boiler and Condenser Pressure  168
4.4  Methods of Improving Efficiency  170
4.4.1 Reheat Cycle 170
4.4.2  Effect of Pressure Drop in the Reheater  171
4.5 Regeneration 174
4.5.1  Regenerative Cycle with Open Heaters  176
4.5.2  Regenerative Cycle with Closed Heaters  177
4.6 Reheat-Regenerative Cycle 182
4.7  Properties of an Ideal Working Fluid  183

A01_THERMAL ENGINEERING_SE_XXXX_A01.indd 12 6/20/2018 9:29:14 AM


xiii
Contents

4.8  Binary Vapour Cycles  184


4.9 Combined Power and Heating Cycle-Cogeneration  187
Summary for Quick Revision ·  Multiple-choice Questions ·  Review
Questions ·  Exercises ·  Answers to Multiple-choice Questions

Chapter 5  Steam Engines                   214


 5.1 Introduction 214
  5.2  Classification of Steam Engines  214
  5.3  Constructional Features of a Steam Engine  215
5.3.1  Steam Engine Parts  216
  5.4  Terminology Used in Steam Engine  217
  5.5  Working of a Steam Engine  218
 5.6 Rankine Cycle 219
  5.7  Modified Rankine Cycle  221
  5.8  Hypothetical or Theoretical Indicator Diagram  222
  5.9  Actual Indicator Diagram  223
5.10  Mean Effective Pressure  224
5.10.1 Without Clearance 224
5.10.2 With Clearance 225
5.10.3  With Clearance and Compression  227
5.10.4 With Clearance and Polytropic Expansion
 and Compression 228
5.11  Power Developed and Efficiencies  230
5.11.1 Indicated Power 230
5.11.2 Brake Power 231
5.11.3  Efficiencies of Steam Engine  232
5.12  Governing of Steam Engines  232
5.13  Saturation Curve and Missing Quantity  243
5.14  Heat Balance Sheet  244
5.15 Performance Curves 246
Summary for Quick Revision ·  Multiple-choice Questions ·  Review
Questions ·  Exercises ·  Answers to Multiple-choice Questions

Chapter 6  Flow Through Steam Nozzles           253


6.1 Introduction 253
6.2 Continuity Equation 254
6.3  Velocity of Flow of Steam Through Nozzles  254
6.3.1  Flow of Steam Through the Nozzle  255
6.4  Mass Flow Rate of Steam  255
6.5  Critical Pressure Ratio  256
6.6 Maximum Discharge 257
6.7  Effect of Friction on Expansion of Steam  258

A01_THERMAL ENGINEERING_SE_XXXX_A01.indd 13 6/20/2018 9:29:14 AM


xiv Contents

 6.8 Nozzle Efficiency 259
  6.9  Supersaturated or Metastable Flow Through a Nozzle  260
6.10  Isentropic, One-Dimensional Steady Flow Through a Nozzle  269
6.10.1 Relationship between Actual and Stagnation
 Properties 270
6.11  Mass Rate of Flow Through an Isentropic Nozzle  273
6.11.1 Effect of Varying the Back Pressure on Mass Flow Rate  274
6.12  Normal Shock in an Ideal Gas Flowing Through a Nozzle  277
Summary for Quick Revision ·  Multiple-choice Questions ·  Explanatory
Notes ·  Review Questions ·  Exercises ·  Answers to Multiple-choice
Questions

Chapter 7  Steam Turbines                  316


  7.1  Principle of Operation of Steam Turbines  316
  7.2  Classification of Steam Turbines  316
  7.3  Comparison of Impulse and Reaction Turbines  318
  7.4  Compounding of Impulse Turbines  319
  7.5  Velocity Diagrams for Impulse Steam Turbine  321
7.5.1  Condition for Maximum Blade Efficiency  324
7.5.2  Maximum Work Done  325
7.5.3  Velocity Diagrams for Velocity Compounded
 Impulse Turbine 325
7.5.4  Effect of Blade Friction on Velocity Diagrams  327
7.5.5  Impulse Turbine with Several Blade Rings  328
  7.6  Advantages and Limitations of Velocity Compounding  329
7.6.1 Advantages 329
7.6.2 Limitations 329
  7.7  Velocity Diagrams for Impulse-Reaction Turbine  330
 7.8 Reheat Factor 333
  7.9  Losses in Steam Turbines  335
7.10 Turbine Efficiencies 335
7.11  Governing of Steam Turbines  336
7.12 Labyrinth Packing 338
7.13  Back Pressure Turbine  339
7.14  Pass Out or Extraction Turbine  340
7.15 Co-Generation 341
7.16  Erosion of Steam Turbine Blades  355
Summary for Quick Revision ·  Multiple-choice Questions ·  Explanatory
Notes ·  Review Questions ·  Exercises ·  Answers to Multiple-choice
Questions

Chapter 8  Steam Condensers               386


8.1 Definition 386
8.2  Functions of a Condenser  386

A01_THERMAL ENGINEERING_SE_XXXX_A01.indd 14 6/20/2018 9:29:14 AM


xv
Contents

8.3  Elements of Steam Condensing Plant  387


8.4  Types of Steam Condensers  388
8.4.1 Jet Condensers 388
8.4.2 Surface Condensers 390
8.5  Requirements of Modern Surface Condensers  393
8.6  Comparison of Jet and Surface Condensers  393
8.6.1 Jet Condensers 393
8.6.2 Surface Condensers 394
8.7 Vacuum Measurement 394
8.8  Dalton’s Law of Partial Pressures  395
8.9  Mass of Cooling Water Required in a Condenser  396
8.10  Air Removal from the Condenser  397
8.10.1  Sources of Air Infiltration in Condenser  397
8.10.2  Effects of Air Infiltration in Condensers  397
8.11 Air Pump 398
8.11.1  Edward’s Air Pump  398
8.12 Vacuum Efficiency 399
8.13 Condenser Efficiency 399
8.14 Cooling Tower 399
Summary for Quick Revision ·  Multiple-choice Questions ·  Review
Questions ·  Exercises ·  Answers to Multiple-choice Questions

Chapter 9  Gas Power Cycles                


423
 9.1 Introduction 423
 9.2 Piston-cylinder Arrangement 423
 9.3 Carnot Cycle 425
 9.4 Stirling Cycle 426
 9.5 Ericsson Cycle 427
 9.6 Atkinson Cycle 428
  9.7  Otto Cycle (Constant Volume Cycle)  429
 9.8 Diesel Cycle 431
 9.9 Dual Cycle 434
9.10 Brayton Cycle 436
9.11  Comparison Between Otto, Diesel, and Dual Cycles  438
Fill in the Blanks ·  Answers ·  True or False ·  Answers ·  Multiple-
choice Questions ·  Review Questions ·  Exercises ·  Answers to
Multiple-choice Questions

Chapter 10  Internal Combustion Engine Systems      485


10.1 Introduction 485
10.2  Classification of Internal Combustion Engines  485
10.3 Construction Features 487

A01_THERMAL ENGINEERING_SE_XXXX_A01.indd 15 6/20/2018 9:29:15 AM


xvi Contents

  10.4  Working of IC Engines  489


10.4.1  Four-stroke Spark-ignition Engine  489
10.4.2  Four-stroke Compression-ignition Engine  490
10.4.3  Two-stroke Spark-ignition Engine  491
10.4.4  Two-stroke Compression-ignition Engine  491
  10.5  Comparison of Four-stroke and Two-stroke Engines  492
  10.6  Comparison of SI and CI Engines  493
  10.7 Merits and Demerits of Two-stroke Engines Over Four-stroke
  Engines 494
10.7.1 Merits 494
10.7.2 Demerits 494
  10.8  Valve Timing Diagrams  495
10.8.1  Four-stroke SI Engine  495
10.8.2  Four-stroke CI Engine  495
10.8.3  Two-stroke SI Engine  495
10.8.4  Two-stroke CI Engine  497
 10.9 Scavenging Process 498
10.10  Applications of IC Engines  499
10.11  Theoretical and Actual p-v Diagrams  500
10.11.1  Four-stroke Petrol Engine  500
10.11.2  Four-stroke Diesel Engine  501
10.11.3  Two-stroke Petrol Engine  503
10.11.4  Two-stroke Diesel Engine  504
10.12 Carburetion 505
10.12.1 Simple Carburettor 506
10.12.2 Compensating Jet 506
10.12.3  Theory of Simple Carburettor  507
10.12.4  Limitations of Single Jet Carburettor  510
10.12.5 Different Devices Used to Meet the Requirements
  of an Ideal Carburettor  511
10.12.6 Complete Carburettor 511
10.13  Fuel Injection Systems in SI Engines  527
10.13.1 Continuous Port Injection System (Lucas Mechanical
  Petrol Injection System)  527
10.13.2  Electronic Fuel Injection System  528
10.13.3  Rotary Gate Meter Fuel Injection System  530
10.14  Fuel Injection in CI Engines  531
10.14.1  Types of Injection Systems  531
10.14.2  Design of Fuel Nozzle  535
10.15 Fuel Ignition 540
10.15.1  Requirement of Ignition System  540
10.15.2 Ignition Systems 541
10.16  Combustion in IC Engines  552
10.16.1  Stages of Combustion in SI Engines  553

A01_THERMAL ENGINEERING_SE_XXXX_A01.indd 16 6/20/2018 9:29:15 AM


Contents
xvii

10.16.2  Ignition Lag (or Delay) in SI Engines  556


10.16.3  Factors Affecting the Flame Propagation  557
10.16.4  Phenomena of Knocking/Detonation in SI Engines  561
10.16.5  Factors Influencing Detonation/Knocking  562
10.16.6  Methods for Suppressing Knocking  564
10.16.7  Effects of Knocking/Detonation  564
10.17  Combustion Chambers for SI Engines  565
10.17.1 Basic Requirements of a Good Combustion
 Chamber 565
10.17.2  Combustion Chamber Design Principles  565
10.17.3  Combustion Chamber Designs  566
10.18  Combustion in CI Engines  567
10.18.1  Stages of Combustion  567
10.18.2  Delay Period or Ignition Delay  568
10.18.3  Variables Affecting Delay Period  569
10.19  Knocking in CI Engines  570
10.19.1  Factors Affecting Knocking in CI Engines  571
10.19.2  Controlling the Knocking  571
10.19.3  Comparison of Knocking in SI and CI Engines  572
10.20  Combustion Chambers for CI Engines  574
10.21 Lubrication Systems 577
10.21.1  Functions of a Lubricating System  577
10.21.2  Desirable Properties of a Lubricating Oil  577
10.21.3  Lubricating Systems Types  577
10.21.4  Lubricating System for IC Engines  579
10.21.5  Lubrication of Different Engine Parts  581
10.22  Necessity of IC Engine Cooling  584
10.22.1  Types of Cooling Systems  584
10.22.2 Precision Cooling 591
10.22.3  Dual Circuit Cooling  591
10.22.4  Disadvantages of Overcooling  591
10.23 Engine Radiators 592
10.23.1 Radiator Matrix 592
10.23.2  Water Requirements of Radiator  593
10.23.3 Fans 594
10.24  Cooling of Exhaust Valve  595
10.25  Governing of IC Engines  596
10.26  Rating of SI Engine Fuels-Octane Number  597
10.26.1 Anti-knock Agents 598
10.26.2 Performance Number 599
10.27  Highest Useful Compression Ratio  599
10.28  Rating of CI Engine Fuels  601
10.29  IC Engine Fuels  601
10.29.1  Fuels for SI Engines  602
10.29.2  Fuels for CI Engines  603

A01_THERMAL ENGINEERING_SE_XXXX_A01.indd 17 6/20/2018 9:29:15 AM


xviii Contents

10.30  Alternative Fuels for IC Engines  603


 10.30.1 Alcohols 604
  10.30.2  Use of Hydrogen in CI Engines  604
 10.30.3 Biogas 605
  10.30.4  Producer (or Water) Gas  605
 10.30.5 Biomass-generated Gas 605
  10.30.6  LPG as SI Engine Fuel  605
  10.30.7  Compressed Natural Gas  606
  10.30.8  Coal Gasification and Coal Liquefaction  606
  10.30.9  Non-edible Vegetable Oils  607
10.30.10  Non-edible Wild Oils  607
10.30.11 Ammonia 607
Summary for Quick Revision ·  Multiple-choice Questions ·  Review
Questions ·  Exercises ·  Answers to Multiple-choice Questions

Chapter 11  Performance of Internal Combustion Engines   622


 11.1 Performance Parameters 622
  11.2  Basic Engine Measurements  624
  11.3  Heat Balance Sheet  626
  11.4  Willan’s Line Method  626
 11.5 Morse Test 627
  11.6  Performance of SI Engines  627
11.6.1 Performance of SI Engine at Constant Speed
  and Variable Load  629
  11.7  Performance of CI Engines  630
 11.8 Performance Maps 631
  11.9  Measurement of Air Consumption by Air-box Method  636
11.10  Measurement of Brake Power  638
11.11  Supercharging of IC Engines  658
11.11.1 Thermodynamic Cycle 658
11.11.2  Supercharging of SI Engines  659
11.11.3  Supercharging of CI Engines  660
11.11.4  Effects of Supercharging  660
11.11.5  Objectives of Supercharging  661
11.11.6  Configurations of a Supercharger  661
11.11.7  Supercharging of Single Cylinder Engines  662
11.12  SI Engine Emissions  663
11.12.1 Exhaust Emissions 665
11.12.2 Evaporative Emission 666
11.12.3 Crankcase Emission 666
11.12.4 Lead Emission 666
11.13  Control of Emissions in SI Engine  666
11.14  Crank Case Emission Control  669
11.15  CI Engine Emissions  669

A01_THERMAL ENGINEERING_SE_XXXX_A01.indd 18 6/20/2018 9:29:15 AM


Contents
xix

11.15.1  Effect of Engine Type on Diesel Emission  670


11.15.2  Control of Emission from Diesel Engine  673
11.15.3 NOx−Emission Control  674
11.16  Three-Way Catalytic Converter  676
11.16.1  Function of a Catalyst in a Catalytic Converter  676
11.17 Environmental Problems Created by Exhaust Emission
  from IC Engines  677
11.18  Use of Unleaded Petrol  678
11.18.1  Use of Additives  678
Summary for Quick Revision ·  Multiple-choice Questions ·  Explanatory
Notes ·  Review Questions ·  Exercises ·  Answers to Multiple-choice
Questions

Chapter 12  Reciprocating Air Compressors         690


12.1 Introduction 690
12.2  Uses of Compressed Air in Industry  691
12.3 Working Principle of Single-stage Reciprocating
 Compressor 691
12.4 Terminology 691
12.5  Types of Compression  692
12.5.1 Methods for Approximating Compression Process
 to Isothermal 692
12.6 Single-Stage Compression 693
12.6.1 Required Work 693
12.6.2 Volumetric Efficiency 696
12.6.3 Isothermal Efficiency 697
12.6.4 Adiabatic Efficiency 699
12.6.5  Calculation of Main Dimensions  699
12.7 Multi-Stage Compression 699
12.7.1 Two-stage Compressor 700
12.7.2  Heat Rejected to the Intercooler  702
12.7.3 Cylinder Dimensions 703
12.7.4  Intercooler and Aftercooler  703
12.8  Indicated Power of a Compressor  705
12.9 Air Motors 705
12.10 Indicator Diagram 706
12.11 Heat Rejected 707
12.12  Control of Compressor  707
Summary for Quick Revision ·  Multiple-choice Questions ·  Explanatory
Notes ·  Review Questions ·  Exercises ·  Answers to Multiple-choice
Questions

Chapter 13  Rotary Air Compressors            735


13.1 Introduction 735
13.2  Working Principle of Different Rotary Compressors  735

A01_THERMAL ENGINEERING_SE_XXXX_A01.indd 19 6/20/2018 9:29:15 AM


xx Contents

13.2.1  Roots Blower or Lobe Compressor  735


13.2.2  Vanes Type Blower  737
13.2.3 Lysholm Compressor 737
13.2.4 Screw Compressor 738
13.3  Comparison of Rotary and Reciprocating Compressors  740
Summary for Quick Revision ·  Multiple-choice Questions ·  Review
Questions ·  Exercises ·  Answers to Multiple-choice Questions

Chapter 14  Centrifugal Air Compressors           744


14.1 Introduction 744
14.2 Constructional Features 744
14.3 Working Principle 745
14.4  Variation of Velocity and Pressure  745
14.5  Types of Impellers  745
14.6  Comparison of Centrifugal and Reciprocating Compressors  746
14.7  Comparison of Centrifugal and Rotary Compressors  746
14.8  Static and Stagnation Properties  747
14.9  Adiabatic and Isentropic Processes  748
14.9.1 Isentropic Efficiency 749
14.10 Velocity Diagrams 749
14.10.1  Theory of Operation  750
14.10.2  Width of Blades of Impeller and Diffuser  753
14.11  Slip Factor and Pressure Coefficient  754
14.12 Losses 755
14.13  Effect of Impeller Blade Shape on Performance  755
14.14 Diffuser 756
14.15 Pre-Whirl 757
14.16 Performance Characteristics 757
14.17  Surging and Choking  760
Summary for Quick Revision ·  Multiple-choice Questions ·  Explanatory
Notes ·  Review Questions ·  Exercises ·  Answers to Multiple-choice
Questions

Chapter 15  Axial Flow Air Compressors           789


15.1 Introduction 789
15.2 Constructional Features 789
15.3 Working Principle 790
15.4  Simple Theory of Aerofoil Blading  790
15.5 Velocity Diagrams 792
15.6  Degree of Reaction  794
15.7  Pressure Rise in Isentropic Flow Through a Cascade  794

A01_THERMAL ENGINEERING_SE_XXXX_A01.indd 20 6/20/2018 9:29:15 AM


Contents
xxi

15.8 Polytropic Efficiency 795
15.9 Flow Coefficient, Head or Work Coefficient, Deflection Coefficient,
  and Pressure Coefficient  797
15.10  Pressure Rise in a Stage and Number of Stages  798
15.11  Surging, Choking, and Stalling  799
15.12 Performance Characteristics 800
15.13  Comparison of Axial Flow and Centrifugal Compressors  802
15.14  Applications of Axial Flow Compressors  802
15.15  Losses in Axial Flow Compressors  802
Summary for Quick Revision ·  Multiple-choice Questions ·  Explanatory
Notes ·  Review Questions ·  Exercises ·  Answers to Multiple-choice
Questions

Chapter 16  Gas Turbines                   840


16.1 Introduction 840
16.2  Fields of Application of Gas Turbine  840
16.3  Limitations of Gas Turbines  840
16.4  Comparison of Gas Turbines with IC Engines  841
16.4.1 Advantages 841
16.4.2 Disadvantages 841
16.5  Advantages of Gas Turbines Over Steam Turbines  841
16.6  Classification of Gas Turbines  842
16.6.1  Constant Pressure Combustion Gas Turbine  842
16.6.2  Constant Volume Combustion Gas Turbine  843
16.7  Comparison of Open and Closed Cycle Gas Turbines  844
16.8  Position of Gas Turbine in the Power Industry  845
16.9 Thermodynamics of Constant Pressure Gas Turbine: Brayton
 Cycle 845
16.9.1 Efficiency 845
16.9.2 Specific Output 847
16.9.3  Maximum Work Output  848
16.9.4 Work Ratio 849
16.9.5 Optimum Pressure Ratio for Maximum Specific
 Work Output 849
16.10  Cycle Operation with Machine Efficiency  850
16.10.1 Maximum Pressure Ratio for Maximum Specific
 Work 850
16.10.2 Optimum Pressure Ratio for Maximum Cycle
 Thermal Efficiency 851
16.11  Open Cycle Constant Pressure Gas Turbine  853
16.12 Methods for Improvement of Thermal Efficiency
  of Open Cycle Constant Pressure Gas Turbine  854
16.12.1 Regeneration 854

A01_THERMAL ENGINEERING_SE_XXXX_A01.indd 21 6/20/2018 9:29:15 AM


xxii Contents

16.12.2 Intercooling 857
16.12.3 Reheating 861
16.12.4  Reheat and Regenerative Cycle  863
16.12.5  Cycle with Intercooling and Regeneration  865
16.12.6  Cycle with Intercooling and Reheating  867
16.12.7 Cycle with Intercooling, Regeneration
 and Reheating 869
16.13  Effects of Operating Variables  890
16.13.1  Effect of Pressure Ratio  891
16.13.2 Effect of Efficiencies of Compressor and Turbine
  on Thermal Efficiency  892
16.14 Multi-Shaft Systems 894
16.15  Multi-Shaft System Turbines in Series  895
16.16  Gas Turbine Fuels  895
16.17 Blade Materials 895
16.17.1 Selection 895
16.17.2  Requirements of Blade Material  896
16.18  Cooling of Blades  896
16.18.1  Advantages of Cooling  896
16.18.2  Different Methods of Blade Cooling  896
Summary for Quick Revision ·  Multiple-choice Questions ·  Explanatory
Notes ·  Review Questions ·  Exercises ·  Answers to Multiple-choice
Questions

Chapter 17  Jet Propulsion                  922


17.1  Principle of Jet Propulsion  922
17.2  Jet Propulsion Systems  922
17.2.1 Screw Propeller 922
17.2.2 Ramjet Engine 923
17.2.3  Pulse Jet Engine  924
17.2.4 Turbo-jet Engine 925
17.2.5 Turbo-Prop Engine 926
17.2.6 Rocket Propulsion 927
17.3  Jet Propulsion v’s Rocket Propulsion  927
17.4  Basic Cycle for Turbo-jet Engine  928
17.4.1 Thrust 930
17.4.2 Thrust Power 930
17.4.3 Propulsive Power 931
17.4.4 Propulsive Efficiency 931
17.4.5 Thermal Efficiency 931
17.4.6 Overall Efficiency 931
17.4.7 Jet Efficiency 932
17.4.8  Ram Air Efficiency  932

A01_THERMAL ENGINEERING_SE_XXXX_A01.indd 22 6/20/2018 9:29:15 AM


Contents
xxiii

17.5 Thrust Work, Propulsive Work, and Propulsive Efficiency


  for Rocket Engine  932
Summary for Quick Revision ·  Multiple-choice Questions ·  Explanatory
Notes ·  Review Questions ·  Exercises ·  Answers to Multiple-choice
Questions

Chapter 18  Introduction to Refrigeration           961


18.1 Introduction 961
18.2 Refrigeration Systems 961
18.3  Methods of Refrigeration  962
18.3.1  Vapour Compression Refrigeration System  962
18.3.2  Vapour Absorption System  963
18.3.3 Ejector-Compression System 963
18.3.4 Electro-Lux Refrigeration 964
18.3.5 Solar Refrigeration 964
18.3.6 Thermo-electric Refrigeration 964
18.3.7  Vortex Tube Refrigeration  965
18.4  Unit of Refrigeration  965
18.5 Refrigeration Effect 965
18.6  Carnot Refrigeration Cycle  966
18.7 Difference Between a Heat Engine, Refrigerator
  and Heat Pump  967
18.8  Power Consumption of a Refrigerating Machine  969
18.9  Air Refrigeration Cycles  970
18.9.1  Open Air Refrigeration Cycle  971
18.9.2  Closed (or dense) Air Refrigeration Cycle  971
18.10  Reversed Carnot Cycle  971
18.10.1 Temperature Limitations for Reversed
 Carnot Cycle 972
18.10.2  Vapour as a Refrigerant in Reversed Carnot Cycle  973
18.10.3  Gas as a Refrigerant in Reversed Carnot Cycle  975
18.10.4  Limitations of Reversed Carnot Cycle  977
18.11  Bell-Coleman Cycle (or Reversed Brayton or Joule Cycle)  977
18.11.1  Bell-Coleman Cycle with Polytropic Processes  979
18.12 Refrigerants 988
18.13  Classification of Refrigerants  988
18.14  Designation of Refrigerants  989
18.15  Desirable Properties of Refrigerants  989
18.16  Applications of Refrigerants  990
18.17 Eco-friendly Refrigerants 993
18.18 Refrigerant Selection 995
Multiple-choice Questions ·  Review Questions ·  Exercises ·  Answers
to Multiple-choice Questions

A01_THERMAL ENGINEERING_SE_XXXX_A01.indd 23 6/20/2018 9:29:15 AM


xxiv Contents

 apour Compression and Vapour Absorption


Chapter 19  V
Systems                      999
19.1 Introduction 999
19.2 Comparison of Vapour Compression System
  with Air Refrigeration System  999
19.3  Simple Vapour Compression Refrigeration System  1000
19.4  Vapour Compression Refrigeration System  1001
19.5  Use of T-s and p-h Charts  1004
19.6  Effect of Suction Pressure  1014
19.7  Effect of Discharge Pressure  1015
19.8  Effect of Superheating of Refrigerant Vapour  1016
19.8.1 Superheat Horn 1017
19.9 Effect of Subcooling (or Undercooling) of
 Refrigerant Vapour 1017
19.10  Vapour Absorption System  1020
19.11 Working Principle of Vapour Absorption Refrigeration
 System 1021
19.11.1 Working 1022
19.12 Advantages of Vapour Absorption System Over Vapour
  Compression System  1022
19.13 Coefficient of Performance of an Ideal Vapour Absorption
 System 1023
19.14 Ammonia-Water (or Practical) Vapour Absorption System
 (NH3 – H2O) 1025
19.15 Lithium Bromide-Water Vapour Absorption System  
 (LiBr-H2O) 1026
19.15.1 Working Principle 1027
19.15.2  Lithium Bromide-Water System Equipment  1028
19.16 Comparison of Ammonia-Water and Lithium Bromide-Water
 Absorption Systems 1029
Exercises

Chapter 20  Air-Conditioning and Psychrometrics       


1034
20.1 Introduction 1034
20.2  Principles of Psychrometry  1034
20.3 Psychrometric Relations 1036
20.4  Enthalpy of Moist Air  1038
20.5  Humid Specific Heat  1039
20.6 Thermodynamic Wet Bulb Temperature or Adiabatic
  Saturation Temperature (Ast) 1043

A01_THERMAL ENGINEERING_SE_XXXX_A01.indd 24 6/20/2018 9:29:15 AM


Contents
xxv

20.7 Psychrometric Chart 1046
20.8 Psychrometric Processes 1048
20.8.1  Sensible Heating or Cooling Process  1048
20.8.2  Humidification or Dehumidification Process  1049
20.8.3  Heating and Humidification  1050
20.8.4  Sensible Heat Factor-SHF  1051
  20.8.5  Cooling and Dehumidification  1051
 20.8.6 Air Washer 1052
  20.8.7 Cooling with Adiabatic Humidification  1054
  20.8.8 Cooling and Humidification by Water Injection
 (Evaporative Cooling) 1056
  20.8.9  Heating and Humidification by Steam Injection  1057
20.8.10  Heating and Adiabatic Chemical Dehumidification  1057
20.9  Adiabatic Mixing of Two Air Streams  1058
20.10  Thermal Analysis of Human Body  1065
20.10.1  Factors Affecting Human Comfort  1066
20.10.2  Physiological Hazards Resulting from Heat  1066
20.11 Effective Temperature 1067
20.11.1 Comfort Chart 1067
20.11.2 Factors Affecting Optimum Effective
 Temperature 1070
20.12  Selection of Inside and Outside Design Conditions  1070
20.12.1  Selection of Inside Design Conditions  1070
20.12.2  Selection of Outside Design Conditions  1071
20.13  Cooling Load Estimation  1072
20.13.1  Heat Transfer Through Walls and Roofs  1072
20.13.2  Heat Gain from Solar Radiation  1073
20.13.3  Sol Air Temperature  1073
20.13.4  Solar Heat Gain Through Glass Areas  1074
20.13.5  Heat Gain due to Infiltration  1074
20.13.6  Heat Gain from Products  1074
20.13.7  Heat Gain from Lights  1075
20.13.8  Heat Gain from Power Equipments  1075
20.13.9  Heat Gain Through Ducts  1075
20.13.10 Empirical Methods to Evaluate Heat Transfer
  Through Walls and Roofs  1075
20.14  Heating Load Estimation  1077
20.15  Room Sensible Heat Factor (Rshf) 1077
20.15.1  Estimation of Supply Air Conditions  1078
20.16  Grand Sensible Heat Factor  1078
20.17  Effective Room Sensible Heat Factor  1079

A01_THERMAL ENGINEERING_SE_XXXX_A01.indd 25 6/20/2018 9:29:15 AM


xxvi Contents

20.18  Air Conditioning Systems  1080


20.18.1 Summer Air-conditioning System with Ventilation
  Air and Zero By-pass Factor  1080
20.18.2 Summer Air-conditioning System with Ventilation
  Air and By-pass Factor  1082
20.18.3  Winter Air-conditioning System  1082
20.18.4  Comfort Air-conditioning System  1084
20.18.5  Industrial Air-conditioning System  1084
Review Questions ·  Exercises

Appendix A 1091
Index1121

A01_THERMAL ENGINEERING_SE_XXXX_A01.indd 26 6/20/2018 9:29:15 AM


Preface

It gives us immense pleasure to present the book on Thermal Engineering. This book is intended for
undergraduate students of mechanical, chemical or process, power, aerospace and automobile engi-
neering. The content covers the syllabi of all the subjects pertaining to thermal engineering such as
applied thermodynamics, power plant engineering, energy conversion and management, internal com-
bustion engines, turbomachinery, gas turbines and jet propulsion, and refrigeration and air-conditioning
taught at different levels of engineering curriculum.
From the vast experience of teaching, we feel that the students are lacking in fundamental concepts
of different aspects of thermal engineering. The basic objective of an introductory course of thermal
engineering should be to build fundamental understanding together with analytical and problem solv-
ing abilities. It is, therefore, imperative to accentuate the basic issues to develop a solid foundation
during the different time-frame starting from the first year to the final year of undergraduate studies.
The present text is an endeavour towards the goal by offering the students a text book on thermal engi-
neering in a most comprehensive manner.
Salient features of the present text are given below:
· Detailed description of intricate physics behind every physical phenomenon
· Stresses on physical understanding to develop mathematical modelling skills for solving real-life
problems related to thermal engineering
· After completion of illustrations, summaries are highlighted for quick revision of the topics and
findings and conclusion are made therein
· A large number of Solved Examples, Multiple-choice Questions, Review Questions and Exercises
in each chapter for easy assimilation of the subject matter
· Solved examples based on questions from reputed institutes/universities as well as from Previous
Question Papers of GATE/IES/IAS
· Multi-choice Questions from GATE/IES/IAS with explanatory notes to help the students to
prepare for different competitive examinations
This book provides a single core level text on ‘Thermal Engineering’. An effort has been made for a
well-balanced coverage of physical concepts, mathematical operations and practical demonstrations
within the scope of the course. Throughout the text, SI units have been used. This book is divided into
twenty chapters.

A01_THERMAL ENGINEERING_SE_XXXX_A01.indd 27 5/17/2018 2:32:12 PM


xxviii Preface

Chapter 1 starts with the description of fuels and the detailed analysis of combustion. The analysis
of flue gas and procedure for determination of calorific value of fuel using different types of calorim-
eter are discussed. Chapter 2 deals with properties of steam. The property changes during transfor-
mation of steam into different phase and for different processes are shown through different plots like
p–v, T–s and h–s. Dryness fraction of steam and its measurement using different calorimeter are dis-
cussed. Chapter 3 discusses different types of steam generators used for power generation. Chapter 4
deals with steam power cycle where basic and improved cycles are discussed. Chapter 5 is devoted to
describe the working of steam engine together with the mathematical analysis. An elaborate in-depth
discussion on flow through steam nozzle is presented in Chapter 6.
The next two chapters (Chapters 7 and 8) deal with the principle of operation of different types
of steam turbines and steam condensers. Chapter 9 discusses the gas power cycles utilised in vari-
ous cycles. The working principle and the function of different components of internal combustion
engines are discussed at length in Chapter 10. Two stroke and four stroke engines as well as spark
ignition and compression ignition engines are considered. Chapter 11 deals with the analysis of per-
formance characteristics of internal combustion engines at different working conditions.
The next four chapters (Chapters 12, 13, 14 and 15) deal with the working principle of four dif-
ferent types of compressors, namely, reciprocating, rotary, centrifugal and axial flow compressors.
Chapter 16 is concerned with the details of different components of gas turbine and their functions.
Chapter 17 is devoted to present different jet propulsion systems together with the associated cycles.
Different refrigeration systems and their components are discussed at length in Chapters 18 and 19.
Chapter 20 deals with psychrometrics where the properties of atmospheric air together with different
processes are presented by psychrometric chart. Fundamentals of different air-conditioning systems
are discussed.
Apart from undergraduate students, this book will provide fundamental information as required by
the practising engineers in the field actual operation in different industries. Students pursuing diploma
can also be benefited out of this text. The text would be extremely helpful for various competitive
examinations such as GATE, IES, IAS, etc.
The complete solutions manual is also available for the faculty members at www.pearsoned.co.in/
sadhusingh
Prof. Singh places on record with gratitude the moral support received from his family members,
children and grandchildren. Dr Pati is indebted to his colleagues and students at NIT Silchar for many
fruitful interactions. Dr Pati expresses his gratitude to his mother, father-in-law, mother-in-law, wife
Munmun, son Soumik and relatives for their persistent encouragement and support and the tolerance
they have meted out during long absence from family life while perusing the present endeavour. The
cooperation received from the publishers and in particular from Ms. Harsha Singh and Mr. M. Balakr-
ishnan is highly appreciated.
Despite all our efforts, it is possible that some unintentional errors are still remaining in the text.
We would gracefully acknowledge if any such errors are pointed out for necessary corrections and
would be extremely thankful to the readers for their constructive suggestions and healthy criticism for
further improvement of the text. If you have comments, suggestions or corrections, please email us at
sukumarpati@gmail.com.

Sadhu Singh
Sukumar Pati

A01_THERMAL ENGINEERING_SE_XXXX_A01.indd 28 6/20/2018 9:29:15 AM


About the Authors

Late Dr Sadhu Singh retired as Director (Colleges), Punjab Technical University, Jalandhar. He had
attained his B.Sc. Mechanical Engineering (Honours) from Punjab University, Chandigarh, M.Sc.
(Mechanical Design and Production Engineering) and Ph.D. from Kurukshetra University. His teach-
ing experience spans 15 years at Regional Engineering College, Kurukshetra (now NIT Kurukshetra)
and 19 years at Pantnagar University. He had been Professor and Head of Mechanical Engineering
Department and Dean, Faculty of Engineering and Technology at G. B. Pant University of Agriculture
and Technology, Pantnagar.

Sukumar Pati is currently working as Assistant Professor in the Department of Mechanical Engineer-
ing at National Institute of Technology (NIT) Silchar, India. Prior to joining at NIT Silchar, he worked
as a Post-doctoral Fellow for almost one year at Indian Institute of Technology Kharagpur after com-
pletion of his Ph.D. from Mechanical Engineering Department of the same Institute. He was awarded
a medal for attaining the second rank in B.E. in Mechanical Engineering from the University of North
Bengal. He has a teaching and research experience of more than seventeen years.
Dr Pati has authored a textbook titled A Textbook on Fluid Mechanics and Hydraulic Machines,
and is the co-author of the textbooks like Engineering Mechanics and Engineering Thermodynamics
and Fluid Mechanics. He has published several research papers in peer-reviewed international jour-
nals, including papers in top-ranked prestigious journals such as International Journal of Thermal
Sciences, International Journal of Heat and Mass Transfer, Physical Review E, International Commu-
nications in Heat and Mass Transfer, Energy, ASME Journal of Heat Transfer, Numerical Heat Trans-
fer, Part A: Applications, Nanoscale and Microscale Thermophysical Engineering and Proceedings
of the Institution of Mechanical Engineers, Part E: Journal of Process Mechanical Engineering. His
field of expertise is thermal science and engineering, and in particular, evaporation and condensation,
micro-scale fluid flow and heat transfer, natural and mixed convection, computational fluid dynamics
and non-Newtonian fluid mechanics.

A01_THERMAL ENGINEERING_SE_XXXX_A01.indd 29 6/20/2018 9:29:16 AM


Thispageisintentionallyleftblank
Chapter 1

Fuels and Combustion

1.1 ❐ INTRODUCTION
A fuel may be defined as a substance, which on burning with oxygen produces a large amount of heat.
A fuel is mainly composed of carbon and hydrogen. The energy produced by burning of fuel in the
form of heat is known as chemical energy.

1.2 ❐ CLASSIFICATION OF FUELS


Fuels may be classified as follows:
1. Based on their occurrence, they are classified as natural fuels and artificial (or prepared) fuels.
  Natural fuels occur in nature and they are also called primary fuels, whereas artificial fuels are
prepared by further processing of primary fuels and they are also called secondary fuels.
2. Based on their state, they are classified as solid fuels, liquid fuels, and gaseous fuels.
A detailed classification of fuels is given in Table 1.1.

Table 1.1  Classification of fuels

Type of fuel Natural or primary fuels Artificial or secondary fuels


1. Solid Wood Coke
Peat Charcoal
Lignite (Brown coal) Briquettes coal
Bituminous coal Pulverised coal
Anthracite
2. Liquid Petroleum Gasoline or petrol
Diesel
Kerosene
Alcohol
(Continued)

M01_THERMAL ENGINEERING_SE_XXXX_CH01(part-1).indd 1 6/15/2018 12:52:47 PM


2 Chapter 1

Table 1.1  (Continued )

Type of fuel Natural or primary fuels Artificial or secondary fuels


Benzol
Heavy fuel oil
Shale oil
3. Gaseous Natural gas Petroleum gas, LPG
LNG Producer gas
Coal gas (or town gas)
Coke–oven gas
Blast furnace gas
Bio–gas
Sewer gas

1.3 ❐ SOLID FUELS


1.3.1  Primary Fuels
1. Wood: It is the most commonly used and easily available natural fuel. It has been used for cook-
ing and other purposes over centuries. The important constituents of wood are cellulose fibre and
water. On burning, it gives ash, which is used for cleaning utensils in rural and even urban areas.
The calorific value of wood varies with its type and moisture content. It ignites easily at about
250°C and is used for igniting other fuels.
2. Peat: It is a mixture of water and decayed vegetable matter. It contains a large amount of mois-
ture and requires drying in sun for about one to two months before use. It is the first stage in the
formation of coal from wood.
3. Lignite: It is transformed from peat. It is brown in colour and is also known as brown coal. Air-
dried lignite contains 10%–20 % moisture. It is used as low-grade fuel. It is non–caking and burns
with a large smoky flame.
4. Bituminous coal: It is transformed from lignite. It has a shining, black appearance and comes in
caking and non–caking varieties. It ignites easily and burns with a long yellow and smoky flame.
5. Anthracite coal: It is transformed from bituminous coal. It is hard, brittle, and lustrous in appear-
ance. It is non–caking and burns without smell, smoke, or flame. It is difficult to ignite. This coal
has minimum ash, volatile matter, and moisture. Its calorific value is the highest and is suitable
for steam generation in thermal power plants.
The calorific value of solid fuels is given in Table 1.2.

Table 1.2  Calorific value of solid fuels

Fuel Higher calorific value, kJ/kg


Wood (dry) 19,300
Peat (dry) 19,700
(Continued)

M01_THERMAL ENGINEERING_SE_XXXX_CH01(part-1).indd 2 6/15/2018 12:52:47 PM


Fuels and Combustion 3

Table 1.2  (Continued )

Fuel Higher calorific value, kJ/kg


Lignite (Brown coal) 21,650
Bituminous coal 33,500
Anthracite 34,600
Coke 33,600
Charcoal 34,300

1.3.2 Secondary Fuels


1. Wood charcoal: It is obtained by destructive distillation of wood in a crude method. It is prepared
by slow burning of logs in a domed type of earthen structure under controlled conditions of air
supply. It takes many days to obtain charcoal. During the process, the volatile matter and water are
expelled to the atmosphere. The temperature of heated air is not allowed to increase beyond about
300°C. It burns rapidly with a clear flame without smoke. It is soft and dark black in colour. It is
mainly used for domestic purposes.
2. Coke: It is prepared by removing the volatile matter from bituminous coal. It is hard, brittle, and
porous. It is generally prepared by heating in an electric furnace, followed by water sprays. It
contains carbon, 2% Sulphur, and small quantities of hydrogen, nitrogen, and phosphorus. It is
mainly used in blast furnace to produce heat and reduce iron ore.
3. Briquetted coal: It consists of finely ground coal or coke mixed with a suitable binder and
pressed together to form blocks or briquettes of any shape. In the briquetted form, the heating
value of low-grade coal is increased.
4. Pulverised Coal: The crushed coal to powder is called pulverised coal. The fineness of powdered
coal is so adjusted that it floats during the burning process. This gives better contact between air
and fuel, which results in very high combustion efficiency. The pulverising method is used for
steam raising in boilers by using low grade and rough fuels. The advantages of coal pulverising
are as follows:
(a)  Flexibility of control
(b)  Complete combustion with lesser excess air
(c)  High flame temperature

1.3.3 Desirable Properties of Coal


A good coal should have the following properties:
1. Low ash content
2. High calorific value
3. Small percentage of sulphur (<1%)
4. Good burning characteristics, that is, it should burn freely without agitation
5. High grindability index for pulverising purposes
6. High weatherability

M01_THERMAL ENGINEERING_SE_XXXX_CH01(part-1).indd 3 6/15/2018 12:52:47 PM


4 Chapter 1

1.3.4 Ranking of Coal


Rank is an inherent property of the fuel, depending upon its relative progression in the classification
process. The ranking of coal is based on the fixed carbon and heating value of the mineral matter free
analysis. The ASME (American Society of Mechanical Engineers) and ASTM (American Society of
Testing and Materials) have accepted the following ranking specifications for coal:
1. Higher ranking: It is done on the basis of heating value (moist basis).
2. Lower ranking: It is done on the basis of heating value (most basis).
For example, A coal ranked as 62–20,000 means that the coal contains C = 62% and C.V. = 20,000
kJ/kg.

1.3.5 Grading of Coal


The grading of coal is done on the following basis:
1. Size
2. Heating value
3. Ash content
4. Ash softening temperature
5. Sulphur content
For example, a coal grade written as 5–10 cm, 20,000 – A8-F24 – F1.6 indicates that the coal has
Size: 5–10 cm
C.V.: 20,000 kJ/kg
Ash: 8–10%
Ash softening temperature: 2400–2500°F
Sulphur content: 1.4–1.6%

1.4 ❐ LIQUID FUELS


The main source of liquid fuels is petroleum which is obtained from drilled wells under the Earth’s
crust. Petroleum has originated probably from organic matter such as fish and plant either by bacte-
rial action or distillation under pressure and heat. It consists of a mixture of gases, liquids, and solid
hydrocarbons with small amounts of nitrogen and sulphur compounds.
Liquid fuels are classified as follows:
1. Natural or crude oils: These oils are further distilled to obtain petrol, benzene, diesel, and other
light oils such as kerosene, alcohol, and so on.
2. Distilled artificial oils: These oils include coal–tar, tar–oil, shale–oil, and natural gas oil.
Various oils obtained from natural or crude oils are briefed below.
1. Petrol: It is the lightest and the most volatile liquid fuel. It is mainly used for spark ignition petrol
engines. It is obtained by fractional distillation of petroleum at 65°C to 220°C. It is also known as
gasoline.
2. Kerosene: It is obtained by fractional distillation of crude oil at 345°C to 365°C. It is also known
as paraffin oil. It is heavier and less volatile than petrol. It is used for heating and lighting purposes.

M01_THERMAL ENGINEERING_SE_XXXX_CH01(part-1).indd 4 6/15/2018 12:52:47 PM


Fuels and Combustion 5

3. Diesel: It is obtained by fractional distillation of crude oil at 345°C to 470°C. It is mainly used in
diesel engines and oil–fired furnaces and boilers. It is also known as heavy fuel oil.
4. Benzene: It is obtained by the redistillation of tar, which is the byproduct collected during the
production of coal gas. It is used as an alternate fuel for internal combustion (IC) engines. It is
less prone to detonation than standard petrol.
5. Alcohol: It is formed by the fermentation of vegetable matter. The calorific value of alcohol is
lower than that of petrol and the cost is higher. It is mainly used for industrial purposes.

1.4.1 Advantages and Disadvantages of Liquid Fuels Over Solid Fuels


The advantages of liquid fuels over solid fuels are as follows:
1. Requirement of less space for storage
2. Higher calorific value
3. Easy handling and transportation
4. Better control of consumption by using valves and number of burners
5. Absence of danger from spontaneous combustion
6. Better cleanliness and absence from dust
7. Absence of ash problems
8. No problem of corrosion and deterioration during storage
9. Higher combustion efficiency
The disadvantages of liquid fuels over solid fuels are as follows:
1. Higher cost
2. Greater risk of fire
3. Require costly containers for storage and transportation

1.4.2 Calorific Value of Liquid Fuels


The calorific value of liquid fuels is given in Table 1.3.

Table 1.3  Calorific value of liquid fuels


Fuel HCV kJ/kg LCV kJ/kg
Gasoline 46,820 43,700
Petrol 47,300 44,000
Diesel oil 46,000 43,250
Heavy fuel oil 44,000 41,300
Paraffin (Kerosene oil) 46,250 43,250
Propane (LPG) – 45,900

1.4.3 Desirable Properties of Liquid Fuels


Liquid fuels should have the following desirable properties:
1. Low ash content
2. High calorific value

M01_THERMAL ENGINEERING_SE_XXXX_CH01(part-1).indd 5 6/15/2018 12:52:47 PM


6 Chapter 1

3. Low gum content


4. Less corrosive tendency
5. Low sulphur content
6. Low pour point

1.5 ❐ GASEOUS FUELS


1. Natural gas: The main constituents of natural gas are methane (CH4) and ethane (C2H6). Its
calorific value is nearly 21 MJ/m3. It is used as an alternate fuel for internal combustion engines
as liquefied natural gas (LNG).
2. Coal gas: It mainly consists of hydrogen, carbon monoxide, and hydrocarbons. It is prepared
by carbonisation of coal. It is also known as town gas. It is largely used in towns for street and
domestic lighting, cooking, and for running gas engines.
3. Coke–oven gas: It is obtained during the production of coke by heating bituminous coal. The
volatile content of coal is driven off by heating. It is produced by the destructive distillation of
coal in air–tight coke ovens made of silica bricks. It is used in heating ovens. This gas must be
thoroughly filtered before using in gas engines.
4. Blast furnace gas: It is obtained as a byproduct from the smelting operation in which air is forced
through layers of coke and iron during the manufacture of pig iron. It contains about 20% carbon monox-
ide. The gas leaving the blast furnace has a high dust content, which must be removed by dust collectors
and washing operation. It has a low calorific value. It is mainly used as a fuel for metallurgical furnaces.
5. Producer gas: It is obtained from the partial oxidation of coal, coke, or peat when burnt with an
insufficient quantity of air. It has low heating value and in general is suitable for large installa-
tions. It is also used in steel industry for firing open hearth furnaces.
6. Water gas: It is produced by blowing steam into white hot coke or coal. It is a mixture of hydro-
gen and carbon monoxide. It burns with a blue flame and is also known as blue water gas. It does
not contain unsaturated hydrocarbons. In order to use this gas for domestic lighting, unsaturated
hydrocarbons are added. In such cases, it is also known as carburetted water gas or enriched water
gas. This is achieved by passing the gas through a carburettor into which a gas oil is sprayed. It is
usually mixed with coal gas to form town gas. It is used in furnaces and for welding.
7. Sewer gas: It is obtained from sewage disposal vats in which fermentation and decay occur. It
consists of mainly marsh gas (CH4) and is collected at large disposal plants. It is used as a fuel for
gas engines which, in turn, drives plant pumps and agitators.

1.5.1 Calorific Value of Gaseous Fuels


The calorific value of gaseous fuels is given in Table 1.4.

Table 1.4  Calorific value of gaseous fuels

Fuel Calorific value at S.T.P., MJ/m3


HCV LCV
Coal gas 20.00 17.50
Blast furnace gas 3.41 3.37
(Continued)

M01_THERMAL ENGINEERING_SE_XXXX_CH01(part-1).indd 6 6/15/2018 12:52:47 PM


Fuels and Combustion 7

Table 1.4  (Continued )

Fuel Calorific value at S.T.P., MJ/m3


HCV LCV
Producer gas 6.04 4.80
Coke oven gas 21.30 19.30
LNG 36.20 32.60
Hydrogen 11.85 10.00
Methane – 33.95
Carbon monoxide – 11.97
Ethane – 60.43
Propane – 86.42
n - Butane – 112.41

1.5.2 Advantages and Disadvantages of Gaseous Fuels


The advantages of gaseous fuels are as follows:
1. Better control of combustion
2. Excess air required for complete combustion is very less
3. Can be directly used in IC engines
4. Give higher combustion efficiency
5. Free from solid and liquid impurities.
6. Do not produce ash and smoke.
7. No problem of shortage if supply is available from public supply line.
8. Distribution of gaseous fuels even over a wide area is easy with the help of pipe line.
The disadvantages of gaseous fuels are as follows:
1. They are highly inflammable.
2. The storage space required is huge if public supply line is not available.

1.5.3 Important Properties of Gaseous Fuels


The important properties of gaseous fuels are as follows:
1. Calorific value
2. Viscosity
3. Specific gravity
4. Density
5. Diffusibility

1.6 ❐ LIQUEFIED GASES


Petroleum gases can be liquefied by slightly compressing them. When these gases liquefy, they con-
tract and occupy less space. They can then be transported more easily in liquid form in pressurised

M01_THERMAL ENGINEERING_SE_XXXX_CH01(part-1).indd 7 6/15/2018 12:52:47 PM


8 Chapter 1

containers. If the pressure is released by opening the valve, the liquid from the container begins to
evaporate and provides a continuous supply of gas.
There are only two families of petroleum gases which can be liquefied by compression at atmos-
pheric pressure. These are as follows:
1. Propanes (C3H8) and Propylenes (C3H6)
2. Butanes (C4H10) and Butylenes (C4H8)
Propane and butane are paraffin hydrocarbons and have the general formula as CnH2n + 2. Methane
(CH4) and ethane (C2H6) must be chilled before they can be liquefied.
Another series of hydrocarbons associated with the aforementioned paraffin is called Olefins and
have the general formula as CnH2n. These include ethylene, propylene, butylene, etc.

1.6.1 Liquefied Petroleum Gas


Liquefied petroleum gas (LPG) mainly contains propane and some fractions of butane. It must be
stored under pressure to keep it in liquid form. LPG is gaining popularity as a gasoline substitute in
IC engines because it costs 60% of petrol and gives 90% mileage of its fellow gasoline. It burns more
clearly, has a higher octane number. Large quantities of LPG are now available from gas and petro-
leum industries. They are often used as fuel for tractors, trucks, and as domestic fuel for cooking. The
calorific value of LPG is about 45,360 kJ/kg.

1.6.2 Liquefied or Compressed Natural Gas


LNG comes from dry natural reservoirs, mainly constituting about 85–90% of methane (CH4) with
very small percentages of ethane and propane. Its critical temperature is about 73°C. It is found in coal
reserves in tight sands and is trapped in geo-pressurised zones within the earth. LNG is obtained when
natural gas (NG) is placed under very high pressure. On the release of high pressure, LNG converts to
NG. At atmospheric pressure and above freezing temperature, LPG is in gas form.
Due to the low critical temperature of NG, it cannot be liquefied at ordinary temperature. To liq-
uefy NG, it must be cooled to cryogenic (ultra-low) temperatures. It must be stored in well-insulated
containers so as to hold it in liquid form. The pressure must be maintained at about 50–60 bar. For
automobiles, a pressure of 220–250 bar is required. The NG used in automobiles is known as CNG.
NG used for domestic and industrial purposes is supplied in pipes and is hence known as piped natural
gas (PNG). The calorific value of LNG is 40.7–41.2 MJ/m3.

1.7 ❐ BIOFUELS
The organic material produced by plants and their derivatives is called biomass. It includes forest
crops and residues, energy crops, and animal manure. It reacts with oxygen in combustion and natural
metabolic process to release heat. The material may be transformed by chemical and biological pro-
cesses to produce intermediate biofuels such as methane gas, ethanol, or charcoal solid. Sugarcane,
poplar, eucalyptus, animal, and human waste include biomass which are used as biofuels.
Biofuel production is economical if the production process uses materials as by-products. They are
available at low cost. Some examples are waste from animal enclosures, offcuts and trimmings from

M01_THERMAL ENGINEERING_SE_XXXX_CH01(part-1).indd 8 6/15/2018 12:52:47 PM


Fuels and Combustion 9

sawmills, municipal sewage, rice husk and straw, wheat straw, shells from coconuts, and straw from
cereal crops. The main dangers of extensive use of biomass fuel are deforestation, soil erosion, and
displacement of food crops by fuel crops.

1.8 ❐ ANALYSIS OF FUELS


The analysis of solid fuels may be carried out in two ways: proximate analysis and ultimate analysis.

1.8.1 Proximate Analysis


The proximate analysis of coal is carried out to determine its behaviour when heated. It gives the per-
centage of fixed carbon, volatile matter, moisture content, and ash content. This analysis is sufficient
for commercial purposes.
The following procedures are adopted to estimate the various contents of coal:
1. Moisture content: One gram sample of coal is heated to a temperature of about 105°C for a
period of one hour. The loss in weight gives the moisture content of the coal.
2. Volatile matter: One gram sample of coal is placed in a covered platinum crucible and heated to
950°C. This temperature is maintained for about seven minutes. There is a loss in weight due to
vapourisation of moisture and volatile matter. Since the moisture content has already been calcu-
lated, the volatile matter can be obtained as
Volatile matter = total loss in weight – moisture content
3. Ash content: One gram sample of coal in an uncovered crucible is heated to a temperature of
about 720°C until the coal is completely burned. Complete combustion of coal is ascertained by
repeated weighing of sample. This ensures that there is only ash remaining in the crucible.
4. Fixed carbon: Fixed carbon can be obtained as
Fixed carbon = (100% by mass of coal) – (moisture content + volatile matter + ash)
However, this difference does not represent all the carbon present in the coal, because some of
the carbon in the form of hydrocarbons may have been lost while estimating the volatile matter.

1.8.2 Ultimate Analysis


The ultimate analysis gives the percentage of each chemical element in coal, along with ash and mois-
ture. This analysis gives the following components on mass basis:
Carbon (C), hydrogen (H2), oxygen (O2), nitrogen (N2), sulphur (S), moisture (M), and ash (A).
Thus, C + H2 + O2 + N2 + S + M + A = 100 % by mass.

1.9 ❐ CALORIFIC VALUE OF FUELS


The calorific value of a solid or liquid fuel is defined as the heat evolved by the complete combus-
tion of unit mass of fuel. The calorific value of a gaseous fuel is expressed as the heat evolved by the
complete combustion of one cubic metre of gas at standard temperature and pressure. There are two
calorific values of fuels.

M01_THERMAL ENGINEERING_SE_XXXX_CH01(part-1).indd 9 6/15/2018 12:52:47 PM


10 Chapter 1

Higher Calorific Value


All fuels containing hydrogen produce water vapour during combustion. If these products of combus-
tion are cooled to the room temperature, the water vapour will condense evolving its latent heat of
vapourisation, producing the maximum amount of heat per kg of fuel. This heat is known as higher
calorific value (HCV).

Lower Calorific Value


The cooling of flue gases to room temperature is not possible in most combustion processes. There-
fore, the amount of latent heat of water vapour goes waste. Accordingly, for the calculation of the
lower calorific value, we assume that the water vapour formed during combustion leaves as vapour.
LCV = HCV − heat carried away by water vapour formed per kg of fuel burned.
= HCV − mw × hfg (1.1)
where mw = mass of water vapour formed per kg fuel burned, hfg = latent heat of vapourisation at the
partial pressure of water vapour in the combustion products = 2395 kJ/kg.
The HCV of fuel is given by

1   O 
H.C.V. =  35000 C + 143000  H −  + 9160 S kJ/kg  (1.2)
100   8  

where C, H, O, and S are percentages of carbon, hydrogen, oxygen, and sulphur, respectively.
The LCV of fuel is given by

LCV = HCV − (9/100) × H × 2460 kJ/kg. (1.3)

1.10   ❐   COMBUSTION OF FUELS


The following chemical equations are used to calculate the amount of oxygen required and the amount
of gases produced by the combustion of fuel, by using the molecular weight of the elements in kg:
1. C + O2 → CO2
12 + 32 → 44
8 11
1+

3 3
2. 2C + O2 → 2CO
24 + 32 → 56
4 7
1+

3 3
3. 2CO + O2 → 2CO2
56 + 32 → 88
4 11
1+ →
7 7

M01_THERMAL ENGINEERING_SE_XXXX_CH01(part-1).indd 10 6/15/2018 12:52:48 PM


Fuels and Combustion 11

4. 2H2 + O2 → 2H2O
4 + 32 → 36
1 + 8 → 9
5. S + O2 → SO2
32 + 32 → 64
1 + 1 → 2

1.11   ❐   COMBUSTION OF HYDROCARBON FUEL


For any hydrocarbon fuel, we have
xCnH2n + 2 + yO2 → aCO2 + bH2O
where x, y, a, and b are constants to be evaluated.
Equating the weights of C, H, and O on both sides of the equation, we have
xn = a
or   a = xn
x(2n + 2) = 2b
or    b = [x/2] (2n + 2) = x(n + 1)
2y = 2a + b
or      y = a + [b/2] = xn + [x/2] (n + 1) = [x/2] (3n + 1)
Substituting there values in combustion equation, we get
x
xCn H2n + 2 + (3n + 1) O2 → xn CO2 + x(n + 1)H2O
2

 3n + 1
or CnH2n + 2 +  O → nCO2 + (n + 1) H2O(1.4)
 2  2

1.12 ❐ MINIMUM AIR REQUIRED FOR COMPLETE


COMBUSTION OF SOLID/LIQUID FUELS
Let C, H, O, and S = percent by mass of C, H2, O2, and S in fuel. Then, the mass of O2 required per
kg of fuel for complete combustion
1 8  O 
=  C + 8  H −  + S
100  3  8 
Percentage of O2 by mass in air = 23%
∴ Minimum amount of air required in kg per kg of fuel for complete combustion
1 8  O  100
=  C + 8  H −  + S ×
100  3  8  23
1 8  O 
=  C + 8  H −  + S (1.5)
23  3  8 

M01_THERMAL ENGINEERING_SE_XXXX_CH01(part-1).indd 11 6/15/2018 12:52:50 PM


12 Chapter 1

1.13 ❐ CONVERSION OF VOLUMETRIC ANALYSIS TO MASS


(OR GRAVIMETRIC) ANALYSIS AND VICE-VERSA
Let the fraction of CO, CO2, N2, and O2 by volume of dry exhaust gases by C1, C2, N, and O, respec-
tively, obtained by the Orsat apparatus. The volumetric analysis can be converted into gravimetric
analysis, as shown is Table 1.5.

Table 1.5  Conversion from volumetric to gravimetric analysis

Constituent Volume Molecular Proportional Mass fraction


(1) fraction weight mass (4)
(2) (3) (4) = (2) ë (3) (5) =
ΣX

28 C1
CO C1 28 28 C1
∑X

44 C2
CO2 C2 44 44 C2
∑X

28 N
N2 N 28 28 N ∑X

32O
O2 O 32 32 O
∑X

Σ(4) = ΣX = 28 C1 + 44 C2 + 28 N + 32 O
The conversion from gravimetric to volumetric analysis is given in Table 1.6.

Table 1.6  Conversion of gravimetric analysis to volumetric analysis

Constituent Mass Molecular Proportional Volume fraction


(1) fraction weight volume (4)
(2) (3) (4) = (2)/(3) (5) =
ΣY
C1
CO C1 28 C1/28
28∑Y

C2
CO2 C2 44 C2/44 44 ∑Y

N
N2 N 28 N/28 28∑Y

O
O2 O 32 O/32 32 ∑Y

M01_THERMAL ENGINEERING_SE_XXXX_CH01(part-1).indd 12 6/15/2018 12:52:53 PM


Fuels and Combustion 13

C1 C2 N O
Σ(4) = ΣY = + + +
28 44 28 32

1.14  ❐  DETERMINATION OF AIR SUPPLIED


1.14.1 Percentage of Carbon by Mass in Fuel and Volumetric
Analysis is Known
28 C1 44 C2
We know that 1 kg of flue gas contains kg of CO and kg of CO2
ΣX ΣX

28 C1 12 28C1 12C1
Amount of carbon in kg of CO = × =
ΣX 28 ΣX ΣX
44 C2 12 44 C2 12 C2
Amount of carbon in kg of CO2 = × =
ΣX 44 ΣX ΣX
12
Total mass of carbon in 1 kg of flue gas = (C + C2 ) kg
ΣX 1
Let C = mass fraction of carbon in fuel and there is no nitrogen in fuel.
mg = mass of flue gases formed per kg of fuel burned.

12
Then, C = ( C + C 2 ) mg
∑X 1
C∑X
∴     mg =
12(C1 + C2 )

Now, nitrogen in 1 kg of air supplied = 0.77 kg


28N
N2 in 1 kg of flue gas =
∑X
28 N 1
∴ Air supplied per kg of flue gas = × kg
∑ X 0.77
Mass of air supplied per kg of fuel = mass of air supplied per kg of flue gas
× mass of flue gases formed per kg of fuel.

28 N 1 28 N × 1 × C ΣX
ma = × × mg =
ΣX 0.77 ΣX 0.77 12( C1 + C2 )

1  CN 
=   (1.6)
0.33 C + C2 
   1

M01_THERMAL ENGINEERING_SE_XXXX_CH01(part-1).indd 13 6/15/2018 12:52:59 PM


14 Chapter 1

1.14.2  Excess Air Supplied


The oxygen carried in the flue gases comes only from the air supplied for the combustion of fuel. In
addition, the oxygen necessary to convert CO to CO2 should be taken into account in calculating the
excess air supplied.
28 C1
CO in 1 kg of flue gas = kg
∑X
32 O
O2 in 1 kg of flue gas = kg
∑X
28 C1 4 28 C1 16 C1
O2 required for burning kg of CO to CO2 = × =
∑X 7 ΣX ΣX
32 O 16 C1
Excess O2 available per kg of flue gas formed = −
∑X ∑X
16
= ( 2O − C1 )
∑X
Excess O2 supplied per kg of fuel = Excess O2 per kg of flue gas × mg
16 C∑X 4C  2O − C1 
= ( 2O − C1 ) × =
∑X 12(C1 + C2 ) 3  C1 + C2 
Excess air supplied per kg of fuel
4C  2O − C1  100 400 C  2O − C1 
    = × = kg (1.7)
3  C1 + C2  23 69  C1 + C2 

1.15 ❐ DETERMINATION OF PERCENTAGE OF CARBON


IN FUEL BURNING TO CO AND CO2
1. Let the volumetric composition of dry flue gases be known.
Percentage of C burning to CO2
Mass of C in CO 2 per kg of flue gas
=
Mass of C in 1 kg of flue gas
44 C2 12
× C2
= ΣX 44 × 100 = × 100
44 C2 12 28 C1 12 C1 + C2
× + ×
ΣX 44 ΣX 28
Similarly, percentage of C burning to CO
C1
= × 100
C1 + C2

M01_THERMAL ENGINEERING_SE_XXXX_CH01(part-1).indd 14 6/15/2018 12:53:07 PM


Fuels and Combustion 15

2. When gravimetric analysis is known, then


7C2
Percentage of C burning to CO 2 = (1.8)
11C1 + 7C2

11C1
Percentage of C burning to CO = (1.9)
11C1 + 7C2

1.16 ❐ DETERMINATION OF MINIMUM QUANTITY OF AIR


REQUIRED FOR COMPLETE COMBUSTION OF
GASEOUS FUEL
Let the volumetric analysis of gaseous fuel be known. The chemical equations for gaseous fuel are as
follows:
1. 2H2 + O2 → 2H2O
2 volumes of H2 + 1 volume of O2 → 2 volumes of H2O
1
or 1 volume of H2 + volume of O2 → 1 volume of H2O
2
2. 2CO + O2 → 2CO2
2 volumes of CO + 1 volume of O2 → 2 volumes of CO2
3. CH4 + 2O2 → 2H2O + CO2
1 volume of CH4 + 2 volumes of O2 → 2 volumes of H2O + 1 volume of CO2
4. C2H4 + 3O2 → 2H2O + 2CO2
1 volume of C2H4 + 3 volumes of O2 → 2 volumes of H2O + 2 volumes of CO2
Let the volume fraction of the gaseous fuel contain H2, CO, CH4, C2H4, CO2, and N2, the last two
being incombustible gases.
H CO 
The quantity of O2 required =  2 + + 2CH 4 + 3C2 H 4  m3/m3 of gas
 2 2 
Air contains 21% of O2 by volume.
100  H 2 CO 
Quantity of air required = × + + 2CH 4 + 3C2 H 4 
21  2 2 
1
= [H2 + CO + 4CH4 + 6C2H4] m3 of air/m3 of gas. (1.10)
0.42

1.17 ❐ DETERMINATION OF EXCESS AIR SUPPLIED


FOR GASEOUS FUEL
Let the volumetric fractions of gaseous fuel and dry flue gases be known.
Let V = volume of flue gases formed in m3 per m3 of fuel when air supplied is just sufficient for
complete combustion.

M01_THERMAL ENGINEERING_SE_XXXX_CH01(part-1).indd 15 6/15/2018 12:53:09 PM


16 Chapter 1

V1 = volume of excess air supplied in m3 per m3 of fuel gas.


Total volume of dry flue gases formed per m3 of gaseous fuel = V + V1
V
Volume of excess air in flue gases per m3 of flue gas = m3 of air/m3 burned gases.
3 3
Let O = quantity of O2 in m /m of exhaust gas V + V1

O 3
Then air in the exhaust gas = m of air/m3 of flue gases
21

V1 O
∴ =
V + V1 21

O×V 3
or V1 = m of excess air/m3 of fuel (1.11)
21 − O

1.18   ❐   FLUE GAS ANALYSIS


1.18.1  Orsat Apparatus Construction
The construction of the Orsat apparatus is shown in Fig. 1.1. It consists of three flasks a, b, and c each
containing different chemicals for absorbing CO2, O2, and CO. The percentage of N2 is obtained by
difference. The percentage of SO2 in the gases cannot be measured by this apparatus. The absorbents
used in flask a are NaOH or KOH solution to absorb CO2. The tube b contains alkaline solution of
pyrogallic acid to absorb O2. The flask c contains cuprous chloride to absorb CO. Keeping values X,
Y, and Z closed, the three-way valve A is opened and the aspirator bottle B is moved down so that the
flue gas enters the eudiometer tube. The flue gas is drawn in until the level in the eudiometer reads
zero. The three-way valve A is closed and then the value X of the flask a is opened and the aspirator

Wooden frame
Handle Single way
3 way value, A Front header
valve, C
U-tube
Flue
gases

CaCl
Z Y X
Single way 100 cc
glass valve
Water
c b a jacket
Glass bottle
B
Glass tubes

Levelling
Eudiometer
screws Rubber tubes
Aspirator bottle

Figure 1.1  Orsat apparatus

M01_THERMAL ENGINEERING_SE_XXXX_CH01(part-1).indd 16 6/15/2018 12:53:12 PM


Fuels and Combustion 17

bottle is moved up and down several times pushing the gas into the flask a containing KOH solution
to absorb CO2 gas. Now, the flue gas is taken into the eudiometer by lowering the aspirator bottle and
the valve X is closed, keeping the original level of the solution. The aspirator is then brought near the
eudiometer and placed at such a position that the water level in both places is the same and the reading
of the eudiometer tube is taken. The difference in reading gives the percentage of CO2 in the flue gas.
The procedure is repeated with the flasks b and c to find the percentage of O2 and CO. The remainder
of the flue gas is taken as N2.
This apparatus gives the percentages of dry flue gases only, as water vapour is condensed at room
temperature when the gas is drawn over water. The flue gases may be passed through a U-tube con-
taining calcium chloride before passing into the Orsat apparatus to ensure that only dry gas is being
analysed.

Test Procedure
Before starting the experiment it is essential that the chemical solutions in the flask are fresh and free
from pollution. Then, water is filled in the eudiometer jacket and salt water in the aspirator bottle. To
expel the residual gases, if any, from the eudiometer, the three-way valve is opened to the atmosphere
and the aspirator bottle is raised to fill the eudiometer to the 100 cc mark. The three-way valve is then
closed and the valve x is opened. The aspirator bottle is lowered which, in turn, lowers the level of salt
water in the eudiometer resulting in the rise in the level of chemical solution in flask a. As soon as the
chemical solution touches a mark on the capillary connecting the common header tube, the valve x
is closed. The same procedure is repeated for flasks b and c. By doing so, all the residual gases in the
flasks are expelled to the eudiometer and the common header. The residual gases from the eudiometer
are expelled by opening the single way valve C to the atmosphere and raising the aspirator bottle so that
salt water reaches up to the 100 cc mark. The apparatus is now ready for taking readings of flue gases.

1.19   ❐   BOMB CALORIMETER


1.19.1 Construction
This is used to determine the higher calorific value of solid and liquid fuels. The bomb calorimeter
shown in Fig. 1.2 consists of a strong steel shell, called the bomb, which can withstand a pressure of
about 100 atm. The electric supply is provided at the bottom of the bomb. The silica or quartz cruci-
ble is supported on two pillars. It is also provided with a non-return and release valves. The bomb is
placed in water bath and the water bath itself is placed in another container which acts as heat insula-
tor. A thermometer and a stirrer are inserted in the outer vessel.

1.19.2 Working
A known quantity of fuel is taken in the crucible such that the fuse touches the fuel. The bomb is then
charged with oxygen to a pressure of 30 bar. The oxygen charging non-return valve and release valve
are closed tightly and the bomb is placed in water bath and closed with the cover. The water in the
calorimeter is stirred and when the temperature remains steady, the fuel is ignited by passing a current
through the fuse wire. The temperature of the bath starts increasing very quickly after the ignition and
the readings on the thermometer are taken at regular intervals till the maximum temperature is reached.

M01_THERMAL ENGINEERING_SE_XXXX_CH01(part-1).indd 17 6/15/2018 12:53:12 PM


18 Chapter 1

Thermometer Pulley
Release Copper- Oxygen
valve calorimeter supply
valve

Stirrer
Fuse wire

Sleeves

Bomb

Water

Crucible

Electric wiring leads To supply

Rheostat

Figure 1.2  Bomb calorimeter

The temperature gradually starts falling. When the temperature fall shows a steady rate, the read-
ings are taken again for another five minutes to determine the cooling correction, as shown in
Fig. 1.3. The cooling correction should be added to the measured temperature rise.
Temperature, q

qm
Equal
areas

Time, t

Figure 1.3  Temperature-time curve

Considering heat balance, we have


Heat given by the fuel due to combustion + Heat given by the combustion of fuse wire
= Heat absorbed by the water and calorimeter.
mf × HCV + mfw × C.V. = (mw + ma)∆q cp
where mf = mass of fuel

M01_THERMAL ENGINEERING_SE_XXXX_CH01(part-1).indd 18 6/15/2018 12:53:14 PM


Fuels and Combustion 19

mfw = mass of fuse wire


mc = water equivalent of calorimeter
mw = mass of water in calorimeter
cp = specific heat of water
qm = recorded temperature rise
HCV = higher calorific value of fuel
C.V. = calorific value of fuse wire
∆q = true temperature rise

( mw + mc ) ∆q − m fw × C.V.
∴ HCV =  (1.12)
mf

The water equivalent of calorimeter is determined by burning a fuel of known calorific value
and using the aforementioned equation. The fuels used for this purpose are benzoic acid
(C.V. = 111573 kJ/kg) and naphthalene (C.V. = 40690 kJ/kg).

1.19.3  Cooling Correction


The radiation correction is computed from Newton’s law of cooling. It states that the rate of heat loss
due to radiation is proportional to temperature difference between the hot body and the surroundings.
Thus,
d Q ∝ (q 2 − q1 )dt
The rate of temperature drop is determined from the temperature-time graph for the bomb calorimeter.
Let it be r°C after the maximum temperature is reached. Therefore, during the period the temperature
r
rises, it can be assumed that °C fall in temperature occurs due to radiation. If t is the time required
2
for the maximum temperature to be reached, the temperature fall during that period due to radiation is
r
× t . Thus, for temperature fall,
2
r
True temperature rise = × t + recorded temperature rise (qm)  (1.13)
2

1.20   ❐   BOYS GAS CALORIMETER


1.20.1 Construction
It is used to determine the higher calorific value of gaseous fuels. Figure 1.4 shows the components of
the calorimeter. It consists of a gas burner B with arrangements to measure flow quantity and pressure
of gas supplied to burner. A gas pressure regulator is used in the gas supply line to control gas flow and
pressure fluctuation. The fuel burns within a cylindrical container surrounded by a cooling coil. Cool-
ing water is supplied to the cooling coil from a constant heat tank. The outer casing of the calorimeter
is provided with heavy insulation to prevent any heat loss to the surroundings.

M01_THERMAL ENGINEERING_SE_XXXX_CH01(part-1).indd 19 6/15/2018 12:53:16 PM


20 Chapter 1

Outer Cooling
casing water out

Cooling coil

Condensate Flowmeter
Condensate Thermometer
overflow
Cooling
water in
Burned
Gas supply
gases Air Air hw

Manometer

Figure 1.4  Boys gas calorimeter


1.20.2 Working
The flue gas moves up the cylindrical container and flows down from the top of the container. The
steam formed due to the combustion of hydrogen in the fuel and carried with the gases condenses
around the cooling coil and drips down into the trap below. The overflow of the condensate is taken
out in a glass beaker.

Calculations
Let Vg′ = volume of gas consumed, m3
mw = mass of water circulated, kg
( ∆q )w = rise in temperature of water circulated, °C
hw = pressure of gas above atmosphere, cm of water
hb = barometer reading, cm of Hg
qg = temperature of gas supply, °C
Volume of gas at normal temperature and pressure (NTP),
 hw 
273  hb + 13.6 
Vg = Vg′ × ×  
( 273 + q g )  76 
 
The calorific value of fuel is given by
Vg × HCV = mw × ( ∆q )w
mw × ( ∆q )w
or HCV = kJ/m3 at N.T.P (1.14)
Vg

Example 1.1
A coal has the following composition by mass:
C = 0.89, H2 = 0.03, S = 0.01, O2 = 0.02, and N2 = 0.03
and remaining is ash. Calculate the HCV and LCV of coal.

M01_THERMAL ENGINEERING_SE_XXXX_CH01(part-1).indd 20 6/15/2018 12:53:18 PM


Fuels and Combustion 21

Solution
 O
HCV = 35000 C + 143000  H −  + 9160 S
 8
 0.02 
= 35000 × 0.89 + 143000  0.03 −  + 9160 × 0.01
 8 
= 35174 kJ/kg
LCV  = HCV – 9H × 2460 = 35174 – 9 × 0.03 × 2460
= 34510 kJ/kg

Example 1.2
The following observations were made to find the HCV and LCV of a solid fuel by using a bomb
calorimeter:
Mass of fuel in crucible, mf    = 0.8 gram
Mass of fuse wire, mfw    = 0.02 gram
CV of fuse wire    = 6500 kJ/kg
Mass of water in calorimeter, mw     = 1.90 kg
Water equivalent of calorimeter, mc = 0.35 kg
Observed temperature rise, ∆q    = 2.97°C
Cooling correction     = 0.03°C
The fuel contains 90% C and 4% H2, the rest being ash.

Solution
Corrected temperature rise, ∆q c = 2.97 + 0.03 = 3°C
mf × HCV + mfw × C.V. = cpw (mw + mc). ∆q c

or 0.8 × 10–3 × HCV + 0.02 × 10–3 × 6500 = 4.2 (1.90 + 0.35) × 3


or HCV = 35275 kJ/kg
LCV  = HCV – 9H × 2460 = 35275 – 9 × 0.04 × 2460
      = 34389.4 kJ/kg

Example 1.3
The following observations were made by using a Boys gas calorimeter to determine the LCV of
a gaseous fuel:
Gas burned = 50 litres
Gas pressure = 4.5 cm of water above atmospheric pressure
Barometer reading = 750 mm of Hg
Temperature of gas = 30°C
Water circulated through the calorimeter = 19 kg
Rise in temperature of water = 10°C
Condensate collected during the test = 60 gram.
Determine the HCV and LCV of the gas at 0°C and 760 mm of mercury pressure.

M01_THERMAL ENGINEERING_SE_XXXX_CH01(part-1).indd 21 6/15/2018 12:53:20 PM


22 Chapter 1

Solution
273 1  h 
Volume at N.T.P., Vg = Vg′ × ×  hb + w 
Tg 76  13.6 

273 1  4.5 
= 50 × 10–3 × ×  75 +  = 0.04465 m
3
303 76  13.6 
C pw ⋅ mw ⋅ ∆t 4.2 × 19 × 10
HCV = = = 17872 kJ/m3 at N.T.P.
Vg 0.04465
Condensate formed per m3 of gas used at N.T.P.
50 × 10 −3
= = 1.1198 kg
0.04465
LCV = HCV –1.1198 × 2460 = 15117 kJ/m3

Example 1.4
The percentage composition by mass of a solid fuel is as follows: C = 90%, H2 = 3.5%, O2 = 3%,
N2 = 1%, S = 1%, and the remainder being ash. Find (a) the mass of air required per kg of fuel of
complete combustion, (b) the mass analysis of the dry products of combustion, (c) If 60% excess air
is supplied in actual combustion, determine the mass of flue gases per kg of fuel, and (d) Volumetric
analysis of the products per kg of fuel.

Solution
(a)
Constituent Mass Mass of O2 required Mass of products formed, kg
fraction for complete
combustion N2 CO2 SO2

8 11
C 0.9 0.9 × = 2.4 0.9 × = 3.3
3 3
H2 0.035 0.035 × 8 = 0.28
S 0.01 0.01 × 1 = 0.01 0.01 × 2 = 0.02
Total Σ = 2.69 Σ = 3.3 Σ = 0.02
O2 0.03 0.03
N2 0.01 0.01

Total O2 required for complete combustion = 2.69 – 0.03 = 2.66 kg/kg of fuel
100
Air required = 2.66 × = 11.58 kg/kg of fuel
23
(b) Mass analysis of dry products of combustion:
N2 = 11.58 × 0.77 + 0.01 = 8.92 kg

M01_THERMAL ENGINEERING_SE_XXXX_CH01(part-1).indd 22 6/15/2018 12:53:22 PM


Fuels and Combustion 23

Constituent Mass % by mass


CO2 3.3 3.3
× 100 = 26.97
12.24
SO2 0.02 0.02
× 100 = 0.164
12.24
N2 8.92 8.92
× 100 = 72.86
12.24
Σ = 12.24 kg

(c) For 60% excess air,


N2 = 11.58 × 1.6 × 0.77 + 0.01 = 14.276 kg
O2 = 11.58 × 0.6 × 0.23 = 1.598 kg

Constituent Mass kg % by mass


CO2 3.3 17.193
SO2 0.02 0.104
N2 14.276 74.377
O2 1.598 8.326
Σ = 19.194 kg
(d) Volumetric analysis

Constituent % by mass Molecular Proportional Percentage by


(1) (2) weight volume volume
(3) (2) (4)
(4) = (5) =
(3) Σ (4)
CO2 17.193 44 0.391 11.817
SO2 0.104 64 0.001625 0.049
N2 74.377 28 2.656 80.276
O2 8.326 32 0.260 7.858
Σ = 3.3086 Σ = 100

Example 1.5
The ultimate analysis of a coal is as follows:
C = 82%, H2 = 5%, O2 = 8%, and the rest is ash.
The volumetric analysis of dry flue gases is:
CO2 = 10%, CO = 2%, O2 = 8%, and N2 = 80%.
Find (a) air supplied per kg of fuel, (b) flue gases formed per kg of fuel, (c) percentage of excess
air supplied, (d) heat carried away by the products of combustion per kg of fuel if exit temperature
of flue gases is 380°C, and (e) heat lost by incomplete combustion. Assume C.V. of carbon = 35000
kJ/kg when it burns to CO2 and 10200 kJ/kg when it burns to CO. Air contains 23% of oxygen by
mass. cp for dry flue gas = 1 kJ/kgK and for steam, cp = 2.10 kJ/kgK.

M01_THERMAL ENGINEERING_SE_XXXX_CH01(part-1).indd 23 6/15/2018 12:53:24 PM


24 Chapter 1

Solution
(a) Mass of carbon per kg flue gas = mass of C from CO2 + mass of C from CO
3 3
= × 0.147 + × 0.0187
11 7
= 0.0481 kg/kg of flue gas
Carbon in 1 kg of fuel = 0.82 kg
0.82
Mass of dry flue gases formed per kg of fuel, mg = =17.048 kg
0.0481
Water formed by 0.05 kg of H2 per kg of fuel = 0.05 × 9 × 0.45 kg
Mass of combustible matter per kg of fuel + air supplied per kg of fuel = mass of products
0.82 + 0.05 + 0.08 + ma = 17.048 + 0.45
CN 0.82 × .80
or   ma = = = 16.55/kg fuel
0.33(C1 + C2 ) 0.33(0.10 + 0.02)

(b)
Constituent Volume Molecular Proportional Mass fraction
(1) fraction weight mass (5) = (4)
(2) (3) (4) = (2) ë (3) Σ (4)
CO2 0.10 44 4.4 0.1470
CO 0.02 28 0.56 0.0187
O2 0.08 32 2.56 0.0856
N2 0.80 28 22.4 0.7487
Σ = 29.92 Σ = 29.92
(c) Theoretical air required for complete combustion of fuel
100  8  O 
=  C + 8  H −  + S
23  3  8 
100  8  0.08  
= × 0.82 + 8  0.05 −  + 0  = 10.9 kg/kg of fuel
23  3  8  
Excess air supplied = 16.55 – 10.9 = 5.65 kg of fuel
5.65
Percentage of excess air = × 100 = 51.83%
10.9
(d) Heat carried by flue gasses = Heat carried by dry flue gases + Heat carried by steam formed
= (1 × 17.048 × 380) + [2460 + 2.1 × (380 –15) × 0.5183]
= 9335.5 kJ/kg of fuel burned
C1 0.02
(e) Carbon in fuel burned to CO = ×C= × 0.82 = 0.1367 kg
C2 + C1 0.10 + 0.02
The burning of carbon to CO instead of CO2 causes a loss of (35000 – 10200)
= 24800 kJ/kg of C burned to CO
Heat loss = 24800 × 0.1367 = 3389.3 kJ/kg of fuel

M01_THERMAL ENGINEERING_SE_XXXX_CH01(part-1).indd 24 6/15/2018 12:53:27 PM


Fuels and Combustion 25

Example 1.6
The flue gas analysis by volume for burning of C6H14 is as follows:
CO2 = 8.5%, CO = 7.8%, and N2 = 83.7%
Calculate (a) the air-fuel ratio required for chemically correct combustion, and (b) the air-fuel
(A:F) ratio as a percentage of the chemically correct mixture A:F ratio.

Solution
2C6H14 + 19O2 = 12CO2 + 14H2O
2 × 86 + 19 × 32 = 12 × 44 + 14 × 18
For complete combustion of 1 kg of fuel, oxygen required
19 × 32
= = 3.535 kg
2 × 86
3.535 × 100
or = 15.37 kg of air
23
\ A:F ratio for complete combustion = 15.37: 1
72
The percentage of carbon in the fuel  = × 100 = 83.72%
86
Mass of air supplied per kg of fuel burned,
CN 83.72 × 83.7
ma = = = 13.03 kg
33(C1 + C2 ) 33(7.8 + 8.5)
\ A:F ratio of the mixture supplied = 13.03:1
A:F ratio of mixture 13.03
= = 0.847
A:F ratio of chemically correct mixture 15.37

Example 1.7
The volumetric analysis of a sample of producer gas is as follows:
H2 = 20%, CH4 = 3%, CO = 22%, CO2 = 8%, and N2 = 47%.
(a) Find the volume of air required for complete combustion of 1 m3 of the gas.
(b) If 50% excess air is supplied, find the percentage contraction in volume after the products of
combustion have been cooled.
Solution
(a) Minimum volume of air required for complete combustion of 1 m3 of fuel
1  H CO  1  20 22 
Va =  + + 2CH 4  =  + + 2 × 3
21  2 2  21  2 2 
= 1.29 m3/m3 of gas
(b) The constituents formed with 50% excess air are:
CO2 = 0.08 + CO2 formed from CO + CO2 formed from CH4
= 0.08 + 0.22 × 1 + 0.03 × 1 = 0.33 m3
    N2 = 0.47 + N2 from the air supplied
 = 0.47 + 1.29 × 1.5 × 0.79 = 2 m3

M01_THERMAL ENGINEERING_SE_XXXX_CH01(part-1).indd 25 6/15/2018 12:53:30 PM


26 Chapter 1

  O2 = O2 from the excess air only


 = (1.29 × 0.5) × 0.21 = 0.135 m3
Total dry flue gases formed = CO2 + N2 + O2 = 0.33 + 2.00 + 0.135 = 2.465 m3
Total volume before combustion = volume of gas + volume of air supplied
     = 1 + 1.29 × 1.5 = 2.935 m3
Percentage contraction in volume after combustion and cooling
 2.935 − 2.465 
=  × 100 = 16%
 2.935

Example 1.8
A sample of coal has the following percentage composition by weights:
C = 82%, H2 = 10%, O2 = 6%, N2 = 1%, and S = 1%.
Calculate the minimum quantity of air required, for the complete combustion of 1 kg of coal. If
20% excess air is used in the combustion, what is the air-fuel ratio?
Solution
Minimum air per kg of fuel
100  8  O 
=  C + 8  H −  + S
23  3  8 
100  8  0.06  
=  × 0.82 + 8  0.1 −  + 0.01 = 12.765 kg/kg of fuel
23  3  8  
With 20% excess air,
A:F = 12.765 × 1.2 = 15.318 kg/kg of fuel
Example 1.9
A fuel gas has composition by volume: H2 = 50%, CO = 40%, CO2 = 6% and N2 = 4%. (a) Calcu-
late the volume of air required for complete combustion of 1 m3 of the gas, (b) If the dry exhaust
gas from an engine using this fuel contains 9.2% by volume of CO2 and no CO, calculate the air to
gas ratio which has been used. Calculate the percentage of O2 by volume in the dry exhaust.

Solution
(a) For complete combustion of fuel gas, the chemical equation is:
 79   79 
50H2 + 40CO + 6CO2 + xO2 +  x + 4 N 2 → aCO2 + bH2O +  x + 4 N2
 21   21 
x
where = oxygen – fuel ratio by volume.
100
Comparing coefficients of H2, C and O2 on both sides, we get
b = 50, a = 40 + 6 = 46
40 b
+ 6 + x = a + = 46 + 25 = 71
2 2
x = 45 m3 of O2 per 100 m3 of fuel gas.
45 100
A:F = × = 2.14 m3 of air/m3 of gas.
100 21

M01_THERMAL ENGINEERING_SE_XXXX_CH01(part-1).indd 26 6/15/2018 12:53:33 PM


Fuels and Combustion 27

 79   79 
(b) 50H2 + 40CO + 6CO2 + yO2 +  y + 4 N2 → dCO2 + 50H2O + eO2 +  y + 4 N2
 21   21 
y
where = new oxygen – fuel ratio by volume
100
C balance: d = 40 + 6 = 46
46
CO2 balance: 0.092 =
79
46 + e + y+4
21
40
O2 balance: + 6 + y = 46 + e
2
Solving for y, we get y = 104 and e = 59
104 100
Air to gas ratio by volume = × = 4.95
100 21
59 × 100 59
% volume of O2 in dry gas = = × 100 = 11.8%
79 500
46 + 59 + × 104 + 4
21
Example 1.10
The composition of a hydrocarbon fuel by mass is: C = 85%, H2 = 13%, and O2 = 2%. Calculate
(a) the chemically correct mass of air required for the complete combustion of 1 kg of fuel, (b) the
volumetric and gravimetric analysis of the products of combustion, and (c) dew point temperature
if the total pressure is 1 bar and (d) If the products of combustion are cooled to 15°C, calculate the
mass of water vapour condensed.
Solution
C + O2 → CO2
1
H2 + O2 → H2O
2
0.85 0.13 0.02
Moles of O2 required = + + = 0.103958 moles of O2/kg of fuel
12 2 × 2 32
100
(a) Mass of air required = 0.103958 × × 29 = 14.356 kg of air/kg fuel
21
(b) The volumetric and gravimetric analysis of products of combustion:
Constituent Moles per kg Molal analysis kg per kg of Gravimetric
of fuel (a) fuel analysis %
(b) =
(a) Σ (a) (c) (c)
(d) = ë 100
Σ (c)
CO2 0.85 0.1344 or 13.44% 44 × 0.85 20.46
= 0.07083 = 3.117
12 12
0.13 18 × 0.13
H2O = 0.065 0.1234 or 12.34% = 1.17 7.68
2 2
79
N2 × 0.103958 = 0.391 0.7422 or 74.22% 0.391 × 28 = 10.948 71.86
21
S (a) = 0.52683 S (b) = 1.00 S (c) = 15.235 S (d) = 100.00

M01_THERMAL ENGINEERING_SE_XXXX_CH01(part-1).indd 27 6/15/2018 12:53:39 PM


28 Chapter 1

nH 2O
(c) Partial pressure of H2O, pH O = × p = 0.1234 × 1 = 0.1234 bar
2
Σn
Dew point is the saturation temperature corresponding to 0.1234 bar. From steam tables,
tdp = 49.97°C.
(d) Temperature of products of combustion on cooling (15°C) is less than tdp. Hence, some water
vapour will condense. The vapour pressure will then be equal to the saturation vapour pressure
corresponding to 15°C.
From steam tables, ps = 0.017051 bar and vg = 77.926 m3/kg.
The volume occupied by the water vapour is equal to the volume occupied by CO2 and N2 at their
partial pressure of (1 – 0.017051) = 0.982949 bar.
nRT (0.07083 + 0.391) × 8.314 × 288
Volume, V = = = 11.25 m3
2
p 0.982949 × 10
V 11.25
Mass of vapour in combustion products = = = 0.14437 kg
u g 77.926
Mass of condensed vapour = 1.17 – 0.14437 = 1.0256 kg

Example 1.11
Octane, C8H18’ is burned with dry air. The molar analysis of the dry products is CO2 = 10%; CO
= 0.4%; O2 = 3%, and N2 = 86.6%. Determine the air fuel ratio on a mass basis and the dew point
temperature of the products if the products are at 1 atm pressure. Dry air contains 3.76 moles of
nitrogen per mole of oxygen. [IES, 1994]

Solution
Constituent Percentage Molecular Proportional Percentage by
by volume weight mass mass
(a) (b) (c) = (a) ë (b) (c)
(d) =
Σ (c)
CO2 10 44 440.0 14.805
CO 0.4 28 11.2 0.377
O2 3.0 32 96.0 3.230
N2 86.6 28 2424.8 81.588
Σ(c) = 2972.0 Σ(d) = 100.00
Mass of C per kg of dry flue gas = Mass of C from CO2 + Mass of C from CO
12 12
= × 0.14805 + × 0.00377 = 0.042 kg
44 28
96
Percentage of C in fuel = × 100 = 84.21%
114
84.21
Mass of dry flue gases formed per kg of fuel = = 20.05 kg
100 × 0.042

M01_THERMAL ENGINEERING_SE_XXXX_CH01(part-1).indd 28 6/15/2018 12:53:42 PM


Fuels and Combustion 29

NC
Mass of air supplied per kg of fuel =
33 (CO 2 + CO)
86.6 × 84.21
= = 21.25 kg
33(10 + 0.4)
Air/Fuel ratio = 21.25:1.
The dew point temperature of products is the saturation temperature at the partial pressure of water
vapour.
C8H18 + 12.5O2 + 12.5 (3.76) N2 → 8CO2 + 9H2O + 47N2
12.5 moles of O2 are required to burn 1 mole C8H18 and 8 moles of H2O are produced.
Stoichiometric mole fraction of water vapour ( xH 2O ) formed,
nH 2O 9 9
xH 2 O = = = = 0.1636
nCO2 + nCO + nO2 + nN 2 8 + 0 + 0 + 47 55
pH 2O = xH 2O atm = 0.1636 bar
The saturation temperature of water vapour corresponding to their partial pressure of 0.1636 bar is,
ts = 54°C.

Example 1.12
An oil fired boiler uses a fuel which contains 84% carbon and 16% hydrogen by weight. The vol-
umetric analysis of the dry flue gases was found to be CO2 = 9%, CO = 2%, O2 = 5%, and N2 =
84%. Assuming complete combustion of the hydrogen in the fuel, determine the quantity of carbon
remaining unburnt in each kg of the fuel fired in the furnace. Determine the actual air-fuel ratio and
the excess air factor. [IES, 1983]

Solution
C + O2 = CO2
12 + 32 = 44
32
0.84 kg of C requires × 0.84 = 2.24 kg of O2
12
2H2 + O2 = 2H2O
4 + 32 = 36
32
0.16 kg of H2 requires × 0.16 = 1.28 kg of O2
4
Total O2 required for complete combustion of 1 kg of fuel = 2.24 + 1.28 = 3.52 kg
Amount of air required for complete combustion of 1 kg of fuel
100
= 3.52 × = 15.3 kg
Air/fuel ratio = 15.3:1 23

M01_THERMAL ENGINEERING_SE_XXXX_CH01(part-1).indd 29 6/15/2018 12:53:46 PM


30 Chapter 1

Mass analysis of dry flue gases:

Constituent % by volume Molecular weight Proportional % by mass


(a) (b) mass (d) = (c)
(c) = (a) ë (b) Σ (c)
CO2 9 44 396 13.36
CO 2 28 56 1.89
O2 5 32 160 5.40
N2 84 28 2352 79.35
Σ(c) = 2964
Mass of C per kg of dry flue gas
12 12
= × 0.1336 + × 0.0189 = 0.0445 kg
44 28
Mass of C per kg of fuel = 0.84 kg
0.84
Mass of dry flue gases per kg of fuel = = 18.876 kg
0.0445
Mass of N2 per kg of fuel = 18.876 × 0.7935 = 14.978 kg
14.978
Mass of air required per kg of fuel = = 19.45 kg
0.77
Excess air supplied = 19.45 – 15.3 = 4.15 kg
4.15
Excess air factor = = 0.2714 or 27.14%
15.3

Example 1.13
The composition of coal burnt during a boiler trial was as follows:
C = 62%, H2 = 3.6%, O2 = 7.4%, and ash = 27%.
The volumetric analysis of the fuel gas showed the following:
CO2 = 10.25%, CO = 0.80%, O2 = 8.54%, and N2 = 80.41%.
Determine (a) the air-fuel ratio
(b) the percentage of excess air, and
(c) the mass of CO2, CO, O2, and N2 in the flue gases per kg of coal burnt. [IES, 1986]
Solution
Conversion of volumetric analysis to mass analysis.

Constituent Percentage by Molecular Proportional mass Percentage of mass


volume weight (c) = (a) ë (b) (d) = (c) ë 100
(a) (b) Σ (c)
CO2 10.25 44 451.0 15.04
CO 0.80 28 22.4 0.75
O2 8.54 32 273.3 9.12
N2 80.41 28 2251.5 75.09
Σ(c) = 2998.0

M01_THERMAL ENGINEERING_SE_XXXX_CH01(part-1).indd 30 6/15/2018 12:53:48 PM


Fuels and Combustion 31

(a) Mass of carbon per kg of dry flue gas


= mass of C in CO2 + mass of C in CO
3 3
= × 0.1504 + × 0.0075 = 0.0442 kg/kg of flue gas
11 7
Mass of dry flue gases formed per kg of fuel
0.62
= = 14.03 kg
0.0442
Mass of steam formed per kg of fuel = 0.036 × 9 = 0.324 kg
Mass of combustible matter + air supplied per kg of fuel = mass of products C + H2 + O2 + air
supplied = mass of products
0.62 + 0.036 + 0.074 + ma = 14.03 + 0.324
or ma = 13.624 kg/kg of fuel
Air/fuel ratio = 13.624:1.
1 8  O 
(b) Theoretical air required =  C + 8 H −  
23  3  8

1 8  7.4  
= × 62 + 8  3.6 − 
23  3  8  
= 8.12 kg/kg of fuel
Excess air supplied = 13.624 – 8.12 = 5.504 kg/kg of fuel
5.504
Percentage of excess air supplied = × 100 = 67.8%
8.12
(c) Flue gas analysis
Constituent % by mass kg/kg of fuel (b) = 14.03 ë (a)/100
(a)
CO2 15.04 2.1
CO 0.75 0.105
O2 9.12 1.280
N2 75.09 10.535

Example 1.14
The mass analysis of a hydrocarbon fuel is as follows:
C = 84%, H2 = 5% and the balance is incombustible material. Find (a) the mass of air required
per kg of fuel for complete combustion, (b) the analysis of the wet exhaust gases, by mass and
volume, if 20 kg fuel is supplied, (c) the partial pressure of the steam formed in the exhaust gases
if the total pressure of the exhaust gases is 103 kPa, (d) the heat carried away by dry exhaust gases
formed per kg of fuel if the temperature of exhaust gas is 375°C and the ambient temperature is
24°C. Take cp for dry gases is 1.005 kJ/kgK.  [IES, 1988]

M01_THERMAL ENGINEERING_SE_XXXX_CH01(part-1).indd 31 6/15/2018 12:53:50 PM


32 Chapter 1

Solution
(a) Oxygen required for complete combustion of 1 kg of fuel
32 32 32 32
= C × + H2 × = 0.84 × + 0.15 × 0.15 × = 3.44 kg
12 4 12 4
Air required per kg of fuel for complete combustion
100
= 3.44 × = 14.96 kg
23
(b) The products of combustion per kg of fuel are:
44
CO2 = 0.84 × = 3.08 kg
12
18
H2O = 0.15 × = 1.35 kg
2
N2 = 14.96 × 0.77 = 11.52 kg
Wet products

Constituent kg/kg of fuel Molecular Proportional Percentage by kg/20 kg of


W weight W volume fuel
volume
(1) M M (4) = (3) × 100 (5) = (1)
(2) (1) Σ3 × 20
(3) =
(2)
CO2 3.08 44 0.070 12.58 61.6
H2O 1.35 18 0.075 13.49 27.0
N2 11.52 28 0.411 73.93 230.4
Σ 3 = 0.556

(c) Partial pressure of steam formed in the exhaust gases


0.075
= 103 × = 13.89 kPa
0.556
(d) Mass of dry exhaust gases formed per kg of fuel
= (mass of CO2 + mass of N2) per kg of fuel
= 3.08 + 11.52 = 14.60 kg
Heat carried away by dry exhaust gases per kg of fuel burned
= 14.60 × 1.005 × (375 – 24) = 5150.223 kJ

Example 1.15
A sample of gaseous fuel has the following composition by volume:
H2 = 28%, CO = 10%, CH4 = 2%, CO2 = 16%, O2 = 2%, N2 = 42%.
If this fuel is burned with 50% excess air, determine (a) the volume of air per cubic metre of gas
at the same temperature and pressure, (b) the volumetric analysis of dry products, (c) the mass of
products per kg of fuel, (d) the mass of dry products per kg of fuel, and (e) the mass of air supplied
per kg of fuel.

M01_THERMAL ENGINEERING_SE_XXXX_CH01(part-1).indd 32 6/15/2018 12:53:53 PM


33
Fuels and Combustion

Solution
(a) Minimum volume of air required for complete combustion of 1 m3 of fuel,
1  Η 2 CO  1  28 10 
Va =  + + 2CH 4  =  + + 2 × 2
21  2 2  21  2 2 
= 1.095 m3/m3 of gaseous fuel
Air required with 50% excess air = 1.095 × 1.5 = 1.643 m3/m3 of gas
(b) The constituents formed with 50% excess air are:
CO2 = 0.16 + CO2 formed from CO + CO2 formed from CH4
= 0.16 + 0.10 × 1 + 0.02 × 1 = 0.28 m3
N2 = 0.42 + N2 from air supplied = 0.42 + 1.643 × 0.79 = 1.718 m3
O2 = 0.02 + O2 from excess air supplied = 0.02 + 1.095 × 0.5 × 0.21 = 0.135 m3
Total dry flue gases formed = CO2 + N2 + O2 = 0.28 + 1.718 + 0.135 = 2.133 m3
(c)
Constit- % of dry prod- Molecular Proportional % of dry products by
uent ucts by volume weight mass mass
(a) (b) (d) = (c)
Σ (c)

CO2 0.28 44 577.588 19.029


= 13.127
2.133

N2 1.718 28 2255.232 74.299


= 80.544
2.133
O2 0.135 32 202.528 6.672
= 6.329
2.133

Σ (a) = 100.000 Σ (c) = 3035.348 Σ (d) = 100.00

1   O 4 
(d) Mass of air supplied per kg of fuel = 8  H −  + CO + 4CH 4 
23   8 7 

Constitu- % by volume of Molecular Proportional % by mass


ent fuel weight mass (d) = (c)
(a) (b) (c) = (a) ë (b) Σ (c)
H2 28 2 56   2.422
CO 10 28 280 12.111
CH4 2 16 32   1.384
CO2 16 44 704 30.450
O1 2 32 64   2.768
N2 42 28 1176 50.865
Σ (c) = 2312 Σ (d) = 100.00

M01_THERMAL ENGINEERING_SE_XXXX_CH01(Part-II).indd 33 6/15/2018 12:58:49 PM


34 Chapter 1

1   2.768  4 
(e) Mass of air supplied = 8  2.422 −  + × 12.111 + 4 × 1384 
23   8 7 
= 0.6318 kg/kg of fuel
With 50% excess air, mass of air supplied = 0.6318 × 1.5 = 0.9478 kg/kg of fuel

Example 1.16
A hydrocarbon fuel when burned with air gave the following Orsat analysis:
CO2 = 11.94%, O2 = 2.26%, CO = 0.41%, N2 = 85.39%
Determine (a) the air-fuel ratio on mass basis, (b) the percent of carbon and hydrogen in the fuel
on mass basis, and (c) the percent of theoretical air supplied. Assume that air contains 21% oxygen
by volume. [IES, 1998]
Solution
Let 100x moles of air be used with fuel per 100 moles of dry exhaust products.
(aC + bH2)fuel + (21 × O2 + 79 × N2)air = 11.94 CO2 + 2.26 O2 + 0.41 CO + 85.39 N2 + dH2O
By molar balance, we have
Carbon: a = 11.94 + 0.41 = 12.35
Hydrogen: b = d
0.41 d
Oxygen: 21x = 11.94 + 2.26 + +
2 2
    = 14.405 + 0.5d
Nitrogen: 79x = 85.39 or x = 1.08
From the above equation, we get
21 × 1.081 = 14.405 + 0.5d
d = b = 16.592
100 x × molecular weight of air 100 × 1.081 × 29
(a) Air-fuel ratio = = = 17.28
aC + bH 2 12.35 × 12 + 16.592 × 2
aC × 100 12.35 × 12 × 100
(b) % Carbon in fuel = = = 81.7%
aC + bH 2 12.35 × 12 + 16.592 × 2
bH 2 16.592 × 2 × 100
% Hydrogen in fuel = = = 18.3%
aC + bH 2 12.35 × 12 + 16.592 × 2
100( 21 × O 2 + 79 × N 2 )
(c) % Theoretical air supplied = + (21 ×O2 + 79 × N2)
aC + bH 2
100 × 1.081 × ( 21 × 32 × +79 × 28)
 =
[(12.35 × 12 + 16.592 × 2) + ( 21 × 32 + 79 × 28) × 1.081]
= 94.5%

Example 1.17
The chemical formula for alcohol is C2H6O. Calculate the stoichiometric air/fuel ratio by mass and
the percentage composition of the products of combustion per kg of C2H6O.[GATE, 1998]

M01_THERMAL ENGINEERING_SE_XXXX_CH01(Part-II).indd 34 6/15/2018 12:58:52 PM


35
Fuels and Combustion

Solution
The chemical equation for complete combustion of given fuel can be written as follows:
C2H6O + 3O2 → 2CO2 + 3H2O
(1 × 46) + (3 × 32) = (2 × 44) + (3 ×18)
For complete combustion of 1 kg of C2H6O, oxygen required
3 × 32
= 2.087 kg of oxygen
46
100
= 2.087 × = 9.074 kg of air
23
\  A:F ratio for complete combustion = 9.074: 1
Also 46 kg of fuel produces products of combustion = 88 + 54 = 142 kg
142
\  1 kg of fuel produces = 3.087 kg of products of combustion
46
(i.e., CO2 and H2O)
88
Hence, CO2 produced by fuel = × 100 = 1.913 or 191.3%
46
54
H2O produced by fuel = × 100 = 1.174 or 117.4%
46
Example 1.18
The products of combustion of an unknown hydrocarbon CxHy have the following composition as
measured by an Orsat apparatus.
CO2 = 8.0%, CO = 0.9%, O2 = 8.8%, and N2 = 82.3%.
Determine (a) the composition of the fuel (b) the air-fuel ratio; and (c) the percentage of excess
air used.

Solution
(a) Let a moles of oxygen by supplied per mole of fuel. The chemical reaction can be written as follows:
 y
CxHy + aO2 + 3.76 aN2 → 8CO + 0.9CO2 + 8.8O2 + 82.3H2 +   H2O
 2
Nitrogen Balance gives 3.76a = 82.3
or a = 21.89
y
Oxygen Balance gives: 2a = 16 + 0.9 + 17.6 +
2
or      y = 18.5
Carbon balance gives
x = 8 + 0.9 = 8.9
Therefore, the chemical formula of the fuel is C8.9H18.5.
The composition of the fuel is
8.9 × 12
% Carbon = × 100 = 85.23%
8.9 × 12 + 18.5 × 1

M01_THERMAL ENGINEERING_SE_XXXX_CH01(Part-II).indd 35 6/15/2018 12:58:55 PM


36 Chapter 1

% Hydrogen = 14.77%
32a + 3.76 a × 28
(b) Air fuel ratio =
12 x + y
21.89(32 + 3.76 × 28)
= = 24
12 × 8.9 + 18.5
88 × 32
(c) % Excess air used= × 100 = 67.22%
2189 × 32 − 88 × 32

Example 1.19
The volumetric analysis of a gas is 14% CO2, 1% CO, 5% O2, and 80% N2. Calculate the fuel gas
composition by mass.
Solution
Given: CO2 = 14% = 0.14 m3; CO = 1% = 0.01 m3; O2 = 5% = 0.05 m3; N2 = 80% = 0.8 m3
The volumetric analysis may be converted into mass analysis by completing the table as follows:

Constituent Volume Molecular Proportional mass Mass in kg per kg % by mass


in 1 m3 of mass flue gas of flue gas = (d) ë 100
flue gas (b) (c) = (a + b) (d) = (c)
(a) Σ (c)
6.16
CO2 0.14 44 6.16 = 0.202 20.2%
30.14
0.28
CO 0.01 28 0.28 = 0.009 0.9%
30.44

O2 0.05 32 1.60 1.60 5.3%


= 0.053
30.44
22.40
N2 0.80 28 22.40 = 0.736 73.6%
30.44
Total 1.00 S(c) = 30.44 1.00 100.0
The fuel gas composition, by mass is given in the last column, i.e., CO2 = 20.2%, CO = 0.9%,
O2 = 5.3%, and N2 = 73.6%.

Example 1.20
A blast furnace gas has the following volumetric composition:
CO2 = 11%, CO = 27%, H2 = 2%, and N2 = 60%.
Find the theoretical volume of air required for the complete combustion of 1 m3 of the gas. Find
the percentage composition of dry flue gases by volume. Assume that air contains 21% of O2, and
79% of N2 by volume.

M01_THERMAL ENGINEERING_SE_XXXX_CH01(Part-II).indd 36 6/15/2018 12:59:00 PM



Fuels and Combustion 37

Solution
Given: CO2 = 11% = 0.11 m3, CO = 27% = 0.27 m3, H2 = 2% = 0.02% m3, N2 = 60% = 0.6 m3
We know that theoretical volume of air required
100
= = [(0.5CO + 0.5H2 + 2CH4 + 3C2H4) – O2] m3
21
100
= = [0.5 × 0.27 + 0.5 × 0.02] = 0.69 m3
21
(∴ CH4, C2H and O2 are equal to zero)
We know that 1 m3 CO produces 1 m3 of CO2, therefore
Volume of CO2 obtained from the given 0.27 m3 of CO = 0.27 m3
and volume of CO2 already present in the fuel = 0.11 m3 (Given)
∴  Total volume of CO2 in the flue gas = 0.11 + 0.27 = 0.38 m3
79
Now volume of N2 from the theoretical air supplied = × 0.69 = 0.545 m3
100
and volume of N2 already present in the fuel = 0.6 m3
∴  Total volume of N2 in the flue gas = 0.6 + 0.545 = 1.145 m3
and volume of the dry flue gas = Total volume of CO2 + Total volume of N2
= 0.38 + 1.145 = 1.525 m3
0.38
∴  Percentage of CO2 in the dry flue gas = × 100 = 24.92%
1.525
1.145
and percentage of N2 in the dry flue gas = × 100 = 75.08%
1.525

Example 1.21
The percentage composition by mass of a sample of coal as found by analysis is given as: C = 90%,
H2 = 3.3%, O2 = 3.0%, N2 = 0.8%, S = 0.9%, and ash 2.0%.
Calculate the minimum mass of air required for the complete combustion of 1 kg of this fuel. If
50% excess air is supplied, find the total mass of dry flue gases per kg of fuel and the percentage
composition of the dry flue gases by volume.

Solution
Given: C = 90% = 0.9 kg; H2 = 3.3% = 0.033 kg; O2 = 3% = 0.03 kg; N = 0.8% = 0.008 kg; S = 0.9%
= 0.009 kg; Ash = 2% = 0.02 kg; Excess air supplied = 50%
We know that minimum mass of air required for complete combustion of 1 kg of fuel.
100  8  
=  C + 8H 2 + S − O 2  kg
23  3 
100  8  
=   × 0.9 + 8 × 0.033 + 0.009 − 0.03 = 11.5 kg
23  3 
Total mass of dry flue gases per kg of fuel
Since 50% excess air is supplied, therefore actual amount of air supplied per kg of coal = 11.5 ×
1.5 = 17.25 kg
∴  Excess air supplied = 17.25 – 11.5 = 5.75 kg

M01_THERMAL ENGINEERING_SE_XXXX_CH01(Part-II).indd 37 6/15/2018 12:59:02 PM


38 Chapter 1

The products of combustion are represented by the following chemical equations:


C + O2 = CO2
2H2 + O2 = 2H2O
S + O2 = SO2
In addition to carbon dioxide, water, and sulphur dioxide, the excess oxygen and nitrogen will be
available in the products of combustion. It may be noted that H2O (water vapour) is a wet gas, there-
fore, the dry flue gases are only carbon dioxide, sulphur dioxide, excess oxygen, and nitrogen. Let us
now find the mass of each of these flue gases per kg of fuel.
We know that 1 kg of carbon produces 11/3 kg of carbon dioxide and 1 kg of sulphur produces
2 kg of sulphur dioxide.
11
∴  Mass of CO2 contained in 0.9 kg of carbon per kg of fuel = × 0.9 = 3.3 kg(i)
3
and mass of SO2 in 0.009 kg of sulphur per kg of fuel = 2 × 0.009 = 0.018 kg(ii)
23
We also know that the mass of excess O2 per kg of fuel = × Excess air supplied
100
23
= × 5.75 = 1.323 kg
100
and mass of nitrogen in the products of combustion per kg of fuel
77 77
= × Actual air supplied = × 17.25 = 13.283 kg
100 100
∴  Total mass of dry flue gases per kg of fuel = 3.3 + 0.018 + 1.323 + 13.283 kg
= 17.924 kg
First, let us find out the percentage composition of dry flue gases from the aforementioned data by
mass. We know the following composition:
3.3
 CO2 = = 0.184 = 18.4%
17.924
0.018
SO2 = = 0.001 = 0.1%
17.924
1.323
Excess O2 = = 0.074 = 7.4%
17.924
13.283
N2 = = 0.741 = 74.1%
17.924
Now, let us convert this mass analysis of dry flue gases into volumetric analysis as follows:
Constituent % Mass Molecu- Proportional volume Volume in 1 m3 % Volumetric
analysis lar mass (a) of flue gas analysis
(c) =
(a) (b) (b) (c) (d) ë 100
(d) =
Σ (c)

18.4 0.418
CO2 18.4 44 = 0.418 = 0.1268 12.68
44 3.297

0.1 0.0012
SO2 0.1 64 = 0.0012 = 0.0006 0.06
64 3.297

(Continued)

M01_THERMAL ENGINEERING_SE_XXXX_CH01(Part-II).indd 38 6/15/2018 12:59:06 PM


39
Fuels and Combustion

(Continued )

Constituent % Mass Molecu- Proportional volume Volume in 1 m3 % Volumetric


analysis lar mass (a) of flue gas analysis
(c) =
(a) (b) (b) (c) (d) ë 100
(d) =
Σ (c)
7.4 0.231
Excess O2 7.4 32 = 0.231 = 0.0701 7.01
32 3.297

74.1 2.646
N2 74.1 28 = 2.646 = 0.8025 80.25
28 3.297

Total 100.0 Σ(c) = 3.297 100.00

 he percentage composition of dry flue gases (by volume) is given in last column, that is,
T
CO2 = 12.68%, SO2 = 0.06%; Excess O2 = 7.01%; and N2 = 80.25%.
C = 85%, H2 = 12.5%, O2 = 2% and the residue 0.5%.

Example 1.22
A liquid fuel C7H16 is burned with 10% more air than the stoichiometric air. Assuming complete
combustion, calculate (a) the mass of air supplied per kg of fuel and (b) the volumetric analysis of
the dry products of combustion. Assume air contains 23% O2 by mass. [IES, 2009]

Solution
(a) The chemical equation of combustion for the given fuel is:
C7H16 + 11O2 = 7CO2 + 8H2O
100 kg + 11 × 32 kg = 308 kg + 144 kg
11 × 32
1 kg of fuel requires = kg of O2
100
11 × 32 100
= × = 15.3 kg of air/kg of fuel
100 23
When 10% more air than the stoichiometric air is burned, then actual mass of air supplied = 1.1 ×
15.3 = 16.83 kg of air/kg of fuel
308
Mass of CO2 formed = = 3.08 kg/kg of fuel
100
O2 comes in exhaust from complete air supplied,
77
N2 = 16.83 × = 12.96 kg/kg of fuel
100
O2 comes in exhaust from only excess air supplied,
23
O2 = 15.3 × 0.1 × = 0.352 kg
100

M01_THERMAL ENGINEERING_SE_XXXX_CH01(Part-II).indd 39 6/15/2018 12:59:10 PM


40 Chapter 1

(b) The mass is converted into volume as listed below in the table:

Products of Mass kg Molecular weight Proportional Percentage by


combustion (a) M volume volume
(b) (a) (c)
(c) = (d) = ë 100
(b) Σ (c)
N2 12.96 28 0.463 85.11
CO2 3.08 44 0.070 12.87
O2 0.352 32 0.011 2.02
Total Σ(c) = 0.544 Σ(d) = 100.00

Example 1.23
A four-cylinder engine of an automobile is converted to run on propane (C3H8) fuel. A dry analysis
of engine exhaust gives volumetric percentage of CO, CO2 and O2, respectively at 9.79%, 4.90%
and 2.45%. Write the resulting chemical reaction and find the equivalence ratio. [IES, 2010]

Solution
Given: Volumetric analysis: CO = 9.7%, CO2 = 4.90%, O2 = 2.45%
The volumetric analysis of exhaust analysis can be converted to mass analysis as follow:

Constituent Percentage by Molecular weight Proportional mass Percentage by


volume (b) (c) = (a) ë (b) mass
(a) (c)
(d) = ë 100
Σ (c)
CO 9.79 28 274.12 9.49
CO2 4.90 44 215.6 7.46
O2 2.45 32 78.4 2.71
N2 (by difference) 82.86 28 2320.08 80.34
Total Σ(c) = 2888.2 Σ(d) = 100.00

3 × 12 36
Percentage of C in C3H8 = = = 0.8182 or 81.82%
3 × 12 + 8 × 1 44
Percentage of H2 in C3H8 = 100 – 81.82 = 18.18%
Mass of air supplied per kg of fuel,
NC 82.86 × 81.82
ma = = = 13.98 kg
33 (CO + CO 2 ) 33(9.79 + 4.90)
A:F = 13.98:1
C3H8 + 5O2 = 3CO2 + 4H2O
(3 × 12 + 8 × 1) + 5 × 2 × 16
44 + 160

M01_THERMAL ENGINEERING_SE_XXXX_CH01(Part-II).indd 40 6/15/2018 12:59:12 PM


41
Fuels and Combustion

Oxygen required for complete combustion of 1 kg of fuel


160
= = 3.636 kg/kg of fuel
44
3.636 × 100
= = 15.81 kg of air
23
Minimum air required for complete combustion of fuel
1 8 
= C + 8H 
23  3 
1 8 
= × 81.82 + 8 × 18.18 = 15.81 kg/kg of fuel
23  3 
Stoichiometric A/F ratio 15.81
Equivalence ratio, p = = = 1.131
Actual A/F ratio 13.98

Example 1.24
Coal, having the following composition, is burnt in a furnace:
Carbon = 86.1%, hydrogen = 3.9%, oxygen = 1.4%, rest ash.
The volumetric analysis of dry products was as follows:
CO2 = 12.7%, CO = 1.4%, O2 = 4.1%, N2 = 81.8%. Determine the percentage of excess air.
Determine the loss due to incomplete combustion, if CO burning to CO2 will release 24000 kJ/kg
of carbon in CO. [IAS, 1998]

Solution
Given: Composition of coal: C = 86.1%, H2 = 3.9%, O2 = 1.4%, Rest = 8.6% ash
Volumetric analysis of dry products:
CO2 = 12.7%, CO = 1.4%, O2 = 4.1%, N2 = 81.8%.

2CO + O2 → 2CO2 + 24,000 kJ/kg of C in CO


CN
Mass of air supplied per kg of fuel, ma =
0.33(C1 + C2 )
0.861 × 0.818
= = 15.927 kg/kg of fuel
0.33(0.014 + 0.12)
Theoretical air required for complete combustion of fuel
100  8  O 
= C + 8 H −  + S
23  3  8 
100  8  0.014  
=  × 0.861 + 8  0.039 − + 0
23  3  8  
= 11.278 kg/kg of fuel
Excess air supplied = 15.927 – 11.278 = 4.649 kg/kg of fuel.

M01_THERMAL ENGINEERING_SE_XXXX_CH01(Part-II).indd 41 6/15/2018 12:59:16 PM


42 Chapter 1

C1 0.014
Carbon in fuel burned to CO = = = 0.10448 kg
C2 + C1 0.12 + 0.014
Loss due to incomplete combustion = 0.10448 × 24000
= 2507.5 kJ/kg of fuel.

Example 1.25
The mass analysis of the petrol used in an engine was 85% C and 15% H2. The dry exhaust gas
analysis showed that the percentage by volume of carbon dioxide was six times that of oxygen and
that no carbon-monoxide was present. Calculate: (a) the air-fuel ratio by mass and (b) the percent-
age excess air supplied.
Assume air contains 23.2% O2 by mass or 20.9% O2 by volume. [IAS, 1999]

Solution
Given: C = 0.85, H2 = 0.15% by mass
(a) Maximum mass of air required for complete combustion,
1 8 
(ma)th = C + 8H 
0.232  3 
1 8 
=  × 0.85 + 8 × 0.15
0.232  3 
= 14.94 kg
The combustion equation for the given fuel can be written as:
 76.8 
0.85C + 0.15H2 + aO2 +  a ×  N = bCO2 + cO2 + dN2 + eH2O
 23.2  2
where b, c, d and e are the products formed and ‘a’ is the amount of O2 supplied per kg of fuel.
Balancing the mass of carbon from both sides, we have
12
b× = 0.85 as 44 kg of CO2 contains 12 kg of carbon.
44
44
b = 0.85 × = 3.117 kg
12
Balancing the mass of hydrogen from both sides, we have
2
e× = 0.15 as 18 kg of H2O contains 2 kg of hydrogen.
18
e = 1.35 kg
Balancing the mass of O2 from both sides, we have
32 16
a=b× +c+e× as 44 kg of CO2 contains 32 kg of O2 and 18 kg of water contains 16 kg
44 18
of O2.

M01_THERMAL ENGINEERING_SE_XXXX_CH01(Part-II).indd 42 6/15/2018 12:59:19 PM



Fuels and Combustion 43

a = 0.7273 b + c + 0.889 e
Now balancing the mass of N2 from both sides, we have
76.8
a× =d
23.2
a = 0.302 d
Thus, we have
a = 0.7273 × 3.117 + c + 0.889 × 1.35
= 2.267 + c + 1.2
= 3.467 + c
Volumetric analysis of exhaust gases:
Constituent Mass of gas Molecular weight Proportional volume (4) % by volume
(1) (2) (3) = (2)/(3) (5) = (4)/S(4)
CO2 3.117 44 0.0708 0.0708/Σ(4)
O2 c 32 c/32 (c/32)/Σ(4)
N2 d 28 d/28 (d/28)/Σ(4)
Total S(4) = 0.0708 + c/32 + d/28

As per the given condition, percentage by volume of CO2 = 6 × percentage by volume of O2


0.0708 (c/32)
=6×
∑ ( 4) ∑( 4)
c
or 0.0708 = 6 ×
32
c = 0.3776 kg of O2 in the exhaust gases when one kg of fuel is burnt.
We have a = 3.467 + 0.3776 = 3.8446 kg of O2 is supplied per kg of fuel.
100
Amount of actual air supplied per kg of fuel, (ma)act = 3.8446 × = 16.57 kg
23.2
∴ A:F ratio = 16.57:1
16.57 − 14.94
(b) % Excess air supplied = = 0.1091 or 10.91%
14.94

Example 1.26
A four cylinder engine of a truck has been converted to run on propane fuel. A dry analysis of the
engine exhaust gives the following volumetric percentages:
CO2 = 4.90; CO = 9.79 and O2 = 2.45.
Calculate the equivalence ratio at which the engine is operating. [IAS, 2001]

Solution
Given: Volumetric analysis of engine exhaust: CO2 = 4.90%, CO = 9.79%, O2 = 2.45%

M01_THERMAL ENGINEERING_SE_XXXX_CH01(Part-II).indd 43 6/15/2018 12:59:21 PM


44 Chapter 1

Constituent % by volume Molecular weight Proportional mass % by mass


(a) (b) (c) = (a) ë (b) (c)
(d) =
Σ (c)
CO2 4.90 44 215.60 7.46
CO 9.79 28 274.12 9.49
O2 2.45 32 78.40 2.72
N2 82.86 28 2320.08 80.33
Total 100.00 Σ(c) = 2888.2 Σ(d) = 100.00

CN
Mass of air supplied, ma =
0.33(C1 + C2 )
Propane: C5H12 + 8O2 = 5CO2 + 6H2O
Minimum amount of air required in kg per kg of fuel for complete combustion
1 8  O 
(ma)th = C + 8 H −  + S
23  3  8 
5 × 12
% C in C5H12 = = 83.33%
5 × 12 + 12
12
% H2 in C5H12 = = 16.67%
5 × 12 + 12
1 8 
ma = × 83.33 + 8 × 16.67 = 15.46 kg/kg of fuel
23  3 

Actual A:F ratio = 15.46:1


Stoichiometric amount of air required per kg of fuel burned,
0.8333 × 0.8033
mstoi = = 11.967 kg/kg of fuel
0.33 (0.0949 + 0.0746)
stoichiometric A/F ratio 11.967
Equivalence ratio, p = = = 0.774
Actual A/F ratio 15.46

Summary for Quick Revision


1. Fuels may be classified as solid, liquid, and gaseous fuels.
2. The calorific value of a solid or liquid fuel is the heat evolved by the complete combustion of unit mass of
the fuel.
3. The calorific value of a gaseous fuel is the heat evolved by the complete combustion of one cubic metre of
gas at STP.
4. Higher calorific value (HCV) of a fuel is obtained when the water vapour produced during the process of
combustion due to hydrogen present in the fuel is allowed to condense evolving their latent heat.
5. Lower calorific value (LCV) of fuel is obtained when the latent heat of water vapour goes waste.

M01_THERMAL ENGINEERING_SE_XXXX_CH01(Part-II).indd 44 6/15/2018 12:59:24 PM


45
Fuels and Combustion

6. LCV = HCV – mw LCVfg


where mw = mass of water vapour formed per kg of fuel burned.
hfg = latent heat of vapourisation at the partial pressure of water vapour in the products of combustion
1   O 
7. HCV =  35, 000 C + 143, 000  H −  + 9, 605 where C, H, O, S = percentage of carbon, hydrogen,
100   8  
oxygen, and sulphur in the fuel.
9
8. LCV = HCV – × H × H.V. = HCV
100
9. Combustion of hydrocarbon fuel:
xCn H2n + 2 + yO2 O a CO2 + bH2O
x
where a = xn, b = x(n + 1), y = (3n + 1)
2
10. Minimum air required for complete combustion of solid/liquid fuels
1 8  O 
=
 C + 8  CH −  + S  kg/kg of fuel
23  3  8  
11. Mass of air supplied per kg of fuel,
1  CN 
ma =
 
0.33  C1 + C2 
where C = mass fraction of carbon in fuel
   C1, C2 = fraction of CO and CO2 by volume in dry flue gases.
400  2O − C1 
12. Excess air supplied = C kg
69  C1 + C2 
7C2
13. Percentage of C burning to CO2 =
11C1 + 7C2
11C1
Percentage of C burning to CO =
11C1 + 7C2
14. The Orsat apparatus is used for flue gas analysis. The percentage of SO2 cannot be measured by this appa-
ratus. NaOH or KOH is used to absorb CO2, alkaline solution of pyrogallic acid absorbs O2, and cuprous
chloride absorbs CO1. N2 is obtained by difference.
15. The bomb calorimeter is used to measure different HCVs of solid and liquid fuels.
( mw + mc ) ⋅ ∆q − m fw × CV
HCV =
mf
where mf, mfw, mw, mc = mass of fuel, fuse wire, water in calorimeter, and water equivalent of calorimeter,
respectively.
Dq = true temperature rise
C.V. = calorific value of fuse wire.
16. The Boys gas calorimeter is used to determine the HCV of gaseous fuels.
mw × ( ∆q ) w
HCV = kJ/m3 at NTP
Vg
where (Dq )w = rise in temperature of water circulated,
mw = mass of water circulated, kg
Vg = volume of gas at NTP

M01_THERMAL ENGINEERING_SE_XXXX_CH01(Part-II).indd 45 6/15/2018 12:59:28 PM


46 Chapter 1

Multiple-choice Questions

1. The percentage of oxygen contained in air on vol- 11. Bomb calorimeter is used to determine
ume basis is (a) HCV at constant volume
(a) 21 (b) 23 (c) 79 (d) 77 (b) HCV at constant pressure
(c) LCV at constant volume
2. The percentage of oxygen contained in air on
(d) LCV at constant pressure
mass basis is
(a) 21 (b) 23 (c) 79 (d) 77 12. The ultimate analysis of fuel is determination of
(a) C, H2, N2, S, and moisture
3. The mass of oxygen required to convert 1 kg of (b) fixed carbon ash, volatile matter, and moisture
carbon to CO2 is will be (c) higher calorific value
3 11 3 8 (d) lower calorific value
(a) kg (b) kg (c) kg (d) kg
11 3 8 3 13. Pour point of a fuel oil is the
4. A gas is formed by mixing equal masses of O2 (a) lowest temperature at which oil will flow of
and N2 gas. The ratio of N2 and O2 by volume its own
will be (b) storage temperature
8 7 7 8 (c) temperature of transportation of oil
(a) (b) (c) (d) (d) temperature of pumping of oil through burners
15 15 8 7
14. Orsat apparatus is used to determine in flue gases
5. CO produced by 1 kg of C is
the percentage of
11 3 3 7 (a) CO2, CO, and O2
(a) kg (b) kg (c) kg (d) kg
3 11 7 3 (b) CO2, CO, O2, and N2
6. The ratio of air required for complete combustion (c) CO2
of C to CO2 as compared to that for CO is (d) CO
(a) 1.5:1 (b) 2:1 (c) 3:1 (d) 4:1 15. In the Orsat apparatus, CO2, O2 and CO are
absorbed in first three pairs of bulbs by filling
7. Which law is associated with combustion of gas-
them with
eous fuels?
(a) sodium hydroxide solution, pyrogallol, and
(a) Avogadro’s hypothesis
(b) Dalton’s law cuprous chloride solutions, respectively
(c) Charles’s law (b) pyrogallol, sodium hydroxide, and cuprous
(d) Boyle’s law chloride, respectively
(c) cuprous chloride, sodium hydroxide, and
8. The Orsat apparatus is used for pyrogallol, respectively
(a) gravimetric analysis of flue gases (d) sodium hydroxide, cuprous chloride, and
(b) volumetric analysis of flue gases pyrogallol, respectively
(c) determining mass flow rate of flue gases
(d) measuring smoke density of flue gases 16. Orsat apparatus is employed to determine
(a) ultimate analysis of fuel
9. Alkaline pyrogallol is used in Orsat apparatus for (b) gravimetric analysis of fuel
absorption of (c) volumetric analysis of dry products of com-
(a) CO2 (b) CO (c) O2 (d) N2 bustion
10. The proximate analysis of fuel is determination of (d) gravimetric analysis of dry products of com-
percentage of bustion
(a) C, H2, N2, S, and moisture 17. In the Orsat apparatus,
(b) fixed carbon, ash, volatile matter, and moisture (a) CO2 is absorbed in cuprous chloride
(c) higher calorific value (b) CO is absorbed in caustic potash solution
(d) lower calorific value

M01_THERMAL ENGINEERING_SE_XXXX_CH01(Part-II).indd 46 6/15/2018 12:59:32 PM


47
Fuels and Combustion

(c) O2 is absorbed in pyrogallic acid 23. The minimum weight of air required per kg of
(d) N2 is absorbed in hot nickel-chrome com- fuel for complete combustion of a fuel is given by
pound  O
(a) 11.6C + 34.8  H −  + 4.35 S
 8
18. Items given in List I and II pertain to gas analysis.
Match List I with II and select the correct answer.  O
(b) 11.6C + 34.8  H −  + 4.35 S
 8
List I List II
 O
A. CO2 1. Alkaline pyrogallol (c) 11.6C + 34.8  H −  + 4.35 S
 8
B. Orsat apparatus 2. KOH solution  O
(d) 11.6C + 34.8  H +  + 4.35 S
C. CO 3. Wet analysis  8

D. O2 4. Ammoniacal 24. The mass of air required for complete combustion


cuprous chloride of unit mass of fuel can always be calculated from
the formula, where C, H, O and S are in percent-
5. Dry analysis
age. Which of the following is the correct option?
Codes: (a) 0.1152C + 0.3456H
A B C D (b) 0.1152C + 03456 (H – 0.125O)
(a)
2 3 1 4 (c) 0.1152C + 0.3456 (H – 1.25O) + 0.0432S
(b)
1 3 2 4 (d) 0.1152C + 0.3456 (H + 0.125) + 0.0432S
(c)
1 5 4 2
25. List I gives the different terms related to combus-
(d)
2 5 4 1
tion while List II gives the outcome of the events
19. If methane undergoes combustion with the sto- that follows. Match List I with List II and select
ichiometric quantity of air, the air-fuel ratio on the correct answer.
molar basis would be
List I List II
(a) 15.22: 1 (b) 12.30: 1
(c) 14.56: 1 (d) 9.52: 1 A. Association 1. Pseudo shock
B. Dissociation 2. Knock
20. The presence of nitrogen in the products of com-
bustion ensures that C. Flame front 3. Endothermic
(a) complete combustion of fuel takes place D. Abnormal 4. Exothermic
(b) incomplete combustion of fuel occurs combustion
(c) dry products of combustion are analysed Codes:
(d) air is used for combustion A B C D
21. In the combustion process, the effect of dissocia- 3 4 1 2
(a)
4 3 1 2
(b)
tion is to
(c)
3 4 2 1
(a) reduce the flame temperature (d)
4 3 2 1
(b) separate the products of combustion
(c) reduce the proportion of carbon monoxide in 26. The calorific value determined by the bomb
gases calorimeter is
(d) reduce the use of excess air (a) lower calorific value at constant pressure
(b) higher calorific value at constant pressure
22. Volumetric analysis of sample dry products of (c) lower calorific value at constant volume
combustion gave the following results:
(d) higher calorific value at constant volume
CO2 = 10% CO = 1% O2 = 8% N2 = 81%
Their proportion by weight will be 27. Incomplete combustion is indicated by
(a) 440: 28: 256: 2268 (a) high percentage of carbon monoxide in the
(b) 22: 14: 256: 1134 exhaust gases
(c) 440: 14: 28: 2268 (b) high percentage of carbon dioxide in the
(d) 22: 28: 14: 1134 exhaust gases

M01_THERMAL ENGINEERING_SE_XXXX_CH01(Part-II).indd 47 6/15/2018 12:59:34 PM


48 Chapter 1

(c) high temperature of exhaust gases IV. In the combustion of liquid fuels, the number
(d) low temperature of exhaust gases of moles of the products are invariably greater
than the number of moles of the reactants.
28. In order to burn 1 kg of CH4 completely, the min-
Of these statements:
imum number of kilograms of oxygen needed is
(a) I, II, and IV are correct
(take atomic weights of H, C, and O as 1, 12, and
(b) I, III, and IV are correct
16 respectively).
(c) II, III, and IV are correct
(a) 3 (b) 4 (c) 5 (d) 6
(d) I and III are correct
29. Consider the following statements:
30. Which one of the following gaseous fuels does not
I. For the combustion of pulverised coal,
have different higher and lower calorific values?
5%–10% excess air is required.
(a) Methane (b) Ethane
II. Air contains 21% oxygen by weight.
(c) Carbon monoxide (d) Hydrogen
III. The flue gases from a coal-fired furnace con-
tain around 70% nitrogen by volume.

Review Questions
1. What is the proximate analysis of coal?
2. What do you understand by ultimate analysis of coal?
3. What is a bomb calorimeter?
4. How does the LCV of a fuel differ from its HCV?
5. What is a pour point of a fuel?
6. What is the flash point?
7. What is LPG?
8. What is combustion?
9. Why is excess air always required to be supplied for combustion of fuel?

Exercises
1.1 Calculate the HCV and LCV of a fuel having the following composition C = 80%, H2 = 10%, O2 = 3%,
S = 2%, N2 = 20% and rest in combustible matter. [Ans. 41946.95 kJ/kg, 39732.95 kJ/kg]
1.2 The following data refer for the determination of calorific value of a gaseous fuel:
Barometer pressure = 740 mm Hg
Volume of fuel used = 15 litres
Pressure of gas = 40 mm of water above atmospheric
Temperature of gas = 20°C
Volume of cooling water circulated = 12.5 litres
Rise in temperature of cooling water = 5°C
Volume of condensate collected = 0.01 litres
Latent heat of steam = 2460 kJ/kg
Specific gravity of Hg = 13.6
Calculate HCV and LCV of the fuel. [Ans. 19213.3 kJ/kg, 16512 kJ/kg]

M01_THERMAL ENGINEERING_SE_XXXX_CH01(Part-II).indd 48 6/15/2018 12:59:34 PM



Fuels and Combustion 49

1.3  The following data refer to determination of calorific value of coal by using bomb calorimeter:
Mass of coal sample = 1.01 gram
Mass of water = 2500 gram
Water equivalent of calorimeter = 744 gram
Temperature rise of water = 2.59°C
Temperature correction for cooling = 0.01°C
Calculate HCV of coal.[Ans. 35073.7 kJ/kg]
  1.4 The gravimetric analysis of a sample coal is given as follows: C = 80%, H2 = 12%, and ash = 8%. Calculate
the stoichiometric A:F ratio and the analysis of products by volume.
[Ans. 13.3: 1, CO2, = 13.6%, H2 = 12.2%, N2= 74.2%]
1.5 A sample of coal gave the following ultimate analysis by mass: C = 81.9%, H2 = 4.9%, O2 = 6%, N2 =
2.3%, ash = 4.9%. Determine (a) the stoichiometric A:F ratio, and (b) the analysis by volume of the wet dry
products of combustion when air supplied is 25% in excess of that required for complete combustion.
[Ans. 10.8: 1; CO2 = 14.16%, H2O = 5.07%, CO = 4.04%, N2 = 76.73%; CO2 = 14.9%, N2, = 85.1%]
1.6 The dry exhaust analysis of a petrol engine gave CO = 1.5% and O2 was negligible if the fuel had an ultimate
analysis by weight of C = 84% and H2 = 16%, calculate the air–fuel ratio supplied.  [Ans. 14.75°C]
1.7 Methane is burnt with 50% excess air. The products of combustion exist at 1 bar, 250°C. Calculate the
dew-point temperature of water vapours in the combustion products.  [Ans. 51°C]
1.8 The composition of a sample of coal was found to be: C = 89.1%, H2 = 5%, O2 = 4.2%, N2 = 1.5%, and
remainder ash. Calculate the stoichiometric air-fuel ratio by mass if 30% excess air is supplied, calculate the
percentage composition of dry gases by volume. [Ans. 11.9: 1; CO2 = 14.1%, O2 = 4.9%, N2 = 81%]
1.9 During a boiler test, it was found that flue gases were leaving the boiler at a temperature of 300°C and inlet
air temperature was 20°C. The volumetric composition of the flue gases was: CO2 = 11.6%, O2 = 6.8, CO =
0.5%. The coal supplied to the furnace has 80% of carbon by mass. Calculate the heat carried away by the
exhaust gases per kg of coal burned. Assume the average specific heat of flue gases to be 1 kJ/kg.
[Ans. 4647.3 kJ/kg of coal burned]
1.10 A sample of coal has the following composition by weight: C = 77.7%, H2 = 6.8%, N2 = 1.2%, S = 2.2%,
O2 = 8.8%, and incombustible matter = 3.3%. Determine the minimum quantity of air required for complete
combustion of 1 kg of coal and the constituent products of combustion in kg per kg of coal.
[Ans. 11.086 kg/kg of coal; CO2 = 2.849, H2O = 0.612, SO2 = 0.044, N2 = 0.012 kg/kg of coal]
1.11 The analysis of a coal contains; C = 82%, H2 = 6%, O2 = 4%, H2O = 2%, and ash = 6%. Determine the
theoretical minimum air required for complete combustion of 1 kg of coal. If the actual air supplied is
18 kg per kg of coal, the hydrogen is completely burned and 80% of C is burned to CO2, and the remainder
to CO, determine the volumetric analysis of the dry products of combustion.
[Ans. 11.412 kg/kg of coal; CO2 = 8.84%, CO = 2.22%, O2 = 8.74%, N2 = 80.2%]
1.12 A coal has the following analysis: C = 81%, H2 = 4.5%, O2 = 8%, and remainder incombustible mat-
ter. The Orsat analysis of the dry flue gases was: CO2 = 8.3%, CO = 1.4%, O2 = 10%, and N2 = 80.3%
(by difference). Calculate the mass of air supplied per kg of coal and the percentage of excess air.
[Ans. 20.15 kg/kg of coal, 89.8%]
1.13 A coal of calorific value 29,725 kJ/kg has a composition by weight of C = 80%, H2 = 5%, O2 = 8%, S = 2%,
N2 = 2%, and remainder is ash. It is burnt in a furnace with 50% excess air. The flue gases enter the chimney
at 325°C and the atmosphere temperature is 15°C. Calculate the proportion of heat carried away by the flue
gases cp for air = 1.005 kJ/kg.K and for dry products of combustion 1.046 kJ/kgK. Heat carried away by
moisture in flue gases = 29.0 kJ per kg of moisture. [Ans. 25.4%]
1.14 An unknown hydrocarbon fuel has the following Orsat analysis: CO2 = 12.5%, CO = 0.3%, O2 = 31%,
N2 = 84.1%. Determine the air-fuel ratio, fuel composition on mass basis, stoichiometric air-flue ratio, and
percentage of excess air. [Ans. 34.44: 1; C = 85.33%, H2 = 14.67%; 15:1; 129.6%]

M01_THERMAL ENGINEERING_SE_XXXX_CH01(Part-II).indd 49 6/15/2018 12:59:34 PM


50 Chapter 1

1.15 The percentage analysis of a gaseous fuel by volume is given as follows: CO2 = 8%, CO = 22%, O2 = 4%,
H2 = 30%, and N2 = 36%. Determine the (a) the minimum volume of air required for complete combustion
of 1 m3 of gas, (b) the percentage composition by volume of the dry products of combustion, and (c) if 1.4 m3
of air is supplied per m3 of gas, what will be the percentage by volume of CO2 in the dry products of com-
bustion?[Ans. 1.05 m3; CO2 = 20.14%, N2 = 79.86%; 16.3%]
1.16 During the trial on a boiler, the dry flue gas analysis by volume was obtained as: CO2 = 13%, CO = 0.3%,
O2 = 6%, N2 = 80.7%. The coal analysis by weight was reported as: C = 62.4%, H2 = 4.2%, O2 = 4.5%,
moisture = 15%, and ash 13%. Determine (a) the theoretical air required to burn 1 kg of coal, (b) the mass
of air actually supplied per kg of coal, and (c) the amount of excess air supplied per kg coal burnt.
[Ans. 8.5 kg/kg of fuel, 11.47 kg/kg of fuel, 3.18 kg/kg of fuel]
1.17 The percentage composition by mass of a crude oil is given as follows: C = 90%, H2 = 3.3%, O2 = 3%, N2 =
0.8%, S = 0.9%, and the remaining is incombustible matter if 50% excess air is supplied, find the percentage
of dry exhaust gases formed by volume. [Ans. CO2 = 12.7%, SO2 = 0.05%, O2 = 7%, N2 = 80.25%]
1.18 The volumetric composition of the dry exhaust gases coming from a petrol engine is: CO2 = 12.4%, CO =
3%, N2 = 84.6%. Assuming the fuel to be a pure hydrocarbon, find the ratio of carbon to hydrogen by mass
in the fuel and the A:F ratio. [Ans. C = 84.4%, H2 = 15.6%, 14.1: 1]

ANSWERS TO MULTIPLE-CHOICE QUESTIONS


1. (a) 2. (b) 3. (d) 4. (d) 5. (d)
6. (b) 7. (a) 8. (b) 9. (c) 10. (b)
11. (a) 12. (a) 13. (a) 14. (a) 15. (a)
16. (c) 17. (c) 18. (d) 19. (d) 20. (d)
21. (a) 22. (a) 23. (a) 24. (c) 25. (d)
26. (d) 27. (a) 28. (b) 29. (a) 30. (c)

M01_THERMAL ENGINEERING_SE_XXXX_CH01(Part-II).indd 50 6/15/2018 12:59:34 PM


Chapter 2

Properties of Steam

2.1 ❐ PURE SUBSTANCE


A pure substance may be defined as a system that has the following characteristics:
1. Homogeneous in composition: It means that the composition of every part of the system is same
throughout. By composition, we mean the relative proportions of the chemical elements into
which the sample of the system can be analysed.
2. Homogeneous in chemical aggregation: It means that the chemical elements must be combined
chemically in the same way in all parts of the system.
3. Invariable in chemical aggregation: It means that the state of chemical combination of the sys-
tem does not change with time and position.
Examples of pure substance are atmospheric air, mixture of steam-water, combustion prod-
ucts of a fuel, etc.

2.2 ❐ CONSTANT PRESSURE FORMATION OF STEAM


Consider 1 kg of ice under atmospheric pressure and at a temperature of −10°C contained in a vessel.
If we heat the ice gradually keeping the pressure constant, the following changes will take place, as
shown in Fig. 2.1:

b′c′ - Latent heat of fusion


Vapour
state c′d′ - Sensible heat of water
f
d′e′ - Latent heat of vapourisation
Soild Melting Liquid Vapourisation
Temperature

e′f′ - Super heat


state state
d e
100°C

b c
0°C
a
−10°C
a′ b′ c′ d′ e′ f′
Heat added

Figure 2.1  Constant pressure temperature-heat added diagram for steam and water

M02_THERMAL ENGINEERING_SE_XXXX_CH02.indd 51 6/15/2018 1:03:30 PM


52 Chapter 2

1. The temperature of ice will gradually increase till it just reaches the freezing temperature (repre-
sented by point ‘b’), which is 0°C at atmospheric pressure. This is shown by the straight line ab.
2. On addition of more heat at point b, the ice starts melting, and with no rise in temperature till
the whole ice has melted, up to point c, the ice is converted into water. The heat added during the
process bc is called the latent heat of fusion of ice or simply latent heat of ice.
3. Gradual addition of more heat beyond the point c leads to gradual increase in temperature till the
water reaches the vapourisation or boiling point ‘d’. Under atmospheric pressure of 1.01325 bar,
it is 100°C.
4. On further heating beyond point d, the water will gradually turn into steam, with no rise in tem-
perature. This steam contains some water particles and is called the wet steam. With further addi-
tion of heat, all the water particles in the steam disappear and this steam is called dry saturated
steam or simply dry steam. This corresponds to point ‘e’. The heat added during de is called the
latent heat of vapourisation of steam or simply latent heat of steam.
5. If the dry saturated steam is further heated beyond point e, keeping the pressure constant, the pro-
cess is called superheating. The steam so obtained is called the superheated steam. Let the steam
be superheated to temperature ts°C, represented by point ‘f’. Then (ts − t)°C is called the degree
of superheat. The heat added during process ef is called the heat of superheat.

2.3 ❐ PROPERTIES OF STEAM


1. Enthalpy or total heat of water: It is defined as the quantity of heat required to raise the temper-
ature of 1 kg of water from 0°C to its boiling point or saturation temperature corresponding to the
pressure applied. It is also called enthalpy of saturated water or liquid heat and is represented by hf.
hf = specific heat of water, cpw × rise in temperature
= 4.187 × Dt, kJ/kg
Note that enthalpy of saturated water can also be found from the steam table based on saturation
temperature (Appendix A.1.1).
2. Heat of steam or latent heat of vapourisation: Latent heat of steam at a particular pressure may
be defined as the quantity of heat required to convert 1 kg of water at its boiling point into dry
saturated steam at the same pressure. It is denoted by hfg.
3. Dryness fraction or quality: Dryness fraction of a liquid−vapour mixture is defined as the ratio
of mass of dry steam actually present to the mass of the total mixture. It is denoted by x.
Let ms = mass of dry steam (saturated vapour), kg
mw = mass of water vapour in steam (saturated liquid), kg
x = dryness fraction of the sample
ms
Then, x = (2.1)
ms + mw

For dry steam, mw = 0 and x = 1.


The quality of steam is the dryness fraction expressed as a percentage.
Wetness fraction = 1 − x
4. Specific volume of wet steam: The specific volume of steam is the volume of 1 kg of dry steam.

M02_THERMAL ENGINEERING_SE_XXXX_CH02.indd 52 6/15/2018 1:03:30 PM


Properties of Steam 53

Let V be the total volume of the liquid-vapour mixture, V f be the volume of the saturated liquid
and Vg be the volume of the saturated vapour.
  Total volume of liquid and vapour can be expressed in terms of their specific volume as
V f = mw v f
Vg = ms v g
where v f and v g are the specific volumes of saturated liquid and saturated vapour, respectively.
  Specific volume of the mixture is then
V +V
v = V = f g
m m
mw v f + ms v g
=
mw + ms
mw v f ms v g
= +
mw + ms mw + ms
= (1 − x )v f + xv g

= v f + x( v g − v f )
     or vwet = v f + xv fg (2.2a)
     Now vf << vg
Volume of 1 kg of wet steam = xvg (2.2b)
1
Density of wet steam = kg/m3  (2.3)
v f + xv fg
5. Enthalpy of wet steam: It may be defined as the quantity of heat required to convert 1 kg of water
at 0°C, at constant pressure, into wet steam. Similar to the analysis of specific volume, one can write

hw = hf + xhfg (2.4)
where hf is the specific enthalpy of saturated water and hfg is the change in specific enthalpy dur-
ing vapourisation. When steam is dry saturated, then x = 1
     and hg = hf + hfg (2.5)
6. Total enthalpy of superheated steam:
Let cps = specific heat of superheated steam
ts = temperature of formation of steam, °C
tsup = temperature of superheated steam, °C
Then heat of superheat = cps (tsup − ts)
   hsup = (hf + hfg) + cps (tsup − ts)(2.6a)
         = hg + cps (tsup − ts)(2.6b)
Note that enthalpy of superheated steam can also be found from the steam table (Appendix A.1.3).

M02_THERMAL ENGINEERING_SE_XXXX_CH02.indd 53 6/15/2018 1:03:37 PM


54 Chapter 2

7. Internal energy of steam: The internal energy of steam may be defined as the actual heat energy
stored in steam above the freezing point of water. It is the difference between the total heat of
steam and the external work of evaporation. Similar to the analysis of specific volume, one can
write for 1 kg of wet steam,
uwet = uf + xufg(2.7)
where uf is the specific internal energy of saturated water and ufg is the change in specific internal
energy during evaporation. From the known value of enthalpy, internal energy can be found as
follows:
uwet = h − pv1

     = h f − p( v f + xv fg ) for wet steam (2.8a)


For superheated steam,
hsup = hg + cps (tsup − ts)

usup = hsup − pvsup

  = hg + cps (tsup − ts) − pvsup, for superheated steam  (2.8b)


8. Entropy of water: Consider 1 kg of water to be heated from a temperature Ts to T at a constant
pressure.
T
     Then, change of entropy = c pw ln s .
T
where cpw = specific heat of water
The entropy of water is always reckoned above the freezing point of water.
      Therefore, Ts = saturation temperature of formation of steam
T = absolute temperature of freezing water = 273 K
Entropy of saturated water,
T
  s f = c pw ln s (2.9)
273
9. Entropy of wet steam: Consider 1 kg of wet steam at an absolute pressure p bar.
    Let x = dryness fraction of steam
Ts = temperature of formation of steam
sf = the specific enthalpy of saturated water
sfg = change in entropy during evaporation
The entropy of wet steam can be calculated as
swet = sf + xsfg(2.10)
When steam is dry saturated, then x = 1
    and   sg = sf + sfg (2.11)
10. Entropy of superheated steam: Consider 1 kg of dry saturated steam at Ts temperature of
formation, to be superheated to a temperature Tsup.
Let cps = specific heat of steam at constant pressure during superheating

M02_THERMAL ENGINEERING_SE_XXXX_CH02.indd 54 6/15/2018 1:03:38 PM


Properties of Steam 55

Change of entropy per kg of steam during superheating


Tsup
= c ps ln
Ts
Total entropy of superheated steam
ssup = Entropy of dry saturated steam + Change of entropy during superheating
Tsup
    = sg + c ps ln (2.12)
Ts

2.4 ❐ STEAM TABLES


In actual practice, it is quite cumbersome to calculate the relation between various quantities such as
pressure, temperature, volume, enthalpy, latent heat and entropy, etc., of steam. Such quantities have
been determined experimentally and recorded in the form of tables known as steam tables. Steam
table is a complete data book that contains various properties, like volume, internal energy, enthalpy
and entropy for 1 kg of water in different phases for a given pressure or temperature. Steam table is
given in the Appendix A. The properties of steam are given in three different tables—saturated steam
table based on temperature (Appendix A.1.1), saturated steam table based on pressure (Appendix A.1.2),
and superheated steam table (Appendix A.1.3). When the temperature is selected as independent varia-
ble and the properties of steam are tabulated, the steam table is referred to as the saturated steam table
based on temperature. When the pressure is chosen as independent variable, it is referred to saturated
steam table based on pressure. For superheated steam, there is a separate table for different amount of
degree of superheat.
Values for unknown pressure or temperature can be calculated by linear interpolation between
two consecutive values. Saturated steam table based on pressure for a particular pressure (= 1 bar) is
shown in Table 2.1.

Table 2.1  Saturated steam table based on pressure


Pres- Temper- Specific Specific internal
sure ature, Specific enthalpy Specific entropy
volume energy
bar (p) °C (t) kJ/kg kJ/kg.K
m3/kg kJ/kg
Sat. Sat. Sat. Evap. Sat. Sat. Evap. Sat. Sat. Evap. Sat.
Liq. Vap. Liq. ufg Vap. Liq. hfg Vap. Liq. sfg Vap.
vf vg uf ug hf hg sf sg

1.00 99.62 0.001043 1.694 417.33 2088.7 2506.1 417.44 2258.0 2675.5 1.3025 6.0568 7.3593

2.5 ❐ T
 EMPERATURE−ENTROPY DIAGRAM
FOR WATER AND STEAM
It is the plot of the saturation temperature of water and steam corresponding to various absolute pres-
sure vs entropies at those saturation temperatures, as shown in Fig. 2.2. The curve on the left marked
water line (saturated liquid line) shows the relationship between entropy and temperature before steam

M02_THERMAL ENGINEERING_SE_XXXX_CH02.indd 55 6/15/2018 1:03:39 PM


56 Chapter 2

Critical point

Satruated
p3
liquid line
p3 Superheated
T p2 region
p2 Constant dryness
fraction line
p1
p1 Saturated
m vapour line
Liquid - vapour
region
x=0 x = 0.25 x = 0.50 x = 0.75 x = 1.0

Figure 2.2 Temperature−entropy diagram for water and steam

is formed. The dry steam line (saturated vapour line) shows the relationship between entropy and
temperature when all the water has been converted into steam. The area in between two regions is
called the wet region. The area to the right of the dry steam lines is called the region of superheat. The
lines of constant dryness, constant volume, and constant total heat are also drawn on the diagram. The
temperature at which the distinction between the entropy of water and that of dry steam disappears is
called the critical temperature (374.15°C). The pressure corresponding to this temperature is called
the critical pressure (221.2 bar).

2.6 ❐ E
 NTHALPY−ENTROPY OR MOLLIER DIAGRAM
OF STEAM
In Mollier diagram (Fig. 2.3), the vertical ordinate represents the enthalpy and the horizontal ordinate
represents the entropy. The regions above and below the saturation line represent the superheated and
wet conditions of steam, respectively. The lines of constant dryness fraction are shown in the wet
steam region, whereas the lines of constant temperature and pressure in the region of both wet and
superheat are shown.
From the thermodynamic property relationship, one can write,
Tds = dh − vdp
For constant pressure process, we have
Tds = dh

 ∂h 
or         = T (2.13)
∂s P
From Eq. (2.13), it is clear that the slope of the constant pressure lines is equal to the corresponding
saturation temperatures. It may be noted that as the pressure increases, the saturation temperature

M02_THERMAL ENGINEERING_SE_XXXX_CH02.indd 56 6/15/2018 1:03:41 PM


Properties of Steam 57

v=c p=c
v=c
Critical point
T=c

h p=c T=c
p=c
=c

T=c

x=c x=c
s

Figure 2.3  Mollier diagram for steam

also increases. This is the reason for which constant pressure lines (isobar) are divergent on the h−s
diagram. The saturated liquid line and saturated vapour line meet at critical point.

2.7 ❐ VARIOUS PROCESSES FOR STEAM


2.7.1 Constant Volume Process
The constant volume heating process is shown on the p−v and T−s diagrams in Fig. 2.4. Consider 1 kg
of wet steam at a certain pressure p1, and dryness fraction x1 is heated to state 2 in the wet region and
then to state 3 in the superheated region.
Initial volume of steam v1 = vf 1 + x1(vg1 − vf 1)
Final volume of steam in wet region v2 = vf 2 + x2(vg2 − vf 2)
Since the volume remains constant,

vf 1 + x1(vg1 − vf 1) = vf 2 + x2(vg2 − vf 2)

P T

P3 3
3
2
2
P2 1

P1 1

O v s
(a) (b)

Figure 2.4 Constant volume heating: (a) p−v diagram,


(b) T−s diagram

M02_THERMAL ENGINEERING_SE_XXXX_CH02.indd 57 6/15/2018 1:03:43 PM


58 Chapter 2

v f 1 + x1 ( v g1 − v f 1 ) − v f 2
or x2 = (2.14)
(vg 2 − v f 2 )
From steam table, internal energy of steam can be found as follows:
Initial internal energy of steam, u1 = uf1 + x1 ufg1
Final internal energy of steam in the wet region, u2 = uf2 + x2 ufg2
Since dv = 0, therefore dW = 0
q = u2 − u1(2.15)
For constant volume heating process, v2 = v3. After knowing the value of v3, the condition of steam can
be found from superheated steam table. If pressure of state 3 is given (=p3), then from known values of p3
and v3, temperature of state 3 can be obtained. Similarly, if temperature at state 3 (=T3) is given, pressure
can be found from known values of T3 and v3.

2.7.2 Constant Pressure Process


The constant pressure heating process is shown in Fig. 2.5. In this case, p1 = p2 = p (say).
For wet steam, during heating process 1–2,
Work done per kg of steam, w = pdv = p(v2 − v1)
where v1 = vf + x1(vg − vf ) and v2 = vf + x2(vg − vf )
The specific enthalpy of steam, h1 = hf + x1 hfg and h2 = hf + x2 hfg
From first law of thermodynamics, heat transfer per kg of steam during constant pressure heating
q = h2 − h1
q = (hf + x2hfg) − (hf + x1hfg)
q = (x2 − x1) hfg(2.16a)
For superheated steam, specific volume and specific enthalpy of steam can be found from the superheated
steam table. Then, during heating process 2-3,
Work done per kg of steam, w = pdv = p(v3 − v2) = p(vsup− v2)
and heat transfer per kg of steam
q = h3 − h2 = hsup − h2(2.16b)

p=c

3
1 2

Figure 2.5  Constant pressure expansion

M02_THERMAL ENGINEERING_SE_XXXX_CH02.indd 58 6/15/2018 1:03:44 PM


Properties of Steam 59

2.7.3 Isothermal Process


The isothermal heating process is shown in Fig. 2.6. The isothermal process for wet steam will be
same as the constant pressure process. However, in the superheated region, the steam will behave as a
gas and shall follow Boyle’s law, pv = const. Therefore, the process will be hyperbolic.
In the wet region,
q = h2 − h1 and w = p1 (v2 − v1)
In the superheated region,
q = h3 − h2 and w = p1 (v3 − v2)

T
p1
p2

T =c
1 2
T =c

Figure 2.6  Isothermal process

2.7.4 Hyperbolic Process


During a hyperbolic process, pv = const. If the superheated vapour behaves as an ideal gas, then the
process during which pv = const. may be regarded isothermal, as shown in Fig. 2.7.
For 1 kg of steam in the wet region,
Initial volume of steam v1 = vf 1 + x1(vg1 − vf 1)
Final volume of steam v2 = vf 2 + x2(vg2 − vf 2)
As pv = const., therefore
p1  v f 1 + x1 ( v g1 − v f 1 )  = p2  v f 2 + x2 ( v g 2 − v f 2 ) 

( p1v f 1 − p2 v f 2 ) + p1 x1 ( v g1 − v f 1 )
or x2 =
p2 ( v g 2 − v f 2 )

T p1
p2
1
2

Figure 2.7  Hyperbolic expansion

M02_THERMAL ENGINEERING_SE_XXXX_CH02.indd 59 6/15/2018 1:03:45 PM


60 Chapter 2

Now, h1 = hf 1 + x1hfg1 and h2 = hf 2 + x2hfg2 for wet steam


and hs2 = hg2 + cp (Tsup − T2) for superheated steam
Change is internal energy,
u2 − u1 = (h2 − p2v2) − (h1 − p1v1)(2.17a)
    = h2 − h1 for wet steam (2.17b)
    = hs2 − h1 for superheated steam (2.17c)
v
Work done, W = p1v1 ln 2 (2.18)
v1
where v1 and v2 are the specific volumes of steam before and after heating.
Total heat supplied, q = w + (u2 − u1)
v
= p1v1 ln 2 + (u2 − u1 )
v1
v2
= ( h2 − h1 ) + p1v1 ln
   (2.19)
v1

2.7.5 Reversible Adiabatic or Isentropic Process


The isentropic process of expansion (1-2) is shown in Fig. 2.8 on T−s diagram. The initial condition of
steam may be wet, dry, or superheated. For initially wet steam as shown in Fig. 2.8(a), we have
s1 = s f 1 + x1s fg1
s2 = s f 2 + x2 s fg 2

Since s1 = s2, therefore


s f 1 + x1s fg1 = s f 2 + x2 s fg 2

( s f 1 − s f 2 ) + x1s fg1
x2 = (2.20a)
s fg 2
For initially dry steam (Fig. 2.8(b)),      s1 = sg1

T p1 T p1 T p1
1
1 p2 1 p2 p2

2 2
2

s s s
(a) (b) (c)

Figure 2.8 Isentropic expansion: (a) Expanded steam wet, (b) Expanded steam dry
saturated, (c) Expanded steam superheated

M02_THERMAL ENGINEERING_SE_XXXX_CH02.indd 60 6/15/2018 1:03:48 PM


Properties of Steam 61

   s2 = s f 2 + x2 s fg 2

sg1 = s f 2 + x2 s fg 2
Then,  

( sg1 − s f 2 )
    x2 = (2.20b)
s fg 2

For initially superheated steam at temperature Tsup, s1 can be calculated from superheated steam table.
After expansion, state 2 may be superheated (Fig. 2.8(c)), dry or wet. If the state after expansion is
superheated, then the condition of steam can be calculated from the relationship s1 = s2. Note that if
pressure is specified, then temperature can be calculated or otherwise if temperature is specified, then
pressure can be calculated.
Now q = w + (u2 − u1)
where u1 = u f 1 + x1u fg1 for wet steam

   = u g1 for dry steam

   = usup for superheated steam

Similarly, u2 can be found depending on the condition of steam after expansion.


But q=0
w = u1 − u2

2.7.6 Polytropic Process


Let the steam expand from condition 1 to 2 according to the law pvn = constant. Then for wet steam,
Initial volume of steam, v1 = vf 1 + x1(vg1 − vf 1)
Final volume of steam v2 = vf 2 + x2(vg2 − vf 2 )

p1v1n = p2 v2n (2.21)


Now    
or n n
p1 =  v f 1 + x1 ( v g1 − v f 1 )  = p2  v f 2 + x2 ( v g 2 − v f 2 ) 
1
 p  n ( v f 1 − v f 2 ) + x1 ( v g1 − v f 1 )
or x2 =  1  × (2.22)
 p2  (vg 2 − v f 2 )
Also u1 = u f 1 + x1u fg1
      
u2 = u f 2 + x2 u fg 2
p1v1 − p2 v2
       w = (2.23)
n −1
p v − p2 v2
q = w + (u2 − u1) = 1 1 + (u2 − u1)(2.24)
n −1
n = 1.13 for wet steam and 1.3 for superheated steam

M02_THERMAL ENGINEERING_SE_XXXX_CH02.indd 61 6/15/2018 1:03:53 PM


62 Chapter 2

For superheated steam on heating from wet steam, v2 can be calculated from superheated steam table
for the known pressure and temperature conditions. Similarly, the condition of steam can also be
found after knowing the value of v2 from Eq. (2.21).

2.7.7 Throttling Process


The throttling process occurs when steam is expanded through a small aperture as in the case of throat
of a nozzle. During this process, no work is done and the enthalpy remains constant. The throttling
process is shown on T−s and h−s diagrams in Fig. 2.9.
h1 = hf1 + x1hfg1
h2 = hf2 + x2hfg2 for wet steam after expansion

= hf2 + hfg2 + cps (Tsup − Ts), for superheated steam after expansion
Now h1 = h2
For wet steam after expansion
hf 1 + x1hfg1 = hf 2 + x2hfg2 (2.25a)
For superheated steam after expansion
hf 1 + x1hfg1 = hg2 + cps (Tsup − Ts)(2.25b)

T p1 h p1

p2 p2
1 1′ 1′
2′
1
2 2′ 2

s s
(a) (b)

Figure 2.9 Throttling process: (a) T−s diagram,


(b) h−s diagram

2.8 ❐ DETERMINATION OF DRYNESS FRACTION OF STEAM


The different methods commonly used for the measurement of dryness fraction of steam are as follows:
1. Barrel Calorimeter
2. Separating Calorimeter
3. Throttling Calorimeter
4. Combined Separating and Throttling Calorimeter.
The working principle of all the calorimeters is to bring the state of the substance from the two-phase
region to the single phase region (either compressed liquid or superheated vapour region). This is
because in the two-phase region, pressure and temperature are not independent variable, one is inde-
pendent and the other is dependent. On the other hand, both in compressed liquid and superheated
vapour region, both pressure and temperature are independent variables.

M02_THERMAL ENGINEERING_SE_XXXX_CH02.indd 62 6/15/2018 1:03:54 PM


Properties of Steam 63

2.8.1 Barrel Calorimeter


The barrel calorimeter, shown in Fig. 2.10, consists of a copper barrel placed on wooden block and
covered by a wooden cover. The barrel contains a known quantity of cold water and is surrounded by
an outer vessel with air space in between, which acts as an insulator. The temperature is measured
by a thermometer that passes through one of the holes in the wooden cover. The whole assembly is
placed on the platform of a weighing bridge. The steam from the main steam pipe enters through the
sampling tube via the control valve and flows into the cold water through fine exit holes of the ring
provided at the end of the pipe. The condensation of steam takes place as it comes in contact with cold
water, and as a result, the temperature of water rises. The quantity of steam condensed can be known
from the difference of readings of the weighing bridge before and after the condensation of steam. The
pressure of entering steam is indicated by the pressure gauge.
Main steam pipe

Sampling tube
Control valve

Pressure Gauge
Thermometer
Wooden cover

Copper calorimeter
Water Outer vessel

Air space
Ring

Wooden blocks

Platform

Figure 2.10  Barrel calorimeter


Let ms = mass of steam condensed
x = dryness fraction of steam
hfg = latent heat of steam
 ts = temperature of formation of steam (corresponding to pressure indicated by the pressure
gauge)
 mw = mass of cold water in the calorimeter in the beginning
 mc = water equivalent of calorimeter
  t1 = initial temperature of cold water in the calorimeter
  t2 = final temperature of the mixture after steam condensation
Heat lost by steam = ms  x × h fg + c ps (t s − t2 ) 

Heat gained by cold water and calorimeter = ( mw + mc ) × c pw × (t2 − t1 )

M02_THERMAL ENGINEERING_SE_XXXX_CH02.indd 63 6/15/2018 1:03:56 PM


64 Chapter 2

Heat lost by steam = Heat gained by cold water and calorimeter

ms  x × h fg + c ps (t s − t2 )  = ( mw + mc ) × c pw × (t2 − t1 )

( mw + mc )c pw ( t2 − t1 ) − ms c ps (t s − t2 )
x= (2.26)
ms h fg

This method gives only approximate results. The value of x calculated is always lower than the actual
value as heat losses due to convection and radiation are not accounted for.

2.8.2 Separating Calorimeter


This calorimeter, shown in Fig. 2.11, consists of two concentric chambers that communicate with each
other through an opening at the top. The steam to be tested flows from the main steam pipe through
a sampling tube to the calorimeter pipe through the valve which must be fully open when testing.
The metal basket has a large number of perforations through which the steam discharges. The water
particles due to their heavier momentum get separated from the steam and get collected in the inner
chamber. The quantity of water collected is indicated by the level in the gauge glass and the pointer that
moves on a scale. The dry steam in the inner chamber moves up and then down again through the annu-
lar space between the two chambers. The gauge fitted has two scales: the inner one indicates the pres-
sure, whereas the outer one indicates the rate of discharge of dry saturated steam during a predefined
time interval. Then the steam then flows through the calibrated orifice to the bucket calorimeter which
is placed on the platform balance by which the most of steam discharged can be further confirmed.
Sampling tube
V
Pressure
Gauge
Main steam
pipe

Metal basket
Scale
Inner chamber
Pointer
Gauge Outer chamber
glass, G

Calibrated orifice

Bucket calorimeter

Platform
balance

V = Valve ( to be fully opened when testing for dryness)


G = Gauge giving rate of steam discharge

Figure 2.11  Separating calorimeter

M02_THERMAL ENGINEERING_SE_XXXX_CH02.indd 64 6/15/2018 1:03:57 PM


Properties of Steam 65

Let ms = mass of dry steam condensed


mw = mass of suspended moisture collected
ms
Then x = (2.27)
ms + mw
It is not possible to remove all the water particles from steam, and the dryness fraction calculated by
this method is always greater than the actual. However, it is a quick method and is used for the meas-
urement of dryness fraction for very wet steam.

2.8.3 Throttling Calorimeter


The throttling calorimeter, shown in Fig. 2.12, consists of the sampling tube through which the steam
from the main steam pipe flows through the throttling valve and becomes superheated. The steam
then flows into the inner chamber, flows down, and rises up again to enter the annular space. The
loss of heat by radiation from the inner chamber is minimised by the hot steam around the outside of
the inner chamber. The temperature of throttled steam is measured by the thermometer placed in the
pocket filled with cylinder oil. To obtain good results, the steam issuing from the throttle valve must be
superheated. To ensure this, a manometer is attached to measure the pressure of steam. Corresponding
to this pressure, the saturation temperature of steam must be lower than the temperature indicated by
the thermometer. The steam finally escapes through the exhaust. This calorimeter is suitable for steam
having high dryness fraction.
 Let p1 = initial pressure of wet steam before expansion
x1 = dryness fraction of wet steam
 hfg1 = latent heat of vapourization of wet steam
tsup = temperature of superheated steam after expansion
 hfg2 = latent heat of vapourization of steam after expansion
ts2 = saturation temperature of steam after expansion
cps = specific heat of superheated steam at constant pressure
Enthalpy of steam before throttling = hf 1 + x1 hfg1

Throttling
valve Thermometer
Sampling
tube Manometer

Main steam
pipe H

Annular space

Outer chamber

Exhaust

Figure 2.12  Throttling calorimeter

M02_THERMAL ENGINEERING_SE_XXXX_CH02.indd 65 6/15/2018 1:03:58 PM


66 Chapter 2

Enthalpy of steam after throttling = hf 2 + hfg2 + cps (tsup − ts2)


Enthalpy before throttling = Enthalpy after throttling
hf 1 + x1 hfg1 = hf 2 + hfg2 + cps (tsup − ts2)
( h f 2 − h f 1 ) + h fg 2 + c ps (tsup − t s 2 )
x1 = (2.28)
h fg1

2.8.4 Combined Separating and Throttling Calorimeter


The combined separating and throttling calorimeter is shown in Fig. 2.13. The steam is first passed
through the separating calorimeter where it losses most of its moisture and becomes comparatively
drier. It is then passed through the throttling calorimeter where superheating takes place without
change of enthalpy. The temperature and pressure of steam after throttling are measured by using a
thermometer and pressure gauge, respectively.
Let p1 = pressure of wet steam entering the throttling calorimeter
  x1 = dryness fraction of steam
   hfg1 = latent heat of entering steam
  mw = mass of suspended moisture collected
    ms = mass of steam leaving the separating calorimeter and entering the throttling calorimeter
    p2 = pressure of steam after throttling
  hfg2 = latent heat of steam at pressure p2
Enthalpy of steam entering throttling calorimeter = Enthalpy of steam leaving throttling calorimeter
h f 1 + x1h fg1 = ( h f 2 + h fg 2 ) + c ps (tsup − t s 2 )

Pressure
Gauge

Valve
Thermometer
Throttle
valve Manometer

Main steam
pipe

Metal
basket
Scale
Inner
chamber
Pointer
Gauge glass
Outer
chamber

Exhaust

Figure 2.13  Combined separating and throttling calorimeter

M02_THERMAL ENGINEERING_SE_XXXX_CH02.indd 66 6/15/2018 1:04:00 PM


Properties of Steam 67

( h2 − h f 1 ) + c ps (tsup − t s 2 )
x1 = (2.29)
h fg1
ms
Let x2 = = apparent dryness fraction of steam (2.30)
ms + mw
and x = dryness fraction of steam entering the separating calorimeter
Mass of dry steam entering the whole apparatus = ms x1
Total mass of dry steam
x=
Total mass of steam entering the throttling calorimeter
ms x1  ms 
=
= x1  = x1 x2
ms + mw  ms + mw 
∴ x = x1x2(2.31)
This calorimeter gives quite satisfactory results when the steam is considerably wet.

Example 2.1
Determine the condition of steam in the following cases:
 (a)  At a pressure of 10 bar and temperature 200°C.
(b)  At a pressure of 10 bar and specific volume 0.175 m3/kg.

Solution
Given p = 10 bar; t = 200°C; v = 0.175 m3/kg
(a) Condition of steam at temperature of 200°C:
  From steam table (A.1.2 in the Appendix), corresponding to a pressure of 10 bar, we find the
saturation temperature ts = 179.91°C
  Since the saturation temperature at 10 bar is (179.91°C) lower than the given temperature of the
steam (200°C), therefore the given steam is superheated.
The degree of superheat = 200 − 179.91= 20.09°C
(b) Condition of steam at a volume of 0.175 m3/kg:
From steam table (A.1.2 in the Appendix), vf = 0.001127 m3/kg, vg = 0.19444 m3/kg.
  Since the volume of given steam (0.175 m3/kg) is less than the specific volume of the dry sat-
urated steam (0.19444 m3/kg), therefore the given steam is wet. Therefore, the dryness fraction
can be found to be
0.175 = 0.001127 + x ( 0.19444 − 0.001127)
or     x = 0.8994

Example 2.2
Calculate the quantity of heat required to generate 1 kg of steam at a pressure of 8 bar from water
at 30°C (a) when dryness fraction is 0.9, (b) when steam is just dry, and (c) when the temperature
of steam is 300°C.

M02_THERMAL ENGINEERING_SE_XXXX_CH02.indd 67 6/15/2018 1:04:02 PM


68 Chapter 2

Solution
From steam tables (A.1.2 in the appendix) at 8 bar: hf = 721.1 kJ/kg, hfg = 2048.0 kJ/kg
And at 30°C (A.1.1 in the appendix), hf = 125.79 kJ/kg
(a) h = hf + x hfg = 721.1 + 0.9 × 2048.0 = 2564.3 kJ/kg
Net heat required = h − hf = 2564.3 − 125.79 = 2438.51 kJ/kg
(b) h = hf + hfg = hg = 2769.1 kJ/kg
Net heat required = h − hf = 2769.1 − 125.79 = 2643.31 kJ/kg
(c) ts = 170.4°C, tsup = 300°C
Therefore, the steam is in superheated condition. From superheated steam table (A.1.3 in the appen-
dix), we have h = 3056.4 kJ/kg
Net heat required = h − hf = 3056.4 − 125.77 = 2930.63 kJ/kg

Example 2.3
A pressure cooker contains 2 kg of steam at 5 bar and 0.9 dry. Find the quantity of heat that must
be rejected so that the quality of steam becomes 0.5 dry.

Solution
The pressure cooker is a constant volume vessel. Therefore, W = 0
∴ Q = ∆U = U2 − U1
From steam tables (A.1.2 in the appendix) at 5 bar: vf = 0.001093 m3/kg , vg= 0.3749 m3/kg,
uf = 639.66 kJ/kg, ufg = 1921.6 kJ/kg,
u1 = uf + x1ufg = 639.66 + 0.9 × 1921.6 = 2369.1 kJ/kg
U1 = 2 × 2369.1 = 4738.2 kJ
Specific volume of the steam at 5 bar
v1 = vf + x1 (vg − vf ) = 0.001093 + 0.9 × (0.3749 − 0.001093) = 0.3375 m3/kg

From steam tables (A.1.2 in the appendix), the properties of steam at 2.5 and 2.75 bar are given below:

p, bar vf , m3/kg vg, m3/kg uf , kJ/kg ufg, kJ/kg


2.5 0.001067 0.7187 535.08 2002.1
2.75 0.001070 0.6573 548.57 1992.0

For constant volume of steam, we have


v1 = v2= 0.3375 m3/kg
Specific volume of the steam at 2.5 bar
v = vf + x (vg − vf ) = 0.001067 + 0.9 × (0.7187 − 0.001067) = 0.3599 m3/kg
Specific volume of the steam at 2.75 bar
v = vf + x (vg − vf ) = 0.001070 + 0.9 × (0.6573 − 0.001070) = 0.3292 m3/kg
Using linear interpolation, the pressure corresponding to v2 = 0.3375 m3/kg is obtained as
p2 − 2.5 0.3375 − 0.3599
=
2.75 − 2.5 0.3292 − 0.3599
or p2 = 2.68 bar

M02_THERMAL ENGINEERING_SE_XXXX_CH02.indd 68 6/15/2018 1:04:02 PM


Properties of Steam 69

Using linear interpolation, we get


2.68 − 2.5 u f 2 − 535.08
=
548.57 − 535.08 2.75 − 2.5
or u f 2 = 544.79 kJ/kg
2.68 − 2.5 u fg2 − 2002.1
=
1992.0 − 2002.1 2.75 − 2.5
or u fg2 = 1994.83 kJ/kg
∴ u2 = uf 2 + x2 ufg2 = 544.79 + 0.5 × 1994.83 = 1542.205 kJ/kg
    U2 = m u2 = 2 × 1542.205 = 3084.41 kJ
   Q = U2 − U1 = 3084.41 − 4738.2 = − 1653.79 kJ

Example 2.4
About 5400 m3/h of wet steam with dryness fraction 0.92 and at 10 bar is to be supplied by a boiler
for a processing plant. Calculate (a) the mass of steam supplied per hour and (b) the quantity of
coal of calorific value 15 MJ/kg to be burnt in the boiler if overall efficiency of the boiler is 30%.

Solution
(a) From steam tables (A.1.2 in the appendix) at 10 bar: vf = 0.001127 m3/kg, vg= 0.19444 m3/kg
Specific volume of the steam v = vf  + x (vg − vf ) = 0.001127 + 0.92 × (0.19444 − 0.001127)
= 0.179 m3/kg
V 5400
Mass flow rate of wet steam, m = = = 30167.6 kg/h
v 0.179
(b) At p = 10 bar, hf = 762.79 kJ/kg, hfg = 2015.3 kJ/kg
  h = hf + x hfg = 762.79 + 0.92 × 2015.3 = 2616.866 kJ/kg
H = mh = 30167.6 × 2616.866 = 78944566 kJ/h
H 78944566
Heat supplied to boiler, Hb = = = 263148553 kJ/h
hb 0.3
Hb 263148553
Quantity of coal burnt = = = 17.54 tonnes/h
C.V. 15 × 106

Example 2.5
Find the internal energy of 1 kg of superheated steam at a pressure of 10 bar and 300°C. If this
steam is expanded to 1.5 bar and 0.9 dryness, then find the change in internal energy.

Solution
From steam table (Appendix A.1.3), at p = 10 bar and tsup = 300°C, for superheated steam,
u1 = 2793.2 kJ/kg
For dry saturated steam, at p = 1.5 bar, we have from table A.1.2 in the Appendix
uf = 466.92 kJ/kg, ufg = 2052.7 kJ/kg,
u2 = uf 2 + x2ufg2 = 466.92 + 0.9 × 2052.7 = 2314.35 kJ/kg
∆u = u2 − u1 = 2314.35 − 2793.2 = − 478.85 kJ/kg

M02_THERMAL ENGINEERING_SE_XXXX_CH02.indd 69 6/15/2018 1:04:05 PM


70 Chapter 2

Example 2.6
A rigid vessel is initially divided into two parts A and B by a thin partition. Part A contains 1 kg of
steam at 4 bar, dry and saturated, and part B contains 2 kg of steam at 8 bar with dryness fraction of
0.90. The partition is removed and the pressure in the vessel after sometimes is found to be 6 bar.
Find (a) the volume of the vessel and the state of steam and (b) the amount of heat transfer from
steam to the vessel and the surroundings.

Solution
(a) At  pA = 4 bar, from steam table A.1.2 in the Appendix, vgA = 0.4625 m3/kg, uA = 2553.6 kJ/kg
VA = mAvgA = 1 × 0 .4625 = 0.4625 m3
At pB = 8 bar, from steam table A.1.2 in the Appendix, vf B = 0.001115 m3/kg, vgB = 0.2404 m3/kg
                                                   

VB = mB ( vf B + xBvgB ) = 2 × [0.001115 + 0.9 × (0.2404 − 0.001115)] = 0.433 m3


Volume of vessel, V = VA + VB = 0.4625 + 0.433 = 0.8955 m3
At p = 6 bar, from steam table A.1.2 in the Appendix, vf = 0.001101 m3/kg, vg = 0.3157 m3/kg,
m = 1 + 2 = 3 kg
Dryness fraction at 6 bar, x can be found to be
V
v = = v f + x( v g − v f )
m
0.8955
or = 0.001101 + x ( 0.3157 − 0.001101)
3
or x = 0.945
(b) Now UA = mAuA = 1 × 2553.6 = 2553.6 kJ
UB = mB (hfB + xBhfgB − pBxBvgB × 102)
= 2 (721.1 + 0.9 × 2048.0 − 8 × 0.9 × 0.240 × 102)
= 4783.0 kJ
At 6 bar,  U = m[hf + x hfg − pxvg × 102]
= 3[670.6 + 0.944 × 2086.3 − 6 × 0.944 × 0.316 × 102)
= 7383.25 kJ
Q = U − (UA + UB)
= 7383.25 − (2553.3 + 4783.0) = 46.95 kJ

Example 2.7
Calculate enthalpy and entropy for 5 kg of steam at 8 bar under the following conditions:
(a) dry and saturated, (b) wet steam having wetness of 40%, and (c) superheated steam at 250°C.

Solution
From steam table (Appendix A.1.2), at p = 8 bar, we have

M02_THERMAL ENGINEERING_SE_XXXX_CH02.indd 70 6/15/2018 1:04:06 PM


Properties of Steam 71

hf = 721.1 kJ/kg, hfg = 2048.0 kJ/kg, hg = 2769.1 kJ/kg, sf = 2.0461 kJ/kgK, sfg = 4.6166 kJ/kgK,
sg = 6.6627 kJ/kgK
At tsup = 250°C and p = 8 bar, we have from table A.1.3
h = 2950.0 kJ/kg, s = 7.0384 kJ/kgK
(a) H = m hg = 5 × 2769.1 = 13845.5 kJ
S = m sg= 5× 6.6627 = 33.3135 kJ/K
(b) x = 1 − 0.4 = 0.6
H = m (hf + x hfg) = 5(721.1 + 0.6 × 2048.0) = 9749.5 kJ
S = m (sf + xsfg) = 5(2.0461 + 0.6 × 4.6166) = 24.08 kJ/K
(c) H = mh = 5 × 2950.0 = 14750.0 kJ
S = ms = 5 × 7.0384 = 35.192 kJ/K

Example 2.8
About 1 kg of steam 0.85 dry at a pressure of 1 bar and dryness fraction 0.85 is compressed in a
cylinder to a pressure of 2 bar according to pv1.25 = const. Determine the final condition of steam
and the heat transfer through the cylinder walls.

Solution
From steam table (Appendix A.1.2), at p = 1 bar, vf 1 = 0.001043 m3/kg, vg1 = 1.694 m3/kg
uf 1 = 417.33 kJ/kg, ufg1 = 2088.7 kJ/kg
Specific volume of steam
v = vf + x (vg − vf ) = 0.001043 + 0.85 × (1.694 − 0.001043) = 1.44 m3/kg
Now p1v1n = p2v2n
1 1
 p n  1  1.25
v2 =  1  × v1 =   × 1.44 = 0.8271
 2
p  2
At p2 = 2 bar, vf 2 = 0.001061 m3/kg, vg2 = 0.8857 m3/kg, uf 2 = 504.47 kJ/kg, ufg2 = 2025 kJ/kg
Now v2 = 0.8271 = vf 2+ x2 (vg2 − vf 2) = 0.001061 + x2 (0.8857− 0.001061)
or x2 = 0.934
Now u1 = uf 1 + x1ufg1 = 417.33 + 0.85 × 2088.7 = 2192.72 kJ/kg
u2 = uf 2 + x2ufg2 = 504.47 + 0.934 × 2025 = 2395.82 kJ/kg
∆u = u2 − u1 = 2395.82 − 2192.72 = 203.1 kJ/kg
Work done,
(1 × 1.44 − 2 × 0.8271) × 10 2
W= = − 85.68 kJ/kg
1.25 − 1
Heat transfer through the cylinder walls,
q = w + (u2 − u1) = − 85.68 + 203.1 = 117.42 kJ/kg
Example 2.9
Steam at a pressure of 10 bar and 0.9 dryness fraction expands to 1 bar hyperbolically. Find
(a) work done and heat absorbed and (b) internal energy and change in enthalpy.
Take specific heat of steam at constant pressure = 2 kJ/kg.K.

M02_THERMAL ENGINEERING_SE_XXXX_CH02.indd 71 6/15/2018 1:04:08 PM


72 Chapter 2

Solution
From steam table (Appendix A.1.2), we have

p bar hf kJ/kg hfg kJ/kg vf m3/kg vg m3/kg


10 762.79 2015.3 0.001127 0.19444
1 417.44 2258.0 0.001043 1.694

v1 = vf 1+ x1 (vg1 − vf 1) = 0.001127 + 0.9 × (0.19444 − 0.001127) = 0.1751 m3/kg


Now, p1v1 = p2v2
v2 p
= 1 = 10
v1 p2
v2
Expansion ratio, r = = 10
v1
Work done, w = p1v1 ln r = 10 × 102 × 0.1751 ln 10 = 403.18 kJ/kg
v2 = 10v1 = 10 × 0.1751 = 1.751 m3/kg
Now, v2 > vg2. Therefore, the steam is superheated at p2 = 1 bar.
Using linear interpolation, we have
Tsup 2 − 100 1.751 − 1.6958
=
150 − 100 1.9364 − 1.6958
Tsup 2 = 111.47°C
h1 = hf 1 + x1hfg1 = 762.79 + 0.9 × 2015.3 = 2576.56 kJ/kg
Using linear interpolation, we get
h2 − 2676.2 111.47 − 100
=
2776.4 − 2676.2 150 − 100
h2 = 2699.19 kJ/kg
∆h = h2 − h1 = 2699.19 − 2576.56 = 122.63 kJ/kg
∆u = ∆h for hyperbolic process
q = w + ∆u = 403.18 + 122.63 = 525.81 kJ/kg

Example 2.10
Steam, which is initially dry saturated, expands isentropically from pressure of 15 bar to 0.15 bar.
Find the index of isentropic expansion.

Solution
From steam table (Appendix A.1.2), we have
p bar vf m3/kg vg m3/kg sf kJ/kg.K sfg kJ/kg.K
15 0.001154 0.13177 2.315 4.130
0.15 0.001014 10.022 0.7548 7.2536

M02_THERMAL ENGINEERING_SE_XXXX_CH02.indd 72 6/15/2018 1:04:09 PM


Properties of Steam 73

For isentropic expansion, s1 = s2


s1 = sf 1 + sfg1 = 2.315 + 4.130 = 6.445 kJ/kg.K
s2 = sf 2 + x2sfg2 = 0.7548 + x2 × 7.2536 = 6.445
or x2 = 0.7845
v1 = vg1 = 0.13177 m3/kg
v2 = vf 2+ x2 (vg2 − vf 2) = 0.001014 + 0.7845 × (10.022 − 0.001014) = 7.8625 m3/kg
Now, p1v1n = p2 v2n
n
v  p
or  1  = 2
 2
v p1
n
 0.13177  0.15
or  =
 7.8625  15

or n ln 
0.13177 
= ln 
1 
 7.8625   100 
or n = 1.126

Example 2.11
Steam at a pressure of 8 bar and 0.95 dry is expanded to a pressure of 1.8 bar. Find the final condi-
tion of steam and the heat drop for each of the following methods of expansion:
(a) Adiabatic expansion
(b) Throttling expansion

Solution
Given that p1 = 8 bar, x1 = 0.95, p2 = 1.75 bar
From steam tables (Appendix A.1.2), at 8 bar, sf 1 = 2.0461 kJ/kg.K, sfg1 = 4.6166 kJ/kg.K,
hf 1 = 721.1 kJ/kg, hfg1 = 2048 kJ/kg, and at 1.75 bar, sf 2 = 1.4848 kJ/kg.K, sfg2 = 5.6868 kJ/kg.K,
hf 2 = 486.97 kJ/kg, hfg2 = 2213.6 kJ/kg
(a) During an adiabatic expansion, s1 = s2
sf 1 + x1sfg1 = sf 2 + x2sfg2
2.0461 + 0.95 × 4.6166 = 1.4848 + x2 × 5.6868
x2 = 0.8699
(b) During throttling process, h1 = h2
hf 1 + x1hfg1 = hf 2 + x2hfg2
721.1 + 0.95 × 2048.0 = 486.97 + x2 × 2213.6
x2 = 0.9847

Example 2.12
Steam at 10 bar flows into a barrel containing 150 kg of water at 10°C. The final temperature of
water is 25°C and the increase in mass is 4 kg. Calculate the dryness fraction of steam. Take cpw =
4.187 kJ/kg.K.

M02_THERMAL ENGINEERING_SE_XXXX_CH02.indd 73 6/15/2018 1:04:11 PM


74 Chapter 2

Solution
Given that ms = 4 kg, p = 10 bar, mw = 150 kg, tw1 = 10°C, tw2 = 25°C, and cps = 4.187 kJ/kg.K
From steam table (Appendix A.1.2), at 10 bar, hfg = 2015.3 kJ/kg and ts = 179.91°C
Heat lost by steam = Heat gained by water
ms[x hfg + cpw(ts − tw2)] = mw (tw2 − tw1)cpw
4[x × 2015.3 + 4.187(179.91 − 25)] = 150 (25 − 10) × 4.187
Dryness fraction of steam, x = 0.847

Example 2.13
In a separating and throttling calorimeter, the steam main pressure is 8 bar absolute and the tem-
perature after throttling is 130°C. The pressure in the throttling calorimeter is 0.001 bar gauge, and
the barometer reading is 75 cm. If 0.15 kg of water is trapped in the separator and 1.8 kg of steam
is passed through the throttling calorimeter, then determine the dryness fraction of steam in the
steam main.

Solution
After throttling, p2 = 0.001 + 0.75 × 9.81 × 13600 × 10−5 = 1.01 bar
The saturation temperature for p2 = 1.01 bar, ts2 = 99.62°C
Degree of superheat = tsup − ts2 = 130 − 99.62 = 30.38°C
For superheated steam, from steam tables (Appendix A.1.3), for tsup = 130°C
p = 1.01 bar, we have by linear interpolation
hsup2 = 2736.32 kJ/kg
From steam tables (Appendix A.1.2),
At 8 bar hf 1 = 721.1 kJ/kg, hfg1 = 2048 kJ/kg
h1 = hf 1 + x1hfg1
Now h1 = hsup2
721.1 + x1 + 2048.0 = 2736.32
x1 = 0.984
ms
x2 =
ms + mw
1.8
= = 0.923
1.8 + 0.15
Dryness fraction, x = x1x2
= 0.984 × 0.923 = 0.908

Example 2.14
Calculate the enthalpy of 1 kg of steam at a pressure of 8 bar and dryness fraction of 0.8. How much
heat would be required to raise 2 kg of this steam from water at 20°C?
Solution
Given that ms = 1 kg, p = 8 bar, x = 0.8, and tw = 20°C
Enthalpy of 1 kg of steam:
From steam table (Appendix A.1.2), corresponding to a pressure of 8 bar, we find that,

M02_THERMAL ENGINEERING_SE_XXXX_CH02.indd 74 6/15/2018 1:04:12 PM


Properties of Steam 75

hf = 721.1 kJ/kg and hfg = 2048 kJ/kg


Enthalpy of 1 kg of wet steam,
h = hf + x hfg = 721.1 + 0.8 × 2048 = 2359.5 kJ
Heat required to raise 2 kg of this steam from water at 20°C:
We have calculated above the enthalpy or total heat required to raise 1 kg of steam from water at
0°C. Since the water, in this case, is already at 20°C, therefore,
Heat already in water = 4.2 × 20 = 84 kJ
Heat required per kg of steam = 2359.5 − 84 = 2275.5 kJ
and heat required for 2 kg of steam = 2 × 2275.5 = 4551 kJ

Example 2.15
Determine the quantity of heat required to produce 1 kg of steam at a pressure of 6 bar at a temper-
ature of 25°C, under the following conditions:
(a) When the steam is wet having a dryness fraction 0.9
(b) When the steam is dry saturated
(c) When it is superheated at a constant pressure at 250°C, assuming the mean specific heat of
superheated steam to be 2.3 kJ/kg.K.

Solution
Given that p = 6 bar, tw = 25°C, x = 0.9, tsup = 250°C, and cps = 2.3 kJ/kg.K
From steam table (Appendix A.1.2), corresponding to a pressure of 6 bar, we find that
hf = 670.4 kJ/kg, hfg = 2086.3 kJ/kg, and ts = 158.85°C
(a) When the steam is wet:
Enthalpy or total heat of 1 kg of wet steam,
h = hf + xhfg = 670.54 + 0.9 × 2086.3 = 2548.21 kJ
Since the water is at a temperature of 25°C, therefore from steam tables (Appendix A.1.1)
Enthalpy of water 104.87 kJ
∴ Heat actually required = 2548.21 − 104.87 = 2443.34 kJ
(b) When the steam is dry saturated:
From steam tables (Appendix A.1.2), enthalpy or total heat of 1 kg of dry saturated steam,
 hg = 2756.8 kJ
∴ Heat actually required = 2756.8 − 104.87 = 2651.93 kJ
(c) When the steam is superheated:
From steam tables (Appendix A.1.3), enthalpy or total heat of 1 kg of superheated steam,
hsup = 2957.2 kJ
∴ Heat actually required = 2957.2 − 104.87 = 2852.33 kJ

Example 2.16
Steam enters an engine at a pressure of 12 bar with a 67°C of superheat. It is exhausted at a pressure
of 0.15 bar and 0.95 dry. Find the drop in enthalpy of the steam. Assume specific heat of super-
heated steam cps = 2.1 kJ/kgK.

M02_THERMAL ENGINEERING_SE_XXXX_CH02.indd 75 6/15/2018 1:04:12 PM


76 Chapter 2

Solution
Given that p1 = 12 bar, tsup − ts = 67°C, p2 = 0.15 bar, and x = 0.95
From steam table (Appendix A.1.2), corresponding to a pressure of 12 bar, we find that
hf = 798.64 kJ/kg; hfg = 1986.2 kJ/kg ; hg = 2784.8 kJ/kg
Enthalpy or total heat of 1 kg of superheated steam,
hsup = hg + cps (tsup − ts)
= 2784.8 + 2 × 67 = 2925.5 kJ/kg
Similarly, from steam tables (Appendix A.1.2), corresponding to a pressure of 0.15 bar
hf = 225.91 kJ/kg; hfg = 2373.1 kJ/kg
Enthalpy or total heat of 1 kg of wet steam,
h = hf + xhfg = 225.91 + 0.95 × 2373.1 = 2480.36 kJ/kg
\ Drop in enthalpy of the steam = hsup − h = 2925.5 − 2480.36 = 445.14 kJ/kg

Example 2.17
A steam engine obtains steam from a boiler at a pressure of 15 bar and 0.98 dry. It was observed
that the steam loses 21 kJ of heat per kg as it flows through the pipe line and pressure remaining
constant. Calculate the dryness fraction of the steam at the engine end of the pipeline.

Solution
Given that p = 15 bar, x = 0.98, and heat loss = 21 kJ/kg
From steam table A.1.2 in the Appendix, corresponding to a pressure of 15 bar we find that
hf = 844.87 kJ/kg; hfg = 1947.3 kJ/kg
Enthalpy of wet steam at the boiler end.
h = hf + xhfg = 844.87 + 0.98 × 1947.3 = 2753.22 kJ/kg
Since the steam loses 21 kJ/kg of steam, therefore enthalpy of wet steam at the engine end,
h2 = 2753.22 − 21 = 27332.22 kJ
Let x2 = Dryness fraction of steam at the engine end.
Since the pressure remains constant, therefore hf and hfg is same. We know that
h2 = hf + x2hfg
2732.22 = 844.87 + x2 × 1947.3
or     x2 = 0.9692

Example 2.18
A coal fired boiler plant consumes 400 kg of coal per hour. The boiler evaporates 3200 kg of water
at 45°C into superheated steam at a pressure of 12 bar and 274.5°C. If the calorific value of the fuel
is 32600 kJ/kg of coal, then determine the following:
(a) Equivalent evaporation
(b) Efficiency of boiler
Assume specific heat of superheated steam as 2.1 kJ/kg.K.

M02_THERMAL ENGINEERING_SE_XXXX_CH02.indd 76 6/15/2018 1:04:12 PM


Properties of Steam 77

Solution
Given that mf = 400 kg/h, ms = 3200 kg/h, feed water temperature, t1 = 45°C, p = 12 bar,
Tsup = 274.5°C, C.V. = 32600 kJ/kg, cps = 2.1 kJ/kg.K
From steam table A.1.2 in the Appendix, at 12 bar, we have
Ts = 187.99°C, hg = 2784.2 kJ/kg
From steam table A.1.3 in the Appendix, using linear interpolation, enthalpy of superheated steam
at exit of boiler (12 bar and 274.5°C) is found to be
hsup = 2989.29 kJ/kg
From steam table A.1.1 in the Appendix, enthalpy of water at 45°C,
hf1 = 188.42 kJ/kg
(a) Equivalent evaporation:
ms ( hsup − h f 1 ) 3200( 2989.29 − 188.42)
me = = = 9.928 kg/kg of coal
m f × 2257 400 × 2257
(b) Boiler efficiency:
ms ( hsup − h f 1 ) 3200( 2989.29 − 188.42)
h= = = 0.6873 or 68.73%
m f × C.V. 400 × 32600

Example 2.19
During a trial on an oil-fired smoke tube boiler for 1 h, the following data were recorded:
Steam pressure, p = 14 bar
Amount of water evaporated, m1= 5400 kg
Condition of steam, x = 0.92 dryness fraction
Amount of fuel burnt, mf = 540 kg
Calorific value of fuel used, C.V. = 42000 kJ/kg
Temperature of steam leaving the superheater, Tsup = 250°C
Temperature of feed water, t1 = 50°C
Determine the equivalent evaporation from and at 100°C with and without superheater, boiler
efficiency, and the percentage of heat utilised in the superheater.

Solution
Given that p = 14 bar, m1 = 5400 kg, x = 0.92, mf = 540 kg, C.V. = 42000 kJ/kg, Tsup = 250°C, and
t1 = 50°C
Amount of steam generated per kg of fuel,
m 5400
m= 1 = = 10 kg/kg of fuel
mf 540
Enthalpy of wet steam at 15 bar, 0.92 dry,
h = hf + x.hfg = 845.2 + (0.92 × 1947.2) = 2636.6 kJ/kg
From steam table A.1.2 in the Appendix, enthalpy of wet steam at 14 bar, 0.92 dry,
h = hf + x.hfg = 830.29 + (0.92 × 1959.7) = 2633.2 kJ/kg
From steam table A.1.3 in the Appendix, Enthalpy of superheated steam at 14 bar, 250°C,
hsup = 2927.2 kJ/kg

M02_THERMAL ENGINEERING_SE_XXXX_CH02.indd 77 6/15/2018 1:04:14 PM


78 Chapter 2

Equivalent evaporation without superheated from and at 100°C.


m( h − 4.19t ) 10( 2633.2 − 4.19 × 50)
me = = = 10.739 kg/kg of fuel
2257 2257
Equivalent evaporation with superheater from and at 100°C.
m( hsup − 4.19t ) 10( 2927.2 − 4.19 × 50)
me = = = 12.04 kg/kg of fuel
2257 2257
m( hsup − 4.19t )
Boiler efficiency, h =
C.V.
10( 2927.2 − 4.19 × 50)
h= = 0.6471 or 64.71%
42000
Percentage of heat utilized in the superheater,
m( hsup − h) 10( 2927.2 − 2633.2)
= × 100 = × 100 = 0.07 or 7.0 %
C.V. 42000

Example 2.20
Following particulars refer to a steam power plant consisting of a boiler, superheater, and an
economiser:
Steam pressure, p = 20 bar
Mass of steam generated, m1 = 10000 kg/h
Mass of coal used, mf = 1300 kg/h
Calorific value of coal, C.V. = 29000 kJ/kg
Temperature of feed water entering the economiser, t1 = 35°C
Temperature of feed water leaving the economiser, t2 = 105°C
Temperature of superheated steam leaving the superheater, tsup = 350°C
Dryness fraction of steam leaving the boiler, x = 0.98
Determine the following:
(a) Overall efficiency of the plant
(b) Equivalent evaporation from and at 100°C
(c) The percentage of heat utilised in boiler, superheater, and economiser
Solution
Given that p = 20 bar, m1 = 10000 kg/h, mf = 1300 kg/h, C.V. = 29000 kJ/kg, t1 = 35°C, t2 = 105°C,
tsup = 350°C, x = 0.98
From steam table A.1.1 in the Appendix, enthalpy of feed water entering the economiser,
hf 1 = 146.66 kJ/kg
Enthalpy of feed water leaving the economiser at 105°C,
hf 2 = 440.13 kJ/kg
From steam table A.1.2 in the Appendix, enthalpy of steam leaving the boiler,
h = hf + x.hfg = 908.77 + 0.98 × 1890.7 (at 20 bar, 0.98 dry)
= 2761.66 kJ/kg

M02_THERMAL ENGINEERING_SE_XXXX_CH02.indd 78 6/15/2018 1:04:17 PM


Properties of Steam 79

From steam table A.1.3 in the Appendix, enthalpy of steam leaving the superheater,
hsup = 3137.0 kJ/kg (at 20 bar, 350°C)
Mass of steam generated per kg of fuel,
m1 10000
m= = = 7.691 kg/kg of fuel
mf 1300
(a) Overall efficiency of the plant:
m( hsup − h f 1 )
h=
C.V.
7.691(3137 − 146.66)
h= = 0.7931 or 79.31%
29000
(b) Equivalent evaporation from and at 100°C:
m( hsup − h f 2 ) 7.691(3137 − 440.13)
me = = = 9.19 kg
2257 2257
(c) Percentage of heat utilised in the boiler:
Heat supplied in 1 kg of fuel = 29000 kJ
m( h − h f 2 ) 7.691( 2761.66 − 440.13)
= × 100 = × 100 = 0.6157 or 61.57%
C.V. 29000
Percentage of heat utilised in superheater:
m( hsup − h) 7.691(3137 − 2761.66)
= × 100 = × 100 = 0.0995 or 09.95%
C.V. 29000
Percentage of heat utilised in economiser:
m( h f 2 − h f 1 )7.691( 440.13 − 146.66)
= × 100 = × 100 = 0.0778 or 07.78%
C.V. 29000
Note: Superheater is considered as a part of the boiler for calculation of equivalent evaporation.

Example 2.21
The following readings were recorded during a boiler trial of 6 h duration:
Mean steam pressure = 12 bar
Mass of steam generated = 40000 kg
Mean dryness fraction of steam = 0.85
Mean feed water temperature = 30°C
Coal used = 4000 kg
Calorific value of coal = 33400 kJ/kg
Calculate the following:
(a) Factor of equivalent evaporation
(b) Heat rate of boiler in kJ/hr
(c) Equivalent evaporation from and at 100°C
(d) Efficiency of the boiler

M02_THERMAL ENGINEERING_SE_XXXX_CH02.indd 79 6/15/2018 1:04:21 PM


80 Chapter 2

Solution
Given that p = 12 bar, m = 40000 kg, x = 0.85, t1 = 30°C, mf = 4000 kg, C.V. = 33400 kJ/kg, and
duration of trial, T = 6 h
From steam table A.1.2 in the Appendix at 12 barhf = 798.64 kJ/kg, hfg = 1986.2 kJ/kg
\ Enthalpy of steam, h = hf + x.hfg = 798.64 + 0.85 × 1986.2 = 2486.91 kJ/kg
From steam table A.1.1 in the Appendix, enthalpy of feed water at 30°C, hf 1 = 125.77 kJ/kg
(a) Factor of equivalent evaporation:

fe =
h − hf 1
=
( 2486.91 − 125.77) = 1.046
2257 2257
(b) Heat rate of boiler:
mf 4000
=
× C.V. = × 33400 = 2226666.7 kJ/h = 2226.67 MJ/h
T 6
(c) Equivalent evaporation from and at 100°C:
m( h − h f 1 ) 40000( 2486.91 − 125.77)
me = = = 10.46 kg/kg of coal
m f × 2257 4000 × 2257
(d) Boiler efficiency:
m( h − h f 1 ) 40000( 2486.91 − 125.77)
h= = = 0.7069 or 70.69%
m f × C.V. 4000 × 33400

Example 2.22
During a trial for 8 h on a boiler, the following data were obtained:
Pressure of steam leaving the boiler = 14 bar
Condition of steam = 0.97 dryness fraction
Steam produced = 26700 kg
Temperature of feed water = 50°C
Mass of coal fired = 4260 kg
Calorific value of coal = 28900 kJ/kg
Air fuel ratio = 17
Temperature of flue gases leaving the boiler = 344°C
Boiler house temperature = 21°C
Specific heat of flue gases = 1.1 kJ/kg.K
Determine the following:
(a) Boiler efficiency
(b) Equivalent evaporation
(c) Heat lost to flue gases expressed as percentage of heat supplied

Solution
Given that p = 14 bar, x = 0.97, m = 26700 kg, t1 = 50°C, mf = 4260 kg, C.V. = 28900 kJ/kg,
m
air−fuel ratio, a = 17, tf  = 344°C, t0 = 21°C, Cpg = 1.1 kJ/kg.K, and duration of boiler trial, T = 8 h
mf

M02_THERMAL ENGINEERING_SE_XXXX_CH02.indd 80 6/15/2018 1:04:23 PM


Properties of Steam 81

(a) From steam table A.1.2 in the Appendix, at 14 bar, hf = 830.29 kJ/kg and hfg = 1959.7 kJ/kg
Enthalpy of steam, h = (hf + x.hfg) at 14 bar = 830.29 + 0.97 × 1959.7 = 2731.2 kJ/kg
From steam table A.1.1 in the Appendix, enthalpy of water at 50°C , hf 1 = 209.31 kJ/kg
\ Efficiency of boiler,
m( h − h f 1 ) 26700( 2731.2 − 209.31)
h= = = 0.5469 or 54.69%
m f × C.V. 4260 × 28900
(b) Equivalent evaporation:
m( h − h f 1 ) 26700( 2731.2 − 209.31)
me = = = 7.003 kg/kg of coal
m f × 2257 4260 × 2257
mf 4260
(c) Fuel burnt/hour, m f = = = 532.5 kg/h
t 8
Heat supplied in fuel = m f × C.V. = 532.5 × 28900 = 15389250.0 kJ/h
Mass of flue gases formed/hour,
 m 
mg = m f 1 + 1  = 532.5 (1 + 17) = 9585.0 kg/h
 mf 
Heat carried away by flue gases,
= mg× cpg (tf − t0) = 9585 × 1.1 (344 − 21) = 3405550.5 kJ/h
Heat loss of flue gases as per cent of heat supplied
Heat carried away by flue gases 3405550.5
= × 100 = × 100 = 22.13%
Heat supplied in fuel 15389250.0

Example 2.23
A gas-fired boiler operates at a pressure of 100 bar. The feed water temperature is 255°C. Steam is
produced with a dryness fraction of 0.9, and in this condition, it enters a superheater. Superheated
steam leaves the superheater at a temperature of 450°C. The boiler generates 1200 tonne of steam
per hour with an efficiency of 92%. The gas used has a calorific value of 38 mJ/m3. Determine the
following:
(a) The heat transfer per hour in producing wet steam in the boiler
(b) The heat transfer per hour in producing superheated steam in the superheater
(c) The gas used in m3/hour

Solution
Given that p = 100 bar, t1 = 256°C, x = 0.9, tsup = 450°C, m = 1200 tonne = 1200 × 103 kg
h = 0.92, C.V. = 38 MJ/m3 = 38 × 103 kJ/m3
(a) From table A.1.1 in the Appendix, enthalpy of feed water: hf 1 = 1109.72 kJ/kg
From table A.1.2 in the Appendix, enthalpy of steam at 100 bar, 0.9 dry
h = hf + x.hfg = 1407.53 + 0.9 × 1317.1 = 2592.92 kJ/kg

M02_THERMAL ENGINEERING_SE_XXXX_CH02.indd 81 6/15/2018 1:04:28 PM


82 Chapter 2

From table A.1.3 in the Appendix, Enthalpy of superheated steam at 100 bar, 450°C
hsup = 3240.8 kJ/kg
∴ Heat transfer/h producing wet steam in boiler
= m(h − hf1) = 1200 × 103 (2592.92 − 1109.72)
= 17.7984 × 108 kJ/h
(b) Heat transfer/hour in producing superheated steam in superheater:
= m(hsup − h) = 1200 × 103 (3240.8 − 2592.92)
= 77.7456 × 107 kJ/h
(c) Let the gas used in m3/hour be Vf.
m( hsup − h f 1 )
h=
V f × C.V

1200 × 103 (3237.3 − 1072.6)


0.92 =
V f × 38 × 103
Vf = 73149.2 m3/h

Summary for Quick Revision

1. A pure substance may be defined as a system which is homogeneous in composition and chemical aggrega-
tion and invariable in chemical aggregation.
2. Properties of steam are as follows:
(a) Latent heat of steam, hfg = Quantity of heat required to convert 1 kg of liquid water at its boiling point
into dry saturated steam at the same pressure.
mass of dry steam, ms
(b) Dryness fraction, x =
mass of dry steam, ms + mass of water in steam, mw
Quality of steam = x × 100
Wetness fraction = 1 − x
(c) Enthalpy of wet steam, hwet = hf + x hfg
1
(d) Density of wet steam = kg/m3
v f + x(v g − v f )
1
Density of dry saturated steam, rg =
vg
Volume of wet steam, vwet = (1 − x)vf + xvg = vf + xvfg
(e) Internal energy of wet steam, uwet = u f + xu fg

(f) Entropy of wet steam, swet = s f + xs fg

M02_THERMAL ENGINEERING_SE_XXXX_CH02.indd 82 6/15/2018 1:04:30 PM


Properties of Steam 83

3. Steam processes are as follows:


(a)  Constant volume process,
v1 = v2; q = u2 − u1
(b)  Constant pressure process,
w = p (p2 − p1)
q = (x2 − x1) hfg, for wet steam
   = hsup − hg, for superheated steam.
(c)  Isothermal process,
In the wet region, q = h2 − h1 and w = p1 (v2− v1)
(d)  Hyperbolic process,
p1v1 = p2v2
(e)  Isentropic process,
s1 = s2
(f)  Polytropic process,
n = 1.13 for wet steam
= 1.3 for superheated steam
p1(v1)n = p2(v2)n
(g)  Throttling process,

h1 = h2
h1 = hf 1 + x1 hfg1 for wet steam
h2 = hf 2 + x2 hfg2 for wet steam
= hg + cps (Tsup − Ts) for superheated steam.
4. Determination of dryness fraction
(a)  Barrel calorimeter,
( mw + me )c pw (t 2 − t1 ) − msc ps (t s − t 2 )
x=
ms h fg
It gives approximate results which are always lower than the actual value.
(b)  Separating calorimeter,
ms
x=
ms + mw
The value of x calculated is always greater than the actual.
(c)  Throttling calorimeter,
( h f 2 − h f 1 ) + h fg 2 + c ps (tsup − t s 2 )
x1 =
h fg1
It is suitable for steam with high dryness fraction.
(d)  Combined separating and throttling calorimeter.
( h2 − h f 1 ) + c ps (tsup − t s 2 )
x1 =
h fg1
ms
x2 =
ms + mw
x = x1 x2
It is suitable for considerably wet steam.

M02_THERMAL ENGINEERING_SE_XXXX_CH02.indd 83 6/15/2018 1:04:32 PM


84 Chapter 2

Multiple-choice Questions

1. The dryness fraction of 1 kg of steam containing (a) a positive slope


0.8 kg of dry steam is (b) a negative slope
(a) 0.2 (b) 0.8 (c) zero slope
(c) 1.0 (d) 0.4 (d) both positive and negative slope
2. If x1 and x2 be the dryness fractions of steam 7. Specific volume of wet steam with dryness frac-
obtained in the separating and throttling calorim- tion x is
eters, respectively, then the dryness fraction is (a) xvf (b) vf + x vfg
(a) x1 + x2 (b) x1x2 (c) x2vg (d) x3vg
x +x
(c) x1 − x2 (d) 1 2 8. Entropy of wet steam is given by
2 (a) sf + x sfg (b) x sg
3. Superheating of steam is done at constant
(a) volume (b) pressure (c) sg + x sfg (d) x sf
(c) temperature (d) enthalpy 9. The phase change at constant pressure (or constant
4. With the increase of pressure, the enthalpy of temperature) from liquid to vapour is referred
evaporation of water to as
(a) decreases (a) melting
(b) increases (b) solidification
(c) remains same (c) sublimation
(d) changes randomly (d) vapourization

5. Only throttling calorimeter is used for measuring 10. The point that connects the saturated liquid line to
(a) very low dryness fraction up to 0.7 the saturated vapour line is called the
(b) very high dryness fraction up to 0.98 (a) triple point
(c) dryness fraction of only low pressure steam (b) critical point
(d) dryness fraction of only high pressure steam (c) superheated point
(d) saturated point
6. Constant pressure lines in the superheated region
of the Mollier diagram have

Review Questions

1. Define a pure substance.


2. Define latent heat of fusion and latent heat of vapourisation.
3. Define dryness fraction and wetness of steam.
4. What is quality of steam?
5. Write expressions for internal energy of steam when it is wet and when it is superheated.
6. Write expressions for entropy of steam for wet and superheated.
7. What is a Mollier diagram?
8. When steam is heated at constant volume and its end condition is still wet, what is heat transfer?
9. In the wet region, constant temperature process is also a constant pressure process. Say true or false.
10. Hyperbolic process is also an isothermal process in the superheat region. Say true or false.
11. What remains constant during isentropic process?
12. What is throttling of steam?
13. What are the drawbacks of barrel calorimeter?
14. Which calorimeter is used for a very wet steam?

M02_THERMAL ENGINEERING_SE_XXXX_CH02.indd 84 6/15/2018 1:04:33 PM


Properties of Steam 85

Exercises

2.1 A vessel of 1.35 m3 capacity is filled with steam at 13 bar absolute and 95% dry. The vessel and its contents
cool until the pressure is 2 bar absolute. Calculate the mass of the contents in the vessel and the dryness
fraction of steam after cooling. Neglect the volume of water present. [Ans. 9.2 kg, 0.1625]
2.2 Steam at a pressure of 5 bar and temperature 200°C is expanded adiabatically to a pressure of 0.7 bar abso-
lute. Determine the final condition of steam. For superheated steam, cps = 2.2 kJ/kg.K. [Ans. 0.93]
2.3 Steam at a pressure of 20 bar absolute and dryness fraction 0.8 is throttled to a pressure of 0.5 bar. Deter-
mine the final condition of steam. [Ans. 0.905]
2.4 About 2 kg of wet steam at 10 bar and 90% dry is expanded according to the law pv = constant to a pressure
of 1 bar. Determine the final condition of steam and the change in internal energy.
[Ans. Superheated to 112°C, 257 kJ]
2.5 While conducting the dryness fraction test with a throttling calorimeter, it was found that the entering steam
was at a pressure of 12 bar and a sample after being reduced to 1 bar in the calorimeter was at a temperature
of 120°C. Estimate the dryness fraction of steam assuming cps = 2.1 kJ/kg.K. [Ans. 0.965]
2.6 In a test with a separating and throttling calorimeter, the following observations were made:
Water separated = 2.04 kg
Steam discharged from throttling calorimeter = 20.6 kg at 150°C
Initial pressure of steam = 12 bar absolute
Final pressure of steam = 12.3 cm of mercury above atmospheric pressure
Barometer = 76 cm of mercury
Calculate the dryness fraction of steam entering the calorimeter. [Ans. 0.905]
2.7 Determine the change in internal energy when 1 kg of steam expands from 10 bar and 300°C to 0.5 bar and
0.9 dry. Take cps = 2.1 kJ/kg.K. [Ans. − 669 kJ/kg]
2.8 One kg of steam at 10 bar exists at 200°C. Calculate the enthalpy, specific volume, density, internal energy,
and entropy. Take cps = 2.1 kJ/kg.K.
[Ans. 2818 kJ/kg, 0.203 m3/kg, 4.93 kg/m3, 2615 kJ/kg, 6.673 kJ/kg.K]
2.9 A vessel contains one kg of steam which contains one-third liquid and two-third vapour by volume. The
temperature of steam is 151.86°C. Calculate the quality, specific volume, and specific enthalpy of the mix-
ture. [Ans. 0.0576, 0.003245 m3/kg, 650.5 kJ/kg]
2.10 A cylinder fitted with a piston contains 0.5 kg of steam at 4 bar. The initial volume of steam is 0.1 m3.
Heat is transferred to steam at constant pressure until the temperature becomes 300°C. Determine the heat
transferred and work done during the process. [Ans. 771 kJ, 91 kJ]
2.11 Steam at 10 bar and 0.9 dry initially occupies 0.35 m3. It is expanded according to the law pv1.25 = con-
stant until the pressure falls to 2 bar. Determine the mass of steam used in the process, the work done,
the change in internal energy, and the heat exchange between steam and surroundings.
[Ans. 2 kg, 192 kJ, −896 kJ, −704 kJ]
2.12 One kg of steam at 8.5 bar and 0.95 dry expands adiabatically to a pressure of 1.5 bar. The law of expansion
is pv1.2 = constant. Determine the final dryness fraction of steam and the change in internal energy during
the expansion. [Ans. 0.792, −230 kJ/kg]

M02_THERMAL ENGINEERING_SE_XXXX_CH02.indd 85 6/15/2018 1:04:33 PM


86 Chapter 2

2.13   A throttling calorimeter is used to measure the dryness fraction of steam in the steam main where the
steam is flowing at a pressure of 6 bar. The steam after passing through the calorimeter comes out at 100
kPa pressure and 120°C temperature. Calculate the dryness fraction of steam in the main. Assume cps =
2.09 kJ/kg.K. [Ans. 0.985]
2.14  A separating and throttling calorimeter was used to determine the dryness fraction of steam flowing through
a steam main at 900 kPa. The pressure and temperature after throttling were 105 kPa and 115°C, respec-
tively. The mass of steam condensed after throttling was 1.8 kg and mass of water collected in the separating
calorimeter was 0.16 kg. Determine the dryness fraction of steam flowing through the steam main. Take cps
= 2.09 kJ/kg.K. [Ans. 0.89]

ANSWERS TO MULTIPLE-CHOICE QUESTIONS


1. (b) 2. (b) 3. (b) 4. (a) 5. (b)
6. (a) 7. (b) 8. (a) 9. (a) 10. (a)

M02_THERMAL ENGINEERING_SE_XXXX_CH02.indd 86 6/15/2018 1:04:33 PM


Chapter 3

Steam Generators

3.1 ❐ INTRODUCTION
A steam generator, popularly known as a boiler, is a closed vessel made of high-quality steel in which
steam is generated from water by the application of heat. The water receives heat from the hot gases
formed by burning fuel through the heating surface of the boiler. Steam is mainly required for power
generation, process heating, and space heating purposes.

3.2 ❐ CLASSIFICATION OF STEAM GENERATORS


Steam generators are classified based on the following:
1. Water tube and fire tube: In a water tube boiler, water or steam flows through the tubes and
heat is supplied to the external surface. Babcock and Wilcox and Stirling boilers fall under this
category.
In a fire tube boiler, hot gases flow through the tubes and water circulates outside the tubes.
There may be one large tube surrounded by water as in a Cornish boiler, or two large tubes sur-
rounded by water in a big tube of water as in the case of Lancashire boiler. There may be many
smaller tubes through which hot gases pass and are surrounded by water as in the case of vertical
Cochran or locomotive boiler.
2. Natural/forced circulation: Natural circulation of water takes place by natural convection cur-
rents produced by application of heat as in the case of Lancashire boiler and Babcock and Wilcox
boiler.
In forced circulation, the fluid is forced ‘once through’ or controlled with partial recirculation
as in the case of LaMont boiler, Velox boiler, and Benson boiler.
3. Horizontal, vertical, or inclined: The disposition on the principal axis of the boiler decides
whether the boiler is horizontal, vertical, or inclined.
4. Single/multiple tube(s): A boiler may have only one fire/water tube or multiple tubes.
5. Stationary/mobile: Boilers are called stationary (land) or mobile (marine and locomotive). Sta-
tionary boilers are used for power plant steam generation. Mobile boilers are portable boilers.
6. Internally/externally fired: A boiler is said to be an external combustion boiler when combus-
tion takes place outside the region of boiling water. A boiler is said to be an internal combustion

M03_THERMAL ENGINEERING_SE_XXXX_CH03_(Part1).indd 87 6/15/2018 1:51:29 PM


88 Chapter 3

boiler if the furnace region is completely surrounded by water-cooled surface as in the case of
Lancashire boiler.
7. Source of heat: Boilers may be classified according to the source from which heat is supplied
to water for evaporation. For example, coal fired, oil fired, gas fired, electrical energy, or nuclear
energy.

3.3 ❐ COMPARISON OF FIRE TUBE AND WATER


TUBE BOILERS
Table 3.1 shows the comparison of fire tube and water tube boilers.

Table 3.1  Comparison of fire tube and water tube boilers

S. no. Criteria Fire tube boilers Water tube boilers


1. Raising of steam Less rapid More rapid

2. Suitability for large power Unsuitable because of the Suitable because of higher
plants lower rate of steam raised. rate of steam raised

3. Operating pressure ≤25 bar >125 bar

4. Floor area required per kg Less More


of steam

5. Reliability and cost Greater reliability and Require careful attention


low cost and maintenance costs are
higher

6. Suitability for rapid Suitable Unsuitable


changes in load

7. Damage due to failure in No damage to boiler as The boiler shall be


feed-water supply it contains large quantity overloaded
of water

8. Risk of damage to prop- Serious because of its Less serious


erty in case of bursting large water capacity

9. Transportation Very inconvenient Can be easily transported


because of large size and erected as various
parts can be separated

10. Chances of bursting Less More

11. Maintenance Difficult to maintain All parts are easily


accessible for cleaning,
inspection, and repairing

12. Treatment of feed water Not essential Essential

M03_THERMAL ENGINEERING_SE_XXXX_CH03_(Part1).indd 88 6/15/2018 1:51:29 PM


Steam Generators 89

3.4 ❐ REQUIREMENTS OF A GOOD BOILER


a good boiler should possess the following qualities:
1. It should be able to generate steam at the required pressure and quantity with minimum fuel
consumption.
2. Its initial cost, installation, maintenance, and operational cost should be as low as possible.
3. It should occupy a small floor area and should be light in weight.
4. It should be able to meet the fluctuating demands of steam without fluctuations in steam pressure.
5. All parts of the boiler should be easily accessible for cleaning, inspection, and maintenance.
6. It should have minimum joints to avoid leakage of steam.
7. Its erection time should be reasonable with minimum labour.
8. The velocities of water and flue gas should be high for better heat transfer with minimum pressure
drop through the system.
9. There should be no deposition of mud and foreign materials inside the surface and soot deposits
on the outside surface of heat-transferring parts.
10. The boiler should conform to safety precautions as laid down in the Boiler Regulations.

3.5 ❐ FACTORS AFFECTING BOILER SELECTION


The factors affecting boiler selection are as follows:
1. Power to be generated, that is, steam to be raised per hour
2. Working pressure
3. Initial capital investment
4. Facilities available for erection
5. Availability of floor area
6. Location of power house
7. Availability of fuel and water
8. Probable load factor
9. Operating and maintenance cost
10. Accessibility for inspection, cleaning, and maintenance

3.6 ❐ DESCRIPTION OF BOILERS


3.6.1  Fire Tube Boilers
Lancashire Boiler
It is a stationary, fire tube, horizontal straight tube, internally fired, natural circulation boiler. Its
normal working pressure range is up to 15 atmospheres and evaporative capacity up to approximately
8000 kg/h. The size varies from 7−9 m in length and 2−3 m in diameter.
The three sectional views of the Lancashire boiler are shown in Fig. 3.1. The gaseous products of
combustion formed as a result of burning of fuel on the grate move along the furnace tube or main flue.
They are deflected up by the brick wall bridge. After reaching the back of the main flue, they deflect
downwards and travel through the bottom flue. The bottom flues are situated below the water shell.

M03_THERMAL ENGINEERING_SE_XXXX_CH03_(Part1).indd 89 6/15/2018 1:51:29 PM


90 Chapter 3

1 4 21 2 3 8
5 20 16
6 19 19
7
8 8 22
11 15
13 13 23
8 19
Sludge 10
Blow 12
17
Down
18
x

13 14
16
12 15

16
13 14

x′

1. Dead weight safety valve connection


2. Low water and high steam safety valve connection
3. Manhole 4. Steam stop valve connection
5. Pressure gauge connection 6. Water level gauge connection
7. Feed check valve connection 8. Flue tubes for first pass
9. Boiler shell 10. Grate
11. Brick-wall bridge 12. Bottom flue
13. Side flue 14. Dampers
15. Main flue 16. Cleaning doors
17. Blow-off cock connection 18. Blow off pit
19. Gusset stays 20. Perforated feed pipe
21. Anti-priming device 22. Fusible plug
23. Ash pit

Figure 3.1  Lancashire boiler

These flue gases heat the lower portion of the shell. After travelling from the back to the front, these
gases bifurcate into two separate paths in the side flues. They travel from the front to the back again
in the main flue from where they discharge to the chimney and atmosphere. The flues are built of
ordinary brick-work and lined with refractory material or bricks.
Superheater
The superheater used with Lancashire boiler is shown in Fig. 3.2. The arrows show the path of the
gases, when the superheater is in commission. 
When superheated steam is to be produced, the dampers are closed and opened so that the flue
gases pass on to the superheater flue at the back and travel towards the bottom flue. In this case, valve 2
is closed and valves 3 and 4 are opened so that steam from the shell passes to the superheater and after

M03_THERMAL ENGINEERING_SE_XXXX_CH03_(Part1).indd 90 6/15/2018 1:51:31 PM


Steam Generators 91

2 1 4

3
6

1. Main steam pipe connection


2–4. Stop valves 8
5. Main pipe line
6. Steamheaders 7
7. Superheater tubes
8. Dampers 9
9. Superheater flue

Figure 3.2 Superheater

getting superheated, they are passed to the main pipe 1. If the superheated steam is not required, valve 2
is opened and valves 3 and 4 are closed and the damper to the superheater flue is closed and the other
damper is opened, thus bypassing the superheater and steam is passed through valve 2 to main pipe 1.

Cornish Boiler
In construction and appearance, it is similar to the Lancashire boiler except that here, instead of two
flue tubes, only one is used. Its capacity and working pressure range is low. Its shell is usually 4−8 m
in length and 1.25−1.75 m in diameter. Its working pressure is about 11.5 bar and can raise steam up
to 1350 kg/h.

Locomotive Boiler
The locomotive boiler shown in Fig. 3.3 is an internally fired horizontal multi-tubular portable fire
tube boiler. It is specially designed to cope up with sudden variations in demand of steam even at the
cost of efficiency. The pressure range is up to about 21 bar and can raise up to 55−70 kg per square
metre of heating surface per hour.

Cochran Boiler
The Cochran boiler, shown in Fig. 3.4, is a multi-tubular, internal furnace vertical fire tube boiler
having several horizontal fire tubes. Its normal size is about 2.75 m shell diameter and 6 m height of
the shell. The steaming capacity is about 3500 kg/h.
The flame and hot gases produced as a result of burning fuel on the grate, rise in the combustion
furnace which is dome shaped. The unburnt fuel, if any, will be deflected. The gases are also deflected
back and pass through the flue to the fire tubes from where they pass out to smoke box and hence to
chimney.
The water circulation is shown by arrows. Water flows down by cooler wall of the shell and rise up
past the fire tubes by natural circulation due to convection currents.

M03_THERMAL ENGINEERING_SE_XXXX_CH03_(Part1).indd 91 6/15/2018 1:51:31 PM


92 Chapter 3

11
0 19 20
12

13
15
16
14

4
17 10

18 7 6 2

5 21

1. Fire box 2. Grate 3. Fire hole


4. Brick-set bridge 5. Damper 6. Smoke tubes
7. Barrel or shell 8. Smoke box 9. Chimney
10. Blast pipe 11. Steam dome 12. Regulator
13. Lever for regulator 14. Superheater tubes 15. Header
16. Outlet pipe for superheated steam 17. Smoke box door 18. Feed check valve
19. Safety valve connection 20. Whistle connection 21. Ash pan

Figure 3.3  Locomotive boiler

1. Shell
2. Flue pipe 14 13
3. Combustion chamber
4. Smoke box
5. Chimney Manhole
6. Grate 5
12
7. Ash pit
8. Fire hole
9. Furnace 3
10. Feed check valve 4
connection
11. Internal feed pipe
12. Pressure gauge
connection
13. Water gauge connection 2
14. Safety valve connection 9
10 6 8
15. Steam stop valve connection
16. Blow-off valve connection 16 17
17. Damper 7
18. Fusible plug

Figure 3.4  Cochran boiler

M03_THERMAL ENGINEERING_SE_XXXX_CH03_(Part1).indd 92 6/15/2018 1:51:33 PM


Steam Generators 93

3.6.2  Water Tube Boilers


For pressures above 10 bar and capacities in excess of 7000 kg of steam per hour, the water tube boiler
is used almost exclusively.

Babcock and Wilcox Boiler


Figure 3.5 shows a stationary type Babcock and Wilcox boiler. It consists of a large number of paral-
lel tubes inclined at an angle which varies from 5° to 15° to the horizontal which connect the uptake
header with the downtake header. These are connected to the shell having a substantial quantity of
water in it. The uptake header is connected to the shell, through a short tube, whereas a long tube is
employed to connect the downtake header with the shell. The coal is fed through the fire hole on to the
chain grate stoker. The velocity of the chain is adjusted so as to ensure complete combustion of coal
by the time it reaches the other end of the grate. The flue gases first rise up, move down, and rise up
again due to the presence of the baffles. The hot water and steam moisture rise up through the uptake
header into the boiler shell, where steam separates from water and collects in the steam space. The
cold water flows down into the tubes through the downtake header. Hence, a continuous circulation of
water is maintained by the convection currents set up.
Superheater
A set of superheater tubes is provided to superheat steam which enters these tubes from the steam
space in the boiler shell. The superheated steam can be taken out to the steam stop valve, through the
steam pipe. The lower part of the downtake header has a mud box attached to it to collect the sedi-
ments. The damper chain controls the quantity of air flowing through the boiler. Two soot doors are
provided for internal cleaning of boiler.

5 12

1. Water tubes
11
2. Uptake header
6 15
3. Downtake header
4. Grate
5. Damper chain 2 9
6. Steam pipe
13
7. Soot doors 9
8. Mud box
9. Baffles
3
10. Damper
11. Shell
14 1 16
12. Stop valve
4 10
13. Superheater tubes
7
14. Fire hole 8
15. Tube
16. Side flue

Figure 3.5  Babcock and Wilcox boiler

M03_THERMAL ENGINEERING_SE_XXXX_CH03_(Part1).indd 93 6/15/2018 1:51:33 PM


94 Chapter 3

21 26
23
18 22 19
1, 2, 3. Steam drums 1 3
2
4, 5. Mud drums 20
6, 7, 8, 9. Bent tubes banks
10. Brick work
11. Fire door 28
12. Damper
13. Fire brick arch 24
15
14, 15, 16. Baffles
17. Water tubes to mud drums 16 7
18, 19. Steam tubes 14
29 25
20. Water tubes connecting 8
front and middle drums
21. Safety valve connection 9
22. Steam chest 30
23. Header
24. Superheater 17 13
25. Bottom box header for 4 5
superheater Sludge
26. Steam stop valve 27
27. Blow down valve 12 Blow down
28, 29, 30. Inspection and 11
cleaning doors.

Figure 3.6  Stirling boiler

Stirling Boiler
Figure 3.6 shows the Stirling boiler, which comes in a water tube type in which three steam drums are
connected together by the banks of bent water tubes. These tubes stand nearly in a vertical position
near the rear end while they slope steeply over the fire at the front end. The baffles deflect the products
of combustion from the furnace over the banks of tubes into the mud drum. The steam generated
collects in the steam spaces of the three drums. The steam can be raised at a working pressure of
60 bar with the temperature as high as 450°C. An evaporation capacity of 50,000 kg/h has been
achieved with a Stirling boiler.

3.7 ❐ HIGH PRESSURE BOILERS


In all modern power plants, boilers raising steam at pressures greater than 100 bar are universally
used. These are called high pressure boilers. They offer the following advantages:
1. The efficiency and the capacity of the plant can be increased as reduced quantity of steam is
required for the same power generation if high pressure steam is used.
2. The forced circulation of water through the boiler tubes provides freedom in the arrangement of
furnace and water walls in addition to the reduction in the heat exchange area.

M03_THERMAL ENGINEERING_SE_XXXX_CH03_(Part1).indd 94 6/15/2018 1:51:34 PM


Steam Generators 95

3. The tendency of scale formation is reduced due to high velocity of water.


4. The danger of overheating is reduced as all the parts are uniformly heated.
5. The differential expansion is reduced due to uniform temperature and this reduces the possibility
of gas and air leakages.

3.7.1  Boiler Circulation


There are four types of boiler circulation as follows.
1. Natural circulation: Water circulates naturally in a boiler when its density in one part of the
circuit is less than that in another part at the same level. The boiler drums, tubes, and water wall
make up the multi-passage circuit. During steam formation in a simple drum, the water near the
sides of the drum gets heated and forms steam bubbles, lowering its density. The water in the
drum centre being heavier displaces the lighter water-bubble mix and, in turn, gets heated up. This
sets up a constant circulation that releases steam into the upper drum region.
  Similarly, in a water tube circuit, the water from the drum flows through the downcomer tube
to the bottom of the heated riser. Heat forms steam bubbles in water passing through the riser,
lowering the density of the mixture. The heavier downcomer water displaces the riser mixer to
establish continuous flow. The force of gravity to produce flow in a natural circulation comes
from the difference between the densities of the fluids in the downcomer and riser portions of the
circuit. The natural circulation is depicted in Fig. 3.7(a).
2. Forced circulation: Forced circulation in various circuits of boiler units is produced by mechan-
ical pumps as depicted in Fig. 3.7(b).
3. Once-through forced circulation: This type of circulation, as depicted in Fig. 3.7(c), receives
water from the feed supply, pumping it to the inlet of heat-absorbing circuit. Fluid heating and
steam generation take place along the length of the circuit until evaporation is completed. Further
process through the heated circuits results in superheating the vapour.
4. Once-through with recirculation (forced): The ‘recirculating’ forced-circulating type unit has
water supplied to the heat absorbing circuits through a separate circulating pump. The water
pumped is considerably in excess of steam produced and requires a steam-and-water drum for
steam generation. This type of system is depicted in Fig. 3.7(d).

3.7.2  Advantages of Forced Circulation Boilers


The advantages of forced circulation boilers are as follows:
1. Smaller bore and therefore, lighter tubes
2. Absence from scaling troubles due to high circulation velocity
3. Lighter for a given output
4. Steam can be raised quickly and load fluctuations met rapidly
5. Uniform heating of all parts eliminates the danger of overheating
6. Greater flexibility in layout of boiler parts
7. Boiler can be operated at desired conditions
8. Boiler can be started rapidly

M03_THERMAL ENGINEERING_SE_XXXX_CH03_(Part1).indd 95 6/15/2018 1:51:34 PM


96 Chapter 3

Economizer Economizer

Steam
Furnace
drum
walls
Furnace
walls Orifice
Circulating Distributing
pump header
(a) (b)

Economizer Economizer

Mixing
vessel

Circulating Orifice
pump
Distributing
Condenser vessel
Feedwater pump
(c) (d)

Figure 3.7  Types of boiler circulation: (a) natural circulation, (b) forced circulation,
(c) once-through, (d) once-through with recirculation (force)

3.7.3  LaMont Boiler


This is a forced circulation boiler whose arrangement of water circulation and different components
are shown in Fig. 3.8. Small diameter tubes are used in the evaporating section which gives flexibil-
ity in placing the heat transfer surfaces. The drum may be placed well away from the furnace. These
boilers have been built to generate 45−50 tonnes of superheated steam per hour at a pressure of
150 bar, 500°C.
The feed water from the hot well is supplied to the boiler through the economiser. A pump circu-
lates the water through the evaporator tubes and a part of the vapour is separated in the separator drum.
The centrifugal pump delivers the water to the headers at a pressure above the drum pressure which
distributes the water through the nozzle into the evaporator. The steam separated in the boiler is further
passed through the superheater and then supplied to the prime mover.
Feed pump

Economiser
Superheater
Storage Steam outlet
drum
G
Sludge
Combustion
Header chamber

Circulating
pump

Figure 3.8  LaMont boiler

M03_THERMAL ENGINEERING_SE_XXXX_CH03_(Part1).indd 96 6/15/2018 1:51:35 PM


Steam Generators 97

3.7.4  Benson Boiler


The major drawback in the LaMont boiler is the formation and sticking of bubbles in the inner surface
of the heating tubes. These attached bubbles reduce the heat flow and steam generation as it offers
higher thermal resistance as compared to water film. If the boiler pressure is raised to critical pressure
(221.2 bar), the steam and water would have same density, and therefore, the danger of bubble forma-
tion can be completely eliminated.
The arrangement of the boiler components is shown in Fig. 3.9. The feed water is passed through
the economiser and the radiant evaporator where a major part of water is converted into steam. The
remaining water is evaporated by hot gases in the convective evaporator. The supercritical pressure
steam is then passed through the superheater after which it is supplied to the prime mover.
The maximum pressure and temperature of steam obtained from this boiler are 500 bar and 650°C,
respectively, and generating capacity of 150 tonnes/h. The major difficulty in the operation of this
boiler is the deposition of salt and sediments on the inner surface of water tubes.

3.7.5  Loeffler Boiler


The major difficulty of Benson boiler is avoided in the Loeffler boiler by preventing the flow of water
into the boiler tubes. Most of the steam is generated outside from the feed water by using a part of the
superheated steam coming out from the boiler. The arrangement of different components and steam
circulations is shown in Fig. 3.10.
The pressure feed pump draws the water through the economiser and delivers to the evaporator
drum. About 65% of the steam coming out of superheater is passed through the evaporator drum to

Exhaust gases
Economiser

Exhaust gases
Air Chimney
Feed preheater Fan
pump Oil fuel or Feed pump
Superheater pulverised Feed tank
fuel
Convective Economiser
evaporator
Convection surface
To prime mover
circulating

Radiant surface
Steam

pump

Main steam
Superheater coils
Radiant
evaporator

Drain Boiler drum

Figure 3.9  Benson boiler Figure 3.10  Loeffler boiler

M03_THERMAL ENGINEERING_SE_XXXX_CH03_(Part1).indd 97 6/15/2018 1:51:36 PM


98 Chapter 3

evaporate the feed water coming from the economiser. This steam is drawn by the steam circulating
pump and passed through the radiant superheater and then convective superheater.
Loeffler boilers are available with generating capacity of 94.5 tonnes/h and operating at 140 bar.
This boiler can handle high drum salt concentration without trouble.

3.7.6  Schmidt-Hartmann Boiler


The arrangement of this boiler is shown in Fig. 3.11. In the primary circuit, a feed pump supplies
water to drum through the economiser, which, in turn, discharges saturated steam to a convective
superheater and then to load. A closed secondary circuit heated by furnace resembles a sectional
header boiler. This section supplies the heated vapour in a coil to evaporate the main feed water. It has
a tube evaporating section. These boilers are built for pressure ranging from 35−125 bar.

3.7.7  Velox Boiler


The arrangement of a Velox boiler is shown in Fig. 3.12. The pressurised combustion chamber (fur-
nace) of this boiler uses low excess air and has high heat transfer rates. The air for the furnace is
supplied by the air compressor. After heating water and steam, the combustion gases pass through
a gas turbine to atmosphere. This gas turbine drives the axial flow compressor that raises incoming
combustion air from atmosphere to the furnace pressure. The other important components are feed
pump, economiser, steam separating section, water circulating pump, and convective superheater. This
boiler is built for pressure ranging from 70−80 bar and 500°C.

E O
Flue
gas

CS
Flue gas

WS T
Flue
gas

D
D
FP

WC AC GT
Secondary
circuit
E
FP
Velox

Figure 3.11  Schmidt-Hartmann boiler Figure 3.12  Velox boiler

M03_THERMAL ENGINEERING_SE_XXXX_CH03_(Part1).indd 98 6/15/2018 1:51:42 PM


Steam Generators 99

3.7.8  Once-through Boiler


The arrangement of this boiler is shown in Fig. 3.13. It has inclined coils arranged in a spiral. Forty
coils are paralleled around the furnace. Steam generated in the headers flows into headers and then to
the convective superheater. Other essential components are the feed pump and economiser.

3.8 ❐ CIRCULATION
The flow of water and steam within the boiler circuit is called circulation. The insulated downcomers
carry water from the steam drum to the header and are located outside the furnace. The risers are
located inside the furnace and carry steam to the steam drum from the header. Risers installed all
around the four-walls of the furnace carry away the heat from the furnace. A simple downcomer−riser
circuit connecting a drum and a header is shown in Fig. 3.14.
Downcomers are fewer in number and bigger in diameter, whereas risers are more in number and
smaller in diameter. Downcomers are meant to make the water fall by gravity. Bigger the diameter,
lesser the pressure drop due to friction, since pressure drop is inversely proportional to the tube diam-
eter. The pressure drop is given by
fL v 2
∆p = r
D 2

where D = diameter of downcomer, L = length of downcomer, r = density of water, v = velocity of


water, f = Darcy’s friction factor. So, the downcomers are made bigger in diameter.
Risers absorb heat from the furnace. For the same total cross-sectional area, the smaller the diame-
ter, the larger the surface area exposed to hot gas for heat transfer. Therefore, the risers are of smaller
diameter, as compared to that of downcomers, and more in number.

Feed water To superheater


from
economiser
O
E Steam drum Riser tube

CS
Flue Water Steam
gas

T Downcorner
Furnace
wall
FP Header

Figure 3.13  Once-through boiler Figure 3.14  Downcomer-riser circuit

M03_THERMAL ENGINEERING_SE_XXXX_CH03_(Part1).indd 99 6/15/2018 1:51:46 PM


100 Chapter 3

3.9 ❐ STEAM DRUM


The purposes of drum used in boiler are as follows:
1. To store water and steam sufficiently to meet varying load requirements.
2. To aid in circulation.
3. To separate vapour or steam from water-steam mixture, discharged by the risers.
4. To provide enough surface area for liquid vapour disengagement.
5. To maintain a certain desired ppm in the drum by phosphate injection and blow down.
A mixture of water, steam, foam and sludge is discharged into the drum by the risers. Steam must
be separated from the mixture before it leaves the drum. Any moisture carried with steam to the
superheater tubes contains dissolved salts. In the superheater, water evaporates and the salts remain
deposited on the inside surface of the tubes to form a scale, which is difficult to remove. This scale
reduces the rate of heat absorption which ultimately leads to the failure of the superheater tubes by
overheating and rupture. The superheater tubes are exposed to the highest steam pressure and tem-
perature on the inside and the maximum gas temperature on the outside. The superheater tubes are
made of very costly material and utmost care should be exercised so that no damage is done to them
by the excessive moisture in steam. Some of the impurities in steam may be vaporised silica, which
may cause turbine blade deposits.
No vapour bubbles should flow along with saturated water from the drum to the downcomers. This will
reduce the density difference and the pressure head for natural circulation. The bubbles tending to flow
upward may also impede the flow in the downcomer and thus affect circulation. The drum has to secure
moisture-free steam going to superheater and bubble-free water going to the downcomers as shown in
Fig. 3.15.
Feed water Moisture-free steam
from to superheater
economiser

Steam drum

Risers
Bubble-free
water to downcorner

Figure 3.15  Functions of steam drum


3.9.1  Mechanism of Separation of Moisture in Drum
The separation of steam from steam-water mixture can be done by the following methods:
1. Gravity separation: At low pressures up to 20 bar, gravity separation is used if sufficient dis-
engaging surface is provided, as shown in Fig. 3.16(a). Water separates out by gravity from
steam-water mixture, being heavier. But at high pressure, separating force due to gravity is low
and it is difficult to achieve adequate separation, and there is considerable moisture carried with
steam.

M03_THERMAL ENGINEERING_SE_XXXX_CH03_(Part1).indd 100 6/15/2018 1:51:47 PM


Steam Generators 101

Baffles
To Superheater
To Superheater

Riser Screen

Riser

Downcomer
Riser Riser
Downcomer

(a) (b)

To Superheater

Scrubber Scrubber
Cyclone Cyclone

Riser Riser

Riser
Riser
Blowdown
Downcorner
(c)

Figure 3.16 Steam drum separation: (a) Gravity separator, (b) Baffle and screen,
(c) Cyclone and scrubber
2. Mechanical separators
(a) Baffles and screens: Baffle plates shown in Fig. 3.16(b) act as primary separators. They
change or reverse the steam flow direction, thus assisting gravity separation, and act as
impact plates that cause water to drain off. Screens made of wire mesh act as secondary sep-
arators where the individual wires attract and intercept the fine droplets. The accumulating
water drops then fall by gravity back to the main body of water.
(b) Cyclone separators: These separators shown in Fig. 3.16(c) utilises the centrifugal forces
for separation of two-phase mixture, which is entered tangentially to direct the water down-
ward and to make the steam flow upward. The steam then goes through the zigzag path in
corrugated plates, called the scrubber or dryer, on the way out to help remove the last traces
of moisture. Finally, perforated plates or screens under the drum exit provide the final dry-
ing action.
The internal details of a controlled circulation boiler are shown in Fig. 3.17.

M03_THERMAL ENGINEERING_SE_XXXX_CH03_(Part1).indd 101 6/15/2018 1:51:48 PM


102 Chapter 3

Steam outlet Steam and


Steam and water mixture
water mixture

Screen
drier

Turbo steam
separators

Feedwater

Downcomers

Figure 3.17  Drum internals of a controlled circulation boiler

3.10 ❐ FLUIDISED BED BOILER


Fluidised bed boilers produce steam from fossil and waste fuels by using a technique called fluidised
bed combustion (FBC).

3.10.1  Bubbling Fluidised Bed Boiler (BFBB)


A schematic diagram of BFBB is shown in Fig. 3.18. In this boiler, crushed coal (6−20 mm) is injected
into the fluidised bed of limestone just above an air-distribution grid at the bottom of the bed.
The air flows upwards through the grid from the air plenum into the bed, where combustion of
coal occurs. The products of combustion leaving the bed contain a large proportion of unburnt carbon
particles which are collected in cyclone separator and fed back to the bed. The boiler water tubes are
located in the furnace.
Since most of the sulphur in coal is retained in the bed by the bed material used (limestone),
the gases can be cooled to a lower temperature before leaving the stack with less formation of acid
(H2SO4). As a result of low combustion temperatures (800−900°C), inferior grades of coal can be used
without slagging problem and there is less formation of NOx. Cheaper alloy materials can also be used,
resulting in economy of construction. Further economies are achieved since no pulveriser is required.

M03_THERMAL ENGINEERING_SE_XXXX_CH03_(Part1).indd 102 6/15/2018 1:51:49 PM


Steam Generators 103

Flue gas and dust to


Convective
particulate removal system
tube bank
Drum

Furnace water Primary cyclone


walls
Submerged
tube bank Cyclone fines
Coal and limestone recycle feed line
feed hopper
Fluidized bed
(Combustion zone)
Transport air
supply
Plenum chamber
Air fan

Grid plate
Bed drain tube

Figure 3.18  Bubbling f  luidised bed boiler

The volumetric heat release rates are 10 to 15 times higher and the surface heat transfer rates are 2 to
3 times higher than a conventional boiler. This makes the boiler more compact.

3.10.2  Advantages of BFBB


1. The unit size and hence capital cost are reduced due to better heat transfer.
2. It can respond rapidly to changes in load demand.
3. Low combustion temperatures (800−950°C) restricts the formation of NOx pollutants.
4. Fouling and corrosion of tubes is reduced considerably due to low combustion temperatures.
5. Cost of coal to fine grind is reduced as it is not essential to grind the coal very fine.
6. Low grade fuels and high-sulphur coal be used.
7. Fossil and waste fuels can be used.
8. Combustion temperature can be controlled accurately.

3.11 ❐ BOILER MOUNTINGS


Different fittings and devices necessary for the operation and safety of a boiler are called boiler
mountings. The various boiler mountings are as follows:
1. Water- level indicator
2. Pressure gauge
3. Steam stop valve
4. Feed check valve
5. Blow-down cock

M03_THERMAL ENGINEERING_SE_XXXX_CH03_(Part1).indd 103 6/15/2018 1:51:49 PM


104 Chapter 3

6. Fusible plug
7. Safety valve: spring loaded, dead weight, lever type
8. High steam and low water safety valve.

3.11.1  Water Level Indicator


The water-level indicator indicates the level of water in the boiler constantly. Every boiler is normally
fitted with two water-level indicators at its front end. Figure 3.19 shows a water-level indicator used in
low pressure boilers. It consists of three cocks and a glass tube. The steam cock D keeps the glass tube
in connection with the steam space and cock E puts the glass tube in connection with the water space
in the boiler. The drain cock K is used to drain out the water from the glass tube at intervals to ascertain
that the steam and water cocks are clear in operation. The glass is generally protected with a shield.
For the observation of water level in the boiler, the steam and water cocks are opened and the drain
cock is closed. The rectangular passage at the ends of the glass tube contains two balls. If the glass
tube is broken, the balls are carried along its passage to the ends of the glass tube and flow of water
and steam out of the boiler is prevented.

3.11.2  Pressure Gauge


The pressure gauge is used to indicate the steam pressure of the boiler. The gauge is normally mounted
in the front top of the steam drum. The commonly used pressure gauge is the Bourdon type pressure
gauge shown in Fig. 3.20. It consists of an elastic metallic tube of elliptical cross-section bent in the
form of circular arc. One end of the tube is fixed and connected to the steam of the boiler and the other
end is connected to a sector wheel through a link. The sector remains in mesh with a pinion fixed on
a spindle. A pointer is attached to the spindle to read the pressure on a dial gauge.

M
B D - Steam cock
Steam N E - Water cock
K - Drain cock
D A A - Bourdon tube
Boiler G - Pointer
plate F G F - Pinion
G L
Water D - Quadrant
level F B - Socket
D
E Link
Water C
P
B
E R
Pipe to boiler
K From
boiler

Figure 3.19  Water-level indicator Figure 3.20  Pressure gauge

M03_THERMAL ENGINEERING_SE_XXXX_CH03_(Part1).indd 104 6/15/2018 1:51:51 PM


Steam Generators 105

When high pressure steam enters the elliptical tube, the tube section tends to become circular,
which causes the other end of the tube to move outward. The movement of the closed end of the tube
is transmitted and magnified by the link and sector. The sector is hinged at a point on the link. The
magnitude of the movement is indicated by the pointer on the dial.

3.11.3  Steam Stop Valve


The function of the stop valve is to regulate the flow of steam from the boiler to the prime mover as
per requirement and shut off the steam flow when not required.
A commonly used steam stop valve is shown in Figure 3.21. It consists of a main body, valve, valve
seat, nut, and spindle, which pass through a gland to prevent leakage of steam. The spindle is rotated
by means of a hand wheel to close or open.

3.11.4  Feed Check Valve


The function of the feed check valve is to allow the supply of water to the boiler at high pressure con-
tinuously and to prevent the back flow of water from the boiler when the pump pressure is less than
boiler pressure or when the pump fails.
A commonly used feed check valve is shown in Fig. 3.22. It is fitted to the shell slightly below the
normal water level of the boiler. The lift of the non-return valve is regulated by the end position of the
spindle which is attached to the hand wheel. The spindle can be moved up or down with the help of
hand wheel which is screwed to the spindle by a nut. Under normal conditions, the non-return valve is
lifted due to the water pressure from the pump and water is fed to the boiler. If the case pump pressure
falls below boiler pressure, the valve is closed automatically.

Hand wheel

Nut

Stud Gland
Hand wheel

Spindle

Body

Steam
out

Spindle

Valve Non-return
Valve valve
seat
Steam in Water in

Figure 3.21  Steam stop valve Figure 3.22  Feed check valve

M03_THERMAL ENGINEERING_SE_XXXX_CH03_(Part1).indd 105 6/15/2018 1:51:52 PM


106 Chapter 3

3.11.5  Blow-Down Cock


The function of the blow-down cock is to remove sludge or sediments collected at the bottommost
point in the water space in a boiler, while the boiler is steaming. It is also used for complete draining
of the boiler for maintenance.
A commonly used type of blow-down cock is shown in Fig. 3.23. It consists of a conical plug fitted
accurately into a similar casing. The plug has a rectangular opening which may be brought into the
line of the passage of the casing by rotating the plug. This causes the water to be discharged from the
boiler. The discharge of water may be stopped by rotating the plug again. The blow-down cock should
be opened when the boiler is in operation for quick forcing out of sediments.

3.11.6  Fusible Plug


The main function of the fusible plug is to put off the fire in the furnace of the boiler when the water
level in the boiler falls below an unsafe level and thus avoid the explosion which may occur due to
overheating of the tubes and shell. The plug is generally fitted over the crown of the furnace or over
the combustion chamber.
A commonly used fusible plug is shown in Fig. 3.24. It consists of a hollow gun metal body
screwed into the fire box crown. The body has a hexagonal flange to tighten it into the shell. A gun
metal plug is screwed into the gun metal body by tightening the hexagonal flange formed into it. There
is yet another solid plug made of copper with conical top and rounded bottom. The fusible metal holds
this conical copper plug and the gun metal plug together due to depressions provided at the mating
surfaces.
During normal operation, the fusible plug remains submerged in water and on no account, its tem-
perature can be more than the saturation temperature of water.
The fusible metal is protected from direct contact with water or furnace gases. When the water
level in the boiler shell falls below the top of the plug, the fusible metal melts due to overheating.

Solid copper plug


Fusible
Hexagonal
metal
flange
Gun metal
Guard plug

Spinde
Gland

Casing Gun metal Hexagonal


body flange
Ribs

Conical
plug

Fire box crown plate

Figure 3.23  Blow-down cock Figure 3.24  Fusible plug

M03_THERMAL ENGINEERING_SE_XXXX_CH03_(Part1).indd 106 6/15/2018 1:51:53 PM


Steam Generators 107

Thus, the copper plug drops down and is held within the gun-metal body by the ribs. The steam space
gets communicated to the fire box and extinguishes the fire. The fusible plug can be put into position
again by interposing the fusible metal.

3.11.7  Safety Valves


The function of a safety valve is to prevent the steam pressure in the boiler exceeding the desired rated
pressure by automatically opening and discharging steam to atmosphere till the pressure falls back to
normal rated value. There are three types of safety valves—spring loaded (Ramsbottom) type, dead
weight type, and lever type.
1. Spring-loaded safety valve: The spring-loaded safety valve of the Ramsbottom type is shown in
Fig. 3.25. The spring holds the two valves on their seats by pulling the lever down. The lever is
provided with two conical pivots, one integrally forged with the lever and the other pin connected
to one end. The upper end of the spring is hooked to the lever midway between the two pivots.
The lower end is hooked to the shackle fixed to the valve chest by studs and nuts. The shackle
and the lever are also connected by two links, one end of which is pin-jointed and the other end
has a slot cut into it to allow for the pin to slide in it vertically, thus allowing the lever to be lifted
and retaining the link connection. These links are provided as a safety measure in case the spring
breaks, the lever should not fly off. The lever projects on one side for manual operation to check
the satisfactory working of the device.
2. Dead weight safety valve: In this type of device, as shown in Fig. 3.26, the steam pressure in
the upward direction is balanced by the downward force of dead weights acting on the valve. It
is generally used on low capacity boilers like the Lancashire boiler. The bottom flange is directly
connected to seating block on the boiler shell.
3. Lever safety valve: A typical lever safety valve is shown in Fig. 3.27. The lever is the second kind
with effort in the middle of fulcrum and load. It is suitable for stationary boilers.

Lever
Pin
Pivot Spring end Pivot

Valve seat Valve


Pin
Slot
Spring
Link
Pin
Shackle
Spring end

Valve chest

Base

Figure 3.25  Spring loaded safety valve

M03_THERMAL ENGINEERING_SE_XXXX_CH03_(Part1).indd 107 6/15/2018 1:51:54 PM


108 Chapter 3

C C

G
W K
E

A - Flange
B - Vertical steel pipe
C - Valve
E - Feathers B To waste pipe
F - Weight carrier
G - Cover
J - Discharge casing L J
L - Drain
W - Weight A

Figure 3.26  Dead weight safety valve

H
C G
W

A - Block (valve body)


C - valve D
F
D - Valve seat
E - Lever
F - Fulcrum
G - Slotted lever guide A
H - Strut
W - Dead weight
Steam
pressure

Figure 3.27  Lever safety valve

3.11.8  High Steam and Low Water Safety Valve


The functions of water safety valve are of two, namely to blow out if the steam pressure is higher than
the working pressure and to blow out steam when the water level in the boiler is low. It is perhaps the
most important mounting on the boiler. Figure 3.28 shows the details of Hopkinson’s high steam and
low water safety valve.

M03_THERMAL ENGINEERING_SE_XXXX_CH03_(Part1).indd 108 6/15/2018 1:51:59 PM


Steam Generators 109

F1
W 1 2

Waste W

Low water 11
valve

High steam valve 4

5 9
6
F2

7 8

10

Figure 3.28  High steam and low water safety valve

F1 is the fulcrum, W the weight suspended on one end of the lever, and W is the adjustable balance
weight on the other end, which can be fixed in position on the lever 2 by a set screw. A predetermined
force is exerted through the strut 1 on the outer valve.
The arrangement of the inner valve with spindle 4 and a dead weight 8 acts as dead weight safety
valve. However, the load on the outer valve is both due to dead weight and strut thrust. If the pressure
in the boiler exceeds the limit, the valve lifts and the steam escapes to waste.
On one end 9, of lever 6−9 with fulcrum F2, is attached a float 10 and on the other end is fixed a
balance weight 7. When the float 10 is submerged in water, the lever is balanced about fulcrum F2.
As soon as the float is uncovered, it gets unbalanced and tilts the lever to the right. This lever has a
projecting knife edge which is clear of the collar 5 when the float is submerged. As soon as it tilts
towards right, it establishes contact with the collar and pushes the spindle 4 up, thus raising the inner
valve from its seat. Thus steam starts escaping out through the waste pipe. A drain connection 11 is
provided to drain off the condensed steam.

3.12 ❐ BOILER ACCESSORIES


These are the appliances installed to increase the overall efficiency of the steam power plant. The
various accessories installed are pressure reducing valve, steam trap, steam separator, economiser, air
preheater, superheater, feed pump, and injector.

M03_THERMAL ENGINEERING_SE_XXXX_CH03_(Part1).indd 109 6/15/2018 1:52:00 PM


110 Chapter 3

The functions of the various accessories are as follows:


1. A pressure reducing valve maintains constant pressure on its delivery side with fluctuating boiler
pressure.
2. The function of steam trap is to drain of water resulting from the partial condensation of steam
from steam pipes and jackets without allowing steam to escape through it.
3. The function of the steam separator is to separate suspended water particles carried by steam on
its way from the boiler to the prime mover.
4. The function of an economiser is to recover some of the heat carried away in the flue gases up the
chimney and utilise for heating the feed water to the boiler.
5. An air preheater recovers some portion of the waste heat of the flue gases by preheating the air
supplied to the combustion chamber.
6. Superheaters are used to increase the temperature of steam above its saturation temperature.
7. The function of a feed pump is to pump water to the water space of the boiler.
8. The function of an injector is to feed water to the boiler with the help of a steam jet.
We shall discuss the air preheater, economiser, and superheater in detail.

3.12.1  Air Preheater


In the air preheater, air is heated by the heat carried away by the flue gases and goes as a waste
through the chimney. It is situated between the economiser and the chimney. Air preheaters are of
three types—tubular, plate type, and regenerative.
In the tubular type of air preheater, shown in Fig. 3.29, the air passes down outside the tubes and
the flue gases through the tubes before going to the induced draught fan at the base of the chimney.
Baffles are provided across the tubes to make the air follow a zigzag path a number of times to utilise
more heat of flue gases.
In the plate type of air preheater, there are alternate chambers provided by the plate surfaces for
the flow of flue gases upwards to the induced draught fan and for the flow of combustion air down of
combustion air downwards.
In the regenerative type, the air preheater chamber is divided into two rotating compartments. The
heating surfaces are made up of a series of corrugated sheets between which the hot flue gases pass

Flue gas out

Air in

Baffle

Tubes

Air out
Flue gas in

Figure 3.29  Tubular air preheater

M03_THERMAL ENGINEERING_SE_XXXX_CH03_(Part1).indd 110 6/15/2018 1:52:01 PM


Steam Generators 111

upwards on one side and the combustion air passes down on the other side. Due to the rotation of pre-
heater at a very low speed, the heater surface comes in contact with combustion air and the cooled
surface is again heated by the flue gases. This alternate heating and cooling of heating surface is called
regenerative heating and is more effective than other types.

3.12.2 Economiser
The Green’s economiser shown in Fig. 3.30 is used for boilers of a medium pressure range up to about
25 bar. It consists of groups of vertical cast iron pipes fitted at their two ends to cast iron boxes, at
the top and bottom. All the tubes are enclosed within the brick work of the economiser. There are two
pipes, C and D, outside the brick work E. Rows of top and bottom boxes are connected to these top and
bottom pipes. The feed water is pumped to the bottom pipe D. From here, it goes to the bottom boxes
and rise up in the groups of vertical pipes into the top boxes. From the top boxes, it goes to the pipe
C from where it passes to the water space of the boiler through the check valve. F is a stop valve for
entry and G is a stop valve for exit of the water from the economiser. S is a safety valve. These tubes

Water S
outlet

E E

Flue gas
out

Flue gas in S

A
G F
Water
intlet
D
L

Figure 3.30 Economiser

M03_THERMAL ENGINEERING_SE_XXXX_CH03_(Part1).indd 111 6/15/2018 1:52:02 PM


112 Chapter 3

are continuously scraped by scrapers J to remove soot collected on them due to the passage of flue
gases through the economiser. A pair of scrapers is connected by a chain passing over a pulley such
that one scraper comes down and the other attached to the same chain goes up. L is the soot chamber
where soot is collected and removed through a soot door.

3.12.3 Superheater
The superheater commonly used in a Lancashire boiler is shown in Fig. 3.30. It consists of two headers
and a set of superheater tubes made of high-quality steel in the form of U-tube. It is located in the
path of furnace gases just before the gases enter the bottom flue. The amount of hot gases passed over
the superheater tubes should be in proportion to the amount of superheated steam passing through the
tubes. Otherwise, the tubes would be over heated. To avoid this, the hot gases are diverted as shown in
Fig. 3.31. The superheater is put out of action by turning the damper upward to the vertical position.
In this position of the damper, the gases coming out from the central flue pass directly into the bottom
flue without passing over the superheater tubes.
For getting superheated steam, the valves A and B are opened and valve C is closed. The damper is
kept open as per the quantity of steam flowing through the pipe. For this position, the flow direction
of steam is shown in Fig. 3.31. If wet steam is required, valves A and B and the gas damper are kept
closed, and valve C is kept open. In this case, steam comes directly from the boiler through valve C.

Stop valve
Steam out
C B

Main steam pipe

A
Stop valve

Handle Headers
Water level
Boiler

Hot gases
Chimney flue

Bottom flue

Damper Gas damper

Figure 3.31 Superheater

M03_THERMAL ENGINEERING_SE_XXXX_CH03_(Part1).indd 112 6/15/2018 1:52:02 PM


Steam Generators 113

By adjusting the gas damper, the temperature of steam coming out of superheater is always maintained
constant, irrespective of the amount of steam passing through the superheater.

3.13 ❐ STEAM ACCUMULATORS


Boilers are not always required to supply steam for economic or rated load but also required to meet
the fluctuating demand of load, that is, peak fractional load. The boiler takes an appreciable time to
change the rate of steam generation but the demand for a considerable change of power may occur
instantly if the boiler could meet such a sudden increase in load; there is always the problem of
disposing the steam once the peak has been passed. Moreover, in meeting the fluctuating load, the
capacity and efficiency of boiler are reduced. To meet these requirements, it is a general practice
to incorporate steam accumulators to equalise the load on the boilers. Thus, the steam accumula-
tors acting as a thermal flywheel store energy during periods when the output of the boiler exceeds
the demand and restores or supplies back the energy when the demand is more than the output
of the boiler.
The steam accumulators are of two types, namely variable pressure system (Ruth’s accumulator)
and constant pressure system (Kiesselbach accumulator).

3.13.1  Variable Pressure Accumulator


This accumulator (Fig. 3.32) was first introduced by Dr Ruth, a Swedish technician, in 1916. It con-
sists of a steel shell filled with about 99% water under low pressure required by the low turbine or
process work.
During charging, high pressure steam from the boiler enters the pipe C through the value V1.
Under this condition, the non-return valve provided in pipe C is closed and the high pressure steam
enters into the shell through the non-return valve B. Specially shaped nozzles surrounded by pipes are
employed to discharge the steam silently into the water and thereby bring about condensation. This
increases the pressure in the accumulator. The more the pressure rises in the accumulator, the greater
becomes the thermal storage capacity.

Charging line
(Steam inlet)
H.P. Main

N.R.V.A C V1

Discharge line
B V2
(Steam outlet)
L.P. Main
Steam

Water

Figure 3.32  Ruth’s accumulator

M03_THERMAL ENGINEERING_SE_XXXX_CH03_(Part1).indd 113 6/15/2018 1:52:03 PM


114 Chapter 3

During the discharging process (the demand of steam is more than output of boiler), the pressure
in the pipe C falls below the pressure of water in the vessel when the low pressure mains valve is in
communication with the industrial plant. In such circumstances, the pressure in the accumulator is
also lowered, whereas the temperature remains same as before. This causes evaporation of water into
steam, thereby increasing the pressure and reducing temperature until equilibrium condition between
temperature and pressure is again attained. The non-return valve B is closed and steam passes to the
low pressure turbine or industrial plant through the dome E and the non-return valve A. When the
output of the steam equals the consumption of steam, there is no flow into and out of the accumulator.
Ruth’s accumulator gives a large discharge over a short period and a small discharge over a large
period. This accumulator is widely used in mines and steel works where the reciprocating steam
engines direct their intermittent exhausts into a low pressure receiver which is placed in series with
accumulators. Low pressure turbines consume the steam supply from the accumulators.
The main advantages of this system are as follows:
1. Increases the efficiency of boiler
2. Increase in rate of evaporation, less repairs, and economy in fuel
3. Rapid pick up
4. Improvement in boiler load factor

3.13.2  Constant Pressure Accumulator


Figure 3.33 shows a constant pressure accumulator. This is used in conjunction with small water tube
boilers, it stores heat in water during charging period (i.e., when the demand is less) but delivers water
(not steam) during discharge period (i.e., when the demand is more than out-put). Therefore, this is
essentially a feed-water accumulator.
In this system, the rate of firing of the boiler is maintained constant but the quantity of feed is var-
ied. When there is fall in the demand for steam, an extra quantity of feed water is circulated through
the boiler where the feed-water temperature rises to that of steam, and after that the surplus water is
passed into the accumulator.

Main stream

Over
flow
Boiler
Feed pump A B
regulator

Cold water
pump
Feed pump

Figure 3.33  Kiesselbach accumulator

M03_THERMAL ENGINEERING_SE_XXXX_CH03_(Part1).indd 114 6/15/2018 1:52:04 PM


Steam Generators 115

3.14 ❐ PERFORMANCE OF STEAM GENERATOR


3.14.1  Evaporation Rate
The quantity of water evaporated into steam per hour is called the evaporation rate. It is expressed
as kg of steam/h, kg of steam/h/m2 of heating surface, kg of steam/h/m3 of furnace volume, or kg of
steam/kg of fuel fired.

Equivalent Evaporation
It is the equivalent of the evaporation of 1 kg of water at 100°C to steam at 100°C. It requires 2257 kJ.

Factor of Evaporation
The ratio of actual heat absorption above feed-water temperature for transformation to steam (wet,
dry, or superheated) to the latent heat of steam at atmospheric pressure (1.01325 bar) is known as a
factor of evaporation.
Let h = specific enthalpy of steam actually produced
hf = specific enthalpy of feed water
hf (atm) = specific enthalpy of evaporation at standard atmospheric pressure
ms = actual evaporation expressed in kg/kg of fuel or kg/h of steam
me = equivalent evaporation expressed in kg/kg of fuel or kg/h
Then, the equivalent evaporation = actual evaporation × factor of evaporation

 h1 − h f 
or me =   × ms = F × ms (3.1)
 h fg (atm) 
where F = factor of evaporation
h1 = hf1 + xhfg1 for wet steam actually generated
= hg1 for saturated steam actually generated
= hsup for superheated steam actually generated.

3.14.2 Performance
The performance of boiler may be explained on the basis of any of the following terms:
1. Efficiency: It may be expressed as the ratio of heat output to heat input.
2. Combustion rate: It is the rate of burning of fuel in kg/m3 of grate area/h.
3. Combustion space: It is the furnace volume in m3/kg of fuel fired/h.
4. Heat absorption: It is the equivalent evaporation from and at 100°C in kg of steam generated/m2
of heating surface.
5. Heat liberated: It is the heat liberated/m3 of furnace volume/h.

M03_THERMAL ENGINEERING_SE_XXXX_CH03_(Part1).indd 115 6/15/2018 1:52:04 PM


116 Chapter 3

3.14.3  Boiler Thermal Efficiency


It is the ratio of heat absorbed by steam from the boiler per unit time to the heat liberated by the com-
bustion of fuel in the furnace during the same time.
m s ( h1 − h f )
Boiler efficiency, hb = (3.2)
m f × C.V.

where m s = mass of steam generated in kg/h


m f = mass of fuel burned in kg/h
C.V. = calorific value of fuel in kJ/kg

3.14.4  Heat Losses in a Boiler Plant


1. Heat used to generate steam
Q1 = ms (h − hf1)
2. Heat lost to flue gases: The flue gases contain dry products of combustion and the steam gener-
ated due to the combustion of hydrogen in the fuel.
Heat lost to dry flue gases,
Q2 = mg cpg(Tg − Ta)(3.3)
where m g = mass of gases formed per kg of fuel
cpg = specific heat of gases
Tg = temperature of gases, °C
Ta = temperature of air entering the combustion chamber of the boiler, °C
3. Heat carried by steam in flue gases
Q3 = ms1(hs1 − hf1)(3.4)
where ms1 = mass of steam formed per kg of fuel due to combustion of H2 in fuel
    hf1 = enthalpy of water at boiler house temperature
     hs1 = enthalpy of steam at the gas temperature and at the partial pressure of the vapour in
the flue gas
4. Heat loss due to incomplete combustion: If carbon burns to CO instead of CO2, then it is known
as incomplete combustion. 1 kg of C releases 10,200 kJ/kg of heat if it burns to CO, whereas it
releases 35,000 kJ/kg if it burns to CO2. If the percentages of CO and CO2 in flue gases by vol-
ume are known, then
CO × C
Mass of C burnt to CO =
CO + CO 2
where CO, CO2 = % by volume of CO and CO2, respectively, in flue gases
  C = fraction of carbon in 1 kg of fuel
Heat lost due to incomplete combustion of carbon per kg of fuel,

M03_THERMAL ENGINEERING_SE_XXXX_CH03_(Part1).indd 116 6/15/2018 1:52:05 PM


Steam Generators 117

CO × C
Q4 = × (35,000 − 10,200)
CO + CO 2

CO × C
= × 24,800 kJ/kg of fuel (3.5)
CO + CO 2
5. Heat lost due to unburnt fuel
Q5 = mf1 × C.V.
where mf1 = mass of unburnt fuel per kg of fuel burnt.
6. Convection and radiation losses: Heat unaccounted for due to convection and radiation losses,

Q6 = Q − (Q1 + Q2 + Q3 + Q4 + Q5)(3.6)

where Q = mf × C.V.
  = heat released per kg of fuel

3.14.5  Boiler Trial and Heat Balance Sheet


There are three purposes of conducting the boiler trial.
1. To determine and check the specified generating capacity of the boiler when working at full load
conditions.
2. To determine the thermal efficiency of the plant.
3. To draw up the heat balance sheet so that suitable corrective measures may be taken to improve
the efficiency.
The following measurements should be observed during the boiler trial.
1. The fuel supplied and its analysis.
2. Steam generated and its quality or superheat.
3. Flue gases formed from exhaust analysis.
4. Air inlet temperature and exhaust gases temperature.
5. Volumetric analysis of exhaust gases.
6. Mass of fuel left unburnt in ash.
7. Feed-water temperature.
The heat balance sheet is a systematic representation of heat released from burning of fuel and heat
distribution on minute, hour or per kg of fuel basis. A pro forma for heat balance sheet is given in
Table 3.2.

M03_THERMAL ENGINEERING_SE_XXXX_CH03_(Part1).indd 117 6/15/2018 1:52:06 PM


118 Chapter 3

Table 3.2  Heat balance sheet for boiler

Heat kJ % Heat utilisation kJ %


supplied

Heat supplied Q = mf 1. Heat used to Q1 = ms (h − hf1) Q1


100% generate steam. × 100
by fuel × CV Q

2. Heat carried by dry Q2 = mgcpg(Tg − Ta) Q2


flue gases. × 100
Q

3. Heat carried by Q3 = ms1(hs1 − hf1) Q3


steam in flue gases. × 100
Q

4. Heat lost due CO × C Q4


to incomplete Q4 = × C.V. × 100
CO + CO 2 Q
combustion.
5. Heat lost due to Q5 = mf1× C.V. Q5
unburnt fuel. × 100
Q

6. Heat unaccounted Q6 = Q − (Q1 + Q2 + Q3 Q6


× 100
for + Q4 + Q5) Q

Total Q 100% – Q 100%

Example 3.1
A boiler generates 4000 kg/h of steam at 20 bar, 400°C. The feed-water temperature is 50°C. The
efficiency of the boiler is 80%. The calorific value of fuel used is 44,500 kJ/kg. The steam gener-
ated is supplied to a turbine developing 450 kW and exhausting at 2 bar with dryness fraction 0.96.
Calculate the fuel burnt per hour and turbine efficiency. Also find the energy available in exhaust
steam above 50°C.

Solution
From steam tables at 20 bar, 400°C, h1 = 3245.5 kJ/kg
Enthalpy of feed water at 50°C, hf = 209.3 kJ/kg
m s ( h1 − h f ) 4000 × (3245.5 − 209.3)
Fuel used per hour = = = 341.146 kg/h
h s × C.V. 0.8 × 44, 500

Enthalpy of exhaust steam at 2 bar, 0.96 dry,


h2 = hf  2 + x2hfg2 = 504.7 + 0.96 × 2201.9 = 2618.5 kJ/kg
Enthalpy drop in turbine, ∆h = h1 − h2 = 3245.5 − 2618.5 = 627.0 kJ/kg

M03_THERMAL ENGINEERING_SE_XXXX_CH03_(Part1).indd 118 6/15/2018 1:52:09 PM


Steam Generators 119

P × 3600 450 × 3600


Turbine efficiency, ht = = = 0.646 or 64.6%
∆h × m s 627 × 4000
Energy available in the exhaust steam above 50°C = h2 − hf = 2618.5 − 209.3 = 2409.2 kJ/kg

Example 3.2
The following readings are taken during trial on a boiler for 1 h:
Steam generated = 5000 kg
Coal burnt = 650 kg
CV of coal = 31,500 kJ/kg
Dryness fraction of steam entering the superheater = 0.90
Rated boiler pressure = 10 bar
Temperature of steam leaving the superheater = 250°C
Temperature of hot well = 40°C
Calculate: (a) equivalent evaporation per kg of fuel without and with superheater, (b) thermal
efficiency of the boiler without and with superheater, and (c) amount of heat supplied by the super-
heater per hour.

Solution
Given that ms = 5000 kg/h, mf = 650 kg/h, C.V. of fuel = 31,500 kJ/kg, x = 0.90, p = 10 bar
( h1 − h f )ms
Equivalent evaporation, me =
2260
5000
where ms = mass of steam generated per kg of fuel = = 7.692 kg/kg of fuel
650
ms ( h f 1 + x1h fg1 − h f )
(a) Without superheater, me =
2260
At 10 bar,      hf 1 = 762.8 kJ/kg, hfg1 = 2015.3 kJ/kg
At 40°C, hf = 167.6 kJ/kg

7.692[762.8 + 0.9 × 2015.3 − 167.6]


me = = 8.2 kg
2260

ms [hsup − h f 1 ]
With superheater,  me =
2260
At 10 bar, 250°C,  hsup = 2942.6 kJ/kg

7.692[2942.6 − 167.6]
me = = 9.445 kg
2260
ms ( h1 − h f )
(b) Thermal efficiency, hth =
C.V.

M03_THERMAL ENGINEERING_SE_XXXX_CH03_(Part1).indd 119 6/15/2018 1:52:12 PM


120 Chapter 3

7.692[762.8 + 0.9 × 2015.3 − 167.6]


Without superheater, hth = = 0.5882 or 58.82%
31, 500

7.692( 2942.6 − 167.6)


With superheater, hth = = 0.6776 or 67.76%
31, 500

(c) Heat supplied by the superheater per hour = ms [(1 − x)hfg1 + hsup]

= 5000 [(1 − 0.9) × 2015.3 + 2942.6] = 15,720,650 kJ/h

Example 3.3
The following data refer to a boiler trial:
Feed water = 700 kg/h
Feed-water temperature = 25°C
Steam pressure = 15 bar
Steam temperature = 300°C
Coal burnt = 90 kg
CV of coal = 30,500 kJ/kg
Ash and unburnt coal in ash pit = 4 kg/h with C.V. = 2200 kJ/kg
Flue gas formed = 20 kg/kg of coal burnt
Flue gas temperature at chimney = 300°C
Ambient temperature = 30°C
Mean specific heat of flue gases = 1.025 kJ/kgK
Calculate (a) the boiler efficiency, (b) the equivalent evaporation, and (c) the percentage heat unac-
counted for.

Solution
From steam tables, at 15 bar, 300°C, hsup = 3038.9 kJ/kg
At 25°C, hf = 108.77 kJ/kg
Heat absorbed by 1 kg of feed water to transform into steam at 300°C and 15 bar

= hsup − hf = 3038.9 − 108.77 = 2930.13 kJ/kg


ms ( hsup − h f ) 700 × 2930.13
(a) Boiler efficiency, hb = = = 0.7472 or 74.72%
m f × C.V. 90 × 30, 500

(b) Equivalent evaporation,

hsup − h f m s 2930.13 700


me = × = × = 10.084 kg/kg of fuel
h fg (atm) mf 2260 90

Heat carried away by flue gases = mg cpg (Tg − Ta)


= 20 × 1.025 × (300 − 30) = 5535 kJ/kg of coal

M03_THERMAL ENGINEERING_SE_XXXX_CH03_(Part1).indd 120 6/15/2018 1:52:14 PM


Steam Generators 121

(c) Heat unaccounted for per kg of fuel


= CV of fuel − heat in steam − heat in flue gases − heat in ash pit
700 4 × 2200
= 30500 − 2930.13 × − 5535 −
90 90
= 30500 − 22790 − 5535 − 97.8 = 2077.2 kJ/kg

Example 3.4
The following observations were made during a boiler trial:
Mass of feed water and its temperature = 640 kg/h, 50°C
Steam pressure = 10 bar
Fuel burnt = 55 kg/h
HCV of fuel = 44,100 kJ/kg
Temperature of flue gases = 300°C
Boiler house temperature = 30°C
Dryness fraction of steam = 0.97
Heating surface of boiler = 18.6 m2
Specific heat for flue gases = 1.1 kJ/kgK
Specific heat for superheated steam = 2.1 kJ/kgK
Composition of liquid fuel used by mass: C = 85%, H2 = 13%, and ash = 2%
The flue gas analysis by volume done by using Orsat apparatus:
CO2 = 12.5%, O2 = 4.5%, and N2 = 83%
Calculate (a) the equivalent evaporation per kg of fuel per m2 of heating surface area per hour,
(b) the boiler efficiency, and (c) the draw the heat balance sheet on 1 kg of fuel basis. Take the
partial pressure of steam in exhaust gases = 0.07 bar.

Solution
ms ( h1 − h f )
(a) Equivalent evaporation, me =
m f × 2260

h1 = hf1 + x1hfg1
At 10 bar, hf1 = 762.6 kJ/kg, hfg1 = 2013.6 kJ/kg
For feed water at 50°C, hf = 209.3 kJ/kg

640  762.6 + 0.97 × 2013.6 − 209.3 


me =  = 12.9 kg
55  2260 

ms ( h1 − h f ) 640 × 2506.5
(b) Boiler efficiency, hb = = = 0.6613 or 66.13%
m f × C.V. 55 × 44100
(c) Heat used to generate steam per kg of fuel
ms 640
= × 2506.5 = × 2506.5 = 29,166 kJ
mf 55

M03_THERMAL ENGINEERING_SE_XXXX_CH03_(Part1).indd 121 6/15/2018 1:52:17 PM


122 Chapter 3

N×C 0.83 × 0.85 × 100 2


Air supplied per kg of fuel burnt = = = 17.1 kg
33(C1 + C2 ) 33(0 + 12.5)
Dry flue gases formed per kg of fuel burnt = 17.1 + 0.85 = 17.95 kg
Heat carried by dry flue gases = mg × cpg (Tg − Ta)
= 17.95 × 1.1 (300 − 30) = 5331 kJ/kg of fuel
Moisture formed per kg of fuel = 0.13 × 9 = 1.17 kg
Heat carried by moisture in the flue gases per kg of fuel
= 1.17 [hg + cps (Tsup − Ts) − hf] at 0.07 bar
= 1.17 [2572.6 + 2.1 (300 − 39) − 125.8]

= 3504 kJ
Heat unaccounted for = 44,100 − (29,166 + 5331 + 3504) = 6099 kJ
Heat balance sheet:

Heat supplied kJ % Heat utilised kJ %


Heat supplied by fuel 44,100 100 1. To generate steam 29,166 66.14
2. Carried by flue gases 5331 12.09
3. Carried by moisture in flue gases 3504 7.94
4. Unaccounted for 6099 13.83
Total 44,100 100 44,100 100.00

Example 3.5
The following data was recorded during a boiler trial for 1 h:
Steam generated = 4500 kg, Steam pressure = 10 bar gauge
Record of throttling calorimeter:
Steam pressure = 30 mm of Hg above barometer reading.
Steam temperature = 105°C
Barometer reading = 735 mm of Hg
Specific heat of superheated steam = 2.1 kJ/kgK
Feed-water temperature = 58°C
Temperature of steam leaving the superheater = 202°C
Coal fired = 450 kg, HCV of coal = 35,700 kJ/kg
Flue gas temperature = 260°C, Boiler house temperature = 30°C
Ultimate analysis of dry coal:
C = 82 %, H2 = 14 %, and ash = 4%
Volumetric analysis of dry flue gases by Orsat apparatus:
CO2 = 12 %, CO = 1.5 %, O2 = 7%, N2 = 79.5 %
Specific heat of dry flue gases = 1.0 kJ/kgK
Partial pressure of steam going with flue gases = 0.07 bar

M03_THERMAL ENGINEERING_SE_XXXX_CH03_(Part1).indd 122 6/15/2018 1:52:18 PM


Steam Generators 123

Moisture content of coal at the time of feeding into the boiler = 4%


(a) Calculate the heat absorbed by the superheater per minute.
(b) Draw up the heat balance sheet on the basis of 1 kg of wet coal fired.

Solution
Quality of steam generated by the boiler:
(735 + 30)
Pressure of steam after throttling = × 1.01325 = 1.02 bar
760
735
Absolute pressure of steam generated = 10 + × 1.01325 = 10.98 bar
760
Total heat in steam before throttling at 10.98 bar = Total heat in steam after throttling at 1.02 bar
hf 1 + x1 hfg1 = (hf 2 + hfg2) + cps (Tsup2 − Ts2)
or 780.936 + x1 × 2000.7 = 2675.696 + 2.1 (105 − 100)
or x1 = 0.952
Heat absorbed in the superheater per minute:
4500
= [(1 − x1) hfg1 + cps (Tsup − Ts1)] at 10.98 bar
60
4500
= [(1 − 0.952) × 2000.7 + 2.1 (202 − 184)]
60
= 10,037.5 kJ/min
Heat generated per kg of coal fired:
1. Hg = Content of dry coal × HCV
= (1 − 0.04) × 35,700 = 34,272 kJ/kg
2. Heat absorbed in boiler and superheater per kg of coal to generate steam
4500
= [hg + cps (Tsup − Ts) − hf 1] at 10.98 bar
450
= 10 [2781.6 + 2.1 (202 − 184) − 4.187 × 58] = 25,765.8 kJ/kg
3. Air supplied per kg of coal burned
NC 79.5 × 82(1 − 0.04)
= = = 14 kg/kg of coal
33(C1 + C2 ) 33(1.5 + 12)
Dry flue gases formed per kg of coal fired
= 14 + (1 − 0.04) × 0.82 = 14.787 kg/kg of coal
4. Heat carried away by dry flue gases per kg of coal burned
= 14.787 × 1.0 (260 − 30) = 3401.0 kJ/kg of coal

M03_THERMAL ENGINEERING_SE_XXXX_CH03_(Part1).indd 123 6/15/2018 1:52:20 PM


124 Chapter 3

5. Moisture carried away by flue gases per kg of coal burned


= Moisture in the fuel + Moisture formed due to combustion of H2
 3 
= 0.04 + (1 − 0.04 × × 9 = 0.299 kg
 100 
Heat carried by moisture going with flue gases
= 0.299 [hg + cps (Tsup − Ts)] at 0.07 bar

= 0.299 [2572.6 + 2.1 (260 − 39)] = 908 kJ

 C1   1.5 
6. Part of carbon burnt to CO =  ×C =   × (1 − 0.04) × 0.82 = 0.0875 kg
 C1 + C2  1.5 + 12 

C.V. when C burns to CO2 = 35,000 kJ/kg, and when burns to CO = 10,250 kJ/kg
Heat lost due to incomplete combustion = 0.0875 (35,000 − 10,250)
= 2165.5 kJ/kg
Heat balance sheet on the basis of 1 kg of coal burned is as follows:

Heat supplied kJ % Heat utilised kJ %


Heat supplied per kg of coal 34,272 100 1. Heat used to generate steam 25,765.8 75.18

2. Heat carried away by dry flue gases 3401.0 9.92

3. Heat carried by moisture with flue 908.0 2.65


gases.

4. Heat lost due to incomplete 2165.5 6.32


combustion.

5. Heat unaccounted for 2031.7 5.93

Total 34,272 100 34,272 100.00

Example 3.6
A power plant producing 125 MW of electricity has steam condition at boiler outlet as 110 bar,
500°C and condenser pressure is 0.1 bar. The boiler efficiency is 92 % and consumes coal of calo-
rific value 25 MJ/kg. The feed-water temperature at boiler inlet is 150°C. The steam generator has
risers in the furnace wall 30 m high and unheated downcomers. The quality at the top of the riser
is 8.5 % and minimum exit velocity of mixture leaving the riser and entering the drum is required
to be 2 m/s. The risers have 50 mm outside diameter and 3 mm wall thickness. Neglecting pump
work and other losses, estimate (a) the steam generation rate, (b) the fuel burning rate, (c) the
evaporation factor, (d) the pressure head available for natural circulation, (e) the circulation ratio,
(f) the number of risers required, and (g) the heat absorption rate per unit projected area of the riser.

M03_THERMAL ENGINEERING_SE_XXXX_CH03_(Part1).indd 124 6/15/2018 1:52:21 PM


Steam Generators 125

Solution
Refer Fig. 3.34.
Given that P = 125 MW, p1 = 110 bar, t1 = 500°C, p2 = 0.1 bar, ve = 2 m/s
hb = 92 %, C.V. = 25 MJ/kg, tfw = 150°C, xtop = 0.085
From steam tables:
At p1 = 110 bar, 500°C, h1 = 3361 kJ/kg, s1 = 6.540 kJ/kgK
p2 = 0.1 bar, hf 2 = 191.83 kJ/kg, hfg2 = 2392.8 kJ/kg
sf 2 = 0.6493 kJ/kgK, sfg2 = 7.5009 kJ/kgK
At tfw = 150°C, hf 5 = 632.2 kJ/kg
s1 = s2 for isentropic process
6.540 = 0.6493 + x2 × 7.5009
or x2 = 0.7853
h2 = hf 2 + x2 hfg2 = 191.83 + 0.7853 × 2392.8 = 2070.89 kJ/kg
h3 = h4 = hf 2 = 191.83 kJ/kg

(a) Let m s = mass flow rate of steam in kg/s

Then m s (h1 − h2) = P

or m s (3361 − 2070.89) = 125 × 103

or m s = 96.89 kg/s

m s ( h1 − h f 5 )
(b) Boiler efficiency, hb =
m f × CV

T
1
773 K

110 bar

Tfw = 423 K 5
4

0.1 bar
3 2

Figure 3.34  Temperature-entropy diagram

M03_THERMAL ENGINEERING_SE_XXXX_CH03_(Part1).indd 125 6/15/2018 1:52:28 PM


126 Chapter 3

96.89(3361 − 632.2)
or 0.92 =
m f × 25, 000

Rate of fuel burned, m f = 11.495 kg/s

m s 96.99
(c) Evaporation factor = = = 8.43
m f 11.495
(d) Pressure head available for natural circulation,
∆p = gH (rf − rm)
At 110 bar, vf = 0.001489 m3/kg, vf = 0.015987 m3/kg
vtop = 0.001489 + 0.085 (0.015987 − 0.001489) = 0.002721 m3/kg
1 1
r top = = = 367.47 kg/m3
vtop 0.002721

1 1
rf = = = 671.6 kg/m3
vf 0.001489

1 1
rm = ( r f + r top ) = (671.6 + 367.47) = 519.53 kg/m3
2 2
∆p = 30 × 9.81 (671.6 − 519.53) = 44.754 kPa or 0.44754 bar
1 1
(e) Circulation ratio, CR = = = 11.765
xtop 0.085
(f) di = d0 − 6 = 50 − 6 = 44 mm
Rate of steam formation in riser,

p  p
m sr =  di2  r top ve × xtop = (0.044)2 × 367.47 × 2 × 0.085 = 0.095 kg/s
4  4
96.89
Number of risers = m s /m sr = = 1020
0.095
(g) At 110 bar, hfg = 1255.5 kJ/kg from steam tables.
Heat absorption rate per unit projected area of the riser
m sr × h fg 0.095 × 1255.5
= = = 79.51 kW/m2
d0 H 0.05 × 30

Example 3.7
An oil fired boiler uses 52 kg/h of oil having an HCV of 44,900 kJ/kg and composition/kg:
C = 0.847, H2 = 0.13, S = 0.0125, and generates 635 kg of steam/h at 10.5 bar pressure from

M03_THERMAL ENGINEERING_SE_XXXX_CH03_(Part1).indd 126 6/15/2018 1:52:34 PM


Steam Generators 127

feed water supplied at 338 K. The flue gases having dry gas analysis/unit volume: O2 = 0.043,
CO2 = 0.124, N2 = 0.833, and specific heat 1.005 kJ/kg-K, leave the boiler at 635 K. The pres-
sure and temperature of steam after throttling in a throttling calorimeter is 1.15 bar and 398 K,
respectively. Taking the partial pressure of steam in flue gases as 0.07 bar and the specific heat of
superheated steam = 2.1 kJ/kg-K, determine (a) the equivalent evaporation/kg of fuel and (b) boiler
efficiency.
Draw up a complete heat balance sheet. Take boiler room temperature = 234 K. [IES, 2000]

Solution
Given: m f = 52 kg/h, HCV = 44,900 kJ/kg, m s = 635 kg/h, p1 = 10.5 bar,
Tf = 338 K, p2 = 1.15 bar, Tsup = 398 K, pvs = 0.07 bar, Tg = 635 K,
Ta = 234 K, cps = 2.1 kJ/kgK, cpg = 1.005 kJ/kgK,
Fuel: C = 0.847, H2 = 0.13, S = 0.0125 per kg
Flue gas: O2 = 0.043, CO2 = 0.124, N2 = 0.833
Throttling calorimeter:
hf 1 + x1 hfg1 = hf 2 + hfg2 + cps (Tsup − Ts2)
From steam tables, we have
At p1 = 10.5 bar, hf 1 = 772.2 kJ/kg, hfg1 = 2007.7 kJ/kg

At p2 = 1.15 bar, hf 2 = 434.0 kJ/kg, hfg2 = 2247.5 kJ/kg, ts2 = 102.05°C

or Ts2 = 375.05 K

772.2 + x1 × 2007.7 = 434.0 + 2247.5 + 2.1 (398 − 375.02) = 2729.76

1957.56
or x1 = = 0.975
2007.7
Dryness fraction of steam = 0.975
(a) Equivalent evaporation:
h1 = hf 1 + x1 hfg1 = 772.2 + 0.975 × 2007.7 = 2729.7 kJ/kg

At Tf = 338 K, hf = cpw (Tf − 273) = 4.187 (338 − 273)

= 272.1 kJ/kg
hfg(atm) = specific enthalpy of evaporation at standard atmospheric pressure of 1.01325 bar
= 2256.9 kJ/kg
 h1 − h f  m s
Equivalent evaporation, me =  ×
 h fg ( atm )  m f

M03_THERMAL ENGINEERING_SE_XXXX_CH03_(Part1).indd 127 6/15/2018 1:52:35 PM


128 Chapter 3

 2729.7 − 272.1 635


=   × 52 = 13.29 kg/kg of fuel
 2256.9 

m s ( h1 − h f )
(b) Boiler efficiency, hb =
m f × HCV

635( 2729.7 − 272.1)


= = 0.6684 or 66.84%
52 × 44900
Heat balance sheet
m s
(i) Heat used to generate steam per kg of fuel = (h − h f )
m f 1
635
= = (2729.7 − 272.1) = 30011 kJ/kg of fuel
52
N×C
(ii) Air supplied per kg of fuel burned =
33(C1 + C2 )

83.3 × 84.7
= = 17.242 kg
33(0 + 12.4)

Dry flue gases formed per kg of fuel burned, mg = 17.242 + 0.847 = 18.089 kg
Heat carried away by dry flue gases = mg × cpg (Tg − Ta)
= 18.089 × 1.005 (635 − 234) = 7253.7 kJ/kg of fuel
(iii) Moisture formed per kg of fuel, mw = 0.13 × 9 = 1.17 kg
Heat carried away by moisture in flue gases per kg of fuel
= mw [hg + Cps (Tsup − Ts) − hf] at p = 0.07 bar
= 1.17 [2572.5 + 2.1 (635 − 312) − 163.4] [∴ Ts = 39°C at 0.07 bar]
= 3087.4 kJ/kg of fuel
(iv) Heat un-accounted for = 44,900 − (30,011 + 7253.7 + 3087.4)
= 4547.9 kJ/kg of fuel

Heat supplied kJ/kg of fuel Heat utilised kJ/kg of fuel %


Heat supplied by fuel 44,900 1. To generate steam 30,011.0 66.84
2. Carried away by flue gases 7253.7 16.16
3. Carried by moisture in gases 3087.4 6.87
4. Unaccounted for 4547.9 10.13
Total 44,900 44,900 100.0

M03_THERMAL ENGINEERING_SE_XXXX_CH03_(Part1).indd 128 6/15/2018 1:52:43 PM


Steam Generators 129

Example 3.8
During a boiler trial, steam was produced at a pressure of 20 bar and superheat temperature of
360°C using feed water at 30°C at the rate of 90000 kg/h. Coal with 3% moisture having calorific
value of 34000 kJ/kg of dry coal was burnt at the rate of 1000 kg/h. The temperature of flue gases
at the exit was 200°C, whereas inlet air was at 30°C. Coal contains 80% carbon, 6% hydrogen,
and rest ash. Volumetric analysis of dry flue gases is: CO2 = 12 %, O2 = 10% and N2 = 78 %.
Enthalpy of steam at 20 bar, 360° C = 3159.3 kJ/kg, enthalpy of water at 30°C = 126 kJ/kg. In the
flue gas, the enthalpy of steam at exit condition is 2879.7 kJ/kg. Specific heat of gas = 1.01 kJ/kg.
Draw the heat balance sheet. [IES, 1998]

Solution
Steam condition: 20 bar, 360°C
Feed water: 30°C, 90,000 kg/h
Moisture in coal = 3 %, CV of coal = 34000 kJ/kg of dry coal, m· f = 1000 kg/h
Exhaust temperature of flue gases = 200°C
Inlet temperature of air = 30°C
Coal composition: C = 80 %, H2 = 6 %, rest is ash.
Volumetric analysis of dry flue gases:
CO2 = 12%, O2 = 10 %, N2 = 78 %
Enthalpy of steam at 20 bar, 360°C = 3159.3 kJ/kg
Enthalpy of water at 30°C = 126 kJ/kg
Enthalpy of steam in flue gases at exit = 2879.7 kJ/kg
Specific heat of gas = 1.01 kJ/kg
Heat supplied by coal per kg, Q = (1 − 0.03) C.V. = (1 − 0.03) × 34,000 = 32,980 kJ/kg
Heat used to generate steam per kg of fuel,
m s 90, 000
Q1 = (h − hf) = (3159.3 − 126) = 28,144 kJ/kg
m f 10000 × 0.97
N×C 78 × 80
Air supplied per kg of fuel burnt = = = 15.76 kg
33(C1 + C2 ) 33(0 + 12)
Dry flue gases formed per kg of fuel burned, mg = 15.76 + 0.80 = 16.56 kg
Heat carried away by dry flue gases, Q2 = mg × cpg (Tg − Ta)
= 16.56 × 1.01 (200 − 30) = 2843.3 kJ
Moisture formed per kg of fuel = 0.06 × 9 = 0.54 kg
Heat carried away by moisture in flue gases, Q3 = 0.54 × 2879.7 = 1555 kJ
Heat lost to moisture in coal, Q4 = 0.03 × 2879.7 = 86.39 kJ
Q1 + Q2 + Q3 + Q4 = 28,144 + 2843.3 + 1555 + 86.39 = 32628.69 kJ
Heat unaccounted for, Q5 = 32980 − 32628.69 = 351.31 kJ

M03_THERMAL ENGINEERING_SE_XXXX_CH03_Part2.indd 129 6/15/2018 3:05:52 PM


130 Chapter 3

Heat balance sheet:

Heat supplied kJ/kg % Heat utilised kJ/kg %


Heat supplied by 32,980 100 1.  Heat used to generate steam, Q1 28,144 85.34
fuel
2.  Heat carried away by dry flue gases, Q2 2843.3 8.62
3. Heat carried away by moisture in flue 1555.0 4.71
gases, Q3
4.  Heat lost to moisture in coal, Q4 86.39 0.26
5.  Heat unaccounted for, Q5 351.31 1.07
32,980 100 32,980 100.00

3.15  ❐  STEAM GENERATOR CONTROL


The purpose of steam generator control is to provide the steam flow required by the turbine at design
pressure and temperate. The variables that are controlled are (i) fuel firing rate, (ii) air flow rate, (iii)
gas flow distribution, (iv) feed-water flow rate, and (v) turbine valve-setting.
The key measurements that describe the plant performance are (i) steam flow rate, (ii) steam
pressure, (iii) steam temperature, (iv) primary and secondary air flow rates, (v) fuel firing rate, (vi)
feed-water flow rate, (vii) steam drum level, and (viii) electrical power output.
The control system must act on the measurement of these plant parameters so as to maintain plant
operation at the desired conditions.
We shall discuss only a few basic control systems related to the following:
1. Feed-water and drum level control: Feed water and steam flow are controlled to meet load
demand by the turbine. Therefore, the level of water in the steam drum has to be maintained,
normally half-full up to the diametral plane.
  A schematic diagram for the three-element feed-water control system is shown in Fig. 3.35. It
comprises a drum-level sensor, feed-water flow sensor, and steam flow sensor.
Steam

Normal
Drum flow C water
sensor O level
N
T Steam drum
Feed water flow R Control
sensor O valve
Trimming
L Feed water
Signal L
Steam flow E
sensor R

Figure 3.35  Schematic diagram for a three-element feed-water control system

M03_THERMAL ENGINEERING_SE_XXXX_CH03_Part2.indd 130 6/15/2018 3:05:54 PM


Steam Generators 131

Steam flow Air flow


sensor sensor

Steam Trimming signal


pressure

Pulverized coal Forced draught


power drive fan drive

Figure 3.36  Schematic diagram of a steam pressure control system


2. Steam pressure control: A schematic diagram of steam pressure control system is shown in
Fig. 3.36. It maintains the steam pressure by adjusting fuel and combustion air flow to meet the
desired pressure. When pressure drops, the flows are increased. A steam pressure sensor acts
directly on the fuel flow and air flow controls. A trimming signal from fuel flow and air flow sen-
sors maintains the proper fuel-air ratio.
3. Steam temperature control: For efficient power plant operation, an accurate control of super-
heat temperature of steam is essential. The principal variables affecting superheat temperature
are (i) furnace temperature, (ii) cleanliness of radiant and pendant superheaters, (iii) temperature
of gases entering the convective superheater, (iv) cleanliness of convective superheater, (v) mass
flow rate of gases through the convective superheater, (vi) feed-water temperature, and (vii) vari-
ation of load on the unit.
A reduction in steam temperature results in loss in plant efficiency. On the other hand, a rise in steam
temperature above design value may result in overheating and failure of superheater, reheater tubes,
and turbine blades.
The temperature of the saturated steam leaving the drum corresponds to the boiler pressure and remains
constant if the steam pressure controls are working properly. It is the superheater-reheater responses to
load changes which need to be corrected. Some of the methods to correct them are as follows:
1. Combined radiant-convective superheaters
2. Desuperheating and attemperation
3. Gas by-pass or damper control
4. Gas recirculation
5. Excess air
6. Tilting burners
7. Burner selection
8. Separately fired superheater

3.16  ❐  ELECTROSTATIC PRECIPITATOR


The basic elements of an electrostatic precipitator are shown in Fig. 3.37. It comprises two sets of elec-
trodes insulated from each other. The first set is composed of rows of electrically grounded vertical

M03_THERMAL ENGINEERING_SE_XXXX_CH03_Part2.indd 131 6/15/2018 3:05:54 PM


132 Chapter 3

High voltage 40-80 kV dc


transformer
Insulators
440 V, 50 Hz
Clean gas

3-phase supply Emitting


electrode
Rectifier Dirty gas
Control
cabinet Collecting
electrode

Collected dust

Figure 3.37  Basic elements of an electrostatic precipitator


parallel plates, called the collection electrodes, between which the dust–laden gas flows. The second
set of electrodes consists of wires, called the discharge or emitting electrodes that are centrally located
between each pair of parallel plates. The wires carry a unidirectional negatively charged high voltage
current from an external DC source. The applied high voltage generates a unidirectional, non-uniform
electrical field whose magnitude is the greatest near the discharge electrodes. When that voltage is
high enough, a blue luminous glow, called a corona, is produced around them. Electrical forces in
the corona accelerate the free electrons present in the gas so that they ionise the gas molecules, thus
forming more electrons and positive gas ions. The new electrons create again more free electrons and
ions, which result in a chain reaction.
The positive ions travel to the negatively charged wire electrodes. The electrons follow the elec-
trical field toward the grounded electrodes, but their velocity decreases as they move away from the
corona region around the wire electrodes towards the grounded plates. Gas molecules capture the low
velocity electrons and become negative ions. As these ions move to the collecting electrode, they col-
lide with the fly ash particles in the gas stream and give them negative charge. The negatively charged
fly ash particles are driven to the collecting plate. Collected particulate matter is removed from the
collecting plates on a regular schedule.
The arrangement of an electrostatic precipitator is shown in Fig. 3.38.

High voltage
Shell electrodes

Gas flow

Plan High voltage electrode


Collecting
electrodes support insulators

Figure 3.38  Arrangement of an electrostatic precipitator


Collecting High voltage
electrode electrodes

M03_THERMAL ENGINEERING_SE_XXXX_CH03_Part2.indd 132 6/15/2018 3:05:55 PM


Shell electrodes

Gas flow

Steam Generators 133


Plan High voltage electrode
Collecting
electrodes support insulators

Collecting High voltage


electrode electrodes

Gas flow

Hopper

Elevation

Figure 3.38  (Continued)

Example 3.9
A boiler generates 8 kg of steam per kg of coal burnt at a pressure of 10 bar, from feed water
having a temperature of 65°C. The efficiency of the boiler is 75% and factor of evaporation 1.2.
Specific heat of steam at constant pressure is 2.2 kJ/kgK. Calculate (a) the degree of superheat and
temperature of steam generated, (b) the calorific value of coal used in kJ/kg, and (c) the equivalent
evaporation in kg of steam per kg of coal.

Solution
Given: ms = 8 kg/kg of coal, ps = 10 bar, tfw = 65°C, hb = 75%, Fe = 1.2, cps = 2.2 kJ/kgK
(a) At p = 10 bar, from steam tables:
    hg = 2778.1 kJ/kg, Ts = 273 + 179 = 452 K
Also, hf1 = cpw tfw = 4.187 × 65 = 272.155 kJ/kg of feed water
     hfg = 2256.9 kJ/kg at atmospheric pressure.
hg + C ps (Tsup − Ts ) − h f 1
Factor of evaporation, Fe =
h fg
2778.1 + 2.2(Tsup − 452) − 272.155
 1.2 =
or     
2256.9
or 2708.28 = 2505.945 + 2.2 (Tsup − 452)
or Tsup = 543.97 K or tsup = 270.97°C
Degree of superheat = Tsup − Ts = 543.97 − 452 = 91.97°C
ms [hg + c ps (Tsup − Ts ) − h f 1 ]
(b) Boiler efficiency, hb =
C.V.

M03_THERMAL ENGINEERING_SE_XXXX_CH03_Part2.indd 133 6/15/2018 3:05:56 PM


134 Chapter 3

8 [2778.1 + 2.2 × 91.97 − 272.155] 21, 330.192


or 0.75 = =
C.V C.V
or C.V. = 28,440 kJ/kg of coal
ms [h1 − h f 1 ]
(c) Equivalent evaporation, me =
m f × 2256.9
21, 330.192
= = 9.45 kg of steam/kg of coal
1 × 2256.9

Example 3.10
The following observations were made during the trial of a gas-fired boiler of a steam power plant:
Generator output = 50,000 kW
Steam conditions = pressure 50 bar temperature 500°C
Feed-water temperature = 80°C
Steam output = 65 kg/s
Percentage composition of fuel (natural gas), by volume CH4 = 96.5, C2H6 = 0.5 (rest
incombustibles)
HCV of gas = 38,700 kJ/m3, at 1.013 bar and 15°C
Gas consumption = 6.5 m3/s
Calculate the boiler efficiency and the overall thermal efficiency based on the lower calorific
value of the fuel. [IES, 1996]
Solution
Combustion equations:
CH4 + 2O2 → CO2 + 2H2O

C2H6 + 2.5O2 → 2CO2 + 3H2O


1 mole of CH4 burns to give 2 moles of water
Therefore, 0.965 moles give 2 × 0.965 × 18 = 34.74 kg of H2O
Similarly 0.005 moles of C2H6 burns to give 3 × 0.005 × 18 = 0.27 kg of H2O
Assuming steam as a perfect gas, 1 mole of gas at 1.013 bar and OC is 22.41 m3
288
∴ 1 mole of gas at 15°C = 22.41 × = 23.64 m3
Mass of steam produced, 273
(34.74 + 0.27)
ms = = 1.481 kg/m3 of fuel
23.64
HCV = LCV + heat in steam
HCV = 38,700 − ms hfg at 15°C
= 38,700 − 1.481 × 2465.9 [hfg at 15°C = 2465.9 kJ/kg]
= 35,043.07 kJ/kg
At 50 bar and 500°C, h1 = 3433.8 kJ/kg
At 80°C, hfg for feed water = 334.98 kJ/kg
Enthalpy of steam = h1 − hfg = 3433.8 − 334.9 = 3098.9 kJ/kg
Heat to water in boiler
Boiler efficiency =
Heat supplied to boiler

M03_THERMAL ENGINEERING_SE_XXXX_CH03_Part2.indd 134 6/15/2018 3:05:59 PM


Steam Generators 135

mw ( h1 − h fg ) 65 × 3098.9
= = = 0.8843 or 88.43 %

V f × HCV 6.5 × 35043.07
Generator output
Overall thermal efficiency =
Heat supplied
50, 000
= = 0.2195 or 21.95 %
6.5 × 35043.07

Example 3.11
In a trial on a boiler fitted with an economiser, the following results were obtained:

Volumetric analysis, %
CO2 O2 N2
Gas entering the economiser 8.3 11.4 80.3
Gas leaving the economiser 7.9 11.5 80.6

The fuel contains 80 % carbon and it does not contain nitrogen.


Calculate (a) the air leakage into the economiser per minute if the fuel used in the boiler is
60 kg/min. (b) the reduction in temperature of the gas due to air leakage if the atmospheric air
temperature is 20°C and the flue gas temperature is 350°C.
For air: specific heat, cpa = 1.005 kJ/kgK and for gas, cpg = 1.1 kJ/kgK.
Percentage of incombustible in fuel = 15%

Solution
Air supply per kg of fuel for the gas analysis entering the economiser,
NC 80.3 × 80
mai = = = 23.4 kg
33(C1 + C2 ) 33(0 + 8.3)
Air supply per kg of fuel for the gas analysis leaving the economiser,
80.6 × 80
mao = = 24.7 kg
33(0 + 7.9)
Air leakage into the economiser per kg of fuel burned in boiler
= mao − mai = 24.7 − 23.4 = 1.3 kg
Air leakage in the economiser per minute = 1.3 × 60 = 78 kg
Mass of flue gas products formed per kg of coal = 23.4 + (1 − 0.15) = 24.25 kg
∴ cpg mg Tg + cpa ma Ta = cpg (mg + ma) Tm
where Tm = temperature of exhaust gases leaving the economiser
1.1 × 24.25 × 623 + 1.005 × 1.3 × 293 = 1.1 (24.25 + 1.3) × Tm
or Tm = 604.9 K or 331.9°C
Drop in temperature of exhaust gases due to air leakage into the economiser
= 350 − 331.9 = 18.1°C

M03_THERMAL ENGINEERING_SE_XXXX_CH03_Part2.indd 135 6/15/2018 3:06:01 PM


136 Chapter 3

3.17  ❐  DRAUGHT
In order to maintain the continuous flow of fresh air into the combustion chamber, it is necessary to
exhaust the products of combustion from the combustion chamber of a boiler. A pressure difference
has to be maintained to accelerate the products of combustion to their final velocity and to overcome
the pressure losses in the flow system. This pressure difference so maintained is called “draught”.

3.17.1 Classification of Draught


The method of producing draught may be classified as follows:
1. Natural (or chimney) draught
2. Artificial draught
(a) Steam jet draught
     (i)  Induced draught
     (ii)  Forced draught
(b) Mechanical draught
   (i)   Induced fan draught
   (ii)  Forced fan draught
(iii)   Balanced draught

3.17.2 Natural Draught


Natural draught is obtained by the use of a chimney. A chimney is a vertical tubular structure of brick,
masonry, steel, or reinforced concrete, built for the purpose of enclosing a column of hot gases, to
produce the draught. The draught produced by the chimney is due to the density difference between
the column of hot gases inside the chimney and the cold air outside.
A diagrammatic arrangement of a chimney of height H in m above the grate is shown in
Fig. 3.39.
Pressure at the grate level on the chimney side,
p1 = patm + ggh
where patm = atmospheric pressure at the chimney top

patm patm

Chimney

H Combustion H
chamber
U-tube water
Grate level
manometer

hw

Figure 3.39  Diagrammatic arrangement of natural chimney draught

M03_THERMAL ENGINEERING_SE_XXXX_CH03_Part2.indd 136 6/15/2018 3:06:02 PM


Steam Generators 137

gg = specific weight of chimney hot gases.


Pressure acting on the grate on the open (atmospheric) side,
p2 = patm + gah
where ga = specific weight of air outside the chimney.
Net pressure difference causing the flow through the combustion chamber, static draught,
∆p = p2 − p1 = (ga − gg)H(1)
Static draught can be measured by a water manometer
Let hw = difference in water level in the two legs of U-tube manometer, mm
g h
Then p = w w N/m2
103
103 × hw
= = hw N/m2(2)
103
Equating the above two equations, we get
∆p = hw = (ga − gg)H

3.17.3 Height and Diameter of Chimney


It can be assumed that the volume of combustion products is equal to the volume of air supplied when
both reduced to the same temperature and pressure conditions.
Let ma = mass of air supplied per kg of fuel
Tg = average absolute temperature of chimney gases
Ta = absolute temperature of atmospheric air
Density of hot gases rg ma + 1
= =
Density of air ra ma
Density of air at atmospheric conditions,
p 1.01325 × 105 353
ra = = =
RTa 287 × Ta Ta
Density of hot gases at temperature, Tg,
p  ma + 1
rg =
RT  m  g a

1.01325 × 105  ma + 1
    =
287 × Tg  ma 
353  ma + 1
     =
Tg  ma 
1 1  ma + 1 
∆p = hw = 353H  −  
 Ta Tg  ma  
Let ∆p be equivalent to H1 metre height of burnt gases.
353H1  ma + 1
Then ∆p = g g H1 =
Tg  ma 

M03_THERMAL ENGINEERING_SE_XXXX_CH03_Part2.indd 137 6/15/2018 3:06:05 PM


138 Chapter 3

 ma  Tg 
∴ H1 = H   . − 1
 ma + 1 Ta 
The velocity of gases passing through the chimney is given by,
v = 2 gH1 ,neglecting losses
If the pressure loss in the chimney is equivalent to a hot gas column of height h1, then
v = 2 g ( H1 − h1 )
h1
= 4.43 H1 × 1 − = k H1
H1
h1
where k = 4.43 1 −
H1
= 0.825 for brick chimney
= 1.1 for steel chimney
Mass of gas flowing through any cross-section of chimney is given by,
mg = Avrg kg/s
p 2
∴     D × v + rg = mg
4
mg
D = 1.128
or   
vrg

3.17.4 Condition for Maximum Discharge Through Chimney


Neglecting losses in chimney,
u = 2 gH1
 ma  Tg 
= 2 gH   − 1
 ma + 1 Ta 
p
Density of hot gases, rg =
RTg
Mass of gases discharged per second, mg = Avrg

Ap  ma  Tg 
mg = 2 gH   . − 1
RTg  ma + 1 Ta 

K1  ma  Tg 
=   − 1
Tg  ma + 1 Ta 
Ap 2 gH
where K1 = is a constant
R dmg
The value of mg will be maximum when = 0, as Ta and ma are fixed.
dTg

M03_THERMAL ENGINEERING_SE_XXXX_CH03_Part2.indd 138 6/15/2018 3:06:10 PM


Steam Generators 139

d  k1  ma  Tg 
  m + 1 T − 1 =0
dTg  Tg 
 a a
 ma  1
Let C =  ×
 ma + 1 Ta

d  (CTg − 1) 
1/ 2
∴       =0
dTg  Tg 
 
1
Tg × (CTg − 1)−1/2 × C − 1 × (CTg − 1)1/2 = 0
2
or     CTg − 2(CTg − 1) = 0
or    CTg = 2
 ma  Tg
or    m + 1 × T = 2
a a
Tg 2( ma + 1)
or     =
Ta ma
1 1  H
∴   w max = 353H 
    (h ) −  = 176.5
 Ta 2Ta  Ta
 ma  2( ma + 1) 
(H1)max = H  × m − 1 = H
 ma + 1 a 
Ap
mg = Arg 2 gH1 = 2 gH1
RTg
Maximum discharge,
Ap 1  ma 
( mg ) max = × × 2 gH
R 2T  m + 1a a
Ap  ma  H
= 2.22 ×
R  ma + 1 Ta

3.17.5 Efficiency of Chimney


A certain minimum flue gas temperature is required to produce a given draught with a given height of
chimney. Therefore, the temperature of flue gases leaving the chimney in case of natural draught has
to be higher than that of flue gases temperature in the case of artificial draught.
Let Tn = absolute temperature of flue gases leaving the chimney to create the draught of hw mm of
water.
Tm = absolute temperature of flue gases leaving the chimney in case of artificial draught.
cpg = mean specific heat of flue gases
Extra heat carried away per kg of flue gases due to higher temperature required in natural draught
= cpg (Tn − Tm) as Tn > Tm

M03_THERMAL ENGINEERING_SE_XXXX_CH03_Part2.indd 139 6/15/2018 3:06:14 PM


140 Chapter 3

Draught pressure produced by natural draught system in m of hot gases,


 ma  Tg 
H1 = H   − 1
 ma + 1 Ta 
Maximum energy given by H1 to one kg of air at the cost of extra heat
 ma  Tg 
      = 1 × H   − 1
 ma + 1 Ta 
  ma  Tg 
  − 1
 ma + 1 Ta
Efficiency of chimney, hch = H  
 c (T − T ) 
 pg n m 
 
where Tn = Tg
Generally hch < 1%

3.17.6 Advantages and Disadvantages of Natural Draught


Advantages of natural draught are as follows:
1. No power is required to produce draught.
2. The life of chimney is quite long.
3. The chimney does not require much maintenance.
Disadvantages of natural draught are as follows:
1. There is very low efficiency.
2. It requires a tall chimney (30.48 m minimum).
3. Efficiency is dependent on atmospheric temperature.
4. There is no flexibility on draught.

3.17.7 Draught Losses


The causes of the losses in draught are as follows:
1. Frictional resistance offered by the gas passages during flow of gas.
2. Loss near the bends in the gas flow circuit.
3. Frictional head loss in different equipment, for example, grate, economiser, superheater, etc.
4. Heat lost to impact velocity to flue gases.
The total loss is nearly 20% of the total static draught produced by chimney.

3.17.8 Artificial Draught


A draught produced by artificial means which is independent of the atmospheric conditions is called
artificial draught. It gives greater flexibility to take the fluctuating loads on the plant. The artificial
draught may be of mechanical type or of steam jet type. If the draught is produced by a fan, it is called
fan (or mechanical) draught, and if it is produced by a steam jet, it is called steam jet draught. Steam
jet draught is used for small installations such as locomotives, whereas mechanical draught is invari-
ably used in central power stations.

M03_THERMAL ENGINEERING_SE_XXXX_CH03_Part2.indd 140 6/15/2018 3:06:15 PM


Steam Generators 141

To exhaust

Furnace
Blower
Grate
Boiler Economiser Air-preheater Chimney
or
stack

Figure 3.40  Forced draught


Forced Draught
In a forced draught system, a blower is installed near the base of the boiler and air is forced to pass
through the furnace, flues, economiser, air-preheater, and to the stack. It is called forced positive
draught system because the pressure of air throughout the system is above atmospheric pressure and
air is forced to flow through the system. The arrangement of the system is shown in Fig. 3.40. A stack
or a chimney is also used to discharge gases high into the atmosphere for better dispersion of ash
particles and pollutants.

Induced Draught
In this system, the blower is located near the base of the chimney. The air is sucked into the system
by reducing the pressure through the system below the atmospheric pressure. The induced draught
fan sucks the gases from the furnace and the pressure inside the furnace is reduced below that of the
atmospheric pressure, thus inducing the atmospheric air to flow through the furnace. The draught
produced is independent of the temperature of hot gases. Therefore, the gases may be discharged as
cold as possible after recovering as much heat as possible in the economiser and the pre-heater. The
arrangement of the system is shown in Fig. 3.41.
To exhaust

Furnace

Grate
Air in Air in Economiser Air-preheater
Boiler Blower Chimney
or
stack

Figure 3.41  Induced draught

M03_THERMAL ENGINEERING_SE_XXXX_CH03_Part2.indd 141 6/15/2018 3:06:16 PM


142 Chapter 3

3.17.9 Comparison of Forced and Induced Draughts


The comparison of forced and induced draughts in given in Table 3.3.

Table 3.3  Comparison of forced and induced draughts

Parameters Forced draught Induced draught


1.  Power required Less More
2. Size Small About three times larger
3.  Cooling of bearings Not required Required
4. Chances of air leakage in furnace No chance Continuous leakage
5.  Rate of burning Better Poor
6.  Effect of opening of doors No effect on efficiency Efficiency decreases
7.  Erosion of fan blades Slow Fast
8.  Blow out Yes No

3.17.10  Comparison of Mechanical and Natural Draughts


The comparison of mechanical and natural draughts is given in Table 3.4.

Table 3.4  Comparison of mechanical and natural draughts

Parameter Mechanical draught Natural draught


1.  Rate of combustion High Low
2.  Air flow control Yes No
3.  Effect of atmospheric temp No effect Seriously affected
4.  Use of low-grade fuel Possible Impossible
5.  Overall thermal efficiency More Less
6.  Height of chimney Less More
7. Efficiency About 7% Less than 1%
8.  Fuel consumption per kW 15% less More
9.  Fuel burning capacity More Less
10.  Formation of smoke No Yes

3.17.11  Balanced Draught


A balanced draught is a combination of forced and induced draughts. The forced draught overcomes
the resistance of the fuel bed and therefore sufficient air is supplied to the fuel bed for proper and
complete combustion. The induced draught fan removes the gases from the furnace, maintaining the
pressure in the furnace just below atmospheric pressure. This prevents to blow-out of flames and leak-
age of air inwards, when the furnace doors are opened. The balanced draught is shown in Fig. 3.42.

M03_THERMAL ENGINEERING_SE_XXXX_CH03_Part2.indd 142 6/15/2018 3:06:16 PM


Steam Generators 143

To exhaust

Furnace

Forced fan
Grate
Economiser Air-preheater
Boiler Induced Chimney
fan

Figure 3.42  Balanced draught

3.17.12  Steam Jet Draught


A steam jet draught of the induced type is shown in Fig. 3.43. A steam nozzle located near the smoke
box induces the flow of gases through the tubes, ash pit, grate, and flues. It is generally used for a
locomotive boiler.
The steam jet creates pressure below that of the atmosphere in the smoke box. The advantages of
steam jet draught are as follows:
1. Very simple and economical
2. Low-grade fuel can be used
3. Forced type system keeps the fire bars cool and prevents the adhering of clinker to them
4. Maintenance cost is nil
5. Occupies minimum space
However, it cannot be started until high pressure steam is available.

Smoke box Chimney

Water

Hot gases

Exhaust Steam
steam from Nozzle
steam engine

Figure 3.43  Steam jet draught

M03_THERMAL ENGINEERING_SE_XXXX_CH03_Part2.indd 143 6/15/2018 3:06:17 PM


144 Chapter 3

Example 3.12
A boiler is equipped with a chimney of 25 m height. The ambient temperature is 27°C. The tem-
perature of flue gases passing through the chimney is 300°C. If the air flow through the combustion
chamber is 20 kg per kg of fuel burned, then find the following:
(a) Theoretical draught in cm of water
(b) The velocity of flue gases passing through the chimney if 50% of theoretical draught is lost in
friction at grate and passage.
Solution
Given that H = 25 m, Ta = 27 + 273 = 300 K, Tg = 300 + 273 = 573 K, ma = 20 kg/kg fuel
1 1  ma + 1 
(a) hw = 353 H  −    mm of H2O
 Ta Tg  ma  
 1 1 21 
= 353 × 25  − ×  = 13.245 mm of water or 1.3245 cm of H2O
 300 573 20 
Equivalent gas head,
 ma  Tg 
H1 = H   × − 1
 ma + 1 Ta 
 20 573 
= 25  × − 1 = 20.476 m
 21 300 
(b) Available head, Hav = 20.476 × 0.5 = 10.238 m
 Velocity of flue gases, v = 2gH av = 2 × 9.81 × 10.238 = 14.17 m/s

Example 3.13
Determine the height of the chimney required to produce a draught equivalent to 2 cm of water if
the flue gas temperature is 240°C and ambient temperature is 25°C. The minimum amount of air
per kg of fuel is 18 kg.

Solution
Given that hw = 2 cm of water, Tg = 240 + 273 = 513 K, Ta = 25 + 273 = 298 K, ma = 18 kg/kg fuel
1 1  ma + 1 
hw = 353 H  −    mm of H2O
 Ta Tg  ma  
 1 1  18 + 1 
or 20 = 353 H  −  
 298 513  18  
or H = 43.646 m

Example 3.14
Find the mass of flue gases flowing through the chimney when the draught produced is equal to
2 cm of water. The temperature of flue gases is 305°C and the ambient temperature is 27°C. The flue
gases formed per kg of fuel burned are 20 kg. The diameter of the chimney is 2 m. Neglect losses.

M03_THERMAL ENGINEERING_SE_XXXX_CH03_Part2.indd 144 6/15/2018 3:06:20 PM


Steam Generators 145

Solution
Given that hw = 2 cm, Tg = 305 + 273 = 578 K, Ta = 27 + 273 = 300 K,
1 + ma = 20 kg/kg of fuel.
 ma  Tg 
H1 = H   − 1
 ma + 1 Ta 
1 1  ma + 1 
hw = 353 H  −  
 Ta Tg  ma  
 1 1 20 
or 20 = 353H  − × 
 300 578 19 
or   H = 37.467 m
 19 578 
H1 = 37.467  × − 1 = 31.11 m of air
 20 300 
v = 2 gH1 = 2 × 9.81 × 31.11 = 24.7 m/s
p 1.01325 × 105
= = 0.6108 kg/m3
rg =
RTg 287 × 578
mg = Avrg
p
or mg = × 22 × 24.7 × 0.6108 = 47.4 kg/s
4
Example 3.15
The height of a chimney used in a plant to provide natural draught is 15 m. The ambient air temper-
ature is 20°C. (a) Find the draught in mm of water when the temperature of chimney gases is such
that the mass of gases discharged is maximum and (b) if temperature of gases should not exceed
330°C, then find the air supplied per kg of fuel for maximum discharge.
Solution
Given that H = 15 m, Ta = 20 + 273 = 293 K, Tg = 330 + 273 = 603 K
H 15
(a) (hw)max = 176.5 = 176.5 × = 9 mm of H2O
Ta 293
(b) For maximum discharge,
Tg  m + 1
= 2 a 
T  m 
a a

603  m + 1
or    = 2 a 
293  ma 
or 1.029 ma = ma + 1
or     ma = 34.47 kg/kg of fuel

Example 3.16
A boiler is equipped with a chimney of 30 m height. The ambient temperature is 25°C. The tem-
perature of flue gases passing through chimney is 300°C. If the air flow is 20 kg/kg of fuel burnt,

M03_THERMAL ENGINEERING_SE_XXXX_CH03_Part2.indd 145 6/15/2018 3:06:24 PM


146 Chapter 3

then find (a) draught produced and (b) the velocity of flue gases passing through chimney if 50%
of the theoretical draught is lost in friction.
Solution
H = 30 m, Ta = 273 + 25 = 298 K, Tg = 273 + 300 = 573 K, ma = 20 kg, mf = 1 kg
1 1  ma + 1 
(a) Draught produced, hw = 353 H  −    mm of H2O
 Ta Tg  ma  
 1 1  20 + 1 
= 353 × 30  −  
 298 573  20  
= 16.13 mm of H2O or 1.613 cm of H2O
 ma  Tg 
(b) Equivalent gas head,H1 = H   − 1
 ma + 1 Ta 
 20  573 
= 30  × − 1 = 24.9376 m
 20 + 1 298 
Available head, Hav = 0.5 H1 = 12.469 m
Velocity of flue gases, v = 2gH au = 2 × 9.81 × 12.469 = 15.64 m/s

Example 3.17
A boiler uses 18 kg of air per kg of fuel. Determine the minimum height of chimney required to
produce a draught of 25 mm of water. The mean temperature of chimney gases is 315°C and that of
outside air 27°C.
Solution
Given: ma = 18 kg, mf = 1 kg, hw = 25 mm of H2O, Tg = 273 + 315 = 588 K,
1 1  ma + 1 
hw = 353H  −    mm of H2O
 Ta Tg  ma  
 1 1  18 + 1 
or 25 = 353H  −  
 300 588  18  
or H = 46.04 m

Example 3.18
With a chimney of height 45 m, the temperature of flue gases with natural draught was 370°C.
The same draught was developed by induced draught fan and the temperature of the flue gases was
150°C. The air flow through the combustion chamber is 25 kg per kg of coal fired. The boiler house
temperature is 35°C. Assuming cp = 1.004 kJ/kg K for the flue gases, determine the efficiency of
the chimney.

Solution
Given that H = 45 m, Tgn = 273 + 370 = 643 K, Tgi = 273 + 150 = 423 K
mg = 25 kg/kg of coal fired, Ta = 273 + 35 = 308 K, cpg = 1.004 kJ/kgK

M03_THERMAL ENGINEERING_SE_XXXX_CH03_Part2.indd 146 6/15/2018 3:06:27 PM


Steam Generators 147

 mg  Tgn 
H   − 1
 mg + 1 Ta 
Efficiency of Chimney =
c pg (Tgn − Tgi )

 25  643 
45  × − 1
=  25 + 1 308  × 100 = 0.02052%
1.004 × 103 (643 − 423)

Example 3.19
A boiler is provided with a chimney of 24 m height. The ambient temperature is 25°C. The temper-
ature of flue gases passing through the chimney is 300°C. If the air flow through the combustion
chamber is 20 kg/kg of fuel burnt, then find (a) the theoretical draught in cm of water and (b) the
velocity of flue gases passing through the chimney if 50% of theoretical draught is lost in friction
at grate and passage.

Solution
Given that H = 24 m, Ta = 273 + 25 = 298 K, Tg = 273 + 300 = 573 K
ma = 20 kg/kg of fuel burnt,
1 1  ma + 1 
(a) hw = 353 H  −  
 Ta Tg  ma  
 1 1 21 
= 353 × 24  − ×  = 12.9 mm of water
 298 573 20 
(b) Equivalent gas head,
 ma  Tg 
H1 = H   − 1
 ma + 1 Ta 
 20 573 
= 24  × − 1 = 19.9 m
 21 298 
Hav = 0.5 H1 = 9.95 m
v = 2 gH au = 2 × 9.81 × 9.95 = 13.972 m/s

Example 3.20
The following data relate to a boiler using induced draught system:
Length of the duct carrying flue gases = 150 m
Mean size of the square duct = 75 cm2
Mean flue gas velocity in the duct = 900 m/min
Mean temperature of gases passing through duct = 227°C
Plenum pressure = 15 cm of water
Atmospheric pressure = 75 cm of Hg
Number of 90° bends = 4
Number of 45° bends = 4

M03_THERMAL ENGINEERING_SE_XXXX_CH03_Part2.indd 147 6/15/2018 3:06:29 PM


148 Chapter 3

Loss of draft in every 90° bend = 0.1 cm of water


Draft available from chimney = 1.5 cm of water
Fuel bed resistance for under fired stoker = 10 cm of water
Fan efficiency = 60%
Motor efficiency driving fan = 92.5%
Characteristic gas constant for gases = 0.294 kJ/kgK
Friction factor, f, corresponding to round section duct = 0.006
Assuming that a 45° bend equal one-half of 90° bend and that the resistance offered by square duct
is 20 % greater than a similar round duct, determine the following:
(a)  Draft to be produced due to friction in square cross-section duct in mm of water
(b)  Draft due to velocity head of flue gases in mm of water
(c)  Total draft for the boiler
(d)  Draft to be produced by induced fan
(e)  Power required to drive fan [IES, 2011]
Solution
Given that L = 150 m, f = 0.006, As = 75 cm2, vg = 900 m/min, C = 1.2, ∆p = 15 cm of water,
tg = 227°C, patm = 75 cm of Hg, hf = 0.6, hm = 0.925, Rg = 0.294 kJ/kgK

4 fLu g2
(a) Frictional head loss in duct carrying flue gases, h f = ×C
2 gd
4 As 4 × 75
Equivalent diameter of round duct, d = = = 9.772 cm
p p
4 × 0.006 × 150 × 152 × 1.2
hf = = 506.97 m of flue gas
2 × 9.81 × 9.772 × 10 −2
Chimney draft produced, pc = 15 mm of water or 15 × 9.81 = 147.15 N/m2
1.01325 × 105 × 75
Atmospheric pressure, patm = = 0.99992 × 105 N/m2
76
Pressure of hot flue gases, pg = patm − pc = 0.9992 × 105 − 147.15
= 99,844.6 N/m2
pg 99, 844.6
Density of hot flue gases, rg = = = 0.6792 kg/m3
Rg Tg 294 × ( 273 + 227)
Frictional head loss in square duct in terms of mm of water
h f × rg 506.97 × 0.6792
= = = 0.344 m of water
rw 1000
= 344 mm of water
Plenum pressure drop in duct = 150 mm of water
Draft to be produced due to friction in duct = 344 + 150 = 494 mm of water
(b) Draft due to velocity head of flue gases:
0.9u g2 152
4 bends of 90° = 4 × = 4 × 0.9 × = 41.28 m of hot flue gases
2g 2 × 9.81
4 bends of 45° = 0.5 × 41.28 = 20.64 m of hot flue gases

M03_THERMAL ENGINEERING_SE_XXXX_CH03_Part2.indd 148 6/15/2018 3:06:32 PM


Steam Generators 149

Total draft due to velocity head = 41.28 + 20.64 = 61.92 m of hot flue gases
61.92 × 0.6792
= = 0.042 m of H2O or 42 mm of H2O
103
Loss of draft due to 90° bends = 4 × 1 = 4 mm of water
Loss of draft due to 45° bends = 2 mm of water
(c) Fuel bed resistance for under fired stoker = 100 mm of water
Total draft for the boiler = 344 + 150 + 42 + 4 + 2 = 542 mm of water
(d) Draft to be produced by induced fan, h = 542 + 100 = 642 mm of H2O
(e) Volume flow rate of flue gases through the duct, V = As × vg
= 75 ×10−4 × 15 = 0.1125 m3/s
ghV 9.81 × 642 × 0.1125
Power required to drive fan = = = 1.277 kW
3
10 × h f × h m 103 × 0.6 × 0.925

Example 3.21
In a steam power plant, the steam generator generates steam at the rate of 120 t/h at a pressure of
100 bar and temperature of 500°C. The calorific value of fuel used by steam generator is 41 MJ/kg
with an overall efficiency of 85 %. In order to have efficient combustion, 17 kg of air per kg of fuel
is used for which a draught of 25 mm of water gauge is required at the base of stack. The flue gases
leave the steam generator at 240°C. The average temperature of gases in the stack may be taken as
200°C and the atmospheric temperature is 30°C. Determine (i) the height of stack required, and
(ii) the diameter of stack at its base. Take the following steam properties for solution: h = 3375 kJ/kg,
hf = 632.2 kJ/kg. [IAS, 2010]

Solution
Given that ms = 120 t/h, p = 100 bar, t = 500°C, CV = 41 MJ/kg, hoverall = 85%, m = A/F = 17 kg/kg
of fuel, hw = 25 mm, tg = 240°C, tgs = 200°C, tatm = 30°C, h = 3375 kJ/kg, hf = 632.2 kJ/kg

patm 1.01325 × 105


(a) Density of air, ra = = = 1.165 kg/m3
RTatm 287 × ( 273 + 30)
Ta  m + 1 303  17 + 1 3
Density of flue gases, rg × ×  = 1.165 × ×  = 0.652 kg/m
Tg  m  ( 273 + 200)  17 
∆p = 103 × ghw × 10−3 = gH (ra − rg)
25 = H(1.165 − 0.652)
Height of stack, H = 48.73 m
(b) Velocity of gases passing through the chimney, v = 2 gH = 2 × 9.81 × 48.73 = 30.92 m/s
m s × h = m f × CV × h0

120 × 103 × 3375


m f = = 11.62 × 103 kg/h or 3.228 kg/s
41 × 106 × 0.85
Mass of air, m a = 17 × m f = 54.878 kg/s
Mass of flue gases, mg = m f + m a = 3.228 + 54.878 = 58.106 kg/s

M03_THERMAL ENGINEERING_SE_XXXX_CH03_Part2.indd 149 6/15/2018 3:06:36 PM


150 Chapter 3

p 2
m g = D × v × rg
4
p 2
or 58.106 = D × 30.92 × 0.652
4
or D = 1.91 m

Summary for Quick Revision


1. A boiler is a closed vessel in which steam is produced from water by combustion fuel at the desired temper-
ature and pressure. It consists of a fire place and a steam-raising vessel.
2. In a fire tube boiler, hot gases flow through the tubes and water is heated to raise steam outside the tubes.
Cochran, Lancashire, Cornish, Locomotive, and Scotch boilers are fire tube boilers.
3. In water tube boilers, water flows through the tubes and hot gases heat water from the outside to raise steam.
Babcock and Wilcox and Stirling boilers are water tune boilers.
4. High pressure boilers work up to 200 bar pressure and raise steam from 30−650 tons/h. Generally no drum
is used in these boilers.
5. Features of boilers are summarised as follows:
Boiler name Working Steam capac- Efficiency Constructional Uses
pressure bar ity kg/h % features
A. Fire tube boilers
1. Cochran 6.5−15 3500−4000 70−75 Vertical, horizontal Process industry
tubes, non-
portable, more tubes,
natural circulation
2. Lancashire 16 (max) 8000−9000 50−70 Horizontal, station- Sugar mills,
ary natural circula- textile industry,
tion, Has two tubes power generation,
chemical industry
3. Cornish 10.5 6500 50−70 Has only one Process industry
tube Horizontal,
stationary
4. Locomotive 14−25 9000 70 Horizontal, port- Locomotive
able tubes, forced engines
circulation
5. Scotch up to 17 1000 50−70 Vertical horizontal Marine
tubes, portable, force applications
circulation
B. Water tube boilers
1. Babcock and 42 (max) 40,000 60−80 Horizontal, station- Large
Wilcox ary natural circu- applications
lation of water and
forced circulation of
air and hot gases
2. Stirling up to 60 50,000 60−80 Three steam drums
inclined tube banks

M03_THERMAL ENGINEERING_SE_XXXX_CH03_Part2.indd 150 6/15/2018 3:06:37 PM


Steam Generators 151

6. Critical and supercritical boilers work at more than 221.2 bar pressure.
7. LaMont, Benson, Loeffler, Schmidt–Hartmann, and Velox are high pressure boilers.
8. Boiler mountings are different fittings and devices necessary for the operation and safety of a boiler. The
various boiler mountings are: Water level indicator, pressure gauge, steam stop valve, feed check valve, blow
down cock, fusible plug, and safety valve.
9. The functions of various boiler mountings are as follows:
(a)  Water-level indicator: To indicate the level of water in the boiler.
(b)  Pressure gauge: To indicate steam pressure of the boiler.
(c)  Steam stop valve: To regulate the flow of steam from the boiler to the prime mover as per requirement.
(d) Feed check valve: To allow the supply of water to the boiler at high pressure continuously and to prevent
the backflow of water from the boiler when the pump pressure is less than the boiler pressure.
(e) Blow–down cock: To remove sludge or sediments collected at the bottommost point in the water space
of boiler, when the boiler is working.
(f) Fusible plug: To put off the fire in the furnace of the boiler when the water level in the boiler falls below
an unsafe level.
(g)  Safety valve: To prevent the steam pressure in the boiler exceeds the desired rated pressure.
10. Boiler accessories are appliances installed to increase the overall efficiency of the steam power plant. The
various accessories are as follows:
(a) Pressure-reducing valve, steam trap, steam separator, economiser, air preheater, superheater, feed pump,
and injector.
11. The functions of various accessories are as follows:
(a)  Pressure-reducing valve: To maintain constant pressure on delivery side with fluctuating boiler pressure.
(b) Steam trap: To drain-off water resulting from the partial condensation of steam without steam to escape
through it.
(c) Steam separator: To separate suspended water particles carried by steam on its way from the boiler to
prime mover.
(d)  Economiser: To recover some heat carried away by flue gases and use for feed-water heating.
(e) Air preheater: To recover some heat of flue gases by preheating the air supplied to the combustion
chamber.
(f)  Superheater: To increase the temperature of steam above its saturation temperature.
(g)  Feed pump: To pump water to the water space of the boiler.
(h)  Injector: To feed water to the boiler with the help of a steam jet.
12. Evaporation rate is the quantity of water evaporated into steam per hour.
13. Equivalent evaporation is the equivalent of 1 kg of water at 100°C to steam at 100°C. It requires approxi-
mately 2257 kJ of heat.
14. Factor of evaporation is the ratio of actual heat absorption above feed-water temperature for transformation
to steam to the latent heat of steam at atmospheric pressure.
Equivalent evaporation = actual evaporation × factor of evaporation
 h1 − h f 
me = ms ×   × ms × F
 ( h fg )atm 
15. Boiler performance parameters:
Heat output
(a)  Efficiency =
Heat input
Rate of burning of fuel per hour
(b)  Combustion rate =
Grate area
Furnance volume
(c)  Combustion space =
Fuel burnt per hour

M03_THERMAL ENGINEERING_SE_XXXX_CH03_Part2.indd 151 6/15/2018 3:06:38 PM


152 Chapter 3


Heat absorption is the equivalent evaporation from and at 100°C in kg of steam generated per m2 of
heating surface
Heat liberated per hour
(d)  Heat liberated =
Furnace volume
ms ( h1 − h f )

16. Boiler efficiency hb =
m f × C .V .
where m s = mass of steam generated in kg/h
m f = mass of fuel burned in kg/h
C.V. = calorific value of fuel, kJ/kg
Equivalent evaporation from and at 100°C per hour
17. Boiler power = kW
21.296
18. Heat balance sheet for boiler:
(a)  Heat supplied by fuel, Q = mf × C.V.
(b)  Heat used to generate steam, Q1 = ms × (h − hf1)
(c)  Heat carried away by flue gases, Q2 = mgcpg (Tg − Ta)
(d)  Heat carried away by steam in flue gases, Q3 = mst × (h1 − hf1)
CO × C
(e)  Heat lost due to incomplete combustion, Q4 = × C.V.
CO + CO 2
(f)  Heat lost due to unburnt fuel, Q5 = mf1 × C.V.
(g)  Unaccounted for heat, Q6 = Q − (Q1 + Q2 + Q3 + Q4 + Q5)
19. Draught is the small pressure difference maintained in the boiler to exhaust the products of combustion form
the combustion chamber.
20. The draught may be natural draught produced by a chimney or artificial draught produced by a steam jet or
mechanically.
21. Steam jet draught may be induced or forced type.
22. Mechanical draught may be induced, forced and balanced type.
23. In the induced draught, the blower is located near the base of the chimney, whereas, in the forced draught,
the blower is installed near the base of the boiler.
24. Natural draught:
Static draught, ∆p = hw = (ga − gg ) H
1 1  m + 1 
∆p = hw = 353  −  a  
 Ta Tg  ma  
25. Natural draught in terms of height of burnt gases:
 ma  Tg 
H1 = H   − 1
 ma + 1 Ta 
26. Velocity of hot gases passing through the chimney,
u = k H1
h1
where k = 4.43 1 −
H1
= 0.825 for brick chimney.
= 1.1 for steel chimney.
mg
27. Chimney diameter, D = 1.128
v rg

M03_THERMAL ENGINEERING_SE_XXXX_CH03_Part2.indd 152 6/15/2018 3:06:42 PM


Steam Generators 153

28. Condition for maximum discharge through chimney.


Tg 2( ma + 1)
=
Ta ma
H
( hw ) max = 176.5
Ta
( H1 ) max = H

Ap  ma  H
Maximum discharge, ( mg ) max = 2.22
R  ma + 1 Ta
  ma  Tg 
  − 1
 m + 1 Ta
29. Chimney efficiency, hch = H  a 
 c (T − T ) 
 pg n m

 
where Tn = Tg and Tm = absolute temperature of flue leaving the chimney in case of artificial draught.
hch is generally less than 1%.

Multiple-choice Questions
1. Consider the following statements: 5. In fluidised bed combustion velocity of fluid is
Blowdown is necessary on boilers, because proportional to (r = radius of the particle) as
1. The boiler water level is lowered rapidly in 1 1
case it accidentally rises too high. (a) (b)
r r
2. The precipitated sediment of sludge is removed
while the boiler is in service. (c) r (d) r
3. The concentration of suspended solids in the 6. Which one of the following is the steady flow
boiler is controlled. energy equation for a boiler?
Of these statements:
(a) 1, 2, and 3 are correct v12 v2
(a) h1 + = h2 + 2
(b) 1 and 2 are correct 2g 2g
(c) 3 alone is correct (b) Q = (h2 − h1)
(d) 1 and 3 are correct
2. Once through boiler is named so because v12 v2
(c) h1 + + Q = h2 + 2
(a) Flue gas passes only in one direction 2g 2g
(b) There is no recirculation of water (d) Ws = (h2 − h1) + Q
(c) Air is sent through the same direction
(d) Steam is sent out only in one direction 7. The output of a boiler is normally stated as
(a) Evaporative capacity in tonnes of steam that
3. Economiser is generally placed between
can be produced from and at 100°C
(a) Last superheater/reheater and air-preheater
(b) Weight of steam actually produced at rated
(b) Air-preheater and chimney
pressure in tonnes per hour
(c) Electrostatic precipitators
(c) Boiler horse power
(d) induced draft fan and forced draft fan
(d) Weight of steam produced per kg of fuel
4. Attempering is
(a) Reduction of steam pressure 8. Which one of the following is the fire-tube boiler?
(b) Reduction of steam temperature (a) Babcock and Wilcox boiler
(c) Discharge of flue gases at certain height (b) Locomotive boiler
(d) Conditioning of disposable fluids for mini- (c) Stirling boiler
mum pollution (d) Benson boiler

M03_THERMAL ENGINEERING_SE_XXXX_CH03_Part2.indd 153 6/15/2018 3:06:47 PM


154 Chapter 3

9. Which one of the following factors has the max- Codes:



imum effect on the equivalent evaporation of a A B C D
boiler? (a) 2 4 3 5
(a) Steam generator pressure (b) 1 3 2 5
(b) Feed-water inlet temperature (c) 3 2 1 4
(c) Heating surface of the boiler (d) 2 3 1 5
(d) Quality of steam produced
14. In forced circulation boilers, about 90% of water
10. Consider the following: is recirculated without evaporation. The circula-
1.  Safety valve tion ratio is
2.  Steam trap (a) 0.1 (b) 0.9
3.  Steam separator (c) 9 (d) 10
4. Economiser
Among these, the boiler accessories would include 15. Given that h is draught in mm of water, H is chim-
(a) 1, 2, and 3 (b) 2, 3, and 4 ney height in meters, and T1 is atmospheric tem-
(c) 1 and 4 (d) 1, 2, 3, and 4 perature in K, the maximum discharge of gases
through a chimney is given by
11. Match List I and List II and select the correct (a) h = 176 T1/H (b) h = H/176.5 T1
answer using the codes given below the lists: (c) h = 1.765 H/T1 (d) h = 176.5 H/T1
List I List II 16. The excess air required for combustion of pulver-
A. Babcock and Wilcox 1. Forced circulation ised coal is of the order of
B. Lancashire 2. Fire tube (a) 100 to 150% (b) 30 to 60%
C. LaMont 3. Water tube (c) 15 to 40% (d) 5 to 10%
D. Cochran 4. Vertical 17. Consider the following:
1. Increasing evaporation rate using convection
Codes:
heat transfer from hot gases.
A B C D 2.  Increasing evaporation rate using radiation.
(a) 1 2 3 4 3.  Protecting the refractory walls of the furnace.
(b) 2 3 4 1 4.  Increasing water circulation rate.
(c) 3 2 1 4 The main reasons for providing water wall enclo-
(d) 2 4 1 3 sures in high pressure boiler furnaces would
12. Consider the following statements: include:
1.  Boiler mounting are mainly protective devices. (a) 2 and 3 (b) 1 and 3
2.  Steam stop valve is an accessory. (c) 1 and 2 (d) 1, 2, 3, and 4
3.  Feed-water pump is an accessory. 18. Consider the following statements:
Of these statements: Expansion joints in steam pipelines are installed to
(a) 1, 2, and 3 are correct 1.  Allow for future expansion of plant.
(b) 1 and 2 are correct 2.  Take stresses away from flanges and fittings.
(c) 2 and 3 are correct 3. Permit expansion of pipes due to temperature
(d) 1 and 3 are correct rise.
13. Match List I with List II select the correct answer Of these statements:
using the codes given below the list: (a) 1, 2, and 3 are correct
(b) 1 and 2 are correct
List I List II
(c) 2 and 3 are correct
A. Soot Blower 1. Removal of solids from (d)  1 and 3 are correct
boiler drums
19. In high pressure natural circulation boilers, the
B. Electrostatic 2. To clean the tube surfaces flue gases flow through the following boiler
precipitator of fly ash accessories:
C. Blow down 3. Cleaning of fluid gas 1.  Superheater 2.  Air heater
3. Economiser 4. ID fan
D. Zeolite 4. Air cleaning
 The correct sequence of the flow of flue gases
5. Water purification through these boiler accessories is:

M03_THERMAL ENGINEERING_SE_XXXX_CH03_Part2.indd 154 6/15/2018 3:06:47 PM


Steam Generators 155

(a)  1, 3, 4, 2 (b)  3, 1, 4, 2 Codes:


(c)  3, 1, 2, 4 (d)  1, 3, 2, 4 (a) 2 (b) 1 and 3
20. Consider the following components: (c) 2, 3, and 4 (d) 1, 2, 3, and 4
1.  Radiation evaporator 25. Which of the following power plants use heat
2. Economiser recovery boilers (unfired) for steam generation?
3.  Radiation superheater 1.  Combined cycle power plants
4.  Convection superheater 2.  All thermal power plants using coal
In the case of Benson boiler, the correct sequence 3.  Nuclear power plants
of the entry of water through these components is: 4.  Power plants using fluidised bed combusting
(a) 1, 2, 3, 4 (b) 1, 2, 4, 3 Select the correct answer using the codes given
(c) 2, 1, 3, 4 (d) 2, 1, 4, 3 below:
(a) 1 and 2 (b) 3 and 4
21. Coal fired power plant boilers manufactured in
(c) 1 and 3 (d) 2 and 4
India generally use
(a) Pulverised fuel combustion 26. Benson boiler is one of the high pressure boilers
(b) Fluidised bed combustion having
(c) Circulating fluidised bed combustion (a) One drum
(b) One water drum and one steam drum
22. Match List–I (name of the boiler) with List–II (c) Three drums
(special features) and select the correct answer (d) No drum
using the code given below the lists:
27. Forced draught fans of a large steam generator
List I List II have
A. Lancashire 1. High pressure water tube (a) Backward curved blades
(b) Forward curved blades
B. Cornish 2. Horizontal double fire tube (c) Straight or radial blades
C. La Mont 3. Vertical multiple fire tube (d) Double curved blades
D. Cochran 4. Low pressure inclined water 28. The device used to heat feed-water by utilising
tube the heat of the exhaust flue gases before leaving
5. Horizontal single fire tube. through the chimney is called
(a) Superheater (b) Economiser
Codes: (c) Air preheater (d) ID fan
A B C D 29. Consider the following statements:
(a) 2 5 1 3    I. Forced circulation is always used in high
(b) 2 4 3 1 pressure power boilers.
(c) 1 5 2 3   II. Soot blowers are used for cleaning tube sur-
(d) 5 4 1 3 faces at regular intervals.
23. Which of the following safety devices is used to III. Electrostatic precipitator is used to remove
protect the boiler when the water level falls below fly ash from flue gases.
a minimum level? Which of these statements are correct?
(a) Water-level indicator (a) I, II, and III (b) II and III
(b) Fusible plug (c) I and III (d) I and II
(c) Blow-off cock 30. Once-through boilers operate at
(d) Safety valve (a) Subcritical pressure
(b) Supercritical pressure
24. Which of the following form part (s) of the boiler
(c) Subcritical as well as supercritical pressures
mountings?
(d) Critical pressure only
1.  Economiser 2.  Feed check valve
3. Steam trap 4. Superheater 31. Match List I (components) with List II (func-
Select the correct answer using the codes given tions) and select the correct answer using the
below: codes given below the lists:

M03_THERMAL ENGINEERING_SE_XXXX_CH03_Part2.indd 155 6/15/2018 3:06:47 PM


156 Chapter 3

List I List II List I List II


A.  Steam trap 1.  Controls steam flow rate (Boilers) (Type/Description)
B. Fusible 2. Controls rate of water flow C. Babcock and 3.  Bent tube, water-tube boiler
plug to boiler Wilcox
C. Blow-off 3. Puts off furnace fire when D. Stirling 4.  High pressure boiler
cock water level reaches unsafe limit Codes:
D. Feed check 4. Removes mud and dirt col- A B C D
valve lected at the bottom of boiler (a) 4 2 1 3
5. Drains off water collected by (b) 1 4 2 3
partial condensation of steam (c) 4 2 3 1
in pipes (d) 1 4 3 2
Codes: 35. A device which is used to drain off water from
steam pipes without escape of steam is called
A B C D
(a) Steam separator
(a) 5 1 4 2 (b) Steam trap
(b) 1 3 5 4 (c) Pressure reducing valve
(c) 5 3 4 2 (d) Injector
(d) 1 2 5 4
36. Blowing down of boiler water is the process to
32. Which of the following sequences indicates the (a) Reduce the boiler pressure
correct order for flue gas flow in the steam power (b) Increase the steam temperature
plant layout? (c) Control the solids concentration in the boiler
(a) Superheater, economiser, and air preheater water
(b) Economiser, air preheater, and superheater (d) Control the drum level
(c) Air preheater, economiser, and superheater 37. Consider the following statements regarding flu-
(d) Economiser, superheater, and air preheater idised bed combustion boilers:
33. Which one of the following statements is not 1. The combustion temperatures are low, around
correct? 900°C.
In a fluidised bed boiler, 2.  The formation of oxides of nitrogen is low.
(a) The combustion temperatures are higher than 3. It removes sulphur from coal during combus-
those in the conventional boilers tion process.
(b) Inferior grade of coal can be used without 4.  It requires high quality of coal as fuel.
slagging problems Which of these statements are correct?
(c) The formation of NOx is less than that in the (a) 1, 2, 3 and 4 (b) 1, 2 and 3
(c) 2, 3 and 4 (d) 1 and 4
conventional boilers
(d) The volumetric heat release rates are higher 38. The correct gas flow path in a typical large mod-
than those in the conventional boilers ern natural circulation boiler is
(a) Combustion chamber – reheater – superheater
34. Match List I (boilers) with List II (type/descrip- – economiser – air preheater – ID fan – elec-
tion) and select the correct answer using the codes trostatic precipitator – stack
given below the lists: (b) Combustion chamber – superheater – reheater
– economiser – air preheater – electrostatic
List I List II precipitator – ID fan – stack
(Boilers) (Type/Description) (c) Combustion chamber – reheater – superheater
A. Lancashire 1. Horizontal straight tube, – air preheater – economiser – electrostatic
fire-tube boiler precipitator – ID fan – stack
B. Benson 2. Horizontal straight tube, (d) Combustion chamber – superheater – reheater
– economiser – air preheater – ID fan – elec-
water-tube boiler
trostatic precipitator – stack

M03_THERMAL ENGINEERING_SE_XXXX_CH03_Part2.indd 156 6/15/2018 3:06:47 PM


Steam Generators 157

39. The draught in locomotive boilers is produced by Which of these improve overall steam power plant
(a) Chimney (b) Centrifugal fan efficiency?
(c) Steam jet (d) Locomotion (a) Only 1, 2, and 3 (b) Only 2 and 3
40. Consider the following statements regarding (c) Only 1 and 4 (d) 1, 2, 3, and 4
waste-heat boilers: 44. Consider the following:
1. Waste-heat boilers are placed in the path of 1. Injector 2. Economiser
exhaust gases. 3. Blow-off clock 4. Steam stop valve
2. These are fire tube boilers.
3. The greater portion of the heat transfer in such Which of the above is/are not boiler mountings?
boilers is due to convection. (a) Only 1 (b) Only 1 and 2
Which of the statements given are correct? (c) 1, 2, and 3 (d) 2 and 4
(a) 1, 2, and 3 (b) 1 and 2 45. The draught produced by the chimney is called
(c) 2 and 3 (d) 1 and 3 (a) Natural (b) Induced
41. Which of the following statements is not true for (c) Forced (d) Balanced
a supercritical steam generator?
46. The draught produced by a steel chimney as com-
(a) It has a very small drum compared to a con-
ventional boiler pared to that produced by a brick chimney for the
(b) A supercritical pressure plant has higher effi- same height is
ciency than a subcritical pressure plant (a) More (b) Less
(c) The feed pressure required is very high, (c) Same (d) Unpredictable
almost about 1.2 to 1.4 times the boiler pres- 47. Artificial draught is produced by
sure (a) Induced fan (b) Forced fan
(d) As it requires absolutely pure feed water, (c) Steam jet (d) Both (a) and (b)
preparation of feed water is more important
than in subcritical pressure boiler 48. The draught in locomotive is produced by
(a) Forced fan (b) Induced fan
42. Which one of the following is the correct path of (c) Steam jet (d) Chimney
water flow through various components of boiler
of a modern thermal power plant? 49. The artificial draught produces
(a) Economiser – boiler drum – water walls – (a) Less smoke
boiler drum – superheater – turbine (b) More draught
(b) Economiser – boiler drum – water walls – (c) Less chimney gas temperature
superheater – turbine (d) All of these
(c) Economiser – water walls – boiler drum –
50. The chimney efficiency is approximately
superheater – turbine
(a) 10% (b) 5%
(d) Economiser – water walls – superheater –
(c) 2% (d) 0.25%
turbine
43. Consider the following: 51. The pressure at the furnace is minimum is case of
1. Superheater 2. Economiser (a) Forced draught (b) Induced draught
3. Air pre-heater 4. Condenser (c) Balanced draught (d) Natural draught

Explanatory Notes
recirculation 90
14. (c) Circulation ratio = =9
circulation 10

M03_THERMAL ENGINEERING_SE_XXXX_CH03_Part2.indd 157 6/15/2018 3:06:48 PM


158 Chapter 3

Review Questions
1. What is a steam generator?
2. How do you classify steam generators?
3. What are the merits of water tube boilers?
4. What are the demerits of fire tube boilers?
5. List the requirements of a good boiler.
6. Enumerate the factors affecting boiler selection.
7. What are the advantages of high pressure boilers?
8. List the advantages of forced circulation boilers.
9. What is a boiler mounting? List the various boiler mountings.
10. What are the functions of boiler accessories?
11. Name the various accessories used on a boiler.
12. Define equivalent evaporation and factor of evaporation.
13. Define boiler efficiency.
14. What are the purposes of carrying out boiler trial?
15. Differentiate between natural and artificial draught.
16. Describe the consumption features of a Lancashire boiler. How can superheated steam be produced with this
boiler?
17. Describe the construction features of a Cochran boiler. Discuss its working briefly.
18. When is a water tube boiler exclusively used? Discuss the construction and working of Babcock and Wilcox
boiler.
19. With the help of a neat sketch, describe a Loeffler boiler. What is the working pressure of such a boiler?
20. Draw a schematic flow diagram of a modern steam generator. Discuss the installation of connection and
radiation superheaters and their exit temperature response with varying steam flow.
21. With the help of a neat sketch, discuss the working principle of Velox high pressure boiler.

Exercises
3.1 In a boiler trial, 1360 kg of coal was consumed in 24 hours. The mass of water evaporated was 13600 kg
and steam pressure was 7.5 bar. The feed-water temperature was 35°C. The calorific value of 1 kg of coal is
30000 kJ/kg. Calculate the boiler efficiency and equivalent evaporation. [Ans. 87.31%, 11.6 kg]
3.2 A boiler generates 8.5 kg of steam per kg of coal burned at a pressure of 13.5 bar from feed water having
absolute temperature of 350 K. The boiler efficiency is 70% and factor of evaporation 1.17. Taking specific
heat of steam at constant pressure to be 2.1 kJ/kgK, calculate (a) the degree of superheat and temperature
of steam generated, (b) the calorific value of coal, and (c) the equivalent evaporation.
[Ans. 84°C, 550.4°C, 32070 kJ/kg, 9.94 kg]
3.3 The data obtained during a boiler trial is as follows:
Duration = 1 hour
Steam generated = 35,500 kg
Steam pressure = 12 bar
Steam temperature = 250°C
Temperature of water entering economiser = 17°C
Temperature of water leaving economiser = 77°C
Oil burnt = 3460 kg
CV of oil = 39,500 kJ/kg
Calculate (a) equivalent evaporation, (b) thermal efficiency of plant, and (c) percentage of heat energy of
fuel energy utilised by the economiser. [Ans. 13.01 kg, 74.4%, 6.52%]

M03_THERMAL ENGINEERING_SE_XXXX_CH03_Part2.indd 158 6/15/2018 3:06:48 PM


Steam Generators 159

  3.4  The following data relate to a boiler trial:


Coal burnt per hour = 6750 kg
Moisture content in coal = 2%
HCV of coal = 35,490 kJ/kg
Ultimate mass analysis of dry coal: C = 84%, O2 = 4%, ash = 8%
Dry flue gas analysis by volume: CO2 = 9.5%, O2 = 10.48%, N2 = 80.02%
Temperature of flue gases = 305°C
Room temperature = 32°C
Specific heat of flue gases = 1 kJ/kgK
Specific heat of steam in flue gases = 2 kJ/kgK
Mass of steam generated at 16.5 bar from feed water at 47°C = 60,500 kg/h
Draw up the heat balance sheet per kg of dry coal. Assume that air contains 23.1% by mass of oxygen.
Ans.

Heat kJ % Heat utilised kJ %


Heat of dry coal 35,490 98.40 Steam formation 24,365 67.61
Heat in feed water 573.6 1.60 Carried away by dry flue gases 3136 8.70
Carried by excess air 2877 7.98
Carried by moisture 1123 3.12
Unaccounted for 4535.6 12.59
Total 36,063.6 100 36,063.6 100
  3.5 Calculate the height of chimney required to produce a draught equivalent to 1.7 m of water if the fuel gas
temperature is 270°C and ambient temperature is 22°C and minimum amount of air per kg of fuel is 17 kg.
[Ans. 33.45 m]
  3.6  A boiler is equipped with a chimney of 30 m height. The flue gases passing through the chimney are at
temperature of 228°C, whereas the atmospheric temperature is 21°C. If the air flow through the combustion
chamber is 18 kg/kg of fuel burnt, find the theoretical draught produced in mm of water and velocity of flue
gases if 50% of theoretical draught is lost. [Ans. 1.37 cm of water, 13.45 m/s]
  3.7 Calculate the mass of flue gases flowing through the chimney when the draught produced is equal to 1.9 cm
of water. Temperature of flue gases is 290°C and ambient temperature is 20°C. The flue gases formed per kg
of fuel burnt are 23 kg. Neglect the losses and take the diameter of the chimney as 1.8 m.  [Ans. 38 kg/s]
  3.8  A chimney is 42 m high and the temperature of hot gases in the chimney is 310°C. The temperature of
outside air is 35°C. The furnace is supplied with 20 kg of air per kg of fuel burnt. Calculate the draught in
cm of water and velocity of flue gases.
  3.9 In a chimney of height 45 m, temperature of flue gases with natural draught is 365°C. The temperature of
waste gases by using artificial draught is 130°C. The temperature of outside air is 35°C. If air supplied is
20 kg per kg of fuel burnt, determine the efficiency of chimney. Take cpg = 1.004 kJ/kgK for flue gases.
3 .10  Calculate the thermal efficiency and equivalent evaporation of a boiler for the following data:
  Steam pressure = 10.5 bar
  Steam temperature = 250°C
  Feed-water temperature = 35°C
  Water evaporated = 10 kg/kg of coal burnt
  Calorific value of coal = 33,400 kJ/kg
3 .11  The following data refers to a steam power plant.
  Height of Chimney = 35 m
Draught = 18 mm of water gauge

M03_THERMAL ENGINEERING_SE_XXXX_CH03_Part2.indd 159 6/15/2018 3:06:48 PM


160 Chapter 3

Temperature of flue gases = 370°C


Boiler house temperature = 32°C
Determine the quantity of air used per kg of fuel burnt in the boiler.
3.12  In a boiler trial, the following observations were made:
Steam pressure = 10 bar, steam generated = 550 kg/h
Fuel used = 66 kg/h, Moisture in fuel = 1.5% by weight
Weight of dry flue gases = 8.5 kg/kg of fuel, LCV of fuel = 34 MJ/kg
Temperature of flue gases = 320°C, Boiler house temperature = 27°C
Feed-water temperature = 50°C, Specific heat of flue gases = 1.1 kJ/kgK
Dryness fraction of steam = 0.95
Draw up the heat balance sheet for the boiler.
3.13  The following data was obtained during a boiler trial:
Feed-water temperature = 45°C, feed water supplied = 4200 kg/h
Steam pressure = 10 bar, dryness fraction of steam = 0.98
Coal fired = 450 kg/h, CV of dry coal = 37,500 kJ/kg
Moisture in coal = 4%, temperature of flue gases = 280°C
Boiler house temperature = 15°C, barometer pressure = 1 bar,
Ultimate analysis of coal:
C = 86 %, H2 = 4 %, Ash = 5 %, incombustible matter = 5%
Volumetric analysis of dry flue gases:
CO2 = 10.4 %, CO = 1.2 %, O2 = 9 %, N2 = 79.4 %
Draw the heat balance sheet on 1 kg of coal used basis.

Answers to Multiple-choice Questions


1. (b) 2. (b) 3. (a) 4. (b) 5. (c)
6. (b) 7. (b) 8. (b) 9. (a) 10. (a)
11. (c) 12. (c) 13. (d) 14. (c) 15. (c)
16. (d) 17. (c) 18. (c) 19. (d) 20. (c)
21. (a) 22. (a) 23. (b) 24. (a) 25. (c)
26. (d) 27. (a) 28. (a) 29. (a) 30. (d)
31. (c) 32. (a) 33. (d) 34. (d) 35. (b)
36. (c) 37. (b) 38. (b) 39. (c) 40. (b)
41. (c) 42. (c) 43. (a) 44. (b) 45. (a)
46. (a) 47. (d) 48. (c)
49. (d) 50. (d) 51. (c)

M03_THERMAL ENGINEERING_SE_XXXX_CH03_Part2.indd 160 6/15/2018 3:06:48 PM


Chapter 4

Steam Power Cycles

4.1 ❐ INTRODUCTION
In steam power plants and refrigeration cycles, the working fluid changes from liquid to vapour and
back to liquid state. This succession of processes is called vapour cycle. In steam power plants, water
is the working fluid in the form of steam and vapour. In refrigeration cycles gasses such as Freon, CO2,
and ammonia (aqua-ammonia) are used as working substances.

4.2 ❐ CARNOT VAPOUR CYCLE


The elements of steam power plants are as follows:
1. Boiler
2. Steam turbine
3. Condenser
4. Feed pump
The schematic diagram of a steam power plant is shown in Fig. 4.1. The p-v and T-s diagrams for the
Carnot vapour power cycle are shown in Fig. 4.2(a) and (b). Dry and saturated steam at temperature
TH = T1 enters the steam turbine and is expanded isentropically to sink temperature TL = T2 to do work.
The exhaust steam from the turbine is condensed and cooled in the condenser to state 3 at which the
working fluid is in a two-phase mixture of water and its vapour. The water and the vapour are then
pumped by the boiler feed pump to state 4 in a saturated state to the boiler, where it is converted into
steam.
Consider 1 kg of the working fluid for analysis.
Work done by steam turbine wt = w1-2 = h1 - h2
Work done on the pump, wp = w3-4 = hf4 - h3
Net work, wnet = w1 - wp = (h1 - h2) - (hf4 - h3)
= (h1 - hf4) - (h2 - h3) = qs - qr
Heat supplied, qs = h1 - hf4
and heat rejected, qr = h2 - h3

M04_THERMAL ENGINEERING_SE_XXXX_CH04.indd 161 6/15/2018 2:33:53 PM


162 Chapter 4

TH = T1

1 kg
Steam 1 wt

Boiler Turbine

qs
CEP: Condensate extraction pump
BFP: Boiler feed pump 2
Hot water

Condenser qr

Cold water

4 Saturated 3
water
T L = T2
wp BFP CEP
Water and vapour

Figure 4.1  Schematic diagram of steam power plant

p T
Isothermal
qs
4 TH 1
4 1 1′
1-2, 1′-2′: Isentropic
1′
qs expansion wt
3-4: Isentropic
wt
wp compression wp
TL qr
3 2 2′ 2′
qr 3 2
Isothermal v s
(a) (b)

Figure 4.2 p-v and T-s diagrams for Carnot vapour cycle: (a) p-v diagram,
(b) T-s diagram

Work done w
Thermal efficiency, hth = = t
Heat supplied qs

( h1 − h f 4 ) − ( h2 − h3 ) qs − qr
= = (4.1)
h1 − h f 4 qs
From the second law of thermodynamics, we have
q q
s1 - s4 = 4 −1 = s
TH TH

M04_THERMAL ENGINEERING_SE_XXXX_CH04.indd 162 6/15/2018 2:33:56 PM


Steam Power Cycles 163

or qs = TH (s1-s4)
q q
s2 - s3 = 2 −3 = r
Similarly,   
TL TL
or qr = TL (s2 - s3)
Since, s1- s4 = s2 - s3, one can write
w q − qr TH − TL
hth = t = s =
qs qs TH
TL T
=1- = 1- 2  (4.2)
TH T1

1. Work Ratio (rw): It is defined as the ratio of net work done to turbine work done in the cycle.
wnet wt − w p wp
Work ratio, rw = = = 1-
wt wt wt
h f 4 − h3
=1-  (4.3)
h1 − h2
2. Specific Steam Consumption (SSC): It is defined as the flow rate of steam per unit of power
developed. It is expressed in kg/kWh.
.
Steam flow rate ms
SSC = =
Power developed wnet

1  kg/s  3600 kg
=  =  (4.4)
wnet  kJ  wnet kWh

3. Heat Rate: It is defined as the rate of heat supplied to produce unit work output.
3600 qs 3600
Heat rate = = kJ/kWh(4.5)
wt − w p h th

4.2.1 Drawbacks of Carnot Cycle


The major drawbacks of Carnot cycle are as follows:
1. It is very difficult to build a pump which can pump isentropically a two-phase mixture of water
and its vapour at state point 3 and to deliver saturated water at state point 4.
2. It is very difficult to superheat steam at constant temperature along path 1–1′.
3. The steam leaving the turbine is of very low dryness fraction. It causes erosion and pitting on
turbine blades.
4. The volume of water vapours to be handled by the pump is quite large. It requires a very large-
sized pump, thus resulting in high power consumption.
5. It requires high specific steam consumption, giving low thermal efficiency. Hence, it is not
economically viable.

M04_THERMAL ENGINEERING_SE_XXXX_CH04.indd 163 6/15/2018 2:34:01 PM


164 Chapter 4

Example 4.1
A steam power plant operates on an ideal Carnot cycle. Dry saturated steam at 20 bar pressure is
supplied to a turbine. It expands isentropically to a condenser pressure of 0.075 bar. Assuming that
saturated water enters the boiler, calculate (a) the thermal efficiency, (b) the work ratio, and (c) the
specific steam consumption.

Solution
For dry saturated steam at 20 bar, from the steam tables, we have
h1 = hg = 2799.5 kJ/kg, s1 = sg = 6.3408 kJ/kg.K
At 0.075 bar condenser pressure, from the steam tables, we have
hf  2 = 168.77 kJ/kg, hfg2 = 2406.0 kJ/kg,
sf  2 = 0.5763 kJ/kg.K, sfg2 = 7.6751 kJ/kg.K
Now s1 = s2 = sf  2 + x2 sfg2 for the isentropic expansion process 1-2. (Fig.4.2)
6.3408 = 0.5763 + x2 × 7.6751
or x2 = 0.751
∴ h2 = hf  2 + x2hfg2 = 168.77 + 0.751 × 2406.0 = 1975.676 kJ/kg
At 20 bar, we have
hf4 = 908.77 kJ/kg, s4 = sf4 = 2.4473 kJ/kg.K
For the isentropic compression process 3-4, we have
s3 = s4
or sf3 + x3 sfg3 = s4
Now sf 3 = sfg3 and sfg3 = sfg2
 0.5763 + x3 × 7.6751 = 2.4473
or x3 = 0.244
hf 3 = hf 2 and hfg3 = hfg2
h3 = hf 3 + x3 hfg3 = 168.77 + 0.244 × 2406.0 = 755.834 kJ/kg
Pump work, wp = hf4 - h3 = 908.77 - 755.834 = 152.936 kJ/kg
Turbine work, wt = h1 - h2 = 2799.5 - 1975.676 = 823.824 kJ/kg
Net work, wnet = wt - wp = 823.824 - 152.936 = 670.888 kJ/kg
Heat supplied, qs = h1 - hf4 = 2799.5 - 908.77 = 1890.73 kJ/kg
wnet 670.888
(a)  Thermal efficiency hth = = = 0.3548 or 35.48%
qs 1890.73
wnet 670.888
(b)  Work ratio, rw = = = 0.814
wt 823.824
3600 3600
(c)  Specific steam consumption, SSC = = = 5.366 kg/kWh
wnet 670.888

M04_THERMAL ENGINEERING_SE_XXXX_CH04.indd 164 6/15/2018 2:34:03 PM


Steam Power Cycles 165

4.3 ❐ RANKINE CYCLE


The schematic of Rankine cycle is shown in Fig. 4.3(a). It consists of a turbine, boiler, condenser,
and pump. The p-v diagram is shown in Fig. 4.3(b) and the T-s diagram in Fig. 4.3(c). The deliv-
ery of steam from the boiler takes place at state 1 when assumed dry saturated or 1′ when assumed
superheated. The steam expands isentropically in the prime mover to state point 2 from point 1 and 2′
from 1′.
After doing work at point 2 or 2′, the steam is condensed in the condenser to saturated water,
represented by point 3 at pressure p2. This water is compressed isentropically to pressure p2 by the
pump represented by the process 3-M. Thus, the boiler receives water at pressure p1 in a sub-cooled
state, heats it to temperature corresponding to point 4, and further transforms it into steam at constant
pressure p1. M-4-1 is the operation carried out in the boiler for dry saturated steam and M-4-1′ for
superheated steam. All processes except the process of mixing of cold water at point M with hot water
at point 4 are reversible . However, when we consider the flow of constant mass, this process gets
deleted. The cycle begins again at 1 or 1′.

1
Turbine Wt p
Saturated liquid line
2

Qin M 1 1′
Qout p1
4 pvg = C
Cooling water
Boiler
4 Condenser
p2
3 3 2 2′
M
Wp
A B v
(a) (b)

T
1′
4 p1 = C 1
M

p2 = C
3 2 2′

s
N R R′
(c)

Figure 4.3 Rankine cycle: (a) Schematic diagram,


(b) p-v diagram, (c) T-s diagram

M04_THERMAL ENGINEERING_SE_XXXX_CH04.indd 165 6/15/2018 2:34:03 PM


166 Chapter 4

4.3.1 Analysis of Rankine Cycle


Consider one kg of mass flow.
Let h1 = enthalpy of steam at point 1
h2 = enthalpy of steam at point 2
hf3 = enthalpy of water at point 3
hf4 = enthalpy of water at point 4
hfM = enthalpy of water at point M
Heat added at constant pressure p1 represented by M-4-1,
qa = h1 - hf M = (h1 - hf   3) - (hf   M - hf   3)
= area N-M-4-1-R-N on T-s diagram
Heat rejected at constant pressure p2 represented by 2-3,
qr = h2 - hf   3
= area R-2-3-N-R on T-s diagram
Net work done, wnet = qa - qr
= (h1 -hf M) - (h2 - hf 3)
= area 1-2-3-M-4-1
Pump work, wp = hf M - hf   3
wnet = (h1 - h2) -wp
qa = (h1 - hf   3 ) - wp
v f 3 ( p1 − p3 ) × 105
Now wp = = vf  3 (p1 - p3) × 102 kJ/kg, where p is in bar.
103
= vf 3 (p1 - p2) × 102 kJ/kg
q − qr w
Thermal efficiency, hth= a = net
qa qa
h1 − h2 − w p
= (4.6)
h1 − h f 3 − w p
Neglecting pump work, which is small as compared to other quantities.
h1 − h2
hth = (4.7)
h1 − h f 3
1 × 3600
Specific steam consumption, SSC = kg/kWh (4.8)
wnet
Overall heat rate (OHR) = Specific steam consumption × heat supplied per kg of throttle steam

1 × 3600  h1 − h f M 
    = kJ/kWh (4.9)
( h1 − h2 ) − ( h f M − h f 3 )
Pump work h − hf 3
Back work ratio, BWR = = fM (4.10)
Turbine work h1 − h2

M04_THERMAL ENGINEERING_SE_XXXX_CH04.indd 166 6/15/2018 2:34:06 PM


Steam Power Cycles 167

Example 4.2
In an ideal Rankine cycle, saturated steam vapours enter the turbine at 160 bar and saturated liq-
uid exits the condenser at 0.05 bar. The net power output of the cycle is 130 MW. Determine
for the cycle (a) the thermal efficiency, (b) the steam mass flow rate, and (c) the specific steam
consumption.
Solution
The Rankine cycle is shown in Fig. 4.4.
From steam tables,
For p1 = 160 bar, h1 = hg = 2580.6 kJ/kg, s1 = sg1 = 5.2454 kJ/kg.K
p2 = 0.05 bar, vf   3 = 0.001005 m3/kg, vg3 = 28.193 m3/kg, hf   3 = 137.79 kJ/kg,
  hfg3 = 2423.7 kJ/kg, sf   3 = 0.4763 kJ/kg.K, sfg3 = 7.9187 kJ/kg.K
For insentropic process 1-2,
s1 = s2 = sf   3 + x2sfg3
or 5.2454 = 0.4763 + x2 × 7.9187
or x2 = 0.602
h2 = hf   3 + x2hfg3 = 137.79 + 0.602 × 2423.7 = 1596.86 kJ/kg
h3 = hf   3 = 137.79 kJ/kg
hM = h3 + vf   3 (p1 - p2) × 102
= 137.79 + 0.001005 (160 - 0.05) × 102 = 153.86 kJ/kg
Turbine work, wt = h1 - h2 = 2580.6 - 1596.86 = 983.74 kJ/kg
Pump work,   wp = hM - h3 = 153.86 - 137.79 = 16.07 kJ/kg
Heat added  qa = h1 - hM = 2580.6 - 153.86 = 2426.74 kJ/kg
wt − w p 983.74 − 16.07
(a)  Thermal efficiency = = = 0.3987 or 39.87%
qa 2426.74
. Wcycle 130 × 103 × 3600
(b)  Steam mass flow rate, ms = = = 483635.95 kg/h
wt − wp 983.74 − 16.07
1 × 3600 1 × 3600
(c)  Specific steam consumption, SSC = = = 3.72 kg/kWh
wt − wp 983.74 − 16.07
T

4 160 bar 1
M

0.05 bar

3 2

Figure 4.4  Ideal Rankine cycle

M04_THERMAL ENGINEERING_SE_XXXX_CH04.indd 167 6/15/2018 2:34:09 PM


168 Chapter 4

Example 4.3
In a steam power plant working on ideal Rankine cycle, the steam turbine receives steam at 10 bar
250°C and discharges at 0.5 bar. Find the thermal efficiency.

Solution
At p1 = 10 bar, ts = 179.91°C. Therefore, steam is superheated. The T-s diagram is shown in Fig. 4.5.

T
1′
4 10 bar
M 1

0.5 bar

3 2′
x2
s

Figure 4.5  Rankine cycle

From steam tables, at 10 bar, 250°C,


h′1 = 2942.6 kJ/kg, s′1 = 6.9246 kJ/kg.K
At p2 = 0.5 bar, vf   3 = 0.001030 m3/kg, hf   3 = 340.47 kJ/kg, hfg3 = 2305.4 kJ/kg, s   f   3 = 1.091 kJ/kg.K,
sfg3 = 6.5029 kJ/kg.K,
s′1 = s′2 = sf   3 + x′2sfg3
or 6.9246 = 1.091 + x′2 × 6.5029
or x′2 = 0.897
∴ h′2 = hf3 + x′2hfg3 = 340.47 + 0.897 × 2305.4 = 2408.4 kJ/kg
Pump work, wp = vf   3 (p1-p2) × 102 = 0.001030 (10 - 0.5) × 102 = 0.9785 kJ/kg
Net work output wnet = wt -wp = (h′1-h′2) - wp
= (2942.6 - 2408.4) - 0.9785 = 533.22 kJ/kg
Heat added qa = h′1 - hf   3 - wp = 2942.6 - 340.47 - 0.9785 = 2601.15 kJ/kg
wnet 533.22
Thermal efficiency hth = = = 0.205 or 20.5%
qa 2601.15

4.3.2 Effect of Boiler and Condenser Pressure


The Rankine cycle comprises internally reversible processes. Thus, the equation for thermal efficiency
can be written in terms of average temperatures of heat addition and heat rejection processes.
With reference to Fig. 4.6, we have
Heat addition, qa = area 1-b-c-M-4-1
1
= ∫ Tds = (Tavg)in (s1-sM)
M

M04_THERMAL ENGINEERING_SE_XXXX_CH04.indd 168 6/15/2018 2:34:11 PM


Steam Power Cycles 169

1′
4
1
M

3 2 2′

s
c b

Figure 4.6  Heat addition and rejection in a Rankine cycle

Heat rejected,   qr = area 2-b-c-3-2


= (Tavg)out (s2-s3)
= (Tavg)out (s1-sM)
qr T
Thermal efficiency, hth = 1 - = 1 - out  (4.11)
qa Tin
Thus hth increases if Tout decreases or Tin increases. Therefore, Eq. (4.11) can be used to study the
effects on the performance of the changes in boiler and condenser pressures.
1. Effect of boiler pressure: Consider two different cases having same condenser pressure but dif-
ferent boiler pressures as shown in Fig. 4.7(a). The average temperature of heat addition is higher
for higher pressure cycle 1′-2′-3-M′-1′ than for cycle 1-2-2-3-M-1. Thus, increase of boiler
pressure leads to increase in the thermal efficiency of Rankine cycle.
2. Effect of condenser pressure: Figure 4.7(b) shows two ideal Rankine cycles with the same
boiler pressure but two different condenser pressures. One condenser operates at atmospheric
pressure and the other at pressure less than atmospheric. The temperature of heat rejection
for the cycle 1-2′′-3′′-M′′-1 is correspondingly lower than that for cycle 1-2-3-M-1. Thus,
the cycle 1-2′′-3′′-M′′-1 has higher thermal efficiency. Therefore, decreasing the condenser
pressure increases the thermal efficiency.

T p1′ T p1
4′ 1′ p1 4 1
1
M′ p2
M
4
M p2 M′′ p2′′
3 2

3 2′ 2 3′′ 2′′

s s
(a) (b)

Figure 4.7 Effect of boiler and condenser pressure on Rankine cycle efficiency:


(a) Effect of increase of boiler pressure, (b) Effect of decrease of
condenser pressure

M04_THERMAL ENGINEERING_SE_XXXX_CH04.indd 169 6/15/2018 2:34:13 PM


170 Chapter 4

4.4 ❐ METHODS OF IMPROVING EFFICIENCY


The efficiency of the Rankine cycle can be improved by the following methods:
1. Reheating
2. Regeneration
3. Combination of reheating and regeneration

4.4.1 Reheat Cycle


Figure 4.8(a) shows the schematic diagram for the reheat cycle. The corresponding h-s and T-s diagrams
are shown in Fig. 4.8(b) and Fig. 4.8(c), respectively. The steam is extracted at a suitable point and is
reheated with the help of flue gases in the boiler furnace. This increases the dryness fraction of steam
passing through the LP turbine.
Heat supplied, qs = (h1 - hf  6) + (h3 - h2)
Pump work, wp = hf  6 - hf  5
Turbine work, wt = (h1 - h2) + (h3 - h4)
Net work output, wnet = wt - wp

Flue gases out

Reheater L.P.
3 turbine
2
Superheater 1

4
Boiler H. P.
turbine Condenser

6 5
pump
(a)
h p1 T
p2′
1 1
3
3 7
p2
6 2
2
5 4
4

s s
(b) (c)

Figure 4.8 Reheat cycle: (a) Schematic diagram, (b) h-s diagram,


(c) T-s diagram

M04_THERMAL ENGINEERING_SE_XXXX_CH04.indd 170 6/15/2018 2:34:13 PM


Steam Power Cycles 171

T p1 h
p2
p2 − ∆p p1
6 1 p2
2′ (p2 − ∆p2)
3
2 1 3
p3
p3
2′
2
4′
5 4 4′ 4

s s
(a) (b)

Figure 4.9 Reheat cycle with pressure drop between stages:


(a) T-s diagram, (b) h-s diagram

wnet ( h − h ) + ( h3 − h4 ) − ( h f 6 − h f 5 )
Efficiency of reheat cycle, hreheat = = 1 2  (4.12)
qs ( h1 − h f 6 ) + ( h3 − h2 )
If pump work is neglected,
( h1 − h2 ) + ( h3 − h4 )
hreheat = (4.13)
( h1 − h f 6 ) + ( h3 − h2 )

4.4.2 Effect of Pressure Drop in the Reheater


The T-s and h-s diagrams for a reheat cycle with pressure drop is shown in Fig. 4.9(a) and (b), respec-
tively. Steam enters the prime mover first stage at point 1. Line 1-2 is the isentropic process. However,
in actual practice, it is an irreversible process due to internal frictional resistance offered to the flow of
steam. Thus, the condition after the irreversible adiabatic expansion is shown by point 2′. The steam at
condition 2′ is passed on the reheater. Since steam cannot pass through the reheater without a pressure
drop, the exit from the reheater at point 3 account for the reheater pressure drop from p2 to p2 - ∆p.
The steam then expands irreversibly and adiabatically to point 4′.
h − h′
For the first stage, turbine efficiency, ht1 = 1 2
h1 − h2
h3 − h4′
And for the second stage, ht2 =
h3 − h4

( h1 − h2′ ) + ( h3 − h4′ )
hreheat = (4.14)
( h1 − h f 5 ) + ( h3 − h2′ )
Example 4.4
A high pressure boiler delivers steam at 90 bar and 480°C. The steam is expanded in the first stage
of turbine to 12 bar and withdrawn and passed on the reheater. The expansion now takes place in
the second stage of the steam turbine, down to the condenser pressure of 0.07 bar. Calculate (a) the
efficiency of the ideal cycle and (b) the work output and efficiency of the reheat cycle operating
through the same states neglecting pump work.

M04_THERMAL ENGINEERING_SE_XXXX_CH04.indd 171 6/15/2018 2:34:16 PM


172 Chapter 4

p1
T pi
1
3
90 bar
7
12 bar
2 p2
6
0.07 bar
5 4 x4

Figure 4.10  Reheat Rankine cycle

Solution
The cycle on T-s diagram is shown in Fig. 4.10.
At 90 bar, 480°C, from steam tables,
h1 = 3336.5 kJ/kg, s1 = 6.5942 kJ/kg.K
Now, s1 = s2
Since, sg at 12 bar is less than 6.5942, the state 2 is in the superheated region. then, we get
h2 = 2814.4 kJ/kg, t2 = 201°C
At 12 bar, 480°C, h3 = 3432.8 kJ/kg, s3 = 7.6198 kJ/kg.K
At 0.07 bar, sf4 = 0.5589 kJ/kg.K, sfg4 = 7.7179 kJ/kg.K
s3 = sf4 + x4sfg4

or 7.6198 = 0.5589 + x4 × 7.7179


or x4 = 0.915
 h4 = hf4 + x4hfg4 = 163.40 + 0.915 × 2409.1 = 2367.7 kJ/kg
hfs = hf4 = 163.4 kJ/kg
 h6 = h5 + wp
 wp = nfs (p1 - p2) × 102
= 0.001007 (90 - 0.07) × 102 = 9 kJ/kg
 h6 = 163.4 + 9 = 172.4 kJ/kg
(a)  qa = (h1 - h6) + (h3 - h2) = (3336.5 - 172.4) + (3432.8 - 2814.4) = 3782.5 kJ/kg
            wnet = (h1 - h2) + (h3 -h4) - wp
= (3336.5 - 2814.4) + (3432.8 - 2367.7) - 9 = 1578.2 kJ/kg
w 1578.2
hreheat = net = = 0.4172 or 41.72%
qa 3782.5
(b)  Neglecting pump work
qa = 3782.5 + 9 = 3781.5 kJ/kg
wnet = 1578.2 + 9 = 1587.2 kJ/kg
1578.2
hreheat = = 0.4186 or 41.86%
3791.5

M04_THERMAL ENGINEERING_SE_XXXX_CH04.indd 172 6/15/2018 2:34:18 PM


Steam Power Cycles 173

Example 4.5
In Example 4.4, steam enters the reheater at 12 bar, 210°C, and leaves at 11.5 bar, 480°C. The
combined steam rate is 3 kg/kWh, the generator efficiency is 94%, and the heat loss through the
turbine casing is 1% of the throttle enthalpy. Determine the turbine thermal efficiency and condi-
tion of exhausted steam.
Solution
The processes in the T-s diagram is shown in Fig. 4.11.

T 1
3
2′
2

4′

5 4
s

Figure 4.11  Reheat Rankine cycle with turbine efficiency

Refer to Fig. 4.11


t′2 = 210°C, p′2 = 12 bar, t3 = 480°C, p3 = 11.5 bar
Now h1 = 3336.5 kJ/kg, h2 = 2814.4 kJ/kg
h′2 at 12 bar, 210°C from steam tables
= 2839.8 kJ/kg
h3 at 11.5 bar, 480°C = 3433.4 kJ/kg
1 × 3600
Now wnet = = 1276.6 kJ/kg
3 × 0.94
For energy balance of turbine,
h1 + h3 = h′2 + h′4 + wnet + qloss
h′4 = h1 - h′2 + h3 - wnet - 0.01 × h1
= 3336.5 - 2839.8 + 3433.4 - 1276.6 - 0.01 × 3336.5 = 2620.1 kJ/kg
At p4 = 0.07 bar, h′4 = 2620.1 kJ/kg, t ′4 = 64°C
Since saturation temperature at 0.07 bar is 39.02°C, the exhaust steam is superheated and the degree
of superheat is 24.98°C.
Efficiency of turbine upto extraction point,
h1 − h2′ 3336.5 − 2836.5
ht = = = 0.95 or 95%
h1 − h2 3336.5 − 2814.4
Overall efficiency of turbine,

( wnet )actual 1276.6


ht0 = = = 0.8089 or 80.89%
( wnet )ideal 1578.2

M04_THERMAL ENGINEERING_SE_XXXX_CH04.indd 173 6/15/2018 2:34:20 PM


174 Chapter 4

Example 4.6
Steam at 100 bar and 500°C enters a prime mover that has one stage of reheat. The steam is
exhausted from the prime mover at 0.07 bar and 85% dry. The net work developed by the prime
mover is 1600 kJ/kg of steam. Calculate the thermal efficiency of the prime mover.
Solution
The cycle on T-s diagram is shown in Fig. 4.12.

T p1
1 pi
100 bar 3

2 p2

0.07 bar
5 4 x4 = 0.85
s

Figure 4.12  Reheat Rankine cycle

Refer to Fig. 4.12.


From steam tables for p1 = 100 bar, 500°C,
h1 = 3373.6 kJ/kg
At 0.07 bar, hf5 = 163.38 kJ/kg
h4 = hf4 + x4 hfg4
= 163.38 + 0.85 × 2409.2 = 2211.2 kJ/kg
wnet = 1600 kJ/kg
wnet
hth =
( h1 − h f 5 ) + ( h3 − h2 )

wnet = (h1 - h2) + (h3 - h4)


or h3 - h2 = wnet + h4 - h1 = 1600 + 2211.2 - 3373.6 = 435.6 kJ/hg
1600
Thermal efficiency hth = = 0.4386 or 43.86%
( 3373 . 6 − 163 .38) + 435.6

4.5 ❐ REGENERATION
In the Rankine cycle and reheat, the condensate which is at a fairly low temperature is pumped to the
boiler. Thus, there is an irreversible mixing of the cold condensate with hot boiler water. This results
in loss of cycle efficiency. Regeneration is a method to heat the feed water from the hot well of the
condenser reversibly by interchange of heat within the system, thus improving the cycle efficiency.
The cycle is called regenerative cycle.

M04_THERMAL ENGINEERING_SE_XXXX_CH04.indd 174 6/15/2018 2:34:21 PM


Steam Power Cycles 175

Heat transferring Insulation


surface

Rotor blades

Water 1 2
4 Steam

Boiler Casing in Condenser


3
turbine

Pump

Figure 4.13  Regenerative heating

In Fig. 4.13, steam is drawn from the boiler at state point 1 and passes through the turbine and
goes to the condenser at state point 2. If no heat is transferred from this steam to the surrounding,
which include the casing, and the expansion is isentropic. This process is shown by line 1-2′ in the
T-s diagram (Fig. 4.14). However, in the regenerative cycle, it is assumed that the condensate after
being pumped to the boiler pressure at state point 4 passed through the hollow casing surrounding the
turbine rotor to the boiler. Therefore, the steam losses heat to the surrounding water which gets heated.
The temperature of steam entering the turbine and temperature of water leaving the turbine are same.
Let T1 = saturation temperature of steam at the commencement of expansion at state point 1.
T3 = temperature of saturated water condensate entering the hollow casing of turbine at state point 3.
Thus, 1 kg of water is gradually heated along the path 3-4 and 1 kg of steam gradually loses the same
amount of heat during expansion process 1-2. Such a heat exchange within the system is called regen-
erative heating. Therefore, the net heat supplied to the system is in the boiler during process 4-1 and
the net heat rejected from the system is in the condenser during process 2-3 (Fig. 4.14).
Heat added, qa = T1 (s1 -s4)
Heat rejected,  qr = T2 (s2 - s3)
Net work done,
wnet = qa - qr
= T1 (s1 - s4) - T2 (s2 - s3)
But s1 - s4 = s2 - s3
∴ wnet = (T1 - T2) (s1 - s4)

4 1

3 2 2′

Figure 4.14  T-s diagram for regenerative Rankine cycle

M04_THERMAL ENGINEERING_SE_XXXX_CH04.indd 175 6/15/2018 2:34:22 PM


176 Chapter 4


wnet
Regenerative efficiency, hreg =
(T − T ) ( s − s )
= 1 2 1 4 =1- 2 
T
(4.15)
qa T1 ( s1 − s4 ) T1
This method of regeneration is not practicable because the steam becomes extremely wet at the later
stages of expansion.

4.5.1 Regenerative Cycle with Open Heaters


The regeneration effect can be achieved by bleeding or extracting small quantities of steam at different
points during the expansion and exploiting the energy of bled steam rather than whole steam. Thus,
that part of the steam which continues to expand and to do work, does not condense excessively. A
regenerative cycle with three bleeding points and therefore, three open type heaters, where feed water
and steam mix, is shown in Fig. 4.15. The corresponding T-s diagram is shown in Fig. 4.16.
Let 1 kg of steam enter the turbine at state point 1 and expand to the state point 2 where m1 kg of
steam is extracted from the turbine and passed on to the heater 3. The remainder (1 - m1) kg of steam
continues to expand isentropically to state points 3 where m2 kg of steam is extracted and passed on
the heater 2. The remainder (1 - m1 - m2) kg of steam further continues to expand isentropically to
state point 4 where m3 kg of steam is bled and passed on to heater 1. Thus, only (1 - m1 - m2 - m3) kg

1 kg steam
Turbine
1
wout
qin Boiler
2 3 4
b 1 kg m2 5 (1−m1−m2−m3) kg
m1 m3
O.H. O.H. qout
O.H.
3 1 6
2
7 Condenser
9 8
wp
4 wp wp
3 wp 1
2
(1−m1) kg (1−m1−m2) kg (1−m1−m2−m3) kg water

Figure 4.15  Regenerative feed water heating with open heaters

T
1
1 kg
wp b9 10 1 kg
4 m1
wp b8 9 2 (1−m1) kg
3 m2
w p b7 8 3
2 m3 (1−m1−m2) kg
w p b6 7 4
1 (1−m1−m2−m3) kg water
6 (1−m1−m2−m3) kg 5

Figure 4.16  T-s diagram for feed heating with open heaters

M04_THERMAL ENGINEERING_SE_XXXX_CH04.indd 176 6/15/2018 2:34:24 PM


Steam Power Cycles 177

of steam does the remaining isentropic expansion to point 5 before entering the condenser, where it is
condensed to point 6. The condensate (1 - m1 - m2 - m3) kg at enthalpy b6 is mixed in heater 1. m3
kg of steam bled at state point 4 condenses to state point 7 and (1 - m1 - m2 -m3) kg of water gets
heated from b6 to point 7. (1 - m1 - m2) kg of feed water at state point 7 is pumped to heater 2 at state
point by. In the heater (1 - m1 - m2) kg of feed water is heated from state point 7 to state point 8 and
m2 kg of bled steam from point 3 condenses to point 8. Thus, heater 2 has (1 - m1) kg of feed water at
point 8 which is pumped to heater 3 raising the enthalpy of pumped feed water to b8. This feed water
in heater 3 gets heated to point 9 by condensation of m1 kg of steam bled from point 2, leaving 1kg
of feed water in heater 3 to be pumped to boiler pressure by raising its enthalpy to b9. Heating of 1 kg
of feed water from b9 to point 10 is done in the boiler where it is further heated to steam at point 1.
For simplified analysis, points 6 and b6, 7 and b7, 8, and b8, 9 and b9 are treated as coincident
points, thus ignoring the very small quantity of pump work. Thus, we have
Heat gained by feed water = Heat lost by bled steam
Heater 1: (1 - m1 - m2 - m3) (h7 - h6) = m3 (h4 - h7)
Heater 2: (1 - m1 - m2) (h8 - h7) = m2 (h3 - h8)
Heater 3: (1 - m1) (h9 - h8) = m1 (h2 - h9)
Net work output, wout = (h1 - h2) + (1 - m1) (h2 - h3) + (1 - m1 - m2) (h3 - h4) + (1 - m1 - m2 - m3)
(h4 - h5)
wnet = wout - ∑wp
Heat added, qa = h1 - hb9 = h1 - h9 - wp4
w
Regenerative cycle efficiency, hreg = net (4.16)
qa
Now ∑wp = (1 - m1 - m2 - m3) (hb6 - h6) + (1 - m1 - m2) (hb7 - h7) + (1 - m1) (hb8 - h8) + (hb9 - h9)
_ vf 6(p1 - p6)

where p1 = throttle pressure, p6 = condenser pressure and vf 6 = specific volume of water at condenser
pressure.

4.5.2 Regenerative Cycle with Closed Heaters


In this type of heater, the feed water is contained in the tubes about which the bled steam passes and
condenses. One pump can be used for two or more heaters placed in series. The heaters can be placed
in various combinations. One such arrangement is shown in Fig. 4.17. The corresponding T-s diagram
is shown in Fig. 4.18.
Work done per kg of steam supplied to turbine,
wt = (h1 - h2) + (1 - m1) (h2 - h3) + (1 - m1 - m2) (h3 - h4)
Total pump work, ∑wp = wp1 + wp2 + wp3
= (1 - m1 - m2) (h6 - h5) + m2 (h10 - h9) + m1 (h12 - h11)
where h6 - h5 = vf5 (p1 - p4) × 102 kJ/kg
h10 - h9 = vf  9 (p1 - p3) ×102 kJ/kg
h12 - h11 = vf11 (p1 - p2) × 102 kJ/kg
where vf is in m3/kg and p is in bar.
Now, vf 5 = vf 9 = vf 11 = saturated volume of water

M04_THERMAL ENGINEERING_SE_XXXX_CH04.indd 177 6/15/2018 2:34:25 PM


178 Chapter 4

Turbine
1 kg steam
1 2 3 4
m1 m2 (1−m1−m2) kg

13 Boiler
Condenser
1 kg 5
8 C.H.2 7 C.H.1
6 wp
12 m1 11 9 1

m2
wp
3

10 wp
2

Figure 4.17  Regenerative feed water heating with closed heaters


T
1
13 1 kg, p1
1 kg
12 m1, p2
8 2
10 m2, p3 (1−m1) kg
7 3
6 9 (1−m1−m2) kg
5 4
(1−m1−m2), p4

Figure 4.18  T-s diagram for regenerative Rankine cycle with closed heaters

Net work done, wnet = wt - ∑wp


Heat added in the boiler, qa = h1 - hf 13
wnet wt − ∑ w p
Regenerative efficiency, hreg = =  (4.17)
qa h1 − h f 13
Neglecting heat losses and considering heat balance at feed heaters, we have
m2 (h3 - hf 9) = (1 - m1 - m2) (hf 9 - hf 5)
and m1 (h2 - hf 11) = (1 - m1 - m2) (hf 11 - hf 9)
Neglecting enthalpy increase due to pump,
hf 6 = hf 5
also hf 7 = hf 9
and hf 8 = hf 11
The condition of feed water h13 entering the boiler is given by,
(1 - m1 - m2) hf 11 + m2hf 9 + m1hf 11 = 1 × h13
or h13 = (1 - m2) hf 11 + m2hf 9

M04_THERMAL ENGINEERING_SE_XXXX_CH04.indd 178 6/15/2018 2:34:27 PM


Steam Power Cycles 179

Example 4.7
A boiler feeds a turbine at 56 bar and 600°C. Before being passed on to the condenser at 30°C, the
steam is bled for regenerative feed heating at 6.5 bar. For an ideal regenerative cycle and 1kg/s of
throttle steam, determine (a) the amount of bled steam, (b) net work done, and (c) the ideal effi-
ciency of the cycle.
Solution
The T-s diagram is shown in Fig. 4.19.
At p1 = 56 bar, 600°C from steam tables,
h1 = 3659.5 kJ/kg, s1 = 7.2011 kJ/kg.K
At 6.5 bar, h2 = 2987 kJ/kg
and t2 = 265°C
s3 = s1 = sf 3 + x3 sgh3
At 30°C, sf 3 = 0.4369 kJ/kg.K, sfg3 = 8.01674 kJ/kg.K
or 7.2011 = 0.436 + x3 × 8.0164
or x3 = 0.8438
h3 = hf 3 + x3hgh3 = 125.79 + 0.8438 × 2430.5 = 2176.6 kJ/kg
At 6.5 bar, hf 6 = 684.26 kJ/kg, vf 6 = 0.001104 m3/kg
h7 = hf 6 + vf 6 (p1 - p2) × 102 = 684.26 + 0.001104 (56 - 6.5) × 102
= 684.26 + 5.47 = 689.72 kJ/kg
wp2 = 5.47 kJ/kg
vf 4 = 0.001004 m3/kg
p4 corresponding to 30°C = 0.042461 bar
wp1 = vf 4 (p2 - p4) = 0.001004(6.5 - 0.042461) × 102
= 0.648 kJ/kg
∑wp = wp1 + wp2 = 0.648 + 5.47 = 6.118 kJ/kg

T
1 56 bar
8 1 kg 6.5 bar
7
wp m1 kg
2 2
6
5
wp 30°C (1−m1)kg
1
4 3

Figure 4.19  Regenerative Rankine cycle with a single heater

M04_THERMAL ENGINEERING_SE_XXXX_CH04.indd 179 6/15/2018 2:34:28 PM


180 Chapter 4

(a) Neglecting pump work, for heat balance of heater,


m1 (h2 - hf6) = (1 - m1) (hf7 - hf4)

hf 6 − hf 4 684.26 − 125.77
m1 =
= = 0.195 kg
h2 − h f 4 2987 − 125.77
(b) wnet = wt -∑wp = (h1 - h2) + (1 - m1) (h2 - h3) -∑wp
= (3659.5 - 2987) + (1 - 0.195) (2987 - 2176.6) - 6.118
= 1318.754 kJ/kg
(c) Heat added, qa = (h1 - h7) = 3659.5 - 689.72 = 2969.78 kJ/kg
wnet 1318.754
Cycle efficiency, hideal = = = 0.444 or 44.4%
qa 2969.78

Example 4.8
Determine the improvement of efficiency which would results if a single stage of regenerative feed
heating were added to a steam cycle having terminal conditions of 16 bar, 300°C and 0.05 bar. The
steam for feed heating is to be extracted at 1.16 bar. The drain from the heat exchanger at satura-
tion temperature of bled steam pressure is returned to the system at a point downstream from the
heat exchanger, the feed water getting heated to saturation temperature at pressure of bled steam.
Neglect change in enthalpy due to liquid pump work.
Solution
The regenerative cycle and h-s diagram is shown in Fig. 4.20(a) and (b).
From the Mollier diagram,
h1 = 3035 kJ/kg, h2 = 2575 kJ/kg, h3 = 2098 kJ/kg
From steam tables, hf  1 = 856.6 kJ/kg, hf  2 = 475.4 kJ/kg, hf  3 = 137.8 kJ/kg
For heat balance of heater,
m (h2 - hf  3) = (hf  2 - hf  3)

1 kg
1

Turbine
2 h, kJ/kg
3 1 300°C
3035
(1−m) kg
m
Condenser
(1−m) (1−m) Pump 2575 2
r

1 kg
ba
16

CH.
r
ba

2098 r
ba 3
6

5
1.

m
Hot well 0.0

Pump s
(a) (b)

Figure 4.20 Single stage regenerative heating Rankine cycle: (a) Schematic


diagram, (b) Mollier diagram

M04_THERMAL ENGINEERING_SE_XXXX_CH04.indd 180 6/15/2018 2:34:30 PM


Steam Power Cycles 181

hf 2 − hf 3 475.4 − 137.8
or m= = = 0.138 kg
h 2− hf 3 2575 − 137.8
wnet = (h1 - h2) + (1 - m) (h2 - h3)
= (3035 - 2575) + (1 - 0.138) (2575 - 2098) = 871 kJ/kg
qa = h1 - hf  2 = 3035 - 475.4 = 2559.6 kJ/kg
wnet 871
Thermal efficiency with regeneration, hreg = = = 0.34 or 34%
qa 2559.6
Without feed water heating,
wnet = h1 - h3 = 3035 - 2098 = 937 kJ/kg
qa = h1 - hf  3 = 3035 - 137.8 = 2897.2 kJ/kg
937
Thermal efficiency without regeneration, hr = = 0.323 or 32.3%
2897.2
34 − 32.3
Improvement in thermal efficiency = = 5.36%
32.3
Example 4.9
A regenerative cycle with three-stage bleed heating works between 30 bar, 450°C, and 0.04 bar.
The bleed temperatures are chosen at equal temperature ranges. Determine the efficiency of the
cycle. Neglect the pump work.
Solution
The schematic diagram of the cycle, T-s, and h-s diagrams are shown in Fig. 4.21(a) to (c).
At p = 0.04 bar, ts = 30°C
Temperature range = 450 - 30 = 420°C
420
Equal temperature difference = = 105°C
4
The temperatures of bled steam are: 450 - 105 = 345°C, 240°C, 135°C
From Mollier chart,
h1 = 3340 kJ/kg, h2 = 3150 kJ/kg, h3 = 2935 kJ/kg,
h4 = 2720 kJ/kg, hs = 2120 kJ/kg
The pressure at 2, 3, 4 points are noted from the h-s chart. Then from steam tables,
hf  1 = 1008.35 kJ/kg, hf  2 = 837.45 kJ/kg, hf  3 = 684.0 kJ/kg,
hf  4 = 517.62 kJ/kg, hf  5 = 121.4 kJ/kg
For heat balance of heaters,
m3 (h4 - hf  4) = (1 - m1 - m2 - m3) (hf  4 - hf  5)
m2 (h3 - hf  3) = (1- m1 - m2 - m3) (hf  3 - hf  4)
m1 (h2 - hf  2) = (1 - m1) (hf  2 - hf  3)
∴ m3 (2720 - 517.62) = (1 - m1 - m2 - m3) (517.62 - 121.4)
or     2202.38 m3 = (1 - m1 - m2 - m3) × 396.22
m2 (2935 - 684) = (1 - m1 - m2) (684 - 517.62)

M04_THERMAL ENGINEERING_SE_XXXX_CH04.indd 181 6/15/2018 2:34:32 PM


182 Chapter 4

2
3
4
(1−m1−m2−m3) kg
10 m1 m2 m3 5
H1 H2 H3 Condenser
9 8 7 6

Pump
(a)

T h
1 1 45°C
10 2 345°C
bar 3 240°C
9 2 30 bar 4
. 5 135°C
8 3 14 ar
. 5 b
6 bar
. 6 5
7 2 ar
4 4b
0.0

6 5
s s
(b) (c)

Figure 4.21 Regenerative Rankine cycle with three-stage bleeding:


(a) Schematic diagram, (b) T-s diagram, (c) h-s diagram

or     2251 m2 = (1 - m1 - m2) × 166.38


m1 (3120 - 837.45 ) = (1 - m1) (837.45 - 684)
or 2282.55 m1 = (1 - m1) × 153.45
Solving the above three equations, we get
m1 = 0.063 kg, m2 = 0.0645 kg, m3 = 0.1332 kg
Net work done, wnet = 1 × (h1 - h2) + (1 - m1) (h2 - h3) + (1 - m1 - m2 ) (h3 - h4) + (1 - m1 - m2 - m3)
(h4 - h5)
= (3340 - 3120) + (1 - 0.063) (3120 - 2935) + (1 - 0.063 - 0.0645) (2935 - 2720) + (1 - 0.063
- 0.0645 - 0.13327) (2720 - 2120) = 1024.6 kJ/kg
Heat added, qa = h1 - hf  2 = 3340 - 837.45 = 2502.55 kJ/kg
w 1024.6
Thermal efficiency of the cycle, hreg = net = = 0.4094 or 40.94%
qa 2502.55

4.6 ❐ REHEAT-REGENERATIVE CYCLE


Reheating is mainly adopted to gain advantage of drier steam in the low pressure stages, and regener-
ation is adopted for increasing the thermal efficiency of the cycle. Therefore, it is advisable to execute
cycle adopting both reheat and regeneration. Figure 4.22(a) shows the schematic diagram of such a
cycle with two-stage regeneration and one-stage reheating at first bleeding point. The same cycle is
represented on T-s diagram in Fig. 4.22(b).

M04_THERMAL ENGINEERING_SE_XXXX_CH04.indd 182 6/15/2018 2:34:34 PM


Steam Power Cycles 183

Reheater

1 kg steam

1 (1−m1)
Wout T
2 3
1 3 (1−m1)kg
1 kg 1kg
qin Turbine wpb8 9
2 5 3 m1
Boiler 4 (1−m1−m2)kg b 2
wp 7 8 m2 4
9 b6
2
(1−m1−m2−m3)kg
1 kg m1 m2 7
H2 wp
O.H. O.H. 1
qout 5′
2 1 6 (1−m1−m2) kg 5
8 7 6 Condenser
s
wp wp wp
3 2 1

(a) (b)

Figure 4.22 Reheat-regenerative cycle: (a) Schematic


diagram, (b) T-s diagram

At point 2, m1 kg of steam is extracted from the turbine and is passed on to open heater 2 and the
remaining (1 - m1) kg steam goes to reheater. The steam from the reheater comes back to the turbine
at state point 3. Thereafter, (1 - m1) kg of steam expands from state point 3 to state point 4. At point
4, again m2 kg of steam is extracted and passed on to open heater 1. From state point 4, (1 - m1 - m2)
kg of steam expands to condenser pressure state point 5, and further condenses in the condenser to
water down to state point 6. Neglecting pump work, net work output from the prime mover is given by,
wout = (h1 - h2) + (1 - m1) (h3 - h4) + (1 - m1 - m2) (h4 - h5)
Net work of the cycle, wnet = wout - ∑wp
where ∑wp = wp1 + wp2 + wp3
= (1 - m1 - m2) (hb6 - h6) + (1 - m1) (hb7 - h7) + (hb8 - h8)
_ v (p - p ) × 102 kJ/kg
∼ f6 1 6
Heat added, qa = (h1 - hf  8) + (1 - m1) (h3 - h2) - ∑wp
wnet
Thermal efficiency, hth =
qa

4.7 ❐ PROPERTIES OF AN IDEAL WORKING FLUID


An ideal working fluid should have the following favourable properties:
1. Plentiful and cheap availability
2. Non-toxic
3. Chemical stability
4. Non-corrosiveness
5. High critical temperature
6. Saturation pressure should be above atmospheric pressure to guard against leakage

M04_THERMAL ENGINEERING_SE_XXXX_CH04.indd 183 6/15/2018 2:34:36 PM


184 Chapter 4

4.8 ❐ BINARY VAPOUR CYCLES


In a binary vapour cycle, two working substances are used—one with good characteristics at high
temperature and the other with good at low temperature characteristics. Figures 4.23(a) and (b) show
two Rankine cycles in a combined arrangement. The heat is rejected from the high temperature cycle
(mercury cycle) as energy supplied to the low temperature cycle (steam cycle). This occurs in a heat
exchanger which functions as the condenser for mercury cycle and boiler for the water cycle. Specific
enthalpy increase of water as it passes through the heat exchanger is several times the magnitude of
the specific enthalpy decrease of mercury. Thus, the mass of mercury circulated in mercury cycle is
several times more than mass of water in the water cycle. Binary vapour power cycles have a great
advantage of its operation at high average heat addition temperature than conventional water cycles
and give higher thermal efficiency.
Economiser
4

4′
Steam superheater
and mercury boiler Wt
1 Steam 2
Water
turbine
5
A Wt
1

B Steam
Mercury condenser
Mercury turbine
Mercury
condenser
and steam boiler C
4′ 4
D Wp Water pump
1
Wp
(a) 2

A
m kg Hg
D 1
B
C
4′ 1 kg 5
4 H 2O
3 2
s
(b)

Figure 4.23 Binary vapour cycles: (a) Schematic diagram,


(b) Mercury and steam cycles
Mercury cycle A-B-C-D-A is a simple Rankine cycle using saturated vapour. Heat is supplied
to mercury in the process D-A. Mercury expands in process A-B and is condensed in process B-C.
Mercury-saturated liquid is compressed in feed pump from condenser pressure to boiler pressure in
process C-D. Thus, the Rankine cycle on mercury is completed.

M04_THERMAL ENGINEERING_SE_XXXX_CH04.indd 184 6/15/2018 2:34:36 PM


Steam Power Cycles 185

Heat rejected during condensation in the mercury condenser is supplied to water to transform water
into saturated steam in process 4-5. Process 4-5 may take place partly in the economiser located
in the exhaust gases of mercury boiler and partly in the mercury condenser. The saturated vapour
is superheated in the superheater located in the mercury boiler furnace process 5-1. Ideal process
4-4′occurs in the economiser and 4′-5 in the mercury condenser. Superheated steam expands in the
turbine, process 1-2, and then condenses in the steam condenser, process 2-3. The condensate or feed
water is then pumped by process 3-4, and heated till it is saturated liquid in the economiser as shown
by process 4-4′, before going to the mercury condenser - steam boiler where enthalpy of evaporation
is absorbed.
Let m = flow rate of mercury in the mercury cycle for 1 kg steam being circulated in steam cycle.
Heat added, Qa = m (hA - hD) + (h1 - h5) + ( h′4 - h4)
Heat rejected, Qr = (h2 - h3)
Workdone by steam and mercury turbines,
Wt = m (hA - hB) + (h1 - h2)
Workdone on the feed pumps for mercury liquid and water,
Wp = m (hD - hC) + (h4 - h3)
Thermal efficiency of binary cycle,
Qa − Qr Wt − W p W
hth = = = net  (4.18)
Qa Qa Qa
3600
    Steam rate = kg/kWh (4.19)
Wnet
The mercury cycle is known as topping cycle and the steam cycle is known as bottoming cycle.

Example 4.10
A binary vapour cycle consists of two ideal Rankine cycles with mercury and steam as working
substances. The following data is given:
Saturated mercury vapour pressure at entry to turbine = 4.5 bar
Mercury vapour exit pressure from turbine = 0.04 bar
Saturated steam generated in mercury condenser pressure = 15 bar
Steam superheated temperature in superheater in mercury boiler = 300°C
Condensate water pressure pumped through economiser located in exhaust flue of mercury
boiler to saturation temperature = 15 bar
Isentropic efficiency of mercury turbine = 0.85
Isentropic efficiency of steam turbine = 0.88
Calculate (a) the overall thermal efficiency of the cycle and (b) the rate of flow of mercury turbine
in kg/h.
Properties of saturated mercury

p t hf hg sf sg vf vg
bar °C kJ/kg kJ/kg kJ/kg.K kJ/kg.K 3
m /kg 3
m /kg
4.5 450 62.93 355.98 0.1352 0.5397 79.9 × 10-6 0.068
0.04 216.9 29.98 329.85 0.0808 0.6925 76.5 × 10-6 5.178

M04_THERMAL ENGINEERING_SE_XXXX_CH04.indd 185 6/15/2018 2:34:38 PM


186 Chapter 4

T D′ 4.5 bar A
450°C
m kg
Hg
D 300°C
1
216.9°C
C 0.04 bar B B′
198.29°C 4′ 15 bar
1 kg H2O 5
4
0.05 bar
3 2″ 2 2′
s

Figure 4.24 Hg - H2O binary vapour cycle

Solution
Refer to Fig. 4.24.
Mercury cycle
hA = 3559 kJ/kg, pA = 4.5 bar, tA = 450°C, sA = 0.5397 kJ/kg.K
Now sA = sf  B + xB (sgB - sf  B)
or 0.5397 = 0.0808 + xB (0.6925 - 0.0808)
or xB = 0.75
hB = hf  B + xB (hgB - hf  B) = 29.98 + 0.75 (329.85 - 29.98) = 254.88 kJ/kg
hB′ = hA - (hA - hB) ht1 = 355.98 - (355.98 - 254.88) × 0.85 = 270.04 kJ/kg
pB′ = 0.04 bar, tB′ = 216.9°C
hc = hfc = 29.98 kJ/kg, tc = 216.9°C
hD = hc + vfc (pD - pC) × 102 = 29.98 + 76.5 × 10-6 (4.5 - 0.04) × 102 = 30.0142 kJ/kg
wp1 = vfc (pD - pC) × 102 = 0.03412 kJ/kg
Steam cycle
p1 = 15 bar, t1 = 300°C, h1 = 3038.9 kJ/kg, s1 = 6.9207 kJ/kg. K
p2 = 0.05 bar, hf  2 = 137.77 kJ/kg, hg2 = 2561.6 kJ/kg, sf  2 = 0.4763 kJ/kg.K,
h5 = 2789.9 kJ/kg, h4′ = 844.66 kJ/kg
hfg2 = 2423.8 kJ/kg, sg2 = 8.3960 kJ/kg.K, vf2 = 0.0010052 m3/kg
s1 = s2 = sf2 + x2 sfg2
or 6.9207 = 0.4763 + x2 (8.3980 - 0.4763)
or x2 = 0.8137
h2 = hf  2 + x2hfg2 = 137.77 + 0.8137 × 2423.8 = 2110 kJ/kg
h2′ = h1 - (h1 - h2) ht2 = 3038.9 - (3038.9 - 2110) × 0.88 = 2221.47 kJ/kg
h3 = hf  3 = 137.77 kJ/kg
wp2 = vf  3 (p4 - p3) × 102 = 0.0010052 (15 - 0.05) × 102 = 1.503 kJ/kg

M04_THERMAL ENGINEERING_SE_XXXX_CH04.indd 186 6/15/2018 2:35:24 PM


Steam Power Cycles 187

h4 = h3 + wp2 = 137.77 + 1.503 = 139.27 kJ/kg


h5 − h4′ 2789.9 − 844.66
mHg = = = 8.103 kg Hg/kg of steam
hB′ − hC 270.04 − 29.98
Total heat added, (qa)total = mHg (hA - hD) + 1 [(h1 - h5) + h4′ - h4)]
= 8.103 (355.98 - 30.0142) + 1 [(3038.9 - 2789.9) + (844.66 - 139.27)] = 3595.7 kJ/kg
Work done, (wt)Hg = mHg (hA - hB′) = 8.103 (355.98 - 270.04) = 696.37 kJ/kg of steam
(wt)steam = 1 × (h1 - h′2) = 1 × (3038.9 - 2221.47) = 817.43 kJ/kg
(wt)total = 696.37 + 817.43 = 1513.8 kJ/kg
Neglecting pump work being very small,
( wt )total 1513.8
hoverall = = = 0.421 or 42.1%
( qa )total 3595.7
Considering pump work,
( wt )total − ∑ w p 1513.8 − ( 0.03412 + 1.503)
hoverall = = = 0.4206 or 42.06%
( qa )total 3595.7

4.9 ❐ COMBINED POWER AND HEATING


CYCLE-COGENERATION
In the Rankine cycle, the heat rejected is considered to be of no value. However, this heat can be sup-
plied by steam, for the heating of buildings, and heating required by many industrial processes. Thus,
the functions of power generation and heating can often be combined effectively. This combination is
often called cogeneration.
In such a cycle, the process heater replaces the condenser of an ordinary Rankine cycle. The
pressure of the exhaust from the turbine is the saturation pressure corresponding to the temperature
desired in the process heater. Such a turbine is called back pressure turbine. The cycle for a single
plant carrying varying heat and power loads is shown in Fig. 4.25.

m1

Turbine Wt
1 PRV
m6 2
6 m2 m3
qin Steam
3
generator 7 Condenser
Process
heater qOUT
qr
8
9 Wp m4
4 Cooling
5 water
m5 Wp

Figure 4.25  Combined power and heating cycle

M04_THERMAL ENGINEERING_SE_XXXX_CH04.indd 187 6/15/2018 2:35:27 PM


188 Chapter 4

When the power load is zero, all the steam passes through the pressure reducing valve (PRV) and
none through the turbine. When heating load is zero, the steam expands through the turbine and flows
into the condenser. In order to meet heating and power loads, the steam flow distribution is deter-
mined as follows:
The steam is extracted from the turbine at point 2 where the pressure is held constant by an auto-
matic valve arrangement. The flow rate in the extraction line is computed from energy balance on the
heating system as:
The power produced by this steam flowing through the turbine from the extraction point is,
W = m2 (h1 - h2)
Now h1 = h6 and h2 = h1 - w, so that h6 > h2.
The steam distribution to compute m2, m3 and m6 must satisfy the First Law of Thermodynamics
for power required.
W = m2 (h1 - h2) + m3 (h1 - h3)(4.20a)
and for heating load, Q = m2 (h2 - h8) + m6 (h6 - h8)(4.20b)

Example 4.11
In a binary-vapour cycle, the steam cycle operates between 30 bar, 0.07 bar and uses a superheated
temperature of 350°C. The mercury cycle works between 12.68 bar and 0.07 bar. The mercury
vapour entering the turbine is in a dry saturated condition. Calculate the efficiency of the combined
cycle assuming expansion in both cycles to be isentropic. Data for mercury is given below:

p bar Saturation temp. hf hg sf sg


°C kJ/kg kJ/kg kJ/kg.K kJ/kg.K
0.07 236.5 32.395 326.667  0.08548 0.662906
12.68 537.5 71.9796 360.734 0.145798 0.50185

Solution
The binary vapour cycle is shown in Fig. 4.26.
Mercury cycle: sa = sb = sf b + xb (sgb - sf b)
0.50185 = 0.08548 + xb (0.662906 - 0.08548)
or xb = 0.721
T
d a
Hg
cycle
c b
1
4 1′
Steam
cycle
3 2

Figure 4.26  Binary vapour cycle neglecting pump work

M04_THERMAL ENGINEERING_SE_XXXX_CH04.indd 188 6/15/2018 2:35:27 PM


Steam Power Cycles 189

hb = hf b + xb (hgb - hf b)


= 32.395 + 0.721 (326.667 - 32.395) = 244.565 kJ/kg of Hg
Isentropic work done, wm = ha - hb = 360.734 - 244.565 = 116.169 kJ/kg Hg
Heat rejected, qrm = hb - hf b = 244.565 - 32.395 = 212.17 kJ/kg Hg
Heat supplied, qsm = ha - hf b = 360.734 - 32.395 = 328.339 kJ/kg Hg
Steam cycle: h1 = 3115.3 kJ/kg, s1 = 6.743 kJ/kg.K, h′1 = 2804.2 kJ/kg
sf  2 = 0.5589 kJ/kg.K, sfg2 = 7.7179 kJ/kg.K, hf  3 = 163.40 kJ/kg
s1 = s2 = sf  2 + x2 sfg2
or 6.743 = 0.5589 + x2 × 7.7179
or x2 = 0.80127
h2 = hf 2 + x2 hfg2 = 163.40 + 0.80127 × 2409.1 = 2093.733 kJ/kg
Work done, ws = h1 - h2 = 3115.3 - 2093.733 = 1021.567 kJ/kg
Heat supplied, qss = h1 = h1′ = 3115.3 - 2804.2 = 311.1 kJ/kg
Heat rejected by Hg/kg of steam = Heat received by water/kg steam
mm (hb - hf b) = 1 (h1′ - hf 3)
2804.2 − 163.4
or mm = = 12.447 kg Hg/kg steam
212.17
Total work done per kg of steam,
w1 = ws + mm wm = 1021.567 + 12.447 × 116.161 = 2467.379 kJ/kg steam
Heat supplied in the cycle,
qs = mm qsm + qss = 12.447 × 328.339 + 311.1 = 4397.936 kJ/kg steam
wt 2467.379
Cycle efficiency = = = 0.5610 or 56.10%
qs 4397.936
Example 4.12
In a single-heater regenerative cycle, the steam enters the turbine at 32 bar, 350°C and the exhaust
pressure is 0.12 bar. The feed water heater is a direct contact type which operates at 4.5 bar. Find
(a) the efficiency and the steam rate of the cycle and (b) the increase in efficiency and steam rate as
compared to Rankine cycle. Neglect pump work.
Solution
From steam tables, h1 = 3113.2 kJ/kg, s1 = 6.712 kJ/kg.K = s2 = s3
sg at 4.5 bar = 6.875 kJ/kg.K
The schematic diagram is shown in Fig. 4.27 (a). The processes on the T-s and h-s diagrams are shown
in Figs. 4.27(b) and (c) respectively. Since s2 < sg, state point 2 lies in the wet region.
sf 2 = 1.821 kJ/kg.K, sfg2 = 5.036 kJ/kg.K
s1 = s2 = sf  2 + x2 sfg2
or 6.712 = 1.821 + x2 × 5.036
or x2 = 0.9712
h2 = hf 2 + x2 hfg2 = 623.3 + 0.9712 × 2120.6 = 2682.83 kJ/kg

M04_THERMAL ENGINEERING_SE_XXXX_CH04.indd 189 6/15/2018 2:35:29 PM


190 Chapter 4

1 kg
1

Boiler Turbine
4.5 bar
0.12 bar, (1 − m1) kg
m1 kg
2 3
7
Heater Condenser

5 4
6

Pump
(1 − m1) kg
1 kg Pump

(a)

T h
1 350°C 1
32 bar 350°C
1 kg 1 kg bar
7 32 1 kg
4.5 bar 7 1 kg
2
6 m1 kg 2 bar ) kg
5 (1 − m1) kg 4.5 (1 − m 1
0.12 bar 5 m 1 kg
r 3
1 (1 − m1) kg 3
6 0.12 ba
) kg
(1 − m 1
4
s s
(b) (c)

Figure 4.27 Single stage regenerative cycle: (a) Flow schematic


diagram, (b) T-s diagram, (c) h-s diagram

sf  3 = 0.6963 kJ/kg.K sfg3 = 7.3900 kJ/kg.K


s1 = s3 = sf  3 + x3 sfg3
or 6.712 = 0.6963 + x2 × 7.3900
or x3 = 0.814
h3 = hf  3 + x3 hfg3 = 206.92 ÷ 0.814 × 2384.1 = 2147.58 kJ/kg
Since wp = 0
∴ h4 = hf  3 = 206.92 kJ/kg = h5
h6 = hf  2 = 623.3 kJ/kg = h7
Energy balance for heater gives,
m1 (h2 - h6) = (1 - m1) (h6 - h5)
or m1 (2682.83 - 623.3) = (1 - m1) (623.3 - 206.92)

M04_THERMAL ENGINEERING_SE_XXXX_CH04.indd 190 6/15/2018 2:35:29 PM


Steam Power Cycles 191

or 2059.53 m1 = (1 - m1) × 416.38


or m1 = 0.168 kg
wt = (h1 - h2) + (1 - m1) (h2 - h3)
  = (3113.2 - 2682.83) + (1 - 0.168) (2628.83 - 2147.58) = 875.698 kJ/kg
qs = h1 - h6 = 3113.2 - 623.3 = 2489.9 kJ/kg
wt 875.698
(hth)reg = = = 0.3517 or 35.17%
qs 2489.9
3600 3600
Steam rate = = = 4.11 kg/kWh
wt 875.698
Without regeneration, wt = h1 - h3 = 3113.2 - 2147.58 = 965.62 kJ/kg
wt 965.62
hth = = = 0.3322 or 33.22%
h1 − h4 2906.28
Increase in cycle efficiency due to regeneration = 35.17 - 33.22 = 1.95%
3600
Steam rate = = 3.728 kg/kWh
965.62
Increase in steam rate due to regeneration = 4.11 - 3.728 = 0.382 kg/kWh

Example 4.13
A steam power plant working on Rankine cycle is supplied with dry saturated steam at a pressure
of 12 bar and exhausts into the condenser at 0.1 bar. Neglecting the pump work, calculate the cycle
efficiency.

Solution
Corresponding to p = 12 bar, from steam tables we find that (see Fig. 4.3)
hf  1 = 798.4 kJ/kg; hg1 = 1984.3 kJ/kg
s1 = sg1 = 6.519 kJ/kg°K
Corresponding to 0.1 bar, we find that
hf  2 = 191.8 kJ/kg; hfg2 = 2393 kJ/kg
sf  2 = 0.649 kJ/kg K; sfg2 = 7.502 kJ/kg·K
Process 1–2 is an isentropic expansion
s1 = s2
s1 = sg1 = 6.519 kJ/kg K
s2 = sf  2 + x2 sfg2
s2 = 0.649 + x2 × 7.502
6.519 = 0.649 + 7.502 x2
x3 = 0.783
h2 = hf  2 + hfg2

M04_THERMAL ENGINEERING_SE_XXXX_CH04.indd 191 6/15/2018 2:35:31 PM


192 Chapter 4

=191.81 + 0.783 × 2392.8 = 2065.5 kJ/kg


h1 = hg2 = 2782.7 kJ/kg
h1 − h2 2782.7 − 2065.5
hth = = = 0.277 or 27.7%
h1 − h f 2 2782.7 − 191.8

Example 4.14
Dry saturated steam at 10 bar is supplied to a prime mover working on Rankine cycle and the
exhaust takes place at 0.2 bar. Determine the cycle efficiency, efficiency ratio, and specific steam
consumption of the prime mover, if the indicated thermal efficiency is 20%. Determine the percent-
age change in the cycle efficiency if the steam is initially 90% dry.

Solution
The h-s diagram is shown in Fig. 4.28.
From Mollier diagram (Fig. 4.28), we have
h1 = 2775 kJ/kg, h2 = 2150 kJ/kg
From steam tables,
hf  2 = 251.5 kJ/kg corresponding to 0.2 bar

h1 − h2 2775 − 2150
Thermal efficiency hth = = = 0.247 = 24.7%
h1 − h f 2 2775 − 251.5
Indicated thermal efficiency 0.2
Efficiency ratio = = = 0.81 or 81%
Rankine efficiency 0.247
3600 3600
Specific steam consumption = = = 5.76 kJ/kWh
h1 − h2 2775 − 2150
Percentage change in cycle efficiency if steam is initially 90% dry
From Mollier diagram, h1 = 2580 kJ/kg, h2 = 2030 kJ/kg (see Fig. 4.29)
h1 − h2 2580 − 2030
Thermal efficiency hth = = = 0.236 = 23.6%
h1 − h f2 2580 − 215.5
24.7 − 23.6
∴ Percentage change in Rankine efficiency = × 100 = 4.45%
24.7

h r
ba
0
1 1
h1
r
sa

ba
t.

0.2
l
in
e

h2
2

Figure 4.28  Mollier diagram for dry saturated steam

M04_THERMAL ENGINEERING_SE_XXXX_CH04.indd 192 6/15/2018 2:35:35 PM


Steam Power Cycles 193

sa
h1

t.
lin
r

e
ba
0.2

x=
h2
2

0.
9
s

Figure 4.29  Mollier diagram for wet steam

Example 4.15
The steam consumption of steam engine is 20 tonnes per shift of 8 hours when developing 220 kW.
Dry and saturated steam enters the engine at 10 bar pressure and leaves at 0.1 bar pressure. Esti-
mate the cycle efficiency and thermal efficiency of engine.
Solution
Given that ms = 20/8 tonne/h = 2.5 tonne/h = 2500 kg/h, P = 220 kW
The h-s diagrams is shown in Fig. 4.30.
From steam tables corresponding to 10 bar
h1 = hg1 = 2778.1 kJ/kg; s1 = sg1 = 6.5864 kJ/kg.K
and corresponding to 0.1 bar, we find that
hf  2 = 191.81 kJ/kg; hfg2 = 2392.8 kJ/kg;
sf  2 = 0.6492 kJ/kg.K; sg2 = 7.5010 kJ/kg.K
Since 1-2 is an isentropic process
s1 = s3
or 6.5864 = 0.6492 + x2 × 7.5010
or x2 = 0.791
∴ h3 = hf  2 + x2 hfg2 = 191.81 + 0.791 × 2392.8 = 2084.5 kJ/kg
h1 − h2 2778.1 − 2084.5
Cycle Efficiency (hR) = = = 0.2682 or 26.82%
h1 − h f 2 2778.1 − 191.81
Thermal efficiency of engine;
3600 × P 3600 × 220
hth = = = 0.1225 or 12.25%
ms ( h1 − h f 2 ) 2500 [ 2778.1 − 191.81]
r
h ba
0
1 1
h1
r
sa

ba
t.

0.1
lin
e

h2
2

Figure 4.30  Mollier diagram for dry saturated steam

M04_THERMAL ENGINEERING_SE_XXXX_CH04.indd 193 6/15/2018 2:35:38 PM


194 Chapter 4

Example 4.16
Steam at 50 bar, 400°C expands in a Rankine engine to 0.34 bar. For 150 kg/s of steam; determine
(a) the power developed, (b) the thermal efficiency neglecting the pump work, and (c) the specific
steam consumption.

Solution
The h-s diagrams is shown in Fig. 4.31.

h
1 400°C
50 bar

ar
4b

Sa
0.3

t.
lin
e
2

Figure 4.31  Mollier diagram for superheated steam

(a) Power developed = m (h1 - h2)


From Mollier diagram (Fig. 4.31), we have
h1 = 3198.3 kJ/kg, h2 = 2257.75 kJ/kg
hf  2 = 301.5 kJ/kg (from steam tables)
Power developed = 150 [3198.3 - 2257.75] = 141082 kW
(b) Thermal efficiency
h1 − h2 3198.3 − 2257.75
hth = = = 0.3246 = 32.46%
h1 − h f 2 3198.3 − 301.5
3600 3600
(c) Specific steam consumption = = = 3.83 kg/kWh
h1 − h2 3198.3 − 2257.75
Example 4.17
A steam engine is supplied with dry saturated steam at 15 bar. The pressure at release is 3 bar and
the back pressure is 1 bar. Determine the efficiency of modified Rankine cycle.

Solution
Consider the Mollier diagram, as shown in Fig. 4.32.
From the Mollier diagram, we have
h1 = 2790 kJ/kg, h2 = 2510 kJ/kg, x2 = 0.9
From steam tables, corresponding to 3 bar, we have
vf  2 = 0.001073 m3/kg, vg2 = 0.6058 m3/kg
∴ Volume of steam at point 3,
v2 = vf  2 + x2 (vg2 - vf  2) = 0.001073 + 0.9 × (0.60585 - 0.001073) = 0.545 m3/kg

M04_THERMAL ENGINEERING_SE_XXXX_CH04.indd 194 6/15/2018 2:35:40 PM


Steam Power Cycles 195

h r
ba
15
1
h1 r
ba
3

sa
h2

t.
lin
2

e
x2
=0
.9
s

Figure 4.32  Mollier diagram for dry saturated steam


From steam tables corresponding to pressure of 1 bar, we find that sensible heat of water,
hf  4 = 417.44 kJ/kg
hMR = Thermal efficiency of modified Rankine cycle
(h − h ) + 100( p2 − p3 )v2
= 1 2
h1 − h f 3
( 2790 − 2510) + 100(3 − 1) × 0.545
= = 0.164 or 16.4%
2790 − 417.44

Example 4.18
Steam at a pressure of 15 bar and 250°C is first expanded through a turbine to a pressure of 4 bar.
It is then reheated at constant pressure to the initial temperature of 250°C and is finally expanded
to 0.1 bar. Estimate the work done per kg of steam flowing through the turbine and the amount of
heat supplied during the process of reheat.
Find the work output when there is direct expansion from 15 bar to 0.1 bar without any reheat.
Assume all expansion processes to be isentropic.
Solution
Given that p1 = 15 bar; T1 = 250°C; p2 = 4 bar; T2 = 250°C; p3 = 0.1 bar
The reheating of steam is represented on the Mollier chart as shown in Fig. 4.33. From the chart,
we find that,
h1 = 2930 kJ/kg; h2 = 2660 kJ/kg; h3 = 2965 kJ/kg; h4 = 2345 kJ/kg; and h5 = 2130 kJ/kg
From steam tables, corresponding to a pressure of 0.1 bar, we find that sensible heat of water at D,
hf4 = hf5 = 191.81 kJ/kg
Workdone per kg of steam:
We know that workdone per kg of steam,
w = (h1 - h2) + (h3 - h4)
= (2930 -2660) + (2965 - 2345) = 890 kJ/kg
Heat supplied during the process of reheat:
We know that the heat supplied during the process of reheat,
h = Heat supplied between 2 and 3
  = (h3 - h2) - (hf4) = (2965 - 2660) - 191.81 = 113.2 kJ/kg

M04_THERMAL ENGINEERING_SE_XXXX_CH04.indd 195 6/15/2018 2:35:41 PM


196 Chapter 4

h
3 250°C
h3
h1
1

p1 sat
h2 . lin
2 e
h4
p2 4
h5
p3 5
s

Figure 4.33  Mollier diagram for superheated steam with reheating

Work output when the expansion is direct:


The direct expansion from 15 bar to 0.1 bar is shown by the line AE in Fig. 4.33. We know that work
output
= Total heat drop = h1 - h5 = 2930 - 2130 = 800 kJ/kg

Example 4.19
In a thermal plant, steam is supplied at a pressure of 30 bar and temperature of 300°C to the
high pressure side of steam turbine where it is expanded to 5 bar. The steam is then removed and
reheated to 300°C at a constant pressure. It is then expanded to the low pressure side of the turbine
to 0.5 bar. Find the efficiency of the cycle with and without reheating.
Solution
Given that p1 = 30 bar; T1 = 300°C; p2 = 5 bar; T2 = 300°C; p3 = 0.5 bar
Efficiency of the cycle with reheating:
The reheating of steam is represented on the Mollier chart as shown in Fig. 4.34.
From the chart, we find that
h1 = 2990 kJ/kg; h2 = 2625 kJ/kg; h3 = 3075 kJ/kg; h4 = 2595 kJ/kg; and h5 = 2280 kJ/kg
From steam tables, corresponding to a pressure of 0.5 bar, we find that sensible heat of water at D,
hf  4 = hf  5 = 340.47 kJ/kg

h
3 300°C
h3
h1
1

p1 sat
h2 . lin
2 e
h4
p2 4
h5
p3 5
s

Figure 4.34  Mollier diagram for superheated steam with reheating

M04_THERMAL ENGINEERING_SE_XXXX_CH04.indd 196 6/15/2018 2:35:42 PM


Steam Power Cycles 197

We know that efficiency of the cycle with reheating,


( h1 − h2 ) + (h3 − h4 )
h1 = = ( 2990 − 2625) + (3075 − 2595)
h1 + ( h3 − h2 ) − h f 4  2990 + [(3075 − 2625) − 340.47]

845
= = 0.2726 or 27.26%
3099.53
Efficiency of the cycle without reheating:
We know that efficiency of the cycle without reheating
h1 − h5 2990 − 2280
h2 = = = 0.268 or 26.8%
h1 − h f 5 2990 − 340.47

Example 4.20
Steam supplied to a 10 MW turbo-alternator at 40 bar and 400°C. The auxiliaries consume 7% of
the output. The condenser pressure is 0.05 bar and condensate is sub-cooled to 30°C. Assuming the
boiler efficiency as 85%, the relative efficiency of turbine as 80%, and the mechanical efficiency of
the alternator as 95%, determine (a) the steam consumption per hour, (b) the overall efficiency of
the plant, and (c) the quality of steam at exit from turbine.

Solution
The h-s diagrams is shown in Fig. 4.35.
(a) From Mollier chart,
h1 = 3215 kJ/kg, h2 = 2064 kJ/kg
Therefore, (h1 - h2) = 3215 - 2064 = 1151 kJ/kg
Thus actual enthalpy drop,
(h1 - h2) = hrelative × (h1 - h2)
= 0.8 × 1151 = 920.8 kJ/kg
Output 10, 000
Input to the alternator = = = 10526 kW
h mech 0.95

h, kJ/kg 400°C
1
3215
r
ba

3
40

2′

2064 r 2
5 ba
0.0

Figure 4.35  Mollier diagram for superheated steam

M04_THERMAL ENGINEERING_SE_XXXX_CH04.indd 197 6/15/2018 2:35:45 PM


198 Chapter 4

Therefore, steam consumption per hour,


10526 × 60 × 60
m=
( h1 − h3 )
10526 × 60 × 60
= = 41154 kg/h
920.8
1
(b) Heat supplied to the boiler/hour = Steam/h × (h1 - hf  2) ×
h boiler
1
= 41154 × (3215 - 137.79) × = 1.48988 × 108 kJ/h
0.85
where hf2 = enthalpy of liquid at saturation at 0.05 bar from steam tables
The auxiliaries consume 7% of the output.
Therefore, useful output = (10,000 × 0.93) = 9300 kW = 9300 × 60 × 60 kJ/h
9300 × 60 × 60
Thus overall efficiency = = 0.225 or 22.5%
1.48988 × 108
(c) The dryness fraction at exit, that is, B′ is read at h2′ = 3215 - 920.8 = 2294.2 kJ on 0.05 bar line.
Thus, x2′ = 0.889

Example 4.21
Two turbines, A and B, operate with steam at an initial pressure of 100 bar and temperature of
500°C. In each turbine, the steam is expanded in a high pressure turbine to 8.5 bar with efficiency
ratio 80%. In turbine A, the expansion is further continued in low pressure turbine from 8.5 bar
to 0.035 bar with efficiency ratio 0.75. In turbine B, steam is reheated after expansion in the high
pressure turbine and is then fed to the low pressure unit at 7 bar and 500°C, after which it expands
to 0.035 bar with efficiency ratio 0.85. Compare the two power cycle with respect to (a) thermal
efficiency and (b) steam consumption for a full load output of 50,000 kW.
Solution
Turbine A: The expansion is shown on h-s chart in Fig. 4.36(a)
ha = 3377 kJ/kg and hb = 2753 kJ/kg
hc = hd = 3377 - 0.8 (3377-2753) = 2878 kJ/kg
he = 2060 kJ/kg
hf = hg = 2878 - (2878 - 2060) × 0.75 = 2264.5 kJ/kg
Total work done = (ha - hd) + (hd - hg) = (ha - hg)
= 3377 - 2264.5 = 1112.5 kJ/kg
Heat supplied   = ha - hg
where hg is the sensible heat of steam at 0.035 bar = 110 kJ
∴ Heat supplied = 3377 - 110 = 3267 kJ
1112.5
Thermal efficiency = = 34.1%
3267
Total work to be done = 53,000 kW = 50,000 kJ/s
= 50,000 × 3600 kJ/h

M04_THERMAL ENGINEERING_SE_XXXX_CH04.indd 198 6/15/2018 2:35:47 PM


Steam Power Cycles 199

h, kJ/kg 100 bar


h, kJ/kg 100 bar
8.5 bar
3490 e
a 8.5 bar a
3377 500°C

0.035 bar 0.035 bar


c d c d
2753
b b g h
f
g
2060 e 0.088 2385 f
Saturation
line
s s
(a) (b)

Figure 4.36 Mollier diagrams: (a) h-s diagram for Turbine A,


(b) h-s diagram for Turbine B
50, 000 × 3600
∴ Steam consumption = = 161800 kg/h
1112.5
Turbine B: The expansion is shown on h-s chart in Fig. 4.36(b)
ha = 3377 kJ/kg: hb = 2753 kJ/kg; hc = hd = 2878 kJ/kg
he = 3490 kJ/kg; hf = 2385 kJ/kg.
hh = 3490 - (3490 - 2315) 0.85 = 2551 kJ/kg
Total work done = (3377 - 2878) + (3490 - 2551) = 1438 kJ/kg
Total heat supplied = (3377 - 110) + (3490 - 2878) = 3879 kJ/kg.
1438
∴ Thermal efficiency hth = = 37%
3879
50, 000 × 3600
Steam consumption = = 125500 kg/h
1438
Example 4.22
In a regenerative cycle, having one feed water heater, the dry saturated steam is supplied from the
boiler at a pressure of 30 bar and the condenser pressure is 1 bar. The steam is bled at a pressure
of 5 bar. Determine the amount of bled steam per kg of steam supplied and the efficiency of the
cycle. What would be the efficiency without regenerative feed heating? Determine the percentage
increase in efficiency due to regeneration.

Solution
Given that p1 = 30 bar; p3 = 1 bar; p2 = 5 bar
From Mollier diagram, as shown in Fig. 4.37, we find that
Enthalpy of steam at 30 bar, h1 = 2800 kJ/kg
Enthalpy of steam at 5 bar, h2 = 2460 kJ/kg
Enthalpy of steam at 1 bar, h3 = 2220 kJ/kg
From steam tables, we also find the enthalpy or sensible heat of water at 5 bar.

M04_THERMAL ENGINEERING_SE_XXXX_CH04.indd 199 6/15/2018 2:35:48 PM


200 Chapter 4

h1 Sat.
line
r
ba
30

h2
ar
5b
h3
1 bar
s

Figure 4.37  Mollier diagram for dry saturated steam

hf  2 = 640.21 kJ/kg


and enthalpy or sensible heat of water at 1 bar,
hf  3 = 417.44 kJ/kg
Amount of bled steam per kg steam supplied
We know that amount of bled steam per kg of steam supplied,
hf 2 − hf 3 640.21 − 417.44
m= = = 0.109 kg
h2 − h f 3 2460 − 417.44
Efficiency of the cycle: We know that efficiency of the cycle,

( h1 − h2 ) + (1 − m)( h2 − h3 )
h1 =
h1 − h f 2
( 2800 − 2460) + (1 − 0.109)( 2460 − 2220) 553.84
= = = 0.256 or 25.6 %
2800 − 640.21 2159.79
Efficiency of the cycle without regenerative feed heating: We know that efficiency of the cycle,

h1 − h3 2800 − 2220
h2 = = = 580
h1 − h f 3 2800 − 417.44 2382.56
= 0.243 or 24.3 %
Percentage increase in efficiency due to regeneration: We know that percentage increase in efficiency
due to regeneration
0.256 − 0.243
= = 0.0535 or 5.35 %
0.243

Example 4.23
In a steam turbine plant, the steam is generated and supplied to the turbine at 50 bar and 370°C.
The condenser pressure is 0.1 bar and the steam enters the condenser with dryness fraction of 0.9.
Two feed water heaters are used, the steam in the heaters being bled at 5 bar and 0.5 bar. In each
heater, the feed water is heated to saturation temperature of the bled steam. The condensate is also
pumped at this temperature into the feed line immediately after the heater. Find the masses of the

M04_THERMAL ENGINEERING_SE_XXXX_CH04.indd 200 6/15/2018 2:35:52 PM


Steam Power Cycles 201

steam bled in the turbine per one kg of steam entering the turbine. Assuming the condition line for
the turbine to be straight, calculate the thermal efficiency of the cycle.
Solution
Given that p1 = 50 bar; T1 = 370°C; p4 = 0.1 bar; x4 = 0.9; p2 = 5 bar; p3 = 0.5 bar
First, let us draw the Mollier diagram and condition line for the cycle, as shown in Fig. 4.38. From
this diagram, we find that,
h1 = 3110 kJ/kg, h2 = 2780 kJ/kg, h3 = 2510 kJ/kg, h4 = 2350 kJ/kg
From steam tables, we also find that
hf  2 = 640.21 kJ/kg (at 5 bar)
hf  3 = 340.47 kJ/kg (at 0.5 bar)
hf  4 = 191.81 kJ/kg (at 0.1 bar)
Mass of steam bled in the turbine:
We know that mass of steam bled at 2.
hf 2 − hf 3 640.21 − 340.47
m1 = = = 0.123 kg
h2 − h f 3 2780 − 340.47
and mass of steam bled at 3.
(1 − m1 )( h f 3 − h f 4 )
m2 = = (1 − 0.123)(340.47 − 191.81)
h3 − h f 4 2510 − 191.81
= 0.056 kg
Thermal efficiency of the cycle:
We know that work done from 1 to 2 per kg of feed water
= h1 - h2 = 3110 - 2780 = 330 kJ/kg
Similarly, work done from 2 to 3 per kg of feed water
= (1 - m1)(h2 - h3) = (1 - 0.123) (2780 - 2510) kJ/kg
= 236.8 kJ/kg
and work done from 3 to 4 per kg of feed water
= (1 - m1 - m2) (h3 - h4) = (1 - 0.123 - 0.056) (2510 - 2350) kJ/kg
= 131.4 kJ/kg
h
1
h1 370°C

r 2
ba
h2 50
r
h3 5 ba Sa
t.li
3 ne
ba r
h4 0.5

0.1 bar 4
s

Figure 4.38  Mollier diagram

M04_THERMAL ENGINEERING_SE_XXXX_CH04.indd 201 6/15/2018 2:35:54 PM


202 Chapter 4

∴ Total work done = 330 + 236.8 + 131.4 = 698.2 kJ/kg


Heat supplied = h1 - hf  2 = 3110 - 640.1 = 2469.9 kJ/kg
∴ Thermal efficiency of cycle,
Work done 698.2
hth = = = 0.283 or 28.3%
Heat supplied 2469.9
Example 4.24
In the Rankine cycle, steam leaves the boiler and enters the turbine at 4 MPa and 400°C. The con-
denser pressure is 10 kPa. Neglecting pump work, determine the cycle efficiency and the Carnot
efficiency for the same temperature limits.
Solution
Refer to Fig. 4.39. Rankine cycle is represented by 1-2-3-4 ignoring pump work, Carnot cycle for the
same temperature limits may be reproduced by 1-2′-3′-4′.
Carnot efficiency is given by

T1 − T2 ( 400 + 273) − ( 45.83 + 273)


hcarnot = = = 0.5263 or 52.63%.
T2 ( 400 + 273)
(It may be noted that Carnot efficiency is independent of the working substance.)
h1 − h2
hRankine =
h1 − h f 3
From steam tables, h1 at 4MPa and 400°C = 3213.5 kJ/kg , s1 = 6.7689 kJ/kg.K
At 10 kPa
hf = 191.81 kJ/kg, hfg = 2392.8 kJ/kg, sf = 0.6492 kJ/kg.K, sfg = 7.5010 kJ/kg.K
Also s1 = s2
or 6.7689 = 0.6492 + x2 × 7.5010
6.7689 − 0.6492
or x3 = = 0.816
7.5010
∴ h2 = hf  3 + x2hfg = 191.81 + 0.816 × 2392.8 = 2144.33 kJ/kg
3213.5 − 2144.33
Thermal efficiency hRankine = = 0.3538 or 35.38%
3213.5 − 191.81

T, K
4′ 400°C 1
673

4 MPa
4

3 0.01 MPa
318.83
3′ 2
s

Figure 4.39  T-s diagram for Rankine cycle

M04_THERMAL ENGINEERING_SE_XXXX_CH04.indd 202 6/15/2018 2:35:56 PM


Steam Power Cycles 203

Example 4.25
A steam power plant has the range of operation from 40 bar dry saturated to 0.05 bar. Determine
(a) the cycle efficiency and (b) the work ratio and specific fuel consumption for (i) Carnot cycle
and (ii) Rankine cycle.
Solution
From steam tables:
At 40 bar, tg = 250.4°C, vf = 0.001252 m3/kg, vg = 0.049778 m3/kg, hf = 1087.29 kJ/kg,
hg = 2801.40 kJ/kg, sg = 6.07 kJ/kg.K
At 0.05 bar, tg = 32.88°C, vf = 0.0010005 m3/kg, vg = 28.193 m3/kg, hf = 137.79 kJ/kg
hfg = 2423.7 kJ/kg, sf = 0.4763 kJ/kg.K, sfg = 7.9187 kJ/kg.K
(i)  Refer of Fig. 4.40. for Carnot cycle analysis on T-s plane.
Work done Heat added − Heat rejected
Cycle efficiency = =
Heat added Heat added
( h1 − h4 ) − ( h2 − h3 )
=
( h1 − h4 )
(Note: Processes 1-2 and 3-4 are reversible adiabatic.)

Also s1 = s2 = sf + x2sfg

6.07 − 0.4763
or  x2 = = 0.706.
7.9187
Also s4 = s3 = sf + x3sfg

2.7963 − 0.4763
or   x3 = = 0.293.
7.9187

T, K

40 bar
523.4 1
4

0.05 bar
305.88
3 2

3.272
s, kJ/kgK
2.796 6.068

Figure 4.40  Carnot cycle on T-s plot

M04_THERMAL ENGINEERING_SE_XXXX_CH04.indd 203 6/15/2018 2:35:58 PM


204 Chapter 4

4 40 bar 1

M
0.06 bar
3 2

kJ/kgk
2.796 0.069

Figure 4.41  Rankine cycle on T-s plot

∴ h2 = 137.79 + 0.706 × 2423.7 = 1848.9 kJ/kg.


and h3 = 137.79 + 0.293 × 2423.7 = 847.9 kJ/kg.
∴ hCarnot cycle = ( 2801.4 − 1087.29) − (1848.9 − 847.9)
( 2801.4 − 1087.29)

= 1714.11 − 1001 = 0.416 or 41.6%


1714.11
Alternatively,

hCarnot cycle =
T1 − T2
=
( 273.15 + 250.4) − ( 273.15 + 32.88) = 0.415 or 41.5%
T2 ( 273.15 + 250.4)
(ii) Refer to Fig. 4.41 for Rankine cycle analysis
Pump work = hM - hf  3
105
= vf  3(pM - p3)× kJ/kg = 0.001005 (40 - 0.05) × 102
103
= 0.001005 × 39.95 × 102 = 4.015 kJ/kg
Therefore, net work
wnet = h1 - h2 - wpump = 2801.4 - 1848.9 - 4.015 = 948.485 kJ/kg
wnet 948.485 948.485
and hRankine cycle = = =
h1 − hM 2801.4 − (137.79 + 4.015) ( 2659.595)
= 0.3566 or 35.66%
Net work 948.485
(b) Also work ratio = = = 0.9958
Turbine work 2801.4 − 1848.9
Specific steam consumption/kWh
1kW-hr 1 × 60 × 60 kJ/kW-hr 3600
= = = kg/kWh = 3.795 kg/kWh
wnet kJ/kg 948.485 kJ/kg 948.485

M04_THERMAL ENGINEERING_SE_XXXX_CH04.indd 204 6/15/2018 2:36:03 PM


Steam Power Cycles 205

Example 4.26
Steam at 50 bar, 400°C expands in a steam tubine to 0.34 bar. For 150 kg/s of steam; determine
(a) the power developed, (b) the thermal efficiency, and (c) the specific steam consumoption (i) for
the Rankine cycle and (ii) for the Rankine engine. For an actual engine with same specifications,
the brake steam rate is 4.75 kg/kWh and the driven electric generator has an electro mechanical
efficiency of 94%.
Determine (a) the brake thermal efficiency, (b) the internal efficiency (expansion efficiency),
(c) the power in kW, and (d) the exhaust dryness fraction of steam.

Solution
Refer to Fig. 4.42.
At 50 bar and 400°C
h1 = 3195.6 kJ/kg, s1 = 6.6458 kJ/kg.K, v1 = 0.05781 m3/kg
At 0.34 bar, hf = 301.48 kJ/kg, hfg = 2328.9 kJ/kg, sf = 0.9795 kJ/kg.K, sfg =6.7471 kJ/kg.K
And s1 = s2 for ideal expansion
6.6458 = 0.9795 + x2 ×6.7471
or x2 = 0.84
Thus h2 = hf  3 + x2hfg = 301.43 + 0.84 × 2328.9 = 2257.756 kJ/kg
(a) For cycle wpump = Pump work = hM - hf  3

0.0010241 × (50 − 0.34 ) × 105


= vf  3 (pM - pf  3) = kJ/kg = 5.0857 kJ/kg
1000
(i) Power developed = Steam per sec × (h1 - h2 - wpump)
= [150 (3195.6 - 2257.756 - 5.0857)] = 139913.7 kW
Net work per kg h1 − h2 − wpump
hRankine =
(ii) =
Heat added per kg h1 − h f 3 − wpump

3195.6 − 2257.75 − 5.0857


= = 0.3229 or 32.29%
3195.6 − 301.48 − 5.0857

T 400°C 1

50 bar
4

M
0.34 bar
3 2 2′
s

Figure 4.42  Rankine cycle on T-s plot

M04_THERMAL ENGINEERING_SE_XXXX_CH04.indd 205 6/15/2018 2:36:05 PM


206 Chapter 4

150 × 3600
(iii) Steam rate or specific consumption = = 3.86 kg/kWh
139913.7
(b) For engine
(i) Power developed = Steam per sec × (h1 - h2) = 150 × (3195.6 - 2257.756)
= 140676.6 kW
h −h 3195.6 − 2257.756
(ii) hRankine engine = 1 2 = = 0.3240 or 32.4%
h1 − h f 3 3195.6 − 301.48
(iii) Steam rate or specific steam consumption
150 × 3600
= = 3.838 kJ/kWh
140676.6
(c) Actual engine
Brake specific steam consumption = 4.75 kg/kWh
3600
hbrake thermal = = 0.2619 or 26.19%
4.75 × (3195.6 − 301.48)
Ideal steam consumption 3.838
Expansion efficiency = = = 0.808 or 80.8%.
Actual steam consumption 4.75
Actual enthalpy drop = 0.808 × Isentropic enthalpy drop
= 0.808 (3195.6 - 2257.756) = 757.8 kJ/kg
Thus, h′2 = h1 - 757.8 = 3195.6 - 757.8 = 2437.8 kJ/kg
Also, h′2 = hf2 + xhfg2
2437.8 − 301.48
Thus, x = = 0.9173 or 91.73%
2328.9
Example 4.27
In a steam power plant operating on Rankine cycle, the steam enters the turbine at 70 bar and 550°C
with a velocity of 30 m/s. It discharges to the condenser at 0.20 bar with a velocity of 90 m/s. If the
steam flow rate is 35 kg/s find the thermal efficiency and the net power produced. Neglect pump
work.[IES, 1992]
Solution
Given that p1 = 70 bar, t1 = 550°C, c1 = 30 m/s, p2 = 0.20 bar, c2 = 90 m/s, m. s = 35 kg/s
The simple Rankine cycle is shown in Fig. 4.43(a) and T-s diagram in Fig. 4.43(b).
T
2 1
T 70 bar
1 4

Boiler
Condenser
3
3 0.2 bar 2
Pump s
(a) (b)

Figure 4.43 Rankine cycle: (a) Schematic diagram, (b) T-s diagram

M04_THERMAL ENGINEERING_SE_XXXX_CH04.indd 206 6/15/2018 2:36:08 PM


Steam Power Cycles 207

From steam tables, we have


At p1 = 70 bar and t1 = 550°C, h1 = 3530.9 kJ/kg, s1 = 6.9486 kJ/kg.K
At p2 = 0.2 bar, hf  2 = 251384 kJ/kg, hfg2 = 2358.3 kJ/kg, sf  2 = 0.8319 kJ/kg.K, sfg2 = 7.0766 kJ/kg.K
Now s1 = s2 = sf  2 + x2 sfg2
or 6.9486 = 0.8319 + x2 × 7.0766
or x2 = 0.8644
∴ h2 = hf  2 + x2 hfg2
  = 251.38 + 0.8644 × 2358.3 = 2289.89 kJ/kg
Net power produced = m. s (h1 - h2) = 35 (3530.9 - 2289.89) = 43435.35 kW
h1 − h2
Thermal efficiency, hth =
h1 − h f 3
Now hf  3 = hf  2
3530.9 − 2289.89
∴ hth = = 0.3784 or 37.84%
3530.9 − 251.38

Example 4.28
The following data refer to a steam turbine power plant employing one stage of regenerative feed
heating:
State of steam entering HP stage: 10 MPa, 600°C
State of steam entering LP stage: 2 MPa, 400°C
State of steam at condenser : 0.01 MPa, x = 0.9
The correct amount of steam is bled for feed heating at exit the HP stage. Calculate the mass of
steam bled per kg of steam passing through the HP stage and the amount of heat supplied in the
boiler per second for an output of 10 MW. Neglect pump work. [IES, 1993]

Solution
The schematic diagram and T-s diagram are shown in Fig.4.44(a) and (b).
H.P. stage: p1 = 10 MPa, t1 = 600°C
L.P. stage: p2 = 2 MPa, t2 = 400°C
Condenser: p3 = 0.01 MPa, x3 = 0.9
Wout = 10 MW
From steam tables, we have (Fig. 4.44(a))
h1 = 3625.3 kJ/kg
h2 = 3247.6 kJ/kg
h3 = hf  3 + x3 hfg3
= 191.81 + 0.9 × 2392.8 = 2345.35 kJ/kg
Heat balance for the heater:
(1 - m1) (h5 - h4) = m1(h2 - h5)
From steam tables,
h5 = 908.77 kJ/kg at p2 = 2 MPa
h4 = 191.81 kJ/kg at p3 = 0.01 MPa
∴ (1 - m1)(908.77 - 191.81) = m1(3247.6 - 908.77)
or (1 - m1) × 716.96 = 2338.83 m1

M04_THERMAL ENGINEERING_SE_XXXX_CH04.indd 207 6/15/2018 2:36:10 PM


208 Chapter 4

1
T
1
Turbine Wout
1 kg
qm 6
Boiler 2 m1 1 kg
p1
b5
3 (1 − m) m1 kg
Heater Wp
6 25 2
qout p2
b4 (1 − m1) kg
5 Condenser p3
Wp
4 1
4 3
(1 − m1) kg

Wp Wp
2 1 s
Feed pump Heater pump
(a) (b)

Figure 4.44 Regenerative feedwater heating cycle:


(a) Schematic diagram, (b) T-s diagram

716.96
or m1 = = 0.235 kg/kg of steam
3055.79
Heat supplied in boiler,
qin = h1 - h5 = 3625.3 - 908.8 = 2716.5 kJ/kg
Work output from boiler,
Wout = (h1 - h2) + (1 - m1) (h2 - h3)
= (3625.3 - 3247.6) + (1 - 0.235) (3247.6 - 2345.35) = 1067.92 kJ/kg
Wout 1067.92
Cycle efficiency, h = = = 0.3931 or 39.31%
qin 2716.5
10, 000
Heat supplied in the boiler for 10 MW output = = 25,437 kW
0.3931
Example 4.29
A small power plant produces 25 kg/s steam at 3 MPa, 600°C in the boiler. It cools the condenser
with ocean water coming in at 12°C and returned at 15°C. Condenser exit is 45°C. Find (a) the net
power output and (b) the required mass flow rate of ocean water. [IAS 2012]
Solution
.
Given that ms = 25 kg/s, p1 = 3 MPa, t1 = 600°C, twi = 12°C, two = 15°C, T1 = 600 + 273 = 873 K,
T2 = 45 + 273 = 318 K
The T-s diagram is shown in Fig. 4.45.
(a) h1 = 3682 kJ/kg, hf  3 = 188.4 kJ/kg, p1 = 9.5934 kPa, s1 = 7.5084 kJ/kg.K
vf  3 = 0.00101 m3/kg, hfg3 = 2394.8 kJ/kg
sf  3 = 0.6386 kJ/kg.K, sfg3 = 7.5261 kJ/kg.K
Now s1 = s2 = sf  3 + x2 sfg3

M04_THERMAL ENGINEERING_SE_XXXX_CH04.indd 208 6/15/2018 2:36:12 PM


Steam Power Cycles 209

T, K

873 1
4
1′
3 MPa

m
2′
318
3 9.59 kPa 2

Figure 4.45 T-s diagram

or 7.5084 = 0.6386 + x2 × 7.5261


or x2 = 0.9128
h2 = hf 3 + x2 hfg3 = 188.42 + 0.9128 × 2394.8 = 2374.39 kJ/kg
Pump work, wp = vf 3 (p1 - p2) = 0.00101 (300 - 9.59) = 0.29 kJ/kg
Turbine work, wt = h1 - h2 = 3682.3 - 2374.39 = 1307.91 kJ/kg
Net work, wnet = wt - wp = 1307.91 - 0.29 = 1307.62 kJ/kg
.
Net power output = m s × wnet = 25 × 1307.62 = 32.69 MW
(b) Heat rejected constant pressure p2, qr = h2 - hf 3 = 2374.39 - 188.42 = 2185.97 kJ/kg
.
Let m w = mass of ocean water circulated
. .
m w cpw × ∆tw = m s × qr
.
or m w × 4.187 × (15 - 12) = 25 × 2185.97
.
or m w = 4350.7 kg/s

Summary for Quick Revision


1. The Rankine cycle is a vapour pressure cycle which operates on steam and is used for steam power plant. It
consists of a boiler, turbine, condenser, and a pump.
2. Net work, wnet = qa - qr = (h1 - hf M) - (h2 - hf 3) = (h1 - h2) - wp = wt - wp
Pump work, wp = hf M - hf 3 = vf 3 (pM - p3) × 102 kJ/kg
= vf 3 (p1 - p3) × 102 kJ/kg [ pM = p1]

where vf is in m3/kg and p is in bar.
3. Thermal efficiency of Rankine cycle considering pump work,
(h1 − h2 ) − w p
hr =
(h1 − h f 3 ) − w p
3600
Heat rate =
h th
4. Thermal efficiency of Rankine cycle neglecting pump work,
h −h
hr = 1 2
h1 − h f 3

M04_THERMAL ENGINEERING_SE_XXXX_CH04.indd 209 6/15/2018 2:36:14 PM


210 Chapter 4

1 × 3600
5. Specific steam consumption, SSC = kg/kWh
wt − w p
6. Overall heat rate, OHR = SSC × heat supplied per kg of throttle steam
1 × 3600 ( h1 − h fM )
=
( h1 − h2 ) − ( h fM − h f 3 )
1 × 3600 ( h1 − h f 3 )
= neglecting pump work
( h1 − h2 )
wp h fm − h f 3 hf 4 − hf 3
7. Back work ratio, BWR = = and work ratio, rw = 1 -
wt h1 − h2 h1 − h2
8. Increase of boiler pressure leads to increase in thermal efficiency of ideal Rankine cycle.
9. Decrease in condenser pressure leads to increase in thermal efficiency of ideal Rankine cycle.
10. Thermal efficiency of Rankine cycle can be improved by reheating and regeneration.
(h1 − h2 ) + (h3 − h4 ) − (h f 6 − h f 5 )
11. Efficiency of reheat cycle, hreheat =
(h1 − h f 6 ) + (h3 − h2 )
(h1 − h2 ) + (h3 − h4 )
= , neglecting pump work
(h1 − h f 6 ) + (h3 − h2 )
12. In the reheat cycle, steam is extracted at a suitable point after expanding in high pressure turbine and is
reheated with the help of flue gases in the boiler furnace.
13. Regeneration is a method to heat the feed water from the hot well of condenser reversibly by interchange of
heat with the system to improve the cycle efficiency.
14. In open heaters, the bled steam is allowed to mix with feed water.
15. In closed heaters, the bled steam is not allowed to mix with feed water. The feed water flows through the
tubes in the heater and bled steam condenses on the outside of tubes.
16. In a binary vapour cycle, two cycles with different working fluids are coupled in a series. Mercury is used in
the topping cycle and steam in the bottoming cycle.
17. In co-generation, some part of the expanded steam in the HP turbine is bled to generate electric power.

Multiple-choice Questions
1. The efficiency of Rankine cycle can be increased by 3. In the Rankine cycle, with the maximum steam
(a) decreasing initial steam pressure and temper- temperature being fixed from metallurgical con-
ature sideration, as the boiler pressure increases,
(b) increasing exhaust pressure (a) the condenser load will increase
(c) increasing expansion ratio (b) the quality of turbine exhaust will decrease
(d) increasing regenerative heaters (c) the quality of turbine exhaust will increase
2. Choose the wrong statement: (d) the quality of turbine exhaust will remain
(a) bleeding increases the thermodynamic effi- unchanged
ciency of turbine 4. Rankine cycle comprises
(b) bleeding increases the net power developed (a) two isentropic and two reversible constant
by turbine volume processes
(c) bleeding decreases the net power developed (b) two isentropic and two reversible constant
by turbine pressure processes
(d) boiler is supplied with hot water due to
bleeding.

M04_THERMAL ENGINEERING_SE_XXXX_CH04.indd 210 6/15/2018 2:36:17 PM


Steam Power Cycles 211

(c) two reversible isothermal and two reversible III. Improved quality of exhaust steam
constant pressure processes    IV. Reduced condenser load
(d) two reversible constant pressure and two Select the correct answer using the code given
reversible constant volume processes. below:
5. Regeneration effect on Rankine cycle (a) I and III only
(a) decreases thermal efficiency (b) II only
(b) increases thermal efficiency (c) II and IV
(c) has no effect on thermal efficiency (d) I, II and III
(d) increases or decreases thermal efficiency 12. In a regenerative feed heating cycle, the economic
depending on the point extraction of steam. number of the stages of regeneration
6. Superheating of steam before expansion in a (a) increases as the initial pressure and tempera-
Rankine cycle ture increase
(a) decreases thermal efficiency (b) decreases as the initial pressure and tempera-
(b) has no effect on thermal efficiency ture increase
(c) increases thermal efficiency (c) is independent of the intitial pressure and
(d) may increase or decrease thermal efficiency. temperature
(d) depends only on the condenser pressure
7. Reheating of steam
(a) increases thermal efficiency 13. Consider the following statements:
(b) increases work output of turbine The reheat cycle helps to reduce
(c) decreases work output of turbine I.    fuel consumption
(d) increases thermal efficiency but decreases II.    steam flow
work output. III. the condenser size
Which of these statements are correct?
8. In a binary vapour cycle, (a) I and II
(a) mercury is used in the bottoming cycle (b) I and III
(b) steam is used in the topping cycle (c) II and III
(c) mercury is used in the topping cycle (d) I, II, and III
(d) either mercury or steam may be used in the
topping cycle. 14. The main advantage of a reheat Rankine cycle is
(a) reduced moisture content in LP side of tur-
9. In a steam power plant, feed water heater is a heat bine
exchanger to preheat feed water by (b) increased efficiency
(a) live steam from steam generator (c) reduced load on condenser
(b) hot flue gases coming out of the boiler furnace (d) reduced load on pump
(c) hot air from air preheater
(d) extracting steam from turbine 15. Consider the following statement pertaining to
the features of regenerative steam cycle plant as
10. Consider the following statements regarding
compared to a non-regenerative plant:
effects of reheating of steam in a steam turbine: I. It increases the cycle efficiency.
I. It increases the specific output of the turbine. II. It requires a bigger boiler.
II. It decreases cycle efficiency. III. It requires a smaller condenser.
III. It increases blade erosion. Which of the statements are correct?
IV. It improves the quality of exit steam. (a) I, II, and III
Which of these statements are correct? (b) I and II
(a) I and II (b) II and III (c) II and III
(c) III and IV (d) I and IV (d) I and III
11. In a steam power plant, what is the outcome of 16. Which of the following statements is not correct
regenerative feed water heating? for regenerative steam cycle?
I. Increase in specific output (a) It increase thermodynamic efficiency.
II. Increase in cycle efficiency (b) It reduces boiler capacity for a given output.

M04_THERMAL ENGINEERING_SE_XXXX_CH04.indd 211 6/15/2018 2:36:17 PM


212 Chapter 4

(c) It reduces temperature stresses in the boiler 19. Consider the following statements:
due to hotter feed. The purpose of reheating the steam in a steam tur-
(d) The efficiency increases with increased num- bine power plant is to
ber of feed heaters. I. increase specific output
17. Consider the following for a steam turbine power II. increase turbine efficiency
plant: III. reduce turbine speed
I. Reduction in blade erosion IV. reduce specific steam consumption
II. Increase in turbine speed Which of the statements are correct?
III. Increase in specific output (a) II and IV
IV. Increase in cycle efficiency (b) I and III
Which of these occurs due to reheating of steam? (c) I, II, and IV
(a) Only I (d) I, III, and IV
(b) I and II 20. In the bottoming cycle of cogeneration, low-grade
(c) I, III, and IV waste heat is used for
(d) II and III (a) processing
18. When is the greatest economy obtained in a (b) power generation
regenerative feed heating cycle? (c) feed water heating
(a) Steam is extracted from only one suitable (d) None of these
point of a steam turbine. 21. In steam and other vapour cycles, the process of
(b) Steam is extracted only from the last stage of removing non-condensables is called
steam turbine. (a) scavenging process
(c) Steam is extracted only from the first stage of (b) deaeration process
a steam turbine. (c) exhaust process
(d) Steam is extracted from several places in dif- (d) condensation process
ferent stages of steam turbines.

Review Questions
1. Draw the T-s diagram for a simple Rankine cycle and write down the expression for its efficiency.
2. What is the effect of boiler pressure and condenser pressure on Rankine cycle efficiency?
3. What are the various methods of improving thermal efficiency of Rankine cycle?
4. Draw the reheat Rankine cycle with superheated steam and write down the expression for its efficiency.
5. Explain regeneration. What are its advantages?
6. What is the difference between open heater and closed heater?
7. What are the properties of an ideal working fluid for the Rankine cycle?
8. What is a binary vapour cycle?
9. What is co-generation?
10. List the advantages of co-generation.
11. What is the difference between bleeding and extraction of steam?

Exercises
4.1 A simple Rankine cycle works between 30 bar and 0.04 bar pressures, the initial condition of steam being
dry saturated. Caculate the cycle efficiency, work ratio, and specific steam consumption.
[Ans. 35%, 0.997, 3.84 kg/kWh]
4.2 In a Rankine cycle, the steam at inlet to turbine is saturated at pressure of 30 bar and the exhaust pressure is
0.25 bar. Calculate (a) the pump work, (b) the turbine work, (c) the cycle efficiency, and (d) the condenser
heat flow per kg of steam. [Ans. 3 kJ, 741 kJ, 29.2%, 1790 kJ]

M04_THERMAL ENGINEERING_SE_XXXX_CH04.indd 212 6/15/2018 2:36:18 PM


Steam Power Cycles 213

  4.3  Steam at 90 bar, 480°C is supplied to a steam turbine. The steam is reheated to its original temperature
by passing the steam through a reheater at 12 bar. The expansion after reheating takes place to condenser
pressure of 0.07 bar. Calculate the efficiency of the reheat cycle and work output per kg of steam.
[Ans. 42.2%, 1611 kJ/kg]
  4.4  A reheat cycle operating between 30 bar and 0.04 bar has a superheat and reheat temperature of 450°C. The
first expansion takes place till the steam is dry saturated and then reheat is given. Determine the ideal cycle
efficiency neglecting feed pump work. [Ans. 38.7%]
  4.5  In a regenerative cycle, the steam pressure at turbine inlet is 30 bar and the exhaust is at 0.04 bar. The steam
is initially saturated. Enough steam is bled off at the optimum pressure to heat the feed water. Neglecting
pump work, determine the cycle efficiency. [Ans. 37.3%]
  4.6   Steam is supplied to a turbine at pressure of 32 bar, 420°C. It expands isentropically to a pressure of 0.08
bar. Determine the thermal efficiency of the cycle.
        If the steam is reheated at 6 bar to 400°C and then expanded isentropically to 0.08 bar, then find the
thermal efficiency. [Ans. 35.4%, 36.6%]
  4.7  Steam of 28 bar and 50°C superheat is passed through a turbine and expanded to a pressure where the steam
is dry saturated. It is then reheated at constant pressure to its original temperature and then expanded to
the condenser pressure of 0.2 bar, the expansion being isentropic. Calculate work done per kg of steam and
thermal efficiency with and without reheat. [Ans. 810 kJ/kg, 880 kJ/kg, 30%, 30.3%]
  4.8  In a binary vapour cycle, the pressure in the mercury boiler is 15.4 bar and in the mercury condenser is
0.2 bar. The pressure limits for steam cycle are 54 bar and 0.08 bar. If both mercury and steam enter their
respective turbines in a dry saturated state, find mass of mercury in kg per kg of steam and thermal effi-
ciency of the cycle. [Ans. 11.4 kg, 57.61%]
  4.9  A binary vapour cycle consists of a mercury operating between 14 bar and 0.1 bar and steam cycle operat-
ing between 30 bar, 450°C and 0.04 bar. Calculate the ideal cycle efficiency of the cycle. [Ans. 51.78%]
4.10  Steam is supplied to a regenerative cycle as dry and saturated at 32 bar the condenser pressure is 0.05 bar. If
steam is bled at 2.8 bar, find the quality of steam bled, the cycle efficiency, and the specific steam consump-
tion.[Ans. 0.718, 35.27%, 4.342 kg/kWh]

ANSWERS TO MULTIPLE-CHOICE QUESTIONS


1. (a) 2. (c) 3. (c) 4. (b) 5. (b)
6. (c) 7. (b) 8. (c) 9. (d) 10. (d)
11. (c) 12. (a) 13. (a) 14. (a) 15. (d)
16. (b) 17. (d) 18. (c) 19. (c) 20. (b)
21. (b)

M04_THERMAL ENGINEERING_SE_XXXX_CH04.indd 213 6/15/2018 2:36:18 PM


Chapter 5

Steam Engines

5.1 ❐ INTRODUCTION
A steam engine uses steam as the working medium. It is the earliest prime mover developed to convert
thermal energy into mechanical energy. It is called a prime mover because it is used to drive other
devices like locomotives, compressors, lathe, shear presses, etc. On account of their low efficiency
steam engines have become obsolete. However, they are still being used in locomotives and some
industries.
A steam engine is called an external combustion engine as the fuel is burnt in the boiler and the
steam raised is used to reciprocate the engine piston for generating power.

5.2 ❐ CLASSIFICATION OF STEAM ENGINES


A steam engine is of the reciprocating type, in which the piston moves to-and-fro in the cylinder due
to the force applied by the expansion of steam. The reciprocating steam engines may be classified as
follows:
1. According to class of service:
(a)   Stationary
(b)  Marine
(c)  Locomotive
(d)  Pumping or hoisting
2. According to speed:
(a)   Low speed: below 125 rpm
(b)   Medium speed: 125 to 300 rpm
(c)   High speed: above 300 rpm
  Note that the speed of the engine means speed of the crankshaft.
3. According to arrangement of cylinders:
(a)   Horizontal
(b)   Inclined
(c)    Vertical

M05_THERMAL ENGINEERING_SE_XXXX_CH05.indd 214 6/15/2018 3:08:20 PM


Steam Engines 215

4. According to type of valve design:


(a) Simple plate valve
(b) Balance plate valve
(c) Piston valve
(d) Riding cut-off valve
(e) Poppet valve
(f) Corliss valve
5. According to method of governing:
(a) Manual or automatic governing
(b) throttle or cut-off governing
(c) Centrifugal or inertia type governor
6. According to type of exhaust:
(a) Condensing type
(b) Non-condensing type
7. According to steam condition:
(a) High or low pressure
(b) Saturated or superheated
8. According to the action of steam upon the piston:
(a) Single acting
(b) Double acting
9. According to the nature of expansion:
(a) Expansive engines
(b) Non-expansive engines
10. According to the range of expansion of steam:
(a) Simple steam engine
(b) Compound steam engine

5.3 ❐ CONSTRUCTIONAL FEATURES OF A STEAM ENGINE


A schematic line diagram of a vertical steam engine is shown in Fig. 5.1
The parts of a steam engine may be broadly classified as:
1. Stationary parts
(a) Engine bedplate
(b) Engine frame
(c) Cylinder
(d) Steam chest
(e) Steam jacket
(f) Stuffing box
(g) Cross-head guides
(h) Main bearings
2. Moving parts
(a) Piston
(b) Piston rod
(c) Cross-head

M05_THERMAL ENGINEERING_SE_XXXX_CH05.indd 215 6/15/2018 3:08:20 PM


216 Chapter 5

Steam
inlet pipe
Stuffing
box

Steam supply Flywheel


D-slide valve
Valve rod Valve rod Ecentric rod
Eccentric
Steam chest

Inlet port Balance


Crank web
Small end web weight
Exhaust Packing Guides
port Crank pin

Piston

Shoes Big end


Piston Gland Cross Bearings
Connecting
Piston rod head
rod
rings Cylinder cover
Cylinder
cover Cylinder with stuffing box
Pulley

Figure 5.1  Schematic diagram of a steam engine

(d) Connecting rod


(e) Crank and crank shaft
(f) D-slide valve
(g) Valve rod
(h) Eccentric and eccentric rod
(i) Flywheel
(j) Governor

5.3.1  Steam Engine Parts


The various parts are briefly described below:
Frame: It is a heavy casting made of cast iron and supports cross-head guides and main bearing. It
generally rests on the engine foundation.
Cylinder: It is also a casting to the frame in which the piston reciprocates.
Its both ends are closed and made steam tight. The stuffing box is fixed in one end through which
piston rod reciprocates.
Steam chest: It is compartment in a steam engine through which steam is delivered from the boiler
to a cylinder. It is connected to the cylinder through the valve passage known as ports. It contains the
D-slide valve. The steam is supplied alternately to the cylinder through the ports and exhausted to the
condenser. The same port serves as inlet port and exhaust port in double acting engines.
D-slide valve: It connects the cylinder to the steam chest and to the exhaust side through the ports
at the correct crank positions. The valve is operated by an eccentric.

M05_THERMAL ENGINEERING_SE_XXXX_CH05.indd 216 6/15/2018 3:08:21 PM


Steam Engines 217

Piston: It is made of cast iron. The steam pressure exerts force on it on both sides to reciprocate the
piston rod. Piston rings are provided on the piston for steam tightness.
Piston Rod: One end of piston rod is connected to the piston and the other end to the cross-head.
It transmits the force from the piston to the cross-head.
Stuffing Box and Gland: It is placed at the point where the piston rod passes through the cylinder
cover. It prevents the leakage of steam from the cylinder to atmosphere, at the same time allowing the
piston rod to reciprocate freely.
Cross-Head: It connects the piston rod and connecting rod. It guides the piston rod along a straight
line. The thrust on the cross-head is taken of by the cross-head guides.
Connecting Rod: One end of it is connected to the cross-head by a gudgeon pin and the other
end is connected to the crank. It transmits force from the cross-head to the crank.It has oscillatory
motion.
Crankshaft: The crank converts the reciprocating motion to rotary motion of the crankshaft.The
crank shaft is supported on main bearings and carries the flywheel and eccentric
Eccentric: It converts rotary motion of crankshaft into reciprocating motion of D slide valve.
Valve rod and Eccentric rod: They connect the eccentric to D-slide valve.
Flywheel: It is connected to the crankshaft and helps to maintain a constant angular speed of the
engine by storing excess energy during idle stroke of the device being operated.
Governor: It helps in maintaining uniform speed of the engine by regulating supply of stream to
the cylinder.

5.4 ❐ TERMINOLOGY USED IN STEAM ENGINE


A line diagram of steam engine mechanism is shown in Fig. 5.2.
Cylinder Bore, D: It is the inside diameter of the cylinder
Piston stroke, L: It is the distance travelled by the piston from the cover end of the cylinder to the
crank end of cylinder. L = 2r.
Single Acting: Steam is supplied on one side of piston only
Double acting: Steam is supplied on both sides of piston.
Dead centres: The position of piston within the cylinder, when the crank and connecting rod are
in the same straight line.
Crank radius, r: It is the distance between the centre of crankshaft and centre of crank pin

Crank
Connecting
Cylinder
Piston rod
Piston
rod
D

Cross head
LC L
Crank shaft
IDC ODC

Figure 5.2  Line diagram of a steam engine

M05_THERMAL ENGINEERING_SE_XXXX_CH05.indd 217 6/15/2018 3:08:21 PM


218 Chapter 5

Swept volume, VS: It is the volume swept by the piston in one stroke

p 2
VS = D L
4
Mechanical Clearance, LC: The mechanical clearance is the distance between the cylinder cover
and inner dead centre position of the piston. It is specified as percentage of stroke length.
Volumetric Clearance, VC: It is the volume between the cylinder cover and the
piston at the inner dead centre position of piston.

p 2
VC = D LC
4
Engine speed, N: It is measured in terms of revolutions of crankshaft per minute.
Piston speed: It is linear speed of the piston.
2 LN
Piston speed = m/s
60
Compression Ratio,a : It is the fraction of stroke volume completed at the start of compression.
V − Vc
a = s
Vs
Expansion Ratio, r: It is the ratio of swept volume to col nine at cut-off of steam.
1
Cut off Ratio, q : It is the ratio of volume at cut-off steam to the swept volume r =
r
Mean Effective Pressure, Pm: It defined as that hypothetical constant pressure which will produce
same work for the same piston displacement with the variable conditions of temperature pressure and
volume.
work output
Pm = (5.1a)
swept volume
card area
= × spring constant (5.1b)
card length
Where card area = area of indicator diagram, card length = length of stroke in indicator diagram.
Eccentric Throw: It is the distance between the centre of eccentric and centre of crankshaft.
Valve Travel: It is the maximum distance travelled by the valve along one direction. It is equal to
twice the eccentric throw.
Back pressure, pb: The pressure of exhaust steam acting on the other side of the piston is known
as back pressure.

5.5 ❐ WORKING OF A STEAM ENGINE


The working of a double acting steam engine is a shown in Fig. 5 3. As the piston reaches the inner dead
centre (IDC) position A, the D-slide valve clears the inlet port C and admits the steam into the cylinder.
This high pressure steam pushes the piston on the forward direction and performs work by rotating
the crank and eccentric. The steam continues to enter into the cylinder maintaining a constant pressure
inside. With the further rotation of crank and eccentric, D-slide valve is moved backwards and closes

M05_THERMAL ENGINEERING_SE_XXXX_CH05.indd 218 6/15/2018 3:08:24 PM


Steam Engines 219

D-slide valve
Valve rod
Steam chest

Connecting rod
E
Crank
C D
Cylinder

Piston Cross head


A B
Stroke
IDC ODC

Figure 5.3  Working of a steam engine

the inlet port C. The steam supply to the cylinder is stopped and this is known as cut-off of steam. After
the cut-off the steam in the cylinder expands pushing the piston in the forward direction till the piston
reaches the outer dead centre (ODC) position B. Just before the ODC position, the valve connects
the cylinder through the port to exhaust E. Therefore the pressure on this side of cylinder falls to the
atmospheric pressure or condenser pressure. After the ODC position the piston reverses its direction
of motion and the return stroke starts. The steam is admitted through port D and exhausted on the IDC
side. Just before the piston reaches the IDC position, the port D closes and low pressure steam in the
cylinder is compressed and acts as a cushion to the piston at the end of the stroke. As the piston reaches
the IDC position, the D-slide valve opens the port C and the cycle is repeated. When admission and
expansion take place on head end side, exhaust and compression takes place on the crank end side.

5.6 ❐ RANKINE CYCLE


Rankine cycle is the theoretical cycle on which the steam engine works. The Rankine cycle is shown
in Fig. 5.4.
The p-v, T-s and h-s diagram for the Ranking cycle are shown in Fig. 5.5.

1
Steam
we
engine
q1 Boiler
2
Condenser

4 Cooling water out


q2
Cooling water in

Feed pump
wp 3

Figure 5.4  Rankine cycle

M05_THERMAL ENGINEERING_SE_XXXX_CH05.indd 219 6/15/2018 3:08:25 PM


220 Chapter 5

p T
1″
4 1′ 1 1″ Superheated steam p1 1′ 1
p1
Dry saturated steam

Wet steam
4

p2 p2
3 2′ 2 2″
3 2′ 2 2″
v s
(a) (b)

h
p1

1″
1
1′ p2

4
2″
2
2′
3

s
(c)

Figure 5.5 Rankine cycle in various ordinates: (a) p-v diagram,


(b) T-s diagram, (c) h-s diagram
The various processes of Rankine cycle are:
Process 1-2: Reversible adiabatic expansion in the engine.
Process 2-3: Reversible constant pressure heat transfer in the condenser.
Process 3-4: Reversible adiabatic pumping process in the feed pump.
Process 4-1: Reversible constant pressure heat transfer in the boiler.
The steam on entry into the engine can be wet, dry saturated or superheated. Considering 1 kg of
steam and applying steady flow energy equation to boiler, engine, condenser and pump, we have:
Boiler:
hf  4 + q1 = h1

q1 = h1 - hf 4
Engine:
h1 = we + h2

M05_THERMAL ENGINEERING_SE_XXXX_CH05.indd 220 6/15/2018 3:08:26 PM


Steam Engines 221

Engine work,     we = h1 - h2


Condenser:

h2 = q2 + hf 3
q2 = h2 - hf 3
Feed pump:
hf 3 + wp = hf4
Pump work,     wp = hf 4 - hf 3
Net work, wnet = we - wp
net work w
Rankine cycle efficiency,    h Rankine = = net
Heat input q1

( h1 − h2 ) − ( h f 4 − h f 3 )
=  (5.3a)
( h1 − h4 )
The feed pump handles liquid water which is incompressible. This implies that with the increase in
pressure its density or specific volume undergoes a small change. For reversible adiabatic compres-
sion, we have
T ds = ah - v dp
ds = 0
dh = v dp
or         ∆h = v ∆p  [∵ ∆v = 0]
hf 4 - hf 3 = v3 (p1 - p2) ×105 J/kg
or    
Now,      hf4 - hf 3 = wp << we, and can be neglected
h1 − h2
∴ h Rankine =
  (5.3b)
h1 − h f 4

5.7 ❐ MODIFIED RANKINE CYCLE


In modified Rankine cycle to reduce friction the stroke length is reduced.
Figure 5.6(a) and 5.6(b) show the modified Rankine cycle on p-v and T-s diagrams respectively, by
neglecting pump work.
It may be observed that p-v diagram is very narrow at the toe. i.e., point 3′. The work obtainable
near to 3′ is very small which is too inadequate to overcome friction due to reciprocating parts of
engine. Therefore the adiabatic process 2-3′ is terminated at point 3. The pressure drop decreases
suddenly whilst the volume remains constant. This operation is represented by the line 3-4. By doing
so the stroke length is reduced. The loss of work is represented by the hatched area 3-3′-4-3, which is
negligibly small.
The work done during the modified Rankine cycle can be calculated as follows:
Let p1, v2, u2, h2 represents the initial conditions at point 2.
p2, v3, u3, h3 corresponds to condition of steam at point 3.

M05_THERMAL ENGINEERING_SE_XXXX_CH05.indd 221 6/15/2018 3:08:28 PM


222 Chapter 5

p1 1 2 T
1
2

3
3 3′
p2 5 4
p3 5 3′
b a4 v s
O v2 v3
(a) (b)

Figure 5.6  Modified Rankine cycle: (a) p-v diagram, (b) T-s diagram

p3, h4 correspond to condition of steam at point 4.


Work done during the cycle per kg of steam = area 1 – 2 −3 – 4 – 5 – 1
= area 1 – 2 – b – 0 – 1 + area 2 – 3 – a – b – 2 area – 0 – b – a – 4 – 5 – 0
= p1 v2 + (u2 – u3) – p3 v3
Heat supplied, qs = h2 – hf4
Work done p v + (uu2 – u3 ) – p3 v3
Modified Rankine efficiency = = 1 2 (5.4)
Heat supplied h2 − h f 4

5.8 ❐ H
 YPOTHETICAL OR THEORETICAL INDICATOR
DIAGRAM
The indicator diagram is a representation of the variations of pressure and volume of steam inside the
cylinder on p-V diagram for one complete cycle of operation. The theoretical p-v diagram for a single
acting steam engine is shown in Fig. 5.7 without and with clearance.
The sequence of operations is as follows:
Process 1-2: The steam is supplied to cylinder at constant pressure p1. The steam is cut-off at point 2.
Process 2-3: The steam expands in the cylinder till the piston reaches the ODC position. The
expansion process is hyperbolic isothermal, i.e. pv = const.
Process 3-4: The pressure falls from point 3 to point 4 at constant volume instantaneously due to
the opening of the exhaust. The point 3 is called the point of release as the pressure is allowed to fall
suddenly to the back pressure pb.
Process 4-5: It represents the exhaust of used steam at constant pressure
Process 5-1: It represents steam admission to cylinder at constant volume. The pressure suddenly
rises form pb to p1.
The working cycle is completed within two strokes of piston or one revolution of crank.

M05_THERMAL ENGINEERING_SE_XXXX_CH05.indd 222 6/15/2018 3:08:29 PM


Steam Engines 223

p p
1 p1 2
p1 2 Point of
1 cut-off
pv = const
pv = const

3
Point of
3 release pb
pb 5 4
5
4

v v1 v2 v
(a) (b)

Figure 5.7 Theoretical indicator diagram for a steam engine: (a) Without clearance,
(b) With clearance

5.9 ❐ ACTUAL INDICATOR DIAGRAM


Indicator diagram is simply a graph between pressure of steam in a cylinder against the steam volume.
The following assumptions are made in drawing the theoretical indicator diagram:
1. Clearance is neglected.
2. Steam supplied to cylinder is at constant pressure.
3. Ports are opened and closed instantaneously.
4. There is no drop in pressure due to condensation.
5. Expansion of steam in cylinder follows the hyperbolic law.
The theoretical and actual indicator diagrams are shown in Fig. 5.8. The actual indicator diagram is
found to be different from the theoretical diagram due to following reason:
1. A definite clearance is necessary to prevent the piston striking the cylinder head. Provide cushion-
ing effect to piston and allow small quantity of water to be collected in the cylinder due to conden-
sation of steam. Therefore the actual supply of steam starts from point ‘a’ instead of point A. The
admission pressure is also lower due to condensation of Steam on the cooler cylinder walls and
throttling effect during flow of steam through the inlet port. Therefore, the supply steam pressure
is represented by ab instead AB.
2. The instantaneous cut-off is not possible as it takes place during few degrees of crank rotation.
Therefore, the diagram gets rounded up near point b.
3. The expansion of steam in the cylinder is not perfectly hyperbolic and is represented by some
other curve bc instead of BC. This is due to condensation of steam during expansion.
4. The exhaust opens before the piston reaches ODC position (say point C) as instantaneous drop
in pressure is not possible. Therefore, fall in pressure starts from C and actual pressure drop is
represented by ‘cd’.
5. The exhaust pressure is slightly above the condenser or back pressure as the steam has to be
forced out from cylinder. The actual exhaust pressure is represented by ‘de’.
6. The supply of exhaust steam to exhaust port is stopped at point ‘c’ by closing the port before the
piston reaches IDC positions. The entrapped steam is compressed during the remaining stroke

M05_THERMAL ENGINEERING_SE_XXXX_CH05.indd 223 6/15/2018 3:08:30 PM


224 Chapter 5

p
B
A
a Actual diagram
b
Theoretical
diagram

C
f
c
e
d
E
D
v

Figure 5.8  Theoretical and actual indicator diagrams

and provides a cushioning effect. This is represented by ‘ef ’. The inlet port opens just before the
piston reaches the IDC position and the pressure inside the cylinder increases.
The area between the theoretical and actual indicator diagrams represents the lost work. The ratio
of the area of the actual indicator diagram to the area of theoretical indicator diagram is called the
diagram factor.
Area of actual indicator diagram
Diagram factor, K d =  (5.5)
Area of theoretical indicator diagram

5.10 ❐ MEAN EFFECTIVE PRESSURE


The mean effective pressure is a quantity related to the operation of an internal combustion engine. It
is a valuable measure of an engine’s capacity to do work that is independent of engine displacement.

5.10.1  Without Clearance


The theoretical p-v diagram for a single acting steam engine without clearance is shown in Fig. 5.9(a).
It can be obtained by drawing a rectangle of area equal to the area of theoretical indicator diagram and
base equal to the stroke volume, as shown in Fig 5.9(b).
Area under p-v diagram gives the work done per cycle by an engine, Work done per cycle w = (area
l−2−2′−0−l) + (area 2−3 −3′−2′−2) − area (5−4−4′−0−5)
v3
ln − pb v4 = p1v2 + p1v2
v2
= p1v2 (1 + ln r) − pb v4
where p1 = steam admission pressure
p2 = release or cut-off pressure

M05_THERMAL ENGINEERING_SE_XXXX_CH05.indd 224 6/15/2018 3:08:31 PM


Steam Engines 225

1 2 p
p1

p3 3

pb 4
5 pm
2′
0 v v
3′, 4′ vs
vs

(a) (b)

Figure 5.9  p-v diagram without clearance

pb = back pressure
v
  r = 3 = expansion ratio
v2
 vs = v3 = swept volume
v2 = cut-off volume
v4 = v3, therefore
w p1
Now = (1 + ln r ) − pb
vs r
Work done per cycle as represented by Fig. 5.9(b) is given by
w = pm . vs
w
or      = pm
vs
From the above two equations, we get
p
pm = 1 (1 + ln r ) − pb (5.6)
r

5.10.2  With Clearance


The p-v diagram for a single acting steam engine with clearance is shown in Fig. 5.10(a). Work done
per cycle is given by,
w = area 1-2-3-4-5 -1
= (area 1-2-2′-5′-1) + (area 2-3-3′-2′-2) - (area 5-4-3′-5′-5)
v
= p1 ( v2 − v1 ) + p1v2 ln 3 − pb ( v3 − v1 ) 
v2
   vs = v3 - v4 = v3 - vc

M05_THERMAL ENGINEERING_SE_XXXX_CH05.indd 225 6/15/2018 3:08:33 PM


226 Chapter 5

vc vc
c = =
vs v3 − vc
Clearance ratio, v3 = vc + vs = c vs + vs = (1 + c) vs
vs
r=
Expansion ratio,   
v2 − vc
1 v2 − vc
Cut-off ratio, r = =
r vs
vs vs  1
    v2 = + vc = + cvs = vs  c + 

r r r
Substituting the values of vc, v2 and v3, we get
 
  1   1  v (1 + c) 
w = p1  vs  c +  − cvs  + p1vs  c +  ln  s  − pb vs
  r   r   1 
 vs  c + r  
 
 
 1   1   (1 + c ) 
= p1vs c + − c  + p1vs c +  ln   − pb vs
 r   r    c + 1 
  
r  

 
p1vs  1   (1 + c) 
= + p1vs c +  ln − pb vs
r  r    c + 1 
  
r  

1 2
p1
p1

3 pm
pb 4
5
pb
5′ 2′ 3′ 4′
0 v
v1 v2 v3 v
0
vc vs vc vs
(a) (b)

Figure 5.10  p-v diagram with clearance

M05_THERMAL ENGINEERING_SE_XXXX_CH05.indd 226 6/15/2018 3:08:36 PM


Steam Engines 227

Mean effective pressure,


 
w p1  1   (1 + c) 
pm = = + p1 c +  ln − pb
vs r  r    c + 1 
  
r  

  
1  +  
 1   (1 c )
   = p1  +  c +  ln    − pb (5.7)
r  r  1 
c+  

   r   
5.10.3  With Clearance and Compression
Figure 5.11 shows a hypothetical indicator diagram for a steam engine with clearance and compres-
sion. Clearance means the volume of gas left in the cylinder at the discharge end of the stroke. It
includes the space between the piston and cylinder head, the volume of the valves, valve pockets etc.
The various processes are:
Process 1-2: Steam admission at p1
Process 2-3: Hyperbolic expansion
Process 3-4: Steam released
Process 4-5: Exhaust of steam into condenser.
Process 5-6: Compression of remaining steam in cylinder
Process 6-1: entry of fresh from boiler into the cylinder rising the cylinder pressure suddenly to
boiler pressure p1.
Work done per cycle w, = area l-2-3-4-5-6-1
   = (area 1-2-2′-6′-1) + (area 2-3-3′-2′-2) - (area 5′-5-4-3′-5′)
− (area 6′-6-5-5′-6′)
V V
  = p1 (V2 − Vc ) + p1V2 ln 3 − pb (V3 − V5 ) − pbV5 ln 5
V2 Vc

p1 1 2
pv = const

pv = c
p6 6
p3 3
pb 4
5

0 v
6′ 5′ 2′ 3′ 4′
vc vs

Figure 5.11  p-v diagram with clearance and compression

M05_THERMAL ENGINEERING_SE_XXXX_CH05.indd 227 6/15/2018 3:08:38 PM


228 Chapter 5

 V  V
= p1V2 1 + ln 3  − p1Vc − pb (Vc + Vs − V5 ) − pbV5 ln 5
 V2  Vc
Vs
Let expansion ratio, r=
V2 − Vc
Swept volume,       Vs = V3 − Vc
V Vc
Clearance ratio,         c = c =
Vs V3 − Vc
V3 = Vc + Vs = cVs + Vs = (1 + c)Vs
V2 − Vc 1
Cut-off ratio      = =
Vs r
Vs Vs  1
V2 = + Vs = + cVs = Vs  c + 
r r  r
Vs − Vc Vs
Compression ratio, a = = −c
Vs Vs
Vs = (a + c)Vs
  
 1   (1 + c)    (a + c)Vs 
w = p1Vs  c +  1 + ln    − p1cVs − pb {cVs + Vs − (a + c)Vs } − pb (a + c)Vs ln  
 r   1 
 c +     cVs 
   r   

 
w  1   1  1 + c  aa ++ cc
= p1  c + − c + p1  c +  ln  − p
− p (c
(c ++ 1 −− a − c)
a − c) −− ppbb((a
a ++ cc))ln
1  b
ln  
Vs  r   r 
c+  cc 
 r
 
p1  1  1 + c   a + c  
  = + p1  c +  ln   − pb (1 − a ) − (a + c) ln  
r  r  1
c +    c 
 r
  
1  1  1 + c   a + c   (5.8)
pm = p1  +  c +  ln    − pb (1 − a ) + (a + c) ln  
 r  r  1
 c +    c 
  r  
5.10.4  With Clearance and Polytropic Expansion and Compression
A polytropic process can be expressed pvn = constant
The indicator diagram is shown in Fig. 5.12.
Work done per cycle.
w = area l-2-3-4-5-6-1
    = (area 1-2-2′-6′-1) + (area 2-3-3′-2′-2) - (area 5-4-3′-5′-5) - (area 6-5-5′-6′-6)
p V − p3V3 p V − p6V6
= p1 (V2 − Vc ) + 1 2 − pb (V3 − V5 ) + 5 5
n1 − 1 n2 − 1

M05_THERMAL ENGINEERING_SE_XXXX_CH05.indd 228 6/15/2018 3:08:43 PM


Steam Engines 229

p3V3n1 = p2V2n1
n1 n1
V  V 
p3 = p2  2  = p1  2 
 V3   V3 
and p5V5n2 = p6V6 n2
n2 n2
V  V 
p6 = p5  5  = pb  5 
 V6   Vc 
Vs
Let expansion ratio, r=
V2 − Vc

Swept volume, Vs = V3 − Vc
Vc Vc
Clearance ratio,    c = =
Vs V3 − Vc
   V3 = Vc + Vs = cVs + Vs = (1 + c)Vs
V2 − Vc 1
Cut-off ratio = =
Vs r
V6 = Vc , p2 = p1 , p5 = pb

 1
V2 =  c +  Vs
 r
V −V V
Compression ratio, a = 5 c = 5 −c
Vs Vs

p1 1 2

pvn1 = const

pvn2 = const

p6 6
p3 3

pb 4
5

6′ 5′ 2′ 3′
0 v
v6 v5 v2 v3
vc vs

Figure 5.12  p-v diagram with clearance and polytopic expansion and compression

M05_THERMAL ENGINEERING_SE_XXXX_CH05.indd 229 6/15/2018 3:08:49 PM


230 Chapter 5

V5 = (a + c)Vs
  1 1
n 
 c+ 
 p1  c + 1  Vs − p1  r  (11 + c)V 
  r  (1 + c)  s

   
 1 
w = p1  c + − c Vs +  
 r   n1 − 1 
 
 
 
 
  a + c 2
n 
 pb (a + c )Vs − pb   cVs 
 c 
− pb (1 + c − a − c)Vs −  
 n2 − 1 
 
 
  1 1 
n
  c+  
w p p1  1  r  pb  (a + c )n2 
pm = = 1+  c +  − − p (1 − a ) − (a + c ) − 
Vs r ( n1 − 1)  r  (1 + c) n1 −1  b
( n2 − 1)  c n2 −1 
   
 

n −1
 1   1 1 
c +  c +  n −1
 (a + c )   (a − c ) 2 
p1  r   r 

=
    + p1  1−   − pb (1 − a ) − pb  1−  (5.9)
r n1 − 1   1 + c    ( n2 − 1)   c 
    
   
 

5.11 ❐ POWER DEVELOPED AND EFFICIENCIES


5.11.1  Indicated Power
The power developed by the steam engine based on the indicator diagram is the indicated power.
Work done by the steam engine per cycle is given by,
w = Force × distance travelled by the piston
Work done per second is the indicated power
pm LA1N
IP = 1 kW for single acting. (5.10a)
60 × 103
pm LA1N pm2 LA2 N
  = 1 + kW for double acting. (5.10b)
60 × 103 60 × 103
p
where A1 = D 2 on cover end side.
4
p
    A2 = ( D 2 − d 2 ) on crank end side.
4
d = diameter of piston rod.

M05_THERMAL ENGINEERING_SE_XXXX_CH05.indd 230 6/15/2018 3:08:52 PM


Steam Engines 231

If area of piston rod is neglected, then


2 pm LAN
kW IP =
60 × 103
Ai = area of indicator diagram
Let  
Li = length of indicator diagram

Ai
The mean height of indicator diagram,   h = cm
Li
Mean effective pressure,   Pm = h × s
where s = spring scale or spring number, (N/cm2)/cm.

5.11.2  Brake Power


A part of the power developed in the engine cylinder is lost in overcoming friction at different parts
of the engine. Therefore the power available at the engine crankshaft is less than the indicated power.
The power available at the crankshaft is known as the brake power of the engine because it is generally
measured by some type of brake.
A rope is wound around the brake drum whose one end is connected to the spring balance sus-
pended from a support and the other end carries the load W, as shown in Fig. 5.13:
Let W = weight on rope, N
  S = spring pull, N
D = outer diameter of brake drum
      dr = diameter of rope
The torque applied on brake drum,
 D + dr 
T = (W − S ) 
 2 

Rope
Drum
D

Figure 5.13  Rope brake dynamometer

M05_THERMAL ENGINEERING_SE_XXXX_CH05.indd 231 6/15/2018 3:08:54 PM


232 Chapter 5

2pNT p (W − S )( D + dr ) N
Brake power, BP = = kW (5.11)
3
60 × 10 60 × 103

5.11.3  Efficiencies of Steam Engine


1. Mechanical efficiency,
BP
h mech = (5.12)
IP
Frictional power,   FP = IP − BP
IP − FP BP
h mech = =  (5.13)
IP IP
Work done
2. Thermal efficiency =
Heat supplied
  Heat supplied to engine per kg of steam = h1 − h2
 Where h1 = heat per kg of steam entering the engine
hf 2 = heat per kg of condensate coming out of condenser
ms = mass of steam supplied per hour
 Heat supplied per hour = ms (h1− hf 2)
 Thermal efficiency of engine on IP basis, i.e., indicated thermal efficiency
( IP ) kW × 3600 3600 3600 × ( IP ) kW
(hi )th = = = (5.14)
ms ( h1 − h f 2 ) ms ms ( h1 − h f 2 )
(h − h f 2 )
( IP ) kW 1
 where msi = specific consumption of steam (kg/kWh) on IP basis

3600 3600 × ( BP ) kW
 Similarly, (hb )th = = (5.15)
ms msb ( h1 − h f 2 )
(h − h f 2 )
( BP ) kW 1
 where msb = specific consumption of steam (kg/kWh) on BP basis
hb BP
= = h mech (5.16)
hi IP
BP in kW
3. Overall efficiency,    hoverall = (5.17)
m f × CV
  where mf = mass of fuel burnt per second in the boiler
   CV = calorific value of fuel in kJ/kg

5.12 ❐ GOVERNING OF STEAM ENGINES


The automatic variation of steam supply with the variation of load is called governing and the device
used for this purpose is known as a governor. There are two methods commonly used for steam engine
governing.

M05_THERMAL ENGINEERING_SE_XXXX_CH05.indd 232 6/15/2018 3:08:58 PM


Steam Engines 233

1. Throttle governing: In this type of governing, the pressure of a steam admitted to the engine is
reduced by throttling before it passes into the engine (Fig. 5.14) and the device used for this pur-
pose is called the throttle valve. The lower pressure of steam during admission process reduces
the work developed by the engine. It has been observed that the steam consumption rate is linearly
proportional to the indicated power. This line is called Willams line.
2. Cut-off governing: In this system of governing, the period of admission of steam entering the
engine cylinder is reduced depending on the load on engine (Fig. 5.14b). The volume of steam
admitted is proportional to the mass of steam.
The indicated power vs steam consumption per minute for throttle and cut-off governing are shown
in Fig. 5.15. It is obvious from the figure that the steam consumption rate at full load is same for both
types of governing, but part load steam consumption of throttle governing is higher than that of cut-off
governing engine.
The governor used for throttle governing is simple and cheaper. But the thermal efficiency of cut-
off governed engine is higher at part load conditions.

p Full load p
p1 3/4th load p1 Full load
p2
p3 1/2 load At reduced load
p4
1/4th load

(a) (b)

Figure 5.14 Methods of governing steam engine: (a) Throttle governing,


(b) Cut-off governing

ing
Steam consumption rate

ov ern
ott le g
Thr
g
nin
go ver
t -off
Cu

IP

Figure 5.15  Effect of method of governing on indicated power

M05_THERMAL ENGINEERING_SE_XXXX_CH05.indd 233 6/15/2018 3:08:59 PM


234 Chapter 5

Example 5.1
The steam supply to a steam engine is at 15 bar dry and saturated. The condenser pressure is 0.4 bar.
Calculate the Rankine efficiency of the cycle. Neglect pump work.
Solution
Given: p1 = 15 bar, p2 = 0.4 bar, x1 = 1.0
From steam tables at 15 bar, h1 = hg = 2789.9 kJ/kg, s1 = sg = 6.4406 kJ/kg.K
At 0.4 bar, hf 2 = 317.7 kJ/kg, hfg2 = 2319.2 kJ/kg
sf 2 = 1.0261 kJ/kg.K sfg2 = 6.6448 kJ/kg.K
For isentropic expansion of steam,
s1 = s2 = sf  2 + x2 sfg2
6.4406 = 1.0261 + x2 × 6.6448
x2 = 0.815
h2 = hf  2 + x2 hfg2 = 317.7 + 0.815 × 2319.2 = 2207.8 kJ/kg
h1 − h2 h −h 2789.9 − 2207.8
h Rankine = = 1 2 = = 0.2354 or 23.54%
h1 − h f 4 h1 − h f 2 2789.9 − 317.7

Example 5.2
A simple Rankine cycle works between 28 bar and 0.06 bar. The initial condition of steam is dry
and saturated. Calculate the cycle efficiency, work ratio and specific steam. consumption.

Solution
Given: p1 = 28 bar, x1 = 1.0 bar, p2 = 0.06 bar
The T-s diagram for the cycle is shown in Fig. 5.16.
From steam tables:
At 28 bar, h1 = 2802.0 kJ/kg, s1 = 6.2101 kJ/kg.K
At 0.06 bar, hf  2 = hf 3 = 151.5 kJ/kg, hfg2 = 2415.9 kJ/kg
sf  2 = 0.521 kJ/kg.K sfg2 = 7.809 kJ/kg.K
vf 3 = vf 2 = 0.001 m3/kg
T

p1
5 1

p2
3 2

Figure 5.16

M05_THERMAL ENGINEERING_SE_XXXX_CH05.indd 234 6/15/2018 3:09:00 PM


Steam Engines 235

For isentropic process 1-2, we have


s1 = s2 = sf 2 = x2 sfg2
6.2104 = 0.521 + x2 × 7.809
x2 = 0.728
h2 = hf 2 + x2 hfg2 = 151.5 + 0.728 × 2415.9 = 1910.27 kJ/kg
Engine work, wc = h1 – h2 = 2802.0 – 1910.27 = 891.73 kJ/kg
Pump work, wp = hf 4 − hf 3 = vf 3 (p1 – p2)
  = 0.001 (28 – 0.06) × 102 = 2.79 kJ/kg
Net engine work, wnet = wc − wp = 891.73 – 2.79 = 888.94 kJ/kg
hf4 = hf3 + wp = 151.5 + 2.79 = 154.29 kJ/kg
wnet wnet
Rankine efficiency, h Rankine = =
q1 h1 − h f 4
888.94
= = 0.3357 or 33.57%
2802.0 − 154.29
w 888.94
Work ratio, rw = net = = 0.997
we 891.73
3600 3600
Specific steam consumption, ssc = = = 4.049 kg/kWh
wnet 888.94
Example 5.3
A single cylinder double acting engine has a bore of 250 mm and a stroke 300 mm. Steam is admit-
ted at a pressure of 1.3 MPa. Cut-off occurs at 30% of stroke and the exhaust pressure is 0.12 MPa.
The diagram factor is 0.82 and the mechanical efficiency is 80%. The engine runs at 150 rpm.
Calculate of the power developed.

Solution
1
Given: D = 0.5 m, L = 0.3 m, p1 = 1.3 MPa, r = , K = 0.82
0.3 d
h mech = 0.8, pb = 0.12 MPa, N = 150 rpm

p 
Mean effective pressure, pm =  1 (1 + ln r ) − pb  × K d
 r 
   1  
= 1.3 × 0.3 1 + ln    − 0.12 × 0.82
   0.3   
= 0.6064 MPa
2 p LAN
Brake power developed, BP = m × h mech
60 × 103
p
2 × 0.6064 × × ( 250)2 × 0.3 × 150
= 4 × 0.80
60 × 103
= 35.72 kW

M05_THERMAL ENGINEERING_SE_XXXX_CH05.indd 235 6/15/2018 3:09:04 PM


236 Chapter 5

Example 5.4
Dry saturated steam at 0.9 MPa is supplied to a single cylinder double acting steam engine develop-
ing 20 kW at 240 rpm. The exhaust is 0.15 MPa. Cut-off takes place at 40% of stroke. The diagram
factor is 0.8 and stroke to bore ratio is 1.25. Assuming hyperbolic expansion determine (a) bore and
stroke of engine, and (b) steam consumption per hour.

Solution
Given: p1 = 0.9 MPa, IP = 20 kW, N = 240 rpm, pb = 0.15 MPa
1 L
= 0.4 , Kd = 0.8, = 1.25
r D
p 
Mean effective pressure, pm =  1 (1 + ln r ) − pb  K d
r 
= [ 0.9 × 0.4 (1 + ln 2.5) − 0.15] × 0.8
 = 0.4319 MPa
2 p ALN
Power developed,   IP = m
60 × 103 p
20 × 60 × 10 = 2 × 0.4319 × 106 × × D2 × 12.5 D × 240
3
4
D3 = 5.896 × 10−3
D = 0.18 m, L = 0.225 m
stroke volume × cut-off ratio
Steam consumption/h = × (strokes / hour )
specific volume of stream at 0.9 MPa
p 0.225 × 0.4 × 2 × 2.10 × 60
= × (0.18)2 ×
4 0.2148
= 307.07 kg/h
Specific volume of dry saturated steam at 0.9 MPa = 0.2148 m3/kg from steam tables.

Example 5.5
A single cylinder double acting steam engine receives steam at 1.0 MPa and exhaust at 0.055 MPa.
The dryness of inlet steam is 96%. The power developed by the engine when running at 220 rpm
is 45 kW, with steam consumption of 450kg/h. The engine bore is 0.25 m and stroke is 0.375 m.
The expansion ratio is 6. Calculate the diagram factor and the indicated thermal efficiency of the
engine.
Solution
Given: p1 = 1.0 MPa, pb = 0.055 MPa, x = 0.96, IP = 45 kW, N = 220 rpm, ms = 450 kg/h, D = 0.25 m,
L = 0.375 m, r = 6
p 1
Theoretical m.c.p. pmt = 1 (1 + ln r ) − pb = (1 + ln 6) − 0.055 = 0.4103 MPa
r 6
Actual m.e.p pm = pmt × K d = 0.4103 × K d
2 p ALN
Power developed, IP = mt × Kd
60 × 103

M05_THERMAL ENGINEERING_SE_XXXX_CH05.indd 236 6/15/2018 3:09:08 PM


Steam Engines 237

p
45 × 60 × 103 = 2 × 0.4103 × 106 × × (0.25)2 × 0.375 × 220 × K d
4
Kd = 0.81
Enthalpy of steam at 1 MPa and 0.96 dry, from steam tables,
h1 = hf 1 + x hfg = 762.6 + 0.96 × 2013.6 = 2717.6 kJ/kg
Enthalpy of water at exhaust pressure of 0.055 MPa, hf 3 = 350.6 kJ/kg
IP × 103 × 3600
Indicated thermal efficiency =
ms ( h1 − h f 3 )

45 × 103 × 3600
= = 15.21%
450( 2717.6 − 350.6) × 103
Example 5.6
A single cylinder double acting condensing type steam engine delivers 20 kW brake power at 240
rpm. The diameter and stroke of the engine are 0.2 m and 0.3 m respectively. The steam is supplied
at 10 bar and cut-off takes place at 50% of stroke. The condenser vacuum is 56 cm of Hg while the
barometer reads 76 cm of Hg. Mechanical efficiency is 80% clearance is 20 % of stroke and piston
rod diameter is 5 cm. Determine the actual mean effective pressure and diagram factor.
Also determine the specific steam consumption on I.P. basis by neglecting clearance and piston
rod area.
Solution
Given: BP = 20 kW, N = 240 rpm, D = 0.2 m, d = 0.5 m,
1
L = 0.3 m p1 = 10 bar, = 0.5 , c = 0.2, h mech = 0.8 x = 0.95
r
76 − 56
Pb = × 1.01325 = 0.267 bar
76
  
1  1  c + 1 
Indicated, m.e.p pmi = p1  +  c +  ln    − pb
r  r 
c+
1 
  r  
  0.2 + 1  
    = 10 0.5 + (0.2 + 0.5) ln    − 0.267
  0.2 + 0.5  
  = 8.506 bar
p LN ( 2 A − a)
Indicated power, IP = ma
60 × 103
 p p 
8.506 × 105 × 0.3 × 240  2 × × (0.2)2 − × (0.05) 2 
=  4 4 
3
60 × 10
= 62.13 kW
BP 20
Actual I .P = = = 25 kW
h mech 0.8

M05_THERMAL ENGINEERING_SE_XXXX_CH05.indd 237 6/15/2018 3:09:12 PM


238 Chapter 5

25
Actual m.e.p., pma = 8.506 × = 3.42 bar
62.13
p 3.42
Diagram factor, K d = ma = = 0.1
pmi 8.506
Neglecting clearance and piston rod area, volume of steam supplied per revolution of engine at 10 bar.
p 1
= D2 L × × 2
4 r
p 2 1 p
supplied per hour = D L × × 2 × N × 60 = × (0.2) × 0.3 × 0.5 × 2 × 240 × 60 = 135.72 m3/ h
4 r 4
135.72
Steam supplied in kg/h =
xu g

where u g = 0.198 m3 /kg at 10 bar from steam tables

135.72
= = 721.5 kg/h
0.95 × 0.198
721.5
Specific steam consumption on IP basis, msi = = 28.86 kg/KWh
25

Example 5.7
Calculate the diameter and stroke of a single cylinder double acting steam engine developing 50 kW
power at 120 rpm with mechanical efficiency of 80%. The steam is supplied at 8 bar pressure and
back pressure of the engine is 1.2 bar. Cut-off takes place at 40% of the stroke and clearance is 15%
of stroke volume.
Assume diagram factor 0.8, and stroke to bore ratio as 1.5.
Solution
Given: BP = 560 kW, N = 120 rpm,h mech = 0.8 , p1 = 8 bar, pb = 1.2 bar,
1 L
= 0.4 , c = 0.15, Kd = 0.8, = 1.5
r D
  
1  1  1 + c 
Indicated, m.e.p pmi = p1  +  c +  ln    − pb
r  r  1
c + 
  r  
  1 + 0.15  
   = 8 0.4 + (0.15 + 0.4) ln    − 1.2
  0.15 + 0.4  
   = 5.245 bar
pma ALN p 2 120
IP = = 4.196 × 105 × D 1.5 D ×
3
60 × 10 4 60 × 103

M05_THERMAL ENGINEERING_SE_XXXX_CH05.indd 238 6/15/2018 3:09:18 PM


Steam Engines 239

= 9.8874 × 102 × D3 kW
62.5 = 9.8874 × 102 × D3
D = 0.398 m or 39.8 cm
        L = 1.5 × 0.398 = 59.75 cm

Example 5.8
The steam enters a steam engine at 15 bar and exhausts at 1.5 bar.
The steam supply is cut-off at 40% of the stroke. The clearance volume is 5% of the swept vol-
ume. Calculate the mean effective pressure.
Solution
1
Given: p1 = 15 bar, pb = 1.5 bar, = 0.4 , c = 0.05
r
  
1  1  c + 1 
Indicated, m.c.p pmi = p1  +  c +  ln    − pb
r  r 
c+
1 
  r  

  0.05 + 1  
= 15 0.4 + (0.05 + 0.4) ln    − 1.5
  0.05 + 0.4  
=10.219 bar

Example 5.9
In a steam engine, the clearance volume is 5% of swept volume and the back pressure is 1.15 bar. If
the compression is at 0.3 of the stroke, find the pressure .at the end of compression stroke.
Find also the mean effective pressure, if the steam supply pressure is 13.7 bar and cut-off occurs
at 40% of stroke.

Solution
The p-v diagram is shown in Fig. 5.17
1 V −V
Given: c = 0.05, p1 = 13.7 bar, pb = 1.15 bar, = 0.4 , V5 - Vc = 0.3 Vs, a = 0.3 = 5 c
r Vs
p5 V5 = p6V6 = 0.05 Vs + 0.3 Vs = 0.35 Vs
V6 = Vc = 0.05 Vs
p5 = pb
pb × 0.35Vs
p6 = = 7 pb = 7 × 1.15
0.05Vs
     = 8.05 bar

M05_THERMAL ENGINEERING_SE_XXXX_CH05.indd 239 6/15/2018 3:09:20 PM


240 Chapter 5

p
1 2
p1
pv = const

6
p6 pv = const 3
pb 4
5

3′
V
VC Vc V5
Vs

Figure 5.17

  
1  1  1 + c   a + c  
pm = p1  +  c +  ln    − pb (1 − a ) + (a + c) ln  
 r  r  1
 c +    c 
  r  
  1 + 0.05  
= 13.7 0.4 + (0.05 + 0.4) ln  
  0.05 + 0.4  
  0.3 + 0.05  
−1.15 (1 − 0.3) + (0.3 + 0.05) ln  
  0.05  
= 10.7036 − 1.5882

= 9.115 bar

Example 5.10
The area of indicator diagram 25 cm2. The swept volume is 0.15 cm2. Calculated the theoretical
mean effective pressure. The indicator diagram is drawn to the following scales:
1 cm = 1 bar along the pressure axis
1 cm = 0.02 m3 along the volume axis
Spring constant = 1 bar/cm.
Solution
Given: A1 = 25 cm2, Vs = 0.15 cm2, S = 1 bar/cm

M05_THERMAL ENGINEERING_SE_XXXX_CH05.indd 240 6/15/2018 3:09:22 PM


Steam Engines 241

Vs
Length of indicator diagram,   Li =
Scale on volume axis
0.15
= = 7.5 cm
0.02
Ai 25
M.E.P., pm = ×s = × 1 = 3.33 bar
Li 7.5

Example 5.11
Find the diagram factor for steam engine with the following data.
Inlet pressure = 10 bar
Back pressure = 1 bar
Expansion ratio = 3
Area of indicator diagram = 12.1 cm2
Length of indicator diagram = 7.5 cm
Pressure scale = 3 bar/cm
Solution
Given: p1 = 10 bar, pb = 1 bar, r = 3, A1 = 12.1 cm2, L1 = 7.5 cm
p
Theoretical m.c.p., pml = 1 (1 + ln r ) − pb
r
10
  = (1 + ln 3) − 1 = 5.9954 bar
3
Ai
Actual m.e.p., pma = × spring scale
Li
12.1
    = × 3 = 4.84 bar
7.5
p 84
Diagram factor, K d = ma = = 0.807
pml 5.9954

Example 5.12
Determine the brake power of a simple double acting steam engine having 400 mm diameter and
500 mm stroke length operating at 350 rpm. The initial and back pressure of steam are 9.5 bar and
1.5 bar respectively. The mechanical efficiency is 80% and the expansion ratio is 2.5.

Solution
Given: D = 0.4 m, L = 0.5 m, N = 350 rpm, p1 = 9.5 bar, pb = 1.5 bar

p1
Indicated m.c.p., pim = (1 + ln r ) − pb
r
9.5
= (1 + ln 2.5) − 1.5
2.5
= 5.782 bar

M05_THERMAL ENGINEERING_SE_XXXX_CH05.indd 241 6/15/2018 3:09:25 PM


242 Chapter 5

2 Pm ALN p 350
IP = = 2 × 5.782 × 105 × (0.4)2 × 0.5 ×
3
60 × 10 4 60 × 103
= 423.843 kW
B.P = IP × h mech = 423.843 × 0.8 = 339.074 kW

Example 5.13
A double acting steam engine has a cylinder bore 200 mm stroke 300 mm and cut-off takes place
at 0.4 stroke. The steam admission and the exhaust pressure are 7 bar and 0.38 bar respectively. If
the diagram factor is 0.8, calculate the indicated power at 200 rpm. Neglect clearance and assume
hyperbolic expansion.

Solution

Given: D = 0.2 m, L = 0.3 m, 1 = 0.4 , p1 = 7 bar, pb = 0.38 bar, Kd = 0.8, N = 200 rpm.
r
p
Theoretical m.e.p.,     pml = 1 (1 + ln r ) − pb
r
  1 
= 7 × 0.4 1 + ln  − 0.38
  0.4  
= 4.9856 bar
Actual m.e.p.. pma = pmt × Kd = 4.9856 × 0.8 = 3.9885 bar
2 Pma LAN p 200
Power developed, IP = = 2 × 3.9885 × 105 × 0.3 × (0.2)2 ×
3
60 × 10 4 60 × 103
= 25.06 kW

Example 5.14
The following readings were taken during the test at full load on a single cylinder, double acting,
condensing type, throttle governed steam engine
Diameter of cylinder = 400 mm
Stroke of engine = 600 mm
Cut-off = 50% of stroke
Pressure of steam supplied = 11 bar
Back pressure = 0.8 bar
Brake wheel diameter = 4.5 m
Net load on the bake = 4900 N
Speed of engine = 150 rpm
Diagram factor = 0.82
Find the indicated power, brake power and mechanical efficiency of the engine.

Solution
1
Given: D = 0.4 m, L = 0.6 m, = 0.5, p1 = 11 bar, pb = 0.8 bar, Db = 4.5m, Wnet = 4900 N, N = 150 rpm,
Kd = 0.82 r

M05_THERMAL ENGINEERING_SE_XXXX_CH05.indd 242 6/15/2018 3:09:28 PM


Steam Engines 243

p1
Indicated theoretical m.e.p., pml = (1 + ln r ) − pb
r
= 11 × 0.5 (1 + ln 2) − 0.8 = 8.5123 bar
Actual m.e.p., pma = pml × Kd = 8.5123 × 0.82 = 6.98bar
2 P ALN
Indicated Power, IP = ma
60 × 103
p
2 × 6.98 × 105 × (0.4)2 × 0.6 × 150
= 4 = 263.18 kW
60 × 103
Wnet × pDb × N 4900 × p × 4.5 × 150
Brake power, BP = = = 173.18 kW
3
60 × 10 60 × 103
BP 173.18
Mechanical efficiency, h mech = = = 0.658 or 65.8 %
IP 263.14

5.13 ❐ SATURATION CURVE AND MISSING QUANTITY


Saturation Curve: The saturation curve is the curve showing the volume the steam in the cylinder
would occupy, during the expansion stroke if the steam is perfectly dry and saturated at all the points.
It is plotted on p-V diagram and the wetness of stream can be seen on it at a glance.
Figure 5.18 shows a calibrated indicator. The total mass of steam in cylinder during expansion
stroke = mC + m.
On the expansion curve, consider any point B and read off from the diagram the pressure (pB) and
volume (VB) at this point. From steam tables, obtain the specific volume υ of dry steam at pressure pB.
The volume the steam at B would occupy if dry saturated = mυ. Let this volume be represented by AC
to the volume scale of the p-V diagram. Then, the point C represents the volume the steam at B would
occupy if dry saturated.
Volume of dry steam in mixture AB
∴ Dryness fraction at B = =
Total volume of steam AC
Following this way, a number of points may be obtained and plotted. The curve passing through
these points to known as saturation curve (because all the points on this line represent the condition
of steam dry and saturated).
From this saturation curve, the dryness fraction for all points on expansion curve can be obtained.
For instance, the dryness fraction at E will be given by
DE
Dryness fraction at E, x =
DF
From Fig. 5.18, it can be seen that the steam is wet at the beginning of the expansion stroke and
becomes drier towards the end. This is owing to the fact that high pressure steam in the initial stage of
expansion is better than the cylinder walls; this causes the steam to condense. During the expansion
stroke the steam pressure falls and towards the end of the stroke the walls will be hotter than the steam;
consequently the condensed steam re-evaporates and as a result the dryness fraction is improved. The
improvement in dryness fraction will not be there if cylinder walls are not jacketed.
Missing quantity: The missing quantity is the horizontal distance between the actual expansion curve
and the saturation curve (Fig. 5.18). At a pressure pD, the missing quantity is represented by EF (m3).

M05_THERMAL ENGINEERING_SE_XXXX_CH05.indd 243 6/15/2018 3:09:31 PM


244 Chapter 5

Saturation
curve
1
A B C

D E F
Expansion
curve

V
VB

Figure 5.18  Calibrated indicator diagram

The missing quantity is mainly due to condensation of the steam, but a small amount will be due to
leakage past the piston. Due to this missing quantity there is a loss of work represented by the area
between the expansion curve and saturation curve.
The missing quantity can be reduced in the following ways
1. By steam jacketing the cylinder walls efficiently.
2. By reducing the temperature range of the steam during the stroke this can be accomplished by
compounding the expansion of steam in two cylinders instead of allowing the whole pressure to
drop to occur in one cylinder.

5.14 ❐ HEAT BALANCE SHEET


Procedure: For preparing a heat balance sheet for a steam engine cylinder, the engine should be
tested over a period of time under conditions of constant load and steam supply. An indicator diagram
should be taken and the steam pressure noted at regular intervals of time, an account should also he
kept of the steam supply to the jackets. The mass of the steam supplied to the cylinder supplied can be
obtained from the steam condensed by the condenser.
Analysis: The heat balance for the steam in engine is more easily drawn up than that of the internal
combustion engine. The working fluid in the steam engine does not undergo any chemical change, and
consequently changes in properties may be ascertained with reference to an arbitrary datum.
Figure 5.19 shows, diagrammatically, a steam engine and various quantities entering and leaving
have been indicated there on. Treating the engine as a flow system as shown in Fig. 5.20, an energy
equation may be written as follows:
ms1 hs1 + mc hc1 = ms2 hs2 + mc hc2 + W + QR(5.18)

ms1 hs1 = ms2 hs2 + mc (hc2 − hc1) + W + QR (5.19)


where m = mass, W = work, h = specific enthalpy, Q = heat transferred, and suffices have the
following meanings, s1 = steam supplied, s2 = condensate discharge, c = cooling water, R = radiation.

M05_THERMAL ENGINEERING_SE_XXXX_CH05.indd 244 6/15/2018 3:09:32 PM


Steam Engines 245

Radiation out

Steam
in

Cooling
water in

Work out Condenser

Cooling
water out

Condensate out

Figure 5.19  A steam engine

Steam

Condensate

Cooling
water
Work Cooling
Radiation water

Figure 5.20

The Eq. (5.19) in tabular form is expressed as follows:

Heat in Heat out


In steam ms1 hs1 Heat as shift work W
Heat to condensate ms2 hs2
Heat to coolant mc (hc2 − hc1)
Heat to radiation QR

The value of enthalpy may be obtained from the steam tables. The heat to coding water is given by
mc × cpw (tout − tin) and the heat to radiation obtained by the difference. In the absence of leakage
ms1 = ms2.

M05_THERMAL ENGINEERING_SE_XXXX_CH05.indd 245 6/15/2018 3:09:33 PM


246 Chapter 5

The heat balance may therefore be written as follows:

Heat in Heat out


Heat in steam Heat as B.P.
   Heat to condensate.
Heat to cooling water.
Heat to radiation, by difference.

It is to be noted that friction terms are absent, since any work done against friction is included in heat
to the condensate, cooling water and radiation loss.

5.15 ❐ PERFORMANCE CURVES


Typical performance curves of a reciprocating steam engine under test conditions are shown in
Fig. 5.21.
Most of the curves are self-explanatory and should be carefully scrutinised for analysis of the
variables involved.
One salient fact is that the speed curve as shown is practically horizontal with a slight drop as
the load is increased, this shows good governing. The Willian’s line as shown is straight when the
governing is by throttling with a fixed cut-off, and indicates a linear relationship between the total
steam consumption is kg/h and the power if steam pressure is varied to suit the load. This fact is
also of great advantage in predicting the part load steam consumption of steam turbines, most of
which use throttle governing. 1f the steam consumption is plotted for an automatic engine with
cut-off governing, the line will not be strictly straight.

Speed, r.p.m.
Sp m ra
ste
ec
a
ific e

y
enc
le ffici
nica
t

ha
Mec y
ienc
e effic
Engin
kg/lPh
ne
s li
ll an’
Wi

I.P.

Figure 5.21

M05_THERMAL ENGINEERING_SE_XXXX_CH05.indd 246 6/15/2018 3:09:34 PM


Steam Engines 247

Example 5.15
In a single cylinder double acting steam engine steam is supplied at a pressure of 12 bar and
exhaust takes place at 1.1 bar. The cut off takes place at 10% of the stroke which is equal to 1.25
times the cylinder bore and the engine develops and indicated power of 100 kW at 90 r.p.m., calcu-
lated the bore and stroke of the engine assuming hyperbola expansion and a diagram factor of 0.8.
Also determine the theoretical steam consumption in m3/mm

Solution
Given: p1 = 12 bar, pb = 1.1 bar,v2= 0.4v3, L = 1.25D, IP = 100 kW, N = 90 rpm, Kd =0.8
The p-v diagram is shown in Fig. 5.22.
Mean effective pressure without clearance
p 
pm = K d  1 (1 + ln r ) − pb 
r 
v3 1
Expansion ratio, r= = = 2.5
v2 0.4
   pm = 0.8 [12 × 0.4 (1+ ln 2.5) + 1.1] = 6.4786 bar
2 Pm LAN
Indicated power developed,     IP =
60 × 103
p
2 × 6.4786 × 105 × 1.25 D × D 2 × 90
100 = 4
3
60 × 10
D3 = 5240.8 × 10-5
D = 0.374 m
L = 1.25 D = 0.468 m
Theoretical steam consumption/min
Stroke volume × cut-off ratio
    = × Number of stroke/min
Specific volume of steam at 12 bar
p
(0.374)2 × 0.468 × 0.4 × 2 × 90
= 4 = 22.71 kg/min
0.163

1 2
p1 pv = const

pb
5 4

v
v2 v3

Figure 5.22

M05_THERMAL ENGINEERING_SE_XXXX_CH05.indd 247 6/15/2018 3:09:37 PM


248 Chapter 5

Summary for Quick Revision

1. A steam engine is an external combustion heat engine as the fuel is burnt in the boiler. The steam raised is
used to reciprocate the piston in the cylinder for generating power.
2. The mean effective pressure is the hypothetical constant pressure which will produce same work for the same
piston displacement with the variable conditions of temperature, pressure and volume.
area of indicator diagram
pm = × spring constannt
length of stroke
p1
= (1 + ln r ) − pb , without clearance and compression
r
  
1  1  1 + c 
= p1  +  c +  ln    − pb
r  r   c + 1 
  r  
With clearance and without compression.
  
1  1  1 + c    a + c 
= p1  +  c +  ln    − pb (1 − a ) + (a + c)ln  without clearance and compression
 r  r  1
 c +    c  
  r  
v3
where r = = expansion ratio
v2
v5 − vc
a = = compression ratio
vs
pb = back pressure
p1= admission pressure of steam
vc
c = clearance ratio =
vs − vc
h1 − h2 − w p
3. Efficiency of simple Rankine cycle, h Rankine =
h1 − h f 4
where, Pump work, w p = h f 4 − h f 3 = v f 3 ( p1 − p2 )

p1v2 + (u2 − u3 ) − p3v3


4. Efficiency of modified Rankine cycle =
h2 − h f 4
5. The indicator diagram is a representation of the variation of pressure and volume of steam inside the cylin-
der on p-v diagram for one complex cycle of operation.
6. Diagram factor is the ratio of the area of actual indicator diagram to the area of theoretical indicator diagram.
Pm LAN
7. Indicated power, IP = kW for single acting
60 × 103
P LA N + Pm 2 LA2 N
= m1 1 kW for double acting
60 × 103

M05_THERMAL ENGINEERING_SE_XXXX_CH05.indd 248 6/15/2018 3:09:43 PM


Steam Engines 249

p (W − S )( D + dr ) N
8. Brake power, BP = kW
60 × 103
where D = drop drum dia, dr = rope dia, N = rpm, W = weight on rope, S = spring pull.
BP IP − FP
9. Mechanical efficiency, h mech = =
IP IP
3600 × ( IP ) kW
Indicated thermal efficiency, (h th )i =
ms ( h1 − h f 4 )
3600 × ( BP ) kW
Brake thermal efficiency, (h th )b =
ms ( h1 − h f 4 )
( BP ) kW
Overall efficiency, hoverall =
m f × CV
10. In throttle governing, the pressure of steam admitted to the engine is reduced by throttling before it passes
into the engine.
11. In cut-off governing, the period of admission of steam entering the engine cylinder is reduced depending on
the load on engine.
12. Saturation curve is the curve showing the volume the steam in the cylinder would occupy during the expan-
sion stroke if the steam is perfectly dry and saturated at all the points.
13. Missing quantity is the horizontal distance between the actual expansion curve and the saturation curve.
14. Heat balance sheet shows the balance between the amount of heat input to the engine and heat utilised for
various purposes.

Multiple-choice Questions

1. Diagram factor is defined as the ratio of 5. In order to reverse a steam engine, the eccentric to
(a) actual m.e.p. and swept volume be shifted for an angle advance of 40°is
(b) theoretical m.e.p. and swept volume (a) 180° (b) 200°C
(c) actual m.c.p. and theoretical m.e.p. (c) 160° (d) 190
(d) theoretical m.e.p. and actual m.e.p.
6. A steam engine works on
2. Choose the quantity which is varied during throt- (a) Carnot cycle
tle governing of steam engines (b) Rankine cycle
(a) pressure (b) volume (c) modified Rankine cycle
(c) temperature (d) dryness fraction (d) reheat cycle
3. In a compound steam engine 7. The cross-head in a steam engine is used to
(a) steam expands twice connect
(b) two engines are combined together (a) piston rod and connecting rod
(c) steam expands in several stages (b) valve rod and piston rod
(d) two units are put together (c) eccentric rod and piston rod
4. In the tandem compound steam engine, the axes (d) connecting rod and valve rod
of the two cylinders 8. A D-slide valve in a steam engine is used to
(a) are at 90° to each other (a) admit steam into cylinder from steam chest.
(b) are inclined at 45° (b) exhaust steam from cylinder to condenser
(c) are at 0° to each other (c) to admit and exhaust steam from cylinder
(d) lie in different plane (d) none of the above.

M05_THERMAL ENGINEERING_SE_XXXX_CH05.indd 249 6/15/2018 3:09:46 PM


250 Chapter 5

9. The cut-off ratio in a steam engine is defined as (a) to-and-fro motion of D-slide valve to rotary
the ratio of volume at cut off to motion of crankshaft
(a) swept volume (b) cylinder volume (b) oscillatory motion of connecting rod to rotary
(c) clearance volume (d) none of above. motion of crankshaft
(c) rotary motion of crankshaft to to-end-fro
10. The clearance ratio in a steam engine is defined as
motion of D-slide valve
the ratio of clearance volume to
(d) rotary motion of crankshaft to oscillatory
(a) swept volume (b) cut-off volume
motion of eccentric rod
(c) cylinder volume (d) none of the above
16. The main function of a stuffing box in a steam
11. Willian’s line is the relation between steam mass engine is to
flow late and (a) prevent leakage of steam
(a) indicated power (b) guide the piston rod
(b) brake power (c) guide the valve rod
(c) compression ratio (d) receive exhaust steam from engine
(d) thermal efficiency
17. The motion to D-slide valve is imparted by
12. In receiver type compound steam engine, the (a) eccentric (b) cam
cranks are placed at (c) flywheel (d) crank
(a) 45° (b) 90°
(c) 180° (d) 270° 18. Rankine efficiency of a steam engine may be in
the range of
13. In a Woolfe compound steam engine, the cranks (a) 15–20% (b) 25–35%
are placed at (c) 70–80% (d) 90–95%
(a) 0° (b) 90°
(c) 180° (d) 270° 19. Rankine cycle comprises of
(a) two isentropic and two reversible constant
14. The function of piston in steam engine is to trans- volume processes
fer motion in (b) two isentropic and two reversible constant
(a) cross head (b) connecting rod pressure processes
(c) D-slide valve (d) eccentric (c) two isothermal and two reversible constant
15. The function of eccentric in a steam engine is pressure processes
convert (d) none of the above

Review Questions
1. Explain the difference between single acting and double acting steam engines
2. Differentiate between condensing and non-condensing steam engines
3. What are the functions of the following
(a) cross head (b) Eccentric
(c) stuffing box (d) D-slide valve
4. Explain the following therms:
(a) Cut-off (b) Release
(c) Dead centres (d) Back pressure.
5. Define diagram factor.
6. Define indicated thermal efficiency and brake thermal efficiency.
7. List the methods for governing of steam engines.
8. Compare and contrast the methods for governing of steam engine
9. How do you determine the brake power of a steam engine.
10. What is frictional power of a steam engine.

M05_THERMAL ENGINEERING_SE_XXXX_CH05.indd 250 6/15/2018 3:09:50 PM


Steam Engines 251

Exercises
5.1 A single cylinder, double acting, condensing type steam engine 0.3 bore and 0.4 m stroke runs at 150 rpm.
Steam at 10 bar is supplied upto 40% of the stroke. Back pressure of the engine is 0.35 bar. Neglecting clear-
ance and assuming a diagram factor of 0.8 and a mechanical efficiency of 80% determine the brake power
of the engine.
5.2 The following readings were taken from a test on a single cylinder double-acting steam engine having 0.25 m
bore and 0.35 m stroke
Pressure of steam supplied = 9.5 bar
Engine speed = 240 rpm
Area of indicator card = 10.5 cm2
Length of indicator card = 8 cm
Spring number = 25 N/cm2/cm
Net load on brake wheel = 1000 N
Diameter of brake wheel = 3 m
Find IP, BP and mechanical efficiency
5.3 A single-cylinder, double-acting, non-condensing steam engine 25 cm bore 50 cm stroke develops 40 kW
indicated power at 100 rpm. The clearance is 10% and cut-off takes place at 40% of stroke. The steam pres-
sure at the point of cut-off is 5 bar. The compression starts at 80% of the return stroke. The pressure of steam
on compression curve at 90% of the return stroke is 1.4 bar and steam is dry and saturated. Determine the
actual and minimum theoretically possible specific steam consumption on indicated power basis.
5.4 Determine the brake power of a single cylinder double acting, condensing type steam engine having 0.3 m
bore and 0.6 m stroke, running at 250 rpm. The steam 8 bar dry and saturated is supplied upto 50% stroke.
The back pressure is 1 bar. Assume mechanical efficiency 70%, diagram factor = 0.75, and piston rod diame-
ter = 5 cm. Also find the specific steam consumption on indicated power basis if indicated thermal efficiency
= 15 %.
5.5 Single cylinder, double acting condensing type steam engine developed 2.10 indicated power at 120 rpm.
Dry and saturated steam at 10 bars is supplied for 40% of the stroke. The exhaust pressure is 0.2 bar. Assum-
ing a diagram factor of 0.85 and ratio of stroke to bore as 2 determine the bore and stroke of the engine.
5.6 A single-cylinder, double acting steam engine develops 150 kW indicated power at 240 rpm. The pressure
of steam supplied is 10 bar and exhaust pressure is 0.15 bar. The cutoff is at 40 % of the stroke, clearance is
10 % of the stroke and compression starts at 80% of the return stroke. Expansion follows the low: PV1.25 = C,
5
Compression follow law PV1.35 = C, ratio of stroke to bore = diagram factor = 0.85. Calculate the bore
3
and stroke of the engine.
5.7 The following data were obtained from the trail on a steam engine governed by throttle governor steam
admission pressure = 10.5 bar dry and saturated
Back pressure = 0.2 bar
Steam consumption at no load = 44 kg/h
Indicated power developed at full load = 7 kW
Specific steam consumption at full load = 18 kg/kWh
Find the specific steam consumption and indicated thermal efficiency when the engine develops 5 kW indi-
cated power.
5.8 A single cylinder, double acting steam engine having 25cm bore and 50 cm stroke is governed by cut-off
governing. The maximum cut-off possible is 50 % of stroke. The steam is supplied at 8 bar and exhausts at
1 bar. Calculate the percentage decrease in power when the cut-off is reduce to 40% of stroke. Assume that
the diagram factor is 0.8 and mechanical efficiency is 80%.

M05_THERMAL ENGINEERING_SE_XXXX_CH05.indd 251 6/15/2018 3:09:51 PM


252 Chapter 5

5.9  A single-cylinder, double-acting, steam engine develops 40 kW when running at 120 rpm. The steam is
admitted at 5 bar and exhausted at 0.35 bar. Cut-off occurs at 1/3 of stroke. Assuming a diagram factor of
0.75 and mechanical efficiency of 0.85 determine the swept volume cylinder.
5.10  Steam is admitted to an engine for 30% of the stroke with pressure of 7 bar, the law of expansion followed
is law pV1.5 = C the compression commences at 60% of return stoke and follows the law F = C. The clear-
ance volume is 20% of the displacement volume and the back pressure is 6.18 mm of Hg vacuum, when
barometer reads 760 mm of Hg. Estimate the mean elective pressure and the indicated power of a double
acting engine with cylinder diameter 30 cm stroke 45 cm and speed 200 rpm.

ANSWERS TO MULTIPLE-CHOICE QUESTIONS


1. (c) 2. (b) 3. (d) 4. (c) 5. (d)
6. (b) 7. (d) 8. (a) 9. (a) 10. (a)
11. (a) 12. (b) 13. (c) 14. (a) 15. (c)
16. (a) 17. (a) 18. (b) 19. (b)

M05_THERMAL ENGINEERING_SE_XXXX_CH05.indd 252 6/15/2018 3:09:51 PM


Chapter 6

Flow Through Steam


Nozzles
6.1 ❐ INTRODUCTION
The steam nozzle is a passage of varying cross-section by means of which a part of the enthalpy of
steam is converted into kinetic energy as the steam expands from a higher pressure to a lower pressure.
Therefore, a nozzle is a device designed to increase the velocity of steam. Steam nozzles are of three
types, namely convergent nozzle, divergent nozzle, and convergent–divergent nozzle. If the cross-
section of the nozzle decreases continuously from the entrance to exit, then it is called a convergent
nozzle, as shown in Fig. 6.1(a). If the cross-section increases, then it is called a divergent nozzle, as
shown in Fig. 6.1(b). If the cross-section of the nozzle first decreases and then increases, it is called a
convergent–divergent nozzle, as shown in Fig. 6.1(c). The least area of cross-section of the nozzle is
called the throat. The divergent section has to be long as the divergent angle is limited to about 7° in
order to prevent separation at the wall.
The main purpose of steam nozzles is to produce a high velocity jet of steam which is used in steam
turbine injectors for pumping feed water into boilers and to maintain high vacuum in power plant
condensers or steam jet refrigeration condensers.

Entry throat Exit

Convergent part Divergent part

(a) (b) (c)

Figure 6.1 Types of nozzles: (a) Convergent nozzle (b) Divergent nozzle


(c) Convergent-Divergent nozzle

M06_THERMAL ENGINEERING_SE_XXXX_CH06.indd 253 6/15/2018 3:18:08 PM


254 Chapter 6

6.2 ❐ CONTINUITY EQUATION


Consider the flow of steam through a nozzle.
Let ṁ = steady state mass flow rate of steam, kg/s
A = cross-sectional area of nozzle normal to the direction of steam flow at any section, m2
v = specific volume of steam at the same section, m3/kg
c = velocity of steam across the section, m/s
Then for steady flow of steam through the nozzle,
Ac A1c1 A c
ṁ = = = 2 2 (6.1)
v v1 v2
Equation (6.1) is the continuity equation for steam flow through the nozzle.

6.3 ❐ VELOCITY OF FLOW OF STEAM THROUGH NOZZLES


The following assumptions are made:
1. The steam flows through the nozzle without any work or heat transfer.
2. The frictional forces are neglected.
Consider the flow of steam through a nozzle, as shown in Fig. 6.2. Applying the energy equation to
sections 1 and 2 at entrance and exit of nozzle, we have

c12 c2
h1 + = h2 + 2
2 2

c2 = 2( h1 − h2 ) + c12 (6.2a)

Usually c1 = 0,

c2 = 2( h1 − h2 ) = 44.72 ( ∆h)isen (6.2b)


where (∆h)isen = h1 − h2 = isentropic enthalpy change, kJ/kg

2
A1
A2
c1
c2
p2
p1
v2
v1 Exit
Entry

Figure 6.2  Flow through a nozzle

M06_THERMAL ENGINEERING_SE_XXXX_CH06.indd 254 6/15/2018 3:18:11 PM


Flow Through Steam Nozzles 255

6.3.1 Flow of Steam Through the Nozzle


Let    pvn = constant
where n = 1.035 + 0.1 x1 for wet steam
= 1.135 for saturated steam
= 1.3 for superheated steam
As the steam pressure drops while passing through the nozzle, its enthalpy is reduced. This reduction
of enthalpy of steam must be equal to the increase in kinetic energy. Hence,
c22 − c12  n 
= (p v – p2v2)
2  n − 1 1 1
where p1, p2 = pressure of steam at entry and exit, respectively, v1, v2 = specific volume at entry and
exit, respectively.
If c1 << c2, then
c22  n   n   p v 
=  ( p1v1 − p2 v2 ) =   p1v1 1 − 2 2 
2  n − 1  n − 1  p1v1 
But p1v1n = p2 v2n
1
v  p n
    2 =  1 
v1  p2 
 1  n −1 
c22  n   p2  p1  n   n    p2  n 
    =   p v 1− ⋅ =  p v 1− 
2  n − 1 1 1  p1  p2    n − 1 1 1   p1  
   
 n −1 
 2n    p2  n 
      c2 =  p v 1−  (6.3)
 n − 1 1 1   p1 
 
 

6.4 ❐ MASS FLOW RATE OF STEAM


The mass flow of steam per second is flowing with velocity c2 through a cross-sectional area A2, and
specific volume v2 is:
A c
ṁ = 2 2
1 v2
 p n
Now, v2 = v1  1 
p  2
Also, from Eq. (6.3), we have

 2n 
c2 =   ( p v − p2 v2 )
n − 1 1 1
A 2  2n 
ṁ =   ( p v − p2 v2 )
v2  n − 1 1 1

M06_THERMAL ENGINEERING_SE_XXXX_CH06.indd 255 6/15/2018 3:18:15 PM


256 Chapter 6

A2  2n 
=   ( p v − p2 v2 )
1 n − 1 1 1
 p1  n
v1  
 p2 
1
  2 n +1   2
= A2   2 n  p2   p2  n  p2 n   (6.4)

    −   
 n − 1 v1  p1   p1  
  

6.5 ❐ CRITICAL PRESSURE RATIO


From Eq. (6.4), we have
1
  2 n +1   2
m  2n  p2
 p2  n  p2  n  
=     −   
A 2  n − 1 v1
 p1   p1  
  
The mass flow rate per unit area is maximum at the throat because it has minimum area of cross-
 2 n +1 
m  p2  n  p2  n 
section. Therefore, will be maximum when   −    is maximum
A2  p1   p2  
 

 2 n +1 
 p2  n  p2  n  d
or  −  =0
 p2   p1   p1  
d   
 p1 
2 n +1
−1 −1
2 p n  n + 1  p2  n
or  2  − =0
n  p1   n   p1 
2−n 1
2 p  n  n + 1  p2  n
or  2  =
n  p1   n   p1 

n 2−n
p   n + 1  p2 
    2 
or     =
 p1   2   p1 

1− n n
p   n + 1
  2 
or   =
 p  1  2 
n n
p  n + 1 1− n  2  n −1
or       2 =   = (6.5)
p1  2   n + 1

M06_THERMAL ENGINEERING_SE_XXXX_CH06.indd 256 6/15/2018 3:18:21 PM


Flow Through Steam Nozzles 257

Equation (6.5) gives the critical pressure ratio at which the discharge through the nozzle is maximum.
p
For saturated steam, n = 1.135, then 2 = 0.5774 (6.5a)
p1
p
For superheated steam, n = 1.3, then 2 = 0.5457 (6.5b)
p1
1
  1− n  2
 2n    2n   p2  n 
Now, c2 =  ( p v − p2 v2 ) =   p2 v2   − 1 
 n − 1 1 1   n − 1 
 1 p  
  
p2
Substituting for from Eq. (6.5), we get
p1
1 1
  2n   n + 1   2  2n   n − 1 2
c2 =   p2 v2  − 1  =     p2 v2 

 n −1  2  
 n −1 2 

                    = np2 v2 = nRT2 = cs2(6.6)


where cs2 = velocity of sound at exit of the nozzle.
1
  n −1   2
  2n    p2  n  
Also, c2 =   p1v1 1 −   
 n − 1   p1  
  

 2n   2n 
=  ⋅ np1v1 =  ⋅ nRT1
 n + 1  n + 1

 2n 
=  c  (6.7)
 n + 1 s1
where cs1 = velocity of sound at entry of nozzle.

6.6 ❐ MAXIMUM DISCHARGE


From Eq. (6.5), maximum mass flow rate of steam can be obtained as
1
  2 n +1   2
A 2   2n   2  n −1  2  n −1  
(ṁ )max =   p v   − 
v1  n − 1 1 1  n + 1  n + 1 

  
1
  2 n +1   2
 2n  p1  2  n −1  2  n −1  
= A 2   − 
 n − 1 v1  n + 1  n + 1 

  

M06_THERMAL ENGINEERING_SE_XXXX_CH06.indd 257 6/15/2018 3:18:26 PM


258 Chapter 6

1
  n +1
−1
1− n   2
 2n  p1  2  n −1  2  n −1  
= A 2   − 
 n − 1 v1  n + 1  n + 1 

  
1
 n +1 2
−1
 2n   p1   2  n −1  2   
= A 2   − 1 
 n − 1  v1   n + 1  n + 1
   

1
 n +1 2
 2n   p1   2  n −1  n + 1  
= A 2   − 1
 n − 1  v1   n + 1   2   
 
1
 n +1 2
 2n   p1   2  n −1  n − 1 
= A 2 
 n − 1  v1   n + 1  
2 
 
1
 n +1  2
 np1  2  n −1 
= A2     (6.8)
v  n + 1
 1 

As p2 is gradually reduced, the discharge gradually increases and becomes maximum as critical
pressure is approached, as shown in Fig. 6.3.

Maximum discharge
Discharge

Critical pressure ratio

Pressure ratio p2/p1

Figure 6.3  Discharge v’s pressure ratio in a nozzle

6.7 ❐ EFFECT OF FRICTION ON EXPANSION OF STEAM


The exit velocity of steam for a given pressure drop is reduced due to the following reasons:
1. Friction between nozzle surface and steam
2. Internal fluid friction in the steam
3. Shock losses

M06_THERMAL ENGINEERING_SE_XXXX_CH06.indd 258 6/15/2018 3:18:28 PM


Flow Through Steam Nozzles 259

Most of these losses occur beyond the throat in the divergent section of the nozzle as the length and the
velocity of steam is much higher there. The effects of these friction losses are as follows:
1. The expansion does not remain isentropic and the enthalpy drop is reduced, resulting in lower exit
velocity.
2. The final dryness fraction of steam is increased as a part of kinetic energy gets converted into heat
due to friction and is absorbed by steam, which increases the enthalpy.
3. The specific volume of steam is increased due to frictional reheating.
The effect of friction of steam flow through a nozzle is shown in h–s (Mollier) diagram in
Fig. 6.4. The point A represents the initial condition of steam and the point E represents the throat of a
convergent–divergent nozzle. AB represents the isentropic expansion without friction, and AC repre-
sents the expansion with friction. Isentropic enthalpy drop is (hA – hC) and actual enthalpy drop with
friction is (hA – hc). If the actual enthalpy drop as percentage of theoretical enthalpy drop is known,
then point C can be located. However, expansion must end at same pressure as at B. The horizontal
line drawn through C to cut the back pressure line pb at point D represents the final condition of steam.
The h–s diagram shows that the dryness fraction of steam at point D is greater than that at point B and
the specific volume of steam also increases.
Most of the friction occurs in the divergent part of the nozzle and actual expansion is represented
by the line AED. AE represents the expansion in the convergent part, whereas ED represents in the
divergent part of the nozzle. Lines AEB and AED represent expansion of steam for initially saturated
steam. Likewise, A1E1B1 and A1E1D1 are for initially superheated steam.

h
A1
A E1

D1 x = 1
D B1
B C

Figure 6.4  Effect of friction on expansion of steam in a convergent–divergent nozzle

6.8 ❐ NOZZLE EFFICIENCY


Nozzle efficiency, hn is a factor that takes into account the effect of friction during expansion of
steam in the nozzle. It is defined as:
Actual enthalpy drop h − hD
hn = = A (6.9)
Isentropic enthalpy drop hA − hB

M06_THERMAL ENGINEERING_SE_XXXX_CH06.indd 259 6/15/2018 3:18:30 PM


260 Chapter 6

The exit velocity of steam considering friction is:

c2 = 44.72 h n × ( ∆h)isen (6.10)


= 44.72 K ( ∆h)isen

where K = hn .

6.9 ❐ SUPERSATURATED OR METASTABLE FLOW


THROUGH A NOZZLE
The isentropic expansion of superheated steam from supply pressure p1 to back pressure pb can be
represented on the Mollier diagram by line AE, as shown in Fig. 6.5. During expansion, change of
phase must start at point B where the pressure line p2 meets the saturation line. However, in nozzles,
under certain conditions, this phenomenon of condensation does not occur at point B as the time
available is very short due to very high velocity of steam (nearly sonic) through the nozzle. The equi-
librium between the vapour phase and liquid is, therefore, delayed and the vapour continues to expand
in dry state even beyond point B upon point B1. The pressure at B1 can be found by extending the
superheated constant pressure line p3 up to B1. The steam during the expansion BB1 remains dry and
condensation is suppressed.
The vapours between pressure p2 and p3 are said to be supersaturated or supercooled and such a
flow in nozzles is called supersaturated or metastable flow of steam. A limit to the supersaturated state
was observed by Wilson and the line drawn on the Mollier chart through the observed points is known
as Wilson line. For all practical purposes, this line has become the saturation line.
The flow is also called supercooled flow because at any pressure between p2 and p3, the tempera-
ture of the vapour is always lower than the saturation temperature corresponding to that pressure. The
difference in this temperature is called the degree of undercooling.
When the expansion reaches at point B, on the Wilson line, the condensation occurs at constant
enthalpy, and the pressure remaining constant, as shown by horizontal line BC. Further isentropic
expansion to the exit pressure is represented by CD. The ratio of saturation pressures corresponding to
the temperatures at B and B1 is called the degree of supersaturation.

h
p1
A p2
B
p3

B1 pb
C
Supersaturated
field
E D
x=1
Wilson line
s

Figure 6.5  Super-saturated flow of steam in a nozzle

M06_THERMAL ENGINEERING_SE_XXXX_CH06.indd 260 6/15/2018 3:18:32 PM


Flow Through Steam Nozzles 261

Velocity of steam at the end of expansion,


1
  n −1   2
  2n    p2  n  
c2 =   p1v1 1 −   
 n − 1   p1  
  
1
 p n
Specific volume, v2 = v1  1 
 p2 
n −1
p  n
Temperature, T2 = T1  2 
 p1 
 2
mv
and A2 =
c2
The superheated expansion law, pv1.3 = constant, is followed in the supersaturated flow.

Example 6.1
Steam expands from 2.5 bar to 1 bar in a nozzle. The initial velocity of steam is 80 m/s and initial
temperature is 200°C. Taking nozzle efficiency as 96%, find the exit velocity.

Solution
Given that p1 = 2.5 bar, p2 = 1 bar, c1 = 80 m/s, t1 = 200°C, hn = 0.96
The h – s diagram is shown in Fig. 6.6.
From steam tables for superheated steam, we have
h1 = 2868 kJ/kg at p1 = 2.5 bar and 200°C
s1 = 7.3593 kJ/kg.K
At p2 = 1 bar, sf 2 = 1.3025 kJ/kg.K and sfg2 = 6.0568 kJ/kg.K
Since s1 > sfg2, steam is superheated. At 1 bar and s1 = 7.3593 kJ/kg.K, the enthalpy for superheated
steam by interpolation is:
 2776.4 − 2676.2 
h2 =   (7.3593 – 7.3614) + 2676.2
 7.6133 − 7.3614 
= 2691.95 kJ/kg
h
2.5 bar
1 1 bar
h1

h2′
h2 2 2′

Figure 6.6  Steam flow through a nozzle

M06_THERMAL ENGINEERING_SE_XXXX_CH06.indd 261 6/15/2018 3:18:34 PM


262 Chapter 6

Enthalpy drop = h1 – h2 = 2868 – 2691.95 = 176.05 kJ/kg


Actual enthalpy drop, (∆h)actual = hn (h1 – h2) = 0.96 × 176.05 = 169.0 kJ/kg

c2 = 2( ∆h)actual + c12 = 2 × 169.0 × 103 + 80 2


= 586.86 m/s

Example 6.2
Superheated steam enters a convergent–divergent nozzle at 20 bar and 300°C. The exit pressure is
4.5 bar. Assuming frictionless flow up to the throat (pv1.3 = const.) and a nozzle efficiency of 90%,
determine (a) the flow rate for a throat area of 30 cm2 and (b) exit area.

Solution
Given that p1 = 20 bar, t1 = 300°C, p3 = 4.5 bar, hn = 0.9, A2 = 30 cm2
n 1.3
p  2  n −1  2  0.3
Critical pressure ratio, 2 =   = = 0.5457
p1  n + 1  2.3 
Throat pressure, p2 = 20 × 0.5457 = 10.91 bar
From Mollier diagram (Fig. 6.7), we get
h1 = 3025 kJ/kg, h2 = 2880 kJ/kg, v2 = 0.2 m3/kg, h3 = 2700 kJ/kg, vs3′ = 0.4 m3/kg, and x3′ = 0.995
Thus, c2 = 2( h1 − h2 )

= 2(3025 − 2880) × 103


= 538.5 m/s
Ac
ṁ = 2 2
v2
30 × 10 −4 × 538.5
=
0.2
= 8.077 kg/s

c3 = 2h n ( h2 − h3 ) + c22 = 2 × 0.9 ( 2880 − 2700) × 103 + (538.5)2 = 783.6 m/s

h, kJ/kg
20 bar
1
3025 300°C 10.91 bar

2880 2
Dry 4.5 bar
satu
rate
d
2700
3 3′ x3′ = 0.995
s

Figure 6.7  Superheated steam flow through a nozzle

M06_THERMAL ENGINEERING_SE_XXXX_CH06.indd 262 6/15/2018 3:18:38 PM


Flow Through Steam Nozzles 263

v3 = x3′ vs3′ = 0.995 × 0.4 = 0.398 m3/kg


 3′ 8.077 × 0.398
mv
A3 = = = 4.1 × 10–3 m2 or 41 cm2
c3 783.6

Example 6.3
Dry saturated steam at 3 bar is expanded through a convergent nozzle to 1.5 bar. The exit area
is 2.5 cm2. Calculate the exit velocity and mass flow rate, assuming (a) isentropic expansion,
(b) supersaturated flow, and (c) degree of under-cooling at exit.

Solution
(a) At p1 = 3 bar
s1 = 6.992 kJ/kg.K, h1 = 2724.3 kJ/kg, v1 = 0.6058 m3/kg
At p2 = 1.5 bar
s1 = s2 = sf 2 + x2 sfg2
6.9918 = 1.4335 + x2 × 5.7897
x2 = 0.96
h2 = hf2 + x2hfg2 = 467.08 + 0.96 × 2226.5 = 2604.52 kJ/kg
v2 = vf2 + x2 (vg2 − vf2) = 0.001053 + 0.96 × (1.159 − 0.001053) = 1.113 m3/kg

c2 = 2( h1 − h2 ) = 2 ( 2725.3 − 2604.52) × 103 = 491.49 m/s

A 2 c2 2.5 × 10 −4 × 491.49
 ṁ = = = 0.11 kg/s
v2 1.113
(b) For supersaturated flow, pv1.3 = const.
1
  n −1   2
  2n    p2  n  
c2 =    p1v1 1 −   
 n − 1   p1  
  
1
  0.3   2
 2 × 1.3    1 .5  1.3  
c2 =  × 3 × 105 × 0.6058 1 −     = 482.52 m/s
 0.3    3  

  
1 1
 p n  3  1.3
v2 = v1  1  = 0.6058   = 1.0325 m3/kg
 p2   1.5 

2.5 × 10 −4 × 482.52
ṁ = = 0.117 kg/s
1.0325

M06_THERMAL ENGINEERING_SE_XXXX_CH06.indd 263 6/15/2018 3:18:41 PM


264 Chapter 6

n −1
 p  n
(c) T2 = T1  1 
 p2 
T1 = 273 + 133.55 = 406.55 K
0.3
 1.5  1.3
T2 = 406.55   = 346.45 K or 73.45°C
 3
Saturation temperature at 1.5 bar = 111.37°C
Degree of under cooling at exit = 111.37 − 73.45 = 37.92°C.

Example 6.4
Steam at a pressure of 10 bar, 0.96 dry is expanded through a convergent–divergent nozzle and
leaves the nozzle at 0.3 bar.
(a) Calculate the velocity of steam at throat for maximum discharge. Take n = 1.134.
(b) Calculate the exit area and steam discharge if the throat area is 1.5 cm2. Assume isentropic
flow and ignore friction losses.
Solution
The nozzle is shown in Fig. 6.8.
(a) v1 = vf1 + x1 (vg1– vf1) = 0.001127 + 0.96 × (0.19444 − 0.001127) = 0.1867 m3/kg
n 1.135
 2  n −1  2  0.135
For maximum discharge, p2 = p1  = 10  = 5.774 bar
 n + 1 
 2.135 

 2n 
Velocity of steam at throat, c2 =  pv
 n + 1 1 1
1
 2 × 1.135  5 2
c2 =   × 10 × 10 × 0.1867

 2.135 
= 445.54 m/s

1 2 3

p1 p2 p3
c1 c2 c3
v1 v2 v3
A1 A2 A3

Inlet Throat Exit

Figure 6.8  Convergent–divergent nozzle

M06_THERMAL ENGINEERING_SE_XXXX_CH06.indd 264 6/15/2018 3:18:44 PM


Flow Through Steam Nozzles 265

(b) For isentropic flow,


s1 = sf1 + x1sfg1 = 2.1386 + 0.96 × 4.4478 = 6.4085 kJ/kg
s1 = s2
sf1 + x1sfg1 = sf2 + x2sfg2

p(bar) vf (m3/kg) vg (m3/kg) hf (kJ/kg) hfg (kJ/kg) sf (kJ/kg.K) sfg (kJ/kg.K)


5.5 0.001097 0.3427 655.9 2097.0 1.8972 4.8920
6.0 0.001101 0.3157 670.54 2086.3 1.9311 4.8289

At p2 = 5.774 bar

(0.001101 − 0.001097)
vf = 0.001097 + × (5.774 − 5.5) = 0.001099 m3/kg
0.5
 670.54 − 655.9 
hf = 655.9 + 
   (5.774 − 5.5) = 663.92 kJ/kg
 0.5
 2086.3 − 2097.0 
hfg = 2097.0 +   (5.774 − 5.5) = 2091.14 kJ/kg
 0.5
 1.9311 − 1.8972 
sf = 1.8972 +   (5.774 − 5.5) = 1.9157 kJ/kg.K
 0.5
 4.8289 − 4.8920 
sfg = 4.8920 +   (5.774 − 5.5) = 4.857 kJ/kg.K
 0.5
6.4085 = 1.9157 + x2 × 4.857
x2 = 0.925
h2 = hf2 + x2hfg2 = 663.92 + 0.925 × 2091.14 = 2598.22 kJ/kg
h1 = hf1 + x1hfg1 = 762.79 + 0.96 × 2015.3 = 2697.5 kJ/kg

c2 = 2( h1 − h2 ) = 2( 2697.5 − 2598.22) × 103 = 454.6 m/s


v2 = vf2 + x2 (vg2 − vf2) = 0.001099 + 0.925 × (0.3279 − 0.001099) = 0.3034 m3/kg

A 2 c2 1.5 × 10 −4 × 445.54
(ṁ)max = = = 0.22 kg/s
v2 0.3034
At p3 = 0.8 bar, sf3 = 1.233 kJ/kg.K, sfg3 = 6.202 kJ/kg.K
s2 = s3
sf2 + x2 sfg2 = sf3 + x3 sfg3
1.9157 + 0.925 × 4.857 = 1.233 + x3 × 6.202
x3 = 0.833
h3 = hf3 + x3hfg3 = 391.7 + 0.833 × 2274.1 = 2286 kJ/kg

M06_THERMAL ENGINEERING_SE_XXXX_CH06.indd 265 6/15/2018 3:18:47 PM


266 Chapter 6

c3 = 2( h1 − h3 ) = 2( 2697.5 − 2286) × 103 = 907.2 m/s


v3 = x3vg3 = 0.833 × 2.087 = 1.732 m3/kg
 3 0.226 × 1.732
mv
A3 = = = 4.315 × 10–4 m2 or 4.315 cm2
c3 907.2

Example 6.5
A convergent–divergent nozzle is supplied with steam at 10 bar and 250°C. The divergent portion of
the nozzle is 4 cm long and throat diameter is 6 mm. Find the semi-cone angle of the divergent sec-
tion so that steam may leave the nozzle at 1.2 bar. The frictional loss in the nozzle is 10 percent of the
total enthalpy drop. Assume that the frictional loss occurs only in the divergent part of the nozzle.
Solution
Assuming maximum discharge, the throat pressure,
n
 2  n −1
p2 = p1 
 n + 1
For superheated steam, n = 1.3
1.3
 2  0.3
p2 = 10  = 5.457 bar
 2.3 

The nozzle and the h − s (Mollier) diagram is shown in Fig. 6.9. Locate point ‘1’ on the Mollier dia-
gram corresponding to p1 = 10 bar and 250°C. Draw a vertical line from point ‘1’ to cut the p2 line at
point ‘2’ and p3 line at point 3.
Length (1 − 4) = 0.9 × Length (1 − 3)
Draw horizontal line 4 − 3′ to cut p3 line at point 3′. Point 3′ gives the exit condition of steam.
From the Mollier diagram, we have
p1 = 10 bar: h1 = 2940 kJ/kg
p2 = 5.457 bar: h2 = 2830 kJ/kg, v2 = 0.4 m3/kg
p3 = 1.2 bar: h3 = 2530 kJ/kg, x3′ = 0.955, vs3′ = 1.5 m3/kg
h1 − h3 = 2940−2530 = 410 kJ/kg
h4 − h3 = 0.1 × 410 = 41 kJ/kg
c2 = 2 ( h1 − h2 ) = 2 ( 2940 − 2830) × 103 = 469 m/s

A 2 c2 p (6 × 10 −3 )2 × 469
ṁ = = × = 0.03315 kg/s
u2 4 0.4

c3 = 2 × 0.9 ( h2 − h3 ) + c22 = 2 × 0.9 ( 2830 − 2530) × 103 + 4692


= 871.76 m/s
v3′ = x3′ vs3′ = 0.955 × 1.5 = 1.4325 m3/kg

M06_THERMAL ENGINEERING_SE_XXXX_CH06.indd 266 6/15/2018 3:18:50 PM


Flow Through Steam Nozzles 267

1 2 3

Inlet Throat Exit

p = 10 bar p = 1.2 bar


h (kj/kg)
10 bar

2940 1
5.457 bar
250°C

2 1.2 bar
2830

4
h4
h3 3′
3
x3′ = 0.995

Figure 6.9  Superheated steam flow through a convergent–divergent nozzle

 3′ 0.03315 × 1.4325
mv
A3 = = = 5.447 × 10–5 m2
c3 871.76
p 2
d = 5.447 × 10–5
4 3
d3 = 8.33 × 10–3 m or 8.33 mm
D − d 8.33 − 6 2.33
tan q = = = = 0.02991
2L 2 × 40 80
 q = 1.67°
 2q = 3.34°

Example 6.6
Dry saturated steam at pressure of 5 bar flows through a convergent–divergent nozzle at the rate of
4 kg/s and discharges at a pressure of 1.5 bar. The loss due to friction occurs only in the diverging
portion of the nozzle and its magnitude is 15% of the total isentropic enthalpy drop. Assume the
isentropic index of expansion n = 1.135. Determine the area of cross-section at the throat and exit
of the nozzle.

M06_THERMAL ENGINEERING_SE_XXXX_CH06.indd 267 6/15/2018 3:18:52 PM


268 Chapter 6

Solution
Refer to Fig. 6.10.
For maximum discharge,
n
p2  2  n −1
= 
p1  n + 1
1.135
 2  0.135
p2 = 5  = 2.887 bar
 1.135 + 1
From steam table, we get
h1 = 2748.7 kJ/kg, s1 = 6.8212 kJ/kg.K
Now, s1 = s3 = sf3 + x3sfg3
6.8212 = 1.4335 + x3 × 5.7997
x3 = 0.93
h3 = hf 3 + x3hfg3 = 467.08 + 0.93 × 2226.5 = 2537.7 kJ/kg
(h1 − h3)isen = 2748.7 − 2537.7 = 211 kJ/kg
h1 − h3′ = h1 − h4 = 0.85 × 211 = 179.35 kJ/kg
From Mollier diagram (Fig. 6.10), x2 = 0.965, x3′ = 0.945, h2 = 2662 kJ/kg
vs2 = 0.7 m3/kg, vs3′ = 1.1 m3/kg, h3′ = 2585 kJ/kg
v2 = x2vs2 = 0.965 × 0.7 = 0.676 m3/kg
v3′ = x3′vs3′ = 0.945 × 1.1 = 1.04 m3/kg

c2 = 2( h1 − h2 ) = 2( 2748.7 − 2662) × 103 = 416.4 m/s


 2′ 4 × 0.676
mv
A2 = = = 6.49 × 10–3 m3 or 64.9 cm2
c2 416.4

c3′ = 2( h2 − h3′ ) + c22 = 2( 2662 − 2585) × 103 + ( 416.4)2 = 572.2 m/s


 3′ 4 × 1.04
mv
A3 = = = 7.27 × 10–3 m2 or 72.7 cm2
c3′ 572.2

h (kJ/kg) 5 bar
2.887 bar
1
h1 1.5 bar
h2 2

h3′
h3 3′
4 3
x=1
x2
x3′
s

Figure 6.10  Dry saturated steam flow through a convergent–divergent nozzle

M06_THERMAL ENGINEERING_SE_XXXX_CH06.indd 268 6/15/2018 3:18:56 PM


Flow Through Steam Nozzles 269

6.10  ❐ ISENTROPIC, ONE-DIMENSIONAL STEADY FLOW


THROUGH A NOZZLE
A nozzle with both converging and diverging section is shown in Fig. 6.11. For the control volume
shown, the following relations can be written:
First law:
dh + cdc = 0 (6.11)
Property relation
dp
Tds = dh − = 0 (6.12)
r
Continuity equation:
rAc = ṁ = const.
By logarithmic differentiation, we get
d r dA dc
+ + = 0(6.13)
r A c
Combining Eqs (6.11) and (6.12), we have

dp
dh = = − cdc
r
1
dc = − dp
rc
Substituting this in Eq. (6.13), we have

dA  d r dc  d r  dp  1
= − − =−   + dp
A  r c r  dp  rc 2
 
 
d r  d r 1  dp  1 1
        = − − = − +
r  dp c 2  r   dp  c 2 
Since the flow is isentropic,    
  dr  
dp c2
= = cs2
d r M2
and therefore,
dA dp
= (1 − M 2 )(6.14a)
A rc 2

p
p + dp
c T c + dc
T + dT
s s + ds

Control surface

Figure 6.11  One-dimensional isentropic flow through a nozzle

M06_THERMAL ENGINEERING_SE_XXXX_CH06.indd 269 6/15/2018 3:19:00 PM


270 Chapter 6

(g RT)dp cs2 dp
= (1 − M 2 ) = ⋅ (1 − M 2 )
2 g p
g pc 2 c
dA 1 dp  1 − M 2 
or   =   (6.14b)
A g p  M2 

where cs = velocity of sound


steam velocity
M = Mach number =
sonic velocity
g  = ratio of specific heats = cp/cv
This is a very significant equation, and from it, we can draw the following conclusions about the
proper shape for nozzles and diffusers:
1. For a nozzle, dp < 0. Therefore,
for a subsonic nozzle, M < 1, dA < 0, and the nozzle is converging;
for a supersonic nozzle, M > 1, dA > 0, and the nozzle is diverging.
2. For a diffuser, dp > 0. Therefore,
for a subsonic diffuser, M < 1, dA > 0, and the diffuser is diverging;
for a supersonic diffuser, M > 1, dA < 0, and the diffuser is converging.
3. When M = 1, dA = 0, which means that some velocity can be achieved only at the throat of a
nozzle or diffuser. These conclusions are summarised in Fig. 6.12.

M<1 p decreases M>1 p decreases


Subsonic A decreases Supersonic A increases

(a)

M<1 p increases M>1 p increases


Subsonic A increases Supersonic A decreases

(b)

Figure 6.12  Required area changes for (a) nozzles and (b) diffusers

6.10.1 Relationship between Actual and Stagnation Properties


c2
The relation between enthalpy h, stagnation enthalpy h0, and kinetic energy is:
2
c2
h + = h0 (6.15)
2

M06_THERMAL ENGINEERING_SE_XXXX_CH06.indd 270 6/15/2018 3:19:03 PM


Flow Through Steam Nozzles 271

For an ideal gas with constant specific heat, Eq. (6.15) can be written as:
g RT  T0 
c2 = 2 cp0 (T0 − T) = 2 − 1
g − 1  T 
Since cs2 = g RT, where g = cp0/cv0

2cs2  T0 
c2 = − 1
g − 1  T 
c2 2  T0 
= M2 = − 1
cs2 g − 1  T 
T0  g − 1 2
= 1+  M (6.16)
T  2 
For an isentropic process,
g 1
 T0  g −1 p0  T  g −1 r0
 T  = and  0  =
p T r
g
p0  (g − 1) 2  g −1
Therefore, = 1+ M  (6.17)
p  2 
1
r0  (g − 1) 2  g −1
= 1+ M  (6.18)
r  2 
p r T
Values of , , and are given as a function of M in Table 6.1 for the value of g  = 1.40.
p0 r0 T0
Table 6.1  O
 ne-dimensional insentropic compressible-flow functions for an ideal gas with constant
specific heat and molecular weight and g  = 1.4

M M* A/A* p/p0 r/r0 T/T0


0.0 0.00000 ∞ 1.00000 1.00000 1.00000
0.1 0.10944 5.82183 0.99303 0.99502 0.99800
0.2 0.21822 2.96352 0.97250 0.98028 0.99206
0.3 0.32572 2.03506 0.93947 0.95638 0.98232
0.4 0.43133 1.59014 0.89561 0.92427 0.96899
0.5 0.53452 1.33984 0.84302 0.88517 0.95238
0.6 0.63481 1.48820 0.78400 0.84045 0.93284
0.7 0.73179 1.09437 0.72093 0.79158 0.91075
0.8 0.82514 1.03823 0.65602 0.73999 0.88652
0.9 0.91460 1.00886 0.59126 0.68704 0.86059
(Continued)

M06_THERMAL ENGINEERING_SE_XXXX_CH06.indd 271 6/15/2018 3:19:08 PM


272 Chapter 6

Table 6.1  (Continued )

M M* A/A* p/p0 r/r0 T/T0


1.0 1.0000 1.00000 0.52828 0.63394 0.83333
1.1 1.0812 1.00793 0.46835 0.58170 0.80515
1.2 1.1583 1.03044 0.41238 0.53114 0.77640
1.3 1.2311 1.06630 0.36097 0.48290 0.74738
1.4 1.2999 1.11493 0.3424 0.43742 0.71839
1.5 1.3646 1.17617 0.27240 0.39498 0.68966
1.6 1.4254 1.25023 0.23527 0.35573 0.66138
1.7 1.4825 1.33761 0.20259 0.31969 0.63371
1.8 1.5360 1.43898 0.17404 0.28682 0.60680
1.9 1.5861 1.55526 0.14924 0.25699 0.58072
2.0 1.6330 1.68750 0.12780 0.23005 0.55556
2.1 1.6769 1.83694 0.10935 0.20580 0.53135
2.2 1.7179 2.00497 0.93522E-01 0.18405 0.50813
2.3 1.7563 2.19313 0.79973E-01 0.16458 0.48591
2.4 1.7922 2.40310 0.68399E-01 0.14720 0.46468
2.5 1.8257 2.63672 0.58528E-01 0.13169 0.44444
2.6 1.8571 2.89598 0.50115E-01 0.11787 0.42517
2.7 1.8865 3.18301 0.42950E-01 0.10557 0.40683
2.8 1.9140 3.50012 0.36848E-01 0.94626E-01 0.38941
2.9 1.9398 3.84977 0.31651E-01 0.84889E-01 0.37286
3.0 1.9640 4.23457 0.277224E-01 0.76226E-01 0.35714
3.5 2.0642 6.78962 0.13111E-01 0.45233E-01 0.28986
4.0 2.1381 10.7188 0.65861E-02 0.27662E-01 0.23810
4.5 2.1936 16.5622 0.34553E-02 0.17449E-01 0.19802
5.0 2.2361 25.0000 0.18900E-02 0.11340E-01 0.16667
6.0 2.2953 53.1798 0.63336E-03 0.51936E-02 0.12195
7.0 2.3333 104.143 0.24156E-03 0.26088E-02 0.09259
8.0 2.3591 190.109 0.10243E-03 0.14135E-02 0.07246
9.0 2.3772 327.189 0.47386E-04 0.81504E-02 0.05814
10.0 2.3905 535.938 0.23563E-04 0.49482E-03 0.04762
∞ 2.4495 ∞ 0.0 0.0 0.0

The conditions at the throat of the nozzle can be found by putting M = 1 at the throat. The properties
at the throat are denoted as an asterisk (*) and are referred to as critical properties.
T* 2
Therefore, = (6.19)
T0 g + 1

M06_THERMAL ENGINEERING_SE_XXXX_CH06.indd 272 6/15/2018 3:19:08 PM


Flow Through Steam Nozzles 273

g
p*  2  g −1
= (6.20)
p0 g + 1
g
p*  2  g −1
= (6.21)
r 0 g + 1
Table 6.2 gives these ratios for various values of k.

Table 6.2  Critical properties for isentropic flow of an ideal gas

Ratio g = 1.1 g = 1.2 g = 1.3 g = 1.4 g = 1.67


P*/p0 0.5847 0.5644 0.5457 0.5283 0.4867
r*/r0 0.6139 0.6209 0.6276 0.6340 0.6497
T*/T0 0.9524 0.9091 0.8696 0.8330 0.7491

6.11 ❐ MASS RATE OF FLOW THROUGH AN ISENTROPIC


NOZZLE
From the continuity equation, we have
m pc g T0 pc g T0 1
= rc = =
A RT g T0 g RT R T T0

pM g g −1 2
   = 1+ M (6.22)
T0 R 2
Substituting Eq. (6.17) in Eq. (6.22), the flow per unit area can be expressed in terms of stagnation
pressure, stagnation temperature, Mach number, and gas properties.
m p g M
     = 0 × (6.23)
A T0 R (g +1)
 g −1 
2 (g −1)
2
1 + 2 M 

m
At the throat, M = 1, and therefore, the flow per unit area at the throat, , can be found by setting
M = 1 in Eq. (6.23). A*
m p0 g 1
    = × (6.24)
(g +1)
A* T0 R
g + 1 2(g −1)
 2 
A
The area ratio can be obtained by dividing Eq. (6.24) by Eq. (6.23).
A*

M06_THERMAL ENGINEERING_SE_XXXX_CH06.indd 273 6/15/2018 3:19:12 PM


274 Chapter 6

A/A*
4
3
g = 1.4
2
1

0.5 1.0 1.5 2.0 2.5 3.0 M

Figure 6.13  Area ratio as a function of Mach number for isentropic nozzle
(g +1)
1  2   g − 1 2   2(g −1)
A
=   1 + 2 M   (6.25)
A∗ M  g + 1  
The values of A/A* as a function of M are given in Table 6.1. Fig. 6.13 depicts the variation of A/A*
with M, which shows that a subsonic nozzle is converging and a supersonic nozzle is diverging.

6.11.1 Effect of Varying the Back Pressure on Mass Flow Rate


Consider first a convergent nozzle as shown in Fig. 6.14, which also shows the pressure ratio p/p0
along the length of the nozzle. The conditions upstream are the stagnation conditions, which are
assumed to be constant. The pressure at the exit plane of the nozzle is designated pE and the back
pressure (the pressure outside the nozzle exit) pB. As the back pressure pB is decreased, the variation
of the mass flow rate and the exit plane pressure pE/p0 are plotted in Fig. 6.15.
Where pB/p0 = 1, there is, of course, no flow, and pE/p0 = 1 as designated by point 'a'. Next, let the
back pressure pB be lowered to point b, so that pB/p0 is greater than the critical pressure ratio. The
mass flow rate has a certain value and pE = pB. The exit Mach number is less than one. Next, let pB
be lowered to the critical pressure at point c. The Mach number at the exit is now unity and pE = pB.
When pB is decreased below the critical pressure, designated by point d, there is no further increase
in the mass rate of flow, and pE remains constant at a value equal to the critical pressure, and the exit
Mach number is unity. The drop in pressure from pE to pB takes place outside the nozzle exit. Under

p0
pE pB
T0

p
a
p0
b
c
d

Figure 6.14  Pressure ratio as a function of back pressure for a convergent nozzle

M06_THERMAL ENGINEERING_SE_XXXX_CH06.indd 274 6/15/2018 3:19:14 PM


Flow Through Steam Nozzles 275

pE
p0
d c
m 1.0
b a
b
d c

a pB pB
1.0 p0 1.0 p0

Figure 6.15 Mass rate of flow and exit pressure as a function of back


pressure for a convergent nozzle

c=0
p0 pE pB
T0

Throat
a
b
p c
p0 d
e

Figure 6.16 Nozzle pressure ratio as a function of back pressure


for a convergent–divergent nozzle
these conditions, the nozzle is said to be choked, which means that for given stagnation conditions, the
nozzle is passing the maximum possible mass flow.
Consider next a convergent–divergent nozzle as shown in Fig. 6.16. Point ‘a’ designated the condi-
tion when pB = p0 and there is no flow. When pB is decreased to the pressure indicated by point b, so
that pB/p0 is less than unity but considerably greater than that the critical pressure ratio, the velocity
increases in the convergent section, but M < 1 at the throat. Therefore, the diverging section acts as
a subsonic diffuser in which the pressure increases and velocity decreases. Point ‘c’ designated the
back pressure at which M = 1 at the throat, but the diverging section acts as a subsonic diffuser (with
M = 1 at the inlet) in which the pressure increases and velocity decreases. Point ‘d’ designates one
other back pressure that permits isentropic flow, and in this case, the diverging section acts as a super-
sonic nozzle with a decrease in pressure and an increase in velocity. Between the back pressure desig-
nated by points c and d, an isentropic solution is not possible, and shock waves will be present. When
the back pressure is decreased below that designated by point d, the exit pressure pE remains constant,
and the drop in pressure from pE to pB takes place outside the nozzle. This is designated by point e.

Example 6.7
A convergent nozzle has an exit area of 500 mm2. Air enters the nozzle with a stagnation pressure
of 1000 kPa and a stagnation temperature of 360 K. Determine the mass rate of flow for back pres-
sures of 800 kPa, 528 kPa, and 300 kPa, assuming isentropic flow. For air, k = 1.4.

Solution
g
p∗  2  g −1  2 
3.5
Critical pressure ratio, = =  = 0.528
p0 g + 1  2.4 

M06_THERMAL ENGINEERING_SE_XXXX_CH06.indd 275 6/15/2018 3:19:17 PM


276 Chapter 6

p* = 1000 × 0.528 = 528 kPa


Therefore, for a back pressure of 528 kPa, M = 1 at the nozzle exit and the nozzle is choked.
Decreasing the back pressure below 528 kPa will not increase the flow.
For a back pressure of 528 kPa,
T* 2 2
= = = 0.8333
T0 g + 1 2.4
T * = 360 × 0.8333 = 300 K
*
At the exit, c = cs = g RT = 1.4 × 287 × 300 = 347.2 m/s
p* 528
r* = = = 6.1324 kg/m3
RT* 0.287 × 300
Discharge at the exit section,
ṁ = r*Ac = 6.1324 × 500 × 10–6 × 347.2 = 1.0646 kg/s
p 800
For a back pressure of 800 kPa, E = = 0.8
p0 1000
p T
From Table 6.1 for = 0.8, ME = 0.573 and E = 0.9381 by interpolation
p0 T0
TE = 360 × 0.9381 = 337.7 K
Sonic velocity at exit,
cEs = g RTE = 1.4 × 287 × 337.7 = 368.4 m/s
Velocity at exit,
cE = MEcEs = 0.573 × 368.4 = 211.1 m/s
Density of air at exit,
pE 800
rE = = = 8.2542 kg/m3
RTE 287 × 337.7
ṁ = rEAEcE = 8.2542 × 500 × 10–6 × 211.1 = 0.8712 kg/s
For a back pressure less than the critical pressure (528 kPa), the nozzle is choked and the mass rate
of flow is the same as that for the critical pressure. Therefore, for an exhaust pressure of 300 kPa, the
mass flow rate is 1.0646 kg/s.
Example 6.8
A converging–diverging nozzle has an exit area to throat area ratio of 2. Air enters this nozzle
with a stagnation pressure of 1000 kPa and a stagnation temperature of 360 K. The throat area is
500 mm2. Determine the mass rate of flow, exit pressure, exit temperature, exit Mach number, and
exit velocity for the following conditions:
(a) Sonic velocity at the throat, diverging section acting as a nozzle.
(b) Sonic velocity at the throat, diverging section acting as a diffuser.
Solution
(a) From Table 6.1, we have
A p T
For E = 2, M*E = 2.197, E = 0.0939, E = 0.5089
* p0 T0
A

M06_THERMAL ENGINEERING_SE_XXXX_CH06.indd 276 6/15/2018 3:19:23 PM


Flow Through Steam Nozzles 277

Therefore, pE = 0.0939 × 1000 = 93.9 kPa


TE = 0.5089 × 360 = 183.2 K
csE = g RTE = 1.4 × 287 × 183.2 = 271.3 m/s
cE = ME* csE = 2.197 × 271.3 = 596.1 m/s
Critical pressure at throat, p* = p0 × 0.528 = 1000 × 0.528 = 528 kPa
Critical temperature, T * = 0.8333 × 360 = 300 K
At throat, c = csE = g RT* = 1.4 × 287 × 300 = 347.2 m/s
p* 528
r* = * = = 6.1324 kg/m3
RT 0.287 × 300

ṁ = r∗Ac = 6.1324 × 500 × 10–6 × 347.2 = 1.0646 kg/s


(b) From Table 6.1, we have
A p T
For E = 2, ME = 0.308, E = 0.936, E = 0.9812
A* p0 T0
Therefore, pE = 0.936 × 1000 = 936 kPa
 TE = 0.9812 × 360 = 353.3 K
 csE = g RTE = 1.4 × 287 × 353.3 = 376.8 m/s
 cE = MCsE = 0.308 × 376.8 = 116 m/s
Since M = 1 at the throat, mass rate of flow is the same as in (a).

6.12  ❐ NORMAL SHOCK IN AN IDEAL GAS FLOWING


THROUGH A NOZZLE
A shock wave involves an extremely rapid and abrupt change of state. In a normal shock, this change
of state takes place across a plane normal to the direction of flow. Figure 6.17 shows a control surface
that includes such a normal shock. Let subscripts x and y denote the conditions upstream and down-
stream of shock, respectively, and assuming steady-state, steady-flow with no heat and work transfer
across the control surface, then the various relations are as follows:
2
cx2 cy
= hy + (6.26)
First law: hx +
2 2
hox = hoy
or    

Control surface
Normal shock

cx cy

Figure 6.17  One-dimensional normal shock

M06_THERMAL ENGINEERING_SE_XXXX_CH06.indd 277 6/15/2018 3:19:27 PM


278 Chapter 6

Continuity equation:
m
= r x cx = r y c y (6.27)
A
Momentum equation:
A(px − py) = ṁ (cy − cx)(6.28)
Second law:
sy − sx ≥ 0 (6.29)
The energy and continuity equations can be combined to establish an equation that when plotted on
the h − s diagram is called the Fanno line. Similarly, the momentum and continuity equations can be
combined to establish an equation the plot of which on the h − s diagram is known as the Rayleigh line.
Both these lines are shown on the h − s diagram in Fig. 6.18. The point of maximum entropy on each
line, points ‘a’ and ‘b’, corresponds to M = 1. The lower part of each line corresponds to supersonic
flow, whereas the upper part corresponds to subsonic flow.
The two points where all three equations are satisfied are points x and y, where x being in the super-
sonic region and y in the subsonic region. Since sy − sx ≥ 0, the normal shock can proceed only from
x to y. This means that the velocity changes from supersonic (M > 1) before the shock to subsonic
(M < 1) after the shock.
Assuming constant specific heats, the energy Eq. (6.26) gives,
Tox = Toy(6.30)
That is, there is no change in stagnation temperature across a normal shock. Using Eq. (6.16), we have
Tox g −1 2
= 1+ Mx
Tx 2
Toy g −1 2
= 1+ My
Ty 2

and substituting into Eq. (6.30), we have


g −1 2
Ty 1+ Mx
  = 2 (6.31)
Tx g −1 2
1+ My
2
h
hox hoy
hox = hoy
cy2
ox

y
po
p

2 M < 1 above points a and b


b M > 1 below points a and b
a
cx2 M = 1 at a and b
2
Rayleigh line

x Fanno line
hx
s

Figure 6.18  End states for a one-dimensional normal shock on an enthalpy–entropy diagram

M06_THERMAL ENGINEERING_SE_XXXX_CH06.indd 278 6/15/2018 3:19:30 PM


Flow Through Steam Nozzles 279

The continuity equation is,


rxcx = rycy
px py
But     rx = , ry =
RTx RTy

Ty py cy p y M y csy p y M y Ty
= = =
Tx px c x px M x csx px M x Tx

2 2
Ty  p y   M y 
or    = (6.32)
Tx  px   M x 

Combining energy Eq. (6.31) and continuity Eq. (6.32) gives the equation of the Fanno line.

g −1 2
py Mx 1+ Mx
= 2 (6.33)
px g −1 2
My 1+ My
2

The momentum and continuity equations can be combined as follows to give the equation of the
Rayleigh line.
m
px – py = (c – c ) = r y c 2y − r x cx2
A y x
px + rx cx2 = py + ry c 2y

px + rx M 2 c 2 = p + r M 2 c 2
x sx y y y sy

px M 2x p y M 2y
px + (g RTx ) = p y + = (g RTy )
RTx RTy

px (1 + g M 2x ) = p y (1 + g M 2 )

py 1 + g M2
  = (6.34)
px 1 + g M 2y
Combining Eqs (6.33) and (6.34), we get
2
M 2x +
g −1
  M 2y = (6.35)
2g
M 2x − 1
g −1
Table 6.3 gives the normal shock functions, which includes My as a function of Mx for g = 1.4.

M06_THERMAL ENGINEERING_SE_XXXX_CH06.indd 279 6/15/2018 3:19:35 PM


280 Chapter 6

Table 6.3  O
 ne-dimensional normal shock functions for an ideal gas with constant specific heat
and molecular weight and g = 1.4

Mx My py/px ry/rx Ty/Tx p0y/p0x poy/px


1.00 1.00000 1.0000 1.0000 1.0000 1.00000 1.8929
1.10 0.91177 1.2450 1.1691 1.0649 0.99893 2.1328
1.20 0.84217 1.5133 1.3416 1.1280 0.99280 2.4075
1.30 0.78596 1.8050 1.5157 1.1909 0.97937 2.7136
1.40 0.73971 2.1200 1.6897 1.2547 0.95819 3.0492
1.50 0.70109 2.4583 1.8621 1.3202 0.92979 3.4133
1.60 0.66844 2.8200 2.0317 1.3880 0.89520 3.8050
1.70 0.64054 3.2050 2.1977 1.4583 0.85572 4.2238
1.80 0.61650 3.6133 2.3592 1.5316 0.81268 4.6695
1.90 0.59562 4.0450 2.5157 1.6079 0.76736 5.1418
2.00 0.57735 4.5000 2.6667 1.6875 0.72087 5.6404
2.10 0.56128 4.9783 2.8119 1.7705 0.67420 6.1654
2.20 0.54706 4.4800 2.9512 1.8569 0.62814 6.7165
2.30 0.53441 6.0050 3.0845 1.9468 0.58329 7.2937
2.40 0.52312 6.5533 3.2119 2.0403 0.54014 7.8969
2.50 0.51299 7.1250 3.3333 2.1375 0.49901 8.5261
2.60 0.50387 7.7200 3.4490 2.2383 0.46012 9.1813
2.70 0.49563 8.3383 3.5590 2.3429 0.42359 9.8624
2.80 0.48817 8.9800 3.6636 2.4512 0.38946 10.569
2.90 0.48138 9.6450 3.7629 2.5632 0.35773 11.302
3.00 0.47519 10.333 3.8571 2.6790 0.32834 12.061
4.00 0.43496 18.500 4.5714 4.0469 0.13876 21.068
5.00 0.41523 29.000 5.0000 5.8000 0.06172 32.653
10.00 0.38758 116.50 5.7143 20.387 0.00304 129.22
∞ 0.37796 ∞ 6.0000 ∞ 0.0 ∞

Example 6.9
For the convergent–divergent nozzle of Example 6.8 in which the diverging section acts as a super-
sonic nozzle (Fig. 6.19), a normal shock stands in the exit plane of the nozzle. Determine the static
pressure and temperature and the stagnation pressure (a) just downstream of the normal shock and
(b) at a point where M = 1.4.
Solution
(a) From Table 6.3, we have
py Ty po y
Mx = 2.197, My = 0.547, = 4.46, =1.854, = 0.630
px Tx pox

M06_THERMAL ENGINEERING_SE_XXXX_CH06.indd 280 6/15/2018 3:19:36 PM


Flow Through Steam Nozzles 281

Normal shock
pOX = 1000 kPa
TOX = 360 K

Mx = 2.197
pox = 1000 kPa
px = 93.9 kPa
Tx = 183.2 K

Figure 6.19  Convergent–divergent nozzle

py = 4.46 × 93.9 = 512.7 kPa


Ty = 1.854 × 183.2 = 339.7 K
poy = 0.630 × 1000 = 630 kPa
(b) From Table 6.1 at point x, as the flow is isentropic to point x,
px T A
Mx = 1.5, = 0.2724, x = 0.6897, x = 1.1762
pox Tox A*x
Therefore, px = 0.2724 × 1000 = 272.4 kPa
Tx = 0.6897 × 360 = 248.3 K
The properties at y can be determined from the normal shock functions (Table 6.3) as
py Ty po y
My = 0.7011, = 2.4583, = 1.320, = 0.9298
px Tx pox
py = 2.4583 × 272.4 = 669.6 kPa
Ty = 1.320 × 248.3 = 327.8 K
poy = 0.9298 × 1000 = 929.8 kPa
Tox = Toy = 360 K, as there is no change in stagnation temperature across a normal shock. From
y to E, the diverging section acts as a diffuser.

Example 6.10
Steam at stagnation pressure of 800 kPa and a stagnation temperature of 350°C expands in a nozzle
to 200 kPa. When the mass flow rate is 3 kg/s, determine the throat area and exit area for isentropic
flow.
Solution
p*
Critical pressure ratio at the throat, = 0.545
p0
p* = 0.545 × 800 = 436 kPa
s* = s0 = 7.4089 kJ/kg. K
h0 = 3161.7 kJ/kg
T * = 268.7°C
h* = 3001.4 kJ/kg

M06_THERMAL ENGINEERING_SE_XXXX_CH06.indd 281 6/15/2018 3:31:18 PM


282 Chapter 6

c *2
h0 = h* +
2
c* = 2( h0 − h*) = 2(3161.7 − 3001.4) × 103 = 566.2 m/s
v*= 0.5724 m3/kg
 * 3 × 0.5724
mv
A* = = = 3.033 × 10–3 m2 or 30.33 cm2
c* 566.2
At the nozzle exit,
pE = 200 kPa, sE = s0 = 7.4089 kJ/kg.K
TE = 178.5°C
hE = 2826.7 kJ/kg
vE = 1.0284 m3/kg

cE = 2( h0 − hE ) = 2(3161.7 − 2826.7) × 103 = 818.5 m/s

 E 3 × 1.0284
mv
AE = = = 3.769 × 10–3 m3 or 37.69 cm2
cE 818.5

Example 6.11
5 kg/s of steam at 30 bar and 350°C is supplied to a group of 6 nozzles in a wheel diameter main-
tained at 4 bar. Calculate the following:
(a) The dimensions of the nozzles of rectangular cross-sectional flow area with aspect ratio of
2.5:1. The expansion may be considered metastable and friction is neglected.
(b) Degree of undercooling and supersaturation
(c) Loss in available heat drop due to irreversibility
(d) Increase in entropy
(e) Ratio of mass flow rate with metastable expansion to that if expansion is in thermal equilibrium.
Solution
The h-s diagram is shown in Fig. 6.20.
h1 = 3115.3 kJ/kg
v1 = 0.09053 m3/kg
s1 = 6.74257 kJ/kg.K
s3 = s1 = sf3 + x3sfg3
6.7427 = 1.7766 + x3 × 5.1193
x3 = 0.97
h3 = hf3 + x3hfg3
= 604.73 + 0.97 × 2133.8
= 2674.5 kJ/kg
v3 = vf3 + x3(vg3– vf3) = 0.0010840.97 × (0.4625 – 0.001084)
= 0.45 m3/kg

M06_THERMAL ENGINEERING_SE_XXXX_CH06.indd 282 6/15/2018 3:31:20 PM


Flow Through Steam Nozzles 283

h (kJ/kg)

3115.3 30 bar
350° C

4 bar

2 2′
2674.5
3

Figure 6.20  Mollier diagram for superheated steam

(a) For supersaturated steam, n = 1.3


 n −1 
 n    p2  n 
h1 − h2 =  p v 1−
 n − 1 1 1   p1  
 
 
 0.3 
1.3   4  1 .3 
= × 30 × 105 × 0.09053 × 10−3 1 −   
0.3  30 
 
= 437.5 kJ/kg

c2 = 2( h1 − h2 ) = 2 × 437.5 × 103 = 935.4 m/s

1 1
 30  1.3
v2 = v1  p1  n = 0.09053   = 0.4263 m3/kg
 p   4
2
A 2 c2
ṁ =
v2
5 × 0.4263
A2 = = 2.279 × 10−3 m2 or 22.79 cm2
935.4
Let breadth = b
Then length, l = 2.5 b
Area of 6 nozzles = 6 × 2.5b × b = 15b2
15b2 = 2.279 × 10–3
b = 0.0123 m or 12.3 mm
l = 0.0308 m or 30.8 mm
n −1 0.3
p  n  4  1.3
(b) T2 = T1  2  = (273 + 350)   = 391.34 K or 118.34°C
 1
p  30 
At 4 bar, ts = 143.65°C
Degree of undercooling = 143.65 − 118.34 = 24.31°C

M06_THERMAL ENGINEERING_SE_XXXX_CH06.indd 283 6/15/2018 3:31:24 PM


284 Chapter 6

ps corresponding to 118.3°C = 1.9 bar


4
Degree of super saturation = = 2.1
1.9
(c) h1 = h3 = 3115.3 − 2674.5 = 440.8 kJ/kg
Loss of available heat drop = (h1 − h3) − (h1 − h2)
   (∆h)loss = 440.8 − 437.5 = 3.3 kJ/kg

( ∆h) less 3.3 3.3


(d) Increase in entropy = = = = 0.0079 kJ/kg.K
Ts 273 + 143.6 416.6

(e) c3 = 2( h1 − h2 ) = 2 × 440.8 × 103 = 938.9 m/s

Mass flow rate for metastable flow



Mass flow rate for thermaal equilibrium flow
Area of flow × c2 v3
=
×
v2 Area of flow × c3
c2 v2 935.4 × 0.45
= = = 1.05
v2 c3 0.4263 × 938.9

Example 6.12
Dry saturated steam at 5 bar with negligible velocity expands isentropically in a convergent nozzle
to 1 bar and dryness fraction 0.94. Determine the velocity of steam leaving the nozzle.
Solution
Given that p1 = 5 bar; p2 = 1 bar; x2 = 0.94
For p1 = 5 bar, enthalpy of dry saturated steam from steam tables,
h1 = hg1 = 2748.7 kJ/kg
and for p2 = 1 bar, we find that,
hf 2 = 417.44 kJ/kg, hfg2 = 2258.0 kJ/kg
∴  h2 = hf2 + x2 hfg2
= 417.44 + 0.94 × 2258.0 = 2539.96 kJ/kg
Enthalpy drop, ∆h = h1 − h2
= 2748.75 − 2539.96 = 208.74 kJ/kg
Velocity of steam leaving the nozzle
c2 = 44.72 ∆h = 44.72 208.74 = 646.1 m/s

Example 6.13
Dry saturated steam at a pressure of 15 bar enters in a nozzle and is discharged at a pressure of 1.5
bar. Find the final velocity of steam when initial velocity of steam is negligible. If 15% of the heat
drop is lost in friction, then find the percentage reduction in the final velocity.

M06_THERMAL ENGINEERING_SE_XXXX_CH06.indd 284 6/15/2018 3:31:27 PM


Flow Through Steam Nozzles 285

Solution
For p1 = 15 bar from steam tables,
h1 = 2792.1 kJ/kg
and for p2 = 1.5 bar,
h2 = 2693.5 kJ/kg
Enthalpy drop, ∆h = h1 − h2
= 2792.19 − 2693.5 = 98.6 kJ/kg
Final velocity of steam,
c2 = 44.72 ∆h = 44.72 98.6 = 444.06 m/s
Percentage reduction in the final enthalpy = 15% = 0.15
Nozzle coefficient or nozzle efficiency, hn = 1 − 0.15 = 0.85
Final velocity of steam,
c2 = 44.72 h n × ∆h

= 44.72 0.85 × 98.6


c2 = 409.4 m/s
Percentage reduction in final velocity
444.06 − 409.4
= × 100 = 7.8%
444.06

Example 6.14
Steam approaches a nozzle with velocity of 250 m/s at a pressure of 3.5 bar and dryness fraction
0.95. If the isentropic expansion in the nozzle proceeds till the pressure at exit is 2 bar, then deter-
mine the change of enthalpy and dryness fraction of steam. Calculate also the exit velocity from
the nozzle and the area of exit of nozzle for flow of 0.75 kg/s.
Solution
Given that c1 = 250 m/s, p1 = 3.5 bar, x1 = 0.95, p2 = 2 bar, ṁ = 0.75 kg/s
From Mollier diagram, as shown in Fig. 6.21,

r
h (kJ/kg) ba
.5
=3 ba
r
p
=2
p
1 Dr
2624 y
sa
tu
ra
te
2534 d
2 lin
e
x1 = 0
x2 = 0.92 .95
s

Figure 6.21  Mollier diagram

M06_THERMAL ENGINEERING_SE_XXXX_CH06.indd 285 6/15/2018 3:31:29 PM


286 Chapter 6

h1 = 2624 kJ/kg
h2 = 2534 kJ/kg
∆h = h1 − h2 = 2624 − 2534 = 90 kJ/kg
Dryness fraction, x2 = 0.92
From steam tables, vf2 = 0.001061 m3/kg, vg2 = 0.8857 m3/kg at p2 = 2 bar
Specific volume at exit
 v2 = vf2 + x2 (vg2 − vf2) = 0.001061 + 0.92 × (0.8857 − 0.001061) = 0.8149 m3/kg

c22 − c12
Now,      = h1 − h2
2 × 103

c22 − ( 250)2
= 90
2 × 103
c22 = 90 × 2 × 103 + (250)2

c2 = 90 × 2000 + ( 250)2 = 492.4 m/s


Steam mass flow rate (ṁ ) is given by,
A 2 c2
ṁ =
v2
A 2 × 492.4
ṁ =
0.8149
A 2 × 492.4
0.75 =
0.8146
A2 = 0.00124 m2 = 12.4 cm2
Example 6.15
Dry saturated steam at a pressure of 6 bar flows through a convergent–divergent nozzle at a rate of
4.5 kg/s and discharges at a pressure of 1.6 bar. The loss due to friction occurs only in the diverging
portion of nozzle and its magnitude is 12% of total isentropic enthalpy drop. Assuming the isen-
tropic index of expansion, n = 1.135, determine the cross-sectional area at throat and exit of nozzles.

Solution
Given that p1 = 6 bar, ṁ = 4.5 kg/s, p3 = 1.6 bar, n = 1.135, hn = 1 − 0.12 = 0.88
From Mollier diagram shown in Fig. 6.22,
h1 = 2750 kJ/kg, h3 = 2520 kJ/kg
Refer to h − s diagram (Mollier chart) as shown in Fig. 6.22.
Let suffix 1, 2 and 3 denote entrance, throat, and exit.
n / n −1 1.135 /.135
p2  2   2 
We have =  = = 0.578
p1  n + 1  2.135 
p2 = 6 × 0.578 = 3.468 bar and
h2 = 2650 kJ/kg

M06_THERMAL ENGINEERING_SE_XXXX_CH06.indd 286 6/15/2018 3:31:32 PM


Flow Through Steam Nozzles 287

ar
h (kJ/kg) 6b
p= bar
.4 68
p =3
1 ar
2750 1. 6b
Dr
y p=
sa
2 tu
2650 ra
te
d
lin
3′ e
2520
3
s

Figure 6.22  Mollier diagram

For Section 1–2:


h1 − h2 = 2750 − 2650 = 100 kJ/kg
c2 = 44.72 100 = 447.2 m/s
A2 = Area of throat
v2 = 0.964 m3/kg
From Mollier diagram, we have
vg2 = 0.5321 m3/kg corresponding to p2 = 3.468 bar

v2 = x2 vg2 = 0.964 × 0.5321 = 0.513 m3/kg


A 2 c2
ṁ =
v2
A 2 × 447.2
4.5 =
0.513
A2 = 0.005162 m2 = 51.62 cm2
For Section 2–3:
(h1 − h3) isentropic = 230 kJ/kg

h1 − h3′ = (h1 − h3) × hn = 230 × 0.88 = 202.4 kJ/kg

Velocity at exit, c3 = 2 × 103 ( h1 − h3′ )

= 44.72 202.4 = 636.24 m/s


From Mollier diagram,
x3′ = 0.9365
vg3 = 1.0911 m3/kg corresponding to pressure of 1.6 bar.
 v3 = x3′ vg3 = 0.9365 × 1.0911 = 1.022 m3/kg
A 3c3
ṁ =

v3

M06_THERMAL ENGINEERING_SE_XXXX_CH06.indd 287 6/15/2018 3:31:37 PM


288 Chapter 6

A 3 × 636.24
4.5 =
1.022
A3 = 0.007228 m2 = 72.28 cm2

Example 6.16
Dry saturated steam enters a nozzle at a pressure of 10 bar and velocity of 100 m/s. The discharge
pressure is 5 bar and discharge velocity is 500 m/s. Heat loss from the nozzle is 5 kJ/kg. Determine
the final dryness fraction of steam.
Solution
Given that p1 = 10 bar, c1 = 100 m/s, p2 = 5 bar, c2 = 500 m/s, q = −5 kJ/kg
Using steady flow energy equation
c12 c2
h1 + + q = h2 + 2 + w
2000 2000
Since w = 0

c22 − c12
h1 − h2 + q =
2000
At p1 = 10 bar, x1 = 1.0, h1 = 2778.1 kJ/kg
500 2 − 100 2
2778.1 − h2 − 5 =
2 × 103
h2 = 2653.1 kJ/kg
At 5 bar; hf2 = 640.21 kJ/kg, hfg2 = 2108.5 kJ/kg
h2 = hf2 + x2 h f g
2

2653.1 = 640.21 + x2 × 2108.5


2653.1 − 640.21
x2 = = 0.955
2108.5
Example 6.17
Steam enters a group of nozzles of a steam turbine at 12 bar and 220°C and leaves at 1.2 bar. The
steam turbine develops 220 kW with a specific steam consumption of 13.5 kg/kWh. If the diameter
of nozzles at throat is 7 mm, then calculate the number of nozzles.

Solution
Given that p1 = 12 bar, t1 = 220°C, p3 = 1.2 bar, power developed = 220 KW, m3 = 13.5 kg/kWh, and
d2 = 7 mm
Refer to Mollier diagram, as shown in Fig. 6.23.
The steam is initially superheated.
n = 1.3 and p2/p1 = 0.5457
p2 = 0.5457 p1 = 0.5457 × 12 = 6.548 bar
From Mollier diagram, we have
h1 = 2860 kJ/kg, h2 = 2750 kJ/kg, x2 = 0.992

M06_THERMAL ENGINEERING_SE_XXXX_CH06.indd 288 6/15/2018 3:31:39 PM


Flow Through Steam Nozzles 289

bar
12
h (kJ/kg) p=

1
2680 ar
4 8b
6.5
p=
bar
1.2
p=
2750 2 Dr
y
sa
tu
ra
te
3 x2 d
s

Figure 6.23  Mollier diagram

vg2 = 0.29 m3/kg


Δh2 = h1 − h2 = 2860 − 2750 = 110 kJ/kg

c2 = 44.72 ∆h2

= 44.72 110 = 470 m/s


p
A2 = Area of throat = (p /4)d22 = (7 × 10–3)2 = 38.48 × 10–6 m2
Mass flow rate per nozzle 4

A 2 c2 A c 38.48 × 10 −6 × 470
= = 2 2 = = 0.0629 kg/s
v2 x2 v g 2 0.992 × 0.29
Total mass flow rate,
13.5 × 220
ṁ = = 0.825 kg/s
3600
Total mass flow rate 0.825
Number of nozzles = = = 13.1 say 14
Mass flow rate per nozzle 0.0629

Example 6.18
Dry saturated steam at a pressure of 8 bar enters a convergent–divergent nozzle and leaves it at a
pressure of 1.5 bar. If the flow is isentropic and the corresponding expansion index is 1.135, then
find the ratio of cross-sectional area at exit and throat for maximum discharge.

Solution
Given that p1 = 8 bar, p3 = 1.5 bar, n = 1.135
Let A2 = Area of cross-section at throat
A3 = Area of cross-section at exit
m = mass of steam discharged per second
Refer to Mollier diagram as shown in Fig. 6.24.

M06_THERMAL ENGINEERING_SE_XXXX_CH06.indd 289 6/15/2018 3:31:43 PM


290 Chapter 6

p1 = 8 bar
h p2 = 4.619 bar
A
h1
p3 = 1.5 bar

h2 B
Dry saturated
h3 x2 = 0.965
C
x3 = 0.982
s

Figure 6.24  Mollier diagram

For dry saturated steam, n = 1.135


p2
= 0.5774
p1
p2 = 8 × .5774 = 4.619 bar
From Mollier chart, h1 = 2775 kJ/kg
h2 = 2650 kJ/kg

h3 = 2465 kJ/kg
x2 = 0.965, x3 = 0.902
From steam tables, vg2 = 0.405 m3/kg, v g2
′ = 1.159 m3/kg
  Δh2 = h1 − h2 = 2775 − 2650 = 125 kJ/kg

 c2 = 44.72 ∆h2 = 44.72 125 = 500 m/s


  
A c
    ṁ = 2 2 [Neglecting vf 2]
x2 v g 2

 2 vg 2
mx m × 0.965 × 0.405
A2 = =
c2 500
A2 = 0.000786 ṁ
Δh3 = h1 – h3 = 2775 − 2465 = 310 kJ/kg

c3 = 44.72 310 = 787.4 m/s


A 3c3
ṁ =
x3 v g 3

 3vg 3
mx m × 0.902 × 1.159
A3 = = = 0.00133 ṁ
c3 787.4
A3 0.00133 m
Ratio of cross-sectional area at exit and throat = = = 1.7
A 2 0.000786 m

M06_THERMAL ENGINEERING_SE_XXXX_CH06.indd 290 6/15/2018 3:31:48 PM


Flow Through Steam Nozzles 291

Example 6.19
Steam at a pressure of 10 bar and dryness fraction of 0.98 is discharged through convergent–
divergent nozzle to a back pressure of 0.1 bar. The mass flow rate is 10 kg/kWh. If the power devel-
oped is 220 kW, then determine the following:
(a) Pressure at throat
(b) Dimensions at exit of the nozzle if the throat is of rectangular cross-section of 5 mm × 10 mm.
The loss due to friction is 10% of the overall isentropic enthalpy drop in the divergent portion.

Solution
Refer to the Mollier diagram as shown in Fig. 6.25.
(a) The isentropic index of expansion for wet steam:
   n = 1.035 + 0.1x1
     n = 1.035 + 0.1 × 0.98 = 1.133
n /( n −1)
p  2 
Throat-pressure 2 =  
p1  n + 1
1.133
p2  2  0.133
=  = 0.5778
p1  1.133 + 1
Now p1 = 10 bar,
p2 = 10 × 0.5778 = 5.778 bar
(b) From Mollier diagram, we have
h1 = 2762 kJ/kg, h3 = 2075 kJ/kg, h2 = 2660 kJ/kg
Loss in heat drop due to friction = 0.1 (h1 − h3) = 0.1 (2762 − 2075) = 68.7 kJ/kg
h3′ − h3 = 68.7

h3′ = h3 + 68.7 = 2075 + 68.7 = 2143.7 kJ/kg

Velocity at exit, c3 = 44.72 h1 − h3′ = 44.72 2762 − 2143.7

     = 44.72 618.3 = 1112 m/s

p1 = 10 bar
h p2 = 5.778 bar

1
p3 = 0.1 bar
2 Dr
ys
atu
ra
x1 = 0.98 ted
3 3′
x2 = 0.953
x3 = 0.816
s

Figure 6.25  Mollier diagram

M06_THERMAL ENGINEERING_SE_XXXX_CH06.indd 291 6/15/2018 3:31:52 PM


292 Chapter 6

A 3c3
Mass flow rate, ṁ =
x3′ v g 3
 3′ v g 3
mx
A3 =
c3
ṁ = 10 kg/kWh
10 × 220
ṁ = = 0.5556 kg/s
3600
x3′ = 0.816
vg3 = 14.674 m3/kg at 0.1 bar (from steam tables)
0.5556 × 0.816 × 14.67
A3 = = 59.81 × 10−4 m2[Neglecting vf3]
1112
Velocity at throat, c2 = 44.72 h1 − h2

= 44.72 2762 − 2600 = 451.66 m/s

 2 vg 2
mx
Total area of throat, A2 =
c2
x2 = 0.953
vg2 = 0.34 m3/kg at 5.778 bar (from steam tables)
0.5556 × 0.953 × 0.34
A2 = = 3.9858 × 10–4 m2
451.66
Throat area per nozzle = 5 × 10 × 10−6 = 0.5 × 10−4 m2
3.9858 × 10 −4
Number of nozzles = = 7.97 = 8 (say)
0.5 × 10 −4
59.81 × 10 −4
Exit area/nozzle = = 7.476 × 10−4 m2
8
Keeping the same aspect ratio of 1:2 for rectangle and let x be the smaller side
2x × x = 7.476 × 10−4
2x2 = 7.476 cm2
7.476
= 1.933 cm
x=
2
Therefore, exit rectangle is 19.33 mm × 38.66 mm.

Example 6.20
The dry saturated steam is expanded in a nozzle from pressure of 10 bar to pressure of 5 bar. If
the expansion is supersaturated, then find (a) the degree of under cooling, and (b) the degree of
supersaturation.

M06_THERMAL ENGINEERING_SE_XXXX_CH06.indd 292 6/15/2018 3:31:56 PM


Flow Through Steam Nozzles 293

Solution
Given that p1 = 10 bar and p2 = 5 bar
From steam tables corresponding to p1 = 10 bar, saturation temperature of steam.
T1 = 179.91°C = 452.91 K
T2′ = Temperature at which supersaturation occurs
p1 p2
=
1.3/ 0.3
(T1 ) (T2′ )1.3/ 0.3

0.3/1.3 0.3/1.3
T2′  p2   5
= =  = 0.852
T1  p1   10 
T2′ = 452.91 × 0.852 = 385.88 K = 112.88°C
From steam tables corresponding to a pressure of 5 bar, we find saturation temperature
T2 = 151.86°C
∴ Degree of undercooling
= T2 − T2′ = 151.86 − 112.88 = 38.98°C

Example 6.21
Steam enters a nozzle in a dry saturated condition and expands from a pressure of 2 bar to 1 bar. It
is observed that supersaturated flow is taking place and steam flow reverts to normal flow at 1 bar.
What is the degree of undercooling and increase in entropy and loss in available heat drop due to
irreversibility?

Solution
For supersaturated flow, 0.3/1.3
T1  p1 
=
T2′  p2 
0.3/1.3
T1  2 
=  = 1.1738
T2′  1 
From steam tables at 2 bar,
T1 = (120.23 + 273) = 393.23 K
T1
Hence,      T2′ = = 335 K
1.1738
From steam tables corresponding to 1 bar, T2 (saturation temperature)
= 99.62°C = 99.62 + 273 = 372.62 K
Degree of undercooling
= T2 − T2′ = 372.62 − 335 = 37.62 K
Refer to the Mollier diagram as shown in Fig. 6.26.
Corresponding to 335 K, saturation pressure from steam tables = 0.22 bar.
∴ p2′ = 0.22 bar

M06_THERMAL ENGINEERING_SE_XXXX_CH06.indd 293 6/15/2018 3:32:01 PM


294 Chapter 6

h
2 bar
1
h1

1 bar
h′2 2′
h2
2
s

Figure 6.26  Mollier diagram


p2 1
Degree of supersaturation = = = 4.545
p2′ 0.22
For supersaturated flow, the formula for superheated steam is used.
 n −1 
n p1v1   p2  n 
Thus, h1 − h2′ = 1−
n − 1 1000   p1  

 
where n = 1.3
v1 = 0.8857 m3/kg from steam tables
p2 1
=
p1 2

1.3 2 × 106 × 0.8857   1 


0.3/1.3 
h1 − h2′ = × 1 −    = 113.5 kJ/kg
0.3 1000   2  
Isentropic heat drop = (h1 − h2) = 117.5 kJ/kg (from Mollier diagram)
Therefore, loss in availability = 117.5 − 113.5 = 4 kJ/kg
4
Increase in entropy = = 0.01073 kJ/kg.K
372.62
Example 6.22
Find the percentage increase in discharge from a convergent–divergent nozzle expanding steam
from 8.75 bar dry to 2 bar, when (a) the expansion is taking place under thermal equilibrium and
(b) the steam is in metastable state during part of its expansion.
Take area of nozzle as 2500 mm2.
Solution
Given that p1 = 8.75 bar, p2 = 2 bar, A2 = 2500 mm2 = 2500 × 10–6 m2
Refer to the Mollier diagram shown in Fig. 6.27.
(a) Mass of steam discharged when expansion is under thermal equilibrium.
Let m 1 = Mass of steam discharged
From Mollier diagram, h1 = 2770 kJ/kg, h2 = 2515 kJ/kg, and x2 = 0.91
From steam tables,

M06_THERMAL ENGINEERING_SE_XXXX_CH06.indd 294 6/15/2018 3:32:05 PM


Flow Through Steam Nozzles 295

h(kJ/kg)
p1 = 8.75 bar
1
2770
p2 = 2 bar

2515
2

Figure 6.27  Mollier diagram

vg2 = 0.885 m3/kg


Δh2 = h1 − h2 = 2770 − 2515 = 255 kJ/kg
Velocity of steam at exit,
c2 = 44.72 ∆h2 = 44.72 255 = 714 m/s

v2 = x2 vg2 = 0.91 × 0.885 = 0.805 m3/kg

A 2 c2 2500 × 10 −6 × 714
ṁ 1== = 2.21 kg/s
v2 0.805
(b) Mass of steam discharged when it is in metastable state.
Let ṁ 2 = mass of steam discharged
Volume of steam at inlet,
0.0023( h1 − 1943)
v1 =
p1
0.0023 [2770 − 1943]
v1 = = 0.217 m3/kg
8.75
Volume of steam at exit,
1/1.3 1/1.3
v2  p1   8.75 
=  =
v1  p2   2 

1/1.3
v2 = v1  8.75 
 
2 
1/1.3
 8.75 
= 0.217  = 0.675 m3/kg
 2 
0.0023 ( h2 − 1943)
Furthermore,      v2 =
p2
0.0023 ( h2 − 1943)
0.675 =
2
h2 = 2530 kJ/kg

M06_THERMAL ENGINEERING_SE_XXXX_CH06.indd 295 6/15/2018 3:32:10 PM


296 Chapter 6

Heat drop from inlet to exit


Δh2 = h1 − h2 = 2770 − 2530 = 240 kJ/kg

Velocity of steam at exit, c2= 44.72 ∆h2

c2 = 44.72 240 = 693 m/s


A 2 c2 2500 × 10 −6 × 693
ṁ 2 = = = 2.57 kg/s
v2 0.675
∴ Percentage increase in discharge
m 2 − m 1 2.57 − 2.21
= = = 0.163 or 16.3%
m 1 2.21

Example 6.23
Steam expands through a nozzle from 5 bar and dry saturated to a back pressure of 0.2 bar. Mass
flow is 2 kg/s. Calculate the exit and the throat areas under the following conditions:
(a) Isentropic expansion with negligible velocity
(b) Isentropic expansion with initial velocity of 100 m/s
(c) Friction loss at pressure amounts to 10% of the total heat drop up to that pressure and initial
velocity negligible

Solution
(a) Pressure at throat is given by
p2 = 0.5774 × p1 = 0.5774 × 5 = 2.887 bar
Drop a vertical from point 1 to cut pressure for 2.887 bar at point 2 and the pressure line for
0.2 bar, i.e., the back pressure at point 3. Then from the Mollier diagram (Fig. 6.28), the
following values are obtained:
At point 1,   h1 = 2745 kJ/kg
At point 2,   p2 = 2.887 bar, h2 = 2645 kJ/kg, and v2 = 0.63 m3/kg
At point 3,   p3 = 0.2 bar, h3 = 2248 kJ/kg, and v3 = 6.6 m3/kg

h (kJ/kg) 5 bar

1 2.88 bar
2745

2 0.2 bar
2645

2248
3

Figure 6.28  Mollier diagram

M06_THERMAL ENGINEERING_SE_XXXX_CH06.indd 296 6/15/2018 3:32:13 PM


Flow Through Steam Nozzles 297

Hence, the area at throat for a mass flow of 2 kg/s is given by,
Mass flow/sec × Specific volume at throat
A2 =
Velocity at throat
m × v2 m × v2
= =
c2 2 × 1000 × ( h1 − h2 )
2 × 0.63
= = 0.00281 m2 = 28.1 cm2
2 × 1000 × ( 2745 − 2645)
Similarly, area at exit for a mass flow of 2 kg/s is given by,
m × v3 2 × 6.6
A3 = =
c3 2 × 1000 × ( h1 − h2 )
2 × 6.6
= = 0.0132 m2 = 132 cm2
2 × 1000 × ( 2745 − 2248)

(b) Refer Fig. 6.29.


c12 100 2
(h0 − h1) = = = 5.0 kJ/kg
2 × 1000 2 × 1000
Hence,  h0 = 5.0 + h1 = 5 + 2745 = 2750 kJ/kg
Corresponding to h0 = 2750 kJ/kg
    p0 = 5.1 bar as shown on the chart
Thus, the problem is resolved to the condition of p0 = 5.1 bar and h0 = 2750 kJ and velocity of
zero at entry to the nozzle.
Therefore, pressure at throat is given by,
n /( n −1)
p2  2 
=  = 0.5774 or p2 = 0.5774 × 5.1 = 2.945 bar
p0  n + 1
p2 = 2.945 bar
And at     
h2 = 2653 kJ/kg, v2 = 0.61 m3/kg, from the Mollier chart
p3 = 0.2 bar
h3 = 2248 kJ/kg, v3 = 6.6 m3/kg, from the Mollier chart

p1
h p0 p2
h0
p3
h1
1
h2
2
h3 3

Figure 6.29  Mollier diagram

M06_THERMAL ENGINEERING_SE_XXXX_CH06.indd 297 6/15/2018 3:32:17 PM


298 Chapter 6

Area at throat is given by,


m × v2 2 × 0.61
A2 = =
2 × 1000 × ( h0 − h2 ) 2 × 1000 × ( 2750 − 2653)
= 0.00277 m2 = 27.7 cm2
Similarly, area at the exit from the nozzle is given by,
m × v3 2 × 6.6
A3 = =
2 × 1000 × ( h0 − h3 ) 2 × 1000 × ( 2750 − 2248)
= 0.01317 m2 = 131.7 cm2
The nozzle efficiency hn = 0.9 and n = 1.135 for dry saturated steam.
n 1.135
Therefore,     m = = = 1.119
h n + n (1 − h n ) 0.9 + 1.135 (1 − 0.9)
The critical pressure ratio is given by,
m /( m +1) 1.119 /(1.119 −1)
p2  2   2 
rp = =  = = 0.5807
p1  m + 1  1.119 + 1
And the discharge per unit area is given by,

Mass 2m p
= h 1  r 2 / m − rp( m +1)/ m 
A ( m − 1) n v1  p 

2 × 1.119 5 × 105 
= × 0.9 × (0.5807) 2 /1.119 − (0.5807)(1.199+1)/1.119 
1.119 − 1 0.3816  

= 2.18 × 107 × 0.020 = 4.36 × 105

2
Area at throat, A2 = = 0.00303 m2 = 30.3 cm2
5
4.36 × 10
p3 0.2
Using the same formula and putting the value of rp = = , we get
p1 5
Mass
= 2.18 × 107 × 0.9 × 10 −3
A
Therefore, area at exit is given by,
2
A3 = = 0.01425 m2 = 142.5 cm2
4
2.18 × 0.9 × 10
The area at the exit may also be found with the help of Mollier diagram (Fig. 6.30). Isentropic
enthalpy drop as found in part (a) = 497 kJ/kg.
Therefore, the actual heat drop = 0.9 × (2745 − 2248) kJ/kg
= 497 + 0.9 = 447.3 kJ/kg

M06_THERMAL ENGINEERING_SE_XXXX_CH06.indd 298 6/15/2018 3:32:24 PM


Flow Through Steam Nozzles 299

h 5 bar
1

0.2 bar

3′
3

Figure 6.30  Mollier diagram

i.e., h1 − h3′ = 447.8 kJ/kg


and    v3′ = 6.7 m3/kg

Therefore, area at exit is given by,
2 × v3′
A3 =
2 × 1000 × 447.3
2 × 6.7
=
2 × 1000 × 447.3
 = 0.01416 m2 = 141.6 cm2 (checks very well)
Example 6.24
A convergent–divergent nozzle is required to discharge 2 kg of steam per second. The nozzle is
supplied with steam at 7 bar and 180°C and discharge takes place against a back pressure of 1 bar.
The expansion up to throat is isentropic and the frictional resistance between the throat and the exit
is equivalent to 63 kJ/kg of steam. Taking approach velocity of 75 m/s and throat pressure of 4 bar,
estimate: (a) suitable areas for the throat and exit and (b) overall efficiency of the nozzle based on
the enthalpy drop between the actual inlet pressure and temperature and the exit pressure.
Solution
Given that m = 2 kg/s; p1 = 7 bar, T1 = 180°C, p3 = 1 bar, frictional resistance = 63 kJ/kg of steam,
c1 = 75 m/s, p2 = 4 bar
(a) Suitable areas for the throat and exit
Let A2 = area at the throat
A3 = area at the exit
The expansion of steam through the nozzle on the Mollier diagram is shown in Fig. 6.31. From
the Mollier diagram, we find that
h (kJ/kg)
1 7 bar
2810
180°C
2
2680 1 bar
2470 Dry saturated
x2 = 0.97
x3 = 0.934
s

Figure 6.31  Mollier diagram

M06_THERMAL ENGINEERING_SE_XXXX_CH06.indd 299 6/15/2018 3:32:27 PM


300 Chapter 6

h1 = 2810 kJ/kg, h2 = 2680 kJ/kg, h3 = 2470 kJ/kg, x2 = 0.97, and x3 = 0.934


From steam tables, we also find that the specific volume of steam at throat corresponding to 4 bar,
vf2 = 0.001084 m3/kg, vg2 = 0.4625 m3/kg
v2 = vf2 + x2(vg2 − vf2) = 0.001084 + 0.97(0.4625 − 0.001084) = 0.449 m3/kg
and specific volume of steam corresponding to 1 bar,
     vf3 = 0.001043 m3/kg, vg3 = 1.694 m3/kg
     v3 = vf3 + x3(vg3 − vf3) = 0.001043 + 0.934(1.694 − 0.001043) =1.582 m3/kg
Heat drop between entrance and throat,
   Δh2 = h1 − h2 = 2810 − 2680 = 130 kJ/kg
∴ Velocity of steam at throat,

c2 = c12 + 2 × 103 × ∆h2 = (75)2 + 2 × 103 × 130 = 515 m/s


    

A 2 c2
and    ṁ =
x2 v g 2

 2 vg 2
mx 2 × 0.97 × 0.449
A2 =
or   = = 1.74 × 10−3 m2
c2 515
         = 1740 mm2
Since there is a frictional resistance of 63 kJ/kg of steam between the throat and the exit,
     h3 − h3′ = 63 or h3 = h3′ + 63 = 2470 + 63 = 2533 kJ/kg
and heat drop between entrance and exit,
   ∆h3 = h1 − h3 = 2810 − 2533 = 277 kJ/kg
∴ Velocity of steam at exit,

   c3 = c12 + 2 × 103 × ∆h3 = (75)2 + 2 × 103 × 277 = 748 m/s

A 3c3
and ṁ =
x3 v g 3

 3vg 3
mx 2 × 0.934 × 1.582
    A3 = =
c3 748
  = 4.23 × 10−3 m2 = 4230 mm2
(b) Overall efficiency of the nozzle
Overall efficiency of the nozzle

Useful heat drop h −h


   hn = = 1 3
Isentropic heat drop h1 − h3′

2810 − 2533
=
      = 0.815 or 81.5%
2810 − 2470

M06_THERMAL ENGINEERING_SE_XXXX_CH06.indd 300 6/15/2018 3:32:32 PM


Flow Through Steam Nozzles 301

Example 6.25
Air is expanded reversibly and adiabatically in a nozzle from 13 bar and 150°C to a pressure of
6 bar. The inlet velocity of the nozzle is very small and the process occurs under steady-state flow
conditions. Calculate the exit velocity of the nozzle. [IES, 1992]

Solution
Given that p1 = 13 bar, T1 = 273 + 180 = 423 K, p2 = 6 bar, and c1 = 0
SFEE for unit mass flow rate is:
 1 2   1 2 
 h1 + c1 + gz1  + q =  h2 + c2 + gz2  + w
2 2
2
1

p1,T1,c1
p2,T2,c2

Figure 6.32  Flow through steam nozzle


Since the air expands reversibly and adiabatically from state 1 to 2, as shown in Fig. 6.32, we have
q = 0, w = 0, and z1 = z2
1 2
h1 = h2 + c
2 2
c2 = 2( h1 − h2 ) = 2c p (T1 − T2 )
g −1
T2  p2  g
Now,    =
T1  p1 
1.4 −1
 6 1.4
T2 = 423  
    = 339.156 K
 13 
Exit velocity of the nozzle,
c2 = 2 × 1.005 × 103 ( 423 − 339.156) = 410.52 m/s

Example 6.26
A steam nozzle receives steam at 40 bar and 400°C at an initial velocity of 40 m/s. The final pres-
sure of steam is 10 bar. The mass flow rate of steam is 2 kg/s. The nozzle efficiency is 90%. The
cross-section of the nozzle is circular. The angle of divergence is 6°. Calculate the throat and exit
diameters and the length of the divergent portion. Show the representation of process on h − s dia-
gram and sketch the nozzle and label the dimensions calculated.  [IES, 2005]

M06_THERMAL ENGINEERING_SE_XXXX_CH06.indd 301 6/15/2018 3:33:24 PM


302 Chapter 6

3
1

2

p1 = 40 bar
c1 = 40 m/s
t1 = 400°C I

p2 p3
c2 c3
d2 d3

Figure 6.33  Flow through convergent-divergent steam nozzle


Solution
The convergent–divergent nozzle is shown in Fig. 6.33 and h − s diagram in Fig. 6.34.
From steam tables and Mollier diagram, we have
h1 =3213.5 kJ/kg for p1 = 40 bar and t = 400°C
p2 = 0.5457, p1 = 0.5457 × 40 = 21.828 bar, for superheated steam
h2 = 3025 kJ/kg, vs2 = 0.125 m3/kg
c2 = [2(h1 − h2) × 103 + c12 ]0.5
= [2(3213.5 − 3025) × 103 + 402]0.5 = 615.3 m/s
A2 c2 2 × 0.125
ṁ s = , A2 = = 4.06 × 10−4 m2
vs 2 615.3
0.5 0.5
4A   4 × 4.06 × 10 −4 
d2 =  2  =  = 0.002274 m = 22.74 mm
 p   p 

h
r
ba
40
=
p1
h1 1 400°C r
ba
2
.8
vs2 21
=
p2

r
h2
vs3 ba
2 10
=
p3

h3 4
h4′ 3
3′
x=1

Figure 6.34 h − s diagram

M06_THERMAL ENGINEERING_SE_XXXX_CH06.indd 302 6/15/2018 3:33:26 PM


Flow Through Steam Nozzles 303

In Fig. 6.34,
h2 − h4 = 0.9(h2 − h3′)
h3 = 2880 kJ/kg, vs3 = 0.214 m3/kg
c3 = [2(3025 − 2880) × 103 + (618.87)2]0.5
= 820 m/s
2 × 0.214
A3 = = 5.22 × 10–4 m2
820
d3 = 25.8 mm
d3 − d2 25.8 − 22.74
l= = = 14.56 mm

2 tan 6 2 tan 6

Summary for Quick Revision

1. A steam nozzle may be defined as a passage of varying cross-section by means of which a part of enthalpy
of steam is converted into kinetic energy as the steam expands from a higher pressure to a lower pressure.
Ac Ac
2. Continuity equation: ṁ = 1 1 = 2 2
v1 v2
3. Velocity of steam through a nozzle:
c2 = 2( h1 − h2 ) + c12

For c1 = 0. c2 = 2( h1 − h2 ) = 44.72 ( ∆h)isen


where h1 and h2 are in kJ/kg
4. Isentropic flow through a nozzle
(a) pvn = constant
where n = 1.035 + 0.1 x1 from wet steam
= 1.135 for saturated steam
= 1.3 for superheated steam
 n −1 
 2n    p2  n 
(b) Velocity, c2 =  p v 1−
 n − 1 1 1   p1  
 
 
(c) Mass of steam discharged,
1
  2 n +1   2
ṁ = A2   2 n  p 2   p2  n  p2 n 

    −   
 n − 1 v1   p1   p1 
  
n
p2  2  n −1
(d) Critical pressure ratio, =  , for maximum discharge.
p1  n + 1

M06_THERMAL ENGINEERING_SE_XXXX_CH06.indd 303 6/15/2018 3:33:31 PM


304 Chapter 6

(e)  Velocity at exit for maximum discharge,

 2n 
c2 =  nRT1
 n + 1
1
 n +1  2
(f)  Maximum discharge, (ṁ )max = A2  np1  2  n −1 
 v  n + 1 
 1 
5. Most of the friction losses occur beyond the throat in the divergent section of the nozzle.
Actual enthalpy drop
6. Nozzle efficiency, hn =
Isentropic enthalpy drop
7. Exit velocity considering friction in nozzle,

c2 = 44.72 h n × ( ∆h)isen
= 44.72 × K ( ∆h)isen , K = h n
8. Supersaturated (or meta stable) flow:
In steam nozzles, the condensation of steam beyond the saturation line is suppressed or delayed due to very
high velocity of steam and the vapours continue to expand in dry state even beyond the saturation line. The
steam vapours are said to be supersaturated or supercooled in this region.
  Velocity of steam at the end of expansion,
1
  n −1   2
  2n    p2  n   mu 2
c2 =   p1u1 1 −     , A2 =
  n − 1   p1   c2
  
9. Wilson line represents the limit to the supersaturated state on the Mollier chart.
10. Isentropic, one-dimensional steady flow through a nozzle:
dA 1 dp  1 − M 2 
(a)  = ×  
A k p  M2 

c dp cp
where M = , cs = = g RT , g =
cs dr cu
(b)  For a nozzle, dp < 0.
Subsonic nozzle: M < 1, dA < 0, and nozzle is converging.
Supersonic nozzle: M > 1, dA > 0, and nozzle is diverging.
(c)  For a diffuser, dp > 0.
       Subsonic diffuser, M < 1, dA > 0, and diffuser is diverging.
       Supersonic diffuser, M > 1, dA < 0, and diffuser is converging.
(d)  When M = 1, dA = 0, which occurs at the throat of nozzle and diffuser.
11. Stagnation properties:
c2
ho = h +
2
To  g − 1 2
      = 1 +  M
T  2 

M06_THERMAL ENGINEERING_SE_XXXX_CH06.indd 304 6/15/2018 3:33:38 PM


Flow Through Steam Nozzles 305

g
po  T  g −1
=  o
p T
1
ro  T  g −1
=  o
r T
12. Properties at the throat, M = 1
g 1
T* 2 p*  2  g −1 r *  2  g −1
= , =  , = 
To g + 1 po  g + 1 ro  g + 1
13. Mass rate of flow through isentropic nozzle.

m p g M
= o × × (g +1)
A To R
 g − 1 2  2(g −1)
1 + 2 M 
 
At the throat, M = 1
(g +1)
A 1  2    g − 1 2   2(g −1)
14. Area ratio, =  1 +   M 
A* M  g + 1   2  
15. A shock wave involves an extremely rapid and abrupt change of state.
(a) In a normal shock, the abrupt change of state takes place across a plane normal to the direction of flow.
(b) The normal shock proceeds from supersonic to subsonic region.
(c) There is no change in stagnation temperature across a normal shock, i.e., Tox = Toy.
g − 1 2
1+  M
Ty  2  x
(d) =
Tx g − 1 2
1+  M
 2  y
1
 g − 1 2  2
1+ M
p y  M x    2  x  1 + g M x2
(e) =   =
px  M y    g − 1 2  1 + g M y2
1 +  M
2  y 

 2 
M x2 + 
2
 g − 1
(f) M y =
 2g  2
 g − 1 M x − 1

Multiple-choice Questions
1. A nozzle has velocity head at outlet of 10 m. If it 2. The effect of friction on flow of steam through a
is kept vertical, the height reached by the stream is nozzle is to
(a) 100 m (b) 10 m (a) decrease the mass flow rate and to increase
the wetness at the exit
1
(c) 10 m (d) m (b) increase the mass flow rate and to increase
10 the exit temperature

M06_THERMAL ENGINEERING_SE_XXXX_CH06.indd 305 6/15/2018 3:33:48 PM


306 Chapter 6

(c) decrease the mass flow rate and to decrease (a) 1200 m/s
the wetness of the steam (b) 900 m/s
(d) increase the exit temperature, without any (c) 60 m/s
effect on the mass flow rate (d) the same as the sonic velocity
3. For adiabatic expansion with friction through a 10. Consider the following statements:
nozzle, the following remains constant I. De Laval nozzle is a subsonic nozzle.
(a) Entropy II. Supersonic nozzle is a converging passage
(b) Static enthalpy III. Subsonic diffuser is a diverging passage
(c) Stagnation enthalpy Which of these statements is/ are correct?
(d) Stagnation pressure (a) I and II (b) II and III
4. The critical pressure ratios for flow of dry satu- (c) I alone (d) III alone
rated steam and superheated steam through a noz- 11. In a steam nozzle, inlet pressure of superheated
zle are, respectively, steam is 10 bar. The exit pressure is decreased
(a) 0.5279 and 0.550 (b) 0.577 and 0.550 from 3 bar to 1 bar. The discharge rate will
(c) 0.577 and 0.546 (d) 0.5279 and 0.546 (a) remain constant
5. At which location of a converging–diverging noz- (b) decrease
zle, does the shock-boundary layer interaction (c) increase slightly
take place? (d) increase or decrease depending on whether the
(a) Converging portion nozzle is convergent or convergent–divergent
(b) Throat 12. Air from a reservoir is to be passed through a
(c) Inlet supersonic nozzle so that the jet will have a Mach
(d) Diverging portion number of 2. If the static temperature of the jet is
6. The effect of friction in a steam nozzle is to not to be less than 27°C, the minimum tempera-
(a) increase velocity and increase dryness fraction ture of air in the reservoir should be
(b) increase velocity and decrease dryness fraction (a) 48.6°C (b) 167°C
(c) decrease velocity and increase dryness fraction (c) 267°C (d) 367°C
(d) decrease velocity and decrease dryness fraction 13. Consider the following statements:
7. Consider the following statements: For supersaturated flow through a steam nozzle,
When dry saturated or slightly superheated the
steam expands through a nozzle, I. Enthalpy drop reduces further
I.  The coefficient of discharge is greater than unity. II. Exit temperature increases
II.  It is dry up to Wilson’s line. III. Flow rate increases
III. Expansion is isentropic throughout. Which of these statements are correct?
Of these statements (a) I, II, and III (b) I and II
(a) I, II, and III are correct (c) II and III (d) I and III
(b) I and II are correct 14. The critical pressure ratio for maximum discharge
(c) I and III are correct through a nozzle is given by
(d) II and III are correct n n
 n + 1 n −1  2  n −1
8. The total and static pressures at the inlet of a (a)  (b) 
 2   n + 1
steam nozzle are 186 kPa and 178 kPa, respec-
n −1 n −1
tively. If the total pressure at the exit is 180 kPa  n + 1 n  2  n
(c)  (d) 
and static pressure is 100 kPa, then the loss of  2   n + 1
energy per unit mass in the nozzle will be
(a) 78 kPa (b) 8 kPa 15. Consider the following statements in relation to
(c) 6 kPa (d) 2 kPa a convergent–divergent steam nozzle operating
9. Under ideal conditions, the velocity of steam at under choked conditions:
the outlet of a nozzle for a heat drop of 400 kJ/kg I. In the convergent portion, steam velocity is
will be approximately less than sonic velocity.

M06_THERMAL ENGINEERING_SE_XXXX_CH06.indd 306 6/15/2018 3:33:50 PM


Flow Through Steam Nozzles 307

II. In the convergent portion, steam velocity is n / n −1 1/ n −1


greater than sonic velocity.  2   1 
(c)  (d) 
III. In the divergent portion, the steam velocity is  n + 1  n + 1
less than sonic velocity. 21. What is the critical pressure ratio for isentropic
IV. In the divergent portion, the steam velocity is nozzle flow with ratio of specific heats as 1.5?
greater than sonic velocity. (a) (0.8)3 (b) (0.8)0.6
Which of the above statements are correct? 0.33
(c) (1.25) (d) (1.25)3
(a) I and III (b) I and IV
(c) II and III (d) II and IV 22. A compressible fluid flows through a passage as
shown in Fig. 6.35. The velocity of the fluid at the
16. For maximum discharge through a convergent point A is 400 m/s.
p
nozzle, the pressure ratio 2 should be (where
p1
n is the index for expansion) A B
2 n
 n  n +1  2  n −1
(a)  (b) 
 n + 1  n + 1 Figure 6.35
n n
Which one of the following is correct?
 n + 1 n −1  2  n +1
(c)  (d)  At point B, the fluid experiences
 2   n + 1
(a) an increase in velocity and decrease in pres-
17. Wilson line is associated with which one of the sure
following? (b) a decrease in velocity and increase in pres-
(a) Total steam consumption with respect to sure
power output (c) a decrease in velocity and pressure
(b) Supersonic flow of steam through a nozzle (d) an increase in velocity and pressure
(c) Nozzle flow with friction
23. If the cross-section of a nozzle is increasing in the
(d) Supersaturated flow of steam through a nozzle
direction of flow in supersonic flow, then in the
18. In a steam nozzle, to increase the velocity of downstream direction
steam above sonic velocity by expanding steam (a) both pressure and velocity will increase
below critical pressure (b) both pressure and velocity will decrease
(a) a vacuum pump is added (c) pressure and increase but velocity will decrease
(b) ring diffusers are used (d) pressure will decrease but velocity will increase
(c) divergent portion of the nozzle is necessary
24. If the velocity of propagation of small distur-
(d) abrupt change in cross-section is needed
bances in air at 27°C is 330 m/s, then at a temper-
19. Which one of the following is the correct state- ature of 54°C, its speed would be
ment? To get supersonic velocity of steam at noz- (a) 660 m/s (b) 300 × 2m/s
zle exit with a large pressure drop across it, the
327
duct must (c) 330/ 2m/s (d) 330 × m/s
(a) converge from inlet to exit 300
25. For one-dimensional isentropic flow in a diverg-
(b) diverge from inlet to exit
ing passage, if the initial static pressure is p1 and
(c) first converge to the throat and then diverge
the initial Mach number is M1 (M1 < 1), then for
till exit
the downstream flow
(d) remain constant in cross-section
(a) M2 < M1; p1 < p2 (b) M2 < M1; p2 > p1
20. Which one of the following is the correct expres- (c) M2 > M1; p2 > p1 (d) M2 > M1; p2 < p1
sion for the critical pressure ratio of a nozzle?
26. Which of the following diagrams correctly depict
1/ n −1 n / n −1 the behaviour of compressible fluid flow in the
 2   1 
(a)  (b) 
 n + 1  n + 1 given geometries (Fig. 6.36)?

M06_THERMAL ENGINEERING_SE_XXXX_CH06.indd 307 6/15/2018 3:33:55 PM


308 Chapter 6

(c) stagnation enthalpy


(d) stagnation pressure
30. The identical convergent–divergent nozzles A and
1 c1 c2 2 c1 c2 B are connected in series, as shown in Fig. 6.38 to
carry a compressible fluid.
A B
M<1 c2 > c1 M > 1 c2 > c1

3 c1 c2 4 c1 c2

Figure 6.38
Figure 6.36 Which one of the following statements regarding
the velocities at the throats of the nozzles is correct?
Codes: (a) Sonic and supersonic velocities exist at the
(a) 1 and 4 (b) 2 and 4 throats of the nozzles A and B, respectively.
(c) 2 and 3 (d) 1 and 3 (b) Sonic velocity can exist at throats of both
27. Figure 6.37 represents a schematic view of the nozzles A and B.
arrangement of supersonic wind tunnel section. A (c) Sonic velocity will always exist at the throat
normal shock can exist without affecting the test of nozzle A, whereas subsonic velocity will
conditions exist at throat of nozzle B.
(d) Sonic velocity exists at the throat of nozzle B,
1 2 3 4 5 whereas subsonic velocity exists at the throat
of nozzle A.
31. Semi-angle of a Mach cone is
Supersonic  1 
test (a) sin–1  (b) sin–1 M
section  M 
 1  1
(c) sin–1   (d) cos–1  
1 2 3 4 5  M  M
Supersonic Supersonic
where M is the Mach number.
nozzle diffuser
32. Which of the following statements are correct?
Figure 6.37 I. Mach wave is a very weak shock wave.
(a) Between Sections 4 and 5 II. Entropy change across a shock wave is nearly
(b) At Section 4 zero.
(c) Between Sections 4 and 3 III. Total pressure behind a shock wave is less
(d) Between Sections 1 and 2 than that ahead of it.
IV. Mach number behind a normal shock is less
28. The Mach number for nitrogen flowing at 195 m/s than one.
when the pressure and temperature in the undis-
turbed flow are 690 kN/m abs and 93°C, respec- Select the correct answer using the codes given
tively, will be below:
(a) 0.25 (b) 0.50 Codes:
(c) 0.66 (d) 0.75 (a) I, II, and III (b) I, III, and IV
(c) I, II, and IV (d) II, III, and IV
29. The Mach number at inlet of gas turbine diffuser
33. Introduction of a Pitot tube in a supersonic flow
is 0.3. The shape of the diffuser would be
would produce
(a) converging
(a) normal shock at the tube nose
(b) diverging
(b) curved shock at the tube nose

M06_THERMAL ENGINEERING_SE_XXXX_CH06.indd 308 6/15/2018 3:33:57 PM


Flow Through Steam Nozzles 309

(c) normal shock at the upstream of the tube nose 38. Consider the following statements:
(d) curved shock at the upstream of the tube nose A convergent–divergent nozzle is said to be
34. In the Fanno line shown in Fig. 6.39, choked when:
I. Critical pressure is attained at the throat.
h
P II. Velocity at throat becomes sonic.
III. Exit velocity becomes supersonic.
Of these statements
Q
(a) I, II, and III are correct
(b) I and II are correct
R (c) II and III correct
s (d) I and III are correct
Figure 6.39 39. In flow through a convergent nozzle, the ratio of
(a) subsonic flow proceeds along PQR back pressure to the inlet pressure is given by the
(b) supersonic flow proceeds along PQR relation
(c) subsonic flow proceeds along PQ and super- g /g −1

pB  2 
sonic flow proceeds along RQ =  
(d) subsonic flow proceeds along PQ and super- p1 g +1
sonic flow proceeds along QR If the back pressure is lower than pB given in the
35. Given k = ratio of specific heats, for Rayleigh line, above equation, then:
the temperature is maximum at a Mach number of (a) The flow in the nozzle is supersonic
1 (b) A shock wave exists inside the nozzle
(a) (b) k (c) The gases expand outside the nozzle and a
k shock wave appears outside the nozzle
(c) 1/k (d) k (d) A shock wave appears at the nozzle exit
36. What is/are the effect(s) of supersaturation in noz- 40. Consider the following statements:
zle flow? Across the normal shock, the fluid properties
I. It increases the mass flow change in such a manner that the:
II. It increases friction in the nozzle I. Velocity of flow is subsonic
III. It increases exit velocity II. Pressure increases
IV. It reduces dryness fraction of steam. III. Specific volume decreases
Select the correct answer using the code given below: IV. Temperature decreases
(a) I only (b) I and III Of these statements:
(c) I, II and IV (d) III only (a) II, III, and IV are correct
37. Consider the following statements pertaining to (b) I, II, and IV are correct
isentropic flow: (c) I, III, and IV are correct
I. To obtain stagnation enthalpy, the flow need (d) I, II, and III are correct
not be decelerated isentropically but should 41. Match List I (Property ratios as the critical and
be decelerated adiabatically. stagnation conditions) with List II (values of
II. The effect of friction in an adiabatic flow is ratios) and select the correct answer using the
to reduce the stagnation pressure and increase codes given below the lists:
entropy.
III. A constant area tube with rough surfaces can List I List II
be used as a subsonic nozzle. • 1
T
Of these statements: A.   2  g −1
T0 1.  
(a) I, II, and III are correct  g +1
(b) I and II are correct
(c) I and III are correct r• 2. 
2
B.  g +1
(d) II and III are correct r0

M06_THERMAL ENGINEERING_SE_XXXX_CH06.indd 309 6/15/2018 3:34:01 PM


310 Chapter 6

(a) Supersonic, sonic, subsonic, and supersonic


List I List II
(b) Sonic, supersonic, subsonic, and supersonic
p• (c) Subsonic, supersonic, sonic, and subsonic
C.  3. 1 (d) Subsonic, sonic, supersonic, and subsonic
p0
g 46. Acoustic velocity in an elastic gaseous medium is
s•
D.   2  g −1 proportional to
s0 4.  
 g + 1 (a) absolute temperature
(b) stagnation temperature
Codes: (c) square root of absolute temperature
A B C D (d) square root of stagnation temperature
(a) 2 1 4 3 47. Which of the following statement(s) is/are rele-
(b) 1 2 3 4 vant to critical flow through a steam nozzle?
(c) 2 1 3 4 I. Flow rate through the nozzle is minimum.
(d) 1 2 4 3 II. Flow rate through the nozzle is maximum.
42. For oblique shock, the downstream Mach number III. Velocity at the throat is supersonic.
(a) is always more than unity IV. Velocity at the throat is sonic.
(b) is always less than unity Select the correct answer using the codes given
(c) may be less or more than unity below:
(d) can never be unity Codes:
43. Fanno line flow is a flow in a constant area duct (a) I alone (b) I and III
(a) with friction and heat transfer but in the (c) II and IV (d) IV alone
absence of work 48. Which of the following statements is/are true
(b) with friction and heat transfer and accompa- in case of one-dimensional flow of perfect gas
nied by work through a converging–diverging nozzle?
(c) with friction but in the absence of heat trans- I. The exit velocity is always supersonic.
fer and work II. The exit velocity can be subsonic or supersonic.
(d) without friction but accompanied by heat III. If the flow is isentropic, then the exit velocity
transfer and work must be supersonic.
44. Reyleigh line flow is a flow in a constant area duct IV. If the exit velocity is supersonic, then the
(a) with friction but without heat transfer flow must be isentropic.
(b) without friction but with heat transfer Select the correct answer from the codes given
(c) with both friction and heat transfer below:
(d) without either friction or heat transfer Codes:
(a) II and IV
45. The plot of the pressure ratio along the length of
(b) II, III, and IV
the convergent–divergent nozzle is shown in Fig.
(c) I, III, and IV
6.40. The sequence of the flow conditions labelled
(d) II alone
(1), (2), (3), and (4) in the figure is, respectively
49. In a normal shock, in a gas
(a) the stagnation pressure remains the same on
both sides of the shock
(b) the stagnation density remains the same on
both sides of the shock
1 2 3 4
p/pb (c) the stagnation temperature remains the same
on both sides of the shock
(d) the Mach number remains the same on both
sides of the shock
50. A normal shock
Length (a) causes a disruption and reversal of flow pattern
Figure 6.40 (b) may occur only in a diverging passage

M06_THERMAL ENGINEERING_SE_XXXX_CH06.indd 310 6/15/2018 3:34:03 PM


Flow Through Steam Nozzles 311

(c) is more severe than an oblique shock (c) remain unaltered


(d) moves with a velocity equal to the sonic velocity (d) first increase to a maximum and then will
51. In a normal shock, in a gas decrease
(a) both the downstream flow and the upstream 57. In isentropic flow between two points
flow are supersonic (a) the stagnation pressure decreases in the direc-
(b) only the upstream flow is supersonic tion of flow
(c) the downstream flow is sonic (b) the stagnation temperature and stagnation
(d) the upstream flow is subsonic pressure decrease with increase in the velocity
52. Which of the following parameters decrease (c) the stagnation temperature and stagnation
across a normal shock wave? pressure may vary
I. Mach number (d) the stagnation temperature and stagnation
II. Static pressure pressure remain constant
III. Stagnation pressure 58. Steam flows at the rate of 10 kg/s through a super-
IV. Static temperature sonic nozzle. Throat diameter is 50 mm. Density
Select the correct answer using the code given ratio and velocity ratio with reference to throat
below: and exit are respectively 2.45 and 0.8. What is the
(a) Only I and III (b) Only II and IV diameter at the exit?
(c) I, II, and III (d) II, III, and IV (a) 122.5 mm (b) 58 mm
53. When are shock waves formed in air compressors? (c) 70 mm (d) 62.5 mm
(a) Mach number < 0.9 59. Which of the following is caused by the occur-
(b) Mach number > 0.9 rence of a normal shock in the diverging section
(c) Mach number = 2 of a convergent-divergent nozzle?
(d) Mach number changes suddenly from one I. Velocity jump
value to another II. Pressure jump
54. It is recommended that the diffuser angle should III. Velocity drop
be kept less than 18° because IV. Pressure drop
(a) pressure decrease in flow direction and flow Select the correct answer using the codes given
separation may occur below:
(b) pressure decreases in flow direction and flow (a) I only (b) I and II
may become turbulent (c) II and III (d) I and IV
(c) pressure increases in flow direction and flow 60. Total enthalpy of stream at the inlet of a nozzle is
separation may occur 2800 kJ while static enthalpy at the exit is 2555 kJ.
(d) pressure increases in flow direction and flow What is the steam velocity at the exit if expansion
may become turbulent is isentropic?
55. A converging diverging nozzle is connected to a (a) 70 m/s (b) 245 m/s
gas pipeline. At the inlet of the nozzle (converging (c) 450 m/s (d) 700 m/s
section) the Mach number is 2. It is observed that 61. What is/are the effect(s) of supersaturation in noz-
there is a shock in the diverging section. What is zle flow?
the value of the Mach number at the throat? I. It increases the mass flow
(a) < 1 (b) Equal to 1 II. It increases friction in the nozzle
(c) > 1 (d) ≥ 1 III. It increases exit velocity
56. A nozzle is discharging steam through critical IV. It reduces dryness fraction of steam.
pressure ratio. When the back pressure is further Select the correct answer using the code given
decreased, the nozzle flow rate will below:
(a) decrease (a) I only (b) I and III
(b) increase (c) I, II, and IV (d) III only

M06_THERMAL ENGINEERING_SE_XXXX_CH06.indd 311 6/15/2018 3:34:03 PM


312 Chapter 6

Explanatory Notes

v2
1. (b) Velocity head = = h = 10 cm
2g
8. (c) Pressure loss = poi − poe = 186 – 180 = 6 kPa

9. (b) Velocity of steam at outlet of nozzle = 2 × 103 × 400 = 894.4 ∼


− 900 m/s

11. (a) Critical pressure ratio for superheated steam = 0.5457


p2 = 10 × 0.5457 = 5.457 bar
Exit pressure of 3 bar is less than p2. With further decrease of pressure from 3 bar to 1 bar, discharge rate
will remain the same.

Temperature of air in reservoir, Tr


12. (a) = 1 + 0.5 (g – 1) M2
Static temperature of jet, T j
= 1 + 0.5 (1.4 − 1) × 22 = 1.8

Tr = 27 × 1.8 = 48.6°C
n 1.5
 2  n −1  2  0.5
21. (a) Critical pressure ratio =  = = (0.8)3
 n + 1  1.5 + 1

273 + 54 327
24. (d) v ∝ T , v2 = v1 T2 = 330 = 330 m/s
T1 273 + 27 300

8314.3
28. (b) For nitrogen, R = = 296.94 J/kg.K
28
Velocity of sound in nitrogen, vs = g RT = 1.4 × 296.94 × 366 = 390 m/s
v 195
M= = = 0.5
vs 390

1
36. (d) Flight Mach number of Aircraft = =2
sin 30°
58. (c) At ct rt = Ae ce re
ct rt
Ae = × ×At
ce re

de2 = 0.8 × 2.45 × 502 = 4900


de = 70 mm

60. (d) ce = 2 × 103 × ( 2800 − 2555) = 700 m/s

M06_THERMAL ENGINEERING_SE_XXXX_CH06.indd 312 6/15/2018 3:34:10 PM


Flow Through Steam Nozzles 313

Review Questions
1. Define a nozzle.
2. Differentiate between a nozzle and a diffuser.
3. What is a throat of nozzle?
4. What is the value of n in pv n = constant for saturated steam?
5. Write the formula for critical pressure ratio in terms of index of expansion.
6. Define Mach number.
7. What are the reasons for reduction in velocity of steam in the nozzle?
8. What are the effects of friction losses in the nozzle?
9. Define nozzle efficiency.
10. What is metastable steam flow?
11. What is Wilson line?
12. Write the conditions for a nozzle to be subsonic and supersonic.
13. Define stagnation enthalpy and stagnation pressure.
14. Define back pressure.
15. What is a normal shock?
16. Explain Fanno line and Rayleigh line.

Exercises
6.1 Steam is expanded in a set of nozzles from 10 bar and 200°C to 5 bar. What type of nozzle is it? Neglecting
the initial velocity find minimum area of the nozzle required to allow a flow of 3 kg/s under the given con-
ditions. Assume isentropic expansion of steam. [Ans. Convergent, 21 cm2]
6.2 Steam expands in a nozzle from 4 bar and 200°C to 1 bar. If the velocity of steam at entry is 60 m/s and
nozzle efficiency is 92%, determine the exit velocity. [Ans. 687.7 m/s]
6.3 Dry saturated steam enters a convergent–divergent nozzle at 11 bar and exit at a pressure of 2 bar. The flow
is frictionless and isentropic, following the law pv1.135 = constant. Calculate (a) the exit velocity of steam
and (b) the ratio of cross-section areas at exit and throat. [Ans. 774.6 m/s, 1.62]
6.4 A convergent–divergent nozzle is to be designed in which steam at 14 bar and 275°C is to be expanded to
1.05 bar. Calculate the necessary throat and exit diameter of the nozzle for steam discharge of 500 kg/h.
Assume that 12% of the total heat drop is lost due to friction only in the divergent part of the nozzle.
[Ans. 9.9 mm, 17.2 mm]
6.5 Steam at 15 bar, 350°C enters a nozzle of the convergent–divergent type and leaves at 1 bar. The throat
diameter is 6 mm and length of the diverging part is 80 mm. Determine the cone angle of the divergent part.
Assume that 12 per cent of the total available enthalpy drop is lost in friction in the divergent part only.
[Ans. 4° 22′]
6.6 A convergent–divergent nozzle is required to discharge 2 kg/s of stream. The nozzle is supplied with steam
at 6.9 bar, 180°C and discharge takes place against a back pressure of 0.98 bar. Expansion up to throat is
isentropic and the frictional resistance between the throat and the exit is equivalent to 62.76 kJ/kg of steam.
Taking approach velocity of 75 m/s and throat pressure 3.9 bar, estimate: (a) suitable areas for the throat and
exit and (b) overall efficiency of the nozzle based on the enthalpy drop between the actual inlet pressure,
temperature and the exit pressure. [Ans. 17.66 cm2, 41.67 cm2, 82.25%]
6.7 The nozzles in the stage of an impulse turbine receive steam at 17.5 bar, 300°C and pressure in the wheel
chamber is 10.5 bar. If there are 16 nozzles, then find the cross-sectional area at the exit of each nozzle for a
total discharge to be 280 kg/min. Assume nozzle efficiency of 90%. [Ans. 1.36 cm2]

M06_THERMAL ENGINEERING_SE_XXXX_CH06.indd 313 6/15/2018 3:34:11 PM


314 Chapter 6

 6.8   Steam at a pressure of 10 bar and 95% dry is supplied through a convergent–divergent nozzle to the wheel
chamber where the pressure is maintained at 0.12 bar. The mass flow rate through the nozzle is 8.16 kg/kWh
and the work developed by the wheel is 110 kW. Determine the following:
(a) Pressure of the throat for maximum discharge
(b) Number of nozzles required if throat diameter of each nozzle is 5 mm
(c) The exit diameter of nozzle if 10% of overall enthalpy drop overcomes friction in the divergent portion
[Ans. 5.93 bar, 9, 18.1 mm]

 6.9   Five kilogram of steam per minute passes through a convergent–divergent nozzle. The pressure and temper-
ature of steam supplied to the nozzle is 10 bar and 200°C, respectively. The discharge pressure is 0.1 bar.
The expansion is supersaturated up to throat and in thermal equilibrium afterwards. Calculate (a) the area
of nozzle at exit, (b) the maximum degree of super-saturation, and (c) the degree of under cooling at the
throat. Assume pv1.3 = constant for super-saturated flow. [Ans. 8.4 cm2, 1.65, 18°C]
6.10  Supersaturated expansion occurs in a nozzle supplied with steam at 20 bar and 325°C. Exit pressure is 3.6
bar. For a flow rate of 7.5 kg/s, determine (a) the throat and exit areas and (b) the degree of undercooling at
the exit. [Ans. 28.9 cm2, 42.8 cm2, 10.9°C]
6.11  A converging nozzle has an exit area of 0.001 m2. Air enters the nozzle with negligible velocity at a pressure
of 1.0 MPa and a temperature of 360 K. For isentropic flow of an ideal gas with k = 1.4, determine the mass
flow rate and the exit Mach number for back pressure of (a) 500 kPa and (b) 784 kPa.
[Ans. 2.13 kg/s, 1.79 kg/s]
6.12   Air flows through a convergent–divergent nozzle. At some section in the nozzle the pressure is 2 bar, the
velocity is 171.1 m/s and the temperature is 1200 K. The area of flow at this section is 1000 cm2. Assuming
isentropic flow conditions, determine the following:
 (a) Stagnation temperature and stagnation pressure.
(b) Sonic velocity and Mach number at this section.
 (c) Velocity, Mach number, and area of flow at the exit section if the pressure at that section is 1.013 bar.
(d) Pressure, temperature, velocity, and area of flow at the throat of the nozzle.
Assume cp = 1.005 kJ/kg.K and cv = 0.718 kJ/kg.K [ESE, 1983]
6.13  A set of steam nozzles in an impulse turbine stage is supplied with steam at 20 bar and 230°C. The mass
flow of steam is 60 kg/s. The steam is expanded from rest to a back pressure of 14 bar with an efficiency of
98%. The mean diameter of nozzle disc is 800 mm and the nozzle angle is 20 degrees. Assuming that steam
is admitted all around the periphery of the nozzle disc, determine:
 (a)  whether a convergent or a convergent–divergent nozzle is to be used,
(b)  the velocity of flow of steam at the exit of the nozzle,
 (c)  the area of flow normal to the axis of the stage, and
(d)  the height of the nozzles.
Assume adiabatic index for superheated steam to be 1.3. What would be the mass flow through the set of
nozzles when the back pressure is reduced to 9 bar keeping the upstream conditions same as before?
[ESE, 1984]
6.14  Air is isentropically expanded in a convergent–divergent nozzle from an initial pressure of 5 bar and 25°C
to a back pressure of 1.5 bar. The velocity of the air entering the nozzle is 100 m/s. The mass flow rate of the
air is 2 kg/s. Determine (a) Mach number at inlet to the nozzle, (b) pressure at the throat, (c) area of flow at
the throat, and (d) the area of flow at the exit of the nozzle. Assume for air the ratio of specific heats to be
1.4 and R to be 0.287 kJ/kg.K. [ESE, 1986]
6.15  Air enters a constant-area combustion chamber at a Mach number of 0.25 with a total pressure of 5.5 bar.
Combustion gas leaves the chamber with a Mach number of 0.35. Neglecting friction and the mass of the
fuel, determine the drop in total pressure of air in the combustion chamber. Assume cp/cv = 1.35.
[ESE, 1995]

M06_THERMAL ENGINEERING_SE_XXXX_CH06.indd 314 6/15/2018 3:34:11 PM


Flow Through Steam Nozzles 315

ANSWERS TO MULTIPLE-CHOICE QUESTIONS


1. (b) 2. (c) 3. (c) 4. (c) 5. (d)
6. (c) 7. (d) 8. (c) 9. (b) 10. (d)
11. (a) 12. (a) 13. (d) 14. (b) 15. (b)
16. (b) 17. (d) 18. (c) 19. (b) 20. (c)
21. (a) 22. (d) 23. (d) 24. (d) 25. (a)
26. (c) 27. (d) 28. (b) 29. (b) 30. (b)
31. (c) 32. (a) 33. (a) 34. (d) 35. (a)
36. (d) 37. (a) 38. (b) 39. (c) 40. (d)
41. (a) 42. (c) 43. (c) 44. (b) 45. (d)
46. (c) 47. (c) 48. (a) 49. (c) 50. (c)
51. (b) 52. (a) 53. (b) 54. (c) 55. (b)
56. (c) 57. (a) 58. (c) 59. (d) 60. (d)
61. (b)

M06_THERMAL ENGINEERING_SE_XXXX_CH06.indd 315 6/15/2018 3:34:11 PM


Chapter 7

Steam Turbines

7.1 ❐ PRINCIPLE OF OPERATION OF STEAM TURBINES


A steam turbine works on the dynamic action of steam. Steam is caused to fall in pressure in a
passage or nozzle. Due to this fall in pressure, certain amount of heat energy is converted into kinetic
energy to give it high velocity. The high velocity steam impinges on the moving blades of the turbine
and changes the direction of motion and thus gives rise to a change in momentum, therefore, a force.
Essentially, the nozzles direct the steam so that it flows in a well-­formed high velocity jet. Moving
buckets convert this high-velocity jet to mechanical work in a rotating shaft.
In impulse turbines, there is a drop of steam pressure and consequent development of kinetic
energy takes place solely in the stationary nozzles, and the work is obtained by the conversion of this
kinetic energy into work on moving blades. In the reaction turbine, only a part of the kinetic energy
conversion occurs in the stationary nozzles, whereas the remainder of the kinetic energy conversion is
accomplished by a pressure drop in steam as it passes through the moving blades.

7.2 ❐ CLASSIFICATION OF STEAM TURBINES


On the basis of principles of operation, steam turbines are classified as follows:
1. Impulse turbine
2. Reaction turbine
3. Impulse-reaction turbine
1. Impulse Turbine: This type of turbine is called a simple impulse turbine because the expansion
of steam takes place only in one set of nozzles, as shown in Fig. 7.1. The pressure of steam falls
from that in the steam nozzle to that existing in the condenser, whereas the steam flows through it.
Hence, the pressure in the wheel chamber is practically equal to the condenser pressure.
The high-velocity jet coming out from the nozzle impinges on the blades, so that the whole
kinetic energy is converted into mechanical energy. In practice, this type of turbine is used for
small power ratings. The rotor diameter is kept small and consequently, the rotational speed
becomes very high. This is known as De-Laval turbine.

M07_THERMAL ENGINEERING_SE_XXXX_CH07-Part-1.indd 316 6/15/2018 3:36:57 PM


Steam Turbines 317

Nozzle
Moving
Steam blades
from
boiler Exit

Velocity
Pressure
Pressure
of steam
at inlet Velocity of steam
at exit

Velocity of Condenser pressure


steam at inlet

Figure 7.1  Simple impulse turbine

2. Reaction Turbine: In a reaction turbine, there are guide blades instead of nozzles which convert the
pressure energy into kinetic energy. The steam passing over the moving blades has the difference of
pressure at the inlet tip and exit tip; hence, there is a drop of pressure in steam while passing over
the moving blades. The fixed blade serves the purpose of nozzles which changes the direction of
steam and at the same time, allows it to expand to a higher velocity. The pressure of steam falls as it
passes over the moving blades. The diameter of each stage of a reaction turbine must increase after
each group of blade rings in order to accommodate the increased volume of steam at lower pressure.

Fixed Moving Fixed Moving

Inlet Exit

Pressure

Pressure
of steam Velocity
at inlet

Condenser
pressure

Velocity of Velocity of
steam at inlet steam at exit

Figure 7.2  Impulse-reaction turbine

M07_THERMAL ENGINEERING_SE_XXXX_CH07-Part-1.indd 317 6/15/2018 3:36:59 PM


318 Chapter 7

3. Impulse-Reaction Turbine: If the pressure of steam at outlet from the moving blades of the tur-
bine is less than that at the inlet side of the blades, the drop in pressure suffered by steam during
its flow through the moving blades causes a further generation of kinetic energy within the blades
and adds to the preparing force which is applied to the turbine shaft; such a turbine is called
impulse-reaction turbine.
This type of turbine is shown in Fig. 7.2. In this type of turbine, there is application of both
principles, namely, impulse and reaction. There are several rows of moving blades fixed to the
shaft and equal number of fixed blades attached to the casing. The fixed blades correspond to
the nozzles of an impulse turbine. Steam is admitted for the entire circumference and therefore,
there is all-round admission. In passing through the first set of fixed blades, the steam undergoes
a small pressure drop and the velocity is increased. It enters the first set of moving blades and
suffers the change of direction and therefore momentum. This gives an impulse to the blades.
However, here, the passage to the blades is so designed that there is also a small drop in pressure
in the moving blades, giving rise to an increase in kinetic energy. This drop in pressure gives rise
to reactions in the direction opposite to that of added velocity. Thus, the driving force is the vector
sum of impulse and reaction turbines. Normally, the turbine is known as reaction turbine. This is
also called Parson’s reaction turbine.

7.3 ❐ COMPARISON OF IMPULSE AND REACTION


TURBINES
The comparison of impulse and reaction turbines is given in Table 7.1.

Table 7.1  Comparison of impulse and reaction turbines

S. no. Particulars Impulse turbine Reaction turbine


 1. Expansion of steam Steam expands completely in the Partially in the fixed blades and
(Pressure drop) nozzles subsequently in the rotor blades
 2. Relative velocity of steam Remains constant over the blades Increases as steam passes over the
blades
 3. Blade profile Symmetrical, area of flow Aerofoil section, area of flow
remains constant changes
 4. Pressure on both ends of Same Different
moving blades
 5. Number of stages for More Less
same power developed
 6. Blade efficiency Less More
 7. Admission of steam At fixed locations of nozzles All-round or complete
 8. Power developed Low High
 9. Space required Less More
10. Blade manufacture Not difficult Difficult

M07_THERMAL ENGINEERING_SE_XXXX_CH07-Part-1.indd 318 6/15/2018 3:36:59 PM


Steam Turbines 319

7.4 ❐ COMPOUNDING OF IMPULSE TURBINES


If the entire pressure drop from boiler pressure to condenser pressure is carried out in a single stage
nozzle, the velocity of steam entering the turbine blades will be very high and consequently, the tur-
bine speed will be very high. Such high speed turbine rotors are not useful for practical purposes and
a reduction gearing is necessary between the turbine and the generator driven by the turbine. There
is also a danger of structural failure of the blades due to excessive centrifugal stresses. Further, the
velocity of steam at the exit of the turbine is also high when a single stage of blades is used. This
gives rise to considerable loss of kinetic energy, reducing the efficiency of the unit. This loss of kinetic
energy is termed as ‘carry over losses’ or ‘leaving losses’.
Compounding is a method employed for reducing the rotational speed of the impulse turbine to
practical limits by using more than one stage. There are three methods of compounding impulse tur-
bines as follows:
1. Velocity compounding
2. Pressure compounding
3. Pressure and velocity compounding
1. Velocity Compounding: This system consists of a nozzle or a set of nozzles and a wheel fitted with
two or more rows of blades as shown in Fig. 7.3. It has two rings of moving blades and two rows
of fixed or guide blades placed. Steam entering the nozzle expands from the initial pressure to the
exhaust pressure. On passing through the first row of moving blades, the steam gives only a part
of kinetic energy and issues from this row of blades with a fairly high velocity. It then enters the
guide blade and the moving blade. There is a slight drop in velocity in the guide blade due to friction.

Nozzle Moving Fixed


Moving
Steam Fixed
from
boiler

Specific
volume

Pressure Velocity
of steam
at inlet Velocity of
steam at exit

Velocity of Pressure Condenser


steam at inlet pressure

Figure 7.3  Velocity-compounded impulse turbine

M07_THERMAL ENGINEERING_SE_XXXX_CH07-Part-1.indd 319 6/15/2018 3:37:00 PM


320 Chapter 7

While passing through the second row of moving blades, the steam suffers a change in momen-
tum and gives a part of kinetic energy to rotor. The steam leaving from the second row of
moving blades is redirected to the second row of guide blades. By doing work on the third
row of moving blades, the steam finally leaves the wheel in more or less axial direction with
a certain residual velocity, about 2% of the initial velocity of steam at nozzle exit. Thus, the
whole of velocity is not utilised on one row of moving blades but in a number of stages, as in a
Curtis turbine.
2. Pressure Compounding: This arrangement consists of allowing the expansion of steam in a
number of steps by having a number of simple impulse turbines in the series on the same shaft, as
shown in Fig. 7.4. The exhaust steam from one turbine enters the nozzles of succeeding turbines.
Each of the simple impulse turbines is called a ‘stage’ of the turbine comprising its sets of noz-
zles and blades. This is equivalent to splitting up the whole pressure drop into a series of smaller
pressure drops, hence, the term ‘pressure compounding’.
The pressure compounding causes a smaller transformation of heat energy into kinetic energy
to take place in each stage than in a simple impulse turbine. Hence, the steam velocity is much
lower, with the result that the blade velocity and rotational speed may be lowered. The leaving
loss is only 1–2% of the total available energy. Rateau turbine is a pressure compounded turbine.
3. Pressure and Velocity Compounding: It is a combination of pressure compounding and velocity
compounding. The total pressure drop of steam is divided into stages and the velocity of each
stage is compounded. This allows a bigger pressure drop in each stage and hence, less stages are

Moving Moving
blades Nozzle blades
Nozzle

Inlet Exit

Specific volume

Velocity
Pressure
of steam Pressure
at inlet
Velocity of
steam at exit

Velocity of Condenser
steam at inlet pressure

Figure 7.4  Pressure compounded impulse turbine

M07_THERMAL ENGINEERING_SE_XXXX_CH07-Part-1.indd 320 6/15/2018 3:37:01 PM


Steam Turbines 321

Nozzle Moving Fixed Moving Nozzle Moving Fixed Moving Nozzle

Specific
volume
Velocity

Velocity
Pressure Pressure of steam
of steam at exit
at inlet

Velocity First Second


of steam stage stage Condenser
Condenser
at inlet pressure
pressure

Figure 7.5  Pressure- and velocity-compounded impulse turbine

necessary which require a shorter turbine for a given pressure drop. Such a turbine is shown in
Fig. 7.5. The diameter of such a turbine increases in each stage in order to accommodate for a
larger volume of steam at the lower pressure. The pressure is constant in each stage.

7.5 ❐ VELOCITY DIAGRAMS FOR IMPULSE STEAM


TURBINE
Let      u = circumferential or tangential linear velocity of blades

va1 = absolute velocity of steam at inlet of moving blade


 va2 = absolute velocity of steam at outlet of moving blade


vw1 = velocity of whirl at the entry of moving blade
= va1 cos a1
vw2 = velocity of whirl at the exit of moving blade
= va2 cos a 2
  vf 1 = velocity of flow at inlet of moving blade

= va1 sin a1
  vf 2 = velocity of flow at outlet of moving blade

= va2 sin a 2


  vr1 = relative velocity of steam at entrance to moving blade
  vr2 = relative velocity of steam at exit of moving blade

M07_THERMAL ENGINEERING_SE_XXXX_CH07-Part-1.indd 321 6/15/2018 3:37:02 PM


322 Chapter 7

m = mass rate of flow of steam, kg/s.


   D = diameter of blade drum.
     h = height of blade.
  a1 = angle which the absolute velocity of steam at inlet makes with the plane of moving blades,
or nozzle angle or outlet angle of fixed blades.
  a2 = angle which the absolute velocity at outlet makes with the plane of moving blades or inlet
angle of fixed blade.
  b1 = inlet angle of moving blade.
  b2 = exit angle of moving blade.
The jet of steam impinges on the moving blades at angle a1 to the tangent of wheel with a velocity va1.
This velocity va1 has the following components.
1. Tangential or whirl component vw1 and since it is in the same direction as the motion of blades, it
is the actual component which does work on the blade.
2. Axial or flow component vf 1 and since it is perpendicular to the direction of motion of blade, it
does no work. However, this component is responsible for the flow of steam through the turbine.
This component also causes axial thrust on the rotor.
The velocity diagrams at inlet and outlet of a moving blade are shown in Fig. 7.6(a). Figure 7.6(b)
shows the combined velocity diagrams.

vw1
u

a1 b1

vr1 vf1
va1 vw2 vw1
u
A B
F E
Moving a2 a1 b2 b1
blade
u
vf2
Va2 vr2 ≤ vr1 vf1
vr2 vr1
vr2 va1
va2 vf2
D
b2 a2

u vw2 C
(a) (b)

Figure 7.6 Velocity diagrams for impulse turbine: (a) Velocity diagrams at inlet and exit,
(b) Combined velocity diagrams

M07_THERMAL ENGINEERING_SE_XXXX_CH07-Part-1.indd 322 6/15/2018 3:37:03 PM


Steam Turbines 323

For blades with a smooth surface, it can be assumed that friction loss is very less or zero. However,
there is always a certain loss of velocity during the flow of steam over the blade and this loss is taken
into account by introducing a factor called blade velocity coefficient, K. It is given by
Blade velocity coefficient, K = vr2/vr1, where vr2 < vr1.
Note that K accounts for the loss in relative velocity due to friction of blades.
1. Power Developed by the Turbine:
Work done per kg of steam,
w = Force in the direction of blade motion × Distance travelled in the direction of force
= Rate of change of momentum × Distance travelled
= [vw1 − (− vw2)] × u
= (vw1 + vw2) u
Power developed by the turbine for a mass rate of flow of m kg/s of steam,
m ( vw1 + vw 2 )u
P= kW (7.1)
103
2. Diagram or Blade Efficiency, gd or gb:
For a single blade stage,
Work done on blades per kg of steam
hb =
Energy supplied per kg of steam
( vw1 + vw 2 )u 2( vw1 + vw 2 )u
= = (7.2)
va21 / 2 va21

3. Gross or Stage Efficiency, gs:


Work done per kg of steam
hs =
Theoretical enthalpy drop in thhe nozzle per kg of steam
( vw1 + vw 2 )u
= (7.3)
( ∆h)isen
Stage efficiency takes into account the losses in the nozzle.
∴ hs = hb.hn (7.4)
Actual enthalpy drop in nozzle
where nozzle efficiency, hn =
Isentropic enthalpy drop durring expansion
2
= va1 /[2(∆h)isen ]
4. Axial Thrust: The axial thrust on the wheel is generated due to the difference between the veloc-
ity of flow at inlet and outlet.
Axial thrust, Fa = Mass rate of flow of steam × Change in axial velocity
    = m (vf 1 − vf  2)(7.5)

M07_THERMAL ENGINEERING_SE_XXXX_CH07-Part-1.indd 323 6/15/2018 3:37:06 PM


324 Chapter 7

5. Energy converted into heat due to blade friction:


= Loss of kinetic energy during flow of steam over the blades
1
= m ( vr21 − vr22 ) (7.6)
2
1
6. Energy lost at exit = m vr22 (7.7)
2

7.5.1  Condition for Maximum Blade Efficiency


2( vw1 + vw 2 )u
Blade efficiency, hb =
va21
From velocity diagrams given in Fig. 7.6(b), we have
vw1 + vw2 = AE + AF = (AB + BE) + (BF − AB)
= BE + BF
= vr1 cos b1 + vr2 cos b2
 v cos b 2 
= vr1 cos b1 1 + r 2 
 vr1 cos b1 
= vr1 cos b1 [1 + KC]
vr 2 cos b 2
where K = and C =
vr1 cos b1
Now BE = AE − AB = va1 cos a1 − u
2u( va1 cos a1 − u )(1 + KC ) u  u  
∴ hb = =2  cos a1 −  (1 + KC ) 
va21 va1  va1  
u
Let speed ratio, r =
va1
∴ hb = 2r [(cos a1 − r) (1 + KC)(7.8)
d
For hb to be maximum, (h ) = 0
dr b
∴          
cos a1 − 2r = 0
cos a1
or              r = (7.9)
2
 cos a1  
(hb ) max = cos a1  cos a1 −  (1 + KC ) 
  2 
cos 2 a1
= (1 + KC ) (7.10)
2

M07_THERMAL ENGINEERING_SE_XXXX_CH07-Part-1.indd 324 6/15/2018 3:37:11 PM


Steam Turbines 325

For symmetrical blades, b1 = b2, so that C = 1 and with no friction over the blades, K = 1
∴ (hb)max = cos2 a1(7.11)

7.5.2  Maximum Work Done


Now, w = (vw1 + vw2)u
= (va1 cos a1 − u) (1 + KC) u
= 2u (va1 cos a1 − u) for K = 1 and C = 1
wmax = 2u [va1 × 2h − u)
 2u 
= 2u  va1 × − u
 va1 
2
= 2u (7.12)
Equations (7.11) and (7.12) represent parabola. The blade efficiency has been plotted against speed
ratio in Fig. 7.7, without losses and with losses for a1 = 20°, b1 = b2 and K = 0.85.

a
b
1.0
0.8
0.6 a 1= 20°
h b 0.4
b1= b2
0.2 K = 0.85 a - without losses
b - with losses
O
0.2 0.4 0.6 0.8
r

Figure 7.7  Blade efficiency v’s speed ratio.

7.5.3  Velocity Diagrams for Velocity Compounded Impulse Turbine


The velocity diagrams for the first and second stage moving blades of a velocity-compounded impulse
turbine are shown in Fig. 7.8(a) and Fig 7.8(b), respectively. Consider that the final absolute velocity
of steam leaving the second row is axial. The velocity u of the blades for both the rows is the same as
they are mounted on the same shaft and are of equal height.
Work done in the first row of moving blades,
w1 = u(vw1 + vw2) = u(vr1 cos b1 + vr2 cos b2)
If there is no friction loss, vr1 = vr2 and for symmetrical blades, b1 = b2.
∴              w1 = 2u vr1 cos b1 = 2u(vr1 cos a1 − u)
Now,            va3 = va2
Work done in the second row of moving blades,
w2 = u vw3 as vw2 = 0 and a4 = 90°.
= u (vr3 cos b3 + vr4 cos b4)

M07_THERMAL ENGINEERING_SE_XXXX_CH07-Part-1.indd 325 6/15/2018 3:37:12 PM


326 Chapter 7

vw1 vw3
vw2 u u

b4 b3
a2 a1 b2 b1 a3

a4 = 90°
vf2
va2 vf4 = va4
vr2 ≤ vr1 vf1 vr4
vr1
vr2 va1 vr3
va3

(a) (b)

Figure 7.8 Velocity diagrams for velocity compounded impulse turbine:


(a) First stage, (b) Second stage

For no friction, vr3 = vr4, and for symmetrical blades, b3 = b4.


∴ w2 = 2uvr3 cos b3 = 2u (va3 cos a3 − u)
For a3 = a 2
va3 cos a3 = va2 cos a 2
= vr2 cos b2 − u = vr1 cos b1 − u
= (va1 cos a1 − u) − u = va1 cosa1 − 2u
∴ w2 = 2u [(va1 cos a1 − 2u) − u] = 2u (va1 cos a1 − 3u)
Total work done, wt = w1 + w2
= 2u (va1 cos a1 − u) + 2u (va1 cos a1 − 3u)
= 2u (2va1 cos a1 − 4u)
= 4u (va1 cos a1 − 2u)
wt 8u
Blade efficiency, hb = = ( va1 cos a1 − 2u )
2
va1 / 2 va21

= 8r (cos a1 −2r)(7.13)
For hb to be maximum,
cos a1 − 4r = 0

cos a1
r= (7.14)
4

M07_THERMAL ENGINEERING_SE_XXXX_CH07-Part-1.indd 326 6/15/2018 3:37:14 PM


Steam Turbines 327

8 cos a1  cos a1 
(hb ) max =  cos a1 − = cos 2 a1 (7.15)
4  2 
4u
For va1 =
cosa1

 4u 
( wt ) max = 4u  . cos a1 − 2u  = 8u 2 (7.16)
 cos a 1 
cos a1
In general,  r = (7.17)
2n

Total work done


and work done in the last row = (7.18)
2n
where n = number of rotating blade rows in series.

7.5.4  Effect of Blade Friction on Velocity Diagrams


In an impulse turbine, the relative velocity at the outlet will be the same as the relative velocity at the
inlet, if friction is neglected. In practice, there is a frictional resistance to the flow of steam jet over the
blade, the effect of which is to cause a slowing down of the relative velocity. Usually, there is a loss of
10−15% in the relative velocity due to friction. Owing to frictional resistance of the blades, it will be
found that vr2 = Kvr1, where K is a coefficient, which takes the blade loss due to friction into account.
The velocity diagrams considering blade friction are shown in Fig. 7.9. Here, the inlet diagram is first
drawn and the line BC, of an unknown length, is drawn at the correct angle b 2. Mark off on line BD =
vr1, the friction loss of relative velocity DD′, then BD′ = Kvr1. With B as centre, draw an arc of radius
BD′ to cut BC at C. Then BC = vr2 = Kvr1.
By joining A and C, the line AC representing va2 is obtained. This completes the outlet velocity
diagram.

vw2 vw1
A u B
E a1 b2 F
a2 b1

vf2
va2
vf1
vr2
vr1
C va1 D′

Figure 7.9  Velocity diagrams for impulse turbine considering blade friction

M07_THERMAL ENGINEERING_SE_XXXX_CH07-Part-1.indd 327 6/15/2018 3:37:17 PM


328 Chapter 7

7.5.5  Impulse Turbine with Several Blade Rings


The stage of an impulse turbine, which is compounded for velocity, consists of alternate rings of
nozzles, and moving and fixed blades. As the blade velocity u is constant for all the moving blade
rings of the stage, the velocity diagrams for all the blade rings can be superimposed on the same base,
represented by u. Figure 7.10 shows the velocity diagram for a stage consisting of two moving and one
fixed blade rings. Let us assume that the following data is known:
1. Blade velocity, u.
2. Nozzle angle, a1.
3. The moving blade angles, b1 and b 2, which are assumed to be same for both blade rings.
4. The velocity of steam va1 discharged from nozzle.
5. Blade friction loss, 10%, i.e., K = 0.9.
The following steps may be followed for drawing the velocity diagrams:
1. Draw AB = u to any convenient scale. Draw AD inclined at angle a1 to AB so that AD = va1 to the
same scale. Join BD. Then BD = vr1 for the first moving blade ring.
2. Mark off DD′ = 0.1 × BD so that vr2 = 0.9 vr1 = BD′. With B as centre, draw an arc of radius BD′
to cut BC at C where the line BC is drawn at angle of b 2 to BA. Join AC. Then AC = va2 for the
first moving blade ring.
3. The steam now flows over the fixed blade ring and will lose 10% of its velocity during the pas-
sage. Hence, mark off CC′ = 0.1 × AC. With A as centre, draw on arc of radius AC′ to cut BD at G.

vw2 vw1
vw3
vw4
u
E J A B K F
a4 a1 b2 b1
a3
a2
vf2 vf4 vf3 vf1
va4 va3
vr3

G′
vr2
G
H
va2

C′ vr4 Second moving blade

C vr1
Fixed blade va1
D′

D
First moving blade

Figure 7.10  Velocity diagrams for impulse turbine considering friction with several blade rings

M07_THERMAL ENGINEERING_SE_XXXX_CH07-Part-1.indd 328 6/15/2018 3:37:19 PM


Steam Turbines 329

Then AG = va3 representing the steam velocity when entering the second moving blade ring. The
velocity diagram for the second blade ring is triangle AGB. BG = vr3, the relative velocity.
4. The steam now flows over the second moving blade and loses one-tenth of its relative velocity due
to friction., Hence, mark off GG′ = 0.1 × BG so that BG′ = 0.9 BG. With B as centre and radius
BG′, draw an arc to cut BC at H, then AH = va4 and BH = vr4.
  a 2 = angle of discharge from first moving blade
    a4 = angle of discharge from second moving blade
  a 3 = outlet angle of fixed blade.
Work done per kg of steam for first moving blade ring,
w1 = (vw1 + vw2)u
Work done per kg of steam for second moving blade ring.
w2 = (vw3 + vw4)u
Total Work done, wt = w1 + w2
= [(vw1 + vw2) + (vw3 + vw4)]u
m u ∑ ( vw1 + vw 2 )
Power developed per stage = kW
103

2u ∑ ( vw1 + vw 2 )
Blade efficiency, hb =
va21

u ∑ ( vw1 + vw 2 )
Stage efficiency, hs =
( ∆h)isen
Total axial thrust = m ∑ ( v f 1 − v f 2 )

7.6 ❐ ADVANTAGES AND LIMITATIONS OF VELOCITY


COMPOUNDING
7.6.1 Advantages
1. Fewer number of stages are required and therefore, the initial cost is less.
2. The space required is less.
3. The pressure in the housing is considerably less which requires cheaper and thin casing for the
turbine.

7.6.2 Limitations
1. The friction losses are larger due to the high velocity of steam.
2. The maximum blade efficiency and efficiency range decrease with the increase in the number of
stages.
3. The power developed in each successive blade row decreases with increase in number of rows.
4. All the stages are not used with equal economy.

M07_THERMAL ENGINEERING_SE_XXXX_CH07-Part-1.indd 329 6/15/2018 3:37:20 PM


330 Chapter 7

7.7 ❐ VELOCITY DIAGRAMS FOR IMPULSE-REACTION


TURBINE
The steam passes to the fixed blades with velocity va1. Fixed blades also act as nozzles with pressure
drop occurring, while steam passes through them so that there is gain in kinetic energy and steam
leaves the fixed blades with velocity va2. The expansion of steam and the enthalpy drop occur in fixed
and moving blades. The velocity diagrams for an impulse-reaction turbine are shown in Fig. 7.11.
1. Degree of Reaction: The degree of reaction Rd of a reaction turbine is defined as the ratio of
enthalpy drop over moving blades to the total enthalpy drop in the stage. Thus

( ∆h) moving blade ∆hm


Rd = = (7.19)
( ∆h) moving blades + ( ∆h)fixed blades ∆hm + ∆h f

va21 − va22
Enthalpy drop through fixed blades, ∆hf =
2

vr 2 2 − vr 2 2
Enthalpy drop along the moving blades, ∆hm =
2
1 2
Kinetic energy supplied to moving blades, K.E. = v + ( vr 22 − vr12 ) 
2  a1 

vr 22 vr12
Neglecting friction of blade surface, = + ∆hm
2 2
Axial thrust on rotor = (vf 1 − vf 2) + (∆p)m × area of blade disc

Now total enthalpy drop in the stage = Work done by steam in the stage

∆hf + ∆hm = u (vw1 + vw2)

vw1
vw2 u

a2 a1 b2 b1

vf1
vf2 vr1
va2 vr2 > vr1
va1

vr2

Figure 7.11  Velocity diagrams for impulse-reaction turbine

M07_THERMAL ENGINEERING_SE_XXXX_CH07-Part-1.indd 330 6/15/2018 3:37:23 PM


Steam Turbines 331

vr22 − vr21
∴ Rd = (7.20)
2u( vw1 + vw 2 )
From Fig. 7.11, we have
vr2 = vf 2 cosec b2 and vr1 = vf 1 cosec b1
and vw1 + vw2 = vf 1 cot b1 + vf 2 cot b2
The velocity of flow generally remains constant through the blades.
∴ vf 1 = vf 2 = vf

v 2f (cosec2 b 2 − cosec 2 b1) v f  (cot 2 b 2 + 1) − (cot 2 b1 + 1) 


Rd = =  
2u v f (cot b1 + cot b 2 ) 2u  cot b1 + cot b 2 

v f  cot 2 b 2 − cot 2 b1  v f
=  = (cot b 2 − cot b1) (7.21)
2u  cot b1 + cot b 2  2u

1 vf
If Rd = 0.5, then = (cot b 2 − cot b1)
2 2u
or u = vf (cot b2 − cot b1)(7.22)
From Fig. 7.11, we have
u = vf (cot b2 − cot a2) = vf (cot a1 − cot b1)
Hence, b1 = a2 and b2 = a1(7.23)
This means that the moving and fixed blades must have the same shape if the degree of reaction
is 50%. This condition gives symmetrical velocity diagrams. This type of turbine is known as
Parson's Reaction Turbine.
2. Efficiency: We assume that the degree of reaction is 50%, that is, ∆hf  = ∆hm, the moving and
fixed blades are of the same shape, and the velocity of steam at exit from the preceding stage is
the same as the velocity of steam at the entrance to the succeeding stage.
The work done per kg of steam, w = u(vw1 + vw2) = u [va1 cos a1 + (vr2 cos b2 − u)]
Now, b2 = a1 and vr2 = va1
 2u v cosa u2 
w = u(2va1 cos a1 − u) = va21  a1 1
− 
 va21 va21 
2
= va1 [2r cos a1 − r2](7.24)
u
where r = is the speed ratio.
va1
2
va1
K.E. supplied to fixed blade =
2
vr22 − vr21
K.E. supplied to moving blade =
2

M07_THERMAL ENGINEERING_SE_XXXX_CH07-Part-1.indd 331 6/15/2018 3:37:27 PM


332 Chapter 7

Total energy supplied to stage,


va21 vr22 − vr21
∆h = +
2 2
For symmetrical blades, vr2 = va1
va21 va21 − vr21 v2
∆h = + = va21 − r1 (7.25)
2 2 2
From Fig. 7.11, we have
vr21 = va21 + u2 − 2va1 u cos a1
1
∴ ∆h = va21 − [va21 + u 2 − 2va1 u cos a1 ]
2
1 2
= [va1 + 2va1 u cos a1 − u 2 ]
2
v2
= a1 [1 + (2u/va1) cos a1 − u 2 va21 ]
2
2
va1
= [1 + 2 r cos a1 − r 2 ] (7.26)
2
W 2 r cos a1 − r 2
Blade efficiency, hb = =
∆h 1
(1 + 2 r cos a1 − r 2 )
2
2 r ( 2 cos a1 − r )
= (7.27)
1 + 2 r cos a1 − r 2
dhb
For hb to be maximum, =0
dr
(1 + 2r cos a1 − r 2) (4 cos a1 − 4r) −2r (2 cos a1 − r) (2 cos a1 − r) = 0
4 (cos a1 − r) (1 + 2r cos a1 − r 2) − 4r (cos a1 − r) (2 cos a1 − r) = 0
(cos a1 − r) [(1 + 2r cos a1 − r 2) − r (2 cos a1 − r)] = 0
∴ cos a1 − r = 0
or r = cos a1(7.28)
2 cos a1 ( 2 cos a1 − cos a1 ) 2 cos 2 a1
(hb ) max = = (7.29)
1 + 2 cos 2 a1 − cos 2 a1 1 + cos 2 a1

3. Height of Turbine Blading:


The height and thickness of blading are shown in Fig. 7.12.
Volume flow = Area × flow velocity
∴           mv  s = (p d − nt1) h1 vf 1 = (p d − nt2) h2 vf 2(7.30)
where d = mean diameter of turbine wheel
h1, h2 = blade heights at inlet and outlet respectively
t1, t2 = thickness of blades at inlet and outlet respectively.
n = number of blades

M07_THERMAL ENGINEERING_SE_XXXX_CH07-Part-1.indd 332 6/15/2018 3:37:32 PM


Steam Turbines 333

h2
h1

vf1 p vf2

(a) (b)

Figure 7.12  Height of turbine blading

For most of the turbines,


vf 1 = vf 2 = vf , h1 = h2 = h and t1 = t2 = t

∴  s = (p d − nt) hvf(7.31)


mv
Equation (7.31) gives height of blades.
The pitch of blades, p is given by:
 s = [n (p − t)h] vf
mv
Generally t << p,
∴  s = n ph vf = p dhvf
mv

pd
or   p= (7.32)
n

7.8 ❐ REHEAT FACTOR


The expansion of steam through a number of stages of turbine is shown in Fig. 7.13. A1B1 represents
isentropic expansion in the first stage. The actual state of steam with frictional reheating is shown by
point A2. Therefore, the actual heat drop in the first stage is A1C1. Similarly, the isentropic and actual
heat drops in the succeeding stages are shown by A2B2, A3B3, ….., A2C2, A3C3, ….., and so on. A1D
represents the isentropic heat drop between supply and condenser state of steam.
Actual enthalpy drop
Stage efficiency, h s =
Isentropic enthalpy drop
A1C1
For first stage, h s1 =
A1 B1

M07_THERMAL ENGINEERING_SE_XXXX_CH07-Part-1.indd 333 6/15/2018 3:37:35 PM


334 Chapter 7

h
A1
Condition
A2 curve
C1
A3
B1 C2
Enthalpy
C3 A4
B2 C4 A5 x=1
B3
B4

s
Entropy

Figure 7.13  Expansion of steam through a number of turbine stages

A2C2
For second stage, h s 2 = and so on.
A2 B2

Comulative enthalpy drop


Reheat factor =
Isentropic heat drop
n

A1 B1 + A2 B2 + .....
∑ Ai Bi
i =1
Rf = =  (7.33)
A1D A1D

Total actual turbine work


Turbine internal efficiency =
Isentropic work
n

A1C1 + A2C2 + .....


∑ AiCi
i =1
hi = = (7.34)
A1D A1D
If hs1 = hs2 = ….. = hs′ then
A1C1 A2C2 A C + A2C2 + .....
hs = = = ..... = 1 1
A1 B1 A2 B2 A1 B1 + A2 B2 + .....
n
∑ AiCi
i =1
or hs = (7.35)
n
∑ Ai Bi
i =1

M07_THERMAL ENGINEERING_SE_XXXX_CH07-Part-1.indd 334 6/15/2018 3:37:38 PM


Steam Turbines 335

From Eqs. (7.34) and (7.35), we get


n
∑ Ai Bi
i =1
hi = × h s (7.36)
A1D
Combining Eqs (7.33) and (7.36), we get
hi = hsRf
Reheat factor is always greater than one. Normally Rf lies between 1.02 to 1.05. The line passing
through A1, A2, … is called the condition curve.

7.9 ❐ LOSSES IN STEAM TURBINES


The various losses in steam turbines and their causes are as follows:
1. Residual Velocity Loss: This loss occurs due to the absolute exit velocity of steam and is equiv-
v2
alent to a2 ,where va2 is the absolute exit velocity of steam. In a single stage turbine, it may be
2
about 10−12% and can be reduced by using multi-stages.
2. Frictional and Turbulence Loss: Friction loss mainly occurs in nozzles and turbine blades. The
nozzle efficiency is used to account the friction loss. The loss due to friction and turbulence is
about 10%.
3. Leakage Loss: It occurs at the following points:
(a) Between the turbine shaft and bearings
(b) Between the shaft and stationary diaphragms carrying nozzles, and blade tips
(c) Through the labyrinth glands
The total leakage loss is about 1−2%.
4. Mechanical Friction Loss: Friction between shaft, bearing, and regulating valves accounts for
this loss. It can be reduced by proper lubrication.
5. Wet Steam Loss: The velocity of water particles is less than that of steam; therefore, water par-
ticles have to be dragged with the steam causing loss of kinetic energy.
6. Radiation Loss: This is due to much higher temperature of turbine as compared to the surround-
ings. This loss can be reduced by proper insulation.

7.10 ❐ TURBINE EFFICIENCIES


Work done on the blade
1. Blade or diagram efficiency,hb =
Energy entering the blade
Net work done on shaft per stage
2. Stage efficiency, h s =
Adiabatic heat drop per stage

M07_THERMAL ENGINEERING_SE_XXXX_CH07-Part-1.indd 335 6/15/2018 3:37:39 PM


336 Chapter 7

Net work done on blades − Disc friction and windage


=
Adiabatic heat drop per stage
Heat converted into useful work
3. Internal efficiency, hi =
Total isentropic heat droop
Work delivered at turbine coupling
4. Overall or turbine efficiency, hoverall =
Total adiabatic heat drop
Brake thermal efficiency
5. Net efficiency or efficiency ratio, h net =
Rankine thermal efficiency

7.11 ❐ GOVERNING OF STEAM TURBINES


The purpose of governing of steam turbines is to maintain the speed of a turbine sensibly constant,
irrespective of the load. The various methods of governing are as follows:
1. Throttle governing
2. Nozzle control governing
3. By-pass governing
4. Combination of throttle governing and nozzle control governing
5. Combination of throttle governing and by-pass governing

1. Throttle governing: The line diagram of throttle governing is shown in Fig. 7.14. The steam
flow to the turbine is throttled by a balanced throttle valve actuated by a centrifugal governor. An
oil differential relay is incorporated to magnify the small force produced by the governor for a
small change of speed to actuate the throttle valve. The throttle valve is moved by a relay piston.
A floating differential lever is fixed to its one end and a piston valve is fixed at some intermediate
point. The pilot piston valve consists of two piston valves covering ports without any overlap. The
piston valves are also operated by lubricating oil supplied by a pump at about 3 to 4 bar. The oil
from this chamber is returned by the oil drain.
Operation: Let the turbine work at full-rated load at the rated constant speed. If the load is
reduced, the energy supplied to the turbine will be in excess and the turbine rotor will acceler-
ate. Thus, the governor sleeve will lift. As the throttle valve position is assumed to be the same
momentarily, the pilot piston valve spindle will get lifted, opening the upper port to oil pressure
and lower port to oil return. The relay piston will thus close the throttle valve partially. The lower-
ing of the throttle valve spindle will lower the pilot piston spindle and close the ports. As soon as
the ports are closed, the relay piston gets stabilised in one position corresponding to the reduced
load. The change in the available enthalpy for throttle governing is shown in Fig. 7.15.
2. Nozzle control governing: The principle of nozzle control governing is accomplished by uncov-
ering as many steam passages as are necessary to meet the load by poppet valves. An arrangement
is shown in Fig. 7.16. The nozzles are divided into groups N1, N2, and N3 under the control valves
V1, V2, and V3, respectively. The number of nozzle groups may vary from three to five or more.
3. By-pass governing: The principle of by-pass governing is shown in Fig. 7.17. Steam entering the
turbine passes through the main throttle valve which is under the control of the speed governor
and enters the nozzle box or the steam chest. For loads greater than the economical load, a by-pass

M07_THERMAL ENGINEERING_SE_XXXX_CH07-Part-1.indd 336 6/15/2018 3:37:41 PM


Steam Turbines 337

Hand
Centrifugal wheel
governor

Maximum Differential
Full load
speed lever

Pilot
Minimum No load
piston
speed
Relay piston
Pilot to
close valve Cylinder
Oil pump

Pipe to
Oil return open valve Balanced
throttle
Steam valve
Gearing from
boiler Steam to
Turbine shaft turbine

ST

Exhaust

Figure 7.14  Throttle governing


Enthalpy

Throttling
re

re
su

su

re
∆h Part load

su
es

s
re

es
pr

rp

pr
∆h Full load

ly

se

ly
pp

pp
en
su

su
d
on
ad

ad
C

lo
l lo

t
ar
ul

,P
,F

p2
1
p

Supply pressure

Entropy

Figure 7.15  T-s diagram for throttling

M07_THERMAL ENGINEERING_SE_XXXX_CH07-Part-1.indd 337 6/15/2018 3:37:43 PM


338 Chapter 7

N2
N1

V3 V2 V1

N3

Steam inlet

Figure 7. 16  Nozzle governing

By-pass
valve Nozzle
By-pass steam
diaphragms

Steam chest
To lower
stages

Throttle valve

Rotor discs

Figure 7.17  By-pass governing

valve is opened, allowing steam to pass from the first stage, nozzle box into the steam chest, and
so into the nozzles of the fourth stage. The by-pass valve is not opened until the lift of the throttle
valve exceeds a certain amount. As the load diminishes, the by-pass valve closes first. The by-pass
valve is under the control of speed governor for all loads.

7.12 ❐ LABYRINTH PACKING


Labyrinth seals are characterised as controlled clearance seals without rubbing contact with the moving
parts and with some tolerable leakage. The fluid throttling is achieved in steps, using a series of small
chambers, where a sudden irreversible acceleration with subsequent deceleration of the leaking fluid
takes place. Every step down in this process of pressure dissipation is accompanied by a loss in pressure.

M07_THERMAL ENGINEERING_SE_XXXX_CH07-Part-1.indd 338 6/15/2018 3:37:45 PM


Steam Turbines 339

(c)

(a)

c (d)
12 34

c
(b) (e)
c

Figure 7.18 Labyrinth seal designs: (a) Straight shaft and straight housing, (b) Staggered,
(c) Staggered and stepped, (d) Stepped, (e) Interference

Due to the lack of direct rubbing contact with the moving shaft, the labyrinth seal is well suited for
sealing shafts operating at high rotational speeds, as in centrifugal compressors and steam turbines.
It requires neither lubrication nor maintenance. The simplest design of a labyrinth packing is with a
straight shaft and a straight housing is shown in Fig. 7.18(a). The modified labyrinth designs are of the
staggered, stepped, and interference configurations (Figs. 7.18(b)–(e)).

7.13 ❐ BACK PRESSURE TURBINE


There are several industries, such as paper making, textile, chemical, dyeing, sugar refining, and so on,
in which there is a dual demand for power and steam for heating and process work. Producing steam

F
c
e
Boiler Turbine

a Water supply Feed


Heater pump
Desuperheater

Figure 7.19  Back pressure turbine plant

M07_THERMAL ENGINEERING_SE_XXXX_CH07-Part-1.indd 339 6/15/2018 3:37:47 PM


340 Chapter 7

bc
Work
done in
turbine
Heat p0
Supplied p1
in boiler Heat
Enthalpy p2 available
for
process
a

Entropy

Figure 7.20  Thermodynamics of back pressure turbine

separately for power and heating is wasteful. If the turbine is operated with normal exhaust pressure
and the temperature of the exhaust steam is too low to be of any use for heating purposes, by suitable
modification of the initial and exhaust pressures, it would be possible to generate the required power
and still have available for process work a large quantity of heat in the exhaust steam.
The back pressure turbine may be used in cases where the power, which may be generated by
expanding steam from an economical initial pressure down to the heating pressure, is equal to, or
greater than the power requirements. The layout of such a plant is shown in Fig. 7.19.
Steam is generated in the boiler at a suitable working pressure and admitted to the turbine. The
exhaust steam from the turbine will normally be superheated and in most cases, is not suitable for
process work. A de-superheater is used to make it suitable for process work by spraying a jet of water,
thermostatically controlled, on the entering steam. The steam is cooled and the spray water evaporated.
The new saturated steam enters the heaters and is entirely condensed. The condensed steam may or
may not be returned to the boiler. The thermodynamics of back pressure turbine is shown in Fig. 7.20.

7.14 ❐ PASS OUT OR EXTRACTION TURBINE


In many cases, the power available from a back pressure turbine through which the whole of the heat-
ing steam flows is appreciably less than that required in the factory. In such cases, it would be possible
to install a back pressure turbine to generate additional power. However, generally, the functions of
both machines are combined in a single turbine. Such a turbine is shown in Fig. 7.21. The steam enters
the turbine and expands through the high pressure stages before the extraction branch. Here, a certain
quantity of steam is continuously being extracted for heating purposes, the remainder passes through
the pressure control valve into the low-pressure part of the turbine.

M07_THERMAL ENGINEERING_SE_XXXX_CH07-Part-1.indd 340 6/15/2018 3:37:48 PM


Steam Turbines 341

Pressure control valve

p1
w3

w1

p2 p3
High pressure
steam
w2 w3
Pass out steam To condenser

Figure 7.21  Pass out or extraction turbine

7.15 ❐ CO-GENERATION
The exhaust steam from a steam power plant is often rejected to the atmosphere. When the heat of
exhaust steam is used for heating of buildings, and heating required by many industrial processes,
the functions of heating and power production can often be combined effectively. This combination is
often called co-generation (Fig. 7.22). The process heater replaces the condenser of an ordinary Rank-
ine cycle. The pressure at the exhaust from the turbine is the saturation pressure corresponding to the
temperature desired in the process heater. Such a turbine is called back pressure turbine.

Turbine
Steam
generator
Process
heater

Pump

Figure 7.22 Co-generation

M07_THERMAL ENGINEERING_SE_XXXX_CH07-Part-1.indd 341 6/15/2018 3:38:02 PM


342 Chapter 7

Example 7.1
Steam at 4.9 bar and 160°C is supplied to a single stage impulse turbine at the rate of 60 kg/min.
From there, it is exhausted to a condenser at a pressure of 0.196 bar. The blade speed is 300 m/s.
The nozzles are inclined at 25° to the plane of the wheel and outlet blade angle is 35°. Neglect
friction losses and estimate (a) the theoretical power developed by the turbine, (b) the diagram
efficiency, and (c) the stage efficiency.  [IES, 1990]

Solution
Given that p1 = 4.9 bar, t1 = 160°C, m = 60 kg/min, p2 = 0.196 bar, u = 300 m/s, a1 = 25°, b 2 = 35°, K = 1
From Mollier diagram, at 4.9 bar and 160°C, h1 = 2700 kJ/kg
Assuming isentropic expansion, at 0.196 bar, h2 = 2200 kJ/kg
Inlet velocity of steam to turbine = outlet velocity of steam from nozzle

∴ va1 = v2 = 2( h1 − h2 ) = 2( 2700 − 2200) × 103 = 1000 m/s

The velocity diagrams are shown in Fig. 7.23.


vw1 = va1 cos a1 = 1000 cos 25° = 906.3 m/s
vw2 = vr2 cos b 2 − u
vr2 = vr1 for K = 1

vr21 = va21 + u2 − 2va1 u cos a1


= 10002 + 3002 − 2 × 1000 × 300 × cos 25° = 546215
or vr1 = 739 m/s
vw2 = 739 cos 35° − 300 = 305.4 m/s

m ( vw1 + vw 2 )u
(a) Theoretical power developed =
103
1(906.3 + 305.4) × 300
= = 363.5 kW
103

vw1
vw2 u
A a1 b2

va2
vr1
vr2 va1

C
Scale: 1 cm = 100 m/s

Figure 7.23  Velocity diagrams for a single stage impulse turbine

M07_THERMAL ENGINEERING_SE_XXXX_CH07-Part-1.indd 342 6/15/2018 3:38:06 PM


Steam Turbines 343

2u( vw1 + vw 2 ) 2 × 300 (906.3 + 305.4)


(b) Blade efficiency, hb = = = 0.727 or 72.7%
va21 1000 2
(c) Stage efficiency, hs = Work output/Heat input
= 363.5/(2700 − 2200) = 0.727 or 72.7%

Example 7.2
The first stage of an impulse turbine is compounded for velocity and has two rings of moving
blades and one ring of fixed blades. The nozzle angle is 20° and the leaving angles of the blades
are respectively: first moving 20°, fixed 25° and second moving 30°. The velocity of steam leaving
the nozzle is 600 m/s, the blade speed is 125 m/s, and the steam velocity relative to the blades is
reduced by 10% during the passage through each ring. Find the diagram efficiency under these
conditions and the power developed for a steam of 4 kg/s.

Solution
Given that u = 125 m/s, va1 = 600 m/s, K = 0.9, a1 = 20°, b2 = 20°, b3 = 25°, b4 = 30°, m = 4 kg/s
Choose a scale of 1 cm = 50 m/s. The velocity diagrams are drawn in Fig. 7.24.
ab = u = 125 m/s = 2.5 cm
∠bad = 20°, ad = va1 = 600 m/s = 12 cm
Join bd. By measurement, bd = 9.7 cm, bd′ = 9.7 × 0.9 = 8.7 cm
∠abc = 20°, bc = bd′
Join ac. By measurement, ac = 6.4 cm, ac′ = 0.9 × 6.4 = 5.7 cm

vw2 vw1
vw4 u vw3
20°
e j a 20° 25° b k f
30°
va2 vf4 vf3 vf1
vf2 va4
vr4 vr3 vr1
c′ vr2 va3 g′
g va1 d′
h
c d

Second moving blade

Fixed blade Scale: 1 cm = 50 m/s

First moving blade

Figure 7.24  Velocity diagrams for a two stage impulse turbine

M07_THERMAL ENGINEERING_SE_XXXX_CH07-Part-1.indd 343 6/15/2018 3:38:07 PM


344 Chapter 7

∠bag = 25°
bg = ac′
bg = 3.6 cm, bg′ = 0.9 × 3.6 = 3.2 cm
∠abh = 30°, bh = bg′
Join ah
vw1 = af = 11.3 cm, vw2 = ea = 5.8 cm, vw3 = ak = 5.1 cm, vw4 = aj = 0.4 cm

2u ∑ ( vw1 + vw 2 )
Blade efficiency, hb =
va21

2 × 125 × 50 × (11.3 + 5.8 + 5.1 × 0.4)


= = 0.785 or 78.5%
600 2

m u ∑ ( vw1 + vw 2 ) 4 × 125 × 50 × 22.6


Power developed = kW = = 565 kW
3
10 103

Example 7.3
In an impulse turbine, the mean diameter of the blades is 1.05 m and the speed is 3000 rpm. The
nozzle angle is 18°, speed ratio is 0.42, and the friction factor is 0.84. The outlet blade angle is to
be made 3° less than the inlet angle. The steam flow is 10 kg/s. Draw the velocity diagrams for the
blades and calculate (a) the tangential thrust, (b) the axial thrust, (c) the resultant thrust, (d) the
power developed, and (e) the blading efficiency.

Solution
Given that dm = 1.05 m, N = 3000 rpm, a1 = 18°, r = 0.42, K = 0.84, b2 = b1 − 3°, m = 10 kg/s
pdm N p × 1.05 × 300
Blade speed, u = = = 164.93 m/s
60 60
Draw the velocity diagrams as shown in Fig. 7.25.

vw2 vw1
u
e a b f
18° b 2 b1
vf2
va2 vf1
vr1
vr2
d c′
va1
c
Scale: 1 cm = 50 m/s

Figure 7.25  Velocity diagrams for an impulse turbine

M07_THERMAL ENGINEERING_SE_XXXX_CH07-Part-1.indd 344 6/15/2018 3:38:11 PM


Steam Turbines 345

u
r=
va1
164.93
or va1 = = 392.7 m/s
0.42
vr21 = u 2 + va21 − 2uva1 cosa1
  
= (164.93)2 + (392.7)2 − 2 × 164.93 × 392.7 × cos 18°= 58219
or vr1 = 241.28 m/s
vf 1 = va1 sin a1 = 392.7 sin 18° = 121.35 m/s
vf1 121.35
sin b1 = = = 0.50295
vr1 241.28
or           b1 = 30.2°
b2 = 30.2 – 3 = 27.2°
vr2 = K vr1 = 0.84 × 241.28 = 202.67 m/s
vf 2 = vr2 sin b2 = 202.67 sin 27.2° = 92.64 m/s
vw1 = va1 cos a1 = 392.7 cos 18° = 373.48 m/s
vw2 = vr2 cos b2 − u = 202.67 cos 27.2° − 164.93 = 15.33 m/s
(a) Tangential thrust = m (vw1 + vw2) = 10 (373.48 + 15.33) = 3888 N
(b) Axial thrust = m (vf 1 − vf 2) = 10 (121.35 − 92.64) = 287.1 N
(c) Resultant thrust = [(3888)2 + (287.1)2]1/2 = 3898.6 N
 ( vw1 + vw 2 ) 10 × 164.93(373.48 + 15.33)
mu
(d) Power developed = = = 641.26 kW
103 103
2u( vw1 + vw 2 ) 2 × 164.93 (373.48 + 15.33)
(e) Blade efficiency, hb = = = 0.8316 or 83.16%
va21 (392.7)2

Example 7.4
One stage of impulse turbine consists of a converging nozzle ring and moving blades. The nozzles
are inclined at 22° to the blades whose tip angles are both 35°. If the velocity of steam at exit from
nozzle is 660 m/s, find the blade speed so that the steam passes on without shock. Find the diagram
efficiency neglecting losses if the blades are run at this speed. [IES, 1992]

Solution
Given that a 2 = 22°, b1 = b2 = 35°, va1 = 660 m/s
The velocity diagrams are shown in Fig. 7.26.
vf 1 = va1 sin 22° = vr1 sin 35°

660 sin 22°


vr1 = = 431.08 m/s
sin 35°

M07_THERMAL ENGINEERING_SE_XXXX_CH07-Part-1.indd 345 6/15/2018 3:38:15 PM


346 Chapter 7

vw2 vw1
u
22° 35° 35°

vf2 vf1
vr1
va2
va1
vr2
35°

Scale: 1 cm = 50 m/s

Figure 7.26  Velocity diagrams for a single stage impulse turbine

vw1 = va1 cos a1 = 660 cos 22° = 611.94 m/s


vr21 = u 2 + va21 − 2u va1 cosa1

or       (431.05)2 = u2 + (660)2 − 2u × 660 cos 22°


or      185804 = u2 + 435600 − 1223.88 u
or       u2 − 1223.88 u + 249796 = 0

1223.88 ± (1223.88)2 − 4 × 1 × 249796


or        u =
2
1223.88 ± 706.18
= = 258.85 m/s (taking –ve sign)
2
For K = 1, vr2 = vr1 = 431.05 m/s
vw2 = vr2 cos b2 − u = 431.05 cos 35° − 258.85 = 94.24 m/s
2u ( vw1 + vw 2 ) 2 × 258.85 (611.94 + 94.24)
Diagram efficiency, hd = = = 0.8392 or 83.92%
va21 660 2

Example 7.5
A single-stage steam turbine is supplied with steam at 5 bar, 200°C at the rate of 50 kg/min. It
expands into a condenser at a pressure of 0.2 bar. The blade speed is 400 m/s. The nozzles are
inclined at an angle of 20° to the plane of the wheel and the outlet blade angle is 30°. Neglecting
friction losses, determine the power developed, blade efficiency, and stage efficiency. [IES, 1994]
Solution
Given that p1 = 5 bar, t1 = 200oC, p2 = 0.2 bar, m· = 50 kg/min, u = 400 m/s, a1 = 20°, b2 = 30°,
vr1 = vr2 as K = 1

M07_THERMAL ENGINEERING_SE_XXXX_CH07-Part-1.indd 346 6/15/2018 3:38:18 PM


Steam Turbines 347

From steam tables,


At 5 bar, 200°C, h1 = 2855.4 kJ/kg, s1 = 7.0592 kJ/kg.K
At 0.2 bar, sf 2 = 0.8319 kJ/kg. K, sfg2 = 7.0766 kJ/kg.K
s2 = s1 = sf 2 + x2sfg2
or              7.0592 = 0.8319 + 7.0766 x2
or               x2 = 0.88
h2 = hf 2 + x2 hfg2 = 251.38 + 0.88 × 2358.3 = 2326.68 kJ/kg
Enthalpy drop, ∆h = h1 − h2 = 2855.4 − 2326.68 = 528.7 kJ/kg

va1 = 2 × ∆h = 2 × 528.7 × 103 = 1028.3 m/s


vw1 = va1 cos a1 = 1028.3 cos 20° = 965.3 m/s
vf 1 = va1 sin a1 = 1028.3 sin 20° = 351.7 m/s
vf1 351.7 315.7
tan b1 = = = = 0.621
vw1 − u 966.3 − 400 566.3
b1 = 31.84°
or              
vf1 351.7
vr1 = = = 665.6 m/s
sin b1 sin 31.84°
vr2 = vr1 = 655.6 m/s
vw2 = vr2 cos b2 − u = 665.6 cos 30° − 400 = 177 m/s
vw = vw1 + vw2 = 965.3 + 177.3 = 1143.6 m/s
m u vw 50 1143.6
Power developed, P = = × 400 × = 381.2 kW
3
10 60 103
2u vw 2 × 400 × 1143.6
Blade efficiency, hb = = = 0.8652 or 86.52%
va21 (1028.3)2

Example 7.6
A simple impulse turbine has a mean blade speed of 200 m/s. The nozzles are inclined at 20° to the
plane of rotation of the blades. The steam velocity from nozzle is 600 m/s. The turbine uses 3600 kg/h
of steam. The absolute velocity at exit is along the axis of the turbine. Determine: (a) the inlet and
exit angles of blades, (b) the power output of the turbine, (c) the diagram efficiency, and (d) the axial
thrust (per kg steam per second). Assume inlet and outlet angles to be equal. [IES, 1998]

Solution
Given that u = 200 m/s, a1 = 20°, va1 = 600 m/s, m· = 3600 kg/h, va 2 = vf 2
The velocity diagrams are shown in Fig 7.27.
(a) vf 1 = va1 sin a1 = 600 sin 20° = 205.2 m/s
vw1 = va1 cos a1 = 600 cos 20° = 563.8 m/s
vf1 205.2
tan b1 = = = 0.564
vw1 − u 563.8 − 200

M07_THERMAL ENGINEERING_SE_XXXX_CH07-Part-1.indd 347 6/15/2018 3:38:21 PM


348 Chapter 7

vw1
u
a1 = 20°b2 b1
vf2 = va2

vr2
vf1
vr1

va1
Scale: 1 cm = 50 m/s

Figure 7.27  Velocity diagrams for a simple impulse turbine

or                b1 = 29.42°
b2 = b1 = 29.42°
vw2 = 0
 ( vw1 + vw 2 )
mu 3500 563.8
(b) Power output of turbine = = × 200 × = 109.62 kW
3
10 3600 103
2u( v w1 + vw 2 ) 2 × 200 × 563.8
(c) Diagram efficiency, hd = = = 0.6264 or 62.64%
va21 600 2
(d) Axial thrust, Fa = m ( v f 1 − v f 2 )
vf 2 = u tan b2 = 200 tan 29.42° = 112.79 m/s
Fa = 1 × (205.2 − 112.79) = 92.41 N

Example 7.7
Steam at 2.94 bar absolute and dry saturated comes out of rotor of a reaction turbine having iden-
tical bladings. The velocity of steam entering into the turbine is 100 m/s. The mean blade height
is 4 cm and the exit angle of the moving blade is 20°. The blade velocity is 4/3 times of axial flow
velocity at the mean radius. If the steam flow rate is 2.5 kg/s, find (a) the rotor speed, (b) the power
developed, (c) the diagram efficiency, (d) the percentage increase in relative velocity in the moving
blades and (e) the enthalpy drop of steam in blade passage. [IES, 1999]

Solution
The velocity triangles are shown in Fig. 7.28.
Given that p = 2.94 bar, h = 4 cm, va1 = 100 m/s, m· = 2.5 kg/s, a1 = b2 = 20°, b1 = a2, u = (4/3)vf
From velocity triangles, we have (Fig. 7.28)
v f = v f 1 = v f 2 = va1 sin a1 = 100 sin 20° = 34.2 m/s

4 4 × 34.2
u=
.v = = 45.6 m/s
3 f 3
Specific volume of steam at 2.94 bar (from steam tables), vs = 0.6173 m3/kg

M07_THERMAL ENGINEERING_SE_XXXX_CH07-Part-1.indd 348 6/15/2018 3:38:25 PM


Steam Turbines 349

vw2 vw1
u
a2 20° b2 b1

vf2
va2 vf1
vr1
vr2 Scale: 1 cm = 25 m/s
va1

Figure 7.28  Velocity diagrams for a reaction turbine having symmetrical blades

vs = p dhv f

2.5 × 0.1673
d= = 0.359 m or 35.9 cm
p × 4 × 10 −2 × 34.2
p dN
(a) u =
60
45.6 × 60
or N= = 2426 rpm
(p × 0.359)
(b) Power developed, P = mu
 ( vw1 − vw 2 )
vw1 = va1 cos a1 = 100 cos 20° = 93.97 m/s
vf1 34.2
tan b1 = = = 0.707
vw1 − u 93.97 − 45.6
or b1 = a 2 = 35.26°
vf 2 34.2
vw 2 = = = 48.37 m/s
tana 2 0.707
P = 2.5 × 45.6(93.97 + 48.37)= 16226.76 W =16.226 kW
vw 2 48.37
va 2 = = = 59.23 m/s
cos a 2 cos 35.26°

(c) Diagram efficiency = u( vw1 + vw 2 ) = 2u( vw1 + vw 2 )


2 2
vr 2 − vr1 va21 − u 2
+ 2va1u cos a1
va21 +
2
2 × 45.6 × ( 93.97 + 48.37)
= = 0.7872 or 78.72%
(100)2 − ( 45.6)2 + 2 × 100 × 45.6 × cos 20°
vf1 34.2
(d) vr1 = = = 48.37 m/s
tan b1 tan 35.26°
vf 2 34.2
vr 2 = = = 93.97 m/s
tan b 2 tan 20°
(93.97 − 48.37)
Increase in relative velocity = = 0.9425 or 94.25%
48.37

M07_THERMAL ENGINEERING_SE_XXXX_CH07-Part-1.indd 349 6/15/2018 3:38:32 PM


350 Chapter 7

(e) Enthalpy drop:


m ( va21 − va22 ) 2.5(100 2 − 59.232 )
Fixed blades, ∆h1 = = = 8.115 kJ
2 2
m ( vr22 − vr21 ) 2.5(93.972 − 48.372 )
Moving blades, ∆h2 = = = 15.224 kJ
2 2

Example 7.8
In a multi-stage Parson’s reaction turbine, at one of the stages, the rotor diameter is 125 cm and
the speed ratio 0.72. The speed of the rotor is 3000 rpm. Determine: (a) the blade inlet angle if the
blade outlet angle is 22°, (b) diagram efficiency, and (c) percentage increase in diagram efficiency
and rotor speed if the turbine is designed to run at the best theoretical speed. [IES, 2000]

Solution
u
Given that d = 1.25 m, N = 3000 rpm, = 0.72, a1 = b2 = 22°
va1
p dN p × 1.25 × 3000
(a) u = = = 196.35 m/s
60 60
u 196.35
va1 = = = 272.7 m/s
0.72 0.72
In Parson’s reaction turbine, (see Fig. 7.29)
vr 2 = va1 , vr1 = va 2 and a 2 = b1

v f 1 = va1 sina1 = 272.7 sin 22° = 102.15 m/s

vw1 = va1 cosa1 = 272.7 cos 22° = 252.84 m/s


vf1 102.15
tan b1 = = = 1.8082
vw1 − u 252.84 − 196.35
or b1 = 61°

vr2 = 272.7 m/s

vw2 vw1
u
b1 22° 22° b1

vf2 vf1
va2 vr1
vr2 va1
Scale: 1 cm = 30 m/s

Figure 7.29  Velocity diagrams for Parson’s reaction turbine

M07_THERMAL ENGINEERING_SE_XXXX_CH07-Part-1.indd 350 6/15/2018 3:38:39 PM


Steam Turbines 351

(b) v w 2 = vr 2 cos b 2 − u = 272.7 cos 22° − 196.35 = 56.5 m/s


u( vw1 + vw 2 ) 2u( vw1 + vw 2 )
Diagram efficiency, = =
vr22 − vr21 va21 − u 2 + 2va1u cos a1
va21 +
2
2 × 196.35 × ( 252.84 + 56.5)
= = 0.899 or 89.9%
( 272.7) − (196.35)2 + 2 × 272.7 × 196.35 × cos 22°
2

 ( vw1 + vw 2 )
mu 1 × 196.35 × 309.25
(c) Power developed, P = = = 60.69 kW
3
10 103
At best theoretical rotor speed, u = va1 cos a1 = 272.6 cos 22° = 252.84 m/s

 252.84 − 196.35 
Percentage increase in rotor speed =   = 0.2877 or 28.77%
 196.35

2 cos 2 a1 2 cos 2 22° 2 × 252.75 × 309.25


Diagram efficiency = = = 0.9245 or 92.45% = 2.1
2 2
1 + cos a1 1 + cos 22° ( 272.6)2

 0.9245 − 0.899 
Percentage increase in diagram efficiency =   = 2.84%
 0.899

Example 7.9
A De-Laval turbine has a mean blade velocity of 180 m/s. The nozzles are inclined at 17° to the
tangent. The steam flow velocity through the nozzles is 550 m/s. For a mass flow of 3300 kg/hr
and for axial exit conditions, find (a) the inlet and outlet angles of the blade system, (b) the power
output, and (c) the diagram efficiency. [IES, 2001]

Solution
Given that u = 180 m/s, va1 = 550 m/s, m = 3300 kg/h, a1 = 17°, a 2 = 90°
The velocity diagrams are shown in Fig. 7.30.
(a) v f 1 = va 2 (see Fig. 7.30)

v f 1 = va1 sina1 = 550 sin 17° = 160.8 m/s

tan b 2 = v f 2 /u = 160.8/180 = 0.89336

or b 2 = 41.78°
tan b1 = v f 1 / ( va1 cos a1 − u )

= 160.8/(550 cos 17° − 180) = 0.46478


or b1 = 24.93°

M07_THERMAL ENGINEERING_SE_XXXX_CH07-Part-1.indd 351 6/15/2018 3:38:46 PM


352 Chapter 7

vw1
u
17° b2 b1
vf2 = va2

vf1
vr1
vr2
va1
Scale: 1 cm = 50 m/s

Figure 7.30  Velocity of diagrams for De-Laval turbine

 w1 = (3300/3600) × (180 × 550 cos 17°/103) = 85.785 kW


(b) Power output, P = muv

(c) Diagram efficiency = 2uvw1/ va21 = 2 × 180 × 550 cos 17°/5502 = 0.626 or 62.6%

Example 7.10
A steam turbine is governed by throttling. The full load output of a steam turbine, measured at the
coupling is 5 MW and the losses due to bearing friction, the governor and oil pump drive, etc., are
200 kW. The steam is supplied at a pressure of 20 bar and 300°C. The exhaust pressure of steam is
0.07 bar. The internal efficiency ratio at full load is 0.75. Calculate the coupling power of turbine
when the steam flow through the turbine is 20% of that at full load. Assume that the external losses
are the same as that at full load, exhaust pressure is the same, and the internal efficiency ratio is
reduced to 70% at this load. [IES, 2005]

Solution
The h–s (Mollier) diagram is shown in Fig 7.31.
At 20 bar, 300°C, h1 = 3020 kJ/kg
At 0.07 bar, h2 = 2100 kJ/kg
At full load:
h −h ∆hi
Internal efficiency, hi = 1 2′ =
h1 − h2 ∆hisen
3020 − h2′
or 0.75 =
3020 − 2100
or h2′ = 2330 kJ/kg
∆hi = 3020 – 2330 = 690 kJ/kg
Internal power developed, Pi = m ∆hi

or 5000 = m × 690

or m = 7.246 kg/s
Nozzle box pressure at part load p1′ = steam mass flow ratio × p1

M07_THERMAL ENGINEERING_SE_XXXX_CH07-Part-1.indd 352 6/15/2018 3:38:50 PM


Steam Turbines 353

h, kJ/kg 20 bar

300°C 4 bar
1
3020 1′

0.07 bar
2350 3′
3

2330
2100 2′
2

Figure 7.31  Mollier diagram for a steam turbine

p1′ = 0.2 × 20 = 4 bar

∆hisen
′ = h1′ − h3′ = 3020 − 2350 = 670 kJ/kg

P1 P1′ = hi′ × m × ∆hisen



= 0.7 × 7.246 × 670 × 0.2 = 679.71 kW
Coupling power = 679.71 – 200 = 479.71 kW

Example 7.11
The turbine rotor has a mean blade ring diameter of 500 mm and the blade angles are equal.
The nozzle angle is 20° and the steam leaves the nozzles with a velocity of 900 m/s. Assuming a
blade friction factor of 0.85, determine the (a) the best blade angles, (b) the turbine speed in rpm,
(c) the steam consumption rate in kg/h if the turbine generates 10 kW, and (d) the maximum blade
efficiency.[IES, 2006]
Solution
Given that a1 = 20°, dm = 0.5 m, va1 = 900 m/s, K = 0.85, b = b1 = b2 so that C = 1, P = 10 kW
The velocity diagrams are shown in Fig. 7.32.
For maximum blade efficiency, the speed ratio,
u cos a1
r= =
va1 2

900 cos 20°


or u= = 422.86 m/s
2
p dm N
u=
60
422.86 × 60
or N= = 16152 rpm
p × 0.5

M07_THERMAL ENGINEERING_SE_XXXX_CH07-Part-1.indd 353 6/15/2018 3:38:55 PM


354 Chapter 7

vw1
e u
vw2 b
a f
a1 = 20° b2 b1

va2 vf2
vr1 vf1
vr2
d va1 c′
c

Scale: 1 cm = 100 m/s

Figure 7.32  Velocity diagrams for an impulse turbine

vw = vw1 +vw2
= vr1 cos b1 + vr2 cos b2
= vr1 cos b (1 + KC) = 18.5 vr1 cos b
u = va1 cos a1 − vr1 cos b
   vr1 cos b = 900 cos 20° − 422.86 = 422.86 m/s
  vw = 1.85 × 422.86 = 782.25 m/s
 vw
mu
Power developed by turbine, P =
103

10 × 103
Mass flow rate, m = = 0.03023 kg/s or 108.83 kg/h
422.86 × 782.25

cos 2 a1
Maximum blade efficiency, (hb ) max = (1 + KC)
2
1.85 cos 2 20°
= = 0.8168 or 81.68%
2
vr21 = va21 + u 2 − 2va1u cosa1
= 9002 + (422.86)2 − 2 × 900 × 422.86 cos 20° = 273564
vr1 = 523.03 m/s
or                

422.86 422.86
cos b = = = 0.80844
vr1 523.03
or b = 36°

M07_THERMAL ENGINEERING_SE_XXXX_CH07-Part-1.indd 354 6/15/2018 3:39:05 PM


Steam Turbines 355

Worst affected areas

Blade
Blade speed
edge

Figure 7.33  Steam turbine blades erosion

7.16 ❐ EROSION OF STEAM TURBINE BLADES


The steam turbine blades are subjected to high pressure and temperature. In the intermediate pressure
stages, the steam is wet. Therefore, the material of blades should be able to withstand corrosion and
erosion due to the presence of water particles. In addition to corrosion and erosion, blades are sub-
jected to high centrifugal stresses. When the speed is high and the dryness fraction is less than 0.9, the
effect of moisture is the most prominent. The most affected position is the back of the inlet edge of the
blade, where either grooves are formed or even some portion breaks away. Due to centrifugal force,
the water particles tend to concentrate in the outer annulus and their tip speed is greater than the root
speed. Hence, erosion effect is the most prominent on the tips, as shown in Fig. 7.33.
The following methods may be adopted to prevent erosion:
1. By raising the temperature of steam at inlet so that at the exit of the turbine, the dryness fraction
does not fall below 0.9.
2. By adopting reheat cycle so that the dryness fraction at exit remains within limits.
3. By providing drainage belts on the turbine so that the water droplets which are on outer periph-
ery, due to centrifugal force, are drained.
4. By providing a shield of a hard material on the leading edge of the turbine.
The most satisfactory solution to prolong the blade life is providing tungsten shield.

Example 7.12
In a De-Laval turbine, steam enters the wheel through a nozzle with a velocity of 500 m/s and at an
angle of 20° to the direction of motion of the blade. The blade speed is 200 m/s and the exit angle
of the moving blade is 25°. Find the inlet angle of the moving blade, exit velocity of steam and its
direction and work done per kg of steam.
Solution
Given that va1 = 500 m/s; a1 = 20°; u = 200 m/s; b2 = 25°, vr2 = vr1.
Now let us draw the combined velocity triangles, as shown in Fig. 7.34, as explained below:
1. First, draw a horizontal line and cut off AB equal to 200 m/s, to some suitable scale, representing
the blade speed, u.
2. Now at B, draw a line BC at an angle of 20° (nozzle angle, a1) and cut off BC equal to 500 m/s to
the same scale to represent the velocity of steam jet entering the blade (va1).
3. Join AC, which represents the relative velocity at inlet (vr1).
4. At A, draw a line AD at an angle of 25° (exit angle of the moving blade, b2). Now with A as cen-
tre, and radius equal to AC, draw an arc meeting the line through A at D.

M07_THERMAL ENGINEERING_SE_XXXX_CH07-Part-2.indd 355 6/15/2018 3:45:18 PM


356 Chapter 7

va D
1
vr2
vf1 vr1 va2 vf2
a1
b1 25° 20° a2
E A B F
u

Figure 7.34  Velocity diagrams for De-Laval steam turbine


5. Join BD, which represent the velocity of steam jet at outlet (va2).
6. From C and D, draw perpendiculars meeting the line AB produced at E and F respectively. CE and
DF represent the velocity of flow at inlet (vf 1) and outlet (vf 2) respectively.
The following values are measured from the velocity diagram:
b1 = 32°, a2 = 59°; va2 = BD = 165 m/s
vw1 = BE = 470 m/s and vw2 = BF = 90 m/s
By measurement from the velocity diagram, the inlet angle of the moving blade, b1 = 32°
By measurement from the velocity diagram, the exit velocity of steam, va2 = 165 m/s
By measurement from the velocity diagram, the direction of the exit steam, a2 = 59°
Work done per kg of steam = m· (vw1 + vw2)
= 1 (470 + 90) = 560 N-m/kg(m·  = 1 kg)

Example 7.13
The velocity of steam leaving the nozzles of an impulse turbine is 1200 m/s and the nozzle angle
is 20°. The blade velocity is 375 m/s and the blade velocity coefficient is 0.75. Assuming no loss
due to shock at inlet, calculate for mass flow of 0.5 kg/s and symmetrical blading: (a) blade inlet
angle; (b) driving force on the wheel; (c) axial thrust on the wheel; and (d) power developed by
the turbine.

Solution
Given that va1 = 1200 m/s; a1 = 20°; u = 375 m/s; K = vr2/vr1 = 0.75; m·  = 0.5 kg/s; b1 = b2, for sym-
metrical blading.
Now draw the combined velocity triangle, as shown in Fig. 7.35, as explained below:
1. First, draw a horizontal line, and cut off AB equal to 375 m/s to some suitable scale representing
the velocity of blade(u).
2. Now at B, draw a line BC at an angle of 20° (Nozzle angle, a1) and cut off BC equal to 1200 m/s
to the scale to represent the velocity of steam jet entering the blade (va1).

M07_THERMAL ENGINEERING_SE_XXXX_CH07-Part-2.indd 356 6/15/2018 3:45:19 PM


Steam Turbines 357

C
D
X va1 vr2
b2
vf1 vr1 va2 vf2
a1
a2
b1 20°
E A B F
u
vw1 vw2

Figure 7.35  Velocity diagrams for impulse turbine

3. Join CA, which represents the relative velocity at inlet (vr1). By measurement, we find that
CA = vr1 = 860 m/s. Now cut off AX equal to vr2 = vr1 × K = 860 × 0.75 = 645 m/s to the scale to
represent the relative velocity at exit (vr2).
4. At A, draw a line AD at an angle b2 equal to the angle b1, for symmetrical blading. Now with
A as centre, and radius equal to AX, draw an arc meeting the line through A at D, such that
AD = vr2.
5. Join BD, which represents the velocity of steam jet at outlet (va2).
6. From C and D, draw perpendiculars meeting the line AB produced at E and F respectively. CE and
DF represents the velocity of flow at inlet (vf 1) and outlet (vf 2), respectively.
The following values are measured from the velocity diagram:

b1 = 29°, vw1 = BE = 1130 m/s; vw2 = BF = 190 m/s

vf 1 = CE = 410 m/s and vf 2 = DF = 310 m/s.


(a) By measurement from the velocity diagram, the blade angle at inlet, b1 = 29°
(b) Driving force on the wheel, Fx = m (vw1 + vw2) = 0.5 (1130 + 190) = 660 N
(c) Axial thrust on the wheel, Fy = m (vf 1 − vf 2) = 0.5 (410 − 310) = 50 N
(d) Power developed by the turbine,

P = m·  (vw1 + vw2) u = 0.5 (1130 + 190) 375 = 247500 W or 247.5 kW

Example 7.14
A single row impulse turbine receives 3 kg/s steam with a velocity of 425 m/s. The ratio of blade
speed to jet speed is 0.4 and the stage output is 170 kW. If the internal losses due to disc friction
etc., amount to 15 kW, determine the blade efficiency and the blade velocity coefficient. The nozzle
angle is 16° and the blade exit angle is 17°.
Solution
Given that m = 3 kg/s; va1 = 425 m/s; u/va1 = 0.4; stage output = 170 kW; internal losses = 15 kW; a1
= 16°; b2 = 17°.

M07_THERMAL ENGINEERING_SE_XXXX_CH07-Part-2.indd 357 6/15/2018 3:45:20 PM


358 Chapter 7

C
va1 =
425 m
/s
vf1
vr1 D
vr2 va2
b1 16°
17°
E A F B
u = 170 m/s
363 m/s vw2

Figure 7.36  Velocity diagrams for single row impulse turbine

(a) Blade speed, u = va1 × 0.4 = 425 × 0.4 = 170 m/s


Total power developed, P = Stage output + Internal losses
= 170 + 15 = 185 kW
Let vw1 + vw2 = Change in the velocity of whirl
Power developed, P = m (vw1 + vw2) u
185 × 103 = 3 (vw1 + vw2) 170
\ vw1 + vw2 = 363 m/s
Now, draw the combined velocity triangle, as shown in Fig. 7.36, and explained below:
  (i) First, draw a horizontal line and cut off AB equal to 170 m/s, to some suitable scale, to rep-
resent the blade speed (u).
  (ii) Now, draw the inlet velocity triangle ABC on the base AB with a1 = 16° and va1 = 425 m/s,
to the scale, chosen.
(iii) Similarly, draw the outlet velocity triangle ABD on the same base AB with b2 = 17° and
(vw1 + vw2) = 363 m/s to the scale.
 (iv) From C and D, draw perpendiculars to meet the line AB at E and F. From the geometry of the
figure, we find that vw2 is in the opposite direction of vw1. Therefore, (vw1 − vw2) = 363 m/s.
By measurement from the velocity diagram, we find that
Relative velocity at inlet, vr1 = 265 m/s
and relative velocity at outlet, vr2 = 130 m/s
Blading efficiency,
2( vw1 − vw 2 ) 2 × 363 × 170
hb = = = 0.683 or 68.3%
va21 ( 425)2
(b) Blade velocity coefficient,
vr 2 130
K= = = 0.49
vr1 263

Example 7.15
In one stage of a reaction steam turbine, both the fixed and moving blades have inlet and outlet
blade tip angles of 35° and 20°, respectively. The mean blade speed is 80 m/s and the steam con-
sumption is 22500 kg per hour. Determine the power developed in the pair, if the isentropic heat
drop for the pair is 23.5 kJ per kg.

M07_THERMAL ENGINEERING_SE_XXXX_CH07-Part-2.indd 358 6/15/2018 3:45:22 PM


Steam Turbines 359

C D
vr2
vf1 va1 va2
vr1 vf2

35° 20° 20°


E 35° F
A 80 B
vw1 vw2

Figure 7.37  Velocity diagrams for reaction turbine

Solution
Given that b1 = a 2 = 35°; b 2 = a1 = 20°; u = 80 m/s; m = 22500 kg/h = 6.25 kg/s; Δh = 23.5 kJ/kg.
Now, let us draw the combined velocity triangle, as shown in Fig. 7.37 and explained below:
1. First, draw a horizontal line and cut off AB equal to 80 m/s (u) to some suitable scale.
2. Now at B, draw in line BC at an angle a1 = 20° with AB. Similarly, at A draw a line AC at an angle
b1 = 35° with BA meeting the first line at C.
3. At A, draw a line AD at angle b2 = 20° (because b 2 = a1) with AB. Similarly, at B draw a line BD
at an angle a2 = 35° (because a 2 = b1) with AB meeting the first line at D.
4. From C and D, draw perpendiculars meeting the line AB produced at E and F.
By measurement, we find that the change in the velocity of whirl,
Δw = vw1 + vw2 = 235 m/s
Power developed in the pair,
P = m (vw1 + vw2) u = 6.25 × 235 × 80 = 117500 W = 117.5 kW

Example 7.16
A 50% reaction turbine with a mean blade diameter of 1 m runs at a speed of 50 rps. The blades
are designed with exit angles of 50° and inlet angles of 30°. If the turbine is supplied with steam
at the rate of 20 kg/s and gross efficiency is 85%, determine the following: (a) power output of the
stage; (b) specific enthalpy drop in the stage; and (c) percentage increase in relative velocity in the
moving blades due to steam expansion.

Solution
Given that dm = 1 m; N = 50 rps; b1 = a2 = 50°; b2 = a1 = 30°; m = 20 kg/s; hs = 85%
(a) Blade velocity, u = p dm N = p × 1 × 50 = 157 m/s
Now let us draw the combined velocity triangle, as shown in Fig. 7.38, and explained below:
    (i) First, draw a horizontal line and cut off AB equal to 157 m/s, to some suitable scale, to rep-
resent the blade velocity u.
  (ii)  Now, draw the inlet velocity triangle ABC on the base AB with a1 = 30° and b1 = 50°.
(iii)  Similarly, draw the outlet velocity triangle ABD on the same base AB with b2 = 30° and a2= 50°.
(iv)  From C and D, draw perpendiculars to meet the line AB produced at E and F.
By measurement from the velocity triangle, we find that change in the velocity of whirl.
vw1 + vw2 = EF = 450 m/s
Relative velocity at inlet, vr1 = CA = 230 m/s

M07_THERMAL ENGINEERING_SE_XXXX_CH07-Part-2.indd 359 6/15/2018 3:45:25 PM


360 Chapter 7

C D
vr2
va1
vf1 vr1 va2 vf2

50° 30° 30° 50°


E A 157 B F
vw1 + vw2

Figure 7.38  Velocity diagrams for symmetrical blades reaction turbine

and relative velocity at outlet, vr2 = DA = 350 m/s


Power output of the stage,
P = m (vw1 + vw2) u = 20 × 450 × 157 = 1413000 W = 1413 kW.
(b) Let Δh = specific enthalpy drop in the stage.
We know that stage efficiency (hs);
( v + v )u 450 × 157 70.65
0.85 = w1 w 2 = =
1000 × ∆h 1000 × ∆h ∆h

Δh = 70.65/0.85 = 83.1 kJ
(c) Increase in the relative velocity in the moving blades due to steam expansion.

vr 2 − vr1 351.6 − 229.5


= = 0.532 or 53.2%
vr1 229.5

Example 7.17
At a stage of a 50% reaction turbine, the rotor diameter is 1.4 m and speed ratio 0.7. If the blade
outlet angle is 20° and the rotor speed 3000 rpm, find the blade inlet angle and diagram efficiency.
Find the percentage increase in diagram efficiency and rotor speed, if the turbine is designed to
run at the best theoretical speed.
Solution
Given that D = 1.4 m; r = u/va1 = 0.7; a1 = b2 = 20°; N = 3000 rpm.
Blade velocity,

p DN p × 1.4 × 3000
u= = = 220 m/s
60 60
and velocity of steam at inlet to the blade.
va1 = u/0.7 = 220/0.7 = 314.3 m/s
Now draw the combined velocity triangle, as shown in Fig. 7.39, as explained below:
1. First, draw a horizontal line and cut off AB equal to 220 m/s, to some suitable scale, to represent
the blade velocity (u).

M07_THERMAL ENGINEERING_SE_XXXX_CH07-Part-2.indd 360 6/15/2018 3:45:28 PM


Steam Turbines 361

C D
va
1 = 31
4.3 vr2
m/s
vf1 va2 vf2
vr1
b1 20° = b2 a1 = 20° a2
E F
A u = 250 m/s B
vw1 + vw2

Figure 7.39  Velocity diagrams for reaction turbine with Rd = 50%

2. Now, draw the inlet velocity triangle ABD on the same base AB with a1 = 20° and va1 = 314.3 m/s,
to the scale.
3. Similarly, draw the outlet velocity triangle ABD on the same base AB with b2 = 20° and vr2 =
314.3 m/s to the scale.
4. From C and D, draw perpendiculars to meet the line AB produced at E and F.
By measurement from velocity diagram, we find that the blade inlet angle.
b1 = 55°
By measurement from velocity diagram, we find that velocity of steam at outlet,
va2 = 130 m/s
Diagram efficiency,
va21 − va22 (314.3)3 − (130)2
hb = = = 0.829 or 82.9%
va21 (314.3)2
Maximum efficiency of the turbine,
2 cos 2 a1 2 × cos 2 20° 2(0.9397)2
h max = = = = 0.938
1 + cos 2 a1 1 + cos 2 20° 1 + (0.9397)2
0.938 − 0.829
∴ Percentage increase in diagram efficiency = = 0.131 or 13.1%
0.829
Let N1 = Maximum rotor speed.
For best theoretical speed (or in other words, for maximum efficiency), the blade velocity,
u = va1 cos a1 = 314.3 cos 20° = 314.3 × 0.9397 = 295.3 m/s
Blade velocity (u);
p DN1 p × 1.4 × N1
u= =
60 60
p × 1.4 × N1
or 295.3 =
60
or N1 = 4044 rpm
4044 − 3000
∴ Percentage increase in rotor speed = = 0.348 or 34.8%
3000

M07_THERMAL ENGINEERING_SE_XXXX_CH07-Part-2.indd 361 6/15/2018 3:45:33 PM


362 Chapter 7

Example 7.18
One stage of an impulse turbine consists of a converging nozzle ring and one ring of moving
blades. The nozzles are inclined at 22° to the blades whose tip angles are both 35°. If the velocity of
steam at exit from the nozzle is 660 m/s, find the blade speed so that the steam shall pass on without
shock. Find the diagram efficiency neglecting losses if the blades are run at this speed. [IES, 1992]

Solution
Given that a1 = 22°, b1 = b2 = 35°, va1 = 660 m/s
The velocity triangles for impulse turbine are shown in Fig. 7.40.
Maximum blade efficiency,

cos 2 a1
(hb)max = (1 + KC)
2
For b1 = b2, C = 1 and for no friction on the blades, K = 1.
∴              (hb)max = cos2 a1 = cos2 22° = 0.8597 or 85.97%

u cosa1
Speed ratio, r = =
va1 2
cos 22
Blade speed, u = 660 × = 305.97 m/s.
2
vw2 vw1

u
a2 a1 b2 b1

vf2 va2 vr1 vf1

vr2 va1

Figure 7.40  Velocity triangles for one-stage impulse steam turbine

Example 7.19
Steam expands in a turbine from 40 bar, 450°C to 0.1 bar isentropically. Assuming ideal conditions,
determine the mean diameter of the wheel if the turbine were of (a) single impulse stage, (b) single
50% reaction stage, (c) four pressure (or Rateau) stages, (d) one two-row Curtis stage, and (e) four
50% reaction stages. Take the nozzle angle as 15° and speed 3000 rpm.

Solution
Given that p1 = 40 bar, t1 = 450°C, p2 = 0.9 bar, a1 = 15°, N = 3000 rpm
At 40 bar, 450°C, from steam tables, we have
h1 = 3030.2 kJ/kg, s1 = 6.9362 kJ/kg.K

M07_THERMAL ENGINEERING_SE_XXXX_CH07-Part-2.indd 362 6/15/2018 3:45:35 PM


Steam Turbines 363

At 0.1 bar: sf 2 = 0.6492 kJ/kg.K, sfg2 = 7.5010 kJ/kg.K,


hf 2 = 191.81 kJ/kg, hfg2 = 2392.8 kJ/kg
Now for isentropic flow,     s1 = s2 = sf 2 + x2 sfg2
or  6.9362 = 0.6492 + x2 × 7.5010
or   x2 = 0.838
 h2 = hf 2 + x2 hfg2 = 191.81 + 0.838 × 2392.8 = 2196.98 kJ/kg

∆h = h1 − h2 = 3030.2 − 2196.98 = 833.22 kJ/kg

(a) va1 = 2 × ∆h = 2 × 833.22 × 103 = 1290.91 m/s


u cosa1 cos15°
= = = 0.48296
va1 2 2
u = 1290.91 × 0.48296 = 623.5 m/s
pDm N p Dm × 3000
 = =
60 60
Dm = 3.97 m

2 × ∆h
(b) va1 = = 833.22 × 103 = 912.81 m/s
2
u
= cos a1
va1
or u = 912.81 × cos 15° = 881.71 m/s
pDm × 3000
= 881.71
60
or Dm = 5.61 m

833.22 × 103
2 × ∆h
(c) va1 = == 645.45 m/s
2 4
cosa1 cos15°
u = va1 × = 645.45 × = 311.728 m/s
2 2
60 × 311.728
Dm = = 1.98 m
p × 3000
∆h 833.22
(d) (∆h)st = = = 208.305 kJ
4 4
2 × ( ∆h) st
va1 = × 103 = 456.4 m/s
2
u = va1 cos a1 = 456.4 × cos 15° = 440.848 m/s
440.848 × 60
Dm = = 2.8 m
p × 3000

M07_THERMAL ENGINEERING_SE_XXXX_CH07-Part-2.indd 363 6/15/2018 3:45:42 PM


364 Chapter 7

Example 7.20
Steam at 20 bar, 500°C expands in a steam turbine to 0.01 bar. There are four stages in the turbine
and the total enthalpy drop is divided equally among the stages. The stage efficiency is 75% and
it is the same in all the stages. Calculate the interstage pressures, the reheat factor, and the turbine
internal efficiency.
Solution
Given that p1 = 20 bar, t1 = 500°C, p2 = 0.09 bar, hs = 0.75
At 20 bar, 500°C, h1 = 3467.6 kJ/kg, s1 = 7.4316 kJ/kg.K
At 0.1 bar, hf6 = 191.81 kJ/kg, hfg6 = 2392.8 kJ/kg, sf6 = 0.6492 kJ/kg.K, sfg6 = 7.5010 kJ/kg.K
Now s1 = s6 = sf6 + x6 sfg6
or 7.4316 = 0.6491 + x6 × 7.5010
or x6 = 0.9042
h6 = hf 6 + x6 hfg6 = 191.81 + 0.9042 × 2392.8 = 2355.4 kJ/kg
h1 − h6 = 3467.6 − 2355.4 = 1112.2 kJ/kg
1112.2
h1 − h2 = = 278.05 kJ/kg
4
The Mollier diagram is shown in Fig. 7.41.
The inter stage pressures read from Mollier chart shown in Fig. 7.41 are:
p2 = 9 bar, p3 = 3 bar, p4 = 0.75 bar

h1 − h2′ = 0.75 × 278 = 208.5 kJ/kg

h2′ = 3467.6 − 208.5 = 3259.1 kJ/kg

h2′ − h3 = 3259.1 − 2960 = 299.1 kJ/kg

h2′ − h3′ = 0.75 × 299.1 = 224.3 kJ/kg


h3′ = 3259.1 − 224.3 = 3034.8 kJ/kg

h3′ − h4 = 3259.1 − 2740 = 519.1 kJ/kg

h3′ − h4′ = 0.75 × 519.1 = 389.3 kJ/kg

h4′ = 3259.1 − 389.3 = 2869.8 kJ/kg

h4′ − h5 = 2869.8 − 2520 = 349.8 kJ/kg

h4′ − h5′ = 0.75 × 349.8 = 262.3 kJ/kg

h5′ = 2869.8 − 262.3 = 2607.5 kJ/kg

x5′ ∼
− 1.0
( h1 − h2 ) + ( h2′ − h3 ) + ( h3′ − h4 ) + ( h4′ − h5 )
Reheat factor =
h1 − h6

M07_THERMAL ENGINEERING_SE_XXXX_CH07-Part-2.indd 364 6/15/2018 3:45:51 PM


Steam Turbines 365

h p1 = 20 bar
1 p2 = 9 bar

2′ p3 = 3 bar
2
3′ p4 = 0.75 bar
3 4′ p5 = 0.1 bar
4 5′
5

Figure 7.41  h-s diagram

278 + 299.1 + 519.1 + 349.8 1446


= = = 1.3
1112 1112
h1 − h5′ 3467.6 − 2607.5
Turbine internal efficiency, hit = = = 0.7735 or 77.35%
h1 − h6 1112

Example 7.21
Find the maximum blade efficiency and corresponding blade angles for a single row impulse steam
turbine assuming equiangular blades, when the nozzle angle a1 = 20° and blade velocity coeffi-
cient K = 0.9.
If the blade efficiency is 85% of the maximum value, what are the possible blade speed ratios
for the same nozzle angle a1, blade velocity coefficient, K and equiangular blades? Find the corre-
sponding blade angles. [IAS, 2003]

Solution
Given that a1 = 20°, K = 0.9
cos 2 a1
Maximum blade efficiency, (hb)max = (1 + KC)
2
vr 2 cos b 1
where K = and C =
vr1 cos b 2
cosa1
, r=
2
u cosa1
or =
va1 2
cosa1
   u = va1
or  
2
vw1 = va1 cos a1, vf 1 = va1 sin a1

M07_THERMAL ENGINEERING_SE_XXXX_CH07-Part-2.indd 365 6/15/2018 3:45:56 PM


366 Chapter 7

vw2 vw1
u
A B
F E
a2 a1 b2 b1

vf2
va2 vr2 ≤ vr1 vf1
vr2 vr1
va1
D

Figure 7.42  Velocity diagrams for impulse turbine

vf1 va1 sin a 1 sin a 1 tan a1


tan b1 = = = = = 2 tan a1 = 2 tan 20° = 0.72794
vw1 − u cos a 1 cos a 1 0.5
va1 cos a 1 − va1 cos a 1 −
2 2
or                 b1 = 36°
Let the blades be symmetrical so that b2 = b1 = 36° and C = 1

cos 2 20°
(hb)max = (1 + 0.9) = 0.8389 or 83.89 %
2
 he velocity diagrams at inlet and outlet of a moving blade are shown in Fig. 7.42.
T

Example 7.22
Steam expands in a steam turbine isentropically from inlet to exhaust having an enthalpy drop
= 12000 kJ/kg. Assuming ideal conditions, determine the mean diameter of the wheel if the turbine
were of:
(a) Single impulse stage
(b) Single 50% reaction stage
(c) One two-row Curtis stage
(d) Ten 50% reaction stages
Take the nozzle angle as 18° and blade speed as 4000 rpm [IAS, 2002]

Solution
Given that a1 = 18°, N = 4000 rpm, ∆h = 12000 kJ/kg,
(a) Single impulse stage turbine:
For ideal conditions,
va1 = 2 × ∆h = 2 × 12000 × 103 = 4898.98 m/s

M07_THERMAL ENGINEERING_SE_XXXX_CH07-Part-2.indd 366 6/15/2018 3:45:58 PM


Steam Turbines 367

cos a1 cos 18°


Speed ratio, r= = = 0.47553
2 2
u
=
va1
or u = 0.47553 × 4898.98 = 2329.6 m/s

p dm N
=
60
2329.6 × 60
or dm = = 11.123 m
p × 4000
(b) Single 50% reaction stage:
u
r= = cos a1
va1
or u = 4898.98 × cos 18° = 4659.2 m/s
60 × 4659.2
dm = = 22.24 m
p × 4000
(c) One two-row Curtis stage:
va1 = ∆h = 12000 × 103 = 3464 m/s
cosa1
r= = 0.47553
2
u = 0.47553 × 3464 = 1647.23 m/s

60 × 1647.23
dm = = 7.86 m
p × 4000
(d) Ten 50% reaction stage
12000 × 103
va1 = ∆h / 5 = = 1549.2 m/s
5
u = 1549.2 cos 18° = 1473.37 m/s
60 × 1473.37
dm = = 7.03 m
p × 4000

Example 7.23
The velocity of steam entering a simple impulse turbine is 1000 m/s and the nozzle angle is 20°.
The mean peripheral velocity of blades is 400 m/s. The blades are asymmetrical. If the steam is to
enter the blades without shock, what will be the blade angles?
Neglecting the friction effects on the blades, calculate the tangential force on the blades and
the diagram power for a mass flow of 0.75 kg/s. Calculate the axial thrust and diagram efficiency.
[IAS, 2001]

M07_THERMAL ENGINEERING_SE_XXXX_CH07-Part-2.indd 367 6/15/2018 3:46:02 PM


368 Chapter 7

Solution
Given that va1 = 1000 m/s, a1 = 20°, u = 400 m/s, vr1 = vr2, m· = 0.75 kg/s. For symmetrical blades,
b 2 = a1 = 20°, a 2 = b 2
v
K = r2 = 1
vr1
Steam is to enter the blades without shock.
The velocity diagrams are shown in Fig. 7.43.
vw1 = va1 cos a1 = 1000 cos 20° = 939.7 m/s
vf 1 = va1 sin a1 = 1000 sin 20° = 342.02 m/s
2 2
vr1 = u2 + vr1 − 2 u va1 cos a1
= 4002 + 10002 − 2 × 400 × 1000 × cos 20°
= 408, 245.9
or vr1 = 638.94 m/s
342.02 vf1
sin b1 = = = 0.53529
vr1 638.94
or b1 = 32.36°
∴ a 2 = 32.36°
vr2 = vr1 = 638.94 m/s
vw2 = vr2 cos b 2 − u = 638.94 cos 20° − 400 = 200.4 m/s
vf 2 = vr2 sin b 2 = 638.94 sin 20° = 218.53 m/s
Tangential force on the blades, F = m· (v + v ) = 0.75 (939.7 + 200.4) = 855.07 N
t w1 w2
 ( vw1 + vw 2 )
mu 855.07 × 400
Diagram power = = = 342.03 kW
3
10 103

vw2 vw1
A u B
F E
a2 a1 b2 b1

vf2
va2 vf1
vr2 vr1
va1
D

Figure 7.43  Velocity diagrams for simple impulse turbine

M07_THERMAL ENGINEERING_SE_XXXX_CH07-Part-2.indd 368 6/15/2018 3:46:05 PM


Steam Turbines 369

2u vw 2 × 400 (939.7 + 200.4)


Diagram efficiency = = = 0.9121 or 91.21%
va21 1000 2
Axial thrust, Fa = m· (vf 1 − vf 2) = 0.75 (342.02 − 218.53) = 92.6 N

Summary for Quick Revision

1. Steam turbines are of three types: impulse turbines, impulse reaction turbines, and reaction turbines.
2. Compounding of steam turbines is a method employed for reducing the rotational speed of the impulse
turbine to practical limits by using more than one stage.
3. Impulse steam turbines can be compounded by velocity compounding, pressure compounding, and mixed
velocity and pressure compounding.
4. Impulse turbine:
m ( vw1 + vw 2 )u
(a) Power developed, P = kW
103
2( vw1 + vw 2 )u
(b) Diagram or blade efficiency, hb =
va21
( vw1 + vw 2 )u
(c) Gross or stage efficiency, hs =
( ∆h)isen
2
va1
(d) Nozzle efficiency, hn =
2( ∆h)isen
(e) Axial thrust, Fa = m ( v f 1 − v f 2 )
(f) Energy converted into heat by blade friction
1
= m ( vr21 − vr22 )
2
u cosa 1
(g) For maximum blade efficiency: speed ratio, r = =
va1 2

cos 2 a 1
(hb)max = (1 + KC)
2
vr 2 cos b 2
where K = ,C=
vr1 cos b1
where b1 = inlet angle of moving blade, b2 = exit angle of moving blade, a1 = outlet angle of fixed blade
For symmetrical blades, b1 = b2 and C = 1
(hb)max = cos2 a1
Work done, w = (va1cos a1 − u) (1 + KC)u
wmax = 2u2 for K = 1 and C = 1
5. Velocity compounded impulse turbine:
(a) For symmetric blades and no friction loss, i.e. b1 = b2 and vr1 = vr2
Total work done, wt = 4u(va1cos a1 − 2u)

M07_THERMAL ENGINEERING_SE_XXXX_CH07-Part-2.indd 369 6/15/2018 3:46:10 PM


370 Chapter 7

(b)  Blade efficiency, hb = 8r (cos a1 − 2r)


cosa 1
For maximum blade efficiency, r =
4
and           (hb)max = cos2a1
(wt)max = 8u2
(c) If n = number of rotating blade rows in series, then
cosa 1
r=
2n
Total work done
and work done in the last row =
2n
6. Reaction turbine:
( ∆h) m
(a)  Degree of reaction, Rd =
( ∆h) m + ( ∆h) f

vr22 − vr21 vf
= = (cot b 2 − cot b1 )
2u( vw1 + vw 2 ) 2u
(b) For Rd = 0.5,
u = vf (cot b2 − cot a2) = vf (cot a1 − cot b1)
∴ b1 = a2 and b2 = a1
Also vr2 = va1
2
(c)  Work done per kg of steam, w = va1 [2r cos a1 − r 2]
2 r ( 2 cos a 1 − r )
(d)  Blade efficiency, hb =
1 + 2 r cos a 1 − r 2
(e)  For maximum blade efficiency, r = cos a1
2 cos 2 a 1
and (hb)max =
1 + cos 2 a 1
pd
(f) If n = number of blades, and d = mean diameter of turbine wheel then pitch of blades, p =
n
(g)  Volume flow rate, m vs = (pd − nt1)h1vf 1 = (pd − nt2) h2vf 2
For vf 1 = vf 2 = vf   , t1 = t2 = t and h1 = h2 = h
m vs = (p d − nt) hvf
m vs
Height of blades, h =
(p d − nt )v f

cumulative heat drop


(h)  Reheat factor, Rf  =
Isentropic heat drop
  (i)  Turbine internal efficiency, hi = hs × Rf

where hs = stage efficiency
7. Turbine efficiencies:
Work done on blade
(a)  hb =
Energy entering the blade
Net shaft work per stage Net work done on blades − Disc friction and windage
(b)  hs =
  =
Adiabatic heat drop per stage Adiabatic heat drop per stage

M07_THERMAL ENGINEERING_SE_XXXX_CH07-Part-2.indd 370 6/15/2018 3:46:17 PM


Steam Turbines 371

Heat converted into useful work


(c) hi =
Total isentropic heat drop

Work delivered at turbine coupling


(d) Overall efficiency, hoverall =
Total adiabatic heat drop

Brake thermal efficiency


(e) Efficiency ratio or net efficiency hnet, =
Rankine thermal efficiency
8. Governing of steam turbines:
(i) Throttle governing (ii) Nozzle control governing, and (iii) By-pass governing.

Multiple-choice Questions
1. In steam turbine terminology, diaphragm refers to (b) relative velocity at the inlet of the moving
(a) The separating wall between rotors carrying blade is equal to that at the outlet
nozzles (c) axial velocity at the inlet is equal to that at the
(b) The ring of guide blades between rotors outlet
(c) A partition between low and high pressure (d) whirl velocity at the inlet is equal to the outlet
dies
6. For a Parson’s reaction turbine, if a1 and a 2 are
(d) The flange connecting the turbine exit to the
fixed angles at inlet and exit, respectively, and b1
condenser
and b2 are the moving blade angles at entrance
2. A three-stage Rateau turbine is designed in such and exit, respectively, then
a manner that the first two stages develop equal (a) a1 = a 2 and b1 = b2
power with identical velocity diagram, whereas (b) a1 = b1 and a 2 = b2
the third one develops more power with higher (c) a1 < b1 and a 2 > b2
blade speed. In such a multistage turbine, the (d) a1 < b2 and b1 > a 2
blade ring diameter
(a) is the same for all the three stages 7. The isentropic enthalpy drop in moving blade is
two-thirds of the isentropic enthalpy drop in pin
(b) gradually increases from the first to the third
fixed blades of a turbine. The degree of reaction
stage
will be
(c) of the third stage is greater than that of the
(a) 0.4 (b) 0.6
first two stages
(c) 0.66 (d) 1.66
(d) of the third stage is less than that of the first
two stages 8. The efficiency of the nozzle-governed turbines is
affected mainly by losses due to
3. In a De Laval nozzle expanding superheated steam
(a) partial admission
from 10 bar to 0.1 bar, the pressure at the mini-
(b) throttling
mum cross-section will be
(c) inter-stage pressure drop
(a) 3.3 bar (b) 5.46 bar
(d) condensation in last stages
(c) 8.2 bar (d) 9.9 bar
9. The main aim of compounding steam turbine is to
4. A single-stage impulse turbine with a diameter of
(a) improve efficiency
120 cm runs at 300 rpm. If the blade speed ratio is
(b) reduce steam consumption
0.42, then, the inlet velocity of steam will be
(c) reduce motor speed
(a) 79 m/s (b) 188 m/s
(d) reduce turbine size
(c) 450 m/s (d) 900 m/s
10. A throttle-governed steam develops 20 kW with
5. In an ideal impulse turbine, the 281 kg/h of steam and 50 kW with 521 kg/h of
(a) absolute velocity at the inlet of moving is steam. The steam consumption in kg/hr when
equal to that at the outlet developing 15 kW will be nearly

M07_THERMAL ENGINEERING_SE_XXXX_CH07-Part-2.indd 371 6/15/2018 3:46:19 PM


372 Chapter 7

(a) 150 kg/hr (b) 156 kg/hr (b) the blade heights increase as the specific
(c) 241 kg/hr (d) 290 kg/hr volume of steam increases
11. Governing of steam turbines can be done by the (c) the blade heights decrease as the specific vol-
following: ume of steam increases
1. Nozzle control (d) the blade heights decrease as the specific
2. Throttle control volume of steam decreases
3. Providing additional value and passage 17. Which of the following statements are correct?
The correct answer will be 1. Impulse turbine rotor blades are thick at the
(a) 1, 2, and 3 (b) 1 and 2 only centre
(c) 2 and 3 only (d) 1 and 3 only 2. Rateau turbine is more efficient than Curtis
turbine
12. What is the cause of reheat factor in a steam
3. Blade velocity coefficient for an impulse tur-
turbine?
bine is the order of 60%
(a) Reheating (b) Superheating
(c) Supersaturation (d) Blade friction Select the correct answer using the codes given
below:
13. In a steam power plant, feed water heater is a heat
(a) 1, 2, and 3 (b) 1 and 2
exchanger to preheat feed water by
(c) 1 and 3 (d) 2 and 3
(a) live steam from steam generator
(b) hot flue gases coming out of the boiler furnace 18. Given that a1 = nozzle angle, n = number of rows
(c) hot air from air preheater of moving blades in a velocity compounded,
(d) extracting steam from turbine impulse turbine, the optimum blade speed ratio is
14. Match List I (turbines) with List II (classification) ncosa 1
(a) 2 cos a1n (b)
and select correct answer using the codes given 2
below the List: cos a 1 cosa 1
(c) (d)
List I List II 2( n + 1) 2n
A. Parson’s 1. Pressure compounded 19. In a Parson’s turbine stage, blade velocity is 320
B. De- Laval 2. Reaction m/s at the mean radius and the rotor blade exit
C. Rateau 3. Simple impulse angle is 30°. For minimum kinetic energy of the
steam leaving the stage, the steam velocity at the
D. Curtis 4. Velocity compounded
exit of the stator will be
Codes: (a) 640/ 3 m/s (b) 640 m/s
A B C D
(a) 3 2 1 4 (c) 320/ 3 m/s (d) 160/ 3 m/s
(b) 2 3 4 1 20. For a free vortex design of blade in the rotor of a
(c) 2 3 1 4 reaction axial turbine, the specific work along the
(d) 3 2 4 1 blade height is
15. The outward radial flow turbine in which there (a) higher at the blade hub and lower at the blade tip
are two rotors rotating in opposite directions is (b) constant from hub to tip
known an (c) lower at the hub and tip but different from the
(a) 50% reaction radial turbine mean section
(b) Cantilever turbine (d) the same at the hub and tip but different from
(c) Ljungstrom turbine the mean section
(d) Pass-out turbine 21. Which of the following sketches represents an
16. In reaction turbines, with reduction of inlet impulse turbine blade?
pressure (a)
(a) the blade heights increase as the specific vol-
ume of steam decreases

M07_THERMAL ENGINEERING_SE_XXXX_CH07-Part-2.indd 372 6/15/2018 3:46:21 PM


Steam Turbines 373

(b) (c) De -Laval, Curtis, Rateau, Parson’s


(d) Parson’s, Curtis, Rateau, De- Laval
26. Consider the following statements regarding the
(c) nozzle governing of steam turbines:
1. Working nozzles receive steam at full pressure
2. High efficiency is maintained at all loads
(d)
3. Stage efficiency suffers due to partial admis-
sion
22. The graph shown in Fig. 7.44 represents the varia- 4. In practice, each nozzle of the first stage is
tion of absolute velocity of steam along the length governed individually
of a steam turbine. Of these statements,
(a) 1, 2, and 3 are correct
(b) 2, 3, and 4 are correct
(c) 1, 3, and 4 correct
(d) 1, 2, and 4 are correct
Velocity

27. Among other things, the poor part-load perfor-


mance of De Laval turbines is due to the
(a) formation of shock waves in the nozzle
Distance from inlet
(b) formation of expansion waves at the nozzle
Figure 7.44 (c) turbulent mixing at the nozzle exit
(d) increase in profile losses in the rotor
The turbine in question is
28. List I gives the various velocities in the velocity
(a) Curtis turbine
diagrams of a two-stage impulse turbine. List II
(b) De-Laval turbine
gives the blade angles. Match the velocity from
(c) Radial turbine
List I with the angle in List II and select the cor-
(d) Parson’s turbine
rect answer using the codes given below the lists:
23. In a simple impulse turbine, the nozzle angle at
the entrance is 30°. What will be the blade-speed List I List II
ratio for maximum diagram efficiency?
A. Relative velocity of 1. Nozzle angle
(a) 0.433 (b) 0.25 steam an inlet tip of
(c) 0.5 (d) 0.75 blade
24. A reaction turbine stage has angles a1, b1, b2 as B. Absolute velocity of 2. M
 oving blade leading
nozzle angle, inlet blade and outlet blade angle, steam at inlet tip of edge angle
respectively. The expression for maximum effi- blade
ciency of the turbine is given by C. Relative velocity of 3. M
 oving blade trailing
steam at outlet tip of edge angle
2 cos 2 b1 2 cos 2 b 2 blade
(a) 2
(b)
1 + cos b1 1 + cos 2 b 2 D. Absolute velocity of 4. F
 ixed blade leading
steam at outlet tip of edge angle
2 cos 2 a 1 cos(a 1 + b1 ) blade
(c) (d)
1 + cos 2 a 1 cos 2 b 2 Codes:
A B C D
25. The correct sequence of the given steam turbines
(a) 1 2 4 3
in the ascending order of efficiency at their design
(b) 2 1 4 3
point is
(c) 2 1 3 4
(a) Rateau, De- Laval, Parson’s, Curtis
(d) 1 2 3 4
(b) Curtis, De- Laval, Rateau, Parson’s

M07_THERMAL ENGINEERING_SE_XXXX_CH07-Part-2.indd 373 6/15/2018 3:46:25 PM


374 Chapter 7

29. Which of the following relationship between (a) 200 N (b) 175 N
angles of fixed blades and moving blades corre- (c) 150 N (d) 125 N
sponds to that of Parson’s turbine?
36. At a particular section of a reaction turbine, the
(a) a1 = a 2 (b) a1 = b2 diameter of the blade is 1.8 m, the velocity of flow
(c) a 2 = b2 (d) b1 = b2 of steam is 49 m/s and the quantity of steam flow
30. The following data refer to an axial flow turbine is 5.4 m3/s. The blade height at this section will be
stage: approximately:
(a) 4 cm (b) 2 cm
Relative velocity of steam at inlet to the rotor = 79.0
(c) 1 cm (d) 0.5 cm
m/s., Relative velocity at the rotor exit = 152 m/s.
What is the approximate degree of reaction? 37. Consider the following statements:
(a) 0.9 (b) 0.8 If steam is reheated during the expansion through
(c) 0.7 (d) 0.6 turbine stages,
31. The clearance flow between the blade tips and 1. Erosion of blade will decrease
casing of a steam turbine is 2. The overall pressure ratio will increase
(a) greater in the reaction turbine than in the 3. The total heat drop will increase
impulse type Of these statements,
(b) greater in the impulse turbine than in the (a) 1, 2, and 3 are correct
reaction type (b) 1 and 2 are correct
(c) independent of type of the turbine (c) 2 and 3 are correct
(d) independent of the size of the turbine (d) 1 and 3 are correct
32. Running speeds of steam turbine can be brought 38. In an impulse-reaction turbine stage, the heat
down to practical limits drop in fixed and moving blades are 15 kJ/kg and
1. By using heavy flywheel 30 kJ/kg respectively. The degree of reaction for
2. By using a quick response governor this stage will be
3. By compounding (a) 1/3 (b) 1/2
4. By reducing fuel feed to the furnace (c) 2/3 (d) 3/4
(a) Only 3 (b) 1, 2, 3, and 4
39. If ‘D’ is the diameter of the turbine wheel and ‘U’
(c) 1, 2, and 4 (d) 2 and 3
is its peripheral velocity, then the disc friction loss
33. The net result of pressure-velocity compounding will be proportion to
of steam turbine is (a) (DU)3 (b) D2U3
(a) less number of stages (c) D3U2 (d) DU4
(b) large turbine for a given pressure drop
(c) shorter turbine for a given pressure drop 40. In a two-row Curtis stage with symmetrical
blading.
(d) lower friction loss
(a) work done by both rows of moving blades are
34. Given, vb = Blade speed equal
va1 = Absolute velocity of steam entering the blade, (b) work done by the first row of moving blades
a1= Nozzle angle is double of the work done by second row of
The efficiency of an impulse turbine is maximum moving blades
when (c) work done by the first row of moving blades
(a) vb = 0.5 va1 cosa1 is three times the work done by second row of
(b) vb = va1 cosa1 moving blades
(c) vb = 0.5 v2a1 cosa1 (d) work done by the first row of moving blades
(d) vb = v2a1 cosa1 is four times the wok done by the second row
of moving blades
35. An impulse turbine produces 50 kW of power
when the blade mean speed is 400m/s. What is 41. The compounding of steam turbines is done to
the rate of change of momentum tangential to the (a) improve efficiency
rotor? (b) reduce turbine speed

M07_THERMAL ENGINEERING_SE_XXXX_CH07-Part-2.indd 374 6/15/2018 3:46:25 PM


Steam Turbines 375

(c) increase blade speed ratio 46. Consider the following statements regarding a
(d) refuse axial thrust 100% reaction turbine:
42. The expression for the maximum efficiency of a 1. Change in absolute velocity of steam across
Parson’s turbine is (a1 is the angle made by abso- the moving blades is zero.
lute velocity an inlet) 2. Change in absolute velocity of steam across
the moving blades is negative.
cos 2 a 1 2 + cos 2 a 1
(a) (b) 3. Enthalpy drop in fixed blades is zero.
2(1 + cos 2 a 1 ) 2 cos 2 a 1 Which of these statements is/are correct?
2 cos a 1 2 cos 2 a 1 (a) Only 1 (b) Only 2
(c) (d) (c) 2 and 3 (d) 1 and 3
1 + cos 2 a 1 1 + cos 2 a 1
47. Which one of the following pairs is not correctly
43. Consider the following statements regarding
matched?
effects of heating of steam in a steam turbine:
1. It increases the specific output of the turbine (a) Internal efficiency of steam turbine: Product
2. It decreases the cycle efficiency of stage efficiency and reheat factor
3. It increases blade erosion (b) Stage efficiency of a turbine: Ratio of adia-
4. It improves the quality of exit steam. batic heat drop to the isentropic heat drop per
Which of these statements are correct? stage
(a) 1, 2, and 3 (b) 2 and 3 (c) Dryness fraction of steam within a stage:
(c) 3 and 4 (d) 1 and 4 Decreases due to reheating
44. Match List I (Different turbine stages) with List II (d) Steam condensation during expansion
(Turbines) and select the correct answer using the through the turbine: Enhances blade erosion
codes given below the lists: 48. Consider the following characteristics:
List I List II 1. High steam and blade velocities
2. Low steam and blade velocities
A. 50% reaction stage 1. Rateau
3. Low speeds of rotation
B. Two-stage velocity 2. Parson 4. High carry-over loss
compounded turbine Which of these characteristics are possessed by a
C. Single-stage impulse 3. Curtis simple impulse turbine?
D. Two-stage pressure 4. De-Laval (a) 1 and 2 (b) 2 and 3
compounded turbine (c) 1 and 4 (d) 3 and 4
5. Hero 49. Velocity triangle for a reaction turbine stage is
shown in the given Fig. 7.45 (AB = ua1 = absolute
Codes:
A B C D velocity at rotor blade inlet; CB = vr1 = relative
(a) 5 1 2 3 velocity at rotor blade inlet; CE = vr2 = relative
(b) 5 3 2 1 velocity at rotor blade exit and CD = CB)
(c) 2 3 4 1 C u A
(d) 3 1 4 2 b1 b2 a1
45. Partial admission steams turbine refers to the sit- vr1 va1 va2
vr2
uation where the D
(a) steam is admitted partially into the blades
through nozzles B E
(b) nozzle occupy the complete circumference
leading into the blade annulus Figure 7.45
(c) nozzle do not occupy the complete circumfer-
ence leading into the blade annulus The ratio of reaction force to impulse force is
(d) steam is admitted partially into the blades (a) CE/CB (b) CD/CE
directly (c) DE/BD (d) AE/AB

M07_THERMAL ENGINEERING_SE_XXXX_CH07-Part-2.indd 375 6/15/2018 3:46:27 PM


376 Chapter 7

50. Consider the following statements: u a1 cos a 1


1. Throttle governing improves quality of steam (a) (b) 2ua1 cosa1
2
in the last few stages
2. Internal efficiency of steam is not seriously (c) ua1 cos2a1 ua1 cosa1
(d)
affected by throttle governing 55. Which one of the following is the correct state-
3. Throttle governing is better than nozzle gov- ment? The degree of reaction of an impulse turbine
erning (a) is less than zero
Which of these statements are correct? (b) is greater than zero
(a) 1, 2, and 3 (b) 1 and 3 (c) is equal to zero
(c) 2 and 3 (d) 1 and 2 (d) increases with steam velocity at the inlet
51. Which one of the following statements is correct? 56. Why is compounding of steam turbines done?
(a) Reheat factor is zero if efficiency of the tur- (a) To improve efficiency
bine is close to unity (b) To reduce the speed of rotor
(b) Lower the efficiency, higher will be the reheat (c) To reduce exit losses
factor (d) To increase the turbine output
(c) Reheat factor is independent of steam condi-
57. A four-row velocity compounded steam turbine
tions at turbine inlet
develops 6400 kW. What is the power developed
(d) Availability of reheat is higher at low pressure
by the last row?
end
(a) 200 kW (b) 400 kW
52. For maximum blade efficiency of a single-stage (c) 800 kW (d) 1600 kW
impulse turbine, the blade speed ratio, (a1 is, the
angle made by absolute velocity at inlet) should be 58. Which of the following is used to bring down the
speed of an impulse steam turbine to practical
cos 2a 1
(a) cos 2a1 (b) limits?
2 (a) A centrifugal governor
cosa 1 2 (b) Compounding of the turbine
(c) (d)
2 cosa 1 (c) A large flywheel
(d) A gear box
53. Figure 7.46 shows the variation of certain steam
parameter in case of a simple impulse turbine. 59. Consider the following for a steam turbine power
The curve A-B-C represents the variation of plant:
1. Reduction in blade erosion
B 2. Increase in turbine speed
3. Increase in specific output
4. Increase in cycle efficiency
Which of the above occur/occurs due to reheating
of steam?
A C
(a) Only 1 (b) 1 and 2
(c) 1, 3, and 4 (d) 2 and 3
Nozzles Blades 60. Ratio of enthalpy drop in moving blades to the
total enthalpy drop in the fixed and moving blades
Figure 7.46 is called
(a) pressure in nozzle and blades (a) reheat factor
(b) velocity in nozzle and blades (b) blade efficiency
(c) temperature in nozzle and blades (c) degree of reaction
(d) enthalpy in nozzle and blades (d) internal efficiency
54. For a reaction turbine with degree of reaction 61. Employing superheated steam in turbines leads to
equal to 50%, (ua1 is the absolute steam veloc- (a) increase in erosion of blading
ity an inlet and a1 is the angle made by it to the (b) decrease in erosion of blading
tangent on the wheel) the efficiency is maximum (c) no erosion in blading
when the blade speed is equal to (d) no change in erosion of blading

M07_THERMAL ENGINEERING_SE_XXXX_CH07-Part-2.indd 376 6/15/2018 3:46:30 PM


Steam Turbines 377

62. Steam enters a De laval steam turbine with an (a) Low efficiency at low rotational speed
inlet velocity of 30 m/s and leaves with an out- (b) High efficiency with low fluid velocities
let velocity of 10 m/s. The work done by 1 kg of (c) High efficiency with high fluid velocities
steam is (d) Low efficiency at high rotational speed
(a) 400 Nm (b) 600Nm
67. In Parson’s reaction turbines, the velocity dia-
(c) 800 Nm (d) 1200 Nm
gram triangles at the inlet and outlet are
63. In a 50% reaction stage, absolute velocity angle at (a) asymmetrical
inlet is 45°, mean peripheral speed is 75 m/s and (b) isosceles
the absolute velocity at the exit is axial. The stage (c) right-angled
specific work is (d) congruent
(a) 2500 m2/s2 (b) 3270 m2/s2
2 2 68. In Parson’s turbine if a1 is nozzle angle, then what
(c) 4375 m /s (d) 5625 m2/s2
is the maximum efficiency of the turbine?
64. Consider the following statements:
2 cos a 1 2 cos 2 a 1
Steam turbines are suitable for use as prime mov- (a) (b)
ers for large steam power plants because (1 + cos a 1 ) (1 + cos 2 a 1 )
1. a single steam turbine can be designed for a
2 cos 2 a 1 cos 2 a 1
capacity of 1000 MW or more (c) (d)
2. much higher speed may be possible as com- (1 − cos a 1 ) (1 + 2 cos 2 a 1 )
pared to a reciprocating engine
69. What is the value of the reheat factor in multi-
3. they are more compact when compared to a
stage turbine?
gas turbine power plant
4. the maintenance cost and running cost may (a) 1.03 to 1.04 (b) 1.10 to 1.20
not increase with years of service (c) 0.90 to 1.00 (d) 1.20 to 1.25
Which of these statements are correct? 70. In which one of the following steam turbines, is
(a) 1, 2 and 3 (b) 2, 3 and 4 steam taken from various points along the turbine,
(c) 1, 2 and 4 (d) 1, 3 and 4 solely for feed-water heating?
65. Expansion line EFG of a 2-stage steam turbine (a) Extraction turbine
is shown in Fig. 7.47 (h-s diagram). The reheat (b) Bleeder turbine
factor for this turbine is (c) Regenerative turbine
p1 (d) Reheat turbine
E
71. Velocity diagram shown in Fig. 7.48 is for an
Enthalpy (h) in kJ/kg ( × 10−2 )

30
p2 impulse turbine stage.
28
27 F p3 4 m/s
v1
=9
60
23 m/
G s
20.2
20
Saturation
line a
450 m/s
910 m/s
Entropy (s)

Figure 7.47 Figure 7.48


(a) 1.08 (b) 0.7 What are the tangential force and axial thrust per
(c) 0.648 (d) 1.43 kg/s of steam, respectively?
66. Which of the following is the features of pres- (a) 450 N, 8 N (b) 560 N, 8 N
sure-compounding (Rateau staging)? (c) 680 N, 4 N (d) 910 N, 4 N

M07_THERMAL ENGINEERING_SE_XXXX_CH07-Part-2.indd 377 6/15/2018 3:46:33 PM


378 Chapter 7

72. Consider the following statements in respect of is 0.92 and rotor efficiency is 0.90, the isentropic
impulse steam turbines: static enthalpy drop will be
1. Blade passages are of constant cross-sectional (a) 352 kW (b) 347 kW
area. (c) 338 kW (d) 332 kW
2. Partial admission of steam is permissible.
78. Shock waves in nozzles would occur while tur-
3. Axial thrust is only due to change in flow
velocity of steam an inlet and outlet of moving bines are operating
blade. (a) at overload conditions
Which of the statements given above are correct? (b) at part load conditions
(a) 1, 2, and 3 (b) Only 1 and 2 (c) above critical pressure ratio
(c) Only 2 and 3 (d) Only 1 and 3 (d) at all off-design conditions

73. In an axial flow impulse turbine, energy transfer 79. Consider the following statements:
takes place due to The erosion of steam turbine blades increases
(a) change in relative kinetic energy with the increase of
(b) change in absolute kinetic energy 1. moisture in the steam
(c) change in pressure energy 2. blade speed
(d) change in energy because of centrifugal force Which of these statement(s) is/are correct?
(a) 1 alone (b) alone
74. In a reaction turbine the enthalpy drop in a stage (c) 1 and 2 (d) Neither 1 nor 2
is 60 kJ/kg. The enthalpy drop in the moving
blades is 32 kJ/kg. What is the degree of reaction? 80. Consider the following statements regarding an
(a) 0.533 (b) 0.284 impulse turbine:
(c) 0.466 (d) 1.875
1. Relative velocity at the inlet and exit of the
75. An emergency governor of a steam turbine trips rotor blades are the same.
the turbine when 2. Absolute velocity at the inlet and exit of the
1. Shaft exceeds 100% of its rated speed rotor blades are the same.
2. Condenser becomes hot due to inadequate 3. Static pressure within the rotor blade channel
cooling water circulation is constant
3. Lubrication system fails 4. Total pressure within the rotor blade channel
4. Balancing of turbine is not proper is constant
Select the correct answer form the codes given Of these statements,
below: (a) 1 and 4 are correct
Codes: (b) 2 and 3 are correct
(a) 1, 2, and 3 (b) 2, 3 and 4 (c) 1 and 3 are correct
(c) 3, 4, and 1 (d) 4, 1, and 2 (d) 2 and 4 are correct
81. If in an impulse turbine designed for free vortex
76. Steam enters the rotor of a reaction turbine with flow, the tangential velocity of steam at the root
an absolute velocity of 236 m/s and relative veloc- radius of 250 mm is 430 m/s and the blade height
ity of 132 m/s. It leaves the rotor with a relative is 100 mm, then the tangential velocity of steam
velocity of 2.32 m/s and absolute velocity of 136
at the tip will be
m/s. The specific work output is
(a) 602 m/s (b) 504 m/s
(a) 35.8 kW (b) 40.1 kW
(c) 409 m/s (d) 307 m/s
(c) 43.8 kW (d) 47.4 kW
82. The blade passage for the nozzle blade row of
77. The work output from a reaction turbine stage is the first stage of an impulse turbine is best repre-
280 kW per kg/s of steam. If the nozzle efficiency sented an

M07_THERMAL ENGINEERING_SE_XXXX_CH07-Part-2.indd 378 6/15/2018 3:46:33 PM


Steam Turbines 379

(a) and 2 refer to inlet and outlet, which one of the


following velocity triangles could be a reaction
turbine stage with reaction more than 50%?
(a)

va1
vr2
va2 vr1

(b)
u
(b)

vr2 va1

va2 vr1

(c) u

(c)

vr2 va1
va2
vr1
(d)

u
(d)
vr2 va1

va2 vr1
83. If u, ua, and ur represent the peripheral, absolute,
and relative velocities, respectively, and suffix 1 u

Explanatory Notes
n
p  2  n −1
3. (b) t =  
p1  n + 1
For superheated steam, n = 1.3
1.3
 2  0.3
pt = 10  = 5.457  5.46 bar
 2.3 
p × 1.2 × 3000
4. (c) u = = 188.5 m/s
60

u
Speed ratio = = 0.42
va1

M07_THERMAL ENGINEERING_SE_XXXX_CH07-Part-2.indd 379 6/15/2018 3:46:40 PM


380 Chapter 7

188.5
Inlet velocity of steam, va1 = = 448.8  450 m/s
0.42
2
7. (a)  ( ∆h) m = ( ∆h) f
3

( ∆h) m ( ∆h) m 2
Degree of reaction, R d = = = = 0.4
3
( ∆h) m + ( ∆h) f ( ∆h) + ( ∆h) 5
m m
2
10. (c)  Let steam consumption, y = A + Bx
where x = kW
281 = A + 20 B
521 = A + 50 B
Solving we get, A = 121, B = 8
y = 121 + 15 × 8 = 241 kg/h
19. (c) u = 320 m/s, b2 = 30°
320
va2 = u tan b 2 = 320 tan 30° = m/s
3
cos a 1 cos 30°
23. (a)  For maximum diagram efficiency, r = = = 0.433
2 2
vr22 − vr21 1522 − 792
30. (d)  Degree of reaction, Rd = = = 0.6
2u ( vw1 + vw 2 ) 2 × 14100

35. (d) P = m u (vw1 − vw2)/103 kW


103 P 50 × 103
Rate of change of momentum = m (vw1 − vw2) = = = 125 N
u 400
2
 = p dhv , h = 5.4 × 10 = 2 cm
36. (b)  V f
p × 1.8 × 49
( ∆h)m 30 30 2
38. (c)  Degree of reaction = = =
( ∆h)st 30 + 15 45 3
Total power 6400
57. (b)  Power developed in the last row = = = 400 kW
2 n
24
where n = number of rows
1 1
62. (a)  Work done by 1 kg of steam =  v12 − v22 ) = × 1(30 2 − 10 2 ) = 400 N.m
m(
2 2
63. (d) u = vw = vw1 + vw2 = 75
Specific work = u (vw1 + vw2) = u2 = 752 = 5625 m2/s2
Cumulative heat drop (30 − 27) + ( 28 − 20.2))
65. (a)  Reheat factor = =
Isentropic heat drop 30 − 20
3 + 7.8 10.8
= = = 1.08
10 10

M07_THERMAL ENGINEERING_SE_XXXX_CH07-Part-2.indd 380 6/15/2018 3:46:45 PM


Steam Turbines 381

71. (d) Tangential force = m (vw1 − vw2) = 1(910 − 0) = 910 N


Axial thrust = m (vf 1 − vf 2) = 1 × 4 = 4N

( ∆h) m 32
74. (a) R d = = = 0.533
( ∆h) st 60
76. (a) va1 = 236 m/s, vr1 = 132 m/s, vr2 = 232 m/s, va2 = 136 m/s
1 1
Specific work output = ( va21 − va22 ) + ( vr22 − vr21 )
2 2
1 1  1
=  ( 236 2 − 136 2 ) + ( 2322 − 1322 ) × 3
 2 2  10
= 35.8 kW
77. (c) P = 280 kW, hn = 0.92, hb = 0.90
P 280
Isentropic static enthalpy drop = = = 338.16 kW
h n hb 0.92 × 0.90
81. (d) r1 = 250 mm, u1 = 430 m/s, h = 100 mm
1
In a free vortex flow, u ∝
r
or u1r1 =u2r2
r2 = 250 + 100 = 350 mm
430 × 250
u2 = = 307 m/s
350

Review Questions

1. Explain the principle of operation of a steam turbine.


2. How do you classify steam turbines?
3. Compare impulse and reaction turbines.
4. What do you mean by compounding of steam turbines? What are the various methods of compounding
impulse steam turbines?
5. Define blade efficiency and stage efficiency.
6. What is the condition for maximum blade efficiency of an impulse turbine considering blade friction?
7. Define speed ratio and write the expression for a turbine with n blade rings.
8. What are the advantages of velocity compounding?
9. What are the limitations of velocity compounding?
10. What is the necessity of compounding impulse turbine?
11. Define degree of reaction.
12. What is the condition for blade angles for 50% degree of reaction?
13. Write the formula for maximum blade efficiency of a reaction turbine.
14. Define reheat factor and turbine internal efficiency.
15. List the various losses in a steam turbine.
16. What are the various methods for governing of a steam turbine?
17. Explain the working of a back-pressure turbine.
18. What is an extraction turbine?
19. Explain co-generation.
20. What is the role of Labyrinth packing?

M07_THERMAL ENGINEERING_SE_XXXX_CH07-Part-2.indd 381 6/15/2018 3:46:49 PM


382 Chapter 7

Exercises

7.1 In a single stage impulse turbine, the blade angles are equal and the nozzle angle is 20°. The velocity coef-
ficient for the blade is 0.83. Find the maximum blade efficiency possible.
If the actual blade efficiency is 90% of maximum blade efficiency, find the possible ratio of blade speed to
steam speed. [Ans. 0.813, 0.3235]
7.2 The following data refer to a compound impulse turbine having two rows of moving blades and one row of
fixed blades in between them.
Nozzle angle = 15°, Exit velocity of steam from the nozzle = 450 m/s
Moving blades tip discharge angles = 30°
Fixed blade discharge angle = 20°
Friction loss in each blade rows = 10% of the relative velocity
Calculate the blade velocity, blade efficiency, and specific steam consumption for the turbine.
[Ans. 98m/s, 78.8%, 45 kg/kWh]
7.3 In a stage of impulse-reaction turbine, steam enters with a speed of 250 m/s at an angle of 30° in the direc-
tion of blade motion. The mean blade speed is 150 m/s when the rotor is running at 3000 rpm. The blade
height is 10 cm. The specific volume of steam at nozzle outlet and blade outlet are 3.5 m3/kg and 4 m3/kg,
respectively. The turbine develops 250 kW. Assuming the efficiency of nozzle and blades together consid-
ered as 90% and carry over coefficient as 0.8, find (a) the enthalpy drop in each stage, (b) degree of reaction,
and (c) stage efficiency. [Ans. 29.3 kJ/kg, 0.39, 71%]
7.4 At a stage of reaction turbine, the mean diameter of the rotor is 1.4 m. The speed ratio is 0.7. Determine the
blade inlet angle if the blade outlet angle is 20°. The rotor speed is 3000 rpm. Find the diagram efficiency,
percentage increase in it, and rotor speed if the rotor is designed to run at the best theoretical speed, the exit
angle being 20°. [Ans. 53.8°, 90.5%, 3.65%, 93.8%]
7.5 In a 50% reaction turbine stage running at 50 rps, the exit angles are 30° and the inlet angles are 50°. The
mean diameter is 1 m. The steam flow rate is 104 kg/min and the stage efficiency is 85%. Calculate (a) the
power output of the stage, (b) the specific enthalpy drop in the stage, and (c) the percentage increase in the
relative velocity of steam when it flows over the moving blades. [Ans. 11.6 kW, 82 kJ/kg, 52.2%]
7.6 A single row impulse turbine develops 130 kW at a blade speed of 180 m/s using 2 kg/s of steam. The steam leaves
the nozzle at 400 m/s. The friction coefficient for blades is 0.9 and steam leaves the blades axially. Determine
(a) the nozzle angle and (b) the blade angles at entry and exit, assuming no shock. [Ans. 25.47°, 43.5°, 35.8°]
7.7 A simple impulse turbine has one ring of moving blades running at 150 m/s. The absolute velocity of steam
at exit from the stage is 80 m/s at an angle of 75° from the tangential direction. The blade speed coefficient
for blades is 0.85 and the rate of steam flowing through the stage is 3 kg/s. If the blades are equiangular,
determine (a) the blade angles, (b) the nozzle angle, (c) the absolute velocity of steam issuing from the
nozzle, and (d) the axial thrust. [Ans. 24.35°, 14.5°, 362.4 m/s]
7.8 In a single stage impulse turbine, the nozzle angle is 20° and blade angles are equal. The velocity coefficient
for blades is 0.85. Calculate the maximum blade efficiency possible. If the actual blade efficiency is 92% of the
maximum blade efficiency, find the possible ratio of blade speed to steam speed. [Ans. 81.6%, 0.336 or 0.603]
7.9 In a single stage steam turbine saturated steam at 10 bar is supplied through a convergent-divergent steam
nozzle of 20° angle. The mean blade speed is 400 m/s. The steam pressure leaving the nozzle is 1 bar. Find
(a) the best blade angles if they are equiangular and (b) the maximum power developed by the turbine if 5
nozzles are used of 0.6 cm2 area at the throat. Assume nozzle efficiency 90% and blade friction coefficient
0.87. [Ans. 35.46°, 115.8 kW]
7.10 The first stage of an impulse turbine is compounded for velocity and has two rows of moving blades and one
ring of fixed blades. The nozzle angle is 15° and leaving angles of blades are respectively, first moving 30°,

M07_THERMAL ENGINEERING_SE_XXXX_CH07-Part-2.indd 382 6/15/2018 3:46:49 PM


Steam Turbines 383

fixed 20°, second moving 30°. The velocity of steam leaving the nozzle is 540 m/s. The friction loss in each
blade row is 10% of the relative velocity. Steam leaves the second row of moving blades axially. Calculate
(a) the blade velocity, (b) the blade efficiency, and (c) the specific steam consumption.
[Ans. 117.3 m/s, 78.6%, 31.39 kg/kWh]
7.11    The first stage of a turbine is a two-row velocity compounded impulse wheel. The steam velocity at inlet is
600 m/s and the mean blade speed is 120 m/s. The nozzle angle is 16° and the exit angles for the first row
of moving blades, the fixed blades, and the second row of moving blades are 18°, 21° and 35°, respectively.
Calculate (a) the blade inlet angles for each row, (b) the driving force for each row of moving blades and
axial thrust on the wheel for a mass flow rate of 1 kg/s, (c) the diagram efficiency and diagram power for the
wheel per kg/s steam, and (d) the maximum possible diagram efficiency. Assume blade velocity coefficient
as 0.9 for all blades.
[Ans. First row: Moving blade 20°, fixed blade 24.5°, Second row: moving blade 34.5°; 875 N, 294 N;
42 N; 140.28 kW, 77.93%, 92.4%]
7.12    The following data refer to a stage of a Parson’s reaction turbine:
Speed of turbine = 1500 rpm
Mean diameter of rotor = 1 m
Stage efficiency = 80%
Blade outlet angle = 20°
Speed ratio = 0.7
Calculate the isentropic enthalpy drop in the stage. [Ans. 13.08 kJ]
7.13    A stage of a Parson’s reaction turbine delivers dry saturated steam at 2.7 bar from the fixed blades at 90 m/s.
The mean blade height is 4 cm, and the moving blade exit angle is 20°. The axial velocity of steam is 3/4 of
blade velocity at the mean radius. Steam is supplied to the stage at the rate of 2.5 kg/s. The effect of blade tip
thickness on the annulus area can be neglected. Calculate (a) the wheel speed, (b) the diagram power, (c) the
diagram efficiency, and (d) the enthalpy drop in this stage. [Ans. 1823 rpm, 13.14 kW, 78.7%, 5.25 kJ/kg]
7.14    In a stage of a reaction turbine, the mean rotor diameter is 1.5 m, speed ratio = 0.72, blade outlet angle = 20°
and rotor speed = 3000 rpm. Calculate (a) the diagram efficiency and (b) the percentage increase in diagram
efficiency and rotor speed if the rotor is designed to run at the best theoretical speed, the exit angle being 20°.
 [Ans. 91%, 2.96%, 3914.7 rpm]
7.15    The nozzle angle of a single stage impulse turbine is 20°. The moving blade angles are equal and the
velocity coefficient for the blade is 0.83. Find the maximum blade efficiency possible. If the actual blade
efficiency is 90% of the maximum blade efficiency, find the possible ratio of blade speed to steam speed.
[ESE, 1979]
7.16    The total power developed by a 20-stage reaction turbine is 11200 kW. Steam is supplied at 15 bar abs. and
300°C. The pressure of steam leaving the turbine is 0.1 bar abs. The turbine has a stage efficiency of 75%
and a reheat factor of 1.05. Determine the mass flow rate of steam through the turbine. Assume that all
stages develop equal power. [ESE, 1980]
7.17    At a certain stage of the turbine in Exercise 7.16, the pressure of steam is 1 bar and it is dry saturated. If the
blade exit angle is 20° and the ratio of the blade speed to steam speed is 0.7, find the mean, diameter of the
1
rotor of this stage, and the rotor speed. Assume the blade height as the mean drum diameter and neglect
10
the thickness of the blades. The velocity diagrams for the blades are symmetrical.[ESE, 1980]
7.18    An impulse stage of a steam turbine is supplied with dry and saturated steam at 15 bar. The stage has a single
row of moving blades running at 3600 rpm. The mean diameter of the blade disc is 9.0 m. The nozzle angle
is 15° and the axial component of the absolute velocity leaving the nozzles is 93.42 m/s. The height of noz-
zles at their exit is 100 mm. The nozzle efficiency is 0.9 and the blades velocity coefficient is 0.965. The exit

M07_THERMAL ENGINEERING_SE_XXXX_CH07-Part-2.indd 383 6/15/2018 3:46:49 PM


384 Chapter 7

angle of the moving blades is 2° greater than that at the inlet. Determine (a) the blade inlet and outlet angles,
(b) the isentropic heat drop in the stage, (c) the stage efficiency, and (d) power developed by the stage.
[ESE, 1982]
7.19    A single row impulse turbine stage running at 3000 rpm has a blade to steam speed ratio of 0.48. The nozzles
are set at angle of 15° with respect to the place of the blade disc. The mean velocity of the blades is 144 m/s.
The moving blades are symmetrical about their axis parallel to the direction of rotation. The nozzle efficiency
is 0.92 and the blade velocity coefficient is 0.97. The height of the nozzles at their exit section is 100 mm.
Steam leaving in the nozzles is saturated and dry and its pressure is 10 bar. Determine (a) the isentropic heat
drop in the stage, (b) the energy lost in the nozzles and the moving blades due to friction, (c) energy lost due
to finite velocity of steam leaving the stage, (d) mass flow rate, (e) power developed by the stage, and (f) the
efficiency of the stage.
Assume that steam is accelerated from rest in the nozzles. [ESE, 1983]
7.20    An impulse stage of steam turbine has a mean diameter of 1.2 m. The speed of rotor is 3000 rpm. The mass
flow rate of steam is 20 kg/s. Steam is supplied to the stage at 15 bar and 300°C, where it expands to 10 bar.
Determine the efficiency ant the power output of the stage if the nozzle efficiency is 0.9 and the blade velocity
coefficient is 0.92. Assume acceleration from rest for the steam expanding in the nozzle. Assume nozzle angle
to be 25°. [ESE, 1986]
7.21    Steam enters a single-stage impulse turbine at 380 m/s and the blade speed is 170 m/s. The steam flow
rate is 2.2 kg/s and turbine develops 112 kW. Assume the blade velocity coefficient to be 0.8. For an axial
discharge of steam, find (a) nozzle angle, (b) blade angles, and (c) diagram efficiency. [ESE, 1987]
7.22    Steam enters in a stage of impulse-reaction turbine with a speed of 280 m/s at an angle of 22° in the direction
of blade motion. The mean diameter of the rotor, which rotates at 3200 rpm, is 1.0 m. The blade height is
10 cm. The specific volume of steam at nozzle outlet and blade outlet are 3.6 m3/kg and 4.1 m3/kg,
respectively. The turbine develops 462 kW. Find (a) the weight of steam used per second, (b) blade angle,
(c) enthalpy drop in each stage, (d) degree of reaction, and (e) stage efficiency. Assume combined efficiency
of nozzles and blades as 90% and carry over coefficient as 0.8. [ESE, 1988]
7.23    The nozzles of a Lavel turbine deliver steam at the rate of 0.9 kg/s with a velocity of 730 m/s to a set of
blades revolving at the rate of 30000 rpm The diameter of the wheel is 11.5 cm. The nozzles are inclined
at an angle of 20° to the plane of wheel rotation. Calculate (a) the diagram efficiency, (b) power developed
by the blades, (c) energy lost in the blades per second, and (d) the condition for maximum efficiency of the
turbine. Assume the blade velocity coefficient as 0.72 and outlet blade angle 25°C. [ESE, 1989]
7.24    The nozzles of a two-row velocity-compounded stage have outlet angles of 22° and utilise an isentropic
enthalpy drop of 200 kJ per kg of steam. All moving and guide blades are symmetrical, and the mean blade
speed is 150 m/s. Assuming an isentropic efficiency for the nozzles of 90%, find graphically all the blade
angles and calculate the specific power output produced by the stage. The velocity at inlet to the stage can
be neglected. [ESE, 1991]
7.25    One stage of an impulse turbine consists of a converging nozzles ring and one ring of moving blades. The
nozzles are inclined at 22° to the blade whose tip angles are both 35°. If the velocity of steam at exit from
the nozzle is 660 m/s, find the blade speed so that the steam shall pass on without shock. Find the diagram
efficiency neglecting losses if the blades are run at this speed. [ESE, 1992]
7.26    A single steam turbine is supplied with steam at 5 bar, 200°C at the rate of 50 kg/min. It exhausts into a
condenser at a pressure of 0.2 bar. The blade speed is 400 m/s. The nozzles are inclined at an angle of 20°
to the plane of the wheel and the outlet blade angle is 30°. Neglecting friction losses, determine the blade
efficiency, the stage efficiency and the power developed by the turbine. [ESE, 1994]

M07_THERMAL ENGINEERING_SE_XXXX_CH07-Part-2.indd 384 6/15/2018 3:46:49 PM


Steam Turbines 385

Answers to Multiple-choice Questions


1. (a) 2. (b) 3. (b) 4. (c) 5. (b)
6. (c) 7. (a) 8. (a) 9. (c) 10. (c)
11. (a) 12. (d) 13. (d) 14. (c) 15. (c)
16. (b) 17. (b) 18. (d) 19. (c) 20. (a)
21. (a) 22. (d) 23. (a) 24. (c) 25. (c)
26. (d) 27. (b) 28. (b) 29. (b) 30. (d)
31. (a) 32. (d) 33. (a) 34. (a) 35. (d)
36. (b) 37. (d) 38. (c) 39. (c) 40. (a)
41. (b) 42. (d) 43. (d) 44. (a) 45. (c)
46. (c) 47. (c) 48. (c) 49. (c) 50. (c)
51. (b) 52. (a) 53. (b) 54. (d) 55. (d)
56. (b) 57. (b) 58. (a) 59. (d) 60. (c)
61. (a) 62. (a) 63. (d) 64. (c) 65. (a)
66. (b) 67. (c) 68. (c) 69. (b) 70. (b)
71. (d) 72. (d) 73. (a) 74. (a) 75. (c)
76. (a) 77. (c) 78. (a) 79. (c) 80. (a)
81. (d) 82. (c) 83. (c)

M07_THERMAL ENGINEERING_SE_XXXX_CH07-Part-2.indd 385 6/15/2018 3:46:49 PM


Chapter 8

Steam Condensers

8.1  ❐  DEFINITION
A condenser is a closed vessel in which steam is condensed by abstracting the heat and where the
pressure is maintained below atmospheric pressure.

8.2  ❐  FUNCTIONS OF A CONDENSER


There are two main functions of a condenser, which are explained below:
1. To reduce the back pressure considerably upon the prime mover, and thereby increasing the work
output of steam during expansion.
  Consider the ideal p−v diagram of a non-condensing steam engine, as shown in Fig. 8.1(a), in
which steam is supplied at pressure p1 and the exhaust takes place at atmospheric pressure p2. The
work output is given by the area 1-2-3-4-5-1. If we reduce the back pressure p2 to p2′ , which is
below the atmospheric pressure, keeping p1 same, then the work output will be given by the area
1-2-3-4′-5′-1, as shown in Fig. 8.1(b). This work output is obviously greater, than the previous
work output. The additional work output is 4-4′-5′-5-4.
2. The condensate condensed from the exhaust steam in the condenser is free from impurities and
can be directly used as feed water for the boiler without any treatment. This recovery of feed water
is an incidental advantage.
p p
p1 1 2 p1 1 2

3 3
p2 p2 5 4
4
5 p′2 4′
5′
v v
(a) (b)

Figure 8.1  Effect of back pressure on work output of a steam engine

M08_THERMAL ENGINEERING_SE_XXXX_CH08.indd 386 6/15/2018 3:51:41 PM


Steam Condensers 387

8.3  ❐  ELEMENTS OF STEAM CONDENSING PLANT


A steam power plant using a condenser is shown in Fig. 8.2. The various components of a steam
condensing plant are described below.
1. Condenser: It is a closed vessel heat exchanger in which steam coming from the turbine is con-
densed by abstracting heat by cooling water where the pressure is maintained below atmospheric
pressure.
2. Condenser cooling water pump: The cooled water from the cooling tower is circulated through
the condenser by this pump. This pump is not required if water from the cooling tower is not
re-circulated.
3. Make up water pump: This pump circulates the required quantity of water through the condenser.
4. Condensate extraction pump: The function of the condensate pump is to extract the condensate
from the condenser where the pressure is below atmospheric pressure and feed it to the hot well,
where the pressure is at atmospheric level.
5. Hot well: It is a sump between the condenser and the boiler where the condensate coming from
the condenser is collected.
6. Air-extraction pump: The function of the air-extraction pump is to extract the air which has
leaked into the condenser by various paths and to maintain the required vacuum inside.
7. Boiler feed pump: Its function is to pump the condensate from hot well into the boiler by increas-
ing the pressure of condensate above the boiler pressure.
8. Cooling tower: It is essential when large amount of cooling water required is not available from
the river throughout the year. Its function is to cool the hot water coming out of the condenser
instead of discharging it to waste. The hot water is cooled by evaporative cooling.

Steam
Steam Cooling
turbine tower

Boiler
Cooling water
Condenser pump
River

Make up
water pump
Air
extraction
pump
Condensate
Boiler water pump
feed pump

Feed water
pump

Heater

Figure 8.2  Steam power plant using a condenser

M08_THERMAL ENGINEERING_SE_XXXX_CH08.indd 387 6/15/2018 3:51:42 PM


388 Chapter 8

8.4  ❐  TYPES OF STEAM CONDENSERS


Steam condensers may be broadly classified into two categories.
1. Jet condensers: They are contact-type condensers in which the steam to be condensed mixes
with the cooling water and the temperature of the condensate and the cooling water is the same
when leaving the condenser. The condensate cannot be recovered for use as feed water to the
boiler. Heat transfer is by direct conduction between steam and water.
  These condensers may be further classified as follows:
(a) Low-level, counter flow type condenser
(b) Low-level, parallel flow type condenser
(c) High-level jet condenser
(d) Ejector condenser
2. Surface condensers: There is no direct contact between steam to be condensed and the circulat-
ing cooling water in surface condensers. Heat is transferred through a wall interposed between
steam and cooling water. The temperature of condensate may be higher than the cooling water.
The condensate can be used as feed water to the boiler.
  These condensers may be further classified as follows:
(a) Cooling water in tubes and steam surrounding it
(b) Central flow or regenerative condenser
(c) Evaporative condenser

8.4.1  Jet Condensers


1. Low level counter-flow jet condenser: This type of condenser is shown in Fig. 8.3. The cold
water is drawn up in the condenser shell from the cooling pond due to the vacuum head created in
the shell. The level of the cooling pond should not be much below the delivery pipe to the shell as
no pump is used. The shell is arranged with two or three water trays with perforations to break up
water into small jets. The exhaust steam and any mixed air enters at lower portion of the shell and
tries to ascend up through the falling sprays. Thus, steam gets condensed and air ascends up and
cools down. The air is removed by a separate suction pump at the top. The mixture of condensate
and cooling water descends down through a vertical pipe to the extraction pump and pumped to
the hot well. From the hot well, the boiler feed pump delivers water to the boiler and surplus water
overflows to the cooling pond. Since the boiler feed and the cool injection water mix, this type of
condenser may be used only where cheap pure water is available. The non-condensable air and
gas are continuously removed by air pump suction to maintain vacuum.
2. Low level parallel flow Jet condenser: It is similar to the counter flow jet condenser except that
steam enters the condenser shell at the top and just below it the cooling water is delivered. It is
less efficient than the counter flow type.
3. High level jet condenser: This is also called Barometric condenser and is shown in Fig. 8.4.
The shell is placed at a height greater than 10.363 m, the barometric height of water column. If a
pipe of height more than 10.363 m is held vertical with one end immersed in water vessel open
to atmosphere and the other end subjected to suction pressure, the atmospheric pressure will hold
the column of water in the pipe equal to the suction pressure. This fact is made use of in this con-
denser by making the tail pipe more than 10.363 m in height and thus making it impossible for

M08_THERMAL ENGINEERING_SE_XXXX_CH08.indd 388 6/15/2018 3:51:42 PM


Steam Condensers 389

Air pump
suction

Injection
water sprays
Trays to break
up the sprays
Exhaust Condenser
steam inlet Shell

Normal water
level

Submergence Removal
Boiler pump suction
feed pipe

Extraction
pump

Hot well Over flow Cooling pond

Figure 8.3  Low level counter flow jet condenser

Air pump suction

Steam
inlet

Tail pipe
water column
Height of

Injection
Over pump
flow

Dry air
pump
Hot well

Cooling pond

Figure 8.4  High-level jet condenser

M08_THERMAL ENGINEERING_SE_XXXX_CH08.indd 389 6/15/2018 3:51:43 PM


390 Chapter 8

Water inlet

Non-return valve
Converging guide cones

Exhaust steam
Hollow truncated
cones

Diverging cones

Discharge to hot well

Figure 8.5  Ejector condenser

any vacuum in the condenser to cause the water to rise high in the tail pipe of the water leg and
flood the engine.
The height of the shell much above the barometric height requires a separate pump for inject-
ing cool water. The condensate and water will fall under gravity to the hot well and maintain a
column of water in the water leg, depending on the vacuum in the condenser. A separate air pump
to remove air is employed.
4. Ejector condenser: It is a low-level type of condenser (as shown in Fig. 8.5). This condenser
works on the principle that by discharging a smooth jet of cold water under a head of about 6 m
through a series of converging guide cones, the steam and the associated air are drawn in through
the hollow truncated cones and led to the diverging cones. In the converging cones, the pressure
energy is partly converted to kinetic energy. In the diverging cones, the kinetic energy is again
partly converted to pressure energy so as to obtain pressure greater than the atmospheric to enable
the condensate and water mixture to be discharged to the hot well exposed to atmospheric pressure.
A non-return valve is fitted on the exhaust steam inlet to condenser so that water from the hot
well does not rush back to the engine in case of cold injected water failure.

8.4.2  Surface Condensers


Water Tube Surface Condenser
In this type of surface condenser, water flows in the tubes and steam surrounds it. Thus, the conden-
sate is directly available as an ideal boiler feed. A surface condenser produces more vacuum and any
type of cooling water can be used. However, it is bulky and the initial capital cost is higher than jet
condensers. But, this is justified by the saving in the running cost due to high efficiency.
This type of condensers may be further classified as follows:
1. Single pass, two pass condensers
2. Down flow: central flow regenerative condensers
A longitudinal section of a two pass, down flow condenser is shown in Fig. 8.6. It consists of a cast
iron shell, cylindrical in shape, with the two ends covered by cover plates. A nest of brass tubes is fixed
in the two tube plates at the end. They are fixed by brass ferrules so that they can be replaced easily.

M08_THERMAL ENGINEERING_SE_XXXX_CH08.indd 390 6/15/2018 3:51:44 PM


Steam Condensers 391

The space between the tube plates and the cover plates is known as water boxes. One of the water
boxes has baffle partitioning the box into two sections—one upper half and the other lower half.
The cold water is sent through the lower half section tubes and comes out through the upper half
section tubes. Exhaust steam enters the shell at the top and flows down surrounding the cold water
tubes. The condensate is removed at the bottom by an extraction pump.
The surface condensers may work on the wet vacuum system or the dry vacuum system. For the
dry vacuum system, three pumps are required—one for the circulating cooling water, the other for
removing the condensate, and the third, the dry air pump, for removing air. The cross-section of such
a dry vacuum system is shown in Fig. 8.7. The air exit is shielded from the down flow of steam by
means of a baffle.
The main requirements of a surface condenser are as follows:
1. Air should be removed at a cooler section and it should be as far as possible dry.
2. The pressure drop of steam in the condenser should be minimum.

Exhaust steam
from turbine

Water box
Tube plate Cooling water
outlet
Water box

Cooling water
inlet

To condensate pump

Figure 8.6  Water tube surface condenser

Exhaust steam from turbine

Baffle for air


Tube cooling section

To dry
air pump

To condensate
pump

Figure 8.7  Down flow surface condenser

M08_THERMAL ENGINEERING_SE_XXXX_CH08.indd 391 6/15/2018 3:51:45 PM


392 Chapter 8

Central Flow or Regenerative Condenser


In such a condenser as shown in Fig. 8.8, the steam passages are all around the periphery of the shell.
The exhaust steam converges to the centre of the shell where there are no tubes. The air is removed
by the air pump suction from the centre. Some of the exhaust steam also passes to the centre and gets
condensed. The condensate is reheated to nearly the temperature of the exhaust steam. The pressure
drop of steam is also reduced due to large flow passages.

Exhaust steam from turbine

Air pump
suction

To condensate
pump

Figure 8.8  Central flow surface condenser

Evaporative Condenser
In evaporative condenser, steam circulates in pipes surrounded by water spray, as shown in Fig. 8.9.
It is used when there is scarcity of cooling water. The cooling water is allowed to evaporate under a
small partial pressure. It consists of gilled piping bent to form many rows and is placed vertically.

Air out

Fan

Drift eliminators
Water spray
Exhaust
steam in

To condensate Condenser coils


pump
Air in Air in
Air louvers

Water pump
Cooling pond

Figure 8.9  Evaporative condenser

M08_THERMAL ENGINEERING_SE_XXXX_CH08.indd 392 6/15/2018 3:51:47 PM


Steam Condensers 393

Steam passes through these pipes. Pumped water is sprayed from the top and descends down forming
a thin film over the pipes. A natural or forced air draught causes the evaporation of the water film.
Steam while circulating inside the pipes, latent heat of evaporation is removed from steam and thereby
the steam gets condensed. This mode of heat transfer reduces the cooling water requirement of con-
denser to 10% of the requirement for surface condensers. The water particles carried with air due to
high velocity of air are removed with the help of eliminator. This type of condenser works better in
dry weather.

8.5  ❐  REQUIREMENTS OF MODERN SURFACE


CONDENSERS
1. The steam should be evenly distributed over the entire cooling surface of the condenser with
minimum pressure loss.
2. There should be no undercooling of the condensate.
3. Water should be passed through the tubes and steam must surround the tubes from outside.
4. There should be no air leakage into the condenser to maintain proper vacuum.
5. Air must be cooled to the maximum possible extent before extraction.
6. For better thermal efficiency, the rise in temperature of the cooling water should be limited to
10°C.

8.6  ❐  COMPARISON OF JET AND SURFACE CONDENSERS


8.6.1  Jet Condensers
Advantages
1. Mixing of steam and cooling water is more intimate.
2. Less quantity of circulating water for condensing exhaust steam is required.
3. Condensing plant is simple and less costly.
4. Spare parts required are less.
5. Condensate extraction pump is not needed in barometric jet and ejector condensers.
Disadvantages
1. Condensate is wasted unless cooling water is pure.
2. In the low-level jet condenser, if the condensate extraction pump fails, then the engine would get
flooded unless safe guarded.
3. Barometric condenser cost is high due to long piping.
4. There is vacuum loss in barometric condenser due to leakage in long piping.
5. There are limitations of vacuum of up to 650 mm of Hg due to liberation of dissolved gases in
cooling water.
6. The air extraction pump requires comparatively high power.

M08_THERMAL ENGINEERING_SE_XXXX_CH08.indd 393 6/15/2018 3:51:47 PM


394 Chapter 8

8.6.2  Surface Condensers


Advantages
1. Vacuum obtained is comparatively very high, resulting in high efficiency.
2. Condensate can be used directly as boiler feed.
3. There is no need of water treatment plant.
4. It is more suitable for high capacity plants.
Disadvantages
1. It requires more space.
2. Maintenance cost is more.

8.7  ❐  VACUUM MEASUREMENT


A vacuum gauge is used to measure the vacuum in the condenser. Figure 8.10 shows a vacuum gauge.
It consists of a glass tube containing mercury to indicate the barometric pressure and another mercury
tube connected to the condenser and vacuum gauge. The vacuum gauge has readings marked in cm of
Hg. It represents the height at which a column of mercury, the upper surface of which is in communi-
cation with the cylinder, will stand when supported by barometric pressure. The vacuum is dependent
upon the barometric pressure and the absolute pressure in the condenser.

Vacuum gauge

Condenser
Torricelli’s
shell
vacuum

Absolute condenser
pressure
Barometric height

Vacuum
Barometric
pressure

Mercury

Figure 8.10  Vacuum gauge

M08_THERMAL ENGINEERING_SE_XXXX_CH08.indd 394 6/15/2018 3:51:47 PM


Steam Condensers 395

Absolute pressure in the condenser = barometric pressure - vacuum gauge pressure


Usually, the vacuum gauge readings are corrected to standard barometer reading of 76 cm of Hg.
Thus,
Corrected vacuum in cm of Hg = 76 - absolute pressure in cm of Hg
= 76 - (actual barometric height - vacuum)
Let Hb = actual barometric height in cm of Hg
Hg = actual vacuum gauge reading in cm of Hg
Hb - Hg = absolute pressure in condenser in cm of Hg
p = absolute condenser pressure in bar
Now, 76 cm of Hg = 1.01325 bar
1.01325
1 cm of Hg = = 0.0133322 bar
76
p = 0.133322 (Hb - Hg) bar (8.1)
Corrected or standard vacuum = 76 - (Hb - Hg) cm of Hg (8.2)

8.8  ❐  DALTON’S LAW OF PARTIAL PRESSURES


Dalton’s law states that
1. The total pressure in a container having mixture of gas and vapour is the sum of the partial pres-
sure of the vapour at the common temperature and the partial pressure of the gas.
If the degree of saturation of air at any point inside the condenser is 100%, the partial pressure
of vapour is the same as the saturation pressure.
Let t°C = temperature of mixture in a container.
pa = absolute pressure of any gas or air at t°C.
ps = saturation pressure of any vapour corresponding to t°C.
pt = total pressure in the container
\ pt = pa + ps(8.3)
or pa= pt - ps (8.4)
2. Each constituent of the mixture in the container occupies the whole volume of the container and
exerts its own partial pressure.
Let V = volume of a container, m3
ma = mass of air in the container, kg
mg = mass of water vapour in the container, kg
m = total mass of mixture in the container, kg
vg = specific volume of saturated vapour at t°C and pa in m3/kg
va = specific volume of air at t°C in m3/kg
Then V = mava = mgvg (8.5a)
ma vg
or      = (8.5b)
mg va
m 1
Mass of air per m3 of container = a = (8.6)
V va
mg 1
Mass of vapour per m3 of container = = (8.7)
V vg

M08_THERMAL ENGINEERING_SE_XXXX_CH08.indd 395 6/15/2018 3:51:50 PM


396 Chapter 8

 m   vg 
Mass of mixture, m = ma + mg = mg 1+ a  = mg 1+ v  (8.8)
 mg   a

 mg   va 
or m = ma 1+  = ma 1+  (8.9)
 ma   vg 

Example 8.1
A vacuum of 67 cm of Hg was obtained with the barometer reading of 75 cm of Hg. The conden-
sate temperature is 20°C. Correct the vacuum to a standard barometer of 76 cm and hence deter-
mine the partial pressure of air and steam. Also find the mass of air present with 1 kg of steam.

Solution
Corrected vacuum = 76 - (actual barometric reading - actual vacuum)
= 76 - (75 - 67) = 68 cm of Hg
Pressure of dry saturated steam at 20°C from steam tables is 0.023385 bar
76 × 0.023385
or = 1.754 cm of Hg
1.01325
and specific volume is v = 57.79 m3/kg
Total absolute pressure in the condenser = 75 - 67 = 8 cm of Hg
Partial pressure of steam, pg = 1.754 cm of Hg
\ Partial pressure of air, pa = 8 - 1.754 = 6.246 cm of Hg
According to Dalton’s law the volume of air present per kg of steam
= 57.79 m3/kg at 20°C
Also for air paV = maRT
(0.0133322 × 6.246 × 105 ) × 57.79
ma = = 5.723 kg of air per kg of steam
287 × 293

8.9  ❐  M
 ASS OF COOLING WATER REQUIRED
IN A CONDENSER
Let m w = mass flow rate of circulating water, kg/h
m s = mass flow rate of steam condensed, kg/h
t = temperature of wet exhaust steam, °C
tc = temperature of condensate, °C
hf = enthalpy of water at t°C, kJ/kg
hfg = enthalpy of evaporation of steam at t°C, kJ/kg
x = dryness fraction of exhaust steam
tiw = inlet temperature of cooling water, °C
tow = outlet temperature of cooling water, °C
hc = enthalpy of condensate, kJ/kg = cpw tc
cpw = specific heat of cooling water, 4.1868 kJ/kg.K

M08_THERMAL ENGINEERING_SE_XXXX_CH08.indd 396 6/15/2018 3:51:52 PM


Steam Condensers 397

Now Heat given by steam = Heat absorbed by water


For jet condenser,
tow = tc
m s ( h f + xh fg − hc )
mw = (8.10)
c pw (tc − tiw )
For surface condenser,
ms (hf + xhfg - hc) = mw cpw (tow - tiw)
m s ( h f + xh fg − hc )
mw = (8.11)
c pw (tow − tiw )

Example 8.2
A 200 kW steam engine has a steam consumption of 10 kg/kWh. The back pressure of the engine
is equal to the condenser pressure of 0.15 bar. The condensate temperature is 32°C. The cooling
water temperature at inlet and outlet are 20°C and 30°C, respectively. Calculate the mass of cooling
water required per hour if the exhaust steam is dry saturated.

Solution
At 0.15 bar, hf = 225.91 kJ/kg, hfg = 2373.1 kJ/kg, tc = 32°C, tow = 30°C, tiw = 20°C
ms ( h f + h fg − hc ) 1( 225.91 + 1 × 2373.1 − 4.1868 × 32)
mw = = = 58.88 kg/h/kg of steam
c pw (tow − tiw ) 4.1868 (30 − 20)
Total steam used per hour = 200 × 10 = 2000 kg
Total circulating water used = 58.88 × 2000 = 117760 kg/h

8.10  ❐  AIR REMOVAL FROM THE CONDENSER


8.10.1  Sources of Air Infiltration in Condenser
1. Air may pass into the condenser with the steam.
2. Air leakage may happen through the packings.
3. The cooling water contains dissolved gases which are released under vacuum.
The amount of air leakage should not be more than 5-15 kg/10000 kg of steam.

8.10.2  Effects of Air Infiltration in Condensers


1. The leaked air in the condenser increases the back pressure on the prime mover, and consequently,
the thermal efficiency of the plant is lowered.
2. The infiltrated air lowers the partial pressure of steam which means a lower saturation tempera-
ture and increased latent heat. Hence, it will require greater amount of cooling water.
3. Because of poor thermal conductivity of air, the rate of heat transfer from the vapour is reduced
which requires increased surface area of tubes for a given condenser duty.

M08_THERMAL ENGINEERING_SE_XXXX_CH08.indd 397 6/15/2018 3:51:54 PM


398 Chapter 8

8.11  ❐  AIR PUMP


The function of the air pump is to maintain a desired vacuum in the condenser by removing air and
condensed steam. Edward’s air pump is commonly used for this purpose.

8.11.1  Edward’s Air Pump


This pump is shown in Fig. 8.11. It consists of a plunger or piston D having a conical head E which
slides inside a barrel or cylinder liner C having a cover B which has a number of delivery valves A.
The passage G is connected to the condenser. The barrel has a number of parts, F pierced in the cyl-
inder liner C. These parts communicate to the air pump suction pipe. H is the water weir where the
discharged condensate collects and overflows to the hot well. This retention of sufficient water above
the cylinder head acts as a water seal against leakage of air. J is the relief valve.
On the downward stroke of the piston, partial vacuum is created above the piston because the head
valves are closed and sealed by water. As soon as the ports are uncovered, the water vapour and air
push into the space above the piston. The condensate collected at the piston is displaced by further
motion of the piston with conical head and pushed rapidly through the ports to the top of the piston.
On the upward stroke, the water, air, and water vapour above the piston are compressed to pressure
above the atmospheric pressure and discharged outside through the head valves.
p 2
Capacity of the pump, V = D LNh v
4
where D = piston diameter, L = stroke length, N = rpm, h v = volumetric efficiency.

B A

F F
G

D J

Figure 8.11  Edward’s air pump

M08_THERMAL ENGINEERING_SE_XXXX_CH08.indd 398 6/15/2018 3:51:55 PM


Steam Condensers 399

8.12  ❐  VACUUM EFFICIENCY


The lowest pressure which can exist in a condenser is the saturation pressure of steam corresponding
to the temperature of water entering the condenser. However, the actual pressure is always greater than
the ideal pressure by an amount equal to the partial pressure of air present in the condenser.
Let ps = saturation pressure of steam corresponding to the temperature of water entering the
condenser.
pa = partial pressure of air in the condenser.
pb = barometric pressure
pt = pa + ps
= total pressure of air and steam in the condenser
Then ideal vacuum possible without air leakage = pb - ps
Actual vacuum existing in condenser due to air leakage
= pb − pt = pb − (pa + ps)
The “vacuum efficiency” is defined as the ratio of actual vacuum to ideal vacuum.
p − ( pa + ps )
\ Vacuum efficiency, hvac = b (8.12)
pb − ps
The factors affecting the vacuum efficiency are as follows:
1. Air leakage increases pa, and hence, it decreases vacuum efficiency.
2. The vacuum efficiency decreases with increase in barometric pressure, keeping pa and ps the same.
3. With insufficient cooling water, the pressure in the condenser increases and subsequently, reduces
the vacuum efficiency of the condenser.

8.13  ❐  CONDENSER EFFICIENCY


It is defined as the ratio of the difference between the outlet and inlet temperatures of the cooling water
to the difference between the temperature ts corresponding to the vacuum in the condenser and the
inlet temperature of cooling water.
t −t
Condenser efficiency, hc = ow iw (8.13)
t s − tiw
where tiw , tow = temperature at inlet and outlet respectively of cooling water, ts = temperature of steam
corresponding to the actual absolute pressure in the condenser.

8.14  ❐  COOLING TOWER


There is a need to recycle the cooling water through the condenser when cooling water supply is lim-
ited. Cooling tower is an artificial device used to cool hot cooling water coming out from the condenser.
There are different types of cooling towers. The induced draft type cooling towers are commonly
used in high capacity power plants. The schematic diagram of an induced draft cooling tower is shown
in Fig. 8.12. The hot water coming out from the condenser is sprayed at the top of the tower and air is
induced to flow through the tower with the help of induced draft fans mounted at the top of the tower.

M08_THERMAL ENGINEERING_SE_XXXX_CH08.indd 399 6/15/2018 3:51:56 PM


400 Chapter 8

Air out

Induced draft
fan
Eliminator
Hot water
from Spray nozzle
condenser
Air in Air in

Cold water to Air louvers Float valve


Air flow
condenser
Make-up
water
Water
pump

Figure 8.12  Induced draft cooling tower

The amount of water supply lost in such a tower ranges from 1% - 2% by evaporation and 0.5% - 2%
by drift losses. To compensate these losses, make-up water is supplied from external sources. The
cooled water is collected at the bottom.

Example 8.3
The following readings were taken during a test on a surface condenser:
Vacuum in condenser = 71 cm of Hg
Barometer reading = 76 cm of Hg
Temperature in condenser = 33°C
Hot well temperature = 30°C
Cooling water circulated = 48,000 kg/h
Inlet temperature of cooling water = 15°C
Outlet temperature of cooling water = 28°C
Condensate = 1500 kg/h
Calculate:  (a)  the mass of air in kg/m3 of condensate volume,
(b)  dryness fraction of steam entering the condenser, and
(c)  the vacuum efficiency
Solution
(a) Absolute pressure in condenser,
pt = (76 - 71) × 0.01333 = 0.0667 bar
At t = 33°C, ps = 0.05106 bar from steam tables = 3.83 cm of Hg
Partial pressure of air, pa = pt − ps= 0.0667 - 0.05106 = 0.01564 bar
pa V 0.01564 × 105 × 1
Mass of air, ma = = = 0.0178 kg/m3
RT 287 × ( 273 + 33)
mw h f + h fg − hc )
(b) =
ms c pw (tow − tiw )

M08_THERMAL ENGINEERING_SE_XXXX_CH08.indd 400 6/15/2018 3:51:58 PM


Steam Condensers 401

At p = 0.0667 bar, hf = 159.47 kJ/kg, hfg = 2411.4 kJ/kg


48, 000 159.47 + x × 2411.4 − 4.1868 × 30
=
1500 4.1868 ( 28 − 15)
x = 0.708
pb − pt 7 6−5
(c) Vacuum efficiency = = = 0.9837 or 98.37%
pb − ps 76 − 3.772

Example 8.4
A steam turbine discharges 6000 kg of steam per hour at 45°C and 0.82 dryness. The estimated air
leakage is 15 kg/h. The temperature at the suction of air pump is 30°C and temperature of conden-
sate is 33°C. Find (a) the vacuum gauge reading, (b) capacity of air pump, (c) loss of condensate
per hour, and (d) the quantity of cooling water required by limiting its temperature rise by 10°C.

Solution
(a) Total pressure in the condenser, pt = pa + ps
At 45°C, ps = 0.095934 bar from steam tables
vg = 15.258 m3/kg
Volume of 6000 kg steam, V = 6000 × xvg = 6000 × 0.82 × 15.258 = 75069.36 m3/h
= volume of air
ma RT 15 × 287 × ( 45 + 273)
Partial pressure of air, pa = = = 18.236 × 10-5 bar
V 5
75069.36 × 10
pt = 18.236× 10-5 + 0.09582 = 0.096 bar
1.01325 − 0.096
Vacuum in condenser = = 68.8 cm of Hg
0.0133322
(b) Partial pressure of steam at air pump suction at 30°C = 0.04242 bar
pa = pt − ps = 0.096 - 0.042461= 0.0535 bar
Volume of air at 30°C and 0.0535 bar,
ma RT 15 × 287 × (30 + 273)
V= = = 243.82 m3
pa 0.0535 × 105
Air pump capacity = 243.82 m3/h
(c) Specific volume of steam at 30°C, vg = 32.893 m3/kg
V 243.82
Loss in condensate = = = 7.41 kg/h
v g 32.893
(d) At 45°C, hf = 188.42 kJ/kg, hfg = 2394.8 kJ/kg
ms ( h f + xh fg − hc ) 6000(188.42 + 0.82 × 2394.8 − 4.1868 × 33)
Mass of cooling water, mw = =
c pw (tow − tiw ) 4.1868 × 10 × 103
= 288.62 tonnes/h

M08_THERMAL ENGINEERING_SE_XXXX_CH08.indd 401 6/15/2018 3:52:02 PM


402 Chapter 8

Example 8.5
The air leakage into a surface condenser operating with a steam turbine is estimated as 80 kg/h.
The vacuum near the inlet of air pump is 70 cm of Hg when barometer reads 76 cm of Hg. The
temperature at the inlet of vacuum pump is 20°C. Calculate (a) the minimum capacity of the air
pump, (b) dimensions of the reciprocating air pump to remove air if it runs at 200 rpm. Take L:D::
3:2 and volume efficiency = 98%, (c) mass of vapours extracted per hour.

Solution
(a) Total pressure at inlet of air pump at 1–1, (Fig. 8.13)
 76 − 70 
pt =  × 1.01325 = 0.08 bar
 76 
Partial pressure of water vapour at 1–1 corresponding to saturation temperature of 20°C, from
steam tables,
ps = 0.023385 bar
Partial pressure of air at 1–1,
pa = pt − ps = 0.08 - 0.023385 = 0.0566 bar
Volume of 80 kg/h of air at 20°C and 0.0566 bar,
m RT 80 × 287 × 293
V= a = = 1188.6 m3/h
pa 0.0566 × 105
Capacity of air pump = 1188.6 m3/h
p
(b) Now V = D2LNhv
4
1188.6 p
= × D2 × 1.5 D × 200 × 0.98
60 4
D3 = 0.0858
D = 0.441 m or 46.1 cm
L = 66.2 cm
Exit from
steam turbine

Surface To air pump


Condenser

Condensate

Figure 8.13  Surface condenser

M08_THERMAL ENGINEERING_SE_XXXX_CH08.indd 402 6/15/2018 3:52:05 PM


Steam Condensers 403

(c) Mass of water vapour going with air in the air pump,

m s = V
vg
At 20°C, vg = 57.79 m3/kg

m s = 1188.6 = 20.57 kg/h


57.79
Example 8.6
The quality of steam entering a jet condenser is 0.92. It is condensed by using cooling water at
15°C. The mass of air in the condenser is 35% of the mixture. Assuming that only latent heat of
steam is absorbed by the cooling water, calculate (a) the temperature of mixture of condensate and
water leaving the condenser, and (b) the mass of water required per kg of steam condensed.
Take for air, R = 287 J/kg.K and for steam, R = 463 J/kg.K. Condenser vacuum = 61 cm of Hg,
Barometer reading = 76 cm of Hg.

Solution
(a) Given that pa = 0.35 pt
pt = (76 - 61) × 0.0133 = 0.2 bar
pa = 0.35 × 0.2 = 0.07 bar
ps = pt - pa = 0.2 - 0.07 = 0.13 bar
Saturation temperature of steam at 0.13 bar, from steam tables,
ts = 50.71°C
As only latent heat of steam is to be absorbed, therefore, outlet temperature of cooling water,
tow = ts = 50.71°C
Since steam and cooling water mix together in the jet condenser,
Mixture temperature coming out of condenser = 50.71°C
mw h f + xh fg − c pw × tow
(b) Now =
ms c pw (tow − tiw )
At ps = 0.13 bar, hf  = 212.27 kJ/kg, hfg = 2380.98 kJ/kg from steam tables
mw 212.27 + 0.92 × 2380.98 − 4.1868 × 50.71
= = 14.65
ms 4.1868(50.71 − 15)
Minimum quantity of cooling water required per kg of steam condensed
= 14.65 kg

Example 8.7
In a trial on a surface condenser, the vacuum gauge reading was 660 mm of Hg when barometer
read 760 mm of Hg. The temperature in the condenser was 45°C.
(a) Determine the alteration in vacuum if the quantity of air entering the condenser is reduced by
100 kg/h.
(b) If the mass of condensate is 960 kg/h, calculate the quantities of air and vapour which the
pump has to handle.

M08_THERMAL ENGINEERING_SE_XXXX_CH08.indd 403 6/15/2018 3:52:07 PM


404 Chapter 8

Solution
Absolute condenser pressure = 760 - 660 = 100 mm of Hg
100
pt = × 1.01325 = 0.1333 bar
760
At 45°C, from steam tables,
Partial pressure of steam, ps = 0.095934 bar
Specific volume of steam, vsg = 15.258 m3/kg
Partial pressure of air, pa = 0.1333 - 0.095934 = 0.037366 bar
Volume of steam per hour, Vs = 960 × 15.258 = 14647.68 m3
= Volume of air, Va
pa Va 0.037366 × 105 × 14647.68
Mass of air, m a = = = 599.7 kg/h
RT 287 × ( 273 + 45)
If the mass of air is reduced to 100 kg/h, then partial pressure of air,
100
pa′ = ×0.037366 = 0.00623 bar
599.7
New absolute pressure of condenser, pt′ = 0.095934 + 0.00623 = 0.102 bar
0.102 × 760
= = 76.52 mm of Hg
1.01325
New vacuum reading = 760 - 76.52 = 683.48 mm Hg
Alteration in vacuum = 683.48 - 660 = 23.48 mm Hg
Capacity of pump = 960 + 100 = 1060 kg/h

Example 8.8
The following observations were recorded during a condenser test;
Vacuum reading = 700 mm of Hg; Barometer reading = 760 mm of Hg;
Condensate temperature = 34°C
Find:  (a) partial pressure of air and (b) mass of air per m3 of condenser volume.
Take R for air = 287 J/kg K

Solution
Given that Vacuum reading = 700 mm of Hg: Barometer reading = 760 mm of Hg; T = 34°C = 34 +
273 = 307 K
(a) Pressure in the condenser
pc = Barometer reading - Vacuum reading
= 760 - 700 = 60 mm of Hg
= 60 × 0.00133 = 0.0798 bar (∵ 1 mm of Hg = 0.001 33 bar)
From steam tables, corresponding to a temperature of 34°C, we find that pressure of steam,
ps = 0.0535 bar
\ Partial pressure of air,
pa = pc - ps = 0.0798 - 0.0535 = 0.0263 bar

M08_THERMAL ENGINEERING_SE_XXXX_CH08.indd 404 6/15/2018 3:52:10 PM


Steam Condensers 405

(b) Mass of air per m3 of condenser volume.


paV 0.0263 × 105 × 1
ma = = = 0.03 kg
RT 287 × 307

Example 8.9
A vacuum gauge fitted to a condenser reads 680 mm of Hg, when the barometer reads 750 mm of Hg.
Determine the corrected vacuum in terms of mm of Hg and bar.

Solution
Given that Vacuum gauge reading = 680 mm of Hg; Barometer reading = 750 mm of Hg:
Pressure in the condenser = 750 - 680 = 70 mm of Hg
Corrected vacuum = 760 - 70 = 690 mm of Hg
= 690 × 0.00133 = 0.918 bar

Example 8.10
Calculate the vacuum efficiency from the following data:
Vacuum at steam inlet to condenser = 700 mm of Hg
Barometer reading = 760 mm of Hg
Hot well temperature = 30°C

Solution
Given that Vacuum reading or actual vacuum = 700 mm of Hg; Barometer reading = 760 mm of Hg;
t = 30°C
Pressure in the condenser = 760 - 700 = 60 mm of Hg
From steam tables, corresponding to a temperature of 30°C, we find that ideal pressure of steam,
0.042461
0.042461 bar =
0.00133
= 31.93 mm of Hg [∵ 1 mm of Hg = 0.00133 bar ]

Ideal vacuum = Barometer reading - Ideal pressure


= 760 - 31.93 = 728.07 mm of Hg
Actual vacuum 700
Vacuum efficiency, h vac = = = 0.9614 or 96.14%
Ideal vacuum 728.07

Example 8.11
The vacuum efficiency of a condenser is 96%. The temperature of condensate is 40°C. If the barom-
eter reads 752 mm of Hg, find the vacuum gauge reading of the condenser.
Solution
Given that hv = 96% = 0.96%; t = 40°C;
0.073837
We find that ideal pressure of steam, = 0.073837 bar = = 55.52 mm of Hg
0.00133
\ Ideal vacuum = Barometer reading - Ideal pressure
= 752 - 55.52 = 696.48 mm of Hg

M08_THERMAL ENGINEERING_SE_XXXX_CH08.indd 405 6/15/2018 3:52:12 PM


406 Chapter 8

Actual vacuum
Vacuum efficiency,   hvac =
Ideal vacuum
Actual vacuum
0.96 =
696.48
\ Actual vacuum or vacuum gauge reading of the condenser
= 0.96 × 696.48 = 668.62 mm of Hg

Example 8.12
In a surface condenser, the vacuum maintained is 700 mm of Hg. The barometer reads 754 mm. If
the temperature of condensate is 18°C, determine: (a) mass of air per kg of steam and (b) vacuum
efficiency.

Solution
Given that Actual vacuum = 700 mm of Hg; Barometer reading = 754 mm of Hg; T = 18°C
= 18 + 273 = 291 K
Pressure in the condenser, pc = 754 - 700 = 54 mm of Hg
From steam tables, corresponding to 18°C, we find that absolute or ideal pressure of steam,
0.02085
ps = 0.02085 bar = = 15.68 mm of Hg
0.00133
Specific volume of steam, vs = 65.844 m3/kg
(a) Pressure of air (as per Dalton’s law)
pa = pc - ps = 54 - 15.68 = 38.32 mm of Hg
= 38.32 × 0.00133 = 0.051 bar = 0.051 × 105 N/m2
Mass of air per kg of steam,
paV 0.051 × 105 × 65.844
ma = = = 4.02 kg (∵ pV = mRT)
RT 287 × 291
(b) Ideal vacuum = Barometer reading - Ideal pressure
= 754 - 15.68 = 738.32 mm of Hg
Actual vacuum 700
Vacuum efficiency hvac = = = 0.9481 or 94.81%
Ideal vacuum 738.32

Example 8.13
A surface condenser, fitted with separate air and water extraction pumps, has a portion of the tubes
near the air pump suction screened off from the steam so that the air is cooled below the conden-
sate temperature. The steam enters the condenser at 38°C and the condensate is removed at 37°C.
The air removed has a temperature of 36°C. (a) If the total air infiltration from all sources together
is 5 kg/h, determine the volume of air handled by the air pump per hour. (b) What would be the
corresponding value of the air handled if a combined air and condensate pump was employed?
Assume uniform pressure in the condenser.

M08_THERMAL ENGINEERING_SE_XXXX_CH08.indd 406 6/15/2018 3:52:14 PM


Steam Condensers 407

Solution
Given that Ts = 38°C = 38 + 273 = 311 K; Tc = 37°C = 37 + 273 = 310 K; Ta = 36°C = 36 + 273
= 309 K; ṁa = 5 kg/h
(a)  Since the pressure at entry to the condenser (pc) is equal to the pressure of steam corresponding
to 38°C, therefore, from the steam tables,
pc = 0.0668 bar
Pressure of steam at the air pump suction, corresponding to 36°C (from steam tables),
ps = 0.0598 bar
\ Pressure of air at the air pump suction (as per Dalton’s law),
pa = pc - ps = 0.0668 - 0.0598 = 0.007 bar
= 0.007 × 105 = 700 N/m2
Volume of air handled by the air pump,
m RT 5 × 287 × 309
Va = a a = = 633.45 m3/h
pa 700
(b)  F
 rom steam tables corresponding to a condensate temperature of 37°C, we find that pressure of
steam,
ps = 0.0633 bar
\ Pressure of air (as per Dalton’s law),
pa = pc - ps = 0.0668 - 0.0633 = 0.0035 bar
= 0.0035 × 105 = 350 N/m2
Volume of air handled,
m RT 5 × 287 × 310
V˙ a = a c = = 127 m3/h
pa 350

Example 8.14
The air leakage into a surface condenser operating with a steam turbine is estimated as 84 kg/h.
The vacuum near the inlet of air pump is 700 mm of Hg when barometer reads 760 mm of Hg.
The temperature at inlet of vacuum pump is 20°C. Calculate the following:
(a) The minimum capacity of the air pump in m3/h;
(b) The dimensions of reciprocating air pump to remove the air if it runs at 200 rpm. Take L/D
ratio = 1.5 and volumetric efficiency = 100%
(c) The mass of vapour extracted per minute

Solution
Given that ṁa = 84 kg/h; Vacuum = 700 mm of Hg; Barometer reading = 760 mm of Hg; T = 20°C =
20 + 273 = 293 K
(a) Pressure in the condenser,
pc = Barometer reading - Condenser vacuum

M08_THERMAL ENGINEERING_SE_XXXX_CH08.indd 407 6/15/2018 3:52:15 PM


408 Chapter 8

= 760 - 700 = 60 mm of Hg
= 60 × 0.00133 = 0.0798 bar
From steam tables, corresponding to a temperature of 20°C, we find that pressure of steam,
ps = 0.023385 bar
\ Pressure of air (as per Dalton’s law),
pa = pc - ps = 0.0798 - 0.023385 = 0.056415 bar
= 0.056415 × 105 = 5641.5 N/m2
Minimum capacity of the air pump,
m RT 84 × 287 × 293
V˙ a = a = = 1252.1 m3/h
pa 5641.5

(b) Let  D = Diameter of the cylinder in metre,


  L = Length of the stroke in metres = 1.5 D
hv = Volumetric efficiency = 100% = 1
N = Speed of the pump = 200 r.p.m.
Minimum capacity of the air pump (Va),
1252.1 p 2 p
= D ×L×N= × D2 × 1.5 D × 200 = 235.6 D3(Va is taken in m3/min)
60 4 4
\ D3 = 0.0886 or D = 0.446 m
and L = 1.5 D = 1.5 × 0.446 = 0.669 m
(c) From steam tables, corresponding to a temperature of 20°C, we find that specific volume of steam,

vg = 57.79 m3/kg
\ Mass of vapour extracted per minute
v 1252.1
= a = = 0.361 kg/min
v g 60 × 57.79

Example 8.15
The vacuum at the extraction pipe in a condenser is 710 mm of mercury and the temperature is
35°C. The barometer reads 760 mm of mercury. The air leakage into the condenser is 4 kg per
10,000 kg of steam. Determine: (a) the volume of air to be dealt with by the dry air pump per kg
of steam entering the condenser, and (b) the mass of water vapour associated with this air. Take
R = 287 J/kg K for air.

Solution
Given that vacuum = 710 mm of Hg; T = 35°C = 35 + 273 = 308 K; Barometer reading = 760 mm of Hg;
ma = 4 kg per 10000 kg of steam = 0.0004 kg/kg of steam.
(a) Let Va = Volume of air per kg of steam entering the condenser.
Pressure in the condenser
pc = Barometer reading - Condenser vacuum

M08_THERMAL ENGINEERING_SE_XXXX_CH08.indd 408 6/15/2018 3:52:16 PM


Steam Condensers 409

= 760 - 710 = 50 mm of Hg = 50 × 0.001 33


= 0.0065 bar
From steam tables, corresponding to the temperature of 35°C, we find that the pressure of steam,
ps = 0.05628 bar
\ Pressure of air (as per Dalton’s law),
pa = pc - ps = 0.0665 - 0.05628 = 0.01022 bar
= 0.01022 × 105 = 1022 N/m2
Now paVa = ma RT
ma RT 0.0004 × 287 × 308
\ Va = =
pa 1022
   = 0.0346 m3/kg of steam
(b) From steam tables, corresponding to a temperature of 35°C, we find that specific volume of steam,
vg = 25.216 m3/kg
\ Mass of water vapour associated with the air
Va 0.0346
= = 0.00137 kg
v g 25.216

Example 8.16
The amount of steam condensed in a condenser is 6800 kg/h and the air leakage into the condenser
is 12 kg/h. The air pump suction is screened off. The exhaust steam temperature is 32°C, the con-
densate temperature is 30°C and the temperature at the air pump suction is 25°C. Determine the
following:
(a) The mass condensed in the air cooler section per hour
(b) The volume of air in m3 handled by air pump
(c) The percentage reduction in air pump capacity due to cooling of the air.

Solution
The saturation pressure corresponding to steam temperature 32°C from steam table is
pa = 0.04799 bar
Specific volume corresponding to 32°C, vs = 29.82 m3/kg
\ Volume of steam handled, Vs = 6800 × 29.82 = 202776 m3/h
According to Dalton’s law, the volume of air handled per hour is equal to the volume of steam handled
per hour. Hence the partial pressure of air,
m a RT 12 × 287 × ( 273 + 32)
pa = = = 0.0000518 bar
5
Va × 10 202776 × 105
We observe that the value of partial pressure of air is very small and hence the total condenser pressure
may be taken as due to steam alone i.e. 0.04799 bar.

M08_THERMAL ENGINEERING_SE_XXXX_CH08.indd 409 6/15/2018 3:52:17 PM


410 Chapter 8

Since the temperature at the air pump suction is 25°C, hence the steam pressure at the air pump
suction corresponding to 25°C from steam tables,
ps = 0.031691 bar
Specific volume corresponding to 25°C, vs = 43.359 m3/kg
\ Partial pressure of air handled,
pa = 0.04799 - 0.031691 = 0.0163 bar
\ Volume of air handled per hour by the pump,
m a RT 12 × 287 × ( 273 + 25)
V˙ a = = = 629.64 m3/h
5 5
pa × 10 0.0163 × 10
According to Dalton’s law, the volume of steam will also be equal to 629.64 m3/h.
629.64
\    The mass of the steam with air = =14.52 kg/h
43.359
The temperature may be assumed to be that of the condensate, i.e., ts = 30°C at the point where the
saturated steam enters the air cooling section. Corresponding to 30°C, the pressure is:
ps = 0.042461 bar
Specific volume, vs = 32.893 m3/h
\ Partial pressure of air is given by,
pa = (0.04799 - 0.042461) = 0.00553 bar
Hence, the volume of air handled
m R.T 12 × 287 × ( 273 + 30)
Va = a = = 1887.04 m3 /h
5
pa × 10 0.00553 × 105
According to Dalton’s law, this volume of air is equal to the volume of steam hence the mass of steam
associated with the air,
m s = 1887.04 = 57.37 kg/h
32.893
\ Mass of steam condensed in the cooler section
= 57.37 - 14.52 = 42.85 kg/h
Hence, the percentage reduction in volume handled by the air pump due to cooling section is:
1887.04 − 629.64
= × 100 = 66.63%
1887.04

Example 8.17
In a condenser test, the following observations were made
Vacuum = 70 cm of Hg
Barometer = 76 cm of Hg
Mean temperature of condensation = 35°C
Hot well temperature = 29°C
Mass of cooling water = 45500 kg/h

M08_THERMAL ENGINEERING_SE_XXXX_CH08.indd 410 6/15/2018 3:52:19 PM


Steam Condensers 411

Inlet water temperature = 16.5°C


Outlet water temperature = 31°C
Mass of condensate = 1200 kg/h
Find (a) the mass of air present per unit condenser volume, (b) the state of steam entering the con-
denser, (c) the vacuum efficiency, (d) the condensate undercooling and (e) condenser efficiency.

Solution
(a) Corresponding to 35°C, the partial pressure of steam from steam table = 0.05628 bar
1.013
Absolute pressure of mixture in condenser = (76 - 70) × = 0.07997 bar
76
\ Partial pressure of air = 0.07997 - 0.05628 = 0.02369 bar
pa × 105 × V 0.02369 × 105 × 1
Mass of air = = = 0.0268 kg/m3
RTa 287 × ( 273 + 35)
(b) Heat absorbed by circulating water = 45500 × 4.18 (31 - 16.5) = 2727755 kJ/h
Heat rejected by steam in condensing at 35°C and in being under cooled from 35°C to 29°C
= mc (hw1 + xL1 - hwc)
= 1200 (146.66 + 2418.6x - 121.59)
= 1200 (25.07 + 2418.6x)
Now, Heat absorbed by circulating water = Heat rejected by steam
2757755 − 25.07 × 1200
x = = 0.9295
1200 × 2418.6
Actual vacuum 70
(c) Vacuum efficiency = = × 100 = 97.52%
Ideal vacuum 76 − 0.05628 × ( 76 / 1.013)
(d) Condensate undercooling = 35 - 29 = 6°C
Actual temp. rise of water 31 − 16.5
(e) Condenser efficiency = = × 100 = 78.37%
Max. permissible temp. rise 35 − 16.5

Example 8.18
In a surface condenser 2500 kg/h of steam is condensed and the air leakage is 2 kg/h. A separate
air and water extraction pumps are fitted which draws the air in air cooler and a portion of the tube
near the air pump suction is screened off from the steam so that the air is cooled below the con-
densate temperature. The steam enters the condenser dry and saturated at 38°C and the condensate
is extracted at the lowest point of the condenser at a temperature of 37°C (i.e., the temperature at
the entrance of air cooler is 3°C). The temperature at the air pump suction is 31°C. Assuming a
constant vacuum throughout the condenser, find the following:
(a) The mass of steam condensed/min in the air cooler
(b) The volume of air to be dealt with per minute by the dry pump
(c) The reduction in the necessary air pump capacity following the cooling of air.

M08_THERMAL ENGINEERING_SE_XXXX_CH08.indd 411 6/15/2018 3:52:22 PM


412 Chapter 8

Solution
Partial pressure of steam at 38°C, ps = 0.0668 bar
Specific volume of steam at 38°C, vs = 21.8 m3/kg
21.8 × 2500
\ Volume of steam, V˙ s = = 908.33 m3/min
60
This must be the volume of 2 kg/h of air when exerting its partial pressure, i.e., Va = Vs = 908.33 m3/min
m a RaT 2 × 287 × 311
Partial pressure of air, pa = = = 0.000033 bar
Va 908.33 × 60 × 105
Hence, total pressure in the condenser = ps + pa = 0.0668 + 0.000033 = 0.06683 bar
At condensate extraction:
Partial pressure of steam at 37°C = 0.0633 bar
Specific volume of steam at 37°C, vs = 22.94 m3/kg
\ Partial pressure of air = 0.06683 - 0.0633 bar = 0.00353 bar
2 287( 273 + 37)
\ Volume of air, V˙ a = × = 8.473 m3/min = Vs
60 0.00353 × 105
\ The mass of the steam associated with air,
V 8.473
m s = s = = 0.369 kg/min
vs 22.94
At air pump suction:
Partial pressure of steam of 31°C = 0.04522 bar
\ Partial pressure of air = 0.06683 - 0.04522 = 0.02161 bar
2 287 × ( 273 + 31)
(a) \ Volume of air removed, V˙ a = × = 1.346 m3/min
60 0.02161 × 105
Mass of steam associated with this air and which is partially condensed in the air cooler
1.346
= = 0.0429 kg/min
31.36
(b) Mass of steam condensed in the air cooler or saving in condensate by using the separate extraction
method
= 0.369 - 0.0429 = 0.3261 kg/min
(c) Air capacity without air cooler = 8.473 m3/min
Air pump capacity with air cooler = 1.346 m3/min
\ Percentage reduction in pump capacity
8.473 − 1.346
= × 100 = 84.11%
8.473
Example 8.19
A barometric jet condenser maintains a vacuum of 64.2 cm of Hg when the barometer reads 76 cm
of Hg. The condenser handles 4600 kg of steam/h and 0.97 dryness fraction. The inlet temperature
of the cooling water is 15°C. The mixture of condensate and cooling water leaves at 43°C. Calculate
(a) the minimum height of the tail pipe above the level of the hot well and (b) the amount of cooling
water required. Assume no undercooling.

M08_THERMAL ENGINEERING_SE_XXXX_CH08.indd 412 6/15/2018 3:52:24 PM


Steam Condensers 413

Solution
(a) Absolute condenser pressure of steam = (76 - 64.2) × 0.0133 = 1.6048 m of water
13.6
Barometer height = 76 × = 10.336 m of water
100
Hence the length of the tail pipe = 10.336 - 1.6048 = 8.7312 m
(b) The saturation temperature of steam entering the condenser will be 43°C in the absence of any
undercooling of the condensate. Corresponding to 43°C,
hwc = hw = 180.079 kJ/kg and L = hfg = 2399.56 kJ/kg
mc ( hw + xL − hwc ) 4600(180.07 + 0.97 × 2399.56 − 180.07)
mw = = = 91480.15 kg/h
c pw (tw 2 − tw1 ) 4.18( 43 − 15)

Example 8.20
A condenser is equipped in a steam turbine which handles 14500 kg of steam per hour and develops
2484.3 kW. The initial conditions of steam entering to turbine are 27 bar and 350°C. The exhaust from
the turbine is condensed in the condenser and the vacuum maintained is 72.5 cm of Hg while barom-
eter reads 75.8 cm of Hg. The temperature of the circulating water is increased from 8 to 28°C while
the condensate is removed from the condenser at a temperature of 29°C. Calculate the following:
(a)  Dryness fraction of steam entering the condenser
(b)  Mass of circulating water per hour and the cooling ratio
(c)  Minimum quantity of cooling water required per kg of steam.
Solution
(a) Absolute condenser pressure = (75.8 - 72.5) × 1.013/76 = 0.04398 bar
From the steam tables, corresponding to 0.043398 bar, hf  = 327.19 kJ/kg and hfg = 2313.46 kJ/kg
Corresponding to 27 bar and 350°C, h = 3124.84 kJ/kg
If x be the dryness fraction of steam entering the condenser, then drop of enthalpy
14500
= [3121.84 - (327.19 + 2313.4x)] × = 2484.3 kJ/kg
3600
\ x = 0.9414
(b) Heat given by steam = Heat received by water
14500[(121.48 + 0.9414 × 2433.1) - 117.31] = mw × 4.18 (28 - 8)
Mass of cooling water = mw = 398002.5 kg/h
398002.5
\ Cooling ratio = = 27.448
14500
(c) The maximum possible temperature of cooling water at outlet will be 29.4°C corresponding to
condenser pressure of 0.04398 bar.
Hence maximum rise in temperature of cooling water = 29.4 - 8 = 21.4°C
If under this condition mw1 be the quantity of cooling water required per kg of steam, by the heat
balance equation, then
[(123.22 + 0.942 × 2432.4 - 121.48)] mw1 × 4.18 × 21.4
mw1 = 25.634 kg of water/kg of steam

M08_THERMAL ENGINEERING_SE_XXXX_CH08.indd 413 6/15/2018 3:52:26 PM


414 Chapter 8

Example 8.21
The following observations were recorded during a trial on a steam condenser:
Condenser vacuum recorded = 70 cm (0.93325 bar)
Barometer reading = 76.5 cm Hg (1.02 bar)
Mean condenser temperature = 35°C
Hot well temperature = 28°C
Condensate formed/hr. = 1800 kg
Circulating cooling water inlet temperature = 15°C
Circulating cooling water outlet temperature = 27°C
Quantity of cooling water/h = 80000 kg
Determine the following:
(a) Vacuum corrected to standard barometer of 76 cm Hg (1.01325 bar)
(b) Vacuum efficiency
(c) Under cooling of condensate
(d) Condenser efficiency
(e) State of steam entering condenser
(f) Mass of air per cubic metre of condenser volume.
(g) Mass of air present per kg of uncondensed steam
Assume R for air as 287 kJ/kgK.

Solution
(a) Vacuum corrected to standard barometer
= Standard barometric pressure - (Barometric pressure - Gauge pressure)
= 1.01325 bar - (1.02 bar - 0.93335 bar)
= 1.01325 - 0.8665 = 0.9266 bar
(b) From steam table, the pressure corresponding to condenser temperature of 35°C = 0.0562 bar
0.93325 0.93325
\ hvac = = = 0.9685 or 96.85%
1.02 − 0.0562 0.9636
(c) Undercooling of condensate = 35° - 28° = 7°C
(d) Absolute condenser pressure = Barometer pressure - Vacuum reading
= 1.02 - 0.93325 = 0.08675 bar
Saturation temperature corresponding to pressure of 0.08665 bar from steam tables is 42.9°C
Thus, cooling water temperature can get raised to 42.9°C
Actual cooling water temperature rise 27 − 15 12
hc = = = = 0.43 or 43%
Maximum possible temperatuure rise 42.9 − 15 27.9
(e) From steam tables for pressure of 0.08665 bar
hf = 179.8 kJ/kg, hfg = 2399.6 kJ/kg.
Enthalpy of condensate corresponding to hot well temperature of 28°C is 117 kJ/kg.
Heat absorbed by cooling water = Heat given by steam
mcpw (t2 - t1) = ms (hf + xhfg - hc)

M08_THERMAL ENGINEERING_SE_XXXX_CH08.indd 414 6/15/2018 3:52:27 PM


Steam Condensers 415

80000 × 4.1868 (27 - 15) = 1800 (179.8 + x × 2399.6 - 117)


80000 × 4.1868 × 12
i.e., = 62.8 + x × 2399.6
1800
or 2232.96 = 62.8 + x × 2399.6
2232.96 − 62.8
Thus x = = 0.9044 or 90.44%.
2399.6
(f) At condenser temperature of 35°C, the partial pressure of steam,
ps = 0.0562 bar.
0.03766 × 105 × 43.40
ma =
Hence,  = 1.911 kg per kg of vapour
287 × 298
1
mv =
or = 0.523 kg of vapour per kg of air.
1.911
With external cooling from 25°C to 20°C we have, from steam tables, at 20°C
ps = 0.023366 bar = 1.7526 cm Hg, vg = 57.834 m3
\ pa = 5.2 - 1.7526 = 3.4474 cm Hg
p V
(g) ma = a a
RT
1.01325
where pa = 3.4474 × = 0.04596 bar
76
3
Va = vg = 57.834 m per kg of vapour
R = 287 J/kg ×K; T = (273 + 20) = 293 K
0.04596 × 57.834
Hence, ma = = 3.161 kg per kg of vapour
287 × 293
1
mv = = 0.316 kg of vapour per kg of air
3.161
These calculations show that by cooling the mixture form 25°C to 20°C, vapour accompanying each
kg of air withdrawn from the condenser is reduced from 0.523 kg to 0.316 kg. This will have a signif-
icant effect both on the ejector capacity and work done for air pump suction.

Example 8.22
In a condenser test, the following observations were made:
Vacuum = 690 mm of Hg; Barometer reading = 750 mm of Hg; Mean temperature of
condensation = 35°C; Hot well temperature = 28°C; Mass of cooling water = 50,000 kg/h; Inlet
temperature = 17°C; Outlet temperature = 30°C; Mass of condensate per hour = 1250 kg.
Find: (a) the mass of air present per m3 of condenser volume, (b) the state of steam entering the
condenser and (c) the vacuum efficiency.
Take R for air = 287 J/kg K.

Solution
Given that vacuum = 690 mm of Hg; Barometer reading = 750 mm of Hg; tc = 35°C; th = 28°C;
m w = 50000 kg/h; tf = 17°C; to = 30°C; m s = 1250 kg/h; R= 287 J/kg K

M08_THERMAL ENGINEERING_SE_XXXX_CH08.indd 415 6/15/2018 3:52:30 PM


416 Chapter 8

(a) We know that pressure in the condenser,


pc = 750 - 690 = 60 mm of Hg = 60 × 0.00133= 0.08 bar
From steam tables, corresponding to a condensation temperature of 35°C, we find that the pres-
sure of steam,
ps = 0.05628 bar
\ Pressure of air (as per Dalton’s law),
pa = pc - ps = 0.08 - 0.05628 = 0.02372 bar = 0.02372 × 105 = 2372 N/m2
Mass of air per m3 of condenser volume,
pV 237280 × 1
ma = a = = 0.0268 kg
RT 287 × (35 + 273)
(b) Let  x = Dryness fraction (i.e., state) of steam entering the condenser.
From steam tables, corresponding to a pressure of 0.05628 bar (or 35°C), we find that,
hf = 146.66 kJ/kg and hfg = 2418.6 kJ/kg
Corresponding to a hot well temperature of 28°C,
hf 1 = 117.3 kJ/kg
Total heat of entering steam,
h = hf + xhfg = 146.66 + x × 2418.6 kJ/kg
Mass of cooling water (mw).
m s ( h − h f 1 ) 1250[146.66 + x × 2418.6 − 117.3]
500000 = = = 22.894 (29.36 + x × 2418.6)
c pw (to − t1 ) 4.2(30 − 17)
or 29.36 + x × 2418.6 = 50 000/22.894 = 2184
2184 − 29.36
\ x= = 0.89
2418.6
(c) Corresponding to a condensation temperature of 35°C,
0.05628
Ideal pressure of steam, = 0.05628 bar = = 42.32 mm of Hg
0.00133
\ Ideal vacuum = Barometer reading - Ideal pressure
= 750 - 42.32 = 707.68 mm of Hg
Actual vacuum 690
Vacuum efficiency, hvac = = = 0.975 or 97.5%
Ideal vacuum 707.68

Example 8.23
The following observations were recorded during a test on a steam condenser:
Barometer reading = 765 mm of Hg
Condenser vacuum = 710 mm of Hg
Mean condenser temperature = 35°C
Condensate temperature = 28°C

M08_THERMAL ENGINEERING_SE_XXXX_CH08.indd 416 6/15/2018 3:52:31 PM


Steam Condensers 417

Condensate collected per hour = 2 tonnes


Quantity of cooling water per hour = 60 tonnes
Temperature of cooling water at inlet = 10°C
Temperature of cooling water at outlet = 25°C
Find: (a) vacuum corrected to the standard barometer reading, (b) vacuum efficiency of the con-
denser, (c) undercooling of the condensate, (d) condenser efficiency, (e) quality of the steam
entering of condenser, (f) mass of air per m3 of condenser volume and (g) mass of air per kg of
uncondensed steam.
Solution
Given that Barometer reading = 765 mm of Hg; Condenser vacuum = 710 mm of Hg; T = 35°C
= 35 + 273 = 308 K; tc = 28°C; m s = 2 t/h = 2000 kg/h; m w = 60 t/h = 60000 kg/h; ti = 10°C; to = 25°C
(a) Absolute pressure in the condenser = Barometer reading - Condenser vacuum
= 765 - 710 = 55 mm of Hg
Vacuum corrected to the standard barometer reading (assuming 760 mm of Hg)
= 760 - 55 = 705 mm of Hg
(b) From steam tables, corresponding to the mean condenser temperature of 35°C, we find that ideal
pressure of steam,
0.05628
ps = 0.05628 bar = = 42.32 mm of Hg
0.00133
Ideal vacuum = Barometer pressure - Ideal pressure
  = 765 - 42.32 = 722.68 mm of Hg
Actual vacuum 710
Vacuum efficiency, hvac = = = 0.9824 or 98.24%
Ideal vacuum 722.68
(c) Undercooling of the condensate = Mean condenser temperature - Condensate temperature
= 35 - 28 = 7°C
(d) Pressure in the condenser, pc = 765 - 710 = 55 mm of Hg
= 55 × 0.00133 = 0.073 bar
From steam tables, corresponding to a pressure of 0.073 bar, we find that vacuum temperature,
tv = 39.83°C
Temperature rise of cooling water
\ Condenser efficiency, hc =
vacuum temperature − inlet cooling temperature
t o − ti 25 − 10
= = = 0.503 or 50.3%
tv − ti 39.83 − 10
(e) Let x = Quality of steam entering the condenser.
From steam tables, corresponding to a pressure of 0.073 bar, we find that,
hf = 166.7 kJ/kg; and hfg = 2407.4 kJ/kg
Corresponding to a condensate temperature of 28°C, heat in condensate,
hf 1 = 117.3 kJ/kg

M08_THERMAL ENGINEERING_SE_XXXX_CH08.indd 417 6/15/2018 3:52:33 PM


418 Chapter 8

Total heat of entering steam,


h = hf + xhfg = 166.7 + x × 2407.4

Mass of cooling water (mw),
m s ( h − h f 1 ) 2000(166.7 + x × 2407.4 − 117.3)
60000 = =
C pw (to − ti ) 4.2( 25 − 10)
= 31.7 (49.4 + x × 2407.4)
60000
or x × 2407.4 = - 49.4 = 1843.3
31.7
\ x = 0.76
(f) Absolute pressure of air (as per Dalton’s law),
pa = pc - ps = 0.073 - 0.0562 = 0.0168 bar
= 0.0168 × 105 = 1680 N/m2
\ Mass of air per m3 of condenser volume,
pV 1680 × 1
ma = a = = 0.019 kg (∵ pV = mRT and V = 1 m3)
RT 287 × 308
(g)  From steam tables, corresponding to 35°C (i.e., mean condenser temperature), specific volume of
steam,
vg = 25.245 m3/kg
Air associated with 1 kg of steam at 35°C will occupy the same volume, i.e., 25.245 m3
\ Mass of air per kg of uncondensed steam,
pa v g 1680 × 25.245
ma = = = 0.48 kg
RT 287 × 308

Example 8.24
The following observations refer to a surface condenser:
Mass flow rate of condensate = 20 kg/min
Mass flow rate of cooling water = 800 kg/min
Mean temperature of condensation = 35°C
Condenser vacuum = 0.954 bar
Barometer reading = 1.03248 bar
Intel cooling water temperature = 20°C
Outlet cooling water temperature = 30°C
Temperature of the hot well = 29°C
Calculate the following:
(a)  Weight of air per unit volume of condenser
(b)  Entering condition of steam to the condenser
(c)  Vacuum efficiency of the condenser.

M08_THERMAL ENGINEERING_SE_XXXX_CH08.indd 418 6/15/2018 3:52:35 PM


Steam Condensers 419

Properties of saturated steam:

T (°C) p (bar) Sp. Vol Enthalpy Entropy


(m3/kg) (kJ/kg) (kJ/kg. K)
Vf Vg hf hg sf sg
35 0.056 0.0001 25.245 146.56 2565.4 0.5049 8.3543

Take R for air = 0.287 kJ/kg. K. [IAS, 2009]

Solution
Given that m s = 20 kg/min, m w = 800 kg/min, T = 273 + 35 = 308 K, twi = 20°C, two = 30°C,
ph = 1.03248 N/m2, pcu = 0.954 N/m2, thw = 29°C
Absolute pressure in condenser, pt = (ph − pcu) = 1.03248 − 0.954 = 0.07848 bar
At t = 35°C, ps = 0.05628
Partial pressure of air, pa = pt − ps = 0.07848 − 0.05628 = 0.0222 bar

paV 0.0222 × 105 × 1


(a) Mass of air, ma = = = 0.0251 kg/m3
RT 287 × 308

m w h f + xh fg − hhw
(b) =
m s c pw (two − twi )

At pt = 0.07848 bar, hf = 169.15 kJ/kg, hfg = 2405.80 kJ.kg


hhw = cpw × thw = 4.187 × 29 = 121.42 kJ/kg
800 169.15 + x × 2405.80 − 121.42
=
20 4.187(30 − 20)

47.73 + 2405.80 × x
40 =
41.87
1674.8 = 47.73 + 2405.80 × x
x = 0.0676
ps − pt 1.03248 − 0.07848
(c) Vacuum efficiency, hvac = = = 0.9772 or 97.72%
pb − ps 1.03248 − 0.05628

Summary for Quick Revision


1. A condenser is a closed vessel in which steam is condensed by abstracting heat and the pressure is main-
tained below atmospheric pressure.
2. Steam condensers are basically of two types—jet condensers and surface condensers.
3. In jet condensers, the steam to be condensed mixes with the cooling water.
4. In surface condensers, steam does not mix with the cooling water.

M08_THERMAL ENGINEERING_SE_XXXX_CH08.indd 419 6/15/2018 3:52:37 PM


420 Chapter 8

5. Jet condensers can be classified as low-level counter flow or parallel flow type; high-level type, and ejector type.
6. Surface condensers can be classified as water tube type, central flow or regenerative type, and evaporative type.
7. Vacuum is sub-atmospheric pressure. It is measured by a vacuum gauge.
8. Absolute condenser pressure. p = 0.133322 (Hb − Hg) bar
where Hb = actual barometric height, cm of Hg
Hg = actual vacuum gauge reading, cm of Hg.
Corrected or standard vacuum = 76 − (Hb − Hg) cm of Hg.
9. According to Delton’s law of partial pressures,
Total pressure in the condenser,
pt = absolute pressure of air, pa + saturation pressure of steam, ps
 
10. Mass of mixture, m = ma 1 + va = m 1 + g 
v
  g
 vg   va 

where ma, mg = mass of air and water vapour respectively.


va, vg = specific volume of air and water vapour respectively.
ms ( h f + xh fg − hc )
11. Mass of cooling water required, mw = for jet condensers
c pw (tc − tiw )
ms ( h f + xh fg − hc )
= for surface condensers.
c pw (tow − tiw )
12. An Edward’s air pump is used to maintain the desired vacuum in the condenser.
13. Vacuum efficiency is defined as the ratio of actual vacuum to ideal vacuum.
pb + ( pa − ps )
hvacuum =
pb − ps
where pb = barometric pressure
pa = partial pressure of air
ps = saturation pressure of steam
tow − tiw
14. Condenser efficiency, hc =
t s − tiw
where tiw, tow = inlet and outlet temperatures of cooling water
ts = steam temperature in the condenser
15. A cooling tower is an artificial device used to cool hot cooling water coming out from the condenser.

Multiple-choice Questions
1. In a steam condenser, the partial pressure of steam (c) reducing the temperature of superheated
and air are 0.060 bar and 0.007 bar, respectively. steam
The condenser pressure is (d) reducing the temperature of cooling water
(a) 0.067 bar (b) 0.060 bar used in condenser
(c) 0.053 bar (d) 0.007 bar 3. The function of the surface condenser is to
2. Cooling tower in a steam power plant is a device for (a) lower the engine thermal efficiency
(a) condensing steam into water (b) lncrease the engine thermal efficiency
(b) cooling the exhaust gases coming out of the (c) lncrease the back pressure of the engine
boiler (d) cool the exhaust gases

M08_THERMAL ENGINEERING_SE_XXXX_CH08.indd 420 6/15/2018 3:52:39 PM


Steam Condensers 421

4. In a jet condenser, (c) removes only uncondensed vapour from con-


(a) steam and cooling water mix together denser
(b) steam and cooling water do not mix together (d) removes air along with vapour and condensed
(c) steam passes through tubes and cooling water water from the condenser
surrounds them 8. In a surface condenser used in a steam power sta-
(d) cooling water passes through tubes and steam tion, undercooling of condensate is undesirable as
surrounds them this would
5. A surface condenser is a (a) not absorb the gases in steam.
(a) water tube device (b) reduce efficiency of the plant.
(b) steam tube device (c) increase the cooling water requirements.
(d) increase thermal stresses in the condenser
(c) steam and cooling water mix to give the con-
densate 9. Consider the following statement:
(d) All of the above The effect of fouling in a water-cooled steam con-
denser is that it
6. The air removal from the surface condenser leads 1. reduces the heat transfer coefficient of water
to 2. reduces the overall heat transfer coefficient
(a) fall in condenser pressure 3. reduces the area available for heat transfer
(b) rise in condenser pressure 4. increases the pressure drop of water
(c) no change in condenser pressure Of these statements:
(d) rise in condenser temperature (a) 1, 2, and 4 are correct
7. Edward’s air pump (b) 2 and 4 are correct
(a) removes only air from the condenser (c) 2 and 4 are correct
(b) removes air and vapour form the condenser (d) 1 and 3 are correct.

Review Questions
1. Define a condenser.
2. What are the functions of a condenser?
3. Name the various elements of a steam condensing plant.
4. What are the various types of steam condensers?
5. Differentiate between jet and surface condensers.
6. What is an evaporative condenser?
7. What are the requirements of a modern surface condenser?
8. List the advantages and disadvantages of a jet condenser.
9. What are the advantages and disadvantages of surface condensers?
10. State Dalton’s law of partial pressures.
11. How the mass of cooling water can be estimated in a surface condenser?
12. What are the sources of air infiltration in a condenser?
13. What are the effects of air infiltration in condensers?
14. Define vacuum efficiency and condenser efficiency.
15. What is the role of cooling towers in surface condenser?

Exercises
8.1 The vacuum in a condenser handling 9000 kg/h of steam is found to be 72 cm of Hg when the barometer
reading is 76 cm of Hg and the temperature is 25°C. The air leakage amounts to 2 kg for every 5000 kg of
steam. Determine the capacity of a suitable dry air pump in m3/min required for the condenser. Take volu-
metric efficiency of the pump as 85%. [Ans. 2.83m3/min]

M08_THERMAL ENGINEERING_SE_XXXX_CH08.indd 421 6/15/2018 3:52:39 PM


422 Chapter 8

8.2  A steam condenser has separate air and condensate pumps. The entry to the air pump suction is screened.
Steam enters the condenser at 38°C and the condensate is removed at 37°C. The air removed has a temper-
ature of 36°C. If the quantity of air infiltration from various sources is 5 kg/h, determine the volume of air
handled by the air pump per hour. Compare this with the quantity that would have to be dealt with by using
a combined air and condensate pump. Neglect the pressure due to air at entry of steam and assume uniform
pressure in the condenser.  [Ans. 647.3m3/h, 1267.38 m3/h]
8.3  A surface condenser deals with 13625 kg of steam per hour at a pressure of 0.09 bar. The steam enters 0.85
dryness fraction and the temperature at the condensate and air extraction pipes is 36°C. The air leakage
amounts to 7.26 kg/hour. Determine the following:
(a)  The surface area required if the average heat transmission rate is 4 kJ/cm per second.
(b) The cylinder diameter for the dry air pump, if it is to be single acting and running at 60 rpm with a
stroke-bore ratio of 1.25 and volumetric efficiency of 85%. [Ans. (a) 1956.57 cm2, (b) 74 cm]
8.4  A surface condenser, having an absolute pressure of 0.10 bar, is supplied with cooling water at the rate of
40kg of steam condensed. The rise in the temperature of cooling water is 14°C. Find the dryness fraction of
steam entering the condenser. The condensate leaves at 44°C. [Ans. 0.98]
8.5  A steam turbine discharges 5000 kg/h of steam at 40°C and 0.85 dry. The air leakage in the condenser is
15 kg/h. The temperature at the suction of air pump is 32°C and temperature of condensate is 35°C. Find
(a) vacuum gauge reading, (b) capacity of air pump, (c) loss of condensate in kg/h, and (d) quantity of cool-
ing water if its temperature rise is limited to 10°C.
 [Ans. (a) 70.45 cm, (b) 502.5 m3/h, (c) 17 kg/h, (d) 254 × 103 kg/h]
8.6  A steam turbine uses 50,000 kg/h of steam. The exhaust steam with dryness fraction 0.9 enters the condenser
fitted with water extraction and air pumps. When the barometer reads 76 cm of Hg, vacuum of air pump
suction is 72 cm of Hg and temperature is 32°C. The air leakage is estimated at 500 kg/h. Calculate (a) net
capacity of air pump and (b) quantity of water circulated per minute if temperature rise is limited to 15°C.
 [Ans. (a) 1291.83 m3.min, (b) 27160 kg/min]
8.7  The temperature in a surface condenser is 40°C and the vacuum is 69 cm of Hg while the barometer reads
75 cm of Hg. Determine the partial pressure of steam and air and the mass of air present per kg of steam.
 [Ans. 0.07375 bar, 0.00625 bar, 0.136 kg/kg vapour]
8.8  In a condenser, to check the leakage of air, the following procedure is adopted:
 After running the plant to reach the steady conditions, the steam supply to the condenser and also the air
and condensate pumps are shut down, thus completely isolating the condenser. The temperature and vacuum
readings are noted at shut down and also after a period of 5 minutes. They are 39°C and 68.5 cm Hg; and
28°C and 27 cm Hg respectively. The barometer reads 75 cm Hg. The effective volume of condenser is 1 m3.
Calculate (a) the quantity of air leakage into the condenser during the period of observation, and (b) the
quantity of water vapour condensed during the period. [Ans. 0.3523 kg, 0.0214 kg]

ANSWERS TO MULTIPLE-CHOICE QUESTIONS


1. (a) 2. (d) 3. (b) 4. (a) 5. (a)
6. (a) 7. (d) 8. (c) 9. (b)

M08_THERMAL ENGINEERING_SE_XXXX_CH08.indd 422 6/15/2018 3:52:39 PM


Chapter 9

Gas Power Cycles

9.1 ❐ Introduction
The gas as the working fluid does not undergo any phase change. The engines working on the gas
cycles may be either cyclic or non-cyclic. The working fluid is assumed to obey all laws of a perfect
gas.
In most of the gas power cycles, the working fluid consists mainly of air. Therefore, it is convenient
to analyse the gas power cycles by devising an idealized cycle known as Air Standard Cycle, in which
the working fluid is pure air.
The following assumptions are made in the air-standard cycles:
1. The air as the working fluid follows the perfect gas law: pV = mRT.
2. The working fluid is homogeneous throughout and no chemical reaction takes place.
3. The specific heats of air do not vary with temperature.
4. The mass of air in the cycle remains fixed.
5. The combustion process is replaced by an equivalent heat addition process.
6. The exhaust process is replaced by an equivalent heat rejection process.
7. All processes are internally reversible.
The thermal efficiency, hth, of air standard cycle is given by
Work Done Heat Supplied − Heat Rejected
h th or h a = =
Heat Supplied Heat Supplied
Qs − Qr qs − qr
= =
Qs qs
The thermal efficiency is also called air-standard efficiency (h a )
Relative efficiency, (h r ) = Actual thermal efficiency/Air standard efficiency.

9.2 ❐ Piston-cylinder Arrangement


The piston-cylinder arrangement is shown in Fig. 9.1. The various definitions are:
Bore (d): is the cylinder or piston diameter.
Stroke (L): is the distance moved by the piston in one direction. It is equal to twice the crank radius.

M09_THERMAL ENGINEERING_SE_XXXX_CH09.indd 423 6/15/2018 4:03:30 PM


424 Chapter 9

TDC (IDC) Cylinder BDC (ODC)

Exhaust
valve (EV) Piston
Piston rod
Cylinder head

Inlet valve
(IV)

Clearance Vc Stroke, L
Vs

Figure 9.1  Piston-cylinder arrangement

Top Dead centre (T.D.C.): is the extreme position of the piston near to the head of cylinder. It is
also called inner dead centre (I.D.C) position
Bottom Dead centre (B.D.C.): is the extreme position of the piston opposite to the head of
cylinder. It is also called outer dead centre (O.D.C) position
L = T.D.C. − B.D.C.
Clearance Volume (Vc): is the minimum volume of clearance between the cylinder head and the
piston at T.D.C. position.
Swept Volume (Vs): is the maximum volume swept by the piston in moving from T.D.C. to B.D.C.
p
position or vice versa. Vs = d 2 L.
4
Cylinder Volume (V): is the sum of clearance volume and swept volume
V = Vc + Vs
Compression ratio (r): is the ratio of cylinder volume to the clearance volume. Thus
V Vc + Vs V
r= = =1 + s
Vc Vc Vc
Clearance ratio (c): is the ratio of clearance volume to stroke volume. Thus
V 1 1+ c
c = c and r = 1 + =
Vs c c
Mean effective pressure (pm): is the average pressure which, if acted on the piston during the
entire power or outward stroke, would produce the same work output as the network output for the
actual cyclic process. Thus
p 4W
Work done per cycle, W = d 2 L. pm or pm =
4 p d2L
Work done per minute = pm L An
p
Where A = d 2
n = number of strokes per minute. 4
= N, for two stroke engines.
= N/2, for four stroke engines.
N = rpm

M09_THERMAL ENGINEERING_SE_XXXX_CH09.indd 424 6/15/2018 4:03:33 PM


Gas Power Cycles 425

pm LAn
Indicated power IP = kW
60
where pm is in kPa.

9.3 ❐ Carnot Cycle


The Carnot cycle consists of two reversible isotherms and two reversible adiabatics, as shown
in Fig. 9.2. For one kg of gas (T1 > T2), we have
q1–2 = 0
w1− 2 = − cv (T2 − T1 )
v 
q2 −3 = RT1 ln  3 
 v2 
v 
w2 −3 = RT1 ln  3 
 v2 
q3-4 = 0
w3− 4 = − cv (T4 − T3 )
v 
q4 −1 = RT2 ln  1 
 v4 
v 
w4 −1 = RT2 ln  1 
 v4 

∫ dq = ∫ dw
∴   
1 1
v3  T2  g −1 v  T  g −1
Now = and 2 =  2 
v4  T1  v1  T1 
v3 v2 v v
∴ = or 3 = 4
v4 v1 v2 v1

p T
2 qs 3
qs T1
2 T1
3
s=c

T1 = C
s=c

T2
1 qr 4
1 T2 = C
T2
qr 4

(a) v (b) s

Figure 9.2  Carnot cycle: (a) p-v diagram, (b) T-s diagram

M09_THERMAL ENGINEERING_SE_XXXX_CH09.indd 425 6/15/2018 4:03:38 PM


426 Chapter 9

v 
Heat supplied, qs = q2 −3 = RT1 ln  2 
 v1 
v 
Heat rejected, qr = q4 −1 = RT2 ln  2 
 v1 
v 
Net work, wnet = qs − qr = R(T1 − T2 ) ln  2 
 v1 
w T
Cycle thermal efficiency, h th = net = (T1 − T2 ) / T1 = 1 − 2  (9.1)
qs T2

9.4 ❐ Stirling Cycle


This cycle works on the principle of regeneration by using a regenerator within the engine itself which
would store the rejected heat energy during heat rejection process and supply the same during heat
addition process. The Stirling cycle consists of two reversible isotherms and two reversible isochors.
The p − v and T − s diagrams are shown in Fig. 9.3. We note that heat addition process 2−3 and heat
rejection process 4−1 cancel each other, since the energy taken from the regenerator is returned in the
latter. An alternative interpretation is to consider the regenerator as a part of the system so that the heat
exchange with the surroundings involves only the source at temperature T1 and the sink at temperature
T2. The heat supplied per cycle is, therefore, during the process from 3−4 and heat rejection is during
the process 1−2. For one kg of ideal gas (T1 > T2), we have

v 
Heat supplied, qs = q3− 4 = w3− 4 = RT1 ln  4 
 v3 
v 
Heat rejected, qr = q1− 2 = w1− 2 = RT2 ln  1 
 v2 
  v4   v1  
 RT1 ln   − RT2 ln   
q −q   v3   v2  
Thermal efficiency, h th =  s r  = 
 qs  v 
RT1 ln  4 
 v3 
v 
But v4 = v3, v1 = v2 and r =  4  , hence
 v1 
T −T  T
h th =  1 2  = 1 − 2 (9.2)
 T1  T1
Generally there is loss of heat due to radiation and poor conductivity of gas. If h r is the efficiency of
the regenerator, then
Heat supplied = RT1 ln r + (1 − h r ) cv (T1 − T2)
Heat rejected = RT2 ln r + (1 − h r ) cv (T1 − T2)
R(T1 − T2 ) ln r
Thermal efficiency, h th =  (9.3)
RT1 ln r + (1 − h r ) cv (T1 − T2 )

M09_THERMAL ENGINEERING_SE_XXXX_CH09.indd 426 6/15/2018 4:03:42 PM


Gas Power Cycles 427

p T
2 2
T1 3
T=C
3
v=c T1

c
c

v=
qr

v=
qs
v=c
1 4
1 T=C T2

4 T2

(a) v (b) s

Figure 9.3  Stirling cycle: (a) p-v diagram, (b) T-s diagram

9.5 ❐ Ericsson Cycle


This cycle also works on the principle of regeneration. It consists of two reversible isothermals and
two reversible isobars. The p-v and T-s diagram are shown in Fig. 9.4. The heat rejected during the heat
rejection process 4 – 1 at constant pressure is stored in the regenerator and the same is supplied during
heat addition process 2 – 3. Assuming 100% efficiency of the regenerator, we have
v 
Heat supplied, qs = RT1 ln  3  = RT1 ln r
 v2 
v 
Heat rejected, qr = RT2 ln  4  = RT2 ln r
 v1 
v  v 
Where r= 3 =  4 
 v2   v1 
 q − q  T −T  T
Thermal efficiency,     h th =  s r  =  1 2  = 1 − 2 (9.4)
 qs   T1  T1

p qs T
1 2 3
2 T1
c

T2 = C T1 = C
p=

p=

1 4 T2
4 3
qr

(a) v (b) s

Figure 9.4  Ericsson cycle: (a) p-v diagram, (b) T-s diagram

M09_THERMAL ENGINEERING_SE_XXXX_CH09.indd 427 6/15/2018 4:03:45 PM


428 Chapter 9

9.6 ❐ Atkinson Cycle


This cycle has two reversible adiabatics (isentropics), one isobar and one isochore. The p-v and T-s
diagrams are shown in Fig. 9.5. For one kg of the working fluid, we have
Heat supplied, qs = cv (T3 – T2)
Heat rejected, qr = cp (T4 – T1)

Net work done, wnet = qs − qr = cv (T3 − T2 ) − c p (T4 − T1 )

wnet cv (T3 − T2 ) − c p (T4 − T1 )


Thermal efficiency,    h th = =
qs cv (T3 − T2 )
g (T4 − T1 )
= 1−
(T3 − T2 )
cp
where    g =
cv
v1
Let compression ratio, r=
v2
p3
Explosion ratio, a=
p2
From compression process 1 − 2, we have
g −1
v 
T2 = T1  1  = T1rg −1
v  2
For constant volume heat addition process 2 − 3, we have
p2 p3
=
T2 T3
p3
T3 = T2 = T .a = a T1rg −1
p2 2

3
p 3 T
v=c
s=c

pvg = c 2
v=c

2 4
s=c

p=c
1

1 p=c 4

v s
(a) (b)

Figure 9.5  Atkinson cycle: (a) p-v diagram, (b) T-s diagram

M09_THERMAL ENGINEERING_SE_XXXX_CH09.indd 428 6/15/2018 4:03:49 PM


Gas Power Cycles 429

From expansion process 3−4, we have


p3 g −1
(g −1)/g
T3  p3  p2  a p2  g
= = =
T4  p4  g −1  p  1
 p2  g
 p 
1
(g −1)/g
(g −1)/g  p2 
=a  p 
1
g
p v 
Also 2 =  1  = rg
p1  v2 
T3 T
= a (g −1)/g µ r (g −1)
∴ ∴ 3 = a (g −1)/g µ r (g −1)
T4 T4
1
T3 a T1rg −1 g
or T4 = = = T1a
g −1 g −1
a × rg −1 a
g g
µrg −1

Substituting the values of T2, T3 and T4, we get


1 − g (T1a g −1 − T1 )
h th =
a T1rg −1 − T1rg −1
  1 
g  a g − 1 
1   
  
    = 1 − r g −1  (9.5)
 a − 1 
 
 
 

9.7 ❐ Otto Cycle (constant volume cycle)


This cycle is composed of four internally reversible processes, two adiabatic and two constant volume
processes. The p-v and T-s diagrams are shown in Fig. 9.6. The various processes are:
Process 1-2: Isentropic compression.
Process 2-3: Constant volume heat addition.
Process 3-4: Isentropic expansion.
Process 4-1: Constant volume heat rejection.
This cycle is used for spark ignition (petrol) engines.
Consider 1 kg of air flowing through the cycle. Since the air in the cylinder acts as a closed system,
from first law of thermodynamics for isentropic compression and expansion, we have
q − w = Δu
For constant volume heat supplied and rejection processes, since w = 0,
q = Δu = cv ΔT

M09_THERMAL ENGINEERING_SE_XXXX_CH09.indd 429 6/15/2018 4:03:52 PM


430 Chapter 9

p T

qs
3 3
pvg = c
v=c s=c
v=c
4 2
qs 2 qr 4
s=c
v=c v=c
1 1
pvg = c
qr

vc vs v s
v2 v1
(a) (b)

Figure 9.6  Otto cycle: (a) p-v diagram, (b) T-s diagram

Heat supplied qs = q2−3 = cv(T3 – T2)


Heat rejected qr = q4−1 = cv (T4 – T1)
Work done per cycle, wnet = qs – qr = cv(T3 – T2) – cv(T4 – T1)
Work done
Air standard (or thermal) efficiency =
Heat supplied
cv (T3 − T2 ) − cv (T4 − T1 )
h a or h th =
cv (T3 − T2 )
T4 − T1
=1−
T3 − T2
v
Now       T2 = T1 1 = T1rg −1
v2
v1 v1
where r = = = compression ratio
v2 vc
T p
T3 = 2 3 = a pT2 = a pT1rg −1
p2
p3 T3
where a p = = = pressure ratio
p2 T2
g −1
 1
T4 = T3   = a pT1
 r
a pT1 − T1
∴ h a or h th = 1 −
a pT1rg −1 − T1rg −1
a p −1
=1−
(a p − 1)rg −1
1
      = 1 − (9.6)
g −1
r

M09_THERMAL ENGINEERING_SE_XXXX_CH09.indd 430 6/15/2018 4:03:56 PM


Gas Power Cycles 431

100
g = 1.67 (Theoretical monotomic gas)

80
g = 1.40 (Air standard)

60
g = 1.20
h th %

40

20

0 1 8 12 16 20 21 28
Compression ratio

Figure 9.7  Otto cycle thermal efficiency v’s compression ratio

The air standard efficiency of Otto cycle depends on compression ratio only and increases as compres-
sion ratio increases (Fig. 9.7). In actual engine working on Otto cycle, the compression ratio varies
from 5 to 8. This engine is used for spark ignition engines working on petrol.

Mean Effective Pressure (m.e.p.). It may be defined as the ratio of work done to the displacement
volume of piston.
Work done, w = cv[(T3 – T2) – (T4 – T1)]
Displacement volume, vs = v1 – v2
w cv (T3 − T2 ) − (T4 − T1 )
M.E.P., pm = = 
vs v1 − v2
 1  RT  r − 1
Now v1 − v2 = v1 1 −  = 1 
 r  p1  r 

p1r[rg −1 (a p − 1) − (a p − 1)]
  pm =
(g − 1)( r − 1)
p1r (a p − 1) ( rg −1 − 1) ( p3 v3 − p4 v4 ) − ( p2 v2 − p1v1 )
= = (9.7)
(g − 1)( r − 1) (g − 1)( v1 − v2 )

9.8 ❐ Diesel Cycle


This cycle is used for compression ignition internal combustion engines working on diesel oil. The p-v
and T-s diagrams are shown in Fig. 9.8. It consists of four internally reversible processes two adiaba-
tic, one constant pressure and one constant volume. The various processes are:
Process 1–2: Isentropic compression of air.

M09_THERMAL ENGINEERING_SE_XXXX_CH09.indd 431 6/15/2018 4:03:58 PM


432 Chapter 9

Process 2–3: Heat addition at constant pressure.


Process 3–4: Isentropic expansion of air.
Process 4−1: Heat rejection at constant volume.
Considering 1 kg of air.
Heat supplied, qs = q2–3 = cp (T3 – T2)
Heat rejected, qr = q4– 1 = cv (T4 – T1)
Work done per cycle, w = qs – qr = cp (T3 – T2) – cv (T4 – T1)
Thermal efficiency,
Work done w
h th = =
Heat supplied qs

c p (T3 − T2 ) − cv (T4 − T1 ) 1  T4 − T1 
= =1−  
c p (T3 − T2 ) g  T3 − T2 

v1 v4
Let       r = = = compression ratio
v2 v2
g −1

v 
T2 = T1  1 
Now    = T1rg −1
v  2
v3
Let    r = = cut off ratio
v2
v3 T3
r= = or T3 = T2 r = T1rg −1 r
v2 T2
g −1 g −1 g −1
v  v v   r
T4 = T3  3  = T3  3 × 2  = T3  
 v4   v 2 v4   r
g −1
 r
= T1rg −1 r   = T1 r g
 r

qs
p T
p=c
2 3 qs 3

p=c s=c
2
pvγ = c 4
s=c
pvγ =c 4 v=c
1 qr
qr
v=c
1
v2 v3 v1 = v4 v s
(a) (b)

Figure 9.8  Diesel cycle: (a) p-v diagram, (b) T-s diagram

M09_THERMAL ENGINEERING_SE_XXXX_CH09.indd 432 6/15/2018 4:04:02 PM


Gas Power Cycles 433

1  T1 r g − T1  1  rg −1 
h th = 1 −  r −1  =1−   (9.8)
g  T1r r − T1rg −1  rg −1 g ( r − 1) 
The thermal efficiency of diesel cycle increases as compression ratio increases but decreases as
cut-off ratio increases (Fig. 9.9). The thermal efficiency of diesel cycle is less than that of Otto cycle.
The compression ratio for diesel cycle varies from 14 to l8.
Work done
Mean effective pressure pm =
Swept volume
c p (T3 − T2 ) − cv (T4 − T1 )
=
v1 − v2

 v   1
v1 − v2 = v1 1 − 2  = v1 1 − 
Now   
 v1   r
RT1  r − 1 T1  r − 1
=   = cv (g − 1) ×  
p1 r p1 r 
c p (T3 − T2 ) − cv (T4 − T1 )
    pm =
T1  r − 1
cv (g − 1) × 
p1  r 
p1r[g (T3 − T2 ) − (T4 − T1 )]
=
(g − 1)T1 ( r − 1)
p1r[g T1 ( rg −1 r − rg −1 ) − T1 ( r g − 1)]
=
(g − 1)T1 ( r − 1)
p r[g r g −1( r − 1) − ( r g − 1)]
= 1
   (9.9)
(g − 1)( r − 1)

80
e)
cycl
Otto
so
1 (Al
60 =
er r =2
l c ycl
se r=
4
Die
40
r increasing
h th %

20

0
1 5 9 13 17 21 25
Compression ratio

Figure 9.9  Diesel cycle thermal efficiency v’s compression ratio

M09_THERMAL ENGINEERING_SE_XXXX_CH09.indd 433 6/15/2018 4:04:06 PM


434 Chapter 9

9.9 ❐ Dual Cycle


In the dual cycle, part of the heat is supplied at constant volume and the rest at constant pressure. This
is also called mixed or limited pressure cycle. The p-v and T-s diagrams are shown in Fig. 9.10. The
various processes are:
Process 1−2: Isentropic compression of air
Process 2−3: Constant volume heat addition
Process 3−4: Constant pressure heat addition
Process 4−5: Isentropic expansion of air
Process 5−1: Constant volume heat rejection
Considering 1 kg of working fluid,
Heat supplied, qs = cv (T3 − T2) + cp (T4 − T3)
Heat rejected, qr = cv (T5 − T1)
Work done per cycle, w = qs – qr = cv (T3 – T2) + cp (T4 – T3) – cv (T5 – T1)

w cv (T3 – T2 ) + c p (T4 – T3 ) – cv (T5 – T1 )


Thermal efficiency, h th = =
qs cv (T3 – T2 ) + c p (T4 – T3 )

=1−
(T5 – T1 )
(T3 – T2 ) + g (T4 – T3 )
v1
Let, r= = compression ratio
v2
p
a p = 3 = pressure ratio
p2
v4 v4
   r = = = cut-off ratio
v3 v2
Then from process 1−2, we have
T2 = T1rγ−1
T3 p3
From process 2 – 3, = =a p or T3 = a pT2 = a p rg −1T1
T2 p2

qs
p p=c 4 T
3
qs 4
v=c =c
qs p s=c
qs 2 3 5
pvg = c
v =c
c
5
2 v=
qr
s=c qr
pvg = c v=c
1 1
v s
(a) (b)

Figure 9.10  Dual cycle: (a) p-v diagram, (b) T-s diagram

M09_THERMAL ENGINEERING_SE_XXXX_CH09.indd 434 6/15/2018 4:04:09 PM


Gas Power Cycles 435

T4 v4 v4
From process 3 – 4, = = =r or T4 = rT3 = ra p rg −1T1
T3 v3 v2
g −1 g −1 g −1 g −1
T5  v4   v   v    v   v    r
From process 4 – 5, = =  4  ×  2   =  4  ×  2   = 
T4  v5   v2   v5    v2   v1    r
g −1
 r
T5 = T4   = T1a p r g
 r
(a p r g − 1)
h th = 1 −
(a p rg −1 − rg −1 ) + g (a p rrg −1 − a p rg −1 )

1  (a p r g − 1) 
=1−
    (9.10)
rg −1  (a p − 1) + g a p ( r − 1) 

The variation of thermal efficiency of dual cycle is shown in Fig. 9.11.


Mean effective pressure,
Work done w
pm = =
Swept volume v1 − v2

 1
Now     v1 − v2 = v1 1 − 
 r
RT1  r − 1
= v1 ( r − 1) / r =  
p1  r 

cv (g − 1)T1  r − 1
=  
p1 r 

cv (T3 – T2 ) + c p (T4 – T3 ) – cv (T5 – T1 )


=
cv (g − 1)T1  r − 1
 
p1 r 

h th
Otto
Dual
ap = 1

Diesel

r =1

Figure 9.11 Variation of thermal efficiency of dual cycle


with compression ratio

M09_THERMAL ENGINEERING_SE_XXXX_CH09.indd 435 6/15/2018 4:04:13 PM


436 Chapter 9

p1r  T3 − T2 g (T4 − T3 ) T5 − T1 
= + −
 
(g − 1)( r − 1)  T1 T1 T1 

p1r (a rg −1 − rg −1 ) + g ( ra rg −1 − a rg −1 ) − (a r g − 1) 
=
(g − 1)( r − 1)  p p p p 

p1r  rg −1{(a − 1) + g a ( r − 1)} − (a r g − 1)  (9.11)


=
(g − 1)( r − 1)  p p p 

9.10 ❐ Brayton Cycle


The air standard Brayton or Joule cycle is a constant pressure cycle used in gas turbine power plants.
The p-v and T-S diagrams are shown in Fig 9.12. It consists of the following processes:
1−2: Isentropic compression in the compressor
2−3: Constant pressure heat addition.
3−4: Isentropic expansion of air
4−4: Constant pressure heat rejection
Consider 1 kg of working fluid
From first law of thermodynamics for steady flow, rejecting ΔKE and ΔPE, we have
d q – d w = dh
Heat added, qs = h3 – h2 = cp (T3 – T2)
Heat rejected qr = h4 – h1 = cp (T4 – T1)
Net work done by turbine, wnet = qs – qr = cp [(T3 −T2) − (T4 − T1)]
Also work done by turbine, wt = h3 − h4 = cp (T3 −T4)
Work consumed by compressor, wc = h2 − h1 = cp (T2 − T1)
wnet = cp [(T3 − T4) − (T2 −T1)] = cp [(T3 −T2) − (T4 − T1)]

Net work w
Thermal efficiency, h th = = net
Heat added qs

p qs T qs
3
2 p=c
3 p=c s=c
2
wt
pvg = c
we 4
pvg = c s=c p=c
qr
1
1 4
p=c
qr
v s
(a) (b)

Figure 9.12  Brayton (or Joule) cycle: (a) p-v diagram, (b) T-s diagram

M09_THERMAL ENGINEERING_SE_XXXX_CH09.indd 436 6/15/2018 4:04:15 PM


Gas Power Cycles 437

c p  T3 ( – T2 ) − (T4 – T1 ) T4 − T1
= 
   =1−
c p T3 – T2  T3 − T2

From isentropic compression process 1 – 2, we have


g −1
T2  p2  g
=
T1  p1 
and from isentropic expansion process 3 – 4,
g −1
T3  p3  g
=
T4  p4 
But p2 = p3 and p1 = p4
g −1
T T p  g T1 T4 T4 − T1
2 = 3 =  3 or = =
T1 T4  p4  T2 T3 T3 − T2
T T T T
h th = 1 − ∴2 = 1 − 4
∴ h th = 1 − 2 = 1 − 4
T1 T3 T1 T3
p2
Let rp =
= pressure ratio
p1 T
2 = ( rp )(g −1)/g
T1

1 1
h∴th = 1 −
∴ h th = 1 − (9.12)
( rp )(g −1)/g ( rp ) −1)/g
(g

The variation of thermal efficiency v’s pressure ratio is shown in Fig. 9.13. The thermal efficiency
increases with increasing values of pressure ratio. This cycle is used in gas turbines.
wt − wc
Work ratio =
wt
 wt = work done by turbine
wc = work supplied to compressor
(T3 − T4 ) − (T2 − T1 ) T −T
= =1− 2 1
T3 − T4 T3 − T4
 
 (g −1)/g 
r − 1
= 1 − 
T1  p
T3 1
 = 1 − T1 r
 T3 p
( )(g −1)/g (9.13)
1 − (g −1)/g 
 rp 
Pressure ratio for maximum work,
W = mcp [(T3 – T2) – (T4 – T1)]
= mcp [(T3 – T4) – (T2 – T1)]

M09_THERMAL ENGINEERING_SE_XXXX_CH09.indd 437 6/15/2018 4:04:20 PM


438 Chapter 9

100

80

Effciency, h th %
g = 1.66
60
g = 1.4
40

20

0
0 2 4 6 8 10 12 14 16
Pressure ratio, rp

Figure 9.13 Efficiency v’s pressure ratio in simple Brayton cycle

g −1
T3 g T2
=r =
T4 p T1

g −1
z=
Let    , mc p = y, then
g

  
1 
W = y T3 1 − 
  rp z 
− T1 rp z − 1 

( )
 
For W to me maximum, dW/drp = 0.
dW
= y[T3 zrp −( z +1) − T1 zrp z −1 ] = 0
drp

g
1/( 2 z )
T   T  2(g −1)
rp =  3  = 3 (9.14)
T  1 T  1

9.11 ❐ Comparison between Otto, Diesel, and


Dual Cycles
The comparison parameters selected are:
1. Equal compression ratio and heat input.
2. Constant maximum pressure and heat input.
3. Constant maximum pressure and work output.
4. Constant maximum pressure and temperature.
5. Equal compression ratio and heat rejection.

M09_THERMAL ENGINEERING_SE_XXXX_CH09.indd 438 6/15/2018 4:04:23 PM


Gas Power Cycles 439

p T p=c
pvγ = c
3 3
3′ 3″
3′
2′
2′
3″ c
2
4″ v=
4″
4′ 2 v=c
4 4′
pv r = c 4
1 1 v=c

v a b b′ b″ s
(a) (b)

Figure 9.14 Comparison for equal compression ratio and heat input:


(a) p-v diagram, (b) T-s diagram
1. Equal compression ratio and heat input. The p-v and T-s diagrams for the three cycles are
shown in Fig.9.14. The cycles have been represented as follows:

Cycle p-v diagram T-s diagram for same heat input Heat rejected
Otto 1 – 2 − 3− 4 1 – 2 − 3− 4 a–1–4−b
Diesel 1 – 2 – 3′′ − 4′′ 1 – 2 – 3′′ − 4′′ a – 1 – 4′′ – b′′
Dual 1 – 2 – 2′ – 3′ – 4′ 1 – 2 – 2′ – 3′ – 4′ a – 1 – 4′ – b′

That cycle will be more efficient which rejects the least amount of heat after expansion, because
q qr
∴ h th = 1 − r = 1 −
Thus the order of efficiencies is: qs Constant
∴ (hth) Otto > (hth)Dual > (hth) Diesel(9.15)
2. Constant maximum pressure and heat input. The p-v and T-s diagrams for Otto and Diesel
cycles are shown in Fig. 9.15 for constant maximum pressure and heat input
Area a – 2 − 3 –b = Area a – 2′ – 3′ –b′ for same heat input.
qr qr
∴ h th = 1 − =1−
qs Constant

Cycle p-v diagram T-s diagram for same heat input Heat rejected
Diesel 1 – 2 − 3− 4′ 1 – 2 − 3− 4′ a–1–4−b
Otto 1 – 2′ – 3′ − 4′ 1 – 2′ – 3′ − 4′ a – 1 – 4′ – b′

Thus cycle will be more efficient that rejects the least amount of heat.
(Area a – 1 − 4 –b) < (Area a – 1 – 4′ –b′)
Thus the Diesel cycle is more efficient than the Otto cycle. A similar argument will show that the
dual cycle performance falls between the other two. Thus
∴ (hth)Diesel > (hth)Dual > (hth)Otto(9.16)

M09_THERMAL ENGINEERING_SE_XXXX_CH09.indd 439 6/15/2018 4:04:26 PM


440 Chapter 9

p T
3 3′ 3′
2
pvg = c
3

p =c
c 4′
2 v=
pvg = c 4′
4 2′
2′ 4
1 1 v=c

v a b b′ s
(a) (b)

Figure 9.15 Comparison for equal maximum pressure and heat input:


(a) p-v diagram, (b) T-s diagram

3. Constant maximum pressure and work output. The p-v and T-s diagrams are shown in
Fig. 9.15 for maximum pressure and output.

Cycle p-v diagram T-s diagram for same heat input Heat rejected
Diesel 1 – 2 − 3− 4 1–2–3−4 a–1–4−b
Otto 1 – 2′ – 3′ − 4′ 1 – 2′ – 3′ – 4′ a – 1 – 4′ – b′

Area 1 – 2 – 3 – 4 = Area 1 – 2′ – 3′ – 4′ for same work output


w w Constant
∴ h th = = =
qs w + qr constant + qr
The cycle will be more efficient which rejects least amount of heat.
(Area a – 1 – 4 – b) < (Area a – 1′ – 4′ – b′)
Hence the Diesel cycle is more efficient than the Otto cycle. Thus
∴ (hth)Diesel > (hth)Dual > (hth)Otto(9.17)

4. Constant maximum pressure and temperature. The p-v and T-s diagrams are shown in
Fig. 9.16 for same maximum pressure and temporal are.

Cycle p-v diagram T-s diagram for same heat input Heat rejected
Diesel 1 – 2 − 3− 4 1–2–3−4 a–1–4−b
Otto 1 – 2′ – 3 − 4 1 – 2′ – 3 – 4 a–1–4–b
Dual 1 – 2′′ – 3′′ – 3 – 4 1 – 2′′ – 3′′ – 3 – 4 a–1–4–b

qr Constant
Now    h th = 1 − =
qs qs

M09_THERMAL ENGINEERING_SE_XXXX_CH09.indd 440 6/15/2018 4:04:28 PM


Gas Power Cycles 441

p T
2 3″ 3
c 3
p=
pvg =c 3″
c
2 v=
2″ c
2″ v= 4
4 2′
2′
c
v=
1 1

v a b s
(a) (b)

Figure 9.16 Comparison for equal maximum pressure and temperature:


(a) p-v diagram, (b) T-s diagram

All the cycles reject equal amount of heat.


Thus cycle will be more efficient which has higher heat addition.
Hence, the Diesel cycle is more efficient than the Otto cycle. Thus
∴ (hth)Diesel > (hth)Dual > (hth)Otto(9.18)

5. Equal compression ratio and heat rejection. The p-v and T-s diagrams are shown in Fig. 9.17,
for equal compression ratio and heat rejection
For same qr, if qs is more, h is more.
∴ (hth)Otto > (hth)Dual > (hth) Diesel(9.19)

Cycle p-v diagram T-s diagram Heat rejected


Otto 1 – 2 − 3− 4 1–2–3−4–1 a–2–3−b
Diesel 1 – 2 – 3′ − 4 – 1 1 – 2 – 3′ – 4 –1 a – 2 – 3′ – b
Dual 1 – 2 – 2′′ – 3′′ – 4 – 1 1 – 2 – 2′′ – 3′′ – 4 – 1 a – 2 – 2′′ – 3′′ – b

p
3 T 3
c
p=
3″ 3′
2″
3″
3′ 2″
2 c
pvg = c v=
c 4
2 p=
4
pvg = c c
v=
1 1

v a b s
(a) (b)

Figure 9.17 Comparison for equal compression ratio and heat rejection: (a) p-v diagram,
(b) T-s diagram

M09_THERMAL ENGINEERING_SE_XXXX_CH09.indd 441 6/15/2018 4:04:30 PM


442 Chapter 9

Example 9.1
A Carnot cycle has lowest temperature and pressure of 20°C and 1 bar. The pressures are: 4 bar
after isothermal compression; 12 bar after isentropic compression and 6 bar after isothermal heat
addition. Calculate (a) the highest temperature in the cycle, (b) thermal efficiency of the cycle,
(c) mean effective pressure, and power developed with 150 cycles per minute.

Solution
Given: p4 = 1 bar, T2 = 293 K, p2 = 12 bar, p1 = 4 bar, p3 = 6 bar (Refer to Fig. 9.2)

RT2 0.287 × 293


v4 = = = 0.8409 m3 /kg
p4 1 × 10 2

p4 v4 1 × 10 2 × 0.8409
v1 = = = 0.2102 m3 /kg
p1 4 × 10 4
1/g 1/14
 p   4
v2 = v1  1  = 0.2102   = 0.0958 m3 /kg
 p2   12 

v2 p2 0.0958 × 12
v3 = = = 0.1916 m3 /kg
p3 6
Stroke volume, vs = v4 – v2 = 0.8409 – 0.0958 = 0.7451 m3/kg
(a)  Highest temperature in the cycle,
g −1 0.4
 v1   0.2102 
T1 = T2  = 293  = 401.2 K
 v 
2  0.0985 

v   0.1916 
qs = p2 v2 ln  3  = 12 × 10 2 × 0.0958 ln 
(b)  Heat supplied,
 v2   0.0958 
  = 79.67 kJ/kg
v   0.2102 
Heat rejected, qr = p4 v4 ln  1  = 1 × 10 2 × 0.8409 ln 
 v4   0.8409 
    = − 58.07 kJ/kg
Net work done, wnet = qs – qr = 79.67 – 58.07 = 21.60 kJ/kg
wnet 21.60
Thermal efficiency, h th = = = 0.2695 or 26.95 %
qs 79.67
wnet
(c)  Mean effective pressure =
stroke volume
21.60
= = 28.98 kg/m 2 or 0.2898 bar
0.7451
(d)  Power developed = wnet × rpm/60 = 21.6 × 150/60 = 54 kW.

M09_THERMAL ENGINEERING_SE_XXXX_CH09.indd 442 6/15/2018 4:04:34 PM


Gas Power Cycles 443

Example 9.2
The following particulars refer to an engine working on Stirling cycle.
Compression ratio = 3, lower temperature = 30°C, speed = 50 rpm, higher temperature = 550°C,
regenerative efficiency =90%, initial pressure = 1 bar, heat added = 2300 kJ/min.
Calculate (a) work done per kg, (b) thermal efficiency, (c) mean effective pressure, and (d) indi-
cated power.

Solution
Refer to Fig. 9.18.
(a) Work done = R(T1 – T2) ln r
   = 0.287 (823 – 300) ln 3
   = 163.956 kJ/kg
  Heat added = RT1 ln r + (1 – hreg) cv(T1 – T2)
        = 0.287 × 823 × ln 3 + (1 – 0.9) × 0.718 × (823 – 303)
           = 296.829 kJ/kg
Work done
(b) Thermal efficiency   =
Heat added
163.956
= = 0.5523 or 55.23%
296.829
2300
(c) Air flow rate     = = 7.7485 kg/min
296.829
50
Working cycles/min = = 25
2
7.7485
Mass of air per working cycle, m = = 0.31 kg
25
mRT2 0.31 × 0.287 × 303
Volume of air at state 1, V1 = = = 0.2694 m3
p1 2
1 × 10
V1 0.2694
V2 = = = 0.0898 m3
r 3

p
3

qs
2 T1 =
C

4
T2 =
C qr
1

Figure 9.18

M09_THERMAL ENGINEERING_SE_XXXX_CH09.indd 443 6/15/2018 4:04:37 PM


444 Chapter 9

Stroke volume Vs = V1 − V2 = 0.2694 − 0.0898 = 0.1796 m3


Work done 163.956 × 0.31
Mean effective pressure, pm = = = 2.978 bar
Stroke volume 0.1796 × 10 2
p LAN 2.978 × 0.1796 × 50 × 10 2
(d) Indicated power = m = = 42.337 kW.
60 60

Example 9.3
An engine working on Atkinson cycle takes in air at 1 bar and 25°C . The air is compressed isentropi-
cally by a compression ratio of 6:1. The heat is added at constant volume increasing the final pressure to
20 bar. Now, the air is expanded isentropically to 1 bar. The heat is rejected at constant pressure. Calcu-
late (a) pressure and temperature at various points, (b) work done per kg of air, and (c) cycle efficiency
Solution
Refer to Fig. 9.19.
(a) p1 =1 bar, T1 = 25 + 273 = 298 K
  T2 = T1rγ – 1 = 298 ×60.4 = 610.2 K
  p2 = p1rγ = 1 × 61.4 = 12.286 bar
 p3 = 20 bar
p3 20
T3 = T2 = 610.2 ×
p2 12.286
= 993.3 K
p4 = p1 = 1 bar
T3 993.3
T4 = = = 422.7 K
g −1
200.286
 p3  g
 p 
4

   Heat added, qs = cv(T3 – T2) = 0.718 (993.3 – 610.2) = 279.4 kJ/kg


   Heat rejected, qr = cp(T4 – T1) = 1.005 (422.7 – 298) = 124.7 kJ/kg
(b) Work done, w = qs − qr = 279.4 – 124.7 = 154.7 kJ/kg
w 154.7
(c) Thermal efficiency = = = 0.5536 or 55.36%
qs 279.4
p
3
pvγ = c
v=c

1 p=c 4

Figure 9.19

M09_THERMAL ENGINEERING_SE_XXXX_CH09.indd 444 6/15/2018 4:04:41 PM


Gas Power Cycles 445

Example 9.4
In an engine working on ideal Otto cycle, the temperature at the beginning and end of compression
are 45°C and 370°C. Find the compression ratio and air standard efficiency of the engine.
Assume γ = 1.4.

Solution
Refer to Fig. 9.6.
1
v  T  g −1
(a) Compression, ratio, r= 1 = 2
v2  T1 
T1 = 45 + 273 = 318 K
T2 = 370 + 273 = 643 K
1
 643  0.4
r = = 5.814
 310 
(b) Air standard efficiency,
1 1
ha = 1 − = 1− = 0.505 = 50.5%
g −1
r 5.8140.4

Example 9.5
In an Otto cycle, air at 15°C and 1 bar is compressed adiabatically until the pressure is 15 bar.
Heat is added at constant volume until the pressure rises to 40 bar. Calculate (a) the air-standard
efficiency, (b) the compression ratio, and (c) the mean effective pressure for the cycle. Assume cv
= 0.718 kJ/kg. K, γ = 1.4 and R = 8.314 kJ/kmol.K.
Solution
The Otto cycle is shown in Fig. 9.20.
For isentropic process 1 – 2,
p1v1γ = pv2γ
1
1
v  p g
Compression ratio, r = 1 =  2  = 151.4 = 6.92
v2  p1 
Air standard efficiency,
1
ha = 1 −
g −1
r
1
= 1− = 0.539 or 53.9%
6.920.4
T1 = 15 + 273 = 288 K
p v  15 × 288
T2 =  2 2  T1 = = 629.3 K
 p1v1  6.92

M09_THERMAL ENGINEERING_SE_XXXX_CH09.indd 445 6/15/2018 4:04:44 PM


446 Chapter 9

p, bar
3
40

15
2
4

1 1

0 V2 m3 V1 v

Figure 9.20
For constant volume process 2 – 3:
p3 40
T3 = T = × 629.3 = 1669.7 K
p2 2 6.92
Heat supplied qs = cv(T3 – T2) = 0.718 (1669.7 – 629.3) = 747.03 kJ/kg
Work done, w = ha qs = 0.539 × 747.03= 402.65 kJ/kg
Mean effective pressure,
Work done w
pm = =
Swept volume v1 − v2
V1 0.8257
  v2 = = = 0.1193 m3 /kg
r 6.92
Specific volume, v1 – v2 = 0.7064 m3/kg
402.65
pm = × 103 = 9.7 × 105 N/m 2 or 9.7 bar
0.7064

Example 9.6
A gas engine working on Otto cycle has a cylinder of diameter 220 mm and stroke 300 mm. The
clearance volume is 1600 cc. Find the air standard efficiency.
Assume cp = 1.004 kJ/kg. K and cv = 0.718 kJ/kg.K for air.

Solution
Swept volume,
p  p 
Vs =   d 2 L =   × 222 × 30 = 11404 cc
 4  4
Vs 11404
Compression ratio, r =1+ =1+ = 8.13
Vc 1600
c p 1.004
Adiabatic index, g = = = 1.4
Air standard efficiency, cv 0.718

1 1
ha = 1 − =1− = 0.5675 or 56.75%
g −1
r (8.13)0.4

M09_THERMAL ENGINEERING_SE_XXXX_CH09.indd 446 6/15/2018 4:14:17 PM


Gas Power Cycles 447

Example 9.7
The pressure limit in an Otto cycle are 1 bar and 20 bar. The compression ratio is 5. Calculate
(a) thermal efficiency and (b) mean effective pressure. Assume γ = 1.4 for air.

Solution
1 1
(a) h th =1−
g −1
=1−
0.4
= 0.4747 or 47.47%
r 5
(b) From Fig. 9.6, we have
g
p2  v1 
= = rg = 51.4 = 9.5182
p1  v2 
p2 = 1 × 9.5182 = 9.5182 bar
p3 = 20 bar
p 20
Pressure Ratio, ap= 3 = = 2.1
p2 9.5182
 (a − 1)( rg −1 − 1)   ( 2.1 − 1)(50.4 − 1) 
p
pm = p1r   =1× 5   = 9.1 bar
 ( r − 1)(g − 1)  
 (5 − 1)(1.4 − 1) 
 

Example 9.8
A petrol engine with compression ratio of 5 develops 20kW indicated power and consumes 8 litres
of fuel per hour. The specific gravity of fuel is 0.78 and its calorific value is 44 MJ/kg. Calculate
the indicated thermal efficiency and relative efficiency. Take γ = 1.4.

Solution
Air standard efficiency,
1 1
ha = 1 −
= 0.4747 = 47.47% =1−
g −1 0.4
r 5
Fuel consumption   = 8 × 0.78 × 1 = 6.24 kg/h
6.24 −3
=
  = 1.733 × 10 kg/s
3600
Indicated thermal efficiency,
indicated work 20
(h th )i = = = 0.2622 or 26.22%
Heat supplied 44 × 103 × 1.733 × 10 −3
(h th )i 0.2622
Relative efficiency = = = 0.5524 or 59.24%
ha 0.4747

Example 9.9
A diesel engine has a compression ratio of 20 and cut-off takes place at 5% of the stroke. Find the
air-standard efficiency. Assume g = 1.4.

M09_THERMAL ENGINEERING_SE_XXXX_CH09.indd 447 6/15/2018 4:14:21 PM


448 Chapter 9

Solution
Refer to Fig. 9.8.
v1
= r = 20
v2
v1 = 20 v2
vs = v1 − v2 = 20 v2 − v2 = 19 v2
v3 = 0.05 vs + v2 = 0.05 × 19 v2 + v2 = 1.95 v2
v
Cut-off ratio,        r = 3 = 1.95
v2
1  rg −1 
ha = 1 −  
rg −1 g ( r − 1) 
1  1.951.4 − 1 
ha = 1 −   = 0.649 or 64.9%
200.4 1.4 (1.95 − 1) 

Example 9.10
A diesel cycle operates at a pressure of 1 bar at the beginning of compression and the volume is
1
compressed to th of the initial volume. Heat is supplied until the volume is twice that of the
15
clearance volume. Calculate the mean effective pressure of the cycle. Assume g = 1.4.

Solution
With reference to Fig. 9.8,
v1
r= = 15
v2
v1 = 15 v2

v3 = 2 v2
vs = v1 − v2 = ( r − 1) v2 = (15 − 1) v2 = 14 v2
Swept volume,
v
or     v2 = s
14
Process 1–2:
g
v 
p3 = p1  1  = 1 × (15)1.4 = 49.31 bar
v  2
 p2 = p3 = 49.31 bar

g g g
v  v  v v 
p4 = p3  3  = p3  3  = p3  3 × 2 
 v4   v1   v2 v1 
1.4
 2
= 49.31   = 2.64 bar
 15 

M09_THERMAL ENGINEERING_SE_XXXX_CH09.indd 448 6/15/2018 4:14:25 PM


Gas Power Cycles 449

1  p3 v3 − p4 v4 p2 v2 − p1v1 
pm =  p2 ( v3 − v2 ) + −
g − 1 

vs  g −1
v2  v3  1  v3 v  1  v 
=  p2  − 1 +  p3 − p4 4  −  p2 − p1 1  
vs  v2  g − 1  2 v2  g − 1  v2  

= 1  49.31( 2 − 1) + 1 ( 49.31 × 2 − 2.64 × 15) − 1 ( 49.31 − 1 × 15) 


14  0.4 0.4 

= 0.071 [49.31 + 122.55 – 79.27] = 6.685 bar

Example 9.11
The mean effective pressure of an ideal diesel cycle is 10 bar. If the initial pressure is 1 bar and the
compression ratio is 14, determine the cut-off ratio and the air-standard efficiency. Assume g = 1.4.

Solution
See Fig. 9.8.
p3 v3 − p4 v4 p2 v2 − p1v1
Work output = pm vs = p2 ( v3 − v2 ) + −
g −1 g −1
v1 v
    r = = 1 + s = 14
v2 vc
vs =13 vc
Now Nowv2 = vc ∴ vs =v13 2 =v2vc ∴ vs = 13v2
And 4 =vcv1 = 14 v2 = 14 vc
Andv4 = v1 = 14 v2 =v14
v3 = rv2 = rvc
p2 g
= r = 141.4 = 40.23
p1
p2 = 1 × 40.23 = 40.23 bar
g
p3  r  141.4 40.23
=  = =
p4  r  r1.4 r1.4
40.23 × r1.4
p4 = = r1.4
40.23
Work done per cycle
p3 v3 − p4 v4 p2 v2 − p1v1
= p2 ( v3 − v2 ) + −
g −1 g −1
1 1.4 1
= ( 40.23 × rvc − r × 14 vc ) − ( 40.23vc − 1 × 14 vc )
0.4 0.4
1 1
pm vs = vc [40.23 ( r − 1) + ( 40.23 r − 14 r1.4 ) − ( 40.23 − 14)]
0.4 0.4
10 × 13 vc = vc [40.23 r − 40.23 + 100.6 r − 35 r1.4 − 100.6 + 35]
130 = 140.83 r − 35 r1.4 − 109.83

M09_THERMAL ENGINEERING_SE_XXXX_CH09.indd 449 6/15/2018 4:14:32 PM


450 Chapter 9

or f ( r ) = r1.4 − 9.024 r + 6.738 = 0

r 2 2.5 2.6 2.64

f (r) 1.329 0.285 0.086 0.007

r = 2.64
We take   

1  rg −1 
h =1−  
rg −1 g ( r − 1) 

1  2.641.4 − 1 
=1−   = 0.5616 or 56.16%
140.4 1.4 ( 2.64 − 1) 

Example 9.12
In an engine working on the diesel cycle, the air-fuel ratio by weight is 50:1. The temperature of air
at the beginning of combustion is 40°C and the compression ratio is 19. What is the ideal efficiency
of the engine. Calorific value of fuel is 42 MJ/kg. Assume cv = 0.717 kJ/kgK, cp = 1.004 kJ/kgK.

Solution
Refer to Fig. 9.8.
cp
1.004
g = =
= 1.4
cv 0.717
v T
r= 3 = 3
v2 T2
g −1
T2  v1 
Process 1 – 2: = = rg −1 = 150.4 = 2.954
T1  v2 
T2 = 929.66 K
F
Process 2 – 3:  qs = C p (T3 − T2 ) = × CV
A
42000
 T3 − T2 = = 836.66 K
1.004 × 50
T3 = 1761.32 K
1761.32
r= = 1.905
924.66
1  rg −1  1  1.9051.4 − 1 
h =1−   = 1 − 0.4  
g −1 g ( r − 1)
r   15 1.4(1.905 − 1) 
= 0.6085 or 60.85%

M09_THERMAL ENGINEERING_SE_XXXX_CH09.indd 450 6/15/2018 4:14:37 PM


Gas Power Cycles 451

Example 9.13
An air standard diesel cycle has a compression ratio of 16. The pressure at the beginning of com-
pression stroke is 1 bar and the temperature is 20°C. The maximum temperature is 1430°C. Deter-
mine the thermal efficiency and the mean effective pressure for this cycle. Take g = 1.4.
Solution
Refer to Fig. 9.8.
v1
r= = 16, p1 = 1 bar, T1 = 20 + 273 = 293 K
v2
T3 = 1430 + 273 = 1703 K
g −1
T2  v1 
= = rg −1 = 160.4 = 9.03
T 1  v2 
T2 = 293 × 9.03 = 887.79 K
v3 T3 1703
Process 2 – 3:       r = = = = 1.918
v1 T2 887.79

1 r g − 1 1 1.9181.4 − 1
h th = 1 −   =1−  
g −1 r − 1
g r   1.4 × 160.4  1.918 − 1 
= 0.618 = 61.8%
g rg −1 ( r − 1) − ( r g − 1) 
pm = p1r  
 ( r − 1)(g − 1) 
1.4 × 160.4 (1.918 − 1) − (1.9181.4 − 1) 
= 1 × 16   = 6.414 bar
 (16 − 1)(1.4 − 1) 

Example 9.14
An engine works on dual combustion cycle, the compression ratio being 11. The pressure at the com-
mencement of combustion is 1 bar and the temperature is 90°C. The maximum pressure in the cycle
1
is 50 bar and the constant pressure heat addition continues for of the stroke. Calculate the work
20
done per kg of air and the ideal thermal efficiency. Assume, cv = 0.718 kJ/kgK, cp=1.005 kJ/kgK.
Solution
cp 1.005
     g = = = 1.4
cv 0.718
T1 = 90 + 273 = 363 K
v
Refer to Fig. 9.21.      r = 1 = 11
v2
vs
       = v1 − v2 = 10 v2
1
     v4 − v3 = (v − v )
20 1 2

M09_THERMAL ENGINEERING_SE_XXXX_CH09.indd 451 6/15/2018 4:14:42 PM


452 Chapter 9

p
3 4
50
qs
qs

2
5
bar qr

1 1

0 v2 v4 v1 v

Figure 9.21

v2  v1 
      = −1
20  v2 
v
  = 2 (11 − 1)
20
v v
  = 2= 3
2 2
v4 1 v 3
− 1 = or 4 = = 1.5
v3 2 v3 2
Process 1 – 2: T2 = T1r 0.4 = 363 × 110.4 = 947.24 K
p2 = p1rg = 1 × 11 = 28.704 bar
p3 50
Process 2 – 3: T3 = T2 = 947.24 × = 1650 K
p2 28.704
 v4 v4 
Process 3 –34–:
Process 4:  3v v =1650
T4 T=4T=3 T × 1×
= 1650 .51=.52475
= 2475
KK
3 3
g −1
v 
T5 = T4  4 
Process 4 – 5: = 1119.46 K
 v5 
  w = qs − qr = cv (T3 – T2) + cp (T4 – T3) - cv (T5 – T1)
         =  [0.718(1650 – 947.24) + 1.005(2475 – 1650)] – 0.718
                  

(1119.46 – 363)
  = 1339.70 – 360.26 = 979.4 kJ/kg
979.4
    h th = w / qs = = 0.7298 = 72.98%
1339.70
Example 9.21
A gas turbine operates on Brayton cycle with in takes air at 1 bar and 15°C. The air is compressed
to 15 bar and heated in a combustion chamber to 800°C. The hot air expands in the turbine to 1 bar.
Find (a) the power developed, (b) thermal efficiency of the cycle. Take g = 1.4 and cp = 1.005 kJ/kgK.

M09_THERMAL ENGINEERING_SE_XXXX_CH09.indd 452 6/15/2018 4:14:47 PM


Gas Power Cycles 453

Solution
Refer to Fig. 9.22
T1 = 15 + 273 = 288 K
0.286
 5
T2 = T1 ( rp )g −1/g = 288   = 456.14 K
 1
Compressor work,   wc = cp (T2 – T1) = 1.005(456.14 - 288) = 168.98 kJ/kg

T3 273 + 800
T4 = = = 677.47 K
g −1
(5)0.286
g
( rp )
Turbine work, wt = h3 - h4 = cp (T3 - T4)
   = 1.005(1073 – 677.47) = 397 .5 kJ/kg
Heat added, qs   = q2-3 = h3 - h2 = cp (T3 - T2)
   = 1.005 (1073 - 456.14) = 619.94 kJ/kg
Network,   w = wt - wc = 397.5 - 168.98 = 228.52 kJ/kg
w 228.52
=
   h th =
= 0.3686 or 36.86 %
qs 619.94
Power output    = 228.52 kJ/kg

Example 9.15
Prove that the optimum pressure ratio rp for maximum net-work done between the temperature T1
and T3 for a Brayton cycle is given by:
g
T  2(g −1)
rp =  3 
T  1
Solution
g −1
From Fig. 9.22, we have g −1
T3 T2  p2  g g
= = = ( rp ) = x (say)
T4 T1  p1 
T3 T3
Let
Let   =y =y
T1 T1
Net-work done/kg of air,
w = (h3 – h2) − (h4 – h1) = cp (T3 – T2) − cp (T4 – T1)
   = cp (T3 – T4) − cp (T2 – T1) = cp T3(1 – 1/x) − cp T1 (x − 1)
w T  1  1
or = 3 1 −  − ( x − 1) = y 1 −  − ( x − 1)
c pT1 T1  x   x
= y – y/x – x – 1
d  w 
  = 0 for maximum work done.
dx  c pT1 

M09_THERMAL ENGINEERING_SE_XXXX_CH09.indd 453 6/15/2018 4:14:50 PM


454 Chapter 9

Here y is constant and x is variable,


y
0+ − 1 = 0 or x = y
x2
g −1
g T3
( rp ) =
T1
g
 T  2(g −1)
rp =  3 
T  1

Example 9.16
An air-standard Brayton cycle has pressure ratio across the compression equal to 6. Air enters the
compressor at 1 bar and 27°C. The maximum temperature of the cycle is 850°C. Calculate the spe-
cific output of the cycle. What will be the power/developed by the unit for a mass flow rate of 10 kg/s.
For air, g = 1.4 and cp = 1.005 kJ/kg.K.

Solution
From Fig. 9.12, we have
p2
rp = = 6, T1 = 27 + 273 = 300 K
p1
p1 = 1 bar, T3 = 850 + 273 = 1123 K

T2 = T1 rp(g  −1)/g  = 300 × 61/9.5 = 500.8 K


g −1 g −1
T3  p3  g p  g
=  = 2 = 61/ 3.5 = 1.669
T4  p4   p1 
T4 = 1123/1.669 = 672.86 K
Specific output,  w = cp (T3 – T2) – cp(T4 – T1)
    = 1.005 [(1123 – 500.8) – (672.86 – 300)] = 250.58 kJ/kg
Power developed, P = mw = 10 × 250.58 = 2505.8 kW

Example 9.17
In an I.C. engine operating on the dual cycle (limited pressure cycle), the temperature of the work-
ing fluid (air) at the beginning of compression is 27°C. The ratio of the maximum and minimum
pressures of the cycle is 70 and the compression ratio is 15. The amounts of heat added at constant
volume and at constant pressure are equal. Compute the air standard thermal efficiency of the cycle.
[IES, 1993]

Solution
The p-v diagram for the dual cycle is shown in Fig. 9.23.
T1 = 273 + 27 = 300 K

M09_THERMAL ENGINEERING_SE_XXXX_CH09.indd 454 6/15/2018 4:14:52 PM


Gas Power Cycles 455

p3 p4 v v
= = 70, 1 = 1 = 15
p1 p1 v2 v3
q2 −3 = q3− 4

g −1
T2  v1 
= = (15)0.4 = 2.954
T1  v2 

T2 = 300 × 2.954 = 886.25 K

g
p2  v1 
= = 151.4 = 44.31
p1  v2 

p3 p3 p1 70
= × = = 1.58
p2 p1 p2 44.31
p3
Process 2 – 3: T3 = T2 × = 886.25 × 1.58 = 1400 K
p2

Now cv (T3 − T2 ) = c p (T4 − T3 )


T3 − T2 1400 − 886.25
∴ T4 = T3 + = 1400 + = 1767 K
g 1.4
v4 T4 v × 1767 v1 1767
Process 3 – 4: = , v4 = 3 = × = 0.0841 v1
v3 T3 1400 15 1400
v5 = v1
Also   
g −1 0.4
T4  v5   v1 
Process 4 – 5: = = = 2.6915
T3  v4   0.0841 v1 
1767
T5 = = 656.5 K
2.6915

p
3 4

Figure 9.23  Dual cycle on p-v diagram

M09_THERMAL ENGINEERING_SE_XXXX_CH09.indd 455 6/15/2018 4:14:58 PM


456 Chapter 9

qr cv (T5 − T1 )
Thermal efficiency, h th = 1 − =1−
qs cv (T3 − T2 ) + c p (T4 − T3 )

T5 − T1
=1−

(T3 − T2 ) + g (T4 − T3 )

656.5 − 300
=1− = 0.653 or 65.3%
(1400 − 886.25) + 1.4(1.767 − 1400)

Example 9.18
In an air-standard Brayton cycle the minimum and maximum temperatures are 300 K and 1200 K,
respectively. The pressure ratio is that which maximizes the network developed by the cycle per
unit mass of air flow. Calculate the compressor and turbine work, each in kJ/kg air, and the thermal
efficiency of the cycle.  [IES, 1994]
Solution
With reference to Fig. 9.25, Tmax = 1200 K and Tmin = 300 K
For maximum work, optimum pressure ratio,
g 1.4
T  2(g −1) 1200  0.8
( rp )opt =  max  =  = 11.314
 Tmin   300 
g −1
g
T2 = T1 ( rp ) = 300 (11.314)0.4 /1.4 = 600 K
Compressor work,
wc = c p (T2 − T1 )
= 1.005(600 – 300) = 301.5 kJ/kg
T3 1200
T4 = = = 600 K
g −1
(11.314)0.4 /1.4
( rp ) g
Turbine work, wt = c p (T3 − T4 ) = 1.005 (1200 − 600) = 603 kJ/kg
Heat supplied, Qs = c p (T3 − T2 ) = 1.005 (1200 − 600) = 603 kJ/kg
w − wc 603 − 301.5
Thermal efficiency, h th = t = = 0.5 or 50%
Qs 603
T 3

4
1

Figure 9.25  Brayton cycle on T-s diagram

M09_THERMAL ENGINEERING_SE_XXXX_CH09.indd 456 6/15/2018 4:15:02 PM


Gas Power Cycles 457

Example 9.19
Derive an expression for air standard efficiency of the following cycle in terms of compression
ratio R, cv and g .
  (a)  an isothermal compression, compression ratio, r
    (b)  an increase of pressure at constant volume.
  (c)  an adiabatic expansion.  [IES, 1997]

Solution
The p-v diagram is shown in Fig. 9.28
Process 1 – 2: Isothermal compression
v1
W1− 2 = Q 1− 2 = mRT1 ln = mRT1 ln r
v2
Process 2 – 3: Constant volume process
3
W2 −3 = ∫ p ⋅ dv = 0
2
Q2 −3 = U 3 − U 2 = mcv (T3 − T2 ) = mcv (T3 − T1 )
Process 3 – 1: Adiabatic expansion
W3−1 = mcv (T3 − T1 )
Q3−1 = 0
Air standard efficiency,
Q + Q3−1 mc (T − T ) + 0 c (T − T )
h = 1 − 2 −3 =1− v 3 1 = 1− v 3 1
Q2 −1 mRT1 ln r RT1 ln r
g −1 g −1
v  v 
Now T3 = T1  1  = T1  1  = T1rg −1
 v3   v2 

C ( rg −1 − 1) Crg (−r1g −−11 − 1) rg −1 − 1


h =1− v

∴ h==11−− v =1−
R ln r (g −R 1)lnlnrr (g − 1) ln r

p
3
pv = c
v=c

T=c 1

v2 v1 v

Figure 9.28  p-v diagram

M09_THERMAL ENGINEERING_SE_XXXX_CH09.indd 457 6/15/2018 4:15:06 PM


458 Chapter 9

Example 9.20
An air standard Otto cycle has a volumetric compression ratio of 6, the lowest cycle pressure of 0.1
MPa and operates between temperature limits of 27°C and 1569°C.
(a) Calculate the temperature and pressure after the isentropic expansion (Ratio of specific heats = 1.4).
(b) Since it is observed that values in (a) are well above the lowest cycle operating conditions, the
expansion process was allowed to continue down to a pressure of 0.1 MPa. Which process is
required to complete the cycle? Name, the cycle so obtained.
(c) Determine by what percentage the cycle efficiency has been improved. [GATE 1994]

Solution
The p-v diagram is shown in Fig. 9.29.

v1
=6
v2
p1 = 0.1 MPa

T1 = 27 + 273 = 300 K

T3 = 1569 + 273 = 1842 K


g
v 
(a)  p2 = p1  1  = 0.1 × 61.4 = 1.2286 MPa
v 
2
g −1
v 
T2 = T1  1  = 300 × 60.4 = 614.3 K
 v2 
p T 1.2286 × 1842
p3 = 2 3 = = 3.684 MPa
T2 614.3
g g
v  v  3.684
p4 = p4  3  = p3  2  = = 0.3 MPa
 v4   v1  61.4

p
3 pv = c

2
4

Figure 9.29  Otto cycle

M09_THERMAL ENGINEERING_SE_XXXX_CH09.indd 458 6/15/2018 4:15:10 PM


Gas Power Cycles 459

p  0.3
T4 = T1  4  = 300 × = 900 K
 p1  0.1
(b) Constant pressure scavenging is required to complete the cycle with expansion process to con-
tinue down to 0.1 MPa as shown in Fig. 9.30. The cycle is called Atkinson cycle.
1 1
(c) hOtto = 1 − =1− = 0.5116 or 51.16%
g −1 0.4
r 6
g (T5 − T1 )
h Atkinson = 1 −
T3 − T2
g −1 0.4
p  g  0.1  1.4
T5 = T3  5  = 1842  = 657.3 K
 p3   3.684 
1.4(657.3 − 300)
h Atkinson = 1 − = 0.5925 or 59.25%
(1842 − 614.3)
Improvement in cycle efficiency = 59.25 – 51.16 = 8.09%

p
3

2 4

1 5

Figure 9.30  Atkinson cycle

Example 9.21
Air enters the compressor of a gas turbine plant operating on Brayton cycle at 1 bar, 27°C. The
pressure ratio in the cycle is 6. If Wt = 2.5 Wc where Wt and Wc  are the turbine and compressor
work respectively, calculate the maximum temperature and tin cycle efficiency.  [GATE, 1996]

Solution
The Brayton cycle is shown in Fig. 9.31
p1 = 1 bar =1 × 105 N/m 2

T1 = 27 + 273 = 300 K
p2
= 6, p2 = 6 × 105 N/m 2
p1

M09_THERMAL ENGINEERING_SE_XXXX_CH09.indd 459 6/15/2018 4:15:13 PM


460 Chapter 9

T p2
3

2 p1

Figure 9.31  Brayton cycle

g −1
T2  p2  g
=
T1  p1 

T2 = 300 × 60.4 /1.4 = 500.55 K


g −1
T3  p2  g
= = 60.4 /1.4 = 1.6685
T4  p1 
Wc = mc p (T2 − T1 )
Wt = mc p (T3 − T4 )
mc p (T3 − T4 ) = 2.5 mc p (T2 − T1 )
T3 − T4 = 2.5 (T2 −T1)
 1 
T3 1 − = 2.5 (500.55 − 300)
 1.6685 
T3 = 1251.4 K, T4 = 750 K
Wt − Wc (T − T ) − (T2 − T1 )
Cycle efficiency = = 3 4
mc p (T3 − T2 ) T3 − T2

(1251.4 − 750) − (500.55 − 300) 501.4 − 200.55


= =
1251.4 − 500.55 750.85
= 0.4007 or 40.07%

Example 9.22
The minimum pressure and temperature in an Otto cycle are 100 kPa and 27°C. The amount of heat
added to the air cycle is 1500 kJ/kg. Determine the pressure and temperatures at all points of the
air standard Otto cycle. Also calculate the specific work and the thermal efficiency of the cycle for
a compression ratio 8:1. (Take cv (air) = 0.72 kJ/kg K and (cp/cv )air = 1.4). [GATE, 1998]

M09_THERMAL ENGINEERING_SE_XXXX_CH09.indd 460 6/15/2018 4:15:18 PM


Gas Power Cycles 461

Solution
The Otto cycle is shown in Fig. 9.32

p
3

2
4

Figure 9.32  Otto cycle

p1 = 100 kPa, T1 = 27 + 273 = 300 K

q2 −3 = 1500 kJ/kg, r = 8
T2 g −1
= r , T2 = 300 × 80.4 = 689.22 K
T1
p2 = p1 rg = 100 × 81.4 = 1837.917 kPa
q2 −3 = cv (T3 − T2 )
1500 = 0.72 (T3 – 689.22)
T3 = 2772.55 K
T3 2772.55
p3 = p2 = 1837.917 ×
T2 689.22
= 7393.46 kPa
T3 2772.55
T4 = = = 1206.82 K
g −1
r 80.4
p3 7393.46
p4 = = = 402.274 kPa
g
r 81.4
1 1
h th = 1 − =1− = 0.5647 or 56.47%
g −1 0.4
r 8
q4 −1 = cv (T4 − T1 ) = 0.72 (1206.82 − 300) = 652.91 kJ/kg
Specific work, wnet = q2 −3 − q4 −1 = 1500 − 652.91 = 847.09 kJ/kg

M09_THERMAL ENGINEERING_SE_XXXX_CH09.indd 461 6/15/2018 4:16:11 PM


462 Chapter 9

Example 9.23
A Brayton cycle (air standard) has a pressure ratio of 4 and inlet conditions of one standard atmos-
pheric pressure and 27°C. Find the air flow rate required for 100 kW power output if the maximum
temperature in the cycle is 1000°C. Assume g = l.4 and cp = l.0 kJ/kg K. [GATE, 2001]

T p2
3

2 p1

Figure 9.33  Brayton cycle


Solution
Refer to Fig. 9.33.
p2
= 4, p1 = 1 atm,
p1
T1 = 27 + 273 = 300 K

T3 = 1000 + 273 = 1273 K


g −1
p  g
T2 = T1  2  = 300 × 40.4 /1.4 = 445.8 K
 p1 
g −1
T3 g
=r
T4 p
1273
T4 = = 856.66 K
40.4 /1.4
Wnet = Wt − Wc
 p [(T3 – T4) − (T2 – T1)]
= mc
= m × 1 [(1273 – 856.66) – (445.8 – 300)]
= 270.5345 × m kW = 100
m = 0.3698 kg/s

M09_THERMAL ENGINEERING_SE_XXXX_CH09.indd 462 6/15/2018 4:16:15 PM


Gas Power Cycles 463

Example 9.24
Calculate the percentage loss in the ideal efficiency of a diesel engine with compression ratio 14 if
the fuel cut-off is delayed from 5% to 8%. [GATE, 2000]

Solution
The diesel cycle is shown in Fig. 9.34
Compression ratio, r = 14, Cut-off ratio, r = 0.05
Efficiency,
1  r g − 1
h1 = 1 −  
g −1
r g  r − 1 

1  0.051.4 − 1 
=1 −  
140.4 × 1.4  0.05 − 1 
= 0.7423
For r = 0.08
1  r g − 1
h2 = 1 −  
rg −1g  r − 1 
1  0.081.4 − 1
=1−  
140.4 × 1.4  0.08 − 1 
= 0.7377
Change in efficiency
 0.7423 − 0.7377 
=  = 0.006197 or 0.6197%
 0.7423 

p
2 3

4
1

vc v
v2 v3 v1 = v4

Figure 9.34  Diesel cycle

Example 9.25
A reciprocating engine works on Otto cycle where the temperature at the beginning and end of the
compression stroke are found to be 40°C and 360°C respectively. Assuming the ratio of specific

M09_THERMAL ENGINEERING_SE_XXXX_CH09.indd 463 6/15/2018 4:16:17 PM


464 Chapter 9

heats at constant pressure and constant volume to be 1.4, determine the air standard efficiency and
the compression ratio. [IES 1984]
Solution
The p-v diagram is shown in Fig. 9.35
T1 = 273 + 40 = 313 K
T2 = 273 + 360 = 633 K
Process 1 – 2: 1
v  T  g −1
Compression ratio, r= 1 = 2
v2  T1 
0.4
 633 
     =  = 5.816
 313 
v3 = v2 and v4 = v1
Also    
v4 v1
= =r
v3 v2
g −1
T4  1  T1 T T
=  = or 4 = 3
T3  r  T2 T1 T2
Heat added − Heat rejected
Air-standard efficiency =
Heat added
cv (T3 − T2 ) − cv (T 4 − T1 ) T −T T
    = =1− 4 1 =1− 1
cv (T3 − T2 ) T3 − T2 T2
1 1
  = 1− =1− = 0.5055 or 50.55%
g −1
r (5.816)0.4

p
3 pvg = c
v=c

2
4
v=c
1

Figure 9.35  p-v diagram for Otto cycle

M09_THERMAL ENGINEERING_SE_XXXX_CH09.indd 464 6/15/2018 4:16:21 PM


Gas Power Cycles 465

Example 9.26
An engine working on the ideal Otto cycle takes in air at 1 bar and 30°C which is compressed to
15 bar at the end of the compression stroke. The temperature attained at the end of constant volume
heat addition is 900°C. Assuming adiabatic index to be 1.4, determine (a) the compression ratio,
(b) the thermal efficiency, (c) the temperature at the end of compression, (d) the pressure at the end
of constant volume heating, and (e) the mean effective pressure.  [IES 1986]

Solution
The p-v diagrams are shown in Fig. 9.37.
p1 = 1 bar, T1 = 273 + 30 = 303 K

p2 = 15 bar, T3 = 273 + 900 = 1173 K,g = 1.4


(a) Compression ratio,
1 1
v  p  g  1.5  1.4
r = 1 =  2  =   = 6.92
v2  p1   1
(b) Thermal efficiency,
1
h th = 1 −
g −1
r
1
=1 − = 0.5387 or 53.87%
(6.92)0.4

(c) T2 = T1rg −1 = 303 × (6.92)0.4 = 656.86 K

T3 1173
(d) p3 = p2 = 15 × = 26.8 bar
T2 656.88
(e) Mean effective pressure,
Work done w
pm = =
Swept volume vs

p
3 pvγ = c

2
4

Figure 9.37  p – v diagram for Otto cycle

M09_THERMAL ENGINEERING_SE_XXXX_CH09.indd 465 6/15/2018 4:16:24 PM


466 Chapter 9

w = cv [(T3 − T2 ) − (T4 − T1 )]

g −1 g −1
v  v  T3 1173
T4 = T3  3  = T3  2  = = = 541.1 K
g −1
 v4   v1  r (6.92)0.4

T T   303 656.88  −2
vs = v1 − v2 = R  1 − 2  = 0.287  −  × 10
 1
p p2  1 15
= 0.74392 m3/kg
w = 0.781[(1173 − 656.88) − (541.1 − 303)] = 199.618 kN m/kg

199.618
    pm = = 268.33 kPa
0.74392

Example 9.27
The diameter and stroke of a gas engine cylinder are 18 cm and 30 cm respectively. The ratio of
expansion is 5. The pressure and temperature of the mixture at the beginning of compression are 1
bar and 100°C respectively. Find the index of the compression process and the weight of the mix-
ture in the cylinder, if the pressure at the end of compression is 7 bar. Also calculate the work done
and heat transferred during the process, indicating the direction of flow. Assume R = 0.287 kJ/kg K
and ratio of specific heats equal to 1.4 for the mixture. Take 1 bar = 100 kPa.  [IES, 1987]

Solution
The p-V diagram is shown in Fig. 9.38
V4 V1
d = 18 cm, L = 30 cm, re = 5 = =
V3 V2
p1 = 1 bar, T1 = 100 + 273 = 373 K

p2 = 7 bar, g = 1.4, R = 0.287 kJ/kg.K

p
3 pvg = c
v=c

2
4
v=c
1

Figure 9.38

M09_THERMAL ENGINEERING_SE_XXXX_CH09.indd 466 6/15/2018 4:16:27 PM


Gas Power Cycles 467

p 2 p
V1 = d L = × 182 × 30 = 7634 cm3
4 4
p1V1 1 × 10 2 × 7634 × 10 −6
m= =
RT1 0.287 × 373
= 71.312 × 10−4 kg
n
p2  V1 
=
p1  V2 
p V
ln 2 = n ln 1
p1 V2
 7
ln  
 1
n=
  = 1.209
ln 5
 7634  −6 2
1 × 7634 − 7 ×  × 10 × 10
p1V1 − p2V2  5 
Work done = = = − 1.461 kJJ
n −1 1.209 − 1
T1 373
T2 = = = 522.16 K
n −1 0.209
 p1  n  1  1.209
 p   
2
7

 g − n
Heat added, Q = mcv  (T − T )
 1 − n  2 1

R 0.287
Now cv = = = 0.718 kJ/kg.K
g − 1 0.4
 1.4 − 1.209 
Q = 71.312 × 10 −2 × 0.718 ×  (522.16 − 373)
 1 − 1.209 
= − 0.6979 kJ/cycle
− ve sign indicates that heat is rejected by the system.

Example 9.28
A four-stroke limited pressure cycle (diesel) engine draws 1.2 kg/s of air at 1 atm and 27°C. Com-
pression ratio of the cycle is 16. Pressure ratio during constant volume heat addition is 2.0. Total
heat added is equal to 2310 kJ/kg of air in the cylinder. Determine
(a) pressure, volume and temperature at all salient points,
(b) % of heat added during constant pressure process,
(c) cut-off ratio,
(d) thermal efficiency, and
(e) mean effective pressure.

M09_THERMAL ENGINEERING_SE_XXXX_CH09.indd 467 6/15/2018 4:16:31 PM


468 Chapter 9

Represent the cycle on p-v and T-s planes.


Assume cp = 1.005 kJ/kg.K and cv = 0.718 kJ/kg.K  [IES. 1988]

Solution
The p-v and T-s diagrams are shown in Fig. 9.39.
p1 = 1 atm =1.03 × 105 Pa
T1 = 27 + 273 = 300 K
p3 v
= 2, r = 1 = 16, ma = 1.2 kg/s
p2 v2
q = 2310 kJ/kg
R = c p − cv = 1.005 − 0.718 = 0.287 kJ/kg.K
cp 1.005
g = = = 1.4
cv 0.718
RT1 0.287 × 300
(a) v1 = = = 0.8359 m3 / kg
p1 1.03 × 10 2
g
v 
p2 = p1  1  = p1rg = 1.03 × 10 2 × 161.4 = 49.958 × 10 2 kPa
v  2
g −1 0.4
p  g  49.958  1.4
T2 = T1  2  = 300  = 909.43 K
 1
p  1.03 
v 0.8359
v2 = 1 = = 0.0522 m3 /kg
16 16
p3 = 2 p2 = 2 × 49.958 × 10 2 = 99.916 × 10 2 kPa

p T
p=c
3 4 3 p=c 4
v=c pvg = c
c s=c
2 v=
2 5
s=c
5 c
v=c 1 v=
1

v s
(a) (b)

Figure 9.39 Limited pressure cycle (diesel) 4-stroke engine:


(a) p-v diagram, (b) T-s diagram

M09_THERMAL ENGINEERING_SE_XXXX_CH09.indd 468 6/15/2018 4:16:35 PM


Gas Power Cycles 469

v3 = v2 = 0.522 m3 / kg
p3
T3 = T2 = 909.43 × 2 = 1818.86 K
p2
Heat added, q = cv (T3 − T2 ) + c p (T4 − T3 )
  2310 = 0.718 (1818.86 – 909.43) + 1.005 (T4 – 1818.86)
T4 = 3467.64 K
T4 3467.64
v4 = v3 = 0.0522 × = 0.09952 m3 / kg
T3 1818.86
g −1 0.4
v   0.09952 
T5 = T4  4  = 3467.64  = 1480.25 K
 0.8359 

v 5
1 1 1
 v g  v g  0.09952  1.4
p5 = p4  4  = p3  4  = 99.916 × 10 2 ×  
 5
v  1
v  0.8359 
= 21.85 × 102 kPa
(b) Heat added during constant pressure process,
q p = c p (T4 − T3 ) = 1.005 (3467.64 − 1818.86) = 1657.024 kJ/kg

1657.024
Percentage of heat added = = 0.7173 or 71.73%
2310
v4 0.09952
(c) Cut-off ratio, r= = = 1.906
v3 0.0522
Heat added − Heat rejected
(d) Thermal efficiency, h th =
Heat added
Heat rejected during process 5 – 1
= cv (T5 − T1 ) = 0.718(1480.25 − 300) = 847.42 kJ/kg

2310 − 847.42
h th = = 0.6331 or 63.31%
2310
(e) Mean effective pressure,
Work done 2310 − 847.42
pm = =
Swept volume 0.8359 − 0.0522
= 1866.25 kPa or 18.662 bar

Example 9.29
A reversible cycle using an ideal gas as the working substance consists of an isentropic compression
from an initial temperature to 555 K, a constant volume process from 555 K to 835 K, a reversible

M09_THERMAL ENGINEERING_SE_XXXX_CH09.indd 469 6/15/2018 4:16:40 PM


470 Chapter 9

adiabatic expansion to 555 K, a constant pressure expansion from 555 K to 835 K, followed by
constant volume process to the initial temperature. Draw the cycle on p-v amd T-s diagrams and
determine the initial temperature. Take g = 1.40. Also compute the work done. [IES, 1989]

Solution
The p-v and T-s diagrams are shown in Fig 9.40 (a) and (b) respectively.
1
2.5
v2  T1  g −1  T1  v3
Process 1 – 2: = = =
v1  T2   555  v5
1
v3  T3  g −1  835  2.5
Process 3 – 4: = = = 2.7764
v4  T4   555 
v4 T4 555
Process 4 – 5: = = = 0.66467
v5 T5 835
v3 v3 v4
Now = × = 2.7764 × 0.66467 = 1.8454
v5 v4 v5
2.5 2.5
 T1   T1 

∴  555   = 1.8454
= 1.8454
555
T1 = 709.13K
R R
Work done = (T − T ) + 0 + (T − T ) + R (T 5 − T4 ) + 0
(g − 1) 1 2 (g − 1) 3 4
0.287 0.287
       = (709.13 − 555) + (835 − 555) + 0.287 (835 − 555)
0.4 0.4
= 110.5883 + 200.9 + 80.36 = 391.848 kJ/kg

p T
pvg = c 3 5
3
s=c
c c
4 p=c v= p=
v=c 5 2
4
2
v=c
s=c
v=c
pvg =c
1
1

v s
(a) (b)

Figure 9.40  (a) p-v diagram, (b) T-s diagram

M09_THERMAL ENGINEERING_SE_XXXX_CH09.indd 470 6/15/2018 4:16:43 PM


Gas Power Cycles 471

Fill in the Blanks


1. The thermal efficiency of the Otto cycle_______as compression ratio increases.
2. The compression ratio of Otto cycle lies between_______.
3. The compression ratio is defined as the ratio of_______volume to_______volume.
4. The cylinder volume is the sum of_______volume and_______volume.
5. The Otto cycle is a_______cycle.
6. The heat addition and rejection in the Otto cycle take place at_______.
7. The heat addition in diesel cycle take place at constant_______.
8. The thermal efficiency of diesel cycle_______as cut off ratio increases.
9. The thermal efficiency of dual cycle equals that of Otto cycle at_______ equal to unity.
10. The thermal efficiency of dual cycle equals that of diesel cycle at_______equal to unity.
11. The heat addition and rejection in Brayton cycle take place at constant_______ .
12. For equal compression ratio and heat input, thermal efficiency of Otto cycle is_______ than diesel and dual
cycles.
13. For constant maximum pressure and heat input, thermal efficiency of diesel cycle is _______than dual and
Otto cycles.
14. A cycle is said to be more efficient which rejects_______amount of heat.

ANSWERS
1. increases 2. 5 to 8 3. cylinder, clearance 4. clearance, swept
5. constant volume 6. constant volume 7. pressure 8. decreases
9. cut-off ratio 10. pressure ratio 11. pressure 12. more
13. more 14. least.

True or False
State true (T) or false (F)
1. Displacement volume is the volume swept by the piston.
2. The Otto cycle is a constant volume cycle.
3. The compression ratio of Otto cycle varies from 5 to 8.
4. The Otto cycle is used in diesel engine.
5. The thermal efficiency of Otto cycle depends on compression ratio only.
6. The thermal efficiency of Otto cycle decreases as compression ratio increases.
7. The heat addition in diesel cycle is at constant pressure.
8. The cut-off ratio of a diesel engine is of the order of 2.
9. The thermal efficiency of diesel engine increases as cut-off ratio increases.
10. The thermal efficiency of a dual cycle lies between Otto and diesel cycle.
11. Joule cycle is used in a gas turbine.
12. Brayton cycle is a constant pressure cycle.

ANSWERS
1. T 2. T 3. T 4. F 5. T
6. F 7. T 8. T 9. F 10. T
11. T 12. T

M09_THERMAL ENGINEERING_SE_XXXX_CH09.indd 471 6/15/2018 4:16:43 PM


472 Chapter 9

Multiple-choice Questions
1. The Otto cycle is also known as: (c) mixture of fuel and air is sucked
(a) constant pressure cycle (d) mixture of air and lubricating oil is sucked
(b) constant temperature cycle
11. The thermal efficiency of petrol engine as com-
(c) constant volume cycle
pared to diesel engine is:
(d) constant entropy cycle
(a) higher
2. The efficiency of Diesel cycle approaches that of (b) lower
Otto cycle when cut-off is: (c) same for same power output
(a) increased (b) decreased (d) same for same speed
(c) constant (d) zero
12. The air standard Otto cycle consists of
3. For the same maximum pressure and temperature (a) two constant pressure and two constant
of the Otto, Diesel and Dual air standard cycles, entropy processes
what will be same for the engine? (b) two constant volume and two constant
(a) compression ratio (b) heal supplied entropy processes
(c) thermal efficiency (d) heat rejected (c) two constant pressure and two constant vol-
4. In a four stroke engine, a working cycle is com- ume processes
pleted in following revolutions of crankshaft: (d) two constant pressure and two constant tem-
(a) one (b) two perature processes
(c) three (d) four. 13. The variation of work output v’s pressure ratio for
5. The air standard cycle on which the petrol engine Otto cycle with fixed compression ratio and adia-
works, is: batic index is shown below. The correct choice is:
(a) Otto cycle (b) Carnot cycle (a) A (b) B
(c) Joule cycle (d) Dual cycle (c) C (d) D
6. The compression ratio in a petrol engine is of the
order of :
C
(a) 5 to 8 (b) 10 to 15
Thermal efficiencyWork output Work output

(c) 15 to 25 (d) 25 to 30
D
7. A carburetor mixes in a petrol engine B
(a) petrol and air A
(b) diesel and air C
(c) petrol and lubricating oil
(d) petrol, lubricating oil and air Pressure
D ratio
B ciency v’s compres-
14. The variation of thermal effi
8. The compression ratio in a diesel engine is of the D
A value of pres-
sion ratio for Otto cycle with fixed
order of
C
(a) 5 to 8 (b) 10 to 15 sure ratio and adiabatic index is shown below. The
(c) 15 to 25 (d) 25 to 30 correct choice is: B
Pressure ratio
9. Which of the following does not relate to spark A D
Thermal efficiency

ignition engine?
C
(a) ignition coil (b) spark plug
(c) distributor (d) fuel injector B
Compression ratio
10. In a four stroke cycle diesel engine, during suc- A
tion stroke
(a) only air is sucked
(b) only fuel is sucked Compression ratio

M09_THERMAL ENGINEERING_SE_XXXX_CH09.indd 472 6/15/2018 4:16:44 PM


Gas Power Cycles 473

(a) A (b) B 17. Match the following:


(c) C (d) D Air standard cycle T-s diagram
15. In a diesel cycle, the thermal efficiency for a fixed (a) Otto cycle 1.
compression ratio and adiabatic index T
(a) increases as cut-off ratio increases
(b) decreases as cut-off ratio increases
(c) remains same with increase in cut-off ratio
(d) none of the above.

16. Match the following s


Air-standard cycle p-v diagram (b) Diesel cycle 2.
(a) Otto cycle 1. T
p
p=c
p=c

s
v
(c) Carnot cycle 3.
(b) Diesel cycle 2. T
p

s
v (d) Brayton cycle 4.
T
(c) Carnot cycle 3. c
p p=

c
v=

v Codes:
   A  B  C  D
(d) Joule cycle. 4. (a)  3    1   2   4
p (b)  3  1   4   2
(c)  1    3   4   2
(d)  1  3   2   4
18. Match the following:
p-v diagrams T-s diagrams
(a)
p v p 1. T 1. T
Codes: c c
   A  B  C  D p= p=
(a)  2  3   1   4
(b)  2  3   4   1 c c
v= v=
(c)    3   
2   4   1
(d)  3  2   1   4 v v    s s

M09_THERMAL ENGINEERING_SE_XXXX_CH09.indd 473 6/15/2018 4:16:48 PM


474 Chapter 9

(b) T 3 T 2 T
p p c
2. T 2. T p= 3 2
2
4 1
c
1 p= 4 1
s s
v v s s (a) (b) (c)
  
(c) T 3 T 2 T 3 T
p p c 3. T 3. T 2 3
p= 3 2
2
1 4
4
p =c 4 1 1 4
1
s s s s
v v (a)  (b) s s (c) (d)
(d)
20. Consider the following four cycles on T-s plane:
p p 4. T 4. T T T T
c c
v= v= c c
v= p=

c c
v= v= c
c v=
v=
v v    s s
s s
Codes: I II III
   A  B  C  D
T T T T
(a)    4   3  
c 2  1 c c
=
v 3   1  2 p= p=
(b)    4  
(c)    
3   4   1  2
(d)    3   4   c c
c 2  1 v= p=
v=
19. A system comprising of as pure substance exe- s s s
cutes reversibly aI cycle 1 – 2 − 3 – 4 −1 consist-
II III IV
ing of two isentropic and two isochoric process The correct sequence is:
as shown in figure below: (a) Otto, Diesel, Carnot, Joule
p
(b) Otto, Diesel, Joule, Carnot
3 (c) Diesel, Otto, Carnot, Joule
(d) Diesel, Otto, Joule, Carnot
4
2 21. In an air-standard Otto cycle, r is the compression
ratio and g is an adiabatic index, the air standard
1
efficiency is given by
v (a) h = 1 −1 / r (g −1)
Which one of the following is the correct rep- (b) h = 1 −1 / rg
resentation of this cycle on the temperature-en-
(c) h = 1 −1 / r (g −1)/g
tropy coordinates?
(d) h = 1 −1 / r (g −1)/ 2g

M09_THERMAL ENGINEERING_SE_XXXX_CH09.indd 474 6/15/2018 4:16:54 PM


Gas Power Cycles 475

22. The order of values of thermal efficiency of Otto, Codes:


Diesel, and Dual cycles, when they have equal com-    A  B  C  D
pression ratio and heat rejection, ratio is given by 4   1   2
(a)  3   
(a) hOtto >hdiesel >hdual (b)  1  4   3   2
(c)  3   
2   1   4
(b) hdiesel >hdual >hOtto (d)  1  2   3   4
(c) hdual >hdiesel >hOtto 25. Otto cycle efficiency is higher than Diesel cycle
(d) hOtto >hdual >hdiesel efficiency for the same compression and heat
input because, in Otto cycle
23. Match List I with List II and select the correct ans- (a) combustion is at constant volume
wer using the codes given below the lists:
(b) expansion and compression air isentropic
List I List II (c) maximum temperature is higher
(Cycles operating (Characteristic of cycle effi- (d) Heat rejection is lower
between fixed tem- ciency g)
perature limits) 26. Which one of the following cycles represent
A.  Otto cycle 1. h
 depends only on temper- Brayton cycle?
ature limits T T T
B.  Diesel cycle 2. h
 depends only on pres- p= v= c
c c p=
sure limits
C.  Carnot cycle 3. h
 depends on volume-com-
p= v=
pression ratio c c
c p=
D.  Brayton cycle 4. h
 depends on cut off ratio
s s
and volume compression
(a) (b) (c)
ratio
T
Codes: T T T
p=
   A  B  C  D v= c
c c p= c
(a)  3   
4   1   2 v=
(b)  1  4   3   2
p= v=
(c)  3   
2  c 1   4 c c
c p= v=
(d)  1  2   3   4
24. Match List I with List II ands select the correct ans- s s s
(a)
wer using the codes given below the lists: (b) (c) (d)

List I List II
27. T-s diagram for an air-standard cycle is shown
(Cycles operating (Dependence of
below
between fixed tem- efficiency)
T 3
perature limits)
A.  Carnot cycle 1.  cut-off ratio and com-
pression ratio
B.  Brayton cycle 2.  volume compression ratio
2 1
C.  Otto cycle 3. pressure ratio
D.  Diesel cycle 4. temperature limits s
5. specific heats of work- The same cycle on p-V diagram will be represented as
ing substances.

M09_THERMAL ENGINEERING_SE_XXXX_CH09.indd 475 6/15/2018 4:16:58 PM


476 Chapter 9

p p p 31. Ericsson cycle pconsists


2 2
of the following four
1 2 2 3 processes
3
(a) two isothermals and two isentropics
(b) two isothermals and two constant volumes
3
(c) two isothermals and two constant pressures
1 (d) two adiabatics and two constant pressures
3 1 1
32. Match List I with List II anil select the correct
v v v v
(a) (b) answers
(c) using the codes given
(d) below the lists
p 2 p 2 List I List II
2 3
3 A. Two constant volumes and two adi- 1. Ericsson
abatics
3
B. 
Two constant pressures and two 2. stirling
adiabatics
1 1
1
C. Two constant volumes and two iso- 3. Joule
v v v
(b) (c) (d) thermals
D. 
Two constant pressures and two 4. Otto
28. Consider the following statements regarding Otto isothermals
cycle Codes:
(a) It is not a reversible process.    A  B  C  D
(b) The efficiency can be improved by using a (a)  3   
4   2   1
working fluid of higher value of ratio of spe- (b)  3  4   1   2
cific heats. (c)  4   
3   1   2
(c) The practical way of increasing its efficiency (d)  4  3   2   1
is to increase the compression ratio.
33. Match List I (details of process of the cycle) with
(d) Carbureted gasoline engines working on Otto
List II (name of the cycle) and select the correct
cycle can work with compression ratios more
answer using the codes given below the lists:
than 12.
Of these statements: List I List II
(a) 1, 3, and 4 are correct A. Two isothermals and two adiabatics 1. Otto
(b) 1, 2 and 3 are correct
B. 
Two isothermals and two constant 2. Joule
(c) 1, 2 and 4 are correct
volumes
(d) 2, 3 and 4 are correct
C. Two adiabatics and two constant vol- 3. Carnot
29. For the same maximum pressure and heat input,
umes
the most efficient cycle is
(a) Otto cycle D. 
Two adiabatics and two constant 4 Stirling
(b) Diesel cycle pressures
(c) Brayton cycle Codes:
(d) Dual combustion cycle      A  B  C  D
30. Atkinson gas cycle has a special feature that (a)  4    3   1   2
(a) isentropic compression and expansion are of (b)  4    3    
2   1
equal volume (c)    3  4    
1    
2
(b) isentropic compression is smaller than expen- (d)    3  4    
2   1
sion 34. For maximum specific output of a constant vol-
(c) isentropic compression is longer than expan- ume cycle (Otto cycle)
sion (a) the working fluid should be air
(d) it employs shortest compression and expan- (b) the speed should be high
sion

M09_THERMAL ENGINEERING_SE_XXXX_CH09.indd 476 6/15/2018 4:16:59 PM


Gas Power Cycles 477

(c) suction temperature should be high (c) Fig. (b)


(d) temperature of the working fluid at the end of v
compression and expansion should be equal
35. For a Brayton cycle, match the following using
U
codes given below:
List I List II
A. Compression 1. Isothermal s
B. Rejection 2. Isobaric (d) Fig. (d)
v
C. Expansion 3. Insentopic
D. Heat addition 4. Constant enthalpy
U
Codes:
     A  B  C  D
(a)  1    2   3   4
(b)  4    3    
2   1 s
(c)    2  3    
4    
1
38. For constant maximum pressure and heat input,
(d)    3  4    
1   2
the air standard efficiency of gas power cycle is in
36. For the same maximum pressure and temperature the order
(a) Otto cycle is more efficient than diesel cycle (a) Diesel cycle, dual cycle, Otto cycle
(b) Diesel cycle is more efficient than Otto cycle (b) Otto cycle, Diesel, cycle, dual cycle
(c) Dual cycle is more efficient than Otto and (c) Dual cycle, Otto cycle, Diesel cycle
diesel cycle (d) Diesel cycle. Otto cycle, dual cycle
(d) Dual cycle is less efficient than Otto diesel 39. The use of regenerator in a gas turbine cycle
cycles. (a) increases efficiency but has no effect on out-
37. A Otto cycle on internal energy (U) and entropy put
(s) diagram is shown in (b) increase output but has no effect on effi-
(a) Fig. (a) ciency
v (c) increase both efficiency and output
(d) increases efficiency but decreases output
40. Consider the following statements with reference
U to gas turbine cycle
1.  Regeneration increases thermal efficiency
2.  Reheating decreases thermal efficiency
s Cycle efficiency increases when maximum
3.  
(b) Fig. (c) temperature of the cycle is increased
v Of these statements
(a) 1, 2 and 3 are correct
(b) 2 and 3 are correct
U (c) 1 and 3 are correct
(d) 1 and 3 are correct
41. Match figures of Column 1 with those given in
s Column II and select the correct answer using the
codes given below the columns:

M09_THERMAL ENGINEERING_SE_XXXX_CH09.indd 477 6/15/2018 4:17:01 PM


478 Chapter 9

Column I Column II (b)


p-v diagrams T-s diagrams
Constant 1 2
(a) 1. volume
p p T T

T
4 3

S
v v  s s
(c)
(b) 2. Constant
p p T T volume 3

T
4
1
v v s s
S
(c) 3.
p p (d)
T T
Constant
3
pressure

v v T
s s 4

Codes: 1
A   B   C S
(a) 1    2    3
(b) 2    3   1
(c) 3    1   2 43. Which one of the following p-T diagrams illus-
(d) 3    2   1 trates the Otto cycle of an ideal gas?
42. Which one of the following diagrams repre-
(a)
sents Otto cycle on temperature (T)-entropy (s)
plane? p
   (a)
1 2

T
4 3 T

M09_THERMAL ENGINEERING_SE_XXXX_CH09.indd 478 6/15/2018 4:17:06 PM


Gas Power Cycles 479

(b) Codes:
     A  B  C  D
p
(a)    3  1   4   2
(b)     5  4      3   2
(c)    2  3     4     5
(d)    1  5     2   3
45. For a heat engine operating on Carnot cycle the
1
work output is th of the heat transferred to the
4
T cold system. The efficiency of the engine is
(a) 20% (b) 25%
(c)
(c) 25% (d) 20%
p 46. Match List I with List II
List I List II
(Heat Engines) (Cycles)
(a) Gas Turbine l. 
Constant volume heat addi-
tion and constant volume
heat rejection
T (b) Petrol Engine 2. Constant pressure heat addi-
tion and constant volume
(d) heat rejection
p (c) Stirling Engine 3. Constant pressure heat addi-
tion and constant pressure
heat rejection
(d) Diesel Engine 4. Heal addition at constant vol-
ume followed by heat addi-
tion at constant temperature
5. Heat rejection at con-
T stant volume followed by
heat rejection at constant
44. Match List I with List II and select the correct
temperature
answer using the codes given below the lists
Codes:
     A  B  C  D
List I List II
(a)    3  1   4   2
(Cycle operating (Characteristics) (b)     5  4      3   2
between fixed tem- (c)    2  3     4     5
perature limits) (d)    1  5     2   3
A. Carnot cycle 1. 
Efficiency depends upon 47. With reference to air standard Otto and Diesel
cut-off ratio and compres- cycles, which of the following statements are
sion ratio true?
B. Brayton cycle 2. 
Efficiency depends upon (a) For a given compression ratio and the same
volume compression ratio state of air before compression. Diesel cycle
C. Otto cycle 3. 
Efficiency depends only is less efficient than an Otto cycle
upon pressure ratio (b) For a given compression ratio and the same
D. Diesel cycle 4. 
Efficiency depends only state of air before compression, Diesel cycle
upon temperature limits. is more efficient than an Otto cycle
(c) The efficiency of a Diesel cycle decreases
5. 
Efficiency depends upon
with an increase in the cut-off ratio
specific heats of the work-
(d) The efficiency of a diesel cycle increases
ing substance.
with an increase in the cut-off ratio

M09_THERMAL ENGINEERING_SE_XXXX_CH09.indd 479 6/15/2018 4:17:08 PM


480 Chapter 9

48. For the same maximum pressure and heat input, translated into temperature-entropy diagram in
the most efficient cycle is figure B. The point X shown on p-v diagram is
(a) Diesel cycle (b) Dual cycle represented on T-s diagram by
(c) Otto cycle (d) Stirling cycle S
49. An Otto cycle operates with volumes of 40 cm3 X
and 400 cm3 at top dead centre (TDC) and bottom
dead centre (BDC) respectively. If the power out-

Temperature
Pressure
put is 100 kW, what is heat input, in kJ/s? Assume P
g = 1.4
R Q
(a) 166 (b) 145
(c) 110 (d) 93 Volume Entropy
50. A cycle consisting of two reversible isothermal S
process and two reversible isobaric processes
X is
known as
(a) Atkinson cycle (b) Stirling cycle

Temperature
Pressure

(c) Brayton cycle (d) Ericsson cycle P


51. The air standard diesel cycle is less efficient than
the Otto cycle for the R Q
(a) same compression ratio and heat addition Volume Entropy
(b) same pressure and heat addition
(c) same rpm and cylinder dimensions (a) points S (b) point P
(d) same pressure and compression ratio. (c) point Q (d) point R
52. A pressure-volume diagram for some engine is
shown in figure A. This diagram has been, closely

Review Questions
1. What do you mean by air-standard cycle? What are the assumptions for air-standard cycle?
2. Define thermal efficiency and relative efficiency of an air cycle.
3. Differentiate between cylinder volume and swept volume.
4. Define compression ratio and clearance ratio.
5. What is mean effective pressure?
6. Write the formula for the thermal efficiency of the Otto cycle.
7. What are the processes involved in the Otto cycle?
8. Draw the p-v and T-S diagrams for the Otto cycle.
9. Write the formula for the thermal efficiency of Diesel cycle.
10. Draw the variation of thermal efficiency against compression ratio of an Otto cycle.
11. How does the efficiency of Diesel cycle vary with cut-off ratio?
12. For what type of engines the diesel cycle is used?
13. What are the processes involved in a diesel cycle?
14. What is a dual cycle?
15. What are the processes involved in a Joule cycle?
16. What are the applications of Brayton cycle?
17. Compare the Otto, Diesel and Brayton cycles with respect to heat addition and heat rejection.

M09_THERMAL ENGINEERING_SE_XXXX_CH09.indd 480 6/15/2018 4:17:09 PM


Gas Power Cycles 481

Exercises
9.1 In an air-standard Otto cycle the compression ratio is 10. The inlet temperature and pressure is 37.8°C and
1 bar. The maximum temperature of the cycle is 1060°C. Calculate heat supplied per kg of air, work done
per kg of air, maximum pressure of cycle, and thermal efficiency.
[Ans. 396.55 kJ/kg, 238.68 kJ/kg, 42.89 kJ/kg, 60.18%]
9.2 The compression ratio of an Otto cycle is 8. The pressure and temperature at the beginning of compression
stroke are 1 bar and 26.7°C. The heat added to the air per cycle is 2079 kJ/kg of air. Calculate the pressure
and temperature at the end of each process of the cycle. Thermal efficiency, and mean effective pressure.
[Ans. 688.5 K; 18.38 bar; 3584 K; 99.678 bar; 1560 K; 9.20 bar; 56.47%; 19.568 bar]
9.3 The compression ratio of an Otto cycle is 10 and the suction conditions are 1 bar and 50°C. If the heat
rejection equals 3517 kJ/kg, calculate the air-standard efficiency and work ratio. [Ans. 60.18%, 0.938]
9.4 An air-standard diesel cycle has a compression ratio of 16 and a cut-off ratio of 2. At begining of compres-
sion the cylinder volume is 0.001415 m3, the air pressure is 1 bar and the temperature is 49.5°C. Calculate
the thermal efficiency. [Ans. 61.38%]
9.5 A diesel engine with compression ratio of 15 is operating with a fuel having a heating value of 44505 kJ/kg.
At half load this engine requires a 70 to 1 ratio of air to fuel.
(a) What value of cut-off ratio should be used if g = 1.4 and temperature at the beginning of compression
is 40°C?
(b) What is the ideal cycle efficiency? [Ans. 1.68, 62%]
9.6 Calculate the thermal efficiency and mean effective pressure of a dual cycle having compression ratio equal
to 10 when the minimum temperature is 2000°C and the maximum pressure is 70 bar. The pressure and
temperature at the start of compression are 1 bar and 17°C respectively. [Ans. 60%, 9.439 bar]
9.7 An engine operates on diesel cycle with a compression ratio of 12. The fuel is injected for 10% of stroke.
The pressure of air entering the cylinder is 0.98 bar and its temperature is l5°C. Calculate cut-off ratio tem-
perature at the end of compression process, and heat input. [Ans. 2.1,778.1 K. 860.3 kJ/kg]
9.8 The compression ratio of an engine operating on dual cycle is 8. The diameter of cylinder is 25 cm and
stroke is 30 cm. At the start of compression the air temperature is 21.6°C and the pressure is 56 bar. If heat
is added at constant pressure during 3% of stroke, calculate a net work of the cycle, and thermal efficiency.
[Ans. 768.6 kJ/kg, 59.93%]
9.9 In an air-standard Brayton cycle, the air enters the compressor at 1 bar and 25°C. The pressure after com-
pression is 3 bar. The temperature at turbine inlet is 650°C. Calculate per kg of air heat supplied, heat
rejected, work available at the shaft, temperature of air leaving the turbine, and cycle efficiency.
[Ans. 519.6 kJ/kg, 378 kJ/kg, 139.5 kJ/kg, 674 K, 26.97%]
9.10 In a Brayton cycle, air enters the compressor at 1 bar and 15°C and leaves the combustion chamber at 1400 K.
Calculate the thermal efficiency. [Ans. 48.20%]
9.11 An engine working on the Otto cycle is supplied with air at 0.1 MPa and 35°C. The compression ratio is 8.
Heat supplied is 2100 kJ/kg. Calculate the maximum pressure and temperature of the cycle, cycle efficiency,
and the mean effective pressure. For air cp = 1.005 kJ/kg.K, cv = 0.718 kJ/kg.K and R = 0.287 kJ/kg.K.
[Ans. 9.435 MPa, 3639.4 K, 56 5%, 1.533MPa]
9.12 In an air-standard diesel cycle, the compression ratio is 16. At the beginning of isontropic compression, the
temperature is 15°C and the pressure is 0.1 MPa. The temperature at the end of the constant pressure pro-
cess is 1480°C. Calculate the cut-off ratio, heat supplied per kg of air, the cycle efficiency, and the m.e.p.
[Ans. 2.01, 889.4 k.J/kg. 76 43%, 871.766 kPa]

M09_THERMAL ENGINEERING_SE_XXXX_CH09.indd 481 6/15/2018 4:17:12 PM


482 Chapter 9

9.13   An air standard dual cycle has a compression ratio of 16. The compression begins at 1 bar and 50°C. The
maximum pressure is 70 bar. The heat transferred to air at constant pressure is equal to that at constant
volume. Calculate cycle efficiency, and m.e.p of cycle cp = 1.005 kJ/kg.K, cv= 0.718 kJ/kg/K.
[Ans. 66.31%, 9.752 bar]
9.14   An internal combustion engine works on diesel cycle with a compression ratio of 14 and cut-off takes place
at 10% of the stroke. Find the ratio of cut-off and the air-standard efficiency. [Ans. 2.3, 66.31%]
9.15   An ideal diesel cycle operates on a pressure of 1 bar and a temperature of 27°C at the beginning of com-
pression and a pressure of 2 bar at the end of adiabatic expansion. Calculate the amount of heat required to
be supplied per kg of air if the ideal thermal efficiency is taken as 60%. Take cv = 0.717 kJ/kg.K.
[Ans. 537.75 kJ/kg]
9.16   The pressure and temperature of a diesel cycle at the start are 1 bar and 17°C. The pressure at the end of
expansion is 2 bar. Find the air-standard efficiency. Assume g = 1.4 [Ans. 61.14%]
9.17   A diesel cycle operates at pressure of 1 bar at the beginning of compression and the volume is compressed
to 1/15 of the initial volume. Heat is then supplied until the volume is twice that of the clearance volume.
Determine the mean effective pressure. Take g = 1.4 [Ans. 6.69 bar]
9.18   A semi diesel engine works on dual combustion cycle. The pressure and temperature at the beginning of the
compression is 1 bar and 27°C respectively and the compressor ratio being 12. If the maximum pressure is
50 bar and heat received at constant pressure is for 1/30th of the stroke, find the work done per kg of air and
the thermal efficiency. Take cp = 1.004 kJ/kg.K, cv = 0.717 kJ/kg.K. [Ans. 537 kJ/kg, 69.27%]
9.19   A compression-ignition engine has a compression ratio of 10 and 2/3 of heat of combustion is liberated at a
constant volume and the remainder at constant pressure. The pressure and temperature at the beginning are
1 bar and 27°C and the maximum pressure is 40 bar. Find the temperature at the end of compression and
expansion, if it follows the law pV1.35 = constant, and g = 1.4.  [Ans. 398.6°C, 347.3°C]
9.20   An Otto cycle engine having a clearance volume of 250 cc has a compression ratio of 8. The ratio of pres-
sure at constant volume is 9. If the initial pressure is 1 bar, find the work done per cycle and the theoretical
mean effective pressure. Takeg = 1.4  [Ans. 1946.1J/cycle, 11.12 bar]
9.21   Find the m.e.p for the ideal air-standard Otto cycle having a maximum pressure of 40 bar and minimum
pressure of 1 bar. The compression ratio is 5:1. Take g = 1.4  [Ans. 9.04 bar]
9.22   An engine working on Otto cycle, the ratio of temperature at the beginning of compression is 300 K. If the
ideal air-standard efficiency = 0.5, calculate the compression ratio of the engine. If the peak temperature of
the cycle is 1150K, calculate the temperature when the piston is at BDC during expansion stroke.
[Ans. 9.66, 575 K]
9.23   A petrol engine working on Otto cycle has a maximum pressure of 50 bar. If the pressure during combus-
tion is 12.286, find the compression ratio and also the ratio of peak temperature to inlet temperature. Take
p1= 1 bar and t1= 27°C.  [Ans. 6 8.34]
9.24   An Otto cycle takes in air at 300 K. The ratio of maximum to minimum temperature is 6. Find out the opti-
mum compression ratio for the maximum work output of the cycle.  [Ans. 9 391]
9.25   The pressure and temperature of a diesel cycle at the start are 1 bar and 20°C respectively and the compres-
sion ratio is 19. The pressure at the end of expansion is 2.5 bar. Find the percentage of working stroke at
which heat is supplied and heal supplied per kg of air. Assume g = 1.4 and cp = 1.004 kJ/kg.K.
[Ans. (i) 7.11% (u) 781.46 kJ/kg]
9.26   An oil engine works on diesel cycle, the compression ratio being 19. The temperature at the start of com-
pression is 17°C and 700 kJ of heat is supplied at constant pressure per Kg of air and attains a temperature
of 417°C at the end of adiabatic expansion. Find the air-standard efficiency of the cycle. What would be the
theoretical work done per kg of air. Take cv = 0.717 kJ/kg K and g = 1.4.  [Ans. 61.32%, 419.20 kJ]

M09_THERMAL ENGINEERING_SE_XXXX_CH09.indd 482 6/15/2018 4:17:15 PM


Gas Power Cycles 483

9.27   An internal combustion engine works on Diesel cycle with a compression ratio of 8 and expansion ratio
of 9. Calculate the air-standard efficiency Assume g = 1.4  [Ans. 51.76%]
9.28   A Diesel engine works on Diesel cycle with a compression ratio of 16 and cut-off ratio of 1.8. Calculate the
thermal efficiency assuming g = 1.4 [Ans. 62.38%]
9.29   A compression-ignition engine works on dual combustion cycle. The pressure and temperature at the begin-
ning of compression are 1 bar and 27°C respectively and the pressure at the end of compression is 25 bar.
If 420 kJ of heat is supplied per kg of air during constant volume heating and the pressure at the end of
adiabatic expansion is found to be 3 bar, find ihe ideal thermal efficiency. Assume cp = 1.004 J/kg.K and cv
= 0 717 k.J/kg K.  [Ans. 58.27%]

9.30  The cycle of an internal combustion engine with isochoric heat supply is performed with the compression
ratio equal to 8. Find heat supplied to the cycle and the useful work, if the removed heat is 500 kJ/kg and
the working fluid is air.  [Ans. 1148.63 kJ/kg, 648.63 kJ/kg]
9.31  The initial parameters (at the beginning of compression) of the cycle of an internal combustion engine with
isobaric heat supply are 0.1 MPa and 80°C. The compression ratio is 16 and the heat supplied is 850 kJ/kg.
Calculate the parameters at the characteristic points of the cycle and the thermal efficiency, if the working
fluid is air.
 [Ans. p2 = 48.5 bar, p3 = 48.5 bar, p3 = 2.26 bar, T1 = 1070.1 K, T3 = 1916 K, T4 = 797.73, hth = 62.5%]
9.32  The pressure ratio a p = 1.5 in the process of isochoric heat supply for the cycle at an internal combustion
engine with a mixed supply of heat = 1034 kJ/kg and the compression ratio = 19. Find the thermal efficiency
and temperature at the characteristic points of the cycle if the initial parameters are 0.09 MPa and 70°C and
the working substance is air. [Ans. hth = 62%, T2 = 956.9 K, T3 = 1439.5 K, T4 = 2122.86 K, T5 = 890 K]
9.33  The parameters of the initial state of one kilogram of an in the cycle of an internal combustion engine are
0.095 MPa and 65°C. The compression ratio is 11. Compare the values of the thermal efficiency for isobaric
and isochoric heat supply in amounts of 800 kJ, assuming that g = 1.4.  [Ans. htp = 59.7 %, htv = 61.7%]
9.34   Find the thermal efficiency of the cycle of an internal combustion engine with a mixed heat supply, if the
minimum temperature of the cycle is 85°C and the maximum temperature is 1700 K. The compression ratio
is 15 and the pressure ratio in the process of heat supply is 1.4. The working fluid is air.
[Ans. hth = 69.25%]
9.35   A reversed Carnot cycle uses 0.1 kg of ammonia as a working fluid. At the beginning of the isothermal
expansion, the ammonia is at 80 kPa, 30 percent quality; at the end of the isothermal expansion, the ammo-
nia is dry and saturated. Heat is rejected at 20°C. Determine the change in internal energy of the ammonia
during the isothermal expansion and the work input per cycle.  [Ans. 89.0 kJ, 23.9 kJ]
9.36   A closed-system Stirling cycle using helium as the working fluid operates with maximum pressure and
temperature of 1200 kPa, 1000°C. The cycle has an overall volume ratio of 4.2 and minimum pressure and
temperature of 75 kPa, 61°C. Determine the thermal efficiency of the cycle.
9.37   For an Ericsson cycle as shown in Fig. 5.11, p1 = 500 kPa, T1 = 600°C, p3 = 100 kPa, T3 = 60°C, m =
0.220 kg/s, and the working fluid is argon. Determine the thermal efficiency.
9.38   A closed-system Stirling cycle using helium as working fluid has an overall volume ratio 3.8 and an overall
temperature ratio of 3.0. Maximum pressure and temperature in the cycle are 2 MPa and 590°C. Determine
the power output of the cycle for a heat input rate of 16 kW.
9.39  The overall temperature and volume ratios of a closed-system Ericsson cycle are 3.0 and 4.8, respectively.
The maximum temperature in the cycle is 640°C and maximum pressure of 3.5 MPa. Determine the cycle
thermal efficiency.

M09_THERMAL ENGINEERING_SE_XXXX_CH09.indd 483 6/15/2018 4:17:16 PM


484 Chapter 9

9.40   In a Stirling cycle the volume varies between 0.03 and 0.06 m3, the maximum pressure is 0.2 MPa, and the
temperature varies between 540°C and 270°C.The working fluid is air
 (a)  Find the work done per cycle and the efficiency for the simple cycle
 (b)  Find the efficiency for the cycle with an ideal regenerator [Ans. (a) 53.7 kJ/kg, 27.7%, (b) 33.2%]
9.41   An Ericsson cycle operating with an ideal regenerator works between 1100 K and 288 K. The pressure at
the beginning of isothermal compression is 1.013 bar. Determine the compressor and turbine work per kg
of air, and the cycle efficiency. [Ans. 121.8 kJ/kg, 465 kJ/kg, 0.738]
9.42   A Stirling regenerative air engine works between temperature of 400°C and 15°C. The ratio of isothermal
expansion is 3. Calculate the ideal efficiency with and without the regenerator of efficiency 80%.
[Ans. 57.3%, 45.4%]

ANSWERS TO MULTIPLE-CHOICE QUESTIONS


1. (c) 2. (b) 3. (d) 4. (b) 5. (a)
6. (a) 7. (d) 8. (c) 9. (d) 10. (a)
11. (b) 12. (b) 13. (d) 14. (c) 15. (b)
16. (a) 17. (b) 18. (c) 19. (c) 20. (a)
21. (a) 22. (d) 23. (a) 24. (c) 25. (d)
26. (c) 27. (a) 28. (b) 29. (b) 30. (b)
31. (c) 32. (d) 33. (c) 34. (d) 35. (d)
36. (b) 37. (c) 38. (a) 39. (a) 40. (a)
41. (c) 42. (c) 43. (a) 44. (c) 45. (b)
46. (a) 47. (a) and (c) 48. (d) 49. (a) 50. (d)
51. (a) 52. (a)

M09_THERMAL ENGINEERING_SE_XXXX_CH09.indd 484 6/15/2018 4:17:16 PM


Chapter 10

Internal Combustion
Engine Systems
10.1 ❐ INTRODUCTION
An engine may be defined as a device which converts one form of energy into mechanical energy.
Mechanical energy can be further easily converted into electrical energy which is easier to transport
and control. Heat engine is a cyclic device which transforms heat energy into mechanical energy.
Heat engines are classified into the following:
1. External combustion engines
2. Internal combustion engines
In an external combustion engine, the fuel is burned outside the engine and the generated heat is
supplied to the working fluid of the engine for power generation. Steam engine is an example of an
external combustion engine in which fuel is burned in the boiler to generate steam, which is used in
the engine for power generation. The working fluid is not mixed with fuel, and the same working fluid
(water in the form of steam) is repeatedly used in the system.
In an internal combustion (IC) engine, the fuel is mixed with air and burned inside the engine to
generate power. In this case, the same working fluid (air-fuel mixture) cannot be used again in the
cycle. Some examples of internal combustion engine are spark ignition (SI) engine, compression
ignition (CI) engine, etc.
IC engines offer the following advantages over external combustion engines:
1. Greater mechanical simplicity
2. Lower weight to output ratio
3. Higher overall thermal efficiency
4. Less water requirement
5. Easy and quick starting

10.2 ❐ CLASSIFICATION OF INTERNAL COMBUSTION


ENGINES
Figure 10.1 shows the classification of IC engines on the basis of cycle of operation into cylinder, type
of fuel, method of supply of fuel, type of ignition, among others.

M10_THERMAL ENGINEERING_SE_XXXX_CH010( Part-I).indd 485 6/15/2018 4:20:33 PM


486 Chapter 10

I.C. Engines

Four stroke engines Two stroke engines


(Reciprocating only)

Spark-ignition (S.I.) Compression


engines ignition (C.I.) engines

Gas engines Petrol engines Ordinary Dual-fuel


and multi-fuel

Carburetted Injection Divided Open


type type chamber chamber
engines engines

Battery Magneto Pre-chamber Open chamber


ignition ignition

Water cooled Air cooled

Reciprocating Rotary (Wankel)

Opposed Inclined Vee Single Two


cylinder cylinder cylinder rotor rotor
(2, 4 or 6) (2, 4 or 6) (4, 6, 8, 12)

Figure 10.1  IC engine classification

1. Basic engine design: Reciprocating engines, rotary (Wankel) engines


2. Working cycle: Otto cycle and diesel cycle
3. Number of strokes: Four-stroke and two-stroke engines
4. Fuel: Gasoline (or petrol), compressed natural gas (CNG), liquefied petroleum gas (LPG), diesel
oil (light diesel oil—LDO and high speed diesel oil—HSD), fuel oil, alcohols (methanol, ethanol)

M10_THERMAL ENGINEERING_SE_XXXX_CH010( Part-I).indd 486 6/15/2018 4:20:33 PM


Internal Combustion Engine Systems 487

(a) (b) (c)

(d) (e)

(f)

Figure 10.2 Classification of IC engines as per cylinder arrangement:


(a) Inline engine, (b) Vee engine, (c) Radial engine, (d) Opposed
piston engine, (e) Opposed cylinder engine, (f) Delta type engine

5. Fuel supply and mixture preparation: Carburetted type and injection type
6. Method of ignition: CI (compression-ignition), SI (spark-ignition) engines: battery ignition or
magneto ignition
7. Method of cooling: Water cooled or air-cooled
8. Cylinder arrangement (Fig. 10.2): Inline, V, or Vee, radial, opposed piston and opposed cylin-
der, delta
9. Valve or port design and location: Overhead valves (I-head), side valve (L-head); in two-stroke
engines: cross scavenging, loop scavenging, uniform scavenging
10. Application: Automotive engines for land transport, marine engines for propulsion of ships,
aircraft engines for aircraft propulsion, industrial engines, prime movers for electrical generators

10.3 ❐ CONSTRUCTION FEATURES


1. Four-stroke spark-ignition engine: The cross section of a four-stroke spark-ignition engine is
shown in Fig. 10.3. The major components of the engine are as follows:
(a) Cylinder block: It is the main supporting structure for the various components. The cylin-
der head is mounted on the cylinder block. The bottom of the cylinder block is called the
crank case. The lubricating oil is kept in the crank case sump.
(b) Cylinder: It is a cylindrical vessel in which the piston reciprocates.

M10_THERMAL ENGINEERING_SE_XXXX_CH010( Part-I).indd 487 6/15/2018 4:20:34 PM


488 Chapter 10

Inlet valve (IV) Exhaust valve (EV)


Spark plug
Inlet manifold Exhaust manifold

Air Products

Fuel Cylinder

Piston

Gudgeon pin
Connecting rod
Cylinder block
Crankshaft

Crankpin Crankcase
Crank Sump

Figure 10.3  Cross-section of a spark-ignition engine

(c) Piston: It is a cylindrical component fitted into the cylinder forming the moving boundary
of the combustion system.
(d) Combustion chamber: It is the space between the cylinder and the piston top where com-
bustion takes place.
(e) Inlet manifold: It is the pipe through which air or air-fuel mixture is drawn in the cylinder.
(f) Exhaust manifold: It is the pipe through which the products of combustion escape into the
atmosphere.
(g) Inlet and exhaust valves: The valves are used for regulating the incoming charge into the
cylinder (inlet valve) or discharging the products of combustion (exhaust valve) from the
cylinder.
(h) Spark plug: It is a component to initiate the combustion process in SI engine.
(i) Carburettor: It is used for mixing fuel and air in correct proportion in SI engine.
(j) Connecting rod: It is used to interconnect the piston and the crank to transmit force from
the piston to the crankshaft.
(k) Crankshaft: It is used to convert reciprocating motion of the piston into rotary motion of
the output shaft.
(l) Piston rings: They provide a tight seal between the piston and the cylinder wall.
(m) Gudgeon pin: It connects the small end of the connecting rod to the piston.
(n) Camshaft: It is a shaft on which cams are mounted to operate the valves. It is driven from
the cam shaft by gears.
(o) Flywheel: It absorbs surplus energy during working stroke and delivers during idle stroke.
2. Four-stroke compression-ignition engine: Except for the spark plug and carburettor, all other
components are the same as for the SI engine. It also requires a fuel pump and fuel nozzle.
Figure 10.4 shows outline of a CI engine.

M10_THERMAL ENGINEERING_SE_XXXX_CH010( Part-I).indd 488 6/15/2018 4:20:35 PM


Internal Combustion Engine Systems 489

Fuel nozzle
Rocker arms Valve spring
Fuel line Exhaust spring
Fuel valve Exhaust
manifold
Inlet
Cylinder
manifold
head
Water
Push rods
jackets
Piston
Fuel pump
Wristpin
Cylinder
liner
Crankshaft
Camshaft drive Frame

Connecting rod
Bearing cap
Crankpin bolt

Crankpin

Main
bearing Bed plate

Crankcase

Figure 10.4  Outline of diesel engine

3. Two-stroke SI engine: In the case of two-stroke engine, the valves are replaced by the exhaust
port and the transfer port. The charging of the cylinder with the air-fuel mixture takes place
through the carburetor and ignition by a spark plug. Other components are same as for a four-
stroke engine.
4. Two-stroke CI engine: Here, the carburettor and the spark plug are replaced by a fuel pump and
fuel nozzle. All other components are same as for two-stroke SI engine.

10.4 ❐ WORKING OF IC ENGINES


10.4.1  Four-stroke Spark-ignition Engine
The details of various processes of a four-stroke spark-ignition engine with overhead valves are shown
in Fig. 10.5. Within the four strokes, there are five events to be completed namely, suction, compres-
sion, combustion, expansion, and exhaust.

M10_THERMAL ENGINEERING_SE_XXXX_CH010( Part-I).indd 489 6/15/2018 4:20:35 PM


490 Chapter 10

Exhaust valve Intel valve


E.V. I.V. E.V. I.V. E.V. I.V.
Spark plug

Piston
Cylinder
Connecting
rod
Crankshaft
Crankpin

(a) (b) (c) (d)

Figure 10.5 Working principle of a four-stroke SI engine: (a) Suction


stroke, (b) Compression stroke, (c) Expansion or power
stroke, (d) Exhaust stroke

1. Suction stroke: It starts when the piston is at the top dead centre (TDC) and is about to move
downwards. The inlet valve is open and the exhaust valve is closed at this time. The charge con-
sisting of the fuel-air-mixture is drawn into the cylinder. When the piston reaches the bottom dead
centre (BDC), the suction stroke ends and the inlet valve closes.
2. Compression stroke: The charge is compressed to the clearance volume by the return stroke of
the piston with both inlet and exhaust valves are closed. At the end of the compression stroke, the
mixture is ignited with the help of spark plug to convert chemical energy of fuel to heat energy.
3. Expansion stroke: The high pressure of burnt gases forces the piston towards BDC with both the
valves in closed position to produce power.
4. Exhaust stroke: At the end of the expansion stroke, the exhaust valve opens with the inlet valve
closed. The piston starts moving towards the TDC and sweeps the burnt gases out from the cylin-
der. The exhaust valve closes when piston has reached TDC.

10.4.2  Four-stroke Compression-ignition Engine


The four cycles of operation of a CI engine are shown in Fig. 10.6.
1. Suction stroke: Only air is induced during the suction stroke with the inlet valve open and the
exhaust valve closed.

I.V. E.V. I.V. E.V. I.V. E.V. I.V. E.V.

(a) (b) (c) (d)

Figure 10.6 Cycle of operation of a four-stroke CI engine: (a) Suction,


(b) Compression, (c) Expansion, (d) Exhaust

M10_THERMAL ENGINEERING_SE_XXXX_CH010( Part-I).indd 490 6/15/2018 4:20:36 PM


Internal Combustion Engine Systems 491

2. Compression stroke: The sucked air is compressed into the clearance volume with both valves
closed.
3. Expansion stroke: Fuel injection starts nearly at the end of the compression stroke, resulting in
combustion. The products of combustion expand and both the valves remain closed.
4. Exhaust stroke: The piston moves from BDC to TDC and pushes out the products of combustion
with the exhaust valve open and intake valve closed.

10.4.3  Two-stroke Spark-ignition Engine


In a two-stroke engine, the cycle is completed in one revolution of the crankshaft. The filling process
is accomplished by the charge compressed in the crankcase or by a blower. The induction of the com-
pressed charge moves out the products of combustion through the exhaust ports. Therefore, two piston
strokes are required for these two operations. Two strokes—one for compressing the fresh charge and
the other for expansion or power stroke—are sufficient to complete the cycle. Figure 10.7 shows the
crankcase scavenged two-stroke engine.

10.4.4  Two-stroke Compression-ignition Engine


Figure 10.8 shows a two-stroke compression-ignition engine. It is a crankcase scavenged type engine.
S is the plate valve for admission of air in the crank case, E is the exhaust port, and A is the port in
the cylinder communicating to the crankcase through the cylinder block casting; cams and valves are
not required. The four operations—air induction, air compression and fuel injection, expansion, and
exhaust are completed in two strokes, that is, in one revolution of crankshaft.
In the upward motion of the piston, suction is created in the crankcase and air enters through the
plate valves for full 180° of crank rotation as shown in Fig. 10.8(a). Above the piston, the compression
starts after both the points have been covered by the piston. At the end of compression, fuel is injected
and ignites at TDC giving products of combustion at high pressure.
On downward stroke, the high pressure products of combustion expand and the air below the
piston compresses, closing the plate valves, as shown in Fig. 10.8(b). As soon as the piston uncovers
the exhaust port, the products of combustion are released into the atmosphere. A little later, on the

Spark plug

Deflector

Transfer Transfer
port port

Inlet valve

Crankcase

Exhaust blow down Scavenging

Figure 10.7  Crankcase scavenged two-stroke SI engine

M10_THERMAL ENGINEERING_SE_XXXX_CH010( Part-I).indd 491 6/15/2018 4:20:37 PM


492 Chapter 10

E A E A E A

S S S S S S

(a) (b) (c)

Figure 10.8  Working of two-stroke C.I. Engine

downward stroke, the other port communicating with the crankcase gets uncovered and the air com-
pressed in the crankcase gets transferred to the space above the piston as shown in Fig. 10.8(c).

10.5 ❐ COMPARISON OF FOUR-STROKE


AND TWO-STROKE ENGINES
Table 10.1 shows a comparison of four-stroke and two-stroke engines.

Table 10.1  Comparison of two-stroke and four-stroke engines

S. no. Feature Four-stroke engine Two-stroke engine


1. Power strokes The thermodynamic cycle is completed The thermodynamic cycle is com-
in four strokes of the piston or in two pleted in two strokes of the piston or
revolutions of the crankshaft. Thus, one in one revolution of the crankshaft.
power stroke is obtained in every two Thus, one power stroke is obtained
revolutions of the crankshaft. in each revolution of the crankshaft.
2. Size of flywheel Due to the aforementioned reason, the Due to the aforementioned reason, the
turning moment is not so uniform and turning moment is more uniform and
hence a heavier flywheel is needed. hence a lighter flywheel is needed.
3. Power developed As one power stroke is required for two As one power stroke is required for
revolutions, power produced for same every revolution, power produced for
size of engine is less, or for the same same size of engine is twice, or for
power, the engine is heavier and bulkier. the same power the engine is lighter
and more compact.
4. Wear and tear Due to one-power stroke in two revolu- Due to one-power stroke in one
tions, less cooling and lubrication are revolution, greater cooling and
required. There are lower rates of wear lubrication are required. There are
and tear. higher rates of wear and tear.
(Continued)

M10_THERMAL ENGINEERING_SE_XXXX_CH010( Part-I).indd 492 6/15/2018 4:20:38 PM


Internal Combustion Engine Systems 493

Table 10.1  (Continued )

S. no. Feature Four-stroke engine Two-stroke engine


5. Valves Four-stroke engines have valves and Two-stroke engines have no valves
valve-actuating mechanisms for opening but only ports (some two-stroke
and closing of the intake and exhaust engines are fitted with conventional
valves. exhaust valve or reed valve).
6. Initial cost Due to comparatively higher weight and Due to light weight and simplicity
complicated valve mechanism, the initial owing to the absence of valve actu-
cost of the engine is more. ating mechanism, the initial cost of
the engine is less.
7. Volumetric Volumetric efficiency is more due to Volumetric efficiency is low due to
efficiency more time for induction. lesser time for induction.
8. Thermal Thermal efficiency is higher; part load Thermal efficiency is lower; part
efficiency efficiency is better. load efficiency is poor.
9. Applications It is used where efficiency is important, It is used where low cost,
namely in cars, buses, trucks, tractors, compactness, and light weight
industrial engine, aeroplanes, power are important, such as in mopeds,
generation, etc. scooters, motorcycles, hand
sprayers, and so on.

10.6 ❐ COMPARISON OF SI AND CI ENGINES


Table 10.2 gives the comparison of SI and CI engines.

Table 10.2  Comparison of SI and CI engines

S. no. Feature SI engine CI engine


1. Basic cycle Otto cycle Diesel cycle
2. Fuel Petrol (Gasoline) Diesel oil
3. Introduction of fuel Fuel and air are mixed in proper Fuel injected directly at the end of
proportion in the carburettor and compression stroke carburettor is
introduced during the suction not required but a high pressure fuel
stroke. The throttle controls the pump and injector are necessary.
quantity of mixture introduced.
4. Ignition Requires ignition system with Self-ignition due to high temperature.
spark plug in combustion chamber.
5. Compression ratio 6−10.5 14−22
range
6. Speed Higher maximum revolution per Maximum rpm lower.
minute due to lighter weight.
7. Efficiency Maximum efficiency lower due to Higher maximum efficiency due to
low compression ratio. higher compression ratio.
8. Weight Lighter Heavier due to higher pressures.

M10_THERMAL ENGINEERING_SE_XXXX_CH010( Part-I).indd 493 6/15/2018 4:20:38 PM


494 Chapter 10

10.7 ❐ MERITS AND DEMERITS OF TWO-STROKE


ENGINES OVER FOUR-STROKE ENGINES
10.7.1 Merits
1. A two-stroke engine gives twice as many power strokes as a four-stroke cycle engine at the same
engine speed; therefore, a two-stroke engine of the same size should develop twice the power of
a four-stroke engine. In practice, the actual power developed by a two-stroke engine is about 1.7
to 1.8 times the power developed by a four-stroke engine of the same dimensions and speed. This
happens because some of the power is used for compressing the charge in the crank case and the
effective stroke is less than the actual stroke.
2. For the same power developed, the two-stroke engine is much lighter, less bulky, and occupies less
floor area. Therefore, it is more suitable for uses in marine engines and transport purposes.
3. It provides mechanical simplicity as valves, rocker arms, push-rods, cam, and cam shafts are not
required. The friction loss is also less, and therefore, it gives higher mechanical efficiency.
4. The two-stroke engines are much easier to start.
5. A crankcase compression and valve-less type two-stroke engine can run in either direction, which
is useful in marine applications.
6. The initial cost of the engine is considerable less.
7. The weight/kW ratio is considerably less.

10.7.2 Demerits
1. The thermodynamic efficiency of an engine is only dependent on the compression ratio. The
effective compression ratio for a two-stroke engine is less than that for four-stroke engine for
the same stroke (actual) and clearance volume. Therefore, the thermodynamic efficiency of two-
stroke cycle is always less than a four-stroke cycle engine.
2. The actual efficiency of a two-stroke cycle is less than a four-stroke cycle engine because greater
overlapping of ports is necessary in a two-stroke engine for effective scavenging. A portion of
fresh charge in the case of an SI engine always escapes unused through the exhaust ports; there-
fore, the specific fuel consumption is usually higher.
3. As the power-strokes per minute are twice the power stroke of four-stroke cycle engines, the
capacity of the cooling system used must be higher. The cooling of the engine also presents dif-
ficulty as the quantity of heat removed per minute is large. Due to firing in each revolution, the
piston is likely to get overheated and oil cooling of the piston is necessary.
4. The consumption of lubricating oil is sufficiently large because of high operating temperatures.
5. Sudden release of the gases makes the exhaust noisier.
6. A two-stroke petrol engine with crankcase compression 50–60% of the swept volume is filled
with fresh charge, whereas a four-stroke petrol engine contains 80–95%. This happens because
the space occupied by the rotating parts in the crankcase prevents a full charge being sucked in.
7. The scavenging is not complete, particularly in high speed engines, as very short time is available
for exhaust; hence, the fresh charge is highly polluted. This can be reduced using an opposed
piston two-stroke diesel engine which provides unidirectional scavenging.
8. The turning moment of a two-stroke engine is more non-uniform as against with four-stroke
engine, so it requires heavier flywheel and strong foundation.

M10_THERMAL ENGINEERING_SE_XXXX_CH010( Part-I).indd 494 6/15/2018 4:20:39 PM


Internal Combustion Engine Systems 495

10.8 ❐ VALVE TIMING DIAGRAMS


10.8.1  Four-stroke SI Engine
The valve timing diagram shows the regulation of the positions in the cycle at which the valves are set
to open and close. The valve timing diagrams for low and high speed four-stroke S.I. engine are shown
in Fig. 10.9. Typical valve timings are given in Table 10.3.

10.8.2  Four-stroke CI Engine


A typical valve timing diagram for a four-stroke CI engine is shown in Fig. 10.10. The typical timing
valves are also displayed.
IVO up to 30° before TDC; IVC up to 40° after BDC
EVO about 45° before BDC; EVC about 30° after TDC
Fuel valve opens (FVO) about 15° before TDC; FVC about 25° after TDC.

10.8.3  Two-stroke SI Engine


The transfer port opens 35° before the BDC and closes 35° after the BDC. The exhaust port opens
45° before the BDC and closes 45° after the BDC. The spark ignition occurs 20° before the TDC. The
typical valve timing diagram for two-stroke engine is shown in Fig. 10.11.
Exhaust Exhaust
Intake closes Intake closes
opens TDC opens TDC
Valve overlap

Spark
Spark

10° 10°
5° 20°
15° 30°
Exhaust valve open
Exhaust valve open

Compression

Intake valve
Compression

Intake valve

open
open

Power Power
60° 55°

25°
Intake
10°
closes Exhaust
opens
Exhaust
Intake BDC opens
closes BDC
(a) (b)

Figure 10.9 Valve timing for low and high speed four-stroke SI engine:
(a) Low speed engines, (b) High speed engines

M10_THERMAL ENGINEERING_SE_XXXX_CH010( Part-I).indd 495 6/15/2018 4:20:39 PM


496 Chapter 10

Table 10.3  Typical valve timings for four-stroke SI engines

Position Theoretical Actual


Low speed engine High speed engine
Inlet valve opens TDC 10°bTDC* 10°bTDC
Inlet valve closes BDC 10°aBDC* 60°aBDC
Inlet valve is open for 180° 200° 250°
Exhaust valve opens BDC 25°bBDC 55°bBDC
Exhaust valve closes TDC 5°aTDC 20°aTDC
Exhaust valve is open for 180° 210° 255°
Valve overlap spark NIL 15° 30°
Spark TDC 15°bTDC 30°bTDC
BDC: Bottom dead centre
TDC:  Top dead centre
*a = after, b = before

TDC

FVO FVC
EVC
15° 25°
en

IVO 35°
Op

30°
n
Ope
Valve

40°
45°
IVC EVO
e
ak
Int
Va
lve

t
aus
Exh
BDC

Figure 10.10  Valve timing diagram four-stroke CI engine

M10_THERMAL ENGINEERING_SE_XXXX_CH010( Part-I).indd 496 6/15/2018 4:20:40 PM


Internal Combustion Engine Systems 497

Rotation
Ignition
occurs TDC

Ex
20°

pa
ion

ns
s

io
Compres

n
Exhaust
Exhaust 35° 35° port opens
port closes
43° 43°
Inlet
Inlet port BDC port opens
closes Inlet en
Port Op
Exh en
aus
t Op
Port

Figure 10.11  Typical valve timing diagram of a two-stroke SI engine

10.8.4  Two-stroke CI Engine


The transfer port opens 45° before the BDC and closes 45° after the BDC. The exhaust port opens 60°
before the BDC and closes 60° after the BDC. The fuel valve opens 15° before the TDC and closes
20° after the TDC. The typical valve timing diagram is shown in Fig. 10.12.

TDC

60°60°
EPC EPO
45° 45°
IPC IPO

BDC

Figure 10.12  Valve timing diagram for a two-stroke CI engine

M10_THERMAL ENGINEERING_SE_XXXX_CH010( Part-I).indd 497 6/15/2018 4:20:41 PM


498 Chapter 10

10.9 ❐ SCAVENGING PROCESS


At the end of the expansion stroke, the combustion chamber of a two-stroke engine is left full of
products of combustion as there is no exhaust stroke available to clear the cylinder of burnt gases. The
process of clearing the cylinder, after the expansion stroke, is called scavenging process. The scaveng-
ing process is the replacement of the products of combustion in the cylinder from the previous power
stroke with fresh-air charge to be burned in the next cycle.
There are three types of scavenging systems as follows:
1. Uniflow scavenging system: In this system, as shown in Figs 10.13(a) and (b), air enters the cylin-
der from one end and leaves from the other end. Air acts like an ideal piston and pushes out the resid-
ual gas in the cylinder and replaces it with fresh charge. Due to absence of any eddies or turbulence,
in a uni-flow scavenging system, it is easier to push the products of combustion out of the cylinder
without mixing with it and short circuiting. Thus, this system has the highest scavenging efficiency.
2. Cross scavenging: This process shown in Fig. 10.14. It employs inlet and exhaust ports placed in
the opposite sides of the cylinder wall. The air moves up to combustion chamber on one side of
the cylinder and then down on the other side to flow out of the exhaust ports. This process requires

Exhaust out

Exhaust out

Scavenge air in Scavenge air in

(a) (b)

Figure 10.13 Uni-flow scavenging system: (a) Exhaust valve,


(b) Opposed piston

Figure 10.14  Cross scavenging

M10_THERMAL ENGINEERING_SE_XXXX_CH010( Part-I).indd 498 6/15/2018 4:20:42 PM


Internal Combustion Engine Systems 499

(a) (b)

Figure 10.15  Loop or reverse scavenging


that air should be guided by the use of either a suitably shaped detector formed on piston top or
by the use of inclined ports. The main disadvantage of this system is that the scavenging air is not
able to get rid of the layer of exhaust gas near the wall, resulting in poor scavenging. Some of the
fresh charge also goes directly in the exhaust port.
3. Loop or Reverse Scavenging: Figures 10.15(a) and (b) show the loop or reverse scavenging
system. This avoids the short-circuiting of the cross-scavenged engine and improves upon the
scavenging efficiency. The inlet and exhaust ports are placed on the same side of the cylinder wall.
The major mechanical problem with this system is that of obtaining an adequate oil supply to the
cylinder wall consistent with reasonable lubricating oil consumption and cylinder wear.

10.10  ❐  APPLICATIONS OF IC ENGINES


The detail applications with capacity and type of engine used are listed in Table 10.4.

Table 10.4  Applications of IC engines

S. no. Applications Power (kW) SI or CI Two-stroke or Cooling


four-stroke engine method
1. Road vehicles
    (i) Mopeds
  0.75−1.5 SI 2 A*
  (ii) Scooters and motor cycles
  3−7 SI 2,4 A
(iii) Small cars 15−75 SI 4 W*
 (iv) Large cars 75−200 SI 4 W
   (v) Lawn movers 0.75−3 SI 2, 4 A
2. Locomotives 400−4000 CI 2, 4 W
3. Small aircraft
  (i) Helicopters 40−1500 SI 4 A
(ii) Airplanes 40−3000 SI 4 A
4. Marine applications
   (i) Motor boats 0.5−75 SI, CI 4 W
(ii) Ships 3000−20,000 CI 2, 4 W

(Continued)

M10_THERMAL ENGINEERING_SE_XXXX_CH010( Part-I).indd 499 6/15/2018 4:20:43 PM


500 Chapter 10

Table 10.4  (Continued )

S. no. Applications Power (kW) SI or CI Two-stroke or Cooling


four-stroke engine method
5. Industrial applications
   (i) Electric power 40−25,000 CI 2, 4 W
(ii) Gas pipe lines 700−5000 SI 2, 4 W
6.

Off-road vehicles
   (i) Light vehicles (factory,
airport)
1.5−15
3−150
SI
SI, CI
2, 4
2, 4
A
A, W
  (ii) Agriculture 40−800 CI 2, 4 W
(iii) Earthmoving machinery 40−2000 CI 2, 4 A, W
 (iv) Military purpose
*A = Air, W = Water

10.11  ❐  THEORETICAL AND ACTUAL p-v DIAGRAMS


10.11.1  Four-stroke Petrol Engine
The theoretical and actual p-v diagrams for a four-stroke petrol engine are shown in Figs. 10.16(a)
and (b), respectively.
The theoretical p-v diagram is drawn with the following assumptions:
1. Suction and exhaust take place at atmospheric pressure through 180° rotation of crank.
2. Compression and expansion take place through 180° rotation of crank.
3. Compression and expansion processes are isentropic.
4. The combustion takes place instantaneously at constant volume at the end of compression stroke.
5. Pressure suddenly falls to the atmospheric pressure at the end of expansion stroke.
The various processes of theoretical p-v diagram are as follows:
5-1: Suction stroke (pa = const)
1-2: Compression stroke (pvg = const)

TDC BDC TDC BDC


3
3 2
A Ignition
2
p p
A1
4 A2 4
5 4′
5 1 1
pa
vc vs vc vs
v v
(a) (b)

Figure 10.16 Theoretical and actual p-v diagrams for a four-stroke petrol


engine: (a) Theoretical, (b) Actual

M10_THERMAL ENGINEERING_SE_XXXX_CH010( Part-I).indd 500 6/20/2018 11:33:35 AM


Internal Combustion Engine Systems 501

2-3: Instantaneous combustion (v = const)


3-4: Expansion stroke (pvg = const)
4-1: Sudden fall in pressure (v = const)
1-5: Exhaust stroke (pa = const)
In practice, the actual conditions differ from the ideal as follows:
1. The suction of mixture in the cylinder is possible only if the pressure inside the cylinder is below
atmospheric pressure.
2. The burnt gases can be pushed out into the atmosphere only if the pressure of the exhaust gases
is above atmospheric pressure.
3. The compression and expansion do not follow the isentropic law, as there will be heat exchange
during these processes.
4. Sudden pressure rise is not possible after the ignition as combustion takes some time for comple-
tion and actual pressure rise is less than theoretical considered. The pressure increase takes place
through some crank rotation, or increase in volume.
5. Sudden pressure release after the opening of expansion valve is not possible and it also takes
place through some crank rotation.
If all these modifications are taken into account, then the cycle can be represented on p-v diagrams as
shown in Fig. 10.16(b).
The area 4′-5-1-4′ representing negative work is called negative loop or pumping loop. This work is
required for admitting the fresh charge and for exhausting the burnt gases. This loss of work is known
as pumping loss and power consumed for this is known as pumping power.
The net work per cycle of the engine is given by the area (A1 − A2). This area (A1 − A2) is always
less than the area A as shown in Fig. 10.16(a) due to the actual deviations of operations from the the-
oretical ones.
If both the areas are represented in the form of rectangles taking vs as base, then the ordinates give
the mean effective pressures as shown in Fig. 10.17.
pma (Actual mean effective pressure) = pmt (theoretical mean effective pressure) × DF (diagram factor)

10.11.2  Four-stroke Diesel Engine


The theoretical and actual p-v diagrams for a four-stroke diesel engine are shown in Figs. 10.18(a)
and (b), respectively.

p pmt
pma

(A1 − A2)
vc vs
v

Figure 10.17  p-v diagrams

M10_THERMAL ENGINEERING_SE_XXXX_CH010( Part-I).indd 501 6/15/2018 4:20:44 PM


502 Chapter 10

TDC BDC TDC BDC


3
2 3 2

p p A
A A2 4
4 4′
5 5 1
vc vs 1 pa vc vs pa
v v
(a) (b)

Figure 10.18 Theoretical and actual p-v diagrams for a four-stroke


diesel engine: (a) Theoretical, (b) Actual

The theoretical p-v diagram for a four-stroke diesel engine is drawn with the following assumptions:
1. Suction and exhaust take place at atmospheric pressure through 180° of crank rotation.
2. Compression and expansion take place during 180° of crank rotation.
3. Compression and expansion are isentropic.
4. The combustion takes place at constant pressure during a small part of expansion stroke.
5. Pressure suddenly falls to atmospheric pressure at the end of expansion stroke.
With the above assumptions, the working cycle can be represented on a p-v diagram as shown in
Fig. 10.18(a), and it is similar to the theoretical diesel cycle. The various processes of theoretical
p-v diagram are as follows:
5-1: Suction stroke (pa = const)
1-2: Compression stroke (pvg = const)
2-3: Constant pressure combustion (p = const)
3-4: Expansion stroke (pvg = const)
4-1: Sudden fall in pressure (v = const)
1-5: Exhaust stroke (pa = const)
However, in practice, the actual conditions differ from the ideal described as follows:
1. The suction of the air inside the cylinder is possible only if the pressure inside the cylinder is
below atmospheric.
2. Exhausting of gasses is possible only if the pressure of the exhaust gases is above atmospheric pressure.
3. The compression and expansion do not follow the isentropic process, as there are heat and pres-
sure losses.
4. The combustion at constant pressure is not possible as the fuel will not burn as it is introduced
into the cylinder.
5. The sudden pressure release after the opening of expansion valve is not possible and it takes place
through some crank rotation.
The operations of the cycle, taking the modifications into account, are represented on the p-v diagram
in Fig. 10.18(b). Actual area (A1 − A2) on p-v diagram per cycle is less than theoretical.

M10_THERMAL ENGINEERING_SE_XXXX_CH010( Part-I).indd 502 6/15/2018 4:20:44 PM


Internal Combustion Engine Systems 503

10.11.3  Two-stroke Petrol Engine


The theoretical and actual p-v diagrams for a two-stroke petrol engine are shown in Fig. 10.19. The
following assumptions are made for drawing the theoretical p-v diagram:
1. The expansion during power stroke and compression during compression stroke are isentropic.
2. The combustion takes place instantaneously at constant volume at the end of compression.
3. The pressure falls instantaneously to the atmospheric pressure as the piston uncovers the exhaust
ports during power stroke.
4. The scavenging takes place at atmospheric pressure.
It may be observed from Fig. 10.19(a) that the compression of the charge starts from ‘1’ instead of
point 6.
Effective compression ratio,
v +v
rce = se c
vc
p 2
where vse = effective stroke volume = d Le
4
p 2
vsa = actual stroke volume = d La
4

D
3
3
2
p
p
Exhaust
2 opens
At Ac
Inlet
1 4 opens
5
5 6 1 6
pa 1 Exhaust 5′ Inlet
closed closed
vc vse(Le ) vc vse(Le )
vsa(La) vsa(La)
v
BDC Le
Le

pme At pme Ac

pma At pma Ac

La La
(a) (b)

Figure 10.19 Theoretical and actual p-v diagrams for a two-stroke


petrol engine: (a) Theoretical, (b) Actual

M10_THERMAL ENGINEERING_SE_XXXX_CH010( Part-I).indd 503 6/15/2018 4:20:46 PM


504 Chapter 10

Le, La = effective and actual stroke lengths, respectively


d = cylinder diameter
The various processes are as follows:
1-2: isentropic compression (pvg = c)
2-3: instantaneous combustion (v = c)
3-4: isentropic expansion (pvg = c)
4-1: release of burned charge to atmosphere (v = c)
1-6 and 6-1: sweeping out of exhaust gases to atmosphere
Point 5: inlet port opens
5-6 and 6-5: charging the cylinder with fresh charge and scavenging action. Actual theoretical mean
effective pressure,
A
pma = t
vsa

vsa
pme = pma = pma La
vse Le
pma, pme = actual mep on the basis of actual and effective stroke respectively. In practice, the actual
conditions differ from the ideal as described below:
1. The compression and expansion processes do not follow the isentropic law strictly.
2. Instantaneous combustion at the end of compression is not possible. The actual pressure rise takes
place through some crank angle, resulting in rounding of the diagram.
3. The scavenging always takes place above atmospheric pressure.
4. Instantaneous fall of pressure at the time of release is not possible.

10.11.4  Two-stroke Diesel Engine


The theoretical and actual p-v diagrams for a two-stroke diesel engine are shown in Fig. 10.20. The
various processes are as follows:
1-2: isentropic compression (pvy = c)
2-3: instantaneous combustion (p = c)
3-4: isentropic expansion (pvy = c)
4-1: release of burned charge to atmosphere (v = c)
1-6 and 6-1: sweeping out of exhaust gases to atmosphere
5-6 and 6-5: charging the cylinder with fresh charge and scavenging action.
Point 5: inlet port opens.

M10_THERMAL ENGINEERING_SE_XXXX_CH010( Part-I).indd 504 6/15/2018 4:20:47 PM


Internal Combustion Engine Systems 505

3 3
2 2

At Ac
4
4 5
6
pa 1 5 6 1 5

vc vse vc vse
vsa vsa

pme At pme Ac

pma At pma Ac
Le Le
La La

(a) (b)

Figure 10.20 Theoretical and actual p-v diagrams for a two-stroke


diesel engine: (a) Theoretical, (b) Actual

10.12  ❐  CARBURETION
In SI engines, the air and fuel are mixed outside the engine cylinder and partly evaporated mixture is
supplied to the engine. The process of preparing this mixture is called carburetion. The device used
for this purpose is known as carburettor. The carburettor atomises the fuel and mixes it with air. This
complicated process is achieved in the induction system, which is shown in Fig. 10.21. The pipe that
carries the prepared mixture to the engine cylinder is called the intake manifold.
During the suction stroke, vacuum is created in the cylinder which causes the air to flow through
the carburettor and the fuel to be sprayed from the fuel jets. Due to high volatility of the SI engine
fuels, most of the fuel vaporises and forms a combustible fuel-air mixture. However, some of the
Exhaust
Throttle Intake
valve
Fuel valve
Exhaust
Air Mixture
Intake
Carburettor manifold Combustion
chamber

Cylinder
Piston

Figure 10.21  Induction system for SI engine

M10_THERMAL ENGINEERING_SE_XXXX_CH010( Part-I).indd 505 6/15/2018 4:20:48 PM


506 Chapter 10

Table 10.5  F:A ratios for various running conditions of SI engines

S. no. Running condition F:A ratio


1. Starting 0.2:1
2. Warming 0.15:1
3. Idling 0.085:1
4. Running with maximum thermal efficiency (0.06-0.07):1
5. Running with maximum power (80% (0.0775 − 0.08):1
throttle)
6. Full throttle 0.085:1
7. Acceleration 0.1:1

larger droplets may reach the cylinder in the liquid form and must be vapourised and mixed with air
during the compression stroke before ignition takes place by the electric spark.
The fuel-air ratios for various running conditions of SI engines are given in Table 10.5.

10.12.1  Simple Carburettor


The details of a simple carburettor are shown in Fig. 10.22. It consists of a float chamber, fuel dis-
charge nozzle and a metering orifice, a venturi, a throttle valve, and a choke. A float and a needle valve
system maintains a constant level of gasoline in the float chamber. If the amount of fuel in the float
chamber falls below the designed level, the float goes down, thereby opening the fuel supply valve and
admitting fuel. When the designed level is reached, the float closes the fuel supply valve, thus stopping
additional fuel flow from the supply valve and stopping additional fuel flow from the supply system.
The float chamber is vented either to the atmosphere or to the upstream side of the venturi.
During suction stroke, air is drawn through the venturi (or choke tube). As the air passes through
the venturi, the velocity increases, reaching a maximum at the venturi throat and pressure decreases to
a minimum. From the float chamber, the fuel is fed to a discharge jet, the tip of which is located in the
throat of the venturi, fuel is discharged into the air stream. To avoid overflow of fuel through the jet, the
level of the liquid in the float chamber is maintained at a level slightly below the tip of the discharge jet.
The throttle valve controls the amount of charge delivered to the cylinder, which is situated after
the venturi tube. As the throttle is closed, less air flows through the venturi tube and less quantity of
the air-fuel mixture is delivered to the cylinder; hence, the power output is reduced. The reverse takes
place when the throttle is opened.

10.12.2  Compensating Jet


The function of the compensating jet is to make the mixture leaner as the throttle opens progressively.
As shown in Fig. 10.23, the compensating jet is connected to the compensating well which is vented
to the atmosphere. The compensating well is supplied with fuel from the main float chamber through a
restricting orifice. With the increase in air flow rate, there is a decrease in the fuel level in the compen-
sating well, with the result that fuel supply through the compensating jet decreases. The compensating
jet, thus, progressively makes the mixture leaner as the main jet progressively makes the mixture richer.

M10_THERMAL ENGINEERING_SE_XXXX_CH010( Part-I).indd 506 6/15/2018 4:20:48 PM


Internal Combustion Engine Systems 507

Air

A 1 A
Idling air bleed

Choke valve Idling air restriction


Fuel discharge Vent to atmosphere
nozzle
Strainer

Z
Fuel in
B 2 B Float Float supply valve
Venturi
Float pivot
Float chamber

Fuel metering orifice


Throttle valve Idling adjustment

Figure 10.22  Simple carburettor

Compensating
well
Concentric jets
Compensating jet

Main jet H
Orifice

Figure 10.23  A compensating jet device

10.12.3  Theory of Simple Carburettor


The air from the atmosphere is sucked through the carburettor by the pressure difference across it
created when the piston moves on its suction stroke. The velocity of the air increases as it passes
through the venturi and reaches maximum at venturi throat (Fig. 10.24). The pressure also changes
and is maximum at section 2-2, because this is the minimum area in the induction track. The fuel is
sucked through the nozzle because of suction created in the venturi.

M10_THERMAL ENGINEERING_SE_XXXX_CH010( Part-I).indd 507 6/15/2018 4:20:49 PM


508 Chapter 10

To engine

Fuel from
fuel tank D
z
2 2
p2
Float
Section 2-2 is
chamber d venturi throat
Float

Air

Figure 10.24  Principles of a simple carburettor

1. Approximate analysis (neglecting compressibility of air): Let z be the height in metre of fuel
nozzle tip higher than the float chamber level. Assuming initial velocity of air to be negligible
(c1 = 0), density of air to be constant and considering sections at entrance and venturi throat, by
applying, Bernoulli’s theorem, we get
p1 p2 c22
= +
ra ra 2
where p1, p2 = pressure at sections 1 and 2, respectively, N/m2
c2 = velocity of air at section 2, m/s
ra = density of air, kg/m3
2( p1 − p2 )
∴ c2 =
ra
Mass of air per second,
m a = A2c2ra = A2 2 ra ( p1 − p2 ) (10.1)
where A2 = area of venturi throat, m2
Similarly, for the flow of fuel, we have
p1 p c 2f
= 2+ + zg
rf rf 2
where rf  = constant density of fuel, kg/m3
cf  = velocity of flow of fuel, m/s
2( p1 − p2 − zg r f )
∴ cf =
rf
Mass flow rate of fuel,

m f  = Af cf rf = Af 2 r f ( p1 − p2 − zg r f ) (10.2)

M10_THERMAL ENGINEERING_SE_XXXX_CH010( Part-I).indd 508 6/15/2018 4:20:52 PM


Internal Combustion Engine Systems 509

where Af  = cross-sectional area of fuel nozzle, m2

m f Af rf zg r f
Fuel-air ratio, = ⋅ × 1− (10.3)
m a A2 ra p1 − p2

Taking the coefficient of discharge of fuel nozzle and venturi into account, we have

m f Cdf Af rf zg r f
= × × × 1− (10.3a)
m a Cda A2 ra p1 − p2
where Cd = coefficient of discharge.
            If z = 0.
m f Cdf A f rf
= × × (10.3b)
m a Cda A2 ra

2. Exact analysis: Considering compressibility of air and applying steady flow energy equation to
sections 1 and 2, we get
c2 − c2
Q − W = h2 − h1 + 2 1
2
where h1, h2 = enthalpies at sections 1 and 2 respectively
Since Q = 0, W = 0 and c1 = 0
∴ c2 = 2( h1 − h2 )

 T 
   = 2c p (T1 − T2 ) = 2c pT1 1 − 2  (10.4)
 T1 
For isentropic flow between the atmosphere and venturi throat, we have
g −1
T2  p2  g
= (10.5)
T1  p1 

 g −1 
  p2  g 
∴ c2 = 2c pT1 1 −    (10.6)
  p1  
 
A1c1 A2 c2
m a =
Now             = (10.7)
v2 v2

where v = specific volume


Since p1v1g = p2 vg2

1 1
 p g RT  p  g
v2 = v1  1  = 1  1  (10.8)
 p2  p1  p2 

M10_THERMAL ENGINEERING_SE_XXXX_CH010( Part-I).indd 509 6/15/2018 4:20:56 PM


510 Chapter 10

Eq. (10.7) becomes

1  g −1 
A p  p g   p2  g 
m a = 2 1  2  2c pT1 1 −   
RT1  p1    p1  
 

 2 y +1 
A2 p1  p2  g  p2  g 
= 2c p   −    (10.9)
R T1  p1   p1  
 

 2 g +1 
 p2  g  p2  g 
2c p   −   
 p1   p1  
m a Cda A2 p1  
= × × × (10.10)
m f Cdf A f R T1 2 r f ( p1 − p2 − zg r f )

10.12.4  Limitations of Single Jet Carburettor


The drawbacks of single jet carburettor are as follows:
1. A single jet carburettor cannot provide a very rich mixture as required at the time of starting the
engine. This is because at low speed (starting or idling), the pressure difference causing the fuel
flow is very small as the throttle is nearly closed. It is not possible to discharge fuel to make the
mixture considerably rich.
2. It cannot provide a very rich mixture required for sudden acceleration of the engine.
3. For gradually increasing pressure difference over the jet (at higher speed of the engine), the weight
of the petrol discharged from a single jet increases at a greater rate than does the air supply.
Hence, a single jet carburettor gives a progressively richer mixture as the air speed increases when
set to given a correct mixture at low air speeds.
4. It cannot reduce the quantity of air flow during starting as required in cold weather conditions.
5. The automatic control of air and fuel according to the required conditions is not possible.
The carburettor used with a variable speed engine must fulfil all the following requirements:
1. It must atomise the fuel and mix it homogeneously with the air.
2. It must be able to run the engine smoothly without hunting or fuel wastage.
3. It must provide rich mixture during starting and idling.
4. It must provide a constant air-fuel ratio during normal running of the engine which is the maxi-
mum period.
5. It must provide a rich mixture required for quick acceleration of the engine.
6. It must be able to start the engine even in very cold weather conditions (during snowfall).
7. All operations should be automatic.
To fulfil the above requirements, the following devices are introduced.
1. A starting or a pilot jet (to start engine)

M10_THERMAL ENGINEERING_SE_XXXX_CH010( Part-I).indd 510 6/15/2018 4:20:58 PM


Internal Combustion Engine Systems 511

2. Compensating devices (to provide constant A:F ratio during normal operation conditions)
3. An automatic control of choke valve (to start the engine in cold weather)

10.12.5 Different Devices Used to Meet the Requirements


of an Ideal Carburettor
The main function of the carburettor is to vapourise the petrol in the current of air created by means
of engine suction and supply the required quantity of air and petrol mixture in proper proportion in
accordance with the load on the engine and its speed.
In the design of a carburettor, the following points should be considered:
1. It is seen that for a gradually increasing pressure difference over the fuel jet, the mass of petrol
discharged from a single jet increases at a greater rate than does the air supply. Hence, a carbu-
rettor of this type will give a progressively rich mixture as air speed is increased. There must be
some device to maintain A: F ratio constant over a wide range of engine speed.
2. To ensure rapid and complete combustion, it is necessary that the fuel should be finely divided
and intimately mixed with air supply. This can be done by proper design of venturi and inlet
manifold.
3. For starting and accelerating, a rich mixture must be supplied momentarily but the supply should
come to the correct mixture strength automatically when the engine attains the desired speed.
4. There must be a provision to supply extra-rich mixture to start the engine in a very cold weather
(where temperature falls to zero or sub-zero temperature).
5. The float in the float chamber of a carburettor must maintain the fuel level constant irrespective
of load or speed of the engine.

10.12.6  Complete Carburettor


In order to satisfy the demands of an engine under all conditions of operations, the following
additional systems are added to the simple carburettor:
1. Main metering system
2. Idling system
3. Power enrichment by economiszer system
4. Acceleration pump systems
5. Choke.

Main Metering System


The main metering system of a carburettor is designed to supply a nearly constant basis fuel-air ratio
over a wide range of speeds and loads. This mixture corresponds approximately to best economy at
full throttle (A/F ratio ≈ 15.6 or F/A ratio 0.064). Since a simple or elementary carburettor tends to
enrich the mixture at higher speeds automatic compensating devices are incorporated in the main
metering system to correct this tendency. These devices are as follows:
1. Use of a compensating jet that allows an increasing flow of air through a fuel passage as the mix-
ture flow increases.
2. Use of emulsion tube for air bleeding. In this device, the emphasis is on air bleeding alone.

M10_THERMAL ENGINEERING_SE_XXXX_CH010( Part-I).indd 511 6/15/2018 4:20:58 PM


512 Chapter 10

3. Use of a tapered metering pin that is moved in and out of the main or auxiliary fuel orifice either
manually or by means of some automatic mechanism changing the quantity of fuel drawn into the
air charge.
4. Back-suction control or pressure reduction in the float chamber.
5. Changing the position or jet in the venturi. The suction action is highest at the venturi throat,
therefore by raising the venturi the nozzle relatively moves to points with smaller suction and the
flow of fuel is decreased.
6. Use of an auxiliary air valve or port that automatically admits additional air as mixture flow
increases.
The main devices are explained in detail.
1. Compensating jet device: This device is shown in Fig. 10.23. In this device, in addition to the
main jet, a compensating jet is provided which is in communication with a compensating well.
  The compensating well is open to atmosphere and gets its fuel supply from the float chamber
through a restricting orifice. As the air flow increases, the level of fuel in the well decreases, thus
reducing the fuel supply through the compensating jet. The compensating jet thus tends towards
leanness as the main jet tends towards richness, the sum of the two remaining constant as shown in
Fig. 10.25. At even higher rates of air flow, when the compensating jet has been emptied, air is
bled through the compensating jet to continue the leanness effect, and incidentally, to assist in fuel
atomisation.
2. Emulsion tube or air bleeding device: In modern carburettors, the mixture correction is done
only by air bleeding. In this arrangement, the main metering jet is fitted about 25 mm below the
petrol level and it is called a submerged jet (see Fig. 10.26). The jet is situated at the bottom of
a well, the sides of which have holes which are in communication with the atmosphere. Air is
drawn through the holes in the well, the petrol is emulsified, and the pressure difference across the
petrol column is not as great as that in the simple or elementary carburettor. Initially, the petrol in
the well is at a level equal to that in the float chamber. On opening the throttle this petrol, being
subject to the low throat pressure, is drawn into the air. This continues with decreasing mixture
richness as the boles in the central tube are progressively uncovered. Normal flow takes place
from the main jet.
Mixture strength (Air/fuell ratio)

Compensating jet

Combined jet

Main jet

Air speed past jet

Figure 10.25  Variation of air-fuel ratio vs air flow with main and compensating jet

M10_THERMAL ENGINEERING_SE_XXXX_CH010( Part-I).indd 512 6/15/2018 4:20:58 PM


Internal Combustion Engine Systems 513

Throat
pressure

Atmospheric
pressure

Well
Main jet submerged jet

Figure 10.26  Correction in modern carburettors by air bleeding

3. Back-suction control or pressure reduction method: A common method of changing the air-
fuel ratio in large carburettors is the back-suction control as shown in Fig. 10.27. In this arrange-
ment, a relatively large vent line connects the carburettor entrance (say point 1) with the top of
the float chamber. Another line, containing a very small orifice line, connects the top of the float
chamber with the venturi throat (say point 2). A control valve is placed in the large vent line.
When the valve is wide open, the vent line is unrestricted the pressure in the float chamber equal
to p1, and the pressure difference acting on the fuel orifice is (p1 − p2). If the valve is closed, the
float chamber communicates only with the venturi throat and the pressure on the fuel surface will
be p2. Then ∆ pf will be zero, and no fuel will flow. By adjusting the control valve, any pressure

Control valve
Throttle
Orifice

Fuel level
h

Air

Figure 10.27  Back-suction control or pressure reduction method

M10_THERMAL ENGINEERING_SE_XXXX_CH010( Part-I).indd 513 6/15/2018 4:20:59 PM


514 Chapter 10

between p1 and p2 may be obtained in the float chamber, thus changing the quantity of fuel dis-
charged by the nozzle.
4. Auxiliary valve carburettor: An auxiliary valve carburettor is illustrated in Fig. 10.28. With
an increase of engine load, the vacuum at the venturi throat also increases. This causes the valve
spring to lift the valve admitting additional air and the mixture is prevented from becoming
over-rich.
5. Auxiliary part carburettor: An auxiliary port carburettor is illustrated in Fig. 10.29. By opening
the butterfly valve, additional air is admitted and at the same time the depression at the venturi
throat is reduced, decreasing the quantity of fuel drawn in. This method is used in aircraft carbu-
rettors for altitude compensation.

Throttle
Valve
spring

Valve Nozzle

Air

Figure 10.28  An auxiliary valve carburettor

Throttle
Butter-fly
valve

Nozzle

Air

Figure 10.29  An auxiliary port carburettor

M10_THERMAL ENGINEERING_SE_XXXX_CH010( Part-I).indd 514 6/15/2018 4:21:00 PM


Internal Combustion Engine Systems 515

Idling System
It has already been shown that at idling and low lead, the engine requires a rich mixture (about A/F 12:1).
However, the main metering system not only fails to enrich the mixture at low air flows but also supplies
no fuel at all at idling. For this reason, a separate idling jet must be added to the basic carburettor. An
example of idling jet is shown in Fig. 10.30. It consists of a small fuel line from the float chamber to a
little on the engine side of the throttle. This line contains a fixed fuel orifice. When the throttle is practi-
cally closed, the full manifold suction operates on the outlet to this jet. In addition, the very high velocity
past be throttle plate increases the suction locally. Fuel can, therefore, be lifted by the additional height
up to the discharge point, but this occurs only at very low rates of air flow. As the throttle is opened, the
main jet gradually takes over while the idle jet becomes ineffective. The desired air-fuel ratio for the
idling jet is regulated manually to idle adjust, which is a needle valve controlling the air bleed.

Power Enrichment of Economiser System


As the maximum power range of operation (75% to 100% load) is approached, some device must
allow richer mixture (A/F about 13: 1, F/A 0.08) to be supplied despite the compensating jet. Such a
device is the meter rod economiser shown in Fig. 10.31. The name economiser is rather misleading.
It stems from the fact that such a device provides a rich uneconomical mixture at high load demand
without interfering with economical operation in the normal power range. The meter rod economiser
shown in Fig. 10.31, simply provides a large orifice opening to the main jet as the throttle is opened
beyond a certain point.
The rod may be tapered or stepped. Other examples provide for the opening of auxiliary jets
through some linkage to the throttle movement or through a spring action when manifold vacuum is
lost as the throttle is opened.

Acceleration Pump System


It has already been shown that when it is desired to accelerate the engine rapidly, a simple carburettor
will not provide the required rich mixture. Rapid opening of the throttle will be immediately followed
by an increased airflow, but the inertia of the liquid fuel will cause at least a momentarily lean mix-
ture just when richness is desired for power. To overcome this deficiency, an acceleration pump is

Engine
suction
Throttle almost closed
high velocity
Air bleed

Idle
jet Idle adjust

Float chamber

Figure 10.30  Idling jet

M10_THERMAL ENGINEERING_SE_XXXX_CH010( Part-I).indd 515 6/15/2018 4:21:01 PM


516 Chapter 10

Open

Orifice

Figure 10.31  Meter rod economiser

provided; an example is shown in Fig. 10.32. The pump consists of a spring-loaded plunger. A linkage
mechanism is provided so that when the throttle is rapidly opened the plunger moves into the cylinder
and forces an additional jet of fuel into the venturi. The plunger is raised again against the spring force
when the throttle is partly closed. Arrangement is provided so that when the throttle is opened slowly,
the fuel in the pump cylinder is not forced into the venturi but leaks past plunger or some holes into
the float chamber.
Instead of the mechanical linkage shown, some carburettors have a plunger held up by a manifold
vacuum. Wherever that vacuum is reduced by the rapid opening of the throttle, a spring forces the
plunger down in a pumping action identical to that of the pump illustrated.

Choke
During cold starting period, at low cranking speed and before the engine has warmed up, a mixture
much richer than usual mixtures (almost five to 10 times more fuel) must be supplied because a large
fraction of the fuel will remain liquid even in the cylinder, and only the vapour fraction can provide
a combustible mixture with the air. The most common means of obtaining this rich mixture is by the
use of a choke, which is a butterfly-type valve placed between the entrance to the carburettor and the

Open
Plunger
Separate
accelerating
jet

Pump
C.V
C.V

Figure 10.32  Acceleration pump

M10_THERMAL ENGINEERING_SE_XXXX_CH010( Part-I).indd 516 6/15/2018 4:21:02 PM


Internal Combustion Engine Systems 517

Throttle

Choke

Figure 10.33  Choke valve with spring-loaded bypass

venturi throat as shown in Fig. 10.33. By partially closing the choke, a large pressure drop can be
produced at the venturi throat that would normally result from the amount of air flowing through the
venturi. This strong suction at the throat will draw large quantities of fuel from the main nozzle and
supply a sufficiently rich mixture so that the ratio of evaporated fuel to air in the cylinders is within
combustible limits. Choke valves are sometimes made with a spring loaded so that high-pressure drops
and excessive choking will not result after the engine has started and has attained a higher speed.
Some manufacturers make the choke operate automatically by means of a thermostat such that
when the engine is cold, the choke is closed by a bimetallic element. After starting and as the engine
warms up, the bimetallic element gradually opens the choke to its fully open position.
An alternative to the choke is the provision of auxiliary fuel jets that are opened manually or auto-
matically only as required.

Example 10.1
A single jet carburettor has to supply 5 kg of air per minute. The air is at a pressure of 1.013 bar
and at a temperature of 27°C. Calculate the throat diameter of the choke for air flow velocity of
90 m/s. Take the velocity coefficient as 0.8. Assume the flow to be incompressible and isentropic.
[IES, 1992]

Solution
Given that m a = 5 kg/min, p1 = 1.013 bar, T1 = 273 + 27 = 300 K, c2 = 90 m/s, Cda = 0.8
Applying Bernoulli’s theorem between inlet and throat, we have
c12 c2
h1 + = h2 + 2
2 2
Taking c1 = 0. c2 = 2( h1 − h2 ) = 2c p (T1 − T2 )

 T 
Actual velocity, c2 = Cda 2c pT1 1 − 2 
 T1 

M10_THERMAL ENGINEERING_SE_XXXX_CH010( Part-I).indd 517 6/15/2018 4:21:04 PM


518 Chapter 10

1
  g −1   2
   p2  g  
= Cda  2c pT1 1 −   
   p1  
  

 0.4 
  p  1 .4 
or 90 = 0.8 2 × 1.005 × 103 × 300 1 −  2  
  p1  
 

1
 p  3.5
or 1 −  2  = 0.021
 p1 
p2
or = 0.9284
p1
or p2 = 0.9284 × 1.013 = 0.9405 bar

p1 1.013 × 105
Density of air at inlet, ra1 = = = 1.1765 kg/m3
RT1 287 × 300
1 1
 p g 1.4
Density of air at throat, ra2 = ra1  2  = 1.1765 ( 0.9284 ) = 1.1157 kg/m3
 p1 

Mass flow rate at throat, m a = ra2 A2c2

5 p
= 1.1157 × d22 × 90
60 4
throat dia, d2 = 0.0325 m or 32.5 mm

Example 10.2
A six-cylinder, four-stroke engine, 80 mm in diameter and 120 mm stroke length runs at 3600 rpm.
The volumetric efficiency of the engine is 0.8. If the maximum head causing the flow is limited
to 11.765 cm of mercury, find the throat diameter of the venturi required. Find the diameter of
the nozzle orifice if the desired A: F ratio is 15:1. Take Cda = 0.9, Cdf = 0.7, ra = 1.3 kg/m3 and
rf = 720 kg/m3.

Solution
Given that i = 6, d = 0.08 m, L = 0.120 m, N = 3600 rpm, hvol. = 0.8, hHg = 11.765 cm
p 2  N
Volume flow rate of air, Va = d  L hvol × i
4 2 × 60
p 3600
= (0.08)2 × 0.120 × 0.8 × 6 × = 0.0868 m3/s
4 2 × 60

M10_THERMAL ENGINEERING_SE_XXXX_CH010( Part-I).indd 518 6/15/2018 4:21:08 PM


Internal Combustion Engine Systems 519

Mass flow rate of air, m a = ra Va = 1.3 × 0.0868 = 0.113 kg/s


hw = hHg × gHg = 11.765 × 13.6 = 160 cm of water

m a = 14 Cda Aa ra hw
p
or 0.113 = 14 × 0.9 × (0.01 da)2 1.3 × 160, da in cm
4
or da = 2.8 cm
2
ma Cda Aa ra Cda  da  ra
= ⋅ =  
m f Cdf A f rf Cdf  df  rf
2
15 0.9  2.8  13
or =  
1 0.7  d f  720
or df  = 0.169 cm or 1.69 mm

Example 10.3
The throat diameter of a carburettor is 80 mm and the nozzle diameter is 5.5 mm.The nozzle lip is
6 mm. The pressure difference causing the flow is 0.1 bar. Find (a) the air-fuel (A:F) ratio supplied
by the carburettor neglecting nozzle lip. (b) A:F ratio considering nozzle lip and (c) the mini-
mum velocity of air required to start the fluid flow. Neglect air compressibility. Take Cda = 0.85,
Cdf = 0.7, ra = 1.2 kg/m3 and rf =750 kg/m3.

Solution
Given that da = 80 mm, df  = 5.5 mm, z = 6 mm, ∆p = 0.1 bar, Cda = 0.85, Cdf  = 0.7, ra = 1.2 kg/m3,
rf  = 750 kg/m3

m a Cda Aa ra 0.85  80  2 1.2


(a) = ⋅ =   = 10.2:1
m f Cdf A f rf 0.7  5.5  750

m a Cda Aa ra  hw 
(b) = ⋅ =
m f Cdf A f r f  hw − hwl 
0.1 × 105
hw = ∆p = 0.1 × 105 N/m2 = = 102 cm of H2O
98
hwl = zgf = 6 × 10−1 × 0.75 = 0.45 cm

m a 0.85  80  2 1.2  102 


= ×    = 10.28:1
m f 0.7  5.5  750  102 − 0.45 

hwl 0.45
(c) ca = 14 = 14 = 8.57 m/s
ra 1.2

M10_THERMAL ENGINEERING_SE_XXXX_CH010( Part-I).indd 519 6/15/2018 4:21:12 PM


520 Chapter 10

Example 10.4
A 4-stroke petrol engine of 2 litres capacity is to develop maximum power at 4000 rpm. The vol-
umetric efficiency at this speed is 0.75 and the air-fuel ratio is 14:1. The venturi throat diameter is
28 mm. The coefficient of discharge of venturi is 0.85 and that for fuel jet is 0.65. Calculate (a) the
diameter of the fuel jet and (b) the air velocity at the throat.
The specific gravity of petrol is 0.76. Atmospheric pressure and temperature are 1 bar and 17°C
respectively.[IAS, 2002]

Solution
Given: Vs = 2 litres or 2 × 10−3 m3, N = 4000 rpm, n = 2, hvol = 0.75, A/F = 14/1, d2 = 28 mm, Cda =
0.85, Cdf  = 0.65, rf = 760 kg/m3 , p1 = 1 bar, T1 = 17 + 273 = 290 K

p1 1 × 105
ra1 = = = 1.2 kg/m3
RT1 287 × 290

2
m f Cdf Af rf Cdf  df  rf
(a) = × × = ×  ×
m a Cda A2 ra Cda  d2  ra

2
1 0.65  d f 
∴ = × × 760 /1.2
14 0.85  da 

df
or = 0.32733
d2

or df = 28 × 0.32733 = 9.165 mm
∴ Diameter of fuel jet = 9.165 mm
(b) Volume of air supplied per cycle = stroke volume × hvol
V1 = 2 × 10−3 × 0.75 = 1.5 × 10−3 m3
p1V1 1 × 105 × 1.5 × 10 −3
Mass of this air per cycle at suction condition, m a = =
RT1 287 × 290

= 1.8 × 10−3 kg/s

ma × N 1.8 × 10 −3 × 4000
Mass of air supplied, m a = = = 0.06 kg/s
60 × n 60 × 2
m a = A2 C2 ra2(ra1 = ra2)
p
or 0.06 = (28 × 10−3)2 × c2 × 1.2
4
or c2 = 81.2 m/s

M10_THERMAL ENGINEERING_SE_XXXX_CH010( Part-I).indd 520 6/15/2018 4:21:16 PM


Internal Combustion Engine Systems 521

Example 10.5
The venturi of a simple carburettor has a throat diameter of 20 mm and the fuel orifice has a
diameter of 1.12 mm. The level of petrol surface in the float chamber is 6.0 mm below the throat
venturi. Coefficient of discharge for venturi and fuel orifice are 0.85 and 0.78 respectively. Specific
gravity of petrol is 0.75. Calculate (a) the air-fuel ratio for a pressure drop of 0.08 bar, (b) petrol
consumption in kg/hr and (c) the critical air velocity. The intake conditions are 1.0 bar and 17°C.
For air cp = 1.005 and cv = 0.718 kJ/kg-K. [IAS, 2003]

Solution
Given: d2 = 20 mm, df  = 1.12 mm, z = 6 m, Cda = 0.85, Cdf = 0.78, sf = 0.75, ∆ p = 0.08 bar, p1 = 1.0
bar, T1 = 290 K, cp = 1.005 kJ/kg.K, cv = 0.718 kJ/kg.K
p1 1 × 105
Density of air ra = = = 1.2 kg/m3
RT1 287 × 290

m f Cdf Af rf zg r f
(a) Fuel-air ratio, = × . × 1−
m a Cda A2 ra ∆p

6 × 10 −3 × 9.81 × 750
2
0.78  1.12  750
= ×  × × 1−
0.85  20 1.2 0.08 × 105
= 0.07174
1
A/F ratio = = 13.94
0.07174

(b) Petrol consumption, m f = Cdf A f 2rf ( ∆p − zg rf )

p
= 0.78 × (1.12 × 10−3)2 2 × 750 (0.08 × 105 − 6 × 10 −3 × 9.81 × 750)
4

= 2.6547 × 10−3 kg/s or 9.557 kg/hour

2 ∆p 2 × 0.08 × 105
(c) Critical air velocity c2 = = = 115.47 m/s
ra 1.2

Example 10.6
The air-fuel ratio of a mixture supplied to an engine by a carburettor is 15. The fuel consumption
of the engine is 7.5 kg/h. The diameter of the venturi is 2.2 cm. Find the diameter of fuel nozzle if
the lip of the nozzle is 4 mm. Take rf  = 750 kg/m3, Cda = 0.82, Cdf  = 0.7, atmospheric pressure =
1.013 bar and temperature = 25°C. Neglect compressibility effect of air.

M10_THERMAL ENGINEERING_SE_XXXX_CH010( Part-I).indd 521 6/15/2018 4:21:19 PM


522 Chapter 10

Solution
Given that A: F = 15:1, m f = 7.5 kg/h, da = 2.2 cm, z = 4 mm, r f = 750 kg/m3, Cda = 0.82, Cdf = 0.7,
p = 1.013 bar, T = 273 + 25 = 298 K
p 1.013 × 105
Density of air, ra = = = 1.185 kg/m3
RT 287 × 298
m f × A:F ratio
Mass flow rate of air,     m a  =
3600
7.5 × 15
= = 0.03125 kg/s
3600

Now m a = Aa Cda


  2ra ⋅ ∆p

p
or 0.03125 = (2.2 × 10−2)2 × 0.82 2 × 1.185 × ∆p
4
or ∆p = 4260 N/m2

m f  = Af Cdf 2r f ( ∆p − zg ⋅ r f )

7.5 p
or = (d × 10−3)2 × 0.7 2 × 750( 4260 − 4 × 10 −3 × 9.81 × 750)
3600 4 f
or df  = 1.245 mm

Example 10.7
A four-stroke petrol engine of 1710 cm3 capacity is to be designed to develop maximum power at
5400 rpm, when the mixture A:F ratio supplied to the engine is 13:1. Two carburettors are to be
fixed. The maximum velocity of air is limited to 107 m/s. Find the diameters of venturi and fuel
nozzle jet, if hvol (engine) = 0.7, Cda = 0.85, Cdf  = 0.66, nozzle lip = 6 mm and rf  = 750 kg/m3.
Atmospheric pressure and temperature are 1.013 bar and 300 K. Take cp = 1 kJ/kgK.

Solution
Given that Ve = 1710 cm3, N = 5400 rpm, A:F = 13, ca = 107 m/s, hvol = 0.7, Cda = 0.85, Cdf = 0.66,
z = 6 mm, rf  = 750 kg/m3, p = 1.013 bar, T = 300 K, cp = 1 kJ/kgK
Volume of air supplied to engine at STP,
N 5400
V =Ve × h = 1710 × 10−6 × × 0.7 = 0.054 m3/s
2 × 60 vol 2 × 60
Air flow through each carburettor,
V 0.054
Va = = = 0.027 m3/s
2 2
Mass flow rate of air,
pVa 1.013 × 105 × 0.027
m a = = = 0.032 kg/s
RT 287 × 300

M10_THERMAL ENGINEERING_SE_XXXX_CH010( Part-I).indd 522 6/15/2018 4:21:26 PM


Internal Combustion Engine Systems 523

Air velocity through venturi,


1/ 2
  g −1  
   p2  g  
ca =  2c pT1 1 −   
   p1  
  
1/ 2
  1 
   p  3 .5  
or 107 =  2 × 1 × 103 × 300 1 −  2   
   p1   
  
p2
or = 0.935
p1
or p2 = 0.935 × 1.013 = 0.947 bar
Assuming the flow to be isentropic,
p1V1g = p2V2g
1 1
 p y  1  1.4
V2 = V1 .  1  = 0.027  = 0.0287 m3/s
 p2   0.935 
Volume of air passing through each carburettor,
V = A c C
2 a a da
p
or 0.0287 = (d × 10−2)2 × 107 × 0.85, da in cm
4 a
or da = 2 cm
Mass flow rate of fuel per carburettor,
m a 0.032
m f = = = 0.00246 kg/s
A: F 13
Now m f = Af Cdf 2r f ( ∆p − zg r f )

∆p = p1 − p2 = 1.013 − 0.947 = 0.066 bar


p
0.00246 = (d × 10−3)2 2 × 750 (0.066 − 6 × 10 −3 × 9.81 × 750) , where df is in mm.
4 f
or df  = 1.23 mm
m f × 3600 6.276 × 10 −3 × 3600
BSFC = = = 0.292 kg/kWh
BP 77.283

M10_THERMAL ENGINEERING_SE_XXXX_CH010( Part-I).indd 523 6/15/2018 4:21:32 PM


524 Chapter 10

Example 10.8
A 4-stroke petrol engine has a swept volume of 2.0 litres and is running at 4000 rpm. The volu-
metric efficiency at this speed is 75% and the air-fuel ratio is 14:1. The venturi throat diameter of
the carburettor fitted to the engine is 30 mm. Estimate the air velocity at the throat if the discharge
coefficient for air is 0.9. The ambient conditions are, pressure = 1.0 bar, temperature = 20°C. Cal-
culate the diameter of the fuel jet if the fuel density is 760 kg/m3. For air cp = 1.005 kJ/kgK and R
= 287 J/kgK. Assume Cdf = 1.0. [IAS, 2005]

Solution
Given: n = 2, Vs = 2 litres or 2 × 10−3 m3, N = 4000 rpm, hvol = 0.75, A/F = 14, d2 = 30 mm, Cda = 0.9,
Cdf = 1.0, p1 = 1.0 bar, T1 = 293 K, rf  = 760 kg/m3, cp = 1.005 kJ/kgK, R = 287 J/kgK
N
Actual volume sucked by the engine is, V1 = Vs ×h vol ×
n × 60

2 × 10 −3 × 0.75 × 4000
= = 0.05 m3/s
2 × 60
V1 0.05
Air velocity at venturi throat, c2 = = = 70.75 m/s
A2 p
(0.03)2
4

 g −1 
  p2  g 
Also c2 = Cda 2c p T1 1 −   
  p1  
 

 0.4 
  p2  1.4  3
or 70.75 = 0.9 2 × 1.005 × 10 × 293 1 −   
  p1  
 
p2
or = 0.964
p1
or p2 = 0.964 bar
∆p = p1 − p2 = 1 − 0.964 = 0.036 bar
Mass of air flowing through the venturi/s,
 g +1 
2 /g
p1  p2   p2  g 
m a = Cda A2 C p   −  
R T1  p1   p1  
 

p 1 × 105
= 0.9 × (0.03)2 × 1.005 × 103 [(0.964)2 /1.4 − (0.964)2.4 /1.4 ] = 0.0408 kg/s
4 287 293

M10_THERMAL ENGINEERING_SE_XXXX_CH010( Part-I).indd 524 6/15/2018 4:21:35 PM


Internal Combustion Engine Systems 525

∴ m f  = 0.0408/14 = 2.916 × 10−3 kg/s


Also m f = Cdf A f 2rf . ∆p

p 2
or 2.916 × 10−3 = 1.0 × d 2 × 760 × 0.036 × 105
4 f
or df  = 1.26 × 10−3 m or 1.26 mm

Example 10.9
An engine fitted with a single jet carburettor having a jet diameter of 1.25 mm has a fuel consump-
tion of 6 kg/hr. The specific gravity of fuel is 0.7. The level of fuel in the float chamber is 5 mm
below the top of the jet when the engine is not running. Ambient conditions are 1 bar and 17°C. The
fuel jet diameter is 0.6 mm. The discharge coefficient of air is 0.85. Air-fuel ratio is 15. Determine
the critical velocity of flow at throat and the throat diameter. Express the pressure at throat in mm
of water column. Neglect compressibility effect. Assume discharge coefficient of fuel flow is 0.60.

Solution
Given: dj = 1.25 mm, m j = 6 kg/hr, Sf  = 0.7, z = 5 mm, p1 = 1 bar, T1 = 273 + 17 = 290 K, df  = 0.6 mm,
Cda = 0.85, A:F = 15, Cdf = 0.60.
pV = mRT

p1 1 × 105
ra = = = 1.20 kg/m3
RT1 287 × 290

rf  = Sf rw = 0.7 × 103 = 700 kg/m3


Applying Bernoulli’s equation for air flow at entrance and venturi throat, we have

p1 p2 ca22
= +
ra ra 2

2( p1 − p2 ) 2 ∆p
or ca 2 = =
ra ra

ca22 ra
or ∆p =
2
Mass of air flow per second,

m = C A c r = c A 2 ra ( p1 − p2 )
a da 2 a2 2 da 2

= Cda A2 2ra ⋅ ∆p
For the flow of fuel, we have

p1 p c 2f 2
= 2+ + zg
rf rf 2

M10_THERMAL ENGINEERING_SE_XXXX_CH010( Part-I).indd 525 6/15/2018 4:21:39 PM


526 Chapter 10

2( p1 − p2 − zg rf )  ∆p − zg r f 
or cf 2 = = 2 
rf  rf 
Mass of fuel flow per second,

m f = Cdf Af cf 2 pf = Cdf Af 2 r f ( p1 − p2 − zg rf ) = Cdf A f 2 r f ( ∆p − zg r f )

m f Cdf A f rf zg r f
F : A ratio, = 1−
m a Cda A2 ra p1 − p2

Critical velocity of flow at throat,


( p1 − p2 ) rf
ca 2 = 2 = 2 gz [ ∴ p1 − p2 = gzrf]
ra ra

700
= 2 × 9.81 × 5 × 10 −3 × = 7.56 m/s
1.2

m f  = Cdf rf Af cf

6 p
or = 0.6 × 700 × (0.6 × 10−3)2 × cf
3600 4

or cf  = 14.035 m/s

1 × 105 p (14.035)2
Now = 2 + 5 × 10−3 × 9.81 +
700 700 2

or p2 = 0.31 bar

2( p1 − p2 ) 2(1 − 0.31) × 105


ca2 = = = 339.12 m/s
ra 1.2

m / m  = 15
a  f

or m = 15 × 6 = 0.025 kg/s
a
3600

Again m = C r C A
a da a a2 2
p 2
or 0.025 = 0.85 × 1.2 × 339.12 × d
4 2
or d2 = 9.6 mm
Pressure at throat = p2 /rw ghw
0.31 × 105
or hw = = 3.16 m of water
103 × 9.81

M10_THERMAL ENGINEERING_SE_XXXX_CH010( Part-I).indd 526 6/15/2018 4:21:45 PM



Internal Combustion Engine Systems 527

10.13  ❐  FUEL INJECTION SYSTEMS IN SI ENGINES


Although the modern carburettor is cheap and reliable, it has several inherent disadvantages that make
the supply of correct A:F mixture always difficult. The problem is further accentuated when a single
carburettor has to supply the mixture to a multi-cylinder engine. In addition, the throat restricts the
flow of air to the engine and the maximum power. The maximum power can be increased by using a
large throat as this affects the economy at low speed because of low air speed and worst control on the
fuel spray and atomisation. Multiple carburettors assist the distribution problem but they also increase
fuel consumption due to low air velocity at low engine speed and capital cost.
For these reasons, many attempts have been made to design a satisfactory petrol injection system,
in which each cylinder is supplied with its correct quantity of petrol for each working cycle under all
operating conditions. It is also claimed that the system devised to do this, allow the engine to produce
more power and gives less vapourisation troubles.
The advantages of fuel injection system over the conventional carburettor have been appreciated
for many years. The quest for improved engine performance linked with fuel economy with legislation
regarding to control of exhaust emission has enormously increased the use of fuel injection system.
The incorporation of electronic control systems has also considerably helped the development of effi-
cient and commercially viable systems.
There are two different injection systems for SI engines which are mechanically operated as follows:
1. Combustion chamber injection
2. Continuous port injection
The combustion chamber injection system is just similar to CI engines. However, this system is not
adopted as today’s emission and fuel economy requirements make it impractical.

10.13.1 Continuous Port Injection System (Lucas Mechanical


Petrol Injection System)
Figure 10.34 shows a simplified line diagram of the aforementioned system. The petrol is sucked from
the tank by a pump and pressurised petrol at 3 bar is supplied through a distributor to the fuel injector
to a particular cylinder. The relief valve shown in figure maintains the pressure and allows excess
petrol to return to the tank. The pump may be driven by engine or by engine or by a separate electric
motor. The latter is preferred because it starts pumping at its normal running speed and pressurises as
soon as ignition is switched on.
In this system, petrol is continuously injected into the inner port as shown in Fig. 10.34 at a varied
rate. The distribution may be made by having a separate metering pump for each cylinder, timed by
the arrangement of a series of cams on one cam shaft or by having one pump operated by a single cam
with a lob for each cylinder (similar to ignition system) that feeds to distribution unit which passes the
petrol to each inlet port in turn. The unit may be driven by shaft, chain, or V-belt.
Nowadays, electronically controlled fuel injection systems are commonly used as they function
rapidly and respond automatically to the change in manifold air pressure, engine speed, crankshaft
angle, and many other secondary factors. The electronic control unit assesses data (manifold pressure,
engine speed, crank angle) received from various sensing devices and then adjusts the A:F supply for
the best performance of the engine.

M10_THERMAL ENGINEERING_SE_XXXX_CH010(Part-II).indd 527 6/15/2018 4:54:37 PM


528 Chapter 10

Metering distribution Air intake


and control unit
Throttle valve
Inlet duct
Nozzle
Pressure Control
Petrol Valve
relief spring
tank
valve Control Pa
Pump wedge Engine
Filter
Distributor cylinder

Figure 10.34  Lucas mechanical petrol injection system


This system has to contain a means of supplying additional fuel for cold starting, during warming,
and enriching the mixture during acceleration. Ice formation is virtually impossible with this system
and the danger of vapourisation is minimised because petrol is under pressure right up to the injection
point.

10.13.2  Electronic Fuel Injection System


The amount of fuel supplied to the engine is controlled by three factors namely, injector cross sec-
tional area, fuel pressure, and duration of injection. Injector orifice size is fixed for a particular design.
Fuel pressure supplied is controlled by electric pump and regulator. Therefore, the only variable fac-
tor in electronic fuel injection is to control the period of injection. This is done by translating the
data supplied to the electronic control unit from the various sensors into electric pulses which are in
turn relayed to the solenoids which operate the injectors thus determining the moment and period of
injection.
The main parts of the system are the injector and electronic control unit.

Injectors
The solenoid operated fuel injector is shown in Fig. 10.35(a). It consists of a valve body and a needle
valve to which the solenoid plunger is rigidly attached. The fuel is supplied to the injector under pres-
sure from the electric fuel pump passing through the filter. The needle valve is pressed against a seat
in the valve body by a helical spring to keep it closed until the solenoid winding is energised. When
a current pulse is received from the electronic control unit, a magnetic field builds up in the solenoid
coil which attracts a plunger and lifts the needle valve from its seat. This opens the path to pressurised
fuel to emerge as a finely atomised spray.

Electronic Control Unit


This unit is the heart of electronic injection system which is responsible for metering the quantity
of fuel supplied to each cylinder. The unit contains a number of printed circuit boards on which, a
series of transistors, diodes, and other electronic components are mounted. This makes the vital data

M10_THERMAL ENGINEERING_SE_XXXX_CH010(Part-II).indd 528 6/15/2018 4:54:38 PM



Internal Combustion Engine Systems 529

A
B
C

E
Q
Cold start mixture F
G Electric
J Solenoid
connection
O P armature
H Solenoid
Nozzle Filter
winding
N valve
S R
T

Starter switch

K M Spring
L

(a) (b)

Figure 10.35  Fuel injection valve: (a) Solenoid operated valve, (b) Electronically controlled valve

analysing circuits responding to various input signals. After processing the input data, the power out-
put circuits in the control unit generates current pulses which are transmitted to the solenoid injectors
to operate the injector for the required period.
All the electrical units of this system are connected to a cable terminating in a 25-pole plug to
the mating socket. Hence, as soon as the ignition switch is actuated, the fuel pump is switched on
and the fuel injection system becomes operational. The arrangement of this system is shown in
Fig. 10.35(b).

Advantages of Petrol Injection System


1. The fuel injection system can precisely match fuel delivery to engine requirements under all load
and speed conditions. This reduces fuel consumption with no loss of engine performance.
2. The manifold in an injection system carries only air, so there is no problem of the air and fuel
separation and design of manifold becomes simple.
3. Due to absence of venturi which abstracts the air passage, the petrol injection results in better
volumetric efficiency and increased power.
4. Fuel injection system is relatively free from icing and surge when tilted, cornering, and braking.
5. Starting and acceleration is much simpler than the carburettor system.
6. It provides an identical A:F ratio mixture to all the cylinders and maintains better balancing. In
addition, the engine can work more economically closer to its lean limit, whereas in a carburet-
tor engine, one or two ratios richer than best economy mixture are to be used to compensate for
driveability. This gives lower specific fuel consumption.
7. By maintaining a precise A:F ratio according to engine requirements, exhaust emissions are low-
ered. The improved air fuel flow in injection system also helps to reduce emission levels.

M10_THERMAL ENGINEERING_SE_XXXX_CH010(Part-II).indd 529 6/15/2018 4:54:40 PM


530 Chapter 10

Disadvantages of Petrol Injection System


1. The major disadvantage is its high capital cost due to precise and complicated components of
electronic circuit.
2. The maintenance of this system is difficult and costly as this system contains 4 times the compo-
nents of mechanical system. A Junker 12 cylinder engine has 1576 parts against 433 parts used
with 12 cylinder Mercedes carburettor system.
3. The weight of the mechanical injection system is considerably higher than that of a
carburettor.
4. The mechanical injection system has many wearing parts such as camshaft, rotor, and so on.
5. The injection system generates more noise.

10.13.3  Rotary Gate Meter Fuel Injection System


We have seen that a conventional fuel injection system requires individual injector to inject fuel
into each cylinder. The additional components increase the overall cost compared to the carburettor
system.
This new system, shown in Fig. 10.36, reduces cost because fuel is injected at a central location
and then distributed to each cylinder. There is a rotary gate valve at the heart of the system, controlled
by engine intake air to regulate fuel injection. As the volume of the air taken into the engine cylinder
increases, the gate valve tips to increase the amount of fuel injected into the engine.
Incidentally, Robert Bosch Corporation, Germany, reported that this system helped in optimising
fuel and distribution as the fuel is injected into the air stream at the point of highest velocity.

Rotary gate is
turned by engine
intake air

Fuel is injected at
point of highest
air velocity
Warm up regulator

Fuel
filter
Pressure regulator

Tank Pump

Figure 10.36  Rotary gate make fuel injection system

M10_THERMAL ENGINEERING_SE_XXXX_CH010(Part-II).indd 530 6/15/2018 4:54:41 PM



Internal Combustion Engine Systems 531

10.14  ❐  FUEL INJECTION IN CI ENGINES


Unlike SI engines, the fuel in CI engines is supplied at a very high pressure, partly during the compression
stroke and partly during the power stroke. The air is taken in during the suction stroke and compressed
to a high pressure (30–70 bar) and high temperature (500°C–700°C) according to the compression ratio
used (12–20). The high temperature of air at the end of the stroke is sufficient to ignite the fuel.
As the fuel is injected in a high pressure air, the pressure of fuel injected lies between 100–200 bar.
During the process of injection, the fuel is broken into very fine droplets. The droplets vapourise, tak-
ing the heat from the hot air and form a combustible mixture and start burning. As the burning starts,
the vapourisation of fuel is accelerated as more heat is available. As the combustion advances, the
amount of oxygen available for burning reduces and therefore, the heat release rate is reduced.
The period between the injection and ignition of fuel is known as delay period (ignition delay). This
lies between 0.001 and 0.002 seconds, according to the speed of the engine. This counts for nearly 25°
crank rotation. The whole performance of the engine is totally dependent on the delay period. The less
is the delay period, better is the performance of the engine.
The main functions of the injection system are as follows:
1. To supply the correct quantity of fuel to be injected as per the load of the engine and increase in
speed for automobile engines.
2. To supply the fuel within a precisely defined period of the cycles.
3. To control the rate of fuel injection such that it should result in the desired heat release pattern.
4. To atomise the fuel into very fine particles.
5. To distribute the fuel uniformly in the combustion chamber of the engine and results in rapid
mixing of fuel and air.
6. Injection starts and stops sharply. There should not be any after injection.

10.14.1  Types of Injection Systems


Injection system may be divided into two general types, as follows:
1. Air injection system
2. Airless or solid or mechanical injection

Air Injection System


It was first developed by Rudolf Diesel. The arrangement of the system is shown in Fig. 10.37. In this
system, air and fuel are injected into the cylinder during the supply of fuel. The required pressure of
the air for injecting the fuel is about 70 bar or higher.
A fuel pump is driven by the engine itself. A cam shaft operates the fuel pump through a cam and the
power required to rotate the cam shaft is taken from the main shaft of the engine with the help of gears
and discharges a definite quantity of fuel into the injection valve as shown in Fig. 10.37. The injection
valve is mechanically opened and high pressure air drives the fuel charge and some air into the com-
bustion chamber. The amount of fuel delivered is under the control of oil pump suction valve, which is
operated by a governor.
The air pressure is raised to about 70 bar by a three-stage compressor (as shown in Fig. 10.37) provid-
ing intercooling. The compressor is also operated by the engine. The high-pressure air projects the fuel
into the combustion chamber and atomises it. Nowadays, such systems are rarely used in diesel engines.

M10_THERMAL ENGINEERING_SE_XXXX_CH010(Part-II).indd 531 6/15/2018 4:54:41 PM


532 Chapter 10

Fuel valve
Cam operating
the fuel valve

Inlet Exhaust
Governor
Starting
bank

3-stage
compressor
Fuel pump

Power from
crank shaft

Figure 10.37  Airless injection system

The advantages and disadvantages of this system are as follows:

Advantages
1. It provides better atomisation and distribution of fuel.
2. As the combustion is more complete, the brake mean effective power (BMEP) is higher than with
other types of injection systems.
3. It allows to use the inferior fuels.

Disadvantages
1. It requires a complicated mechanism to run the compressor.
2. The weight of the engine increases.
3. A part of the power is used to drive the compressor and the BHP of the engine is reduced.

Airless or Solid Injection


In this system, the fuel is supplied at a very high pressure (150 bar) from the fuel pump to the fuel
injector from where it is injected to the combustion chamber with the help of an injector. The main
parts of this system are the fuel pump and the fuel injector. The fuel pump is operated by a cam which
is mounted on a cam shaft. The power required to operate the cam is taken from the engine crank shaft.
Depending on the location of the fuel pumps and injectors, and the method used to meter the fuel,
solid injection may further be classified as follows:
1. Common rail system: In this system, there is a single high pressure pump, which supplies high
pressure fuel to a common header (rail). The accumulator is connected to different cylinders by
a separate fuel line through the fuel nozzle as shown in Fig. 10.38. The pressure in the accumu-
lator is maintained constant with the help of a pressure relief valve. The fuel is supplied to each

M10_THERMAL ENGINEERING_SE_XXXX_CH010(Part-II).indd 532 6/15/2018 4:54:42 PM



Internal Combustion Engine Systems 533

Mechanically
actuated
spray valve

Fuel lines to
Control other cylinders
wedge
High pressure
Camshaft relief valve

Header or “common rail”


Fuel from
day tank High pressure
pump
Pump drive
comshaft

Figure 10.38  Common rail system

cylinder by operating the respective fuel valve with the help of a cam mechanism driven by the
engine crank shaft.
  The quantity of fuel injected and the timing of injection are controlled by fuel valve and not
by the injection pump. The arrangement of the fuel valve (metering meter) and the fuel nozzle
is shown in Fig. 10.38. This system uses spring loaded injection valves which open and close by
mechanical means.
  The pressure used in this system ranges between 110–300 bar, according to the compression
ratio used for the engine design.
  The advantages and disadvantages of the system are listed below.
Advantages
1. It fulfils the requirements of either the constant load with variable speed or constant speed
with variable load.
2. Only one pump is sufficient for a multi-cylinder engine.
3. Variation of pump supply pressure will affect all the cylinders uniformly.
4. The arrangement of the system is very simple and maintenance cost is less.
Disadvantages
1.  Very accurate design and workmanship are required.
2.  There is tendency to develop leaks in the injection valve.
2. Distributor system: This system, like the common rail system, employs a single high pressure
pump as shown in Fig. 10.39. The high pressure pump in this system is used for metering and
compressing the fuel and then the fuel is delivered to the common rotating distributor. The fuel is
supplied to each cylinder by the distributor. In every cycle, the injection strokes of the pump are

M10_THERMAL ENGINEERING_SE_XXXX_CH010(Part-II).indd 533 6/15/2018 4:54:44 PM


534 Chapter 10

Nozzle

Distributor Metering and


block pressure pump

Control
rock

Cams Cam

Primary pump

Figure 10.39  Distributor system


equal to the number of cylinders. The quantity of fuel supplied and the timing of fuel supply are
done by single plunger (main pump). Therefore, equal amount of fuel is supplied to each cylinder
and at the same point in the cycle. The function of the distributor is merely to select the cylinder
to receive the fuel.
  The distributor block selects a particular cylinder according to the cam coming in contact with
the distributor as shown in Fig. 10.39. The appropriate valve opens just before the beginning of
injection and oil is supplied to the required cylinder.
3. Individual pump and nozzle system: This differs from constant pressure injection both in design
and operation of the pump and fuel injector. Each injector has a separate pump and the injector
contains a spring-loaded hydraulically operated automatic plunger valve. No separate mechanism
is required to operate it.
  In this system, separate pumps, each (depending on the number of cylinder) driven individually
or a single pump having four plungers in a common block may be used. In this case, the single
pump is driven by the crank shaft through a single cam shaft having individual cam for each
cylinder.
  The arrangement of the individual pump system is shown in Fig. 10.40 with all pumps in one
block, four plungers in one barrel and a common cam shaft.
  The design of this type of pump must be accurate as the volume of fuel injected per cycle is
1/20,000 of the engine displacement at full load and 1/100,000 of the engine displacement during
idling. The time allowed for injecting such a small quantity of fuel is very limited (about 1/450
second at 1500 rpm of the engine providing injection through a 20° angle). The pressure require-
ments vary from 100–300 bar.
  A comparison of various fuel injection systems is given in Table 10.6.

M10_THERMAL ENGINEERING_SE_XXXX_CH010(Part-II).indd 534 6/15/2018 4:54:45 PM



Internal Combustion Engine Systems 535

Injection
nozzle

Pump with
High pressure
individual
fuel lines
cylinder for
each nozzle

Control
Pump camshaft
rack

Figure 10.40  Individual pump system

Table 10.6  Comparison of fuel injection systems

S. no. Job Air injection Solid injection system


system Individual pump Common rail Distributor
1. Metering Pump Pump Injection valve Pump
2. Timing Fuel cam Pump cam Fuel cam Fuel cam
3. Injection rate Spray valve Pump cam Spray valve Fuel cam
4. Atomisation Spray valve Spray tip Spray tip Spray tip
5. Distribution Spray valve Spray tip Spray tip Spray tip

10.14.2  Design of Fuel Nozzle


The fuel injection into the cylinder through the fuel nozzle is shown in Fig. 10.41.

Injector
1 1′

2 2′

Piston

Cylinder

Figure 10.41  Fuel injection through an injection nozzle

M10_THERMAL ENGINEERING_SE_XXXX_CH010(Part-II).indd 535 6/15/2018 4:54:46 PM


536 Chapter 10

Let,
p1 = pinj = injection pressure, kN/m2
p2 = pcyl = combustion chamber pressure, kN/m2
r f = density of fuel, kg/cm3
t = period of injection, s
Q = fuel sprayed, cm3/cycle/cylinder
df = diameter of injector
Let u f = specific volume of fuel (assumed incompressible)
c1 = velocity at section 1 – 1′
c2 = velocity at section 2 – 2′
c12 c2
Thus, + p1u f = 2 + p2u f
2 2
Neglecting c1, being very small compared with c2,
2( p1 − p2 )
c2 = 2u f ( p1 − p2 ) =
rf

2( p1 − p2 )
Actual fuel velocity, cf = Cdf c2 = Cdf rf (10.11)

where Cdf = co-efficient of discharge for the fuel injector.

Fuel consumed per cycle, mf = BSFC × Power (10.12)


Cycles/h
N
where cycles/h = × 60 for 4-stroke engine (10.13a)
2

= N × 60 for 2-stroke engine


     (10.13b)
Duration of injection in second,
q × 60
t = s (10.14)
N × 360
Mass of fuel supplied per second,

mf
m f = = Cdf A f 2r f × ∆p  (10.15)
t
Volume of fuel injected per second
 p   q 1   N 
Q f =  d 2 × n × c f ×  ×  ×    Cdf (10.16)
 4   360 N i   60  


where d = orifice diameter


n = number of orifices

M10_THERMAL ENGINEERING_SE_XXXX_CH010(Part-II).indd 536 6/15/2018 4:54:49 PM



Internal Combustion Engine Systems 537

q = crank angle during which the fuel is supplied


Ni = number of injections per minute
N
= for a 4-stroke engine  (10.17a)
2
= N for a 2-stroke engine (10.17b)
N = rpm
Fuel consumed per hour, mf = power in kW × SFC in kg/kWh.
Fuel consumed per cylinder per cycle (for 4-stroke engine),
mf
m fc = (10.18)
N
× Number of cylinders × 60
2 
Volume of fuel injected per cylinder per cycle,
m fc × 10 −3
Q = m3 (10.19)
Specific gravity of fuel

Injection period

Period of injection in degrees of crank angle × 60s


t=
360 × N

p 2 m fc
Nozzle hole area, df = (10.20)
4 c f × t × rf

Example 10.10
Calculate the diameter of the injector orifice of a six-cylinder, 4-stroke CI engine using the follow-
ing data:
Brake power = 250 kW; Engine speed = 1500 rpm; BSFC = 0.3 kg/kWh; Cylinder pressure = 35 bar;
Injection pressure = 200 bar; Specific gravity of fuel = 0.88; Co-efficient of discharge of the fuel
orifice = 0.92; Duration of injection = 36° of crank angle. [IES, 2007]

Solution
Given data: BP = 250 kW, N = 1500 rpm, BSFC = 0.3 kg/kWh, p1 = 200 bar, p2 = 35 bar, Specific
gravity of fuel = 0.88, Cdf = 0.92, Duration of injection = 36° of crank angle, Number of cylinders =
6, Number of strokes = 4
Fuel consumed per hour = BP × BSFC = 250 × 0.3 = 75 kg/h
Fuel consumed per cylinder per cycle
75 × 2 1
= = kg
1500 × 6 × 60 3600
Volume of fuel injected per cylinder per cycle
10 −3
= −6 3
3600 × 0.88 = 0.3156 × 10 m

M10_THERMAL ENGINEERING_SE_XXXX_CH010(Part-II).indd 537 6/15/2018 4:54:52 PM


538 Chapter 10

Density of fuel, rf = 1000 × 0.88 = 880 kg/m3

2( 200 − 35) × 105 = 193.65 m/s


c2 =
880

p 2 Mass delivered per cycle


Nozzle hole area, d =
4 f Cdf × c2 × Time × r f

1 × 250
= 3600 × 0.92 × 193.65 × 1 × 880

Diameter of cylinder orifice, df = 0.75 mm.

Example 10.11
A six-cylinder, four-stroke oil engine develops 200 kW at 1200 rpm and consumes 0.3 kg/kWh.
Determine the diameter of a single orifice injector if the injection pressure is 200 bar and com-
bustion chamber pressure is 40 bar. The injection is carried for 30° rotation of a crank. Take
rf = 900 kg/m3, and Cdf = 0.7. Each nozzle on a cylinder is provided with a single orifice.

Solution
BSFC × Power
Fuel consumed per cycle, mf = Cycles/h
0.3 × 200
= = 1.67 × 10−3 kg
1200
× 60
2
30 60
Duration of injection t = × = 4.17 × 10−3 s
1 1200 × 360
mf 1.67 × 10 −3
  m f = = = 0.4 kg/s
t 4.17 × 10 −3

  m f = Cdf A f r f × ∆p

or   0.4 = 0.7 × Af 900 × ( 200 − 40) × 105

or    Af = 0.0476 × 10−4 m2 or 4.76 mm2

4.76 Af
= 0.793 mm2
Area of each nozzle = =
6 6
p
   d 2f = 0.793
4
or       df  = 1.005 mm

M10_THERMAL ENGINEERING_SE_XXXX_CH010(Part-II).indd 538 6/15/2018 4:54:57 PM



Internal Combustion Engine Systems 539

Example 10.12
A 16-cylinder diesel engine has a power output of 800 kW at 900 revolutions per minute. The
engine works on the four stroke cycle and has a fuel consumption of 0.238 kg/kWh. The pressure in
the cylinder at the beginning of injection is 32.4 bar and the maximum cylinder pressure is 55 bar.
The injector is set at 214 bar and maximum pressure at the injector is around 600 bar. The specific
gravity of the fuel is 0.86. Calculate the orifice area required per injector, if the injection takes place
over 10 degree crank angle. [IES, 2001]

Solution
Given: i = 16, Pt = 800 kW, N = 900 rpm, n = 2, m f = 0.238 kg/kWh, pc = 32.4 bar, pmax = 55 bar,
pi = 214 bar, s = 0.86, q =10°, (pimax) = 600 bar

Pt 800
Power per cylinder, P = = = 50 kW
i 16
Fuel consumption per cylinder, mf = m f × P = 0.238 × 50 = 1.19 kg/h or 3.3056 × 10−3 kg/s

mf 3.3056 × 10 −3
Fuel to be injected per cycle, mfi = = = 4.0747 × 10−4 kg
 N 1  900   1 
 ×    
60 n 60   2 

q 60  10   60 
Fuel injection time, t = × =  = 1.852 × 10–3 s
360 N  360   900 

4.40747 × 10 −4
Fuel mass rate per second, m f = mf i /t = = 0.238 kg/s
1.852 × 10 −3
Pressure difference in the beginning, ∆pi = pi – pc = 214 – 32.4 = 181.6 bar

Pressure difference at the end,


∆pe = (pi)max – pmax = 600 – 55 = 545 bar

∆pi + ∆pe 181.6 + 545


Average pressure difference, ∆pa = = = 363.3 bar
e 2
Now m f = Cd Ai [2 r f ∆pa ]1/ 2

or 0.238 = 0.6 × Ai [2 × 0.86 × 103 × 363.3 × 105]1/2


Ai = 1.587 × 10–6 m2 or 1.587 mm2
or   

Example 10.13
An 8-cylinder, 4-stroke diesel engine has a power output of 368 kW at 800 rpm. The fuel con-
sumption is 0.238 kg/kW-hr. The pressure in the cylinder at the beginning of injection is 35 bar
and the maximum cylinder pressure is 60 bar. The injector is adjusted to operate at 210 bar and the

M10_THERMAL ENGINEERING_SE_XXXX_CH010(Part-II).indd 539 6/15/2018 4:55:01 PM


540 Chapter 10

maximum pressure in the injector is set at 600 bar. Calculate the orifice area required per injector if
the injection takes place over 12° crank angle. Assume the coefficient of discharge for the injector
= 0.6, specific gravity of fuel = 0.85 and the atmospheric pressure = 1.013 bar. Take the effective
pressure difference to be the average pressure difference over the injection period. [IAS, 2004]
Solution
Given: i = 8, n = 2, BP = 368 kW, N = 800 rpm, m f = 0.238 kg/kWh, p1 = 35 bar, pmax = 60 bar,
pi = 210 bar, (pi)max = 600 bar, q = 12°, Cdf = 0.6, Sf = 0.85, patm = 1.013 bar, ∆pe = (∆pavg)i
368
Fuel consumed by the engine per cylinder = 0.238 × = 10.948 kg/hr
8
10.948
Fuel consumed per cycle = = 4.56 × 10–4 kg
800
× 60
2
12
Duration of injection = = 2.5 × 10–3 s
800 × 360
60
4.56 × 10 −4
Rate of fuel consumption m f = = 0.1824 kg/s
2.5 × 10 −3
rf = 0.85 × 103 = 850 kg/m3
Density of fuel      
Now           m f = Cdf Af rf ( ∆p)

or          0.1824 = 0.6 × Af 850 × ( 210 − 35) × 105


or             Af = 2.49 × 10–6 m2 or 2.49 mm2
df = 1.78 mm
or              

10.15  ❐  FUEL IGNITION


The ignition of fuel is concerned only with starting the combustion and not with the behaviour of the
combustion flame. For starting the burning of fuel, it is necessary to raise the temperature of the air-
fuel mixture to its ignition temperature. The energy required for this purpose is supplied through an
electric spark. Within the range of mixtures normally used (12:1 to 15:1) in SI engines, a spark energy
of 10 kJ is sufficient to start the combustion process.

10.15.1  Requirement of Ignition System


The important requirements of a spark ignition system are as follows:
1. The voltage across the spark plug electrodes should be sufficiently large to produce an arc required
to initiate the combustion. The voltage necessary to overcome the resistance of the spark gap and
to release enough energy to initiate the self-propagating flame front in the combustible mixture is
about 10,000–20,000 volts.

M10_THERMAL ENGINEERING_SE_XXXX_CH010(Part-II).indd 540 6/15/2018 4:55:03 PM



Internal Combustion Engine Systems 541

2. The intensity of the spark should lie in a specified limit because extremely high intensity may
burn the electrodes and extremely low intensity may not ignite the mixture properly.
3. The volume of the mixture (clearance volume) at the end of compression should not too large,
otherwise the spark produced may not be sufficient to ignite the whole charge. There is definite
relation between the size of the spark and clearance volume.
4. There should be no missing cycle due to failure of spark.
5. In a multi-cylinder engine, there must be arrangement (distributor) to carry this voltage to the
right cylinder at the right time.

10.15.2  Ignition Systems


The basic ignition systems in use are as follows:
1. Battery ignition system—conventional, transistor-assisted
2. Magneto ignition system—low temperature, high temperature
3. Electronic ignition system
Battery and magneto ignition systems differ only in the source of electrical energy. A battery ignition
system uses a battery, whereas a magneto ignition system uses a magneto to supply low voltage, all
other system components being similar.

Battery Ignition System


The function of battery ignition system is to produce high voltage spark and to deliver it to the spark
plugs at regular intervals and at the correct time with respect to the crank position. The required com-
ponents of the system are as follows:
1. A battery of 6–12 volts
2. Ignition coil
3. Contact breaker
4. Condenser
5. Distributor
6. Spark plug
The arrangement of all the components of battery ignition system for 4-cylinder engine is shown in
Fig. 10.41.
The source of current is the storage battery and it is connected to the primary induction coil through
the starting switch as shown in Fig. 10.42. The other end of the primary coil is connected to the
breaker, which is connected to the ground, when the breaker contact points are closed. (In Fig. 10.42,
the breaker contact points are shown in the open position). As one terminal of the battery is grounded,
the circuit is closed by passing the current from the battery through the starting switch, primary coil,
contact breaker, ground and back to the battery when contact points are closed.
The induction coil consists of primary winding, usually 100–200 turns and a secondary winding,
usually 10,000 turns. Both windings are mounted on soft iron core.
The contact breaker consists of contact points, a camshaft on which a cam is mounted which is used
to break and make the contacts between the contact points.
The distributor consists of a distributor arm, as shown in Fig. 10.42. The arm is mounted on a cam
shaft and is rotated at half the speed of crankshaft. The function of the arm is to make the contact with
each spark plug as shown in Fig. 10.42.

M10_THERMAL ENGINEERING_SE_XXXX_CH010(Part-II).indd 541 6/15/2018 4:55:03 PM


542 Chapter 10

Ballast coil Ignition coil


Primary coil
Laminated core
Distributor arm rotates at
Secondary coil half speed of crank shaft

Starting
switch Distributor

Contact
Battery breaker
a
b

Contact
points

Spark plug
Ground Condensor
4-Cams used for
Cam shaft rotates breaking the contacts
at half speed of
crank-shaft

Figure 10.42  Battery ignition system for multi-cylinder engines

The distributor unit generally includes contact breaker to make the unit more compact, as both are
driven by the same cam shaft.
A condenser is included in the circuit as shown in Fig. 10.42.

Principle of Induction
An EMF is produced in the coil due to the relative movement of magnetic lines and coil because the
magnetic field lines are cut by the coil. The EMF produced depends on the relative movement between
the two; higher the movement, greater the EMF.
The principle of induction from one coil into another is shown in Figs 10.43(a) to (c). When current
is allowed to flow through the primary coil, a magnetic field is set up and this field passes through
the secondary coil and induces EMF, sending a current through a closed circuit. The current in the
primary coil quickly attains a steady value and a magnetic field is stabilised. The EMF is not induced
in the secondary coil as there is no relative movement between field (established by primary) and
secondary coil.
If the primary is switched off, the established magnetic field collapses and the EMF is induced
again in the secondary coil in the opposite direction. The greater EMF is induced when the circuit is
broken because the collapse is more rapid.
The EMF induced in the secondary coil can be further increased by collapsing the established field
more rapidly. The magnetic effect in the secondary coil (EMF-produced) is intensified by winding
the primary and secondary coils on a common soft iron bar or a ring. The lines of the magnetic field
are concentrated around the bar or ring and the magnetic effects are intensified. A higher EMF at the
secondary terminal can be obtained by suitable proportioning of primary and secondary coil around
a common iron coil (1:100).

M10_THERMAL ENGINEERING_SE_XXXX_CH010(Part-II).indd 542 6/15/2018 4:55:03 PM



Internal Combustion Engine Systems 543

Magnet

Magnetic
Coil
lines
Coil

Supply G
Galvanometer
(a) (b)

Magnetic field
during collapsing
of fields

G
Battery
Switch
(c)

Figure 10.43  Principle of induction

Principle of Ignition
High voltage can be introduced at the terminals of the secondary coil by collapsing the field estab-
lished by primary and proper proportioning of the turns of primary and secondary as mentioned ear-
lier. If this is connected to the two points providing an air gap between them as shown in Fig. 10.44,
a spark is produced. Ignition can take place in the compressed charge of petrol engine if the spark is
produced in the charge at the end of compression.
Working of Battery Ignition System
When the primary circuit is closed by the contact breaker (shown in open position in Fig. 10.42) a
current begins to flow through the primary coil and magnetise core of the coil. The EMF is induced
in the secondary as the current in the primary increases. The EMF induced in the secondary coil is
proportional to the rate at which the magnetic flux increases. The EMF produced in the secondary coil
due to the growth of current in the primary coil is not sufficient to produce a spark at the spark plug
because the primary circuit has to establish the magnetic flux.
When the primary circuit is opened by the contact breaker, the magnetic field collapses. Electro-
motive force is induced in the secondary which is directly proportional to the rate at which the mag-
netic field of the core collapses which in turn depends on the rate of decrease of the primary current.
A condenser is connected across the contact breaker in the primary circuit as shown in Fig. 10.42.

Air gas
From
Secondary

Figure 10.44  Principle of ignition

M10_THERMAL ENGINEERING_SE_XXXX_CH010(Part-II).indd 543 6/15/2018 4:55:04 PM


544 Chapter 10

This helps to collapse the field very rapidly by absorbing part of the energy of the magnetic field which
is thrown back into the primary winding and produces a very high voltage in the secondary. This EMF
in the secondary is sufficient to ignite the charge by producing the spark.
One end of the secondary coil is connected to the ground and the other end is connected to the
central terminal of the distributor. The distributor connects the secondary coil in turn to the different
spark plugs of the engine in their firing order. The spark plug of a particular cylinder is placed in cir-
cuit of the secondary coil with the help of the distributor when the time comes for the charge in that
cylinder to be ignited and at the same time the primary circuit is opened by contact breaker. A spark
is produced between the points of the spark plug.
The distributor and contact breaker are generally mounted on the same cam-shaft which rotates at
half speed of the crankshaft. The function of the distributor is to connect the secondary coil to each
cylinder of a multi-cylinder engine at the time of ignition. The contact breaker also works simultane-
ously with the distributor and its function is to disconnect the primary circuit exactly at the same time
when the spark in the particular cylinder is required. The distributor arm connects four spark plugs in
one rotation of the cam shaft and therefore, four contact points are required in four cylinder engines.
The contact breaker has to break the contacts four times in four cylinder engines and requires four
cams as shown in Fig. 10.42. If there are n cylinders, the contact points and cams required are also n
in number.
In a single cylinder engine, the distributor is not required as in scooter engine, and single cam is
sufficient for giving the spark. An ignition system used in single cylinder petrol engine is shown in
Fig. 10.45. Instead of the battery, the magneto is used in this system.
The cam is mounted on the crankshaft only as breaking of circuit during each rotation is required
in two stroke engine and there is no necessity of cam-shaft.

Induction coil

Switch

Battery
B

Contact
breaker
Spark plug
Condensor

Figure 10.45  Battery ignition system for single cylinder engine

M10_THERMAL ENGINEERING_SE_XXXX_CH010(Part-II).indd 544 6/15/2018 4:55:05 PM



Internal Combustion Engine Systems 545

Number of Sparks
The number of sparks produced must be equal to the number of working strokes in a single cylinder
engine. If there are Nc, cylinders, the number of sparks produced for that engine are as follows:
Ns, (Number of spark) = n × Nc
where n = number of working strokes and Nc = number of cylinders
N
Further, n = for 4-stroke cycle engine
2
= N for 2-stroke cycle engine where n is the rpm of the engine
NN c
Thus, for 4-stroke engine, Ns = and for 2-stroke engine, Ns = NNc.
2
Advantages of Battery Ignition System
1. Its initial cost is low compared with magneto. This is the main reason for the adoption of coil
ignition on cars and commercial vehicles.
2. It provides better spark at low speeds of the engine during starting and idling. This is because the
maximum current is available throughout the engine speed range, including starting.
3. The maintenance cost is negligible, except for the battery.
4. The spark efficiency (intensity) remains unaffected by advance and retard positions of the timing
control mechanism.
5. The simplicity of the distributor drive is another factor in favour of coil ignition.
Disadvantages of Battery Ignition System
1. The engine cannot be started if the battery runs down.
2. The weight of the battery ignition system is greater than the magneto which is a major consider-
ation in adopting the system in aero-engines.
3. The wiring involved in the coil ignition is more complicated than the one used in a magneto
ignition. This results in more likelihood of defects occurring in the system.
4. The sparking voltage drops with increasing speed of the engine.
Components of a Battery Ignition System
1. Battery and cut-out: A battery is an electro-chemical device which supplies current because of
chemical reactions that occur in it. The common type of battery used in automobiles is a lead acid
storage battery. A 12 V battery has six cells each generating two volts. Each cell consists of group
of positive and negative plates. The positive plate has a grid of lead and antimony alloy filled with
lead peroxide. The negative plate has a grid filled with spongy lead. The positive and negative
plates are immersed in an electrolyte of dilute sulphuric acid. The plates are separated from each
other by PVC or rubber separators. The positive and negative plate grids are connected to lead
antimony strips. These strips separately connect the positive and negative sparks in series and the
positive and negative terminals. The plates are placed in a battery container made of hard rubber
which is acid-proof. The plate tops are supported by insulating and acid-resisting cell covers.
Holes are provided in each cell to fill and unfill electrolytes or distilled water. The filter holes are
covered with plugs containing small holds that allow gases to escape.

M10_THERMAL ENGINEERING_SE_XXXX_CH010(Part-II).indd 545 6/15/2018 4:55:06 PM


546 Chapter 10

  The battery is charged continuously by a dynamo directly mounted on the crankshaft. To avoid
over charging or discharging of the battery, an electric switch is introduced between the battery
and dynamo, which is known as a cut-out.
2. Ignition coil: The purpose of the ignition coil is to step up 6 V or 12 V battery to 5000 V, which
is sufficient to generate the spark. This coil consists of two insulated conducting coils having
primary and secondary windings. The primary winding is connected to the battery through an
ignition switch and contact breaker and the secondary winding is connected to the spark plugs
through the distributor.
3. Contact breaker: As the number of sparks produced increases with an increase in engine speed,
the time available for building up the magnetic field of the spark coil becomes shorter. As a result,
the maximum value attained by the primary coil is lessened with increasing speed and the mag-
netic field produced becomes weaker until there is not enough voltage induced in the secondary
winding to produce the spark. Contact breaker is a device to increase the time during which the
primary circuit remains closed.
4. Condenser: For obtaining the highest voltage in the secondary circuit, a quick collapse of the
magnetic field is essential. In addition, it is also necessary to prevent the arcing and consequently
burning of contact points.
  This is achieved by providing a condenser. The condenser is a device which will absorb and
hold an electric charge when an electromotive force is applied to its terminal. In its simplest form,
it consists of two sheets of conducting material separated by a layer of insulating material.
  The condenser’s capacity is given by C = A × x × p, where A is the area of conducting material,
x is the distance between the sheets of conducting material and p is the specific inductive capacity
of the insulating material. The parts of the condenser are shown in Fig. 10.46.
  In order to get large capacity with limited space, instead of two sheets of conducting material, two
sets of such sheets are used and are separated by an insulating material. The conducting material
used is ‘tinfoil’, whereas the insulating material is mica in condenser used with magneto and wax-
impregnated paper for battery system. All conducting sheets of a set are electrically connected.
The arrangement for the condenser is shown in Fig. 10.47.
  The operation of the condenser is described as follows. The condenser is connected across the
contact breaker. When the contacts points open, instead of passing across the points in the form

Conductors
Paper
Casing

Roll
Rubberized Roll end with sprayed solder
fextolits
washer
Plastic washer

Figure 10.46 Condenser

M10_THERMAL ENGINEERING_SE_XXXX_CH010(Part-II).indd 546 6/15/2018 4:55:06 PM



Internal Combustion Engine Systems 547

Primary coil
Primary coil
Contact
Condenser
breaker

Contact A Ammeter Cam


breaker
Condenser

Cam Battery

Switch
Earth Earth

Figure 10.47  Condenser arrangement

of an arc, the current flows into the condenser, is stored by it, and becomes charged. The charge
in the condenser discharges back immediately in the primary circuit in the direction reverse to the
flow of a battery current, thus assisting in a quicker collapse of magnetic field when the contact
points open.
5. Distributor: The main function of the distributor is to distribute the high voltage surge to dif-
ferent plugs of multi-cylinder engine at the right time. The high tension current first goes to the
control electrode of the distributor and then to the rotor. As the rotor rotates, it passes this high
tension current to metal electrodes which are connected to the spark plugs through high tension
wiring according to the firing order of the engine.
  These are two types of distributors—brush type and gap type. The blow-up of distributor parts
is shown in Fig. 10.48. In the brush type, a carbon brush carried by a rotor arm slides over a metal-
lic segments embedded in the distributor cap of moulded insulating material, thus establishing
electrical connection between the secondary winding of the coil and the spark plug.
  In the gap type, the electrode of the rotor arm passes close to, but does not actually contact the
segments in the distributor cap. There is no appreciable wear and misfiring due to fouled spark
plug.
  When the surface of the spark plug insulator inside the combustion chamber becomes fouled
with conducting carbon and oxides, there is considerable leakage of current through this conduct-
ing layer which prevents the voltage in the secondary from building up.
  In actual practice, the distributor, contact breaker, rotating shaft with breaker cam, condenser,
and the ignition advance mechanism are housed together.
6. Spark plug: The function of the spark plug is to generate the spark in the combustion using a
high voltage communicated by the secondary. The spark plug provides two electrodes with a
proper gap across which high potential is discharged and spark is generated.
  A sectional view of a conventional spark plug is shown is Fig. 10.49. It consists of a steel shell,
an insulator, and two electrodes. The high voltage supply from secondary is given to the central
electrode which is insulated with porcelain. The other electrode is welded to the steel shell of
the plug and thereby automatically grounded when the plug is fitted in the cylinder head of the
engine. The electrodes are made of high nickel alloy to withstand severe corrosion and erosion to
which they are subjected.

M10_THERMAL ENGINEERING_SE_XXXX_CH010(Part-II).indd 547 6/15/2018 4:55:07 PM


548 Chapter 10

Rotor arm
Capacitor
L.T. Terminal

Contacts
Fixed contact plate
securing screw
Cam
Contact breaker
base plate
Distance
collar
Centrifugal timing
control weights

Thrust washer

Bearing bush

Dog 8 pin

Figure 10.48  Distributor, blow up

Terminal nut

Leakage-current Terminal stud


barriers
Insulator

Special conductive
seal

Captive gasket

Internal seal
Center electrode

Insulator nose
Ground electrode

Figure 10.49  Schematic of a typical spark plug

M10_THERMAL ENGINEERING_SE_XXXX_CH010(Part-II).indd 548 6/15/2018 4:55:08 PM



Internal Combustion Engine Systems 549

  The tips of central electrode and insulation are exposed to the burned gases. This results in
high thermal stresses and the insulator may crack. As the tips are subject to high temperature
(2000°C – 25.000°C), heat must flow from the insulator and tip to the surrounding shell in order
to cool the electrodes and prevent pre-ignition.
  The spark plugs are classified as hot plug and cold plug, depending on the temperature at the tip
of the electrodes. The operating temperature of the tip depends on the amount of heat transferred,
which, in turn, depends on the path followed by the heat to flow. A cold plug has a short heat flow
path, whereas a hot plug follows a long flow path for the heat to flow as shown in Fig. 10.50.
  The hot plug is used to avoid cold fouling where combustion chamber temperatures are rela-
tively low such as during low power operation and continuous idling.
  The temperature may be less at idling speed and then tips will be fouled by unburned carbon
deposits or excess lubricating oil. The carbon deposits burn at 350°C, whereas lubricating depos-
its burn at 550°C. If the spark plug runs hot at idling speed to prevent carbon deposits, it may run
too hot at a high speed. This may cause undesirable pre-ignition. If the plug runs above 800°C,
pre-ignition generally occurs.
  The insulator tip length is the most important parameter which controls the operating temper-
ature. Therefore, the tip temperature is generally controlled by varying the insulator tip position
and the electrode material.
  It is necessary in practice to compromise to obtain a proper spark plug which would operate
satisfactorily throughout the engine operating range. An improper spark plug has remained a
major source of engine trouble as misfiring and pre-ignition.
  The factors affecting the operation of spark plug are (i) compression ratio, (ii) electrode tem-
perature, (iii) speed of the engine, (iv) mixture strength, and (v) run of the engine.

Magneto Ignition System


Some of the drawbacks of the battery ignition system are as follows:
1. There are chances of discharging the battery.
2. There are chances of misfiring at higher speed of the engine.
3. It requires complicated wiring.
4. There are chances of failure.
5. There are many mechanical complications in the operation of the system.
These difficulties experienced with battery ignition system can be avoided by using magneto ignition
system.

Insulator

Shell
Cylinder
head
Water

Figure 10.50  Heat transfer path of hot and cold spark plug

M10_THERMAL ENGINEERING_SE_XXXX_CH010(Part-II).indd 549 6/15/2018 4:55:09 PM


550 Chapter 10

Magneto Ignition System—Working


The arrangement of the magneto ignition system is shown in Fig. 10.51. The only difference between
the battery and the magneto system is that the battery is replaced by the rotating magnet. As the mag-
neto rotates, the voltage and current are generated in the primary coil and the circuit is completed by
passing the current through the contact breaking point and the ground. As the current passes through
the primary coil through the contact breaker, the circuit is completed through the ground. As the cam-
shaft rotates, the cam 1 opens the contact breaker and interrupts the flow of current in the primary.
This causes the decay in the magnetic field lines and cuts the lines of magnetic field in the secondary
coil, and a high voltage is generated in the secondary coil. The process of generating the high voltage
in the secondary coil is known as the induction phenomenon. The voltage generation in the secondary
depends on the ration of number of turns in the secondary and primary coils.
N 
e ′∞  s 
 NP 
Further, consider the effect of a firing sequence on engine cooling. When the first cylinder is fired, its
temperature increases. When the next cylinder fired is number 2, the portion of the engine between
the cylinder number 1 and 2 gets overheated. If the third cylinder fired is cylinder number 3, this
over-heating is shifted to the portion between the cylinders 2 and 3. Thus, the task of the cooling
system becomes very difficult because it is required to cool more at a place than at other places and
this place too changes its position with time. If we fire the third cylinder after the first, the overheating
problem can be mitigated.
Next, consider the flow of exhaust gases in the exhaust pipe. After firing the first cylinder, exhaust
gases flow out to the exhaust pipe. If the next cylinder fired is the cylinder number 2, we find that
before the gases exhausted from the first cylinder go out to the exhaust pipe, the gases exhausted from
the second cylinder overtake them. This would require that the exhaust pipe be made bigger. Other-
wise, the back pressure in it would increase and the danger of back flow will arise. If, instead of firing

Distributor
Secondary 1
2
4
Primary 3 2 1 4
3
Rotating
magnet

Contact Spark plug


points

1 4
2 3
Cam
Condenser

Cam shaft

Figure 10.51  Magneto ignition system

M10_THERMAL ENGINEERING_SE_XXXX_CH010(Part-II).indd 550 6/15/2018 4:55:10 PM



Internal Combustion Engine Systems 551

cylinder number 2, cylinder number 4 is fired, by the time the gases exhausted by cylinder 4 come into
the exhaust pipe, the gases from cylinder 1 will have sufficient time to travel the distance between 1
cylinder and 4 and thus, the development of a high back pressure would be avoided.
It should be noted that to some extent, all the three requirements are conflicting and therefore, a
trade-off is necessary. For four cylinder engines, the possible firing orders are 1-2-4-3 and 1-3-4-2. The
latter is in common use. For a six-cylinder engine, the firing orders which can be used are 1-5-3-6-2-4,
1-5-4-6-2-3, 1-2-4-6-5-3, and 1-2-3-6-5-4. The first one is in common use.
Other Firing Orders
For three-cylinder engine 1-3-2
Eight-cylinder in-line engine 1-6-2-5-8-3-7-4
Eight-cylinder V-shape engine 1-5-4-8-6-3-7-2, 1-8-4-3-6-5-7-2, 1-6-2-5-8-3-7-4, 1-8-7-3-6-5-4-2, 1-5-
4-2-6-3-7-8. Cylinder no. 1 is taken from front of the in-line engines, whereas in V-shape, front cylinder
on right side-bank is considered cylinder no. 1 for fixing H.T. leads according to engine firing order.

Ignition Timing
In order to obtain maximum power from an engine, the compressed mixture must deliver its maximum
pressure at a time when the piston is about to commence its downward stroke. Since there is a time lag
between the occurrence of spark and the burning of the mixture, the spark must take place before the
piston reaches TDC on its compression stroke. Usually, it should occur at about 15° bTDC. If the spark
occurs too early, combustion will take place before combustion stroke is completed and the pressure
so developed will oppose the piston motion and thereby, reduce the engine power. If the spark occurs
too late, the piston will have already completed a certain part of the expansion stroke before the pres-
sure rise occurs and a corresponding amount of engine power will be lost.
The correct instant for the introduction of spark is mainly determined by the ignition lag. The
ignition lag depends on many factors such as compression ratio, mixture strength, throttle opening
engine temperature, combustion chamber design, and speed. Some of the important factors affecting
the ignition timings are as follows:
1. Engine speed: When an engine is rotating at 2000 rpm, even one-thousandth of a second repre-
sents 12° of crank rotation. Therefore, the spark must occur, say, 20° before the TDC for maxi-
mum power and economy. As the speed of the engine increases, the combustion time increases
in terms of crank angle degrees and the spark must advance accordingly, that is, the angle of the
advance must increase as the speed increases (see Fig. 10.52).
2. Mixture strength: In general, rich mixtures burn faster. Therefore, as the mixture is made richer,
the optimum spark advance must be retarded, that is, the number of degrees of the crank angle
before TDC is decreased and the spark occurs closer to the TDC.
3. Part load operation: Part-load operation of a spark ignition engine is affected by throttling. Due to
throttling, a smaller amount of charge enters the cylinder and the dilution that occurs due to resid-
ual gases is greater. In addition, higher air-fuel ratio required for the part-load operation causes the
combustion time to increase. Therefore, at part load, the spark advance must be increased.
4. Type of fuel: Ignition delay will depend on the type of fuel used in the engine. For maximum
power and economy, a slow-burning fuel will need a higher spark advance than a fast-burning fuel.
  It is obvious from the above that the point in the cycle where the spark occurs must be regulated
to ensure maximum power and economy at different speeds and loads and must be automatic.
Most engines are fitted with a mechanism which is integral with the distributor and automatically
regulate the optimum spark advance to account for change of speed and load.

M10_THERMAL ENGINEERING_SE_XXXX_CH010(Part-II).indd 551 6/15/2018 4:55:10 PM


552 Chapter 10

After
Ignition timing

Crank angle

Crank angle
required to burn first 10%
TDC

Before
Crank angle required
to burn from10% to 90%

Combustion complete
Engine speed
Idle

Figure 10.52 Combustion time in terms of engine speed and


degrees of crankshaft rotation

10.16  ❐  COMBUSTION IN IC ENGINES


Combustion is defined as a rapid chemical reaction between H2 and C with oxygen in the air, and in
the process, the reaction liberates energy in the form of heat.
It is absolutely essential to burn the fuel supplied completely for the economical working and
safety of the engine. Therefore, the mixture supplied to the engine should possess the required A:F
ratio; otherwise, combustion cannot be initiated or if initiated, it cannot be sustained. In addition, there
must be some means to initiate the combustion and the generated flame should be able to propagate
through the mixture and burn the mixture completely.
It is a known fact that the fuel with any A:F ratio cannot be burned. There is an ignition limit for
any fuel to start the combustion and sustain it till the complete fuel burns by the flame generated with
spark plug. In addition, the temperature of the mixture to initiate the ignition is equally important. It
is also known that the flame will propagate if the temperature of the burnt gases exceeds 1500 K for
SI engine (gasoline) fuels. The ignition limits of the hydrocarbon fuel when temperature of mixture
reaches to 1500 K are shown in Fig. 10.53.
The upper and lower limits of A:F ratio for ignition depend on the temperature of a particular fuel.
The limit becomes wider at higher temperatures because of higher reaction rate and higher thermal
diffusivity coefficients of the mixture. Therefore, it is essential to see that the A:F ratio of the mixture
supplied to the engine lies in the practical limit as shown in Fig. 10.53.

Practical limit for SI engine

Too
Too-lean
rich
0 7 9 15 20 30 40
Ignition limit

Figure 10.53 Ignition limit of hydro-carbon fuels at 1500 K

M10_THERMAL ENGINEERING_SE_XXXX_CH010(Part-II).indd 552 6/15/2018 4:55:11 PM



Internal Combustion Engine Systems 553

10.16.1  Stages of Combustion in SI Engines


It is assumed that the heat is added instantaneously at constant volume in the ideal air-cycle of SI
engines. To achieve this, the burning of fuel in the SI engine should be instantaneous. In actual
engines, combustion occurs over a finite period of time as the flame starting around the spark plug has
to propagate through the entire mixture of air and fuel.
A high intensity spark is produced for a few degrees of crank angle before TDC at the end of
the compression stroke. The function of the spark is to provide a source of heat to the combustible
mixture. The temperature of the combustible mixture surrounding the spark is raised to a sufficient
temperature level to start the combustion. The flame propagation is continued through the combustible
mixture, provided the release of heat from the initial sources is sufficient to heat the adjoining portion
of the mixture to a temperature at which the heat from the reaction is sufficient to overcome the heat
losses. The heat liberated by the burning portion ‘flame front’ prepares the adjacent portion of the
unburned charge for the combustion reaction.
It has been observed experimentally that there is a certain time interval between the instant of spark
and the instant when there is a noticeable rise in pressure due to combustion. This means that the com-
bustion starts some time after the spark. This time lag is known as ‘ignition lag’. Ignition lag is a time
interval in the process of chemical reaction during which the molecules of the fuel get heated up to the
self-ignition temperature, get ignited, and produce a self-propagating nucleus of flame.
The pressure variation in the SI engine combustion chamber during the crank rotation is shown in
Fig. 10.54. This is an unfold p-v diagram.
The ignition is timed to take place at the point a but the burning commences only at the point ‘b’.
The time interval between the points ‘a’ and ‘b’ is known as ‘ignition lag’. This is generally expressed
in terms of crank angle and is given by q1 as shown in Fig. 10.54. The time required for the crank to
rotate through an angle q2 is known as combustion period during which the propagation of flame takes
place. Many times, the combustion is not complete at the point c (when the mixture is too rich) and
the fuel may burn after the point c during the expansion stroke. This burning of fuel after the point of
maximum pressure is known as ‘after burning’. This is not desirable.
The major disadvantages of the ignition lag is to reduce the power developed. If the ignition lag is
long, the peak pressure occurs during the expansion stroke, and complete advantage of expansion is
not achieved. The time lag may vary from 10°–50°, according to the type of the fuel used and several

q2
q1
Point of c
combustion
Pressure

Point of b
ignition a d
Angle of
advance

TDC
Crank angle

Figure 10.54 Crank angle v’s pressure


variation in SI engine

M10_THERMAL ENGINEERING_SE_XXXX_CH010(Part-II).indd 553 6/15/2018 4:55:12 PM


554 Chapter 10

other factors affecting the time lag. To maintain the ‘time lag’ constant at variable speed of the engine,
it is necessary to change the angle of advance. The angle of advance must increase with increase in
speed. This is commonly done in automobile engines by an automatic advancing mechanism.
All considerations should be taken into account in the design of the combustion chamber and
selecting the fuel used to reduce the ignition lag.
The theoretical diagram of combustion is shown in Fig. 10.55(a).
However, the actual process differs from theory as instant combustion is not possible as shown by
bc in Fig. 10.55(a). Some time (order of millisecond) is needed to start the combustion after giving the
spark because of the fact that the surrounding mixture is to be heated up to ignition temperature and
then formed nucleus of flame which starts propagating through the surrounding mixture.
The pressure variation in the engine with respect to crank-angle is shown in Fig. 10.55(b). There
are mainly three stages of combustion in SI engines as shown in Fig. 10.55(b).
1. First phase: This phase is considered between the point of ignition and point of combustion.
As shown in Fig. 10.55(b), ignition is timed at point a and combustion starts at point b. The
time interval between the points a and b is known as ignition lag. The ignition lag is generally
expressed in terms of crank angle (q1). This period of ignition lag is very small and lies between
0.00015 and 0.002 seconds. An ignition lag of 0.002 seconds corresponds to 35° crank rotation
when the engine is running at 3000 rpm and crank angle required (which is known as angle of
advance) increases with an increase in engine speed.

c Expa
nsion
b d
n
essio
a
C ompr
0° 180° 360°
Crank angle

(a)

q1= Ignition lag I II q 2 = Combustion period


q2
q1
Point of c
combustion
Pressure

Compression
Point of b
line
ignition
d
a

e
(Non-
firin g)
80 60 40 20 20 40 60 80
TDC
Crank angle, degree

(b)

Figure 10.55 Pressure vs crank angle diagram:


(a) Theoretical, (b) Actual

M10_THERMAL ENGINEERING_SE_XXXX_CH010(Part-II).indd 554 6/15/2018 4:55:12 PM



Internal Combustion Engine Systems 555

2. Second phase: Once the flame is formed at point b, it should be self-sustained and be able to
propagate through the mixture. This is possible when the rate of heat generation (Qg) by burning
the surrounding mixture of the flame nucleus is higher than the heat lost (Ql) by the flame to the
surrounding. As the difference between (Qg – Ql) is higher, the rate of flame propagation is higher
and complete combustion will occur at the earliest, which is the most desirable requirement of
combustion in SI engines. The propagation of flame also depends on the flame temperature and
the temperature and density of the surrounding mixture as its propagation is directly proportional
to these factors. A weak spark and a low compression ratio (as density of mixture is less) give low
propagation of the flame.
  After point b, the flame propagation is abnormally low in the beginning as heat lost is more
than heat generated. Therefore, the pressure rise is also slow as the mass of mixture burned is
small. Therefore, it is necessary to provide an angle of advance of 30° to 35° if the peak pressure
is to be attained at 5°–10° after TDC.
  After the point c, the pressure starts falling due to the fall in the rate of heat release when the
flame reaches the wall in the last part of combustion and cannot compensate for its fall due to the
gas expansion, and heat is transferred to the walls.
  The time required for the flame to travel 95% of the chamber length with respect to speed of
the engine is shown in Fig. 10.56, when q1 = 30° and A:F = 13.
  It is obvious that the crank angle required for 95% travel increases with an increasing engine
speed (the time available is decreased), therefore, if the combustion is to be completed at point
c, the angle of advance (q ) must be increased with increasing engine speed. The flame speed
increases with increasing engine speed because of the increase in turbulence of the mixture.
  The time required for the crank to rotate through an angle q2 is known as combustion period
during which the propagation of the flame takes place.
  Stages I and II are not entirely distinct. The starting point of stage II is measurable as the rise
in pressure can be seen on the p-q diagram. This is the point where the line of combustion departs
from the line of compression.

0.020
flame to travel 95% distance

100°
Time in seconds for the
Crank angle for 95%

80° 40 0.016
flame travel

(q )
Speed in (m/s)

60° 30 0.012

ed Time
40° 20 s pe 0.008
me
Fla
20° 10 0.004

300 600 900 1200 1500 1800


Engine speed, rpm

Figure 10.56 Time required for the flame to travel 95% of


the chamber length

M10_THERMAL ENGINEERING_SE_XXXX_CH010(Part-II).indd 555 6/15/2018 4:55:13 PM


556 Chapter 10

3. Third phase: Although the point c represents the end of the flame travel, it does not assure the
complete combustion of fuel. In this case, the combustion still continues after attaining the peak
pressure and is known as afterburning. This is continued throughout the expansion stroke. This
generally happens when the rich mixture is supplied to the engine.

10.16.2  Ignition Lag (or Delay) in SI Engines


The ignition delay period is the phase during which some fuel has already been admitted but has not
yet ignited. The fuel does not ignite immediately upon injection into the combustion chamber. There
is a definite period of inactivity between the time when the first droplet of fuel hits the hot air in the
combustion chamber and the time it starts through the actual burning phase. This period is known as
the ignition delay period. In Fig. 10.57, the delay period is shown on the pressure crank angle (or time)
diagram between points a and b. Point a represents the time of injection and point b represents the
time at which the pressure curve first separates from the motoring curve.
The important factors affecting delay period are as follows:
1. Compression ratio: With increase in compression ratio, the minimum auto ignition temperature
of a fuel decreases due to increased density of the compressed air. This results in closer contact
between the molecules of fuel and oxygen, reducing the time of reaction. The increase in the com-
pression temperature as well as the decrease in the minimum auto ignition temperature decreases
the delay period. The peak pressure during the combustion process is only marginally affected by
the compression ratio (because delay period is shorter with higher compression ratio and hence
the pressure rise is lower).
  One of the practical disadvantages of using a very high compression ratio is that the mechani-
cal efficiency tends to decrease due to increase in weight of the reciprocating parts. Therefore, in
practice, engine designers always try to use a lower compression ratio which helps in easy cold
starting and light load running at high speeds.
2. Engine speed: The delay period decreases with an increase in engine speed in a variable speed
operation with a given fuel. With increase in engine speed, the loss of heat during compression
decreases, resulting in the rise of both the temperature and pressure of the compressed air thus
reducing the delay period.
3. Power output: With an increase in engine output, the air–fuel ratio decreases, operating temper-
atures increase, and hence, delay period decreases.

TDC
b
a c
a – Start of injection Maximum
b – Start of combustion pressure
Preesure

Co
a c b c – Ignition delay M m
d – Mixing period ot pr bu
or es st
e – Interaction period in su io
g n
e Area A cu re
d rv
e
Expanded view of area A Time

Figure 10.57  Pressure-time diagram illustrating ignition delay

M10_THERMAL ENGINEERING_SE_XXXX_CH010(Part-II).indd 556 6/15/2018 4:55:13 PM



Internal Combustion Engine Systems 557

4. Quality of fuel: A lower self-ignition temperature results in a lower delay period. In addition,
fuels with higher cetane number give lower delay period, and smoother engine operation. Other
properties such as volatility, latent heat, viscosity, and surface tension also affect the delay period.
Table 10.7 shows the summary of the effects of variables on delay period.

Table 10.7  Effect of variables on delay period

S. no. Increase in variable Effect on delay period Reason


 1. Cetane number of fuel Reduces Reduces the self-ignition
temperature.
 2. Injection pressure Reduces Reduces physical delay due to
greater surface–volume ratio.
 3. Injection timing advance Reduces Reduced pressures and tem-
peratures when the injection
begins.
 4. Compression ratio Reduces Increases air temperature and
pressure and reduces auto-
ignition temperature.
 5. Intake temperature Reduces Increases air temperature.
 6. Jacket water temperature Reduces Increases wall and hence air
temperature.
 7. Fuel temperature Reduces Increases chemical reaction due
to better vapourisation.
 8. Intake pressure (supercharging) Reduces Increases density and
also reduces auto-ignition
temperature.
 9. Speed Increase in terms of crank Reduces loss of heat.
angle. Reduces in terms of
milliseconds
10. Load (fuel-air ratio) Decreases Increases the operating
temperature.
11. Engine size Decreases in terms of crank Larger engines operate
angle. Little effect in terms of normally at low speeds.
milliseconds.
12. Type of combustion chamber Lower for engines with Due to compactness of the
pre-combustion chamber. chamber.

10.16.3  Factors Affecting the Flame Propagation


After the ignition, the rate of flame propagation affects the combustion process in SI engines.
Better the propagation, higher is the combustion efficiency and higher the economy. The propa-
gation depends on the flame velocity. Unfortunately, flame velocities for most fuels range between
10–30 m/s. Therefore, all steps should be taken in the design and operation of the engine so that the
flame velocity is as maximum as possible.

M10_THERMAL ENGINEERING_SE_XXXX_CH010(Part-II).indd 557 6/15/2018 4:55:13 PM


558 Chapter 10

The study of flame propagation rate and the factors affecting the same is important for the following
two reasons:
1. Its effect on the rate of pressure rise in the cylinder.
2. Its effect in connection with abnormal combustion (knocking).
The factors which affect the flame propagation are discussed below:
1. A:F ratio: The mixture strength influences the rate of combustion and the amount of heat gen-
erated. The maximum flame speed for all hydrocarbon fuels occurs at nearly 10% rich mixture.
Flame speed is reduced for lean and rich mixture.
  Lean mixture releases less heat, resulting in lower flame temperature and lower flame speed.
Very rich mixture results in incomplete combustion (C → CO instead of CO2) and in production
of less heat; the flame speed remains low.
  The effects of the A:F ratio on the p-v diagram and p-q diagram are shown in Fig. 10.58.
2. Compression ratio (r): A higher compression ratio increases the pressure and temperature of
the mixture and also decreases the concentration of residual gases. All these factors which are in
favour reduces the ignition lag and helps to speed up the second phase of combustion. The maxi-
mum pressure of the cycle as well as the mean effective pressure of the cycle will increase with an
increase in r. Figure 10.59 shows the effect of the compression ratio on pressure (indirectly on the
speed of combustion) with respect to crank angle for same A:F ratio and same angle of advance.

B Rich
B Rich
p C Stoichiometric C Stoichiometric
D Weak
Ignition D Weak
Compression
with no ignition
TDC

v Crank angle

Figure 10.58  Indicator diagrams for stoichiometric and weak mixtures

TDC
50 r=
6
40 5
Pressure (bar)

4
30

20

10
Angle of advance
0
150 100 50 35 0 50 100 150
Crank angle, deg

Figure 10.59  Effect of compression ratio on pressure rise

M10_THERMAL ENGINEERING_SE_XXXX_CH010(Part-II).indd 558 6/15/2018 4:55:15 PM



Internal Combustion Engine Systems 559

Higher compression ration increases the surface to volume ratio and thereby increases the part of
the mixture which afterburns in the third phase.
3. Load on engine: With increase in load, the cycle pressures increase and the flame speed also
increases.
  In an SI engine, the power developed by an engine is controlled by throttling. At lower load and
higher throttle, the initial and final pressure of the mixture after compression decrease and the
mixture is also diluted by more residual gases. This reduces the flame propagation and prolongs
the ignition lag. Therefore, the advance mechanism is also provided with a change in load on the
engine. This difficulty can be partly overcome by providing rich mixture at part loads but this defi-
nitely increases the chances of after-burning. The after-burning is prolonged with richer mixture.
  In fact, poor combustion at part loads and necessity of providing richer mixture are the main
disadvantages of SI engines which cause wastage of fuel and discharge of a large amount of CO
with exhaust gases.
4. Turbulence and engine speed: Turbulence plays very important role in the combustion of fuel
as the flame speed is directly proportional to the turbulence of the mixture. This is because turbu-
lence increases the mixing and heat transfer coefficient or heat transfer rate between the burned
and the unburned mixture. The turbulence of the mixture can be increased at the end of com-
pression by a suitable design of the combustion chamber (geometry of cylinder head and piston
crown).
  Insufficient turbulence provides low flame velocity and incomplete combustion and reduces the
power output. However, excessive turbulence is also not desirable as it increases the combustion
rapidly and leads to detonation. Excessive turbulence causes to cool the flame generated and
flame propagation is reduced.
  Moderate turbulence is always desirable as it accelerates the chemical reaction, reduces igni-
tion lag, increases flame propagation, and even allows the weak mixture to burn efficiently.
  The turbulence of the mixture increases with an increase in engine speed. Therefore, the effects
of an increase in engine speed are similar to increase in turbulence.
5. Other factors: Among other factors, the factors which increase flame speed include supercharg-
ing of the engine, spark timing, and residual gases left in the engine at the end of exhaust stroke.
  The air humidity also affects the flame velocity but its exact effect is not known. Anyhow, its
effect is not large compared with A:F ratio and turbulence.

Example 10.14
The spark plug is fixed at 18° before top dead centre (TDC) in an SI engine running at 1800 rpm.
It takes 8° of rotation to start combustion and get into flame propagation mode. Flame termination
occurs at 12°C after TDC. Flame front can be approximated as a sphere moving out from the spark
plug which is offset 8 mm from the centre line of the cylinder whose bore diameter is 8.4 cm. Cal-
culate the effective flame front speed during flame propagation. The engine speed is increased to
34000 rpm and subsequently as a result of which the effective flame front speed increases at a rate
such that it is directly proportional to 0.85 times of engine speed. Flame development after spark
plug firing still takes 8° of engine rotation. Calculate how much engine rotation must be advanced
such that the flame termination again occurs at 12° after TDC. [IES, 2010]

M10_THERMAL ENGINEERING_SE_XXXX_CH010(Part-II).indd 559 6/15/2018 4:55:15 PM


560 Chapter 10

Solution
Given that q1 = 8°, q2 = 10° + 12° = 22°, N1 = 1800 rpm, e = 8 mm, D = 8.4 cm, N2 = 34000 rpm,
The combustion process in p-v diagram is shown in Fig. 10.60.

2p N1 2p × 1800
w1 = = = 60 p rad/s
60 60
p
q2 = × 22 = 0.122 × p rad
180
0.122 p
Time taken, t = = 0.002037 s
60 p
0.008
Flame front speed, v1 = = 3.92 m/s
0.002037
When engine speed is increased to 34000 rpm, then
2pN 2 2p × 34000
w2 = = = 1133.3 × p rad/s
60 60
34000
v2 = 3.92 × 0.85 × = 62.94 m/s
1800
0.008 0.008
 q 2 = = = 1.27 × 10–4 rad or 2.6°
v2 62.94
p
q1= Ignition lag
q1 = q2 =
8° 22° Flame
termination
c
Point of
b
combustion
Point of
ignition a
Angle of
advance

18° TDC 12°

Cylinder

SP 8.4 cm
8 mm

Figure 10.60  Combustion process in p-v diagram

M10_THERMAL ENGINEERING_SE_XXXX_CH010(Part-II).indd 560 6/15/2018 4:55:18 PM



Internal Combustion Engine Systems 561

10.16.4  Phenomena of Knocking/Detonation in SI Engines


In a SI engine, combustion is initiated between the spark plug electrodes and then spreads across the
combustible mixture. A definite flame front which separates the fresh mixture from the products of
combustion travels from the spark plug to the other end of the combustion chamber. Heat release due to
combustion increases the temperature and pressure of the burned part of the mixture above those of the
unburned mixture. In order to effect pressure equalisation, the burned part of the mixture will expand
and compress the unburned part adiabatically, thereby increasing the pressure and temperature of the
unburned part further. If the temperature of the unburned mixture exceeds the self-ignition temperature
of the fuel and remains at or above this temperature during the ignition lag, spontaneous ignition (or
auto-ignition) occurs at various pin-point locations. This phenomenon is called detonation or knocking.
The phenomenon of knocking may be explained by referring to Fig. 10.61(a) which shows the
cross-section of the combustion chamber with flame advancing from the spark plug location A without
knock, whereas Fig. 10.61(c) shows the combustion process with knock. In the normal combustion, the
flame travels across the combustion chamber from A towards D. The advancing flame front compresses
the end charge BB′D farthest from the spark plug, thus raising its temperature. The temperature is also
increased due to heat transfer from the hot advancing flame front and the pre-flame oxidation in the
end charge. If the temperature of the end charge does not reach its self-ignition temperature, the charge
would not auto-ignite and the flame will advance further and consume the charge BB′D. This is the nor-
mal combustion process which is illustrated by means of the pressure-time diagram in Fig. 10.61(b).
If the end charge BB′D reaches its auto-ignition temperature and remains for some period of time equal
to the time of pre-flame reactions, the charge will auto-ignite, leading to detonation. In Fig. 10.61(c),
Pressure

B
Ignition

A D n
ssio
mpre Power
Co
BDC TDC BDC
B′ Time
(a) (b)
Pressure

B C
Ignition

A D n
ssio
m pre
Co
BDC TDC BDC
C′
B′ Time
(c) (d)

Figure 10.61 Normal and abnormal combustion: (a) Movement


of flame front for normal combustion, (b) P-q diagram for
normal combustion, (c) Movement of flame front for abnor-
mal combustion, (d) P-q diagram for abnormal combustion

M10_THERMAL ENGINEERING_SE_XXXX_CH010(Part-II).indd 561 6/15/2018 4:55:19 PM


562 Chapter 10

it is assumed that when the flame has reached the position BB′, the charge ahead of it has reached
critical auto-ignition temperature. During the pre-flame reaction period, if the flame front could move
from BB′ to only CC ′ , the charge ahead of CC ′ would auto-ignite. Due to auto-ignition, another flame
front starts travelling in the opposite direction to the main flame front. When the two flame fronts
collide, a severe pressure pulse is generated. The gas in the chamber is subject to compression
and rarefaction along the pressure pulse until pressure equilibrium is restored. This disturbance
can force the walls of the combustion chamber to vibrate at the same frequency as the gas (≈ 5000 Hz).
The pressure-time trace of such a situation is shown in Fig. 10.61(d).

10.16.5  Factors Influencing Detonation/Knocking


The factors influencing knocking are as follows:
1. Density factors: Any factor which reduces the density of the charge tends to reduce knocking by
providing lower energy release.
(a) Compression ratio: Increase in compression ratio increases the pressure and temperature of
the gases at the end of compression stroke which decreases the ignition lag of the end gas and
increases the tendency of knocking.
(b) Mass of induced charge: An increase in the mass of induced charge into the cylinder of
an engine by throttling or by increasing the amount of supercharging increases both tem-
perature and density of the charge at the time of ignition. This increases the tendency
for knocking.
(c) Inlet temperature of mixture: Increase in inlet temperature of mixture makes the compres-
sion temperature higher thereby, increasing the tendency of knocking.
(d) Temperature of combustion chamber walls: Hot spots (such as spark plug and exhaust
valve) in combustion chamber promote knocking.
(e)  Retarding spark timing: Having the spark closer to the TDC reduces knocking. However,
this affects the brake torque and power output of the engine.
(f) Power output of the engine: A decrease in the output of the engine reduces the tendency to
knock.
2. Time factors: The following factors reduce the possibility of knocking:
(a) Turbulence: Turbulence increases the engine speed and reduces the time available for the end
charge to attain auto-ignition conditions thereby decreasing the tendency to knock.
(b) Engine speed: An increase in engine speed increases turbulence and decreases the tendency
to knock.
(c) Flame travel distance: Shortening the time required for the flame front to traverse the com-
bustion chamber reduces the tendency for knocking. Flame travel distance is governed by
combustion chamber size and spark plug position.
(d)  Engine size: A large engine (combustion chamber size) has a greater tendency for knocking
as the flame requires a longer time to travel across the combustion chamber.
(e) Combustion chamber shape: The combustion chamber should be such that it promotes
turbulence to reduce knocking. Spherical chambers minimise knocking tendency.
(f)  Location of spark plug: A centrally located spark plug or multiple spark plugs minimise
flame travel time and reduce knocking.

M10_THERMAL ENGINEERING_SE_XXXX_CH010(Part-II).indd 562 6/15/2018 4:55:19 PM



Internal Combustion Engine Systems 563

3. Composition factors:
(a)  Fuel: F/A ratio affects the reaction time or ignition delay. When the mixture is nearly 10%
richer than stoichiometric (F/A ratio = 0.08), ignition lag of the end gas is minimum, velocity
of flame propagation is maximum, and knocking tendency is maximum. A rich mixture is
effective in decreasing the knocking tendency due to a longer delay period and lower tem-
perature of compression.
(b) Octane rating of fuel: Higher the octane number, lesser is the tendency for knocking. Par-
affin series have the maximum and aromatic series the minimum tendency to knock. The
knocking characteristics of a fuel can be decreased by adding small amounts of additives
called dopes.
(c) Humidity of air: Increasing the humidity of the atmospheric air decreases the tendency to
knock.
4. Temperature factors: Increasing the temperature of the unburned mixture by any factor in
design or operation will increase the possibility of knocking in SI engine. The temperature of the
unburned mixture is increased by the following factors:
(a) Raising the compression ratio: For a given engine setting and fuel, there will be a critical
compression ratio above which knocking occurs. This compression ratio is called the highest
useful compression ratio (HUCR).
(b) Supercharging
(c)  Raising the inlet temperature
(d)  Raising the coolant temperature
(e)  Increasing the load (opening the throttle)
(f)  Raising the temperature of the cylinder and combustion chamber walls
(g)  Advancing the spark timing
A summary of variable affecting knocking in SI engines is given in Table 10.8.

Table 10.8  Summary of variables affecting knock in an SI engine

S. no. Increase in variable Major effect on Action to be taken to Can operator


unburned reduce knocking usually control?
charge
1. Compression ratio Increases temperature Reduce No
and pressure
2. Mass of charge inducted Increases pressure Reduce Yes
3. Inlet temperature Increases temperature Reduce In some cases
4. Chamber temperature Increases temperature Reduce Not ordinarily
5. Spark advance Increases temperature Retard In some cases
and pressure
6. A/F ratio Increases temperature Make very rich In some cases
and pressure
7. Turbulence Decreases time factor Increase Somewhat (through
engine speed)
(Continued)

M10_THERMAL ENGINEERING_SE_XXXX_CH010(Part-II).indd 563 6/15/2018 4:55:19 PM


564 Chapter 10

Table 10.8  (Continued)

S. no. Increase in variable Major effect on Action to be taken to Can operator


unburned reduce knocking usually control?
charge
8. Engine speed Decreases time factor Increase Yes
9. Distance of flame travel Increases time factor Reduce No

10.16.6  Methods for Suppressing Knocking


1. Proper location of spark plug: A compact combustion chamber with proper central location of
the spark plug reduces the path of the flame front travel from the spark plug to the remotest part
of the combustion chamber, and thereby, reduces the tendency of knocking. The same result could
be obtained by multi-fuel injection system.
2. Proper selection of material for piston and cylinder head: The end charge cooling is much
better when the piston and cylinder are made of aluminium alloys as they have good thermal con-
ductivity. A better cooling of the end charge reduces the tendency for knocking.
3. Injecting water into the intake manifold: Injection of water in the cylinder reduces the temper-
ature of the end gas and increases the delay period, thereby decreasing the tendency to knock.
4. Retarding the spark timing: By retarding the spark timing, the peak pressures are reached only
during the power stroke and are of lower magnitude. This reduces knock.
5. Extremely rich or lean mixture: By using extremely rich or lean mixture, the flame temperature
can be kept low, thus considerably eliminating the tendency of knock.
6. Squish recesses in combustion chamber: The provision of squish recess in combustion chamber
cools the last position of the charge and reduces the tendency of knock.

10.16.7  Effects of Knocking/Detonation


1. Mechanical damage: Knocking creates a high pressure wave of large amplitude. This increases
the rate of wear of almost all mechanical parts such as piston, cylinder head, and valves. The
engine parts are also subjected to very high temperatures due to auto-ignition and the piston is
damaged by overheating.
2. Noise: When the intensity of the knock is high, a loud pulsating noise is created due to high inten-
sity pressure wave which vibrates back and forth across the cylinder. The high vibrating motion
of the gases causes crankshaft vibrations and engine runs rough.
3. Heat transfer rate: The heat transfer rate increases as more heat is lost to the coolant.
4. Power output: The power output slightly decreases due to rapid burning of last part of the charge.
5. Pre-ignition: Pre-ignition is the ignition of the charge in the absence of spark as it comes in con-
tact with hot surface. This adds negative work by the piston during compression which increases
high peak pressure and temperature in the cylinder.

M10_THERMAL ENGINEERING_SE_XXXX_CH010(Part-II).indd 564 6/15/2018 4:55:19 PM


Internal Combustion Engine Systems 565

10.17  ❐  COMBUSTION CHAMBERS FOR SI ENGINES


The design of combustion chamber involves the shape of the combustion chamber, location of the
spark plug, and disposition of inlet and exhaust valves.

10.17.1  Basic Requirements of a Good Combustion Chamber


The three main requirements of a SI engine combustion chamber are high power output with mini-
mum octane requirement, high thermal efficiency, and smooth engine operation.
1. Higher power output requires the following:
(a)  High compression ratio
(b)  Small or no excess air
(c)  No dead pockets
(d)  An optimum degree of turbulence
(e)  High volumetric efficiency
2. High thermal efficiency requires the following:
(a)  High compression ratio
(b)  Small heat loss during combustion
(c)  Good scavenging of exhaust gases
3. Smooth engine operation requires the following:
(a)  Moderate rate of pressure rise during combustion.
(b) Absence of detonation: compact combustion chamber, proper location of spark plug and
exhaust valve, and satisfactory cooling of spark plug points.

10.17.2  Combustion Chamber Design Principles


1. To achieve high volumetric efficiency, the largest possible inlet valve should be accommodated
with ample clearance around the valve heads.
2. To prevent detonation, the length of the flame travel from the spark plug to the farthest point in
the combustion space should be as short as possible.
3. To reduce the possibility of detonation, the exhaust valve should be near the spark plug.
4. The exhaust valve should be kept small due to its hot surface, but to compensate for this, a high
lift should be employed.
5. The correct amount of turbulence should be created in the combustion chamber by properly posi-
tioning the inlet valve. Turbulence should be preferably created by squish.
6. The shape of the combustion chamber should be such that the largest mass of the charge burns as
soon as possible after ignition with progressive reduction in the mass of charge burnt towards the
end of combustion.
7. To ensure high thermal efficiency and less air pollution, the surface–volume ratio of the chamber
should be minimum in the beginning.
8. In the end gas region, surface–volume ratio should be large to achieved good cooling in the
‘detonation zone’.

M10_THERMAL ENGINEERING_SE_XXXX_CH010(Part-III).indd 565 6/15/2018 5:00:36 PM


566 Chapter 10

9. The spark plug should be so positioned so that it will be scoured of any residual exhaust products
by the incoming charge.
10. The exhaust valve head should be well-cooled as it is the hottest region of the combustion
chamber.
11. The sparking plug points should be sufficiently cooled to avoid pre-ignition effects.
12. There should be good scavenging of exhaust gases.
13. Thickness of wells must be uniform for uniform expansion.
14. It is desirable to employ a plain flat-topped piston on manufacturing grounds.
15. To achieve maximum thermal efficiency, the highest possible compression ratio should be used.

10.17.3  Combustion Chamber Designs


The various types of combustion chambers are shown in Fig. 10.62.
1. T-head combustion chamber: In the earliest engine, the T-head design shown in Fig. 10.62(a)
was used. Its disadvantages are that it has two camshafts and it is prone to detonation.
2. L-head or side valve combustion chamber: This is shown in Fig 10.62(b). Its advantages are as
follows:
(a) A side valve engine is easy to manufacture.
(b) It is easy to enclose and lubricate the valve mechanism.
(c) Its head can be removed without disturbing the valve-gear and pipe work for decarbonising.
(d) It provides a compact layout.
(e) Cooling of exhaust by inlet charge is more effective as both are situated by the side.
Its shortcomings are as follows:
(a) There is lack of turbulence in the incoming charge.
(b) It is extremely prone to knocking as the flame has to travel long distance.
(c) It is extremely sensitive to ignition timing due to slow combustion.

S
S
S

I E I IE
E

(a) (b) (c)

I E
E S
S

E – Exhaust valve I
I – Inlet valve
S – Spark plug

(d) (e)

Figure 10.62 Examples of typical combustion chambers: (a) T-head type, (b) L-head type,
(c) L-head side valve type, (d) I-head type, (e) F-head type

M10_THERMAL ENGINEERING_SE_XXXX_CH010(Part-III).indd 566 6/15/2018 5:00:37 PM


Internal Combustion Engine Systems 567

3. Ricardo turbulent L-head side valve design: This arrangement is shown in Fig. 10.62(c). This
gives higher flame speed and reduces the knocking tendency. The entire combustion space is
concentrated over the inlet and exhaust valves. This provides sufficient turbulence during the
compression stroke and increased flame velocity. The ratio of area to volume of the space above
the piston is large, which provides sufficient quench area essential to avoid knocking.
4. Overhead valve or I-head combustion chamber: In this arrangement, both the valves are located
in the cylindrical head as shown in Fig. 10.62(d). This arrangement is superior to side valve or
L-head arrangement when high compression ratios are used for the following reasons:
(a) Lower pumping losses and higher volumetric efficiency from better breathing of the engine
from larger valves or valve lifts and more direct passage ways.
(b) Less distance for the flame to travel and therefore greater freedom from knock.
(c) Less force on the head bolts and therefore less possibility of leakage.
(d) Absence of exhaust valve from block results in more uniform cooling of cylinder and piston.
(e) Lower surface-volume ratio and, therefore, less heat loss and less air pollution.
(f ) Easier to cast and hence lower casting cost.
5. F-head combustion chambers: A combustion chamber in which one valve is located in the head
and other in the block is known as F-head combustion chamber, as shown in Fig. 10.62(e). This
arrangement is a compromise between L-head and I-head combustion chambers. Figure 10.62(e)
shows exhaust valve in head and inlet valve in block. This gives a rather poor performance. The
modern F-head engines have inlet valve in the head and exhaust valve in the block. The valves
are inclined with plug located in the flat roof. This provides the shortest flame travel and ensures
minimum knocking tendency.

10.18  ❐  COMBUSTION IN CI ENGINES


In the CI engines, only air is compressed through a large compression ratio during the compression
stroke, raising highly its temperature and pressure. At this stage, one or more jets of fuel are injected
in the liquid state, compressed to a high pressure by means of a fuel pump. Each minute droplet, as
it enters the hot air, is quickly surrounded by an envelope of its own vapour and after an appreciable
interval, is inflamed at the surface of the envelope. As soon as this vapour and the air in contact with it
reach a certain temperature and the local air-fuel ratio is within combustion range, ignition takes place.

10.18.1  Stages of Combustion


The stages of combustion in the CI engine are shown in Fig. 10.63.
1. First stage—ignition delay period: During the first stage, some fuel has been admitted but has
not yet been ignited. The ignition delay is counted from the start of injection to the point where
the pressure-time (crankshaft rotation) curve separates from the pure air compression curve. The
delay period is a sort of preparatory phase.
2. Second stage—rapid or uncontrolled combustion (premixed flame): In the second stage, the
pressure rise is rapid because during the delay period the fuel droplets have had time to speed
over a wide area and they have fresh air all around them. This period is counted from the end of
the delay period to the point of maximum pressure on the indicator diagram. About one-third of
the heat is evolved during this period.

M10_THERMAL ENGINEERING_SE_XXXX_CH010(Part-III).indd 567 6/15/2018 5:00:37 PM


568 Chapter 10

12 3 4 Compression
pressure
Start of
combustion Motoring
(non-firing)
Pressure

Start of
injection

0.001s

Injection
Atmospheric

100° 80° 60° 40° 20° TDC 20° 40° 60° 80° 100°

Figure 10.63  Stages of combustion in a CI engine

3. Third stage—controlled combustion (diffusion flame): At the end of the second stage, the
temperature and the pressure are so high that the fuel droplets injected during the last stage
burn almost as they enter and any further pressure rise can be controlled by the injection rate.
The period of controlled combustion is assumed to end at maximum cycle temperature. About
70%–80% of total heat of the fuel is supplied during this period.
4. Fourth stage—after burning: Due to poor distribution of fuel particles, combustion continues
during the part of the remainder of the expansion stroke. This after-burning can be called the
fourth stage of combustion. The after-burning period is about 70°–80° of crank angle from the
TDC. The total heat evolved during the end of entire combustion process is 95–97% and 3–5% of
heat goes to unburnt fuel.

10.18.2  Delay Period or Ignition Delay


In Fig. 10.64, the delay period is shown on pressure-crank angle (or time) diagram between points
a and b. Point a represents the point of injection and point b represents the time at which the pressure
curve (caused by combustion) first separates from the compression curve (non-firing or motoring
curve). This ignition delay period can be roughly divided into two parts.
1. Physical delay: The period of physical delay is the time between the beginning of injection and
the attainment of chemical reaction conditions. In the physical delay period, the fuel is atomised,
vapourised, mixed with air, and raised in temperature.
2. Chemical delay: In this period, the pre-flame reactions start slowly and then accelerate until local
ignition takes place. Generally, chemical delay is longer than the physical delay. The delay period
refers to the sum of physical and chemical delay. The ignition lag in SI engine is basically equiv-
alent to the chemical delay in CI engine. There is no component like physical delay in SI engine.

M10_THERMAL ENGINEERING_SE_XXXX_CH010(Part-III).indd 568 6/15/2018 5:00:38 PM


Internal Combustion Engine Systems 569

TDC
b
a c
a – Start of injection Maximum
b – Start of combustion pressure

Preesure
Co
c – Ignition delay M m
a c b ot pr bu
d – Mixing period or es st
e – Interaction period in su io
g n
Area A cu re
e rv
d e
Expanded view of area A Time

Figure 10.64 Pressure-time diagram illustrating ignition delay

10.18.3  Variables Affecting Delay Period


1. Fuel: A lower pre-ignition temperature means a wider margin between it and the temperature of
the compressed air and hence delay period. A higher cetane number means a lower delay period
and smoother engine operation. Other properties of fuel which affect delay period are: volatility,
latent heat, viscosity, and surface tension.
2. Injection pressure or size of droplet: The smaller the size and greater the number of droplets,
the larger will be aggregate area of inflammation and therefore the greater the uncontrolled pres-
sure rise. The size of the droplets depends on the injection pressure. Therefore, lower the injec-
tion pressure the lower the rate of pressure rise during the uncontrolled phase and smoother the
running.
3. Injection advance angle: The delay period increases with increase in injection advance angle
because the pressures and temperatures are lower when the injection begins. When the injection
advance angles are small, the delay period reduces and operation of the engine is smoother but the
power is reduced because larger amount of fuel burns during after-burning. The optimum angle of
injection advance is 12°–20° before TDC.
4. Compression ratio: Increase in compression ratio reduces the delay period as it raises both tem-
perature and density. However, higher compression ratio decreases the volumetric efficiency and
power. It also decreases mechanical efficiency. In practice, the engine designer uses the lowest
compression ratio which would satisfy the needs of easy cold starting and light load running at
high speeds.
5. Intake temperature: Increasing the intake temperature would result in increase in the com-
pressed air temperature, which would reduce the delay period. However, it would reduce the
density of air and hence volumetric efficiency and power output.
6. Jacket water temperature: Delay period is reduced with increase in jacket water temperature or
compressed air temperature increases.
7. Fuel temperature: Increase in fuel temperature would reduce both physical and chemical delay
periods.
8. Intake pressure or supercharging: Supercharging reduces the auto-ignition temperature and
hence reduces delay period.
9. Speed: As the engine speed increases, the loss of heat during compression decreases with the
result that both the temperature and pressure of the compressed air tend to rise, thus reducing the
delay period.

M10_THERMAL ENGINEERING_SE_XXXX_CH010(Part-III).indd 569 6/15/2018 5:00:40 PM


570 Chapter 10

Table 10.9  Effect of variables on delay period in CI engine

S. no. Increases in variable Effect on delay period Reason


 1. Cetane number of fuel Reduces Reduces self-ignition temperature
 2. Injection pressure Reduces Greater surface area to volume
ratio hence less physical delay
 3. Injection advance angle Increases Pressures and temperatures lower
when injection begins
 4. Compression ratio Reduces Increases air temperature and
pressure and reduces auto-ignition
temperature
 5. Intake temperature Reduces Increases air temperatures
 6. Jacket water temperature Reduces Increases wall temperature and
hence air temperature
 7. Fuel temperature Reduces Better vapourisation and increases
chemical reaction
 8. Intake pressure (Supercharging) Reduces Increases in density reduces auto-
ignition temperature
 9. Speed Reduces in milliseconds Less loss of heat because of more
crank angle in a given time
10. Load (fuel-air ratio) Increase in crank angle
11. Engine size Little effect Operating temperature increases
12. Type of combustion chamber Lower for precombustion Low rpm
chamber

10. Air-fuel ratio (load): With increase in air-fuel ratio the combustion temperatures are lowered and
hence the delay period increases. With increase in load, air-fuel ratio decreases operating temper-
ature increase and hence, delay period decreases.
11. Engine size: The engine size has little effect on the delay period in milliseconds but crank angle
decreases.
12. Type of combustion chamber: A pre-combustion chamber gives shorter delay period as com-
pared to an open type of combustion chamber.
The summary of the effect of delay period in a CI engine is given in Table 10.9.

10.19  ❐  KNOCKING IN CI ENGINES


The fuel injection takes place over a definite period. Consequently, the first droplets injected are pass-
ing through the ignition delay, additional droplets are being injected into the chamber. If the ignition
delay of the injected fuel is small the first droplets start burning in a relatively short time after injection
and relatively small amount of fuel will be accumulated in the chamber. Therefore, the rate of pressure
rise will be smooth as shown in Fig. 10.65(a). However, if the delay period is longer, the accumula-
tion of fuel will be larger as the actual burning of the first droplets is delayed. When actual burning
starts, the accumulated fuel and additional fuel injected starts burning and can cause too rapid rate of
pressure rise. This rapid rate of pressure rise causes pulsating combustion and creates heavy noise.

M10_THERMAL ENGINEERING_SE_XXXX_CH010(Part-III).indd 570 6/15/2018 5:00:40 PM


Internal Combustion Engine Systems 571

Start of combustion

Start of combustion
Start of ignition

Start of ignition
Pressure

Pressure
TDC TDC
(a) (b)

Figure 10.65 Combustion in CI engine: (a) Without knocking,


(b) With knocking

This pulsating variation of pressure due to sudden burning of accumulated fuel is shown in
Fig. 10.65(b). This type of abnormal combustion is known as knocking. This knocking occurs during
the initial period over the delay period when the burning of the first portion of the fuel is uncontrolled,
thus giving rise to an excessive rate of pressure rise.

10.19.1  Factors Affecting Knocking in CI Engines


The factors which influence the delay period also influence the knocking tendency of CI engines are
as follows.
1. If the injection of fuel is too far advanced, the rate of pressure rise during auto-ignition is very
high and cause knocking.
2. Inferior fuels (having lower cetane number) promote diesel knock but this can be avoided by
using better types of fuel (higher cetane number).
3. Fuel injection parameters (better penetration and distribution) and better combustion chamber
design (split type) also influence knocking tendency of the engine.
4. Initial condition of the air (higher temperature or higher pressure in case of supercharged engine
or both) influences knocking tendency. Higher pressure and temperature reduces knocking
tendency.
5. The fuel having longer delay period and higher self-ignition temperature leads to knocking.
With proper design of combustion chamber and injecting the fuel uniformly over a hot surface of the
combustion chamber, the detonation can be avoided.

10.19.2  Controlling the Knocking


If the delay period is long, a large amount of fuel will be injected and accumulated in the chamber.
The auto-ignition of this large amount of fuel may cause high rate of pressure rise and high maximum
pressure which may cause knocking in diesel engines. The methods for controlling diesel knock are
as follows:
1. Cetane number of the fuel used: Cetane number is a scale for comparing the ignition delay
period of various diesel fuels. A higher Cetane number means a lower delay period, and smoother

M10_THERMAL ENGINEERING_SE_XXXX_CH010(Part-III).indd 571 6/15/2018 5:00:41 PM


572 Chapter 10

mf

Figure 10.66  Required rate of fuel supply to avoid detonation

engine operation, because it reduces the self-ignition temperature. Some compounds called addi-
tives or dopes are used to improve combustion performance of fuels. Various additives used for
diesel fuels are: Isopropyl nitrate, Amyl nitrate, Heptyl peroxide and Butyl peroxide. These dopes
delay the auto-ignition but they increase NOx drastically.
2. Compression ratio: Increase in compression ratio reduces the delay period as it raises both tem-
perature and density. At the same time the minimum auto-ignition temperature decreases which
reduces the time of reaction when fuel is injected. Thus the delay period decreases. With increase
of compression ratio the unused air would be much more decreasing the volumetric efficiency and
power. It further decreases the mechanical efficiency due to increase in weight of reciprocating
parts.
3. Air-fuel ratio: With decrease in air-fuel ratio (richer mixture) the combustion temperatures are
increased and cylinder wall temperatures are increased and hence the delay period decreases.
With increase of load air-fuel ratio decreases, operating temperature increases and hence delay
period decreases.
4. Injection pressure: Lower the injection pressure, the lower the rate of pressure rise during the
uncontrolled phase and smoother the running of engine. Lower injection pressure gives rise to
fuel droplets of smaller size to obtain largest surface-volume ratio resulting in less physical delay.
Large droplets of fuel results in too low subsequent burning.
5. Controlling the rate of fuel supply: It can be reduced by controlling the initial rate of fuel
delivery without affecting efficient combustion condition. A small amount of fuel should be sup-
plied until combustion starts and more fuel should be supplied as the inside combustion chamber
condition is suitable (because of high temperature inside) to burn more fuel. In this case, the cam
shape is so designed that it gives a low initial injection followed by a main injection at a higher
rate as shown in Fig 10.66.
6. Knock reducing fuel injector: This type of injector is developed so that it can avoid the sudden
increase in pressure inside the combustion chamber due to the accumulated fuel. In this case, the
injection pressure of 100 bar is used with a semi-flexible needle valve.

10.19.3  Comparison of Knocking in SI and CI Engines


It is interesting to compare the knocking phenomenon in SI and CI engines. This phenomenon in both
engines is fundamentally similar as it involves auto-ignition, subject to the ignition-lag characteristics
of A:F mixture supplied to the engine.
1. In SI engines, the knocking occurs near the end of combustion, whereas in CI engine, this occurs
at the beginning of combustion as shown in Fig. 10.67.

M10_THERMAL ENGINEERING_SE_XXXX_CH010(Part-III).indd 572 6/15/2018 5:00:43 PM


Internal Combustion Engine Systems 573

Pressure

Pressure
For SI For CI
engine engine

Crank angle Crank angle


TDC TDC

Figure 10.67  Knocking in SI and CI engines

2. The knocking in SI engine takes place in a homogeneous mixture; therefore, the rate of pressure
rise and maximum pressure is considerably high. In case of a CI engine, the mixture is not homo-
geneous and the rate of pressure is lower than in an SI engine.
3. The question of pre-ignition does not arise in CI engines as the fuel is supplied only near the end
of compression stroke.
4. It is easier to distinguish between knocking and non-knocking operations in SI engines as the
human ear easily finds the difference. However, in a CI engine, the normal ignition is by auto-ig-
nition and the rate of pressure-rise under normal operation is considerably higher (10 bar against
2.5 bar for SI engines) and causes high noise. The noise level becomes excessive under knocking
condition. Therefore, there is no definite distinction between normal and knocking combustion.
The knocking in SI engines is due to the auto-ignition of the last part of the charge. To avoid this, the
fuel must have a long-delay period and high self-ignition temperature.
To avoid knock in the CI engine, the delay period should be as small as possible and the fuel
self-ignition temperature should be as low as possible.
The factors which reduce knock in SI and CI engines are given in Table 10.10.
Due to this dissimilarity in the time of starting of the knock in SI and CI engines, the conditions
which reduce the knock tendency in the SI engine increase the knock tendency in CI engines. The fuels
which are better from the point of view of avoiding detonation in SI engines may promote detonation
in CI engines. Therefore, good SI fuels are poor CI fuels. In terms of fuel rating, diesel oil has high
cetane number (40–60) and low octane number (30) and petrol has high octane number (80–90) and
low cetane number (20).

Table 10.10  Factors which reduce knock in SI and CI engines

S. no. Factors affecting knock SI engines CI engines


1. Self-ignition High Low
2. Delay period of fuel Long Short
3. Compression ratio Low High
4. Inlet temperature Low High
5. Inlet pressure Low High
6. Speed High Low
7. Cylinder size Small Large
8. Combustion chamber wall temperature Low High

M10_THERMAL ENGINEERING_SE_XXXX_CH010(Part-III).indd 573 6/15/2018 5:00:43 PM


574 Chapter 10

10.20  ❐  COMBUSTION CHAMBERS FOR CI ENGINES


The most important function of the CI engine combustion chamber is to provide proper mixing of fuel
and air in a short time. For this purpose, an organised air movement, called air swirl, is provided to
produce high relative velocity between the fuel droplets and air.
There are three basic methods of generating swirl in a CI engine combustion chamber as follows.
1. By directing the flow of the air during its entry to the cylinder, known as induction swirl. This
method is used in open combustion chambers shown in Figs 10.68 (a) to (d).
2. By enforcing air through a tangential passage into a separate swirl chamber during the compres-
sion stroke, known as compression swirl. This method is used in swirl chambers. This type of
chamber is shown in Fig. 10.69.

(a) (b) (c) (d)

Figure 10.68 Open combustion chambers: (a) Shallow depth chamber,


(b) Hemispherical chamber, (c) Cylinderical chamber,
(d) Toroidal chamber

Nozzle

Spherical shaped
swirl chamber

Glow plug

Throat

Piston

Figure 10.69  Ricardo swirl chamber comet, mark II

M10_THERMAL ENGINEERING_SE_XXXX_CH010(Part-III).indd 574 6/15/2018 5:00:44 PM


Internal Combustion Engine Systems 575

Intel and exhaust valves Two-part energy cell

Cylinder
head

Injector
Nozzle

Cylinder

Piston
Two-lobed main combustion chamber
Prechamber
Orifice

Piston

(a) (b)

Figure 10.70 Combustion-induced swirl chambers: (a) Precombustion chamber,


(b) Lanova air-cell combustion chamber
3. By use of the initial pressure rise due to partial combustion to create swirl turbulence, known as
combustion induced swirl. This method is used in pre-combustion chambers and air-cell cham-
bers. These type of chambers are shown in Figs 10.70(a) and (b).
The comparison of induction and compression swirl is given in Table 10.11 and comparison of com-
bustion characteristics in Table 10.12.

Table 10.11  Comparison of induction and compression swirl

Induction swirl Compression swirl


Advantages Disadvantages
1. High excess air (low temperature), low tur- Less excess air, lower indicated efficiency 5–8% more
bulence (less heat loss). Therefore, indicated fuel consumption. Decreased exhaust valve life.
thermal efficiency high.
2.  Due to low intensity of swirl easier starting. Cold starting trouble due to high heat loss due to strong
swirl, greater surface area/volume ratio.
3. No additional work for producing swirl. High Work absorbed in producing compression swirl, mechan-
mechanical efficiency and hence high brake ical efficiency lower.
thermal efficiency.
4. Used with low speeds. Therefore low quality Cylinder more expensive in construction.
of fuel can be used.
(Continued)

M10_THERMAL ENGINEERING_SE_XXXX_CH010(Part-III).indd 575 6/15/2018 5:00:45 PM


576 Chapter 10

Table 10.11  (Continued )

Induction swirl Compression swirl


Disadvantages Advantages
1. Weak swirl, multi-orifice nozzle, high Single injector, pintle-type (self-cleaning), less
injection pressure, clogging of holes, high maintenance.
maintenance.

2. Influences minimum quantity of fuel. Compli- Large valves, higher volumetric efficiency.
cation at high loads and idling.

3. Shrouded valves, smaller valves, low volumet- Greater air utilisation due to strong swirl. Smaller
ric efficiency. (cheaper) engine.

4. Weak swirl, low air utilisation (60%), lower Swirl proportional to speed, suitable for variable speed
mep, large size (costly) engine. operation.

5. Swirl not proportional to speed. Efficiency not Smooth engine operation.


maintained in a variable speed engine.

Table 10.12  C
 omparison of combustion chamber characteristics (Mainly for four-stroke
non-supercharged high-speed engines)

S. no. Characteristics Open MAN Swirl Pre- Air-cell


chamber ‘M’-chamber chamber combustion combustion
     1.
  Speed capability, rpm 3500 3500 4000 3000 3500
     2.
  bmep, bar 6.6 6.9 8.3 6.3 6.3
     3.
  Rate of pressure rise, 6.6 6.9 8.3 6.3 6.3
bar/degree crank angle
     4.
  Compression ratio 15–16 16–17 17–19 18 13–16
     5.
  Injection pressure, bar 560 280 350 140 280
     6.
  Nozzle holes 6–8 1 1 1 1
     7.
  Pumping loss Low Medium High High High

     8.
  Volumetric efficiency Low Lowest High High High
     9.
  Coolant load Low Medium Highest High High
10. Air motion Medium High Highest High High
11. Combustion noise High Lowest Low Low Low
12. Cold starting Excellent Fair ←Needs help→

M10_THERMAL ENGINEERING_SE_XXXX_CH010(Part-III).indd 576 6/15/2018 5:00:45 PM


Internal Combustion Engine Systems 577

10.21  ❐  LUBRICATION SYSTEMS


All moving parts of an internal combustion engine have relative motion and rub against each other.
The lubrication is required to reduce this rubbing action and increases the life of engine. The pur-
pose of lubrication is to reduce the rubbing action between different machine parts having relative
motion and to remove the heat generated inside the cylinder. Due to friction between rubbing parts
and combustion of fuel in the cylinder, the temperature of various parts, cylinder, and piston may rise
enormously if the engine is not cooled. The lubrication system will also be seriously affected, leading
to damage to the cylinder and piston. Therefore, it is essential to provide lubrication and cooling to
various engine parts.

10.21.1  Functions of a Lubricating System


The important functions of a lubricating system are as follows:
1. Lubrication: The main function of the lubricating system is to keep the moving parts sliding
freely past each other and thus, reduce the engine friction and wear.
2. Cooling: To keep the surfaces cool by taking away a part of their heat through the oil passing over
them. This requires oil to possess good oxidation stability.
3. Cleaning: To keep the bearings and piston rings clean of the products of wear and combustion by
washing them away.
4. Sealing: The lubricating oil must form a good seal between piston rings and cylinder walls.
Therefore, it must possess adequate viscosity stability.
5. Reduction of noise: Lubrication reduces the noise of the engine and other sliding parts.

10.21.2  Desirable Properties of a Lubricating Oil


The desirable properties of a lubricant are as follows:
1. It should be available in a wide range of viscosities.
2. There should be little change in the viscosity of the oil with a change in temperature.
3. The oil should be chemically stable at all temperatures encountered in its application.
4. The oil should have sufficient specific heat to carry away frictional heat without abnormal rise in
temperature.
5. It should be commercially available at a reasonable cost.

10.21.3  Lubricating Systems Types


There are two types of lubricating systems:
1. Splash lubricating system: The arrangement of a splash lubricating system is shown in
Fig. 10.71. This method is generally used for a vertical engine with a closed crankcase. The sump
is located at the bottom of the crankcase. When the engine crankshaft rotates, the big end of the
connecting rod splashes oil by centrifugal action. The connecting rod big end has a hollow pipe
called a scoop which is fitted to the bearing cap and pointed towards the direction of rotation of
the crankshaft. The lubricating oil passing through the scoop lubricate the big end bearing and

M10_THERMAL ENGINEERING_SE_XXXX_CH010(Part-III).indd 577 6/15/2018 5:00:45 PM


578 Chapter 10

gudgeon pin bearing. All other parts are lubricated by the splash. Excess oil is collected in the
troughs, is provided with overflows, and collected in the main sump. The level of the oil in the
trough is maintained constant. The dripping from the cylinders is also collected in the sump. The
oil from the sump is recirculated with the help of a pump.
The limitations of this system are as follows:
(a) Inability to regulate the quantity of oil splashed against the cylinder wall.
(b) Inability to keep the oil from getting past the piston head into the combustion chamber,
burning with the fuel and passing out with exhaust gases.
2. Pressure feed lubricating system: This system is shown in Fig. 10.72. Such a system supplies
oil under pressure directly to the connecting rod bearing, crankshaft bearings, valve gear, and the
camshaft drive. Indirect supplies reach the cylinder walls, gudgeon pin, the distributor, and pump
drives.
Oil is carried in the sump and circulated by the gear pump which sucks from the sump through a
strainer. The pump delivery pressure is controlled by a relief valve and the oil passes through a very
fine filter before it reaches the main distributor gallery from the various bearings, surfaces, and gears.
After lubricating the big end bearings, the oil is fed to the gudgeon pins through the oilway in
the connecting rod and further squirted into the cylinder wall.
3. Charge lubricating system: It does not require oil-filter and oil sump. In this system, the lubri-
cating oil is pre-mixed with the fuel which carries the lubricating oil in the cylinder to lubricate the
piston and cylinder. Most of the oil burns with the fuel and is carried away with the exhaust gases.

Oil pressure gauge

Camshaft

Connecting rod bearing


Main bearing

Lower
oil pan
Oil troughs

Oil strainer Oil pump Oil sump

Figure 10.71  Splash lubrication system

M10_THERMAL ENGINEERING_SE_XXXX_CH010(Part-III).indd 578 6/15/2018 5:00:46 PM


Internal Combustion Engine Systems 579

To valve
Bypass rocker gear Rocker shaft
filter
Oil return form cylinder head
(drain tubes)
Pressure Camshaft
gauge

Screw gear oil pump


and distributor drive
Oil filter

Oil pump

Sump Strainer Pressure relief


valve
Drain plug Excess oil

Figure 10.72  Wet sump high pressure lubrication system

The lubricating oil cannot be recovered in this system. The system is generally used for two stroke
spark ignition engines of scooters and motorcycles.
  The main disadvantages of this system are as follows:
(a) Carbon deposits due to burning of oil on the spark plug
(b) Non-recovery of the oil used
The advantages of this system are as follows:
(a) It does not require a separate lubricating system and is the most economical one.
(b) There is no risk of failure of lubrication system.
(c) The lubricating oil supplied is regulated at various loads and speeds by the increased fuel
flow.

10.21.4  Lubricating System for IC Engines


Various lubricating systems used for internal combustion engines may be classified as follows:
1. Mist lubrication system: This system is used for two-stroke cycle engines. Most of these engines
are crank-charged, that is, they employ crankcase compression and thus, are not suitable for
crankcase lubrication.
Such engines are lubricated by adding 2–3% lubricating oil in the fuel tank. The oil and the fuel
mixture is inducted through the carburettor. The gasoline is vapourised; and the oil, in its form of
mist, goes through a crankcase into the cylinder. The oil which impinges on the crankcase walls
lubricates the main and connecting and bearing, the rest of the oil which passes on to the cylinder
during charging and scavenging periods, lubricates the piston, piston rings, and the cylinder.

M10_THERMAL ENGINEERING_SE_XXXX_CH010(Part-III).indd 579 6/15/2018 5:00:46 PM


580 Chapter 10

The two-stroke engine is very sensitive to a particular oil and fuel combination. The composi-
tion of fuels and lubricants used influence the exhaust smoke, internal corrosion, bearing life, ring
and cylinder bore wear, ring sticking, exhaust and combustion chamber deposits, and one of the
most irritating and difficult problem of spark plug fouling and whishering. Therefore, specially
formulated ashless oils are used for two-stroke engines.
The fuel/oil ratio used is also important for good performance. A fuel/oil ratio of 40 to 50:1
is optimum. Higher ratios increase the rate of wear and lower ratios result in spark plug fouling.
The main advantage of this system is simplicity and low cost because no oil pump, filter, etc.,
are required. However, this simplicity is at the cost of many troubles some of which are enumer-
ated as follows:
(a) Some of the lubricating oil remains invariably in combustion chamber. This heavy oil when
burned, and still worse, when partially burned in the combustion chamber, leads to heavy
exhaust emissions and formation of heavy deposits on the piston crown, ring grooves, and
the exhaust port which interferes with efficient engine operation.
(b) One of the main functions of the lubricating oil is protection of anti-friction bearings, etc.,
against corrosion. Since the oil comes in close contact with acidic vapours produced during
the combustion process, it rapidly loses its anti-corrosion properties resulting in corrosion
damage of bearings.
(c) For effective lubrication, oil and the fuel must thoroughly mixed. This requires either sepa-
rate mixing prior to use or use of some additive to give the oil good mixing characteristics.
(d) One important limitation of this system is oil starvation of the working parts, especially
when the throttle is closed on a descent on a long hill. A closed throttle means no fuel, and,
hence, no oil. The prolonged absence of oil so produced may result in overheating and piston
seizure. This oil starvation can be controlled if the driver periodically releases the throttle to
replenish for the complete absence of oil while descending a hill.
(e) Due to high exhaust temperature and less efficient scavenging, the crankcase oil is diluted.
In addition, some lubricating oil also burns in the combustion chamber. This results in about
5–15% high lubrication consumption for two-stroke engines as compared to four-stroke
engines of similar size.
(f) Since there is no control over the lubricating oil, once introduced with fuel, most of the two-
stroke engines are over-oiled most of the time.
Some manufacturers use a separate oil injection pump to inject the oil directly into the carburettor
and the amount of the oil is regulated according to engine load and speed. This system completely
avoids the oils starvation problem discussed above. The oil consumption is also reduced. This
results in less deposits and less spark plug fouling problems. Since in this system the main bear-
ings are excluded from the crankcase and instead receive lubricating oil from a separate pump,
the corrosion damage of bearings is also eliminated.
2. Wet sump lubricating system: The wet sump lubricating system is shown in Fig. 10.73. In this
system, the sump is always full of oil. Oil is drawn from the sump by an oil pump through an oil
strainer. A pressure relief valve is provided which automatically maintains the delivery pressure
constant. If the pressure exceeds the predetermined pressure, the valve opens and allows some
of the oil to return to the sump and relieves the oil pressure in the system. The oil from the pump
goes to the bearings and part of it passes through the filter which removes solid particles from the
oil. As all the oil is not passed through the filter, the system is known as by-pass filtering system.
The advantage of this system is that a clogged filter will not restrict the flow of oil to the engine.

M10_THERMAL ENGINEERING_SE_XXXX_CH010(Part-III).indd 580 6/15/2018 5:00:47 PM


Internal Combustion Engine Systems 581

3. Dry sump lubricating system: In the dry sump lubricating system as shown in Fig. 10.74, the
sump does not contain oil and all the oil required for lubrication remains in circulation. High
speed racing cars and military jeeps use this type of system.
An auxiliary tank is used to supply the oil to the main bearings with the help of the pump.
The oil returns to the tray and is then returned to the auxiliary tank by the scavenging pump. If
the filter gets clogged, the pressure relief valve opens, permitting oil to flow by passing the filter
and reaches the supply tank. The oil is then recirculated to the bearings from the supply tank. A
separate oil cooler is used to cool the oil.

10.21.5  Lubrication of Different Engine Parts


1. Lubrication of main bearings: The main bearings are lubricated satisfactorily with the help of
a ring or a chain-type feeder. The arrangement of the system is shown in Fig. 10.75.
The oil ring rests on the main shaft where a small portion of the main bearing shell has been
cut away as shown in the figure. The lower end of the oil ring is allowed to submerge in the oil bath
as shown in Fig. 10.73. The oil ring rotates with the main shaft and carries the oil from the oil bath
to the bearing and is distributed to the bearing through the oil groove. The surplus oil flows to the
ends of the bearing and drops back into the oil bath. Chains or more rings are provided instead of
single ring to carry more oil.

Oil filter
Bypass

Pressure
Oil to regulator
bearing
Breather

Oil pump

Wet
Engine
sump
crankcase
Oil strainer

Drain plug

Figure 10.73  Wet sump lubricating system

M10_THERMAL ENGINEERING_SE_XXXX_CH010(Part-III).indd 581 6/15/2018 5:00:47 PM


582 Chapter 10

Oil cooler Pressure relief valve Vent

Oil
bearings

Supply
Oil pump tank

Pressure relief valve

Dry sump

Strainer Filter
Scavenging
pump

Figure 10.74  Dry sump lubricating system

Oil ring

Main shaft
Main
bearing
Oil bath

Figure 10.75  Ring lubrication

This type of lubrication is more useful for medium speed engines because at high speeds the
oil will be thrown off due to centrifugal force but at low speeds, the amount of oil carried is not
sufficient.
2. Lubrication of cylinder and small end bearing of connecting rod: The cylinder and a
small end bearing (gudgeon pin) are lubricated with the help of side feed lubricator as shown
in Fig. 10.76.
The oil is supplied to the surface of piston through the hole provided in the cylinder wall. The
oil falling on the surface of the piston is spread over the surface of the cylinder walls.
3. Lubrication of crank and gudgeon pin: The scoops are provided to the caps of big-end-bearing
of the connecting rods as shown in Fig. 10.76. These scoops dip into the splash toughs when the
pistons are at BDC and splashes the oil in all directions with the rotation of cranks. A thin mist of
splashed oil settles on the surface of the cylinder and on the other parts which are to be lubricated.

M10_THERMAL ENGINEERING_SE_XXXX_CH010(Part-III).indd 582 6/15/2018 5:00:48 PM


Internal Combustion Engine Systems 583

Oil
Cylinder cup

Connecting
rod

Piston Gudgeon pin

Figure 10.76  Side feed lubricator

The main bearings and cam shaft bearings are provided with small pockets and the oil splashed by
scoops is collected in those pockets and is used for lubrication. The surplus oil falls back into the
oil sump.
Many times, a through hole is provided in the connecting rod and it is connected with the
scoop; therefore, some oil is supplied to the crank pin and through the hole to the gudgeon pin.
The oil level in the splash tough is maintained with the help of oil pump. The arrangement of
scoop with connecting rod is shown in Fig. 10.77.
4. Lubrication of big end (crank pin) and small end (Gudgeon pin) of connecting rod: The
crank pin and gudgeon pin are lubricated by ring as shown in Fig. 10.78.
The arrangement of the system used is shown in Fig. 10.78. In this system, the oil is supplied to the
ring from the oil feed pump which is fitted concentric on the crank shaft. The centrifugal force is given
to the oil as the oil ring rotates with the crank shaft and the lubricating oil is forced to the crank pin
and to the gudgeon pin through the holes provided in the crank pin and connecting rod as shown in
Fig. 10.77. This system of lubrication is known as ring lubrication.

Connecting
Crank rod

Scoop

Oil pan
Over flow
pipe

Figure 10.77  Scoope used in splash lubrication

M10_THERMAL ENGINEERING_SE_XXXX_CH010(Part-III).indd 583 6/15/2018 5:00:49 PM


584 Chapter 10

Gudgeon pin

Connecting rod
with inner hole
Crank

Oil ring
Crank pin

Shaft
Oil from pin

Figure 10.78  Lubrication system for big end of connecting rod

10.22  ❐  NECESSITY OF IC ENGINE COOLING


The cooling is provided to avoid the effects of overheating as follows:
1. The high temperature reduces the strength of the piston and piston rings and uneven expansion of
cylinder and piston may cause the seizure of the piston.
2. The high temperature may cause the decomposition of the lubricating oil and lubrication between
the cylinder wall and piston may break down, resulting in a scuffing of the piston.
3. If the temperature around the valve exceeds 250°C, the overheating of the valves may cause the
scuff of the valve guides, due to lubrication break down. Further increase in temperature may
cause the burning of valves and valve seats.
4. The tendency of the detonation increases with an increase in temperature of the cylinder body.
5. The pre-ignition of the charge is possible in spark ignition engines if the ignition parts initially are
at high temperature.

10.22.1  Types of Cooling Systems


There are two types of cooling system as per the coolant used—air cooling and water cooling system.

Air Cooling System


In this system, air is used as a cooling medium and it is used for small capacity engines. Earlier, it was
used for big capacity air-craft engines as water cooling was not practically possible as the weight of
water cooling system is very high compared with air-cooling system.
As mentioned earlier, the heat transfer coefficient for air-cooling is very low. This heat transfer
coefficient can be increased further by using the forced flow of air over the engine surface as done in
aero-engines. The heat transfer coefficient with air-cooling with forced circulation is also considerably
lower (50 W/m2-K) when compared with water cooling system. The other method of increasing the
heat transfer rate from the cylinder’s surface is to increase the surface area by providing the fins. The
use of fins increases the heat transfer surface area by 5 to 10 times of its original value. The air-cooled

M10_THERMAL ENGINEERING_SE_XXXX_CH010(Part-III).indd 584 6/15/2018 5:00:50 PM


Internal Combustion Engine Systems 585

systems, the forced circulation of air with increased surface area by using the fins, is commonly used
in practice for aero-engines and motor cycle engines.
The sectional view of an engine-cylinder with fins is shown in Fig. 10.79. More fins are used near the
exhaust valve and cylinder head where the possibility of occurrence of maximum temperature exists.
The cooling fins are either cast integral with the cylinder and cylinder head or they are fixed to the
cylinder block separately. The number of the fins used are two to three in case of cast fins and four to
five in case of machined fins per centimetre. The height of the fin depends on the type of material and
the manufacturing process used. Generally, the height of the fin used lies between 2 cm and 5 cm and
the fin spacing is limited to 2.5 mm.
Advantages of Air Cooling
1. Water jacket, radiator and water pump are not required. This reduces the size of the engine and its
weight.
2. Simpler engine design.
3. Less sensitive to climatic conditions engine.
4. Due to small thermal losses, the specific fuel consumption is low.
5. Better warm-up performance results in low wear of cylinder.
6. Sustained engine performance due to reduced carbon deposit on combustion chamber.
7. Easier control of cooling systems compared to water-cooled engines.
8. An air-cooled engine can take up some degree of damage.
Disadvantages of Air Cooling
1. Air fan creates noise.
2. The volumetric efficiency is lower due to higher cylinder head temperature.
3. High specified output engines cannot be air-cooled due to the complex nature of the fins that are
required.
4. Air cooling results in higher engine temperature. This necessitates the provision of bigger clear-
ances between the various parts of the engine.
Exhaust
valve

Inlet
valve Combustion Fins
chamber

Fins

Cylinder

Figure 10.79  Air cooling of engine cylinder

M10_THERMAL ENGINEERING_SE_XXXX_CH010(Part-III).indd 585 6/15/2018 5:00:51 PM


586 Chapter 10

Water Cooling System


The water cooling system is further subdivided into two groups:
1. Thermo-siphon system: The arrangement of the system is shown in Fig. 10.80. The force (pres-
sure head) required to circulate the water through the system is the difference in pressure head due
to hot and cold water. This force is given by
F = h(wc – wh) = h × ∆w.
where h = Height of the radiator tubes through which the water is circulated.
wc = Weight density of cold water.
wh = Weight density of hot water.
The difference in density is limited as the rise in temperature of the water passing through the
engine is limited. The rate of circulation is less as the force causing the flow of water is limited.
This cannot be used for heavy duty engines as the heat carrying capacity of this system is limited.
The water as passed through the radiator is cooled by the flow of air passed over the radiator tubes
by the cooling fans. The cooled water rises to the cylinder jacket; takes the heat from the cylinder
wall and then it enters into the radiator from the top header and comes down. It is cooled as it is
passed through the radiator tubes.
The limitations of this system are as follows:
(a) The engine should be placed as low as possible in relation to the radiator as the force causing
the flow is limited by the temperature difference of hot and cold water.
(b) The water level in the system should not fall below the level of the delivery pipe; otherwise,
the circulation of water in the system will stop. This causes the boiling of water and for-
mation of steam, resulting in further loss of water which may damage the engine in a short
period.
(c) The use of this system is recommended for small capacity engines.
(d) The main drawback of this system is that cooling depends only on the temperature and is
independent of engine speed. The rate of circulation is slow.
Due to these limitations, this system is rarely used at present.

Cooling fan
Top
header

Radiator
Cylinder
jacket Cooling fins

Bottom
Hose connections header

Figure 10.80  Thermo iphon system

M10_THERMAL ENGINEERING_SE_XXXX_CH010(Part-III).indd 586 6/15/2018 5:00:51 PM


Internal Combustion Engine Systems 587

Fan

Air
Engine flow

Radiator

Pump

Figure 10.81  Forced pump system


2. Forced pump system: In a thermo-siphon system, the height of the radiator should be kept above
the engine as the flow of water takes place by natural circulation.
To avoid this limitation, a pump is incorporated in the system for water circulation. The
arrangement of the system is shown in Fig. 10.81. The pump is driven from the engine shaft with
the help of the belt.
The major advantage of this system is effective and positive cooling of all the parts of the
engine. It can easily take the overload as the engine speed increases, water circulation is also
increased and same effective cooling can be maintained by this system.
Advantages of Forced Circulation System
1. The increased rate of coolant flow improves the heat transfer between the gases and water. This
reduces the quantity of water to be circulated and thereby reduces both the bulk and weight of the
engine because smaller coolant passages can be used.
2. The high temperature areas such as spark plug; exhaust valve seats and exhaust ports can be
effectively cooled and reduces local overheating.
3. The formation of steam pockets in critical areas that restrict the coolant circulation and hence the
heat transfer, can be avoided by maintaining the pressure in the system.
4. Increased coolant velocities allow a higher mean temperature difference between the water and
the air passing through radiator matrix. This reduces the size of the radiator.
5. When the vehicle is operated in mountainous regions, loss of water through boiling may occur
in thermo-siphon system is used because the boiling temperature of water decreases with the
decrease in atmospheric pressure at high altitudes.
Disadvantages of Forced Circulation System
1. There is a possibility of overcooling as the cooling is independent of temperature.
2. There is a possibility of overheating the engine when the automobile is climbing a hill where the
fuel burning rate is increased and water flow rate is reduced as part of the head is lost to overcome
the inclination of the road.
3. As soon as engine stops, the cooling also stops. This is undesirable because the cooling should
continue for some time till the temperatures are brought to normal values.

Cooling With Thermostatic Regulator


An important feature of the cooling system of present-day cars is the inclusion of a thermostat. This is
generally fitted in the upper radiator hose and its purpose is to prevent the circulation of water through

M10_THERMAL ENGINEERING_SE_XXXX_CH010(Part-III).indd 587 6/15/2018 5:00:52 PM


588 Chapter 10

radiator until the cylinder block and head have reached a temperature of about 75°C. If the water cir-
culation starts immediately as the engine has been started (as in the absence of thermostat), the engine
cannot run efficiently when all the water in the cooling system has been raised to a temperature near
the boiling point (90°C). By preventing circulation before the engine is warm, the engine gets warm
much more quickly. In consequence, there is less danger of the oil being washed away from the cylin-
der walls by wet petrol drawn into the cylinders while the engine is cold. Less petrol is wasted while
the engine warms up and the car can be driven more quickly after the engine has been started. The
arrangement of this system is shown in Fig. 10.82.
The thermostat consists of a valve attached to a bellows containing volatile liquid such as ether.
Heating of bellow by the water around them causes vapourisation of the liquid, expansion of the bel-
lows, and therefore, opening of the valve.
When the forced circulation is used, it is necessary to use a bypass pipe from the engine side of the
thermostat (as shown in Fig. 10.80) to the lower water hose so that the circulation pressure of the water
cannot force to open the valve before the correct temperature has been reached.

Radiator cap
Header
tank

Thermostat

Engine

Air
flow

Fan

Radiator
hose
Bottom
Drain
tank
tap

Figure 10.82  Thermo-syphon cooling system for IC engines

M10_THERMAL ENGINEERING_SE_XXXX_CH010(Part-III).indd 588 6/15/2018 5:00:53 PM


Internal Combustion Engine Systems 589

Evaporative Cooling System


This is not preferred until there is acute shortage of cooling water. The arrangement is shown in
Fig. 10.83. In this system, the water coming out of engine jacket is allowed to be heated to 100°C. This
method of cooling utilises the latent heat of water to obtain cooling with minimum water. The cooling
circuit is such that the coolant is always maintained in liquid form but the steam formed is flashed off
in a separate vessel as shown in Fig. 10.83. This system is generally used for big-capacity stationary
engines used for power generation (2 MW).
Advantages of Water Cooled Engines
1. The engine design is compact due to high heat transfer coefficient of water.
2. Due to high latent heat of water, overheating troubles are eliminated.
3. The engine can be installed anywhere in the vehicle.
4. The volumetric efficiency is higher than air-cooled engine.
Disadvantages of Water-cooled Engines
1. The need for a radiator and a pump increases the weight and dimensions of the engine.
2. It requires more maintenance.
3. The engine performance is more sensitive to climatic conditions.
4. The warm-up performance is poor resulting in greater cylinder wear.
5. Pump absorbs slightly higher power than fan.

Open and Closed System


In open this system, the hot water coming out from the engine jacket is not circulated through the
pump directly but it is cooled either in cooling tower or spray pond as shown in Fig. 10.84.

Vents in cap

Engine
Steam
Block

Water
Pump

Figure 10.83  Evaporative cooling system

Spray
80°C

Engine
Control valve Spray pond
30°C
Pump

Figure 10.84  Open cooling system

M10_THERMAL ENGINEERING_SE_XXXX_CH010(Part-III).indd 589 6/15/2018 5:00:54 PM


590 Chapter 10

With the help of a spray, the water is cooled by partly evaporating the water (1%) and absorbing the
latent heat required from the remaining water. This evaporated water is carried by the air. The effective
cooling of water depends upon the relative humidity of the surrounding area. This system is generally
used where more than adequate water is available.
In closed system, the hot water coming out from the engine is cooled in the radiator, heat exchanger,
or cooling tower. In this system, the same water is used repeatedly without exposing to the air directly
as in the case of an open system.
Impurities in water and the consequent danger of scale formation are major problems in an open
system. Due to water evaporation, which does not carry impurities, the concentration of salt in the
water after every pass increases and danger of scaling also increases. In closed system, the hot water
cooling methods are shown in Figs 10.85(a) to (c).
In a closed system, the same water remains in the system and recooled repeatedly. If it is pure at the
start, it remains pure throughout the period of use and there is no danger of scaling.

Engine Air
block Fan out

Pump
(a)
Raw water
out

Engine
block Heat
exchanger

Pump

(b)

Heat Natural draught


Engine
exchanger cooling tower
block

Pump

(c)

Figure 10.85 Closed cooling systems: (a) Closed system (radiator type),


(b) Closed system (heat exchanger) using raw water,
(c) Closed system using cooling tower

M10_THERMAL ENGINEERING_SE_XXXX_CH010(Part-III).indd 590 6/15/2018 5:00:55 PM


Internal Combustion Engine Systems 591

Advanced Cooling System


There are two main disadvantages with the conventional cooling systems. First, large volume of cool-
ant in the primary circuit can lead to a slow engine warm up and second, the cooling system tends
to overcool the engine parts when the engine is running at part load conditions. To overcome these
difficulties, advanced cooling systems are used.
The main purpose of these systems is to maintain more uniform temperature and less sensitive to
the load and speed of the engine. The friction loss between the piston and the cylinder is reduced if the
liner temperature is raised. The significance of reduction in friction loss increases as load is reduced
because this leads to reduction in fuel consumption. A rise in liner temperature may increase the emis-
sion of NOx. This can be reduced either by retarding the ignition timing or by using exhaust gas recir-
culation (EGR). Both these remedies impose a fuel consumption penalty and the fuel saving gained by
reducing the frictional loss. The NOx emission is problematic in diesel engine at full load conditions.
With SI engines, another problem that can occur at full load is the reduction in the knock margin
caused by higher cylinder temperatures. This also can be remedied by EGR or retarding the ignition
timing.
A few methods used for this purpose are described in brief.

10.22.2  Precision Cooling


It is developed for cooling diesel engines and to provide cooling only where it is needed; at a rate
proportional to the local heat flux.
A precision-cooled system has small local passages which are used to cool critical regions such as
injector nozzle, exhaust valve region, and valve guides, with much space in the head filled with air.
With precision cooling, many regions are not directly cooled and their temperatures rise, but the
system is designed to keep the temperature within safe limits. The corresponding benefit is a more
even temperature distribution, producing less thermal strain and possible elimination of hot spots to
allow higher compression ratios in SI engine.

10.22.3  Dual Circuit Cooling


In this process of cooling, there are two separate cooling circuits—one for the head and the other for
the cylinder block. The higher block temperature reduces friction losses and reduces fuel consump-
tion, particularly at part loads. The lower coolant temperature in the cylinder head tends to reduce the
indicated thermal efficiency but the risk of knock is reduced in SI engines and higher compression
ratio can be used. The overall effect is an improved efficiency.
Increasing the coolant temperature passing through cylinder block reduces the fuel consumption
and hydrocarbon emissions but increases NOx emissions.

10.22.4  Disadvantages of Overcooling


Should we cool the engine as much as we can? No. The engine must never be overcooled. The engine
must always be kept sufficiently hot to ensure smooth and efficient operation. Extremely low engine
temperatures may be difficult to start and above all, the low temperature corrosion assumes such a

M10_THERMAL ENGINEERING_SE_XXXX_CH010(Part-III).indd 591 6/15/2018 5:07:04 PM


592 Chapter 10

0.4

Water mm/1000 km
0.3

0.2

0.1

0
40 60 80 100 120 140 160
Cylinder temperature °C

Figure 10.86  Effect of temperature on corrosion

significant magnitude (see Fig. 10.86) that the engine life is greatly reduced. At low temperatures, the
sulphurous and sulphuric acids resulting from combustion of fuel (fuel always contains some sulphur)
attack the cylinder barrel. The dew points of these acids vary with pressure, and hence, the critical
temperature, at which corrosion assumes significant properties, varies along the cylinder barrel. To
avoid condensation of acids the coolant temperature should be greater than 70°C. Thus, the cooling
system should not only cool but must also keep the cylinder liner temperature above a minimum level
to avoid corrosion and ensure good warm up performance of the engine.

10.23  ❐  ENGINE RADIATORS


The function of the radiator is to cool hot water coming out of the engine with the help of air. In the
radiator, the heat transfer from hot water to the air takes place by conduction and convection.
The cooling effect in the radiator is enhanced by providing fins over the surface of tubes carrying
the hot water to be cooled. The fins are also so arranged that some turbulence is generated in air pas-
sages which further help to increase heat transfer coefficient to the air side.
A commonly used cross-flow type radiator is shown in Fig. 10.87. The fins are not shown.

10.23.1  Radiator Matrix


Figure 10.87 shows four different types of matrices (fins) commonly used in practice.
In the tube and fin type shown in Fig. 10.87(a), a series of long tubes extending from top to bottom of
the radiator are surrounded by simple straight metallic fins. The matrix has greater structural strength.
The improvement over the tube and fin type is the tube and corrugated fin type matrix as shown
in Fig. 10.87(b). The water tubes are made of an oval-shape section and zig-zag copper ribbons are
used to provide secondary heat transfer areas and air turbulence. This combines the good heat transfer
characteristics of film type matrix and structural strength of the tube and fin type matrix.
Figure 10.87(c) shows the film-type matrix which is also known as the ribbon-cellular matrix. It
consists of a pair of thin metal ribbons soldered together along their edges so as to form a water-way
running from header tank to the bottom tank of the radiator. The zig-zig copper ribbons forms an air
passage. In this case, the metal surface area is significantly increased and higher turbulence is also
created.

M10_THERMAL ENGINEERING_SE_XXXX_CH010(Part-III).indd 592 6/15/2018 5:07:04 PM


Internal Combustion Engine Systems 593

Fins

Fins

(a) (b)

(c) (d)

Figure 10.87 Different types of radiator matrices: (a) Plate type matrix, (b) Ribbon cellular
type matrix, (c) Corrugated fin matrix, (d) Honey comb matrix
Earlier, radiators used honey-comb matrix as shown in Fig. 10.87(d). The hexagons were packed
in contact and bound by solder. This provided a continuous water passage between the circular parts
of the tube. The cooling air was passed through the circular tubes. This type of matrix is rarely used
in present types of radiators.
In all the aforementioned matrices, the air flow was in cross-flow (perpendicular to the paper).
The materials used for matrices must have the following properties:
1. It must have good conductivity (Cast iron = 60 W/m-K, Aluminium = 210 W/m-K and Copper =
400 W/m-K).
2. The material must have good corrosion resistance.
3. It must possess the required strength.
4. It must be easily formable.
5. It must be easily available at a reasonable cost.
Yellow brass and copper meet all these requirements and are widely used.

10.23.2  Water Requirements of Radiator


The quantity of cooling water (Q) required to be circulated is given by
K × BP
Q=
( ∆T )

M10_THERMAL ENGINEERING_SE_XXXX_CH010(Part-III).indd 593 6/15/2018 5:07:07 PM


594 Chapter 10

10

required (m3/kW-min)
Petrol
8

Cooling air
2-Stroke diesel
6
4-Stroke diesel
4

0 50 100 150 200 250 300


Power (KW)

Figure 10.88  Variation of amount of air required with power for air-cooled radiator

where (∆T) is permissible rise in temperature and the value of K depends upon specific fuel consump-
tion and compression ratio.
The cooling water carries 4000 kJ/kWh for large engines and 550 kJ/kWh for small engines. If the
rise in temperature is limited to 10–15°C, the outlet cooling water temperature is limited to 60°C for
small or medium engine and 80°C for automobile engines.
Figure 10.88 shows the amount of air required for air-cooled radiator. It is clear from the graph,
petrol engine requires much more air than diesel engine.
The power required for cooling the engine is given by
Ae ( IP )2.25
Pcooling =
( A f ⋅ ∆T f )5.75 ra2
where Af  and ∆Tf  are the fin area and temperature difference between the fin average surface temper-
ature and air temperature, ra is air density, and Ae is effective area.
The equation is developed with following assumptions.
1. A:F ratio supplied to engine is constant.
2. The temperature change in fin is assumed small.
3. The density and temperature of the cooling media are assumed to be unaffected by its flow through
the radiator fin.

10.23.3 Fans
As mentioned earlier, an engine driven fan is mounted behind the radiator to increase the air flow
over the radiator tubes. This increases the air-side heat transfer coefficient and ultimately overall heat
transfer coefficient.
A radiator fan is usually made of pressed steel blades, four or six in number depending upon the
size of the radiator (capacity of engine). The power consumed by the fan depends upon the amount of
air handled, diameter of the fan, its shape, and speed of the engine.
Figure 10.89 shows the effect of speed and diameter of fan on the quantity of air flow. Figure 10.90
shows the effect of fan diameter and air flow on the power consumed by the fan. For the given flow
rate of air, the power required decreases slightly as the fan diameter increases, whereas for the same
engine speed, the power required increases with the fan diameter.

M10_THERMAL ENGINEERING_SE_XXXX_CH010(Part-III).indd 594 6/15/2018 5:07:09 PM


Internal Combustion Engine Systems 595

240
Speed = 3000 rpm

Air flow (m3/min)


s)
180 lade
(6–b
s)
lade
120 (4–b
s)
lade
(2–b
60

0
35 40 45 50 55
Fan diameter, cm

Figure 10.89  Variation of diameter of fan on the quantity of air flow

1200

900 pm
Power (w)

120m 3
00r /min
30 100m 3
600 /min
80m 3 rpm
60m 3 /min
/mi 3 000
300 n

0
35 40 45 50 55
Fan diameter, cm

Figure 10.90  Effect of fan diameter and air flow on the power consumed

10.24  ❐  COOLING OF EXHAUST VALVE


In the liquid cooled engines, the spaces around the exhaust valve seating should be sufficiently large
for the coolant to flow. Sometimes, the water circulation provides jets of water to be directed on the
valve seating metal.
An efficient valve cooling requires methods of conducting the heat from the valve head to the cyl-
inder, improved guides and metal, also to the lower part of the valve stem operating under cooler air
conditions.
In order to enhance the conductivity of the exhaust valve, the practice of making the valve stem and
even the heads hollow is becoming popular. The hollow stem is filled with sodium which melts at 97°C
and boils at 880°C. Thus, at valve operating temperatures, the valve is filled with conducting material.
Such a valve is shown in Fig. 10.91 with a tapered plug seal driven into the hollow stem. The end of the
stem is provided with a Stellite button, fused on the seat to prolong the wearing life of stem and end.

M10_THERMAL ENGINEERING_SE_XXXX_CH010(Part-III).indd 595 6/15/2018 5:07:11 PM


596 Chapter 10

Stellite
button

Plug

Hollow
stem
Sodium

Stellite

Figure 10.91  Cooling of exhaust valve

10.25  ❐  GOVERNING OF IC ENGINES


Governing is the process of varying the fuel supply to the engine in accordance with the load demand
so that the engine runs at practically constant speed. There are four methods of governing of IC
engines.
1. Hit and miss governing: This method is used for small engines and is illustrated in Fig. 10.92.
The rotational motion of the cam C actuates the rocker R through the roller D. The roller carries
a pecker G which strikes against the pecker block H and lifts the valve V against the spring S.
When the speed of the engine becomes excessive, the pecker block is lifted by the rod E as the
sleeve of the governor rises up. The pecker is now unable to strike against and lift the valve off its
seat and the engine performs an idle cycle because no fuel is now being supplied. This method of

E Sleeve

V G
H
S J
R

C D

Figure 10.92  Hit and miss governing

M10_THERMAL ENGINEERING_SE_XXXX_CH010(Part-III).indd 596 6/15/2018 5:07:12 PM


Internal Combustion Engine Systems 597

governing is quite simple but owing to the violent explosions which usually occur as a result of
extra scavenging which take place immediately after a mixed explosion, produces uneven turning
moment necessitating the use of heavy flywheel which increases the friction at the bearings and
lowers the mechanical efficiency of the engine.
2. Qualitative governing: In this method, a centrifugal governor is used to control the supply of
fuel, whereas the supply of air remains constant. Thus the quality of air-fuel mixture is altered.
This method is widely used in all heavy oil engines.
3. Quantitative governing: In this method, the quantity of air-fuel mixture flowing into the cylinder
is varied. This may be done by decreasing the lift of the inlet valve or by throttling the mixture
before it is made to enter the cylinder. This air-fuel ratio of the mixture is kept constant. This gives
a more even turning moment and closer limits of speed variation. This method is widely used for
governing petrol and large gas engines.
4. Combination method: This method uses qualitative and quantitative methods simultaneously.

10.26  ❐  RATING OF SI ENGINE FUELS-OCTANE NUMBER


The tendency to knock depends on the composition of fuel as mentioned earlier. Fuels differ widely in
their ability to resist knock. The rating of a particular fuel is done by comparing its performance with
that of a standard reference fuel which is usually a combination of iso-octane and n-heptane.
Iso-octane offers great resistance to knock and is arbitrarily assigned a rating of 100 octane number,
n-heptane, a straight chain paraffin, on the other hand, detonates very rapidly therefore it is assigned
a rating of zero-octane number. It is possible to match the knocking tendency of all commonly used
fuels by blending these two reference fuels in different proportions.
The percentage of iso-octane by volume in a mixture of iso-octane and n-heptane, which exactly
matches the knocking intensity of a given fuel, in a standard engine under given standard operating
conditions, is termed as ‘octane number’ of the fuel. An octane number of 80 means that the fuel has
the same knocking tendency as a mixture of 80% iso-octane and 20% n-heptane by volume.
It has already been mentioned that the knocking tendency of an engine increases with increasing
compression ratio. The higher the octane number of the fuel, the greater will be its resistance to knock
and higher will be the compression ratio which may be used without knocking. The power output and
specific fuel consumption are functions of the compression ratio and therefore, these can be consid-
ered as a function of the octane number of the fuel. This fact indicates an importance of the octane
number rating of fuels for SI engines. Higher octane number fuels increase the efficiency of the engine
as high compression ratio can be used with higher octane number fuels.
The knocking tendency of the fuels used in aero-engines is more as they are supercharged engines;
therefore, it is necessary to use the fuels having the highest octane number (largest resistance to knock).
The fuels superior in anti-knock qualities to iso-octane (having octane number greater than 100)
are required. The addition of tetra-ethyl-lead (TEL) to gasoline produces marked an effect in reducing
the knock tendency of the fuel. The addition of various amounts of one of these compounds (TEL is
commonly used) to iso-octane produces fuels of greater anti-knock quality than iso-octane alone. The
anti-knock quality of fuels above 100 octane number in the USA is measured in terms of cc (cu-cm) of
TEL per US gallon of iso-octane. A fuel having an octane number of 110 means that that fuel has the
same tendency to knock as a mixture of 10 cc of TEL in 1 US gallon of iso-octane. The importance of
such fuels (commonly used in aero-engine) has diminished with an introduction of turbojet turbines
in aircraft.

M10_THERMAL ENGINEERING_SE_XXXX_CH010(Part-III).indd 597 6/15/2018 5:07:12 PM


598 Chapter 10

10.26.1  Anti-knock Agents


The knock resistance tendency of a fuel can be increased by adding anti-knock agents. The anti-knock
agents are substances which decrease the rate of preflame reaction by delaying the auto-ignition of
the end mixture in the engine until the flame generated by spark plug passes through the end mixture.
Among all, TEL [Pb(C2H5)4] is the most powerful antiknock agent. It has helped to improve the
efficiency of the engine and to increase the specific output of SI engines. Its use will not improve
the performance of the engine which is not knocking unless the spark is advanced, the compression
ratio is increased, or a higher inlet pressure is used to take the advantage of an increase in octane
number. Advancing the spark will improve the performance of the engine only if optimum spark for a
non-knocking fuel is not already used.
Some other anti-knock agents with their relative effectiveness are listed in Table 10.13.

Table 10.13  Anti-knock agents

Compound Chemical symbol Weight for given Relative weight


effect (gm)
Tetraethyl lead Pb (C2H5)4 0.0295 1
Aniline C6H5NH2 1.00 34
Ethyl iodide C2H5I 1.55 53
Ethyl alcohol C2H5OH 4.75 161
Xylene (C6H4CH3)2 8.00 271
Toluene C6H5CH3 8.8 298
Benzene C6H6 9.8 332

Knocking essentially results in an incomplete and non-uniform combustion of the fuel. It is caused
by the sudden and spontaneous ignition of the unburned mixture, ahead of the flame front advancing
through the combustion chamber. Partial oxidation reactions of the hydrocarbons in this mixture pro-
duce reactive intermediates that lead to such spontaneous ignition and overheating of the engine. This
overheating can result in the wearing of engine parts. Antiknock agents react with these intermediates
and reduce their ability to propagate the oxidation that causes knocking. Octane number of the gaso-
line is a measure of the effectiveness of the antiknock agent it contains. Higher this number lesser is
the knock. Table 10.14 shows some of these agents and their relative effectiveness.

Table 10.14  Relative effectiveness of antiknocks

Compound Relative effectiveness


Tetraethyl lead 100
Methyl cyclopentadienyl 65
Manganese tricarbonyl ironcarbonyl 43
Copper methylaminomethylenecetate 40
Nickelcarbonyl 30
Triethyl bismuth 20
Tetraethyltin 3
N-Methylaniline-Ethyliodide 11

M10_THERMAL ENGINEERING_SE_XXXX_CH010(Part-III).indd 598 6/15/2018 5:07:13 PM


Internal Combustion Engine Systems 599

Gasoline composition, methods of test, and antiknock concentrations affect both the absolute and
relative effectiveness of different compounds, even the same element. Moreover, some compounds
can function as an antiknock under some conditions but promote knock under other. Therefore, the
comparison of active elements in various compounds is representative but subject to considerable
variation.
The effectiveness of an element can vary greatly, depending on the specific compound consid-
ered. Some aromatic amines are not antiknocks while others are twice as effective as N-methylamine
(nitrogen in other forms is not effective). Most metallic compounds possessing the antiknock property
have metal to carbon bonds but there are exceptions. In case of organometallics, it is believed that
the products released by the thermal decomposition of the compound at an appropriate stage of the
precombustion process are the active antiknock species.
TEL is the most widely used (also known longest) of all antiknocks. A hydrogen atom in ethane
is substituted by the heavy metal atom of lead in this organometallic compound. The average TEL
usage, at present, in motor gasoline (2.0 gms lead per gallon in regular and 2.7 gms in premium)
usually takes up the octane number by 6 to 10. It is a mixture of tetra methyl and ethyl leads, and
the intermediate lead alkyls produced by these two are also commercially used as antiknocks. These
higher volatile alkyls offer advantages of better vapourisation and are the most effective in some
multicylinder engines. Concentrated and alkyls are hazardous to handle. Therefore, these antiknock
agents are blended with gasoline in refineries under same conditions; the resulting leaded gasoline can
be handled as a motor fuel safely. The use of leaded gasoline, however, is not a perfect solution to the
problem. It leads to the emission of lead into the atmosphere which is known to be hazardous.

10.26.2  Performance Number


The detonation tendency is also measured in terms of performance number. The performance number
(PN) is defined as
( KLIMEP)test-fuel
PN =
( KLIMEP)Iso-octane
where KLIMEP represents conception of knock limited indicated mean effective pressure.
The performance number is obtained for a specified engine, under a specified set of conditions by
varying the inlet pressure.
The PN method of rating enables to develop a scale beyond 100 octane number. There is a relation-
ship between performance number and Octane number as shown in Fig. 10.93.
It has already been mentioned that by adding TEL to iso-octane increases knock resistance ten-
dency. Figure 10.96 also shows the quantity of TEL by volume in millilitre per litre of gasoline on
X-axis for octane numbers exceeding 100 and performance number on y-axis.

10.27  ❐  HIGHEST USEFUL COMPRESSION RATIO


In the development of SI engines, it was of great interest to use higher compression ratio as the effi-
ciency and power output of the engine increase with an increase in compression ratio. The maximum
compression ratio of any SI engine is limited by its tendency to knock. Therefore, previously, the
rating of SI engine fuels was based on the highest useful compression ratio (HUCR) that could be
employed in a given engine under a given set of conditions. To find the highest compression ratio of a

M10_THERMAL ENGINEERING_SE_XXXX_CH010(Part-III).indd 599 6/15/2018 5:07:13 PM


600 Chapter 10

200 200

180 180

160 160

Performance number
140 140

120 120

100 100

80

60

40

20

0 20 20 20 20 100 0 1 2
mI TEL/Litre

Figure 10.93  Relationship between performance number and octane number


fuel, the compression ratio of the engine is increased (using a variable compression ratio engine) till
knocking became audible with certain temperature conditions and with mixture strength and ignition,
both adjusted to give highest efficiency. The HUCR of a few fuels determined by using Richardo E6
engine are listed in Table 10.15.

Table 10.15  HUCR for various fuels

Fuel Toluene Benzene Iso-octane Cyco-hextane M-heptane


HUCR 15 14.6 10.96 8.20 3.75

Through the measurement of HUCR of various fuels, a measure of their anti-knock value can be
obtained.
The effect of anti-knock agents on HUCR is shown in Fig. 10.94.

8 60
% increase in
HUCR
HUCR

7 40

6 20

5
0 0.1 0.2 0.3 0.4 0.5 0 0.1 0.2 0.3 0.4 0.5
% Addition of TEL % Addition of TEL
(a) (b)

Figure 10.94  Effect of anti-knock agents on HUCR

M10_THERMAL ENGINEERING_SE_XXXX_CH010(Part-III).indd 600 6/15/2018 5:07:14 PM


Internal Combustion Engine Systems 601

200 100
Diesel index
200 80

Boiling point
Aniline point

Diesel index
Viscosity

Viscosity
180 60 Aniline point

160 40
Boiling point
140 20

120
10 20 30 40 50 60 70 80 90 100
Cetone number

Figure 10.95  Variation of properties of fuel with cetane number

10.28  ❐  RATING OF CI ENGINE FUELS


Knocking is also encountered in CI engines, with an effect similar to that in SI engines, although it
is due to a different phenomenon. Knock in the CI engine is due to sudden ignition and abnormally
rapid combustion of accumulated fuel in the combustion chamber. Such a situation occurs because of
ignition lag in the combustion of fuel between the time of injection and actual burning. As the ignition
lag increases, the amount of fuel accumulated in the combustion chamber, before starting of combus-
tion, also increases. When combustion actually takes place, abnormal amount of energy is suddenly
released, causing an excessive rate of pressure rise which results in an audible knock.
A CI engine knock can be controlled by decreasing ignition lag. The shorter an ignition lag, there
is less tendency to knock. A good CI engine fuel, therefore, will ignite more readily, that is, it will
have short ignition delay. Furthermore, ignition lag affects the starting, warm-up, and production of
exhaust smoke in CI engines.
The property of ignition lag is generally measured in terms of cetane number. Cetane, a straight
chain paraffin with good ignition quality, is arbitrarily assigned a rating of 100 cetane number (CN). It
is mixed with alpha-methyl naphthalene, a hydrocarbon with poor ignition quality, which is assigned
zero CN. The mixture is matched with a fuel under test in standard (CFR) engine running under pre-
scribed conditions. The CN of the fuel is then defined as the percent by volume of cetane in a mixture
of cetane and alpha-methyl-naphthalene that produces the same ignition lag as the fuel being tested,
in the same engine and under the same operating conditions.
If a test fuel has a CN of 40, it means a mixture containing 40% cetane and 60% a-methyl naphtha-
lene by volume gives the same ignition delay as tested fuel.
The straight chain paraffins are the most suitable fuels for CI engines.
The graph shown in Fig. 10.95 shows the relationship of the other properties of the fuel with CN
which are also responsible for the knocking of the engine.

10.29  ❐  IC ENGINE FUELS


The desirable properties of IC engine fuels are as follows:
1. High energy density
2. Good combustion qualities

M10_THERMAL ENGINEERING_SE_XXXX_CH010(Part-III).indd 601 6/15/2018 5:07:15 PM


602 Chapter 10

3. High thermal stability


4. Low deposit forming tendencies
5. Compatibility with the engine hardware
6. Good fire safety
7. Low toxicity
8. Low pollution
9. Easy transferability on board vehicle storage

10.29.1  Fuels for SI Engines


Gasoline
It is a mixture of various hydrocarbons such as paraffins, olefins, napthalenes, and aromatics. The
requirements of an ideal gasoline are as follows:
1. It should mix readily with air and afford uniform manifold distribution, i.e., it should easily
vapourise.
2. It must be knock-resistant.
3. It should not pre-ignite easily.
4. It should not tend to decrease the volumetric efficiency of engine
5. It should be easy to handle.
6. It should be cheap and easily available.
7. It should not corrode engine parts.
8. Its calorific value should be high.
9. It should not form gum and varnish.

Knock Rating of SI Engine Fuels


Highest Useful Compression Ratio: The HUCR is the highest compression ratio at which a fuel
can be used without detonation in a specified test engine under specified operating conditions and the
ignition and mixture strength being adjusted to give best efficiency. The HUCR of different fuels is
given in Table 10.16.

Rating of SI Engine Fuels


The rating of fuel is done by comparing its performance with that of a standard reference fuel which is
usually a combination of iso-octane and n-heptane. Iso-octane offers great resistance to deformations
and is arbitrarily assigned a rating of 100-octane number. On the other hand, n-heptane detonates very
rapidly, and is assigned a rating of zero-octane number.

Octane Number
It is percentage of iso-octane by volume in a mixture is iso-octane and n-heptane, which exactly
matches the knocking intensity of a given fuel, in a standard engine under given standard operating
conditions.
The octane scale is extended beyond 100 by adding TEL to iso-octane.

M10_THERMAL ENGINEERING_SE_XXXX_CH010(Part-III).indd 602 6/15/2018 5:07:15 PM


Internal Combustion Engine Systems 603

Table 10.16  HUCR for different fuels

Fuel HUCR
Iso-octane 10.96
N-heptane 3.75
Toluene 15.00
Cyclohexane 8.20

10.29.2  Fuels for CI Engines


Diesel fuels are used for CI engines. These are petroleum fractions that lie between kerosene and
lubricating oils.
The main desirable characteristics of diesel fuels are as follows:
1. Cleanliness—carbon residue, contamination, sulphur, etc.
2. Ignition quality—cetane number
3. Fluidity—viscosity, pour point, etc.
4. Volatility—flash point and carbon residue

Rating of CI Engine Fuels


The rating of a diesel fuel is measured in terms of its cetane number. Cetane, a straight chain paraffin
with good ignition quality, is arbitrarily assigned a rating of 100-cetane number. It is mixed with a-me-
thyl naphthalene, a hydrocarbon with poor ignition quality, which is assigned zero-cetane number.

Cetane Number
This is defined as the percentage of volume of cetane in a mixture of cetane and a-methyl naphthalene
that produces the same ignition lag as the fuel being tested, in the same engine and under the same
operating conditions.

10.30  ❐  ALTERNATIVE FUELS FOR IC ENGINES


The various alternative fuels for IC engines are as follows:
1. Alcohols: ethanol and methanol
2. Hydrogen
3. Biogas
4. Producer (or water) gas
5. Biomass generated gas
6. LPG and LNG
7. CNG
8. Coal gasification and coal liquefication
9. Non-edible vegetable oils
10. Non-edible wild oils
11. Ammonia

M10_THERMAL ENGINEERING_SE_XXXX_CH010(Part-III).indd 603 6/15/2018 5:07:15 PM


604 Chapter 10

10.30.1 Alcohols
Alcohols are of two types—ethanol (C2H5OH) and methanol (CH3OH) which can be produced from
sugarcane waste and other agricultural products. Ethanol can be produced by the termination of veg-
etables and plant materials such as sugarcane, molasses, starchy matter, cellulose material (wood,
sulphite, waste liquor from paper manufacture), hydrocarbon gases, and so on.
The use of ethanol in IC engines gives better performance than on a gasoline engine. The power
output, BSHC, maximum thermal efficiency, and engine torque are higher as compared to petrol
engines. Alcohols are not suitable fuels for CI engines.
Methanol behaves much like petroleum. It can be used directly or mixed with gasoline. Some of the
important features of methanol as fuel are as follows:
1. The specific fuel consumption with methanol as fuel is 50% less than petrol engine.
2. Exhaust CO and HC are decreased continuously with blends.
3. Methanol can be used as supplementary fuel in heavy vehicles.
Methanol can be produced from coal, natural gas, oil shell, farm waste, etc. Methanol emits less
amount of CO2 and other polluting gases as compared to gasoline-fuelled vehicles. However, metha-
nol has high latent heat and gives cold starting problems, is more corrosive, and more toxic as com-
pared to petrol, gives high levels of formaldehyde emission. The important objectionable emissions
are CO, HC, NOx, and aldehydes.

10.30.2  Use of Hydrogen in CI Engines


There are two methods by which hydrogen can be used in CI engines as follows.
1. By introducing hydrogen with air and using a spray of diesel oil to ignite the mixture by the dual fuel
mode. The limiting conditions are when the diesel quantity is too small to produce effective ignition,
that is failure of ignition and when the hydrogen air mixture is so rich that the combustion becomes
violent. In between these limits, a wide range of diesel-to-hydrogen proportions can be tolerated.
2. By introducing hydrogen directly into the cylinder at the end of compression. Since the self-
ignition temperature of hydrogen is very high, the gas spray is made to impinge on a hot glow plug
in the combustion chamber by surface ignition. It is also possible to feed a very lean hydrogen air
mixture during the intake into an engine and then inject the bulk of the hydrogen towards the end
of the compression stroke.

Advantages of Hydrogen as IC Engine Fuel


1. Low emission: Essentially, no CO or HC in the exhaust as there is no carbon in the fuel. Most
exhaust would be H2O and N2.
2. Fuel availability: There are a number of different ways of making hydrogen, including electroly-
sis of water.
3. Fuel leakage to environment is not a pollutant.
4. High energy content per volume when stored as a liquid. This would give a large range vehicle
range for a given fuel tank capacity.

Disadvantages of Hydrogen as IC Engine Fuel


1. Requirement of heavy, bulky fuel storage both in vehicle and at the service station. Hydrogen can
be stored as a cryogenic liquid or as a compressed gas. If stored as a liquid, it would have to be

M10_THERMAL ENGINEERING_SE_XXXX_CH010(Part-III).indd 604 6/15/2018 5:07:15 PM


Internal Combustion Engine Systems 605

kept under pressure at a very low temperature. This would require a thermally super-insulated fuel
tank. Storing in a gas phase would require a high pressure vessel with limited capacity.
2. Difficult to refuel.
3. Poor engine volumetric efficiency. When a gaseous fuel is used in an engine, the fuel will displace
some of the inlet air and less volumetric efficiency will result.
4. Fuel cost would be high in the context of present–day technology and availability.
5. High NOx emissions because of high flame temperature.
6. Can detonate.

10.30.3 Biogas
Biogas is produced from organic waste (cow dung) which is easily available. The primary advantages
of biogas is its ability to operate the engine on lean mixture, which reduces exhaust HC and CO2
concentration in the engine exhaust. It gives less deposits and shows clean burning characteristics as
compared to petrol and diesel oil.
SI engines can be operated on biogas after starting the engine by using petrol. Biogas can be used
in SI engine in two forms as follows:
1. To run the engine entirely on biogas
2. Dual fuel engine operation on biogas and petrol
The advantages of using biogas as fuel in a CI engine are as follows:
1. The gas-air mixture provides uniform mixture in a multi-cylinder engine at all times.
2. There is virtually no CO emission in exhaust due to lean operation of the engine.
3. NOx emissions are reduced by about 60%.
4. Soot is virtually eliminated and exhaust is found to have less pungent odour than diesel oil.

10.30.4  Producer (or Water) Gas


Producer gas is a mixture of CO, H2, and N2. It is obtained from the partial oxidation of coal, or peat
when burnt with insufficient quantity of air. In using the producer gas in an SI engine, the problem
remains as to how and with which fuel it has to be produced economically. The producer gas can be
used with an SI engine but with a lesser output.

10.30.5  Biomass-generated Gas


Biomass comprises all kinds of agricultural waste such as stalks, husks of different crops, residues like
bagasse, and so on. Biomass gasification gives producer gas.

10.30.6  LPG as SI Engine Fuel


LPG consists of hydrocarbons of such volatility that they exist as gases under atmospheric conditions,
but can be readily liquefied under pressure. LPG contains mainly members of paraffin hydrocarbons
up to C4. It consists of propane and butane, to some extent.

M10_THERMAL ENGINEERING_SE_XXXX_CH010(Part-III).indd 605 6/15/2018 5:07:15 PM


606 Chapter 10

Advantages of LPG in SI Engine


1. LPG contains less carbon than petrol. Therefore, emission is much reduced by its use.
2. LPG mixes with air at all temperatures.
3. In multi-cylinder engines, a uniform mixture can be supplied to all cylinders.
4. There is no crankcase dilution because the fuel is in the form of vapour.
5. Automobile engines can use propane if they have high compression ratio.
6. LPG has high antiknock characteristics.
7. Its heat energy is about 80% of gasoline, but its high octane value compensates the thermal effi-
ciency of the engine.
8. Running on LPG translates into a cost saving of about 50%.
9. The engine may have a 50% longer life.

Disadvantages of LPG in SI Engine


1. Engines are normally designed to take in a fixed volume of the mixture and air. Therefore, LPG
will produce 10% less power for a given engine at full throttle.
2. The ignition temperature of LPG is somewhat higher than petrol. Therefore, running on LPG
could lead to a 5% reduction in valve life.
3. A good cooling system is quite necessary, because LPG vapouriser uses engine coolant to provide
the heat to convert the liquid LPG to gas.
4. The vehicle weight is increased due to the use of heavy pressure cylinders for storing LPG.
5. A special fuel feed system is required for LPG.
Liquefied natural gas (LNG) comes from dry natural reservoirs, mainly CH4 (methane) with a very
small percentage of ethane and propane. The major difficulty in using this gas is its very low boiling
point (–161.5°C)

10.30.7  Compressed Natural Gas


Compressed natural gas (CNG) is composed of methane, ethane, and propane with traces of other
gases. Methane is the main constituent. It has very good anti-knock qualities and burns at much higher
temperature as compared with petrol. It can be safely used in engines with a compression ratio up
to 12:1. It gives higher thermal efficiency as compared to gasoline engine. The pollutants emitted by
CNG are also reduced. It is nontoxic and has been widely used in three-wheelers and buses.

Advantages of CNG
1. It is cheaper than petrol.
2. It is engine friendly–gives lower maintenance cost and longer life.
3. It is safe–it is lighter than air and safely dissipates to atmosphere.
4. It mixes easily with air completely and gives proper combustion.
5. It is odourless.

10.30.8  Coal Gasification and Coal Liquefaction


These fuels are mainly used for power generation than IC engines.

M10_THERMAL ENGINEERING_SE_XXXX_CH010(Part-III).indd 606 6/15/2018 5:07:15 PM


Internal Combustion Engine Systems 607

10.30.9  Non-edible Vegetable Oils


The vegetable oils including soya bean oil, cotton seed oil, sunflower oil, rapeseed oil, palm oil, coco-
nut oil, and linseed oil have cetane numbers and calorific values comparable with those of diesel oil
and can be used as alternative fuels for diesel engine. The main disadvantage is their high viscosity
which gives difficulties with fuel injection and cold flow pumping.

10.30.10  Non-edible Wild Oils


Of all the non-edible wild oils, mahua and neem oils have the highest production. Their use as IC
engine fuels has not become popular yet.

10.30.11 Ammonia
Ammonia has been demonstrated as a practical fuel for engine applications. Ammonia does not con-
tain carbon and hence, there is an absence of two major pollutants, HC and CO, in the exhaust. These
are its added advantages. The performance characteristics of engine using NH3 as fuel are similar to
the performance characteristics obtained with petroleum fuels.
The pros and cons of alternative fuels are given in Table 10.17.

Table 10.17  Pros and cons of alternative fuels


Fuel Advantages Disadvantages
Methanol Familiar liquid fuel Range as much as half less, or larger fuel
Vehicle development relatively advanced tanks
Organic emission (ozone precursors) will have Would likely be imported from overseas
lower reactivity than gasoline emissions Formaldehyde emissions a potential prob-
Lower emissions of toxic pollutants, except lem, especially, at higher mileage, requires
formaldehyde improved controls
Engine efficiency should be greater More toxic than gasoline
Abundant natural gas feedstock M100 has non-visible flame, explosive in
enclosed tanks
Less flammable than gasoline
Costs likely somewhat higher than gasoline,
Can be made from coal or wood (as can gaso- especially during transition period
line), though at a higher cost.
Cold starts a problem for M100
Flex fuel ‘transition’ vehicle available
Greenhouse problem if made from coal.
Ethanol Familiar liquid fuel Much higher cost than gasoline
Organic emissions will have lower reactiv- Food/fuel competition at high production
ity than gasoline emission (but higher than levels
methanol)
Supply is limited, especially if made from
Lower emissions of toxic pollutants
corn
Engine efficiency should be greater
Range as much as 1/3 less, or larger fuel
Produced from domestic sources
tanks
Flex fuel ‘transition’ vehicle available
Cold starts a problem for E100
Lower CO with gasohol (10% ethanol blend)
Enzyme-based production from wood being
developed
(Continued)

M10_THERMAL ENGINEERING_SE_XXXX_CH010(Part-III).indd 607 6/15/2018 5:07:15 PM


608 Chapter 10

Table 10.17 (Continued )

Fuel Advantages Disadvantages


Natural Though imported, likely North American source Dedicated vehicles have remaining develop-
Gas for moderate supply (1 mmbd or more gasoline ment needs
displaced) Retail fuel distribution system must be built
Excellent emission characteristics except for Range quite limited, need large fuel tanks
potential of somewhat higher NOx emissions with added costs, reduced space (LNG range
Gas is abundant world-wide not as limited, comparable to methanol)
Modest greenhouse advantage Dual fuel ‘transition’ vehicle has moderate
Can be made from coal performance
slower refuelling
Greenhouse problem it made from cool
Hydrogen Excellent emission characteristics—minimal Range very limited, need heavy, bulky fuel
hydrocarbons storage
Would be domestically produced Vehicle and total costs high
Big greenhouse advantage if derived from pho- Needs new infrastructure
tovoltaic energy
Possible fuel cell use
Refor- No infrastructure change except refineries Emission benefits remain highly uncertain
mulated Probable small to moderate emission reduction Costs uncertain, but will be significant
gasoline
Engine modifications not required No energy security or greenhouse advantage
May be available for use by the entire fleet, not
just new vehicles

Summary for Quick Revision


1. An internal combustion engine is a device in which the fuel is mixed with air and burned inside the engine
to generate power.
2. The IC engines can be either of spark ignition or compression ignition type.
3. The scavenging process refers to clearing the cylinder after the expansion stroke with fresh-air charge.
Actual mean effective pressure
4. Diagram factor =
Theoretical mean effective pressuree
5. The process of mixing air and fuel, outside the engine cylinder, in proper proportions, is called carburetion.
6. For a simple carburettor:
(a) Approximate analysis:
(i) Mass of air per second, m a = Cda A2 2 ra ( p1 − p2 ) , A2 = area of venturi throat

(ii) Mass of fuel per second, m f = Cdf Af 2 r f ( p1 − p2 − zg r f )


Af = area of fuel nozzle, Cd = coefficient of discharge.
m f Cdf Af rf
(iii) For z = 0, = × ×
m a Cda A2 ra

M10_THERMAL ENGINEERING_SE_XXXX_CH010(Part-III).indd 608 6/15/2018 5:07:18 PM


Internal Combustion Engine Systems 609

(b)    Exact analysis:


1/2
  g −1  
   p2  g  
           (i) c2 = 2cp (T1 − T2 ) =  2c pT1 1 −   
   p1  
  
1/2
  2 (g −1) 
A p   p  g p  g 
        (ii) m a = 2 1  2c p  2  − 2 
R T1   p1   p1  
  
1/2
  2 (g −1) 
 2c  p2  g −  p2  g 
 p  p1  p 
1

m a Cda A p    
      (iii) = × 2 × 1 ×
m f Cdf Af R T 2 r f ( p1 − p2 − zg r f )
7. There are two mechanically operated petrol injection systems—combustion chamber injection and continu-
ous port injection. The first method is not being used nowadays.
8. Fuel injection systems for CI engine are air injection system and solid injection.
9. Fuel nozzle design:
BSFC × Power
(a) Fuel consumed per cycle, mf =
Cycles ⁄ h
q × 60
(b) Duration of injection in seconds, t =
N × 360
mf
(c) Mass of fuel supplied per second, m f = = Cdf  Af 2r f × ∆p
t

  
` l d 2n   q I  N
(d) Volume of fuel injected per second, Qf = Cdf  C × × ×
 f  360 N i  60 
 4  
 
where d = orifice diameter, n = number of orifices, q = crank angle duration, Ni = number of injections per
minute, N = rpm, Cdf = coefficient of discharge of fuel nozzle.
10. Fuel ignition systems are battery ignition system, magneto ignition system, and electronic ignition system.
11. Ignition lag (or delay) is the phase during which some fuel has already been admitted but has not been
ignited.
12. The phenomenon of detonation occurs in SI engines when the temperature of unburned mixture exceeds the
self-ignition temperature of the fuel during the ignition lag. This gives rise to spontaneous or auto-ignition
of fuel at various pin-point locations.
13. The phenomenon of pulsating variation of pressure due to sudden burning of accumulated fuel during the
delay period in CI engines is called knocking.
14. There are two types of lubrication systems—splash lubrication systems and pressure feed lubrication system.
15. There are mainly two types of cooling systems—air cooling system and water cooling system.
16. There are four methods used for governing of IC engines—hit and miss governing, qualitative governing,
quantitative governing, and combination methods.

M10_THERMAL ENGINEERING_SE_XXXX_CH010(Part-III).indd 609 6/15/2018 5:07:22 PM


610 Chapter 10

Multiple-choice Questions

1. An Otto cycle on internal energy (U) and entropy


List I List II
(s) diagram is shown in:
C. Open combustion 3. Spark ignition
(a) (b)
chamber
D. F-head combustion 4. Combustion induced
U U chamber
5. M-Chamber
Codes:
S S
A B C D
(c) (d)
(a) 4 5 3 2
(b) 1 3 5 2
U U
(c) 2 3 1 5
(d) 4 1 2 3
S S 5. Which of the following factors increase detona-
2. Which of the following statements is “true”? tion in the SI engine?
(a) The term “Knock” is used for on identical I. Increase spark advance
phenomenon in a spark ignition and compres- II. Increased speed
sion ignition engine. III. Increased air-fuel ratio beyond stoichiometric
(b) “Knock” is a term associated with a phenom- strength
enon taking place in the early part of combus- IV. Increased compression ratio
tion in a spark ignition engine and the later
Select the correct answer using the codes given
part of combustion in a spark ignition engine.
below:
(c) “Knock” is a term associated with a phenome-
non taking place in the early part of combustion Codes:
in a spark ignition engine and the later part of (a) I and III (b) II and IV
combustion in a compression ignition engine. (c) I, II, and IV (d) I and IV
(d) None of the above
6. Which one of the following curves is proper
3. The essential function of the carburettor in a representation of pressure differential (y-axis)
spark ignition engine is to vs velocity of air (x-axis) at the throat of a
(a) meter the fuel into air stream and amount dic- carburettor?
tated by the load and speed (a) (b)
(b) bring about mixing of air and fuel to get a
homogeneous mixture
(c) vapourise the fuel ∆p ∆p
(d) distribute fuel uniformly to all cylinders in a
multi-cylinder engine and also vapourise it.
Velocity Velocity
4. Match List I with List II and select the correct
answer using the codes given below the lists: (c) (d)
List I List II
A. Pre-combustion 1. Compression swirl ∆p ∆p
chamber
B. Turbulent chamber 2. Masked inlet valve Velocity Velocity

M10_THERMAL ENGINEERING_SE_XXXX_CH010(Part-III).indd 610 6/15/2018 5:07:27 PM


Internal Combustion Engine Systems 611

7. Match List I with List II and select the correct    II. Fuels of higher octane number can be
answer using the codes given below the lists: employed at higher compression ratio.
Of these statements
List I List II (a) both I and II are true
(Elements of a com- (Rich-mixture (b) both I and II are false
plete carburettor) requirement) (c) I is true but II is false
A. Idling system 1. To compensate for (d) I is false but II is true
dilution of charge 11. The object of providing masked inlet valve in the
B. Economiser 2. For
 cold starting air passage of compression-ignition engines is to
C. Acceleration pump 3. F
 or meeting maxi- (a) enhance flow rate
mum power range of (b) control in flow
operation (c) induce primary swirl
(d) induce secondary turbulence
D. Choke 4. F
 or meeting rapid
opening of throttle 12. Which one of the following events would reduce
the volumetric efficiency of a vertical compres-
Codes:
sion ignition engine?
A B C D (a) Inlet valve closing after bottom head centre
(a) 1 2 3 4 (b) Inlet valve closing before bottom dead centre
(b) 1 3 4 2 (c) Inlet valve opening before top dead centre
(c) 2 3 4 1 (d) Exhaust valve closing after top dead centre
(d) 4 1 2 3 13. As compared to air standard cycle, in actual work-
8. Match List I with List II and select the correct ing, the effect of variation in specific heats is to
answer using the codes given below the lists: (a) increase maximum pressure and maximum
temperature
List I List II (b) reduce maximum pressure and maximum
(SI engine problem) (Characteristic of temperature
fuel responsible (c) increase maximum pressure and decrease
for the problem) maximum temperature
A. Cold starting 1. Front end volatility (d) decrease maximum pressure and increase
maximum temperature
B. Carburettor icing 2. Mid-range
volatility 14. Reference fuels for knock rating of SI engine
fuels would include
C. Crankcase dilution 3. Tail end volatility
(a) iso-octane and alpha-methyl naphthalene
Codes: (b) normal octane and aniline
A B C (c) iso-octane and n-hexane
(a) 1 2 3 (d) n-heptane and so-octane
(b) 1 3 2
15. Consider the following measures:
(c) 2 3 1
     I. Increasing the compression ratio
(d) 3 1 2
   II. Increasing the intake air temperature
9. The air-fuel ratio for idling speed of automobile III. Increasing the length of dimension of the cyl-
petrol engine is closer to inder
(a) 10:1 (b) 15:1  IV. Increasing the engine speed
(c) 17:1 (d) 21:1 The measures necessary to reduce the tendency to
10. Consider the following statements: knock in CI engines would include
I. The performance of an SI engine can be (a) I, II, and III (b) I, II, and IV
improved by increasing the compression ratio. (c) I, III, and IV (d) II, III, and IV

M10_THERMAL ENGINEERING_SE_XXXX_CH010(Part-III).indd 611 6/15/2018 5:07:27 PM


612 Chapter 10

16. Match List I with List II and select the correct (a) F head, L head, I head
answer using the codes given below the lists: (b) T head, L head, F head
(c) I head, T head, F head
List I List II (d) F head, I head, T head
(Operating (Approximate air
20. The two reference fuels used for cetane rating are
condition) fuel ratio)
(a) cetane and isooctane
A. Idling 1. 16 (b) cetane and tetraethyl lead
B. Part load operation 2. 10 (c) cetane and n-heptane
(d) cetane and a-methyl naphthalene
C. Full load 3. 12.5
D. Cold start 4. 3 21. Match List I with II, in respect of SI engines, and
select the correct answer using the codes given
Codes: below the lists:
A B C D
(a) 2 1 3 4 List I List II
(b) 1 2 4 3 A. Highest useful com- 1. Ignitable mixture
(c) 2 1 4 3 pression ratio
(d) 1 2 3 4 B. Cold starting 2. Knock rating of fuels
17. Generally, in Bosch type fuel injection pumps, C. Limiting mixture 3. Detonation
the quantity of fuel is increased or decreased with strength
change in load, due to change in D. Delay period 4. C
 hain of chemical
(a) timing of start of fuel injection reactions in combus-
(b) timing of end fuel injection tion chamber
(c) injection pressure of fuel Codes:
(d) velocity of flow of fuel A B C D
18. Match List I with List II and select the correct (a) 2 3 1 4
answer using the codes given below the lists: (b) 3 2 1 4
(c) 2 3 4 1
List I List II (d) 3 4 2 1
(Operating mode of (Appropriate air-fuel
22. By higher octane number of SI fuel, it is mean
SI engine) ratio)
that the fuel has
A. Idling 1. 125 (a) higher heating value
B. Cold starting 2. 9.0 (b) higher flash point
C. Cruising 3. 16.0 (c) lower volatility
D. Maximum power 4. 22.0 (d) longer ignition delay
5. 3.0 23. Which of the following factors would increase the
Codes: probability of knock in the CI engines?
A B C D       I. Long ignition delay of fuel

(a) 2 4 5 1    II. Low self-ignition temperature of fuel


(b) 1 3 4 2 III. Low volatility of fuel
(c) 5 1 1 3 Select the correct answer using the codes given
(d) 2 5 3 1 below:
19. Knocking in the SI engine decreases in which one Codes:
of the following orders of combustion chamber (a) I, II, and III (b) I and II
designs? (c) I and III (d) II and III

M10_THERMAL ENGINEERING_SE_XXXX_CH010(Part-III).indd 612 6/15/2018 5:07:27 PM


Internal Combustion Engine Systems 613

24. List I gives the different terms related to combus- Of these statements
tion while List II gives the outcome of the events (a) I and II are correct
that follow. Match List I with List II and select the (b) I and III are correct
correct answer. (c) II and III are correct
(d) II and IV are correct
List I List II
28. Match List I with List II select the correct answer:
A. Association 1. Pseudo shock
B. Dissociation 2. Knock List I List II
(SI engine operat- (Desired air-fuel ratio)
C. Flame front 3. Endothermic
ing mode)
D. Abnormal 4. Exothermic A. Idling 1. 13.0
combustion
B. Cold starting 2. 4.0
Codes: C. Cruising 3. 16.0
A B C D D. Full throttle 4. 9.0
(a) 3 4 1 2
(b) 4 3 1 2 Codes:
(c) 3 4 2 1 A B C D
(d) 4 3 2 1 (a) 4 3 2 1
(b) 2 4 1 3
25. Which one of the following engines will have (c) 4 2 1 3
heavier flywheel than the remaining ones? (d) 2 4 3 1
(a) 40 kW four-stroke petrol engine running at
1500 rpm 29. Compensating jet in carburettor supplies almost
(b) 40 kW two-stroke petrol engine running at constant amount of petrol at all speeds because
1500 rpm (a) the jet area is automatically varied depending
(c) 40 kW two-stroke diesel engine running at on the suction.
750 rpm (b) the flow from the main jet is diverted to the
(d) 40 kW four-stroke diesel engine running at compensating jet with increase in speed.
750 rpm (c) the diameter of the jet is constant and the dis-
charge coefficient is invariant.
26. Consider the following statements: (d) the flow is produced due to the static head in
Knock in the SI engine can be reduced by the float Chamber.
     I. supercharging
   II. retarding the spark 30. If methane undergoes with the stoichiometric
III. using a fuel of long straight chain structure quantity of air, then the air-fuel ratio on molar
 IV. increasing the engine speed basis would be
Of these statement (a) 15.22:1 (b) 12.30:1
(a) I and II are correct (c) 14.56:1 (d) 9.52:1
(b) II and III are correct 31. For maximum specific output of a constant vol-
(c) I, III, and IV are correct ume cycle (Otto cycle)
(d) II and IV are correct (a) the working fluid should be air
27. Consider the following statements: (b) the speed should be high
The injector nozzle of a CI engine required to (c) suction temperature should be high
inject fuel at a sufficiently high pressure in order (d) temperature of the working fluid at the end of
to compression and expansion should be equal
     I. be able to inject fuel in a chamber of high 32. In a SI engine, which one of the following is the
pressure at the end of the compression stroke. correct order of the fuels with increasing detona-
   II. inject fuel at high velocity to facilitate atomi- tion tendency?
sation. (a) Paraffins, olefins, naphthenes, and aromatics
III. ensure that penetration is not high. (b) Aromatics, naphthenes, paraffins, and olefins

M10_THERMAL ENGINEERING_SE_XXXX_CH010(Part-III).indd 613 6/15/2018 5:07:27 PM


614 Chapter 10

(c) Naphthenes, olefins, aromatics, and paraffins (c) h dual > h diesel > h otto
(d) Aromatics, naphthenes, olefins, and paraffins (d) h otto > h dual > h diesel
33. Consider the following statements: 39. In an air-standard Diesel cycle, r is the compression
Detonation in the SI engine can be suppressed by ratio, r is the fuel cut-off ratio and g is the adiabatic
1. retarding the spark timing index g , its air standard efficiency is given by
2. increasing the engine speed
3. using 10% rich mixture  1 ( rg − 1) 
Of these statements (a) h = 1 −  g 
(a) I and III are correct  g r ( r − 1) 
(b) II and III are correct
(c) I, II, and III are correct  1 ( rg −1 − 1) 
(b) h = 1 −  g −1 
(d) I and II are correct
 g r ( r − 1) 
34. For the same maximum pressure and temperature
(a) Otto cycle is more efficient than diesel cycle  1 ( r − 1) 
(c) h = 1 −  g −1 
(b) diesel cycle is more efficient than Otto cycle  g r ( r − 1) 
(c) dual cycle is more efficient than Otto and die-
sel cycles  1 ( rg −1 − 1) 
(d) dual cycle is less efficient than Otto and die- (d) h = 1 −  g 
sel cycles  g r ( r − 1) 
35. Velocity of flame propagation in the SI engine is 40. Consider the following statements:
maximum for a fuel-air mixture which is      I. Octane rating of gasoline is based on isooc-
(a) 10% richer than stoichiometric tane and iso-heptane fuels which are paraf-
(b) equal to stoichiometric fins.
(c) more than 10% richer than stoichiometric   II. Tetraethyl lead is added to gasoline to

(d) 10% leaner than stoichiometric increase octane number.


III. Ethylene dibromide is added as scavenging
36. Divided chamber diesel engines use lower injec- agent to remove lead deposits on spark plugs.
tion pressure compared to open chamber engines IV. Surface ignition need not necessarily cause

because knocking.
(a) pintle nozzles cannot withstand high injec- Which of these statements are corrects?
tion pressures (a) I, II, III, and IV
(b) high air swirl does not require high injection (b) II, III, and IV
pressures for atomisation (c) I and IV
(c) high injection pressures may cause overpene- (d) I, II, and III
tration
(d) high injection pressure causes leakage of the 41. Consider the following statements:
fuel at the pintle      I. Recycling exhaust gases with intake increases
emission of oxides of nitrogen from the
37. For the same maximum pressure and heat input, engine.
the most efficient cycle is   II. When the carburettor throttle is suddenly
(a) Otto cycle opened, the fuel air mixture leans our tempo-
(b) Diesel cycle rarily causing engine stall.
(c) Brayton cycle III. The effect of increase in altitude on carburet-
(d) Dual combustion cycle tor is to enrich the entire part throttle opera-
38. The order of values of thermal efficiency of Otto, tion.
diesel, and dual cycles, when they have equal IV. Use of multiple venturi system makes it pos-
compression ratio and heat rejection, is given by sible to obtain a high velocity air stream when
(a) h otto > h diesel > h dual the fuel is introduced main venturi throat.
(b) h diesel > h dual > h otto Which of these statements are correct?

M10_THERMAL ENGINEERING_SE_XXXX_CH010(Part-III).indd 614 6/15/2018 5:07:29 PM


Internal Combustion Engine Systems 615

(a) I and III (b) I and II 46. Consider the following statements regarding the
(c) II and III (d) II and IV advantages of fuel injection over carburetion in SI
42. Match List I (Air-fuel ratio by mass) with List II engines:
(Engine operation mode) and select the correct       I. Higher power output and increased volumet-
answer using the codes given below the lists: ric efficiency.
   II. Simple and inexpensive injection equipment.
List I List II III. Longer Life of injection equipment.
 IV. Less knocking and reduced tendency for
A. 10:1 1. CI engine part load
back-fire.
B. 16:1 2. SI engine part load Select the correct answer using the codes given
below:
C. 35:1 3. SI engine idling
(a) I, II, and III (b) I, II, and III
D. 12.5:1 4. CI engine full load (c) II and III (d) I and IV
5. SI engine full load 47. Stoichiometric air-fuel ratio by volume for com-
bustion of methane in air is
Codes: (a) 15:1 (b) 17.16:1
A B C D
(c) 9.52:1 (d) 10.58:1
(a) 3 2 1 5
(b) 4 2 1 5 48. Auto-ignition time for petrol-air mixture is min-
(c) 3 1 2 4 imum when the ratio of actual fuel-air ratio and
(d) 4 1 2 3 chemically correct fuel-air ratio is
(a) 0.8 (b) 1.0
43. Consider the following statements:
(c) 1.2 (d) 1.5
In down draft carburettor, a hot spot is formed at
the bottom wall which is common for intake and
49. Consider the following statements regarding
exhaust manifolds. This helps to knock rating of SI engine fuels:
      I. improve evaporation of liquid fuel       I. Iso-octane is assigned a rating of zero octane
   II. provide higher thermal efficiency number.
III. reduce fuel consumption    II. Normal heptane is assigned a rating of hun-
 IV. lower the exhaust gas temperature dred octane number.
Which of these statements are correct? III. Iso-octane is assigned a rating of hundred
(a) I, II, and IV (b) I, II, and III octane number.
(c) I, III, and IV (d) II, III, and IV  IV. Normal heptane is assigned a rating of zero
44. In a petrol engine car, which one of the follow- octane number.
ing performance characteristics is affected by the Which of the above statements are correct?
front-end volatility of the gasoline used? (a) I and II (b) II and III
(a) Hot starting and vapour lock (c) III and IV (d) IV and I
(b) Engine warm-up and spark plug fouling 50. In spark ignition engines knocking can be reduced
(c) Spark plug fouling and hot starting by
(d) Vapour lock, engine warm-up, and spark plug (a) increasing the compression ratio
fouling (b) increasing the cooling water temperature
45. In the operation of four-stroke diesel engines, the (c) retarding the spark advance
term ‘squish’ refers to the (d) increasing the inlet air temperature
(a) injection of fuel in the pre-combustion cham- 51. The tendency of knocking in CI engine reduces
ber by
(b) discharge of gases from the pre-combustion (a) high self-ignition temperature of fuel
chamber (b) decrease in jacket water temperature
(c) entry of air into the combustion chamber (c) injection of fuel just before TDC
(d) stripping of fuel from the core (d) decrease in injection pressure

M10_THERMAL ENGINEERING_SE_XXXX_CH010(Part-III).indd 615 6/15/2018 5:07:29 PM


616 Chapter 10

52. Consider the following statements relevant to the 56. Which of the following factor(s) increase(s) the
ignition system of SI engine: tendency for knocking in the CI engine?
     I. Too small a dwell angle will lead to the burn- (a) Increasing both the compression ratio and the
ing of condenser and contact points. coolant temperature
  II. Too small a dwell angle will result in misfir- (b) Increasing both the speed and the injection
ing. advance
III. Too large a dwell angle will result in burning (c) Increasing the speed, injection advance and
of condenser and contact points. coolant temperature
 IV. Too large a dwell angle will result in misfir- (d) Increasing the compression ratio
ing. 57. Match List I with List II and select the correct
Which of the above statements are correct? answer using the codes given below the lists:
(a) I and II (b) II and III
(c) III and IV (d) IV and I List I List II
53. Consider the following statements in respect of
(SI engine opera- (Air-fuel ratio by mass)
automobile engine with thermosyphon cooling: tional mode)
     I. Heat transfer from gases to cylinder walls A. Idling 1. 4:1
takes place by convection and radiation. B. Cruising 2. 10:1
  II. Most of the heat transfer from radiator to C. Maximum power 3. 12.5:1
atmosphere takes place by radiation. D. Cold starting 4. 16:1
III. Most amount of heat transfer from radiator to 5. 14.8:1
atmosphere takes place by convection.
 IV. Heat transfer from cylinder walls take place Codes:
by conduction and convection. A B C D
(a) 2 4 3 1
Which of the above statements are correct?
(b) 5 4 1 3
(a) I, II and IV (b) I, III, and IV
(c) 2 3 5 1
(c) II, III, and IV (d) I and II
(d) 5 3 1 4
54. Which of the following action(s) increase(s) the
knocking tendency in the SI engine? 58. Consider the following statements for a multi-jet
(a) Increasing mixture strength beyond equiva- carburettor:
lence ratio (f) = 1.4      I. Acceleration jet is located just behind the
(b) Retarding the spark and increasing the com- throttle valve.
pression ratio.   II. Idle jet is located close to the choke.
(c) Increasing the compression ratio and reduc- III. Main jet alone supplies petrol at normal
ing engine speed. engine speeds.
(d) Increasing both mixture strength beyond Which of the statements given above are correct?
equivalence ratio (f) = 1.4 and the compres- (a) I, II, and III (b) I and II
sion ratio (c) II and III (d) I and III

55. Which of the following features(s) is/are used 59. The stoichiometric air-fuel ratio for petrol is 15:1.
in the combustion chamber design to reduce SI What is the air-fuel ratio required for maximum
engine knock? power?
(a) Spark plug located away from exhaust valve, (a) 16:1 – 18:1 (b) 15:1
wedge-shaped combustion chamber, and (c) 12:1 – 14:1 (d) 9:1 – 11:1
short flame travel distance 60. For which of the following reasons, do the indirect
(b) Wedge-shaped combustion chamber injection diesel engines have higher specific out-
(c) Wedge-shaped combustion chamber and put compared to direct injection diesel engines?
short flame travel distance      I. They have lower surface to volume ratio.
(d) Spark plug located away from exhaust valve,   II. They run at higher speeds.
short flame travel distance, and side valve III. They have higher air utilization factor.
design  IV. They have lower relative heat loss.

M10_THERMAL ENGINEERING_SE_XXXX_CH010(Part-III).indd 616 6/15/2018 5:07:29 PM


Internal Combustion Engine Systems 617

Select the correct answer using the code given 66. Consider the following statements:
below:      I. In the SI engines, detonation occurs near the
(a) I and II (b) II only end of combustion, whereas in CI engines,
(c) II and III (d) III and IV knocking occurs near the beginning of com-
61. Consider the following statements: bustion.
     I. In a carburettor, the throttle valve is used to    II. In SI engines, no problems are encountered
control the fuel supply. on account of preignition.
   II. The fuel level in the float chambers is to be III. Low inlet pressure and temperature reduce
about 4 to 5 mm below the orifice level of knocking tendency in SI engines but increase
main jet. the knocking tendency in CI engines.
III. An idle jet provides extra fuel during sudden Which of the statements given above are correct?
acceleration. (a) I, II, and III (b) Only I and II
 IV. A choke valve restricts the air supply to make (c) Only II and III (d) Only I and III
the gas richer with fuel. 67. The tendency of petrol to detonate in terms of
Which of the statements given above are correct? octane number is determined by comparison of
(a) II and IV (b) I and III fuel with which of the following?
(c) I, II, and III (d) II, III, and IV
(a) Iso-octane
62. The knocking tendency in compression ignition (b) Mixture of normal heptane and iso-octane
engines increases with (c) Alpha methyl naphthalene
(a) increase of coolant water temperature (d) Mixture of methane and ethane
(b) increase of temperature of inlet air
(c) decrease of compression ratio 68. For the same indicated work per cycle, mean
(d) increase of compression ratio speed and permissible fluctuation of speed, what
is the size of flywheel required for a multi-cylinder
63. Which of the following cannot be caused by a hot
engine in comparison to a single-cylinder engine?
spark plug?
     I. Pre-ignition (a) Bigger
   II. Post-ignition (b) Smaller
III. Detonation (c) Same
 IV. Run-on-ignition (d) Depends on thermal efficiency of the engine
Select the correct answer using the code given
69. Consider the following statements:
below:
     I. For a diesel cycle, the thermal efficiency
(a) I and IV (b) II only
decreases as the cut-off ratio increases.
(c) II and III (d) III only
   II. In a petrol engine, the high voltage for spark
64. Which of the following combustion chamber is in the order of 1000 V.
design features reduce(s) knocking in SI engines? III. The material for centre electrode in spark
     I. Spark plug located near the inlet valve plug is carbon.
   II. T-Head Which of the statements given above is/are
III. Wedge-shaped combustion chamber correct?
 IV. Short flame travel distance
(a) Only I (b) Only I and II
Select the correct answer using the code given
below: (c) Only II and III (d) I, II, and III
(a) I and III (b) III only 70. Consider the following statements:
(c) III and IV (d) I and II In order to prevent detonation in a spark igni-
65. A 4-stroke diesel engine, when running at 2000 rpm tion engine, the charge away from the spark plug
has an injection duration of 1.5 ms. What is the cor- should have:
responding duration of the crank angle in degrees?      I. low temperature
(a) 18° (b) 9°    II. low density
(c) 36° (d) 15° III. long ignition delay

M10_THERMAL ENGINEERING_SE_XXXX_CH010(Part-III).indd 617 6/15/2018 5:07:29 PM


618 Chapter 10

Which of the statements given above is/are I. It is standard fuel used for knock rating of
correct? diesel engines.
(a) Only I (b) Only II II. Its chemical name is n-hexadecane.
(c) Only III (d) I, II, and III III. It is a saturated hydrocarbon of paraffin
series.
71. Where does mixing of fuel and air take place in
IV. It has long carbon chain structure.
case of diesel engine?
Of the above statements
(a) Injection pump (b) Injector
(a) I, III, and IV are correct
(c) Engine cylinder (d) Inlet manifold
(b) I, II, and III are correct
72. Consider the following statements regarding (c) I, II, and IV are correct
cetane: (d) II, III, and IV are correct

Review Questions
1. Define an engine.
2. Define a heat engine.
3. How IC engines are classified?
4. Define an internal combustion engine.
5. What is an external combustion engine?
6. List the applications of IC engines.
7. List four advantages of internal combustion engine over external combustion engine.
8. Describe the salient features of SI engine.
9. Differentiate between SI and CI engines.
10. What are the main differences between 4-stroke and 2-stroke engines?
11. Explain the working of:
(a) Four-stroke SI engine (b) Four-stroke CI engine
12. Explain the working of 2-stroke SI engine.
13. Compare 4-stroke and 2-stroke engines.
14. Compare SI and CI engines.
15. List the merits of two-stroke engines over 4-stroke engines.
16. Draw the p-v diagram for (a) 4-stroke petrol engine (b) 2-stroke petrol engine.
17. Why the actual p-v diagram for a 4-stroke diesel engine differ from the theoretical diagram?
18. Explain the working of 2-stroke diesel engine.
19. What is the function of flywheel in an internal combustion engine?
20. What is the necessity of valve-timing diagram?
21. What is scavenging process in IC engines?
22. List four applications of Wankel engine.
23. Why do actual p-v diagrams differ from the theoretical diagrams of IC engines?
24. Define diagram factor.
25. Write four assumptions generally made while drawing the theoretical p-v diagram for 2-stroke petrol engine.
26. Suggest the IC engines used for the following:
(a) Bus (b) Motor cycle (c) Scooter (d) Moped
27. What are the essential requirements of an ideal carburettor?
28. What do you understand by compensation? Explain.
29. Explain with a neat diagram how the power and efficiency of an SI engine vary with A:F ratio.
30. Why are rich mixtures required for starting and during idling of an engine? Explain with the help of a neat
sketch.
31. Draw a simple type of carburettor and explain its working.
32. What do you understand by stoichiometric A:F ratio?

M10_THERMAL ENGINEERING_SE_XXXX_CH010(Part-III).indd 618 6/15/2018 5:07:30 PM


Internal Combustion Engine Systems 619

33. What are the requirements of a good injection system?


34. What is solid injection? What are its advantages over air-injection system?
35. What are the basic requirements of an ideal ignition system?
36. Explain the working of a battery ignition system with the help of a neat sketch.
37. What are the main drawbacks of a battery ignition system? How can these be overcome?
38. Explain the working of a magneto ignition system with the help of a neat sketch.
39. Compare battery and magneto ignition systems.
40. Describe the working of an electronic ignition system.
41. What are the lubricating systems for IC engines?
42. Differentiate between splash and pressure lubrication system and describe their operations with the help of
neat diagrams.
43. What is meant by dry and wet sump lubrication? Where is dry sump preferred? Why?
44. Explain the lubrication of the following parts:
(a) Crank pin and gudgeon pin  (b) Main bearing
(c) Cylinder and piston     (d) Exhaust valve
45. What is the necessity of cooling system for an IC engine?
46. Name the cooling systems generally used for I.C. engines.
47. What is the necessity of I.C. engine cooling?
48. Discuss the types of cooling systems for an IC engine.
49. What are the effects of under cooling and over cooling of an engine?
50. Draw a neat diagram of thermostatic-controlled water cooling system and explain its working.
51. Discuss the merits and demerits of water cooling with air cooling system.
52. Define the function of a radiator. Discuss different types of matrices used with these radiators.
53. What is the function of fan in radiator system? Discuss its performance with change of speed and load on
the engine.
54. What do you understand by evaporative cooling? When is it preferred?
55. What is the difference between open system and close system cooling used for IC engines? Discuss their
relative merits and demerits.
56. Discuss the relative merits and demerits of air cooling system over water cooling system.
57. What is overcooling? What are its disadvantages?
58. Define combustion phenomenon in I.C. engines.
59. What are the stages of combustion in S.I. engines?
60. What is ignition lag?
61. List four factors affecting delay period.
62. List four factors which affect the velocity of flame propagation.
63. What is the phenomenon of knocking in S.I. engines?
64. List four factors which reduce knocking in S.I. engines.
65. What are the effects of knocking?
66. Write four design principles for combustion chambers.
67. What are the stages of combustion in C.I. engines?
68. Write four factors which reduce delay period in C.I. engines.
69. What is diesel knock?
70. What is knocking in C.I. engines?
71. List four factors which affect knocking in C.I. engines.
72. Give the names of few anti-knock agents.

M10_THERMAL ENGINEERING_SE_XXXX_CH010(Part-III).indd 619 6/15/2018 5:07:30 PM


620 Chapter 10

Exercises

10.1 The venturi of a single jet carburettor has throat diameter of 75 mm and fuel nozzle diameter of 5 mm.
Find the A:F ratio of the mixture supplied to the engine for the following data: Cda = 0.93, Cdf = 0.68, ∆p
across venturi = 0.15 bar, ra = 1.29 kg/m3, rf = 720 kg/m3. Neglect nozzle lip and air compressibility effect.
[Ans. 13.3:1]
10.2 A four-cylinder, four-stroke petrol engine of 82.5 mm diameter and 115 mm stroke has 80% volumetric
efficiency, when running at 3000 rpm. The pressure head causing the flow is 11.03 cm of Hg. Determine
(a) the size of the venturi, and (b) the fuel nozzle diameter if the A:F ratio of mixture supplied is 14:1. Take
Cda = 0.84, Cdf = 0.7, ra = 1.29 kg/m3, rf = 700 kg/m3. [Ans. 22.3 mm, 4.15 mm]
10.3 The diameter of a simple venturi is 75 mm and fuel nozzle diameter is 5 mm. Nozzle lip = 5 mm, and Cda =
0.85, Cdf = 0.7, ra = 1.29 kg/m3, rf = 720 kg/m3
Determine (a) the air-fuel ratio supplied by the carburettor if the pressure drop causing air flow is 0.14 bar
neglecting nozzle lip and (b) the air-fuel ratio when nozzle lip is considered.
10.4 A simple carburettor fitted on a SI engine consumes 6.4 kg/h. The nozzle lip = 3 mm, df = 1.27 mm,
Cda = 0.8 and Cdf = 0.6. The atmospheric air pressure and temperature are 1.013 bar and 15.5°C and rf =
760 kg/m3. The A:F ratio supplied by the carburettor = 15:1. Determine (a) the venturi diameter, and (b)
the drop in venturi pressure. [Ans. 21.5 mm, 37 cm of H2O]
10.5 A single jet carburettor is to be designed to give an A:F ratio of 15:1 and fuel supply of 24.5 kg/h. The
atmospheric air pressure and temperature are 1.013 bar and 288 K. Calculate (a) the venturi diameter if
the air velocity is limited to 97.5 m/s, and (b) the fuel nozzle diameter.
Take Cda = 0.84 and Cdf = 0.66. [Ans. 33.8 mm, 2 mm]
10.6 A four-stroke, eight-cylinder CI engine develops 500 kW at 1200 rpm and consumes 0.175 kg/kWh of fuel
whose specific gravity is 0.89. Determine the diameter of a single orifice injector if the injection pressure is 160
bar and combustion chamber pressure is 40 bar. The period of injection is 30° crank rotation. Take Cdf = 0.9.
10.7 A four-stroke, six-cylinder oil engine, 115 mm diameter and 140 mm stroke operates with A:F = 16:1.
The suction condition in the engine are 1 bar and 300 K. The volumetric efficiency of the engine is 76%.
Determine the maximum amount of fuel supplied per hour to the engine running at 1500 rpm. The injec-
tion pressure is 125 bar and combustion chamber pressure is 40 bar. The fuel is supplied for 20° rotation
of the crank. Calculate the diameter of the orifice assuming each nozzle has single orifice. Take rf = 760
kg/m3 and Cdf = 0.64. [Ans. 21.7 kg/h, 1.76 mm]
10.8 A six-cylinder, four-stroke engine, 8 cm bore and 12 cm stroke runs at 3600 rpm. The volumetric effi-
ciency of the engine is 0.8. If the maximum head causing the flow is limited to 11.765 cm of mercury, find
the throat diameter of the venturi required. Find the diameter of the nozzle orifice if the desired A:F ratio
is 15:1. Assume Cda = 0.9, Cdf = 0.7, ra = 1.3 kg/m3, rf = 720 kg/m3. [Ans. 28 mm, 1.69 mm]
10.9 The throat diameter of a carburettor is 8 cm and nozzle diameter is 5.5 mm. The nozzle lip is 6 mm. The
pressure difference causing the flow is 0.1 bar. Find (a) A:F ratio supplied by the carburettor neglecting
nozzle lip. (b) A: F ratio considering nozzle lip, and (c) and minimum velocity of air required to start the
fuel flow. Neglect air compressibility. Take Cda = 0.85, Cdf = 0.7, ra = 1.2 kg/m3, rf = 750 kg/m3.
[Ans. 10.2:1, 10.28:1, 8.57 m/s]
10.10 The A:F ratio of a mixture supplied to an engine by a carburettor is 15:1. The fuel consumption of the engine
is 7.5 kg/h. The diameter of the venturi is 2.2 cm. Find the diameter of fuel nozzle if the lip of nozzle is 4 mm.
Assume rf = 750 kg/m3, Cda = 0.82, Cdf = 0.7 atmospheric pressure 1.013 bar and temperature = 25°C.
Neglect compressibility effect of air. [Ans. 1.245 mm]

M10_THERMAL ENGINEERING_SE_XXXX_CH010(Part-III).indd 620 6/15/2018 5:07:30 PM


Internal Combustion Engine Systems 621

10.11  A simple carburettor is designed to supply 6 kg of air and 0.45 kg of fuel per minute to a four stroke single-
cylinder petrol engine. The ambient air is at 1.013 bar and 27°C. (a) Calculate the throat diameter of the
choke (venturi) when the velocity of air is limited to 92 m/s. Take rf = 740 kg/m3 and velocity coefficient
= 0.8 (b) If the pressure drop near the fuel nozzle is 75% of that at the venturi, calculate fuel nozzle
diameter. Take Cdf = 0.6.[Ans. 35.2 mm, 23.4 mm]
10.12  Determine the size of fuel orifice to give air-fuel ratio of 12:1. The diameter of venturi throat is 3.5 cm and
vacuum at the venturi is 6.9 cm of Hg. The pressure and temperature of atmospheric air are 1.013 bar and
25°C. The nozzle lip is 5 mm. Assume Cda = 0.9, Cdf = 0.7, rf = 760 kg/m3. Consider the compressibility
of air. [Ans. 0.63 mm]
10.13  An engine having a simple single jet carburettor consumes 6 kg/h of fuel. The level of fuel in the float
chamber is 3 mm below the top of the jet when engine is not running. Ambient conditions are 76 cm of
Hg pressure and 20°C temperature. The jet diameter is 1.3 mm and its coefficient of discharge is 0.60. The
discharge coefficient of air is 0.84 and A/F ratio is 15. Determine the critical air velocity and the throat
diameter. Express the pressure depression in cm of water. Take density of fuel = 700 kg/m3.
 [Ans. 5.847 m/s, 19 mm, 32 cm of water]
10.14  A six-cylinder, four-stroke oil engine develops 200 kW at 1200 rpm and consumes 0.3 kg/kWh. Determine
the diameter of a single orifice injector if the injection pressure is 200 bar and combustion chamber pres-
sure is 40 bar. The injection is carried out for 30° rotation of the crank. Take rf = 900 kg/m3 and Cdf = 0.7.
Each nozzle on a cylinder is provided with a single orifice.  [Ans. 1.005 mm]
10.15  A four-stroke, six-cylinder oil engine operates on A:F = 20:1. The diameter and stroke of the cylinder are
10 cm and 14 cm, respectively. The volumetric efficiency is 80%. The condition of air at the beginning of
compression are 1 bar and 300 K. Determine (a) the maximum amount of fuel that can be injected in each
cylinder per cycle. (b) If the speed of the engine is 1500 rpm, injection pressure is 150 bar, air pressure
during fuel injection is 40 bar and fuel injection is carried out for 20° of crank rotation, determine the
diameter of the fuel orifice assuming only one orifice being used. Take rf = 760 kg/m3 and Cdf = 0.67.
[Ans. 0.023 kg/s, 0.575 mm]

ANSWERS TO MULTIPLE-CHOICE QUESTIONS


1. (c) 2. (c) 3. (a) 4. (a) 5. (d)
6. (c) 7. (b) 8. (a) 9. (a) 10. (d)
11. (a) 12. (b) 13. (b) 14. (d) 15. (b)
16. (a) 17. (b) 18. (d) 19. (a) 20. (d)
21. (b) 22. (d) 23. (b) 24. (d) 25. (d)
26. (d) 27. (a) 28. (a) 29. (b) 30. (d)
31. (a) 32. (a) 33. (d) 34. (b) 35. (a)
36. (b) 37. (a) 38. (d) 39. (c) 40. (a)
41. (a) 42. (c) 43. (b) 44. (c) 45. (c)
46. (b) 47. (d) 48. (b) 49. (c) 50. (c)
51. (c) 52. (c) 53. (b) 54. (b) 55. (a)
56. (a) 57. (d) 58. (a) 59. (b) 60. (c)
61. (a) 62. (c) 63. (b) 64. (c) 65. (b)
66. (d) 67. (b) 68. (b) 69. (a) 70. (d)
71. (c) 72. (a)

M10_THERMAL ENGINEERING_SE_XXXX_CH010(Part-III).indd 621 6/15/2018 5:07:30 PM


Chapter 11

Performance of Internal
Combustion Engines
11.1 ❐ PERFORMANCE PARAMETERS
The performance of an engine is an indication of the degree of success with which the conversion of
chemical energy contained in the fuel is done into useful mechanical work. The degree of success is
compared on the basis of following parameters:
1. Specific fuel consumption (SFC)
2. Brake mean effective pressure (BMEP)
3. Specific power output (SP)
4. Specific weight (SW)
5. Exhaust smoke and other emissions
However, in the evaluation of engine performance, the following performance parameters are chosen:
1. Power and mechanical efficiency
2. Mean effective pressure and torque
3. Specific output
4. Fuel-air ratio
5. Volumetric efficiency
6. Specific fuel consumption
7. Thermal efficiency and heat balance
8. Exhaust smoke and other emissions
9. Specific weight
Indicated power,
pim LAN
IP = kW(11.1)
60 × 103 × n
where pim = indicated mean effective pressure, Pa

area of indicator diagram, mm 2 × spring constant (bar/mm)


=
length of indicator diagram in mm
L = Length of stroke, m

M11_THERMAL ENGINEERING_SE_XXXX_CH11( Part-I).indd 622 6/15/2018 5:15:20 PM


Performance of Internal Combustion Engines  623

A = Area of stroke, m2
N = rotational speed, rpm
n  = number of strokes per revolution
= 2 for four-stroke engine
= 1 for two-stroke engine
pbm LAN
Brake power, BP = kW(11.2)
60 × 103 × n
where pbm = brake mean effective pressure, Pa
Frictional power, FP = IP − BP (11.3)

BP pbm BP
Mechanical efficiency, hmech = = = (11.4)
IP pim BP + FP

pbm AL
Torque, T = (11.5)
2p n
Specific output: It is defined as the brake output per unit of piston displacement.

BP
Specific output = (11.6)
AL

Volumetric efficiency,
Actual volume inhaled during suction stroke at NTP
hv = (11.7)
Swept volume of the piston
= va /vs
Brake specific fuel consumption,

60 × m f
BSFC = kg /(BP.h)(11.8)
BP × t
where mf  = mass of fuel consumed, kg
t = time in minutes
Indicated specific fuel consumption,
60 × m f
ISFC = kg /(IP.h)(11.9)
IP × t
BP × 60
Brake thermal efficiency, hbt = (11.10)
m f × LCV

where LCV = lower calorific value of fuel, kJ/kg

IP × 60
Indicated thermal efficiency, hit = (11.11)
m f × LCV

M11_THERMAL ENGINEERING_SE_XXXX_CH11( Part-I).indd 623 6/15/2018 5:15:24 PM


624 Chapter 11

hbt h
Relative efficiency, hrel = or it (11.12)
ha ha

where ha = air standard efficiency

Mass of fuel
Fuel-air ratio, F/A = (11.13)
Mass of air

Mass of air
Air-fuel ratio, A /F = (11.14)
Mass of fuel

Actual fuel-air ratio


Relative fuel-air ratio, FR = (11.15)
Stoichiometic fuel-air ratio

Stoichiometic A/F ratio


Equivalence ratio, f = (11.16)
Actual A/F ratio

Weight of the engine


Specific weight = (11.17)
Brake power, BP

Exhaust smoke and other emissions: Oxides in nitrogen, unburned hydrocarbons, etc.

11.2 ❐ BASIC ENGINE MEASUREMENTS


The basic measurements required to be carried out to evaluate the performance of an engine are as
follows:
1. Speed
2. Fuel consumption
3. Air consumption
4. Smoke density
5. Brake power
6. Indicated power and friction power
7. Heat loss to cooling water
8. Heat going to exhaust
9. Exhaust gas analysis

1. Speed Measurement: The angular speed of engine (crankshaft) may be measured by a tachome-
ter. Tachometers may be classified into the following categories:
(a) Mechanical tachometers
(b) Electrical tachometers
They can further be classified as contact type and non-contact type. Contact type tachometers are
of the following types:
(a) Mechanical type: Revolution counter and timer, slipping clutch tachometers, and centrifugal
free tachometers.

M11_THERMAL ENGINEERING_SE_XXXX_CH11( Part-I).indd 624 6/15/2018 5:15:26 PM


Performance of Internal Combustion Engines  625

(b) Electrical type: Drag cup tachometer, electromagnetic tachogenerator. Non-contact type
tachometers are photoelectric tachometer, variable reluctance tachometer, stroboscope, and
capacitance pick up tachometer.
2. Fuel consumption measurement: The fuel consumption of an engine is measured by determining
the volume flow in a given time interval and multiplying by the specific gravity of the fuel.
Another method is to measure the time required for consumption of a given mass of fuel. Accurate
methods for fuel measurement are as follows:
(a) Two glass-chambers method
(b) Rotameter
(c) Positive displacement meter
3. Measurement of air consumption: Accurate measurements for air consumption can be carried
out by the following methods:
(a) Orifice chamber method
(b) Viscous-flow air meter
4. Measurement of exhaust smoke: A smoke meter is a sort density measuring device and may be
used to measure exhaust smoke.
5. Measurement of heat carried away by exhaust gases: An exhaust gas calorimeter is commonly
used for the measurement of heat carried by exhaust gases.
Mass of air supplied per kg of fuel,
N ×C
ma = (11.18)
33(C1 + C2 )
Heat carried away by exhaust gas per kg of fuel,
Qg = (ma + 1)cpg (Tge − Ta) kJ/kg of fuel (11.19)
where (ma + 1) = mass of exhaust gases formed per kg of fuel supplied
cpg = specific heat of exhaust gases, kJ/kgk
Tge = temperature of exhaust gases coming out from engine, °C
Ta = temperature of ambient air, °C
6. Measurement of heat carried away by cooling water: The heat carried away by cooling water
is generally measured by measuring the water flow rate through the cooling jacket and the rise in
temperature of water during flow through the engine. The inlet and outlet temperatures of water
are measured by thermometers inserted in the pockets provided at the inlet and outlet from the
engine. The quantity of water flowing is measured by collecting the water in a bucket for a spec-
ified period.
Heat carried away by cooling water,
Qw = mwcpw (Tw − Twi ) kJ/min(11.20)
0

where mw = mass of water collected /min


cpw = specific heat of water, kJ/kg °C
Twi = inlet temperature of water, °C
Tw0 = outlet temperature of water, °C

M11_THERMAL ENGINEERING_SE_XXXX_CH11( Part-I).indd 625 6/15/2018 5:15:26 PM


626 Chapter 11

11.3 ❐ HEAT BALANCE SHEET


A heat balance sheet is an account of heat supplied and heat utilised during engine trial.
1. Heat supplied to the engine,
Qs = m f × LCV 

where m f = mass of fuel supplied per hour.

LCV lower calorific value of fuel, kJ/kg


2. Heat utilised in various ways:
(a) Heat equivalent of BP,
Q1 = 3600 × BP in kW kJ/h
(b) Heat carried away by cooling water,

Q2 = m w ⋅ cpw (Tw0 − Twi ) kJ/h

(c) Heat carried away by exhaust gases,

Q3 = m g ⋅ cpg (Tge − Ta) kJ/h

(d) Heat unaccounted for,


Q4 = Qs − (Q1 + Q2 + Q3)

11.4 ❐ WILLAN’S LINE METHOD


It is a method to determine the frictional power of an engine. In this method, gross fuel consumption
v’s brake power at a constant speed is plotted and the graph is extrapolated back to zero fuel consump-
tion as shown in Fig. 11.1. The point where this graph cuts the BP axis is an indication of the frictional
power of the engine at that speed. The test is applicable to only compression ignition engines.
Fuel consumption rate

O Brake power
FP

Figure 11.1  Willan’s line method

M11_THERMAL ENGINEERING_SE_XXXX_CH11( Part-I).indd 626 6/15/2018 5:15:29 PM


Performance of Internal Combustion Engines  627

11.5 ❐ MORSE TEST


This is a method to determine the indicated power of a cylinder in a multi-cylinder engine. In this
method, the engine is first run at the required speed and the output is measured. One cylinder is then
cut out by short circuiting the spark plug or by disconnecting the injector, as the case may be. The
output is measured by keeping the speed constant at its original value. The difference in the output is
a measure of the indicated power of the cutout cylinder. Thus for each cylinder the IP is obtain and is
added together to find the total IP of the engine.
IP of n cylinders, IPn = BPn + FPn
IP of (n − 1) cylinders IPn−1 = BPn−1 + FPn
IP of nth cylinder, (IP)nth = BPn − BPn−1 = IPn − IPn−1
Total IP of engine, IPn = ∑ (IP)nth
Frictional power of the engine, FP = IPn − BPn(11.21)

11.6 ❐ PERFORMANCE OF SI ENGINES


The performance of an engine is generally given by a heat balance sheet. The main components of a
heat balance sheet are (i) heat equivalent to the effective (brake) work of the engine, (ii) heat rejected
to the cooling medium, (iii) heat carried away from the engine with the exhaust gases, and (iv) unac-
counted losses. The unaccounted losses include the radiation losses from the various parts of the
engine and heat loss due to incomplete combustion. The friction loss is not shown as a separate item
as it ultimately reappears as heat in cooling water, exhaust, and radiation.
Figure 11.2 shows the heat balance sheet for a petrol engine run at full throttle over its speed range.
In SI engines, the loss due to incomplete combustion included in unaccounted losses can be rather
high. For a rich mixture (A /F ratio 12.5:13) it could be 20%. Figure 11.3 shows the heat balance of an
uncontrolled SI engine at different loads.

100
Unaccounted losses

75
Exhaust losses
Energy input (%)

50
Coolant losses

25
Useful work

0
Low Engine speed High

Figure 11.2  Heat balance diagram for a typical SI engine

M11_THERMAL ENGINEERING_SE_XXXX_CH11( Part-I).indd 627 6/15/2018 5:15:29 PM


628 Chapter 11

100
Exhaust

Wasted energy
80

Energy supplied
60 Cooling

Engine friction
40

Useful work
20 Accessories
Pumping propulsion
work energy
0 20 40 60 80 100 120
Speed km per hour

Figure 11.3  Uncontrolled SI engine


100
550
Machanical efficiency 500
80 450
Specific fuel consumption 400
350
Efficiency %

60
300
250
40 Specific fuel
Indicated thermal efficiency
consumption,
20 g/Kw-hr
Brake thermal efficiency

1000 2000 3000 4000 5000 6000


Engine speed, rpm

Figure 11.4 Efficiency and specific fuel consumption vs speed for a SI engine


at full throttle
Figure 11.4 shows the brake thermal efficiency, indicated thermal efficiency, mechanical efficiency,
and specific fuel consumption for the above SI engine.
Figures 11.5 (a) and (b) show the indicated power (IP), brake power (BP), and friction power (FP)
(by difference), brake torque, brake mean effective pressure, and brake specific fuel consumption of a
high compression ratio (r) automotive SI engine at full or wide open throttle (WOT).
The following conclusions can be drawn from the above figures:
1. At full throttle, the brake thermal efficiency at various speeds varies from 20%–27%, maximum
efficiency being at the middle speed range.
2. The percentage heat rejected to a coolant is more at lower speed (>> 35%) and reduces at higher
speeds (>> 25%). Considerably more heat is carried by the exhaust at higher speeds.
3. Torque and mean effective pressure (MEP) do not strongly depend on the speed of the engine but
depend on volumetric efficiency and friction losses. Maximum torque position corresponds with
the maximum air charge or maximum volumetric efficiency position.
  Torque and MEP curves peak at about half that of the brake power.

M11_THERMAL ENGINEERING_SE_XXXX_CH11( Part-I).indd 628 6/15/2018 5:15:30 PM


Performance of Internal Combustion Engines  629

160
Imep IP 1100
140
bmep 1000
120
BP 900

Power P, kW
100
800

mep, kPa
80
700
60
300
40 fmep
200
20 FP
100
0
1000 2000 3000 4000 5000
Engine speed, rev/min
(a)

hmech
0.9 350

SFC, g/kWh
BSFC
0.8 300
hmech

0.7 BSFC 250


0.6
200
0 1000 2000 3000 4000 5000
Engine speed, rev/min
(b)

Figure 11.5 Variable speed test of automotive SI engine at full throttle (CR = 9):
(a) Power, mep vs engine speed, (b) hmech, SFC vs engine speed

Note: If size (displacement) of the engine were to be double, torque would also double, but MEP
is a ‘specific’ torque—a variable independent of the size of the engine.
4. High brake power arises from high speed. In the speed range before the maximum brake power is
obtained, doubling the speed doubles the power.
5. At low engine speed, the frictional power is relatively low and brake power is nearly as large as
indicated power. As engine speed increases, however, friction power increases at continuously
greater rate and therefore, brake power reaches a peak and starts reducing even though indicated
power is increasing. At engine speeds above the usual operating range, friction power increases very
rapidly. At these higher speeds, indicated power will reach a maximum and then fall off. At some
point, indicated power and friction power will be equal, and brake power will then drop to zero.

11.6.1 Performance of SI Engine at Constant Speed


and Variable Load
The performance of an SI engine at constant speed and variable load is different from the performance
at full throttle and variable speed. Figure 11.6 shows the heat balance of an SI engine at constant speed
and variable load. The load is varied by altering the throttle and the speed is kept constant by resetting
the dynamometer.

M11_THERMAL ENGINEERING_SE_XXXX_CH11( Part-I).indd 629 6/15/2018 5:15:31 PM


630 Chapter 11

Engine speed N = 2500 rpm


100
Radiation etc.
80 Exhaust

Energy %
60

40 Coolant
20
Useful work
0
0 5 10 15
Brake power, kW

Figure 11.6  Heat balance vs load for a SI engine

Closing the throttle reduces the pressure inside the cylinders but the temperature is affected very little
because the air /fuel ratio is substantially constant, and the gas temperatures throughout the cycle are
high. This results in high loss to coolant at low engine loads.
At low loads, the efficiency is about 10%, rising to about 25% at full load. The loss to the coolant
is about 60% at low loads and 30% at full load. The exhaust temperature rises very slowly with load
and as the mass flow rate of exhaust gas is reduced because the mass flow rate of fuel into the engine
is reduced, the percentage loss to exhaust remains nearly constant (about 21% at low loads to 24% at
full load). Percentage loss to radiation increases from about 7% at low loads to 20% at full load.

11.7 ❐ PERFORMANCE OF CI ENGINES


The performance of a CI engine at constant speed and variable load is shown in Fig. 11.7. As the effi-
ciency of the CI engine is more than the SI engine, the total losses are less. The coolant loss is more at
low loads and radiation and other losses are greater at high loads. The BMEP, BP, and torque directly
increase with load, as shown in Figs. 11.8(a) and (b). Unlike the SI engine, the BP and BMEP are
continuously rising curves and are limited only by the smoke. The exhaust temperature is also nearly
proportional to the load. The lowest BSFC and hence the maximum efficiency occurs at about 80%
of the full load.
100
Unaccounted losses
80
Energy input (%)

60 Exhaust losses

40 Coolant losses

20 Useful work

0 25 50 75 100
Power output (%)

Figure 11.7  Heat balance diagram for a typical CI engine

M11_THERMAL ENGINEERING_SE_XXXX_CH11( Part-I).indd 630 6/15/2018 5:15:32 PM


Performance of Internal Combustion Engines  631

1100 1000

200 imep 1000 900


imep

bmep.kPa
mep.kPa
180 bmep 900 800
bmep
160 800 70 700

Power P, kW
Power P, kW

140 700 60 600


IP
120 50 500
100 40
BP
80 30
bsfc
60 240 20

sfc.g/kWh
isfc
200 10
160 0
1000 1500 2000 2500 2700 1000 2000 3000 4000 5000
Speed, rev/min Speed, rev/min
(a) (b)

Figure 11.8  IP, BP, IMEP, BMEP, and SFC for a CI engine

Brake SPEC fuel BMEP Torque-Nm


100 Torque
350
90 300
80 250
BMEP
70 700
cons. kg/kWh kN/m2

600
Power. kW

60 BHP 500
50
40 0.275
30 0.250
SPEC Fuel 0.225
20 0.200
1000 1500 2000 2500 3000
Speed rev/min

Figure 11.9  Performance curves of a diesel engine

Figure 11.9 shows the performance curves of a variable speed GM 7850 cc four cycle V-6 Toro-flow
diesel engine. The maximum torque value is at about 70% of maximum speed compared to about 50%
in the SI engine. In addition, the BSFC is low through most of the speed range for the diesel engine
and is better than the SI engine.

11.8 ❐ PERFORMANCE MAPS


For critical analysis, the performance of an IC engine under all conditions of load and speed is shown
by a performance map. Figure 11.10 shows the performance map of an automotive SI engine and
Fig. 11.11 shows the performance map of a four-stroke pre-chamber CI engine. Figure 11.11 also

M11_THERMAL ENGINEERING_SE_XXXX_CH11( Part-I).indd 631 6/15/2018 5:15:33 PM


632 Chapter 11

9.5 0.05 0.14 0.23 kW/cm2 Piston area

8.4 340

7.0 0
3010
3
5.6 5
31 28

bmep, bar
3 40
3 h
4.2 /kW
5 g s.f.c rve
36 .
b d cu
2.8 Ro
a

1.4
1216
0
5 10 15
Piston speed m/s

Figure 11.10  Form of performance map for a SI engine

9.8

8.4

7.0

5.6
bmep, bar

5
22 0.14 kW/cm2
231 237
4.2 Piston area
243
255
267 g/kWh 0.09
2.8
b.s.f.c
292
1.4 0.045
0.012
0 5 10 15 18
Piston speed, m/s

Figure 11.11  Form of performance map for a diesel engine

includes a typical curve of BMEP versus piston speed for level road operation in high gear. Note that
these maps can be used for comparing engines of different sizes as performance parameters have been
generalised by converting rpm into piston speed and brake power per time of piston area.
Generally, all engines show a region of the lowest specific fuel consumption (highest efficiency) at
a relatively low piston speed with a relatively high BMEP.
SI Engine: Constant speed line: Increased BSFC is obtained by moving upward along the constant
speed line because of mixture enrichment at high load which offsets an increase in mechanical
efficiency. Moving to lower BMEP, the BSFC increases because of the reduced mechanical efficiency
(IMEP decreases, whereas FMEP remains constant).
Constant BMEP line: Moving from the region of the highest efficiency along a line of constant
BMEP, the BSFC increases due to increased friction at higher piston speeds. Moving to the left
towards lower piston speed, although friction MEP decreases, indicated efficiency falls off owing to
poor fuel distribution and increased relative heat losses.

M11_THERMAL ENGINEERING_SE_XXXX_CH11( Part-I).indd 632 6/15/2018 5:15:35 PM


Performance of Internal Combustion Engines  633

CI Engine: In the CI engine, the BSFC increases at high loads owing to increased fuel waste (smoke)
associated with high fuel-air ratios. At lower load, BSFC increases due to decrease in mechanical
efficiency (same in the SI engine).
As speed is reduced from the point of the best economy along a line of constant BMEP, the product
of mechanical and indicated thermal efficiency appears to remain about constant down to the lowest
operating speed. The reduction in FMEP with speed is apparently balanced by a reduction in indicated
thermal efficiency due to poor spray characteristics at very low speed.
An interesting feature of performance curves is that they show that power at maximum economy is
about half of the maximum power.

Example 11.1
A six-cylinder, four-stroke spark-ignition engine of 10 cm × 2 cm (bore × stroke) with a compres-
sion ratio of 6 is tested at 4800 rpm on a dynamometer of arm 55 cm. During a 10 minutes test,
the dynamometer reads 45 kg and the engine consumed 5 kg of petrol of calorific value 45 MJ/kg.
The carburettor receives air at 9°C and 1 bar at the rate of 10 kg /min. Calculate (a) the brake power,
(b) the brake mean effective pressure, (c) the brake specific fuel consumption, (d) the brake specific
air consumption, (e) the brake thermal efficiency, and (f ) the air-fuel-ratio.

Solution
Given that i = 6, n = 2, ∆ = 10 cm, L = 12 cm, r = 6, N = 4800 rpm, l = 55 cm, t = 10 min, w = 45 kg,
mf = 5 kg, CV = 45 MJ/kg, m a = 10 kg /min

2pNT
(a) Brake power,   BP =
60 × 103
T = w × g × l = 45 × 9.81 × 55 × 10−2 = 242.79 N-m

2p × 4800 × 242.79
BP = = 122 kW
60 × 103

(b)      BP = pbm [(LA)N/n] × Number of cylinders


 p × (0.1)2 × 0.12  4800 × 6
122 × 103 = pbm ×  ×
 4  2 × 60
pbm = 5.39 bar
5
(c) m f = kg /min or 30 kg /h
10
30
BSFC = = 0.45 kg /kW.h
122
(d) m a = 10 kg /min or 600 kg /h
600
BSAC = = 4.918 kg /kW.h
122

M11_THERMAL ENGINEERING_SE_XXXX_CH11( Part-I).indd 633 6/15/2018 5:15:38 PM


634 Chapter 11

BP 122 × 103
(e) hbt = = = 0.325 or 32.5%
m f × CV 30 × 45 × 106
3600

m a 600
(f ) A /F ratio = = = 20 : 1
m f 30

Example 11.2
A sharp-edged circular orifice of diameter 3.8 cm and coefficient of discharge as 0.6 is used
to measure air-consumption of a four-stroke petrol engine. Pressure drop through the orifice is
145 mm of water and barometer reads 75.5 cm of Hg. The compression ratio of the engine is 6 and
the piston displacement volume is 2000 cm3.
The temperature of air is taken to be 26°C. At 2600 rpm, the engine brake power recorded is
29.5 kW. The fuel consumption is 0.14 kg /min and the calorific value of fuel used is 43960 kJ/kg.
Determine (a) the volumetric efficiency, (b) the air-fuel ratio, (c) the brake mean effective pressure,
(d) the brake thermal efficiency, (e) the air standard efficiency, and (f ) the relative efficiency.

Solution
Given that d = 3.8 cm, Cd = 0.6, hw = 14.5 cm of water, pa = 75.5 cm of Hg, r = 6, Vs = 2000 cm3,
N = 2600 rpm, BP = 29.5 kW, m f = 0.14 kg /min, CV = 43960 kJ/kg
Density of air at atmospheric conditions,
p 1 × 105
ra = = = 1.1653 kg /m3
RT 287 × 299

 h   r  14.5 × 1000
Head in m of air =  w   w  = = 124.43 m of air
 100   ra  100 × 1.1653
Velocity of air passing through orifice,
ca = [2gH]0.5 = [2 × 9.81 × 124.43]0.5 = 49.41 m /s
Volume of air passing through orifice,
p
Va = A0 caCd = . (3.8)2 × 10−4 × 49.41 × 0.6
4
= 0.033622 m3/s or 2.0173 m3/min

(a) Volumetric efficiency


Va 2.0173
= = 0.3879 or 38.79%
Vs 2000 × 10 −6 × 260
(b) Mass flow rate of air,
m a = Va ra = 2.0173 × 1.1653 = 2.35 kg /min
m a 2.35
A /F ratio = = = 16.79:1
m f 0.14

M11_THERMAL ENGINEERING_SE_XXXX_CH11( Part-I).indd 634 6/15/2018 5:15:41 PM


Performance of Internal Combustion Engines  635

BP 29.5 × 103
(c) BMEP = = = 0.3404 MPa
Vs 2000 × 10 −6 × 2600
60
(d) Brake thermal efficiency
BP 29.5 × 3600
(h th )b = = = 0.2895 or 28.95%
m f × CV 0.14 × 60 × 43960

1 1
(e) Air standard efficiency,  ha = 1 − = 1− = 0.5116 or 51.16%
(g −1) 0.4
r 6
(h th )b 28.85
(f ) Relative efficiency = = = 0.5639 or 56.39%
ha 51.16

Example 11.3
A four-stroke SI engine has six single-acting cylinders of 7.5 cm bore and 9 cm stroke. The engine
is coupled to a brake having a torque arm radius of 38 m. At 3300 rpm, with all cylinders operating
the net brake load is 324 N. When each cylinder, in turn, is rendered in operative, the average
net brake load produced at the same speed of the remaining five cylinders is 245 N. Estimate the
indicated mean effective pressure engine.

Solution
3300
BP when all cylinders are working = 2p NT = 2p × = (324 × 0.38) × 10−3
60
= 42.55 kW
BP when each cylinder is cut-off in turn
3300
= 2p × × (245 × 0.38) × 10−3 = 32.17 kW
60
IP of cylinder cut off = 42.55 − 32.17 = 10.38 kW
Total IP of engine = 6 × 10.38 = 62.3 kW
Indicated mean effective pressure

IP × n 62.3 × 103 × 2 × 60 × 10 −5
= = = 9.5 bar
i × LAN p 2
6 × 0.09 × (0.0745) × 3300
4

Example 11.4
A six-cylinder SI engine operates in a four-stroke cycle. The bore of each cylinder is 70 mm and
the stroke 100 mm. The clearance volume per cylinder is 67 cm3. At a speed of 3960 rpm the
fuel consumption is 19.5 kg /h and the torque developed is 140 N-m. Calculate (a) brake power,
(b) brake mean effective pressure, (c) the brake thermal efficiency if LCV of fuel is 44000 kJ/kg,
and (d) the relative efficiency on brake power basis. Assume g  = 1.4 for air.

M11_THERMAL ENGINEERING_SE_XXXX_CH11( Part-I).indd 635 6/15/2018 5:15:44 PM


636 Chapter 11

Solution
2pNT 2p × 3960 × 140
(a) Brake power BP = = = 58 kW
3
60 × 10 60 × 103
BP × n 6 × 58 × 103 × 2 × 60 × 10 −5
(b) Brake mean effective pressure pbm = = = 7.61 bar
LAN × i p 2
0.01 × (0.07) × 3960 × 6
4
BP 58 × 3600
(c) Brake thermal efficiency (h th )b = = = 0.244 or 24.4%
m f × LCV 19.5 × 44000
(d) Swept volume per cylinder,
p
Vs = AL = (0.07)2 × 0.1 = 3.84810−4 m3
4
Vs + Vc 384.8 + 67
Compression ratio, r = = = 6.743
Vc 67
1 1
Air standard efficiency, ha = 1 − = 1− = 0.534
g −1
r (6.473)0.4
(h th )b 0.244
Relative efficiency, h r = = = 0.457 or 45.7%
ha 0.534

11.9 ❐ MEASUREMENT OF AIR CONSUMPTION


BY AIR-BOX METHOD
The arrangement to measure the consumption of air by air-box method is shown in Fig. 11.12. It con-
sists of an air-tight box fitted with a sharp-edged orifice of known coefficient of discharge. Due to the
suction of engine, there is a pressure depression in the box which causes the flow through orifice for
obtaining a steady flow. The pressure difference causing the flow through the orifice is measured with
the help of a water manometer.
Let
A0 = area of orifice, m2
hw = head of water causing flow, cm
Cd = coefficient of discharge for orifice
d0 = diameter of orifice, cm
ra = density of air, kg /m3
hw rw h 1000 10 hw
Head of air, H = × = w × = m of air
100 ra 100 ra ra
Velocity of air through the orifice,
10 hw
ca = 2 gH = 2 g × m /s
ra

M11_THERMAL ENGINEERING_SE_XXXX_CH11( Part-I).indd 636 6/15/2018 5:15:50 PM


Performance of Internal Combustion Engines  637

Thermometer

Orifice
plate
Air box

hw
Inake

Water
manometer Engine

Figure 11.12  Measurement of air by air box method

Volume of air passing through the orifice,


2 g × 10 hw h
Va = A0 ca ⋅ Cd = A0Cd = 14 A0Cd w m /s
ra ra
Mass flow rate of air passing through the orifice,
p 2 hw
m a = Va ⋅ ra = 14 × d ×10 −4 × Cd × ra
4 0 ra

= 11 × 10 −4 Cd d02 hw ra kg /s

or 0.066 = Ca d02 hw ra kg /min(11.22)


pa
Density of air, ra =
RTa
hw × pa ×105
m a = 0.066 Cd d02
287 Ta

hw pa
= 1.232 Cd d02 (11.23)
Ta
where hw is in cm, pa in bar and Ta in K
Mass of air supplied per kg of fuel used,
N×C
ma /kg of fuel = (11.24)
33(C1 + C2 )
where N = % of N2 by volume in exhaust gases
C = % of carbon in fuel
C1, C2 = % of CO2 and CO by volume in exhaust gases.

M11_THERMAL ENGINEERING_SE_XXXX_CH11( Part-I).indd 637 6/15/2018 5:15:55 PM


638 Chapter 11

11.10  ❐  MEASUREMENT OF BRAKE POWER


Brake power is measured with the following ways:
1. Rope brake dynamometer: A rope brake dynamometer is shown in Fig. 11.13. A rope is wound
around the brake drum, whose one end is connected to the spring balance S suspended from over-
head and the other end carries the load W. In this arrangement, the whole power developed by
the engine is absorbed by the friction produced at the rim of the brake drum. The rim of the brake
drum is generally water-cooled to absorb the heat generated due to rubbing action of rope on rim.
Let
W = dead weight on the rope, N
S = spring pull, N
D = outer diameter of brake drum, m
dr = diameter of rope, m
N = speed of engine, rpm
Net load acting on brake drum = (W − S) N
1
Effective radius of brake drum, R = ( D + dr )
2
Frictional torque rating on brake drum,
Tf  = (W − S) × R Nm
2p NT f
Brake power developed by engine = kW(11.25)
60 × 103

dr

D D

Figure 11.13  Rope brake dynamometer


2. Prony brake: The arrangement of this braking system is shown in Fig. 11.14. It consists of brake
shoes which are clamped on the rim of the brake drum by means of bolts. The pressure on the rim
is adjusted with the help of nuts and springs. A load lever extends from top of the brake and a load
carrier is attached to the end of the load lever. The weights kept on this load carrier are balanced
by the reaction torque in the shoes. The load lever is kept horizontal to keep its length constant.

M11_THERMAL ENGINEERING_SE_XXXX_CH11( Part-I).indd 638 6/15/2018 5:15:56 PM


Performance of Internal Combustion Engines  639

Spring
Load
lever

Brake drum

Acting
W
weight

Tie bolt

Brake
shoe

Figure 11.14  Prony-brake arrangement

2pNT
Brake power = kW
60 × 103
T = WL (11.26)
where
W = load on load carrier, N
  L = distances from centre of shaft to load, m.

Example 11.5
A four-cylinder engine running at 1200 rpm gave 18.6 kW brake power. The average torque when
one cylinder was cut off was 105 N.m. Determine the indicated thermal efficiency if the calorific
value of the fuel is 42,000 kJ/kg and the engine uses 0.34 kg of petrol per kWh of brake power.
Solution
Given that i = 4, N = 1200 rpm, (BP)n = 18.6 kW, T3 = 105 N.m, CV = 42,000 kJ/kg
BSFC = 0.34 kg /kWh

2pNT3 2p × 1200 × 105


(BP)n−1 = = = 13.2 kW
60 × 103 60 × 103
(BP)n − (BP)n−1 = (IP)n − (IP)n−1 = (IP)1
∴ (IP)1 = 18.6 − 13.2 = 5.4 kW
Indicated power of 4 cylinders,
(IP)n = i × (IP)1 = 4 × 5.4 = 21.6 kW
( BP)n 18.6
h mech = = = 0.861 or 86.1%
(IP)n 21.6

M11_THERMAL ENGINEERING_SE_XXXX_CH11( Part-I).indd 639 6/15/2018 5:15:58 PM


640 Chapter 11

BSFC 0.34
ISFC = = = 0.3948 kg /kWh
hmech 0.861
ISFC × (IP)n
Fuel consumption, m f =
3600
0.3948 × 21.6
= = 2.3688 × 10−3 kg /s
3600
Indicated thermal efficiency,
(IP)n
(hth )i =
m f × CV
21.6
= = 0.2171 or 21.71%
2.3688 × 10 −3 × 4200

Example 11.6
A single cylinder four-stroke oil engine works on a diesel cycle. The following readings correspond
to full load conditions:
Area of indicator diagram = 3 cm2, Length of the diagram = 5 cm, Spring constant = 12 bar/cm2.
cm, Speed of the engine = 500 rpm, Load on the brake = 400 N, Spring reading = 50 N
Diameter of brake drum = 120 cm, Fuel consumption = 2.75 kg /h, Calorific value of fuel
= 42000 kJ/kg, Diameter of cylinder = 16 cm, Stroke length of piston = 20 cm. Determine (a) the
friction power, (b) the mechanical efficiency, (c) the brake thermal efficiency, and (d) the brake
mean effective pressure.
Solution
Given that n = 2, Ai = 3 cm2, Li = 5 cm, C = 12 bar/cm2. cm, N = 500 rpm, W = 400 N, S = 50 N,
Db = 120 cm, m f = 2.75 kg /h, CV = 42,000 kJ/kg, d = 16 cm, L = 20 cm
Ai C 3 × 12
IMEP, pim = = = 7.2 bar
Li 5
pim LAN p 500 1
IP = = 7.2 × 105 × 0.2 × (0.16)2 × × = 12.06 kW
3
n × 60 × 10 4 2 60 × 103
Db 2p N ( 400 − 50) × 1.2 × 2p × 500
BP = (W − S) × = = 10.99 kW
2 60 × 103 6 × 103 × 2
(a) Frictional power,
FP = IP − BP = 12.06 − 10.99 = 1.07 kW
BP 10.99
(b) Mechanical efficiency h mech = = = 0.9122 or 91.12%
IP 12.06
BP × 3600 10.99 × 3600
(c) Brake thermal efficiency (h th ) b = = = 0.3425 or 34.25%
m f × CV 2.75 × 42, 000

M11_THERMAL ENGINEERING_SE_XXXX_CH11( Part-I).indd 640 6/15/2018 5:16:05 PM


Performance of Internal Combustion Engines  641

(d) Brake mean effective pressure


BP × 103 × 60 × n 10.99 × 103 × 60 × 2
pbm = = = 0.6559 × 105 N/m2 or 6.559 bar
LAN p 2
0.2 × (0.16) × 500
4

Example 11.7
The following data are known for a four-cylinder four-stroke petrol engine: cylinder dimensions:
11 cm bore and 13 cm stroke, engine speed: 2250 rpm, brake power: 50 kW, friction power: 15 kW,
fuel consumption rate: 10.5 kg /h, calorific value of fuel: 50,000 kJ/kg, air inhalation rate: 300 kg /h,
and ambient condition: 15°C, 1.03 bar. Estimate (a) brake mean effective pressure, (b) volumetric
efficiency, (c) brake thermal efficiency, and (d) mechanical efficiency. [IES, 2008]

Solution
Given that D = 11 cm, L = 13 cm, N = 2250 rpm, BP = 50 kW, FP = 15 kW, m f = 10.5 kg /h.
CV = 50,000 kJ/kg, m a = 300 kg /h, ta = 15°C, pa = 1.03 bar, i = 4
BP pbm LA( N / 2)
(a) =
i 60 × 103
50 × 60 × 103 × 2
or B.M.E.P., pbm = = 0.5396 × 106 Pa
−2 p 2
4 × 13 × 10 × (0.11) × 2250
4
(b) Volumetric efficiency
Actual volume inhaled during suction stroke at NTP Va
= =
Swept vollume of piston Vs
p 2 p
or Vs = D L = × (0.11)2 × 0.13 = 1.2354 × 10−3 m3
4 4
pa 1.03 × 105
Density of air, ra = = = 1.246 kg /m3
RTa 287 × 288
Swept mass of air per second
N
= Vs × ra × ×i
2 × 60

1.2354 × 10 −3 × 1.246 × 2250 × 4


= = 0.1154 kg /s
2 × 60
300 1
hu = × = 0.7221 or 72.21%
3600 0.1154
(c) Brake thermal efficiency,
BP 50 × 103 × 3600
(hth)b = = = 0.3428 or 34.28%
m f × CV 10.5 × 50, 000 × 103

M11_THERMAL ENGINEERING_SE_XXXX_CH11( Part-I).indd 641 6/15/2018 5:16:10 PM


642 Chapter 11

(d) Indicated power,


IP = BP + FP = 50 + 15 = 65 kW
BP 50
Mechanical efficiency hmech = = = 0.7692 or 76.92%
IP 65

Example 11.8
The following data relates to a four-cylinder, four-stroke petrol engine:
Diameter of piston = 80 mm, length of stroke = 120 mm
Clearance volume = 100 × 103 mm3
Fuel supply = 4.8 kg /h. calorific value = 44100 kJ/kg
When the Morse test was performed the following data were obtained:
BP with all cylinders working = 14.5 kW
BP with cylinder 1 cut-off = 9.8 kW
BP with cylinder 2 cut-off = 10.3 kW
BP with cylinder 3 cut-off = 10.14 kW
BP with cylinder 4 cut-off = 10 kW
Find the IP of the engine and calculate indicated thermal efficiency, brake thermal efficiency, and
relative efficiency.

Solution
Given that i = 4, n = 2, d = 80 mm, L = 120 mm, Vc = 100 × 103 mm3, m f = 4.8 kg /k, CV = 44100 kJ/kg,
(BP)n = 14.5 kW, (BP)1 = 9.8 kW, (BP)2 = 10.3 kW, (BP)3 = 10.14 kW, (BP)4 = 10 kW
  (IP)1 = (BP)n − (BP)n−1
∴   (IP)1 = 14.5 − 9.8 = 4.7 kW
  (IP)2 = 14.5 − 10.3 = 4.2 kW
  (IP)3 = 14.5 − 10.14 = 4.36 kW
  (IP)4 = 14.5 − 10 = 4.5 kW
4
Indicated power, IP = Σ (IP)n = 4.7 + 4.2 + 4.36 + 4.5 = 17.76 kW
n =1

Indicated thermal efficiency,


360 × IP 3600 × 17.76
( nth )i = = = 0.3020 or 30.2%
m f × CV 4.8 × 44100

Brake thermal efficiency,


3600 × BP 3600 × 14.5
( nth )b = = = 0.2466 or 24.66%
m f × CV 4.8 × 44100

p 2 p
Swept volume,Vs = d L = (80)2 × 120 = 603.185 × 103 mm3
4 4

Vs 603.185 × 103
Compression ratio, r = 1 + = 1+ = 7.03
Vc 100 × 103

M11_THERMAL ENGINEERING_SE_XXXX_CH11( Part-I).indd 642 6/15/2018 5:16:12 PM


Performance of Internal Combustion Engines  643

Air standard efficiency,


1 1
ha = 1 − = 1− = 0.5416 or 54.16%
g −1
r (7.03)0.4

(h th )i 30.2
Relative efficiency, h r = = = 0.5576 or 55.76%
ha 54.16

Example 11.9
Find the air-fuel ratio of a four-stroke, single-cylinder, air-cooled engine with a fuel consumption
time for 10 cc as 20.4 s and air consumption time for 0.1 m3 as 16.3 s. The load is 17 kg at the speed
of 3000 rpm. Find the brake specific fuel consumption and brake thermal efficiency. Assume the
density of air as 1.175 kg /m3 and specific gravity of fuel to be 0.7. The lower heating value of fuel
is 43MJ/kg and dynamometer constant is 5000.

Solution
10 0.1
Given that V f = = 0.49 c.c./s, Va = = 6.135 × 10−3 m3/s, W = 17 kg, N = 3000 rpm,
20.4 16.3
ra = 1.175 kg /m3, sf = 0.7, LCV = 43 MJ/kg, C = 5000

m f = V f r f = V f × s f × rw = 0.49 × 10−6 × 0.7 × 103 = 0.343 × 10−3 kg /s

m a = Va ra = 6.135 × 10−3 × 1.175 = 7.208 × 10−3 kg /s

m a 7.208 × 10 −3
A /F ratio = = = 21
m f 0.343 × 10 −3

Load on dynamometer × rpm W × N 17 × 3000


Brake power, BP = = = = 10.2 kW
Dynamometer constant C 5000

m f 0.343 × 10 −3 × 3600
Brake specific fuel consumption, BSFC = = = 0.121 kg /kWh
BP 10.2

BP 10.2 × 103
Brake thermal efficiency, (hb)th = = = 0.6915 or 69.15%
m f × CV 0.343 × 10 −3 × 43 × 106

Example 11.10
The following data refers to a test on a single cylinder oil engine working on four-stroke cycle:
Diameter of brake wheel = 60 cm; rope diameter = 3 cm; dead load is 25 kg, spring balance
reading = 5 kg, and the engine is running at 400 rev/min. The indicator diagram has area = 4 cm2,
length = 6 cm, and spring stiffness is 12 bar/cm. The fuel consumption is 0.23 kg /kWh and the
fuel used has a calorific value 43963.5 kJ/kg. Taking cylinder bore 10 cm and piston stroke
15 cm, calculate the brake power, indicated power, mechanical and indicated thermal efficiencies
of the engine.

M11_THERMAL ENGINEERING_SE_XXXX_CH11( Part-I).indd 643 6/15/2018 5:16:17 PM


644 Chapter 11

Solution
Given that Db = 0.6 m, dr = 0.03 m, W = 25 × 9.81 = 245.25 N, S = 5 × 9.81 = 49.05 N, N = 400 rpm,
n = 2, a2 = 4 cm2, li = 6 cm, k = 12 bar/cm, m f = 0.23 kg /kWh, CV = 43963.5 kJ/kg, D = 10 cm,
L = 15 cm
(W − S ) × p ( Db + dr ) × N
Brake power, BP =
60 × 103
( 245.25 − 49.05) × p (0.6 + 0.03) × 400
     = = 2.589 kW
60 × 103
ai k 4 × 12
Indicated mean effective pressure, pim = = = 8 bar
li 6
pim LAN
Indicated power, IP =
60 × 103 × N
p
8 × 105 × 0.15 × (0.1)2 × 400
= 4 = 3.141 kW
60 × 103 × 2
BP 2.589
Mechanical efficiency, hmech = = = 0.8241 or 82.41%
IP 3.141
Total fuel consumption, m = BP × m f

= 2.589 × 0.23 = 0.5955 kg /h


IP × 3600 3.141 × 3600
Indicated thermal efficiency, (hth)i = = = 0.432 or 43.2%
m f × CV 0.5955 × 43963.5

Example 11.11
A four-stroke, single cylinder petrol engine mounted on a motor cycle was put to load test. The
load measured on dynamometer was 30 kg with drum diameter and speed, respectively, at 900 mm
and 2000 rpm. The engine consumed 0.15 kg of fuel in one minute, the calorific value of fuel being
43.5 MJ/kg. The fuel supply to the engine was stopped and was driven by a motor which needed
5 kW of power to keep it running at the same speed, the efficiency of the motor being 80%. The
engine cylinder bore and stroke are respectively at 150 mm and 200 mm. Calculate (a) the brake
power, (b) the indicated power, (c) the mechanical efficiency, (d) the brake thermal efficiency,
(e) the indicated thermal efficiency, (f ) the brake mean effective pressure and (vii) the indicated
mean effective pressure. [IES, 2012]

Solution
Given that four-stroke single cylinder petrol engine, W = 30 kg, Dd = 900 mm, N = 2000 rpm,
m f  = 0.15 kg /min, CV = 43.5 MJ/kg, Pm = 5 kW, hm = 80%, D = 150 mm, L = 200 mm, n = 2.

(a) Torque applied,


Dd 0.9
T=W× = 30 × 9.81 × = 132.435 N⋅m
2 2

M11_THERMAL ENGINEERING_SE_XXXX_CH11( Part-I).indd 644 6/15/2018 5:16:22 PM


Performance of Internal Combustion Engines  645

Break power,
2p NT 2p × 2000 × 132.435
BP = = = 27.737 kW
3
60 × 10 60 × 103
(b) Indicated power,
IP = BP + FP
Frictional power,
Pm 5
FP = = = 6.25 kW
h m 0.8
Indicated power, IP = 27.737 + 6.25 = 33.987 kW
(c) Mechanical efficiency,
BP 27.737
hmech = = = 0.8161 or 81.61%
IP 33.987
(d) Brake thermal efficiency,
BP 27.737 × 60
(hth )b = = = 0.2550 or 25.5%
m f × CV 0.15 × 43.5 × 103

(e) Indicated thermal efficiency,
IP 33.987 × 60
(hth )i = = = 0.3125 or 31.25%
m f × CV 0.15 × 43.5 × 103

pbm × LAN
(f )    BP =
60 × 103 × n

pbm × 0.2 × p (0.15)2 × 2000


27.737 = 4
60 × 103 × 2
BMEP, Pbm = 4.7 bar

4.7 × 33.987
(g)   IMEP = = 5.77 bar
27.737

Example 11.12
During the trial of a single oil engine, cylinder diameter 20 cm, stroke 28 cm, working on the
two-stroke cycle, and firing every cycle, the following observations were made:
Duration of trial = 1 h, total fuel used = 4.22 kg, calorific value of fuel = 44670 kJ/kg, proportion
of hydrogen in fuel = 15%, total number of revolutions = 21,000, mean effective pressure = 2.74 bar,
net brake load applied to a drum of 100 cm diameter = 600 N, total mass of cooling water circulated
= 495 kg, temperature of cooling water: inlet 13°C, outlet 38°C, air used = 135 kg, temperature of
air in test room = 20°C, temperature of exhaust gases = 370°C. Assume cp of gases = 1.005 kJ/kg.K,
cp of steam = 2.093 kJ/kg.K at atmospheric pressure. Calculate the thermal efficiency and draw up
the heat balance sheet. [IES, 1997]

M11_THERMAL ENGINEERING_SE_XXXX_CH11( Part-I).indd 645 6/15/2018 5:16:25 PM


646 Chapter 11

Solution
Given that D = 20 cm, L = 28 cm, n = 1, t = 1 h, mf = 4.22 kg, CV = 44670 kJ/kg, H2 = 15%,
Nt = 21000 rev, pm = 2.74 bar, W = 600 N, d = 100 cm, mw = 495 kg, twi = 13°C, two = 38°C, me = 13.5 kg,
tr = 20°C, tg = 370°C, cpg = 1.005 kJ/kg. K, cps = 2.093 kJ/kg.K
N t 21000
N= = = 350 rpm
60 60
Torque, T = W × d/2 = 600 × 0.5 = 300 N.m
2p NT 2p × 350 × 300
Brake power, BP = = = 10.996 kW
3
60 × 10 60 × 103
Indicated power,
pm LAN 2.74 × 105 × 0.28 × p × (0.2)2 × 350
IP = = = 14.06 kW
60 × 103 × n 4 × 60 × 103 × 4
IP 14.06 × 3600
Thermal efficiency, hth = = = 0.2685 or 26.85%
m f × LCV 4.22 × 44670

p 2 p
Swept volume, Vs = D LN = (0.2) 2 × 0.28 × 350 = 3.08 m3/min
4 4
4.22
Heat input, Qs = mf × C.V./t = × 44670 = 3141.79 kJ/min
60
Heat equivalent of BP, Q1= BP × 60 = 10.996 × 60 = 659.76 kJ/min
495
Heat lost to cooling water, Q2 = m w cw ⋅ ∆t = × 4.18 (38 − 13) = 862.125 kg /min
60
Heat carried away by exhaust gases:
135 + 4.22
Mass of flue gases = m a + m f = = 2.32 kg /min
60
422
Steam in exhaust gases m s = 9 × 0.15 × = 0.095 kg /min
60
Mass of dry exhaust gases m g = 2.32 − 0.095 = 2.225 kg /min
Heat in steam in exhaust gases, Q3 = m s [hg + cps (tg − tsat) − hfw] [∴ tsup = tg] at patm = 1.013 bar
= 0.095 [2676 + 2.093 (370 − 100) − 4.187 × 20] [∵ hfw = cpw × tr]
= 300 kJ/min
Heat in dry exhaust gases = m g × cpg (tg − tr)

= 2.225 × 1.005 × (370 − 20) = 782.6 kJ/min


Unaccounted heat loss = 3141.79 − [659.76 + 862.125 + 300 + 782.6]
= 537.305 kJ/min

M11_THERMAL ENGINEERING_SE_XXXX_CH11( Part-I).indd 646 6/15/2018 5:16:31 PM


Performance of Internal Combustion Engines  647

Heat Balance Sheet:

Heat input kJ/min Heat expenditure kJ/min %


Heat supplied by fuel 3141.79 1.  Heat equivalent to BP 659.76 21.00
2.  Heat in cooling water 862.125 27.44
3.  Heat in steam in exhaust gases 300.00 9.54
4.  Heat in dry exhaust gases 782.6 24.91
5.  Unaccounted heat 537.305 17.11
Total 3141.79 3141.79 100.00

Example 11.13
During a trial of a single cylinder, a four-stroke diesel engine, the following observations were
recorded:
Bore = 340 mm, stroke = 440 mm, rpm = 400, area of indicator diagram = 465 mm2, length
of diagram = 60 mm, spring constant = 0.6 bar/mm, load on hydraulic dynamometer = 950 N,
dynamometer constant = 7460, fuel used = 10.6 kg /h, calorific value of fuel (CV) = 49500
kJ/kg, cooling water circulated = 25 kg /min, rise in temp. of cooling water = 25°C, mass analysis of
fuel: carbon = 84%, hydrogen = 15%, incombustible = 1%, volume analysis of exhaust gas: carbon
dioxide = 9%, oxygen = 10%, temperature of exhaust gases = 400°C, specific heat of exhaust gas
= 1.05 kJ/kg°C, partial pressure of steam in exhaust gas = 0.030 bar, specific heat of superheated
steam = 2.1 kJ/kg°C, saturation temp. of steam at 0.030 bar = 24.1°C. Draw up heat balance sheet
on minute basis. [IAS, 2011]

Solution
Given that n = 2, D = 0.34 m, L = 0.44 m, N = 400 rpm, Ai = 465 mm2, Ld = 60 mm, k = 0.6 bar/mm,
W = 950 N, C = 7460, CV = 49500 kJ/kg, m f = 10.6 kg /h, m w = 25 kg /min, ∆tw = 25°C, C = 84%,
H2 = 15%, incombustible = 1%, CO2 = 9%, O2 = 10%, teg = 400°C, cpg = 1.05 kJ/kg /°C, ps = 0.030
bar, cps = 2.1 kJ/kg°C, ts at 0.030 bar = 24.1°C

Load on hydraulic dynamometer × Dynamometer (or engine) rpm


Brake power, BP =
Dynamometer constant
W × N 950 × 400
= = = 50.938 kW
C 7460
Area of indicator diagram × Spring constant
Indicated mean effective pressure, pim =
Length of indicator diagram
Ai × k 465 × 0.6
= = = 4.65 bar
Ld 60

p
pim LAN 4.65 × 105 × 0.44 × (0.34)2 × 400
Indicated power, IP = = 4
60 × 103 × n 60 × 103 × 2
= 61.92 kW

M11_THERMAL ENGINEERING_SE_XXXX_CH11( Part-I).indd 647 6/15/2018 5:16:33 PM


648 Chapter 11

m f 10.6
(a) Heat input, Qi = × CV = × 49500 = 8745 kJ/min
60 60
(b)   (i)  Heat equivalent of BP, Qb = BP × 60 = 50.938 × 60 = 3056.28 kJ/min
(ii)  Heat lost to cooling water, Qw = m w − cpw ⋅ Δtw = 25 × 4.187 × 25 = 2616.87 kJ/min
Percentage of N2 in exhaust gas = 100 − (9 + 10) = 81%
N ×C
Mass of air supplied per kg of fuel, ma =
33(C1 + C2 )
81 × 84
= = 22.91 kg
33(0 + 9)
Mass of exhaust gases formed per kg of fuel = 1 + 22.91 = 23.91 kg
10.6
Mass of exhaust gases formed per minute = × 23.91 = 4.224 kg /min
60
Mass of steam formed and carried with the exhaust gases per minute due to the combustion
of hydrogen in the fuel,
10.6
m s = 0.15 × × 9 = 0.2385 kg /min
60
Mass of dry exhaust gases formed per minute, m g = 4.224 − 0.2385 = 3.9855 kg /min
  (iii)  Heat carried away by exhaust gases per minute, Qg = m g cpg (tg − ta)
Let ambient air temperature, ta = 27°C
Qg = 3.9855 × 1.05 (400 − 27) = 1560.92 kJ/min
  (iv)  Heat carried away by steam with exhaust gases per minute,
Qs = m s [hg + cps (Tsup − 100)]
At p = 0.030 bar, ts = 24.10°C, hg = 2545.5 kJ/kg
Qs = 0.2385 [2545.5 + 2.1 (400 − 100)] = 757.36 kJ/min
(c) Heat unaccounted for = Qi − (Qb + Qw + Qg + Qs)
= 8745 − (3056.28 + 2616.87 + 1560.92 × 757.36) = 753.57 kJ/min

Heat Balance Sheet:

Heat supplied kJ/min % Heat distributed kJ/min %


Heat supplied by the 8745 100 1.  Heat in BP 3056.28 34.95
fuel
2.  Heat in cooling water 2616.87 29.92
3.  Heat in dry exhaust gases 1560.92 17.85
4. Heat in steam carried 757.36 8.66
with exhaust gases
5.  Heat unaccounted for 753.57 8.62
Total 8745 100 8745.00 100.00

M11_THERMAL ENGINEERING_SE_XXXX_CH11( Part-I).indd 648 6/15/2018 5:16:35 PM


Performance of Internal Combustion Engines  649

Example 11.14
During a test on a two-stroke engine on full load, the following observations were recorded:
Speed =350 rpm, net brake load = 590 N, mean effective pressure = 2.8 bar, fuel oil consumption
= 4.3 kg /h, cooling water required = 500 kg /h, rise in cooling water temperature = 25°C, air used per
kg of fuel = 33 kg, room temperature = 25°C, exhaust gas temperature = 400°C, cylinder diameter
= 220 mm, stroke length = 280 mm, effective brake diameter = 1 m, C.V. of fuel oil = 43900 kJ/kg,
proportion of hydrogen in fuel = 15%, mean specific heat of exhaust gases = 1.0 kJ/kg-K, specific
heat of steam = 2.09 kJ/kg-K
Calculate the following:
(a) Indicated power
(b) Brake power
(c) Draw the heat balance sheet on the basis of kJ/min[IAS, 2010]

Solution
Given that N = 350 rpm, Wb = 590 N, pmi = 2.8 bar, m f  = 4.3 kg /h, (∆t)w = 25°C, m w = 500 kg /h,
ma /mf = 33, t0 = 25°C, tg = 400°C, D = 220 mm, L = 280 mm, db = 1 m, CV = 43900 kJ/kg, H2 = 15%,
cpg = 1.0 kJ/kg.K, cps = 2.09 kJ/kg.K
p
pmi LAN 2.8 × 105 × 0.28 × (0.220)2 × 350
(a) Indicated power. IP = = 4 = 17.38 kW
60 × 103 60 × 103
2p N × Wb × ( db / 2) 2p × 350 × 590 × 0.5
(b) Brake power, BP = = = 10.81 kW
3
60 × 10 60 × 103
(c) Heat balance sheet:
4.3
Heat input = × 43900 = 3146.2 kJ/min
60
Heat equivalent of BP = 10.81 × 60 = 648.6 kJ/min
500
Heat lost to cooling water = m w × cpw × (Δt)w = × 4.187 × 25 = 872.3 kJ/min
60
Heat carried away by exhaust gases:
33 × 4.3 + 4.3
Mass of flue gases = m a + m f = = 2.437 kg /min
60
4.3
Steam in exhaust gases, m s = 9 × 0.15 × = 0.09675 kg /min
60
Mass of dry exhaust gases, m g = 2.437 − 0.09675 = 2.340 kg /min
  hf = cpw (100 − t0)
Heat in steam in exhaust gases = m s [hf + hfg + cps (tg − 100)]

  = 0.09675 [4.187 (100 − 25) + 2256.9 + 2.09 (400 − 100)]


  = 309.4 kJ/min

M11_THERMAL ENGINEERING_SE_XXXX_CH11( Part-I).indd 649 6/15/2018 5:16:38 PM


650 Chapter 11

Heat in dry exhaust gases = m g × cpg (tg − 100) = 2.340 × 1.0 (400 − 100) = 702 kJ/min
Unaccounted heat loss = 3146.2 − (648.6 + 872.3 + 309.4 + 702) = 613.9 kJ/min
Heat input kJ/min Heat expenditure kJ/min %
Heat supplied by fuel 3146.2 Heat equivalent to BP 648.6 20.61
Heat in cooling water 872.3 27.72
Heat in steam in exhaust gases 309.4 9.83
Heat in dry exhaust gases 702.0 22.31
Unaccounted heat 613.9 19.53
Total 3146.2 3146.6 100%

Example 11.15
A four-stroke petrol engine develops 30 kW at 2600 rpm. The compression ratio of the engine is 8
and its fuel consumption is 8.4 kg /h with calorific value of 44 MJ/kg. The air consumption of the
engine as measured by means of a sharp-edged orifice is 2 m3 per min. If the piston displacement
volume is 2 litres, calculate (a) the volumetric efficiency, (b) the air-fuel ratio, (c) the brake mean
effective pressure, (d) the brake thermal efficiency, and (e) the relative efficiency.
The ambient temperature air can be taken as 27°C, R for air as 287 J/kgK, and g  = 1.4. The
barometer reads 755 mm of mercury. [IAS, 2006]

Solution
Given that n = 2, BP = 30 kW, N = 2600 rpm, r = 8, m f = 8.4 kg /h, CV = 44 MJ/kg, Va = 2m3/min,
Vs = 2 litres = 2 × 10−3 m3, T0 = 27 + 273 = 300 K, R = 287 J/kg.K, g = 1.4, pb = 755 mm of Hg

V × N 2 × 10 −3 × 2600
(a) Swept volume per minute, Vs = s = = 2.6 m3/min
2 2
V 2
Volumetric efficiency hVol = a = = 0.7692 or 76.92%
V 2.6 s

(b)    p0 = 1.01325 × 755 = 1.00658 bar


760
p0 1.00658 × 105
ra = = = 1.169 kg /m3
RT0 287 × 300

m a = Va × 60 × ra = 2 × 60 × 1.169 = 140.28 kg /h

140.28
A /F ratio = = 16.7:1
8.4

pbm × ( LA ) N
(c) Brake power BP = kW
60 × n × 103

pbm × 2 × 10 −3 × 2600
or 30 =
60 × n × 103

M11_THERMAL ENGINEERING_SE_XXXX_CH11( Part-I).indd 650 6/15/2018 5:16:41 PM


Performance of Internal Combustion Engines  651

or pbm = 0.6923 MPa


BP × 3600 30 × 3600
(d) Brake thermal efficiency (hth)b = = = 0.2922 or 29.22%
m f × CV 8.4 × 44 × 103

ha = 1− (1/rg −1) = 1 − 1/(0.8)0.4 = 0.5647 or 56.47%


(hth )b 0.2922
hrcl =
(e) Relative efficiency       = = 0.5174 or 51.74%
ha 0.5647

Example 11.16
A six-cylinder, four-stroke petrol engine has a swept volume of 3.0 litres with a compression
ratio of 9.5. Brake output torque is 205 N-m at 3600 rpm. Air enters at 105 N/m2 and 60°C.
The mechanical efficiency of the engine is 85% and air-fuel ratio is 15:1. The heating value of
fuel is 44,000 kJ/kg and the combustion efficiency is 97%. Calculate (a) the rate of fuel flow,
(b) the brake thermal efficiency, (c) the indicated thermal efficiency, (d) the volumetric efficiency, and
(e) the brake specific fuel consumption. [IAS, 2005]

Solution
Given that i = 6, n = 2, Vs = 3 litre = 3 × 10−3 m3, r = 9.5, Tb = 205 N.m, N = 3600 rpm, p = 105 N/m2,
T = 60 + 273 = 333 K, hmech = 85%, A /F = 15, CV = 44000 kJ/kg, hcomb = 97%
2pNTb 2p × 3600 × 205
Brake power, BP = = = 77.283 kW
3
60 × 10 60 × 103
BP 77.283
Indicated power, IP = = = 90.921 kW
hmech 0.85
1 1
 Air standard efficiency, h a = 1 − = 1− = 0.5936 or 59.36%
g −1
r (9.5)0.4
p 105
Density of air, ra = = = 1.046 kg /m3
RT 287 × 333

Air volume flow rate, m a =


Vs N 3 × 10 −3 × 3600 = 0.09 m3/s
=
60 × n 60 × 2
Mass of air taken in per second = 0.09 × 1.046 = 0.09414 kg /s
0.09414
(a) Mass flow rate of fuel for the engine, m f = = 6.276 × 10−3 kg /s
15
BP 77.283
(b) Brake thermal efficiency, (h th )b = =
m f × CV × hcomb 6.276 × 10 −3 × 44000 × 0.97
= 0.2885 or 28.85%
0.2885
(c) Indicated thermal efficiency, (hth )i = = 0.3394 or 33.94%
0.85

M11_THERMAL ENGINEERING_SE_XXXX_CH11( Part-I).indd 651 6/15/2018 5:17:27 PM


652 Chapter 11

(hth )b 0.2885
(d) Relative efficiency, (hr ) = = = 0.486 or 48.6%
ha 0.5936

m f × 3600 6.276 × 10 −3 × 3600


(e) BSFC = = = 0.292 kg /kWh
BP 77.283

Example 11.17
The air flow to a four cylinder, four-stroke engine is measured by means of a 4.5 cm diameter
orifice, having Cd = 0.65. During a test the following data was recorded:
Bore = 10 cm, stroke = 15 cm, engine speed = 1000 rpm, brake torque = 135 Nm, fuel
consumption = 5.0 kg /h, CVfuel = 42600 kJ/kg, head across orifice = 6 cm of water.
Ambient temperature and pressure are 300 K and 1.0 bar, respectively.
Calculate (a) the brake thermal efficiency, (b) the brake mean effective pressure, and (c) the
volumetric efficiency. [IAS, 2004]
Solution
Given that i = 4, n = 2, d0 = 4.5 cm, Cd = 0.65, D = 10 cm, L = 15 cm, N = 1000 rpm, Tb = 135 Nm,
m f = 5 kg /h, CV = 42600 kJ/kg, h = 6 cm of H2O, T0 = 300 K, p0 = 1 bar, R = 287 J/kg.K

2p N Tb 2p × 1000 × 135
(a) Brake power, BP = = = 14.137 kW
3
60 × 10 60 × 103
BP × 3600 14.137 × 3600
Brake thermal efficiency, (hth)b = = = 0.2389 or 23.89%
m f × CV 5 × 42600

BP × n
(b) Brake mean effective pressure, pbm =
LAN × i

14.137 × 2 × 103 × 60 × 10 −5
= = 3.6 bar
p 2
0.15 × (0.1) × 1000 × 4
4
−2
(c) Δp = rgh = 10 × 9.81 × 6 × 10 = 588.6 N/m2
3

p0 1 × 105
ra = = = 1.614 kg /m3
RT0 287 × 300

Air inhaled per second, Va = Cd A0 2∆p.ra

p
= 0.65 × ( 4.5 × 10 −2 ) 2 × 2 × 588.6 × 1.1614 = 0.0382 m3/s
4
p N p 1000
Swept volume/s, Vs = 2 × D 2 L × = 4 × (0.1)2 × 0.15 × = 0.03927 m3/s
4 n × 60 4 2 × 60
Va 0.0382
Volumetric efficiency, hv = = = 0.9727 or 97.27%
Vs 0.03927

M11_THERMAL ENGINEERING_SE_XXXX_CH11( Part-I).indd 652 6/15/2018 5:17:33 PM


Performance of Internal Combustion Engines  653

Example 11.18
An eight-cylinder automobile engine of 80 mm diameter and 90 mm stroke with a compression
ratio of 7, is tested at 4000 rpm on a dynamometer of 600 mm arm length. During a 10-minute test
period at a dynamometer scale reading of 450 N, 4.8 kg of gasoline having a calorific value of 45000
kJ/kg was burnt and air at 27°C and 1.0 bar was supplied to the carburettor at the rate of 6.6 kg /min.
Find (a) the brake power delivered, (b) the brake mean effective pressure, (c) the brake specific fuel
consumption, (d) the brake thermal efficiency, (e) the volumetric efficiency, and (f ) the air fuel ratio.
[IAS, 2003]
Solution
Given that i = 8, D = 80 mm, L = 90 mm, r = 7, N = 4000 rpm,  = 600 mm, t = 10 min, W = 450 N,
mf = 4.8 kg, CV = 45000 kJ/kg, T1 = 27 + 273 = 300 K, p1 = 1 bar, m a = 6.6 kg /min

p1 1 × 105
Density of air, ra = = = 1.16 kg /m3
RT1 287 × 300

(a) Torque on the dynamometer, T = Wl = 450 × 0.6 = 270 N.m


2p NT 2p × 4000 × 270
Brake power, BP = = = 113.1 kW
3
60 × 10 60 × 103

ALN
(b) Also, BP = i × pbm ×
60 × n × 103
p
or 113.1 × 60 × 2 × 103 = 8 × pbm × (0.08)2 × 0.09 × 4000
4
or pbm = 9.375 bar

m f /hr 4.8 × 60 /10


(c) Brake specific fuel consumption BSFC = = = 0.25464 kg /kWh
BP 113.1
BP 113.1 × 600
(d) Brake thermal efficiency, (hth)b = = = 0.3142 or 31.42%
m f × CV 4.8 × 45000


p 2 p
(e) Swept volume, Vs = D L = (0.08)2 × 0.09 = 4.524 × 10−4 m3
4 4
Vs + Vc Vs
Compression ratio, r = = +1=7
Vc Vc
Vs V
= 6, Vc = s = 0.754 × 10−4 m3
Vc 6
N 4000
Swept volume/s, Vs = i × Vs × = 8 × 4.524 × 10 −4 × = 0.1206 m3/s
2 × 60 2 × 60

Air inhaled: p1V = m a RT1

M11_THERMAL ENGINEERING_SE_XXXX_CH11( Part-I).indd 653 6/15/2018 5:17:38 PM


654 Chapter 11

6.6 287 × 300


V = × = 0.0947 m3/s
60 1 × 105

V 0.0947
Volumetric efficiency,h vol = = = 0.7853 or 78.53%
Vs 0.1206
6.6 × 10
(f ) A/F ratio = = 13.75:1
4.8

Example 11.19
A six-cylinder, four-stroke petrol engine, with a bore of 120 mm and stroke of 180 mm under test,
is supplied petrol of composition: C = 82% and H2 = 18% by mass. The Orsat gas analysis indi-
cated that CO2 = 12%, O2 = 4% and and N2 = 84% by volume. Determine (a) the air-fuel ratio and
(b) the percentage of excess air.
Calculate the volumetric efficiency of engine based on intake conditions when the mass flow
rate of petrol is 32 kg /min at 1600 rpm. Intake condition are 1 bar and 17°C. Consider the density
of petrol vapour to be 3.5 times that of air at same temperature and pressure. Air contains 23%
oxygen by mass. [IAS, 2003]

Solution
Given that i = 6, n = 2, D = 120 mm, L = 180 mm, C = 82%, H2 = 18%, CO2 = 12%, O2 = 4%,
N2 = 84%, m f = 32 kg /h, N = 1600 rpm, p0 = 1 bar, T0 = 17 + 273 = 290 K, rf  /ra = 3.5
Minimum mass of air required for complete combustion
1 8  1 8 
=  C + 8H  =  × 82 + 8 × 18 = 15.768 kg /kg of fuel
23  3  23  3 
CN 0.82 × 0.84
Mass of air supplied, ma = = = 17.394 kg /kg of fuel
0.33 (C1 + C2 ) 0.33 × 0.12
Excess air supplied = 17.394 − 15.768 = 1.626 kg /kg of fuel
1.626
Percentage of excess air = × 100 = 10.31%
15.768
Actual A/F ratio = 17.394:1 kg air/kg of fuel
Mass flow rate of petrol = 32 kg /h
p 2 N p 1600
Swept volume per second = i × D L× = 6 × (0.12)2 × 0.18 × = 9.772 m3/min
4 n × 60 4 2
ma RT0 17.394 × 287 × 290
Volume of air, Va = = = 14.47 m3/kg of fuel
p0 5
1 × 10
Va 14.477 × 3.5
Volume of petrol vapour, Vf = × 3.5 = = 2.913 m3/kg of fuel
17.394 17.394
Total volume = 14.477 + 2.913 = 17.39 m3/kg fuel

M11_THERMAL ENGINEERING_SE_XXXX_CH11( Part-I).indd 654 6/15/2018 5:17:42 PM


Performance of Internal Combustion Engines  655

32
Mixture aspirated per minute = 17.39 × = 9.275 m3/min
60
9.275
Volumetric efficiency, h vol = × 100 = 94.91%
9.772

Example 11.20
Two identical petrol engines having the following specifications are used in vehicles:
Engine 1: Swept volume = 3300 cc. normally aspirated, BMEP = 9.3 bar, rpm  4500, compression
ratio = 8.2, efficiency ratio = 0.5, mechanical efficiency  0.9, mass of the engine  200 kg
Engine 2: Supercharged, swept volume = 3300 cc, BMEP  12.0 bar, rpm = 4500, compression
ratio = 5.5, efficiency ratio = 0.5, mechanical efficiency = 0.92, mass of the engine  220 kg
If both the engines are supplied with just adequate quantity of petrol for the test run, determine
the duration of test run so that the specific mass per kW of brake power is same for both the
engines. Calorific value of petrol = 44000 kJ/kg.
Assume both the engines operate on four stroke cycle.  [IAS, 2010]

Solution
Engine 1
Vs = 3300 cc or 33 × 10−4 m3, pbm = 9.3 bar, N = 4500 rpm, n = 2, r = 8.2, hr = 0.5, hmech = 0.9,
M1 = 200 kg, LCV = 4400 kJ/kg
pbm × ( LA) × N 9.3 × 105 × 33 × 10 −4 × 4500
Brake power, BP = = = 115 kW
n × 60 × 103 2 × 60 × 103
Indicated power, IP = BP/hmech = 115/0.9 = 127.8 kW
1 1
Air standard efficiency, ha = 1 − g −1 = 1− = 0.569
r (8.2)0.4
(h )
Efficiency ratio,  h rel = th i
ha
or hth = 0.5 × 0.569 = 0.2845
Again, IP = mf 1 × CV × (hth)i
or 127.8 = mf 1 × 44000 × 0.2865
or mf 1 = 10.96 × 10−3 kg /s
= 10.96 ×10−3 × t × 3600 = 39.46 × t kg /h
39.46 × t + M1 39.46 × t + 200
Specific mass of engine per kW of BP = =
BP 115
Engine 2:
Vs = 3300 cc or 33 × 10−4 m3, pbm = 12 bar, N = 450 rpm, n = 2, r = 5.5, hr = 0.5, hmech = 0.92, M2 =
220 kg
12 × 105 × 33 × 10 −4 × 4500
BP = = 148.4 kW
2 × 60 × 103

M11_THERMAL ENGINEERING_SE_XXXX_CH11( Part-I).indd 655 6/15/2018 5:17:46 PM


656 Chapter 11

148.4
IP = = 161.3 kW
0.92
1
Air standard efficiency, ha = 1 − = 0.494
(5.5)0.4
(hth)i = 0.494 × 0.5 = 0.247
IP = mf 2 × CV × (hth)i
or 161.3 = mf 2 × 44000 × 0.247
or mf 2 = 14.84 × 10−3 kg /s
= 14.84 × 10−3 × t × 3600 = 53.43 × t kg /h
53.43 × t + 220
Specific mass of engine per kW of BP =
148.4
39.46 × t + 200 53.43 × t + 220
Now     =
115 148.4
or 1.2904 (39.46 × t + 200) = 53.43 × t + 220
or 38.08 = 2.511 × t
or t = 15.1 hours
Example 11.21
The following data relates to a test trial of a single-cylinder four-stroke engine:
Cylinder dia. = 24 cm, stroke length = 48 cm, compression ratio = 5.9, number of explosions/
minute = 77, gas used/min at 771 mm of mercury and 15°C = 0.172 m3, lower calorific value of
gas at NTP = 49350 kJ/m3 , mean effective pressure from indicator card = 7.5 bar, weight of Jacket
cooling water/min = 11 kg, temperature rise of cooling water = 34.2°C, specific heat of water
= 4.2 kJ/kg°C.
Net brake load applied at brake wheel having an effective circumference of 3.86 m is 1260 N at
average speed of 227 rpm.
Estimate (a) the mechanical efficiency, (b) the indicated thermal efficiency and (c) the efficiency
ratio, and draw a heat balance sheet for the engine assuming that exhaust gases carry away 24% of
heat.  [IAS, 2000]

Solution
3.86
Given that D = 24 cm, L = 48 cm, r = 5.9, Wbn = 1260 N, rb = = 0.6143 m, Nm = 227 rpm,
2p
Ne = 77/min, Vg = 0.217 m3/min at 15°C and 771 mm of Hg, LCV = 49350 kJ/m3 at NTP, pmi = 7.5 bar,
m w = 11 kg /min, (Δt)w = 34.2°C, cpw = 4.2 kJ/kg.°C

(a) Brake torque, Tb = Wbn × rb = 1260 × 0.6143 = 774 N.m


2p N m Tb 2p × 774 × 227
Brake power, BP = = = 18.4 kW
60 × 103 60 × 103
5 p 2
pmi LAN e 7.5 × 10 × 0.48 × 4 (0.24) × 77
Indicated power, IP = = = 20.9 kW
60 × 103 60 × 103

M11_THERMAL ENGINEERING_SE_XXXX_CH11( Part-I).indd 656 6/15/2018 5:17:49 PM


Performance of Internal Combustion Engines  657

BP 18.4
Mechanical efficiency, hmech = = = 0.88 or 88%
IP 20.9

IP × 60
(b) Indicated thermal efficiency, (hth )i =
m f × LCV

1.01325 × 105
p= × 771 = 1.0279 × 105 N/m2
760
T = 15 + 273 = 288 K

pVg p0V0
=
T T0

p T0 1.0279 × 105 273


or V0 = × × Vg = × × 0.217= 0.20873 m3/min
p0 T 1.01325 × 105 288

Volume of gas at NTP = 0.20873 m3/min


20.9 × 60
Indicated thermal efficiency (hth )i = = 0.1217 or 12.17%
0.20873 × 49350

1
(c) Air standard efficiency,h a = 1 −
g −1
r

1
=1 − = 0.5083 or 50.83%
(5.9)1.4 −1

(h th )i 12.17
Efficiency ratio = = = 0.2394 or 23.44%
ha 50.83

Heat balance sheet:


Heat input = V0 × LCV = 0.20873 × 49350 = 10300.82 kJ/min
Heat equivalent of BP = 18.4 × 60 = 1104 kJ/min
Heat lost to cooling water = m w × cpw × (Δt)w
= 11 × 4.2 × 34.2 = 1580.04 kJ/min
Heat lost to exhaust gases = 0.24 × 10,300.82 = 2472.20 kJ/s
Heat unaccounted for = 10,300.82 − (1104 + 1580.04 + 2472.20) = 5144.58 kJ/min
Heat input kJ/min Heat expenditure kJ/min %
Heat supplied by gas, Q 10300.82 1.  Heat equivalent of BP, Q1 1104.00 10.72
2.  Heat lost to cooling water, Q2 1580.04 15.53

(Continued )

M11_THERMAL ENGINEERING_SE_XXXX_CH11( Part-I).indd 657 6/15/2018 5:17:53 PM


658 Chapter 11

(Continued )

Heat input kJ/min Heat expenditure kJ/min %


3.  Heat lost to exhaust gases, Q3 2472.02 24.00
4. Heat unaccounted for, Q4 = Q − 5144.76 49.75
(Q1 + Q2 + Q3)
10300.82 100.00

11.11  ❐  SUPERCHARGING OF IC ENGINES


The power output of an engine depends on the amount of air inducted into the cylinder per unit time,
the degree of utilisation of this air, and the thermal efficiency of the engine. The amount of air inducted
per unit time can be increased by increasing the engine speed or by increasing the density of air at
intake. The volumetric efficiency decreases as the speed is increased. The method of increasing the
inlet air density is called supercharging. It is generally used to increase the power output of the engine.
It is done by supplying air at a pressure higher than the pressure at which the engine naturally aspirates
air from the atmosphere by using a pressure boosting device called a supercharger.

11.11.1  Thermodynamic Cycle


The p-v diagram for an ideal Otto-cycle supercharged engine is shown in Fig. 11.15(a). The pressure
p1 represents the supercharging pressure and p6 is the exhaust pressure. Area 8-6-7-0-1-8 represents
the work done by the supercharger in supplying air at a pressure p1, whereas the area 1-2-3-4-1 is the
output of the engine. Area 0-1-6-7-0 represents the gain in work during the gas exchange process due
to supercharging. Thus, a part of the supercharger work is recovered. However, the area 1-6-8-1 cannot
be recovered and represents a loss of work. This loss of work causes the ideal thermal efficiency of the
supercharged engine to decrease with an increase in supercharging pressure.
Figure 11.15(b) shows an ideal dual combustion cycle supercharged engine. The pressure p1 rep-
resents the supercharging pressure and p6 = p7, is the exhaust pressure. The engine is supercharged
by the compressor. Area 9-10-1-11-9 represents the work done on the supercharger in supplying air at
3
Pressure

4
1
O
8
7 6
Volume

(a)

Figure 11.15  p -v diagram for supercharged engine: (a) Ideal Otto cycle,
(b) Dual combustion cycle

M11_THERMAL ENGINEERING_SE_XXXX_CH11( Part-I).indd 658 6/15/2018 5:17:54 PM


Performance of Internal Combustion Engines  659

3 4

1-2-3-4-5-1
2 Area representing
output in working
cycle mass per cycle = m

Pressure

8 1
Positive
(gas exchange work)
7 6
Vc Vs
1
11
Negative
(supercharger work)
10
9
Volume

(b)

Figure 11.15 (Continued)
pressure p1. Thus, for the engine, 8-1 represents induction process, 1-2 as the compression process,
2-3-4 as the heat addition process, 4-5 as the expansion process, 5-6 as the blow down to atmosphere
or heat rejection process and 6-7 as the exhaust process.
Work done my mass m by the supercharged engine,
W = Area (1-2-3-4-5-1) + Area (8-1-6-7-8) — Area (9-10-1-11-9).
Thus, a part of the supercharger work is recovered; however, the curve equivalent to Area (9-10-
1-11-9) — Area (8-1-6-7-8) is not recoverable and represents loss of work. This loss of work causes
the ideal thermal efficiency of supercharged engine to decrease with an increase in supercharging
pressure.

11.11.2  Supercharging of SI Engines


As far as SI engines are concerned, supercharging is employed only for aircraft and racing car engines.
This is because the increase in supercharging pressure increases the tendency to detonate and pre-ignite.
Apart from increasing the volumetric efficiency of the engine, supercharging results in an increase
in the intake temperature and pressure of the engine. These reduce the ignition delay and increase the
flame speed and results in greater tendency to detonate or pre-ignite. For this reason, the supercharged
SI engines use a lower compression ratio which results in lower thermal efficiency. The fuel consump-
tion is also greater than naturally aspirated engines.
Due to its poor fuel economy, supercharging of petrol engines is not very popular and is used only
when more power is needed or when more power is need to compensate altitude loss.

M11_THERMAL ENGINEERING_SE_XXXX_CH11( Part-I).indd 659 6/15/2018 5:17:55 PM


660 Chapter 11

11.11.3  Supercharging of CI Engines


Supercharging of CI engines does not result in any combustion problems. Increase in pressure and
temperature of the intake air reduces ignition delay and hence the rate of pressure rise results in a
better, quieter, and smoother combustion. This allows the use of poor quality fuel. The increase in
intake air temperature reduces volumetric and thermal efficiencies but the increase in density due to
pressure compensates for this and intercooling is not required except for highly supercharged engines.
It is possible to use lower fuel-air ratios in a supercharged engine, resulting in lower temperature and
reduced smoke from the engine. This results in an increased life of the engine. Thus, a CI engine is
more suitable for supercharging.
The apparatus used for increasing air density is known as a supercharger. A supercharger is an air
compressor which may be positive displacement, reciprocating (piston-cylinder), rotary type (roots
blower), or rotodynamic compressors (centrifugal or axial flow type). The centrifugal compressor is
widely used as a supercharger for IC engines. The function of a supercharger is either to produce more
power from an engine of a given cylinder size, or to compensate the power loss at high altitudes due
to rarefied atmosphere.

11.11.4  Effects of Supercharging


The effect of supercharging on the power and efficiency is shown in Fig. 11.16.
1. Power output: Supercharging produces more power because the supercharger supplies air at
a pressure and density higher than atmospheric with has the effect of increasing volumetric
efficiency.
2. Mechanical efficiency: The mechanical efficiency of a supercharged engine is higher than that of
one not supercharged.
3. Fuel consumption: It provides better mixing of fuel and air, which results in a specific reduction
of fuel consumption and the thermal efficiency increases.

80
Total compression 6 : 1
70

60
Power and efficiency

50
Power
40
Efficiency
30

20

10

0
1.5 2 3 4 5 6
Ratio of supercharge

Figure 11.16  Effect of supercharging ratio on power and efficiency

M11_THERMAL ENGINEERING_SE_XXXX_CH11( Part-I).indd 660 6/15/2018 5:17:56 PM


Performance of Internal Combustion Engines  661

11.11.5  Objectives of Supercharging


The objectives of supercharging are as follows:
1. To overcome the effect of high altitudes, as in the case of aircrafts and stationary engines in
mountains.
2. To reduce the weight of an engine per kW or power developed, as in the case of racing cars.
3. To reduce the size of the engine to fit into a limited space, as in the case of locomotives or marine
engines.
4. To increase the power output of an existing engine to meet greater power demands.

11.11.6  Configurations of a Supercharger


1. Compressor Coupled of Engine Shaft: The compressor is operated directly by the engine with
set-up gearing to increase the speed of centrifugal compressor. A part of the engine output is used
to drive the supercharger and the net output is calculated by deducing this power from the gross
power of the engine.
2. Turbo-charger: The energy of the exhaust gases from the engine is used to develop power in
a turbine which directly runs the supercharger. There is no mechanical connection between the
engine and the supercharger.
3. Direct Coupling between the Engine, Compressor, and Turbine: The advantage of this
arrangement is that when the turbine output is insufficient to run the compressor, additional power
required is taken from the engine. Additional power from the turbine can also be fed to the engine.
4. Compressor geared with Engine and Free turbine: The engine drives the compressor but the
energy in the exhaust is utilised to develop the power from a separate turbine.
The various configurations are shown in Fig. 11.17(a) to (d).

Exhaust Air in

C
C

Engine G Load
Engine
Air in Gearing
Exhaust
(a) (b)

T G2 Load

T C C
Exhaust
Load G Engine Engine G1 Load
Gearing Exhaust Air in
Air in Gearing
(c) (d)

Figure 11.17 Arrangements for supercharging: (a) Mechanical supercharging,


(b) Turbocharging, (c) Engine-driven compressor and turbocharger,
(d) Engine-driven compressor and free turbine

M11_THERMAL ENGINEERING_SE_XXXX_CH11( Part-I).indd 661 6/15/2018 5:17:57 PM


662 Chapter 11

11.11.7  Supercharging of Single Cylinder Engines


Supercharging of single cylinder engines is not carried out because the thermal efficiency drops to
zero when the ratio of supercharger is about 6. Further, the power is maximum when the ratio of the
supercharger is nearly 2, 5, and then drops to zero at a ratio of about 6.

Example 11.22
A four-stroke diesel engine of 3000 cc capacity develops 14 kW per m3 of free air induced per
minute. When running at 3500 rev/min, it has a volumetric efficiency of 85% referred to free air-
conditions of 1.013 bar and 27°C. It is proposed to boost the power of the engine by supercharging by
a blower (driven mechanically from the engine) of pressure ratio 1.7 and isentropic efficiency of 80%.
Assuming that at the end of induction, the cylinders contain a volume of charge equal to the swept
volume, at the pressure and temperature of the delivery from the blower, estimate the increase in brake
power to be expected from the engine. Take overall mechanical efficiency as 80%, g for air = 1.4,
R = 0.287 kJ/kg K. [IES, 2009]

Solution
The schematic arrangement is shown in Fig. 11.18(a).
Swept volume of engine per minute,
3500
Vs = 3 × 10−3 × = 5.25 m3/min
2
Unsupercharged inducted volume
= Vs × hvol = 5.25 × 0.85 = 4.4625 m3/min
The actual and isentropic compression processes of supercharger are shown in Fig. 11.18(b).
g −1
0.4
p  g
T2 = T1  2  = (273 + 27) (1.7) 1.4 = 349.1 K
 p1 
Isentropic efficiency of supercharger.

Inlet
T
Exhaust
p2

p1
2 2′
Engine

Turbocharger
t
Gear
Air in s

(a) (b)

Figure 11.18  Supercharger: (a) Schematic arrangement, (b) Processes on T-s diagram

M11_THERMAL ENGINEERING_SE_XXXX_CH11( Part-I).indd 662 6/15/2018 5:17:58 PM


Performance of Internal Combustion Engines  663

T2 − T1
hc =
T2 ′ − T1
349.1 − 300
0.8 =
T2 ′ − 300
T2′ = 361.4 K
The supercharger delivers 5.25 m3/min at p2 = 1.013 × 1.7 = 1.72 bar and 361.4 K to the engine. This
volume referred to 1.013 bar and 300 K is,

V = 1.72 × 300 × 5.25 = 7.41 m3/min


1.013 361.4
Increase in inducted volume = V − Vs = 7.41 − 4.4625 = 2.95 m3/min
As indicated power (I.P.) is directly proportional to induced volume, therefore, increase in I.P. due to
increase in inducted volume of air
= 14 × 2.95 = 41.3 kW
Increase in IP due to increase in inducted air pressure because of supercharger

 5.25 
= ∆p × Vs = (1.72 − 1.013) × 105 × × 10−3 = 6.186 kW
 60 
Total increase in IP because of supercharger
= 41.3 + 6.186 = 47.486 kW
Increase in BP = Increase in IP × hmech
= 47.486 × 0.8 = 38 kW
Power required to run the supercharger.
Psup = m a· cpa (T2′ − T1)
where m a = mass of air delivered by the supercharger per second.
p2V2 1.72 × 105 × 5.25
= = = 0.145 kg /s
RT2 ′ 60 × 287 × 361.4
Psup = 0.145 × 1.005 × (361.4 − 300) = 8.95 kW
Net increase in BP = 38 − 8.95 = 29.05 kW
29.05 × 100
Percentage increase in BP = = 324.6%
8.95

11.12  ❐  SI ENGINE EMISSIONS


Spark ignition engine emissions are divided into three categories as exhaust emission, evaporative
emission, and crank case emission.

M11_THERMAL ENGINEERING_SE_XXXX_CH11( Part-I).indd 663 6/15/2018 5:18:01 PM


664 Chapter 11

HC 15 to 25%
(from fuel tank
and carburettor)

Fuel
Engine tank
silencer

HC 50 to 60%
CO 10%
Exhaust NO 10%
emission

Figure 11.19  Emissions from SI engines

The major constituents which contribute to air pollution are CO, NOx, and hydrocarbons (HC) coming
from SI engine exhaust. The percentages of different constituents coming out from the above three
mentioed sources are shown in Fig. 11.19.
The relative amounts depend on the engine design and operating conditions but are of order,
NOx   500 to 1000 ppm (20 g /kg of fuel), CO  1 to 2% (200 g /kg of fuel) and HC  3000 ppm
(25 g /kg of fuel). Fuel evaporation from the fuel tank and the carburettor exists even after engine shut
down and these are unburned HCs. However, in most modern engines, these non-exhaust unburned
HCs are effectively controlled by returning the blow-by gases from the crank case to the engine intake
system by venting the fuel tank and through a vapour-absorbing carbon cannister which is purged as
some parts of the engine intake air during normal engine operation.
The other constituents include SO2 and lead compounds. Petrol rarely contains sulphur; therefore,
SO2 is not a pollutant from the SI engine exhaust. Petrol contains lead in small percentages but its
effect is more serious on human health.
The processes by which pollutants form within the cylinder in a conventional SI engine are quali-
tatively illustrated in Fig. 11.19. It shows the formation of pollutants during four strokes of the cycle.
NO forms throughout the high temperature burned gases behind the flame through chemical reactions.
NO formation rate increases with an increase in gas temperature. As the burned gases cool, during
expansion stroke, the reactions involving NO freeze and leave NO concentrations far in excess of
levels corresponding to equilibrium temperature at exhaust conditions.
CO also forms during combustion process with lean A:F mixtures, and there is sufficient O2 to
burn all the carbon in the fuel to CO2. However, in high temperature products even with lean mixtures,
there is sufficient CO in exhaust because of dissociation of CO2. Later, in expansion stroke, the CO
oxidation process also freezes as the gas temperature falls.
The unburned hydrocarbon emission comes from different sources. During compression and combus-
tion, the increasing cylinder pressure forces some of the gases in the cylinder into crevices connected to
combustion chamber, the volumes between the piston rings and cylinder wall are the largest of these. Most
of this gas entering into crevices is unburned air fuel mixture escaped from primary combustion zone.
This happens because the flame cannot enter these narrow crevices. The gas which leaves these crevices
later in the expansion and exhaust processes is one source of unburned HC emissions. The combustion
chamber walls are another possible source. A quench layer containing unburned and partially burned
A:F mixture is left at the wall when the flame dies as it approaches the wall. This unburned HC in this
layer (0.1 mm) burns rapidly if the combustion chamber walls are clean. The next source of HC is the thin

M11_THERMAL ENGINEERING_SE_XXXX_CH11( Part-I).indd 664 6/15/2018 5:18:02 PM


Performance of Internal Combustion Engines  665

layer of lubricating oil on cylinder wall, and the piston which absorbs HC before and after combustion.
A final source of HC in engines is incomplete combustion due to bulk quenching of the flame in that
fraction of engine cycle where the combustion is especially slow. This unburned HC near the cylinder
wall is exhausted during exhaust stroke as the piston pushes the gases out.

11.12.1  Exhaust Emissions


The major exhaust emissions are as follows:
1. Unburnt hydrocarbons, (HC)
2. Oxides of carbon, (CO and CO2)
3. Oxides of nitrogen, (NO and NO2)
4. Oxides of sulphur, (SO2 and SO3)
5. Particulates
6. Soot and smoke
The various exhaust emissions from petrol engine are as follows:
1. Unburnt hydrocarbons (HC): The causes for the emissions of HC are incomplete combustion,
crevice volumes and flow in crevices, leakage past the exhaust valve, valve overlap, deposits on
walls, and oil on combustion chamber walls.
The reasons for incomplete combustion are improper mixing of air and fuel, and flame quench-
ing at the walls of the cylinder. Low load and idle conditions increase HC.
2. Oxides of Carbon (CO and CO2): Carbon monoxide is generated with a rich fuel-air ratio mix-
ture. This happens during starting and accelerating under load. Poor mixing, local rich regions,
and incomplete combustion are the sources of CO emissions.
3. Oxides of Nitrogen (NOx): NOx are created mostly from nitrogen in the air and fuel blends.
In addition to temperature, the formation of NOx depends on pressure, A/F ratio, combustion
duration, and location of spark plug.
4. Oxides of Sulphur (SOx): These are generated due to the presence of sulphur in the fuel. SO2
and SO3 react with water to give rise to H2SO3 and H2SO4, which causes acid rain.
The variation of emissions from a petrol engine with A/F ratio is shown in Fig. 11.20.

Max. economy
Max. power
(with catalyst)
Emission
NOx control
(no catalyst)
% CO
PPM-NOx
and HC HC
CO

0 10 12 14 16 18
A/F ratio

Figure 11.20  Variation of emissions with A/F

M11_THERMAL ENGINEERING_SE_XXXX_CH11( Part-I).indd 665 6/15/2018 5:18:03 PM


666 Chapter 11

11.12.2  Evaporative Emission


As mentioned earlier, there are two main sources of evaporative emissions—the fuel tank and the
carburettor. The main factors governing the tank emissions are fuel volatility and ambient temperature
but the tank design and location can also influence the emissions as location affects the temperature.
Insulation of the fuel tank and vapour collection systems have all been explored with a view to reduce
the tank emissions.
Carburettor emission may be divided into two categories as running losses and parking losses. Most
internally vented carburettors have an external vent which opens at idle throttle position. The existing
pressure forces prevent outflow of vapours to the atmosphere. Internally vented carburettor may enrich
the mixture which, in turn, increases exhaust emission. Carburettor losses are significant only during
hot conditions when the vehicle is in operation. Fuel volatility also affects the carburettor emissions.

11.12.3  Crankcase Emission


It consists of engine blow by-gases and crank case lubricant fumes. From the point of view of pollu-
tion, blow-by gases are the most important. The blow-by is the phenomenon of leakage past the piston
from the cylinder to the crankcase because of pressure difference. The blow of HC emissions is about
20% of the total HC emission from the engine. This is further increased to 30% if the piston-rings are
worn.

11.12.4  Lead Emission


Lead emissions come only from SI engines. The lead is present in the fuel as lead tetraethyl or tetrame-
thyl, to control the self-ignition tendency of fuel-air mixtures that is responsible for knock.

11.13  ❐  CONTROL OF EMISSIONS IN SI ENGINE


An emission control programme aims at reducing the concentration of CO, HC, and NO. The main
approaches adopted are as follows:
1. Engine design modification: The following steps reduce the exhaust emission by engine design
modification:
(a) Use leaner air-fuel ratio
(b) Retard ignition timing
(c) Avoid flame quenching time by reducing the surface to volume ratio of the combustion
chamber
(d) Lower compression ratio
(e) Reduce valve overlap
(f ) Modify the induction system by using high velocity carburettors or multi-chock carburettors
2. Exhaust gas oxidation: The devices used to reduce HC/CO emissions are as follows:
(a) Use of after-burner
(b) Use of exhaust manifold reactor
(c) Use of catalytic converter
3. Fuel modification: Air-fuel ratios leaner than stoichiometric result in almost insignificant amount
of CO and reduce HC with reduced specific fuel consumption.

M11_THERMAL ENGINEERING_SE_XXXX_CH11( Part-I).indd 666 6/15/2018 5:18:03 PM


Performance of Internal Combustion Engines  667

4. Blow-by control: The crankcase blow-by control is the recirculation of the vapours back to the
intake air cleaner.
To reduce atmospheric pollution, two different approaches are followed:
1. Reduction of formation of pollutants in the emission by redesigning the engine system, fuel sys-
tem, cooling system, and ignition system.
2. Destroying the pollutants after these have been formed.
In petrol engines, the main pollutants which are objectionable and are to be reduced are HC, CO, and
NOx. The methods used are as follows:
1. Modifications in the engine design: Engine modifications improve emission quality. A few
parameters which improve an emission are as follows:
(a) Combustion chamber configuration: Modification in the combustion chamber as reduc-
ing surface/volume ratio can reduce quenching zone and reduce HC emission. This can also
be achieved by reducing dead space around piston ring.
(b) Lower compression ratio: Lower compression ratio also reduces the quenching area and
thus reduces HC emission. Lower compression ratio also reduces NOx emission due to lower
maximum temperature. However, lowering compression ratio reduces thermal efficiency
and increases fuel consumption. By using petrol of lower octane number, it is possible to
phase the lead out of petrol, that is, use of unleaded petrol.
(c) Induction system: The supply of designed A:F ratio mixture to multi-cylinder engine is
always difficult under all operating conditions of load and power. This can be achieved by
proper designing of induction system or using high velocity or multiple-choke carburettors.
(d) Ignition timing: Retarding spark ignition allows increased time for the fuel to burn. Retard-
ing the spark reduces NOx formation by decreasing NOx emission. It also reduces HC emis-
sion by causing higher exhaust temperature. However, retarding the ignition results in loss of
power and consumption of fuel. The controls are designed to retard the spark timing during
idling and provide normal spark advance during acceleration.
(e) Reduced valve overlap: Increased overlap carries fresh mixture with the exhaust and
increases emission level. This can be avoided by reducing the valve overlap.
2. Modifying the fuel used: To reduce the pollution from the exhaust of these engines, the emission
of olefins should be obviated as far as possible. For this, we can change the fuel itself. LPG and
CNG be used instead of gasoline as they produce less pollution than present petrol engines.
3. Exhaust gas treatment: Exhaust Gas Oxidation—The exhaust gases coming out of exhaust
manifold are treated to reduce HC and CO emission. A few devices are discussed below.
(a) Use of after-burner: An after-burner is a burner where air is supplied to the exhaust gases
and the mixture is burned with the help of an ignition system. The HC and CO which are
formed in the engine combustion chamber because of inadequate O2 and inadequate time
to burn are further burned by providing air in a separate box, known as an after-burner. The
after-burner is located close to the exhaust manifold with an intention that the tempera-
ture of the exhaust should not fall. The oxidation of HC in the after-burner depends on the
temperature of the exhaust and the mixing provided in the after-burner. Air injection does
nothing to NOx emission. A simple arrangement of an after-burner is shown in Fig. 11.21.
  The performance of this system was not satisfactory as combustion was not sustained
during low HC emission.

M11_THERMAL ENGINEERING_SE_XXXX_CH11( Part-I).indd 667 6/15/2018 5:18:03 PM


668 Chapter 11

Secondary
air Exhaust
Exhaust out
gases Ignition source

Figure 11.21  Typical after-burner

(b) Exhaust manifold reactor: This is a further development of the after-burner where high
temperature exhaust gases and secondary air are mixed properly and burnt. Here HC carried
with exhaust combines with O2 and forms non-objectionable gases.
  There are different types of after-burners where heat losses are minimised and sufficient
time and mixing of exhaust and secondary air are provided.
  A special after-burner designed by Du-Point, where the entry of exhaust gases is radial
and air flow is peripheral, is shown in Fig. 11.22.
(c) Catalytic converters: Catalytic oxidation of the exhausted HC and CO is accomplished by
placing a common device called catalytic converter in the vehicle exhaust system. The cata-
lytic converter is filled with catalytic material. Exhaust gas hydrocarbons and CO are oxidised
while passing through the bed. The catalytic material itself does not enter into the reaction but
only promotes the oxidation process at a lower temperature. Usually, air compressor is used
to supply additional oxygen necessary for complete oxidation of the exhaust gas stream.
  A catalyst is an agent that aids or speeds a process or a chemical reaction without becom-
ing a part of the reaction during the process—it is a sort of chemical middleman. In a mod-
ern car’s emissions control system, the so-called three-way catalyst helps the three major
evil elements of exhaust—HC, CO, and NOx—react with oxygen and each other. The cata-
lyst helps the HC and CO become non-poisonous CO2 and water vapour, whereas the NOx
is converted into CO2, nitrogen, and water vapour.

Exhaust
from engine

Clean
exhaust
out

Figure 11.22  A typical after-burner

M11_THERMAL ENGINEERING_SE_XXXX_CH11( Part-I).indd 668 6/15/2018 5:18:04 PM


Performance of Internal Combustion Engines  669

Pressure balance Pressure balance


valve To air cleaner valve From air cleaner

Conister Conister

Intake Purge control valve Intake Purge control valve


manifold Fuel tank manifold Fuel tank

To exhaust manifold To exhaust manifold


(a) (b)

Figure 11.23  Fuel-system evaporation loss control device: (a) Hot soak condition,
(b) Purging condition

4. Evaporation Emission Control Device: The purpose of this device is to collect all evaporative
emissions (vapours) and recirculating them at a proper time.
The device is shown in Fig. 11.23. It consists of an absorbent chamber, pressure balancing
valve, and purge control valve. The absorbent chamber contains charcoal which can hold the
hydrocarbon vapour before it escapes into the atmosphere. The fuel tank and carburettor float,
which are main sources of HC emission in the form of vapour, are directly connected to the absor-
bent chamber when the engine is turned off, that is, under hot soak-condition. This causes the
petrol to boil from the carburettor float and a large amount of petrol vapour comes out. All these
vapours during stopping or running the engine are absorbed in the absorber chamber.
When the absorber bed becomes saturated, the air coming out from the air-cleaner is passed
through the absorber bed and the air with vapour is passed to the inlet manifold through the purge
valve. Here, the seat of the pressure balancing valve is so located that there is direct pressure
communication between the internal vent and the top of the carburettor float, maintaining the
designed carburettor metering forces.
The operation of the purge control valve is controlled by the exhaust back pressure as shown
in Fig. 11.23. The fuel supply is cut off under idling condition and the level of HC is reduced.

11.14  ❐  CRANK CASE EMISSION CONTROL


The basic principle of a crank case blow-by control system is the recirculation of vapours back to the
inlet manifold. Figure 11.24 shows a typical closed type known as a positive crank case ventilation
(PCV) system. The gases escaping past the piston and entering into the crankcase are returned to the
inlet manifold and then to the engine. During compression, the HC enters into the crankcase due to
pressure difference between the engine cylinder and crank case (small vacuum) and the HC from the
crank case is taken back into the intake manifold during the expansion stroke and back side of the
piston compresses the gases in the crank cases.

11.15  ❐  CI ENGINE EMISSIONS


The diesel engine is used more than any other type of engine for transportation, thermal power genera-
tion, and many other industrial and agricultural applications. The exhaust emissions from combustion

M11_THERMAL ENGINEERING_SE_XXXX_CH11( Part-I).indd 669 6/15/2018 5:18:06 PM


670 Chapter 11

Carburettor Air cleaner

Air intake
Frash
air line
PCV valve

Combustion chamber

Blowby gases

Figure 11.24  Positive crank case ventilation system

in diesel engine are no different from those of combustion processes in petrol engine, the difference
being only in the level of concentration of individual pollutants. The sample of a diesel exhaust may
be free from smoke, odour, and HC or may be heavily smoke-laden, highly malodorous, and can have
heavy concentration of unburned HC.
The pollutants from a diesel engine can be classified into two types as visible and invisible emis-
sions. Visible emission is the smoke which is objected more by the public. The invisible emissions
include CO, HC, NO, SO2, partially oxidised organics (as aldehydes and ketones), and odours. An
unpleasant odour is also heavily objected by the public. Smoke and odour are not harmful to public
health but are objectionable because of their unsightliness, unbearable smell, and possible reduction
in visibility. Other invisible emissions mentioned above have similar effect on health as they are also
emitted by petrol engines.

11.15.1  Effect of Engine Type on Diesel Emission


The type of the engine and the speed of the engine are two main factors which influence the exhaust
emission from a diesel engine. It has been observed that there is a significant difference in emission
levels from different engines except the odour level.
The following observations can be summarised:
1. A two-stroke air-scavenged engine produces high HC and intermediate NOx. The smoke level
remains low at all load conditions.
2. A four-stroke medium speed engine has the lowest emissions of all constituents except high
smoke intensity.
3. A four-stroke, high speed engine has high HC emissions.
4. A turbo-charged, four-stroke engine is notably low in HC but high in NOx. The smoke level is also
considerably low compared with other engines.

Formation of Smoke and Affecting Factors


Engine exhaust smoke is the result of incomplete combustion. Smoke from exhaust is a visible indica-
tor of the combustion process within the engine. It is generated at any volume in the engine where the

M11_THERMAL ENGINEERING_SE_XXXX_CH11( Part-I).indd 670 6/15/2018 5:18:07 PM


Performance of Internal Combustion Engines  671

mixture is rich. The fuel-air ratio greater than 1.5 and at pressures developed in diesel engine produces
soot. Once soot is formed, it can burn if it finds sufficient O2; otherwise it comes out with exhaust. It
becomes visible if it is dense. The size of the soot particles affects the appearance of smoke. The soot par-
ticles agglomerate into bigger particles which have an objectionable darkening effect on diesel exhaust.
The turbo-charged engine emits less smoke with increasing fuel air ratio as the mixing is much
better in this engine as well as sufficient O2 is also available in the engine cylinder at all times.

Diesel Odour
It has been observed from the experiments that the products of partial oxidation are the main cause of
odour in diesel exhaust. This partial oxidation may due to a very lean mixture during idling or due to
wall-quenching effect. The effect of fuel-air ratio and odour is shown in Fig. 11.25.
The members of the aldehyde family are considered responsible for the pungent odours of diesel
exhaust. The aldehydes in the exhaust are found at maximum 30 ppm but it is observed that even 1
ppm can cause irritation to the nose and eyes. There is no standard method developed yet for measur-
ing the odour. However, several odour producing components such as nepthaldehyde, n-butylbenzene,
and so on, are given standard rating and trained personnel can give odour ratings for the diesel exhaust
sample by comparison.
The factors which affect the odour formation in the diesel engine are as follows:
1. Fuel-air Ratio: It is already mentioned that lean mixtures produce odours.
2. Mode of engine operation: The mode of operation of the engine affects the exhaust odour signif-
icantly. Maximum odour occurs when the engine is accelerated from idling.
3. Engine type: The odour intensity does not change with the type of engine—two-stroke or four-
stroke engine.
It is also claimed by a few researchers that the intensity of odour is reduced by additive compounds.

140

130

120
Input temperature °C

110

100

90
Smoke odour CFR engine
80 or irritation 1000 RPM
CR = 75 : 1
70

60

50
0 0.02 0.04 0.06 0.08 0.10 0.12 0.14
Fuel air ratio

Figure 11.25  Effect of fuel air ratio on odour in diesel exhaust

M11_THERMAL ENGINEERING_SE_XXXX_CH11( Part-I).indd 671 6/15/2018 5:18:08 PM


672 Chapter 11

Unburned Hydrocarbons
The concentration of hydrocarbons in diesel exhaust varies from a few ppm to a several thousand ppm,
depending on the load on the engine and its speed. The hydrocarbons in diesel exhaust are composed
of a mixture of many individual hydrocarbons in the fuel supplied to the engine as well as partly
burned hydrocarbons produced during combustion process.
During the normal operation of the engine, the relatively cold wall quenches the fuel-mixture and
inhibits the combustion, leaving a thick layer of unburned fuel air mixture over the entire surface of
the combustion chamber. The thickness of this layer depends on the combustion pressure, tempera-
ture, mixture ratio, turbulence, and residual gases in the engine at the end of the exhaust stroke. A
greater surface to volume ratio of the combustion chamber leads to the formation of greater fraction
of hydro-carbon from the quenched zone.

Carbon Monoxide
CO is formed when there is insufficient O2 to completely oxidise the fuel during combustion of fuel.
The amount of CO formed in a diesel engine is considerably lower than a petrol engine because of
supply of continuous excess air to the engine. Theoretically, diesel engine should not emit CO at all as
it always operates with excess air. However, CO is present in small quantities in diesel exhaust and this
is, possibly, due to the fact that the fuel injected during later part of injection does not find sufficient
O2 as a result of local depletion in certain parts of the combustion chamber.
The percentage of CO in exhaust varies from 0.1%−0.75%, which is easily acceptable level.

Oxides of Nitrogen
Among the gaseous pollutants emitted by the diesel engine, NOx are the most significant. In this
respect, the diesel engine is not very much behind the gasoline engine. NOx being most hazardous,
the limit is set to 350 ppm in many countries. In many diesel engines, NOx varies from a few hundreds
to 1000 ppm.
The mechanism of formation of NOx is the same as discussed in the petrol engine. The conditions
which create the highest local temperature (2000 K) and have sufficient O2 give the highest NOx con-
centration in diesel engine too.
The pre-combustion chamber diesel engines produce less NOx than a direct injection engines
because of low peak temperature. High fuel-air ratio (rich mixture), an additional fuel tends to cool
the charge, and localised peak temperature falls and reduces the NOx emission.
In addition, injection pattern, injection period, cetane number of the fuel, viscosity, and rate of
burning also affect NOx formation significantly. The effect of load on NOx emission rates for a four-
stroke normally aspirated engine and four-stroke turbo-charged engines are shown in Fig. 11.26.
The NOx emission for turbocharged engine is considerably high compared with a normally aspirated
engine at all load conditions.

Smog
The increase in the levels of SO2, SO3, NOx, and suspended particulate matter creates havoc with
the surrounding atmosphere. Out of these, SO2 is the most potent as it causes bronchitis spasms. O3
causes inflammations of the inner lining of air passages. A cocktail of these lethal substances narrows
and inflames the air passages leads to smelling in the lining of the throat and finally wheezing and
difficulty in breathing.

M11_THERMAL ENGINEERING_SE_XXXX_CH11( Part-I).indd 672 6/15/2018 5:18:08 PM


Performance of Internal Combustion Engines  673

1.6 8

1.2 6

NOx (kg/hr)

NOx (kg/hr)
0.8 4

ne
gi
en
d
rge

al
0.4 2

rm
c ha

No
e
rbo in
Tu eng
0 25 50 75 100
Load (%)

Figure 11.26  E
 ffect of load on NOx emission for four-stroke
normally and turbo-charged engines

11.15.2  Control of Emission from Diesel Engine


The main pollutants from the diesel engine as mentioned earlier are HC, CO, NOx smoke, odour, and
SO2. The methods of reducing HC, CO, and NOx from petrol engine are already discussed in detail and
the same methods are also used for reducing the pollutants from a diesel engine. Therefore, the methods
used to reduce smoke and odour which are additional pollutants from diesel engine are discussed here.

Smoke and Control of Smoke


Formation of smoke is basically a process of conversion of molecules of hydrocarbon fuels into
particles of soot. It should be noted that soot is not carbon but simply an agglomeration of very
large polybenzenoid free radicals. It is also observed that soot formation during the early part of the
actual combustion process is common to all diesel engines but is consumed during the latter part of
combustion.
Pyrolysis of fuel molecules is thought to be responsible for soot formation. Fuel heated with insuf-
ficient O2 will give carbonaceous deposits. It is believed that the ‘heavy ends’ of diesel fuel may pyro-
lyse to yield the type of smoke that is observed from a diesel engine. This is believed to be the path of
formation of polycyclic aromatic hydrocarbons (benzo-pyrene) found in soot.
Many theories have been put forward for the formation of smoke but the basic reactions leading to
the formation of smoke are not fully known.
There is hardly any successful method to control the formation of soot except the engine has to run
at lower load and maintain the engine at best possible condition.
Some methods suggested for the control of smoke are as follows.
1. Smoke-suppressing additives: It has been found that some barium compounds added in fuel reduce
the temperature of combustion and avoid soot formation. It is further observed that if the soot is found,
the barium compounds break them in very fine particles and reduce the smoke. However, berium salts
added in fuel form the deposit on engine parts and reduce the filtering capacity of the filter.
2. Fumigation: Fumigation is a method of injecting a small amount of fuel in the intake manifold. This
helps pre-combustion reactions during compression stroke and reduces the chemical delay because
the intermediate products such as peroxides and aldehydes react more rapidly with O2 than hydrocar-
bons. Reducing the chemical delay curbs thermal cracking which is responsible for soot formation.
The cracking may not even happen which is mainly responsible for soot formation when
fumigation is used because it requires about 350 kJ/mole to break C-C bond and 425 kJ/mole
to break C—H bond. The energy required may not be available due to easy oxidation during
pre-combustion reaction.

M11_THERMAL ENGINEERING_SE_XXXX_CH11( Part-I).indd 673 6/15/2018 5:18:09 PM


674 Chapter 11

3. Catalytic convertors: Catalytic convertors are not effective like in a petrol engine because of
large soot formation which interferes in the oxidation of HC, CO, and NOx. Therefore, these
catalysts have a very small effect on engine smoke. Extensive research is being carried on to use
catalysts for effective removal of soot as well as for removal of other emissions.

Odour Control
It is claimed by many manufacturers that odour additive compounds can reduce odour intensity.
However, it is observed that by using additives, there is hardly any effect on odour formation and is
carried by exhaust gases.
The control of odours by using catalysts are under development and experiments have revealed that
a few oxidation catalysts reduce odour intensity.
Unfortunately, the study of exhaust odour is hampered by lack of standard tests and standard units
to measure the intensity of odour and the type of odour.
Several nations have been undertaking studies on suppressing odours by different methods.

Cleaning Up Diesel Emissions with Plasma and a Converter


While diesel engines are more economical, they produce NOx during combustion and put engine
designers in a catch-22 situation. Presently, the permitted emission of NOx in Europe is 0.7 g /km and
it will be further reduced to 0.57 g /km. This limit is already in force in USA.
The major problem faced to reduce NOx by catalytic converter which has proved successful in conven-
tional engines cannot be applied to lean burn S.I. engines or diesel engines because these engines burn their
fuel with high excess air (A:F = 30:1) and O2 in the exhaust prevents the catalytic decomposition of NOx.
Siemens and Partners have developed an efficient exhaust gas purification process for diesel engine
(SINOx). This system comprises a catalytic converter, a control system, and a dosing device for urea.
The urea undergoes hydrolysis into CO2 and NH3, which act as a reducing agent, transforming NOx
into environmentally compatible N2 and water. However, in order to chemically reduce NOx effec-
tively in a catalytic converter, a minimum temperature of 200°C is essential—a condition that is usu-
ally met in trucks. In passenger cars, on other hand, the low efficiency of the process at temperatures
below 200°C causes problems typical of urban driving and cold start phase.

11.15.3 NOx−Emission Control
The concentration NOx in the exhaust is closely related to the peak cycle temperature. There are dif-
ferent methods by which peak cycle temperature can be reduced and NOx emission can be controlled.
There are mainly three methods which are commonly used as follows: catalyst (which is already
discussed), water injection (rarely used), and exhaust gas recirculation (EGR) method. EGR is com-
monly used to reduce NOx. This method is used in petrol as well as diesel engines. In SI engines, about
10% recirculation reduces NOx emission by 50%. Unfortunately, the consequently poorer combustion
directly increases hydrocarbon emission and calls for mixture enrichment to restore combustion reg-
ularity which gives a further indirect increase of both HC and CO.
Figure 11.27 shows the arrangement of an EGR system. A portion of the exhaust gases is recircu-
lated to the cylinder intake charge. This reduces the quantity of O2 available for combustion.
The exhaust gas for recirculation is taken as shown in Fig. 11.27 through an orifice and passed
through the control valve for the regulation of the quantity of recirculation.

M11_THERMAL ENGINEERING_SE_XXXX_CH11( Part-I).indd 674 6/15/2018 5:18:09 PM


Performance of Internal Combustion Engines  675

Exhaust gas recirculation line

Central filter valve


Air injection

Throttle valve
Re-action
zone

fuel air Combustion chamber


mixture Thermal
reactor

Piston top

Figure 11.27 EGR-system

The effect of A:F ratio on NOx emission takes EGR as a parameter as shown in Fig. 11.28. It can be seen
that maximum emission of NOx occurs during lean mixture when gas recirculation is the least effective.
On the other hand, for less emission of CO and HC, a lean mixture is preferred. About 15% recycling
reduces NOx by 80% but increases HC and CO by 50%−80%. These are conflicting requirements of this
emission control system, and can be solved by adopting a package system to control all emissions.

4000
NOx 0% Exh gas
(A/S range of
3000
A/F range of 5% fuel atoimised
NOx ppm

conventional carburetor)
2000
carburetors
10%
1000
15%
0

600
HC
HC ppm

400

200

6
CO mole %

CO
4
Stoichiometric
A:F ratio
2

0
8 10 12 12 18 18 20 22
Air-fuel ratio

Figure 11.28  Effect of recycling of gas on NOx concentration

M11_THERMAL ENGINEERING_SE_XXXX_CH11( Part-I).indd 675 6/15/2018 5:18:10 PM


676 Chapter 11

11.16  ❐  THREE-WAY CATALYTIC CONVERTER


A catalyst is a substance that accelerates a chemical reaction by lowering the energy needed for it to
proceed. It is not consumed in the reaction. A three-way catalytic converter reduces the concentration
of CO, HC, and NOx in the exhaust. A catalytic converter is usually a stainless steel container mounted
along the exhaust pipe of the engine. There is a porous ceramic (Al2O3) structure inside the container
through which the exhaust gas flows. The ceramic structure is a single honey-comb structure with
many flow passages. The ceramic passages contain small embedded particles of catalytic material
such as platinum, palladium, rhodium, and so on that promote oxidation or reduction reaction in the
exhaust gas. Platinum and palladium promote the oxidation of CO and HC, whereas rhodium pro-
motes the reaction of NOx by reduction process.
The principle of working of a three-way catalytic converter is shown in Fig. 11.29(a) and (b).

11.16.1  Function of a Catalyst in a Catalytic Converter


The catalyst controls the level of various exhaust pollutants from the engine by changing the chem-
ical characteristics of the exhaust gases. Catalyst materials such as platinum or platinum, palla-
dium, and rhodium are used in the converter. CO and HC oxidise the CO2 and H2O by palladium
and platinum and NOx is reduced by rhodium. An oxidation catalyst is placed downstream of the
reduction catalyst.

Secondary
EGR Engine
air pump
intercooler
EGR hose

HC/CO NOx
catalyst catalyst

Muffler Programmed
By-pass valve

(a)

Secondary
Reducing element air Oxidising element
for NOx for HC/CO

Exhaust Exhaust
gas in gas out

(b)

Figure 11.29 Three-way catalytic converter: (a) Catalytic converter


package, (b) Three way catalytic converter

M11_THERMAL ENGINEERING_SE_XXXX_CH11(Part-II).indd 676 6/15/2018 5:20:43 PM



Performance of Internal Combustion Engines 677

Oxidation Reactions
2CO + O2    ⇒ 2CO2
CO + H2O    ⇒ CO2 + H2
CxHy + zO2   ⇒ xCO2 + yH2O
where z = x + 0.25 y
Reduction Reactions
2NO + 2CO    ⇒ N2 + 2CO2
2NO + 5CO + 3H2O  ⇒ 2NH3 + 5CO2
2NO + CO    ⇒ N2O + CO2
2NO + 2H2   ⇒ N2 + 2H2O
2NO + 5H2    ⇒ 2NH3 + 2H2O
2NO + H2    ⇒ N2O + H2O

11.17  ❐ 
ENVIRONMENTAL PROBLEMS CREATED BY
EXHAUST EMISSION FROM IC ENGINES
The emissions exhausted into the surroundings pollute the atmosphere and cause the following
problems:
1. Global warming: The earth surrounding the atmosphere contains a 3 mm-thick layer of ozone
(O3) at 50 km from its surface in stratosphere. This layer of O3 has the specific property to absorb
ultraviolet (UV) rays emitted by the sun. If UV rays enter into the atmosphere and touch the earth,
they will destroy all human, animal, and crop life. It has been observed that the O3 layer is slowly
getting destroyed due to the use of chlorofluorocarbons (CFCs) refrigerants used for refrigera-
tion and air-conditioning purposes. The CFCs have verifying degree of ozone depletion potential
(GWP) as well. The use of fully CFCs that are considered to have high ODP have been banned.
2. Acid rain: The diesel engine emissions contain oxides of sulphur, SOx (SO2 and SO3) in varying
amounts. These emissions are dissolved in water and give rise to sulphuric acid (H2SO4). They
fall on the earth and are very harmful to human and plant life.
3. Smog: Smog means an increase in ‘morbidity’. The increase in the levels of SO2, SO3, NOx, and
suspended particulate matter create huge problem in the surrounding atmosphere. SO2 causes
bronchitis spasms. SO3 causes inflammations of the inner lining of air passages, swelling in the
lining of the throat, and difficulty in breathing.
4. Odours: The members of the aldehyde family are considered responsible for the pungent odours
of diesel exhaust. Even 1 ppm aldehydes in exhaust can cause irritation to the nose and eyes. The
intensity of the odour is reduced by additive compounds.
5. Respiratory and other health hazards

Pollutant Health hazards


(a) Carbon monoxide, Interferes with transfer of oxygen through the body, causes headache, nausea,
CO dizziness, permanent visionary damage, coma, and even death.
(b) Nitrogen oxides (NOx) Tissue changes in lungs, heart, liver, and kidney.
(Continued)

M11_THERMAL ENGINEERING_SE_XXXX_CH11(Part-II).indd 677 6/15/2018 5:20:43 PM


678 Chapter 11

(Continued )

Pollutant Health hazards


(c) Hydrocarbons (HC) Reduces visibility.
(d) Smoke Creates irritation and reduces visibility.
(e) Sulphur oxides (SOx) Thickens the blood and reduces life. When it combines with water, it forms H2SO4,
which is highly corrosive and destroys the crops, equipment, and buildings.
(f) Lead (Pb) It is poisonous.
(g) Odours Give unpleasant smell and cause unpleasant atmosphere.

11.18 ❐ USE OF UNLEADED PETROL


It is desirable that a catalytic converter has an effective life time equal to half to the car life or at least
2 lakh km. Converters lose their efficiency with age due to thermal degradation and poisoning of the
active catalyst material. Just a small amount of lead on a catalyst site reduces HC emission reduction
by a factor of two of three. Use of leaded gasoline filled two times (full tank) would completely poison
a converter and make it totally useless. Therefore, leaded gasoline cannot be used in engines equipped
with catalytic converters.

11.18.1 Use of Additives


To improve the combustion performance of fuels, some compounds called additives or dopes are used.
The requirements of an additive are as follows:
1. It must be knock-resistant, surface ignition-resistant, or both.
2. It should be stable in storage and have no adverse effect on fuel stability.
3. It should be soluble in fuel under all conditions.
4. It should be in liquid phase at normal temperature and volatile to give rapid vapourisation in
manifold.
5. It must not produce harmful deposits.
6. Its water solubility must be minimum to minimise handling losses.
The most commonly used additives are as follows:
1. Tetraethyl lead (TEL), ethylene dibromide (EDB), or ethylene dichloride (EDC) are added to TEL
to avoid lead deposits.
2. Tetra-methyl lead (TML).

Summary for Quick Revision


1. Performance parameters
pim LAN
(a) Indicated power, IP = kW
60 × 103 × n
(i) pim = indicated mean effective pressure (imep)
area of indicator diagram ( mm 2 ) × spring constant ( bar/mm)
=
length of indicator diagram (mm)

M11_THERMAL ENGINEERING_SE_XXXX_CH11(Part-II).indd 678 6/15/2018 5:20:44 PM



Performance of Internal Combustion Engines 679

dm
pbm LAN Load on dynamometer × Engine rpm ( W − S) 2 × 2 kN
(b)  Brake power, BP = kW = = kW
60 × 103 × n Dynamometer constant 60 × 103
(c)  Frictional power, FP = IP − BP
BP
(d)  Mechanical efficiency, h mech =
IP
BP
(e)  Specific output =
AL
Actual suction stroke volume at NTP
(f)  Volumetric efficiency, hvol =
Piston swept volume
m f
(g)  Brake specific fuel consumption, BSFC =
BP
m f
(h)  Indicated specific consumption, ISFC =
IP
BP
(i)  Brake thermal efficiency, (h th )b =
m f × LCV
IP
(j)  Indicated thermal efficiency, (h th )i =
m f × LCV
(h th )b or (h th )i
(k)  Relative efficiency, hrel =
ha
F m f
(l)  Fuel-air ratio, =
A m a
Actual fuel-air ratio
(m)   Relative fuel-air ratio, FR =
Stoichimetric fuel-air ratio
Stoichiometric A/F ratio
(n)  Equivalence ratio, f =
Actual A/F ratio
2. Willans line method is used to determine the FP of an engine. In this method, gross fuel consumption v’s BP
at constant speed is plotted and the line is extrapolated back to zero fuel consumption. The point where the
line cuts the BP axis gives the FP of the engine at that speed. This test is applicable to CI engines only.
3. Morse test
This is a method to determine the IP of a cylinder in a multi-cylinder engine
IP of n cylinders, IPn = BPn + FP
IP of (n − 1) cylinders, IPn−1 = BPn–1 + FP
IP of nth cylinder, IPnth = BPn − BPn–1
Total IP of engine, IPn = ∑ IPnth
4. Air standard efficiency:
1
h a = 1 − g −1 , for petrol engine (Otto cycle)
r
1  1  r g − 1 
  = 1 −  
g −1   , r = cut-off ratio for diesel cycle.
r g  r − 1  
5. Heat balance sheet on minute basis:
(a)  Heat supplied by fuel, Qs = m f × CV
(b)  Heat consumed in the system:

M11_THERMAL ENGINEERING_SE_XXXX_CH11(Part-II).indd 679 6/15/2018 5:20:50 PM


680 Chapter 11

(i) Heat equivalent of BP, Q1 = 60 × BP in kW, kJ/min


(ii) Heat carried away by cooling water, Q2 = cpw × m w (Two − Twi), kJ/min
(iii) Heat carried away by dry exhaust gases, Q3 = m g × cpg (Tge − Ta), kJ/min
where m = m a + m f = mass of exhaust gases, kg/min
Steam in exhaust gases, m s = 9 × H2 × m f kg/min
Mass of dry flue gases, m g = m – m s
(iv) Heat carried away by steam in exhaust gases,
Q4 = m s [cpw (100 − ta) + hfg + cps (tge − ts)]
(v) Heat unaccounted for, Q5 = Qs − (Q1 + Q2 + Q3 + Q4)

Multiple-choice Questions

1. Besides mean effective pressure, the data needed 3. If the approximate average mean pressures during
for determining the indicated power of an engine induction, compression, power and exhaust strokes
would include of an internal combustion engine are, respec-
(a) piston diameter, length of stroke, and calo- tively, 15 kN/m2 below atmosphere, 200 kN/m2
rific value of fuel above atmosphere, 1000 kN/m2 above atmosphere
(b) piston diameter, specific fuel consumption, and 20 kN/m2 above atmosphere, then the result-
and calorific value of fuel ant mean effective pressure, in kN/m2, is
(c) piston diameter, length of stroke, and speed (a) 765 (b) 795
of rotation (c) 800 (d) 805
(d) specific fuel consumption, speed of rotation, 4. Match List I (Performance curves, labelled A, B,
and torque C and D, for a constant speed diesel engine) with
2. For a typical automobile CI engine, for conditions List II (performance parameter) and select the cor-
of increasing engine speed, match List I with List II rect answer using the codes given below the List:
and select the correct answer using codes given List I List II
below the lists:
A 1. Smoke level
List I List II
(Performance (Tendency, B 2. Brake specific fuel
consumption
parameter) qualitatively)
A. Power output 1. Increasing and then C D 3. Brake thermal
decreasing efficiency
B. Torque 2. Decreasing and then 0 4. Brake power
% Load 100
increasing
Codes:
C. Brake specific 3. Increasing throughout
fuel consumption the range A B C D
(a) 3 4 1 2
4. Decreasing throughout (b) 3 4 2 1
the range (c) 4 3 1 2
(d) 4 3 2 2
Codes:
A B C 5. Which one of following quantities is assumed
(a) 1 2 3 constant for an internal combustion engine while
(b) 1 4 3 estimating its friction power by extrapolation
(c) 2 3 4
(d) 3 1 2 through Willans line?

M11_THERMAL ENGINEERING_SE_XXXX_CH11(Part-II).indd 680 6/15/2018 5:20:52 PM



Performance of Internal Combustion Engines 681

(a) Brake thermal efficiency (a) Morse test (b) Prony brake test
(b) Indicated thermal efficiency (c) motorist test (d) heat balance test
(c) Mechanical efficiency 11. In the context of performance evaluation of IC
(d) Volumetric efficiency engine, match List I with List II and select the
6. A gas engine has a swept volume of 300 cc and correct answer.
clearance volume of 25 cc. Its volumetric effi-
ciency is 0.88 and mechanical efficiency is 0.90. List I List II
What is the volume of the mixture taken in per (Parameter) (Equipment for
stroke? measurement)
(a) 248 cc (b) 252 cc A.  Brake power (BHP) 1.  Bomb calorimeter
(c) 264 cc (d) 286 cc
B.  Engine speed 2.  Electrical tachometer
7. The curve shown in the given Fig. 11.30 is char-
C. Calorific value of 3. Hydraulic
acteristic of diesel engines.
fuel dynamometer
What does the Y-axis represent? D.  Exhaust emissions 4. Flame ionisation
(a) Efficiency detector
(b) Specific fuel consumption
(c) Air-fuel ratio Codes:
(d) Total fuel consumption A B C D
y (a) 3 1 2 4
(b) 4 2 1 3
(c) 3 2 1 4
(d) 2 3 4 1

12. The presence of nitrogen in the products of com-


x bustion ensures that
0 Power (a) complete combustion of fuel takes place
(b) incomplete combustion of fuel occurs
Figure 11.30  Characteristic of diesel engines (c) dry products of combustion are analysed
(d) air is used for combustion
8. The correct sequence of the decreasing order of
brake thermal efficiency of the three given basic 13. A two-stroke engine has a speed of 750 rpm. A
type of IC engines is four-stroke engine having an identical cylinder
(a) four-stroke CI engine, four-stroke SI engine, size runs at 1500 rpm. The theoretical output of
two-stroke SI engine the two-stroke engine will
(b) four-stroke SI engine, four-stroke CI engine, (a) be twice that of the four-stroke engine
two-stroke SI engine (b) be half that of the four-stroke engine
(c) four-stroke CI engine, two-stroke SI engine, (c) be the same as that of the four-stroke
four-stroke SI engine (d) depend upon whether it is a CI or SI engine
(d) two-stroke SI engine, four-stroke SI engine, 14. For same power output and same compression
four-stroke CI engine ratio, as compared to two-stroke engines, four-
9. Keeping other parameters constant brake power stroke SI engines have
diesel engine can be increased by (a) higher fuel consumption
(a) decreasing the density of intake air (b) lower thermal efficiency
(b) increasing the temperature of intake air (c) higher exhaust temperature
(c) increasing the pressure of intake air (d) higher thermal efficiency
(d) decreasing the pressure of intake air 15. Which one of the following plots correctly repre-
10. The method of determination of indicated power sents the variation of thermal efficiency (y-axis)
of multi-cylinder SI engine is by the use of with mixture strength (x-axis)?

M11_THERMAL ENGINEERING_SE_XXXX_CH11(Part-II).indd 681 6/15/2018 5:20:53 PM


682 Chapter 11

Codes:
Lean A B C D
(a)
Rich (a) 1 3 2 5
(b) 1 3 2 4
(c) 1 2 3 5
Stoichiometric (d) 2 1 4 3
Mixture 17. Consider the following statements:
  I. Volumetric efficiency of diesel engines is
higher than that of SI engines.
   II.  When a SI engine is throttled; its mechanical
(b) Lean efficiency decreases.
Rich III. Specific fuel consumption increases as the
power capacity of the engine increases.
Stoichiometric  IV. In spite of higher compression ratios, the
Mixture exhaust temperature in diesel engines is
much lower than that in SI engines.
Of these statements,
(a) I, II, III, and IV are correct
(c) Lean (b) I, II, and III are correct
Rich (c) III and IV are correct
(d) I, II, and IV are correct
Stoichiometric 18. In a variable speed SI engine, the maximum
Mixture torque occurs at the maximum
(a) speed
(b) brake power
(c) indicated power
(d) Lean
Rich (d) volumetric efficiency
19. In a Morse test for a two-cylinder, two-stroke, spark
ignition engine, the brake power was 9 kW, whereas
Stoichiometric the brake powers of individual cylinders with spark
Mixture cut off were 4.25 kW and 3.75 kW, respectively.
The mechanical efficiency of the engine is
(a) 90% (b) 80%
16. Match List I with the performance curves and
(c) 45.5% (d) 52.5%
select the correct answer using the codes given
below the List: 20. Match List I (performance Parameter Y) with List II
(Curves labelled 1, 2, 3, 4, and 5 BHP vs. Y) regard-
List I List II ing a CI engine run at constant speed and select the
(Performance (Performance correct answer using the codes given below the lists:
parameters of an IC curves) List I List II
engine) (Performance (Curves BHP vs Y)
A.  Indicated power parameter Y)
1
B. Volumetric-efficiency 2 A. Total fuel consumption 5
3 rate 2 4
C.  Brake power 3
B.  Mechanical efficiency
4 1
D.  Specific fuel C.  Indicated power
5
consumption D. Brake specific fuel
consumption
Speed (rpm)
BHP

M11_THERMAL ENGINEERING_SE_XXXX_CH11(Part-II).indd 682 6/15/2018 5:20:57 PM



Performance of Internal Combustion Engines 683

Codes: Initial volume of dry gas sample 100 cc


A B C D 1. Volume after absorption in pipette con- 89 cc
(a) 5 3 4 2 taining potassium hydroxide solution
(b) 1 3 4 2
(c) 5 4 2 3 2.  Volume after absorption in pipette 84 cc
(d) 1 4 2 3 containing solution of pyrogallic acid
and potassium hydroxide
21. Match List I with List II and select the correct 3. Volume after absorption in pipette 82 cc
answer using the codes given below the lists: containing cuprous chloride solution
The percentage (by volume) of CO2 in the dry
List I List II
products was
A. Supercharging 1. Multi-cylinder engine (a) 2% (b) 5% (c) 11% (d) 18%
B.  Morse test 2.  CI engine 24. The volumetric efficiency of a well-designed SI
C. Heterogeneous 3.  Calorific value engine is in the range of
combustion (a) 40%–50% (b) 50%–60%
(c) 60%–70% (d) 70%–90%
D. Ignition quality of 4.  Aircraft engine
petrol 25. Variation of specific fuel consumption with fuel-
air ratio for spark ignition engine is represented
5.  Octane number
by which of the curves shown in Fig. 11.31?
6. Single cylinder SI 4
engine 3

Codes: 2
Sp. fuel consumption

A B C D
(a) 4 1 2 5
(b) 6 3 2 5
(c) 6 1 5 2 1
(d) 4 3 5 2
22. With respect to IC engine emissions, consider the
following statements:
  I.  Evaporative emissions have no carbon mon-
Lean 1.0 Rich
oxide and oxides of nitrogen Relative fuel-air ratio
  
II. Blow-by emissions are essentially car-
bon monoxide and suspended particulate
matter
Figure 11.31
III.  Exhaust emissions contain 100% of carbon (a) Curve 1 (b) Curve 2
monoxide, 100% of oxide of nitrogen, and (c) Curve 3 (d) Curve 4
around 50%–55% of hydrocarbons emitted 26. Exhaust emissions versus air-fuel ratio curves for
by the engine a petrol engine are shown in Fig. 11.32.
IV. There are no suspended particulate in the
exhaust
Emissions

Of these statements,
(a) I and IV are correct
(b) I and III are correct B
(c) II and III are correct
(d) I, II, III, and IV are correct
A C
23. A hydrocarbon fuel was burnt with air and the
Orsat analysis of the dry products of combustion Air fuel ratio
yielded the following data:
Figure 11.32

M11_THERMAL ENGINEERING_SE_XXXX_CH11(Part-II).indd 683 6/15/2018 5:20:59 PM


684 Chapter 11

The curve C represents If the calorific value of the fuel used is 40,000
(a) hydrocarbon (b) carbon dioxide kJ/kg, then what is the fuel consumption?
(c) carbon monoxide (d) oxides of nitrogen (a) 1.5 kg/h (b) 3.0 kg/h
27. If the performance of diesel engines of different (c) 0.3 kg/h (d) 1.0 kg/h
sizes, cylinder dimensions, and power rating are 32. A 40 kW engine has a mechanical efficiency of
to be compared, which of the following parame- 80%. If the frictional power is assumed to be con-
ters can be used for such comparison? stant with load, what is the approximate value of
(a) Swept volume the mechanical efficiency at 50% of the rated load?
(b) Air fuel ratio (a) 45% (b) 55% (c) 65% (d) 75%
(c) Specific brake fuel consumption 33. Consider the following statements:
(d) Volumetric efficiency       I. Supercharging increases the power output
28. Consider the following statements for NOx emis- and increases the volumetric efficiency
sions from IC engines:    II. Supercharging is more suitable for SI engines
   I.  Formation of NOx depends upon combustion than CI engines
temperature III. The limit of supercharging for an SI engine is
  II.  Formation of NOx depends upon type of cool- set by knock while that for a CI engine is set
ant used by thermal loading
III. Exhaust gas recirculation is an effective Which of the statements given above are correct?
means for control of NOx (a) I and III (b) I, II, and III
IV.  Activated Platinum is used for reduction of NOx (c) II and III (d) I and II
Which of the statements given above are correct? 34. Which one of the following cannot be controlled
(a) I and II (b) I, II, and III by a three-way catalytic converter?
(c) II and IV (d) I and III (a) HC emission (b) CO emission
29. Consider the following statements: (c) NOx emission (d) SPM emission
Exhaust emission of carbon monoxide from spark 35. The discharge of hydrocarbons from petrol auto-
ignition engine is mobile exhaust is minimum when the vehicle is
I.  Mainly fuel-air mixture strength dependent (a) idling (b) cruising
II.  In the range of zero to 10% (c) accelerating (d) decelerating
III. Measured with the help of an instrument
working on the principle of non-dispersive 36. What is the purpose of employing supercharging
infra-red analysis. for an engine?
IV.  Controlled by the use of a two way catalytic (a) To provide forced cooling air
convertor (b) To raise exhaust pressure
(c) To inject excess fuel for coping with higher
Which of the statements given above are correct? load
(a) I and IV (b) II and III (d) To supply an intake of air at a density greater
(c) I and III (d) I, II, III, and IV than the density of the surrounding atmos-
30. An engine using octane-air mixture has N2, phere
O2, CO2, CO, and H2O as constituents in the 37. Consider the following statements:
exhaust gas. Which one of the following can be   I. Supercharging increases the power output of
concluded? an engine
(a) Supply mixture is stoichiometric   II. 
Supercharging increases the brake thermal
(b) Supply mixture has incomplete combustion efficiency considerably
(c) Supply mixture is rich III.  Supercharging helps scavenging of cylinders
(d) Supply mixture is lean Which of the statements given above are correct?
31. An engine produces 10 kW brake power while (a) Only I and II (b) Only II and III
working with a brake thermal efficiency of 30%. (c) Only I and III (d) I, II, and III

M11_THERMAL ENGINEERING_SE_XXXX_CH11(Part-II).indd 684 6/15/2018 5:20:59 PM


Performance of Internal Combustion Engines 685

Explanatory Notes

3. (a) Resultant MEP = 1000 − 200 − (15 + 20) = 765 KN/m2


Vc + Vs
6. (c) Volumetric efficiency, h vol = = 0.88
Vs
Volume of mixture per stroke = 300 × 0.88 = 264 cc
19. (a) IP2 = 9 − 4.25 = 4.75 kW
IP1 = 9 − 3.75 = 5.25 kW
IP = IP1 + IP2 = 4.75 + 5.25 = 10 kW
BP 9
Mechanical efficiency = = = 0.9 or 90%
IP 10
23. (c) CO2 = 100 – 89 = 11 cc, O2 = 89 − 84 = 5 cc
CO = 84 − 82 = 2 cc
11
Percentage of CO2 = = 0.11 or 11%
100
BP
32. (c) h mech =
BP+FP
40
0.8 =
40 + FP
FP = 10 kW
20
At 50% rated load,h mech = = 66.67%  65%
20 + 10

Review Questions

1. Explain the method to measure the brake power of a small engine.


2. Describe the method to measure the heat lost in exhaust gases of an IC engine.
3. Explain the Morse test to measure the indicated power of a multi-cylinder engine.
4. Describe the method commonly used in laboratory to measure the air supplied to an IC engine.
5. Derive the formula used for finding the mass of air supplied to an engine using an orifice meter and tank.
6. Draw the following curves for a single-cylinder, four-stroke petrol, and diesel engines:
(a) BP v’s fuel consumption
(b) BP v’s SFC
(c) BP v’s brake thermal efficiency
7. Discuss the various performance parameters of IC engines.
8. What are the effects of load on the following?
(a) hvol (b) hmech (c) hbt (d) BSFC of SI engines
9. Define the following:
(a) BP (b) IP (c) FP (d) BSFC (e) hr
10. Define the following:
(a) Equivalence ratio (b) ISFC (c) Specific weight (d) pim
11. What are the categories of SI engine emissions?
12. Show the emissions from a four-wheeler having an SI engine on a neat sketch.
13. List the major exhaust emissions from a SI engine.
14. What is evaporative emission?

M11_THERMAL ENGINEERING_SE_XXXX_CH11(Part-II).indd 685 6/15/2018 5:21:02 PM


686 Chapter 11

15. Name four methods for control of emissions in SI engines.


16. What is an after-burner? What is it used for?
17. What is a catalytic converter?
18. What is an evaporation emission control device?
19. Name the emissions from a CI engine.
20. Define smog and fumigation.
21. Explain SOx and NOx.
22. Name the catalyst materials used in a catalytic converter.
23. What are the environmental problems created by exhaust emission from IC engines?
24. What are additives used in IC engine fuels?
25. Name four alternative fuels for IC engines.
26. Define performance number.
27. Define HUCR.
28. List four desirable properties of IC engine fuels.
29. Define octane number and cetane number.
30. What do you understand by rating of fuels?
31. What are the main pollutants emitted by SI engine?
32. What are the various types of exhaust emissions from SI engine? Discuss briefly.
33. How exhaust emissions in SI engines can be controlled? Explain briefly.
34. What are the various types of exhaust emissions from CI engine? Explain briefly.
35. Describe the methods for control of emission from diesel engine.
36. Write a short note on Nox emission control.
37. What is the function of a catalytic converter? Explain the working of a three-way catalytic converter with the
help of a neat sketch.
38. Write short notes on the following:
(a) Environmental problems created by exhaust emissions from IC engines.
(b) Use of Unleaded petrol.
(c) Use of additives in IC engine fuels.
39. Explain the various alternative fuels for IC engines.

Exercises

11.1 A single cylinder, four-stroke cycle oil engine works on diesel cycle. The following data is available from a
test:
Area of indicator diagram = 3 cm3, length of indicator diagram = 4 cm, spring constant = 10 bar/cm2-cm,
engine speed = 400 rpm, load on the brake = 380 N, spring reading = 50 N, diameter of brake drum = 1.2 m,
fuel consumption = 2.8 kg/h, LCV = 42 MJ/kg, cylinder diameter = 16 cm, piston stroke = 20 cm.
Calculate (a) the friction power (b) the mechanical efficiency (c) the brake thermal efficiency, and (d) the
brake mean effective pressure. [Ans. (a) 1.71 kW, (b) 83%, (c) 25%, (d) 6.15 bar]
11.2 A four-cylinder, four-stroke petrol engine 6 cm bore and 9 cm stroke was tested at constant speed. The fuel
supply was fixed to 0.13 kg/min and spark plugs of 4-cylinders were successively short-circuited without
change of speed.
The power measurements were as follows:
With all cylinder firing = 16.25 kW, with first cylinder cut-off = 11.55 kW,
With 2nd cylinder cut-off = 11.65 kW, with 3rd cylinder cut-off = 11.70 kW,
With 4th cylinder cut-off = 11.50 kW.

M11_THERMAL ENGINEERING_SE_XXXX_CH11(Part-II).indd 686 6/15/2018 5:21:02 PM



Performance of Internal Combustion Engines 687

 Determine (a) the indicated power of the engine, (b) the mechanical efficiency, (c) the indicated thermal
efficiency if LCV of fuel used is 42 kJ/kg, and (d) the relative efficiency on IP basis assuming clearance
volume 65 cm3.[Ans. 18.5 kW, 88%, 20.4%]
    11.3  The following readings were taken during the test on a single cylinder, four-stroke IC engine:
 Speed of engine = 240 rpm, orifice diameter of tank = 20 mm, pressure causing flow through the orifice = 10 cm
of H2O, ambient temperature = 30°C, barometer reading = 760 mm of Hg, gas consumption = 3.8 m3/h at
30°C and 10 cm of H2O above atmosphere.
 Find (a) the air consumption per hour, and (b) the A:F ratio by volume and weight. Take Cd = 0.7 for air
orifice, R = 287 J/kg.K for air and 280 J/kg.K for gas. [Ans. (a) 0.544 m3/min, (b) 85:1, 8:1]
    11.4  The following data was obtained from a test on a single-cylinder, four-stroke oil engine.
 Cylinder bore = 15 cm, stroke = 25 cm, area of indicator diagram = 540 mm2, length of indicator diagram =
50 mm, indicator spring rating = 1.2 mm for a pressure of 8.8 N/cm2, engine speed = 400 rpm, brake torque =
225 N-m, fuel consumption = 3 kg/h, calorific value of fuel = 44200 kJ/kg, cooling water flow rate = 4 kg/min,
cooling water temperature rise = 42°C, specific heat of water = 4.1868 kJ/kg.°C.
   Calculate (a) the mechanical efficiency, (b) the brake thermal efficiency, (c) the specific fuel consumption,
and (d) draw heat balance sheet.
[Ans. (a) 87.07%, (b) 25.6%, (c) 0.31 kg/kWh, (d) Qs = 36.83 kW, Qw = 11.72 kW,
Qge = 15.68 kW, Qb = 9.43 kW]
    11.5  The following particulars were obtained in a trial for 1 h on a 4-stroke gas engine:
 Speed = 16000 rpm, missed cycles = 600 rpm, net brake load = 1600 N, brake circumference = 4 m, IMEP
= 8 bar, gas consumption = 2200 litres, CV of gas = 20 kJ/litre, d = 25 cm, L = 40 cm, r = 6.5.
 Find (a) the indicated power and brake power (b) the brake specific fuel consumption, and (c) the brake ther-
mal efficiency and relative efficiency. [Ans. (a) 32.3 kW, 28.4 kW, (b) 773.4 litres/kWh, (c) 23.3%, 44%]
    11.6  A 8-cylinder, 4-stroke petrol engine of 80 mm bore and 90 mm stroke has a compression ratio of 7. It is
tested at 4000 rpm on a dynamometer which has 54 cm arm. During a 10 minute test, the dynamometer
reads 400 N and the engine consumes 4.75 kg of fuel. Air is supplied at 1 bar and 27°C at the rate of
6.5 kg/min. CV of fuel = 44 MJ/kg.
                       Determine (a) the brake power (b) the brake mean effective pressure (c) the volumetric efficiency (d) the
A:F ratio, and (e) the relative efficiency. [Ans. (a) 90.8 kW, (b) 6 bar, (c) 77.32%, (d) 13.68:1, (e) 48%]
    11.7  A six-cylinder gasoline engine operates on four stroke cycle. The bore of each cylinder is 80 mm and the
stroke is 100 mm. At speed of 4000 rpm the fuel consumption is 20 kg/h and the torque developed is 150 Nm.
Calculate (i) the brake power (ii) the brake mean effective pressure (iii) brake thermal efficiency if calorific
value of the fuel is 43000 kJ/kg. [Ans. (a) 62.83 kW, (b) 6.25 bar, (c) 26.3%]
     11.8  
An eight-cylinder, four-stroke automobile engine develops 76.25 kW brake power when tested at
4000 rpm. The compression ratio of the engine is 7. The testing is carried out for 10 minutes and fuel
consumption was 4.5 kg. CV of fuel used = 45000 kJ/kg. The air passing through the carburettor has 20°C
temperature and 1.03 bar pressure, Air measured was 5.45 kg/min. The diameter and stroke are both equal to
8.5 cm. Determine (a) the brake mean effective pressure, (b) the brake specific fuel consumption, (c) the
brake thermal efficiency, and (d) the A:F ratio.[Ans. (a) 5.93 bar, (b) 0.354 kg/kWh, (c) 57.6%, (d) 12.25:1]
    11.9  A six-cylinder, four-stroke petrol engine develops 40 kW when running at 3000 rpm. The volumetric effi-
ciency at NTP is 85% and the indicated thermal efficiency is 25%. The A:F ratio of the mixture supplied
is 15:1. The calorific value of fuel used is 41000 kJ/kg. Assume the diameter to be equal to stroke of the
engine, calculate the bore and stroke. [Ans. 76 mm, 76 mm]
11.10  A single cylinder, four-stroke cycle oil engine works on diesel cycle. The following readings were taken
when the engine was running at full load:

M11_THERMAL ENGINEERING_SE_XXXX_CH11(Part-II).indd 687 6/15/2018 5:21:02 PM


688 Chapter 11

  Area of indicator diagram = 3 cm, length of diagram = 4 cm,


spring constant = 10 bar/cm2. cm, speed of engine = 400 rpm,
  Load on the brake = 380 N, Spring reading = 50 N, diameter of brake drum = 1.2 m, fuel consumption =
2.8 kg/h, calorific value of fuel = 42000 kJ/kg, cylinder diameter = 16 cm, stroke = 20 cm. Calculate (a)
the friction power of the engine, (b) the mechanical efficiency, (c) the brake thermal efficiency, and (d) the
brake mean effective pressure. [Ans. (a) 1.71 kW, (b) 83%, (c) 25%, (d) 6.15 bar]
11.11  A four stroke petrol engine 80 mm bore 100 mm stroke is tested at full throttle at constant speed. The fuel
supply is fixed at 0.08 kg/min and the plugs of the four cylinders are successively short circuited with-
out change of speed, brake torque being correspondingly adjusted. The brake power measurements are the
following:
With all cylinder firing    = 12.5 kW
With cylinder No. 1 cut-off = 9 kW
With cylinder No. 2 cut-off = 9.15 kW
With cylinder No. 3 cut-off = 9.2 kW
With cylinder No. 4 cut-off = 9.1 kW
 Determine the indicated power of the engine under these conditions. Also determine the indicated thermal
efficiency. Calorific value of the fuel is 44100 kJ/kg. Compare this efficiency with the air standard value,
clearance volume of one cylinder is 70 × 103 mm3.[Ans. 13.55 kW, 23.04%, 40.52%]
11.12  A four-cylinder, four-stroke petrol engine, 60 mm bore and 90 mm stroke was tested at constant speed. The
fuel supply was fixed at 0.13 kg/min and plugs of 4-cylinders were successively short-circuited without
change of speed. The power measurements were as follows:
With all cylinders working = 16.25 kW
With 1st-cylinder cut-off = 11.55 kW (BP)
With 2nd-cylinder cut-off  = 11.65 kW (BP)
With 3rd-cylinder cut-off = 11.70 kW (BP)
With 4th-cylinder cut-off = 11.50 kW (BP)
 ind the IP of engine, the mechanical efficiency, the indicated thermal efficiency if CV of fuel used is
  F
42,000 kJ/kg, and the relative efficiency on IP basis assuming clearance volume 65 m3.
[Ans. 1850 kW, 88%, 20.4%, 42%]
11.13   T
 he air flow to a four cylinder, four-stroke petrol engine is measured by means of a 7.5 cm sharp-edged
orifice, Cd = 0.6. During a test on the engine, following data was recorded.
    Bore = 12 cm, stroke = 14 cm, engine speed = 2200 rpm, brake power = 35 kW, fuel consumption =
10 kg/h, LCV = 42000 kJ/kg, pressure drop across the orifice = 4 cm of water. Atmospheric temperature =
20°C, and atmospheric pressure = 1.01325 bar.
 Calculate (a) the thermal efficiency on BP basis, (b) the brake mean effective pressure, and (c) the volu-
metric efficiency based on free air conditions. [Ans. (a) 30%, (b) 1.507 bar, (c) 70.2%]
11.14  The following readings were taken during test on a single cylinder, four-stroke gas engine: speed of engine
= 240 rpm, diameter of orifice of air tank = 20 mm, pressure causing the air flow through orifice =
10 cm of water, ambient temperature = 30°C, barometer reading = 76 cm of Hg, gas consumption =
3.8 m3/h measured at 30°C and 10 cm of water above atmosphere.
 Find (a) air consumption per hour and (b) the A:F ratio by weight and by volume. Take Cda = 0.7,
R = 287 J/kg. K for air, and R = 280 J/kg.K for gas [Ans. (a) 0.544 m3/min, (b) 8.5:1, 8.1]
11.15   The following particulars were obtained in a trial on a 4-stroke gas engine:
 Duration of trial = 1 h, revolutions = 16000, missed cycles = 600, net brake load = 1600 N, brake cir-
cumference = 4 m, MEP = 8 bar, gas consumption = 2200 litres, CV of gas = 20 kJ/litre, D = 25 cm,

M11_THERMAL ENGINEERING_SE_XXXX_CH11(Part-II).indd 688 6/15/2018 5:21:02 PM



Performance of Internal Combustion Engines 689

L = 40 cm compression ratio = 6.5. Calculate the indicated power, the brake power, the brake specific fuel
consumption, the brake thermal efficiency, and the relative efficiency.
[Ans. 32.3 kW, 28.4 kW, 773.4 litres/kWh, 23.3%, 44%]
11.16  The following data is given for a four-stroke, four-cylinder, diesel engine:
 Cylinder diameter = 35 cm, piston stroke = 40 cm, engine speed = 315 rpm, IMEP = 7 bar, BP of engine
= 260 kW, fuel consumption = 80 kg/h, CV of fuel used = 43000 kJ/kg, hydrogen content in fuel = 13%
and remaining is carbon, air consumption = 30 kg/min, cooling water circulated = 90 kg/min, rise in tem-
perature of cooling water = 38°C, piston cooling oil used = 45 kg/min, rise in temperature of cooling oil
= 23°C, cp for cooling oil = 2.2 kJ/kg.K, exhaust gas temperature = 322°C, cp for exhaust gases = 1.1 kJ/
kg.K, ambient temperature = 22°C, cp of superheated steam = 2 kJ/kg.K, latent heat of steam = 2520 kJ/kg.
  Calculate (a) the mechanical and indicated thermal efficiency, (b) find bsfc when load on the engine is 50%
of full load assuming same indicated thermal efficiency, and (c) draw up heat balance sheet on minute and
percentage basis.
      [Ans. (a) 92%, 29.6%, (b) 0.308 kg/kWh, (c) Qs = 57333 kJ/min, Qb = 15600 kJ/min, Qw = 14364 kJ/min,
Qoil = 2277 kJ/min, Qge = 9858 kJ/min, Qsteam = 4540 kJ/min, Qunacc = 10694 kJ/min]

ANSWERS TO MULTIPLE-CHOICE QUESTIONS


1. (c) 2. (d) 3. (a) 4. (a) 5. (b)
6. (c) 7. (d) 8. (a) 9. (c) 10. (a)
11. (c) 12. (d) 13. (c) 14. (d) 15. (c)
16. (a) 17. (a) 18. (c) 19. (a) 20. (a)
21. (a) 22. (c) 23. (c) 24. (d) 25. (c)
26. (d) 27. (c) 28. (c) 29. (c) 30. (b)
31. (b) 32. (c) 33. (b) 34. (a) 35. (c)
36. (d) 37. (c)

M11_THERMAL ENGINEERING_SE_XXXX_CH11(Part-II).indd 689 6/15/2018 5:21:02 PM


Chapter 12

Reciprocating Air
Compressors
12.1 ❐ INTRODUCTION
Compressors are mechanical devices used for increasing the pressure of a gas. Compressors used for
producing high pressure air are called air compressors. Air is drawn from the atmosphere by suction
process, which is then compressed to the required pressure and delivered to the receiver. Air compres-
sors may be classified as shown in Fig. 12.1.

Compressor

Postive displacement type Rotodynamic type

Reciprocating Rotary Centrifugal type Axial flow type

Figure 12.1  Classification of compressors

If compression is done in a conventional cylinder with a closely fitted piston making reciprocating
motion, then the compressor is called a reciprocating compressor. External work must be supplied
from a prime mover to the compressor to achieve the required compression. The general arrangement
of the compressor and prime mover used to run the compressor is shown in Fig. 12.2.

Heat rejected
(Q − W) Heat rejected in
coolant and radiation

Prime W
Compressor Compressed air
mover Work
output
Low pressure
Heat input air
Q

Figure 12.2  Energy flow in an air-compressor

M12_THERMAL ENGINEERING_SE_XXXX_CH12.indd 690 6/15/2018 5:29:14 PM


Reciprocating Air Compressors 691

12.2 ❐ USES OF COMPRESSED AIR IN INDUSTRY


The several uses of compressed air are as follows:
1. To drive a compressed air engine
2. For producing an air blast for a workshop
3. For spraying the fuel (atomizing) into a boiler furnace
4. For operating pneumatic drills and tools
5. For operating pneumatic brakes for locomotives and rolling stock
6. Pneumatic conveying and for pumping of water by compressed air
7. For working compressed air engines especially in mines
8. In gas turbine power plants and air-conditioning plants

12.3 ❐ WORKING PRINCIPLE OF SINGLE-STAGE


RECIPROCATING COMPRESSOR
The schematic arrangement of a single-stage reciprocating compressor is shown in Fig. 12.3. It con-
sists of a cylinder with cooling jacket, piston, connecting rod, crank, suction valve, and delivery valve.
The piston will be making to-and-fro motions through the crankshaft arrangement and generally run
by an electric motor, diesel engine, petrol engine, or steam engine. During the outward motion of the
piston, the pressure inside the cylinder falls below the atmospheric pressure and the suction valve is
opened due to the pressure difference. The atmospheric air is then drawn into the cylinder until the
piston reaches the bottom dead centre position.

High pressure
Connecting
air out
Crank rod Cylinder Delivery
valve
Cooling
O jacket

Piston Cylinder head

Cooling Low pressure


jacket Suction valve air in

Figure 12.3  Single-stage reciprocating air-compressor

As the piston starts move inwards, the suction valve gets closed and the pressure starts increasing until
the pressure inside the cylinder is more than the pressure of the delivery side. Then the delivery valve
opens and high pressure air is delivered to the receiver till the piston reaches the top dead centre. At the
end of the delivery stroke, the small volume of high pressure air left in the clearance volume expands
as the piston starts moving outwards. Hence, the cycle is repeated.

12.4 ❐ TERMINOLOGY
1. Single-acting compressor: If the air admission from the atmosphere is on only one side of the
cylinder.

M12_THERMAL ENGINEERING_SE_XXXX_CH12.indd 691 6/15/2018 5:29:15 PM


692 Chapter 12

2. Double-acting compressor: When, the air from the atmosphere is drawn on both sides of the
piston.
3. Single-stage compressor: If the total compression is done fully in one cylinder.
4. Multi-stage compressor: If the compression is carried out in more than one cylinder, and every
cylinder carries out a part of the compression.
5. Free Air Delivered (FAD): It is the actual volume delivered at the stated pressure, reduced to
intake pressure and temperature, and expressed in cubic metre/minute (m3/min). FAD is used to
measure the capacity of a compressor.
6. Displacement or Swept Volume of the Compressor: The volume displaced by piston movement
between two dead centers is called displacement or swept volume. For a single acting compressor,
swept volume,
p
Vs = d 2 L, where d = diameter of piston and L = length of stroke
4

12.5 ❐ TYPES OF COMPRESSION


The theoretical p−V diagram for single acting compressor is shown in Fig.12.4. The line 4–1 rep-
resents the suction stroke. The air is then compressed adiabatically as shown by the curve 1–2
in Fig. 12.4. It is then forced out of the cylinder at constant pressure p2, as shown by the line 2–3.
The work done is represented by the area 1–2–3–4–1.
p
p2 Adiabatic
2′ 2
3 pVg = const

Isothermal
pV = const
p1
4 1

Figure 12.4  Theoretical p-V diagram for single acting compressor


If the air had been compressed isothermally, as represented by the curve 1–2′, then the work done on
the air would be the area 1–2′–3–4–1, which is considerably less than that due to adiabatic compression.
However, it is not possible in practice to compress the air isothermally because in that case, the compres-
sor would need to run extremely slow. In practice, the compressors are driven at fairly high speeds in
order to compress as much air as possible in a given time. Hence, the compression of air will approximate
to an adiabatic. The work saved by compressing isothermally is shown by the shaded area 1–2–2′–1.

12.5.1  Methods for Approximating Compression Process


to Isothermal
The following practical methods are used to achieve nearly isothermal compression:
1. Cold water spray: In this method, cold water is sprayed onto the cylinder during compression,
thus reducing the temperature of the air. Without the cold water spray, the compression would
have been adiabatic or pVg = constant. This effect is shown in Fig. 12.5, where now the compres-
sion would be between adiabatic and isothermal or pVn = const., where 1 < n < 1.4. The work
saved is represented by 1-2-2′′-1.

M12_THERMAL ENGINEERING_SE_XXXX_CH12.indd 692 6/15/2018 5:29:16 PM


Reciprocating Air Compressors 693

Isothermal
p
p2 2′ 2′′ 2 pV = const
3 Polytropic
pV n = const
Adiabatic
p1 pVg = const
4 1
V

Figure 12.5  Effect of type of compression on work done.

Work saved due


to intercooling
p
4′ 4 a pVg = const
3

3′ 2 pV = const
2′
4 1
V

Figure 12.6  Multi-stage compression

2. Water jacketing: The water is circulated around the cylinder through the water jacket to cool the
air during compression. This method is commonly used for all types of reciprocating compressors.
3. Intercooling by using multi-stage compression: In a multi-stage compressor, the air is com-
pressed in several stages. In principle, it is equivalent to number of compressors in series where
the air passes from one cylinder to the next and the pressure increases in each cylinder. The p − V
diagram for a four-stage compressor is shown in Fig. 12.6. The dotted line 1–4′ is the isothermal.
In the first stage, the air is compressed adiabatically to 2′ and cooled at constant pressure to 2′
in the intercooler, possibly to the initial temperature. For complete intercooling, the point 2′ is
on the isothermal line 1–4′. The air is then drawn into the second cylinder for the second stage
of compression and the process is repeated for the subsequent cylinders. Line 1–a represents the
adiabatic compression in a single cylinder. The compressor work saved by intercooling is repre-
sented by the area 2–a–4–3′–3–2′–2.
4. External fins: Small capacity compressors are provided with fins to increase the heat transfer
from the surface of the cylinder.

12.6 ❐ SINGLE-STAGE COMPRESSION


12.6.1  Required Work
Without Clearance
Consider the theoretical p−V diagram for a single-stage air compressor as shown in Fig. 12.7. The
work done on air per cycle is represented by the area 1–2–3–4–1.

M12_THERMAL ENGINEERING_SE_XXXX_CH12.indd 693 6/15/2018 5:29:17 PM


694 Chapter 12

p p2 2
3
pVg = const
p1
4 1

b c
a V
v2 v1

Figure 12.7 p−V diagram without clearance

     W = area (a − b − 2 − 3 − a) + area (b − c − 1 − 2 − b) − area (a − c − 1 − 4 − a)


p2V2 − p1V1
= p2V2 + − p1V1
g −1
 1 
= ( p2V2 − p1V1 ) 1 + 
 g − 1
 g 
= ( p V − p1V1 )
 g − 1 2 2

 g 
= mR(T2 − T1 )
 g − 1
 g  p V 
=  p1V1  2 2 − 1
 g − 1  p1V1 
 g −1 
 g   p2  g 
=  p1V1   − 1
 g − 1  p1  
 
 g −1 
 g   p2  g 
=
      mRT1   − 1(12.1)
 g − 1 
 1 p  
 
where m = mass of air delivered per cycle.
Work done per kg of air
 g −1 
 g   p2  g 
w =   RT1   − 1 (12.2a)
 g − 1  p1  
 

 g  T 
=
    RT1  2 − 1 (12.2b)
 g − 1  T1 

M12_THERMAL ENGINEERING_SE_XXXX_CH12.indd 694 6/15/2018 5:29:21 PM


Reciprocating Air Compressors 695

If the compression had followed the law, pVn = c, then


 n −1 
 n   p2  n 
w =  RT − 1 (12.3a)
 n − 1 1  p1 
 
 
 n  T 
       =   RT1  2 − 1(12.3b)
 n − 1  T1 
WN
Power required in driving the compressor = kW(12.4)
60 × 103
where N = number of complete cycles per minute
= rpm, if single acting
= number of strokes per minute, if double acting.

With Clearance Volume


The p-V diagram for a single-stage and single-acting air compressor is shown in Fig. 12.8 with
clearance volume.
Let Vc = clearance volume
Vs = swept volume = V1 − Vc
Va = actual volume = V1 − V4
Let the compression and expansion processes follow the same low pV n = const.
Work done per cycle,
  W = area (1–2–3–4–1)
   = area (1–2–5–6–1) − area (3–4–6–5–3)

 n −1   n −1 
 n   p2  n   p3  n 
= pV − 1 − p4V4   − 1
 n − 1 1 1  p1 
   4 
p 
   
Now, p3 = p2 and p4 = p1

p2 5 3 2
pV n = c

p1 6
4 1
V
Va
Vc Vs

Figure 12.8  p-V diagram with clearance

M12_THERMAL ENGINEERING_SE_XXXX_CH12.indd 695 6/15/2018 5:29:23 PM


696 Chapter 12

 n −1 
 n    p 
2 n 
∴ W =  p1 (V1 − V4 )   − 1
 n − 1
 p1  
 
 n −1 
 n   p2  n 
=
    p1Va   − 1(12.5)
 n − 1
 p1  
 
 n −1 
 n   p2  n 
=
    m1RT1   − 1 (12.6)
 n − 1
 p1  
 
 n  T 
=
   RT 2 − 1 for 1 kg of actual air (12.7)
 n − 1 1  T1 
where m1 = actual mass of air delivered per cycle.

12.6.2  Volumetric Efficiency
The volumetric efficiency of a reciprocating compressor is defined as the ratio of the actual free air
delivered to the swept volume of the compressor. The free air delivered is (V1–V4), whereas the swept
volume is (V1–Vc). Thus,
Va V1 − V4
hv = =
Vs V1 − Vc
Vc
Let c= = clearance ratio
Vs V 
Vs − Vc  4 − 1
(V + Vs ) − V4 Vs − (V4 − Vc )  Vc 
Then, hv = c = =
Vs Vs Vs
Now,    p3V3n = p4V4n

or    p2Vcn = p1V4n
1/ n
V p 
or 4 =  2 
Vc  p1 
 p  1/ n 
h v = 1 − c  2  − 1
Thus,   (12.8a)
 p1  
 
1/n
p 
       = 1 + c − c  2  (12.8b)
 p1 
V 
     = 1 + c − c  1  (12.8c)
 V2 

M12_THERMAL ENGINEERING_SE_XXXX_CH12.indd 696 6/15/2018 5:29:28 PM


Reciprocating Air Compressors 697

When referred to ambient conditions,


Volume of air sucked referred to ambient conditions
hv =
Sweptt volume

p1T0  V1 − V4  p1T0  p  
1/ n
      = = 1 + c − c 2 
p0T1  Vs  p0T1   p   (12.9)
 1 

Factors Affecting Volumetric Efficiency


The volumetric efficiency of a compressor can be lowered by any of the following conditions:
1. Very high speed
2. Leakage through piston seals
3. Too large a clearance volume
4. Obstruction at inlet valves
5. Overheating of air by contact with cylinder walls
6. Inertia effect of air
Figure 12.9 shows the variation in volumetric efficiency with clearance ratio (c), and the pressure
ratio (p2/p1), and polytropic index of compression (n) (by changing one factor keeping the other
two factors constant). The volumetric efficiency decreases with increase in both the clearance ratio
(c), and the pressure ratio (p2/p1), whereas it increases with increase in the polytropic index of
compression (n).

n
hv

p2/p1

c, p2/p1, n

Figure 12.9 Variation of volumetric efficiency with clearance ratio,


pressure ratio and polytropic index of compression

12.6.3  Isothermal Efficiency
The p-V and T- s diagrams for isothermal and polytropic compression of air in the compressor, respec-
tively, are shown in Fig. 12.10.
 n −1 
 n   p2  n 
Polytropic work done, Wp =  RT − 1
 n − 1 1  p1 
 
 

M12_THERMAL ENGINEERING_SE_XXXX_CH12.indd 697 6/15/2018 5:29:29 PM


698 Chapter 12

p p2 T p2
2′ 2 p1
3 pv = c

pV n = c 2
2′
p1 1
4 1
V s

Figure 12.10  Isothermal and polytropic compression in a compressor

v1
Isothermal work done Wi = p1v1 ln + p2 v2′ − p1v1
v2 ′
However,    p1v1 = p2v2′

∴ W = p v ln v1 = mRT ln  p2 
i 1 1
v2 ′ 1  p 
1
p 
= RT1 ln  2  per kg of air delivered

 p1 
Work saved,     ∆W = Wp − Wi
 n −1 
 n    p 
2 n  p 
    =   RT1   − 1 − RT1 ln  2 
 n − 1  p1 
 p1  
 
  n −1  
 n   p2  n   p2  
  = RT1     − 1 − ln    (12.10)
 n − 1  p1    p1  
   
The isothermal efficiency (hi)is a measure of the degree to which isothermal compression has been
achieved. It is defined as the ratio of isothermal work to that of actual indicated work and is given by
p 
ln  2 
Wi  p1 
Isothermal efficiency,  hi = = (12.11)
Wp   n −1 
 n  p2  n 
 n − 1  p  − 1
  1 
 
p 
p1v1 ln  2  × N
Isothermal power,  p1 
pi = kW/kg of air(12.12)
3
60 × 10
Isothermal power
Compressor efficiency,        hc = (12.13)
Indicated power

M12_THERMAL ENGINEERING_SE_XXXX_CH12.indd 698 6/15/2018 5:29:33 PM


Reciprocating Air Compressors 699

12.6.4  Adiabatic Efficiency
It is defined as the ratio of the work done on the compressor with reversible adiabatic compression to
the work done with irreversible polytropic compression.
 g −1 
 g   p2  g 
 g − 1  − 1
 p1  
Adiabatic efficiency, ha =   (12.14)
 n −1 
 n   p2  n 
  − 1
n − 1  p1  
 

12.6.5  Calculation of Main Dimensions


The actual volume of air drawn in by the compressor per stroke,
V = (V1 − V3)hv(12.15)
Capacity of a single-acting compressor,

       Q = VN = (V1 − V3 )h v N (12.16)

Q
Vs = V1 − V3 =
Swept volume,       (12.17)
Nhv
p 2
Also,     Vs = d L
4
where d = piston diameter
L = length of stroke
L
= 0.6 − 0.9
d
Therefore, L and d can be determined.

12.7 ❐ MULTI-STAGE COMPRESSION


Single-stage compression suffers from many disadvantages such as (i) handling of very high pressure
range in one cylinder resulting in leakage past the piston, (ii) ineffective cooling of the gas, and (iii)
necessitating robust construction of the cylinder to withstand the high delivery pressure.
The volumetric efficiency of a single-stage compressor with fixed clearance decreases with an
increase in pressure ratio and thus reduces the capacity. Thus the necessity of of multi-stage compres-
sion with intercooling between stages are listed below:
1. The air can be cooled perfectly at pressures intermediate between the suction and delivery pres-
sures resulting in less power required as compared to a single-stage compressor for the same
pressure limits and quantity of free air delivered.

M12_THERMAL ENGINEERING_SE_XXXX_CH12.indd 699 6/15/2018 5:29:35 PM


700 Chapter 12

2. The mechanical balance of the machine is better due to phasing of the operations.
3. The pressure range and hence the temperature range in each stage can be kept within desirable
limits. This results in:
(a) Less loss of air due to leakage past the piston.
(b) More perfect lubrication due to lower temperatures.
(c) Better volumetric efficiency.
(d) The lighter construction of the machine that reduces the cost.

12.7.1  Two-stage Compressor
The schematic arrangement of two-stage compressor with intercooler and p − V diagram are shown in
Fig. 12.11. The air is first taken into the low pressure (L.P.) cylinder at pressure p1. After compression
to intermediate pressure p2, the air at condition 2 is passed through the intercooler and leaves it at
point 3, where its temperature is reduced from T2 to T3. The air may be cooled to point 3′ such that
T3′ = T1. Finally, the air is compressed in high pressure (H.P.) cylinder from condition 3 to 4 and is
discharged to the receiver. Area 2–3–4–6–2 represents the work saved due to intercooling.
Figure 12.12 shows the p − V diagram for two-stage compression with perfect intercooling. Process
1–2 represents the polytropic compression in the L.P. cylinder with pV n1 = constant. Process 2–3 rep-
resents intercooling from T2 to T3 = T1. Process 3–4 represents the polytropic compression in the H.P.
cylinder with pV n2 = constant. Curve 1–3–4′ represents the isothermal compression.
Total work done on the air,
 n1 −1   n2 −1 
 n1   p2  n1   n2   p3  n2 
Wt =   RT1   − 1 +   RT3   − 1 (12.18a)
 n1 − 1  p1    n2 − 1  p2  
   
n −1 n −1
Let 1 = a and 2 =b
n1 n2

RT1  p2   RT  p  b 
a
Wt =  − 1 + 3  3  − 1 (12.18b)
a  p1   b  p2  
   

Water out

p Work saved due


p1 p2 p2 p3 4′ 4
Air from Inter Air to p3 6 to intercooling
atmosphere Cooler receiver
H.P pV n = c
p2 2
3′ 3 pV = c
L.P. H.P. L.P
Water in
p1
1
V
(a) (b)

Figure 12.11 Two stage compression with intercooling: (a) Schematic arrangement,


(b) p-V diagram

M12_THERMAL ENGINEERING_SE_XXXX_CH12.indd 700 6/15/2018 5:29:37 PM


Reciprocating Air Compressors 701

p
4′ 4
p3 pV n = const
5
H.P. pV = c
6 2
p2
3 pV n = const
LP.
p1 1
7
V

Figure 12.12  Two stage compression with perfect intercooling

For p1 and p3 to be constant, intermediate pressure p2 must be determined for minimum work.
Thus,
dWt
=0
dp2
n −1 n −1
T1  p2  1 T3  p3   − p3 
a + b  2  =0
a  p1  p1 b  p2   p2 
p2a −1 p3b
T1 − T3 =0
p1a p2b +1
T1 p2a + b = T3 p1a p3b
1
T  a+b
p2 =  3 ( p1a p3b )  (12.19)
 T1 
For n1 = n2 = n, and a = b.
1
1  1 2
T 
a a2   T3  a 
p2 =  3 ( p1 p3 )  =  p1 p3    (12.20)
 T1    T1  
 
For perfect intercooling, T3 = T1. Thus,
     p2 = p1 p3 (12.21)
 a a 
RT1   p1 p3   p3  
(Wt)min =   +  − 2
a   p1   p1 p3  
 
 a 
2 RT1  p3  2 
= − 1
a  p1  
 
 n −1 
 2n   p3  2n 
=  RT − 1 (12.22a)
 n − 1 1  p1 
 
 

M12_THERMAL ENGINEERING_SE_XXXX_CH12.indd 701 6/15/2018 5:29:41 PM


702 Chapter 12

 n −1 
 2 n    p 
3 2n 
=   p1v1   − 1 (12.22b)
 n − 1
 p1  
 
In general, with i number of stages, we have
p2 p3 p4 p
= = = ... = i +1
p1 p2 p3 pi
 n −1 
 in   pi +1  in 
Minimum shaft work, Wt =  RT − 1  (12.23)
 n − 1 1  pi 
 
 
The minimum indicated powers (I.P.), with imperfect intercooling is:
 n −1 
 n    p3  2n  1
I.P. =   mR  2 T1T3   − (T1 + T3 )  (12.24)
×
 n − 1  p1  3
  10
 
where m = mass of air delivered in kg per second
The number of stage is decided by the delivery pressure for a given inlet pressure (1 bar) as follows:
 1. For delivery pressure upto 5 bar: Single stage compressor
2. For delivery pressure between 5 to 35 bar: Two stage compressor
3. For delivery pressure between 35 to 85 bar: Three stage compressor
4. For delivery pressure more than 85 bar: Four stage compressor

12.7.2  Heat Rejected to the Intercooler


Let m = mass of air in the cylinder
Then  p1V1 = mRT1
pV
or      m = 1 1 
From the compression 1–2, we have RT1

p1V1n = p2V2n
1
 p n
V2 = V1  1  
or    
 p2 
Then p2V2 = mRT2
pV
or     T2 = 2 2 
mR
From the constant pressure line 2–3, we have
p2V2 p3V3
=
T2 T3

M12_THERMAL ENGINEERING_SE_XXXX_CH12.indd 702 6/15/2018 5:29:45 PM


Reciprocating Air Compressors 703

 pV 
T3 = T2  3 3  
 p2V2 
Heat rejected to intercooler = mcp (T2 − T3)(12.25)

12.7.3  Cylinder Dimensions
For steady-state flow, the mass of air passing through each cylinder per stroke must be same.
Let Va = actual volume of air per stroke taken during suction
r = density of air
Then,      Va1 r1 = Va2 r 2 = Va3 r 3 = ... = const.
Va1 p1 Va2 p2 Va3 p3
or = = = ...
RT1 RT2 RT3
For perfect intercooling,      T1 = T2 =T3 =…
∴   Va1 p1 = Va3 p2 = Va3 p3 = ...

Vs1h v1 p1 = Vs2 h v2 p2 = Vs3h v3 p3 = ...


where hv = volumetric efficiency of a cylinder
Vs = stroke volume of the cylinder
p 2 p p
d L h p = d 2 L h p = d 2 L h p = ...
4 1 1 v1 1 4 2 2 v2 2 4 3 3 v3 3
or d12 L1h v1 p1 = d22 L2h v2 p2 = d32 L3h v3 p3 = ...(12.26a)

If h v1 = h v2 = h v3 = ... and L1 = L2 = L3 = ...(12.26b)

Then d12 p1 = d22 p2 = d32 p3 = ...(12.26c)

12.7.4  Intercooler and Aftercooler


Intercooler
An intercooler is a simple heat exchanger in which heat is removed from the air after it has been com-
pressed and its temperature has risen as a result of compression. The intercooler commonly used is of
the counter flow type as shown in Fig. 12.13 because it gives high effectiveness.
Effectiveness of intercooler,
Actual drop in temperature of air
e=
Maximum possible drop inn temperature of air

T2 − T3 T −T
= 2 3 (12.27)
Tw 2 − Tw1 T2 − T1
A simple section showing the principles of construction of an intercooler is shown in Fig. 12.14. The
coolant, which may be water or any other fluid, passes through the tubes secured between two end

M12_THERMAL ENGINEERING_SE_XXXX_CH12.indd 703 6/15/2018 5:29:49 PM


704 Chapter 12

Intercooler
Air in after compression Cooled air out
water out water in

T2
TW2 Air
T1
Water
TW1

Figure 12.13  Counter flow intercooler

Air in Air out


A B

C
Coolant out

Coolant in
D

Tube Baffle
Tubes
plate

Figure 12.14  Principles of construction of inter-cooler

plates and the air circulates over the tubes through a system of baffles. Two passes are used for water
flow and air is made to flow partly parallel and partly cross with the help of baffles. This type of inter-
cooler gives better effectiveness than the ordinary counter-flow type. The purpose of the intercooler is
to reduce the work done on the air.

Aftercooler
An aftercooler is used to cool the air coming out from the compressor before it enters the receiver. The
air coming out from the compressor at pressure p3 is sufficiently hot at temperature T4 (Fig. 12.15). If
this air is cooled in the aftercooler, then the pressure will remain p3, but the temperature will fall from
T4 to T4′′. Therefore, the volume of air leaving the aftercooler will be given by,
p4 ″ v4 ″ p4 v4
= .
T4 ″ T4
However,   p4′′ = p4.
 T ″
Thus,    v4 ″ = v4  4 
 T4 
As T4′′ < T4, v4′′ < v4
Therefore, the purpose of the aftercooler is to reduce the size of the receiver. The position of the
intercooler and aftercooler is shown in Fig. 12.15.

M12_THERMAL ENGINEERING_SE_XXXX_CH12.indd 704 6/15/2018 5:29:51 PM


Reciprocating Air Compressors 705

p1 p2 T3 p3
Inter
cooler p2 T4
T1 T2

d1 d2 After
cooler

T4″
Receiver

Figure 12.15  Position of inter-cooler and after-cooler

12.8 ❐ INDICATED POWER OF A COMPRESSOR


pm LAN
Indicated power, I.P. = kW (12.28)
60 × 103
where A = area of the cylinder, m2
L = length of stroke, m
N = rpm of compressor crank
  pm = mean effective pressure of air, Pa (N/m2)
Theoretically, m.e.p. for a single-acting, single-stage compressor is,
 n −1 
 n    p 
2 n 
pm =   p1h v   − 1 (12.29a)
 n − 1
 p1  
 
where hv = volumetric efficiency of compressor
Using indicator card, the m.e.p. is:

Area of indicator card, cm 2


pm =
    N/m 2 (12.29b)
2
Measured length of the indicator card, cm × Vertical scale, N/m /cm
Indicated power of compressor
Mechanical efficiency, h mech = (12.30)
Brake power of compressor

12.9 ❐ AIR MOTORS


The working of an air motor is similar to that of air compressor. High pressure air is admitted to the
motor cylinder through a mechanically operated inlet valve and drives the piston in the forward direc-
tion. After a part of the stroke of the piston has been performed, the air supply is cut-off and the stroke
is completed after decreasing pressure as the air expands in the cylinder. After the expansion stroke
is completed, the air is allowed to escape into the atmosphere through a mechanically operated dis-
charge valve. The return stroke is performed by compressed air acting on the other side of the piston
in double-acting cylinder.
Figure 12.16 shows the p − V diagram for the air motor with clearance. Process 4–1 is the intake
with the high pressure p2 at cut-off point 1. From 1 to 2, the air expands from higher pressure p2 at 1
to atmospheric pressure p1 at 2, according to the polytropic law, pVn = c. The process 2–3 represents
the exhaust.

M12_THERMAL ENGINEERING_SE_XXXX_CH12.indd 705 6/15/2018 5:29:53 PM


706 Chapter 12

p
p2 4 1 pV n = c

p1
3 2
V

Figure 12.16 p−V diagram for an air motor

Work obtained neglecting clearance,


 n −1 
 n    p2  n 
W =  p V 1−  (12.31a)
 n − 1 1 1   p1 
 
 
 n −1 
 n    p2  n 
  = mRT1 1 −   (12.31b)
 n − 1
  p1  
 
Work done considering clearance,
 n −1 
 n    p2  n 
′V=  p (V − V ) 1 −  (12.32a)
 n − 1 1 1 4   p1 
 
 
 n −1 
   =  n  m RT 1 − p2
  n 
 
n − 1 1 1   p1   (12.32b)

 
where m1 = actual mass of air delivered per cycle.

12.10  ❐  INDICATOR DIAGRAM


The theoretical of p − V diagram for the single-stage reciprocating compressor is represented by
1–2–3–4–1 in Fig. 12.17. The actual indicator diagram is 1–2′–3–4′–1. The difference between the
actual and theoretical indicator diagrams is due to the intake and discharge losses. The intake losses
include the friction losses in pipe, friction loss in inlet valve, and valve inertia loss. Theoretically, the

p 2′ Energy required to
3 force the air out
2

Energy required to
4 draw the air in
1
4′
V
VC VS

Figure 12.17  Actual p−V diagram for single stage compressor

M12_THERMAL ENGINEERING_SE_XXXX_CH12.indd 706 6/15/2018 5:29:56 PM


Reciprocating Air Compressors 707

p
p2
H.P.
Intercooler losses
L.P.
p1

Figure 12.18  Actual p−V diagram for two-stage compressor

inlet valve should open at 4, but actually, it opens little afterwards at 4′ due to the inertia of the valve,
and the pressure inside the cylinder falls below the atmospheric pressure. The oscillating part of the
curve indicates valve flutter due to vibration of spring loaded valve. On the delivery side, the discharge
valve should open at 2, but actually, it opens little afterwards at 2′ due to the inertia of the spring
loaded discharge valve. The effect of these losses is to increase the work required by the compressor.
The actual indicator diagram for two-stage compressor with intercooler is shown in Fig. 12.18.

12.11  ❐  HEAT REJECTED


If the air is cooled to the initial temperature, then there is no change in internal energy per kg mass of
air, and all the work done is rejected to the cooling medium party during the compression process and
the remaining after compression at constant pressure.
Now, q1–2 = du + w1–2
However,      du = 0
For a single-stage compressor, the heat rejected is given by work done − ∫ vdp.
 n −1 
Heat rejected,  n   p2  n 
q1− 2 = w1− 2 = p v − 1
 n − 1 1 1  p1  
 
 
n −1
T p  n
Also, p1v1 = RT1 and 2 =  2 
T1  p1 
 n  T 
∴          q1− 2 = w1− 2 =   RT1  2 − 1
 n − 1  T1 
 n 
   =  R(T2 − T1 )(12.33a)
 n − 1
 c (g − n) 
      = c p + v (T − T )(12.33b)
 n − 1  2 1
     = Heat rejected to the coolant in the intercooler + Heat
rejected to the coolant during compression process

12.12  ❐  CONTROL OF COMPRESSOR


In order to balance the demand and supply of air, it is necessary to incorporate devices for the com-
pressor control. The common methods of control are as follows:

M12_THERMAL ENGINEERING_SE_XXXX_CH12.indd 707 6/15/2018 5:29:59 PM


708 Chapter 12

1. Throttle control
2. Clearance control
3. Blowing air to waste
1. Throttle control: When the demand is less, there is build-up of pressure in the receiver, and
the high pressure air from the receiver is led to piston and cylinder. The movement of piston is
resisted by a spring. However, with excessive pressure, the piston depresses the spring, thus clos-
ing partly the suction valve. Therefore, during the suction stroke, the air intake is partly throttled.
The reverse action takes place when the pressure in the receiver drops due to increase in demand.
2. Clearance control: In this method, clearance pockets are provided for increasing or decreasing
clearance. Therefore, the volumetric efficiency is reduced in proper proportion to control output.
3. Blowing air to waste: In case of excessive pressure built-up in the receiver due to decrease in
demand, a by-pass valve from the high pressure cylinder delivers air directly to atmosphere. When
the pressure in the receiver drops, the relay piston closes the valve.

Example 12.1
A single-stage reciprocating air compressor is required to compress 72 m3 of air per minute from
15°C and 1.0 bar to 8 bar pressure. Find the temperature at the end of compression, work done,
power, and heat rejected during each of the following processes: (a) isothermal, (b) adiabatic, and
(c) polytropic compression following the law pV1.25 = constant.

Solution
(a) Isothermal compression:
p1V1 = p2V2
1 × 72
V2 = = 9 m3/min
8
T2 = T1 = 273 + 15 = 288 K
V1  72 
Work done, Wiso = p1V1 ln = 1 × 105 × 72 ln   = 150 × 105 J/min
V2  9

150 × 105
Power,   Piso = = 250 kW
60 × 103
Heat rejected,     Q = W = 250 kJ/s p1V1g = p2V2g [∵ dU = 0]
(b) Adiabatic compression:
p1V1g = p2V2g ∵
g −1
0.4
T2  p2  g
= = (8) 1.4 = 1.81
T1  p1 
T2 = 2.88 × 1.81 = 521 K
 g −1 
 g   p2  g 
Work done, Wadi =  p1V1   − 1
 g − 1  p1  
 

M12_THERMAL ENGINEERING_SE_XXXX_CH12.indd 708 6/15/2018 5:30:01 PM


Reciprocating Air Compressors 709

 1.4 −1 
 1.4 
= × 1 × 10 × 72 (8) 1.4 − 1 = 204.5 × 105 kJ/miin
5 
     0.4   
 
204.5 × 105
  Power, Padi = = 340.8 kW
60 × 103
  Heat rejected = 0
(c) Polytropic compression:
p1V11.25 = p2V21.25
n −1
0.25
T2  p2  n
= = (8) 1.25 = 1.5157
T1  p1 
T2 = 288 × 1.5157 = 436.53 K
 n −1 
 n   p2  n 
  Work done,      W poly =  pV − 1
 n − 1 1 1  p1 
 
 
 1 .25 −1 
 1.25  5  .25 − 1 = 185.66 × 105 kJ/min
       =  0.25  × 1 × 10 × 72 (8)
1

 
185.66 × 105
  Power,        Ppoly = = 309.43 kW
60 × 103
 g − n  1.4 − 1.25 
 = 
  Heat rejected    ×Wp =  × 185.66 × 105
 g −1   1.4 − 1 
= 6962.2 kJ/min
Note: Example 9.1 illustrates that isothermal compression requires the minimum compression work,
while isentropic compression requires the maximum for the same suction and delivery pressure.

Example 12.2
A double-acting reciprocating compressor with a piston displacement of 0.05 m3 per stroke operates
at 500 rpm. The clearance is 5 per cent and it receives air at 100 kPa, discharges it at 600 kPa. The
compression is polytropic, pV1.35 = c. Determine the power required and the air discharged in m3/s.
Solution
The p–V diagram for the compressor is shown in Fig. 12.19.
 n −1 
 2n   p2  n 
W = pV − 1
 n − 1 1 a  p1 
 
 
Va = Vc + Vs − V4

Vc = 0.05 × 0.05 = 0.0025 m3

M12_THERMAL ENGINEERING_SE_XXXX_CH12.indd 709 6/15/2018 5:30:05 PM


710 Chapter 12

p
3 2
600 kPa
pV n = c

100 kPa 1
4

Va

V
Vc Vs

Figure 12.19  p–V diagram

1 1
 p n  600  1.35
V4 = Vc  3  = 0.0025  = 0.0094 m3
 4
p  100 
        
Va = 0.0025 + 0.05 − 0.0094 = 0.0431 m3
 0.35 
 2 × 1.35   600  1.35 
W = × 100 × 0.0431  − 1
 0.35   100 

 
= 19.659 kJ/cycle
500
Power required = 19.659 × = 163.823 kJ/s or kW
60
N
Volume discharged = 2(V2 − V3 ) ×
60
1 1
 p n  100  1.35
V2 = V1  1  = 1.05 × 0.05  = 0.0139m3
 p2   600 
V3 = Vc
500
Volume discharged = 2(0.0139 − 0.0025) × = 0.19 m3/s
60

Example 12.3
A double-acting reciprocating compressor with complete intercooling delivers air to the main at
a pressure of 30 atm, the suction condition being 1 atm and 27°C. If both cylinders have the same
stroke, then find the ratio of the diameters of the cylinders for the efficiency of compression to be
a maximum. Assume the index of compression to be 1.3.

Solution
Volume of L.P. cylinder = V1
Volume of H.P. cylinder = V3
L1 = L3

M12_THERMAL ENGINEERING_SE_XXXX_CH12.indd 710 6/15/2018 5:30:08 PM


Reciprocating Air Compressors 711

∴ V1 d12
=
V3 d32
d1 V
or   = 1
d3 V3
Now,        pV1.3 = c
     p1V11.3 = p 2 V21.3
1
V1  p2  1.3
or =
V2  p1 
For maximum efficiency,
p2 = p1 p3 = 1 × 30 = 5.477 atm
1
V  5.477  1.3
1 =   = 3.699
V2  1 
n −1
T2  p2  n
= ,
T1  p1 
0.3
 5.477  1.3
T2 = 300   = 444 K
 1 
From constant pressure process 2–3: (see Fig. 12.12)
V T T
3 = 3 = 1 for perfect cooling
V2 T2 T2
300
  =
444
V2
or = 1.48
V3
V1 V1 V2
= × = 3.699 × 1.48 = 5.475
V3 V2 V3
d1
= 3.475 = 2.34
d3

Example 12.4
A double-acting reciprocating compressor with perfect intercooling takes in air at 1 bar and 27°C.
The law of compression in both the stages is pV1.3 = constant. The compressed air is delivered at
9 bar from the H.P. cylinder to an air receiver. Calculate per kg of air (a) the minimum work done,
(b) the heat rejected to the intercooler, and (c) the minimum work done in a three-stage compressor
working under the same conditions. Take cp = 1.005 kJ/ kg.K.

M12_THERMAL ENGINEERING_SE_XXXX_CH12.indd 711 6/15/2018 5:30:12 PM


712 Chapter 12

Solution
T1 = 273 + 27 = 300 k
For 1 kg of air,     p1v1 = RT1,
287 × 300
or   v1 = = 0.861 m3/kg
1 × 105
(a) The minimum work required in two-stage compressor with perfect intercooling is,
 n −1 
 2n   p3  2n 
W = pv − 1
 n − 1 1 1  p1 
 
 
 0.3 
 2 × 1.3  5  9  2.6 
= × 2 × 10 × 0 . 861 − 1
 0.3   1  
 
= 215.3 kJ/kg of air
The intermediate pressure is found to be
(b) p2 = p1 p3 = 1 × 9 = 3 bar
n −1 0.3
T2  p2  n  3  1.3
= =  = 1.289
T1  p1   1
T2 = 300 × 1.289 = 386.6 K
Heat rejected to intercooler,
  q = cp (T2 − T3) = cp (T2 − T1) [∵T3 = T1 ]
= 1.005(386.6 − 300) = 87 kJ/kg of air
(c) The least work done in case of three-stage air compressor working between the same pressure
limits is given by,
 n −1 
 3n   p4  3n 
  W =  pV − 1
 n − 1 1 1  p1 
 
 
p4 = 9 bar, p1 = 1 bar
 0.3 
 3 × 1.3  5  9  3.9 
W = × 1 × 10 × 0. 861 − 1
 0.3   1  
 
= 206 kJ/kg of air

Example 12.5
A single-stage, single-acting reciprocating compressor delivers 15 m3/min of free air from 1 bar to
8 bar at 300 rpm. The clearance volume is 6% of the stroke volume, and compression and expan-
sion follow the law pV1.3 = constant. Calculate the diameter and stroke of the compressor. Take
L = 1.5 D. The temperature and pressure of air at suction are the same as atmospheric air.

M12_THERMAL ENGINEERING_SE_XXXX_CH12.indd 712 6/15/2018 5:30:15 PM


Reciprocating Air Compressors 713

Solution
Clearance ratio, c = 0.06
1
 p n
Volumetric efficiency, hv = 1 + c − c  2 
 p1 
1
 8  1.3
= 1 + 0.06 − 0.06  
 1
= 0.763 or 76.3%

FAD
Now hv =
Vs × N

FAD 15 3 FAD
or Vs = h × N = 0.763 × 300 = 0.06553 m h v = V × N
v s

p 2
Also,  
   D L = Vs
4
p 2 p 2
       D × 1.5 D = 0.06553  [∵ LD= 1×.15.D
5D] = 0.06553 [∵ L = 1.5 D ]
4 4
D = 0.3817 m or 38.17 mm
   
L = 1.5 × 0.3817 = 0.5726 m or 57.26 mm

Example 12.6
A two-cylinder, single-acting reciprocating air compressor is to deliver 15 kg of air per minute at
6.5 bar from suction conditions 1 bar and 13°C. Clearance may be taken as 4 per cent of stroke
volume and the index for both compression and re-expansion as 1.3. The compressor is directly
coupled to a four-cylinder, four-stroke petrol engine that runs at 1800 rpm with b.m.e.p. of 6 bar.
Assuming a stroke–bore ratio of 1.1 for both engine and compressor and a mechanical efficiency of
85% for the compressor, calculate the required cylinder dimensions. Take R = 287 J/kgK.

Solution
The p−V diagram for the compressor is shown in Fig. 12.20.
15
Amount of air delivered per cylinder = = 7.5 kg/min
2
Now,   p1(V1 − V4) = mRT1
7.5 × 287 × 286
V1 − V4 = = 6.156 m3 /min
1 × 105
6.156
= = 3.42 × 10 −3 m3 /stroke
1800
From the expansion curve, we have
1 1
V4  p3  n  6.5  1.3
= =  = 4.22
V3  p4   1 

M12_THERMAL ENGINEERING_SE_XXXX_CH12.indd 713 6/15/2018 5:30:20 PM


714 Chapter 12

V3 = 0.04 Vs
V4 = 0.1688 Vs
Now, V1 = V3 + Vs = 0.04 Vs + Vs = 1.04 Vs
V1 − V4 = (1.04 − 0.1688) Vs = 0.8712 Vs = 3.42 × 10−3
Vs = 3.9256 × 10−3 m3
p 2
D L = Vs
4
p 2
D × 1.1D = 3.9256 × 10 −3
4
D = 0.1656 m or 165.6 mm
L = 1.1 D = 0.1822 m or 182.2 mm
Work done per minute
Now, I.P. of the compressor =
60 × 103
 n −1 
 n   p2  1 
Work done per minute =  mRT1   − 1
 n − 1  p 
 1 
 
 0.3 
1.3  6.5  1.3 
  = × 15 × 287 × 286   − 1
0.3  1 
 

 = 2882491 N.m/min
2882491
Indicated power of compressor = = 48.04 kW
60 × 103
I.P. 48.04
Brake power of engine = = = 56.52 kW
h mech 0.85

p
3 2
6.5 bar
pV n = c

1 bar 1
4

Va

V
Vc Vs

Figure 12.20  p-V diagram

M12_THERMAL ENGINEERING_SE_XXXX_CH12.indd 714 6/15/2018 5:30:23 PM


Reciprocating Air Compressors 715

( b.m.e.p.) × LAN × i
Now, B.P. =
60 × 103
5
6 × 105 6× ×
( LA ( LA )××41800 × 4
10 ) × 1800  1800  1800 
56.52 =56.52 =  N == 4, N =
∵i = 4,∵i
360 × 103 × 2
60 × 10 × 2  2  2 
LA = 15.70 × 10−4 m3
p 2
D × 1.1D = 15.70 × 10 −4
4
D = 0.122 m or 122 mm
L = 0.134 m or 134 mm

Example 12.7
A multistage reciprocating compressor has to be designed to supply air at 135 bar, while atmos-
pheric condition is 1.03 bar and 15°C. The value of compression index may be assumed as 1.35.
Due to practical reasons, the intercoolers are not able to cool the air below 45°C, while the maxi-
mum temperature allowable in the system is 120°C. Calculate the number of stages that are neces-
sary in the compression and the rate of cooling water circulated per kg of air. Take cp = 1 kJ/kg. K.
Solution
The p−V diagram for the compressor is shown in Fig. 12.21.
T1 = 273 +15 = 288 K, p1 = 1.03 bar, n =1.35
Since the maximum allowable temperature is 120°C, the temperature after first stage of
compression,
T2 = 273 + 120 = 393 K
n 1.35
p2  T2  n −1  393  0.35
= , p2 = 1.03  = 3.416 bar
p1  T1   288 

n 1.35
p2  T2  n −1  393  0.35
=
or       , p2 = 1.03  = 3.416 bar
p1  T1   288 

The intercoolers cool the air from T2 = 393 K to T3 = 273 + 45 =318 K. For the second stage, the inlet
temperature is T3 = 318 K and outlet temperature is T4 = 393 K (see Fig. 12.10b).

p Work saved due


4′ 4 6 to intercooling
p3

H.P pV n = c
p2 2
3′ 3 pV = c
L.P
p1
1
V

Figure 12.21  p-V diagram

M12_THERMAL ENGINEERING_SE_XXXX_CH12.indd 715 6/15/2018 5:30:25 PM


716 Chapter 12

n −1
Toutlet  poutlet  n
For each stage, =
Tinlet  pinlet 
n n
 T4  n −1  T  n −1
For the second-stage compression, outlet pressure p4 = p3   p[ 4

= p3 4  ∵ p3= p2]
 T3   T3 
n 0.135
 T  n −1  393  0.35
= p2  4  = 3.416  = 7.73 bar
T 
3  318 

p4 7.73
Pressure ratio = = = 2.26
p3 3.416
The pressure ratio is same for all the subsequent stage till the pressure reaches 135 bar. Hence, the
number of stage i can be calculated from,
135
( 2.26)i =
3.416
i ln 2.26 = ln 3.952
i = 4.51
n
Therefore, the minimum number of stages required including the first stage
 T4  n −1
p4 = p3= 4.51 + 1 = 5.51
∵ 6
 T3 
Heat received by stage coolers per kg of air compressed
     = cp (T2 − T3) (i − 1) = 1(393 − 318) × 5 = 375 kJ
375
Cooling water circulated per kg of air = = 89.7 kg
4.18

Example 12.8
A single-acting, two-stage reciprocating compressor with complete intercooling delivers 10 kg/min
of air at 16 bar. The suction occurs at 1 bar and 27°C. The compression and expansion processes are
reversible with polytropic index n = 1.3. The compressor runs at 450 rpm. Calculate the following:
(a) The power required to drive the compressor.
(b) The isothermal efficiency.
(c) The free air delivered.
(d) The heat transferred in intercooler.
(e) The swept and clearance volumes for each cylinder if the clearance ratios for L.P. and H.P.
cylinders are 0.04 and 0.06, respectively.
Solution
The p − V diagram is shown in Fig. 12.22.
p1 = bar, p3 = 16 bar, T1 = 273 + 27 = 300 K, N = 450 rpm, n = 1.3, c1 = 0.04, and c2 = 0.06
For perfect intercooling, p2 = p1 p3 = 1 × 16 = 4 bar

M12_THERMAL ENGINEERING_SE_XXXX_CH12.indd 716 6/15/2018 5:31:14 PM


Reciprocating Air Compressors 717

p
7 6
16 bar

H.P.
3 5
4 bar 2
8
L.P.

1 bar 1
4

V
(VS)HP
(VC )HP
(VC )LP (VS)LP

Figure 12.22  p − V diagram for two-stage compressor

(a) Work done in two stages with perfect intercooling,


 n −1 
 2n   p3  2n 
W =  mRT1   − 1
 n − 1
 p1  
 
 0.3 
2 × 1.3  16  2.6 
= × 10 × 0.287 × 300   − 1
0.3  1
 
          = 2813.24 kJ/min
W 2813.24
Power required to drive the compressor, Pp = =
60 60
= 46.887 kW
(b) Isothermal power,
p3 10
Pi = mRT1 ln = × 0.287 × 300 × ln 16 = 390.787 kW
p1 60
Pi 39.787
Isothermal efficiency = = × 100 = 84.58%
Pp 46.887

mRT1 10 × 0.287 × 300


(c) Free air delivered = = = 8.61 m3 /min
p1 2
1 × 10

n −1 0.3
p  n  4  1.3
(d) T2 = T1  2  = 300   = 413.1 K
 p1   1

M12_THERMAL ENGINEERING_SE_XXXX_CH12.indd 717 6/15/2018 5:31:17 PM


718 Chapter 12

  Heat transferred in intercooler with perfect intercooling


10
= mc p (T2 − T3 ) = × 1.005( 413.1 − 300) = 18.44 kW
60
1 1 1 1 1 1
 p n  p2  n  p24n 1.3  4  1.3  4  1.3
(e) h v = 1 + c1 −hc1v = 12+ ch1v− 1 −−c10=.04
==1c11++0.c04 1 +0 .04
 =−10=
+.049238
0.04 .04=
 − 0or 92 8% =or0.92
0.3.9238 .38%
9238 or 92.38%
 p 
1    p 1
1 p 1  1 1
FAD FAD FAD
h v =
Now        hv = hv =
Vs × N Vs × N Vs × N
FAD FAD
15 FAD15 15
s =
or       
V V=s = Vs == = 0.06553 = m=3 0.06553 0.306553 m3
=m
h v × N 0.h 763
v ××N h
300 0
v . ×
763
N × 300
0 .763 × 300
FAD
Swept volume, Vs = V1 − V3 =
N × hv
8.61
= = 0.0207 m3
450 × 0.9238
Clearance volume = c1 (V1 − V3) = c1Vs = 0.04 × 0.0207
= 8.285 × 10−4 m3
(f) H.P. stage:
1
 p n
h v = 1 + c2 − c2  3 
 p2 
1
= 1 + 0.06 − 0.06 ( 4 )13 = 0.8857 or 88.57%
FAD
Swept volume,    Vs = V5 − V7 =
Stage pressure ratio × N × h v
8.61
              = = 5.4 × 10 −3 m3
4 × 450 × 0.8857
Clearance volume = c2Vs = 0.06 × 5.4 × 10−3 = 3.24 × 10−4 m3

Example 12.9
A reciprocating compressor is to be designed to deliver 4.5 kg/min of air from 100 kPa and 27°C
to compress through an overall pressure ratio of 9. The law of compression is pV1.3 = constant.
Calculate the saving in power consumption and gain in isothermal efficiency, when a two-stage
compressor with complete intercooling is used in place of a single-stage compressor. Assume equal
pressure ratio in both the stages of the two-stage compressor. Take R = 0.287 kJ/kg K.

Solution
Given: m = 4.5 kg/min, p1 = 100 kPa, T1 = 273 + 27 = 300 K, rp = 9, n = 1.3
 n −1 
 n   n − 1
Rate of work required in single-stage compression =  mRT1 rp
 n − 1  
 

M12_THERMAL ENGINEERING_SE_XXXX_CH12.indd 718 6/15/2018 5:31:20 PM


Reciprocating Air Compressors 719

 1.3−1 
1.3 4.5
   = × × 0.287 × 300 9 1.3 − 1
1.3 − 1 60  
 
      = 5.54 kW
 n −1 
 2n 
Rate of work required in two-stage compression =  mRT1  rp 2n − 1
 n − 1  
 1.3−1 
2 × 1.3 4.5
= × × 0.287 × 300 9 2×1.3 − 1
0.3 60  
 
     = 4.84 kW
Saving in power = 5.54 − 4.84 = 0.70 kW

Example 12.10
A double-acting, single-cylinder reciprocating air compressor has a piston displacement of
0.015 m3 per revolution, operates at 500 rpm, and has 5% clearance ratio. The air is received at 1
bar and delivered at 6 bar. The compression and expansion are polytropic with n = 1.3. If the inlet
temperature of air is 20°C, determine (a) the volumetric efficiency, (b) the power required and
(c) the heat transferred and its direction, during compression.

Solution
Given that Vs = 0.015 m3/rev, N = 500 rpm, c = Vc/Vs = 0.05
p1 = 1 bar, p2 = 6 bar, n = 1.3, T1 = 273 + 20 = 293 K
1
 p n
(a) Volumetric efficiency, h v = 1 + c − c  2 
 p1 
1
 6  1.3
= 1 + 0.05 − 0.05   = 0.8516 or 85.16%
 1
va Va
v = , hv =
(b) Volumetrichefficiency
vs Vs × 2 N
or Va = 0.015 × 0.8516 × 500 × 2 = 12.774 m3/min
 n −1 
 n   p2  n 
Power required,        P =   p1Va  p  − 1
 n − 1   1 
 
 1.3−1 
1.3 2  6  1.3 
= × 1 × 10 × 12.774 ×   − 1
1.3 − 1  1 
 
= 2834.54 kJ/min or 47.242 kW
(c) Mass of air sucked,
p1Vas
m = = 1 × 105 × 12.774 / ( 287 × 293) = 15.19 kg/min
RT1

M12_THERMAL ENGINEERING_SE_XXXX_CH12.indd 719 6/15/2018 5:31:23 PM


720 Chapter 12

( n −1) 0.3
p  n  6  1.3
T2 = T1  2  = 293   = 443 K
 p1   1
Heat transferred during compression,
g − n 
Qc = mC
 v  [T2 − T1 ]
 n −1 
 (1.4 − 1.3) 
= 15.19 × 0.718 ×   [443 − 293]
 (1.3 − 1) 
= 545.321 kJ/min

Example 12.11
Find the optimum intermediate pressure of a two-stage reciprocating compressor, if intercooling is
done up to a temperature T2′, which is greater than the inlet temperature.

Solution
Work done in the L.P. cylinder (Fig. 12.23)
 ( n –1) 
n  p2  n 
W1 = mRT1   – 1
n –1  1 
p 
 

Work done in the H.P. cylinder,


 ( n –1) 
n  p3  n 
W2 = mRT2′   – 1
n –1  p2  
 
Total work done, W = W1 + W2

 ( n –1) ( n –1) 
n   p2  n  p3  n 
= mR × T1   + T2′   – T1 – T2′ 
n –1   p1   p2  
 

p
p3 3′ 3
pV n = const
H.P.
p2 2
pV = const
L.P.
p1 1
V

Figure 12.23 p − V diagram for two-stage compressor

M12_THERMAL ENGINEERING_SE_XXXX_CH12.indd 720 6/15/2018 5:31:27 PM


Reciprocating Air Compressors 721

dW
For maximum work, =0
dp2

n T1 p2 –1/ n
–T2′ p3( )
n –1 / n n – 1 ( 2 n –1)/ n
p =0
n – 1 p ( n –1)/ n n 2
1
n
1  T2′  ( 2n – 2)
or p2 =  p1 p3  2  
 T1 

Example 12.12
A two-stage reciprocating air compressor takes in air at 1.013 bar and 15°C and delivers at 43.4 bar.
The intercooler pressure is 7.56 bar. The intercooling is perfect and the index of compression is 1.3.
Calculate the work done in compressing 1 kg of air. If both cylinders have the same stroke and the
piston diameters are 9 cm and 3 cm and the volumetric efficiency of the compressor is 90%, will the
intercooler pressure be steady or will rise or fall as the compressor continues working?

Solution
Given that p1 = 1.013 bar, T1 = 273 + 15 = 288 K
p3 = 43.4 bar, p2 = 7.56 bar, n = 1.3,
m = 1 kg, d1 = 9cm, d2 = 3 cm, hv = 0.9
Input work per kg for perfect intercooling T3 = T1,
 ( n –1) ( n –1) 
 n   p2  n  p3  n 
W =  RT1   +  – 2
 n – 1  p1   p2  
 
 0.3 0.3 
 1.3   7.56  1.3  43.4  1.3 
=   × 0.287 × 288  + − 2  = 389.3 kJ/kg
 0.3   1.013   
7.56 
 
Compression in L.P. cylinder:
1 1
 p  n  RT   p  n
v2 = v1  1  =  1   1 
 p2   p1   p2 
1
 0.287 × 288   1.013  1.3
=   = 0.17386 m3/kg
 1.013 × 10 2   7.56 
( n –1)
p  n  7.56  0.3
T2 = T1  2  = 288  = 457.97 K
 p1   1.0130  1.3
For constant pressure in the intercooler, the volume of air leaving the intercooler and entering the H.P.
cylinder,

M12_THERMAL ENGINEERING_SE_XXXX_CH12.indd 721 6/15/2018 5:31:30 PM


722 Chapter 12

v2T3 288
v2′ = = 0.17386 × = 0.10933 m3/kg
T2 457.97
 0.287 × 288 
 v1   
1.013 × 10 2 
 v  = = 4.693
2 th 0.17386
Ratio of effective cylinder volumes,
2 2
v1e  vL.P.   d1   9
= × h = h =   × 0.9 = 8.1
v2e  vH.P.  v 
 d2 
v
3
v  v 
As  1e  >  1  , more air is supplied to H.P. cylinder than it can hold and consequently the
 v2e   v2  th
pressure in the intercooler will rise.

Example 12.13
Find the percentage saving in work done by compressing air in two stages from 1 bar to 7 bar
instead of one stage. Assume compression index 1.35 in both the cases and optimum pressure and
complete intercooling in two-stage compression.

Solution
Given that p1 = 1 bar, p3 = 7 bar, n1 = n2 = 1.35
For perfect intercooling, p2 = p1 p3 = 1 × 7 = 2.646 bar and T3 = T1
With compression in one stage (see Fig. 12.7) without clearance, work done per kg of air is given by,
 n –1 
 n   p3  2n 
W1=  RT – 1 
 n – 1 1  p1 
 
 
Work done with perfect intercooling,
 n –1 
 2n   p3  n 
W2 =  RT – 1 
 n – 1 1  p1 
 
 
W1 – W2 W2
Percentage saving in work = = 1–
W1 W1
 n –1 
 p3  2n 
 – 1
 p1 
=1−2  
 n –1 
 p3  n 
 – 1
 p1  

M12_THERMAL ENGINEERING_SE_XXXX_CH12.indd 722 6/15/2018 5:31:34 PM


Reciprocating Air Compressors 723

 0.35 
  7  2.70 
  1  − 1
=1−2  
0.35
 
  7  1.35 − 1 

  1  

 0.2869 
=1−2
 0.6561 
= 0.1255 or 12.55%

Example 12.14
A single-stage, double-acting reciprocating air compressor delivers 3 m3 of free air/min at 1.013
bar and 20°C to 8 bar with the following data:
Rotational speed = 300 rpm, mechanical efficiency hmech = 0.9, pressure loss in passing through
intake valve = 0.04 bar, temperature rise of air during suction stroke = 12°C, clearance volume =
5% of stroke volume. Index of compression and expression = 1.35, and length of stroke = 1.2 times
of the cylinder diameter. Calculate: (a) volumetric efficiency, (b) cylinder dimension, (c) indicated
power, and (d) isothermal efficiency of the compressor, take for air, R = 0.287 kJ/kg K.

Solution
Given that Va = 3 m3/min, pa = 1.013 bar, Ta = 273 + 20 = 293 K, p1 = 1.013 − 0.04 = 0.973 bar, T1 =
Ta + 12 = 293 + 12 = 305 K, Vc = 0.05Vs, n = 1.35, L = 1.2D, R = 0.287 kJ/kg K, N = 300 rpm
The p − V diagram is shown in Fig 12.24.
Mass of free air delivered,
p V 1.013 × 105 × 3
V1 = m = 1 a = = 3.614 kg/min
RTa 0.287 × 103 × 293
Compression process 1–2:
n –1
T2  p2  n
=
T1  p1 

p
3 2
0 bar
pV 1.35 = C

1.013 bar
0.973 bar 1
4

V(m 3)
VC VS 3′

Figure 12.24 p − V diagram for single stage compressor

M12_THERMAL ENGINEERING_SE_XXXX_CH12.indd 723 6/15/2018 5:31:36 PM


724 Chapter 12

0.35
 8  1.35
               T2 = 305  = 526.6 K
 0.973 
Expansion process 3–4:
V3 = Vc = 0.05Vs
1 1
V  p n  8  1.35
               4 =  3  ,V4 = 0.05Vs  = 0.238Vs

V3  p4   0.973 
V – V = (V + V ) – V =(0.05V + V ) − 0.238V = 0.812V
1 4 c s 4 s s s s
Corresponding value of F.A.D. per cycle,
pT 0.973 × 293
V = 1 a (V1 − V4 ) = × 0.812Vs = 0.749Vs
paT1 1.013 × 305
(a) Volumetric efficiency,
V 0.749Vs
hv =
          = = 0.749 or 74.9%
Vs Vs
(b) Volume of air inhaled per cycle,
Va 3
=
              = = 0.005 m3/cycle
2 × N 2 × 300
p 2 p
Stroke volume, Vs = D L = D 2 × 1.2 D = 0.94248D3 m3/cycle
4 4
0.005 0.005
Also stroke volume = = = 6.6756 ×10−3 m3/cycle
hv 0.749
∴       0.94248D3 = 6.6756 × 10−3
D = 0.192 m or 192 mm
L = 1.2 × 192 = 230.4 mm
 n 
(c) Indicated power, I.P. =  V1= mR (T2 − T1)
 n –1
1.35 3.614
= × × 0.287(526.6 - 305)
0.35 60
= 14.776 kW
(d) Isothermal indicated power,
p 
V1 == mRT1 ln  2 
(I.P.)iso
 p1 

3.614  8 
= × 0.287 × 305 ln  = 11.108 kW
60  0.973 
Isothermal efficiency,
11.108
hiso = = 0.7517 or 75.17%
14.776

M12_THERMAL ENGINEERING_SE_XXXX_CH12.indd 724 6/15/2018 5:31:43 PM


Reciprocating Air Compressors 725

Example 12.15
The following data were obtained from a performance test of a 14 cm × 10 cm single stage
reciprocating air compressor having 3 percent clearance: barometer 77 cm Hg, suction pressure
0 bar gauge, suction temperature 22°C, discharge pressure 4.05 bar gauge, discharge temperature
174°C, shaft rpm 1160, shaft power 350 kW, mass of air delivered per minute 1.75 kg. Determine
(a) the actual volumetric efficiency; (b) the approximate indicated power; (c) the isothermal com-
pression efficiency; (d) the mechanical efficiency and (e) the overall efficiency of the unit. Take
patm = 1.031 bar.

Solution
Atmospheric pressure corresponding to 77 cm Hg
1.75 × 287 × 295 77 × 1.0312
= = 1.418 m3 = = 1.0447 bar
5 76
1.0447 × 10
∴ p1 = 0 + 1.0447 = 1.0447 bar
and p2 = 4.05 + 1.0447465 = 5.0947 bar
1.75 × 287 × 295 77 × 1.0312
Air entering the cylinder = = 1.418 m3 = = 1.0447 bar
5 76
1.0447 × 10
2
p  14  10
Piston displacement =   × × 1160 = 1.787 m3
4  100  100
1.418
(a) Volumetric efficiency = = 79.35%
1.787
n −1 n −1
T   p  n 447  5.0965  n
(b)  2  =  2  or = 
 T1   p1  295  1.0465 
∴  Compression index n = 1.353
n mR(T2 − T1 )
Indicated power =
n −1 4500
1.353
 = × 1.75 × 0.287( 447 − 295) = 292.6 kW
0.353
p
(c) Isothermal power = mRT1 ln 2
p1
5.0947
 =1.75 × 0.287 × 295 × loge = 234.76 kW
1.0447
234.76
∴   Isothermal efficiency = = 0.8023 or 80.23%
292.6
indicated power 292.6
(d) Compressor mechanical efficiency = = = 83.6%
shaft power 350
(e) Overall efficiency = isothermal efficiency × mechanical efficiency
= 0.8023 × 0.836 = 67.07%

M12_THERMAL ENGINEERING_SE_XXXX_CH12.indd 725 6/15/2018 5:31:46 PM


726 Chapter 12

Example 12.16
A two-stage reciprocating compressor is used to compress from 1.0 bar to 16 bar. The compression
is as per the law pV1.25 = const. The temperature of air at inlet of compressor is 300 K. Neglecting
the clearance and assuming perfect intercooling find out the minimum indicated power to deliver
5 m3/min air measured at inlet conditions and find the intermediate pressure also.
Solution
Given: p1 = 1 bar v1 = 5m3/min = 0.083 m3/s, p3 = 16 bar, n = 1.25, T1 = 300 K
For complete intercooling, the intercooler pressure,
p2 = p1 p3 = 1 × 16 = 4 bar
Work done in compressing the air,
 n −1 
2n  p3  2n 
W = pV − 1
n − 1 1 1  p1  
 
 0.25 
2 × 1.25 5  16  2×1.25 
= × 10 × 0.083   − 1
0.25  4
 
= 26519.16 J/s = 26.519 W
Power required to drive the compressor is 26.519 kW.

Summary for Quick Revision


1. A reciprocating compressor is a device in which air is compressed in a conventional cylinder with a closely
fitted piston making reciprocating motion.
2. Free air delivered (FAD) is the actual volume delivered by the compressor at the stated pressure, reduced to
intake pressure and temperature and expressed in m3/min.
3. The work done on the compressor to compress a given volume of air at a given pressure shall be the least
when the compression process is isothermal.
4. Approximation to isothermal compression can be achieved by spraying cold water on the cylinder, water
jacketing the cylinder, intercooling by using multi-stage compression and by using external fins of the cyl-
inder surface.
5. Single stage compression:
(a) Required work and power.
(i) Without clearance:
 p  ( n −1)/ n 
n , n = index of compression
W= p1V1  2  − 1
n −1  p1  
 

 p  ( n −1)/ n 
n , m = mass of air delivered per cycle.
= mRT1  2  − 1
n −1  p1  
 

M12_THERMAL ENGINEERING_SE_XXXX_CH12.indd 726 6/15/2018 5:31:49 PM


Reciprocating Air Compressors 727

n T 
= mRT1  2 − 1
n −1 T
 1 
WN
Power required in driving the compressor, P = kW
60 × 103
Where N = rpm, if single acting
  = number of strokes, if double acting.
(ii)  With clearance:
 p  ( n −1)/ n 
 n  
W = p V 2 − 1 , Va = Actual volume delivered
 n − 1 1 a  p1  
 
 p  ( n −1)/ n 
 n  
= m RT 2 − 1
 n − 1 1 1  p1 


 
 n  T 
= m RT 2 − 1
 n − 1 1 1  T1 

Actual free air delivered per stroke


(b)  Volumetric efficiency,h v =
Swept volume

 p  1/ n 
h v = 1 − c  2  − 1
 p1  
 

When referred to ambient conditions,

Volume of air sucked referred to ambient conditions


hv =
  Sweptt volume
 1
p1T0   p2  n 
hv = 1 + c − c   
p0T1   p1  
 

V
where c = c = clearance ratio, Vc = clearance volume, Vs = swept volume.
Vs
(c)  Isothermal efficiency,

Isothermal work ln rp p
hi = = , rp = 2
Polytropic work  n   ( n −1)/ n  p1
  r −1
n − 1  p 

p1V1 ln rp × N
Isothermal power, Pi = kW
60 × 103
Isothermal power
Compressor efficiency, hc =
Indicated power

M12_THERMAL ENGINEERING_SE_XXXX_CH12.indd 727 6/15/2018 5:31:55 PM


728 Chapter 12

 g   (g -1)/g
 g − 1  rp − 1

(d)  Adiabatic efficiency, h a =
 n   ( n −1)/ n 
  r −1
n − 1  p 
(e)  Main dimensions,
Q
Swept volume, Vs = V1 − V3 =
Nhu
p 2 L
= d L, = 0.6 − 0.9
4 d
6. Two-stage compression:
(a)  For perfect intercooling, p2 = p1 p3

 ( n −1) 
 2n   p3  ( 2n) 
(b)  Minimum compression work, (Wt)min =   p1V1   − 1
 n − 1
 p1  
 
(c)  In general, with i number of stages,
p2 p3 p
= = ... = i +1
p1 p2 pi
  n −1 
 in   pi +1   in  
(d)  Minimum shaft work, Wt =  RT − 1
 n − 1 1  pi  

 
(e)  Minimum indicated power with imperfect intercooling,
  n −1 
 n    p3   2n  
I.P. =  mR  T1T3   − (T1 + T3 )
 n − 1   p1  
 
(f)  Heat rejected to intercooler = mcp (T2 − T3)

(g)  Cylinder dimensions:


d12 p1 = d22 p2 = d32 p3 = ...

T2 − T3
(h)  Effectiveness of intercooler, e =
T2 − T1

pm LAN
7. Indicated power, IP = kW
60 × 103
 n 
Where pm =  p h  r ( n −1)/ n − 1
 n − 1 1 v  p 

Area of indicator card, cm 2


= N/m 2
Measured length of indicator card, cm × Vertical scale, N/m 2 /cm
I.P.
8. Mechanical efficiency, h mech =
Brake power

M12_THERMAL ENGINEERING_SE_XXXX_CH12.indd 728 6/15/2018 5:32:01 PM


Reciprocating Air Compressors 729

9. Heat rejected per kg of air,


 c (g − n) 
q =  cp + v (T2 − T1 )
 n − 1 
10. A compressor can be controlled by: Throttle control, clearance control or by blowing air to waste.

Multiple-choice Questions
1. Which one of the following statements is correct? (a) reduced volume flow rate
In reciprocating compressors, one should aim at (b) increased volume flow rate
compressing the air (c) lower suction pressure
(a) adiabatically (b) isentropically (d) lower delivery pressure
(c) isothermally (d) polytropically 6. In a reciprocating air compressor, the compres-
2. Consider the following statements: sion work per kg of air
i. Reciprocating compressors are best suited for (a) increases as clearance volume increases
high pressure and low volume capacity. (b) decreases as clearance volume increases
ii. The effect of clearance volume on power con- (c) is independent of clearance volume
sumption is negligible of the same volume of (d) increases with clearance volume only of mul-
discharge. tistage compressor
iii. While the compressor is idling, the delivery
7. Consider the following statements:
value is kept open by the control circuit.
When air is to be compressed to reasonably high
iv. Intercooling of air between the stages of com-
pressure, it is usually carried out by multistage
pression helps to minimize losses. Of these
compressor with an intercooler between the
statements
stages because
(a) i and ii are correct
1. work supplied is saved.
(b) i and iii are correct
2. weight of compressor is reduced.
(c) ii and iv are correct
3. more uniform torque is obtained leading to the
(d) iii alone is correct
reduction in the size of flywheel.
3. For two-stage compressor in which index of com- 4. volumetric efficiency is increased.
pression for low pressure stage is m and for high Of the four statements listed above
pressure stage in n. The load sharing with perfect (a) 1 alone is correct
intercooling in expressed as: (b) 2 and 4 are correct
W1 m( n − 1) W1 n( n − 1) (c) 1, 2 and 3 are correct
(a) = (b) =
W2 n( m − 1) W2 m( m − 1) (d) 1, 2, 3 and 4 are correct
W1 n W1 m 8. Consider the following statements:
(c) = (d) = The volumetric efficiency of a compressor
W2 m W2 n
depends upon
4. For a two-stage reciprocating compressor, com- 1. clearance volume
pression from pressure p1 to p3 is with perfect 2. pressure ratio
intercooling and no pressure losses. If compres- 3. index of expansion
sion in both cylinders follows the same polytropic Of these statements:
process and the atmospheric pressure is pa then (a) 1 and 2 are correct
the intermediate pressure p2 is given by (b) 1 and 3 are correct
(a) p2 = (p1 + p3)/2 (b) p2 = p1 p3 (c) 2 and 3 are correct
(d) 1, 2 and 3 are correct
p3 p3
(c) p2 = pa (d) p2 = pa 9. The heat rejection by a reciprocating air com-
p1 p1 pressor during the reversible compression process
5. A large clearance volume in reciprocating com- AB, shown in the temperature-entropy diagram, is
pressor results in represented by the area:

M12_THERMAL ENGINEERING_SE_XXXX_CH12.indd 729 6/15/2018 5:32:04 PM


730 Chapter 12

15. A two-stage compressor takes in air at 1.1 bars


T p2 and discharges at 20 bars. For minimum work
input, the intermediate pressure is
B p1 (a) 10.55 bars (b) 7.33 bars
F
(c) 5.5 bars (d) 4.7 bars
G C A
16. Consider the following statements:
The volumetric efficiency of a reciprocating com-
pressor can be enhanced by
O D E S 1.  heating the intake air
(a) ABC 2.  decreasing the clearance volume
(b) ABDE 3.  cooling the intake air
(c) ABFG Which of these statements is/are correct?
(d) ABFOE (a) 1 alone (b) 1 and 2
10. A 3-stage reciprocating compressor has suction (c) 2 and 3 (d) 3 alone
pressure of 1 bar and delivery pressure of 27 bar. 17. Reciprocating compressors are provided with
For minimum work of compression, the delivery (a) simple disc/plate valve
pressure of 1st stage is (b) poppet valve
(a) 14 bar (b) 9 bar (c) spring-loaded disc value
(c) 5.196 bar (d) 3 bar (d) solenoid valve
11. Consider the following factors: 18. If n is the polytropic index of compression and
1. Cylinder size p2
is the pressure ratio for a three-stage com-
2. Clearance ratio p1
3.  Delivery pressure pressor with ideal intercooling, the expression for
4. Compressor shaft power total work of three stage is
The factors which affect the volumetric efficiency  ( n −1) 
of a single-stage reciprocating air compressor 3n  p2  n 
would include (a) p1v1   − 1
( n − 1) 
 1 p  
(a) 1 and 2 (b) 3 and 4  
(c) 2 and 3 (d) 1 and 4
 ( n −1) 
12. A four-stage compressor with perfect intercooling n  p2  3n 
between stages, compresses air from 1 bar to 16 (b) p1v1   − 1
( n − 1) 
 1 p  
bar. The optimum pressure in the last intercooler  
will be
 ( n −1) 
(a) 6 bar (b) 8 bar
n  p2  n 
(c) 10 bar (d) 12 bar (c) p1v1   − 1
( n − 1) 
 1 p  
13. The clearance volume of a reciprocating com-  
pressor directly affects
 ( n −1) 
(a) piston speed
3n  p2  3n 
(b) noise level (d) p1v1   − 1
(c) volumetric efficiency ( n − 1) 
 1 p  
 
(d) temperature of air after compression
19. Consider the following statements:
14. The capacity of an air compressor is specified
Volumetric efficiency of a reciprocating air com-
as 3m3/min. It means that the compressor is capa-
pressor increases with
ble of
1.  increase in clearance ratio
(a) supplying 3 m3 of compressed air per minute
(b) compressing 3 m3 of free air per minute 2.  decrease in delivery pressure
(c) supplying 3 m3 of compressed air at NTP 3. multistaging
(d) compressing 3 m3 of standard air per minute Which of the statements given above is/are
correct?

M12_THERMAL ENGINEERING_SE_XXXX_CH12.indd 730 6/15/2018 5:32:07 PM


Reciprocating Air Compressors 731

(a) Only 1 and 2 (b) Only 2 and 3 23. Consider the following statements:
(c) Only 3 (d) 1, 2 and 3 In a reciprocating compressor, clearance volume
20. What is the preferred intercooler pressure for a is provided
two stage air compressor working between the 1. so that piston does not hit and damage the
suction pressure ps and the delivery pressure pd? valves
2. to account for differential thermal expansion
( ps + pd ) ( pd − ps ) of piston and cylinder
(a) (b)
2 2 3. to account for machining tolerances
1 1 4. to achieve isentropic compression
(c) ( ps × pd ) 2 (d) ( ps × pd ) 4 Which of these statements are correct?
(a) 1, 2 and 3 (b) 1, 2 and 4
21. Which of the following statements are correct for
(c) 1, 3 and 4 (d) 2, 3 and 4
multi-staging in a reciprocating air compressor?
1. It decreases the volumetric efficiency
24. Which of the following can be the cause/causes
2. The work done can be reduced
of an air-cooled compressor getting overheated
3. A small high-pressure cylinder is required.
during operation?
4. The size of flywheel is reduced. 1. Insufficient lubricating oil.
Select the correct answer using the code given 2. Broken valve strip.
below: 3. Clogged intake filter.
(a) 1, 2 and 3 (b) 2, 3 and 4 Select the correct answer using the code given below:
(c) 1, 3 and 4 (d) 1, 2 and 4 (a) Only 3 (b) Only 1 and 2
22. For a two-stage reciprocating air compressor, the (c) Only 2 and 3 (d) 1, 2 and 3
suction pressure is 1.5 bar and the delivery pres- 25. Performance of a reciprocating compressor is
sure is 54 bar. What is the value of the ideal inter- expressed by
cooler pressure? Isothermal work Indicated work
(a) 6 bar (b) 9 bar (a) (b)
Indicated work Isothermal work
9 Adiabatic work Indicated work
(c) 27.75 bar (d) bar (c) (d)
2 Indicated work Adiabatic work

Explanatory Notes

10. (d) p2 = 3 p1 p3 = 3 27 × 1 = 3 bar


3
 p 4 3
12. (b) Optimum pressure in the last intercooler =  5  = (16 ) 4 = 8 bar
 p1 
15. (d) pi = p1 p2 = 1.1 × 20 = 4.6904  4.7 bar

22. (b) pi = p1 p2 = 1.5 × 54 = 81 = 9 bar

Review Questions

1. List at least six uses of compressed air in industry.


2. Explain the working of a single stage reciprocating air compressor.
3. What do you mean by free air delivered (FAD)?
4. Define volumetric efficiency.
5. What are the factors on which volumetric efficiency depends?
6. What are the methods generally adopted for approximating the compression process of a reciprocating air
compressor as isothermal one?

M12_THERMAL ENGINEERING_SE_XXXX_CH12.indd 731 6/15/2018 5:32:13 PM


732 Chapter 12

7. Define clearance ratio. How the volumetric efficiency is dependent on clearance ratio?
8. Define isothermal efficiency and compressor efficiency.
9. Differentiate between isothermal power and indicated power.
10. Define adiabatic efficiency.
11. What are the advantages of multi-stage compression?
12. What is the condition for minimum compressor work with perfect intercooling for a multi-stage reciprocat-
ing air compressor?
13. Distinguish between the function of intercooler and after-cooler.
14. What is an air motor?
15. How heat is rejected in air compressor?

Exercises
12.1 A single-stage single-acting reciprocating air compressor delivers 15 m3 of free air per minute from 1 bar to
1
8 bar at 300 rpm.The index of both compression and expansion is n = 1.3 for and clearance th of swept
16
volume, find the diameter and stroke of the compressor. Take L = 1.5 D. [Ans. 383 mm, 587.5 mm]
12.2 A single-stage double-acting reciprocating air compressor delivers air at 7 bar. The pressure and tem-
perature at the end of suction stroke are 1 bar and 27°C. It delivers 2 m3 of free air per minute when the
compressor is running at 300 rpm. The clearance volume is 5% of the stroke volume. The ambient pressure
and temperature are 1.03 bar and 20°C. Index of compression and expansion is 1.30 and 1.35 respectively.
Calculate (a) the volumetric efficiency, (b) indicated power and brake power if mechanical efficiency is
80%, and (c) diameter and stroke of the cylinder if both are equal.
[Ans. 83.9%, 8.5 kW, 10.64 kW, 174.6 mm]

12.3 A two-stage single-acting reciprocating air compressor delivers air at 20 bar. The pressure and temperature
of air before compression in L.P. cylinder are 1 bar and 27°C. The discharge pressure of L.P. cylinder is 4.7
bar. The pressure of air leaving the intercooler is 4.5 bar and the air is cooled to 27°C. The diameter and
stroke of L.P. cylinder are 0.4 m and 0.5 m respectively. The clearance volume is 4% of stroke in both cyl-
inders. The speed of compressor is 200 rpm. The index of compression and re-expansion in both cylinders
is 1.3. Determine (a) the indicated power to run the compressor, and (b) the heat rejected to intercooler per
minute. [Ans. 68.9 kW, 1716 kJ/min]
12.4 A single-stage, double-acting reciprocating air compressor delivers 15 m3 of air per minute measured at
1.013 bar, 27°C and delivers at 7 bar. At the end of the suction stroke the pressure and temperature are 0.98
bar and 40°C. The clearance volume is 4% of the swept volume and the stroke is 1.3 times the bore. The
compressor runs at 300 rpm. Calculate (a) the volumetric efficiency, (b) cylinder dimensions, (c) indicated
power and (d) isothermal efficiency. n = 1.3 for both compression and expansion. Take R = 0.287 kJ/kg.K
[Ans. 79.6%; 313.3 mm, 407.3 mm; 65.83 kW; 78.92%]

12.5 A single-stage, double acting air reciprocating compressor takes air at 0.98 bar and 32°C and delivers at
6.32 bar. The clearance is 5% of the stroke volume. The compression and expansion follow the law pV1.32
= c. The air handled by the compressor is 17 m3/min when measured at 1 bar and 15°C. Determine the
temperature of air delivered, the stroke volume and the indicated power of compressor if it turns at 500 rpm.
Neglect the area of piston and take R = 0.287 kJ/kg.K.
12.6 The free air delivered of a single cylinder single stage reciprocating air compressor is 2.5 m3/min. The
ambient air is at 0°C and 1.013 bar and delivery pressure is 7 bar. The clearance ratio is 5% and law of
compression and expansion is pv1.25 = c. If L =1.2 D and the compressor runs at 150 rpm, determine the
size of the cylinder. [Ans. 275.5 mm, 330 mm]

M12_THERMAL ENGINEERING_SE_XXXX_CH12.indd 732 6/15/2018 5:32:13 PM


Reciprocating Air Compressors 733

 12.7  A single stage double acting reciprocating air compressor delivers air at 7 bar. The amount of free air
delivered is 2 m3 at 300 rpm. The pressure and temperature at the end of suction stroke are 1 bar and
27°C. The ambient conditions are 1.03 bar and 20°C. The clearance is 5% stroke. The compression and
re-expansion follow the law pV1.3 = const. Determine the brake power required to run the compres-
sor if the mechanical efficiency is 80%, and the diameter and stroke of the cylinder if both are equal.
[Ans. 10.8kW, 179.5 mm]
 12.8  A two-stage double acting reciprocating air compressor delivers air at the rate of 1.35 kg/s. The suction
pressure is 1 bar and inter-stage pressure is 7 bar and delivery pressure 42 bar. Air enters the L.P. cylinder
at 17°C and cooled in the intercooler to 32°C. The clearances in L.P. and H.P. cylinders are 6% and 8% of
respective strokes. The law of compression and re-expansion is pV1.21 = c in both cylinders. The compres-
sor runs at 500 rpm. Calculate the amount of cooling water required per minute in intercooler, if rise in
temperature of water is limited to 20°C, power required, and size of cylinder if length of stroke and bore
are equal. [Ans. 2259 kg/min, 1068 kW, 483 mm]
 12.9  Determine the size of a cylinder for a double acting reciprocating air compressor of 37 kW, in which air is
drawn in at 1 bar, 15°C and compressed according to the law pV1.2 = const. to 6 bar. The compressor runs
at 100 rpm with average piston speed of 152.5 m/min. Neglect clearance. [Ans. 298.5 mm, 762.5 mm]
12.10   A single stage double acting reciprocating air compressor running at 500 rpm handles 17 m3/min of air,
measured at 1 bar and 15°C. The pressure and temperature at the end of suction are 0.98 bar and 32°C.
The air is delivered at 6.325 bar. Assuming a clearance factor of 5% and the compression and expansion
processes to follow the law pV1.32 = const, determine the stroke volume of the compressor. Also calculate
the indicated power of the compressor. Take R = 0.287 kJ/kg.K [Ans. 0.2175 m3, 70.73 kW]
12.11   A single acting compressed air motor works on compressed air at 10.5 bar and 37°C supplied at the rate of
1 kg/min. Cut-off takes place at 25% of the stroke and the expansion follows the adiabatic frictionless law
down to 1.0135 bar. Determine the mean effective pressure, the indicated power and the cylinder volume
if the motor runs at 250 rpm. Neglect clearance. [Ans. 3.37 bar, 1.94 kW, 0.001383 m3]
12.12   The cylinder of an air motor has a bore of 63.5 mm and a stroke of 114 mm. The supply pressure and
temperature are 6.3 bar and 24°C and exhaust pressure is 1.013 bar. The clearance volume is 5% of the
swept volume and the cut-off ratio is 0.5. The air is compressed by the returning piston after it has travelled
through 0.95 of its stroke. The law of compression and expansion is pV1.3 = const. Calculate the temper-
ature at the end of expansion and the indicated power of the motor which runs at 300 rpm. Also calculate
the air supplied per minute. Take R = 0.287 kJ/kgK. [Ans. 244.4 K, 0.746 kW, 0.418 kg/min]
12.13   A water cooled air compressor requires a work input of 200 kJ/kg of air delivered. The enthalpy of air
leaving the compressor is 75 kJ/kg greater than that entering. Heat lost to the cooling water is 105 kJ/kg.
From the first law analysis estimate the heat lost by compressor to atmosphere.
12.14   A two-stage single acting air reciprocating compressor equipped with an intercooler draws air at 1.013
bar and 30°C and delivers it at 20 bar. The mass of air delivered by the compressor is 10 kg/min. The ratio
of clearance volume to swept volume in the low pressure and high pressure cylinders is 0.03 and 0.04
respectively. If the compressor has a mechanical efficiency of 82% and it operates under most efficient
conditions, determine:
   (a)  power supplied at the shaft of the compressor;
   (b)  isothermal efficiency;
   (c)  ratio of cylinder diameters for identical stroke length; and
   (d)  heat rejected in the intercooler.
        Take R = 0.287 kJ/kg.K, cp = 1.005 kJ/kgK and g = 1.4.
12.15   A single acting two-stage reciprocating air compressor is to compress air from 1 bar and 30°C to 12 bar.
The base of the low pressure cylinder is 30 cm. The stroke length of both the low and the high pressure
cylinders is the same and is equal to 40 cm. The compressor runs at 180 rpm. The clearance volume in both
the cylinders is 3% of the stroke volume. Index of compression and expansion is 1.3 in both the cylinders.

M12_THERMAL ENGINEERING_SE_XXXX_CH12.indd 733 6/15/2018 5:32:13 PM


734 Chapter 12

Determine the shaft power required to drive the compressor when (a) the air is cooled to its original tem-
perature before entering the H.P. cylinder, (ii) when the air is cooled to 45°C in the intercooler. Assume
mechanical efficiency to be 85% in both cases. Take R = 0.287 kJ/kg.K.
12.16   A multi-stage reciprocating compressor has to be designed to supply air at 135 bar, while atmospheric
condition is 1.03 bar and 15°C. The value of compression index may be assumed as 1.35. Due to practical
reasons the intercoolers are not able to cool the air below 45°C, while the maximum temperature allowable
in the system is 120°C. Calculate the number of stages that are necessary in the compression and the rate
of cooling water circulated per kg of air. Take cp = 1 kJ/kgK.

ANSWERS TO MULTIPLE-CHOICE QUESTIONS


1. (c) 2. (b) 3. (a) 4. (b) 5. (d)
6. (a) 7. (a) 8. (d) 9. (b) 10. (d)
11. (a) 12. (b) 13. (c) 14. (b) 15. (d)
16. (c) 17. (a) 18. (d) 19. (d) 20. (c)
21. (b) 22. (b) 23. (a) 24. (c) 25. (a)

M12_THERMAL ENGINEERING_SE_XXXX_CH12.indd 734 6/15/2018 5:32:13 PM


Chapter 13

Rotary Air Compressors

13.1 ❐ INTRODUCTION
In a rotary compressor, the compression of air is achieved due to the rotating blades fitted in a rotor. It
requires less starting torque as compared to reciprocating compressors because of direct coupling with
the prime over. Usually, rotary compressors operate at high speed and supplies higher quantity of air
than reciprocating compressors. Rotary compressors may be classified as: (a) Roots blower, (b) Vane
type compressor, (c) Lysholm compressor and (d) Screw compressor.

13.2 ❐ WORKING PRINCIPLE OF DIFFERENT


ROTARY COMPRESSORS
In positive displacement type rotary compressors, the air is compressed by entrapping it between the
reduced space of two sets of engaging surfaces. The pressure rise is either by backflow of air, as in
the case of roots blower, or by both squeezing action and backflow of air, as in the case of vane type.

13.2.1  Roots Blower or Lobe Compressor


The schematic arrangement of a roots blower is shown in Fig. 13.1(a). It consists of two rotors that are
driven externally. One rotor is connected to the driver and the second rotor is gear driven from the first
in the opposite direction. The lobes of the rotors are of epicycloids, hypocycloid, or involute profiles
to ensure a seal between the high and low pressure regions at all angular positions. A small clearance
provided between the rotors and the cylinder surface to reduce the wear reduces the efficiency of the
compressor due to leakage.
The volume Vs of air at atmospheric pressure p1 is entrapped between the left hand rotor and the
cylinder casing. The volume of air once entrapped does not decrease from entry to exit, and therefore,
there is no rise of pressure till the exit port is uncovered. As the exit part opens, some high pressure air
from the receiver rushes back and mixes with the air volume Vs irreversibly until the pressure is equal-
ised. The resulting pressure after mixing will be the receiver pressure p2. The air is then transferred to
the receiver. This happens four times in one revolution in case of two-lobed rotor and six times in case
of three-lobed rotor. The p−V diagram for the roots blower is shown in Fig. 13.1(b).

M13_THERMAL ENGINEERING_SE_XXXX_CH13.indd 735 6/15/2018 5:35:45 PM


736 Chapter 13

Outlet. p2

Receiver

High
pressure

p
vs
p2
Low Irreversible pressure
pressure rise due to backflow
p1 from receiver
vs
V
Inlet. p1
(a) (b)

Figure 13.1  Roots blower

Work done per revolution, W = 2i (p2 − p1)Vs (13.1)


where i = number of lobes
Let V1 = volume of air handled per minute at p1 and T1.
Then work done per minute is,
Wroots = (p2 − p1)V1(13.2)
If the compression is isentropic, then ideal work required per minute is,
  (g −1)/g 
Wideal =  g  p1V1  p2 − 1  (13.3)
 g − 1  p1  
 
The efficiency of roots blower
 p  (g −1)/g 
 g   2 
p V
 g − 1 1 1  p  − 1
1 
hroots =
Wideal
=  
Wroots ( p2 − p1 )V1

 g   (g −1)/g
 g − 1  rp − 1

or   hroots = (13.4)
rp − 1
p2
where rp = is the pressure ratio.
p1
The roots blower is used to supply air from 0.15 to 1500 m3/min with pressure ratio up to 3.6 per
blower. Rotational speeds up to 12,500 rpm are used and can be directly coupled to a steam turbine or
a gas turbine shaft without any intermediate gearing.

M13_THERMAL ENGINEERING_SE_XXXX_CH13.indd 736 6/15/2018 5:35:49 PM


Rotary Air Compressors 737

Receiver
p2

vd p Irreversible pressure
p2 2 rise due to backflow
vc
from receiver
pd d
vb c
b
a Isentropic
va
p1 1
V1
p1 V
(a) (b)

Figure 13.2  Vane type compressor

13.2.2  Vanes Type Blower


The schematic diagram of a vane type blower is shown in Fig. 13.2(a). It consists of a rotor located
eccentrically in a cylindrical outer casing. The rotor carries a set of spring-loaded vanes in the slots
of the rotor. The air of volume V1 at atmospheric pressure p1 is entrapped between two vanes. As the
rotation proceeds counter clockwise, the entrapped air is first compressed reversibly to Va. Afterwards,
the compression continues to Vc (say) and then to Vd. After this, the delivery port is uncovered and
the irreversible compression takes place from pressure pd to p2. The p−V diagram for the compression
process is shown in Fig. 13.2(b). The work done per revolution with i vanes is given by:

 g   p  (g −1)/g 
W = i  p V  d − 1 + i (p2 − pd) Vd (13.5)
 g − 1 1 1  p1  
 
The vane blowers require less power than the root blowers for the same capacity and pressure rise. They
are used to deliver up to 150 m3/min of air at pressure ratio up to 8.5. The speed is limited to 3000 rpm.

13.2.3  Lysholm Compressor


The Lysholm compressor is shown in Fig. 13.3. Its principle of working is similar to that of the roots
blower. The air is admitted through one end of the compressor and trapped between the helical rotors
and the casing. The screw action of the rotors displaces the air axially. It produces constant compres-
sion internally. Its main disadvantage is mechanical complexity.

Figure 13.3  Lysholm compressor

M13_THERMAL ENGINEERING_SE_XXXX_CH13.indd 737 6/15/2018 5:35:51 PM


738 Chapter 13

13.2.4  Screw Compressor


The screw compressor may be single helical or double helical. The advantage of double helical is that
they are balanced axially. The air is carried forward to the discharge along the rotor in pockets formed
between the teeth and the casing as shown in Fig. 13.4. The principle of working is similar to that of
the roots blower.

Figure 13.4  Screw compressor

Example 13.1
Free air of 25 m3/min is compressed from 1 bar to 2.5 bar. Calculate the indicated power required
if the compression is carried out in (a) roots blower and (b) vanes blower. Assume that there is 20%
reduction in volume before the backflow occurs. Also, calculate the isentropic efficiency in each
case.

Solution
The p−V diagrams for roots blower and vanes blower are shown in Fig. 13.5(a) and (b), respectively.
p p

p2 2 p2 2
Irreversible
pd d
Isentropic
p1 1
p1 1
V V
(a) (b)

Figure 13.5 p−V diagrams: (a) Roots blower, (b) Vanes blower

(a) Roots blower,


( p2 − p1 )V1 ( 2.5 − 1) × 105 × 25
I.P. = = = 62.5 kW
60 × 103 60 × 103
(b) Vanes blower,
  p  (g −1)/g  
 g   1
d  
     I.P. =   p1V1   − 1 + ( p2 − p1 )Vd 
10 × 60  g − 1
3   p   
  1 
g g
pd  V1   V 
=  =  1 
p1  Vd   0.8V1 
1.4
 1 
=  = 1.367
 0.8 

M13_THERMAL ENGINEERING_SE_XXXX_CH13.indd 738 6/15/2018 5:35:56 PM


Rotary Air Compressors 739

pd = 1 × 1.367 = 1.367 bar


 1.4
1 
I.P =  × 1 × 105 × 25 (1.367)0.4 /1.4 − 1 + ( 2.5 − 1.367) × 105 × 0.80 × 25
3
10 × 60  0.4   

=
3
10 × 60
1
{
8.175 × 105 + 22.66 × 105 = 51.39 kW }
I.P. if whole compression is carried out isentropically,
  p  (g −1)/g 
1  g   2  
I.P. =   p V
 1 1  p  − 1
103 × 60  g − 1 
  1 
 1.4
1  2.5  0.4 /1.4  
5
=  × 1 × 10 × 25   − 1  = 43.64 kW
103 × 60  0.4  1   
43.64
Isentropic efficiency of roots blower = × 100 = 69.83%
62.5
43.64
Isentropic efficiency of vanes blower = × 100 = 84.92%
51.39

Example 13.2
A rotary vanes blower works between the pressure limits of 1 bar and 1.8 bar, and gives 5 m3/min
of free air delivered when running at 240 rpm. Determine the power required to drive the blower
when (a) ports are so placed that there is no internal compression and (b) when the ports are
so placed that there is 50% pressure rise due to internal adiabatic compression before backflow
occurs. Assuming mechanical efficiency to be 98%, calculate the blower efficiency.

Solution
The p−V diagrams without and with internal compression are shown in Fig. 13.6(a) and (b),
respectively.

p p

p2 2 p2 2
Irreversible
pd d
Isentropic
p1 1
p1 1
V V
(a) (b)

Figure 13.6 p−V diagrams: (a) Without internal compression,


(b) With internal compression

M13_THERMAL ENGINEERING_SE_XXXX_CH13.indd 739 6/15/2018 5:36:00 PM


740 Chapter 13

(a) Without internal compression,


5
I.P. = (p2 − p1) V1 = (1.8 − 1) × 105 × = 6.67 kW
60 × 103
(b) With internal compression,

p2 − p1 1.8 − 1
pd = p1 + =1+ = 1.4 bar
2 2

1/g 1/1.4
 1 
Vd = V1  p1  = 5  = 3.932 m3/min
 p   1.4 
d

  p  (g −1)/g  
I.P. =
1  g   d  
3  p V
 1 1  p  − 1 + ( p2 − p )V
d d
10 × 60   g − 1  
  1 

1  1.4   1.4  0.4 /1.4  


5
=   × 1 × 10 × 5    − 1 + (1. 8 − 1. 4 ) ×1
1 05 × 3 .932 
103 × 60  0.4   1   
1
= [1.766 × 105 + 1.5728 × 105] = 5.565 kW
3
10 × 60
For minimum power requirement, the whole compression should be a reversible adiabatic process.

 g    (g −1)/g 
Ideal I.P. =  p V  p2 − 1
 g − 1 1 1  p1  
 

1 1.4  1.8  0.4 /1.4 


= × × 1 × 105 × 5   − 1 = 5.333 kW
103 × 60 0.4  1  
  Actual I.P. = Ideal I.P. × hmech
= 5.333 × 0.98 = 5.226 kW
5.226
Isentropic efficiency without internal compression = × 100 = 78.35%
6.67

5.226
Isentropic efficiency with internal compression = = 98%
5.333

13.3 ❐ COMPARISON OF ROTARY AND RECIPROCATING


COMPRESSORS
The comparison of rotary and reciprocating compressors is given in Table 13.1.

M13_THERMAL ENGINEERING_SE_XXXX_CH13.indd 740 6/15/2018 5:36:06 PM


Rotary Air Compressors 741

Table 13.1  Comparison of rotary and reciprocating compressors

S. no. Item Reciprocating Rotary


1. Air discharge Low at very high pressure, Large at low pressure, 3000 m3/min up to
300 m3/min up to 1 bar 10 bar
2. Speed Low, require some reduction High, up to 50,000 rpm, can be directly
gear coupled turbines, etc.
3. Receiver required Yes, as supply is intermittent No, as supply is continuous
4. Size Large for the same discharge Small
5. Balancing problem Yes No
6. Quality of air Less clean More clean as it does not come into con-
supplied tact with lubricating oil
7. Lubrication system More complicated Simple
8. Compression process May approach isothermal Isentropic with n  1.4
process

Example 13.3
Free air of 30 m3/min is compressed from 101.3 kPa to 2.23 bar in roots blower. Determine the
power required and the isentropic efficiency.

Solution
The p − V diagrams for roots blower is shown in Fig. 13.7.
p

p2 2

p1 1

Figure 13.7 p − V diagrams

V1 = 30 m3/min, p1 = 101.3 kPa


  p2 = 2.23 bar
Refer Fig. 13.8.
( p − p1 )V1
Indicated power, I.P. = 2
60 × 103
( 2.23 × 105 − 101.3 × 103 ) × 30
= = 60.85 kW
60 × 103
  g −1 
 g 
1  p2  g 
Isentropic indicated power, (I.P.)isen =   p1V1   − 1 
10 × 60  g − 1
3
 p1  
  

M13_THERMAL ENGINEERING_SE_XXXX_CH13.indd 741 6/15/2018 5:36:09 PM


742 Chapter 13

  0.4 
 1.4 
1 3  223  1.4  
=   × 101.3 × 10 × 30   − 1 
103 × 60  0.4  
101.3 
 
 
= 44.83 kW
(I.P.)isen
Isentropic efficiency, hisen = × 100
I.P.
44.83
= × 100 = 73.67%
60.85

Summary for Quick Revision

1. Rotary compressors are of positive displacement type in which air is compressed by entrapping it between
the reduced space of two sets of engaging surfaces.
2. The pressure rise is either by backflow of air (as in roots blower) or by both squeezing action and backflow
(as in vanes blower).
3. Roots blower
Work done per minute, Wroots = (p2 − p1)V1

c p  ( rp )(g −1)/ g − 1
Roots efficiency, hroots =  
R  rp − 1 
 
where V1 = volume of air handled per minute
p
rp = 2 , pressure ratio
p1
4. Vanes type blower   (g −1)/ g 
 g   pd
Work done per revolution = i  p V − 1 + i (p2 − p1) Vd
 r − 1 1 1  
 p1  
where i = number of Vanes  
pd = maximum isentropic pressure
W
5. Indicated power, I.P. = kW
60 × 103

Multiple-choice Questions
1. The efficiency of a roots air blower is (rp = pres- n −1
rp − 1
sure ratio) rp n −1
(c) (d) n −1
g −1 rp − 1
rp − 1 rp n −1
g
rp −1 g −1
(a) (b) 2. In a roots blower with two lobes, the high pressure
rp − 1 g
rp −1
air is delivered in one revolution

M13_THERMAL ENGINEERING_SE_XXXX_CH13.indd 742 6/15/2018 5:36:15 PM


Rotary Air Compressors 743

(a) Once (b) Twice (a) Roots blower


(c) Thrice (d) Four times (b) Vanes blower
3. The efficiency of vane type air compressor as (c) Centrifugal compressor
compared to roots blower for the same pressure (d) axial compressor
ratio is 5. Roots blower is an example of
(a) Less (a) Reciprocating (positive displacement) com-
(b) More pressor
(c) Same (b) Rotary (positive displacement) compressor
(d) May be less or more (c) Centrifugal compressor
4. The p−V diagram shown in Fig. 13.8 below is for (d) Axial compressor
p
p2 2

pd d

p1 1
V

Figure 13.8 p-V diagram

Review Questions
1. What are the types of rotary compressors?
2. Draw the p−V diagram for a roots blower.
3. Which rotary compressor is used for steam/gas turbines?
4. What do you mean by a positive displacement compressor?

Exercises
13.1 What are the various types of compressors?
13.2 What is a positive displacement compressor?
13.3 Explain the working of a roots blower.
13.4 Derive an expression for the work done and efficiency of a roots blower.
13.5 Explain the working of a vane type blower.
13.6 Explain the working of a screw compressor.
13.7 Compare rotary and reciprocating compressors.

ANSWERS TO MULTIPLE-CHOICE QUESTIONS


1. (a) 2. (d) 3. (b) 4. (b) 5. (b)

M13_THERMAL ENGINEERING_SE_XXXX_CH13.indd 743 6/15/2018 5:36:16 PM


Chapter 14

Centrifugal Air
Compressors
14.1 ❐ INTRODUCTION
A centrifugal compressor is of rotodynamic type in which air flows continuously and steadily through
various parts and the rise in pressure is primarily due to the centrifugal action. It is used to supply large
quantities of air but at a lower pressure ratio.

14.2 ❐ CONSTRUCTIONAL FEATURES


Essentially, a centrifugal compressor consists of four elements namely, inlet buckets, impeller, dif-
fuser, and casing as shown in Fig. 14.1. The inlet buckets are attached to the shaft and rotate with it,
which guide air on the impeller. The impeller consists of a disc on which radial blades are attached.
The diffuser surrounds the impeller and provides diverging passages for air flow, thus increasing the
air pressure. The air coming out from the diffuser is collected in the casing and taken out from the
outlet of the compressor.

Difiuser ring

Drive shaft

Air
intake
Buckets

Impeller Impeller
eye Rotation

Figure 14.1  Centrifugal compressor

M14_THERMAL ENGINEERING_SE_XXXX_CH14.indd 744 6/15/2018 5:44:14 PM


Centrifugal Air Compressors 745

14.3 ❐ WORKING PRINCIPLE


The air enters the eye of the compressor at atmospheric pressure and low velocity. The inlet buckets
guide the air to the impeller where it moves radially outward and is guided by the impeller blades.
The impeller increases the momentum of the air flowing through it, causing a rise in pressure and
temperature of the air. The air leaving the impeller enters the diffuser where its velocity is reduced by
providing more cross-sectional area for the flow of air. A part of the kinetic energy of air is converted
into pressure energy and further increases the pressure of air.

14.4 ❐ VARIATION OF VELOCITY AND PRESSURE


As the air flows through the impeller and diffuser, there is a variation of both velocity and pressure as
shown in Fig. 14.2. Nearly half of the total pressure rise takes place in the impeller and the remaining
half occurs in the diffuser. A pressure ratio of 4 can be achieved in single-stage compressors. For higher
pressure ratios, multi-stage compressors are used. A pressure ratio of 12:1 is possible with multi-stage
compressors.

3
Pressure Pressure rise
∆p2
curve in diffuser
2 Pressure rise
∆p1
in impeller
1

Velocity ∆v3
Velocity increases curve Velocity drop
∆v1
in impeller 3 in diffuser

Impeller Diffuser

Figure 14.2 Variation of velocity and pressure of air passing through impeller


and diffuser

14.5 ❐ TYPES OF IMPELLERS


Impellers are of two types—single-eye type and double-eyed type as shown in Fig. 14.3 (a) and (b),
respectively. In a single-eye type, air enters into the compressor from one side only, whereas in a
double-eyed type, air enters from both sides. A double-eyed type impeller sucks in more air and has
the advantage of self-balancing over a single-eye impeller.
In multi-stage compressors, the output of the first stage is passed on to the second stage and so on,
as shown in Fig. 14.3(c).

M14_THERMAL ENGINEERING_SE_XXXX_CH14.indd 745 6/15/2018 5:44:15 PM


746 Chapter 14

m
Air out Air out

F F

Eye tip m
Air in
Eye root
Air in Air in

(a) (b) (c)

Figure 14.3 Types of impellers: (a) Single eye impellers, (b)


Double eye impellers, (c) Multi-stage impellers

14.6 ❐ COMPARISON OF CENTRIFUGAL AND RECIPRO-


CATING COMPRESSORS
The comparison of centrifugal and reciprocating compressors is given in Table 14.1.

Table 14.1  Comparison of centrifugal and reciprocating compressors

S. no. Aspect Reciprocating Compressor Centrifugal Compressor


 1 Vibration problems More due to unbalanced recipro- Less as there are no reciprocating
cating parts parts
 2 Mechanical efficiency Low due to many sliding parts High as sliding parts are less
 3 Installation cost Higher Lower
 4 Pressure ratio per stage 5 to 8 3 to 4.5
 5 Delivery pressure High up to 5000 atm Medium up to 400 atm.
 6 Volume delivered Small up to 300 m3/min High up to 3000 m3/min
 7 Maintenance cost Higher Lower
 8 Isentropic Efficiency About 80% About 70%
 9 Suitability High-pressure low volumes of air Low-pressure high volumes of air
10 Uses Driving pneumatic tools and Supercharging of IC engines, refrig-
air-operated controlling equipment erants, and industrial gases

14.7 ❐ COMPARISON OF CENTRIFUGAL AND ROTARY


COMPRESSORS
The comparison of centrifugal and rotary compressors is given in Table 14.2.

M14_THERMAL ENGINEERING_SE_XXXX_CH14.indd 746 6/15/2018 5:44:16 PM


Centrifugal Air Compressors 747

Table 14.2   Comparison of centrifugal and rotary compressors

S. no. Aspect Rotary compressors Centrifugal compressors


1 Pressure ratio 2 to 3 About 3 to 6
2 Suitability for multi-staging No Yes
3 Operational speed Low High
4 Mechanical efficiency Low High
5 Starting torque required High Low
6 Delivery pressure Low Medium and with multi-staging up to
400 atm
7 Suitability For handling large For handling large volumes of air at low
volumes of air at low and medium pressures
pressures.

14.8 ❐ STATIC AND STAGNATION PROPERTIES


The velocities of air encountered in centrifugal compressors are very high as compared to that in
reciprocating compressors. Therefore, total head quantities should be taken into account in the anal-
ysis of centrifugal compressors. The total head quantities account for the kinetic energy of the air
passing through the compressor.
Consider a horizontal passage of varying area of cross-section, as shown in Fig. 14.4, through
which air flows from left to right. Assuming no external heat transfer and work transfer to the system,
the steady flow energy equation for one kg mass of air flow can be written as:

v12 v2
h1 + = h2 + 2
2 2

v2 v2
cpT1 + 1 = cpT2 + 2
or   
2 2

v2
cpT +
or     = const.
2
Temperature T is called the ‘static temperature’; it is the temperature of the air measured by a ther-
mometer moving with the air velocity. If the moving air is brought to rest under reversible adiabatic

p3
p1
T1 T2

v1

v2

Figure 14.4  Passage of air through varying area of cross-section

M14_THERMAL ENGINEERING_SE_XXXX_CH14.indd 747 6/15/2018 5:44:18 PM


748 Chapter 14

conditions, the total kinetic energy of the air is converted into thermal energy, thereby increasing its
temperature and pressure. This temperature and pressure of the air are known as ‘stagnation’ or ‘total
head’ temperature and pressure. The stagnation quantities are denoted by a suffix notation o.
v2
   cpT + = cpT0(14.1)
2
where T0 = stagnation or total head temperature
v2
         T0 = T + (14.2)
2c p
v2
Similarly,   h0 = h + (14.3)
where h0 = stagnation or total head enthalpy 2
The stagnation pressure can be obtained by using the following equation:
g
p  T  g −1
0 =  0  (14.4)
p T
where p, T = static pressure and temperature
p0, T0 = stagnation pressure and temperature.

14.9 ❐ ADIABATIC AND ISENTROPIC PROCESSES


For an adiabatic compression process, there is no heat exchange with the surroundings. If the adiabatic
process is reversible (frictionless), then the process is called isentropic process in which the entropy
of the system does not change. However, in an actual compressor, during adiabatic compression, there
are losses due to friction in air and blade passages, eddies formation, and shocks at entry and exit.
These factors cause internal heat generation and consequently, the maximum temperature reached
would be higher than that for adiabatic compression. This results in a progressive increase in entropy.
Such a process, although adiabatic, is not reversible adiabatic or isentropic.
The isentropic and adiabatic processes for the static and stagnation values are shown in Fig. 14.5
on the T-s diagram. Process 1-2 is the isentropic process for the static temperature and 01-02 for the
total head temperature. Processes 1-2′ and 01-02′ are the adiabatic processes for the static and total
head temperatures, respectively.

T
02′
T02′
02
T02
T2′ 2′
T2 2 Actual
Isentropic

01
T01
1
T1
s

Figure 14.5  Isentropic and actual compression

M14_THERMAL ENGINEERING_SE_XXXX_CH14.indd 748 6/15/2018 5:44:20 PM


Centrifugal Air Compressors 749

14.9.1 Isentropic Efficiency


Isentropic efficiency may be defined as the ratio of isentropic temperature rise to actual temperature
rise.
( ∆T )isen T −T
Isentropic efficiency, hisen = = 02 01 (14.5a)
( ∆T )actual T02′ − T01

rp(g −1)/g − 1
or hisen = (14.5b)
rp( n −1)/ n − 1

p2
where     rp = and n > g.
p1
During compression process, work has to be done on the impeller. The energy balance then gives,
cpT01 = cpT02 - W
W = cp (T02 - T01)(14.6)
or  

Isentropic work c p (T02 − T01 ) T02 − T01


hisen =
Then       = = (14.7)
Actual work c p (T02′ − T01 ) T02′ − T01

14.10  ❐  VELOCITY DIAGRAMS


The velocity diagrams at inlet and outlet of the impeller for the centrifugal compressor are shown in
Fig. 14.6(a).

v w2 vf2 b2 Impeller vane


va2
u2 Diffuser
b3
a2 vr2
Impeller
b1
a4 a3
= v f1
vr1 v a1
a1 r1
Diffuser vane Diffuser vane
u1
b1
r2

N
r4
b2
b4

(a)

Figure 14.6  (a) Actual velocity diagrams

M14_THERMAL ENGINEERING_SE_XXXX_CH14.indd 749 6/15/2018 5:44:23 PM


750 Chapter 14

Let
   u1 = mean blade velocity at entrance
 u2 = mean blade velocity at exit
va1 = absolute velocity at inlet to rotor
va2 = absolute velocity at outlet to rotor
  vr1 = relative velocity at inlet to rotor
  vr2 = relative velocity at outlet to rotor
vw1 = velocity of whirl at inlet
vw2   = velocity of whirl at outlet
    vf1 = velocity of flow at inlet
vf 2     = velocity of flow at inlet
           D1, D2 = mean diameter of rotor at inlet and outlet, respectively
a1 = exit angle from stator or guide vanes at entrance
b1 = inlet angle to rotor in impeller blade angle at inlet
a2 = inlet angle to the diffuser or the stator
b2 = outlet angle from rotor or impeller blade angle at outlet.
If it is assumed that the entry of the air to the rotor is axial, then whirl component vw1 = 0 and
va1 = vf1.

14.10.1   Theory of Operation
(a) Ideal Velocity Diagrams
By Newton’s second law of motion, the rate of change of angular momentum of air is equal to
the torque applied to the body causing that change. Considering 1 kg/s of mass flow rate of air,
Torque, T = vw2 r2 - rw1r1(14.8)

D1 D
where         r1 = and r2 = 1
2 2
For ideal case, let us assume that the impeller is radial vaned, and there is no frictional loss, no
heat transfer and the air leaves the impeller with a tangential velocity vw2 = u2. Also for axial
entry, vw1 = 0
Torque, T = uw2 = u2r2(14.9)
Work done by impeller for 1 kg/s air flow rate,
w = u2r2w(14.10)

v1 vr1 v2 vr2

b1 a1 a2 b2
u1 u2 = vw2

(b)

Figure 14.6  (b) Ideal velocity diagrams

M14_THERMAL ENGINEERING_SE_XXXX_CH14.indd 750 6/15/2018 5:44:25 PM


Centrifugal Air Compressors 751

where w = angular speed of rotor, rad/s


Now,    r2w = u2
∴       w = u22(14.11)
  Since air cannot leave the impeller at a velocity greater than the impeller tip velocity,
Eq. (14.11) gives the maximum work capacity of the impeller.
Also, by steady flow energy equation, we have

 v2   v2   2   2 
w =  h2 + a 2  -  h1 + a1  = cp T + va 2  - cp T + sa1 
 2   2  2
2c p  1
2c p 
 
= cp (T02 - T01)(14.12a)
= h02 - h01(14.12b)
In most practical problems,
 va1 = va2
∴   w = cp (T2 - T1)(14.13a)
= h2 - h1

Now, w = cpT01  T02 − 1 (14.13b)


 T 
01
g −1
T01 = T1  p01  g
 
 p1 
g −1
T02 = T2  p02  g
 
 p2 
 g −1 
w = cpT01   p 
02 g 
  − 1 (14.14)
  p01  
 
 g −1 
= cpT01 ( r ) g − 1 (14.15)
 op 
 
po2
where rop = = stagnation pressure ratio
po1
 g −1 
If va1 = va2, then  w = cpT1  r g − 1 (14.16)
 p 
 
p
where, rp = 2 = static pressure ratio
p1

M14_THERMAL ENGINEERING_SE_XXXX_CH14.indd 751 6/15/2018 5:44:29 PM


752 Chapter 14

Comparing Eq. (14.11) and (14.14), we have

 g −1 
u22 = cpT01  p02  g − 1 (14.17)
  
 p01  
 
g
p02  u22  g −1
= + 1 (14.18)
p01  c pT01 

If va1 = va2, then


g
p  u2  g −1
2 =  2 + 1 (14.19)
p1  c pT1 

Total temperature increase across an impeller,

u22
    ΔT0 = = const. (14.20)
cp

 22
mu
Indicated power, IP = kW(14.21)
103
where m = mass flow rate of air, kg/s, and u2 is in m/s.
  Therefore, for a centrifugal compressor working under ideal condition, the power input depends
on the following:
1.  The mass flow rate of air
2.  The total pressure ratio of the compressor which, in turn, depends on the square of the impeller
velocity
3.  The total inlet temperature
4.  The total inlet temperature rise between the inlet and outlet
5.  There is a maximum work capacity of an impeller depending on the tip velocity
The maximum pressure produced depends on the following:
1.  Inlet temperature
2.  Square of the impeller velocity
3.  It is independent of the impeller diameter
(b) Actual Velocity Diagrams
The actual velocity diagrams are shown in Fig. 14.7.
Theoretical torque,   T = vw2r2 - vw1r1
Work done on 1 kg/s of air.
w = (vw2r2 - vw1r1)n
where, w = angular speed of rotor, rad/s
w = vw2 u2 = vw1u1 (14.22)

M14_THERMAL ENGINEERING_SE_XXXX_CH14.indd 752 6/15/2018 5:44:34 PM


Centrifugal Air Compressors 753

va2
vf2 vr2
vr2
vf1 va1
a2 b2
b1 a1
vw2
vw1
u1 u2

Figure 14.7  Actual velocity diagrams for centrifugal compressor

2
Now, vr1 = u12 + va1
2
- 2u1va1 cos a1 = u12 + va1
2
- 2u1 vw1
2
vr2 = u22 + va2
2
- 2u2va2 cos a2 = u22 + va2
2
- 2v2 vw2
1
 u1vw1 = (u12 + va1
2 2
- vr1)
2
1
 u2vw2 = (u22 + va2
2 2
- vr2 )
2
va22 − aa21 vr21 − vr22 u22 − u12
w = + + (14.23)
2 2 2
= ΔKE + Δp due to diffusion action + Δp due to centrifugal
action
Therefore, the fraction of kinetic energy imparted to air and converted into pressure energy in
impeller is given by,
vr21 − vr22 u22 − u12 2 dp
2
+
2
= ∫1 r
(14.24)
where r = density of air.
If na1 = diffuser outlet velocity, then
va22 − va21 2 dp

2
= ∫1 r
+ (Δh)diff. loss(14.25)

14.10.2  Width of Blades of Impeller and Diffuser


Air mass flow rate per second,
Circumferential area of flow × v f 1
m a =
Specific volume of air

(pD1 − nt )b1 × v f 1 pD1kb b1v f 1


= = (14.26a)
vs1 vs1

(pD2 − nt )b2 × v f 2 pD2 kb b2 v f 2


= = (14.26b)
vs 2 vs 2

M14_THERMAL ENGINEERING_SE_XXXX_CH14.indd 753 6/15/2018 5:44:45 PM


754 Chapter 14

where,  ns = specific volume of air


b1, b2 = width or height of impeller blades at inlet and outlet, respectively
kb = blade factor
n = number of blades
t = thickness of blades
The width or height of the diffuser blades at the outlet is given by,
(p Dd − ntd )bd × v fd
m a = (14.27)
vsd
where suffix ‘d’ represents the quantities at the diffuser outlet. The width or height of the diffuser
blades at the inlet is the same as that of the impeller blades at outlet.

14.11  ❐  SLIP FACTOR AND PRESSURE COEFFICIENT


In the ideal velocity diagrams, we had assumed that nw2 = u2. However, in actual operation, this condi-
tion is not satisfied in actual practice due to secondary flow effects. Actually, nw2 < u2. The difference
(u2 - nw2) is called the slip. The slip factor is defined as the ratio of actual whirl component to the ideal
whirl component and is denoted by fs.
vw 2
Slip factor, fs = (14.28)
u2
The total head pressure ratio is given by,
g −1 g −1
p02  T02  g  T −T  g
= = 1 + 02 01 
p01  T01   T01 

Substituting the value of (T02 - T01) from Eq. (10.5), we have


g −1
p02  hisen (T02′ − T01 )  g
= 1 +  (14.29)
p01  T01 
The actual work done per kg of air is given by:
cp (T02′ - T01) = u2nw2
The actual work done per kg of air by the compressor is always greater than u2nw2 due to fluid friction
and windage losses. Therefore, the actual work is obtained by multiplying u2nw2 by the factor fw,
called the work factor or power input factor.
cp (T02′ - T01) = fwu2nw2
fw u2 vw 2
T02′ - T01 =
  (14.30)
cp
Eq. (10.29) becomes g −1
p  h f u v  g
      02 = 1 + isen w 2 w 2 
p01  c pT01 

M14_THERMAL ENGINEERING_SE_XXXX_CH14.indd 754 6/15/2018 5:44:49 PM


Centrifugal Air Compressors 755

Now, vw 2 = fs u2
g −1
p  h f f u2  g
02 = 1 + isen w s 2  (14.31)
p01  c pT01 
The pressure coefficient is defined as the ratio of isentropic work to Euler work and is denoted by fp.
Isentropic work c p (T02 − T01 )
Pressure coefficient,  fp = = (14.32)
Euler work u2 vw 2
Using Eq. (14.5) and assuming radial vanes of impeller, i.e., nw2 = u2, we get
c phisen (T02′ − T01 )
fp =
u22
Using Eqs (14.28) and (14.30), we get
fp = fwfshisen(14.33)
  

14.12  ❐  LOSSES
There are two types of losses in a centrifugal compressor: internal and external. The internal losses
sustained in the compressor are manifested by an increase of enthalpy of air. The internal losses which
occur in compressor are as follows:
1. Friction between air and walls of flow passage
2. Disc friction
3. Leakage between impeller and casing
4. Turbulence
5. Shock
The compressor sustains an external loss in the form of bearing friction and windage loss. The friction
losses are proportional to va2 and hence proportional to m a2. The incidence loss in terms of drag coef-
ficient CD is proportional to CD v 2 .
a

14.13  ❐ EFFECT OF IMPELLER BLADE SHAPE


ON PERFORMANCE
The following types of blade shapes are used for impellers of centrifugal compressors:
1. Backward-curved blades (b2 < 90°)
2. Radial blades (b2 = 90°)
3. Forward-curved blades (b2 > 90°).
The characteristics of these blades are shown in Fig. 14.8. Centrifugal effects on the curved blades
create a bending moment and produce increased stresses which limit the maximum speed at which
the impeller can run.
Slightly backward-curved impeller blades (i.e., b2 < 90°) give optimum efficiency because the
degree of reaction increases with decreasing blade tip angles. With increase in degree of reaction, the
part of kinetic energy that is transferred into pressure energy within the impeller is also increased.

M14_THERMAL ENGINEERING_SE_XXXX_CH14.indd 755 6/15/2018 5:44:52 PM


756 Chapter 14

b 2 > 90°
u rved
ard c
Forw

Pressure ratio
Radial b 2 − 90°
Bac
kwa
rd c
urved b 2 < 90°

Mass flow rate

Figure 14.8  Characteristics of blades

By selecting very small tip blade angles b2, the length of impeller channel increases and consequently
the friction loss increases considerably, thereby reducing the efficiency. The optimum blade angle is
about 45°.
Radial blades are used for aircraft centrifugal compressors because they can be manufactured
easily, have the lowest centrifugal stresses, are free from bending stresses, and have equal energy con-
version in impeller and diffuser, giving higher pressure ratio and efficiency.

14.14  ❐  DIFFUSER
The diffuser converts the kinetic energy imparted to air by the impeller into pressure rise. In the vaned
diffuser, the vanes are used to remove the whirl of the fluid at a higher rate than is possible by a sim-
ple increase in radius, thereby reducing the length of flow path and diameter. A typical vaned type
diffuser is shown in Fig. 14.9. A pressure gradient exists in the diffuser in the direction opposite to that
of flow, and the flow streamlines will tend to break away from the diverging passage walls, reversing
their direction and resulting in turbulence. About 20° is accepted as the maximum included angle of
divergence for satisfactory diffusion.
The clearance between the impeller and the vanes leading edges constitutes a vaneless diffuser.
This helps smoothen out velocity variation between the impeller tip and diffuser vanes. This also
reduces the circumferential pressure gradient at the impeller tip.
The flow follows an approximately logarithmic spiral path to the vanes after which it is constrained
by the diffuser channels.

Diffiuser vanes

Impeller

Figure 14.9  Vaned diffuser

M14_THERMAL ENGINEERING_SE_XXXX_CH14.indd 756 6/15/2018 5:44:53 PM


Centrifugal Air Compressors 757

p4 − p3
Diffuser efficiency, hd = (14.34)
1 2 2
r(va3 − va 4 )
2
where suffices 3 and 4 denote upstream and downstream conditions of diffuser and r is the mass
density of air.

14.15  ❐  PRE-WHIRL
In order to avoid acceleration to sonic or supersonic velocities, the Mach number of flow entering the
impeller eye should be kept below the value of 0.9. When the absolute velocity of approach is high
enough or the static absolute temperature of the entering air is low enough to make the Mach number
more than 0.9, the method of providing a pre-whirl and thereby reducing the relative velocity entering
the impeller eye is quite helpful.

Impeller vane

va1 vr1 vr1


vf1 va1 = vf1
a1 a1 = 90°
b1 b1
v1
vw1

Pre-whirl vane
(a) (b)

Figure 14.10 Effect of pre-whirl on inlet velocity triangle: (a) With pre-whirl


(vw ≠ 0), (b) Without pre-whirl (vw = 0)

Pre-whirl is provided by intake guide vane as shown in Fig. 14.10, which shows the velocity triangles
for blades without pre-whirl and with pre-whirl. It may be observed that the magnitude of the relative
velocity with pre-whirl vane gets reduced at the entrance, but the flow enters with a large relative angle.
Thus, the flow inlet Mach number is lowered by pre-whirl vane. However, since vw1 ≠ 0 with pre-whirl
vane, the work done on the air must be greater for the same compressor inlet and outlet conditions.

14.16  ❐  PERFORMANCE CHARACTERISTICS


Let us consider the following variables for the performance of the compressor.
p3 = pressure at the outlet of diffuser
p1 = inlet pressure
T1 = inlet temperature
D = diameter
G = mass rate of flow
N = r.p.m of rotor

M14_THERMAL ENGINEERING_SE_XXXX_CH14.indd 757 6/15/2018 5:44:54 PM


758 Chapter 14

Then, p3 = f (p1, T1, D, G, N)


The following dimensions are chosen:
M = mass (kg), L = length (m), t = time (s)
 kg × m 
p = M1L-1t -2  2
 s × m 2 
Therefore,  

 Heat Nm kg × m × m 
T = L2t -2  Mass = kg = 
 kg × s 2 
D = L; G = M1t-1; N = t-1 (rpm = rad/s)
Number of variables, n = 6
Basic dimensions, m = 3
According to Buckingham’s Pi theorem, the number of dimensionless groups
=n-m=6-3=3
The three dimensionless groups are formed by combining p1, T1, and D with each of the remaining
variables, so that
a b
p1 = p1 1 T1 1 D c1 p3
a b
p2 = p1 2 T1 2 D c2 G
a b
p3 = p1 3 T1 3 D c3 N
Substituting the basic dimensions for the variables in p1, we have
p1 = ( M 1L−1t −2 )a1 ( L2t −2 )b1 Lc1 (M2L-1t-2)1
For M, 0 = a1 + 1
For L, 0 = - a1 + 2b1 + c1 -1
For t, 0 = - a1 - 2b1 -2
From which, a1 = -1, b1 = 0, c1 = 0
1 p3
∴     p1 = p1−1T10 D0 p3 =
p1

G T1
Similarly,        p2 = p1−1T10.5D-2G-1 =
p1D 2
DN
p3 = p10T1−0.5D1N1 =
T1
p1 = f1 (p2, p3)
Thus,    

p3  
= f2 G T1 , DN (14.35)
p1  2 
 p1D T1 
For a given compressor under consideration, D = const.
p3  
= f3 G T1 , N (14.36)
p1  
 p1 T1 

M14_THERMAL ENGINEERING_SE_XXXX_CH14.indd 758 6/15/2018 5:45:00 PM


Centrifugal Air Compressors 759

5
Operating line

4
Surge line Design
point

p3/p1
3

0.6 0.8
0.4
1
0.2 0.4 0.6 0.8 1.0 1.2

G√T1 (Referred t design value)


c
p1

p3 G T1
Figure 14.11  v’s
p1 p1

T3
Similarly, the ratio can be formed to be function of the dimensionless groups of Eqs. (14.35) and
T1
(14.36). Similarly, if stagnation temperature and pressure are considered, the result will again be the
same except that stagnation temperatures and pressures are substituted in place of static pressures and
temperatures.

Lines of constant speed


3.2

Lines of constant
efficiency
2.8 Locus of point of
maximum efficiency

2.4
Surge line
1300
Pressure ratio

12

350 r.p.m
00
11

2.0 Maximum
00

efficiency
10
00

1.6
90
0
80
70

0
0

1.2
60 80 100 120 140 160 180
Volume flow m2 per min

Figure 14.12  Pressure ratio v’s Volume flow rate

M14_THERMAL ENGINEERING_SE_XXXX_CH14.indd 759 6/15/2018 5:45:03 PM


760 Chapter 14

The performance of a given compressor or geometrically similar compressor can be plotted in


terms of three dimensionless parameters provided dynamic similarity is maintained. Fig. 14.11 shows
G T1 p N
a plot of against 3 and . The operating line is the locus of the points of maximum
p1 p1 T1
N
efficiency at various values of . The surge line represents the stable operation, the region to the
T1
right of the surge line being for stable conditions.
The performance of the compressor has been plotted in terms of volume flow rate and pressure
ratio in Fig. 14.12.

14.17  ❐  SURGING AND CHOKING


The delivery pressure v’s mass flow rate in a dynamic compressor is shown in Fig. 14.13. Suppose
a discharge valve is put in the line for this compressor. The mass rate of flow will be zero when the
discharge valve is closed and the static pressure developed is that delivered by the impeller to the air
contained in the compressor. Such a situation is shown by point a in Fig. 14.13. If the valve is now
opened, the flow of air takes place and the diffuser becomes effective in increasing the static pressure.
This is shown by points b and c. The maximum delivery pressure is obtained at point c. As the valve is
opened more, the mass flow rate increases beyond point c and the efficiency of compressor decreases
with decrease in delivery pressure. When the designed mass rate is greatly exceeded, the incidence
between the vane and air angle becomes so large that flow separation and shock occur, accompanied
by rapid decrease in efficiency.
If the compressor operates at some point b to the left of point c, then a decrease in mass rate of flow
is accompanied by decrease in pressure developed by the compressor. If the static pressure of the air at
compressor outlet does not decrease as rapidly as the developed pressure, there is a natural tendency of
the air to flow back into the compressor in the direction of pressure gradient. With the drop in pressure
at the compressor outlet, the pressure gradient is reversed and also the direction of flow is reversed.
Thus, this leads to unstable condition of cyclic reversal taking place at extremely high frequencies.
This pulsating air flow phenomena is called ‘surging’. Such a situation is avoided by keeping the oper-
ating point to the right of point c, because in this region a decrease in mass flow rate is accompanied
by increase in delivery pressure, leading to stability.

c
b d
a
Delivery pressure

Surging region: a-b-c


e
No surging: c-d-e
Choking: e

f
Mass flow rate

Figure 14.13  Surging and choking phenomena

M14_THERMAL ENGINEERING_SE_XXXX_CH14.indd 760 6/15/2018 5:45:05 PM


Centrifugal Air Compressors 761

After point c, any increase in mass flow rate is accompanied by decrease in delivery pressure.
This happens because the rate of increase in pressure loss due to the friction is more than the rate of
increase in pressure gain by diffuser. Theoretically, the decrease in delivery pressure is continued until
the mass flow rate is represented by the point ‘f  ’. In practice, the maximum mass flow is limited by
the point e because if the mass flow exceeds design mass flow, the air angles are widely different from
vane angles and choking takes place. The point e represents the choking of the compressor, that is, the
maximum mass flow rate condition.
Surging does not take place in the region ce as the reduction in mass flow rate is accompanied
with the increase in delivery pressure and flow reversal is not possible and stability of operation is
maintained.

Example 14.1
A centrifugal air compressor having a pressure ratio of 5, compresses air at a rate of 10 kg/s. If the
initial pressure and temperature are 1 bar and 20°C respectively, find the final temperature of air
and power required to drive the compressor. Take g = 1.4 and cp = 1 kJ/kg.K.
Solution
Given: rp = 5, m = 10 kg/s, p1 = 1 bar, T1 = 273 + 20 = 293 K, g = 1.4, cp = 1 kJ/kg.K
g −1
T2 g
= ( rp )
T1

0.4
or    T2 = 293 (5) 1.4 = 464 K or 191°C
Power required to drive the compressor,
P = mc
 p (T2 - T1) = 10 × 1 × (464 - 293) = 1710 kW.

Example 14.2
A centrifugal compressor has a pressure ratio of 4:1 with an isentropic efficiency of 82% when
running at 16,000 rpm. It takes in air at 17°C. Guide vanes at inlet give the air a pre whirl of 20° to
the axial direction at all radii and the mean diameter of the eye is 200 mm, the absolute air velocity
at inlet is 120 m/s. At exit, the blades are radially inclined and the impeller tip diameter is 550 mm.
Calculate the slip factor of the compressor.

p2

2′
2 p1

Figure 14.14  T – s Diagram

M14_THERMAL ENGINEERING_SE_XXXX_CH14.indd 761 6/15/2018 5:45:07 PM


762 Chapter 14

Solution
p2
= 4, T1 = 273 + 17 = 290 K, N = 16,000 rpm, a1 = 90° - 20° = 70°
p1
d1 = 200 mm, va1 = 120 m/s, hisen = 0.82, d2 = 550 mm, b2 = 90°
Temperature after isentropic compression (Fig. 14.14)
g −1 0.1
T2 = T1  p2  g
= 290( 4) .4
0 = 430.94 K
 p 
1
Isentropic temperature rise,
(∆T)isen = 430.94 - 390 = 140.94 K
Actual temperature rise,
( ∆T )isen 140.94
( ∆T )a = =
hisen 0.82
= 171.88 K
Power input per unit mass flow rate
= cp × (∆T)a = 1.005 × 171.88 = 172.74 kJ/kg
An inlet to impeller (Fig. 14.15 (a)),
va1 = 120 m/s
Angle of pre-whirl = 20°
pd1N p × 200 × 16000
u1 = = = 167.58 m/s
3
60 × 10 60 × 103
uw1 = va1 sin 20° = 120 sin 20° = 41.04 m/s
At exit of impeller (Fig. 14.15(b))
pd2 N p × 550 × 16000
u2 = = = 460.77 m/s
3
60 × 10 60 × 103

Real Ideal
Angle of
prewhirl
20°
va1 v′a2
vr1 vr2 = vf2
vf1 v′r2
va2

b ′2
a1 b1 a ′2
a2 b2

vw1 vw2
u1 vw2 = u2

(a) (b)

Figure 14.15 Velocity diagrams for centrifugal compressor: (a) Inlet, (b) Outlet

M14_THERMAL ENGINEERING_SE_XXXX_CH14.indd 762 6/15/2018 5:45:11 PM


Centrifugal Air Compressors 763

For radial discharge, vw2 = u2 = 460.77 m/s


Power input per unit mass flow rate
= u2 v′ - u1vw1
w2

= 460.77 v′ w2 - 167.55 × 41.04 = 172.74 × 103


v′ w2 = 389.82 m/s
v′ w 2 389.82
Slip factor = = = 0.846
v′ w 2 460.77

Example 14.3
A centrifugal compressor running at 15000 rpm takes in air at 15°C and compresses it through a
pressure ratio of 4 with an isentropic efficiency of 80%. The blades are radially inclined and the slip
factor is 0.85. Guide vanes at inlet give the air an angle of pre-whirl of 20° to the axial direction.
The mean diameter of the impeller eye is 200 mm and the absolute air velocity at inlet is 120 m/s.
Calculate the impeller tip diameter. Take cp = 1.005 kJ/kg.K and g = 1.4.

Solution
Given: N = 15000 rpm, T1 = 273 + 15 = 288 K, rp = 4, hisen = 80%, fs = 0.85, va1 = 120 m/s,
d1 = 200 mm, a1 = 70°

T
p2

2′
2 p1

s
(a)

v′a2
20° vf2 = vr2
va1 va2
vr1
vf1
20°

a1 b2 a2 b2

vw1 v′w2
u1 vw2
u2
(b)

Figure 14.16  (a) T-s diagram, (b) Velocity triangles.

M14_THERMAL ENGINEERING_SE_XXXX_CH14.indd 763 6/15/2018 5:45:14 PM


764 Chapter 14

From Fig. 14.16 (a), we have


g −1 0.4
T2 g
= ( rp ) = 4 .4
1 = 1.486
T1
or T2 = 288 × 1.486 = 428 K
T −T
hise = 2 1
T2′ − T1
428 − 288
or ∆T = T2′ - T1 = = 175 K
0.8
Compressor work, wc = cp · ∆T = 1.005 × 175 = 175.875 kJ/kg
p d1N p × 200 × 15000
u1 = = = 157 m/s
3
60 × 10 60 × 103
From Fig. 14.16 (b), vw1 = va1 cos 70°
= 120 cos 70° = 41.04 m/s
For radial discharge, b2 = 90° and
vw2 = u2
vw′ 2
Slip factor,     fs = = 0.85
vw 2
′ = 0.85 u2
vw2
Power input = u2 vw2
′ - u1 vw1
175·875 × 103 = u2 × 0.85 u2 - 157 × 41.04
or u2 = 463.13 m/s
p d2 N p d2 × 15000
         = =
3
60 × 10 60 × 103
d2 = 589.7 mm
or   

Example 14.4
The free air delivered by a centrifugal compressor is 25 kg/min. The suction condition is 1 bar and
25°C. The velocity of air at inlet is 50 m/s. The isentropic efficiency of the compressor is 70%.
If the total head pressure ratio of the compressor is 4, determine (a) the total head temperature of
air at the exit of the compressor, and (b) brake power required to run the compressor assuming
mechanical efficiency of 96%. Pressure and temperature of air at inlet are static. For air g = 1.4 and
R = 0.287 kJ/kg. K.
Solution
cp
cp = R + cn = R +
g

gR 1.4 × 0.287
cp = = = 1.005 kJ/kg. K
g −1 0.4

M14_THERMAL ENGINEERING_SE_XXXX_CH14.indd 764 6/15/2018 5:45:18 PM


Centrifugal Air Compressors 765

T p02
T02′ 02′
p2
T02 02
v22
T2′ 2cp 2′
2
T2 p01
p1
T01 01
v21
T1 2cp
1
S

Figure 14.17  T–s diagram for centrifugal compressor


Refer to Fig. 14.17.
v12
T01 = T1 +
2c p

50 2
= 298 +
2 × 1.005 × 103
= 299.24 K
g −1 1
T02 = T01  p02  g
= 299.24 ( 4) 3.5 = 444.67 K
 p 
01

T02 − T01
hisen =
T02′ − T01

444.67 − 299.24
0.7 =
T02′ − 299.124
T02′ = 507 K
m a c p (T02′ − T01 )
Brake power required to run the compressor =
h mech

25 × 1.005 (507 − 299.24)


= = 90.62 kW
0.96 × 60

Example 14.5
Air at a temperature of 20°C flows into the centrifugal compressor running at 21,000 rpm. The
following data are given:
Slip factor = 0.85
Work input factor = 1.0
Isentropic efficiency = 72%
Outer diameter of blade tip = 60 cm

M14_THERMAL ENGINEERING_SE_XXXX_CH14.indd 765 6/15/2018 5:45:22 PM


766 Chapter 14

Assuming the absolute velocities of air entering and leaving the compressor are same, determine:
(a) Temperature rise of air passing through compressor, and
(b) Static pressure ratio
Take cp = 1.005 kJ/kg. K.

Solution
pD2 N p × 60 × 21000
  u2 = = = 659.7 m/s
60 × 100 60 × 100
Work done per kg of air = u2vw2 - u1vw1
vw 2
 = u22fsfw[∵ vw1 = 0 and fs = ]
u2
= (659.7)2 × 0.85 × 10-3 × 1 = 369.923 kJ/s
Also work done = cp (T2′ - T1) = 1.005 (T2′ - T1) = 369.923 [as va1 = va2]

or      T2′ - T1 = 368.1 K


T −T
hisen = 2 1
T2′ − T1
or T2 = 293 + 0.72 × 368.1 = 558 K
Static pressure ratio, g
3.5
p2  T2  g −1  558 
=  =  = 9.53
p1  T1   293 

Example 14.6
A centrifugal compressor handles 180 kg/min of air. The suction pressure and temperature are 1 bar
and 300 K. The suction velocity is 90 m/s. The delivery conditions are 2 bar, 400 K, and 240 m/s.
Calculate (a) the isentropic efficiency, (b) the power required to drive the compressor, and (c) the
overall efficiency of the unit. Take cp = 1.005 kJ/kg. K. Assume that entire kinetic energy gained in
the impeller is converted into pressure in the diffuser.

Solution
180
m = = 3 kg/s, p1 = 1 bar, T1 = 300 K, va1 = 90 m/s, p2 = 2 bar, T2 = 350 K, va2 = 240 m/s
60
Isentropic work
(a) Isentropic efficiency, hisen =
Actual work
g −1 1
T2  p2  g  2  3.5
= =  = 1.219
T1  p1   1

T2 = 300 × 1.219 = 365.7 K

va22 − va21
Isentropic work done, wisen = cp(T2 - T1) +
2 × 103

M14_THERMAL ENGINEERING_SE_XXXX_CH14.indd 766 6/15/2018 5:56:24 PM


Centrifugal Air Compressors 767

240 2 − 90 2
= 1.005 (365.7 - 300) +
2 × 103
= 66.028 + 24.75 = 90.778 kJ/kg
Actual work done in the impeller,
240 2 − 90 2
wa = 1.005 (400 - 300) +
2 × 103
= 100.5 + 24.75 = 125.25 kJ/kg
90.778
hisen = = 0.7248 or 72.48%
125.25
(b) Power required to drive the compressor = m × wa = 3 × 125.25 = 375.75 kW
(c) Let T3 = temperature at the exit of diffuser
va22 − va21
 cp (T3 - T2) =
2 × 103
or      1.005 (T3 - 400) = 24.75
or T3 = 424.6 K
g
p3  T3  g −1  424.6  3.5
= = = 1.2326
p2  T2   400 

or p3 = 2 × 1.2326 = 2.465 bar


After isentropic compression, the delivery temperature from diffuser is,
g −1 1
T3 ′  p3  g  2.465  3.5
= = = 1.294
T1  p1   1 

T3 ′ = 300 × 1.294 = 388.2 K


or       
T3 ′ − T1 388.2 − 300
Overall efficiency, hoverall = =
T3 − T1 424.6 − 300
88.2
= = 0.7078 or 70.78%
124.6

Example 14.7
A centrifugal compressor handles 540 kg/min of air at 1 bar and 20°C ambient conditions. The air is
compressed from 1 bar static pressure to 4.5 bar total pressure. The air enters the impeller eye with a
velocity of 150 m/s with no pre-whirl. The ratio of whirl speed to tip speed is 0.9. Calculate (a) the rise
in total temperature during compression if the change in kinetic energy is negligible, (b) the tip diam-
eter of the impeller, (c) the power required, and (d) the eye diameter if the hub diameter is 130 mm.
Assume that the compressor runs to 20,000 rpm with isentropic efficiency of 82%.
Solution
540
m = = 9 kg/s, p01 = p1 = 1 bar, T1 = 273 + 20 = 293 K, N = 20,000 rpm, hisen = 0.82,
60

M14_THERMAL ENGINEERING_SE_XXXX_CH14.indd 767 6/15/2018 5:56:29 PM


768 Chapter 14

T p02
T02′ 02′
p2
02
v22
2 2cp 2′
T2′
p01
p1
T01 01
v21
T1 2cp
1
S

Figure 14.18  T–s diagram

vw 2
p02 = 4.5 bar, va1 = 150 m/s, fs = = 0.9, dh = 130 mm = 0.13 m
u2
The T - s diagram is shown in Fig. 14.18.
v2 150 2
(a) T01 = T1 + a1 = 293 + = 304.2 K
2c p 2 × 103 × 1.005
g 3.5
 304.2 
p01 = p1  T01  g −1 = 1  = 1.14 bar
 T   293 
1
g −1 1
T02  p02  g  4.5  3.5
= = = 1.48
T01  p01   1.14 

or T02 = 304.2 × 1.48 = 450.3 K


Isentropic rise in total temperature,
(∆T )isen = 450.3 - 304.2 = 146.13 K
Actual rise in total temperature,
( ∆T )isen 146.13
(∆T )actual = = = 178.2 K
hisen 0.82
(b) Actual work consumed by the compressor,
wactual = cp (∆T)actual = 1.005 × 178.2 = 179.1 kJ/kg
Work consumed given by Euler’s equation without pre-whirl,
u v
wactual = 2 w 2 kJ/kg
103
vw2 = fsu2 = 0.9 × u2
0.9u22
        179.1 =
103
or   u2 = 446.08 m/s
p D2 N
u2 =
Again  
60

M14_THERMAL ENGINEERING_SE_XXXX_CH14.indd 768 6/15/2018 5:56:34 PM


Centrifugal Air Compressors 769

Tip diameter of impeller,


446.08 × 60
D2 = = 0.426 m or 426 mm
p × 20,000

(c) Power required = m × wactual = 9 × 179.1 = 1611.9 kW

p1 1 × 105
(d) r1 = = = 1.189 kg/m3
RT1 287 × 293
p
m = ( D12 − db2 ) × va1 × r1
4
p
[ D 2 − (0.13)2 ] × 150 × 1.189
9=
4 1
or     D1 = 0.2848 m or 284.8 mm

Example 14.8
A centrifugal compressor delivers 720 m3/min of free air while running at 12,000 rpm. The ambi-
ent air conditions are 1 bar and 27°C. The air is compressed to 4.2 bar with an isentropic efficiency
of 0.85. Blades are radial at the outlet of the impeller and the flow velocity is 60 m/s, which may
be assumed constant throughout. The outer radius of impeller is twice the inner and the slip factor
is 0.9. The blade area coefficient is 0.9 at inlet. Determine (a) the final temperature of air, (b) the
theoretical power, (c) the inlet and outlet diameter of impeller, (d) the breadth of impeller at inlet,
(e) the impeller blade angle at inlet, and (f) the diffuser blade angle at inlet.

Solution
p1 = 1 bar, T1 = 273 + 27 = 300 K, p2 = 4.2 bar, hisen = 0.85, vf2 = 60 m/s, r2 = 2r1, fs = 0.9, kb = 0.9.
g −1 1
p  g  4.2  3.5
(a) T2 = T1  2  = 300  = 452 K
 p1   1 
T2 − T1
hisen =

T2′ − T1
452 − 300
or  T2′ = 300 + = 478.8 K
0.85
p1V 1 × 105 × 720
(b) Mass flow rate, m = = = 13.94 kg/s
RT1 60 × 287 × 300
Theoretical power, Pth = mc
 p (T2′ - T1)
= 13.94 × 1.005 (478.8 - 300) = 2504.4 kW

fs u22
(c) Work done = = cp (T2′ - T1)
103
or  0.9 u22 = 103 × 1.005 (478.8 - 300)

M14_THERMAL ENGINEERING_SE_XXXX_CH14.indd 769 6/15/2018 5:56:40 PM


770 Chapter 14

pD2 N
or  u2 = 446.8 m/s =
60
60 × 446.8
or  D2 = = 0.711 m or 711 mm
p × 12000
D
D1 = 2 = 355.6 mm
2
(d) Volume flow rate = p D1b1kbvf1
720
or  b1 = = 0.099 m or 99 mm
60 × p × 0.711 × 0.9 × 60
vf 60
(e) tan b1 = 1 = = 0.26858
u1  446 .8 
 
2 
or  b 1= 15.03°
vf 60
(f) tan a2 = 2 = = 0.1492
fs u2 0.9 × 446.8
or  a2 = 8.48°

Example 14.9
A centrifugal blower compresses 5.4 m3/s of air from 1 bar and 25°C to 1.5 bar. The index of com-
pression is 1.5. The flow velocity of 70 m/s is constant throughout. The inlet and outlet diameters
of impeller are 0.30 m and 0.60 m, respectively. The blower rotates at 9000 rpm. Determine (a) the
blade angles at inlet and outlet of the impeller, (b) the absolute angle at the tip of the impeller, and
(c) the breadth of blade at inlet and outlet.
Assume that no diffuser is employed and the whole pressure increase occurs in the impeller and
the blades have negligible thickness.

Solution
V1 = 5.4 m3/s, p1 = 1 bar, T1 = 273 + 25 = 298 K, n = 1.5, p2 = 1.5 bar, v f1 = v f 2 = 70 m/s, D1 = 0.3 m,
D2 = 0.6 m, N = 9000 rpm
n −1 0.5
 1.5  1.5
T2 = T1  p2  n
= 298   = 341.1 K
 p   1
1

pD1N p × 0.3 × 9000


u1 = = = 141.37 m/s
60 60
u2 = 2u1 = 282.74 m/s
u2 vw 2
Work done = cp (T2 - T1) =
103

1.005(341.1 − 298) × 103


or vw2 = = 153.2 m/s
282.74

M14_THERMAL ENGINEERING_SE_XXXX_CH14.indd 770 6/15/2018 5:56:46 PM


Centrifugal Air Compressors 771

v f170
(a) tan b1 = =
= 0.4951 [see Fig. 14.6(b)]
u1 141.37
or  b1 = 26.34°
v f2 70
tan b2 = = = 0.5404 [see Fig. 14.7]
u2 − vw 2 282.74 − 153.2
or   b2 = 28.38°
vf 2 70
(b) tan a2 = = = 0.4569
vw 2 153.2
or  a2 = 24.56°
(c) V1 = p D1b1vf1
5.4
or  b1 = = 0.0818 m or 81.8 mm
p × 0.3 × 70
p V T 1 × 5.4 × 341.1
V2 = 1 1 × 2 = = 4.12 m3/s
T1 p2 298 × 1.5
V = p D b v
2 2 2 f2
4.12
or  b2 = = 0.0312 m or 31.2 mm
p × 0.6 × 70

Example 14.10
A centrifugal compressor delivers 18 kg/s of air with a total head pressure ratio of 4 : 1. The speed
of the compressor is 16000 rpm. Total head temperature at inlet is 22°C, slip factor 0.9, power input
factor 1.05 and isentropic efficiency 0.82. Calculate (a) the overall diameter of the impeller and (b)
the power input.

Solution
m = 18 kg/s, rop = 4, N = 16000 rpm, T01 = 273 + 22 = 295 K, fs = 0.9, fw = 1.05, hisen = 0.82
g −1
1
T p  g
(a) 02 =  02  = ( 4) 3.5 = 1.486
T01  p01 

or       T02 = 295 × 1.486 = 438.4 K


T02 − T01
hisen =
T02′ − T01
438.4 − 295
or      0.82 =
T02′ − 295
or    T02′ = 469.8 K

2
f fu
Work done = w s 2 = cp (T02′ - T01)
103
or 1.05 × 0.9 × u22 = 103 × 1.005 (469.8 - 295)

M14_THERMAL ENGINEERING_SE_XXXX_CH14.indd 771 6/15/2018 5:56:53 PM


772 Chapter 14

or    u2 = 431.2 m/s
60 × 431.2
D2 = = 0.5147 m or 514.7 mm
p × 16000
(b) Power input, P = m cp (T02′ - T01)
= 18 × 1.005 (469.8 - 295) = 3162.1 kW
Example 14.11
A single-sided centrifugal compressor impeller fitted with a diffuser, without water cooling,
requires 2000 kW when running at 16,500 rpm. The outlet diameter of impeller is 0.5 m and the
width of the casing of the vortex chamber between the impeller and the diffuser is 50 mm. During
a test on this compressor, static temperature and pressure were measured at a radius of 0.28 m and
were found to be 120°C and 3 bar, respectively. Assuming whirl speed at the tip of the impeller to
be 0.95 of the peripheral speed and neglecting friction in vortex chamber, calculate (a) the flow
rate, (b) the resultant speed at the section given, and (c) the total temperature at the section given.
Take cp = 1.005 kJ/kg, K, g = 1.4 and intake pressure = 1 bar.

Solution
P = 2000 kW, N = 16500 rpm, D2 = 0.5 m, b = 50 mm, fs = 0.95, p1 = 1 bar
(a) Work done = vw2u2
pD2 N p × 0.5 × 16500
u2 = = = 432 m/s
60 60
vw2 = fsu2 = 0.95 × 432 = 410.4 m/s

vw 2 u2 410.4 × 432
Power per kg of air = = = 177.28 kW
3
10 103
P 2000
Air mass flow rate, m = = = 11.28 kg/s
177.28 177.28
(b) Using suffix 3 for the given section
Area of flow, Af 3 = 2p   r3b3 = 2p × 0.28 × 50 × 10-3 = 0.088 m2
Volume rate of flow at the given section,
 3 11.28 × 287 × 393
mRT
V3 = = = 4.24 m3/s
p3 3 × 105
V 4.24
vr3 = v f3 = 3 = = 48.19 m/s
A f3 0.088
For free vortex in vaneless space, vw1 × r1 = const.
i.e., vw1r1 = vw2r2 = vw3r3
vw 2 r2 410.4 × 0.25
or     vw3 = = = 366.4 m/s
r3 0.28

   va3 = v 2f 3 + vw2 3 [∵ vf3 = vr3]

M14_THERMAL ENGINEERING_SE_XXXX_CH14.indd 772 6/15/2018 5:57:00 PM


Centrifugal Air Compressors 773

= ( 48.19)2 + (366.4)2 = 369.6 m/s

va23 (369.6)2
(c) T03 = T3 + = 393 + = 461 K or 188°C
2c p 2 × 1.005 × 103

Example 14.12
A centrifugal compressor delivers 580 m3/min of free air when running at 8000 rpm. Use the fol-
lowing data: inlet pressure and temperature of air = 1.013 bar and 20°C, pressure ratio = 3.5, isen-
tropic efficiency = 83 %, flow velocity throughout the compressor = 62 m/s, the blades are radial at
the outlet of the impelier, tip diameter = 2 times eye diameter, blade area coefficient = 0.94. Find
(a) the input power required to run the compressor, (b) the impeller diameters at inlet and outlet,
(c) the breadth of impeller at inlet, and (d) the impeller blade angle at inlet.

Solution
Given that Va = 580 m3/min, N = 800 rpm, p1 = 1.013 bar, T1 = 273 + 20 = 293 K, rp = 3.5, hisen = 0.83, vf =
62 m/s, b2 = 90°, D2 = 2D1, kt = 0.94
g −1
1.4 −1
p  g
T2 = T1  2  = 293(3.5) 1.4 = 419.1 K
 p1 
T2 − T1
hisen =
T2 − T1

419.1 − 293
or       0.83 =
T2′ − 293
T2 ′ = 444.9 K
or   
p1Va 1.013 × 105 × 580
Mass flow rate of air, m a = = = 11.645 kg/s
RT1 287 × 293 × 60
(a) Input power required = m a cp ( T2 ′ - T1) = 11.645 × 1.005(444.9 - 293)
= 1777.72 kW
(b) For radial blades, work done on compressor,
u22
W= = cp ( T2 ′ - T1)
103
or u22 = 103 × 1.005(444.9 - 293) = 152659.5
u2 = 390.72 m/s
or    
pD2 N
      =
60
390.72 × 60
D2 =
or   = 0.933 or 933 mm
p × 8000

M14_THERMAL ENGINEERING_SE_XXXX_CH14.indd 773 6/15/2018 5:57:07 PM


774 Chapter 14

D 2 933
D1 = = = 466.5 mm.
2 2

(c) Va = pD1b1vf kt


580
or   = p × 0.4665 × b1 × 62 × 0.94
60
or                b = 0.113 m or 113 mm
vf  u2 
(d) tan b1 =   u1 = 2 
u1
62 × 2
= = 0.31736
390.72
b1 = 17.6°

Example 14.13
A centrifugal compressor runs at 16,000 rpm and 2300 kW of power is required to run the com-
pressor. The outer diameter of the impeller is 50 cm. The uniform width of the casing of vortex
chamber between impeller and diffuser is 4 cm. Static conditions at a radius of 27 cm were meas-
ured and it were equal to 2.4 bar and 390 K. Surrounding pressure is 1 bar. Assuming slip factor
of 0.94, find (a) the mass flow rate of gas, (b) the resultant velocity and (c) the temperature at the
given radius of 27 cm. Assume cp = 1.005 kJ/kgK and g = 1.4.

Solution
Given that N = 16,000 rpm, P = 2300 kW, D2 = 50 cm, B = 4 cm, At R = 27 cm, p = 2.4 bar, T = 390
K, p1 = 1 bar, fs = 0.94, cp = 1.005 kJ/kg.K, g = 1.4
pD2 N p × 0.5 × 16000
(a) u2 = = = 418.88 m/s
60 60
Work done, W = vw2u2 = fs u22

W fs u22 0.94 × ( 418.88)2


Power per kg of air = = = = 164.93 kW
103 103 103
P 2300
Mass flow rate of air, m a = = = 13.945 kg/s
164.93 164.93
(b) Using suffix 3 for the given section,
Area of flow, Af3 = 2p R3B3 = 2p  × 0.27 × 0.04 = 0.06786 m2

 = m a RT3 = 13.945 × 287 × 390 = 6.5 m3/s


Volume rate of flow, V3
p3 2.4 × 105
V 6.5
     vr3 = v f3 = 3 = = 95.8 m/s
Af 3 0.06786
For free vortex in vaneless space, vw1 × r1 = const.

M14_THERMAL ENGINEERING_SE_XXXX_CH14.indd 774 6/15/2018 5:57:15 PM


Centrifugal Air Compressors 775

i.e., vw1r1 = vw2r2 = vw3r3


vw 2 r2 fsu2 r2 0.94 × 418.88 × 0.25
vw3 = = = = 364.58 m/s
r3 r3 0.27

Absolute velocity, va3 = v 2f 3 + vw2 3 = (95.8)2 + (364.58) 2 = 376.96 m/s

va23 (376.96)2
T03 = T3 +
(i) = 390 +
2c p 2 × 1.005 × 103
= 460.98 K = 461 K or 188°C

Example 14.14
A centrifugal compressor has to deliver 6kg/s of air with pressure ratio of 4:1 at 17500 rpm. Initial
conditions are static air at 1 bar and 15°C. Assuming an adiabatic efficiency of 78%, ratio of whirl
speed to tip speed 0.94 and neglecting all other losses, calculate the tip speed, diameter, and rise in
total pressure. Determine the external diameter of eye for which the internal diameter is 12 cm and
the axial velocity at inlet is 150 m/sec. Assume cp = 1.005 kJ/kgK and g = 1.4.

Solution
p
Given that: m a = 6 kg/s, rp = 2 = 4, N = 17500 rpm, p01= 1 bar, T01 = 273 + 15 = 288 K, hisen = 0.78,
v p1
fs = w2 = 0.94, dh = 12 cm, for radial inlet, a1 = 90° and vf1 = va1 = 150 m/s, cp = 1.005 kJ/kg.K,
u2
g = 1.4
The T-s diagram is shown in Fig. 14.19(a) and the impeller is shown in Fig. 14.19(b)
va21
T01 = T1 +
2c p

150 2
T1 = 288 -
or    
2 × 1.005 × 103
= 288 - 11.2 = 276.8 K
g
p1 = p01  T01  g −1
 T 
01

3.5
 276.2 
=1× = 0.87 bar
 288 

p2 = 4 × 0.87 = 3.48 bar


g −1 0.4
T2 = T1  p2  g
= 276.8( 4) 1.4 = 411.3 K
 p 
1

M14_THERMAL ENGINEERING_SE_XXXX_CH14.indd 775 6/15/2018 5:57:20 PM


776 Chapter 14

Tip
p02
T
02′ p2 Eye

02 2′
p01
2 d2
p1 d1 dh

01
1
s
(a) (b)

Figure 14.19 Diagrams for centrifugal compressor:


(a) T-s diagram, (b) Impeller

Let va1 = va2, then


va22
T02 = T2 + = 411.3 + 11.2 = 422.5 K
2c p
g
p02  T02  g −1
=
p2  T2 
3.5
 422.5 
or        p02 = 3.48  = 3.82 bar
 411.3 
Rise in total pressure, ∆p0 = p02 - p01 = 3.82 - 1 = 2.82 bar
T02 − T01
Now,        hisen =
T02 ′ − T01
422.5 − 288
or     0.78 =
T02 ′ − 288
or        T02 ′ = 460.4 K
Work required per kg of air, w = cp(T02′ - T01)
= 1.005(460.4 - 288) = 173.262 kJ/kg

Also, work required per kg of air, w = u2vw2 = u22fs


= 0.94 u22

or u22 = 173262/0.94 = 184321


u2 = 429.3 m/s
or  

M14_THERMAL ENGINEERING_SE_XXXX_CH14.indd 776 6/15/2018 5:58:15 PM


Centrifugal Air Compressors 777

pd2 N pd2 × 17500


= =
60 60
d2 = 0.468 m or 46.8 cm
or  

p1 0.87 × 10 2
Density of air, r1 = = = 1.095 kg/m3
RT1 0.287 × 276.8
p 2
 m a = (d - dh2 ) × vf1 × r1
4 1
p
or  6 = [d12 - (0.12)2] × 150 × 1.095
4
or d1 = 0.247 m or 24.7 cm

Example 14.15
A single-sided centrifugal air compressor delivers 1800 kg of air per minute. The air enters the eye
of the impeller axially at total pressure of 100 kPa and total temperature of 290 K. The overall diam-
eter of the impeller is 700 mm and it rotates at 1600 rpm. The slip factor is 0.9 and the work input
factor is 1.1. The isentropic efficiency is 85%. Calculate (a) power required by the compressor,
(b) pressure coefficient, and (c) total pressure at delivery. [IES, 2003]

Solution
Given that m· a = 1800 kg/min, a1 = 90, p01 = 100 kPa, T01 = 290 K, D2 = 700 mm, N = 1600 rpm, fs
= 0.9, fw = 1.1, hisen = 85%
pD2 N p × 700 × 1600
u2 = = = 58.64 m/s
3
10 × 60 103 × 60
vw2
fs = , v = 58.64 × 0.9 = 52.78 m/s
u2 w2
0.9
(a) Compressor power = m· a fw u22 fs = (1800/60) × 1.1 × (58.64)2 × = 102.13 kW
103
(b) Pressure efficient, fp = fw fs hisen = 1.1 × 0.9 × 0.85 = 0.8415
g −1
p  h f f u2  g
(c) 02 = 1 + isen w s 2 
p01  c p T01 
1.4 −1
 0.85 × 1.1 × 0.9 × (58.64)2  1.4
= 1 +  = 1.00282
 1.005 × 103 × 290 
p02 = 100 × 1.00282 = 100.82 kPa
or      

M14_THERMAL ENGINEERING_SE_XXXX_CH14.indd 777 6/15/2018 5:58:20 PM


778 Chapter 14

Example 14.16
A centrifugal compressor running at 18,000 rev/min takes in air at 25° C and compresses it through
a pressure ratio of 4.0 with an isentropic efficiency of 80%. The guide vanes at inlet give the air an
angle of pre-whirl of 20° to the axial direction. The mean diameter of impeller eye is 225 mm. The
absolute air velocity at inlet is 130 m/s. At exit the blades are radially inclined. If the slip factor is
0.90, calculate the impeller tip diameter. [IAS, 1999]

Solution
Given that N = 18000 rpm, T1 = 25 + 273 = 298 K, rp = 4, hisen = 80%, pre-whirl at inlet = 20° or a1
= 90° - 20° = 70°, dm = 225 mm, va1 = 130 m/s, b2 = 90°, fs = 0.90
Temperature after isentropic compression,

g −1 1.4 −1
T2 = T1 rp ( )g = 298( 4) 1.4 = 442.83 K
Isentropic temperature rise,
( ∆T)isen = T2 - T1 = 442.83 - 298 = 144.83 K
( ∆T )isen 144.83
Actual temperature rise, (∆T)act = = = 181 K
hisen 0.8
Power input per unit mass flow rate = cp × (∆T)act = 1.005 × 181 = 181.9 kJ/kg
pdm N p × 225 × 18000
u1 = = = 212 m/s
3
10 × 60 103 × 60
The velocity diagrams are shown in Fig. 14.20
vw1 = va1 cos a1 = 130 cos 70° = 44.46 m/s
pd2 N pd2 × 18000
u2 = = = 0.9425 × d2 m/s
3
10 × 60 103 × 60
For radial discharge, vw2 = u2
v′
Slip factor, fs = w 2 = 0.90
vw 2
′ = 0.9 vw2 = 0.9 × 0.9425 d2 = 0.84825 × d2 m/s
vw2

v′r2
va1 v′a2 va2
vr2 vf2 = vr2
vf1
20°
a ′2

a1 a2 b ′2
b1 b2
vw1
v′w2
u1 u2
(a) (b)

Figure 14.20  Velocity diagrams: (a) Inlet, (b) Outlet

M14_THERMAL ENGINEERING_SE_XXXX_CH14.indd 778 6/15/2018 5:58:23 PM


Centrifugal Air Compressors 779

Power input per unit mass flow rate = u2 vw2


′ - u1 vw1
or 181.9 × 103 = 0.9425 × d2 × 0.84825 × d2 - 212 × 44.46
= 0.7995 d22 - 9425.52
d2 = 488 mm
or  

Example 14.17
A double-sided centrifugal compressor has root and tip diameters of 18 cm and 30 cm and is to
deliver 16 kg of air per second at 16,000 rpm. The design ambient conditions are 15°C and 1 bar
and the compressor has to be a part of a stationary power plant. Determine the following:
(a) Suitable values for impeller vane angles at the root and tip of the eye if the air is given 20° of
pre-whirl at all radii. The axial component of the velocity is constant throughout the impeller
and is 150 m/s.
(b) Power required if the power input factor is 1.05 and mechanical efficiency is 95%
(c) Maximum Mach number at the eye
Take for air: cp = 1.005 kJ/kg K and g  = 1.4. [IAS, 1998]

Solution
Given that r1 = 9 cm, r2 = 15cm, m· a = 16 kg/s, N = 16000 rpm, T1 = 273 + 15 = 288 K
p1 = 1 bar, a1 = a2 = 70°, vf1 = vf2 = 150 m/s, fs = 1.05, hmech = 95%, cp = 1.005 kJ/kg K, g  = 1.4
2p N 2p × 16000
(a) Angular speed of rotor, w  = = = 1675.5 rad/s
60 60
The velocity diagrams are shown in Fig. 14.21.
u1 = w r1 = 1675.5 × 0.09 = 150.8 m/s
u2 = w r2 = 1675.5 × 0.15 = 251.3 m/s
a1 = 90° - angle of pre-whirl = 70°
vf1
= tan a1
vw1
150
vw1 =
or       = 54.6 m/s
tan 70o
vf1 150
tan b1 = = = 1.5592
u1 − vw1 150.8 − 54.6
b1 = 57.3°
or  
vf 2 = vf1 = 150 m/s, vw2 = vw1 = 54.6 m/s
vf 2 150
tan b2 = =
u2 − vw 2 251.3 − 54.6
b2 = 37.3°
or  

M14_THERMAL ENGINEERING_SE_XXXX_CH14.indd 779 6/15/2018 5:58:26 PM


780 Chapter 14

Angle of
prewhirl vf2
20°
vr1 20° vr2
va1 20° va2
vf1

a1 b1 a2 b2

vw1 vw2
u1 u2

Figure 14.21 Velocity diagrams

Work done per kg of air, w = vw2 u2 - vw1 u1 = vw1 (u1 - u1)


= 54.6 (251.3 - 150.8) = 5487.3
m a × fw × w 16 × 1.05 × 5487.3
(b) Power required = = = 97 kW
h mech 0.95 × 103
vf1
(c) = sin a1,
va1
150
va1 = = 159.6 m/s
sin 70o

Sonic velocity = g RT1 = 1.4 × 0.287 × 103 × 288 = 340.2 m/s


159.6
Mach number at the eye = = 0.469
340.2

Summary for Quick Revision


1. A centrifugal compressor is of rotodynamic type in which the pressure rise is primarily due to the centrifugal
action.
2. Stagnation or total head values:
v2
(a) Stagnation temperature, T0 = T +
2cp

v2
(b) Stagnation enthalpy, h0 = h +
2
g
(
(c) Stagnation pressure, p0/p = T0 / T g −1 )
T02 − T01
3. Isentropic efficiency, hisen =
′ − T01
T02

M14_THERMAL ENGINEERING_SE_XXXX_CH14.indd 780 6/15/2018 5:58:30 PM


Centrifugal Air Compressors 781

4. Ideal velocity diagrams:


(a)  a1 = 90°, b2 = 90°, va1 = vf1 and vr2 = vf 2, i.e. vw1 = 0 and vw2 = u2
(b)  Torque, T = u2 r2
2
(c)  Work done for 1 kg/s of air flow, w = u2
(d)  From SFEE, w = cp (T02 - T01 ) = h02 - h01

    = cp (T2 - T1) = h2 - h1 for va1 = va2


 g −1 
p
(e) w = cp T01 ( rop ) g − 1 , rop = 02 = stagnation pressure ratio.

  p01
 

 g −1 
  = c T ( r ) g − 1 for v = v , r = p2
  p 1  p  a1 a2 p
p1
 
(f)  Stagnation pressure ratio:
g −1
p  u2  g
(g)  02 =  2 + 1
p01  c p T01 
g −1
p  u2  g
(h)  2 =  2 + 1 for va1 = va2
p1  c p T1 
u2
(i)  Total temperature increase across an impeller, ∆T0 = 2 = const.
cp
2
m ua2
(j)  Indicated power, IP = kW
103
5. Actual velocity diagrams:
v 2 − va21 vr21 − vr22 u22 − u12
(a)  Work done on 1 kg/s of air, w = a 2 + +
2 2 2
1 2 2
(b)  ∆KE = ( va 2 − va1 )
2
1
∆p due to diffusion action = ( vr21 − vr22 )
2
1
∆p due to centrifugal action = (u22 − u12 )
2
6. Width of blades:
(pD − nt )bv f pDkb bv f
m· a = =
vs vs
where kb = blades factor, b = width of height of blades, t = thickness of blades,
vs = specific volume
vw2
7. Slip factor, fs =
u2
Slip = u2 - vw2
Isentropic work
8. Pressure coefficient, fp =
Euler work

M14_THERMAL ENGINEERING_SE_XXXX_CH14.indd 781 6/15/2018 5:58:36 PM


782 Chapter 14

c p (T02 − T01 )
      =
u2 vw 2
c p hisen (T02
′ − T01 )
= when vw2 = u2 for radial vanes.
u22
′ − T01 )
c p (T02 Actual work
9. Work factor, fw = =
u2 vw 2 Euler work

c p (T02 − T01 )
=
u2 vw 2 hisen
10. Relationship between fp, fw, fs and hisen:
fp = fs fw hisen
11. Shape of impeller blades:
(a)  There are three types of impeller blades:
12. Backward curved blades, b2 < 90°.
(a)  Radial blades, b2 = 90°.
(b)  Forward curved blades, b2 > 90°.
Slightly backward curved impeller blades give optimum efficiency. Its consequences are:
(a) Degree of reaction increases which results in increasing part of kinetic energy transferred into pressure
energy within the impeller.
(b)  The length of impeller channel increases which increases the friction loss.
(c)  Radial blades are used for aircraft centrifugal compressor.
13. The diffuser converts the kinetic energy imparted to air by the impeller into pressure rise.

p4 − p3
Diffuser efficiency, hd =
1
p ( v a23 − va24 )
2
14. Pre-whirl is given to intake guide vanes in order to avoid acceleration to sonic or supersonic velocities to
keep the Mach number below 0.9 of the air flow entering the impeller eye. This increases the work done on
the air vw1 ≠ 0.
15. Performance characteristics:
p3 G T1 ∆N
p1 = ,p = p =
p1 2 p1D 2 3 T1
p1 = f (p2, p3)

For D = const
p3  G T1 N 
=f , 
p1  p1 T1 

where G = mass rate of flow, h = rotor rpm, p1 = inlet pressure,


T1 = inlet temperature, p3 = pressure at diffuser outlet.
16. The pulsating air flow phenomena is called surging.
The maximum mass flow rate of air is called choking of the compressor.

M14_THERMAL ENGINEERING_SE_XXXX_CH14.indd 782 6/15/2018 5:58:40 PM


Centrifugal Air Compressors 783

Multiple-choice Questions
1. The inlet and exit velocity diagrams of a turboma- 5. If two geometrically similar impellers of a centrif-
chine rotor are shown in Fig. 14.22. ugal compressor are operated at the same speed,
This turbomachine is their head, discharge, and power will vary with
(a) An axial compressor with backward curved their diameter ratio d as
blades (a) d, d2, and d3, respectively
(b) A radial compressor with backward curved (b) d2, d3, and d5, respectively
blades (c) d, d3, and d5, respectively
(c) A radial compressor with forward curved (d) d2, d, and d3, respectively
blades 6. Which one of the following velocity triangles
(d) An axial compressor with forward curved (Fig. 14.23) represents the one at the exit of a
blades radial impeller with forward curved blades?
(u2 = peripheral velocity, u2 = absolute velocity,
v2 w2 = relative velocity)
v2 vr
1 vr v2 v2
2 w2 v2 w2
(a) (b) (c)

u2 u2 u2 u2 u2

Figure
v2 14.22 Velocity triangles v2 v2
w 2 v2 w2 w2 w2
(a)
2. It is recommended that the(b)diffuser angle should (c) (d)
be kept less thanu218° because u2 u2 u2
(a) Pressure decreases in flow direction and flow
separation may occur Figure 14.23 Velocity triangles
(b) Pressure decreases in flow direction and flow 7. The stagnation pressure rise in a centrifugal com-
may become turbulent press or stage takes place
(c) Pressure increases in flow direction and flow (a) Only in the diffuser
separation may occur (b) In the diffuser and impeller
(d) Pressure increases in flow direction and flow (c) Only in the impeller
may become turbulent (d) Only in the inlet guide vanes
3. When the outlet angle from the rotor of a centrif- 8. For 15 m3/s air flow at 10 mm Hg head, which one
ugal compressor is more than 90, then the blades of the following would be the best choice?
are said to be (a) Centrifugal fan with forward curved blades
(a) Forward curved (b) Axial fan with a large number of blades in rotor
(b) Backward curved (c) Axial propeller fan with a few blades in rotor
(c) Radial (d) Cross-flow fan
(d) Either backward or forward curved
9. What is the ratio of the isentropic work to Euler’s
4. The degree of reaction of a turbomachine is defined work known as?
as the ratio of the (a) Pressure coefficient
(a) Static pressure change in the rotor to that in (b) Slip factor
the stator (c) Work factor
(b) Static pressure change in the rotor to that in (d) Degree of fraction
the stage
(c) Static pressure change in the stator to that in 10. Which of the following is the effect of blade shape
the rotor on performance of centrifugal compressor?
(d) Total pressure change in the rotor to that in (a) Backward curved blade has poor efficiency
the stage (b) Forward curved blades have higher efficiency

M14_THERMAL ENGINEERING_SE_XXXX_CH14.indd 783 6/15/2018 5:58:41 PM


784 Chapter 14

(c) Backward curved blades lead to stable perfor- (c) Rotor inlet velocity is supersonic and exit
mance velocity supersonic; stator inlet is supersonic
(d) Forward curved blades produce lower pres- and exit velocity is subsonic
sure ratio (d) Rotor inlet velocity is supersonic and exit
11. Surging basically implies velocity supersonic; stator inlet velocity is
(a) Unsteady, periodic, and reversed flow subsonic and exit velocity is subsonic
(b) Forward motion of air at a speed above sonic 16. The curve in Fig. 14.24 shows the variation of
velocity theoretical pressure ratio with mass of flow rate
(c) The surging action due to the blast of air pro- for a compressor running at a constant speed. The
duced in a compressor permissible operating range of the compressor is
(d) Forward movement of aircraft represented by the part of the curve from
12. Centrifugal compressors are suitable for large (a) A to B
discharge and wider mass flow range, but at a rel- (b) B to C
atively low discharge pressure of the order of 10 (c) B to D
bars, because of (d) D to E
(a) Low pressure ratio
(b) Limitation of size of receiver

Pressure ratio
(c) Large speed A B C
(d) High compression index D

13. Given: vw2 = velocity of whirl at outlet E


u2 = peripheral velocity of the blade tips.
Mass flow
The degree of reaction in a centrifugal compres- Figure 14.24
sor is equal to
17. In a centrifugal compressor, the highest Mach
vw 2 u2 number leading to shockwave in the fluid flow
(a) 1 − (b) 1 −
2u2 2 vw 2 occurs at
(a) Diffuser inlet radius
2 vw 2 vw 2 (b) Diffuser outlet radius
(c) 1 − (d) 1 −
u2 u2 (c) Impeller inlet radius
(d) Impeller outer radius
14. In a centrifugal compressor assuming the same
overall dimensions, blade inlet angle, and rota- 18. In the centrifugal air compressor design practice,
tional speeds, which of the following blades will the value of polytropic exponent of compression
give the maximum pressure rise? is generally taken as
(a) Forward curved blades (a) 1.2 (b) 1.3
(b) Backward curved blades (c) 1.4 (d) 1.5
(c) Radial blades 19. For centrifugal compressors, which one of the fol-
(d) All three types of bladings have the same lowing is the correct relationship between pres-
pressure rise sure coefficient (fp), slip factor (fs), work input
15. Under which one of the following sets of con- factor (fn) and isentropic efficiency (ha)?
ditions will a surpersonic compressor have the fs × fw fw
(a) fp = (b) fp =
highest efficiency? ha fs × h a
(a) Rotor inlet velocity is supersonic and exit fs × h a
velocity subsonic; stator inlet velocity is sub- (c) fp = fs × fw × ha (d)
fp =
sonic and exit velocity is subsonic fw
(b) Rotor inlet velocity is supersonic and exit 20. At the eye tip of a centrifugal impeller, blade
velocity subsonic; stator inlet velocity is velocity is 200 m/s while the uniform axial veloc-
supersonic and exit velocity is subsonic ity at the inlet is 150m/s. If the sonic velocity is

M14_THERMAL ENGINEERING_SE_XXXX_CH14.indd 784 6/15/2018 5:58:44 PM


Centrifugal Air Compressors 785

300 m/s, the inlet Mach number of the flow will be (c)  4   1 3
(a) 0.50 (b) 0.66 (d)  2   3 4
(c) 0.83 (d) 0.87 26. The flow in the vaneless space between the impel-
21. What will be the shape of the velocity triangle at ler exit and diffuser inlet of a centrifugal compres-
the exit of a radial bladed centrifugal impeller, sor can be assumed as
taking into account slip? (a) Free vortex
(a) Right-angled (b) Forced vortex
(b) Isosceles (c) Solid body rotation
(c) All angles less than 90° (d) Logarithmic spiral
(d) One angle greater than 90° 27. Which portion of the centrifugal compressor
22. What does application of centrifugal air compres- characteristics shown in Fig. 14.25 is difficult to
sors lead to? obtain experimentally?
(a) Large frontal area of aircraft S
(b) Higher flow rate through the engine
(c) Higher aircraft speed T

pr. ratio
(d) Lower frontal area of the aircraft
R U
23. Consider the following statements:
In centrifugal compressors, there is a tendency of
increasing surge when V
i. the number of diffuser vanes is less than the mass flow
number of impeller vanes
ii. the number of diffuser vanes is greater than
Figure 14.25
the number of impeller vanes (a) RS (b) ST
iii. the number of diffuser vanes is equal to the (c) TU (d) UV
number of impeller vanes 28. The pressure rise in the impeller of centrifugal
iv. mass flow is greatly in excess of that corre- compressor is achieved by
sponding to the design mass flow (a) The decrease in volume and diffusion action
Which of these statements is/are correct? (b) The centrifugal action and decrease in volume
24. In centrifugal compressor terminology, vaneless (c) The centrifugal and diffusion action
space refers to the space between (d) The centrifugal and push-pull action
(a) The inlet and blade inlet edge 29. In a radial blade centrifugal compressor, the
(b) Blades in the impeller velocity of blade tip is 400 m/s and slip factor is
(c) Diffuser exit and volute casing 0.9. Assuming the absolute velocity at inlet to be
(d) Impeller tip and diffuser inlet edge axial, what is the work done per kg of flow?
25. Match List-I with List-II (pertaining to blower (a) 36 kJ (b) 72 kJ
performance) and select the correct answer using (c) 144 kJ (d) 360 kJ
the codes given below the list: 30. The power required to drive a turbo-compressor
List I List-II for a given pressure ratio decreases when
A. Slip 1.  Reduction of whirl velocity (a) Air is heated at entry
(b) Air is cooled at entry
B. Stall 2. Fixed mass flow rate regardless of
(c) Air is cooled at exit
pressure ratio
(d) Air is heated at exit
C. Choking 3.  Flow separation
31. In a centrifugal compressor, how can the pressure
4.  Flow area separation ratio be increased?
Codes: (a) Only by increasing the tip speed
A  B C (b) Only by decreasing the inlet temperature
  (a)   4    3 2 (c) By both (a) and (b)
(b)   1   3 2 (d) Only by increasing the inlet temperature

M14_THERMAL ENGINEERING_SE_XXXX_CH14.indd 785 6/15/2018 5:58:45 PM


786 Chapter 14

Explanatory notes

Axial velocity inlet 150


20. (a) Mach number, M = = = 0.5
Sonic velocity 300

u
31. (c) fp = w 2 , vw2 = 0.9 × 400 = 360 m/s
u2
400 × 360
Work done = u2uw2 = = 144 kJ
103

Review Questions
1. What are the constructional features of a centrifugal compressor?
2. Explain the principle of working of a centrifugal compressor.
3. How does the velocity and pressure of air vary through the impeller and the diffuser?
4. What are the various types of impellers?
5. Define total head values.
6. Define isentropic efficiency.
7. Define stagnation pressure ratio.
8. Define slip factor.
9. What is work factor and pressure coefficient?
10. What is the relationship between fp, fw, fs, and fisen?
11. List the losses which generally occur in a centrifugal compressor.
12. What are the effects of impeller blade shapes on performance?
13. What is the role of a diffuser?
14. Explain pre-whirl and its role.
15. Explain the phenomena of surging and choking.

Exercises
14.1 A single eye, single-stage centrifugal compressor delivers 20 kg/s of air with a pressure ratio of 4 when
running at 15,000 rpm. The pressure and temperature of air at the inlet are 1 bar and 17°C. Assume the
following:
Slip factor = 0.9, work input factor = 1.04, isentropic efficiency = 75%.
Calculate (a) the IP required to drive the compressor and (b) the blade angles at the impeller eye if the root
and tip diameters are 20 cm and 40 cm, respectively.
14.2 A centrifugal compressor delivers 600 m3/min of free air when running at 900 rpm. The following data is
given:
Inlet pressure and temperature of air = 1 bar and 20°C
Compression ratio = 3.5
Isentropic efficiency = 83%
Flow velocity throughout the impeller = 60 m/s
Tip diameter = 2 × eye diameter
Blade area coefficient = 0.94
The blades are radial at outlet of impeller.
Calculate (a) the IP required to run the compressor, (b) impeller diameters at inlet and outlet, (c) breadth of
impeller at inlet, and (d) impeller blade angle at inlet.

M14_THERMAL ENGINEERING_SE_XXXX_CH14.indd 786 6/15/2018 5:58:46 PM


Centrifugal Air Compressors 787

 14.3  The first stage of a centrifugal compressor is single-inlet with an eye 60 cm, hub 20 cm and an overall
diameter of 100 cm. When supplied with 20°C air at 1 bar and running at 6000 rpm, it compresses 30 kg/s
of air through a compression ratio of 2.0.
    Calculate (a) the isentropic efficiency of rotor based on impeller tip velocity with slip factor as 0.9, and (b)
the rotor power and shaft power. Take hmech = 0.97.
 14.4  A single-sided centrifugal compressor is to deliver 14 kg/s of air when operating at a pressure ratio of 4:1
and a speed of 12000 rpm. The total head inlet conditions may be taken as 288 K and 1 bar. Assuming
a slip factor of 0.9, a power input factor of 1.04 and an isentropic efficiency (based on the total head) of
80%, estimate the overall diameter of the impeller. If the Mach number is not to exceed unity at the impel-
ler tip and 50% of the losses are assumed to occur in the impeller, find the minimum possible depth of
the diffuser.
 14.5  The following data refer to a single-sided centrifugal compressor:
    Air mass flow rate = 9 kg/s, Eye tip diameter = 0.3 m, Slip factor = 0.9, Isentropic efficiency = 80%.
    Eye root diameter = 0.15 m, overall diameter of impeller = 0.5 m, Power input factor = 1.04, Rotational
speed = 18000 rpm, Inlet stagnation pressure and temperature = 1.1 bar, 295 K.
    Calculate (a) pressure ratio of the compressor, (b) inlet angle of the impeller vane of the root and tip radii
of the eye, and (c) the axial depth of the impeller channel at the periphery of the impeller.
 14.6  An air-craft fitted with a single-sided centrifugal compressor flies with a speed of 850 km/h at an altitude
where the pressure is 0.23 bar and the temperature is 217 K. The inlet duct of the impeller eye contains
fixed vanes which give the air pre-whirl of 25° at all radii. These inner and outer diameters of the eye are
180 mm and 330 mm, respectively, the diameter of the impeller tip is 540 mm, and the rotational speed is
16,000 rpm. Estimate the stagnation pressure at the compressor outlet when the mass flow is 216 kg/min.
    Neglect losses in inlet duct and fixed vanes, and assume that the isentropic efficiency of the compressor is
0.80. Take the slip factor as 0.9 and the power input factor as 1.04.
 14.7  A centrifugal compressor compresses 30 kg/s of air at a rotational speed of 1500 rpm. The air enters the
compressor axially, and the conditions at the exit section are radius = 0.3 m, relative velocity of air at the tip
= 100 m/s at an angle of 80°. Find the torque and power required to drive the compressor and work done.
 14.8  Air at a temperature of 300 K flows in a centrifugal compressor running at 18,000 rpm. Slip factor = 0.8;
isentropic total head efficiency = 0.75; outer diameter of blade tip = 0.5 m. Determine (a) the temperature
rise of air and (b) static pressure ratio.
    Assume that the velocities of air at inlet and exit of the compressor are the same.
 14.9  A single inlet type centrifugal compressor delivers 8 kg/s of air. The ambient air conditions are 1 bar and
20°C. The compressor runs at 22,000 rpm with isentropic efficiency of 82%. The air is compressed in the
compressor from 1 bar static pressure to 4.2 bar total head pressure. The air enters the impeller eye with a
velocity of 150 m/s with no pre-whirl. Assuming that the ratio of whirl speed to tip speed is 0.9, calculate
(a) the rise in total head temperature during compression if the change in kinetic energy is negligible,
(b) the tip diameter of the impeller, (c) power required, and (d) the eye diameter if the hub diameter is 10 cm.
14.10  A centrifugal compressor is to have a pressure ratio of 3.5:1. The inlet eye of the impeller is 30 cm in diam-
eter. The axial velocity at inlet is 130 m/s and the mass flow rate is 10 kg/s. The velocity in the delivery
duct is 115 m/s. The tip speed of the impeller is 450 m/s and runs at 16,000 rpm with total head isentropic
efficiency of 78% and pressure coefficient of 0.72. The ambient conditions are 1 bar and 15°C. Calculate:
(a) the static pressure ratio, (b) the static pressure and temperature at inlet and outlet of compressor,
(c) work of compressor per kg of air, and (d) the theoretical power required.
14.11  The following details refer to a centrifugal air compressor of a total head pressure ratio of 3 and isentropic
efficiency of 70%. Find the power required to run the compressor.
  Suction conditions:
  Pressure = 1 bar,

M14_THERMAL ENGINEERING_SE_XXXX_CH14.indd 787 6/15/2018 5:58:46 PM


788 Chapter 14

  Temperature = 20°C
  Velocity of air at inlet = 60 m/s
  Pressure and temperature are static values.
  Free air delivered = 20 kg/min
  Mechanical efficiency = 95%
  Ratio of specific heats for air = 1.4
  Specific gas constant for air = 0.287 kJ/kg.K
14.12 A centrifugal compressor has a pressure ratio of 4:1 with an isentropic efficiency of 82% when running at
16000 rpm. It takes in air at 17°C. Guide vanes at inlet give the air a pre-whirl of 20° to the axial direction
at all radii and the mean diameter of the eye is 200 mm; the absolute air velocity at inlet is 120 m/s. At
exit, the blades are radially inclined and the impeller tip diameter is 550 mm. Calculate the slip factor of
the compressor.

ANSWERS TO MULTIPLE-CHOICE QUESTIONS


1. (a) 2. (c) 3. (a) 4. (b) 5. (d)
6. (b) 7. (b) 8. (a) 9. (a) 10. (c)
11. (a) 12. (a) 13. (a) 14. (a) 15. (c)
16. (c) 17. (b) 18. (c) 19. (c) 20. (a)
21. (c) 22. (a) 23. (d) 24. (d) 25. (b)
26. (b) 27. (a) 28. (b) 29. (c) 30. (b)
31. (c)

M14_THERMAL ENGINEERING_SE_XXXX_CH14.indd 788 6/15/2018 5:58:46 PM


Chapter 15

Axial Flow Air Compressors

15.1 ❐ INTRODUCTION
In an axial flow compressor, the air flows throughout the compressor parallel to its axis. Axial flow
compressor has many merits over centrifugal compressor and is particularly suitable for super charg-
ing of internal combustion engines, turbojets, and gas turbines.

15.2 ❐ CONSTRUCTIONAL FEATURES


The cross-section of a typical axial flow compressor is shown in Fig. 15.1. It consists of adjacent rows
of moving and fixed blades. The moving blades are mounted on the rotating drum and the fixed blades
are fixed to the casing or stator. One stage of the compressor comprises a row of moving blades, fol-
lowed by a row of fixed blades.
The blades belong to the aerofoil section designed on the basis of aerodynamic theory. Such a
design prevents losses due to shock and turbulence and is free from stalling problems. The blades are
said to be stalled when the air stream fails to follow the blade profile.

Inlet guide Stator


vane casing Delivery vane

Air M F M F M F M Air delivery


inlet
Drive shaft

Air
inlet Air delivery

Moving Fixed Drum Rotor disc


blade blade

Figure 15.1  Typical axial flow compressor

M15_THERMAL ENGINEERING_SE_XXXX_CH15.indd 789 6/16/2018 12:37:20 PM


790 Chapter 15

The blades of a reaction gas turbine have profile formed by a number of circular arcs. This occurs
because the acceleration process carried out in the converging blade passages of a reaction turbine
is much more efficient and stable as against the diffusing or decelerating process carried out in the
diverging passage between the blades of an axial flow compressor.
To keep the flow velocity constant throughout the compressor length, the annular area is reduced
from inlet to outlet of the compressor. In the diverging passage of the moving blades, there is a rise in
temperature due to diffusion and the absolute velocity is also increased due to work input.
The fixed blades serve to convert a part of the kinetic energy of air into pressure energy by diffusion
and also guide and redirect the air stream to enter the next stage without shock.

15.3 ❐ WORKING PRINCIPLE


The axial compressor is generally driven by an internal combustion engine or a turbine. The work
input to the rotor shaft is transferred by the rotor moving blades to the air, thus accelerating it. The
space between the moving blades form diffuser passages so that the velocity of air relative to the
blades is decreased as the air passes through them and consequently, the pressure rises. The air is fur-
ther diffused in the fixed blades which also form diffuser passages. In the fixed blades, the air is turned
through an angle so that its direction is such that it can pass to the next ring of moving blades without
shock. Generally, 5 to 14 stages are used.

15.4 ❐ SIMPLE THEORY OF AEROFOIL BLADING


When an object is placed within a fluid stream, a force is exerted on the object which may be inclined
to the flow direction. The component of this force parallel to the direction of flow is called the drag
and the normal component is called the lift, as shown in Fig. 15.2.
Consider an aerofoil section as shown in Fig. 15.3, which has an angle of attack a, the chord length
C, and the length of the foil perpendicular to the plane of the paper, termed as the span, l. The resultant
force F executed by the fluid on the aerofoil has two mutually perpendicular components: the lift FL
and the drag FD. Consider a surface element of elementary area dA of aerofoil on which pressure p
acts. Let the tangent to dA make an angle q with the flow direction. The differential lift and drag on
this area element are as follows:
dFL = pdA cos q
dFD = pdA sin q

p F2
Body placed F
F2 F
U∞ in the free U∞ dA
stream of flow
F0
F0 Aerofoil
C
a

Figure 15.2  Definition sketch of lift and drag

M15_THERMAL ENGINEERING_SE_XXXX_CH15.indd 790 6/16/2018 12:37:21 PM


Axial Flow Air Compressors 791

−ve Pressure on
upper surface of
aerofoil
p<0

p=0

p>0

+ve Pressure on
lower surface of
aerofoil

Figure 15.3  Pressure distribution on an aerofoil section

Lift, FL = ∫ A pdA cosq


Drag, FD = ∫ A pdA sinq
where ∫ A represents the integration over the entire body surface area A.
We define the following lift and drag coefficients:
FL
Lift coefficient, CL =  (15.1)
1
rU ∞2 A
2
FD
Drag coefficient, CD =  (15.2)
1
rU ∞2 A
2
where r = density of air stream
U∞ = free stream velocity of air
A = projected area of the body perpendicular to the oncoming flow, i.e., the frontal area of the body.
Let
V = free air delivered by the compressor, m3/min
i = number of moving blades per blade ring
p1, T1 = ambient air conditions
Dm = blade ring mean diameter, m
kb = blade factor
h = blade height, m
N = rotor speed, rpm
CL, CD = lift and drag coefficients respectively
A = projected area of blades, m2
p
Then density of air, r= 1
RT1

V =v A a
A = kb p Dmh

M15_THERMAL ENGINEERING_SE_XXXX_CH15.indd 791 6/16/2018 12:37:27 PM


792 Chapter 15

b1
vr 1
va1 = vf1

Figure 15.4  Velocity triangle


V V
va = = m/s
60 A 60 × kb × p Dm h
pDm N
u= m/s
60
From the velocity triangle as shown in Fig. 15.4, we get
u
tan b1 =
va

vr1 = u 2 + va2
1
Lift force,        FL = C rv 2 A(15.3)
2 L r
1
Drag force,       FD = C D rvr2 A(15.4)
2
Power input/stage,
1
P = (FL cos b1 + FD sin b1) u × 10-3 kW (15.5)
2
V r
Mass flow rate,          m = kg/s (15.6)
60
g −1
 pT1  g
mc 
Also,     P =  rp − 1 (15.7)
hc  
 
p2
where rp = and hc = efficiency of the compressor
p1

15.5 ❐ VELOCITY DIAGRAMS


Figure 15.5 shows the velocity triangles for one stage of an axial flow compressor. All angles are
measured from the axial direction. The blade velocity u is taken to be same at blade entry and exit as
the air enters and leaves the blades at almost equal radii. Due to the diffusion process in the moving
blades, vr2 < vr1 but va2 > va1. The air leaves the fixed blades with velocity va3 at an angle a3 and is
redirected to the next page. It is assumed that va1 = va3.
From the inlet velocity triangle, we have
v u − vw1
tan a 1 = w1 , tan b1 =
vf1 vf1

M15_THERMAL ENGINEERING_SE_XXXX_CH15.indd 792 6/16/2018 12:37:33 PM


Axial Flow Air Compressors 793

u
vw1

vf1
va1 a1 vr 1
b1

Moving Rotor
u blades
vw2

vf 2
va2 b
a2 2 vr 2
Fixed Stator
blades

va1 = va3
a3

Figure 15.5  Velocity diagrams for axial flow compressor

u
∴ = tan a 1 + tan b1(15.8)
vf 2

From the outlet velocity triangle, we have


vw 2 u − vw 2
tan a 2 = , tan b2 =
vf 2 vf 2
u
∴ = tan a 2 + tan b2(15.9)
vf 2
Assuming 1 kg flow of air through the compressor stage, we have
Tangential force per kg of air = vw2 - vw1
Work required by the stage per kg of air,
    w = u(vw2 - vw1) = cp (T02′ - T01)(15.10)
Now vf1 = vf2 = vf
∴    
w = uvf (tan a2 - tan b1)(15.11)
uvf (tan b1 - tan b2)(15.12)
        u [u - vf (tan a1 + tan b2)](15.13)
Also, isentropic efficiency of the stage,
hisen = T02 − T01
T02′ − T01
T02 = T01 + hisen (T02′ - T01)
T (T − T01 )
02 = 1 + hisen 02′ (15.14)
T01 T01

M15_THERMAL ENGINEERING_SE_XXXX_CH15.indd 793 6/16/2018 12:37:36 PM


794 Chapter 15

g g
and p02 =  T02  g −1 = 1 + h (T02′ − T01 )  g −1
   isen 
p01  T01   T01 
g
 u( vw 2 − vw1 )  g −1
= 1 + hs   (15.15)
 c pT01 

15.6 ❐ DEGREE OF REACTION


The degree of reaction, Rd, is defined as the ratio of pressure rise in the rotor blades to the pressure
rise in the compressor stage.
1
Pressure rise in the rotor blades = ( vr21 − vr22 )
2
But   vr21 = v 2f + (vf tan b1)2

and vr22 = v 2f + (vf tan b2)2

1 2 v 2f
( vr1 − vr22 ) = (tan2 b1 - tan2 b2)
2 2
Pressure rise in the compressor stage
= uvf (tan b1 - tan b2)
v 2f
(tan 2 b1 − tan 2 b 2 )
Rd = 2
uv f (tan b1 − tan b 2 )

vf
    = (tan b1 + tan b 2 ) (15.16)
2u
1
For 50% degree of reaction,   Rd =
2
u
∴ = tan b1 + tan b 2 (15.17)
vf
From Eqs (15.8) and (15.9), we have
u
= tan a1 + tan b1 = tan a2 + tan b2
vf
∴   a 1 = b2 and a 2 = b1(15.18)
Therefore, with 50% degree of reaction, the blades are symmetrical. This reduces the friction
losses considerably.

15.7 ❐ PRESSURE RISE IN ISENTROPIC FLOW


THROUGH A CASCADE
Consider the incompressible isentropic and steady flow through a cascade from condition 1 to condi-
tion 2. Using Bernoulli’s equation, we have

M15_THERMAL ENGINEERING_SE_XXXX_CH15.indd 794 6/16/2018 12:37:40 PM


Axial Flow Air Compressors 795

p1 v12 p2 v22
+ = +
r 2 r 2
r
(∆p)isen = (p2 -p1)isen = ( v 2f 1 + vw2 1 ) − ( v 2f 2 + vw2 2 ) 
2 
Now,    v f1 = v f2 = v f
r r
∴    (∆p)isen = ( vw2 1 − vw2 2 ) = v 2f (tan2 a1 - tan2 a2)(15.19)
2 2

15.8 ❐ POLYTROPIC EFFICIENCY


It is the isentropic efficiency of one stage of a multistage compressor. This small-stage efficiency is
constant for all stages of a compressor with infinite number of stages.
Consider the compression process of a multistage compressor on a T-s diagram as shown in
Fig. 15.6 from total pressure p01 to p02 in four stages of equal pressure ratio with intermediate pres-
sure p0a, p0b, and p0c.
Overall isentropic stagnation efficiency of machine is,
( ∆T0 )m / c
hisen (m/c) =
( ∆T0′ )m / c
Stagnation isentropic efficiency for the stage,
( dT0 )st
hisen (st) =
( dT0′ )st
Total actual temperature rise,
( ∆T0 )m / c
  ∆T0′ =
hisen ( m / c )
∑( dT0 )st
Also,   
 ∆T0′=
hisen (st )
( ∆T0 )m / c ∑( dT0 )st
Thus =
hisen ( m / c ) hisen (st )

hisen ( m / c ) ( ∆T0 )m / c
or   =
hisen (st ) ∑( dT0 )st
p02
T (dT0′) 1st stage p0c
2
2′ p0b
2 c
a″ p0a
c′ (dT0′) stage
∆T0′ b″ p01
(∆T0′)m/c b′
b
a a′
1
s

Figure 15.6  Concept of polytropic efficiency

M15_THERMAL ENGINEERING_SE_XXXX_CH15.indd 795 6/16/2018 12:37:45 PM


796 Chapter 15

Now (∆T0) m/c = (1 - a) + (a - b) + (b - c) + (c - 2)


Σ (dT0)st = (1 - a) + (a′ - b′′) + (b′ - c′′) + (c′ - 2′′)
On the T-s plot, the constant pressure lines diverge towards the right, therefore,
(a ′ - b′′) > (a - b)
(b′ - c′′) > (b - c)
and so on.
Therefore, we can say that
Σ (dT0)st > (∆T0) m/c
∴ hisen (st) > hisen (m/c)
The small stage efficiency hisen (st) which is constant for all stages is called polytropic efficiency and
is denoted by hp.
Let the law of compression for the irreversible adiabatic path 1 - 2′ be,
p0 v0n = c1 (15.20)
and for the isentropic path 1-2 is,
p0 vg0 = c2
Eq. (15.20) can be written as:
p0 = c1 r 0n
Differentiating, we get
p0
dr0 = c1n r 0n −1 dr0 = n d r (15.21)
r0 0
Now      p0 = r0 RT0′
p
or   r0 = 0

RT0′
Differentiating, we get
1  T dp − p0 dT0′ 
dr0 =  0′ 0  (15.22)
R  T0′ 2 

p0 1  T0′ dp0 − p0 dT0′ 


Eq. (15.21) becomes dp0 = n  
r 0 R  T0′ 2 

1  T0′ dp0 − p0 dT0′ 


= n RT0′  
R  T02′ 
 T dp − p dT  dT0′
= n  0′ 0 0 0′
 = ndp0 - np0
2 T0′
 T0′ 
dp0 (n - 1) = np0 dT0′
or           
T0′
 n − 1 T0′
                dT0′ = dp0   (15.23)
 n  p0

M15_THERMAL ENGINEERING_SE_XXXX_CH15.indd 796 6/16/2018 12:37:52 PM


Axial Flow Air Compressors 797

Similarly, for the ideal compression process p0 vg0 = c2, we have


 g − 1 T0′
dT0 = dp0 
 g  p0
dT0  g −1  n 
Thus       hp = =   (15.24)
dT0′  g   n − 1
n dp T
From Eq. (15.23), we have     = 0 ⋅ 0′
n − 1 T0 dT0′
g − 1 dp0 T0′
hp = × ×
g p0 dT0′
dT0′ g − 1 dp0
hp = ⋅
T0′ g p0
Integrating between the end states 1 and 2′ , we get
 T   g − 1  p ′   g − 1  p 

hp ln  02  =  ln  02  = ln 02
 T ′   g   p ′   g   p01 
01 01
g −1
 g − 1  p02  p  g
 g  ln  p  ln  02 
01  p01 
    hp = = (15.25)
T ′  T ′ 
ln  02  ln  02 
 T   T 
01′ 01′

15.9 ❐ FLOW COEFFICIENT, HEAD OR WORK


COEFFICIENT, DEFLECTION COEFFICIENT,
AND PRESSURE CO-EFFICIENT
Flow Coefficient: It is defined as the ratio of flow (or axial) velocity at inlet to the blade velocity.
Mathematically, we can write
vf
Flow coefficient,   ff = 1
u
Now       u = v f (tan a1 + tan b1)
1
1
∴         f f = (15.26a)
tan a 1+ tan b 1
Also,        v f = v f = vf
1 2

v f2 1
f f = =  (15.26b)
u tan a 2 + tan b 2
Head or Work Coefficient: It is defined as the ratio of actual work done to the kinetic energy corre-
sponding to the mean peripheral velocity. One can write

M15_THERMAL ENGINEERING_SE_XXXX_CH15.indd 797 6/16/2018 12:37:57 PM


798 Chapter 15

Head or work coefficient,


c p ⋅ ∆T 2( vw 2 − vw1 )u
fh = =
2
u u2
2
2( vw 2 − vw1 ) 2(tan a 2 − tan a1 )
or      fh = = (15.27)
u tan a 2 + tan b 2
Deflection Coefficient: fdef: It is defined as,
vw 2 − vw1 f h
u( vw 2 − vw1 )
   fdef = =
= (15.28)
2 u 2
u
Pressure coefficient: It is defined as the ratio of isentropic work done to kinetic energy corres‑
ponding to the peripheral velocity.
c p ∆Tisen
Pressure coefficient,       fp = = hisen ⋅ f h (15.29)
u2
2

15.10  ❐ PRESSURE RISE IN A STAGE AND NUMBER


OF STAGES
The static pressure ratio from Eq. (15.15) is,
g
p2   T ′ − T   g −1
= 1 + hisen  2 1  
p1   T1  
Temperature rise in rotating blades,
vr21 − vr22
∆TR = T2′ - T1 =
2c p
g
p2  ∆TR  g −1
∴       = 1 + hisen 
p1  T1 
Pressure rise in rotating blades,
 g 
 
∆pR = p2 - p1 = p1 1 + h ∆T  g −1 
 isen
R
 − 1 (15.30)
 T1  
 
va22 − va23
∆Ts = T3 - T2 =
2c p
g
p3  ∆T  g −1
∴        = 1 + h s s 
p2  T2 
Pressure rise in stationary blades is,
 g 
 
∆ps = p3 - p2 = p2 1 + h ∆Ts  g −1  (15.31)
 s  − 1
 T2  
 

M15_THERMAL ENGINEERING_SE_XXXX_CH15.indd 798 6/16/2018 12:38:04 PM


Axial Flow Air Compressors 799

Pressure increase in stage is,


   ∆pst = ∆pR + ∆ps(15.32)
and temperature rise in stage, ∆Tst = ∆TR + ∆Ts(15.33)
The stagnation pressure ratio is,
g
p  ∆T ′  g −1
02 = 1 + hisen 0  (15.34)
p01  T01 
If the work done per stage is assumed to be the same, then number of stage z is given by,
∆T0
Number of stages, z= (15.35)
( dT0 )stage
If pressure ratio per stage is the same, then
p p p ( z + 1)
(rp)st = 02 = 03 = ... = 0
p01 p02 p0 z
The overall pressure ratio is,
z
rp = ( rp )st 

ln rp
    z= (15.36)
ln ( rp )st
Note that the pressure rise per stage in axial-flow compressor is less than that of a centrifugal com‑
pressor. This is because of no centrifugal action in axial flow compressor.

15.11  ❐  SURGING, CHOKING, AND STALLING


1. Surging: The delivery pressure v’s mass flow rate in an axial flow compressor is shown in
Fig. 15.7. The mass flow is zero when the discharge valve is closed, but the air entrapped in the
blade area is compressed and pressure of air increases. The condition is represented by point ‘a’
in Fig. 15.7. As the valve is opened, the flow of air starts and the pressure rises upon point ‘b’.
  Any increase in air mass flow rate after point ‘b’ is accompanied by a decrease in delivery pres-
sure. This happens because the rate of increase in pressure loss due to friction is more than the rate of
increase in pressure rise due to diffuser. Theoretically, the decrease in delivery pressure is continued
up to point ‘e’. In practice, the maximum mass flow rate is limited by point ‘c’ because beyond point
‘c’, the mass flow exceeds design mass flow, the air angles are widely different from vane angles,
and choking takes place.
 If the compressor is working between points ‘a’ and ‘b’, as the mass flow is decreased, the
delivery pressure also decreases. However, the pressure in the downstream side of the compres-
sor does not fall quickly, resulting in reversal of flow from downstream side towards the resulting
pressure gradient. When this occurs, the pressure to the downstream side of the compressor falls
and the compressor will start delivering air and the cycle would be repeated continuously. Thus,
the flow would be pulsating between point ‘a’ and ‘b’. This pulsating air flow phenomena is
known as ‘surging’. The surging causes overheating and stress reversal in the blades and dam-
ages the compressor. Within the region bc, the flow is stable. A fall in mass flow rate will result
in rise in delivery pressure to restore to fall.

M15_THERMAL ENGINEERING_SE_XXXX_CH15.indd 799 6/16/2018 12:38:06 PM


800 Chapter 15

Delivery pressure
a
c
ab - Surging region
bc - No surging
e e - Chocking
Mass flow rate

Figure 15.7  Characteristic curve

2. Choking: The maximum mass flow rate possible in a compressor is known as choking. The point
‘e’ in Fig. 15.7 represents the choking of the compressor.
3. Stalling: It is a phenomena which occurs on the suction side of the compressor in which the flow
breaks away from the aerofoil blading. It may be due to lesser flow rate than the designed value
or due to non-uniformity in the profile of the blades. Thus, stalling precedes surging. Stalling is a
local phenomenon, whereas surging is a complete system phenomenon.

15.12  ❐  PERFORMANCE CHARACTERISTICS


The relationship between pressure rise and mass flow rate at various speeds is shown in Fig. 15.8.
At a certain speed, efficiency increases as the mass flow rate increases and reaches a maximum
value after which it decreases, as shown in Fig. 15.9. The power consumed increases as mass flow
increases. Figure 15.10 shows the performance and constant efficiency curves. The performance
G T1 N p N
curves are plotted with dimensionless parameters, , and 3 ⋅ At constant value of
p1 T1 p1 T1
G T1
there is a considerable narrower range of mass flow rate than in centrifugal compressor.
p1

6.0
Surge-line
5.0
0.86
Efficiency 0.825
p3 4.0
N
p1 = 1.0
√T
3.0 0.9 1

0.8 0.80
2.0
0.7
1.0 0.5 0.6
0 10 20 30 40 50
G√ T / p1

Figure 15.8  Pressure ratio v’s mass flow rate

M15_THERMAL ENGINEERING_SE_XXXX_CH15.indd 800 6/16/2018 12:38:09 PM


Axial Flow Air Compressors 801

Efficiency

Power, efficiency
Power consumed

Mass flow rate

Figure 15.9  Power and efficiency v’s mass flow rate

170
160
Lines of constant
150
speed
140
130 Lines of
constant
120
efficiency Design point
110
100
Pressure rise %

90
80
Surge line
70
60
50
40
30
20
10
0
10 20 30 40 50 60 70 80 90 100 110 120
Volume flow %

Figure 15.10  Pressure rise v’s volume flow

The surge point is reached in axial compressors much before the maximum pressure ratio for given
N
value of ⋅ Since the design usually calls for the operating line to be near the maximum point on
T1
N
the curves, it follows that the operating line for axial flow compressors must be very near the
T1
surge line, thus narrowing the range of stable operation.

M15_THERMAL ENGINEERING_SE_XXXX_CH15.indd 801 6/16/2018 12:38:10 PM


802 Chapter 15

15.13  ❐ 
COMPARISON OF AXIAL FLOW
AND CENTRIFUGAL COMPRESSORS
The merits and demerits of axial flow compressors are given in Table 15.1.

Table 15.1  Merits and demerits of axial flow compressor over centrifugal compressor

S. no. Parameter Axial flow Centrifugal


 1. Pressure ratio 1.2 : 1, requires more number of 4 : 1 per stage
stages for higher pressure ratio.
 2. Isentropic efficiency 85%-88% 70%
 3. Power required per kg air flow rate Lower than centrifugal Higher than axial
 4. Flexibility of operation Less More
 5. Frontal area More Less
 6. Effect of deposit formation on the impeller Effects No effect
 7. Starting torque High Low
 8. Efficiency v’s speed curve More steep More flat
 9. Suitability for supercharging No Suitable
10. Applications Large gas turbines Refrigerators and
industrial gases

15.14  ❐  APPLICATIONS OF AXIAL FLOW COMPRESSORS


Axial flow compressors have wide applications in the following:
1. Jet engines
2. Gas turbines for power plants
3. Steel mills
4. Aircraft applications.
Note that the requirements of compressors for aircraft application:
1. High efficiency
2. Smaller frontal area
3. Suitability for multistaging
4. High starting torque
5. Lesser drag

15.15  ❐  LOSSES IN AXIAL FLOW COMPRESSORS


The following losses occur in axial flow compressors:
1. Profile losses on the surface of blades
2. Skin friction loss on the annulus walls
3. Secondary flow losses

M15_THERMAL ENGINEERING_SE_XXXX_CH15.indd 802 6/16/2018 12:38:10 PM


Axial Flow Air Compressors 803

Example 15.1
An axial compressor is fitted with half-reaction blading, the blade inlet and outlet angles being 50°
and 15° when measured from the axial direction. The mean diameter of a certain blade pair is 85 cm
and the rotor runs at 5500 rpm. Calculate the necessary isentropic efficiency of the stage if the pres-
sure ratio of compression is to be 1.4 and the inlet air temperature is 25°C. Take cp = 1.005 kJ/kg.
K, g = 1.4. [CSE, 1987]

Solution
b1 = 50°, b2 = 15°, vf1 = vf2 = vf, a1 = 15°, a2 = 50°
Mean blade speed,
pDN p × 0.85 × 5500
u= = = 244.78 m/s
60 60
From the velocity triangles, (Fig. 15.11)
u − vw1
tan b1 =
vf
u − vw 2 vw1
tan b2 = = for Rd = 0.5
vf vf
tan b1 u − vw1 tan 50°
= =
tan b 2 vwl tan 15°
1.192
= = 4.448
0.268
vw1 = 0.1826 u
vw2 = 0.8164 u
∆vw = vw2 - vw1 = 0.6328 u = 0.6328 × 244.78 = 155 m/s
Work done/kg, wa = u × ∆vw = 244.78 × 155 = 37940.9 N.m/kg
Isentropic work, wisen = cp (T2 - T1)
g −1
1
p  g
T2 = T1  2  = 298 (1.4) 3.5 = 328 K
 p1 

u
vw2
vw1

vf 2

vf1
va1 va2
vr2
15° 15°
50° vr1 50°

Figure 15.11  Velocity diagrams for axial flow compressor

M15_THERMAL ENGINEERING_SE_XXXX_CH15.indd 803 6/16/2018 12:38:16 PM


804 Chapter 15

wisen = 1.005 (328 - 298) = 30.15 kJ/kg


Isentropic efficiency,
wisen 30.15 × 103
hisen = = × 100 = 79.46%
wa 37940.9

Example 15.2
An axial flow compressor comprises a number of similar stages with equal work done per stage,
and the velocity of flow is uniform throughout the compressor. The following data is given:
Overall stagnation pressure ratio = 4
Stagnation inlet temperature = 330 K
Relative air angle at rotor inlet = 130°
Relative air angle at rotor outlet = 100°
Blade velocity = 180 m/s
Degree of reaction = 0.5
Overall stagnation adiabatic efficiency = 0.86
Calculate (a) the stagnation outlet temperature, and (b) number of stages.

Solution
The velocity diagrams are shown in Fig. 15.12.
g −1
p  g
T02 = T01  02 
 p01 
1
 4  3.5
= 330   = 490.38 K
 1
Stagnation adiabatic efficiency,
T02 − T01
ha =
T02′ − T01
490.38 − 330
0.86 =
T02′ − 330

u
vw2 vw1
vw1
50° 50°
80° vf2 80°
vf1
va1
va2 vr2
a1 b2
b1 vr1 a2

Figure 15.12  Velocity diagrams for axial flow compressor

M15_THERMAL ENGINEERING_SE_XXXX_CH15.indd 804 6/16/2018 12:38:19 PM


Axial Flow Air Compressors 805

T02′ = 516.5 K
vf
tan 80° =
Also,   = 5.671
u − vw1
vf
  tan 50° = = 1.192
u − vw1
For degree of reaction = 0.5, the velocity diagrams are similar.
∴      u - vw2 = vw1
vf vf
= 5.671 and = 1.192
vw1 u − vw1
u − vw1 5.671
= = 4.757
vw1 1.192
vw1 = 0.1737 u
vw2 = u - vw1 = 0.8263 u
∆vw = vw2 - vw1 = 0.6526 u
Work done per stage = u · ∆vw = 180 × 0.6526 × 180 = 21144 N.m/kg
Total work = cp (T02 - T01) = 1.005(490.38 - 330) = 161.182 kJ/kg
161.182 × 103
Number of stages = = 7.62 = 8
21144

Example 15.3
Air at a temperature of 300 K enters a 10-stage axial flow compressor at the rate of 3.5 m/s. The
pressure ratio is 6.0 and the isentropic efficiency is 90%. The process is adiabatic and the compres-
sor has symmetrical stages. The axial velocity is uniform across the stage and equals to 120 m/s
and the mean blade speed of each stage is 200 m/s. Assume that the temperature change is same
in each stage.
Determine the direction of the air at entry to and exit from the rotor and the stator blades. Also,
find the power given to the air. Take cp = 1.005 kJ/kg.K and g = 1.4. [IES, 2001]
Solution
The velocity diagrams are shown in Fig. 15.13.
z = 10, T1 = 300 K, m = 3.5 kg/s, rp = 6, hi = 0.9,
vf = 120 m/s, u = 200 m/s
(∆T)stage = (∆T)overall/z
T 
=  1  [rp(g −1)/g − 1]
 zhi 
 300  0.4 /1.4
=  [6 − 1] = 22.284 K
10 × 0.9 

M15_THERMAL ENGINEERING_SE_XXXX_CH15.indd 805 6/16/2018 12:38:23 PM


806 Chapter 15

u
vw2
vw1

vf vf

va2
va1
vr2
a1 b2
b1 vr1 a2

Figure 15.13  Velocity diagrams for axial flow compressor

Increase in temperature per stage,


 uv f 
(∆T)stage =   (tan b1 − tan b 2 )
 cp 
tan b1 - tan b2 = 22.284 × 1.005 × 103/(200 × 120) = 0.93313
u 200
Also,  tan b1 + tan b2 = = = 1.6667
v f 120
tan b1 = 1.2999, b1 = 52.43°, b2 = 20.14°
Power supplied = z mc
 p (∆T)stage = 10 × 3.5 × 1.005 × 22.284 = 783.84 kW

Example 15.4
In an axial flow air compressor: u = 250 m/s, vf = 200 m/s, a1 = 50°, a2 = 15° and r = 1 kg/m3.
Determine (a) the pressure rise, and (b) the work done per kg of air.

Solution
(a) Pressure rise through a moving blade ring,
1
∆p = rv 2f (tan2 a1 - tan2 a2)
2
1
= × 1 × 2002 (tan2 50° - tan2 15°) × 10-5 = 0.2697 bar
2
(b) Work done per kg of air,
w = u(vw1 - vw2) = uv f (tan a1 - tan a2)
= 250 × 200 (tan 50° - tan 15°) × 10-3 = 46.19 kW
Example 15.5
An axial flow air compressor having eight stages and 50% degree of reaction compresses air in
the pressure ratio of 4.5 : 1. Air enters the compressor at 27°C and flows through it with a constant
speed of 100 m/s. The moving blades of compressor rotate with a mean speed of 200 m/s. The
isentropic efficiency of the compressor is 85%. Calculate (a) the work done on the compressor and
(b) blade angles. Take g = 1.4 and cp = 1.005 kJ/kg.K for air.
Solution
z = 8, Rd = 0.5, rp = 4.5, T1 = 273 + 27 = 300 K, vf = 100 m/s, u = 200 m/s, hisen = 0.85

M15_THERMAL ENGINEERING_SE_XXXX_CH15.indd 806 6/16/2018 12:38:25 PM


Axial Flow Air Compressors 807

g −1 1
g
T2 = T1 ( rp ) = 300( 4.5) 3.5 = 461 K
T2 − T1 461 − 300
T2′ = T1 + = 300 + = 489.4 K
hisen 0.85
(a) Work required per kg of air,
w = cp (T2′ - T1) = 1.005 (489.4 - 300) = 190.36 kJ/kg
(b) Total work done = zu (vw2 - vw1) = zu vf (tan a 2 - tan a1)
190.36 = 8 × 200 × 100 (tan a2 - tan a1) × 10-3
tan a2 - tan a1 = 1.1897
For     Rd = 0.5, a1 = b2 and a2 = b1
∴ 1.1897 = tan b1 - tan b2
or      tan b1 - tan a1= 1.1897
Now,      u = vf (tan a1 + tan b1)
u 200
    tan a1 + tan b1 = = =2
v f 100
Adding the above two equations, we get

2 tan b1 = 3.1897
or      tan b1= 1.5949
or   b1 = 57.9° = a2
tan a1 = tan b1 - 1.1897 = 0.4052
or   a1 = 22° = b2

Example 15.6
The overall isentropic efficiency of an axial flow air compressor is 83%. It draws air at 25°C and
1 bar and compresses to 4 bar. Assuming 50% degree of reaction, blade velocity 180 m/s, exit angle
from stator (a1 ) = 15°, inlet angle to rotor (b1 ) = 45° and work input factor as 0.80, calculate the
flow velocity and the number of stages.
Solution
hisen = 0.83, T1 = 273 + 25 = 298 K, p1 = 1 bar, p2 = 4 bar,
Rd = 0.5,a1 = 15°, b1 = 45°, u = 180 m/s, fw = 0.80
g −1 1
p  g  4  3.5
T2 = T1  2  = 298   = 442.8 K
 p1   1
T −T
hisen = 2 1
T2′ − T1
T2 − T1 442.8 − 298
or    T2′ = T1 + = 298 + = 472.5 K
hisen 0.83

M15_THERMAL ENGINEERING_SE_XXXX_CH15.indd 807 6/16/2018 12:38:29 PM


808 Chapter 15

Work required per kg of air


= cp (T2′ - T1) = 1.005 (472.5 - 298) = 175.37 kJ/kg
From inlet velocity triangle (Fig.15.13), we have
u
= tan a1 + tan b1 = tan 15° + tan 45° = 1.268
vf
180
or   vf = = 141.96 m/s
1.268
For Rd = 0.5, a2 = b1 = 45° and a1 = b2 = 15°
vw2 = vf tan a2 = 141.96 tan 45° = 141.96 m/s
vw1 = vf tan a1 = 141.96 tan 15° = 38.04 m/s
Work done per kg per stage = fwu(vw2 - vw1)
= 0.80 × 180 (141.96 - 38.04) × 10-3 = 14.965 kJ/kg
175.37
Number of stages, z= = 11.72 = 12
14.965
Example 15.7
In a 10-stage axial flow air compressor, the overall stagnation pressure ratio is 4 : 1 with an overall
isentropic efficiency of 90%. The inlet stagnation temperature and pressure are 293 K and 1 bar.
The work is divided equally between the stages. The mean blade speed is 180 m/s and 50% reaction
blading are used. The flow velocity throughout the compressor is 90 m/s. Calculate (a) the blade
angles, and (b) the power required.
Solution
z = 10, rop = 4, hisen = 0.9, T01 = 293 K, p01 = 1 bar, u = 180 m/s, Rd = 0.5, vf = 90 m/s
(a) For Rd = 0.5, a1 = b2 and a2 = b1
With isentropic compression, the temperature of air leaving the compressor is,
g −1 1
 p02  g  4  3.5
T02 = T01  p  = 293   = 435.4 K
01  1
T02 − T01 435.4 − 293
T02′ = T01 + = 293 + = 451.2 K
hisen 0.9
Work required by the compressor = cp ( T02′ - T01)
= 1.005 (451.2 - 293) = 159 kJ/kg
= zu (vw2 - vw1)
= zuvf (tan a2 - tan a1)
159 × 103
tan a2 - tan a1 =
= 0.9815
10 × 180 × 90
or     tan b1 - tan a1 = 0.9815

M15_THERMAL ENGINEERING_SE_XXXX_CH15.indd 808 6/16/2018 12:38:48 PM


Axial Flow Air Compressors 809

From inlet velocity triangle, we have,


u 180
= tan a1 + tan b1 = =2
vf 90
Adding the above two equations, we get
2  tan b1 = 2.9815
or      tan b1 = 1.4908
or  b1 = 56.15° = a2
tan a1 = 2 - 1.4908 = 0.5092
or     a1 = 26.98° = b2
(b) Power required = m cp( T02′ - T01) = 1 × 1.005 (451.2 - 293) = 159 kW

Example 15.8
The overall stagnation pressure ratio and isentropic efficiency for an axial flow air compressor are
5 and 82%, respectively. The inlet stagnation pressure and temperature are 1 bar and 303 K. The
mean blade speed is 200 m/s and degree of reaction is 0.5. The relative air angles are 12° and 30°
at the rotor inlet and outlet, respectively. The work input factor is 0.9. Calculate (a) the stagnation
polytropic efficiency, (b) the number of stages, (c) the inlet temperature and pressure, and (d) the
blade height in the first stage if the hub-tip ratio is 0.42 and mass flow rate is 20 kg/s.

Solution
rop = 5, hisen = 0.82, p01 = 1 bar, T01 = 303 K, u = 200 m/s, Rd = 0.5, b1 = 30° =a2, b2 = 12° = a1,
fw = 0.9, m = 20 kg/s, hub-tip ratio = 0.42.

g −1
0.4
 p0 z  g
(a) T0z = T01  p  = 3.3 ( 4) .4
1 = 479.9 K
01
T0 z − T01
T0 z ′ = T01 +
hisen
479.9 − 303
= 303 + = 518.7 K
0.82
Polytropic efficiency,
g −1
p  g 0.4
ln  0 z 
 p01  ln(5) 1.4
hp = =
T   518.7 
ln  0 z ′  ln 
 T01   303 
= 0.8553 or 85.53%
(b) From the inlet velocity triangle (Fig. 15.13), we have
u
= tan a1 + tan b1 = tan 12° + tan 30° = 0.7899
vf
200
or vf = = 253.2 m/s
0.7899

M15_THERMAL ENGINEERING_SE_XXXX_CH15.indd 809 6/16/2018 12:39:01 PM


810 Chapter 15

vw2 = vf 2 tan a2 = 253.2 tan 30° = 146.2 m/s


vw1 = vf tan a1 = 253.2 tan 12° = 53.8 m/s
Work consumed per stage = fw u(vw2 - vw1)
= 0.9 × 200 (146.2 - 53.8) × 10-3 = 16.632 kJ/kg
Total work consumed by the compressor = cp ( T0 z ′ - T01)
= 1.005 (518.7 - 303) = 216.78 kJ/kg
216.78
Number of stages, z = = 13
16.632
vf 253.2
(c) va1 = = = 258.86 m/s
cos a1 cos 12°
Static temperature,
va21
T1 = T01 -
2c p

( 258.86)2
= 303 - = 269.7 K
2 × 1.005 × 103
g
3.5
 T  g −1  269.7 
p1 = p01  1  =1  = 0.665 bar
 T01   303 

p1 0.665 × 105
(d) r1 = = = 0.859 kg/m3
RT1 287 × 269.7
m = r1A1vf
or 20 = 0.859 × p r12 [1 - (0.42)2] × 253.2
or   r1 = 0.1885 m or 188.5 mm
rh
= 0.42, rh = 0.42 × 188.5 = 0.0792 m or 79.2 mm
r1

Example 15.9
A multistage axial flow air compressor delivers 25 kg/s of air. The inlet stagnation condition is
1 bar and 20°C. The power consumed by the compressor is 4500 kW. Calculate (a) the delivery
pressure, (b) the number of stages, and (c) the overall isentropic efficiency of the compressor.
Assume that temperature rise in the first stage is 15° C, the polytropic efficiency of compression
is 0.90 and the stage stagnation pressure ratio is constant.
Solution
m = 25 kg/s, p01 = 1 bar, T01 = 273 + 20 = 293 K, P = 4500 kW, T02 = 293 + 15 = 308 K, np = 0.90
g −1
p  g
ln  02 
(a) Polytropic efficiency hp =  p01 
T 
ln  02 
 T01 

M15_THERMAL ENGINEERING_SE_XXXX_CH15.indd 810 6/16/2018 12:39:06 PM


Axial Flow Air Compressors 811

1
 p  3.5
ln  02 
 p01 
or    0.9 =
 308 
ln 
 293 
p   308 
or     ln  02  = 3.5 × 0.9 × ln  = 0.15727
 p01   293 
p
    02 = 1.17
p01
Power     P = m cp ( T0 z ′ - T01)

or 4500 = 25 × 1.005 ( T0 z ′ - 293)


or    T0 z ′ = 472.1 K
Again, polytropic efficiency can be expressed as
 g − 1  n 
hp =   
 g   n − 1
 1.4 − 1  n 
or 0.9 = 
 1.4   n − 1
n
or        = 3.15
n −1
n
p0 z  T0′z  n −1  472.1 3.15
= = = 4.4934
p01  T01   293 
or p0z = 1 × 4.4934 = 4.4934 bar
p02 p03 p0 z
(b) Now = = ... =
p01 p02 p0 ( z − 1)
z
 p02  p0 z
 p  = p
01 01
p  p 
or z ln  02  = ln  0 z 
 p01   p01 
 4.4934 
ln 
 1 
or z = = 9.57 = 10
ln 1.17
g −1 1
(c) T0z = T01  p0 z  g
= 293 ( 4.4934) .5
3 = 450.1 K
 p 
01 T0 z − T01
(hisen)overall =
T0 z ′ − T01
450.1 − 293
= = 0.8772 or 87.72%
472.1 − 293

M15_THERMAL ENGINEERING_SE_XXXX_CH15.indd 811 6/16/2018 12:47:10 PM


812 Chapter 15

Example 15.10
An axial compressor receives 1000 m3/min of free air at 15°C and 0.9 bar. The blades are of aero-
foil type having projected area and blade length as 19.25 cm2 and 6.75 cm, respectively. The blade
ring mean diameter is 60 cm and the speed is 6000 rpm. On each blade ring, there are 50 blades and
the blades occupy 10% of the axial area of flow. Value of CL and CD are 0.6 and 0.05 respectively
at zero angle of incidence. Assuming isentropic compression, calculate the pressure rise per blade
ring and the power input per stage. Assume axial inlet.
Solution
V = 1000 m3/min, T1 = 273 + 15 = 288 K, p1 = 0.9 bar, A = 19.25 cm2, h = 6.75 cm, kb = 0.9, CL = 0.6,
CD = 0.05, Dm = 60 cm, i = 50

Density of air,
p1 0.9 × 105
r= = = 1.09 kg/m3
RT1 287 × 288
Area across flow, Af = kb p Dmh
= 0.9 × p × 0.6 × 6.75 × 10-2
= 0.1145 m2
V 1000
va1 = = = 145.55 m/s
60 A f 60 × 0.1145
p Dm N
Blade velocity,     u =
60
p × 0.6 × 6000
= = 188.5 m/s
60
For axial inlet, va1 = vf = 145.55 m/s
The velocity triangle at inlet is shown in Fig. 15.14.
u 188.5
tan b1 = = = 1.295
v f 145.55
b1 = 52.325°
vr1 = u 2 + va21

= (188.5)2 + (145.55) 2 = 238.15 m/s

b1

vr1
vf = va1

Figure 15.14  Velocity diagram for axial flow compressor

M15_THERMAL ENGINEERING_SE_XXXX_CH15.indd 812 6/16/2018 12:47:14 PM


Axial Flow Air Compressors 813

1
Lift force, FL = C rv 2 A
2 L r1
1
= × 0.6 × 1.09 × (238.15)2 × 19.25 × 10-4 = 35.7 N
2
1
Drag force, FD = C rv 2 A
2 D r1
1
= × 0.05 × 1.09 × (238.15)2 × 19.25 × 10-4 = 2.975 N
2
Power input per stage,
p = (FL cos b1 + FD sin b1) ui
= (35.7 cos 52.325° + 2.975 sin 52.325°) × 188.5 × 50 × 10-3
= 227.839 kW
Mass flow rate,
1000 × 1.09
m = V r =
= 18.167 kg/s
60
 g −1 
 pT1  rp g − 1
Now,      P = mc
 
 
 1 
or    227.839 = 18.167 × 1.005 × 288  rp3.5 − 1
 
or rp = 1.16
p2 = 0.9 × 1.16 = 1.044 bar
Pressure rise = p2 - p1 = 1.044 - 0.9 = 0.144 bar

Example 15.11
An axial flow compressor gives a pressure rise of 4 : 1 and the total head isentropic efficiency is
86%. Stagnation inlet temperature is 17°C. The inlet and outlet air angles from the rotor blades are
45° and 10°, respectively. The rotor and stator blades are symmetrical. The mean blade and axial
velocity remain constant. Assuming blade speed of 220 m/s and work input factor 0.86, find the
polytropic efficiency, number of stages required and Mach number. Take R = 287 J/kg.K.
Solution
rp = 4, ho isen = 0.86, T01 = 273 + 17 = 290 K, a1 = 45°, a2 = 10°, u = 220 m/s, fw = 0.86
Polytropic efficiency,
 g − 1  n 
hp =   
 g   n − 1
Head isentropic efficiency,
g −1
rp g −1
ho isen =
n −1
rp n −1

M15_THERMAL ENGINEERING_SE_XXXX_CH15.indd 813 6/16/2018 12:47:18 PM


814 Chapter 15

g −1
rp g −1
=
 g −1 1
 g  h
rp p
−1
1
4 3.5 − 1
or       0.86 =
1
3.5×h p
4 −1
0.486
=
1
3.5h p
4 −1
1
or 3.5h p
= 1.565
4
1
or       ln 4 = ln1.565
3.5h p

hp = 0.8843 or 88.43%
or    
The velocity diagrams are shown in Fig. 15.15.
a1 = b2 = 10°, a2 = b1 = 45°

u = vf1 (tan a1 + tan b1)


220
vf1 =
or   = 187 m/s
tan 10° + tan 45°
Power required per kg of air = cp (T2 - T1) = fw uvf (tan a2 - tan a1)

0.86 × 220 × 187(tan 45° − tan 10°)


T2 - T1 = = 29°C
1.005 × 103

vf2
va2 vr2
a2 b2

b1 a1 va1
vr1
vf1

Figure 15.15  Velocity diagrams at inlet and outlet of vane

M15_THERMAL ENGINEERING_SE_XXXX_CH15.indd 814 6/16/2018 12:47:21 PM


Axial Flow Air Compressors 815

n −1 g −1 0.4
T gh
Also, 02 = rp n = 4 p = 41.4 × 0.8843= 1.565
T01
T02 = 290 × 1.565 = 453.86 K
Total temperature rise = T02 - T01
= 453.86 - 290 = 163.86°C
163.86
Number of stages, z= = 5.65 = 6
29
From the inlet velocity diagram,
vf 187
vr1 = = = 264.5 m/s
cos a1 cos 45°
vf 187
va1 = = = 190 m/s
cos b1 cos 10°

va21 190 2
T1 = T01- = 290 - = 272 K
2c p 2 × 1.005 × 103
vr1 264.5
 M = = = 0.8
g RT1 1.4 × 287 × 272

Example 15.12
Air at 1.01325 bar and 288 K enters an axial flow compressor stage with an axial velocity of
150 m/s. There are no inlet guide vanes. The rotor stage has a tip diameter of 60 cm and a hub diam-
eter of 50 cm and rotates at 100 rps. The air enters the rotor and leaves the stator in the axial direc-
tion with no change in velocity or radius. The air is turned through 30° as it passes through rotor.
Assuming constant specific heat and that air enters and leaves the blade at the blade angles, con-
struct the velocity diagrams at mean diameter for this stage, and calculate (a) the mass flow rate,
(b) the power required, and (c) the degree of reaction.
Solution
p1 = 1.01325 bar, T1 = 288 K, vf = 150 m/s, D2 = 60 cm, D1 = 50 cm, N = 100 rps, b2 − b1 = 30°
The velocity triangles are shown in Fig. 15.16.
vw2 u
va2
vf2 a b
2 2 vr2

b1
vr1 vf1 = va1

Figure 15.16  Velocity diagrams at inlet and outlet of vane

M15_THERMAL ENGINEERING_SE_XXXX_CH15.indd 815 6/16/2018 12:47:25 PM


816 Chapter 15

D1 + D2 0.5 + 0.6
Dm = = = 0.55 m
2 2
u = p DmN
= p × 0.55 × 100 = 172.8 m/s
u 172.8
tan b1 = = = 1.152
vf1 150
or      b1 = 49.04°
b2 - b1 = 30° or b1 - b2 = 30°
or b2 = 19.04° or 79.04°
vf 2 tan b2 = u + vw2
150 tan 19.04° = 51.77 < 172.8 m/s
∴ vw2 is -ve
v u − v f tan b 2 121.03
∴ tan a2 = w 2 = = = 0.80
vf vf 150
a2 = 38.9°
va21
Now, T01 = T1 +
2c p
150 2
288 = T1 + = T1 + 11.2
2 × 1.005 × 103
or T1 = 276.8 K
g
p01  T01  g −1
=
p1  T1 
3.5
 276.8 
p1 = 1.01325  = 0.8819 bar
 288 
p1 0.8891 × 105
r1 = = = 1.11 kg/m3
RT1 287 × 276.8
Volume flow rate,
p
V = Af vf = ( D22 − D12 )v f
4
p
= (0.62 - 0.52) × 150 = 12.96 m3/s
4
(a) Mass flow rate,
m = V r = 12.96 × 1.11 = 14.384 kg/s

(b) Power required,


P = m uvf = 14.384 × 172.8 × 150 × 10-3 = 372.85 kW

(c) Degree of reaction,


vf
Rd = (tan b1 + tan b2)
2u
150
= (tan 49.04 + tan 19.04) = 0.65
2 × 172.8

M15_THERMAL ENGINEERING_SE_XXXX_CH15.indd 816 6/16/2018 12:47:30 PM


Axial Flow Air Compressors 817

Example 15.13
An axial flow compressor with pressure compression ratio 4 draws air at 20°C and delivers at
197°C. The mean blade speed and flow velocity are constant throughout the compressor. Assuming
50% reaction and blading velocity as 180 m/s, find the flow velocity and number of stages. Take
work input factor = 0.82, a1 = 20°, b1 = 42°, cp = 1.005 kJ/kg K.

Solution
Given: rp = 4, T1 = 273 + 20 = 293 K, T2 = 273 + 197 = 470 K, u = u1 = u2 and vf1 = vf1 = vf, Rd = 0.5,
u = 180 m/s, fw = 0.82, a1 = 20°, b1 = 42°, cp = 1.005 kJ/kg K
For 50% degree of reaction,
a2 = b1 = 42° and b2 = a1 = 20°
Work input factor, 2( vw 2 − vw1 )
fw =
u
2( vw 2 − vw1 )
or 0.82 =
180
0.82 × 180
or vw2 - vw1 = = 73.8 m/s
2
Refer to Fig. 15.17
vw1 v
= tan a1 and w 2 = tan a2
vf vf
∆vw = vw2 - vw1 = vf (tan a2 - tan a1)
= vf (tan 42° - tan 20°) = 0.5364 vf
∴  0.5364 vf = 73.8
Flow velocity,  vf = 137.6 m/s
Work done per stage = u × ∆vw
  = 180 × 73.8 = 13284 Nm/kg
Total work = cp (T2 - T1)
= 1.005 × 103 (470 - 293) = 177885 Nm/kg
177885
Number of stages, z =
13284
  = 13.39 14

u
vw2
vw1

vf vf
va1 va2
vr2
a1 b2
b1 vr1 a2

Figure 15.17  Velocity diagrams for axial flow compressor

M15_THERMAL ENGINEERING_SE_XXXX_CH15.indd 817 6/16/2018 12:47:33 PM


818 Chapter 15

Example 15.14
An eight-stage axial flow compressor provides an overall pressure ratio of 6: 1 with an overall
isentropic efficiency 90% when the temperature of air at inlet is 20°C. The work divided equally
between the stages. A 50% reaction design is used with a mean blade speed 188 m/s and a constant
axial velocity 100 m/s through the compressor. Estimate the power required and the blade angles.
Take cp = 1.005 kJ/kg.K and g = 1.4.

Solution
Given: z = 8, pz/p1 = 6, (hisen)0 = 0.9, T1 = 273 + 20 = 293 K, Rd = 0.5, u = 188 m/s, vf = 100 m/s

g −1
Refer to Fig. 15.18.
Tz  pz  g
=
T1  p1 
0.4
  Tz = 293 (6 ) 1.4 = 489 K
Tz − T1
(hisen)0 =
T1 − T1
489 − 293
or 0.9 =
Tz ′ − 293
or TzÄ = 510.8 K
u 188
tan a1 + tan b1 = = = 1.88
v f 100
For Rd = 0.5, a1 = b2, a2 = b1
From Fig. 15.19, we have vw1 = vf1 tan a1 and vw2 = vf2 tan a2
Work done per kg of air, w = zu (vw2 - vw1 )
8 × 188 × 100
= (tan a2 - tan a1)
103
= 150.4 (tan a2 - tan a1)
= 150.4 (tan b1 - tan a1)
= cp (TzÄ - T1) = 0.24 × 4.187 (510.8 - 293) = 218.86

T
p2

T2′ 2′ p1
2
T2

T1 1

s
O

Figure 15.18   T-s diagram

M15_THERMAL ENGINEERING_SE_XXXX_CH15.indd 818 6/16/2018 12:47:35 PM


Axial Flow Air Compressors 819

u
vw1

vf2
vr1
va1 a1 b1

u
vw2

vf1
vr2
b2
va2 a2

Figure 15.19  Velocity triangles

∴ tan b1 - tan a1 = 1.455


After solving, we get
2 tan b1 = 3.335
or b1 = 59°
tan a1 = 1.88 - 1.6676 = 0.2124
or a1 = 12°
Power required = Cp (TzÄ - T1) = 0.24 × 4.187 (510.8 - 293)
= 218.86 kW

Example 15.15
The first stage of an axial compressor is designed on free vortex principles with no inlet guide vanes.
The rotational speed is 6000 rpm and the stagnation temperature rise is 20 K. The hub-tip ratio is
0.60, work done factor is 0.93 and isentropic efficiency of the stage is 0.89. Assuming an inlet veloc-
ity of 140 m/s and ambient conditions of 1.01 bar and 288 K, calculate (a) the tip radius and corre-
sponding rotor air angles, if the Mach number relative to the tip is limited to 0.95, (b) the mass flow
entering the stage, (c) the stage stagnation pressure ratio and power input, and (d) rotor air angles at
the root section. Assume cp = 1.005 kJ/kgK and g  = 1.4.

Solution
rh
Given: N = 6000 rpm, ∆T0′ = 20 K, = 0.6, fw = 0.93, (hisen)stage = 0.89, va1 = 140 m/s, p1 = 1.01
rt
bar, T1 = 288 K, M = 0.95, cp = 1.005 kJ/kg.K, g  = 1.4
With no inlet guide vanes, a1 = 0° and vf1 = va1 = 140 m/s, vw1 = 0
For free vortex flow = vw1 × rh = vw2 × rt
The velocity diagrams are as shown in the Fig. 15.20(a).
Also, vf 1 = vf 2 = vf
∴ va2 = va1 and b1 = b2
va21
T01 = T1 +
2c p

M15_THERMAL ENGINEERING_SE_XXXX_CH15.indd 819 6/16/2018 12:47:37 PM


820 Chapter 15

vf2 = va2 b vr2


2

vr1
va1 = vf1 b
1

(a)

T pO2

02′
02

pO1

01
s
(b)

Figure 15.20  (a) Velocity diagrams, (b) T-s diagram for axial flow compressor

140 2
= 288 + = 297.75K
2 × 1.005 × 103
T02Ä = T01 + ∆T0′ = 297.75 + 20
= 317.75 K
T − T01
(hisen)stage = 02
T02′ − T01
T02 = T01 + ( T02′ - T01) × (hisen)stage
= 297.75 + 20 × 0.89 = 315.55 K
(a) Mach number relative to tip, M = 0.95
vr 2
M =
g RT2

va22
 T2 = T02 -
2c p
140 2
= 315.55 - = 306 K
2 × 1.005 × 103
vr2
0.95 =
1.4 × 287 × 306
or  vr2 = 333.1 m/s

M15_THERMAL ENGINEERING_SE_XXXX_CH15.indd 820 6/16/2018 12:47:40 PM


Axial Flow Air Compressors 821

vf 2 va1 140
cos b2 = = = = 0.42
vr 2 vr 2 333.1
      or b2 = 65.14°, a2 = 0

u = vr22 − v 2f 2 = (333.1)2 − (140)2 = 302.25 m/s


pDt N
     Now, u=
60 × 103
p × Dt × 600
  or      302.25 =
60 × 103
   Tip diameter, Dt = 962 mm

 Hub diameter, Dh = 0.6 Dt = 0.6 × 962 = 577.2 mm



u 302.25
tan b1 = = = 2.15893
va1 140
b1 = 65.14°, a1 = 0
(b) Density of air at inlet,
p1 1.01 × 105
r1 = = = 1.222 kg/m3
RT1 287 × 288
p
( D 2 − Dh2 )v f r 1
 Mass flow rate of air entering the stage, m a =
4 t
p
         = (962)2 − (577.2)2  × 10-6 × 140 × 1.222
4 
    = 79.58 kg/s
g 1.4
p  T  g −1  315.55  0.4
(c) 02 =  02  = = 1.225
p01  T01   297.75 
fw m a uu f 0.93 × 79.58 × 302.25 × 140
Power input = = = 3131.7 kW
3
10 103
(d) a1 = a2 = 0°, b1 = b2 = 65.14°

Example 15.16
An axial flow compressor has an overall pressure ratio of 4 and mass flow rate of 3 kg/s. If the
polytropic efficiency is 88% and the stagnation temperature rise per stage must not exceed 25 K,
calculate the number of stages required and pressure ratio of first and last stages. Assume equal
temperature rise in all stages. If the absolute velocity approaching the last rotor is 165 m/s at
an angle of 20° from the axial direction, the work done factor is 0.83, the velocity diagram is
symmetrical and the mean diameter of last stage rotor is 18 cm, calculate the rotational speed and
length of the last stage rotor blade inlet to the stage. Ambient conditions are 1.01 bar and 288 K.

Solution
p2
Given: rp = = 4, m a = 3 m/s, hp = 0.88, (dT0h)stage ≤ 25K, (va1)z = 165 m/s,
p1
a1 = 20°, fw = 0.83, (dm)z = 0.18 m, p1 = 1.01 bar, T1 = 288 K

M15_THERMAL ENGINEERING_SE_XXXX_CH15.indd 821 6/16/2018 12:47:45 PM


822 Chapter 15

T p02
02′ p2
02
2
2 p01
p1
01
1
s

Figure 15.21  T – s diagram


Refer to Fig. 15.21.
Tz
= rp(g −1)g
T1
0.4
Tz = 288 × ( 4) 1.4 = 428 K
( va1 ) 2z
T0z = Tz +
2c p
1652
= 428 + = 428 + 13.5 = 441.5 K
2 × 1.005 × 103
g
p0 z  T0 z  g −1
=
p1  T1 
1.4
 441.5  0.4
p0z = 1.01  = 4.5054 bar
 288 
Now, (va1)1 = (va1)z = 165 m/s
( v )2
T01 = T1 + a1 = 288 + 13.5 = 301.5 K
2c p
g
p01  T01  g −1
=
p0 z  T0 z 
3.5
 301.5 
p 01 = 4.505  = 1.186 bar
 441.5 
Pressure ratio of first and last stage,
p0 z 4.505
= = 3.8
p01 1.186
n −1
T p  n
Now,       0 z ′ =  0 z 
T01  p01 
 g − 1  n 
Polytropic efficiency, hp =   
 g   n − 1

M15_THERMAL ENGINEERING_SE_XXXX_CH15.indd 822 6/16/2018 12:47:52 PM


Axial Flow Air Compressors 823

 1.4 − 1  n 
0.88 = 
or                  
 1.4   n − 1
or n = 1.48

∴                   T0zÄ = 301.5(3.8)0.48/1.48 = 464.9 K


T0 z ′ − T01
Number of stages, z=
( dT0′ ) stage
464.9 − 301.5 163.4
  = = = 6.54 = 7
25 25
Actual stagnation temperature rise per stage
163.4
= = 23.4 K < 25K
7
Hence O.K.
For symmetrical velocity diagram, a1 = b2 (see Fig. 15.22)

Work done per stage, w = cp × (dTÄ0)stage × fw

= 1.005 × 23.4 × 0.83 = 19.519 kJ/kg

Also,       w = (vw2 - vw1)u

= (v - 2vw1)u = (u - 2 × va1 sin a1)u

= (u - 2 × 165 sin 200)u = (u - 112.87)u

or  u2 - 112.87 u = 19519

u2 - 112.87u - 19519 = 0
or    
1
u= 112.87 ± (112.87)2 + 4 × 19519 
2  
= 207.1 m/s

a2 b1 a1
b2
vr2 vr1 v
a2 va1
vf2 vf1

vw1
vw2
u

Figure 15.22  Symmetrical velocity diagrams

M15_THERMAL ENGINEERING_SE_XXXX_CH15.indd 823 6/16/2018 12:47:55 PM


824 Chapter 15

pdm N
Now, u=
60
207.1 × 60
or N= = 21975 rpm
p × 0.18
Area across flow,        Af = pdmh
p1 1.01 × 105
Density of air, r1 = = = 1.222 kg/m3
RT1 287 × 288
Velocity of flow, vf 1 = va1 cos a1 = 165 cos 20° = 155 m/s
m a = r1Af vf 1
   3 = 1.222 × p × 0.18 × h × 155
   h = 0.028 m or 2.8 cm
Length of last stage rotor blade, h = 2.8 cm.

Example 15.17
An axial-flow compressor employed in gas turbine plant delivers air at the rate of 300 kg/s and devel-
ops a total pressure ratio of 20. The inlet stagnation conditions are 1 bar and 300 K and the blade
speed is kept at 200 m/s to minimise noise generation. The stage degree of reaction at the mean blade
height is 50%. The axial velocity of flow is 160 m/s. The work done factor is 0.88. The hub-tip diam-
eter ratio is 0.8. Assume actual temperature rise in each stage. Show the process on T-s diagram and
draw velocity diagram. Find (a) all the fluid angles of the first stage and (b) The hub and tip diameter
including blade height. Take R = 0.287 kJ/kg-K and cp = 1.005 kJ/kg-K. [IES, 2010]

Solution
.
Given: ma = 300 kg/s, rp = 20, T01 = 300 K, p01 = 1 bar, hisen = 0.87, z = 18, u = 200 m/s, Rd = 0.5,
fw = 0.88, hub-tip ratio = 0.8, R = 0.287 kJ/kg-K, cp = 1.005 kJ/kg-K, vf = 160 m/s
The T − s and velocity diagrams are shown in Fig. 15.23.
g −1 1.1−1
g
(a) Toz = T01 ( rp ) = 300( 20) 1.1 = 706.06 K

T u
vw2
poz vw1

oz′
oz vf2
p01 vf1
va1 vr2
va2
a1 b2
o1 b1 a2
vr1
s
(a) (b)

Figure 15.23  (a) T-s diagram, (b) Velocity diagrams

M15_THERMAL ENGINEERING_SE_XXXX_CH15.indd 824 6/16/2018 12:48:01 PM


Axial Flow Air Compressors 825

Toz − T01
hisen =
Toz ′ − T01
706.06 − 300
or 0.87 =
T02′ − 300
Toz′ = 766.7 K
vw2 = vf tan a2, vw1 = vf tan a1
For   Rd = 0.5, a2 = b1, a1 = b2
Work consumed by compressor per kg of air
= u (vw2 - vw1) fw × z
= cp (Toz′ - T01)
200 (tan a2 - tan a1) 160 × 0.88 × 18 = 1.005 (766.7 - 300)
tan a2 - tan a1 = 9.25 × 10-4
or       tan b1 - tan a1 = 9.25 × 10-4
u 200
Also,   tan a1 + tan b1 = = = 1.25
v f 160
Adding the above two equations, we get
2 tan b1 = 1.250925
or b1 = 32.02° = a2
or tan a1 = 1.25 - 0.62546 = 0.62453
a1 = 31.98° = b2
(b) Density of approaching air to first stage.
p 1 × 105
r1 = 01 = = 1.1614 kg/m3
RT01 287 × 300
Applying continuity equation,
.
r1Af vf = ma
1.1614 × p r12 [1 - (0.8)2] × 160 = 300
Tip radius, r1 = 1.194 m
Hub radius, rh = 0.8 r1 = 0.8 × 1.194 = 0.9558 m
Blade height of first stage,
h = r1 - rh = 1.194 - 0.9558 = 0.238 m.

Example 15.18
Air enters an axial flow compressor at 25°C and undergoes a pressure increase of six times that at
inlet. The mean velocity of rotor blades is 200 m/s. The inlet and exit angles of both the moving and
fixed blades are 45° and 20° respectively. The degree of reaction at the mean diameter is 50 percent
and there are 10 stages in the compressor. The isentropic efficiency of compressor is 85% and axial
velocity is constant throughout. Calculate the work done factor of the compressor.
Solution
Given: T1 = 273 + 25 = 298 K, rp = 6, um = 200 m/s, b1 = 45°, b2 = 20°, Rd = 50%, z = 10, (hc)isen =
85%, vf 1 = vf 2 = vf
For Rd = 50%, a2 = b1 and a1 = b2
Also vr1 = va2 and vr2 = va1

M15_THERMAL ENGINEERING_SE_XXXX_CH15.indd 825 6/16/2018 12:48:04 PM


826 Chapter 15

um
vw2 vw1
vw1

vf vf
va1 vr2
a1 vr1 b2
b1
va2 a2

Figure 15.24   Velocity triangles


From Fig 15.24, we have v cos a1
va1 cos a1 = va2 cos a2 or va2 = a1
cos a 2
um = vr2 sin b2 + va2 sin a2
= va1 sin b2 + va1 cos a1 tan a2
200 = va1 [sin 20° + cos 20° tan 45°]
= 1.2817 va1
or va1 = 156 m/s
cos 20o
va2 = 156 × = 207.3 m/s
cos 45o
vw1 = va1 sin a1 = 156 sin 20° = 53.36 m/s
vw2 = va2 sin a2 = 207.3 sin 45° = 146.58 m/s
∆vw = vw2 - vw1 = 146.58 - 53.36 = 93.22 m/s
g −1 0.4
T2
= rp g = (6) 1.4 = 1.6685
T1
 T2 = 298 × 1.6685 = 497.2 K
T −T
(hc)isen = 2 1
T2′ − T1
497.2 − 298
or ∆T = T2Ä - T1 = = 234.35 K
0.85
Work done on compressor, wc = cp × ∆T = 1.005 × 234.35 = 235.5 kJ/kg
Power input to compressor = ∆vw × um × fw × z
= 93.22 × 200 × fw × 10 = 186, 440 × fw
235.5 × 103
Work done factor, fw = = 1.263
186, 440

Example 15.19
An axial flow compressor compresses the air up to overall stagnation pressure 10 bar with overall
stagnation isentropic efficiency of 88%. The inlet stagnation pressure and temperature are 1 bar
and 300 K. The mean blade speed is 200 m/s. The degree of reaction is 0.5 at the mean radius
with air angles of 30° and 10° at rotor inlet and outlet with axial direction respectively. The work

M15_THERMAL ENGINEERING_SE_XXXX_CH15.indd 826 6/16/2018 12:48:52 PM


Axial Flow Air Compressors 827

done factor is 0.88. The hub-tip ratio is 0.4. The mass flow rate is 50 kg/s. Show the compression
process on T-s diagram and draw the inlet and outlet velocity triangles. Find (a) the stagnation poly-
tropic efficiency, (b) the number of stages and (c) the blade height in first stage of the compressor.
 [IAS, 2006]
Solution
Given: p1 = 1 bar, T01 = 300 K, poz = 10 bar, hisen = 88%, um = 200 m/s,
r
Rd = 0.5, b1 = 30°, b2 = 10°, fw = 0.88, h = 0.4, m. = 50 kg/s,
r1
The T-s diagram is shown in Fig. 15.25(a)
g −1 0.4
Toz = T01  poz  g = 300  10  1.4 = 579.2 K
   
 p01  1
Toz − T01
hisen = ,
Toz′ − T01
T − T01 579.2 − 300
Toz′ = T01 + oz = 300 + = 617.3 K
hisen 0.88
The velocity triangles are shown in Fig.15.25(b).
g −1
p  g 0.4
ln  oz 
 p01  ln(10) .4
1 0.65788
(a) Stagnation polytropic efficiency, hp = = =
T′   617.3  0.72157
ln  oz  ln 
T   300 
01
= 0.9117 or 91.17%
(b) For Rd = 0.5
a1 = b2 = 10°, a2 = b1 = 30°
From inlet velocity triangle, we have
u
= tan a1 + tan b1 = tan 10° + tan 30° = 0.7537
vf
200
vf = = 265.4 m/s
0.7537
u
T oz′ poz vw2
vw1
oz pz
v2az
2cp z′

p01
b2
o1 p1 vf
vf
v2a1
va1 a1 b1 a2 vr2
2cp vr1
1
s vr2

(a) (b)

Figure 15.25  (a) T-s diagram, (b) Velocity triangles

M15_THERMAL ENGINEERING_SE_XXXX_CH15.indd 827 6/16/2018 12:48:57 PM


828 Chapter 15

vw2 = vf tan a2 = 265.4 tan 30° = 153.2 m/s


vw1 = vf tan a1 = 265.4 tan 10° = 46.8 m/s
Work consumed per stage = fw u (vw2 - vw1)
= 0.88 × 200 (153.2 - 46.8) × 10-3 = 18.726 kJ/kg
Total work consumed by compressor = cp ( Toz′ - T01)
= 1.005 (617.3 - 300) = 318.89 kJ/kg.
318.89
Number of stages, z = = 17
18.726
vf 265.4
(c) va1 = = = 269.5 m/s
cos a1 cos 10o
va21 ( 269.5)2
Static temperature, T1 = T01 - = 300 - = 263.8 K
2c p 2 × 1005
g
p1  T1  g −1
= ,
p01  T01 
1.4
 263.8  0.4
p1 = 1 ×  = 0.638 bar
 300 
p1 0.638 × 105
r1 = = = 0.843 kg/m3
RT1 287 × 263.8
m. = r1 A1 vf
50 = 0.843 × p r12 [1 - (0.4)2] × 265.4
r1 = 0.291 m or 291 mm
rh = 0.4 r1 = 0.4 × 291 = 116.4 mm

Example 15.20
An air compressor has eight stages of equal pressure ratio 1.35. The flow rate through the compres-
sor and its overall efficiency are 50 kg/s and 82 per cent respectively. If the condition of air at entry
are 1.0 bar and 40°C, determine:
(a) the state of air at the compressor exit,
(b) polytropic or small stage efficiency,
(c) efficiency of each stage,
(d) power required to drive the compressor assuming overall efficiency of the drive as 90%
Take cp = 1.005 kJ/kg K and g = 1.4. [IAS, 2005]
Solution
Given: z = 8, (rp)st = 1.35, m. = 50 kg/s, h0 = 82%, p1 = 1.0 bar, T1 = 313 K, hmech = 90%, cp = 1.005
kJ/kg.K, g  = 1.4
ln rp
(a) z =
ln ( rp ) st
ln rp
8= ,
ln 1.35
rp = 11.03 = overall pressure ratio

M15_THERMAL ENGINEERING_SE_XXXX_CH15.indd 828 6/16/2018 12:49:02 PM


Axial Flow Air Compressors 829

Exit pressure, pa = 1.0 × 11.03 = 11.03 bar


g −1 0.4
g
T9 = T1 ( rp ) = 313 × (11.03) 1.4 = 621.5 K
T9 − T1
h0 = ,
T9′ − T1
621.5 − 313
0.82 = ,
T9′ -313
T9′ = 689.2 K or 416.2°C
g −1
p  g 0.4
ln  9 
 p1  ln (11.03) 1.4
(b) Polytropic efficiency, hp = = = 0.869 or 86.9%
 T ′  689.2 
ln  9  ln 
 T1′   313 

 g − 1  n 
(c) hp =   
 g   n − 1
 1.4 − 1  n 
0.869 =  ,
 1.4   n − 1
n
= 3.0415
n −1
3.0415 n - 3.0415 = n
or n = 1.49
g −1
T2
T1
= rp ( )g
0.4
or     T2 = 313 (1.35) .4
1 = 341 K
n −1
T2′
T1
= rp( ) n

1
or      T2′ = 313 (1.35) 3. 0415 = 345.46 K
T −T 341 − 313
Isentropic efficiency of each stage = 2 1 = = 0.8626 or 86.26%
T2′ − T1 345.46 − 313

(d) Power required to drive the compressor, P = ṁ × cp T9′ − T1


h mech
50 × 1.005(689.2 − 313)
 = = 21004 kW
0.9
Example 15.21
A small compressor has the following data:
Air flow rate = 1.5778 kg/s
Pressure ratio = 1.6

M15_THERMAL ENGINEERING_SE_XXXX_CH15.indd 829 6/16/2018 12:49:08 PM


830 Chapter 15

Rotational speed = 54,000 rpm


Efficiency = 85%
State of air at entry, p0p = 1.008 bar, T0p = 300 K
cp for air = 1.009 kJ/kg K.
(a) Calculate the power required to drive this compressor.
(b) A geometrically similar compressor of three times the size is constructed. Determine, for this
compressor (i) mass flow rate, (ii) pressure ratio, (iii) speed, and (iv) the power required.
Assume same entry conditions and efficiency for the two compressors and also assume kinematic
and dynamic similarities between the two machines. [IAS, 2005]

Solution
Given: m a = 1.5778 kg/s, rp = 1.6, N = 54,000 rpm, h = 85%, p01 = 1.008 bar, T01 = 300 K, cp = 1.009
kJ/kg.K, D2 = 3D1
p02
= 1.6, p02 = 1.6 × 1.008 = 1.6128 bar
p01
g −1 0.4
T02 g
= ( rp ) = (1.6) 1.4 = 1.1437
T01
T02 = 300 × 1.1437 = 343.1 K
T02 − T01
h=
T02′ − T01
343.1 − 300
0.85 =
T02′ − 300
T02′ = 350.725 K
(a) Power required = m a cp( T02′ - T01)  = 1.5778 × 1.009 (350.725 - 300) = 80.75 kW
G T
(b) = const., G = mass flow rate.
pD 2

G1 T01 G2 T02
(i) =
p01D12 p02 D22

p 300
G2 = G1 T01 × 02 × 32 = 1.5778 × × 1.6 × 9 = 21.24 kg/s
T02 p01 343.1

p3 G1N1 G2 N 2
(ii) Now = =
p1 p1 p2

p2 G2 N 2 21.24 19250
= × = × = 4.8
p1 G1 N1 1.5778 54000

M15_THERMAL ENGINEERING_SE_XXXX_CH15.indd 830 6/16/2018 12:49:23 PM


Axial Flow Air Compressors 831

DN D N T02 1 343.1
(iii) 1 1 = 2 2 , N2 = N1 D1 × = 54000 × × = 19250 rpm
T01 T02 D2 T01 3 300

p2 = 21.24 × 1.009 (350.725 - 300) = 1087.6 kW


(iv)

Example 15.22
Air at a temperature of 300 K enters a 10-stage axial flow compressor at the rate of 3.5 kg/s. The
pressure ratio is 6.0 and the isentropic efficiency is 90%. The process is adiabatic and the compres-
sor has symmetrical stages. The axial velocity of 120 m/s is uniform across the stages and the mean
blade speed is 200 m/s. Assume that the temperature change is same in each stage.
Determine the direction of the air at entry to and exit from the rotor and stator blades. Also find
the power given to the air.
For air, take cp = 1.005 kJ/kg.K and g = 1.4. [IAS, 2004]

Solution
Given: T1 = 300 K, z = 10, m a = 3.5 kg/s, rp = 6, hisen = 90%, vf = 120 m/s, u = 200 m/s, cp = 1.005
kJ/kg.K, g = 1.4
Symmetrical stages, same temperature change in each stage
The velocity diagrams are shown in Fig. 15.26
(∆T)stage = (∆T)overall/z
 T  g 
g −1

=  1   rp − 1
 zhisen   
 0.4 
 300   1.4 
=  6 − 1 = 22.284 K
10 × 0.9  
 
Increase in temperature per stage,

 uv f 
(∆T)stage =  [tan b1 - tan b2]
 c p 

u
vw2
vw1

vf
va1 b2
a1 vf
b1 a2 vr2
vr1
va2

Figure 15.26  Velocity triangles

M15_THERMAL ENGINEERING_SE_XXXX_CH15.indd 831 6/16/2018 12:49:37 PM


832 Chapter 15

22.284 × 1.005 × 103


tan b1 - tan b2 = = 0.93313
200 × 120
u 200
Also tan b1 + tan b2 = = = 1.6667
uf 120
tan b1 = 1.2999, b1 = 52.43°, b2 = 20.14°
Power supplied = z m a cp(∆T)stage
= 10 × 3.5 × 1.005 × 22.284
= 783.84 kW
Example 15.23
An axial flow compressor of 50% reaction blading has isentropic efficiency of 82%. It draws air
at 17°C and compresses in the pressure ratio of 4 : 1. The mean blade speed and flow velocity are
constant throughout the compressor. The inlet and outlet angles of blades are 15° and 45° respec-
tively (angles measured from axial direction). Blade speed = 180 m/s and work input factor = 0.84.
Calculate (a) the flow velocity and (b) the number of stages. [IAS, 2002]

Solution
Given: Rd = 50%, hisen = 82%, T1 = 17 + 273 = 290 K, rp = 4:1, um = const, vf = const., b1 = 15°,
b2 = 45°, u = 180 m/s, fh = 0.84
For Rd = 50%, a1 = b2 = 45°, a2 = b1 = 15°
g −1 0.4
T2 = T1 ( r ) g = 290 ( 4) 1.4 = 430.9 K
p

vw1 u − vw1 u
(a) From Fig. 15.27, we get tan a1 = , tan b1 = = - tan a1
vf vf vf
u 180
vf = = = 142 m/s
tan a1 + tan b1 tan 45° + tan 15°
∴ Flow velocity, vf = 142 m/s
wisen = cp (T2 - T1) = 1.005(430.9 - 290) = 141.6 kJ/kg

u
vw2 vw1
vw1

vf
vf
va1 va2
b2 vr2
a1
b1 a2
vr1

Figure 15.27  Velocity triangles

M15_THERMAL ENGINEERING_SE_XXXX_CH15.indd 832 6/16/2018 12:49:41 PM


Axial Flow Air Compressors 833

wisen
hisen =
wact
141.6
wact = = 172.7 kJ/kg
0.82
2( vw 2 − vw1 )
Work input factor, fh =
u
0.84 × 180
∆vw = vw2 - vw1 = = 75.6 kJ/kg
2
(b) Work done per stage = u × ∆vw = 180 × 75.6 = 13.608 kJ/kg
172.7
Number of stages = ∼ 13
= 12.69 -
13.608

Summary for Quick Revision


1. In axial flow compressors, the air flows parallel to its axis throughout.
2. Axial flow compressors are particularly suitable for supercharging of internal combustion engines, gas tur-
bines, and turbojets.
3. Aerofoil theory:
1 2
(a) Lift force, FL = C L rvr A
2
1 2
(b) Drag force, FD = C D rvr A
2
1
4. Power input per stage, P = (F cos b1 + FD sin b1) u × 10-3 kW
2 L
 pT1
mc
=  rp( r −1)/ r − 1
hc  

p2 V r
where rp = , hc = compressor efficiency, m = kg/s,
p1 60

V = FAD in m3/min.
5. Velocity relations:
u u −u w1
(a) tan a1 = w1 , tan b1 =
u f1 u f1

u
6. = tan a1 + tan b1
u f1

u u −u w 2
(a) tan a2 = w 2 tan b2 =
uf2 uf2

u
7. = tan a2 + tan b2
vf 2

M15_THERMAL ENGINEERING_SE_XXXX_CH15.indd 833 6/16/2018 12:49:52 PM


834 Chapter 15

where a1 = exit angle from stator, b1 = inlet angle to rotor, a2 = inlet angle to stator and b2 = outlet angle
from rotor.
8. Work required per stage per kg of air, w = (vw2 - vw1)u = cp ( T02′ - T01)
  = vf (tan a2 - tan a1)u
  = [u - vf (tan a1 + tan b2]u
T02 − T01
9. Stage isentropic efficiency, hisen =
T02′ − T01
g g
p  T02  (g −1)  u( vw 2 − vw1 )  (g −1)
10. Stagnation pressure ratio, 02 =   = 1 + hisen 
p01  T01   c pT01 
Pressure rise in rotor blades
11. Degree of reaction, Rd =
Pressure rise in compressor stage
uf
  = (tan b1 + tan b2)
2u
12. For Rd = 50%, the blades are symmetrical, i.e.,
  a1 = b2 and a2 = b1
13. Isentropic pressure rise through a cascade,
r 2
(a)  (∆p)isen = u f (tan2 a1 + tan2 a2)
2
14. Polytropic efficiency: It is the small stage efficiency (hisen)stage, which is constant for all stages, and is
denoted by hp.
dT0  g − 1  n 
(a)  hp = =  
dT0′  g   n − 1
(g −1)
p  g
ln  02 
 p01 
(b)  =
T 
ln  02′ 
 T01′ 
Flow (or axial) velocity at inlet u f1
15. Flow coefficient, ff = =
Blade velocity u
1 1
(a)  = =
tan a 1 + tan b1 tan a 2 + tan b 2
Actual work done
16. Head or work coefficient, fh or fw =
Kinetic energy corresponding to mean periipheral velocity.
2(uw 2 − uw 1 ) 2(tan a 2 − tan a 1 )
(a)  = =
u tan a 2 + tan b 2
Isentropic work done
17. Pressure coefficient, fp =
Kinetic energy corresponding to periphheral velocity
c p ⋅ ∆Tisen
(a)  = = hisen×fh
 u2 
 2
 

M15_THERMAL ENGINEERING_SE_XXXX_CH15.indd 834 6/16/2018 12:50:01 PM


Axial Flow Air Compressors 835

∆T0
18. Number of stages, z =
( dT0 )stage
p02 p03 p0( z +1)
(a) For same pressure ratio per stage, (rp) st = = = .... =
p01 p02 p0 z

ln rp
19. z =
ln (rp ) st
20. The pulsating air flow phenomenon is known as surging. It causes overheating and stress reversal in the
blades which damages the blades.
21. The maximum mass flow rate possible in a compressor is known as choking.
22. Stalling is a phenomenon in which the flow breaks away from the aerofoil blading. It occurs due to lesser
flow rate than the designed value or due to non-uniformity of blade profile.

Multiple-choice Questions
1. A multistage compressor is to be designed for a (a) suppression of separation of flow on the blade
given flow rate pressure ratio. If the compressor (b) choking of the flow
consists of axial flow stages followed by centrifu- (c) separation of flow on the pressure side of the
gal instead of only axial flow stages, then the blade
(a) overall diameter would be decreased (d) separation of flow on the suction side of the
(b) overall diameter would be increased blade
(c) axial length of the compressor would be 6. Which one of the following is the correct expres-
increased sion for the degree of reaction for an axial-flow
(d) axial length of the compressor would be air compressor?
decreased
Work input to the rotor
2. What is the ratio of the isentropic work to Euler’s (a)
work known as? Work input to thestage
(a) Pressure coefficient Change of enthalpy in the rotor
(b) Slip factor (b)
Change of enthalpy in the sttage
(c) Work factor
(d) Degree of reaction Pressure rise in the rotor
(c)
3. In air-craft gas turbines, the axial flow compres- Pressure rise in the stage
sor is preferred because Isentropic work
(a) of high pressure rise (d)
Actual work
(b) it is stall-free
(c) of low frontal area 7. Which one of the following types of compressors
(d) of higher thrust is mostly used for supercharging of IC engines?
4. In axial flow compressor, exit flow angle devia- (a) Radial flow compressor
tion from the blade angle is a function of (b) Axial flow compressor
(a) blade camber (c) Roots blower
(b) space-chord ratio (d) Reciprocating compressor
(c) both blade camber and space-chord ratio 8. Phenomenon of choking in compressor means
(d) blade camber and incidence angle (a) no flow of air
5. High positive incidence in an axial compressor (b) fixed mass flow rate regardless of pressure
blade row leads to ratio

M15_THERMAL ENGINEERING_SE_XXXX_CH15.indd 835 6/16/2018 12:50:06 PM


836 Chapter 15

(c) reducing mass flow rate with increase in pres-


sure ratio
(d) increased inclination of the chord with air v1 vr1 v2 ur2
steam
9. Degree of reaction in an axial compressor is u1 u2
defined as the ratio of static enthalpy rise in the
(a) rotor to static enthalpy rise in the stator Figure 15.28
(b) stator to static enthalpy rise in the rotor
(c) rotor to static enthalpy rise in the stage The turbomachine is
(d) stator to static enthalpy rise in the stage (a) an axial compressor with radial blades
(b) a radial compressor with radial blades
10. The usual assumption in elementary compressor
(c) a radial compressor with curved blades
cascade theory is that
(d) an axial compressor with forward curved blades
(a) axial velocity through the cascade changes
(b) for elementary compressor cascade theory, 14. The turbomachine used to circulate refrigerant in
the pressure rise across the cascade is given large refrigeration plant is
by equation of state (a) A centrifugal compressor
(c) axial velocity through the cascade does not (b) A radial turbine
change (c) An axial compressor
(d) with no change in axial velocity between inlet (d) An axial turbine
and outlet, the velocity diagram is formed 15. The energy transfer process is
11. Match List 1 with List II (pertaining to blower (a) continuous in a reciprocating compressor and
performance) and select the correct answer using intermittent in an axial compressor
the codes given below the Lists: (b) continuous in an axial compressor and inter-
mittent in a reciprocating compressor
List I List II (c) continuous in both reciprocating and axial
compressors.
A. Slip 1. Reduction of whirl velocity
(d) intermittent in both reciprocating and axial
B. Stall 2. Fixed mass flow rate compressors
regardless of pressure ratio
16. In an axial flow compressor stage, air enters and
C. Choking 3. Flow separation leaves the stage axially. If the whirl component of
4.  Flow area seperation the air leaving the rotor is half the mean periph-
eral velocity of the rotor blades, then the degree
Codes: of reaction will be
A B C (a) 1.00 (b) 0.75
(a) 4 3 2 (c) 0.50 (d) 0.25
(b) 1 3 2 17. If an axial flow compressor is designed for a con-
(c) 4 1 3 stant velocity through all stages, the area of the
(d) 2 3 4 annulus of the succeeding stages will
12. The critical value of Mach number for a subsonic (a) remain the same
airfoil is associated with a sharp increase in drag (b) progressively decrease
due to local shock formation and its interaction (c) progressively increase
with the boundary layer. A typical value of this (d) depend upon the number of stages
critical Mach number is of the order of 18. If the static temperature rise in the rotor and sta-
(a) 0.4 to 0.5 (b) 0.75 to 0.85 tor, respectively, are ∆TA and ∆TB, the degree of
(c) 1.1 to 1.3 (d) 1.5 to 2.0 reaction in an axial flow compressor is given by
13. The inlet and exit velocity diagrams of a turboma- ∆TA ∆TA
(a) (b)
chine rotor are shown in Fig. 15.28. ∆TB ∆TA + ∆TB

M15_THERMAL ENGINEERING_SE_XXXX_CH15.indd 836 6/16/2018 12:50:08 PM


Axial Flow Air Compressors 837

∆TB ∆T 22. Compared to axial compressors, centrifugal com-


(c) (d) B . pressors are more suitable for
∆TA + ∆TB ∆TA
(a) high head, low flow rate
19. Which one of the following pairs of features and (b) low head, low flow rate
compressors type is NOT correctly matched? (c) low head, high flow rate
(a) Intake and delivery ports compression is (d) high head, high flow rate
attained by back flow and internal compres- 23. Stalling of blades in axial-flow compressor is the
sion cylindrical rotor set to eccentric casing: phenomenon of
Vane compressor (a) air stream blocking the passage
(b) Intermittent discharge requires receiver, pro- (b) motion of air at sonic velocity
duces high pressure, slow speed and lubrica- (c) unsteady, periodic and reversed flow
tion problems : Reciprocating compressor (d) air stream not able to follow the blade contour
(c) Continuous flow, radial flow, handles large 24. In an axial flow compressor:
volume, much higher speed and fitted into a1 = exit angle from stator, b1 = inlet angle to
design of aero-engines : Centrifugal com- rotor, a2 = inlet angle to stator, and b2 = outlet
pressor angle from rotor.
(d) Successive pressure drops through contract- The condition to have a 50% degree of reaction is
ing passages, blades are formed from a num- (a) a1 = b2
ber of circular arcs, axial flow : Axial flow (b) a2 = b1
compressor (c) a1 = b2 and b1 = a2
20. In an axial flow compressor design, velocity dia- (d) a1 = a2 and B1 = b2
grams are constructed from the experimental data 25. Consider the following statements with reference
of aerofoil cascades. Which one of the following to supercharging of IC engines:
statements in this regard is correct? (i) Reciprocating compressors are invariably
(a) Incidence angle of the approaching air is used for high degree of super charging
measured from the trailing edge of the blade (ii) Rotary compressors like roots blowers are
(b) d is the deviation angle between the angle of quite suitable for low degree of supercharg-
incidence and tangent to the camber line ing
(c) The deflection e of the gas stream while pass- (iii) Axial flow compressors are most commonly
ing through the cascade is given by e = a1 - a2 employed for supercharging diesel engines
(d) e is the sum of the angle of incidence and used in heavy duty transport vehicles
camber less any deviation angle, i.e., e = i + (iv)  Centrifugal compressors are used for tur-
q-d bo-charging
21. Consider the following statements regarding the Which of the statements given above are correct?
axial flow in an air compressor: (a) (i) and (ii) (b) (ii) and (iii)
(i) Surging is a local phenomenon while stalling (c) (i) and (iv) (d) (ii) and (iv)
affects the entire compressor. 26. While flowing through the rotor blades in an axial
(ii) Stalling is a local phenomenon while surging flow air compressor, the relative velocity of air
affects the entire compressor. (a) continuously decreases
(iii) The pressure ratio of an axial compressor (b) continuously increases
stage is smaller than that of a centrifugal (c) first increases and then decreases
compressor stage (d) first decreases and then increases
Of these statements 27. In an axial flow compressor, when the degree of
(a) (i), (ii) and (iii) are correct reaction is 50%, it implies that
(b) (i) and (ii) are correct (a) work done in compression will be the least
(c) (ii) and (iii) correct (b) 50% stages of the compressor will be ineffective
(d) (i) and (iii) correct (c) pressure after compression will be optimum
(d) the compressor will have symmetrical blades

M15_THERMAL ENGINEERING_SE_XXXX_CH15.indd 837 6/16/2018 12:50:10 PM


838 Chapter 15

28. For a multistage compressor, the polytropic effi- (c) constant throughout for all the stages
ciency is (d) a direct consequence of the pressure ratio
(a) the efficiency of all stages combined together.
(b) the efficiency of one stage.

Explanatory Notes

u f 
16. (d) Degree of reaction, Rd =   (tan b1 + b2)
 2u 

uf 1 1 uf
= , tan b1 = tan b 2 = =
u 2 2 u
1  1 1 1
Rd =  +  = = 0.25
4  2 2 4

Review Questions
1. How do axial flow compressors differ from centrifugal compressors?
2. Explain the working principle of axial flow compressors.
3. Define lift and drag coefficients.
4. Define degree of reaction and write down the mathematical expression for it.
5. For 50% degree of reaction, what is the condition on blade angles?
6. Write the expression for pressure rise in isentropic flow through a cascade.
7. Define polytropic efficiency and write down the mathematical expression for it.
8. Define flow coefficient and work coefficient.
9. Define deflection coefficient and pressure coefficient.
10. Differentiate between choking and stalling.
11. List the various losses in an axial flow compressors.
12. List the main applications of axial flow compressors.
13. List the merits and demerits of axial flow compressors.
14. Draw the efficiency and power consumption curves for the axial flow compressor.

Exercises

15.1 An axial flow air compressor of 50% degree of reaction has blades with inlet and outlet blade angles of 40°
and 15°, respectively. The pressure ratio is 5 : 1 with an overall isentropic efficiency of 85% when the air inlet
temperature is 30°C. The blade speed and axial velocity are constant throughout the compressor. Assuming
a blade speed of 210 m/s, find the number of stages required when the work coefficient is 0.9 for all stages.
15.2 In an axial flow air compressor, the overall stagnation pressure ratio is 4.5 with overall stagnation isentropic
efficiency 85%. The inlet stagnation pressure and temperature are 1 bar and 310 K. The mean blade speed
is 180 m/s and degree of reaction is 0.5 at the mean radius. The relative air angles are 10° and 30° at rotor
inlet and outlet, respectively. The work coefficient is 0.9. Calculate (a) the stagnation polytropic efficiency,
(b) the number of stages, (c) the inlet temperature and pressure, and (d) the blade height in the first stage if
the hub-tip ratio is 0.45 and mass flow rate is 20 kg/s.

M15_THERMAL ENGINEERING_SE_XXXX_CH15.indd 838 6/16/2018 12:50:12 PM


Axial Flow Air Compressors 839

15.3  A multi-stage axial flow compressor delivers 19 kg/s of air. The inlet stagnation condition is 1 bar and 25°C.
The power consumed by the compressor is 4400 kW. Calculate (a) the delivery pressure, (b) the number of
stages, and (c) the overall isentropic efficiency of the compressor.
   Assume that temperature rise in the first stage is 21°C. The polytropic efficiency of compressor is 90% and
stagnation pressure ratio is constant.
15.4  An axial flow compressor compresses the air up to overall stagnation pressure of 10 bar with overall stag-
nation isentropic efficiency of 88%. The inlet stagnation pressure and temperature are 1 bar and 300 K. The
mean blade speed is 200 m/s. The degree of reaction is 0.5 at the mean radius with air angles of 30° and 10°
at rotor inlet and outlet with axial direction respectively. The work done factor is 0.88. The hub-tip ratio is
0.4. The mass flow rate is 50 kg/s. Draw the inlet and outlet velocity triangles and show the compression
process on T-s diagram. Find the following:
(a) The stagnation polytropic efficiency
(b) The number of stages
(c) The blade height in the first stage of the compressor
15.5  An axial flow compressor provides a total head pressure ratio of 4 : 1 with an overall total head isentropic
efficiency of 85%, when the inlet total head temperature is 290 K. The compressor is designed for 50%
reaction with inlet and outlet air angles from the rotor blades of 45° and 10°, respectively. The mean blade
speed and axial velocity are constant throughout the compressor. Assuming a value of 201.16 m/s for the
blade speed and a work done factor of 0.86, find the number of stages required. What is the inlet Mach
number relative to the rotor at the mean blade height of the first stage?
15.6  Air at a temperature of 300 K enters a 10-stage axial flow compressor at the rate of 3.5 kg/s. The pressure
ratio is 6.0 and the isentropic efficiency is 90%. The compression process is adiabatic and the compressor
has symmetrical stages. The axial velocity is uniform across the stages and equals to 120 m/s and the mean
blade speed is 200 m/s. Assume that the temperature change is same in each stage. Determine the direction
of the air at entry and exit from the rotor and stator blades. Also, find the power given to the air. For air, take
cp = 1.005 kJ/kg.K and g = 1.4.
15.7  An axial flow compressor consists of a number of similar stages with equal work done per stage. The veloc-
ity of flow is uniform throughout the compressor. The data given is as follows:
Overall stagnation pressure ratio = 3.5 : 1
Stagnation inlet temperature = 60°C
Relative air angle at rotor inlet = 130°
Relative air angle at rotor outlet = 100°
Blade velocity = 185 m/s
Degree of reaction = 50%
Overall stagnation adiabatic efficiency = 87%
Calculate (a) the stagnation outlet temperature, and (b) the number of stages.

ANSWERS TO MULTIPLE-CHOICE QUESTIONS


1. (b) 2. (a) 3. (c) 4. (c) 5. (a)
6. (a) 7. (b) 8. (b) 9. (c) 10. (c)
11. (b) 12. (c) 13. (a) 14. (c) 15. (c)
16. (d) 17. (a) 18. (b) 19. (c) 20. (a)
21. (d) 22. (a) 23. (c) 24. (c) 25. (d)
26. (a) 27. (d) 28. (a)

M15_THERMAL ENGINEERING_SE_XXXX_CH15.indd 839 6/16/2018 12:50:12 PM


Chapter 16

Gas Turbines

16.1 ❐ INTRODUCTION
The gas turbine is the most satisfactory way of producing very large power in a self-contained and
compact unit. The gas turbine obtains its power by utilising the energy of a jet or burnt gases and air,
the velocity of the jet being absorbed as it flows over several rings of moving blades which are fixed to
a common shaft. The gas turbine requires an air compressor which is driven off its own shafting. This
absorbs a considerable proportion of the power produced and thus lowers the overall efficiency, which
is of the order of 20%–30%.

16.2 ❐ FIELDS OF APPLICATION OF GAS TURBINE


The following are the major fields of application of gas turbines:
1. Aviation: A gas turbine is used in aviation because it is self-contained, light weight not requiring
cooling water, and generally fits into the overall shape of the aircraft structure. It is also suitable
for turbo-jet and turbo-propeller engines.
2. Power generation: It is used for electric power generation because of its simplicity, lack of cool-
ing water, quick installation, and starting. It is also suitable for supercharging, road transport, and
railways.
3. Oil and gas industry: It is used in the oil and gas industry due to cheaper supply of fuel and low
installation cost.
4. Marine propulsion: It is used in marine propulsion due to its light weight and it does not require
cool water.

16.3 ❐ LIMITATIONS OF GAS TURBINES


The limitations of gas turbines are as follows:
1. They are not self-starting.
2. Low efficiency at part loads
3. Higher rotor speeds
4. Low overall plant efficiency

M16_THERMAL ENGINEERING_SE_XXXX_CH16-First part.indd 840 6/16/2018 12:04:06 PM


Gas Turbines 841

16.4 ❐ COMPARISON OF GAS TURBINES WITH IC


ENGINES
16.4.1 Advantages
1. Perfect balancing of the rotating parts is possible.
2. Mechanical efficiency is very high (≈95%) compared with 85% of IC engine because of its large
number of sliding parts.
3. Torque on the turbine shaft is continuous and uniform; therefore, flywheel is not required.
4. Work developed is more because expansion of gases up to atmospheric pressure is possible.
5. The weight of the gas turbine per kW of power developed is less, and therefore, they are superior
for use in aircraft.
6. The compression, combustion, and expansion take place in different units. Therefore, these units
can be designed, tested, and developed individually.
7. The gas turbine can be driven at much higher speeds.
8. The maximum pressure used in gas turbines is very low; therefore, its components can be made
lighter.
9. The lubrication and ignition systems are much simpler.
10. Cheaper fuels can be used in gas turbine.
11. The installation and maintenance costs are less.
12. The exhaust is free from smoke and is less polluting.

16.4.2 Disadvantages
1. The thermal efficiency of a simple gas turbine is low.
2. There is very poor thermal efficiency at part loads.
3. The temperature of gases supplied to the turbine is limited to 1100 K as there are no metals which
can be used for blades to sustain this temperature at high centrifugal stresses.
4. The manufacturing of blades is difficult and costly as Ni-Cr alloy is used.
5. The control of fuel is difficult with wide operating speeds and output.
6. A special cooling system is required for the blades.
7. The starting is more difficult than IC engines.
8. It produces five times exhaust gases than an IC engine for same output.
9. Speed reduction device is always required due to higher operating speed.
10. The life of blades and combustion chamber is low due to high temperature.

16.5 ❐ ADVANTAGES OF GAS TURBINES OVER STEAM


TURBINES
1. Boiler, along with accessories, is not required.
2. The capital and running cost is less.
3. Less space is required for the plant.
4. Starting the plant is more easy and quick.

M16_THERMAL ENGINEERING_SE_XXXX_CH16-First part.indd 841 6/16/2018 12:04:07 PM


842 Chapter 16

5. The weight of the plant per kW generated is less.


6. Control of plant is much easier.
7. The plant can be installed anywhere as cooling water is not required.
8. Gas turbine plant is used as a best type for peak load plant.

16.6 ❐ CLASSIFICATION OF GAS TURBINES


Gas turbines may be classified as follows:
1. Constant pressure combustion (or continuous combustion) gas turbine
(a) Open cycle constant pressure gas turbine
(b) Closed cycle constant pressure gas turbine
2. Constant volume combustion (or explosion) gas turbine

16.6.1 Constant Pressure Combustion Gas Turbine


In this type, the fuel is burned at constant pressure and the cycle used is the Joule cycle. The turbine
belongs to the reaction type using oil fuel and is fitted with an axial flow rotary air compressor which
is coupled to the rotor shaft. The heat is supplied to the working fluid at constant pressure.
The open and closed cycle gas turbines are shown in Figs 16.1(a) and 16.1(b), respectively.
A simple open cycle constant pressure gas turbine consists of a compressor, combustion chamber,
and turbine. The air is taken from the atmosphere by the compressor and is compressed to the com-
bustion pressure of 1 to 4 atm. It is then forced into the combustion chamber. A part of this air is used
as combustion air for the oil which enters the burner, and the remainder is forced through the annular
space between the wall of the combustion chamber and the burner jacket. The air receives heat from
the burner jacket and also mixes with the products of combustion chamber and the burner jacket. This
raises the temperature and volume of the air. The use of a very large quantity of air in excess of the
combustion air prevents the temperature of the mixture from reaching values which are too high for
the metal of the rotor blades. It also prevents the burner from becoming too hot.
The high pressure and high temperature gases now enter the turbine and flow through the blade
rings, where they continuously expand, and the pressure energy is converted into kinetic energy, which
in turn is absorbed by the rotor. On leaving the turbine, the spent gases pass away to the exhaust.
The speed of the turbine is generated by controlling the fuel oil supply and the safety valve which
by-passes some of the mixture. If the pressure becomes too high, the net power developed by the tur-
bine is utilised in driving the generator but a considerable amount of the total power produced by the
rotor is absorbed in driving the compressor. The turbine is started by the electric starter motor.
In the closed cycle constant pressure gas turbine, the compressed fluid coming out from the com-
pressor is heated in the heater by an external source at constant pressure and the high pressure and
temperature fluid develop the work while passing through the turbine. The fluid coming out from the
turbine is cooled to its original temperature in the cooler using an external cooling source before pass-
ing into the compressor. Thus, the working fluid is continuously circulated in the closed cycle. Such a
turbine has now become obsolete.

M16_THERMAL ENGINEERING_SE_XXXX_CH16-First part.indd 842 6/16/2018 12:04:07 PM


Gas Turbines 843

Fuel
Spark plug
3

2
Combustion
chamber
C T
Turbine Generator
1 Compressor
4
Inlet air Exhaust to
from atmosphere atmosphere
(a)

2 Heater

C T
Turbine Generator
Compressor
1 4

Cooler
(b)

Figure 16.1  Constant pressure combustion gas turbine:


(a) Open cycle turbine, (b) Closed cycle turbine

16.6.2 Constant Volume Combustion Gas Turbine


The constant volume combustion gas turbine is shown in Fig. 16.2. Air is taken from the atmosphere
and is compressed by an axial flow compressor driven by a separate steam turbine. The compressed
air coming out of the compressor at about 3 bar is passed to the constant volume combustion cham-
ber. The fuel is injected into the combustion chamber in the current of air by a separate fuel pump.

Spark plug

Gas
turbine
Constant volume Nozzle Generator
Combustion chamber (Water cooled)
Exhaust
Boiler gases to
atmosphere

C
Electric
Axial Steam motor
compressor turbine
Inlet air
from atmosphere

Figure 16.2  Constant volume combustion gas turbine

M16_THERMAL ENGINEERING_SE_XXXX_CH16-First part.indd 843 6/16/2018 12:04:10 PM


844 Chapter 16

The air-fuel mixture formed in the combustion chamber is then ignited by means of a spark plug.
The combustion takes place at constant volume in the combustion chamber and the pressure inside
the chamber increases to nearly 12 bar. The high pressure and temperature gases are fed to the gas
turbine through the water-cooled nozzle. The gases passed through the turbine develop the useful
work. The exhaust gases are fed to the steam boiler to generate steam for the steam turbine driving
the compressor.
The major disadvantage of this turbine is the intermittent combustion which impairs its smooth
functioning. The absolute necessity of a separate steam turbine to recover the heat from exhaust gases
further discourages its use in practice.

16.7 ❐ COMPARISON OF OPEN AND CLOSED CYCLE


GAS TURBINES
The comparison of open and closed cycles gas turbines is given in Table 16.1.

Table 16.1  Comparison of open and closed cycles gas turbines

S. no. Particular Closed cycle Open cycle


 1. Cycle pressure Higher which results in a Lower which results in a bigger size
reduced size of plant of plant
 2. Corrosion and erosion of Avoided due to indirect More due to direct heating
blades heating.
 3. Filtration of incoming air Not required Poses a severe problem
 4. Density of working fluid High density reduces the size Low, which increases the size of
of compressor, turbine, and various components
heat exchanger
 5. Type of fuel used Solid fuel can be used more Fuel has to be a fluid, which is com-
economically paratively costlier
 6. Working fluid Any thermodynamically better Working fluid is restricted to air only
fuel than air can be used
 7. Maintenance costs Low High
 8. Uses Suitable for stationary plants Suitable for aircraft turbines
and marine power as it requires
abundance quantities of water
 9. Weight High, not suitable for moving Low, suitable for moving vehicles
vehicles also
10. Heat exchange coefficient Low High
11. Installation cost High Low
12. Response to changing Poor Good
load

M16_THERMAL ENGINEERING_SE_XXXX_CH16-First part.indd 844 6/16/2018 12:04:10 PM


Gas Turbines 845

16.8 ❐ POSITION OF GAS TURBINE IN THE POWER


INDUSTRY
Constant pressure gas turbines are now used for generating electricity, driving locomotives and aer-
oplane propellers, producing the air stream for wind tunnels, and jet propulsion of air craft. In some
types of aircraft, the exhaust gases from the main engines are used for driving a gas turbine which,
in turn, drives the supercharger. Gas turbines driven by exhaust gases are also used for supplying a
pressure charge to diesel engines.

Combustion
chamber

Heat exchanger

Compressor
Turbine

Figure 16.3  Gas turbine utilising exhaust heat

Overall efficiencies of constant pressure gas turbine plants vary between 18%–25%. This is based on
a turbine efficiency of 75%–78%. The efficiency of this type of turbine can be improved by using heat
from exhaust gases to preheat the compressed air before it enters the combustion chamber. The hot
mixture of air and burnt gases is then expanded through the turbine from which they are exhausted
into the heat exchanger. The available heat is now given to the high pressure air from the compressor.
Such a system is shown in Fig. 16.3.

16.9 ❐ THERMODYNAMICS OF CONSTANT PRESSURE


GAS TURBINE: BRAYTON CYCLE
16.9.1 Efficiency
The Brayton cycle is shown on p-V and T-s diagrams in Fig. 16.4. During this cycle, air is drawn
into the compressor at point 1 and then compressed isentropically along the process 1-2. Process 2-3
represents the burning of the oil at constant pressure p2. Process 3-4 is the isentropic expansion of the
gases through the turbine. Process 4-1 represents the exhausting and cooling of the gases at constant
pressure p1.
For 1 kg of air,
Compressor work, wc = w1-2 = h2 - h1 = cp (T2 - T1)
Heat supplied,     q = q2-3 = h3 - `h2 = cp (T3 - T2)
Turbine work,  wt = w3-4 = h3 - h4 = cp (T3 - T4)
Net work done,       wnet = wt - wc = cp [(T3 - T4) - (T2 - T1)]

T T T 
= c pT1  3 − 4 − 2 + 1
T
 1 T1 T1 

M16_THERMAL ENGINEERING_SE_XXXX_CH16-First part.indd 845 6/16/2018 12:04:11 PM


846 Chapter 16

p T p2
3
2 3
p2
2 p1

1
p1
1 4

v s
(a) (b)

Figure 16.4  Brayton cycle for constant pressure gas turbine:


(a) p-V diagrams, (b) T-s diagrams

p2 T
Let rp = be the pressure ratio and a = 3
p1 T1
For the adiabatic compression process 1-2, we have
(g −1)/g
T1 V2  p1 
= =
T2 V1  p2 

and for the adiabatic expansion process 3-4, we have


(g −1)/g
T4 V3  p4 
= =
T3 V4  p3 
But p1 = p4 and p2 = p3
T2 T3
∴    = = rp(g −1)/g = c
T1 T4

Wnet
Thermal efficiency,    h th =
q
  1 
c pT1 a 1 −  − (c − 1) 
  c  
=
c p (T3 − T2 )

 1
a 1 −  − (c − 1)
 c
=
T3 T2

T1 T1
 1
a 1 −  − (c − 1)
 c
=
a −c

M16_THERMAL ENGINEERING_SE_XXXX_CH16-First part.indd 846 6/16/2018 12:04:16 PM


Gas Turbines 847

Thermal efficiency, hth


0.8 g = 1.66

0.6 g = 1.4

0.4

0.2

0 5 10 15 20
Pressure ratio, rp

Figure 16.5  Variation of thermal efficiency with pressure ratio

 a − c
(a − c) − 
 c 
=
a −c
1 1
or           h th = 1 − = 1− (16.1)
c rp 1)/g
(g −

The variation of thermal efficiency with rp is shown in Fig. 16.5.

16.9.2 Specific Output


wnet T3 T4 T3 T2
Specific output = = − × − +1
c pT1 T1 T3 T1 T1

wnet a  1
or     = a − − c + 1 = a 1 −  − (c − 1)(16.2)
c pT1 c  c

d  wnet 
For maximum specific output,  =0
dc  c pT1 

 1
a  −1 = 0
 c 2 
2
a = c 2 =  rp(g −1)/g  (16.3)
or    
 
 g −1 
or           rp g  = a =c
 
  max

M16_THERMAL ENGINEERING_SE_XXXX_CH16-First part.indd 847 6/16/2018 12:04:21 PM


848 Chapter 16

a = T3 /T1

2.0 rp = p2 /p1

1.5 a =5

a =4
1.0

cpT1
wnet
a =3
0.5
a =2
0
0 5 10 15 20
rp

Figure 16.6  Variation of specific output with pressure ratio

g −1
T2 T3
Also, rp g = =
T1 T4

T T T
a = 2 × 3 = 2a
    
T1 T4 T4
T2 = T4(16.4)
or   
Hence, for maximum work output, the temperature after compression must be equal to the exhaust gas
temperature, which depends on the expansion ratio. The variation of specific output with rp is shown
in Fig. 16.6.

16.9.3  Maximum Work Output


Maximum work output,
(wnet)max = cp [(T3 - T4) - (T2 - T1)]

= cp [T3 + T1 - T4 - T2]

= cp [T3 + T1 - 2T2]( T4 = T2) ∴

= c p  T32 + T12 − 2 T2T4 


 
 T3 T2 
= c p  T32 + T12 − 2 T3T1   ∵ T = T 
  4 1
2 2
= c p  T3 − T1  = c p  Tmax − Tmin  (16.5)

M16_THERMAL ENGINEERING_SE_XXXX_CH16-First part.indd 848 6/16/2018 12:04:24 PM


Gas Turbines 849

16.9.4  Work Ratio


wnet wt − wc w
Work ratio,    Rw = = = 1− c
wt wt wt
g −1
g
T2 − T1 T1 ( rp ) − T1
= 1− = 1−
T3 − T4 T
T3 − g 3−1

rp g
 g −1 
 g  g −1
 T  rp − 1  = 1 − T1 ( r ) g = 1 − c (16.6)
= 1 −  1   
 T3  1 − 1 T3 p a
(g −1)/g 
 rp 

16.9.5 Optimum Pressure Ratio for Maximum Specific Work Output

wnet = wt - wc = cp (T3 - T4) -cp (T2 - T1)

 T  T 
= c pT3 1 − 4  − c pT1  2 − 1
 T3   T1 
 
   g −1 
1

= c pT3 1 −  − c pT1  rp g − 1
 g −1   
 r g   
 p 
dwnet
wnet will be maximum, when =0
drp

 −
 g −1 g −1 
d   g  g 
∴       c pT3 − c pT3 rp − c pT1rp + c pT1  = 0
drp
 
g −1
Let  m =
g d
∴    [ −T3 rp− m − T1rpm ] = 0
drp

mT3 rp− m −1 − mT1rpm −1 = 0


2(g −1)
T g
   3 = rp2m = rp
T1
g
 T  2(g −1)
∴                 ( rp )opt =  3 (16.7)
 T1 

M16_THERMAL ENGINEERING_SE_XXXX_CH16-First part.indd 849 6/16/2018 12:04:31 PM


850 Chapter 16

From Eq. (16.3), we have


g
 T  g −1
( rp ) max =  3
T 
1
Combining with Eq. (16.7), we get
    ( rp )opt = ( rp ) max (16.8)

16.10  ❐  CYCLE OPERATION WITH MACHINE EFFICIENCY


16.10.1  Maximum Pressure Ratio for Maximum Specific Work
It is not possible to achieve isentropic compression and expansion in the actual Brayton cycle because
of inevitable losses due to friction and turbulence in the compressor and turbine. Therefore, the tem-
perature at the end of compression and expansion are higher than that in an ideal cycle for the same
pressure ratio. The actual Brayton cycle is shown in Fig. 16.7.
T −T
Isentropic efficiency of compressor, hc = 2 1
T2′ − T1
T −T
Isentropic efficiency of turbine,   ht = 3 4′
Net specific work available, T3 − T4
wnet = wt - wc = cp (T3 - T4′) - cp (T2′ - T1)
(T2 − T1 )
= c pht (T3 − T4 ) − c p
hc
 T  c pT1  T2 
= c pht T3 1 − 4  − −1
 T3  hc  T1 
 
   g −1 
1  c pT1  g

= c pht T3 1 − − r − 1 (16.9)
 g −1  hc  p 
 r g   
 p 
For specific work output to be maximum for given temperature limits,
d
(w ) = 0
drp net

T
3 p2
2 2′
p1
4 4′
1
s

Figure 16.7  Actual Brayton cycle with machine efficiencies

M16_THERMAL ENGINEERING_SE_XXXX_CH16-First part.indd 850 6/16/2018 12:04:35 PM


Gas Turbines 851

 g −1  g −1
 g − 1 − g −1  T1  g − 1 g −1
ht T3  r − r =0
 g  p  hc  g  p
 
2g +1 −1
− −1 T
g
ht T3 rp − 1 rpg = 0
hc
 1−g 
2  T1
rp  g  =
hcht T3
g
  T   2(g −1)
rp = hcht  3  
  (16.10)
  T1  

16.10.2 Optimum Pressure Ratio for Maximum Cycle Thermal


Efficiency
q = cp (T3 - T2′) = cp (T3 - T2)
Heat supplied,   
wnet c p (T3 − T4′ ) − c p (T2′ − T1 )
Thermal efficiency, h th = =
q c p (T3 − T2 )
1
ht (T3 − T4 ) − (T − T )
hc 2 1
=
T3 − T2
 T  T T 
ht T3 1 − 4  − 1  2 − 1
=  T3  hc  T1 
T T 
T1  3 − 2 
 T1 T1 
g −1
T T
Now        2 = 3 = rp g
T1 T4
 
   g −1 
1 T1  g

ht T3 1 − g −1 −  r − 1
  hc  p 
 r g   
      h th =  p  (16.11)
 g −1 
T
T1  3 − rp g 
 T1 
 
The thermal efficiency of the cycle is dependent only on rp for the fixed values of T1 and T3. The con-
dition for the maximum value of thermal efficiency for given temperature limits is

M16_THERMAL ENGINEERING_SE_XXXX_CH16-First part.indd 851 6/16/2018 12:04:39 PM


852 Chapter 16

dh th
=0
drp
g −1
T3
Let c = rp g and a = ,
T1
 1 1
aht 1 −  − (c − 1)
 c  hc
then h th =
a −c
dh th  1 1   1 1 
(a − c) aht × −  + 1 aht 1 −  − (c − 1)  = 0
dc  c 2 hc    c  hc 
a 2ht a aht c ah c 1
− − + + aht − t − + =0
c 2 hc c hc c hc hc
a 2ht a 2aht 1
− − + aht + =0
c 2 hc c hc
a hcht − ac 2 − 2ahcht c + ahcht c 2 + c 2 = 0
2

c 2 (1 + ahcht − a ) − 2ahcht c + a 2hcht = 0

2ahcht ± [4a 2hc2ht2 − 4a 2hcht (1 + ahcht − a )]1/ 2


c=
2(1 + ahcht − a )

ahcht ± a [hc2ht2 − hcht (1 + ahcht − a )]1/ 2


=
(1 + ahcht − a )
  1 + ah h − a 1/ 2 
a 1 ± 1 − c t
 
  hcht  
= 
1 + ahcht − a
hcht
Taking -ve sign
  1 a  
1/ 2

a 1 − 1 − −a + 
  hcht hcht  
c=  
    
1/ 2
 
1/ 2
1 + 1 − 1 − a + a   1 − 1 − 1 − a + a  
  hcht hcht     hcht hcht  
  
g −1
a
rp g =
1/ 2
 1 
1+  (a − 1) − (a − 1)
hcht 

M16_THERMAL ENGINEERING_SE_XXXX_CH16-First part.indd 852 6/16/2018 12:04:45 PM


Gas Turbines 853

a
=
1/ 2
  1  
1 + (a − 1)  − 1 
  hcht  
g
  g −1
 T3 
 T1 
( rp )opt = 
   (16.12)
1/ 2
  T   1  
 
1 +  3 − 1  − 1  
  T1   hcht   

16.11  ❐  OPEN CYCLE CONSTANT PRESSURE GAS


TURBINE
The open cycle constant pressure gas turbine plant is shown diagrammatically in Fig. 16.8. The fuel
is burned with air coming out from the compressor into the combustion chamber. The products of
combustion from the combustion chamber are passed through the turbine and developed the required
power. Part of the power developed is used to run the compressor. The gases leaving the turbine are
exhausted to atmosphere.
Let m· a = mass rate of flow of air
m· f = mass rate of flow of fuel
Compressor work, W = m·  c (T - T )
c a pa 2 1
Turbine work, Wt = (m· a + m· f) cpg (T3 - T4′)
where cpa, cpg = specific heat of air and gases, respectively
Network Wnet = Wt - Wc = (m· a + m· f) cpg (T3 - T4′) - m· acpa (T2′ - T1)
Heat supplied, Q = m·  × C.V.
f
where CV = calorific value of fuel

Fuel
mf

C.C ma + mf
3 T
2 ma 3 p2

C T 2 2′
p1
1 4 4 4′
Air from Burned gases 1
atmosphere to atmosphere s

(a) (b)

Figure 16.8  Open cycle constant pressure gas turbine:


     (a) Schematic diagram, (b) T-s diagrams

M16_THERMAL ENGINEERING_SE_XXXX_CH16-First part.indd 853 6/16/2018 12:04:47 PM


854 Chapter 16

Wnet
Thermal efficiency,    h th =
Q
( m a + m f )c pg (T3 − T4′ ) − m a c pa (T2′ − T1 )
        = (16.13)
m f × C.V.
T −T
Now,    hc = 2 1
T2′ − T1
T2 − T1
∴  
T2′ − T1 =
hc
T3 − T4′
and     ht =
T3 − T4
∴  
T3 - T4′ = ht (T3 - T4)
m a c pa
( m a + m f ) c pg ht (T3 − T4 ) − (T2 − T1 )
hc
  h th =
Hence,   (16.14)
m f × C.V.

16.12  ❐  METHODS FOR IMPROVEMENT OF THERMAL


EFFICIENCY OF OPEN CYCLE CONSTANT
PRESSURE GAS TURBINE
The thermal efficiency of open cycle constant pressure gas turbine can be improved by the following
methods:
1. Regeneration
2. Intercooling
3. Reheating
4. Combination of above

16.12.1 Regeneration
In this method, the heat of the exhaust gases is used to heat the air coming out from the compressor,
thus reducing the mass of fuel supplied in the combustion chamber. A schematic diagram of such a
plant is shown in Fig. 16.9(a).
1.  Without Machine Efficiencies
The T-s diagram without machine efficiencies is shown in Fig. 16.9(b).
For 1 kg of air
Compressor work, wc = w1 - 2 = cp (T2 - T1)
Heat supplied, qs = q5 - 3 = cp (T3 – T5)
Turbine work,   wt = w3 - 4 =cp (T3 – T4)
Net work,            wnet = wt – wc = cp (T3 – T4) – cp (T2 – T1)

M16_THERMAL ENGINEERING_SE_XXXX_CH16-First part.indd 854 6/16/2018 12:04:56 PM


Gas Turbines 855

6 Regenerator
Burned gases
to atmosphere
c.c
4
2 5
3

C T
1 Wnet

Air from atmosphere


(a)
T T p2
3 3

5 p1
5 4′
2 2 2′
4 4
6 6

1 1
s s
(b) (c)

Figure 16.9 Open cycle gas turbine with regeneration: (a) Schematic diagram,
(b) T-s diagram without machine efficiencies,
(c) T-s diagram with machine efficiencies

wnet (T3 − T4 ) − (T2 − T1 )


  Thermal efficiency, h th = =
qs T3 − T5
  In an ideal regenerator, T4 = T5
 T2 
− 1
T2 − T1  T1   T1
    h th = 1 − = 1−    
T3 − T4  T3  1 − T4 
 T 
 3

 
1  c −1 c
= 1−
    = 1 − (16.15)
a 1
1 −  a
 c
g −1
T
   a = 3,
where        c= rp g
T1

wnet  T3 T4   T2 
=  −  −  − 1
c pT1  T1 T1   T1 

 1
= a 1 −  − (c − 1)(16.16)
  
 c

M16_THERMAL ENGINEERING_SE_XXXX_CH16-First part.indd 855 6/16/2018 12:05:00 PM


856 Chapter 16

1.0
...Simple cycle
0.8 a=5

Thermal efficiency, hth


0.6
a=4
0.4
a=3
0.2 a=2

0 5 10 15 20
Pressure ratio, rp

Figure 16.10  Variation of thermal efficiency with pressure


ratio for open cycle gas turbine with regeneration

The variation of thermal efficiency with pressure ratio is shown in Fig. 16.10. It may be seen that
T
thermal efficiency increases with an increase in a = 3 and decreases with increase in rp.
T1
2. Considering Machine Efficiencies
The T-s diagram is shown in Fig. 16.9(c).
wc = w1 – 2′
= cp (T2′ – T1)
(T2 − T1 )
= cp
hc
qs = q5 – 3
= cp (T3 – T5) = cp (T3 – T4′)
wt = w3 – 4′ = cp (T3 – T4′)
= cp ht (T3 – T4)
T2 − T1
(T3 − T4′ ) −
w − wc hc
h th = t =
qs T3 − T4′
T2 − T1
hc
= 1−
ht (T3 − T4 )
 
T1 1  c − 1  c
= 1− ⋅
     = 1− (16.17)
T3 hcht  1 ahcht
1− 
 c
cp
wnet = wt − wc = c pht (T3 − T4 ) − (T2 − T1 )
hc

M16_THERMAL ENGINEERING_SE_XXXX_CH16-First part.indd 856 6/16/2018 12:05:05 PM


Gas Turbines 857

T p2
3
p1
5
5′
4′
4
2 2′ 6 6′

1
s

Figure 16.11  Actual regeneration process

wnet  T3 T4  1  T2 
= ht  − −  − 1
c pT1  T1 T1  hc  T1 
 1 1
   = ht a 1 −  − (c − 1)(16.18)
 c  hc
3. Effectiveness of Regenerator
In actual practice, the rise in temperature from T2′ to T5 = T4′ is not possible. The actual tem-
perature of air will be T5′ < T5. Hence, actual thermal efficiency [see Fig. (16.11)] of the cycle
becomes,

(T3 − T4′ ) − (T2′ − T1 )


(h th )a = (16.19)
(T3 − T5′ )
The effectiveness of the regeneration is given by
Actual heat transfer to air
e=
Maximum heat transfer theoretiically possible

m a c pa (T5′ − T2′ )
       = (16.20)
m g c pg (T4′ − T2′ )
where m· a, m· g = mass rate of flow of air and exhaust gases, respectively
      cpa, cpg = specific heats of air and exhaust gases, respectively.

16.12.2 Intercooling
The work consumed by the compressor can be reduced by compressing the air in two stages and
incorporating the intercooler in between, as shown in Fig. 16.12(a). The corresponding T-s diagram is
shown in Fig. 16.12(b).
wc = cpa (T2′ – T1) + cpa (T4′ – T3)
c pa c pa
=
  (T2 − T1 ) + (T − T )
hc1 hc 2 4 3

M16_THERMAL ENGINEERING_SE_XXXX_CH16-First part.indd 857 6/16/2018 12:05:09 PM


858 Chapter 16

Fuel

3 5
C.C
2′ Intercooler 4′

C1 C2 T
Power
6′ output
1
Air inlet from To exhaust
atmosphere
(a)

T
p2
5

4′ pi
4

2 2′
p1
6′
3 6

1
s
(b)

Figure 16.12  Gas turbine plant with intercooling:


(a) Schematic diagram, (b) T-s diagram

c paT1  T2  c paT3  T4 
=  − 1 +  − 1
hc1  T1  hc 2  T3 
 g −1   g −1 
c paT1  pi  g  c paT3  p2  g 
=    − 1 +   − 1
hc1  p1   hc 2  pi  
   
T3 g −1
a = and m =
Let   
T1 g

c paT1  pi  m  c paaT1  p2  m 
   wc =  − 1 +  − 1
hc1  p1   hc 2  pi  
   
dwc
For minimum work, =0
dpi

c paT1  m  pi  m −1  c paaT1   p2  m −1 p2 
 + m − =0
hc1  p1  p1   hc 2   pi  p 2
   i 

M16_THERMAL ENGINEERING_SE_XXXX_CH16-First part.indd 858 6/16/2018 12:05:13 PM


Gas Turbines 859

c paT1 m c paaT1 1
or × × pim −1 + ( − mp2m ) × =0
hc1 p1m hc 2 pim +1

pim −1 ap2m
or =
hc1 p1m hc 2 pim +1

hc1
pi2m = a
or    × ( p1 p2 )m
hc 2
1/2m
 ah 
          pi =  c1 × ( p1 p2 )m 
 hc 2 
1/ 2 m
 ah 
      =  c1  × ( p1 p2 )1/ 2
 hc 2 
g
 T h  2(g −1)
      =  3 × c1  × p1 p2  (16.21)
 T1 hc 2 
For perfect cooling, T3 = T1 and if nc1 = nc2 = nc′ then

    pi = p1 p2 (16.22)

c paT1  pi  m  p2  m 
    ( wc ) min =  + − 2 
hc  p1   p  
i
 

 m m 
c paT1  p2  2  p2  2 
=    +   − 2
hc  p1   p1  
 

 g −1 
2c paT1  p2  2g 
   =    − 1 (16.23)
hc  p1  
 

(T5 − T6′ ) − [(T2′ − T1 ) + (T4′ − T3 )]


nth =
(T5 − T4′ )

(T5 − T6′ ) − (T2′ + T4′ − 2T1 )


If T3 = T1′ then      nth = (16.24)
T5 − T4′

M16_THERMAL ENGINEERING_SE_XXXX_CH16-First part.indd 859 6/16/2018 12:05:19 PM


860 Chapter 16

wnet = wt – wc = cp (T5 – T6′) – [cp (T2′ – T1) + cp (T4′ – T3)]

T − T T − T 
    = c pht (T5 − T6 ) − c p  2 1 + 4 3 
 hc1 hc 2 

For perfect intercooling, T3 = T1 and if hc1 = hc2 = hc, then

cp
wnet = c pht (T5 − T6 ) − (T2 + T4 − 2T1 )
hc

 T T   c pT1  T2 T4 
= c pT1ht  5 − 6   −  + − 2
 T1 T1   hc  T1 T1 

wnet  T T  1  T2 T4 
= ht  5 − 6  −  + − 2
c pT1  T1 T1  hc  T1 T1 

T T T
Let                        5 = b , 5 = b, 2 = b1 ,
T1 t6 T1

T4 T
= b2 = 4
T3 T1

wnet  b 1
= ht  b − b  − h [b1 + b2 − 2]
c pT1   c

m m m
p  p  p 
Now, b = b1b2 because b =  2  , b1 =  i  and b2 =  2 
 1
p  p1   p1 

wnet  1 1  b 
∴           
∴ = ht b  1 −  −  b1 + b − 2  (16.25)
c pT1  b  hc 1

d  wnet  1  b
  = − 1 − 2  = 0
db1  c pT1  hc  b1 

(w )  1 1  b 
     net max = ht b 1 −  −  b + − 2
c pT1  b  hc  b 

 1 2
            = ht b 1 −  − ( b − 1)(16.26)
 b  hc

M16_THERMAL ENGINEERING_SE_XXXX_CH16-First part.indd 860 6/16/2018 12:05:29 PM


Gas Turbines 861

( wnet ) max
(h th ) max =
qs
  1 2 
c pT1 ht b 1 −  − ( b − 1) 
  b  hc 
=
c p (T5 − T4′ ) = c p (T5 − T4 )

T5 T4 T5 T4
Now, − = − =b− b
T1 T1 T1 T3

 1 2
h t b  1 −  − ( b − 1)
 b  hc
∴   ∴  (h th ) max = (16.27)
b− b
 1
b 1 −  − 2( b − 1)
 b
                            = for ht = hc = 1
b− b
b b
(b − b) − − b+2 + b −2
= b = 1− b (16.28)
b− b b− b

16.12.3 Reheating
A considerable increase in power output can be achieved by expanding the gases in two stages with
a reheat combustion chamber between the two as shown in Fig. 16.13(a). The corresponding T-s dia-
gram is shown in Fig. 16.13(b).
      wt = cpg (T3 – T4′) + cpg (T5 – T6′)
= cpg ht1 (T3 – T4) + cpg ht2 (T5 – T6)
  

 T   T 
= c pg ht1 T3 1 − 4  + c pg ht 2 T5 1 − 6 
 T3   T5 

  p m   p m
= c pg ht1 T3 1 −  i   + c pght 2T5 1 −  1  
  p2     pi  
g −1  
where m =
g
T5
Let a = ,
T3
  p m   p m
then  wt = c pg ht1 T3 1 −  i   + c pg ht 2 a T3 1 −  1   (16.29)
  p2     pi  
   

M16_THERMAL ENGINEERING_SE_XXXX_CH16-First part.indd 861 6/16/2018 12:06:39 PM


862 Chapter 16

Fuel Fuel
3 5

2′ C.C. 4′ Reheat C.C.

C1 T1 T2
Power
output
1 6′
Air from To exhaust
atmosphere
(a)

T p2
3
pi
2
2′ 4′ 5
4 p1
6′
6

1
s
(b)

Figure 16.13  Gas turbine plant with reheating:


(a) Schematic diagram, (b) T-s diagram

dwt
For wt to be maximum, =0
dpi

m −1 m −1
 p  1 p   −p 
h t1 × ( − m ) ×  i  × + nt 2a ( − m)  1  ×  1 = 0
 p2  p2  pi  2
 pi 

h t1 ht 2ap1m
× pim −1 =
p2m pim +1
ht 2
pi2m = a ( p p )m
h t1 1 2
1 g
1
 a ht 2  2 m h T  2(g −1)
   pi =   × ( p1 p2 ) 2 =  t 2 × 5  × p1 p2 (16.30)
 h t1   ht1 T3 
If ht1 = ht2 = ht and T5 = T3, then
pi = p1 p2 (16.31)
  

M16_THERMAL ENGINEERING_SE_XXXX_CH16-First part.indd 862 6/16/2018 12:06:52 PM


Gas Turbines 863

[(T3 − T4′ ) + (T5 − T6′ )] − (T2′ − T1 )


h th =
(T3 − T2′ ) + (T5 − T4′ )

 m  m
( wt ) max   p1  2    p1  2 
= 1− + 1−
c pght T3   p2     p2  
   

 g −1 
  p1  2g 
= 2 1 −    (16.32)
  p2  
 

16.12.4  Reheat and Regenerative Cycle


This cycle is shown in Fig. 16.14.
wc = cp (T2 – T1)
 qs = q7 – 3 + q4 – 5 = cp (T3 – T7) + cp (T5 – T4)
wt = w3 – 4 + w5 – 6 = cp (T3 – T4) + cp (T5 – T6)

8 Fuel Fuel
To
6
exhaust
3
2 Heat 7 C.C. 4 Reheat C.C. 5
exchanger

C1 T1 T2
Power
output
1
Air from
atmosphere
(a)
p2
T
pi
3 5
p1
7
2 6
4
8

1
s
(b)

Figure 16.14  Gas turbine plant with reheat and regeneration:


(a) Schematic diagram, (b) T-s diagram

M16_THERMAL ENGINEERING_SE_XXXX_CH16-First part.indd 863 6/16/2018 12:06:56 PM


864 Chapter 16

Net work wnet = wt – wc


= cp [(T3 – T4) + (T5 – T6) – (T2 – T1)]

T T T T T 
= c pT1  3 − 4 + 5 − 6 − 2 + 1
T
 1 T1 T1 T1 T1 
p2 p p
Let rp = , r2 = 2 , r1 = i so that rp = r1r2
p1 pi p1

g −1 g −1 g −1
T T T5
c= rp g = 2 , c2 = r2 g = 3 , c1 = r1 g =
T1 T4 T6

So that c = c1c2
wnet T3 T4 T3 T5 T6 T5 T2
= − × × − × − +1
c pT1 T1 T3 T1 T1 T5 T1 T1

T3
Now, T5 = T3 and =a
T1

wnet a a
= 2a − − − c + 1
c pT1 c2 c1

a c2a
= 2a − − − c +1
c2 c

d  wnet 
For maximum output,  =0
dc2  c pT1 

a a
− =0
c22 c

c2 = c = c1
or  
( wnet ) max 2a
Then,    = 2a − − c +1
c pT1 c

 1 
      = 2a 1 −  − (c − 1)(16.33)
 c

M16_THERMAL ENGINEERING_SE_XXXX_CH16-First part.indd 864 6/16/2018 12:07:02 PM


Gas Turbines 865

 2a 
c pT1  2a − c + 1 − 
( wnet ) max  c
(h th ) max = =
qs c p [(T3 − T7 ) + (T5 − T4 )]

2a
2a − c + 1 −
c
 =
T3 T7 T5 T4
− + −
T1 T1 T1 T1
Now, T7 = T4 and T5 = T3

2a 2a
2a − c + 1 − 2a − c + 1 −
c c
(h th ) max = =
T T   a 
2 3 − 4  2 a − 
 T1 T1   c

c −1
or    (h th ) max = 1 − (16.34)
 1 
2a 1 − 
 c

16.12.5  Cycle with Intercooling and Regeneration


The schematic arrangement and T-s diagram of the cycle are shown in Fig. 16.15.
wc = w1 – 2 + w3 – 4 = cp [(T2 – T1) + (T4 – T3)]
qs = q7 – 5 = cp (T5 – T7)
wt = w5 – 6 = cp (T5 – T6)
wnet = wt – wc = cp [(T5 – T6) – (T2 – T1) – (T4 – T3)]

 T T   T  T T 
= c pT1  5 − 6  −  2 − 1 −  4 − 3  
 T1 T1   T1   T1 T1  

T5 T T T
Let = a , 5 = c, 2 = c1 , 4 = c2 so that c = c1c2 and T3 = T1
T1 T6 T1 T3

wnet  a
=  a −  − (c1 − 1) − (c2 − 1)
c pT1  c

 1 c 
     = a 1 −  − (c1 − 1) −  − 1
 c  c1 

M16_THERMAL ENGINEERING_SE_XXXX_CH16-First part.indd 865 6/16/2018 12:07:08 PM


866 Chapter 16

8 Fuel
To
exhaust
6

7 5
2 Intercooler 3 4 Heat Reheat C.C
exchanger

L.P H.P T
Power
Compressor Compressor output
1
Air from
atmosphere (a)
5 p2
T
p1
7
6
pi
4 2
8

3 1

s
(b)

Figure 16.15 Gas turbine plant with inter cooler and regeneration:


(a) Schematic diagram, (b) T-s diagram

d  wnet  c
  = −1 + 2 = 0
dc1  c pT1  c1

or c1 = c = c2
( wnet ) max  1
= a 1 −  − ( c − 1) − ( c − 1)
c pT1  c

 1
= a 1 −  − 2( c − 1)
 c
a
= a − − 2 c + 2(16.35)
or   
c
( wnet ) max
(h th ) max =
qs
qs = cp (T5 – T7)
Now,  

M16_THERMAL ENGINEERING_SE_XXXX_CH16-First part.indd 866 6/16/2018 12:07:14 PM


Gas Turbines 867

qs T T
= 5− 7
c pT1 T1 T1

Also T7 = T6
qs T T T a
∴       = 5 − 6 × 5 =a −
c pT1 T1 T5 T1 c

a
a− −2 c +2
(h th ) max = c
a
a−
c
2( c − 1)
(h th ) max = 1 −
or    (16.36)
 1
a 1 − 
 c

16.12.6  Cycle with Intercooling and Reheating


The schematic arrangement of this cycle is shown in Fig. 16.16(a) and the corresponding T-s diagram
in Fig. 16.16(b).
     wc = cp [(T2 – T1) + (T4 – T3)]
qs = cp [(T5 – T4) + (T7 – T6)]
wt = cp [(T5 – T6) + (T7 – T8)]
    wnet = wt – wc = cp [(T5 – T6) + (T7 – T8) – (T2 – T1) – (T4 - T3)]

T T T T T T T 
= c pT1  5 − 6 + 7 − 8 − 2 + 1 − 4 + 3 
T
 1 T1 T1 T1 T1 T1 T1 
Now, T3 = T1 and T7 = T5 for perfect intercooling and reheating, respectively. Then,
wnet T5 T6 T5 T8 T2 T
= − + − − +1− 4 +1
c pT1 T1 T1 T1 T1 T1 T1

T5 T6 T5 T5 T8 T5 T2 T4
           = − × + − × − − +2
T1 T5 T1 T1 T7 T1 T1 T3

T5 T T T
Let       a = , c = 4 , c1 = 2 , c2 = 4 , also T2 = T3, T5 = T1
T1 T1 T1 T3
T4 T4 T2
= × .
T1 T3 T1
Thus, c = c1c2
For maximum work output,        pi = p1 p2
p p
or         i = 2
p1 pi

M16_THERMAL ENGINEERING_SE_XXXX_CH16-First part.indd 867 6/16/2018 12:07:21 PM


868 Chapter 16

Fuel Fuel

2 Intercooler 3 4 C.C 5 6 Reheat C.C 7

L.P H.P H.P L.P


Power
Turbine output
Compressor Compressor Turbine 8
1
Air from To exhaust
atmosphere (a)
p2
T
5 pi
7
p1
4 2 6 8

3
1

s
(b)

Figure 16.16  Gas turbine cycle with intercooler and reheating:


(a) Schematic diagram, (b) T-s diagram

g −1 g −1
T2  pi  g T p  g
= and 4 =  2 
T1  p1  T3  pi 

T T
∴        2 = 4
T1 T3
g −1
T p  g T4
Also,         5 =  2  = = c2
T6  pi  T3

g −1
T T p g
and         7 = 5 =  i  = c1
T8 T8  p 
 c1 = c2
or    
wnet c1 = c2 = c
∴ c1 = c2 = c
c pT3 w net a a
=a − +a − − c1 − c2 + 2
c pT3 c2 c1

M16_THERMAL ENGINEERING_SE_XXXX_CH16-First part.indd 868 6/16/2018 12:07:28 PM


Gas Turbines 869

( wnet )max a ( wnet )max


a a a
∴   =∴a − +a − =− a c−− c++a2 − − c − c+2
c pT3 c
c p 3 cT c c

( wnet )max  a 
or = 2 a − − c + 1 (16.37)
c pT3  c 

 a 
2c pT1  a − − c + 1
( wnet ) max  c 
     (h th ) max = =
qs c p [(T5 − T4 ) + (T7 − T6 )]

 a   a 
2 a − − c + 1 2 a − − c + 1
 c   c 
= =
T5 T4 T7 T6 a
− + − 2a − c −
T1 T1 T1 T1 c

a 
2(a − c ) − 2  − 1
 c 
=
a
2a − c −
c

a
+ c −2
c
(h th ) max = 1 −
or   (16.38)
a
2a − c −
c

16.12.7  Cycle with Intercooling, Regeneration and Reheating


The schematic arrangement of the cycle and corresponding T-s diagram are shown Fig. 16.17.
wc = cp [(T2 - T1) + (T4 - T3)]
qs = cp [(T5 - T9) + (T7 - T6)]
wt = cp [(T5 - T6) + (T7 - T8)]
wnet = wt - wc = cp [(T5 - T6 + T7 - T8) - (T2 - T1 + T4 - T3)]
  T7 = T5
wnet T5 T6 T5 T5 T8 T5 T2 T T
= − × + − × − +1− 4 + 3
c pT1 T1 T5 T1 T1 T7 T1 T1 T1 T1

T4 T T T T T
Let = c, 1 = 2 = c1 , 5 = 4 = c2 , 5 = a , T6 = T8 , T1 = T3 and T5 = T7 so that c1 = c2
T1 T8 T1 T6 T3 T1

M16_THERMAL ENGINEERING_SE_XXXX_CH16-First part.indd 869 6/16/2018 12:07:33 PM


870 Chapter 16

10 Fuel Fuel
To 8
exhaust

2 9
Intercooler 3 4 Heat C.C 5 6 Reheat C.C 7
exchanger

L.P H.P H.P L.P


Power
Compressor Compressor Turbine Turbine output
1
Air from
atmosphere (a)

T p2

5 7 pi
p1
9
6 8
4 2
10

3 1

s
(b)

Figure 16.17  Gas turbine cycle with intercooling, regeneration and reheating:
(a) Schematic diagram, (b) T-s diagram

Similar to the case of 16.12.6, it can be shown that


c1 = c2 = c
wnet a a
= a − + a − − c1 − c2 + 2
c pT1 c2 c1

( w net ) max a a
=a − +a − − c− c+2
c pT1 c c

( wnet ) max  a 
= 2 a − − c + 1
c pT1  c 
qs = cp (T5 - T9 + T7 - T6)
T6 = T8 = T9
qs T T T T
= 5− 6+ 5− 6
c pT1 T1 T1 T1 T1
a a
=a − +a −
c c

M16_THERMAL ENGINEERING_SE_XXXX_CH16-First part.indd 870 6/16/2018 12:07:38 PM


Gas Turbines 871

 a 
2 a − − c + 1
 c 
(h th ) max =
a a
a− +a −
c c
c −1 c
or       (h th ) max = 1 − = 1− (16.39)
a a
( c − 1)
c

Example 16.1
Calculate the indicated mean effective pressure and efficiency of a Joule cycle if the temperature at
the end of combustion is 2000 K and the temperature and pressure before compression are 350 K
and 1 bar, respectively. The pressure ratio is 1.3 and assume cp = 1.005 kJ/kgK.
Solution
Refer to Fig. 16.18.
p1 = 1 bar, T1 = 350 K, T3 = 2000 K
 rp = 1.3, cp = 1.005 kJ/kgK
g −1 0.4
g
T2 = T1rp = 350 × (1.3) .4
1 = 377.24 K

g −1 0.4
 1 g  1  1.4
T4 = T3   = 2000   = 1855.56 K
 rp   1.3 

wc = cp (T2 - T1) = 1.005 (377.24 - 350) = 27.376 kJ/kg


wt = cp (T3 - T4) = 1.005 (2000 - 1855.56) = 145.162 kJ/kg
Net work done,  wnet = wt - wc = 145.162 - 27.376 = 117.786 kJ/kg
qs = cp (T3 - T2) = 1.005 (2000 - 377.24) = 1630.874 kJ/kg

wnet 117.786
Thermal efficiency h th = = × 100 = 7.22%
qs 1630.874

p
p2 2 3

p1
1 4
V

Figure 16.18  p-v diagram

M16_THERMAL ENGINEERING_SE_XXXX_CH16-First part.indd 871 6/16/2018 12:07:43 PM


872 Chapter 16

RT4 0.287 × 103 × 1855.56


v4 = = = 5.325 m3 /kg
p1 5
1 × 10
RT2 0.287 × 103 × 377.24
v2 = = = 0.833 m3/kg
p2 5
1.3 × 10
wnet 117.86
Mean effective pressure pm = =
v4 − v2 5.325 − 0.833
= 26.235 kPa or 0.26235 bar

Example 16.2
A gas turbine operates on a pressure ratio of 6. The inlet air temperature to the compressor is 300 K
and the temperature of air entering to the turbine is 580°C. If the volume rate of air entering the
compressor is 250 m3/s, calculate the net power output of the cycle. Also, calculate the efficiency.
Solution
The T-s diagram shown in Fig. 16.19
rp = 6, T1 = 300 K, T3 = 273 + 580 = 853 K

V1 = 250 m3 /s
g −1 0.4
g
T2 = T1rp = 300 × 6 .4
1 = 500.55 K
g −1 0.4
 1 g  1  1.4
T4 = T3   = 853   = 511.23 K
r
 p  6
Compressor work wc = cp (T2 - T1) = 1.005 (500.55 - 300) = 201.55 kJ/kg
Turbine work wt = cp (T3 - T4) = 1.005 (853 - 511.23) = 343.48 kJ/kg
Net work output wnet = wt - wc = 343.48 - 201.55 = 141.93 kJ/kg
Heat input qs = cp (T3 - T2) = 1.005 (853 - 500.55) = 354.21 kJ/kg
wnet 141.93
Thermal efficiency h th = = × 100 = 40.07%
qs 354.21

T p2
3
p1
2
4
1
s

Figure 16.19  T-s diagram

M16_THERMAL ENGINEERING_SE_XXXX_CH16-First part.indd 872 6/16/2018 12:07:47 PM


Gas Turbines 873

p1V1 = mRT
 1

1 × 105 × 250
m = = 290.36 kg/s
287 × 300
290.36 × 141.93
= mw
Power output    net = = 41.21 MW
103
Example 16.3
A gas turbine has a perfect heat exchanger. Air enters the compressor at a temperature and pressure
of 300 K and 1 bar and discharges at 475 K and 5 bar. After passing through the heat exchanger,
the air temperature increases to 655 K. The temperatures of air entering and leaving the turbine are
870°C and 450°C, respectively. Assuming no pressure drop through the heat exchanger, compute
(a) the output per kg of air, (b) the efficiency of cycle, and (c) work required to drive the compressor.

Solution
The p – V and T-s diagrams are shown in Fig. 16.20.
T1 = 300 K, p1 = 1 bar,
T2 = 475 K, p2 = 5 bar,
T5 = 655 K, T3 = 870 + 273 = 1143 K,
T4 = 450 + 273 = 723 K
g −1
T2 g
= rp
T1
g −1  475 
or     = ln  ln 5
g  300 
or        g = 1.4, cp = 1.005 kJ/kg
Compressor work wc = cp (T2 - T1) = 1.005 (475 - 300) = 175.9 kJ/kg
Turbine work wt = cp (T3 - T4) = 1.005 (1143 - 723) = 422.1 kJ/kg
Heat input qs = cp (T3 - T5) = 1.005 (1143 - 655) = 490.44 kJ/kg
(a) Net work output, wnet = wt - wc = 422.1 - 175.9 = 246.2 kJ/kg
w 246.2
(b) Thermal efficiency, h th = net = × 100 = 50.2%
qs 290.44
(c) Compressor work, wc = 175.9 kJ/kg

p T
3
2 5 3
5
2 4
6

1 1
6 4 v s
(a) (b)

Figure 16.20  Diagrams for a gas turbine:


(a) p-V diagram, (b) T-s diagram

M16_THERMAL ENGINEERING_SE_XXXX_CH16-First part.indd 873 6/16/2018 12:07:50 PM


874 Chapter 16

Example 16.4
A closed cycle regenerative gas turbine operating with air has the following data: p1 = 1.4 bar,
T1 = 310 K, rp = 5, Tmax = 1050 K, effectiveness of generator = 100%, net output = 3 MW. Calculate
(a) the mass flow rate of air per minute and (b) the thermal efficiency.

Solution
The cycle in T-s diagram is shown in Fig. 16.21.
Refer to Fig. 16.21

T
3
5
2 4
6

1
s

Figure 16.21  T-s diagram for closed cycle regenerative gas turbine
g −1 0.4
g
T2 = T1rp = 310 × 5 .4
1 = 491.2 K

T3 1050
T4 = = = 662.65 K
g −1
50.286
g
rp
For 100% regenerative effectiveness,
T4 = T5 = 662.65 K
(a)     wnet = mc
 p (T3 − T4 ) − mc
 p (T2 − T1 )

3000 = m × 1.005 [(1050 − 662.65) − ( 491.2 − 310)]

m = 14.48 kg/s or 868.8 kg/min


(b) Heat input      Q = mc
 p (T3 − T5 )
= 14.48 × 1.005 (1050 - 662.65) = 5636.87 kJ/s
3000
Thermal efficiency h th = × 100 = 53.22%
5636.87

Example 16.5
The following data refers to a turbine with regenerator:
T1 = 290 K, T2 = 460 K, T3 = 900°C, T4 = 467°C
Calculate (a) the pressure ratio, (b) the specific work output, (c) the efficiency of cycle, and
(d) the compressor work. Assume hmech = 100%.

M16_THERMAL ENGINEERING_SE_XXXX_CH16-First part.indd 874 6/16/2018 12:07:55 PM


Gas Turbines 875

T
3
5
2 4
6

1
s

Figure 16.22  T-s diagram for a gas turbine with regenerator

Solution
The T-s diagram of the cycle is shown in Fig. 16.22.
g −1
T g
(a) 2 = rp
T2

 460   1.4 − 1
ln  = ln rp
 290   1.4 
or    rp = 5

(b) Work output, wnet = cp [(T3 - T4) - (T2 – T1)]


= 1.005[(1173 - 740) - (460 - 290)]
= 264.3 kJ/kg
(c) Heat input, qs = cth (T3 - T5) = cp (T3 - T4)
= 1.005 (1173 - 740) = 435.165 kJ/kg
wnet 264.3
Thermal efficiency h th = = × 100 = 60.74%
qs 435.165
(d) Compressor work, wc = cp (T2 - T1) = 1.005 (460 - 290) = 170.85 kJ/kg

Example 16.6
The ratio of net work to turbine work of an ideal gas turbine plant is 0.563. The temperature of air
at the inlet to the compressor is 300 K. Calculate the temperature drop across the turbine if thermal
efficiency of the unit is 35%. Assume a mass flow rate of 10 kg/s, cp = 1 kJ/kgK and g = 1.4.

Solution
1
Thermal efficiency h th = 1 −
g −1
g
rp
1
or 0.35 = 1 −
0.286
rp
rp = 4.51
or  

M16_THERMAL ENGINEERING_SE_XXXX_CH16-First part.indd 875 6/16/2018 12:08:00 PM


876 Chapter 16

g −1
g
Now          T2 = T1rp = 300 × ( 4.51)0.286 = 461.54 K
Compressor work wc = cp (T2 - T1) = 1 × (461.54 - 300) = 161.54 kJ/kg

wnet w − wc 161.54
= 0.563 = t = 1−
wt wt wt
or wt = 369.66 kJ/kg = cp · ∆T
 ∆T = 369.66°C

Example 16.7
In a Brayton cycle, the pressure ratio in the compressor is rp . The minimum and maximum temper-
atures are T1 and T3. The air is expanded in two stages, each turbine having the same pressure ratio,
with reheat to T3 between the stages. Show that the specific work output will be maximum when
2
T3 g −1
rp = a 3m , where a = ,m=
T1 g

Solution
The T-s diagram is shown in Fig. 16.23.
Refer to Fig. 16.23.
g −1
T2
= rp g = rpm = c
T1

T3 T5
= = ( rp )m = c
, T5 = T3 , T4 = T6
T4 T6

1
= 1−
a
1
= 1−
a
Turbine work    wt = cp (T3 - T4) + cp (T5 - T6)

T 3 5

6
2 4

1
s

Figure 16.23  T-s diagram for Brayton cycle with two-stage turbine

M16_THERMAL ENGINEERING_SE_XXXX_CH16-First part.indd 876 6/16/2018 12:08:08 PM


Gas Turbines 877

= cp (T3 - T4) + cp (T3 - T6)

 T 
= 2cp (T3 - T4) = 2cpT3 1 − 4
 T 
3

 1 
= 2c pT3 1 − 
 c
T 
wc = c p (T2 − T1 ) = c pT1  2 − 1
Compressor work     
 1 
T
= cpT1 (c - 1)

 1 
Specific work output     wnet = wt − wc = 2c pT3 1 −  − c pT1 (c − 1)
 c

wnet  1 
= 2a 1 −  − (c − 1)
c pT1  c

For maximum specific work output


3
d  wnet  1 −
  = 2a × × c 2 − 1 = 0
dc  c pT1  2

3

or       a c 2 =1

2
or          
c=a3
2
m
rp = a 3
or      
2g
2
 T  3(g −1)
or                      rp = a 3m =  3
T 
1

Example 16.8
In a gas turbine, air is taken in at 1 bar pressure and 30°C. It is compressed to 6 bar with an isen-
tropic efficiency of 87%. Heat is added by the combustion of fuel in combustion chamber to raise
the temperature to 700°C. The efficiency of the turbine is 85%. The calorific value of fuel used is
43.1 MJ/kg. For an air flow of 80 kg/min, calculate (a) the air/fuel ratio of turbine gases, (b) the
final temperature of exhaust gases, (c) the net power developed, and (d) the overall thermal effi-
ciency of plant.
Assume cpa = 1.005 kJ/kgK, ga = 1.4, cpg = 1.147 kJ/kgK, gg = 1.33.

M16_THERMAL ENGINEERING_SE_XXXX_CH16-First part.indd 877 6/16/2018 12:08:15 PM


878 Chapter 16

T 3

2 2′ 4′
4
1
s

Figure 16.24  T-s diagram for gas turbine with machine efficiencies

Solution
The T-s diagram is shown in Fig. 16.24.
p1 = 1 bar, T1 = 273 + 30 = 303 K
p2 = 6 bar, hc = 0.87, T3 = 273 + 700 = 973 K
ht = 0.85, CV = 43.1 MJ/kg
ga −1 0.4
p  ga  6  1.4
T2 = T1  2  = 303   = 505.5 K
 p1   1

T2 − T1
hc = ,
T2′ − T1
T2 − T1
T2′ = T1 +
hc

505.5 − 303
= 303 + = 535.8 K
0.87

(a) m f × C.V. = m a c pg (T3 − T2′ )  [∵ m a + m f  m a ]

80 1.147 × (973 − 535.8)


m f = × = 0.0155 kg/s
60 43.1 × 103
A m a 80 1
= = × = 86
F m f 60 0.0155
T3 − T4′
(b) ht =
T3 − T4
T3 973
T4 = = = 623.8 K
g g −1 0.33
gg 6 .33
1
rp

973 − T4′
0.85 =
973 − 623.8
T4′ = 676.2 K

M16_THERMAL ENGINEERING_SE_XXXX_CH16-First part.indd 878 6/16/2018 12:08:20 PM


Gas Turbines 879

(c) wnet = m a c pg (T3 − T4′ ) − m a c pa (T2′ − T1 )

80
   = × [1.147 (973 − 676.2) − 1.005 (535.8 − 303)]
60
wnet 142 × 100
(d) h th = = = 21.25%
Q 0.0155 × 43100

Example 16.9
In a gas turbine plant, air enters the compressor at 1 bar and 7°C. It is compressed to 4 bar with an
isentropic efficiency of 82%. The maximum temperature at the inlet to the turbine is 800°C. The
isentropic efficiency of turbine is 85%. The calorific value of fuel used is 43100 kJ/kg. The heat
losses are 15% of the calorific value. Calculate (a) the compressor work, (b) the heat supplied,
(c) the turbine work, (d) the net work, (e) the thermal efficiency, (f) the air-fuel ratio, (g) the specific
fuel consumption, and (h) the ratio of compressor to turbine work. Assume cpa = 1.005 kJ/kgK,
ga = 1.4, cpg = 1.147 kJ/kgK, gg = 1.33.
Solution
p1 = 1 bar, T1 = 280 K, hc = 0.82, p2 = 4 bar, T3 = 1073 K, ht = 0.85, hcomb = 0.85.
The T-s diagram is shown in Fig. 16.25.
Refer to Fig 16.25
g a −1
p  ga
T2 = T1  2 
 p1 
0.4
= 280 ( 4) 1.4 = 416.1 K
T2 − T1
T2′ = T1 +
hc
416.1 − 280
= 280 + = 446 K
0.182
(a) The compressor work
wc = cpa (T2′ - T1) = 1.005 (446 - 280) = 166.82 kJ/kg

T 3

4′
2 2′
4

1
s

Figure 16.25  T-s diagram for gas turbine with machine efficiencies

M16_THERMAL ENGINEERING_SE_XXXX_CH16-First part.indd 879 6/16/2018 12:08:23 PM


880 Chapter 16

(b) The heat supplied


c pg (T3 − T2′ ) 1.147 (1073 − 446)
qs = = = 846.1 kJ/kg
hcomb 0.85

1073
(c) T4 = 0.33
= 760.7
4 1.33
T4′ = T3 - ηt (T3 - T4) = 1073 - 0.85 (1073 - 760.7) = 807.5 K
The turbine work
wt = cpg (T3 - T4′) = 1.147 (1073 -807.5) = 304.53 kJ/kg
(d) The net work output
wnet= wt - wc = 304.153 -166.83 = 137.7 kJ/kg
(e) The thermal efficiency
wnet 137.7
h th = = × 100 = 16.23%
qs 846.1

(f) m f × C.V. (1 - 0.15) = (m· a + m· f ) Q × hcomb

m a CV 43100
or    = –1 = – 1 = 49.94
m f qs 846.1

(g) The specific fuel consumption

Fuel consumed per hour m f × 3600


SFC = =
Net power m a × wnet

3600
= = 0.524 kg/ kWh
49.94 × 137.7
wc 166.83 × m a
(h) =
wt 304.53 × ( m a + m f )

166.83 × 49.94
= = 0.537
304.53 × 50.94

Example 16.10
In a closed circuit gas turbine plant, the working fluid at 38°C is compressed with an adiabatic
efficiency of 82%. It is then heated at constant pressure to 650°C. The fluid expands down to initial
pressure in a turbine with an adiabatic efficiency of 80%. The pressure ratio is such that work done
per kg is maximum. cpa = 1.005 kJ/kgK, cpg = 1.147 kJ/kgK. Calculate the pressure ratio for max-
imum work output and the corresponding cycle efficiency.

M16_THERMAL ENGINEERING_SE_XXXX_CH16-First part.indd 880 6/16/2018 12:08:28 PM


Gas Turbines 881

Solution
Refer to Fig 16.25.
T3 923
a = = = 2.96
T1 311
For maximum specific work,
g −1
g
( rp )opt = ahcht = 2.96 × 0.82 × 0.80 = 1.3935
or ( rp )opt = (1.395)3.5 = 3.194
g −1
T2 = T1 ( r ) g = 311 × 1.3935 = 433.51 K
p opt
T2 − T1 433.51 − 311
T2′ = T1 + = 311 + = 460.40 K
hc 0.82
T3 923
T4 = = = 692.03 K
g −1 1.3935
g
( rp )opt
T4′ = 923 − 0.8 (923 − 692.03) = 738.22 K
1.147 (923 − 738.22) − 1.005( 460.40 − 311)
hth =
1.147(923 − 460.40)
= 0.1155 or 11.55%
Example 16.11
A constant pressure closed cycle turbine plant works between a temperature range of 800°C and
30°C. The isentropic efficiencies of compressor and turbine are 80% and 90%, respectively. Find the
pressure ratio of the cycle for maximum thermal efficiency and maximum specific output of the cycle.
Solution
T1 = 273 + 30 = 303 K, T3 = 273+ 800 = 1073 K, hc = 0.8, ht = 0.9
Pressure ratio for maximum specific output
g 1.4
 T  2(g −1)  1073  2× 0.4
rp = hcht 3  = 0.8 × 0.9 × = 5.15
 T1  303 
g
  g −1
 T3 
 T1 
Pressure ratio for maximum thermal efficiency rp =  
  T3   1  
1 +  T − 1  h h − 1 
 1 c t 
1.4
  0.4
 1073 
= 303  = 25.5
  1073   1  
1 +  − 1  − 1 
  303   0.8 × 0.9  

M16_THERMAL ENGINEERING_SE_XXXX_CH16-Second part.indd 881 6/16/2018 12:25:36 PM


882 Chapter 16

Example 16.12
A gas turbine plant operates between 5°C and 839°C. Find the following:
(a) Pressure ratio at which cycle efficiency equals to Carnot cycle efficiency
(b) Pressure ratio at which maximum work is obtained
(c) Efficiency corresponding to maximum work output
Solution
T1 278
The efficiency of Carnot cycle hcarnot = 1 − = 1− = 0.75
T2 1112
1
For simple gas turbine cycle, hth = 1 − = 0.75
g −1
g
rp
1
(a) 1 − = 0.75
g −1
g
rp
p2
or rp = = 128
p1
T2 1112
(b) a = = =4
T1 278
wnet  1
= a 1 −  − (c − 1)
cpT1  c

dwnet a
For maximum work output = −1 = 0
dc c2
or c= a = 4=2
1 1
(c) hth = 1 − = 1 − = 0.5 or 50%
c 2

Example 16.13
A simple open cycle gas turbine has a compressor turbine and a free power turbine. It develops
electrical power output of 250 MW. The cycle takes in air at 1 bar and 288 K. The total compressor
pressure ratio is 14. The turbine inlet temperature is 1500 K. The isentropic efficiency of compres-
sor and turbine are 0.86 and 0.89, respectively. The mechanical efficiency of each shaft is 0.98.
Combustion efficiency is 0.98 while combustor pressure loss is 0.03 bar. The alternator efficiency is
0.98. Take calorific value of fuel equal to 42,000 kJ/kg, cpa = 1.005 kJ/kgK and cpg = 1.15 kJ/kgK.
Calculate (a) the air-fuel ratio, (b) the specific work output, (c) the specific fuel consumption, (d)
the mass flow rate of air, and (e) the thermal efficiency of the cycle.  [IES 2004]
Solution
p1 = 1 bar, T1 = 288 K, p2 = 14 bar, p3 = 14 − 0.03 = 13.97 bar, T3 = 1500 K, P = 250 kW, hc = 0.86, ht =
0.89, hmech = 0.98, hcomb = 0.98, ha = 0.98, CV = 42,000 kJ/kg, cpa = 1.005 kJ/kgK, cpg = 1.15 kJ/kgK

M16_THERMAL ENGINEERING_SE_XXXX_CH16-Second part.indd 882 6/16/2018 12:25:39 PM


Gas Turbines 883

T
ba
r ar
7b
14 13.9
3
2 2′ ar
1b
4 4′
1
s

Figure 16.26  T-s diagram for gas turbine with pressure loss in combustion chamber

The T-s is shown in Fig. 16.26.


g −1
T2  p2  g
=
T1  p1 

T2 = 288 (14)0.4/1.4 = 612.15 K


Compressor efficiency,

T2 − T1
hc =
T2′ − T1

or 612.15 − 288
0.86 =
T2′ − T1
or T2′ − T1 = 376.92 K
or T2′ = 664.92 K

T3 1500
T4 = = = 706.14 K
g −1 0.4
 p3  g  13.97  1.4
 p   
4
1 
Turbine efficiency,
T3 − T4′
h=
T3 − T4

1500 − 706.14
or T3 − T4′ = = 891.97 K
0.89
Actual net work done, wnet = Turbine work, wt − Compressor work, wc
c pg (T3 − T4′ )h mech − c pa (T2′ − T1 )
=
ha

1.15 × 891.97 × 0.98 − 1.005 × 376.92


=
0.98
= 618.72 kJ/kg of air

M16_THERMAL ENGINEERING_SE_XXXX_CH16-Second part.indd 883 6/16/2018 12:25:42 PM


884 Chapter 16

Effective heat supplied


qe = cpg ( T3 − T2′)
= 1.15 (1500 − 664.92) = 960.34 kJ/kg
Actual heat supplied,
qe 960.34
qa = = = 979.94 kJ/kg
hcomb 0.98
(a) Air/fuel ratio,
A C.V. of fuel 42000
= = = 42.86 : 1
F qa 979.94
(b) Specific work output = 618.72 kJ/kg of air.
Specific air consumption
(c) Specific fuel consumption =
( A/F )
 3600 
 Specific work output 
=
( A/F )

 3600 
 
618.72 
= = 0.1357 kg/kWh
42.86
(d) Mass flow rate of air, ma × Specific work output
= Electric power developed × Alternator efficiency
250 × 103 × 0.98
or ma = = 395.98 kg/s
618.72
(e) Thermal efficiency of the cycle
Actual net work done 618.72
= = × 100 = 63.14%
actual heat supplied 979.94

Example 16.14
An open cycle gas turbine employs a regenerative arrangement. The air enters the compressor at
1 bar and 288 K and is compressed to 10 bar with a compression efficiency of 85%. The air is heated
in the regenerator and the combustion chamber till its temperature is raised to 1700 K and during
the process pressure falls by 0.2 bar. The air is then expanded in the turbine and passes to regenerate
which has 75% effectiveness and causes a pressure drop of 0.2 bar. The isentropic efficiency of the
turbine is 86%. By sketching the gas turbine system and showing the process on T-s diagram, calcu-
late the thermal efficiency and the power output if mass flow rate of air is 100 kg/s. Take mechanical
and alternator efficiency as 98% each. Take cpg = 1.15 kJ/kgK and cpa = 1.005 kJ/kgK.
[IES 2005]

M16_THERMAL ENGINEERING_SE_XXXX_CH16-Second part.indd 884 6/16/2018 12:25:45 PM


Gas Turbines 885

6
Exhaust gases
to atmosphere
C.C
3 5′
2′ Regenerator 4

C T
Power
1 Compressor Turbine output
Air from atmosphere
(a)
r
T ba 9.8 bar
10
1700 K 4
x
3
2′ 1.2 bar
2 5′
5 1 bar
z
6

278 K
1
s
(b)

Figure 16.27  Open cycle gas turbine: (a) Gas turbine system, (b) T-s diagram

Solution
The gas turbine system and T-s diagram are shown in Fig. 16.27.
(g −1) 0.4
p  g  10  1.4
T2 = T1  2  = 288   = 556 K
 p1   1
(T2 − T1 )
T2′ = T1 + = 288 + (556 − 288) / 0.85 = 603.3 K
hc
p3′ = 10 − 0.2 = 9.88 bar = p4
p5 = 1 + 0.2 = 1.2 bar
g −1 0.4
T4  p4  g  9.8  1.4
= = = 1.822
T5  p5   1.2 
1700
T5 = = 932.96 K
1.822
T5′ = T4 − (T4′ − T5′ )ht
= 1700 − (1700 − 932.96) × 0.86 = 1040.34 K
Work consumed by the compressor,
wc = c pa (T2′ − T1 )
= 1.005 (603.3 − 288) = 316.876 kJ/kg

M16_THERMAL ENGINEERING_SE_XXXX_CH16-Second part.indd 885 6/16/2018 12:25:48 PM


886 Chapter 16

Work done by the turbine


wt = c pg (T4 − T5′ ) = 1.15 (1700 − 1040.34)
= 758.61 k/kg
Net work done = wt – wc = 758.61 − 316.876 = 441.73 kJ/kg
T3 − T2′
Regenerator effectiveness, e = = 0.75
T5′ − T2′

T3 = 0.75(1040.34 − 603.3) + 603.3 = 931.08 K


Heat supplied = c pa (T4 − T3 ) = 1.005 (1700 − 931.08) = 772.76 kJ/kg

Net work 441.73


Thermal efficiency = = = 0.5716 or 57.16%
Heat supplied 772.76
Power developed = 100 × 441.73 × 0.98 × 0.98 = 42423.75 kW

Example 16.15
Air enters the compressor of a gas turbine at 100 kPa, 300 K with a volumetric flow rate of 5 m3/s.
The air is compressed in two stages to 1200 kPa with intercooling to 300 K between stages at a
pressure of 350 kPa. The turbine inlet temperature is 1400 K and the expansion occurs in two stages
with reheat to 1340 K between the stages at a pressure of 350 kPa. The compressor and turbine
stage efficiencies are 87% and 85%, respectively. Draw the schematic diagram of the cycle and
indicate the process on T-s diagram. Determine (a) the thermal efficiency of the cycles, (b) the back
work ratio, (c) the net power developed in kW. Assume effectiveness of the regenerator as 80% and
cp = 1.0045 kJ/kgK for air and gas.  [IES 2006]

Solution
The schematic diagram of the cycle and T-s diagram are shown in Fig. 16.28.
p1 = pa = 100 kPa, T1 = 300 K, V1 = 5 m3/s, p2 = p3 = p7 = p8 = 350 kPa,
p4 = p6 = 1200 kPa, T1 = T3 = 300 K, T6 = 1400 K, T8 = 1340 K,

h c1 = h c 2 = 0.87, ht1 = ht 2 = 0.85, e = 0.8 , cpa = cpg 1.0045 kJ/kgK


Compressor 1st stage:
g −1 1
p  g
 350  3.5
T2 = T1  2  = 300  = 429.11 K
 p1   100 

T2 − T1
hc1 =
T2′ − T1

429.11 − 300
or T2′ = 300 + = 448.4 K
0.87
Work done, wc1 = c pa (T2′ − T1 ) = 1.0045 (448.4 − 300) = 149.07 kJ/kg

M16_THERMAL ENGINEERING_SE_XXXX_CH16-Second part.indd 886 6/16/2018 12:25:52 PM


Gas Turbines 887

Regenerator
10 9′
To
exhaust

C.C C.C
5 7′
4′ 6 8
Compressor Compressor

L.P H.P H.P L.P s


Generator
1 Turbine Turbine
Air from 3
atmosphere
2′
Intercooler
(a)

T 1200 kPa
350 kPa
6 100 kPa
1700 K 8
1340 K
5 7′ 9′
5′ 7 9
4′ 2′
4 2 10′

300 K
3 1
s
(b)

Figure 16.28  Gas turbine with intercooling, reheating, and regeneration:


(a) Gas turbine system, (b) T-s diagram

Compressor 2nd stage: g −1 1


p  g
 1250  3.5
T4 = T3  4  = 300  426.59 K
 p3   350 

426.59 − 300
T4′ = 300 + = 445.50 K
0.87
Work done, wc 2 = c pa (T4′ − T3 ) = 1.0045 (445.50 − 300) = 146.16 kJ/kg
Total compressor work, wc = wc1 + wc2 = 295.23 kJ/kg
Turbine Ist stage:
g −1 1
p  g
 350  3.5
T7 = T6  7  = 1400  = 984.55 k
 p6   1200 

T6 − T7′
h t1 =
T6 − T7

or T7′ = 1400 − 0.85 (1400 − 984.55) = 1046.87 K

M16_THERMAL ENGINEERING_SE_XXXX_CH16-Second part.indd 887 6/16/2018 12:25:54 PM


888 Chapter 16

Work done, wt1 = c pg (T6 − T7′ ) = 1.0045 (1400 − 1046.87) = 354.72 kJ/kg
Turbine 2nd stage:
g −1 1
p  g  100  3.5
T9 = T8  9  = 1340  = 936.82 K
 p8   350 
T8 − T9′
ht 2 =
T8 − T9
or T9′ = 1340 − 0.85 (1340 − 936.82) = 997.3 K

Work done, wt 2 = c pg (T8 − T9′ ) = 1.0045 (1340 − 997.3) = 344.24 kJ/kg


Total turbine work wt = wt1 + wt2 = 698.96 kJ/kg
Net work wnet = wt – wc = 698.96 − 295.23 = 403.73 kJ/kg
T − T4′
Regenerator effectiveness, e = 5′
T9′ − T4′
T5′ − 445.5
or 0.8 =
997.3 − 445.5
or T5′ = 886.94 K

Heat supplied, qs = c pg [(T6 − T5′ ) + (T8 − T7′ )]


= 1.0045 [(1400 − 886.94) + (1340 − 1046.87)]
= 809.82 kJ/kg
(a) Thermal efficiency,
wnet 403.73
hth = = = 0.4985 or 49.85%
qs 809.82
wnet 403.73
(b) Work ratio = = = 0.5776
wt 698.96

(c) p1V1 = m• RT1


100 × 105 × 5
or m = = 580.72 kg/s
287 × 300
Power developed = m × wnet = 580.72 × 403.73 = 234454 kW

Example 16.16
Air enters the compressor of a gas turbine plant operating on Brayton cycle at 1 bar, 27°C. The
pressure ratio in the cycle is 6. If Wt = 2.5 Wc, where Wt and Wc are the turbine and compressor
work, respectively, calculate the maximum temperature and the thermal efficiency of the cycle.
Take g  = 1.4.

M16_THERMAL ENGINEERING_SE_XXXX_CH16-Second part.indd 888 6/16/2018 12:25:59 PM


Gas Turbines 889

Solution
p1 = 1 bar, T1 = 27 + 273 = 300 K, p2 = 6 bar, g  = 1.4
Refer to Fig. 16.4, we get
g -1
T2  p2  g
=
T1  p1 
or T2 = 300 × 60.286 = 500.5 K
g -1
T3  p2  g
= = 60.286 = 1.66
T4  p1 
Wc = mcp (T2 − T1)
Wt = mcp (T3 − T4)
Wt = 2.5 Wc
or T3 − T4 = 2.5 (T2 − T1)
 1 
or T3 1 − = 2.5 (500.5 − 300)
 1.66 
or T3 = 1260.7 K
Wt − Wc
Thermal efficiency =
mc p (T3 − T2 )

(1260.7 − 759.4) − (500.5 − 300)


= × 100 = 39.56%
(1260.7 − 500.5)

Example 16.17
In an air-standard regenerative gas turbine cycle, the pressure ratio is 5. Air enters the compressor
at 1 bar, 300 K and leaves at 490 K. The maximum temperature in the cycle is 1000 K. If the effec-
tiveness of the regenerator and the isentropic efficiency of the turbine are each 80%, determine the
cycle efficiency. Assume g =1.4 for air.

Solution
The cycle on the T-s diagram is shown in Fig. 16.29 assuming compressor efficiency to be 100%.
g −1
p  g
T2 = T1  2 
 p1 
= 300 × 50.286 = 475.14 K
T3 1000
T4 = = = 631.39 K
g −1
50.286
 p2  g
 p 
1

M16_THERMAL ENGINEERING_SE_XXXX_CH16-Second part.indd 889 6/16/2018 12:26:05 PM


890 Chapter 16

T p2
3
p1
4′
5
4
5′
2 6′
6
1
s

Figure 16.29  T-s diagram for gas turbine with regeneration

T3 − T4′
Turbine efficiency, ht =
T3 − T4
1000 − T4
or 0.8 =
1000 − 631.39
or T4′ = 705.11 K
T5′ − T2
Regenerative effectiveness =
T4′ − T2
T5′ − 475.14
or 0.8 =
705.11 − 475.14
or T5′ = 659.116 K
wc = cp (T2 − T1) = 1.005 × (475.14 − 300)
= 176.01 kJ/kg
wt = cp (T3 − T4′ ) = 1.005 (1000 − 705.11)
= 296.36 kJ/kg
Net work, wnet = wt − wc = 296.36 − 176.01
= 120.35 kJ/kg
Heat supplied, qs = cp (T3 − T5′ )
= 1.005 × (1000 − 659.116) = 342.58 kJ/kg
wnet 120.35
Thermal efficiency = = = 0.351 or 35.1%
qs 342.58

16.13 ❐ EFFECTS OF OPERATING VARIABLES


The operating variables for the performance of open cycle gas turbine are as follows:
1. Pressure ratio
2. Compressor inlet temperature
3. Turbine inlet temperature

M16_THERMAL ENGINEERING_SE_XXXX_CH16-Second part.indd 890 6/16/2018 12:26:09 PM


Gas Turbines 891

4. Efficiency of the compressor


5. Efficiency of the turbine
These variables affect the thermal efficiency and the work ratio of the gas turbine.

16.13.1  Effect of Pressure Ratio


For the simple Brayton cycle,

1
Thermal efficiency, h th = 1 −
g −1
g
rp
g −1
T1 g
Work ratio, RW = 1 − (r )
T3 p

The thermal efficiency for various values of g  and rp are given in Table 16.2.
The variation of thermal efficiency with pressure ratio for different values of ratio of specific heats
is shown in Fig. 16.30. It may be observed that the thermal efficiency increases as the pressure ratio
and the ratio of specific heats increases.
The work ratios for different values of rp and T3/T1 ratio are given in Table 16.3.

Table 16.2  Effect of pressure ratio on thermal efficiency

g Thermal efficiency
rp = 1 rp = 5 rp = 10 rp = 15 rp = 20
1.33 0 0.329 0.435 0.489 0.524
1.40 0 0.368 0.482 0.539 0.575
1.66 0 0.472 0.600 0.659 0.696

0.7
1.66
0.6
g =
1 .40
0.5 g =
g = 1.33
Thermal efficiency, h th

0.4

0.3

0.2

0.1

0
5 10 15 20
Pressure ratio, rp

Figure 16.30  Variation of thermal efficiency with pressure ratio of a gas turbine

M16_THERMAL ENGINEERING_SE_XXXX_CH16-Second part.indd 891 6/16/2018 12:26:10 PM


892 Chapter 16

Table 16.3  Effect of pressure ratio and temperature ratio on work ratio (g = 1.4)

T3/T1 Work ratio


rp = 1 rp = 5 rp = 10 rp = 15 rp = 20
2 0.5 0.208 0.035 −0.084 −0.175
3 0.667 0.472 0.357 0.277 0.217
4 0.750 0.604 0.517 0.458 0.412
5 0.80 0.683 0.614 0.566 0.530

0.8
0.7 T3 /
T1 =
5
0.6
0.5 T3 /T
1 =4
Work Ratio, Rw

0.4
T3 /T
0.3 1 =3
0.2
0.1 T3 /
T1
=2
0
5 10 15 20
Pressure ratio, rp

Figure 16.31  Variation of work ratio with pressure ratio of a gas turbine

The variation of work ratio with pressure ratio for different values of temperature ratio is shown in
Fig. 16.31. It may be observed that the work ratio decreases as the pressure ratio increases. However,
the work ratio increases with increase in temperature ratio.

16.13.2 Effect of Efficiencies of Compressor and Turbine


on Thermal Efficiency
From Eq. (16.11), we have

T3 
1
1
ht 1 − rpm  − [rpm − 1]
T1   hc
h th =  
 T3 m
 − rp 
 T1 
g −1
where m=
g

M16_THERMAL ENGINEERING_SE_XXXX_CH16-Second part.indd 892 6/16/2018 12:26:11 PM


Gas Turbines 893

0.4
hc = 0.9, h t = 0.9

hc = 0.85, h t = 0.90
0.3
hc =
ht =

Thermal efficiency, h th
0.85
hc =
0.8
0, h
0.2 t = 0.
hc = 85
0.8
0, h
t = 0.
80
0.1
a=4

0
2 5 10 15 20
Pressure ratio, rp

Figure 16.32  Variation of thermal efficiency with pressure ratio

0.4

= 0.90
0.3 hc = h t
Thermal efficiency, h th

hc = h t = 0.9
0.2

0.1
rp = 5
rp = 2
0
1 2 3 4 5
T
Temperature ratio a = 3
T1
Figure 16.33  Variation of thermal efficiency with temperature ratio

T3
Let = a, then
T1
 1  1
aht 1 − m  − [rpm − 1]
 rp  hc
hth =  
[a − rpm ]

1
For g  = 1.4, m =
3.5

M16_THERMAL ENGINEERING_SE_XXXX_CH16-Second part.indd 893 6/16/2018 12:26:14 PM


894 Chapter 16

The effect of machine efficiencies on thermal efficiency has been shown in Fig. 16.32. The ther-
mal efficiency initially increases, becomes maximum, and then starts decreasing. Figure 16.33 shows
the effect of temperature ratio on thermal efficiency. It may be observed that the thermal efficiency
increases as temperature ratio increases.

16.14 ❐ MULTI-SHAFT SYSTEMS


It is possible to use two turbines in parallel by using a single compressor and a single combustion
chamber. The arrangement for such a system is shown in Fig. 16.34. For 1 kg of air through the com-
pressor, a certain fraction x is required to pass through turbine I in order to drive the compressor. Then
(1 − x) kg passes through turbine II to produce the power output. This is advantageous in that the speed
of the compressor would not be affected by the load. A further advantage in this respect can be had by
the use of two combustion chambers as shown in Fig. 16.35.
In such an arrangement, the inlet temperatures of the two turbines, as well as their speeds, are
separately controllable.

4′

TII
Power
Load output
C.C turbine
2′ 3′
Compressor
C TI turbine

1 4

Figure 16.34  Two turbine in parallel

3″ TII
C.C

4″
C.C
2′ 3′

C TI
1
4′

Figure 16.35  Two turbines of parallel with two combustion chambers

M16_THERMAL ENGINEERING_SE_XXXX_CH16-Second part.indd 894 6/16/2018 12:26:15 PM


Gas Turbines 895

3′
C.C
4′
2′

C TI TII
1
5′

Figure 16.36  Two turbines in series

16.15 ❐ MULTI-SHAFT SYSTEM TURBINES IN SERIES


The arrangement shown in Fig. 16.36 has two shafts with two turbines in series. It gives advantage of
independent shaft speeds.

16.16 ❐ GAS TURBINE FUELS


The various fuels used for gas turbines are as follows:
1. Gaseous fuels: Natural gas, blast furnace, and producer gas
2. Liquid fuels: Liquid fuels of petroleum origin such as distillate oils or residual oils
3. Solid fuels: Pulverised coal and crushed coal

16.17 ❐ BLADE MATERIALS


Materials used in the manufacture of the blades consist of mainly three different elements, namely
iron, nickel, and cobalt with chromium which forms one of the major alloying elements, since it gives
high resistance to oxidation. Other alloying elements that have been used include most of the metals
of the periodic table.
Materials working under high stress and high temperature have a particular rate of creep. It means
elongation under stress is not a fixed quantity as it is in case of normal temperature. Thus, elongation
continues to increase with time and the blades gradually take up the original gap provided at their
tips. Therefore, contact with casing results in failure. The repeated heating and cooling of the material
affect the physical properties of the material.

16.17.1 Selection
The turbine blades are the most conditioned members of a gas turbine. They must withstand the
following:
1. Effects of high operating temperature
2. Centrifugal tensile stresses due to rotational speeds of the order of 8000 to as high as 30,000
r.p.m.

M16_THERMAL ENGINEERING_SE_XXXX_CH16-Second part.indd 895 6/16/2018 12:26:16 PM


896 Chapter 16

3. Bending stress due to equivalent impulse load of the gases acting at a certain distance from fixing
of the cantilever blade.
4. Hot erosive and corrosive effects due to high temperature combustion products e.g., CO2 and CO
with O2.
Turbine blade material must, therefore, be selected from considerations of the working conditions.

16.17.2  Requirements of Blade Material


The material should possess the following characteristics:
1. Maximum strength at high temperature
2. High creep strength
3. High resistance of corrosion
4. Maximum erosion resistance
5. Structural stability when exposed to varying temperatures
6. Castability or forgability
7. Weldability, if welding is used in the manufacture
8. Machinability
9. Absence of embrittlement in the service

16.18 ❐ COOLING OF BLADES


The thermodynamic analysis of the gas cycle shows that the efficiency of the cycle depends on the
range of maximum temperature of the gas. If maximum temperature is increased, efficiency of the cycle
is also increased. Any increase in the maximum temperature of gas results in an increase in the blade
temperature, thereby inducing more thermal stresses in the blade material. Thus, this limits the capacity
of gas temperature when blades are cooled properly.

16.18.1  Advantages of Cooling


1. Specific thrust of jet engine increases by 32% for gas temperature increase from 800°C to 1100°C
at the expense of 7% specific fuel consumption.
2. If the maximum temperature is increased to 1600°C in place of present practice of 900°C, the
specific fuel consumption will be increased by 50% and there is an increase in specific power
output by 200%.

16.18.2  Different Methods of Blade Cooling


1. Internal air cooling: It is done by supplying cold air through hollow blade. It employs an internal
baffle or deflector to direct the flow over the hotter portions on the internal surface (Fig. 16.37).
2. Film cooling: It involves supplying of thin film of cold compressed air through a narrow slit on
the blade’s surface to form a cold boundary layer over the blades (Fig. 16.38).
3. Water cooling: Circulation of water is maintained through a section of the blade from the root
towards the tip.

M16_THERMAL ENGINEERING_SE_XXXX_CH16-Second part.indd 896 6/16/2018 12:18:33 PM


Gas Turbines 897

4. Rim cooling: The rim is cooled by circulating the cooling fluid and thus the blade temperature is
reduced by conduction.
Figure 16.39 shows the distribution of temperature over the blade height.

Tip of Blade

Deflector
or inernal
baffle to
direct the flow
Hollow
Blade air passage

(a)

Root

(b) Air passages Blade cooling

Figure 16.37  Internal cooling of blade

Slit

Figure 16.38  Air film cooling of blade

750°C

150
0
Blade temperature

650°C 110 0 rpm


00
100 rpm
00
550°C rpm

400°C

1/3 1/2 Tip

Figure 16.39  Distribution of temperature over blade height

M16_THERMAL ENGINEERING_SE_XXXX_CH16-Second part.indd 897 6/16/2018 12:18:34 PM


898 Chapter 16

Example 16.18
A gas turbine unit receives air at 100 kPa and 300 K and compresses it adiabatically to 620 kPa.
The fuel has heating value of 44180 kJ/kg and fuel/air ratio is 0.017 kg fuel per kg air. The isen-
tropic efficiencies of the compressor and the turbine are 88% and 90%, respectively. Calculate the
compressor work, the turbine work, and the thermal efficiency.

Solution
The T-s diagram is shown in Fig. 16.40.
Given: p1 = 100 kPa, T1 = 300 K, p2 = 620 kPa, C.V. = 44180 kJ/kg,
F:A = 0.017 kg fuel/kg air, hc = 0.88, ht = 0.90
Refer to Fig. 16.40.
Isentropic process 1−2:
g −1
T2  p2  g
=
T1  p1 
0.4 /1.4
 620 
=  = 1.6842
 100 
or T2 = 300 × 1.6842 = 505.3 K
T2 − T1
Compressor efficiency, hc =
T2′ − T1

505.3 − 300
0.88 =
T2′ − 300
or T2′ = 533.3 K

Combustion process 2′−3:


m· f × C.V. = m· a × cpa (T3 − T2′ ) + m· f × cpg (T3 − T2′ )
Let cpa = 1.005 kJ/kgK and cpg = 1.15 kJ/kgK
0.017 × 44180 = (1 × 1.005 + 0.017 × 1.15) (T3 − 533.3)
= 1.0246 (T3 − 533.3)

T
p2
3

T 2′ 2 2′
T2 p1
4′
4
T1 1
s

Figure 16.40  T-s diagram for gas turbine

M16_THERMAL ENGINEERING_SE_XXXX_CH16-Second part.indd 898 6/16/2018 12:18:38 PM


Gas Turbines 899

or T3 = 1266.4 K
Isentropic process 3−4:
T3 1266.4
T4 = = = 751.9 K
(g −1)/ g 1.6842
( p3 /p4 )
T3 − T4′
Turbine efficiency, ht =
T3 − T4

1266.4 − T4′
or 0.90 =
1266.4 − 751.9
or T4′ = 803.3 K
Compressor work, wc = cpa ( T2′ − T1) = 1.005 (533.3 − 300) = 234.47 kJ/kg

 m f 
Turbine work, wc = cpa (T3 − T4′ ) +   cpg (T3 − T4′ )
 m a 
= (1.005 + 0.017 × 1.15) (1266.4 − 803.3) = 477.47 kJ/kg
Net work, wnet = wt − wc = 474.47 − 234.47 = 240 kJ/kg
Heat supplied, qs = m· f × C.V. = 0.017 × 44180 = 751.06 kJ/kg
wnet 240
Thermal efficiency, hth = = = 0.3195 or 31.95%
qs 751.06

Example 16.19
The pressure ratio of an open cycle constant pressure gas turbine plant is 6. The temperature range
of the plant is 20°C and 850°C. Using the following data; cpa = 1 kJ/kgK, cpg = 1.05 kJ/kgK,
ga = gg = 1.4, C.V. of fuel = 44,000 kJ/kg, hc = 0.85, ht = 0.90, hcomb = 0.95, find (a) the thermal
efficiency of the plant, (b) the net power developed if circulation of air is 5 kg/s, (c) the air-fuel
ratio, and (d) the specific fuel consumption.
Solution
Given: rp = 6, T1 = 273 + 20 = 293 K, T3 = 273 + 850 = 1223 K, cpa = 1 kJ/kgK
cpg = 1.05 kJ/kgK, ga = gg = 1.4, CV = 44,000 kJ/kg,
hc = 0.85, ht = 0.90, hcomb = 0.95, m a = 5 kg/s
Refer to Fig. 16.40
Isentropic process 1−2:
g a −1
ga
T2 = T1 ( rp ) = 293(6)0.4/1.4 = 488.87 K

T2 − T1
hc =
T2′ − T1

M16_THERMAL ENGINEERING_SE_XXXX_CH16-Second part.indd 899 6/16/2018 12:18:43 PM


900 Chapter 16

488.87 − 293
or 0.85 =
T2′ − 293
or T2′ = 523.43 K
Combustion process 2′−3
m f × C.V. × hcomb = ( m a × cpa + m f × cpg) (T3 − T2′ )
 m 
or C.V. × hcomb =  a × c pa + c pg  (T3 − T2′ )
 m f 
or 44000 × 0.95 = [(A/F) × 1 + 1.05](1123 − 523.43)
A
or = 68.67
F
Isentropic process 3−4:
T3 1123
T4 = = = 673 K
(g g −1)/ g g
rp (6)0.4 /1.4

T3 − T4′
ht =
T3 − T4
1123 − T4′
or 0.90 =
1123 − 673
or T4′ = 718 K
Compressor work, wc = cpa ( T2′ − T1) = 1 × (523.43 − 293) = 230.43 kJ/kg
1
Turbine work, wt = 1 × cpa (T3 − T4′ ) + × cpg (T3 − T4′ )
( A/F )
1
= 1 × 1.0 (1123 − 718) + × 1.05(1123 − 718)
68.67
= 411.19 kJ/kg
Net work per kg of air, wnet = wt − wc = 411.19 − 230.43 = 180.76 kJ/kg
1
Heat supplied, qs = m f × C.V. × hcomb = × 44,000 × 0.95 = 608.7 kJ/kg
68.67
(a) Thermal efficiency of plant,
wnet 180.76
hth = = = 0.2969 or 29.69%
qs 608.7
(b) Net power developed,
P = m a × wnet = 5 × 180.76 = 903.8 kW
(c) Specific fuel consumption,
m f × 360 m a 360
SFC = = ×
P ( A/F ) P
5 3600
= × = 0.29 kg/kWh
68.69 903.8

M16_THERMAL ENGINEERING_SE_XXXX_CH16-Second part.indd 900 6/16/2018 12:18:51 PM


Gas Turbines 901

Example 16.20
Determine the efficiency of a gas turbine plant fitted with a heat exchanger of 75% effectiveness.
The pressure ratio is 4:1 and the compression is carried out in two stages of equal pressure ratio
with intercooling back to initial temperature of 290 K. The maximum temperature is 925 K. The
isentropic efficiency of the turbine is 88% and the isentropic efficiency of each compressor is 85%.
For air g = 1.4, cp = 1.005 kJ/kgK.
Solution
The T-s diagram is shown in Fig. 16.41.
p2 p3
Given: e = 0.75, rp = 4, T1 = 290 K = T3, = = 4 = 2,
p1 p2
T6 = 925 K, ht = 0.88, hc = 0.85, g = 1.4, cp = 1.005 kJ/kgK.
Refer to Fig. 16.41.
L.P. compressor: (g −1)/ g
p 
T2 = T1  2  = 290(2)0.4/1.4 = 353.5 K
 p1 
T2 − T1
hc =
T2′ − T1

353.5 − 290
or 0.85 =
T2′ − 290
or T2′ = 364.7 K
p3 p2
Since T3 = T1 and = =2
p2 p1
∴ T4′ = 364.7 K for the H.P. compressor.
H.P. turbine:
(g −1)/ g
T6  p3 
= = (8)0.4/1.4 = 1.811
T7  p1 
925
or T7 = = 510.6 K
1.811

T 6 p3
p2
p1
5′ 7′
5 7
4′ 2′
4 2

3 1
s

Figure 16.41  T-s diagram for gas turbine with intercooling

M16_THERMAL ENGINEERING_SE_XXXX_CH16-Second part.indd 901 6/16/2018 12:18:54 PM


902 Chapter 16

T6 − T7′
ht =
T6 − T7

925 − T7′
or 0.88 =
925 − 510.6
or T7′ = 560.3 K
T − T4′
Heat exchanger effectiveness, e = 5′
T7′ − T4′
T5′ − 364.7
or 0.75 =
560.3 − 364.7
or T5′ = 511.4 K
Heat supplied, qs = cp (T6 − T5′ ) = 1.005 (925 − 511.4) = 415.67 kJ/kg
Compressor work, wc = 2cp ( T2′ −T1) = 2 × 1.005 (364.7 − 290) = 150.147 kJ/kg
Turbine work, wt = cp (T6 − T7′ ) = 1.005 (925 − 500.3) = 366.52 kJ/kg
Net work, wnet = wt − wc = 366.52 − 150.147 = 216.376 kJ/kg
wnet 216.376
Efficiency of the gas turbine, hth = = = 0.5205 or 52.05%
qs 415.67

Example 16.21
Air enters the compressor of a gas turbine plant operating on Brayton cycle at 1 bar and 27°C. The
pressure ratio in the cycle is 6. Assuming the turbine work 2.5 times the compressor work, calculate
the maximum temperature in the cycle and cycle efficiency. Take g   = 1.4
Solution
Given: p1 = 1 bar, T1 = 273 + 27 = 300 K, rp = 6, wt = 2.5 wc
Refer to Fig. 16.4(b)
Process 1−2:
T2 = T1(rp)(g − 1)/g = 300(6)0.4/1.4 = 500.55 K
wc = cp (T2 − T1) = 1.005(500.55 − 300) = 201.55 kJ/kg
Process 3−4:
T3 T3 T3
T4 = = =
(g −1)/ g 0.4 /1.4 1.6685
( rp ) (6 )
 T3 
wt = cp (T3 − T4) = 1.005 T3 −  = 0.40266T3 kJ/kg
 1 .6685 
Now 0.40266 T3 = 2.5 × 201.55
or T3 = 1251.36 K
Maximum temperature is 1251.36 K.
Turbine work wt = 2.5 × 201.55 = 503.87 kJ/kg
Net work, wnet = wt − wc = 503.87 − 201.55 = 302.32 kJ/kg

M16_THERMAL ENGINEERING_SE_XXXX_CH16-Second part.indd 902 6/16/2018 12:18:59 PM


Gas Turbines 903

Heat supplied, qs = cp (T3 − T2) = 1.005(1251.36 − 500.55)


= 754.564 kJ/kg
wnet 302.32
Cycle efficiency, hth = = = 0.40 or 40%
qs 754.564

Example 16.22
A gas turbine cycle takes in air at 25°C and atmospheric pressure. The compression pressure ratio
is 4. The isentropic efficiency of the compressor is 75%. The inlet temperature to turbine is limited
to 750°C. What turbine efficiency would give overall cycle efficiency zero percent?

Solution
Given: T1 = 273 + 25 = 298 K, p1 = 1.013 bar,
rp = 4, hc = 0.75, T3 = 273 + 750 = 1023 K
Refer to Fig. 16.40,
Process 1−2:
T2 = T1 (rp)(g − 1)/g = 298(4)0.4/1.4 = 442.8 K
T2 − T1
hc =
T2′ − T1
442.8 − 298
or 0.75 =
T2′ − 298
  T2′ = 491 K
or             
Process 3−4:
T4 1 1
= = = 0.673
( g −1)/ g
T3 ( rp ) ( 4) .4 /1.4
0

T4 = 1023 × 0.673 = 688.4 K


T3 − T4′ 1023 − T4′ 1023 − T4′
ht = = =
T3 − T4 1023 − 688.4 334.6
T4′ = 1023 − 334.6 ht
Compressor work, wc = cp ( T2′ − T1) = 1.005(491 − 298) = 193.96 kJ/kg
Turbine work, wt = cp (T3 − T4′ ) = 1.005(1023 −1023 + 334.6 ht)
= 336.27ht
Net work, wnet = wt − wc = 336.27ht − 193.96
Heat supplied, qs = cp (T3 − T2′ ) = 1.005(1023 − 491) = 534.66 kJ/kg

M16_THERMAL ENGINEERING_SE_XXXX_CH16-Second part.indd 903 6/16/2018 12:19:03 PM


904 Chapter 16

wnet
Cycle efficiency, hth = =0
qs
or 336.27 ht − 193.96 = 0
or ht = 0.5768 or 57.68%

Example 16.23
In a Brayton cycle gas turbine power plant, the minimum and maximum temperatures of the cycle
are 300 K and 1200 K. The compression is carried out in two stages of equal pressure ratio with
intercooling of the working fluid to the minimum temperature of the cycle after the first stage of
compression. The entire expansion is carried out in one stage only. The isentropic efficiency of both
compressors is 0.85 and that of the turbine is 0.9. Determine the overall pressure ratio that would
give the maximum net work per kg working fluid. Derive the expression you use. g = 1.4.
[IES, 1995]

Solution
p2 p3
Given: T1 = T3 = 300 K, T5 = 1200 K, = , h = hc2 = 0.85, ht = 0.90, g  = 1.4
p1 p2 c1
The T−s diagram is shown in Fig. 16.42.

T p3
5

p2

4′ p1
4
2′
2 6′
6

3 1
s

Figure 16.42  T-s diagram for gas turbine

Compressor 1:
g −1
T2  p2  g y
= =  rp  = rp( y / 2)
T1  p1   
g −1
where y=
g

M16_THERMAL ENGINEERING_SE_XXXX_CH16-Second part.indd 904 6/16/2018 12:19:05 PM


Gas Turbines 905

T2 − T1
hc1 =
T2′ − T1

T2 − T1 T T  T
T2′ − T1 = = 1  2 − 1 = 1  rp( y / 2) − 1
hc1 hc1  T1  hc1  
Compressor 2:
y
T4  p3  T
= = rp( y / 2) = 4
T3  p2  T1

T4 − T3
hc2 =
T4′ − T3

T4 − T1 T  T1  T4 ( y / 2)
T4′ − T1 = = 1  − 1 = r − 1
hc 2 hc 2  p 
 T1  hc 2
Compressor work, wc = wc1 + wc2 = cp [( T2′ − T1) + ( T4′ − T1)]

c p T1 2c p T1
= ( r ( y / 2) − 1) + r ( y / 2) − 1 =  r ( y / 2) − 1
hc  p p  hc  p 
g −1
T p  g
Turbine: 5 =  3  = rpy
T6  p1 
T5 − T6′
ht =
T5 − T6
 T 
T5 − T6′ = ht (T5 − T6) = ht T5 1 − 6  = ht T5 1+ rp− y 
 T5   

Turbine work, wt = cp [T5 − T6] = cp ht T5 1− r − y 


 p 
Net work done, wnet = wt − wc
2c p T1 ( y / 2)
= cp ht T5 1 − rp− y  − r − 1
  hc  p 
dwnet
For maximum net work, =0
drp
 y 
2c p T1  y  2 −1 
or cp ht T5 [ y rp( − y −1) ] − ×r =0
hc  2 p 
 
y 
Tmin  −1
or ht Tmax × rp−( y +1) − ×rp 2  = 0
hc

M16_THERMAL ENGINEERING_SE_XXXX_CH16-Second part.indd 905 6/16/2018 12:19:13 PM


906 Chapter 16

y 
 −1
rp 2  T 
=  max  × hc ht
d −p ( y +1)  Tmin 

 3y
  Tmax
or rp 2  = hc ht
Tmin
3 y 3  g − 1 3(g − 1)
Now =  =
2 2  g  2g
2g
 T   3(g −1)
∴ (rp)opt = hc ht  max  
  Tmin  
2 ×1.4
 120  3(1.4 −1)
= 0.85 × 0.90 × = (3.06)2.333 = 13.594
 300 

Example 16.24
In an open-cycle gas turbine plant, air enters at 15°C and 1 bar, and is compressed in a compressor
to a pressure ratio of 15. The air from the exit of compressor is first heated in a heat exchanger
which is 75% efficient by turbine exhaust gas and then in a combustor to a temperature of 1600 K.
The same gas expands in a two-stage turbine such that the expansion work is maximum. The
exhaust gas from HP turbine is reheated to 1500 K and then expands to LP turbine. The isentropic
efficiencies of compressor and turbine may be taken as 86% and 88%, respectively. The mechanical
efficiencies of compressor and turbine are 97% each. The alternator efficiency is 98%. The output
of turbo-alternator is 250 MW. Sketch the system and show the process on T-s diagram, Determine
(a) the cycle thermal efficiency, (b) the work ratio, (c) the specific power output, and (d) the mass
flow rate of air. [IES, 2007]

Solution
The schematic is shown in Fig. 16.43(a) and T-s diagram in Fig. 16.43(b).
(a) hc = 0.86, ht = 0.88, halt = 0.98, (hmech)c = (hmech)t = 0.97
(g −1)/g
     T2 = T1  p2 
 p 
1
1
 = 288 (15) .5
3 = 624.3 K

T2 − T1 624.3 − 288
T2′ = T1 + = 288 + = 679.1 K
hc 0.86

M16_THERMAL ENGINEERING_SE_XXXX_CH16-Second part.indd 906 6/16/2018 12:19:19 PM


Gas Turbines 907

8 7′
Exhaust
2′
C.C 5′
3 4 C.C
H.E.
6

C HP LP ∼
250 MW
T1 T2
1
15°C
1 bar
(a)

p2
T pi
4
1600 K 6 p1
1500 K
3 5
2′ 5′
7′
679.1 K 2 3′ 7

288 K
1
s
(b)

Figure 16.43  Open cycle gas turbine: (a) Schematic, T-s diagram

Compressor work,
c p (T2′ − T1 ) 1.005 (679.1 − 288)
wc = = = 405.2 kJ/kg
(h mech )c 0.97
For maximum expansion work,

pi = p1 p2 = 1 × 15 = 3.873 bar
p5 = pi = 3.873 bar
(g −1)/g 1
p   3.873  3.5
T5 = T4  5  = 1600  = 1086.7 K
 p4   15 
T4 − T5′
ht1 =
T4 − T5
or T5′ = T4 − ht1 (T4 − T5)
= 1600 − 0.88 (1600 − 1086.7) = 1158.5 K
(g −1)/g
p 
T7 = T6  7 
 p6 

M16_THERMAL ENGINEERING_SE_XXXX_CH16-Second part.indd 907 6/16/2018 12:19:27 PM


908 Chapter 16

p7 = p1 = 1 bar, p6 = pi = 2.873 bar


1
 1  3.5
T7 = 1500  = 1018.8 K
 3.873 

T4 − T7′
ht2 =
T6 − T7
or T7′ = 1500 − 0.88 (1500 − 1018.8) = 1076.5 K
Turbine work,
wt = wt1 + wt2
= 1.005 [(1600 − 1158.5) + (1500 − 1076.5)] × 0.97
= 843.245 kg
Net work, wnet = wt − wc
= 843.245 − 405.2 = 438.045 kJ/kg
T3′ − T2′
Heat exchanger effectiveness =
T7′ − T2′

T3′ − 679.1
or   0.75 =
1076.5 − 679.1
or    T3′ = 977.15 K
Heat supplied = 1.005 [(1600 − 977.15) + (1500 − 1158.5)] = 969.17 kJ/kg
Thermal efficiency,

wnet 438.045
hth = = = 0.452 or 45.2%
qs 969.17

wnet 438.045
(b) Work ratio = = = 0.5195.
wt 843.245

(c) Specific power output


1 × wnet = 1 × 438.045 = 438.045 kW

MW ×halt
(d) Mass flow rate of air =
wnet
250 × 103 × 0.98
= = 559.3 kg/s
438.045

M16_THERMAL ENGINEERING_SE_XXXX_CH16-Second part.indd 908 6/16/2018 12:19:33 PM


909
Gas Turbines

Example 16.25
A gas turbine utilises a two-stage centrifugal compressor. The pressure ratios for the first and second
stages are 2.5 and 2.1, respectively. The flow of air is 10 kg/s, this air being drawn at 1.013 bar and
20°C. If the temperature drop in the intercooler is 60°C and the isentropic efficiency is 90% for
each stage, calculate:(a) the actual temperature at the end of each stage and (b) the total compressor
power. Assume g = 1.4 and cp= 1.005 kJ/kgK for air. [IES, 2012]

Solution
Given: rp1 = 2.5, rp2 = 2.1, m a = 10 kg/s, p1 = 1.013 bar, T1 = 20 + 273 = 293 K, ∆T2 = 60°C,
hi1 = hi2 = 0.9, g = 1.4, cp = 1.005 kJ/kgK.
The T–s diagram is shown in Fig. 16.44.
g −1 g −1 0.4
T p  g g
(a)  2 =  2  = ( rp1 ) = ( 2.5) .4
1 = 1.299
T1  p1 

or T2 = 380.68 K
T −T
hi1 = 2 1
T2′ − T1
380.68 − 293
or  0.9 =
T2′ − 293
or T2′ = 390.42 K or 117.42°C
Temperature at the end of first stage is 117.42°C.
Temperature at the inlet of second stage
= 117.42 – 60 = 57.42°C
T3 = 57.42 + 273 = 330.42 K
g −1 g −1 0.4
T4  p3  g g
= = ( rp 2 ) = ( 2.1) 1.4 = 1.236
T3  p2 

T p3

4′
4 p2

2 p1
3 2′

1
s
Figure 16.44  T-s diagram

M16_THERMAL ENGINEERING_SE_XXXX_CH16-Third part.indd 909 6/16/2018 12:21:05 PM


910 Chapter 16

or T4 = 408.44 K
T4 − T3
hi2 =
T4′ − T3

408.44 − 330.42
0.9 =
T4′ − 330.42
Temperature at the end of second stage,
T4′ = 417.11 K or 144.1°C
(b)  Pc1 = m acp ( T2′ – T1) = 10 × 1.005 (390.42 – 293) = 979.07 kJ/s

Pc2 = m acp ( T4′ – T3) = 10 × 1.005 (417.11 – 330.42) = 877.23 kJ/s


Total compressor power,
Pc = Pc1 + Pc2 = 979.07 + 871.23 = 1850.3 kW

Example 16.26
An open cycle gas turbine takes in air at 300 K and 1 bar and develops a pressure ratio of 20. The
turbine inlet temperature is 1650 K. The polytropic efficiency of the compressor and the turbine
each is 90%. The pressure loss in the combustor is 3% and the alternator efficiency is 97%. Take
cpa= 1.005 kJ/kgK and cpg = 1.128 kJ/kgK for air and gas, respectively. The calorific value of fuel
is 42 MJ/kg. Determine the overall efficiency, the specific power output, the fuel to air ratio and the
specific fuel consumption. [IAS, 2010]
Solution
Given: T1 = 300 K, p1 = 1 bar, rp = 20, T3 = 1650 K, hc = ht = 0.90, ∆p2 = 3%, halt = 0.97,
cpa = 1.005 kJ/kgK, cpg = 1.128 kJ/kgK, C.V. = 42 MJ/kg
The T–s diagram is shown in Fig. 16.45.
Compressor process 1–2′:
g −1 g −1 0.4
T2  p2  g
= = rp g = ( 20) 1.4 = 2.3535
T1  p1 
or T2 = 300 × 2.3535 = 706 K
T2 − T1
hc =
T2′ − T1
706 − 300
or 0.9 =
T2′ − 300
or T2′ = 747.8 K
p2 = 20 × 1 = 20 bar
p3 = 0.97 × 20 = 19.4 bar
Combustion process 2′–3:
m f × CV = m f × cpa (T3 – T2′ ) + m f × cpg (T3 – T2′ )
= ( m a × cpa + m f × cpg) (T3 – T2′ )

M16_THERMAL ENGINEERING_SE_XXXX_CH16-Third part.indd 910 6/16/2018 12:21:11 PM



Gas Turbines 911

T p2
p3
3
2′
2

p1
4′
4

1
s

Figure 16.45   T-s diagram

or m f × 42 × 103 = ( m a × 1.005 + m f × 1.128) (1650 – 747.8)


= 906.7 m a + 1017.7 m f
or 40982.3 m f = 906.7 m a
m a
= 45.2
m f
Turbine process 3–4′:
T3
T4 =
g g −1
 19.4  gg
 
1 
c pg
gg = , R = cpg – cvg, cvg = cpg – R = 1.128 – 0.287 = 0.841 kJ/kgK
cvg

1.128
gg = = 1.34
0.841
1650
T4 = = 777.5 K
0.34
(19.4) .34
1

T3 − T4′
ht =
T3 − T4

1650 − T4′
or   0.9 =
1650 − 777.5

or T4′ = 864.75 K
Specific power output:
wnet = wt – wc = cpg (T3 – T4′ ) – cpa ( T4′ – T1)

M16_THERMAL ENGINEERING_SE_XXXX_CH16-Third part.indd 911 6/16/2018 12:21:18 PM


912 Chapter 16

= 1.128 (1650 – 864.75) – 1.005 (747.8 – 300)


= 435.72 kJ/kg of air
wnet 435.72 435.72
Overall efficiency, hoverall = = = = 0.4281 or 42.81%
qs c pg (T3 − T2′ ) 1.128(1650 − 747.8)

 3600 
specific air consumption  w 
net
Specific fuel consumption = =
(A/F) ( A/F)
(3600 / 435.72)
= = 0.1828 kg/kWh
45.2

Summary for Quick Revision


1. Gas turbines may be classified as: (i) constant pressure combustion (a) open cycle type (b) closed cycle type
and (ii) constant volume combustion
2. Brayton cycle:
(a) Compressor work, wc = cp (T2 – T1)
(b) Heat supplied, qs = cp (T3 – T2)
(c) Turbine work, wt = cp (T3 – T4)
(d) Net work, wnet = wt – wc  g −1
w  
(e) Thermal efficiency, hth = net = 1 − rp  g 
qs
w
(f) Specific output = net
c p T1
(g) For maximum specific output, T2 = T4
2 2
(h) Maximum work output, (wnet)max = cp  T3 − T1  = c p  Tmax − Tmin 
g −1
w T
(i) Work ratio, Rw = net = 1 − 1 ( rp ) g
wt T3
(j) Optimum pressure ratio for maximum specific work output,
g
 T  2(g −1)
(k) rp =  3 
 T1 
g
 T  g −1
( )opt
and rp =  3
 T1 
∴ rp = ( rp )opt
3. Cycle operation with machine efficiencies:
(a) Optimum pressure ratio for maximum specific work output
T2 − T1
Isentropic efficiency of compressor, hc =
T2′ − T1

M16_THERMAL ENGINEERING_SE_XXXX_CH16-Third part.indd 912 6/16/2018 12:21:24 PM



Gas Turbines 913

T3 − T4′
Isentropic efficiency of turbine, ht =
T3 − T4

 For specific work output to be maximum for given temperature limits,


g
  T   2(g −1)
 Pressure ratio, ( rp )opt = hcht  3  
  T1  

(b)  Optimum pressure ratio for maximum cycle efficiency,
g
  2(g −1)
 
 (T3 / T1 ) 
( rp )opt = 1/ 2 
   1  
1 + {(T3 / T1 ) − 1}  − 1  
  hcht − 1   

4. Open cycle constant pressure gas turbine:


 a cpa (T2′ − T1 )
Wc = m

 a + m f) cpg (T3 − T4′ )


 Wt = ( m
      Wnet = wt – wc
 f × C.V.
Q= m

( m a + m f )c pg (T3 − T4′ ) − m ac pa (T2′ − T1 )


         hth =
m f × C.V.

( m a + m f )c pght (T3 − T4 ) − ( m a c pa / hc )(T2 − T1 )


 =
m f × C.V.

5. The thermal efficiency of open cycle gas turbine can be improved by: Regeneration, intercooling, reheating,
and combination of above.
 (a)  Regeneration.
(i)  Without machine efficiencies: T5 = T4
g −1
c g T3
         hth = 1 – a
, c = ( rp ) ,a=
T1
 1 w
    
net
= a 1 −  − (c − 1)
c p T1  c

(ii)  With machine efficiencies.


c
         hth = 1 –
a h c ht

wnet  1  (c − 1)
= ht a
c p T1 1 −  −
c hc

M16_THERMAL ENGINEERING_SE_XXXX_CH16-Third part.indd 913 6/16/2018 12:21:38 PM


914 Chapter 16

m a c pa (T5′ − T2′ ) T4′ = T5 


(iii) Effectiveness of regenerator, e =  
m g c pg (T4′ − T2′ ) T5′ < T5 
(b) Intercooling:
For minimum compressor work.

wc = cpa ( T2′ – T1) + cpa( T4′ – T3)

Intermediate pressure for minimum compressor work,


g
 T h  2(g −1)
pi =  3 × c1  × p1 p2
 T1 hc 2 

For T3 = T1 and hc1 = hc2, pi = p1 p2

 g −1 
2c paT1  p2  2g 
(wc)min = − 1
hc  p1  

 
(c) Reheating: For ht1 = ht2 = ht and T5 = T3, pi = p1 p2 for maximum wt.

 g −1 
  p1  2g 
(wc)max = 2cpg ht T3 1 −   
  p2  
 

Multiple-choice Questions

1. Consider the following statements: (a) (p1p4)1/2 and (p1p2)2/3


I. Intercooling is effective only at lower pressure
( ) ( )
2/3 1/ 2
ratios and high turbine inlet temperatures. (b) p12 p4 and p1 p 24
II. There is very little gain in thermal efficiency
when intercooling is used without the benefit
of regeneration. (c) p1 p4 and p1 p4
III. With higher value of g and cp of the working
(c) ( p1 p4 )1/ 2 and ( p1 p4 ) 2 / 3
fluid, the net power output the Brayton cycle
will increase. 3. Intercooling in gas turbines
Of these statements, (a) decreases net output but increases thermal
efficiency
(a) I, II, and III are correct
(b) increases net output but decreases thermal
(b) I and II are correct
efficiency
(c) I and III are correct
(c) decreases both net output and thermal effi-
(d) II and III are correct ciency
2. In a gas turbine cycle with two stages of reheat- (d) increases both net output and thermal effi-
ing, working between maximum pressure p1 and ciency
minimum pressure p4 the optimum reheat reas- 4. Figure 16.46 shows four plots A, B, C, and D of
sure would be thermal efficiency against pressure ratio.

M16_THERMAL ENGINEERING_SE_XXXX_CH16-Third part.indd 914 6/16/2018 12:21:45 PM



Gas Turbines 915

A (a) Brayton (b) Rankine


(c) Stirling (d) Otto

B 10. Reheating in a gas turbine


hth CD (a) increases the compressor work
(b) decreases the compressor work
(c) increase the turbine work
(d) decreases the turbine work
s 11. Consider the following statements relating to a
rp (Pressure ratio)
closed gas turbine cycle:
Figure 16.46     I. The cycle can employ monatomic gas like
The curve which represents that of a gas turbine helium instead of air to increase the cycle
plant using Brayton cycle (without regeneration) efficiency if other conditions are the same.
is the one labelled   II. The efficiency of heat exchanger increases
(a) A (b) B (c) C (d) D with the use of helium.
III. The turbine blades suffer higher corrosion
5. The optimum intermediate pressure pi for a gas damages.
turbine plant operating between pressure limits p1 IV. Higher output can be obtained for the same
and p2 with perfect intercooling between the two size.
stages of compression (with identical isentropic Which of these statements are correct?
efficiency) is given by (a) I, II, and III (b) I, II, and IV
1
(a) p1 = p2 – p1 (b) p1 = ( p1 + p2 ) (c) II, III, and IV (d) I, III, and IV
2
12. The efficiency of a simple gas turbine can be
(c) p1 = p1 p2 (d) p1 = p22 p12 improved by using a regenerator, because the
6. In a gas turbine cycle, the turbine output is 600 kJ/ (a) work of compression is reduced
kg, the compressor work is 400 kJ/kg and the heat (b) heat required to be supplied is reduced
supplied is 1000 kJ/kg. The thermal efficiency of (c) work output of the turbine is increased
this cycle is: (d) heat rejected is increased
(a) 80% (b) 60% (c) 40% (d) 20% 13. In a regenerative feed heating cycle, the economic
7. In a single-stage open-cycle gas turbine, the mass number of the stages of regeneration
flow through the turbine is higher than the mass (a) increases as the initial pressure and tempera-
flows through compressor, because ture increase
(a) the specific volume of air increases by use of (b) decreases as the initial pressure and tempera-
intercooler ture increase
(b) the temperature of air increases in the reheater (c) is independent of the initial pressure and tem-
(c) the combustion of fuel takes place in the perature
combustion chamber (d) depends only on the condenser pressure
(d) the specific heats at constant pressure for 14. Match List I (Blades) with List II (Features) and
incoming air and exhaust gases are different select the correct answer using the codes given
8. A gas turbine develops 120 kJ of work while the below the Lists:
compressor absorbed 60 kJ of work and the heat
supplied is 200 kJ. If a regenerator which would List I List II
recover 40% of the heat in the exhaust were used, (Blades) (Features)
then the increase in the overall thermal efficiency A. Ceramic blades 1. High creep strength
would be: B. Steam turbine blades 2. Forged and machined
(a) 10.2% (b) 8.6%
(c) 6.9% (d) 5.7% C. Alloy steel blades 3. Precision cast
D. Compressor blades 4. Thick at mid chord
9. A gas turbine works on which one of the follow-
ing cycles? 5. Thin trailing edge

M16_THERMAL ENGINEERING_SE_XXXX_CH16-Third part.indd 915 6/16/2018 12:21:47 PM


916 Chapter 16

  Codes:  g   g 
T   T  
  A B C D (a) 1 − 1 rp g −1 (b) 1 − 3 rp g −1
(a)  
2 1 5 4 T3 T1
(b)  
3 4 5 1
(c)  
2 4 3 5  g −1  g −1
(d)  
3 2 1 5 T3  g  T1  g 
(c) 1 − rp (d) 1 − rp
T1 T3
15. Consider the following statements about modifi-
cation in a gas turbine power plant working on a 19. Consider the following features for a gas turbine
simple Brayton cycle: plant:
   I. Incorporation of the regeneration process       I. Intercooling
increases specific work output as well as ther-    II. Regeneration
mal efficiency. III. Reheat
   II. Incorporation of regeneration process increases Which of the above features in a simple gas tur-
thermal efficiency but specific work output bine cycle increase the work ratio?
remains unchanged. (a) I, II, and III (b) Only I and II
III. Incorporation of intercooling process in a (c) Only II and III (d) Only I and IV
multi-stage compression increases specific
work output but the heat input also increases. 20. Consider the following statements:
IV. Incorporation of intercooling process in a For a large aviation gas turbine, an axial flow com-
multi-stage compression system increases pressor is usually preferred over centrifugal com-
specific work output, the heat addition pressor because
remains unchanged.      I. the maximum efficiency is higher
Which of the above statements are correct?    II. the frontal area is lower
(a) 1 and 3 (b) 1 and 4 III. the pressure rise per stage is more
(c) 2 and 3 (d) 2 and 4  IV. the cost is lower
Which of the statements given above are correct?
16. A power plant, which used a gas turbine followed
(a) I and IV (b) Only I and II
by steam turbine for power generation, is called
(c) I, II, and III (d) II, III, and IV
(a) Topping cycle
(b) Bottoming cycle 21. In a reaction turbine, the heat drop in fixed blade is
(c) Brayton cycle 8 kJ/kg and the total heat drop per stage is 20 kJ/
(d) Combined cycle kg. The degree of reaction is
(a) 40% (b) 66.7%
17. Consider the following statements:
(c) 60% (d) 25%
  I. The speed of rotation of the moving elements
of gas turbines is much higher than those of 22. Consider the following statements:
steam turbines. Which of the following increase the work ratio in
   II. Gas turbine plants are heavier and larger in a simple gas turbine plant?
size than steam turbine plants.   I. Heat exchanger
III. Gas turbines require cooling water for its    II. Inter cooling
operations. III. Reheating
 IV. Almost any kind of fuel can be used with gas Select the correct answer using the code given
turbines. below:
Which of the statements given above are correct? (a) 1 and 2 (b) 2 and 3
(a) I and II (b) I and III (c) 1 and 3 (d) 1, 2, and 3
(c) I and IV (d) III and IV
23. Optimum pressure ratio for maximum specific
18. The thermal efficiency of a gas turbine cycle with
output for ideal gas turbine plant operating at ini-
regeneration in terms of T3 (maximum tempera-
tial temperature of 300 K and maximum tempera-
ture), T1 (minimum temperature), rp (pressure
ture of 1000 K is closer to
 cp  (a) 4 (b) 8 (c) 12 (d) 16
ratio) and g  =  is given by
 cu 

M16_THERMAL ENGINEERING_SE_XXXX_CH16-Third part.indd 916 6/16/2018 12:21:50 PM


Gas Turbines 917

24. The cycle shown in Fig. 16.47 represents a gas Codes:


turbine cycle with intercooling and reheating. A B C D
5
(a) I II IV III
7 (b) II IV V III
4 6 (c) III IV V II
T 8
2 (d) II V IV I
3
1 25. Which one of the following is the correct state-
ment? In a two-stage gas turbine plant with inter-
s cooling and reheating,
Figure 16.47 (a) both work ratio and thermal efficiency improve
(b) work ratio improves but thermal efficiency
Match List-X (Units) with List-Y (Processes) and
decreases
select the correct answer using the codes given
(c) thermal efficiency improves but work ratio
below the Lists:
decreases
List-X List-Y (d) both work ratio and thermal efficiency decrease
A. Intercooler I. 1–2
B. Combustor II. 2–3
C. Reheater III. 3–4
D. High pressure IV. 4–5
compressor
V. 6–7

Explanatory Notes
wt − wc 600 − 400 200
6. (a) Thermal efficiency = = = = 0.2 or 20%
q 1000 1000
wt − wc 120 − 60 60
8. (d) Thermal efficiency = = = = 0.3 or 30%
q 200 200
Heat in exhaust gas = 200 – 120 = 80 kJ
Heat recovered in regenerator = 80 × 0.4 = 32 kJ
Heat supplied = 200 – 32 = 168 kJ
120 − 60 60
New thermal efficiency = = = 0.357 or 35.7%
168 168
Increase in efficiency = 35.7 – 30 = 5.7%
( ∆h) m ( ∆h) st − ( ∆h) f
21. (c) Degree of reaction = =
( ∆h) st ( ∆h) st
20 − 8 12
= = = 0.6 or 60%
20 20
23. (b) Optimum pressure ratio for maximum specific output of a gas turbine,
g 1.4
 T  2(g −1) 1000  0.8
rp =  2  =  = 8.22  8
 T1   300 

M16_THERMAL ENGINEERING_SE_XXXX_CH16-Third part.indd 917 6/16/2018 12:21:53 PM


918 Chapter 16

Review Questions
1. List the fields of application of gas turbines.
2. What are the limitations of a gas turbine?
3. List four advantages of gas turbine over an IC engine.
4. List four disadvantages of gas turbine over an IC engine.
5. Enumerate five advantages of gas turbine over steam turbine.
6. How do you classify gas turbines?
7. Make a comparison of open and closed cycle gas turbines.
8. Explain the position of the gas turbine in the power industry.
9. Write the formula for the thermal efficiency of the Brayton cycle.
10. Define specific output and work ratio.
11. Write the expressions for maximum work output and work ratio of the Brayton cycle.
12. Draw the schematic arrangement of open cycle constant pressure combustion gas turbine.
13. What do you understand by regeneration?
14. Define effectiveness of a regenerator.
15. What is the intermediate pressure for intercooling to have minimum compressor work?

Exercises

16.1 A Brayton cycle operates with air between 1 bar, 300 K and 5 bar, 1000 K. The air is compressed in two
stages with perfect intercooling. Similarly, in the turbine, expansion occurs in two stages with perfect
reheating. Calculate the optimum intermediate pressure, net work output, and the fraction of turbine out-
put that has to be put back to the compressor. [Ans. 2.236 bar, 257.3 kJ/kg, 0.378]
16.2 A gas turbine unit operates at a mass flow of 30 kg/s. Air enters the compressor at a pressure of 1 bar, 288 K
and is discharged from the compressor at a pressure of 10.5 bar. Combustion occurs at constant pressure
and results in a temperature rise of 420 K. If the flow leaves of the turbine at a pressure of 1.2 bar, deter-
mine the net power output from the unit and also the thermal efficiency. [Ans. 5338.74 kW, 42.55%]
16.3 In a regenerative gas turbine cycle, air enters the compressor at a temperature and pressure of 30°C and
1.5 bar and discharges at 220°C and 5.2 bar. After passing through the regenerator, the air temperature
is 395°C. The temperature of air entering and leaving the gas turbine are 900°C and 510°C, respectively.
Assuming no pressure drop through the regenerator, determine (a) the output per kg of air, (b) the effi-
ciency of cycle, and (c) the work required to drive the compressor. Take cp = 1.005 kJ/kgK.
[Ans. 201 kJ/kg, 39.6%, 190.95 kJ/kg]
16.4 A Brayton cycle works between 1 atm, 27°C and 5 atm, 977°C. There are two stages of compression with
perfect intercooling and two stages of expansion. The work output of first expansion stage being used to
drive the two compressors, where the interstage pressure is optimised for the compressor. The air from
the first stage turbine is again heated to 977°C and expanded. Calculate the power output of free power
turbine and cycle efficiency without and with a perfect heat exchanger and compare them. Calculate the
percentage improvement in the efficiency because of the addition if heat exchangers.
[Ans. 350.8 kJ/kg, 33.96%, 69.2%, 50.9%]
16.5 The following data apply to a gas turbine set employing a separate power turbine, regenerator, and inter-
cooler between two-stage compression:
Isentropic efficiency of compression in each stage = 80%
Isentropic efficiency of compressor turbine = 88%

M16_THERMAL ENGINEERING_SE_XXXX_CH16-Third part.indd 918 6/16/2018 12:21:53 PM



Gas Turbines 919

     Isentropic efficiency of power turbine = 88%


     Turbine to compressor transmission efficiency = 98%
     Pressure ratio in each stage of compression = 3:1
     Temperature after intercooler = 297 K
     Air mass flow rate = 15 kg/s
     Regenerator effectiveness = 80%
     Regenerator gas-side pressure loss = 0.1 bar
     Maximum turbine temperature = 1000 K
     Ambient temperature = 327 K
     Ambient pressure = 1 bar
     Calorific value of fuel = 43.1 MJ/kg
     Calculate (a) the net power output, (b) specific fuel consumption, and (c) overall thermal efficiency. Assume
that the pressure losses in the air-side of the regenerator and combustion chamber are accounted for in the
compressor efficiency. Take cpa = 1.005 kJ/kgK, cpa = 1.147 kJ/kgK, g a = 1.4 and g g = 1.33.
[Ans. 1811.67 kW, 0.32 kg/kWh, 26.1%]
  16.6 In a gas turbine plant, air is compressed from 1 bar, 15°C through a pressure ratio of 4:1. It is then heated
to 650°C in a combustion chamber and expanded back to atmospheric pressure of 1 bar in a turbine. Cal-
culate the cycle efficiency and the work ratio if a perfect heat exchanger is used. The isentropic efficiencies
of turbine and compressor are 85% and 80%, respectively. Take cpa = 1.00 kJ/kgK.
[Ans. 31.6%, 0.316]
  16.7 In a gas turbine plant, the pressure ratio through which air at 15°C is compressed is 6. The same air is then
heated to a maximum permissible temperature of 750°C, first in a heat exchanger having effectiveness of
75% , and then in the combustion chamber. The same air at 750°C is expanded in two stages such that the
expansion work is maximum. The air is reheated to 750°C after the first stage. Determine (a) the cycle
thermal efficiency, (b) the work ratio, and (c) the net shaft work per kg of air. The machine efficiencies may
be assumed to be 80% and 85% for the compressor and turbine, respectively. Take cpa = 1.00 kJ/kgK.
[Ans. 33.6%, 0.398, 160 kJ/kg]
  16.8 In a gas turbine plant, the air at 283 K and 1 bar is compressed to 4 bar with compression efficiency of 80%.
The air is heated in the regenerator and in the combustion chamber till its temperature is raised to 973 K,
and during the process, the pressure falls by 0.1 bar. The air is then expanded in the turbine and passes
to the regenerator which has 75% effectiveness and causes a pressure drop of 0.14 bar. If the isentropic
efficiency of the turbine is 85%, determine the thermal efficiency of the plant. [Ans. 23.15%]
  16.9 A gas turbine works on Brayton cycle between 27°C and 827°C. Determine the maximum net work per kg
and the cycle efficiency. Take cpa = 1.005 kJ/kgK. [Ans. 252.34 kJ/kg, 47.77%]
16.10 Atmospheric air enters the compressor of an open cycle constant pressure gas turbine at a pressure of
1 bar, 293 K. The pressure of air after compression is 4 bar. The isentropic efficiencies of compressor
and turbine are 80% and 85%, respectively. The air-fuel ratio used is 90:1. If the mass flow rate of air is
3 kg/s, then calculate (a) the power developed and (b) the thermal efficiency of the cycle. Assume
cpa = cpg = 1.0 kJ/kgK and g a = g g = 1.4. Calorific value of fuel = 41800 kJ/kg.
[Ans. 252.84 kW, 18.14%]
16.11 In a constant pressure open cycle gas turbine, air enters at 1 bar, 20°C and leaves the compressor at 5 bar.
Temperature of gases entering the turbine is 680°C, pressure loss in the combustion chamber is 0.1 bar,
hc = 0.85, ht = 0.80, hcombustion = 0.85, g a = 1.4 and cpa = cpg = 1.024 kJ/kg.K. Calculate (a) the quantity
of air circulated if power developed is 1065 kW, (b) the heat supplied per kg of air circulated, and (c) the
thermal efficiency of the cycle. Neglect the mass of fuel as compared to that of air.
[Ans. 13.43 kg, 552.9 kJ/kg, 14.34%]

M16_THERMAL ENGINEERING_SE_XXXX_CH16-Third part.indd 919 6/16/2018 12:21:53 PM


920 Chapter 16

16.12 An open cycle constant pressure gas turbine draws air at 1.01 bar, 15°C and pressure ratio of 7:1. The
compressor is driven by the H.P. turbine and L.P. turbine drives a separate power shaft. The isentropic
efficiencies of compressor, H.P. and L.P. turbines are 0.82, 0.85, and 0.85, respectively. If the maximum
cycle temperature is 610°C, calculate (a) the pressure and temperature of gases entering the power turbine,
(b) the net power developed (c) the work ratio, and (d) the thermal efficiency of the plant. Assume
cpa = 1.005 kJ/kgK, g a = 1.4; cpg = 1.15 kJ/kgK, g g = 1.333. Neglect mass of fuel.
[Ans. 1.636 bar, 654.4 K; 72.1 kW; 0.25; 18.8%]
16.13 A gas turbine employs a heat exchanger with a thermal ratio of 0:72. The turbine operates between 1.01 bar
and 4.04 bar and ambient temperature is 20°C. The isentropic efficiencies of the compressor and turbine
are 80% and 85%, respectively. The pressure drop on each side of the heat exchanger is 0.05 bar and in
the combustion chamber 0.14 bar. Assume hcombustion = 1.0 and. C.V. of fuel = 41800 kJ/kg. Calculate the
increase in thermal efficiency due to heat exchanger.
16.14 A gas turbine has a pressure ratio of 6:1 and a maximum cycle temperature of 600°C. The isentropic
efficiencies of the compressor and turbine are 82% and 85%, respectively. Calculate the power output
when the air enters the compressor at 15°C at the rate of 15 kg/s. Take cpa = 1.005 kJ/kgK, g a = 1.4;
cpg = 1.11 kJ/kgK, gg = 1.333. [Ans. 920 KW]
16.15 A gas turbine plant operates between the temperatures of 1200 K and 320 K and corresponding pressures
of 6 bar and 1.2 bar. The ratio of turbine and compressor efficiencies is 0.95. Determine the two efficiencies
if the plant operates at optimum pressure ratio for maximum specific output. Assume g = 1.4.
[Ans. ht = 79.72%, hc = 83.90%]
16.16 The maximum and minimum temperatures occurring in a closed cycle gas turbine plant are 927°C and
37°C. If the pressure at the outlet and the inlet of the compressor are 5 bar and 1 bar, respectively. Deter-
mine the compressor work, the turbine work, the heat supplied to the cycle, and the net work done in the
cycle. Also determine the thermal efficiency of the cycle and the optimum pressure ratio for maximum
power output for the given temperatures. Assume g   for air = 1.4 and cp = 1.005 kJ/kgK.
16.17 An open cycle constant pressure gas turbine plant operates with a pressure ratio of 6.5. The temperatures at
the inlet to the compressor and turbine are 17°C and 820°C, respectively. Assume: specific heat at constant
pressure of air and gas to be 1.005 kJ/kgK and 1.079 kJ/kgK, respectively; ratio of specific heats for air and
gas to be 1.4; calorific value of fuel 44350 kJ/kg; efficiency of compressor and turbine to be 0.86 and 0.92,
respectively. Calculate: (i) the power of the plant for air circulation of 6 kg/s, (ii) the thermal efficiency of
plant, (iii) the air-fuel ratio, and (iv) the specific fuel consumption. Take the mass of the fuel into account.
16.18 A gas turbine power plant works between pressures of 1 bar and 5 bar and temperatures of 285 K and 1100 K.
The intercooler cools the air at 2.3 bar to 285 K before the air is sent to the second stage compressor. The
compressor air from the second-stage compressor passes through a regenerator having effectiveness of 0.72
and then through the combustion chamber. The heated air is then expanded in a high pressure turbine to 2.3
bar and is then reheated to 1100 K. The air is finally expanded in the low-pressure turbine to 1 bar. Assum-
ing the compressor and turbine efficiencies to be 85%, determine: (i) the ratio of compression work to the
turbine work, (ii) the power developed for an air flow of 3 kg/s, (iii) the thermal efficiency of the cycle,
(iv) the heat rejected per second to the cooling water in the intercooler, and (v) the heat rejected per second
to the atmosphere. Assume that all the components are mounted on the same shaft. Sketch the flow dia-
gram of the turbine and represent the process on T-s plane. Also, assume cp = 1.005 kJ/kg.K and g = 1.4.
16.19 A gas turbine set takes in air at 27°C and 1 atm. The pressure ratio is 4 and the maximum temperature is
560°C. The compressor and turbine efficiencies are 83% and 85%, respectively. If the regenerator effec-
tiveness is 0.75, determine the overall efficiency.

M16_THERMAL ENGINEERING_SE_XXXX_CH16-Third part.indd 920 6/16/2018 12:21:53 PM



Gas Turbines 921

ANSWERS TO MULTIPLE-CHOICE QUESTIONS


    1. (d) 2. (a) 3. (b) 4. (b) 5. (c)
     6. (d) 7. (c) 8. (d) 9. (a) 10. (c)
11. (b) 12. (b) 13. (a) 14. (a) 15. (d)
16. (d) 17. (c) 18. (d) 19. (d) 20. (b)
21. (c) 22. (b) 23. (b) 24. (b) 25. (b)

M16_THERMAL ENGINEERING_SE_XXXX_CH16-Third part.indd 921 6/16/2018 12:21:53 PM


Chapter 17

Jet Propulsion

17.1 ❐ PRINCIPLE OF JET PROPULSION


Jet propulsion is the thrust produced by passing a jet of fluid in the direction opposite to the motion of
the object. The propulsive thrust in jet propulsion is obtained by the reaction of the engine produced
by the exit of high velocity fluid stream from the nozzle. In case of a jet engine, the atmospheric air
is drawn into the engine and compressed in a compressor driven by a gas turbine. The compressed air
is used in the combustion chamber of the gas turbine and is partly expanded to extract enough work
to drive the air compressor. The balance expansion takes place in the nozzle and the high velocity
exhaust from the nozzle provides the thrust, which is used to propel the vehicles.

17.2 ❐ JET PROPULSION SYSTEMS


Jet propulsion systems may be classified as follows:
1. Air stream jet engines: In these engines, the oxygen necessary for combustion is taken from the
atmosphere. They are also called air-breathing engines. They may be classified into the following:
(a) Screw propeller
(b) Ramjet engine
(c) Pulse jet engine
(d) Turbojet engine
(e) Turboprop engine
2. Self-contained rocket engines: In a rocket engine, the full and the oxidiser are contained in the
propelling body and thus can function in vacuum as well. They are also called non-air breathing
engines. They may be classified into the following:
(a) Liquid propellant
(b) Solid propellant

17.2.1   Screw Propeller


The line diagram of a screw propeller is shown in Fig. 17.1(a). The schematic arrangement and the
corresponding T-s diagram are shown in Fig. 17.1(b) and Fig. 17.1(c), respectively. In this system,

M17_THERMAL ENGINEERING_SE_XXXX_CH17.indd 922 6/16/2018 11:52:32 AM


Jet Propulsion 923

Air from
atmosphere

Direction
of aircraft

Propeller Aircraft Exhaust gases


(a) from turbine

C.C T
Combustion p2
2 3 3
chamber
p1
2 2′ 4′
Compressor Turbine 4
1 4
1
Air from Exhaust gases to
atmosphere atmosphere s
(b) (c)

Figure 17.1  Screw propeller: (a) Line diagram, (b) Schematic diagram, (c) T-s diagram

full expansion takes place in the turbine, and the total power developed by the turbine is used to drive
the compressor and the propeller. The power supplied to the propeller is controlled by controlling the
supply of fuel in the combustion chamber. The rate of increase in efficiency of the screw propeller is
higher at lower speed but its efficiency falls rapidly at higher speed. It has high power for take-off and
higher propulsion efficiency at speed below 600 km/h.

17.2.2   Ramjet Engine


The line diagram of a ramjet engine is shown in Fig. 17.2(a) and the corresponding T-s diagram is
shown in Fig. 17.2(b). It consists of a supersonic diffuser, subsonic diffuser, combustor, and nozzle.
Both supersonic and subsonic diffusers convert the kinetic energy of the entering air into pressure rise.
This energy transformation is called the ram effect and the pressure rise is called the ram pressure.
Air from the atmosphere enters the engine at a very high speed and its velocity gets reduced first
in the supersonic diffuser, thereby its static pressure increases. The air then enters the subsonic dif-
fuser wherein it is compressed further and its pressure and temperature reaches above the ignition
temperature. Later, the air flows into the combustion chamber, the fuel is injected and mixed with the
unburnt air. The high pressure and high temperature gases are passed through the nozzle, converting
the pressure energy into kinetic energy. The high velocity gases leaving the nozzle provide the required
forward thrust to the ramjet.
The major advantages of ramjet engines are as follows: light in weight, no moving parts, and
possibility of using a wide variety of fuels. However, its main drawback is that it cannot be started on its
own from rest and requires some launching device. Due to its high thrust at high operational speed, it is
widely used in high-speed military aircrafts and missiles. Subsonic ramjets are used in target weapons,
in conjunction with turbojets or rockets for getting the starting torque. It can be used for pilotless aircraft.

M17_THERMAL ENGINEERING_SE_XXXX_CH17.indd 923 6/16/2018 11:52:32 AM


924 Chapter 17

Supersonic Subsonic
Combustor Nozzle
diffuser diffuser

Direction
Exhaust
of aircraft

Air
inlet Central body housing
accesories
(a)

T
p3
4

3′ p2
3
p2′
2 re s sureure p1
m air pir press 5′
2′ Ra ent a 5
i
Amb
1
Ram air work
s
(b)

Figure 17.2  Ramjet engine: (a) Line diagram, (b) T-s diagram

17.2.3   Pulse Jet Engine


The line diagram of a pulse jet engine is shown in Fig. 17.3. It consists essentially of a diffuser, valve
grid, combustion chamber, spark plug, and a tail pipe or discharge nozzle. It is an intermittent com-
bustion engine and operates similar to a reciprocating engine. It develops thrust by a high velocity of
jet of exhaust gases without the aid of a compressor or turbine.
The incoming air is compressed by the ram effect in the diffuser and the grid passages which
are opened and closed by V-shaped non-return valves. The fuel is then injected into the combustion
chamber by fuel injectors. The combustion is then initiated by a spark plug. As a result of combustion,
the temperature and pressure of combustion products increase and the non-return valves get closed;

Inlet diffuser Valve grid Combustion chamber

Direction Shock
of aircraft waves Exhaust gases
Air
inlet
Discharge
nozzle
Spark
Fuel plug

Figure 17.3  Pulse-jet engine

M17_THERMAL ENGINEERING_SE_XXXX_CH17.indd 924 6/16/2018 11:52:34 AM


Jet Propulsion 925

consequently, the hot gases flow out. The static pressure in the chamber falls and the high pressure air
in the diffuser forces the valves to open and fresh air is admitted for the new cycle.
Its main advantages are that it is simple in construction, very inexpensive as compared to turbo-jet
engine, and well adaptable to pilotless aircrafts. It is also capable of producing static thrust.
Its major shortcomings are: high intensity of noise, severe vibrations, high rate of fuel consump-
tion, low efficiency, intermittent combustion, limited operational attitude and serious limitations of
mechanical valve arrangement.

17.2.4   Turbo-jet Engine


The line diagram of the turbojet unit is shown in Fig. 17.4(a). The schematic arrangement and the
corresponding T-s diagram are shown in Fig. 17.4(b) and Fig. 17.4(c), respectively. It consists of a
diffuser, compressor, combustion chamber, turbine, and exhaust nozzle.

Diffuser Compressor Fuel Turbine Nozzle

Direction Exhaust
of aircraft gases
Air
inlet
Combustion chamber
Shaft Fuel

(a)
Diffuser Compressor Turbine
Nozzle
2′
1 6′ Exhaust to
atmosphere
3′ 4′
5′
Air from C.C
atmosphere
(b)

T
p4
4 p3
3 3′ p2
5′
5
2 p1
2′ 6′
6
1
s
(c)

Figure 17.4  Turbo-jet engine: (a) Line diagram, (b) Schematic diagram, (c) T-s diagram

M17_THERMAL ENGINEERING_SE_XXXX_CH17.indd 925 6/16/2018 11:52:34 AM


926 Chapter 17

The function of the diffuser is to convert the kinetic energy of the entering air into a static pressure
rise, which is achieved by the ram effect. This air enters the axial flow compressor and combustion
chamber. The gas turbine produces just sufficient power to drive the compressor by partial expansion
in the turbine. The gases coming out from the turbine are expanded in the nozzle and produce a very
high velocity jet which gives a forward motion to the aircraft by the jet reaction.
The advantages of a turbo-jet engine are its simpler construction, absence of vibrations, uninter-
rupted and smooth operation, lesser weight to power ratio, higher rate of climb, and higher propulsion
efficiency at higher speed. However, its disadvantages are less efficiency, shorter life, noisier, expan-
sive, larger length of take-off required, and low thrust at take-off.
It is suitable for piloted operation of aircraft travelling above 800 km/h.

17.2.5   Turbo-Prop Engine


The line diagram of a turbo-prop engine is shown in Fig. 17.5(a) and the corresponding T-s diagram
in Fig. 17.5(b). The engine consists of propeller, reduction gear, diffuser, compressor, combustion
chamber, turbine, and exhaust nozzle. In this system, the gases are partly expanded in the turbine
and partly in the nozzle. The total power produced by the turbine is used to run the compressor and
the propeller. The propeller provides most of the propulsive thrust and the thrust produced due to jet
action is quite small.
The turbo-prop combines the merits of a turbo-jet and a propeller. It is suitable for piloted operation
of aircraft.
Diffuser Compressor Fuel Turbine Nozzle

6′
1 Exhaust
3′ 4 5′
gases
2′

Propeiler Combustion chamber


Reduction Shaft Fuel
gear

(a)

T 4
3′
3 5′
5

2 6′
2′ 6

1
s
(b)

Figure 17.5  Turboprop engine: (a) Line diagram, (b) T-s diagram

M17_THERMAL ENGINEERING_SE_XXXX_CH17.indd 926 6/16/2018 11:52:35 AM


Jet Propulsion 927

Pumps
Fuel Oxidiser Gas Combustion
tank tank turbine chamber Nozzle

Exhaust
gases
Direction
rocket jet

Figure 17.6  Line diagram of a rocket jet

17.2.6   Rocket Propulsion


In a rocket engine, the fuel and the oxidiser are stored in the propelling body. The rocket can function
in a vacuum and is the only device capable of space flight.
Rockets may be of the single-stage or multi-stage type, consisting of one or more rocket motors.
They may be solid propellant or liquid propellant rockets.
Figure 17.6 shows the line diagram of a liquid propellant rocket engine using liquid oxygen and
refined petrol. It consists of a fuel tank, oxidiser tank, gas turbine, pumps, combustion chamber,
and nozzle. Pumps are used to supply propellants to the nozzle at high pressure to obtain higher
thrust. These pumps are operated by a gas turbine where the high pressure and high temperature gases
formed by the fuel and oxidiser are delivered to the gas turbine.
The properties for an ideal propellant are high calorific value, high density, high stability, ease of
handling and storing, non-corrosive, and high boiling point at low pressure.
Rockets find important applications in the field of long range artillery, lethal weapons, signalling and
fireworks, jet assisted take-off, satellites, space ships, and research. They are suitable for pilotless operation.

17.3 ❐ JET PROPULSION V’S ROCKET PROPULSION


Table 17.1 shows the comparison of jet propulsion and rocket propulsion.

Table 17.1  Comparison of jet and rocket propulsion

S. no. Item Jet propulsion Rocket propulsion


1. Propulsive thrust Reaction of the engine produced As for jet propulsion
by the exit of high velocity fluid
stream from the nozzle
2. Oxidant supply Does not carry Carries its own
3. Necessity of a compres- Yes No
sor and gas turbine
4. Operation in vacuum and No, because it requires atmos- Yes, because it carries its own
outer space pheric air for its operation oxidant
5. External source for Requires some external source for Does not require external source
starting starting the compressor and turbine for starting because it can be
started by oxidisation of fuel in
the combustion chamber

M17_THERMAL ENGINEERING_SE_XXXX_CH17.indd 927 6/16/2018 11:52:35 AM


928 Chapter 17

17.4 ❐ BASIC CYCLE FOR TURBO-JET ENGINE


The basic cycle of a turbo-jet engine in T-s diagram is shown in Fig. 17.7. The various processes are
as follows:
Process 1-2: Isentropic diffusion of atmospheric air from velocity c1 to c2 = 0 with diffuser effi-
ciency of 100%. 1-2′ is the actual diffusion with diffuser efficiency hd from p1 to p2.
Process 2′-3: Isentropic compression of air in the compressor from p2 to p4. Process 2′-3′ is the
actual compression with compressor efficiency hc.
Process 3-4: Ideal heat addition at pressure p4. Process 3′-4 is the actual heat addition at pressure p4.
Process 4-5: Isentropic expansion of gas in the turbine from p4 to p3. Process 4-5′ is the actual
expansion in the turbine with turbine efficiency ht.
Process 5′-6: Isentropic expansion of gas in the nozzle from p3 to p1. Process 5′-6′ is the actual
expansion of gas in the nozzle with nozzle efficiency hn.
Consider 1kg of working fluid flowing through the system.
Diffuser:
The energy equation between states 1 and 2 is as follows:
c12 c2
h1 + + q1-2 = h2 + 2 + w1-2
2 2
In an ideal diffuser, c2 = 0, q1-2 = 0 and w1-2 = 0

c12
∴  
h2 = h1 +
  (17.1a)
2
c12
or T2 = T1 + (17.1b)
2c pa
where cpa = specific heat of air at constant pressure
h2 − h1 T −T
Diffuser efficiency,    hd = = 2 1 (17.2)
h2′ − h1 T 2′ − T1
h2 − h1
h2′ = h1 +
hd
T2 − T1
   T2′ = T1 + (17.3)
2c pah d
T p4
4
3′ 5′ p3 Turbine
3
p2 Nozzle
r
5 p1
so
pres
Com
2 s e r 6′
Diffu 6
2′

1
s

Figure 17.7  T-s diagram for turbo-jet

M17_THERMAL ENGINEERING_SE_XXXX_CH17.indd 928 6/16/2018 11:52:40 AM


Jet Propulsion 929

Compressor:
Energy equation between state 2′ and 3:

c22′ c2
h2′ + + q2′−3 = h3 = 3 − w2′−3
2 2
c32 − c22′
Compressed work, wc = w2′−3 = ( h3 − h2′ ) +
2
_ c2′, and q2′-3 = 0, we have
Assuming c3 ∼
wc = h3 - h2′ = cpa (T3 - T2′)
wca = h3′ - h2′
Actual compressor work,      
h −h c pa (T3 − T2′ )
    = 3 2′ = (17.4)
hc hc
Combustion chamber:
Ideal heat supplied,    q = q3′- 4 = h4 - h3′

= cpg (T4 - T3′)(17.5a)


      
 m f 
Actual heat supplied,  qa = 1+ h - h3′(17.5b)
 m a  4

 m f 
   = cpg 1+ T - c T (17.5c)
 m a  4 pa 3′
where cpg = specific heat of gas at constant pressure
Turbine:
Energy equation between states 4 and 5:

c42 c2
h4 + + q4 -5 = h5 + 5 + w4-5
2 2
If q4-5 = 0, then
c42 − c52
Turbine work,        wt = w4-5 = (h4 - h5) +
_ c5, then
If c4 ∼ 2

  wt = h4 - h5 = cpg (T4 - T5)(17.6a)


Actual turbine work,     wta = h4 - h5′ = cpg (T4 - T5)ht(17.6b)
now     wta = wca
c pa (T3 − T2′ )
cpg (T4 - T5)ht =
hc
If cpa = cpg = cp, then
T5′ = T4 - (T4 - T5)ht
T3 − T2′
T5′ = T4 -
or   (17.7)
hc

M17_THERMAL ENGINEERING_SE_XXXX_CH17.indd 929 6/16/2018 11:52:44 AM


930 Chapter 17

Jet nozzle:
Energy equation between states 5′ and 6:
c52′ c2
Ideal case:     h5′ + = h6 + 6
2 2

c52′ c2
Actual case:     h5′ + = h6′ + 6′
2 2

_ 0 as compared to c , then
If c5′ ∼ 6′
c62′
h5′ = h6′ +
2
c6′ = 2( h5′ − h6′ ) = 2h n ( h5′ − h6 )

= 2c ph n (T5′ − T6 ) (17.8)

( h4 − h6′ ) − ( h3′ − h1 )
Thermal efficiency,    hth =
( h4 − h3′ )
(T4 − T6′ ) − (T3′ − T1 )
     = (17.9)
(T4 − T3′ )

17.4.1   Thrust
Let ca = forward velocity of aircraft through air, m/s
  = velocity of approach, assuming surrounding air velocity to be zero.
  cj = velocity of jet of gases relative to the aircraft, m/s
 m f 
m· = 1+ , mass rate of flow of products leaving the nozzle per 1kg of air.
 m a 
Absolute velocity of gases leaving the aircraft = cj - ca
Absolute velocity of air entering the aircraft = 0
∴ Change of momentum = m· (cj - ca)
Thrust, F = m· (c - c ) N/(kg/s) of air
j a (17.10a)
   = (cj - ca) N/(kg/s) of air, neglecting mass of fuel (17.10b)

17.4.2   Thrust Power


It is defined as the rate at which work must be developed by the engine if the aircraft is to be kept
moving at a constant velocity ca against friction force or drag.
Thrust power, T.P. = forward thrust × speed of aircraft

( m .c j − ca ).ca
     = kW/kg of air (17.11)
103

M17_THERMAL ENGINEERING_SE_XXXX_CH17.indd 930 6/16/2018 11:52:48 AM


Jet Propulsion 931

17.4.3   Propulsive Power


The energy required to change the momentum of the mass flow of gases represents the propulsive
power. It is expressed as the difference between the rate of kinetic energies of the entering air and exit
gases.
 mc
 2j ca2 
Propulsive power, P.P = ∆KE =  −  × 10-3 kW/kg of air (17.12a)
 2 2
 

c 2j − ca2
    = kW/kg of air by neglecting mass of fuel. (17.12b)
2 × 103

17.4.4   Propulsive Efficiency


It is defined as the ratio of thrust power to propulsive power.

T.P. ( m .c j − ca )ca 2( m .c j − ca )ca


    hp = = = (17.13a)
P.P m c 2j ca2 m c 2j − ca2

2 2
2ca 2
Neglecting the mass of fuel,      hp = = (17.13b)
c j + ca cj
+1
ca
17.4.5   Thermal Efficiency
It is defined as the ratio of propulsive work to the energy released by the combustion of fuel.

1   m f  2 2

 1 +  c j − ca 
2   m a  
hth = (17.14)
m f
× C.V.of fuel × 103
m a
17.4.6   Overall Efficiency
c 2j − ca2 2ca
Overall efficiency, h0 = hth × hp = ×
m f c j + ca
2× × C.V. × hcomb
m a

ca (c j − ca )
     = (17.15)
m f
× C.V. × hcomb
m a
dh0
for h0 to be maximum, =0
dca
cj - 2ca = 0

M17_THERMAL ENGINEERING_SE_XXXX_CH17.indd 931 6/16/2018 11:52:52 AM


932 Chapter 17

cj
or ca = (17.16)
2
Therefore, for maximum overall efficiency, the aircraft velocity is one-half of the jet velocity.

17.4.7   Jet Efficiency


Jet efficiency is defined as follows:
Final K.E.of the jet
hjet = (17.17)
Isentropic enthalpy drop in the jet nozzle + carry over from the turbine

17.4.8   Ram Air Efficiency


In Fig. 17.2(b), process 1-2′ is the ramming process during which the total energy or stagnation
enthalpy remains unchanged, if the process is assumed adiabatic, while pressure of air increases.
Process 1-2 is the ideal ramming action.
p2′ − p1
Ram efficiency      hram = (17.18)
p2 − p1
ca2
 h2 = h2′ = h1 +
Now,  
2
ca2
or cpT2 = cpT2′ = cp T1 +
     
2
ca2
T2 = T2′ = T1 +
2 × cp

T2′ ca2
= 1+
T1 2 × c pT1

(g −1)ca2
=1+
2g RT1
T2′ (g −1) 2
or =1+ M (17.19)
T1 2

17.5 ❐ THRUST WORK, PROPULSIVE WORK, AND


PROPULSIVE EFFICIENCY FOR ROCKET ENGINE
Neglecting the friction and other losses, we have
Thrust work = cj ca (17.20)
2
(c j − ca ) c 2j + ca2
Propulsive work = c jca + = (17.21)
2 2

M17_THERMAL ENGINEERING_SE_XXXX_CH17.indd 932 6/16/2018 11:52:57 AM


Jet Propulsion 933

c j + ca
Rocket propulsive efficiency =
1 2
(c + ca2 )
2 j
c 
2 a 
2 c ca  cj 
  = 2 j
  =  (17.22)
c j + ca2 c 
2
1+  a 
 cj 

Example 17.1
A turbo-jet unit consists of single stage compressor, single stage turbine, and nozzle. The pressure
and temperature at the inlet of compressor are 0.8 bar and 280 K, respectively. The pressure at the
outlet of compressor is 4.5 bar. The following data is available:
Isentropic efficiency of compressor = 0.82
Maximum cycle temperature = 560°C
Isentropic efficiency of turbine = 0.85
Isentropic efficiency of nozzle = 0.90
Combustion efficiency = 0.98
Mechanical efficiency = 0.95
cpa = 1.005 kJ/ kg.K, ga = 1.4, cpg = 1.2 kJ/ kg.K, gg = 1.35
C.V. of fuel used = 42,000 kJ/ kg
Static back pressure on the nozzle = 0.65 bar
Speed of aircraft = 720 km/h
Mass rate of flow of air = 20 kg/ s
Calculate (a) I.P. required to drive the compressor, (b) A.F. ratio, (c) thrust developed, and (d) the
propulsion efficiency.

Solution
The T-s diagram is shown in Fig. 17.8. 1.4 −1
 4.5  1.4
T2 = 280  = 458.65 K
 0.8 

bar
T 3 4.5
833 K
2′
2 ar
4′ 0.8 b
4 ar
5b
0.6
5′
280 K 5
1

Figure 17.8  T-s diagram for turbo-jet

M17_THERMAL ENGINEERING_SE_XXXX_CH17.indd 933 6/16/2018 11:52:59 AM


934 Chapter 17

T2 − T1
hc = ,
T21 − T1
T −T
T2′ = T1 + 2 1
hc
458.65 − 280
= 280 + = 497.8 K
0.82
T3 = 560 + 273 = 833 K
Heat supplied by the fuel = Heat carried by gases from combustion chamber
mf × C.V. × hcomb = cpg (m· a + m· f) (T3 - T2′)
 m 
4200 × 0.98 = 1.2  a + 1 (833 - 497.8)
 m f 
ma
= 101.3:1
mf
No work developed by turbine = work required to run the compressor

 m a + m f 
cpg   (T3 - T4′) hmech = cpa (T2′-T1)
 m a 
102.3
1.2 × × (833 - T4′) × 0.95 = 1.005 (497.8 - 280)
101.3
T4′ = 642.87 K
T3 − T4′
ht =
T3 − T4
833 − 642.87
or      0.85 =
833 − T4
T4 = 609.3 K
or    
0.35
T3  4.5  1.35
=
T4  p4 
0.35
833  4.5  1.35
or   = 
609.3  p4 
or            p4 = 1.347 bar
0.35
T  1.347  1.35 642.87
  4′ =   =
T5  0.65  T5
T5 = 532.2 k
or     

M17_THERMAL ENGINEERING_SE_XXXX_CH17.indd 934 6/16/2018 11:53:05 AM


Jet Propulsion 935

T4′ − T5′
Nozzle efficiency, hn =
T4′ − T5
642.87 − T5′
or        0.90 =
642.87 − 532.2
T5′ = 543.3 K
or  
I.P. required to run the compressor = m· a × cpa (T2′ - T1)
         = 20 × 1.005 (497.8 - 280) = 4377.78 kW
Heat drop in nozzle,        ∆h = cpg (T4′ - T5′)
= 1.2 (642.87 - 543.3)
= 119.484 kJ/ kg of air
Velocity of gases learning the nozzle relative to aircraft,

cj = 2∆h = 2 × 119.484 × 103 = 488.84 m/s

720 × 103
Velocity of aircraft, ca = = 200 m/s
3600
102.3
Mass of gases passing through the nozzle, m· g = 20 × = 20.197 kg/s
101.3
Thrust produced = m· g (cj - ca) = 20.197 (488.84 - 200) = 5833.7 N

2ca 2 × 200
Propulsive efficiency = = = 0.5807 or 58.07%
ca + c j 200 + 488.84

Example 17.2
In a jet propulsion system, air enters the compressor at 1 bar, 10°C. The pressure leaving the com-
pressor is 4 bar and the maximum temperature is 85°C. The air expands in the turbine to such a
pressure that the turbine work is just equal to the compressor work. On leaving the turbine, the air
expands in a nozzle to 0.8 bar. Calculate the velocity of air leaving the nozzle. Take cp = 1.005kJ/
kg.K and g = 1.4 for both compression and expansion processes.

Solution
The T-s diagram is shown in Fig. 17.9.
g −1
g
T2 = T1rp
0.4
 4  1.4
= 283 ×   = 420.5 K
 1
Compressor work,      wc = cp (T2 - T1)
= 1.005 (420.5 - 283) = 138.2 kJ/ kg of air
T3 = 273 + 850 = 1123 K

M17_THERMAL ENGINEERING_SE_XXXX_CH17.indd 935 6/16/2018 11:53:08 AM


936 Chapter 17

T
3 4 bar
1123 K
p4
2
4 1 bar

0.8 bar
283 K 5
1

Figure 17.9  T-s diagram for jet propulsion

Now wt = wc = cp (T3 - T4)


or 1.005(1123 - T4) = 138.2
or              T4 = 985.46 K
g −1
T3  4  g
=
T4  p4 
0.4
1123  4  1.4
or  =  
985.46  p4 
or     p4 = 2.53 bar
g −1 0.4
 1 g  0.8  1.4
T5 = T3   = 1123  = 709 K
 rp   4 
_ 0, ∆h = c (T -T ) = 1.005 (985.46 - 709) = 277.8 kJ/ kg
Assuming c4 ∼ p 4 5
c5 = 2∆h = 2 × 277.8 × 103 = 745.4 m/s

Example 17.3
A turbojet engine inducts 40 kg/s of air and propels an aircraft with a uniform flight speed of 900
km/h. The isentropic enthalpy change for nozzle is 190 kJ/ kg and its velocity coefficient is 0.96.
The fuel-air ratio is 0.015. The combustion efficiency is 0.95 and the lower heating value of the fuel
is 44,000 kJ/ kg. Calculate the thermal efficiency of the engine, the fuel flow rate and TSFC, the
propulsive power, the thrust power, the propulsive efficiency, and the overall efficiency.

Solution
Jet velocity, cj = Cc 2∆h = 0.95 2 × 190 × 103 = 585.6 m/s

900 × 103
ca =
Aircraft velocity,         = 250 m/s
360
m j
m· = 1+ = 1 + 0.015 = 1.015
m a

M17_THERMAL ENGINEERING_SE_XXXX_CH17.indd 936 6/16/2018 11:53:14 AM


Jet Propulsion 937

m· f = 0.015 ma = 0.015 × 40
= 0.6 kg/s or 2160 kg/h
Thrust, F = (m· a + m· f) cj - m· a ca
= (40 + 0.6) × 585.6 - 40 × 250
= 13775 N or 13.775 kN
2160
Thrust specific fuel consumption, TSFC = = 156.8 kg/kW.h or 0.1568 kg/W.h
13.775
1
Propulsion power, P.P. = ( m + m f )c 2j − m a ca2 
2 a 
1
= ( 40 + 0.6) × (585.6)2 − 40 × ( 250)2 
2 

= 5711 × 103 W = 5711 kW


Thrust power, T.P. = [( m a + m f )c j − m a ca ] × ca

 = (40.6 × 585.6 - 40 × 250) 250


 = 3443.86 × 103 W = 3443.86 kW
T.P. 3443.86
Propulsive efficiency = = = 0.6030 or 60.3%
P.P 5711
T.P.
Overall efficiency =
m × CV. ×h comb

3443.6
 = = 0.1373 or 13.73%
0.6 × 44000 × 0.95
1
( m + m f )c 2j − m a ca2 
2  a 
Thermal efficiency, hth =
m f × C.V. × h comb


1 
× 40.6 × (585.6)2 − 40 × 250 2 
2  
= = 0.2277 or 22.77%
3
0.6 × 44000 × 0.95 × 10

Example 17.4
A turbo-jet unit is flying at a speed of 268 m/s at an altitude where the ambient conditions are 0.2
bar and 220 K. The air enters an ideal diffuser and leaves the compressor at 1350 K and 1 bar.
The fuel supplied has a heating value of 43,000 kJ/ kg. Assume all compression and expansion
processes to be isentropic. Determine (a) the air-fuel ratio, (b) the specific thrust, and (c) the pro-
pulsive efficiency. Take Cpa = 1.005 KJ/ kg.K and ga = 1.4 for compression and Cpg = 1.102 KJ/ kg,
gg = 1.33 for expansion.  [IES, 1998]

M17_THERMAL ENGINEERING_SE_XXXX_CH17.indd 937 6/16/2018 11:53:18 AM


938 Chapter 17

T
4 1 bar
1350 K
3 Turbine
5
pre ssor Nozzle
Com 0.2 bar
2 ser
Diffu 6

220 K 1
s

Figure 17.10 T-s diagram

Solution
The T-s diagram is shown in Fig. 17.10.
Diffuser:
c12
T2 = T1 +
2c pa
2682
= 220 + = 255.73 K
2 × 1.005 × 103
ga
 T  g a −1
p2 = p1  2 
T  1
1.4
 255.73  0.4
= 0.2  = 0.3387 bar
 220 
Compressor:
g a −1 0.4
p  ga  1  1.4
T3 = T2  3  = 255.73  = 348.4 K
 p2   0.3387 

Combustion chamber:
T4 = 1350 K
h3 + q3-4 = h4
m· a cpa T3 + m· f × CV = (m· a + m· f) cpg T4

 
m a × c T + CV = m a + 1 c T
pa 3   pg 4
m f  m f 

m a
(c T - c T ) = cpg T4 - CV
m f pa 3 pg 4

m a
(1.005 × 348.4 -1.102 × 1350) = 1.102 × 350 - 43000
m f
m a
or      = 36.5
m f

M17_THERMAL ENGINEERING_SE_XXXX_CH17.indd 938 6/16/2018 11:53:22 AM


Jet Propulsion 939

Turbine:
wt = wc
(m a + m f) cpg (T4 - T5) = m· a × cpa (T3 - T2)
· ·

 m a  m
 + 1 cpa (T4 - T5) = a × cpa (T3 - T2)
 m f  m f
37.5 × 1.102 (1350 - T5) = 36.5 × 1.005 (348.4 - 255.73)
T5 = 1267.77 K
or  

gg
 T  g g −1
p5 = p4  5 
 T4 
1.33
 1267.77  0.33
=1× = 0.776 bar
 1350 
Let p6 = 0.2 bar
g g −1 0.33
p  gg  0.2  1.33
T6 = T5  6  = 1267.77  = 905.6 K
 p5   0.776 
Work output = (m· a + m· f) cpg (T5 - T6)
Specific thrust = work output per kg of fuel consumed
 m 
=  a + 1 cpg (T5 - T6)
 m f 
    = 37.5 × 1.102 (1267.77 - 905.6)
 = 14966.4 kN/kg
Jet velocity, cj = 2c pg (T5 − T6 )

= 2 × 1.102 × (1267.77 × 905.6) × 103 = 893.4 m/s


Propulsive efficiency, neglecting fuel mass as compared to air mass,
 2ca   2 × 268 
=  =  = 0.4615 or 46.15%
 ca + c j   268 + 893.4 

Example 17.5
A turbojet engine consumes air at the rate of 60 kg/s when flying at a speed of 960 km/h. Calculate
the following:
(a) Exit velocity of the jet when the enthalpy drop in the nozzle is 220 kJ/ kg and velocity coeffi-
cient is 0.95.

M17_THERMAL ENGINEERING_SE_XXXX_CH17.indd 939 6/16/2018 11:53:26 AM


940 Chapter 17

(b) Fuel rate of flow for A/F ratio of 60:1,


(c) Thrust specific fuel consumption (TSFC),
(d) Thermal efficiency of the engine assuming combustion efficiency 92% and calorific value of
fuel used 43,000 kJ/kg,
(e) Propulsive power,
(f) Propulsive efficiency, and
(g) Overall efficiency

Solution
(a) Exit velocity of jet, c j = Cc 2 ∆h = 0.95 2 × 220 × 103 = 630.16 m/s

960 × 103
ca = = 266.67 m/s
3600
m a 60
(b) Fuel rate of flow, m· f = = = 1 kg/s
60 60
(c) Thrust produced, F = (m· a + m· f) cj - m· a ca
= (60 + 1) 630.16 - 60 × 266.67 = 22439.6N
m f 1
TSFC = = = 4.456 × 10-5 kg/N of thrust/s
F 22439.6
c 2j − ca2
(d) Thermal efficiency hth =
m f
2× × CV × h comb × 103
m a

(630.16)2 − ( 266.67)2
= = 0.2475 or 24.72%
1
2 × × 43000 × 0.92 × 103
60
m a 2
(e) Propulsive power = (c j − ca2 )
2
60
= [(630.16)2 - (266.67)2] = 9779.7 kW
2 × 03
2ca 2 × 266.67
(f) Propulsive efficiency = = = 0.5947 or 59.47%
ca − c j 266.67 + 630.16

(c j − ca )ca (630.16 − 266.67) × 266.67


(g) Overall efficiency = = = 0.147 or 14.7%
m f 1 3
× CV × hcomb × 43000 × 0..92 × 10
m a 60

M17_THERMAL ENGINEERING_SE_XXXX_CH17.indd 940 6/16/2018 11:53:33 AM


Jet Propulsion 941

Example 17.6
The following data refer to turbo-jet flying at a height of 9000 m:
Speed of turbo-jet = 800 km/h
Propulsive efficiency = 54%
Overall efficiency of plant = 18%
Propulsive force = 6000 N
Calorific value of fuel = 44,000 kJ/kg
Calculate (a) absolute velocity of jet, (b) volume of air compressed per main, (c) diameter of jet,
(d) power output of the unit, and (e) air-fuel ratio.

Solution
2ca
(a) Propulsive efficiency,   hp =
ca + c j

800 × 103
ca = = 222.2m/s
3600
2 × 222.2
0.54 =
222.2 + c j
cj = 600.76 m/s
  Absolute velocity of jet = cj - ca =600.76 - 222.2 = 378.56 m/s
(b) Propulsive force = m· a (cj - ca)
6000 = m· a × 378.56
  m· a = 5.85 kg/ s

Volume flow rate of air V  = m a = 15.85 × 60 = 5283.14 m3/min


f 0.18
(c) Let jet diameter = d
p 2
d cj = V
4
p 2 5283.14
d × 600.76 =
4 60
d = 0.432 m or 432 mm
600 × 222.2
(d) Thrust power, TP = drag force × ca = = 1333.2 kW
103
T.P. 1333.2
Turbine output = = = 2468.89 kW
hp 0.54
Heat equivalent of output
(e) Overall efficiency =
m f × C. V.

M17_THERMAL ENGINEERING_SE_XXXX_CH17.indd 941 6/16/2018 11:54:38 AM


942 Chapter 17

2468.89
0.18 = ,
m f × 44, 000
m· f = 0.312 kg/s
m a 15.85
A/F ratio = = = 50.8:1
m f 0.312

Example 17.7
A high altitude flight jet propeller aircraft is flying with speed corresponding to Mach number of
1.25. The ambient atmospheric pressure and temperature are 0.01MPa and -40°C. The temperature
and pressure of gases entering the turbine are 827°C and 0.2 MPa. Isentropic efficiency of compres-
sor and turbine are 0.80 and 0.85, respectively. The ram air efficiency is 0.80. The back pressure on
the nozzle may be assumed as the ambient pressure and efficiency of nozzle based on total pressure
drop available is 0.90. Neglecting mass increase due to fuel consumed, calculate (a) compressor
power per kg per second, (b) air-fuel ratio if the fuel calorific value is 41500 kJ/ kg, (c) pressure of
gases leaving the turbine, and thrust per kg per second, (d) propulsive efficiency, and (e) thermal
and overall efficiency. Assume cpa = 1.005 kJ/kgK, ga = 1.4, cpg = 1.12 kJ/kgK, ga = 1.33.

Solution
Sonic velocity at ambient conditions,
cs = g a R a T1
c pa 1.005
Ra = c pa − = 1.005 −
ga 1.4
= 0.287kJ/ kg.K
T1 = 273 - 40 = 233 K
cs = 1.4 × 287 × 233 = 306 m/s
Speed of aircraft,
ca = 1.25 × 206 = 382.5 m/s
Ramming process 1-2′ (see Fig. 17.11)
T2 T2′  g −1 2  1.4 − 1
= = 1+  M = 1+  × (1.25)2 = 1.3125
T1 T1  2   2 
or   T2 = T2′ = 233 × .3125 = 305.8 K
g −1
p T  g
Now, 2 =  2 
p1  T1 
3.5
 305.8 
or p2 = 0.01  = 0.0259 MPa
 233 

M17_THERMAL ENGINEERING_SE_XXXX_CH17.indd 942 6/16/2018 11:54:41 AM


Jet Propulsion 943

T
4 p 3 = p4
1000 K
2M Pa p5 = p5′
3′ 0.
3 5 5′

p2
p2′
2 p1
305.8 K 2′ Pa 6′
0.01 M 6

233 K
1
s

Figure 17.11  T-s diagram for jet propeller


p2′ − p1
hram =
p2 − p1
or        p2′ = 0.8 (0.0259 - 0.01) + 0.01 = 0.0227 MPa
Compressor process 2′-3:
g −1 0.4
 0.2  1.4
T3 = T2′  p3  g
= 305.8  = 569.4 K
 p   0.0227 
2′

T3 − T2′ 569.4 − 305.8


T3′ = + T2′ = + 305.8 = 635.3 K
hc 0.80
Compressor power, Pc = m cpa (T - T)
= 1 × 1.005 (635.3 - 305.8) = 331.18 kJ/kg/s or kW/kg
Combustion process 3′-4:
T4 = 827 + 273 = 1100 K
Heat supplied, q3′-4 = cpg (T4 - T3′) = 1.12 (1100 - 635.3) = 520.5 kJ/ kg
ma cpg (T4 - T3′) = mf × CV

ma 41500
or = = 79.74
mf 520.5
∴ A/F ratio = 79.74:1 = 79.74
Now,            wc = wt
or 331.18 = cpg (T4 - T5′) = 1.12 (1100 - T5′)
or T5′ = 804.3 K

T4 − T5′ 1100 − 804.3


T5 = T4 - = 1100 − = 752.1 K
ht 0.85
ga 1.33
 T  g a −1  752.1 0.33
p5 = p4  5  = 0.2  = 0.0432 MPa
 T4   1100 

M17_THERMAL ENGINEERING_SE_XXXX_CH17.indd 943 6/16/2018 11:54:46 AM


944 Chapter 17

g a −1 0.33
 p6  ga  0.01  1.33
T6 = T5′ = 804.3  = 559.4 K
 p 
5′  0.0432 
T6′ = T5′ - hn (T5′ - T6)

= 804.3 - 0.9 (804.3 - 559.4) = 583.9 K

(∆h)nozzle = cpg (T5′ - T6) = 1.12 (804.3 - 583.09) = 246.83 kJ/kg

cj = c6′ = 2( ∆h) nozzle

= 2 × 246.83 × 03 = 702.6 m/s


Thrust c6′ - ca = 702.6 - 382.5 = 320.1 kN/kg/s
2ca 2 × 382.5
Propulsive efficiency, hp = = = 0.705 or 70.5%
ca + c j 382.5 + 702.6

c 2j − ca2 (702.6)2 − (382.5)2


Thermal efficiency, hth = =
2 × 103 × ( F /A ) × CV 1
2 × 10 2 × × 0.705
79.74
= 0.3337 or 33.37%
Overall efficiency = hth × hp = 0.3337 × 0.705
 = 0.2352 or 23.52%

Example 17.8
A flying turbojet engine propels at speed of 880 km/h and draws air at the rate of 50 kg/s. Isentropic
enthalpy drop in the nozzle is 190 kJ/kg and its velocity coefficient is 0.96. Fuel air ratio used is
0.012. The calorific value of fuel used is 46000 kJ/kg and combustion efficiency is 95%. Find ther-
mal, propulsive, and overall efficiency.
Solution
800 × 103
Given: ca = 880 km/h = = 244.44 m /s, m a = 50 kg/s
3600
Dh = 190 kJ/kg, Cc = 0.96, F/A = 0.012, CV = 46000 kJ/kg, hcomb = 0.95,
c j = Cc 2 × ∆h = 0.96 2 × 190 × 103 = 591.78 m/s

m f = 0.012 × 50 = 0.6 kg/ s

1
m a (c 2j − ca2 )
(a) Thermal efficiency, h th = 2
m f × CV × hcomb × 103

M17_THERMAL ENGINEERING_SE_XXXX_CH17.indd 944 6/16/2018 11:54:50 AM


Jet Propulsion 945

1
× 50 (591.78)2 − ( 244.44)2 
2  
=
3
0.6 × 46000 × 0.95 × 10
= 0.2769 or 2769%
2ca 2 × 244.44
(b) Propulsive efficiency, h p = = = 0.5846 or 58.46%
ca + c j 244.44 + 591.78

m a ca (c j − ca )
(c) Overall efficiency, h0 =
m f × CV × hcomb

50 × 244.44 (591.78 − 244.44)


= = 0.1619 or 16.19%
0.6 × 46000 × 0.95 × 103

Example 17.9
The effective jet exit velocity from jet engine is 2700 m/s. The forward flight velocity is 1350 m/s
and the air flow rate is 78.6 kg/s. Calculate (a) thrust, (b) thrust power, and (c) propulsive efficiency.

Solution
Given: cj = 2700 m/s, ca = 1350 m/s, m a = 78.6 kg/s
(a) Thrust, F = m a (c j − ca ) = 78.6(2700 - 1350) = 106110N

m a (c j − ca )ca 106110 × 1350


(b) Thrust power, TP = 3
= = 143248.5 kW
10 103
m a (c 2j − ca2 ) 78.6( 2700 2 − 1350 2 )
(c) Propulsive power P.P = = = 214872.75 kW
2 × 103 2 × 103

Thrust Power (T.P.)


Propulsive efficiency, h p =
Propulsive power (P.P.)
143248.5
= × 100 = 66.67%
214872.75

Example 17.10
A simple turbo-jet unit operates with a turbine inlet temperature of 1100 K, a pressure ratio of 4:1
and mass flow of 22.7 kg/s under design conditions. The following component efficiencies may be
assumed
Isentropic compressor efficiency = 0.85
Isentropic turbine efficiency = 0.9
Propeller nozzle efficiency = 0.95

M17_THERMAL ENGINEERING_SE_XXXX_CH17.indd 945 6/16/2018 11:54:53 AM


946 Chapter 17

Transmission efficiency = 0.99


Combustion chamber pressure loss = 0.21 bar
Calculate the design thrust and specific fuel consumption when the unit is stationary at sea level
where the conditions may be taken as 1.013 bar and 288 K.

Solution
Given that T3 = 1100 K, rp = 4, m a = 22.7 kg/ s, hc = 0.85, ht = 0.9, hn = 0.95, hmech = 0.99, Dp3 =
0.21 bar, p1 =1.013 bar, T1 = 288 K
Process 1 - 2: Compressor (Fig. 17.12)

g −1
T2  p3  g
=
T1  p1 
1.4 −1
or T2 = 288( 4) 1.4 = 427.97 K

h = T2 − T1
c
T2′ − T1

or     0.85 = 427.97 − 288


T2′ − 288
or T2′ = 452.67 K
p3′ = p3 − ∆p3 = 4 × 1.013 − 0.21 = 3.842 bar
Neglecting the mass of fuel and cpg = cpa

(T3 − T4′ )h mech = T2′ − T1


or (1100 − T4′ ) × 0.99 = 452.67 − 288
or T4′ = 933.67 K
T3 − T4′
ht =
T3 − T4

p3
T p3′
2′ 3 p2
2
p1
4′
4 p0
5′
5
1

Figure 17.12  T-s diagram for simple turbo-jet

M17_THERMAL ENGINEERING_SE_XXXX_CH17.indd 946 6/16/2018 11:54:58 AM


Jet Propulsion 947

1100 − 933.67
or 0.9 =
1100 − T4
T4 = 915.19 K
g −1
T p  g
3 =  3′ 
T4  p2 
1.4 −1
1100  3.842  1.4
or   =
915.19  p2 
p2 = 2.018 bar
or  
T4′ − T5′
       h n =
T4′ − T5
p0 = p1
Taking  
g −1
T p  1
    4′ =  2 
T5  p1 
1.4 −1
933.67  2.018  1.4
or       =
T5  1.0013 
T5 = 7666.79 K
or  
933.67 − T5′
      0.95 =
933.67 − 766.79
or        T5′ = 775.13 K
Enthalpy drops per kg of mass flow rate per second:

  ( ∆h)c = c p (T2′ − T1 )

     ( ∆h)t = c p (T3 − T4′ )

     ( ∆h)n = c p (T4′ − T5′ )

 ∆h = ( ∆h)t + ( ∆h)n  − ( ∆h)c = c p [T3 − T4′ + T4′ − T5′ − T2′ + T1 ]
  

     = c p [T3 − T5′ − T2′ − T1 ] = 1.005[1100 − 775.13 − 452.67 + 288]

= 161 kJ/kg
Velocity of nozzle, c j = 2 × ∆h = 2 × 161 × 103 = 567.45 m/s
(a) Design thrust, F = m a × c j = 22.7 × 567.45 = 12881 N

m f 22.7 × 3600
(b) Specific fuel consumption = = = 6.344 kg/Ns
F 12881

M17_THERMAL ENGINEERING_SE_XXXX_CH17.indd 947 6/16/2018 11:55:04 AM


948 Chapter 17

Example 17.11
In a turbo jet engine, air enters the diffuser at 0.8 bar, 240 K. with a velocity of 1000 km/hr. The
pressure, ratio across the compressor is 8. The turbine inlet temperature is 1200 K and the pressure
at nozzle exit is 0.8 bar. The turbine work just equals the compressor work input. The diffuser,
compressor, turbine and nozzle processes are isentropic and there is no pressure drop for flow
through the combustor. Determine the pressure at exit from the diffuser, the compressor, and the
turbine, and also the velocity at the nozzle exit. Show the various process on a temperature-entropy
diagram. For air, cp 1.001 kJ/kg. K and cp/cu = 1.4. [IES, 1993]

Solution
p3
Given that p1 = 0.8 bar, T1 = 240 K, c1 = 1000 km/h or 277.78 m/s, = 8, T4 = 1200 K, wt =
p2
wc, cpa = 1.001 kJ/kg K, ga = 1.4
From Fig. 17.13 we have
p3 = p4, p1 = p6
Assume cpg = cpa and gg = ga = g
Diffuser:

c12
T 2 = T1 +
2c pa

( 277.78)2
= 240 + = 278.5 K
2 × 1001
g 1.4
 T2  g −1  278.5  0.4
p2 = p1 = 0.8  = 1.347 bar
 T   2402 
1
Compressor:
p3 = 8 × 1.34 = 10.778 bar
g 0.4
T3 = T2  p3  g −1 = 278.5(8) 1.4 = 504.5 K
 p 
2
p4 = p3 = 10.778 bar

T
4 p4 = p3
1200 K
Turbine
3 p5
5 p2
Compressor Nozzle
2 p1 = p6 = 0.8 bar

Diffuser 6
240 K
1
s

Figure 17.13  Turbo-jet engine T-s diagram

M17_THERMAL ENGINEERING_SE_XXXX_CH17.indd 948 6/16/2018 11:55:09 AM


Jet Propulsion 949

Turbine:
wc = cp (T3 - T2) = 1.001 (504.5 - 278.5) = 226.226 kJ/kg

wt = cp(T4 - T5) = 1.001 (1200 - T5)


Now      wt = wc
∴     1.001 (1200 - T5) = 226.226
T5 = 974 K
or    
g
p5  T5  g −1
=
p4  T4 
1.4
 974  0.4
p5 = 10.778  = 5.19 bar
 1200 
g −1 g −1
T p  g  p  g
Nozzle: 6 =  6  =  1
T5  p5   p2 
0.4
 0.8  1.4
T6 = 974  = 570.87 K
 5.19 

Nozzle jet velocity c6 = 2c p (T5 − T6 )

= 2 × 1001(974 − 570.87) = 896.4 m/s

Example 17.12
A jet engine is flying at 300 m/s when the pressure and temperature of the atmosphere are 0.8
bar and 230 K, respectively. The compressor pressure ratio is 4 and the maximum cycle temper-
ature is 1000 K. Calculate, specific thrust, power produced, propulsive efficiency, overall thermal
efficiency, and the fuel consumption. Assume: isentropic efficiency of components unity; nozzle
throat area 0.06 m2; calorific value of fuel 43000 kJ/kg; cp and g for the combustion and expansion
processes 1.15 kJ/kg K and 1.333. [IES, 1997]

Solution
Given: c1 = 300 m/s, p1 = 0.8 bar, T1 = 230 K, p3/p2 = 4, T4 = 1000 K, hisen = 1.0, At = 0.06 m2, C.V.
= 43000 kJ/kg
cpa = 1.005 kJ/kg. K, ga = 1.4, cpg = 1.15 kJ/kg. K, gg = 1.333
The T-s diagram is shown in Fig. 17.14.
Diffuser:
c12 3003
T2 = T1 + = 230 + = 274.87 K
2c pa 2 × 1.005 × 103

M17_THERMAL ENGINEERING_SE_XXXX_CH17.indd 949 6/16/2018 11:55:14 AM


950 Chapter 17

T
4
1000 K
Turbine
3
Compressor 5
0.8 bar
Nozzle
2
Diffuser 6
230 K
1
s

Figure 17.14 T-s diagram for jet engine

ga 1.4
 274.78  0.4
p2 = p1  T2  g a −1 = 0.8  = 1.49 bar
 T   230 
1
Compressor:
p3 = 4p2 = 4 × 1.49 = 5.96 bar
g a −1 0.4
T3 = T2  p3  ga
= 274.78( 4) .4
1 = 408.32 K
 p 
2
T4 = 1000 K
Neglecting mass of fuel as compared to that of air,
Work done by turbine = Work done on compressor
cpg (T4 - T5) = cpa (T3 - T2)

1.15 (1000 - T5) = 1.005 (408.32 - 274.78)


T5 = 883.3 K
or  
Also p4 = p3
gg
p4  T4  g g −1
=
p5  T5 
5.96
or p5 = = 3.63 bar
1.333
 1000  0.333
 
883.3 
Let p6 = p1 = 0.8 bar
g g −1
T5  p5  gg
=
T6  p6 

M17_THERMAL ENGINEERING_SE_XXXX_CH17.indd 950 6/16/2018 11:55:18 AM


Jet Propulsion 951

883.3
or T6 = = 605.4 K
0.333
 3.63  1.333
 
0.8 
Enthalpy drop in nozzle,
h = cpg (T5 - T6) = 1.15 (883.3 - 605.4) = 319.6 kJ/kg
Velocity of jet, cj = c6 = 2.∆h = 2 × 319.6 × 103 = 799.5 m/s
Thrust produced, F = m(c
 j - ca) = 1(799.5 - 800) = 499.5 N/kg of air
Fca 499.5 + 300
Thrust power, TP = = = 149.85 kW
3
10 103
Specific thrust = 149.85 kW/kg of air
Power produced = 149.85 + Turbine output
= 149.85 + 1.15 (1000 - 883.3) = 284.06 kW/kg of air
Propulsive efficiency,
149.85
hprop = = 0.5275 or 52.75%
284.06
Overall thermal efficiency,
149.85 149.85
hth = = = 0.2202 or 22.02%
c pg (T4 − T3 ) 1.15 (1000 − 408.32)

(b) Considering mass of fuel.


Let mf = mass of fuel.
Then mf × CV = (ma + mf) cpg × T4 - cpa ma T3

mf  mf 
or × C.V. = 1+ c × T4 - cpa.ma.T3
ma  ma  pg
mf
or (C.V. - cpg T4) = cpg T4 - cpa T3
ma
mf
or (43000 - 1.15 × 1000) = 1.15 × 1000 - 1.005 × 408.32
ma
mf 739.6384
or = = 0.0176 kg of fuel/kg of air
ma 41850
Now Wt = Wc
(ma + mf) cpg (T4 - T5) = ma cpa (T3 - T2)
or (1 + 0.0176) × 1.15 (1000 - T5) = 1.005 (408.32 - 274.78)
or     T5 = 885.3 K
gg
p4  T4  g g −1
Also =
p5  T5 

M17_THERMAL ENGINEERING_SE_XXXX_CH17.indd 951 6/16/2018 11:55:25 AM


952 Chapter 17

5.96
p5 =
or   = 3.66 bar
1.333
 1000  0.333
 
885.3 
5.96
T6 = = 604.98 K
0.333/1.333
 3.66 
 
0.8 
Nozzle:
∆h = cpg (T5 - T6) = 1.15 (885.3 - 604.98) = 322.37 kJ/kg
cj = c6 = 2.∆h = 2 × 322.37 × 103 = 802.85 m/s
 mf 
Specific thrust, F = 1+ (c - c )
 ma  j a
= (1 + 0.0176) (802.95 - 300) = 511.8 N/kg of air
 g g − 1  1.333 − 1
Now R =   cpg =   × 1.5 = 0.287 kJ/kg. K
g
 g  1.333 
p6u6 = RT6
0.287 × 604.98
u6 = = 2.17 m3/kg of air
0.8 × 10 2
Af c j 0.06 × 802.95
Mass flow rate, m = = = 22.2 kg/s
ug 2.17
Power produced = F × ca × m / 103 = 511.8 × 300 × 22.2/103 = 3408.6 kW

Example 17.13
Air at 25 kPa and 230 K enters a turbojet engine with a velocity of 250 m/s. The pressure ratio
across the compressor is 12. The turbine inlet temperature is 1400 K and the pressure at the nozzle
exit is 25 kPa. The diffuser, compressor, turbine and nozzle processes are isentropic and there is no
pressure drop for flow through the combustor. Draw the line diagram indicating all the components
and show the processes on a T-s diagram. Under steady state operating conditions, determine
(a) the velocity at the nozzle exit and (b) the pressures and temperatures at each state. [IES, 2006]
Solution
Diffuser - compressor: The line diagram of turbo-jet and the T-S diagram are shown in Fig. 17.15.

v02 250 2
T1 = T0 + = 230 + = 261.1 K
2c p 2 × 1000 × 1.005
g 1.4
 261.1 0.4
p1 = p0  T1  g −1 = 25  = 38.87 kPa
 T   230 
0

M17_THERMAL ENGINEERING_SE_XXXX_CH17.indd 952 6/16/2018 11:55:32 AM


Jet Propulsion 953

C.C

2 3 4
5 Exhaust to
C T
atmosphere
1
Nozzle
D
Air from
atmosphere
(a)
p3
T e
3 in
rb p
Tu 2

2 4 zle p1
oz
N
Compressor
1
Diffuser 5

0
s
(b)

Figure 17.15  Turbo-jet engine: (a) Line diagram of turbo-jet, (b) T-s diagram

p2 = 12 p1 = 12 × 38.97= 467.64 kPa


(g −1)
T2 = T1  p2  g
= 261.1 (12)0.4/1.4 = 531.05 K
 p 
1
Combustor:
p3 = p2 = 467.64 kPa
T3 = 1400 K
Turbine: Assuming compressor work = turbine work, we have
cp (T2 - T1) = cp (T3 - T4)
or 531.05 - 262.1 = 1400 - T4
or T4 = 1130.05 K
g 1.4
 1130.05  0.4
p4 = p3  T4  (g −1) = 467.64  = 220.95 kPa
 T   1400 
3
g −1
T  p  g
Nozzle:     5  =  5 
 T4   p4 
0.4
 25  1.4
T5 = 1130.05  = 606.3 K
 220.95 

M17_THERMAL ENGINEERING_SE_XXXX_CH17.indd 953 6/16/2018 11:55:36 AM


954 Chapter 17

c42 c2
h4 + = h5 + 5
2 2
c4 ≈ 0 as compared to c5.
c5 = [2 (h4 - h5)]0.5 = [2 × cp (T4 - T5)]0.5
= (2 × 1000)0.5 × [1.005 (1130.5 - 606.3)]0.5
= 1026 m/s

Summary for Quick Revision


1. In jet propulsion, the work output of a gas turbine is used to produce high velocity jet by expanding gas
through a nozzle. The reaction of the jet produces the propulsive force to propel a vehicle.
2. Jet propulsion systems are of two types: air stream jet engines and self-contained rocket engines.
3. In a screw propeller, full expansion of gas takes place in the turbine and the total power developed is used to
drive the compressor and the propeller.
4. In the ramjet engine, the diffusers convert the kinetic energy of high speed entering air into pressure rise
(called ram effect). The high pressure and high temperature gases are then passed through the nozzle which
converts the pressure energy into kinetic energy to produce a high velocity jet.
5. The pulse jet engine develops thrust by a high velocity jet of exhaust gases without the aid of a compressor
or turbine.
6. The turbine of a turbo-jet engine produces just sufficient power to drive the compressor by partial expansion
in the turbine. The gases coming out from the turbine are expanded in the nozzle to produce a very high
velocity jet to propel the aircraft.
7. In the turbo-prop engine, the gases are expanded partly in the turbine and partly in the nozzle. The turbine
power is totally consumed to drive the compressor and the propeller. The propeller provides most of the
propulsive thrust. It is suitable for pilotless operation of aircraft.
8. In a rocket engine, both the fuel and oxidiser are stored in the propelling body. It is the only device capable
to space flight.
9. Turbo-jet engine:
(a) Thrust, F = m (c j − ca )N/(kg/s) of air
where cj, ca = velocity of jet and aircraft, respectively
m = 1 + m f /m a
m f , m a = mass of fuel and air, respectively
m (c j − ca )ca
(b) Thrust Power, T.P. = kW/kg of air
103
m (c 2j − ca2 )
(c) Propulsive power, P.P = kW/kg of air
2 × 103
2
(d) Propulsive efficiency, h p = , neglecting mass of fuel
(c j /ca ) + 1
c 2j − ca2
(e) Thermal efficiency, hth =
2 × ( m f /m a ) × C.V.
ca (c j − ca )
(f) Overall efficiency, ho =
( m f /m a ) × C.V. × hcomb

M17_THERMAL ENGINEERING_SE_XXXX_CH17.indd 954 6/16/2018 11:55:40 AM


Jet Propulsion 955

cj
(g) For ho to be maximum, ca =
2
Final K.E. of jet
(h) Jet efficiency, h jet =
( ∆h)isen in jet nozzle + carry over from turbine
p2′ − p1  g − 1 2
(i) Ram air efficiency, h ram = = 1+  M
p2 − p1  2 
c
where Mach number, M = a
cs
Sonic velocity, cs = g RT1
10. Rocket engine:
(a) Thrust work = cj ca
c 2j + ca2
(b) Propulsive work =
2
2(ca /c j )
(c) Propulsive efficiency =
1 + (ca /c j ) 2

Multiple-choice Questions
1. In solid propellants rocket, ammonium picrate is Codes:
usually added as: A B C
(a) an additive (b) an inhibitor (a) 1 2 3
(c) a darkening agent (d) a plasticiser (b) 2 1 3
(c) 2 3 2
2. A turbo prop is preferred to turbo-jet because (d) 3 1 2
(a) it has high propulsive efficiency at high speed
(b) it can fly at supersonic speed
Propulsive efficiency

III II
(c) it can fly at high elevations
(d) it has high power for take-off
3. The thrust of a jet propulsion power unit can be I
increased by
(a) injecting water into compressor
(b) burning fuel after gas turbine
(c) injecting ammonia into the combustion chamber
kmph
(d) All of the above
4. Figure 17.16 shows the propulsive efficiencies Figure 17.16
of three different engines. Based on the figure,
match List I with List II and select the correct 5. If V is the jet velocity V is the vehicle velocity, the
answer using the codes given below the List: propulsive efficiency of a rocket is given by
2( v0 / v1 ) ( v0 /v1 )
List I List II (a) (b)
v  v 
2
A. Rocket 1. Curve I 1+  0  1+  0 
 v1   v1 
B. Turbojet 2. Curve II
v0 v1
C. Propeller 3. Curve III (c) (d)
v0 + v1 v0 + v1

M17_THERMAL ENGINEERING_SE_XXXX_CH17.indd 955 6/16/2018 11:55:46 AM


956 Chapter 17

6. Consider the following statements relating to 11. Consider the following statements:
rocket engines: In open cycle turbo-jet engines used in military
I. The combustion chamber in a rocket engine aircraft, reheating the exhaust gas from the tur-
is directly analogous to the reservoir of a bine by burning more fuel is used to increase
supersonic wind tunnel. I. thrust
II. Stagnation conditions exist at the combustion II. the efficiency of engine
chamber. III. the range of aircraft
III. The exit velocities of exhaust gases are much Of these statements:
higher than those in jet engines. (a) I and III are correct
 IV. Efficiency of socket engines is higher than (b) I and II are correct
that of jet engines. (c) II and III are correct
Of these statements: (d) I, II, and III are correct
(a) I, III, and IV are correct
12. In a turbojet engine, subsequent to heat addition
(b) II, III, and IV are correct
to compressed air, to get the power output, the
(c) I, II, and III are correct
working substance is expanded in
(d) I, II, and IV are correct
(a) turbine blades, which is essentially an isen-
7. Consider the following statements about a rocket tropic process
engine: (b) turbine blades, which is polytropic process
I. It is a very simple in construction and opera- (c) exit nozzle, which is essentially an isentropic
tion. process
 II. It can attain very high vehicle velocity. (d) exit nozzle, which is a constant volume pro-
III. It can operate or very long duration. cess
Of these statements:

13. Which one of the following is the correct
(a) I and II are correct sequence of the position of the given components
(b) I and III are correct in a turboprop?
(c) II and III are correct (a) Propeller, compressor, turbine, burner
(d) I, II, and III are correct (b) Compressor, propeller, burner, turbine
8. Only rocket engines can be propelled to space (c) Propeller, compressor, burner, turbine
because (d) Compressor, propeller, turbine, burner
(a) they can generate very high thrust. 14. The absolute jet exit velocity from a jet engine is
(b) they have high propulsion efficiency. 2800 m/s and the forward flight velocity is 1400
(c) these engines can work on several fuels. m/s. The propulsive efficiency is
(d) they not air-breathing engines. (a) 33.33% (b) 40%
9. Consider the following statements: (c) 66.67% (d) 90%
As compared to turboprop, a turbojet 15. The efficiency of jet engine is
I. can operate at higher altitudes (a) higher at high speed
 II. can operate at higher flight velocities (b) lower at low speed
III. is more fuel efficient at lower speeds (c) higher at high altitude
Of these statements: (d) same at all altitude
(a) I, II, and III are correct
16. The propulsive efficiency of a turbojet aircraft
(b) I and II are correct
approaches 100% when the thrust approaches
(c) II and III are correct
(a) Maximum
(d) I and III are correct
(b) 50% of the maximum
10. Propulsion efficiency of a jet engine is given by (c) 25% of the maximum
(where u is flight velocity and V is jet velocity (d) Zero
relative to aircraft).
17. Which one of the following is the correct expres-
(a) 2u [V - u] (b) [V + u] 2u
sion for the propulsive efficiency of a jet plane
(c) 2u [V + u] (d) [V - u] 2u
(neglecting the mass of fuel)?

M17_THERMAL ENGINEERING_SE_XXXX_CH17.indd 956 6/16/2018 11:55:46 AM


Jet Propulsion 957

2 2 Of these statements:
(a) h p = hp =
(b) (a) I and II are correct
 va   j
v
  +1  v  + 1 (b) II and III are correct
 vj  a (c) I and III are correct
(d) I, II, and III are correct
2 2
(c) h p = hp =
(d) 22. The propulsive efficiency of a turbojet aircraft
 va   vj 
  −1  v  − 1 approaches 100% when the thrust approaches
 vj  a (a) maximum
(b) 50% of the maximum
18. The relative jet exit velocity from a rocket is 2700 (c) 25% of the maximum
m/s. The forward flight velocity is 1350 m/s. (d) zero
What is the propulsive efficiency of the unit?
(a) 90% (b) 66.66% 23. An aeroplane travels at 400 km/h at sea level
(c) 50% (d) 33.33% where the temperature is 15°C. The velocity of
the aeroplane at the same Mach number at an alti-
19. Consider the following statements regarding per- tude where a temperature of -25°C is prevailing,
formance of turbojet engines: would be
I. The thrust decreases at higher altitudes due to (a) 126.78 km/h (b) 130.6 km/h
reduced density of air and consequently lower (c) 371.2 km/h (d) 400.10 km/h
mass flow of air. 24. In turbo prop, the expansion of gases takes place
II. Any subsonic speed, the effect of increased approximately
velocity is to increase the air flow and the (a) 100% in the turbine
thrust increases. (b) 80% in the turbine and 20% in the nozzle
III. The relative velocity of jet with respect to the (c) 50% in the turbine and 50% in the nozzle
medium decreases at higher speeds which (d) 100% in the nozzle
tends to reduce the thrust. 25. With reference to turbojet and rocket engines,
 IV. For turbojet engine, the thrust of jet at sub- consider the following statements:
sonic speeds remains relatively constant. I. Efficiency of rocket engines is higher than
Which of the statements given above are cor- that of jet engines.
rect? II. Exit velocities of exhaust gases in rocket
(a) I, II, III, and IV engines are much higher than those in jet
(b) I and III engines.
(c) I, II, and IV III. Stagnation conditions exist at the combustion
(d) II, III, and IV chamber in rocket engines.
20. Consider the following statements:  IV. Rocket engines are air-breathing engines.
The thrust of rocket engine depends on Which of these statements are correct?
 I. Effective jet velocity (a) I and II (b) I, III, and IV
II. Weight of the rocket (c) II, III, and IV (d) I, II, and III
III. Rate of propellant consumption 26. Consider the following statements indicating a
Of these statements: comparison between rocket and jet propulsion
(a) I and II are correct systems:
(b) I and III are correct    I. Both rocket and jet engines carry the fuel and
(c) II and III are correct oxidant
(d) I, II, and III are correct   II. Rockets do not employ compressor or propeller
III. Rockers can operate in the vacuum too
21. Consider the following statements:   IV. Rockers can use solid fuels and oxidants
In a turbojet engine, thrust may be increased by (a) I, II, III, and IV
 I. Increasing the jet velocity (b) Only I and II
 II. Increasing the mass flow rate of air (c) Only II, III, and IV
III. After burning of the fuel (d) Only I, III, and IV

M17_THERMAL ENGINEERING_SE_XXXX_CH17.indd 957 6/16/2018 11:55:48 AM


958 Chapter 17

27. Match List I with List II in respect of a chemical Codes:


rocket engine and select the correct answer using A B C D
the codes given below the Lists: (a) 1 3 2 4
(b) 2 3 4 1
List I List II (c) 2 4 3 1
A. Ethy alcohol 1. Liquid oxidizer (d) 4 1 2 3
B. Nitrocellulose 2. Liquid fuel
C. Ammonium 3. Solid oxidiser
perchlorate 
D. Hydrogen peroxide 4. Solid fuel

Explanatory Notes
2 va 2 × 1400 2800
14. (c) h p = = = = 0.6667 or 66.67%
v j + va 2800 + 1400 4200
2 va 2 × 1350 2700 2
18. (b) h p = = = = = 0.6667 or 66.67%
v j + va 2700 + 1350 4050 3
2 va 2 × 1350 2700 2
22. (c) h p = = = = = 0.66666 = 66.666%
v j + va 2700 + 1350 4050 3
248
23. (c) Sonic velocity, vs ∝ T = 400 = 371.2 km/h
288

Review Questions
1. Explain the principle of jet propulsion.
2. List the various jet propulsion systems.
3. Which are piloted and pilotless jet propulsion systems?
4. How is rocket propulsion different from jet propulsion?
5. Describe the working of a screw propeller. What are the advantages of ramjet engine?
6. Explain ram effect and ram pressure. What are the advantages of ram-ret engine?
7. Explain the working of a pulse-jet engine. What are its limitations?
8. Describe the construction of a turbo-jet engine. What are its advantages and disadvantages?
9. How is a turbo-prop engine different from a turbo-jet engine?
10. Define thrust and thrust power.
11. Define propulsive power and propulsive efficiency.
12. Define thrust specific fuel consumption (TSFC).
13. Define thermal efficiency and overall efficiency.
14. What is jet efficiency and nozzle efficiency?
15. Define thrust work, propulsive work, and propulsive efficiency for a rocket engine.

Exercises
17.1 In a propulsion unit, air is drawn into the compressor at 15°C, 1 bar, and delivered at 4 bar. The isentropic
efficiency of compression is 82% and the compression is uncooled. The air is then heated at constant pres-
sure to 750°C. The air then passes through a turbine which drives the compressor only and has an isentropic

M17_THERMAL ENGINEERING_SE_XXXX_CH17.indd 958 6/16/2018 11:55:50 AM


Jet Propulsion 959

efficiency of 78%, followed by expanding through a nozzle with an efficiency of 88% to atmospheric pres-
sure of 1 bar. Neglecting any mass increase due to the mass of fuel, determine: (a) the power required to
drive the compressor, (b) the air-fuel ratio if calorific value of fuel is 42,000 kJ/kg (c) pressure of gases
leaving the turbine, and (d) thrust per kg of air per second. Assume for air: R = 0.287 kJ/kg K, g = 1.4.
[Ans. 15.1 171.38 kW, 74.1:1, 1.72 bar, 469.95 N/kg/s]
17.2   A jet propelled plane consuming air at the rate of 18.2 kg/s is to fly at Mach number 0.6 of an altitude of 4.5
km (pa = 0.55 bar, Ta = 255 K). The diffuser which has a pressure ratio of 5 and maximum temperature in
the combustion chamber is 1273 K. After expanding in the turbine the gases continue to expand in the nozzle
to a pressure of 0.69 bar. The isentropic efficiencies of compressor, turbine, and nozzle are 0.81, 0.85, and
0.915, respectively. The heating value of fuel is 45870 kJ/kg. Given cpa = 1.005 kJ/kg.K, cpg = 1.147 kJ/kg.K,
ga = 1.4, gg = 1.33. Calculate (a) power input to the compressor, (b) power output of the turbine, (c) fuel air
ratio, (d) thrust provided by the engine, and (e) thrust power developed.
[Ans. 3608.27 kW, 3608.27 kW, 0.0205, 9087 N, 1743.96 kW]
17.3   A turbo-prop aircraft is flying at 800km/h at an altitude, where the ambient conditions are 0.567 bar and
-20°C. Compressor pressure ratio is 8:1. Maximum gas temperature is 1100 K. The intake duct efficiency is
0.95 and total head isentropic efficiency of compressor and turbine is 0.92 and 0.95, respectively. Calculate
the specific power output in kJ/kg.s air/s, the thermal efficiency of the unit taking mechanical efficiency of
transmission as 0.96. Assume that exhaust gases leave the aircraft at 800 km/h relative to the aircraft. Take
cpa = 1.005 kJ/kg.K, ga = 1.4, cpg = 1.147 kJ/kg. K, gg = 1.33. [Ans. 226.23 kJ/kg, 34.17%]
17.4   In a jet-propulsion unit air is compressed by means of uncooled rotary compressor, the pressure at the
delivery being 3.5 times of that at the entrance, and the temperature rise during compression is 1.15 times
of that for frictionless adiabatic compression. The air is then led to the combustion chamber where the
fuel is burned under constant pressure conditions. The products of combustion at 480°C pass through
turbine which drives the compressor. The exhaust gases form the turbine are expanded in the nozzle down
to atmospheric pressure, the ambient conditions are 1 bar, 10°C. Calculate the power required to drive the
compressor per kg of air per second, the air-fuel ratio if the calorific value of fuel is 43300 kJ/kg, and static
thrust developed per kg of air per second. Assume that the values of R and g  after combustion remain same
as that for air. R = 0.287 kJ/kg.K, g = 1.4. [Ans. 140.92 kJ/kg/s, 130.74:1, 414.93 N/kg of air/s]
17.5   The speed of a jet-propulsion aircraft is 200 m/s relative to still air. The total pressure and total temperature
at intake to uncooled compressor are 0.7 bar and 0°C. The total temperature and total pressure of gases
entering the turbine are 760°C and 3.15 bar. The isentropic efficiencies of compressor and turbine are 85%
and 80%, respectively. The static back pressure on the propulsion nozzle is 0.55 bar and the efficiency of
nozzle based on the total pressure drop available is 0.90%.
  Neglecting other losses and mass increases due to fuel consumed, determine (a) the power required to
drive the compressor per kg per second, (b) the air-fuel ratio of the fuel has calorific value of 41860 kJ/kg,
(c) the total pressure of gases leaving the turbine, and (d) the thrust per kg per second. Assume Cpa = 1.005
kJ/kg. K, ga = 1.4., Cpg = 1.17 kJ/kg. K, and gg = 1.33. [Ans. 173.65 kW/kg, 63.1:1, 1.418 bar, 426 N/kg.s]
17.6   A turbojet aircraft is flying at a speed such that the corresponding Mach number is 0./9, where the ambient
conditions are 0.5 bar and -20°C. The compressor pressure ratio is 8:1. The maximum cycle temperature is
1250 K with fuel of calorific value 44000 kJ/kg. The pressure loss in the combustor is 0.1 bar. The ram air
efficiency = 0.9, isentropic efficiency of compressor = 0.85, isentropic efficiency of turbine 0.80, combus-
tion efficiency = 0.98, transmission efficiency of turbine to compressor = 0.98, nozzle efficiency = 0.90, and
outlet area of nozzle = 0.1 m. Calculate mass flow rate, the thrust developed, and specific fuel consumption.
Assume cpa = 1.004 kJ/kg. K, ga = 1.4, cpg = 1.15kJ/kg. K, and gg = 1.335.
[Ans. 19.25 kg/s, 147.5 kJ/kg, 0.04 hg/kwh]
17.7   The exit velocity from a jet unit is 650 m/s for an air flow of 40hg/s through the unit. The aircraft is flying at
250 km/h. Calculate the thrust developed, the thrust power and the propulsion efficiency. Neglect the effect
of fuel. [Ans. 23222.4 N, 1612.56 kW, 193%]

M17_THERMAL ENGINEERING_SE_XXXX_CH17.indd 959 6/16/2018 11:55:50 AM


960 Chapter 17

ANSWERS TO MULTIPLE-CHOICE QUESTIONS


1. (a) 2. (d) 3. (a) 4. (a) 5. (a)
6. (c) 7. (b) 8. (d) 9. (b) 10. (c)
11. (b) 12. (a) 13. (c) 14. (c) 15. (a)
16. (d) 17. (b) 18. (b) 19. (b) 20. (b)
21. (a) 22. (d) 23. (c) 24. (a) 25. (a)
26. (c) 27. (c)

M17_THERMAL ENGINEERING_SE_XXXX_CH17.indd 960 6/16/2018 11:55:50 AM


Chapter 18

Introduction to Refrigeration

18.1 ❐ INTRODUCTION
The process of producing cold by artificial means may be called refrigeration. The term ‘refrigeration’
may be defined as the artificial withdrawal of heat, producing in a substance or within a space, a tem-
perature lower than that which would exist under the natural influence of surroundings. In other words,
the refrigeration means a continued extraction of heat from a body whose temperature is already below
the temperature of its surroundings. In refrigeration, heat is virtually being pumped from a lower tem-
perature to a higher temperature.
According to the second law of thermodynamics, the extraction of heat from a body at lower tem-
perature to a higher temperature can only be performed with the aid of some external device. Such
a device is called refrigerator. A refrigerator is a reversed heat engine or a heat pump which extracts
heat from a cold body and delivers it to a hot body. The substance or medium which works in a heat
pump to extract heat from a cold body and to deliver to a hot body is called a refrigerant.

18.2 ❐ REFRIGERATION SYSTEMS


A refrigeration system is a device to produce the refrigeration effect. Some of the popular refrigera-
tion systems are:
1. Ice refrigeration-used in hotels for keeping the drinks cold.
2. Evaporative refrigeration-desert bag, artificial snow.
3. Air refrigeration-aircrafts.
4. Gas throttling refrigeration-liquefaction of gases, like air, nitrogen and oxygen.
5. Vapour compression refrigeration-domestic refrigerator.
6. Vapour absorption refrigeration-ice making using aqua ammonia or water plus lithium bromide
as the absorbent.
7. Steam jet refrigeration-comfort cooling in air-conditioning installations or in industrial processes.
8. Liquid gases refrigeration-transportation vehicles for perishable items.
9. Dry ice (solid CO2) refrigeration-transportation of perishable items.

M18_THERMAL ENGINEERING_SE_XXXX_CH18.indd 961 6/16/2018 11:04:58 AM


962 Chapter 18

18.3 ❐ METHODS OF REFRIGERATION


The methods of refrigeration are:
1. Vapour compression refrigeration.
2. Vapour absorption refrigeration.
3. Ejector-compression refrigeration.
4. Electro-Lux refrigeration.
5. Solar refrigeration.
6. Thermo-electric refrigeration.
7. Vortex-tube refrigeration.

18.3.1  Vapour Compression Refrigeration System


The schematic diagram of simple vapour compression refrigeration system is shown in Fig. 18.1.
It consists of the following five essential parts:
1. Compressor: The low pressure and temperature vapour refrigerant from evaporator is drawn into
the compressor through the inlet or suction valve A, where it is compressed to a high pressure
and temperature. This high pressure and temperature vapour refrigerant is discharged into the
condenser though the delivery, or discharge valve B.
2. Condenser: The condenser or cooler consists of coils of pipe in which the high pressure and
temperature vapour refrigerant is cooled and condensed. The refrigerant, while passing through

Insulated cold chamber

Low pressure liquid


vapour mixture

Pressure gauge Low pressure vapour Expansion valve


or refrigerant
Pressure gauge control valve

Low pressure side High pressure vapour High pressure liquid


High pressure side
A B

Condenser
Compressor
Receiver
High pressure liquid vapour mixture

Figure 18.1  Simple vapour compression refrigeration system

M18_THERMAL ENGINEERING_SE_XXXX_CH18.indd 962 6/16/2018 11:04:59 AM


Introduction to Refrigeration 963

the condenser, gives up its latent heat to the surrounding condensing medium which is normally
air or water.
3. Receiver: The condensed liquid refrigerant from the condenser is stored in a vessel known as
receiver from where it is supplied to the evaporator through the expansion valve or refrigerant
control valve.
4. Expansion valve: It is also called throttle valve or refrigerant control valve. The function of the
expansion valve is to allow the liquid refrigerant under high pressure and temperature to pass
at a controlled rate after reducing its pressure and temperature. Some of the liquid refrigerant
evaporates as it passes through the expansion valve, but the greater portion is vapourised in the
evaporator at the low pressure and temperature.
5. Evaporator: An evaporator consists of coils of pipe in which the liquid vapour refrigerant at low
pressure and temperature is evaporated and changed into vapour refrigerant at low pressure and
temperature. In evaporating, the liquid vapour refrigerant absorbs its latent heat of vaporisation
from the medium (air, water or brine) which is to be cooled.
Note: In any compression refrigeration system, there are two different pressure conditions. One is
called the high pressure side and other is known as low pressure side. The high pressure side includes
the discharge line (i.e. piping from delivery valve B to the condenser), condenser, receiver and expan-
sion valve. The low pressure side includes the evaporator, piping from the expansion valve to the
evaporator and the suction line (i.e. piping form the evaporator to the suction valve A).

18.3.2  Vapour Absorption System


A simple vapour absorption system consists of a condenser, an expansion device, an evaporator, an
absorber, a pump, a generator and a pressure reducing valve. The system can be improved by incor-
porating a regenerative heat exchanger between the poor and rich solutions. The two commonly used
refrigerant absorbent pairs are ammonia-water (aqua ammonia) and lithium bromide-water. Lithium
bromide-water system is superior to ammonia water system. In aqua ammonia, ammonia works as
the refrigerant and water as the absorber, whereas in the other pair, water acts as the refrigerant and
lithium bromide as the absorber.
A schematic diagram of absorption refrigeration system is shown in Fig. 18.2. The working of the
ammonia absorption system is described below:
Ammonia vapour is produced in the generator at high pressure from aqua ammonia by external
heating. The water vapour are removed by passing through an analyser and rectifier. The dehydrated
ammonia enters the condenser where vapours are condensed. The sub-cooled high pressure liquid
ammonia is passed through a throttle valve to further lower its temperature. Now they enter the evap-
orator and leave as saturated vapour. The saturated vapour goes to absorber to become strong after
absorbing ammonia vapour and the strong solution is pumped to the generator through the pump and
heat exchanger. The solution in the generator becomes weak as ammonia vapour comes out of it.
The weak ammonia solution from the generator is passed to the heat exchanger through the pressure
reducing valve. The cycle is repeated.

18.3.3  Ejector-Compression System


The system using water as the refrigerant in a vapour compression cycle in which compression is
achieved by the principle of jet compression employing its own vapour as the motive vapour is called

M18_THERMAL ENGINEERING_SE_XXXX_CH18.indd 963 6/16/2018 11:04:59 AM


964 Chapter 18

Analyser
Absorber

Generator

Rectifier Heat Pump


exchanger
Pressure
reducing valve Evaporator

Throttle
Condenser
valve

Figure 18.2  Ammonia absorption system

steam-ejector system. Refrigeration effect is obtained by direct evaporation and subsequent self cool-
ing of water.
Bubble point temperature is the temperature at which a liquid mixture begins to boil. Azeotrope is
a constant temperature boiling mixture.

18.3.4  Electro-Lux Refrigeration


This vapour absorption system uses ammonia as the refrigerant and hydrogen as the inert gas medium.
The inert gas is confined only to the low side of the system, i.e., evaporator and absorber. In the evap-
orator, ammonia evaporates in the presence of hydrogen atmosphere. The partial pressure of ammonia
is much lower than its pressure and thus give very low temperature. Hydrogen has the advantage of
being non-corrosive and insoluble in water. This system is used for domestic refrigeration.

18.3.5  Solar Refrigeration


This uses the strong aqua ammonia vapour absorption system by utilizing the solar energy as the heat
source to produce the refrigeration effect. This is an intermittent cycle. Lithium chloride-water system
has also been used.

18.3.6  Thermo-electric Refrigeration


It is based on the thermoelectric effects: Seebeck effect, Peltier effect, and the Thomson effect. These
effects are reversible in nature. The other two irreversible effects associated with thermoelectricity are

M18_THERMAL ENGINEERING_SE_XXXX_CH18.indd 964 6/16/2018 11:05:00 AM


Introduction to Refrigeration 965

the energy. Joule effect and conduction effect. It has low COP but infinite life expectancy, no leakage
problem.

18.3.7  Vortex Tube Refrigeration


Vortex tube is a simple device for producing cold. It consists of a nozzle, diaphragm, valve, hot air
side and cold air side. This is the only device to produce the refrigeration effect by utilizing the waste
heat energy.

18.4 ❐ UNIT OF REFRIGERATION


The capacity of a refrigeration unit is given in tons of refrigeration. A ton of refrigeration is defined
as the quantity of heat to be removed in order to form one ton of ice in 24 hours when the initial tem-
perature of water is 0°C. 1 ton of refrigeration, TR = 211 kJ/min or 3.5167 kW. Taking latent heat of
freezing of water equal to 336 kJ/kg, we have

1000 × 336
1 tonne of refrigeration = = 14, 000 kJ/h or 3.8889 kW (18.1)
24

Coefficient of performance (COP): The COP of a refrigeration system is a ratio of the heat removed
(cooling effect produced) from a system to the work supplied to achieve the heat removal.

Heat removed per unit time, Q ( kJ/s)


COP = (18.2)
Work supplied per unit time,W ( kJ/s)

211Tn ( kJ/min ) 3.5167


COP = = (18.3)
(60 P ) ( kJ/min )  P 
 T 
n

where P = energy required in kW


Tn = capacity of plant in tons

Actual COP
Relative COP = (18.4)
Theoretical COP

18.5 ❐ REFRIGERATION EFFECT


During evaporation, the liquid-vapour refrigerant absorbs its latent heat of vapourisation from the
medium (air, water, brine) which is to be cooled. This heat which is absorbed by the refrigerant is
called the refrigeration effect. In other words, the refrigeration effect is the amount of cooling produced.
Refrigeration effect = Enthalpy of vapour refrigerant − Enthalpy of liquid refrigerant.

M18_THERMAL ENGINEERING_SE_XXXX_CH18.indd 965 6/16/2018 11:05:01 AM


966 Chapter 18

18.6 ❐ CARNOT REFRIGERATION CYCLE


The reversed Carnot cycle is shown in Fig. 18.3 on p-v and T-s. diagrams. The sequence of operations
is given below:
1. The process 1-2 represents the isentropic compression of the working fluid with the aid of exter-
nal work. The temperature of the working fluid rises from T1 to T2.
2. The process 2-3 represents the isothermal compression of the working fluid during which heat Qr
is rejected at constant high temperature T2.
3. The process 3-4 represents the isentropic expansion of the working fluid. The temperature of
working fluid drops from T2 to T1.
4. The process 4-1 represents the absorption of heat Qa by the working fluid from refrigerator at
constant low temperature T1 during isothermal expansion.

Heat absorbed in refrigerator Qa


COP = =
Work done Qr − Qa

T1 ( s1 − s4 ) T1 h1 − h4
= = = (18.5)
T2 ( s1 − s4 ) − T1 ( s1 − s4 ) T2 − T1 ( h2 − h3 ) − ( h1 − h4 )
  
The Carnot cycle is practically not feasible since isothermal energy rejection requires extremely slow
motion followed by isentropic process during which the piston should move at extremely faster rate,
which is mechanically not obtainable.
The COP of reversed Carnot cycle is the highest of all refrigeration cycles. COP increases as
T1 is increased or T2 is lowered. The increase in COP for increased T1is more than for decreased
T2. Since T2 in winter is less than T2 in summer therefore COP in winter is more than that of in
summer.
If the cycle is used as a heat pump, then energy performance ratio,

Heat supplied to room T2


EPR = =  (18.6)
Work done T2 − T1

p T
3 pv = c Condensation
Isentropic
T2 3 2 expansion
Qr
2 Isentropic
pvg = c WE compression
Qa Wc
T1 1
4
1 Evaporation
4
v v
(a) (b)

Figure 18.3  Reversed Carnot cycle: (a) p-v diagram, (b) T-s diagram

M18_THERMAL ENGINEERING_SE_XXXX_CH18.indd 966 6/16/2018 11:05:03 AM


Introduction to Refrigeration 967

18.7 ❐ DIFFERENCE BETWEEN A HEAT ENGINE,


REFRIGERATOR AND HEAT PUMP
Heat Engine: In a heat engine, as shown in Fig. 18.4(a), the heat supplied to the engine is converted
into useful work. If Q2 is the heat supplied to the engine and Q1 is the heat rejected from the engine,
then the net work done by the engine is given by,
We = Q2 − Q1
Efficiency of heat engine,

Work done We Q2 − Q1 Q T
he = = = = 1 − 1 = 1 − 1 (18.7)
Heat supplied Q2 Q2 Q2 T2

Refrigerator: A refrigerator is an equipment used to remove the heat continuously from the space
(sink). It maintains the temperature below atmospheric temperature and reject the heat to the atmos-
phere (source) by increasing the temperature potential of the heat to be rejected with the help of
mechanical work input (compressor).
The refrigeration system is shown in Fig. 18.4(b). In a refrigerator system Q1 quantity of heat is
removed from the source where its temperature T1 is maintained below atmospheric temperature T2.
COP of refrigerator is,

Q1 Q1 Q1 / Q2 (T / T ) T1
Refrigeration effect = (COP) ref = = = = 1 2 = (18.8)
Wref Q2 − Q1 1 − (Q1 / Q2 ) 1 − T1 / T2 T1 − T2

Heat Pump: It is an equipment used to supply the heat continuously to the space. It maintains the
temperature above atmospheric temperature by absorbing the heat from the atmosphere and increasing
its temperature potential with mechanical work input (compressor). The heat pump system is shown in
Fig. 18.4(c). In a heat pump system, Q2 quantity of heat is supplied to the room where the temperature
T2 is maintained above atmospheric temperature T1. The heat Q1 is obtained from the atmosphere.

Sink Sink room


Hot body Source
atmosphere T2
T2
T2

Q2 Q2 Q2

HE We R Wr HP Wp

Q1 Q1 Q1

Source Source
T1
Cold body (Room) (Atmosphere)
Sink
T1 T1

(a) (b) (c)

Figure 18.4  (a) Heat engine, T1> Tatm , (b) Refridgerator T1< Tatm , (c) Heat pump

M18_THERMAL ENGINEERING_SE_XXXX_CH18.indd 967 6/16/2018 11:05:05 AM


968 Chapter 18

COP or Energy performance ratio of heat pump is,

Q2 Q1 + W p Q
(COP) HP or EPR = = = 1 + 1 = 1 + (COP) ref  (18.9a)
Wp Wp Wp

Q2 Q1 T2 1
       = = 1+ = =  (18.9b)
Q2 − Q1 Q2 − Q1 T2 − T1 1 − (T1 / T2 )

Example 18.1
Find the COP of a refrigeration system if the work input is 10 kJ/kg and refrigeration effect pro-
duced is 180 kJ/kg of refrigerant flowing.
Solution

wref =100 kJ/kg, q =180 kJ/kg


q 180
(COP) ref = = = 1.8
wref 100

Example 18.2
A machine working on a Carnot cycle operates between 310 K and 270 K. Calculate the COP when
it is operated as:
(a)  a refrigerator, and
(b)  a heat pump.

Solution
Given: T1 = 270 K, T2 = 310 K
(a) Refrigerator
T1 270 270
(COP) ref = = = = 6.75
T2 − T1 310 − 270 40
(b) Heat pump:
T1 310 310
(COP) HP = = = = 7.75
T2 − T1 310 − 270 40

Example 18.3
A Carnot refrigeration cycle absorbs heat at 268 K and rejects it at 298 K.
(a) Calculate the COP of this refrigeration cycle.
(b) If the cycle is absorbing 1150 kJ/min at 268 K, find the rate of work input required.
(c) How many kJ/s will be delivered if it works as a heat pump at 298 K and absorbs 1150 kJ/min
at 268 K.

M18_THERMAL ENGINEERING_SE_XXXX_CH18.indd 968 6/16/2018 11:05:08 AM


Introduction to Refrigeration 969

Solution
Given: T1 = 268 K, T2 = 298 K, Q1 = 1150 kJ/min
(a) Carnot refrigeration cycle:
T1 268 268
(COP) ref = = = = 8.933
T2 − T1 298 − 268 30
Q1 1150
Wref =
Work required,      = = 128.73 kJ/min or 2.145 kJ/s
(COP) ref 8.933
(b) Heat pump:

(COP) HP = 1 + (COP) ref = 1 + 8.933 = 9.933

Q2 Q2
or 9.933 = =
Q2 − Q1 Q2 − 1150
9.933 Q2 − 11422.95 = Q2

11422.96
Q2 = = 1278.71 kJ/min or 21.31 kJ/s
8.933

18.8 ❐ POWER CONSUMPTION OF A REFRIGERATING


MACHINE
The power consumption of a refrigerating machine is determined in terms of kW. However the power
consumption of the driving motor is sometimes rated in horsepower (H.P). we have
Wref
HP =
0.736
where Wref is in kW and HP is in metric units,
HP
Power consumption per ton refrigeration = (18.10)
kW refrigeration
Wref
  =
   (18.11)
0.736 Q1
where Q1 = refrigerating capacity in kW
Since 1 TR = 3.5167 kW, therefore
HP Wref × 3.5167 1.78
= =  (18.12)
TR 0.736 Q1 (COP) ref

M18_THERMAL ENGINEERING_SE_XXXX_CH18.indd 969 6/16/2018 11:05:11 AM


970 Chapter 18

Example 18.4
The ambient air temperature during summer and winter in a particular locality are 45°C and 15°C
respectively. Find the value of Carnot COP for an air-conditioner for cooling and heating, corre-
sponding to refrigeration temperatures of 5°C for summer and heating temperature of 55°C for
winter. Also find the theoretical power consumption per ton of refrigeration in each case.

Solution
For summer: T1 = 273 + 5 = 278 K, T2 = 45 + 273 = 318 K
T1 278 278
For cooling, Carnot (COP)cooling = = = = 6.95
T2 − T1 318 − 278 40
211
Power consumption,    Wcooling =
= 30.36 kJ/min or 0.506 kW
6.95
For winter:     T1 = 273 + 15 = 288 K, T2 = 273 + 55 = 328 K
T2 328 328
For heating,      (COP) heating = = = = 8.2
T2 − T1 328 − 288 40
211
Power consumption,   Wheating = = 25.732 kJ/min or 0.429 kW
8.2

Example 18.5
A refrigerator has working temperature of -30°C and 35°C. The actual COP is 0.75 of the maximum.
Calculate the power consumption, and heat rejected to the surroundings per ton of refrigeration.

Solution
T1 273 − 30 243
 (COP)Carnot = = = = 3.74
T2 − T1 35 − ( − 30) 65
 Actual COP = 0.75 × 3.74 = 2.8
3.5167
Power consumption per ton = = 1.256 kW
2.8
Heat rejected per ton = 3.5167 + 1.256 = 4.7727 kW

18.9 ❐ AIR REFRIGERATION CYCLES


There are two types of air refrigeration cycles.
1. Open air refrigeration cycle.
2. Closed (or dense) air refrigeration cycle.

M18_THERMAL ENGINEERING_SE_XXXX_CH18.indd 970 6/16/2018 11:05:13 AM


Introduction to Refrigeration 971

18.9.1  Open Air Refrigeration Cycle


In an open air refrigeration cycle, the air is directly (at atmospheric pressure) led to the space to be cooled
(i.e. refrigerator). The air is then allowed to circulate through the cooler and returned to the compressor
to start another cycle. This requires size of the compressor and expander to be large. The moisture is
regularly carried away by the air circulated through the refrigerator. This leads to the formation of frost
at the end of expansion process and clog the line. Therefore, a drier is required in the open cycle system.

18.9.2  Closed (or dense) Air Refrigeration Cycle


In a closed air refrigeration cycle, the air is passed through the pipes and components parts of the sys-
tem at all times. The air is used for absorbing heat from the other fluid (say brine) which is circulated
into the space to be cooled. The air does not come in contact directly with the space to be cooled.
This system has the following advantages over the open cycle:
1. The suction pressure is higher than that of atmospheric pressure Therefore, the volume of air
handled by the compressor and expander is smaller.
2. The operating pressure ratio can be reduced, which results in higher coefficient of performance.

18.10  ❐  REVERSED CARNOT CYCLE


A reversed Carnot cycle, using air as working medium is shown in Fig. 18.5 on p-v and T-s diagrams.
The reversed Carnot cycle is represented by four processes as described below:
1. Process 1-2: Isentropic compression
During this process, the pressure of air increases from p1 to p2, specific volume decreases from v1 to
v2 and temperature increases from T1 to T2. No heat is absorbed or rejected by air during this process.
2. Process 2-3: Isothermal compression
During this process, the pressure of air increases from p2 to p3 and specific volume decreases
from v2 to v3. Temperature remains constant at T2 = T3.
Heat rejected by air per kg of air, q2-3 = area 2-3-3′-2′
   = T3 (s2 – s3) = T2 (s2 – s3)
p
T

p3 3

T2 = T3 3 2
p2 2

wcycle
p4
4
p1 1 T1 = T4 1
4
q4 − 1
s3 = s4 s1 = s2
v s
0 v3 v4 v2 v1 0 3′ 2′
(a) (b)

Figure 18.5  Reversed Carnot Cycle: (a) p-v diagram, (b) T-s diagram

M18_THERMAL ENGINEERING_SE_XXXX_CH18.indd 971 6/16/2018 11:05:14 AM


972 Chapter 18

3. Process 3-4: Isentropic expansion


During this process, the pressure of air decreases from p3 to p4, specific volume increases from
v3 to v4 and temperature decreases from T3 to T4. No heat is absorbed or rejected by air.
4. Process 4-1: Isothermal expansion
During this process, the pressure of air decreases from p4 to p1 and specific volume increases
from v4 to v3. Temperature remains constant at T4 = T1.
Heat absorbed by air (or heat extracted from cold body) per kg of air.
q4 −1 = are 4-1-2′-3′
= T4 ( s1 − s4 ) = T4 ( s2 − s3 ) = T1 ( s2 − s3 )
Work done during the cycle per kg of air, wcycle = q2 −3 − q4 −1 = are 1-2-3-4
= T2 ( s2 − s3 ) − T1 ( s2 − s3 )
= (T2 − T1 ) ( s2 − s3 )
Coefficient of performance of refrigeration system,
Heat absorbed q4 −1
(COP) ref = =
Work done wcycle
T1 ( s2 − s3 )
  =
(T2 − T1 ) ( s2 − s3 )

T1
 =  (18.13)
T2 − T1

18.10.1  Temperature Limitations for Reversed Carnot Cycle


For a reversed Carnot cycle.
T1
(COP) ref =
T2 − T1

where T1 = lower temperature, and


   T2 = higher temperature.
The COP of the reversed Carnot cycle may be improved by the following methods:
1. Decreasing the higher temperature T2 of hot body.
2. Increasing the lower temperature T1 of cold body.
The temperature T1 and T2 cannot be varied at will, due to certain functional limitations. The lowest
possible refrigeration temperature is T1 = 0 (absolute zero) at which (COP)ref = 0. The highest possible
refrigeration temperature is T1 = T2 i.e. when the refrigeration temperature is equal to the temperature
of the surroundings at which ,(COP)ref = ∞. Thus reversed Carnot COP varies between 0 and ∞.
To obtain maximum possible COP in any application,
1. the cold body temperature T1 should be as high as possible, and
2. the hot body temperature T2 should be as low as possible.

M18_THERMAL ENGINEERING_SE_XXXX_CH18.indd 972 6/16/2018 11:05:18 AM


Introduction to Refrigeration 973

The lower the refrigeration temperature required and higher the temperature of heat rejection to the
surroundings, the larger is the power consumption of the refrigerating machine. Also, the lower is the
refrigeration temperature required, the lower is the refrigerating capacity obtained.
The temperature T2 in winter is less than T2 in summer. Therefore, (COP)ref in winter will be higher than
(COP)ref in summer. In other words, the Carnot refrigerator works more efficiently in winter than in sum-
mer. Similarly, if the lower temperature fixed by the refrigeration application is high, the (COP)ref will be
high. Thus a Carnot refrigerator used for making ice at 0°C (273 K) will have less (COP)ref than a Carnot
refrigerator used for air-conditioning plant in summer at 20°C when the ambient air temperature is 40°C.

18.10.2  Vapour as a Refrigerant in Reversed Carnot Cycle


The reversed Carnot cycle can be made almost practical by operating in the liquid-vapour region of a
pure substance as shown in Fig. 18.6 on T-s diagram.
1. Process 1-2: Isentropic compression
The vapours during compression are wet and becomes dry saturated at the end of the process.
Such a process is called wet compression. The temperature rises from T1 to T2.
Compression work per kg of refrigerant, w1− 2 = h2 − h1
2. Process 2-3: Condensation
During this process the temperature remains constant at T2. Heat is rejected from h2 to h3 and
refrigerant gets converted to liquid at the end of process.
q2 −3 = h2 − h3 = ( h fg )t2
3. Process 3-4: Isentropic expansion
During this process the flashing of the liquid refrigerant takes place with consequent temperature
drop from T2 to T1. The refrigerant becomes wet at the end of expansion.
  Work of expander, w3− 4 = h3 − h4
4. Process 4-1: Evaporation
During this process the wet refrigerant evaporates, heat is absorbed from the medium, and refrig-
eration effect is produced.

3 Condensation
T2 2

Isentropic Isentropic
expansion compression
T1 4 1
Evaporation
x4 x1

s3 = s4 s1 = s2
s

Figure 18.6  Reversed Carnot cycle with vapour as a refrigerant

M18_THERMAL ENGINEERING_SE_XXXX_CH18.indd 973 6/16/2018 11:05:19 AM


974 Chapter 18

Heat absorbed, q4 −1 = h1 − h4
Network, wnet = w1− 2 − w3− 4 = ( h2 − h1 ) − ( h3 − h4 )

      = ( h2 − h3 ) − ( h1 − h4 ) = q2 −3 − q4 −1

Refrigerating effect
   (COP)cooling =
Net work
h1 − h4
   =
( h2 − h3 ) − ( h1 − h4 )
1
      =  (18.14)
 h2 − h3 
 h − h  − 1
1 4

Example 18.6
A Carnot refrigerator has working temperatures of -30°C and 35°C. It operates with R-12 refrig-
erant as a working substance. Calculate the work of isentropic compression, isentropic expansion,
refrigeration effect, and COP of the cycle per kg of refrigerant.
If the actual COP is 80 percent of the maximum, calculate the power consumption and heat
rejected to the surroundings per ton of refrigeration.

Solution
Given: t1= -30°C, t2 = 35°C, Refrigerant = R-12, (COP)actual = 0.8(COP)max From the table of prop-
erties of R-12, we have (Refer to Fig. 18.6)
 s1 = s2 = 0.6839 kJ/(kg.K), s3 = s4 = 0.2559 kJ/(kg.K), h2 = 201.5 kJ/kg..
    h3 = 69.5 kJ/kg, s f 1 = s f 4 = 0.0371kJ/(kg.K), sg1 = sg 4 = 0.7171 kJ/(kg.K)
h f 1 = h f 4 = 8.9 kJ/kg, hg1 = hg 4 = 174.2 kJ/kg

s1 = s f 1 + x1 ( sg1 − s f 1 ) = 0.0371 + x1 (0.7171 − 0.371)


Now     

= 0.0371 + 0.68 x1 = 0.6839


x1 = 0.951
h1 = h f 1 + x1 ( hg1 − h f 1 ) = 8.9 + 0.951(174.2 − 8.9) = 166.1 kJ/kg
s4 = s f 4 + x4 ( sg 4 − s f 4 ) = 0.0371 + x4 (0.7171 − 0.0371) = 0.2559
x4 = 0.3218
 h4 = h f 4 + x4 ( hg 4 − h f 4 ) = 8.9 + 0.3218 (174.2 − 8.9) = 62.1 kJ/kg
Work of compression, w1− 2 = h2 − h1 = 201.5 − 166.1 = 35.4 kJ/kg

Work of expression, w3− 4 = h3 − h4 = 69.5 − 62.1 = 7.4 kg

Refrigerating effect, q4 −1 = h1 − h4 = 166.1 − 62.1 = 104 kJ/kg

M18_THERMAL ENGINEERING_SE_XXXX_CH18.indd 974 6/16/2018 11:05:25 AM


Introduction to Refrigeration 975

Heat rejected, q2 −3 = h2 − h3 = 201.5 − 69.5 = 132 kJ/kg

Network, wnet = w1− 2 − w3− 4 = 35.4 − 7.4 = 28 kJ/kg


q4 −1 104
(COP)cooling = = = 3.714
wnet 28
T1 273 − 30 243
or = = = = 3.738
T2 − T1 ( 273 + 35) − ( 273 − 30) 65
(COP)act = 0.8 × 3.738 = 2.99
3.5167
Power consumption per ton of refrigeration = = 1.1761 kW
2.99
Heat rejected per ton of refrigeration to the surroundings = 3.5167 + 1.1761 = 4.6928 kW

18.10.3  Gas as a Refrigerant in Reversed Carnot Cycle


Figure 18.7 shows the p-v and T-s diagrams for the reversed Carnot cycle with a gas as a refrigerant.
1. Process 1-2: Isentropic compression
During this process, the pressure increases from p1 to p2 and volume decreases from v1 to v2. Heat
transfer is zero so that q1− 2 = 0. Temperature of gas increases from T1 to T2.
Work done per kg of gas,
p2 v2 − p1v1 R(T2 − T1 )
w1− 2 = =
g −1 g −1
2. Process 2-3: Isothermal Compression
During this process, the pressure increases from p2 to p3 and volume decreases from v2 to v3. The
temperature remains constant at T2.

p T p3 p4 p2
p3 3
T2 3
2
T=c

2 p1
p2 pvg = c
4
T1 1
p4 T=c pvg = c
4
p1 1

v s
0 v3 v4 v2 v1 0 s3 = s4 s1 = s 2
(a) (b)

Figure 18.7 Reversed Carnot cycle with gas as a refrigerant: (a) p-v diagram,
(b) T-s diagrame

M18_THERMAL ENGINEERING_SE_XXXX_CH18.indd 975 6/16/2018 11:05:28 AM


976 Chapter 18

v2 v
Work done per kg of gas, w2 −3 = p2 v2 ln = RT2 ln 2
v3 v3
Heat rejected, q2 −3 = w2 −3 for a perfect gas

3. Process 3-4: Isentropic expansion


Pressure decreases from p3 to p4, volume increases from v3 to v4 and temperature decreases from
T2 = T3 to T4 = T1
p3 v3 − p4 v4 R(T2 − T1 )
Work done, w3− 4 = =
g −1 g −1
4. Process 4-1: Isothermal expansion
Pressure decreases from p4 to p1, volume increases from v4 to v1, and temperature remains con-
stant at T1.
v v
Work done,     w4 −1 = p4 v4 ln 1 = RT1 ln 1
v4 v4

Refrigerating effect,   q4 −1 = w4 −1 for a perfect gas

Net work of the cycle,  wnet = w2 −3 − w4 −1


v2 v
= RT2 ln − RT1 ln 1
v3 v4
v1
Refrigerating effect,    q4 −1 = RT1 ln
v4
For the isentropic processes 1-2 and 3-4, we have
g −1 g −1
T2  p2  g p  g
= = 3
T1  p1   p4 
g −1 g −1
T2  v1  v 
and = = 3 = rg −1
T1  v2   4
v
where r = compression ratio for the isentropic processes.
p2 p3 v v
Here = and r = 1 = 4
p1 p4 v2 v3
v2 v1 p p
or = = 3= 4
v3 v4 p2 p1
Since      p2 v2 = p3 v3 and p4 v4 = p1v1
v1
RT1 ln
q4 −1 v4
(COP)cooling = =
wnet v2 v
RT2 ln − RT1 ln 1
v3 v4

M18_THERMAL ENGINEERING_SE_XXXX_CH18.indd 976 6/16/2018 11:05:33 AM


Introduction to Refrigeration 977

T1 1 1
=
   = = 
T2 − T2 T2 g −1 (18.15)
−1 r − 1
T1

Thus, COP is a function of compression ratio only.

18.10.4  Limitations of Reversed Carnot Cycle


The limitations of reversed Carnot cycle are:
1. The isentropic compression and expansion processes require high speed while the isothermal
condensation and evaporation processes require an extremely low speed. This variation in speed
of air of a cycle is not practicable.
2. It is difficult to design an expander to handle a mixture of largely liquid and partly vapour.
3. Because of the internal irreversibilities in the compressor and the expander, the actual COP of the
reversed Carnot cycle is very low.
4. With gas as refrigerant, it is not possible to devise, in practice, isothermal processes of heat
absorption and rejection.
5. The stroke volume of gas cycle cylinder is very large resulting in poor actual COP.

18.11 ❐ BELL-COLEMAN CYCLE (OR REVERSED BRAYTON


OR JOULE CYCLE)
The Bell-Coleman air refrigeration cycle was developed by Bell-Coleman and Light Foot by reversing the
Joule’s or Brayton’s air cycle. The schematic diagram of such a cycle is shown in Fig. 18.8. It consists of
a compressor, a cooler, an expander, and a refrigerator. The p-v and T-s diagrams are shown in Fig. 18.9.
1. Process 1-2: Isentropic compression
The cold air from the refrigerator is drawn into the compressor cylinder where it is compressed
isentropically. The pressure and temperature of air increases and the specific volume decreases
from v1 to v2. No heat is absorbed or rejected by the air.
2. Process 2-3: Constant pressure cooling
The warm air from the compressor is passed into the cooler where it is cooled at constant pressure
p3 = p2. The temperature decreases from T2 to T3 and specific volume reduces from v2 to v3.
Heat rejected by air, q2–3= cp(T2 − T3)
3. Process 3-4: Isentropic expansion
The air from the cooler is drawn into the expander cylinder where it is expanded isentropically
from p3 to p4. The temperature falls from T3 to T4 and specific volume increases from v3 to v4. No
heat is absorbed or rejected by air during this process.
4. Process 4-1: Constant pressure expansion
The cold air from the expander is passed to the refrigerator where it is expanded at constant pres-
sure p4 = p1.The temperature of air increases from T4 to T1 and specific volume increases from v4
to v1.
Heat absorbed by air (or extracted from the refrigerator), q4 −1 = c p (T1 − T4 )

M18_THERMAL ENGINEERING_SE_XXXX_CH18.indd 977 6/16/2018 11:05:34 AM


978 Chapter 18

2 1
Compressor

Water out Brine in

Cooler Refrigerator
Water in Brine out

3 4
4
3

Expander

Figure 18.8  Schematic diagram of Bell-Coleman air refrigeration cycle


p T

3 p=c 2
p2 = p3 T2 2
p=c
T3 3
pvg = c g s=c
pv = c
s=c
T1 1
p=c
p1 = p4
1 T4
4 4 p=c

v s
0 v3 v4 v2 v1 s3 = s4 s1 = s2
(a) (b)

Figure 18.9  Bell-Coleman cycle: (a) p-v diagram, (b) T-s diagrame
Work done during the cycle per kg of air,
W = Heat rejected − Heat absorbed
= q2 −3 − q4 −1 = c p [(T2 − T3 ) − (T1 − T4 )]
Heat absorbed
Coefficient of performance, COP =
Work done

M18_THERMAL ENGINEERING_SE_XXXX_CH18.indd 978 6/16/2018 11:05:36 AM


Introduction to Refrigeration 979

q4 −1 c p (T1 − T4 )
= =
w c p [(T2 − T3 ) − (T1 − T4 )]
(T1 − T4 ) 1
= =
(T2 − T3 ) − (T1 − T4 )  T2 − T3 
 T − T  − 1
1 4
1 1
= =
T3  (T2 / T3 − 1)  T3
−1 −1
 
T4  (T1 / T4 − 1)  T4
 (18.16a)
g −1
T2  p2  g
For isentropic compression process 1-2, = and for isentropic expansion process 3-4,
g −1 T1  p1 
T3  p3  g
=
T4  p4 
T2 T3
Since p2 = p3 and p1 = p4, therefore, =
T1 T4

T2 T1
or =
T3 T4
g −1 g −1 g −1
T3  p3  g p  g g
Also   = = 2 = ( rp )
T4  p4   p1 

p2
where rp = = pressure ratio
p1
1 1
∴ COP = = (18.16b)
T3 g −1
−1
T4 ( rp ) g − 1

18.11.1  Bell-Coleman Cycle with Polytropic Processes


Let the compression and expansion processes take place according to the polytropic law. pv n = const.
1. Process 1-2: Polytropic compression
 n   n 
Work done per kg of air, w1− 2 =   ( p2 v2 − p1v1 ) =  R(T2 − T1 )
 n − 1  n − 1

2. Process 3-4: Polytropic expansion


 n   n 
w3− 4 =  ( p v − p4 v4 ) =  R(T3 − T4 )
 n − 1 3 3  n − 1

M18_THERMAL ENGINEERING_SE_XXXX_CH18.indd 979 6/16/2018 11:05:40 AM


980 Chapter 18

Net work done during the cycle per kg of air,


 n 
wnet =  R [(T2 − T1 ) − (T3 − T4 )]
 n − 1
3. Heat absorbed during constant pressure process 4-1.
q4 −1 = c p (T1 − T4 )

Heat absorbed c p (T1 − T4 )


        COP = =
Work done  n 
 n − 1 R [(T2 − T1 ) − (T3 − T4 )]

 c   1  (g − 1)c p
Now R = c p − cv = c p  1 − v  = c p  1 −  =
 cp   g g

c p (T1 − T4 )
∴        COP =
 n   g − 1
 n − 1  g  c p [(T2 − T1 ) − (T3 − T4 )]

T1 − T4
=
 n   g − 1
 n − 1  g  [(T2 − T3 ) − (T1 − T4 )]

1
= (18.17)
n −1
 n   g − 1
 n − 1  g  [( rp )
n − 1]

(a) For n = g, Eq. (18.17) reduces to Eq. (18.16b)


(b) For n < g, (COP)poly > (COP)isen
(c) For n > g, (COP)poly < (COP)isen
Therefore, for obtaining higher COP, the compression and expansion index should be kept less than
1.4.

Example 18.7
A closed cycle refrigeration system working between 5 bar and 20 bar extracts 144 MJ of heat per
hour. The air enters the compressor at 6°C and the expander at 22°C. The unit operates at 320 rpm.
Calculate by assuming isentropic compression and expansion:
(a)  Power required to operate the unit.
(b)  Bore of compressor, and
(c)  Refrigerating capacity in tonnes of ice at 0°C per day.
The following data is available:
Stroke for double acting compressor and expander = 300 mm
Mechanical efficiency of compressor      = 80%
Mechanical efficiency of expander      = 85%

M18_THERMAL ENGINEERING_SE_XXXX_CH18.indd 980 6/16/2018 11:05:43 AM


Introduction to Refrigeration 981

Solution
Given: p1 = p4 = 5 bar, p2 = p3 = 20 bar, Q4-1 = 144 MJ/h or 2400 kJ/min,
T1 = 273 + 6 = 279 K, T3 = 273 + 22 = 295 K, N = 320 rpm, L = 300 mm,
hc = 0.80, he = 0.85, g = 1.4
For p-v and T-s diagrams, refer to Fig. 18.9.
p2 20 p
(a) rp = = =4= 3
p1 5 p4
g −1 0.4
T2 g
= ( rp ) = ( 4) 1.4 = 1.486
T1
T2 = 279 × 1.486 = 414.6 K
g −1
T3 g
= ( rp ) = 1.486
T4
295
T4 = = 198.5 K
1.486
Heat extracted from the refrigeration system per kg of air,
q4 −1 = c p (T1 − T4 ) = 1.005 ( 279 − 198.5) = 80.9 kJ/kg

Q4 −1 2400
Mass of air circulated, m a = = = 29.67 kg/min
q4 −1 80.9
Work done during isentropic compression process 1-2,
 g  1
w1−1 =   R(T2 − T1 ) ×
 g − 1 hc

1.4 1
= × 0.287 ( 414.6 − 279) × = 170.26 kJ/kg
0.4 0.80
Work done during isentropic expansion process 3-4,
 g 
q3− 4 =  R(T3 − T4 )he
 g − 1
1.4
= × 0.287 ( 295 − 198.5) × 0.85 = 82.39 kJ/kg
0.4
Net work done per kg of air supplied to the system,
qnet = w1− 2 − w3− 4 = 170.26 − 82.39 = 87.87 kJ/kg

m a × wnet 29.67 × 87.87


Power required to operate the system, P = = = 43.45 kW
60 60
(b) p1V1 = ma RT1

M18_THERMAL ENGINEERING_SE_XXXX_CH18.indd 981 6/16/2018 11:05:48 AM


982 Chapter 18

29.67 × 0.287 × 279


V1 = = 4.75 m3 /min
2
5 × 10
p 2
Also   V1 = D L × 2 × N  (for double acting compressor)
4
p
4.75 = D 2 × 0.3 × 2 × 320
4
D = 0.1775 m or 177.5 mm
(c) Refrigerating capacity of the system per day = q 4 −1 × ma × 60 × 24
= 80.9 × 29.67 × 60 × 24 = 3456.436 MJ
Assuming latent heat of ice = 336 kJ/kg
3456.436 × 103
Ice formation capacity of system = = 10287 kg or 10.287 tonnes
336

Example 18.8
A closed cycle reversed Brayton refrigerator is required for a capacity of 12TR. The cooler pressure
is 4.5 bar and the refrigerator pressure is 1.5 bar. The air is cooled in the cooler at a temperature
of 45°C and the temperature of air at inlet to compressor is 20°C. Calculate for the ideal cycle:
(a) COP, (b) mass of air circulated per minute, (c) theoretical displacement of compressor piston,
(d) theoretical displacement of expander piston, and (e) net power per tonne of refrigeration. The
compression and expansion are isentropic.
Solution
Given: Q = 12TR, p1 = p4 = 1.5 bar, p2 = p3 = 4.5 bar, T1 = 273 − 20 = 253 K, T3 = 273 + 45 = 318 K.
The p-v and T-s diagrams are shown in Fig. 18.10.
g −1 1.4 −1
T2  p2  g  4.5  1.4
(a) = = = 1.369
T1  p1   1.5 
T2 = 253 × 1.369 = 346.3 K
g −1
T3  p3  g
= = 1.369
T4  p4 
318
T4 = = 232.3 K
1.369
T1 − T4 253 − 232.3 20.7
COP = = = = 2.724
(T2 − T3 ) − (T1 − T4 ) (346.3 − 318) − ( 253 − 232.3) 7.6

(b) Heat extracted per minute, Q = 211 × 12 = 2532 kJ/min


Heat extracted from the refrigerator per kg of air,
q = c p (T1 − T4 ) = 1.005 ( 253 − 232.3) = 20.8 kJ/kg

M18_THERMAL ENGINEERING_SE_XXXX_CH18.indd 982 6/16/2018 11:05:52 AM


Introduction to Refrigeration 983

p T
p=c (K)
3 2 T2 2
4.5
3
318
pvg = c
bar
253 1

1.5 1 T4
4 p=c 4

v s
(a) (b)

Figure 18.10  Reversed Brayton cycle: (a) p-v diagram, (b) T-s diagram

Q 2532
Mass of air circulated, m a = = = 121.73 kg/min
q 20.8
m a RT1 121.73 × 0.287 × 253
(c) V1 = = = 58.926 m3 /min
p1 2
1.5 × 10
m a RT4 121.73 × 0.287 × 232.3
(d) V4 = = = 54.1 m3 /min
p4 2
1.5 × 10
(e) Work done per minute = m a (Heat rejected – Heat extracted)
= m a c p [(T2 − T3 ) − (T1 − T4 )]
= 121.73 × 1.005 [(346.3 – 318) – (253 – 232.3)]
= 929.77 kJ/min
929.77
Net power per tonne of refrigeration = = 1.29 kW/TR.
60 × 12

Example 18.9
Dense air is used as refrigerant in Reverse-Brayton or Bell-Coleman or Joule cycle. Draw T-s and
p-v diagrams for the cycle. Derive the expression for COP in terms of pressure ratio. If tempera-
tures at the end of heat absorption and heat rejection are 0°C and 30°C respectively, the pressure
ratio is 4 and the pressure in the cooler is 4 bar determine the temperatures at all state points and
volume flow rates at inlet to compressor and at exit of turbine for 1 TR cooling capacity.

Solution
The T-s and p-v diagrams are shown in Fig. 18.11(a) and the schematic diagram in Fig. 18.11(b).
qr = q2 −3 = c p (T2 − T3 )

qa = q4 −1 = c4 −1 (T1 − T4 )

M18_THERMAL ENGINEERING_SE_XXXX_CH18.indd 983 6/16/2018 11:05:56 AM


984 Chapter 18

    Work done = qr − qa = c p (T2 − T3 ) − c p (T1 − T4 )

qa c p (T1 − T4 )
COP = =
Work done c p (T2 − T3 ) − c p (T1 − T4 )

T 
T4  1 − 1
 T4 
=
T  T 
T3  2 − 1 − T4  1 − 1
 T3   T4 
(g −1)/g
T2  p2 
=
T1  p1 

p T 2
p=c
3 2 p=c
1
3
pvg = c g
pv = c
p=c
4
4 p=c 1

v s
(a) (b)

2 1
Expander

Compressor
cooler
Air

m⋅ a

3 4

Expander

High Low
pressure pressure
(c)

Figure 18.11 Bell-Coleman cycle: (a) p-v diagram, (b) T-s diagram,


(c) Schematic diagram

M18_THERMAL ENGINEERING_SE_XXXX_CH18.indd 984 6/16/2018 11:05:58 AM


Introduction to Refrigeration 985

(g −1)/g (g −1)/g
T3  p3  p 
= = 2
T4  p4   p1 

T2 T3 T T
Hence   = or 2 = 1
T1 T4 T3 T4

T4
COP =
T3 − T4

p p
rp = 2 = 3 = 4
Let,  
p1 p4
T1 = 273 K, T3 = 303 K

303
T4 = = 203.82 K
0.4 /1.4
4
Heat extracted per minute = 1 × 211 = 211 kJ/min
Heat extracted from cold chamber per kg of air
= c p (T1 − T4 ) = 1.005 × ( 273 − 203.82) = 69.526 kJ/kg

Mass flow rate of air,


211
m a = = 303 kg/min or 182.1kg/h
69.526
Volume handled by the compressor,
m a RT1 182.1 × 287 × 273
Vc = = = 142.68 m3 /h
p1 1 × 105
Volume handled by expander,
m a RT3 182.1 × 287 × 303
Ve = = = 39.59 m3 /h
p3 5
4 × 10

Example 18.10
An air refrigeration used for food storage provides 30 TR. The temperature of air entering the com-
pressor is 10°C and temperature at exit of cooler is 30°C. Calculate:
(a) COP of the cycle, and
(b) Power required per tonne of refrigeration by the compressor.
The quantity of air circulated in the system is 3600 kg/h. The compression and expansion processes
follow the law pv1.3 = const. and g = 1.4, cp = 1.005 kJ/kg.K for air.

M18_THERMAL ENGINEERING_SE_XXXX_CH18.indd 985 6/16/2018 11:06:02 AM


986 Chapter 18

Solution
Given: Q = 30 TR, T1 = 273 + 10 = 283 K, T3 = 273 + 30 = 303 K, m a = 3600 kg/h, n = 1.3
(a) Heat extracted from the refrigerator, Q = 30 × 211 = 6330 kJ/min
3600
= m a c p (T1 − T4 ) = × 1.005 ( 283 − T4 )
60
T4 = 178 K
T3 T2 283 × 303
Now = ,T = = 481.7 K
T4 T1 2 178
T1 − T4
COP =
 n   g − 1
 n − 1  g  [(T2 − T3 ) − (T1 − T4 )]

283 − 178
= = 1.15
1.3 0.4
× [( 481.7 − 303) − ( 283 − 178)]
0.3 1.4

3600
(b) Heat absorbed, Q4 −1 = m a c p (T1 − T4 ) =
× 1.005 ( 283 − 178) = 6331.5 kJ/min
60
Heat absorbed 6331.5
Work done per minute, W = = = 5505.65 kJ/min
COP 1.15
W 5505.65
Power required per ton of refrigeration = = = 3.06 kW/TR
60 × TR 60 × 30

Example 18.11
A dense air refrigeration cycle operates between pressures of 4 bar and 20 bar. The air temperature
after heat rejection to surroundings is 40°C and air temperature at exit of refrigeration is 8°C.
The isentropic efficiencies of compressor and expander are 0.82 and 0.86 respectively. Determine
the compressor and expander work per TR. COP, and power required per TR. Take g = 1.4 and
cp = 1.005 kJ/kg.K.

Solution
Given: p1 = p4 = 4 bar, p2 = p3 = 20 bar, T3 = 273 + 40 = 313 K,
T1 = 273 + 8 = 281 K,
hc = 0.82, he = 0.86, g = 1.4, cp = 1.005 kJ/kg.K.
The p-v and T-s diagrams are shown in Fig. 18.12
Process 1-2: Isentropic compression
g −1 1.4 −1
T2  p2  g  20  1.4
= =  = 1.584
T1  p1   4
T2 = 281 × 1.584 = 445 K

M18_THERMAL ENGINEERING_SE_XXXX_CH18.indd 986 6/16/2018 11:06:06 AM


Introduction to Refrigeration 987

p T
K
2′
3 2 2′
20 T2′ 2
T2
bar 3
313 = T3
1
281 = T1
4
4 4′ 1 T4′ 4′
T4 4

v s
(a) (b)

Figure 18.12  Dense air refrigeration cycle: (a) p-v diagram, (b) T-s diagram

Process 3-4: isentropic expansion


g −1 0.4
T3  p3  g  20  1.4
= =  = 1.584
T4  p4   4

313
T4 = = 197.6 K
1.584
T2 − T1
Isentropic efficiency of compressor,  hc =
T2′ − T1
445 − 281
0.82 =
T2′ − 281
T2′ = 481 K
T3 − T4
Isentropic efficiency of expander,   he =
T3 − T4′
313 − T4′
0.86 =
313 − 197.6
T4′ = 213.7 K
Refrigerating effect per kg of air, q4′–1 = c p (T1 − T4′ ) = 1.005 ( 281 − 213.7) = 67.6 kJ/kg

Q 211
Mass flow rate of air, m a = = 3.12 kg/min
q4′ −1 67.6
Compressor work per TR, Wc = m a c p (T2′ − T1 ) = 3.12 × 7.005 ( 481 − 281) = 627.12 kJ/min

Expander work per TR, We = m a c p (T3 − T4′ ) = 3.12 × 1.005 (313 − 213.7) = 311.36 kJ/min

Net work done, Wnet = Wc − We = 627.12 − 311.36 = 315.76 kJ/min

M18_THERMAL ENGINEERING_SE_XXXX_CH18.indd 987 6/16/2018 11:06:11 AM


988 Chapter 18

Heat extracted per minute 211


COP = = = 0.668
Net work done 315.76

Wnet 315.76
Power required per TR = = = 5.26 kW
60 × TR 60 × 1

18.12  ❐  REFRIGERANTS
A refrigerant may be defined as a substance which absorbs heat through expansion or vaporization
and loses it through condensation in a refrigeration system.

18.13  ❐  CLASSIFICATION OF REFRIGERANTS


1. Primary refrigerants: They take part directly in refrigeration and cool the substance on absorp-
tion of their latent heat. Some of the primary refrigerants are NH3, CO2CH3, C12, CH2C12, SO2,
C2H5C1 and Freon group.
2. Secondary refrigerants: They are first cooled with the help of primary refrigerant and then used
for cooling other substances by absorption of their sensible heat only e.g. ice, solid CO2.
Primary refrigerants may be grouped as follows:
(a) Halo-Carbon compounds: They contain one or more of three halogens, fluorine, chlorine
and bromine. Commercially they are sold under the names as Freon, Genetron, Irontron and
Areton, They are used in domestic commercial and industrial refrigerating devices. Some of
their important examples are:
Carbon tetrachloride − R10 : CC14
Trichloromono-fluro methane − R11 : CC13F
Dichloro-difluro methane − R12 : CC12F2
Monobromotri-fluro methane − R13 : CBrF3
(b) Azeotropes: They are the mixtures of different refrigerants. They do not under go any sep-
aration with changes in temperature and pressure, e.g.
R-500 (73.8% R12 + 26.2% R-152) : CC12F2 + CH3CH F2
R-502 (48.8% R22 + 51.2% R-115) : CHC1F2 + CC1 F2 CF3
(c) Hydrocarbons: They are hydrocarbons, e.g.
Methane − R50 : CH4
Ethane − R170 : C2H6
Propane − R200 : C2H8
(d) Inorganic compounds:
Ammonia − R717 : NH3
Water − R718 : H20
Air − R720 : −
Carbon dioxide − R744 : CO2
(e) Unsaturated organic compounds: They contain ethylene or propylene as main constitu-
ents, e.g.
Ethylene − R1150 : C2H4

M18_THERMAL ENGINEERING_SE_XXXX_CH18.indd 988 6/16/2018 11:06:12 AM


Introduction to Refrigeration 989

Propylene − R1720 : C3H6


Trichloroethylene − R1120 : C2H4Cl3

18.14  ❐  DESIGNATION OF REFRIGERANTS


1. For a compound derived from a saturated hydrocarbon denoted by the chemical formula
CmHnFpC1q
In which (n + p + q) = 2m + 2, the complete designation of a refrigerant is

R(m − l)(n + 1)(p)


2. The brominated refrigerants are denoted by putting an additional B and a number to denote as to
how many chlorine atoms are replaced by bromine atoms.
3. In the case of isomers, i.e., compounds with the same chemical formula but different molecular
structure, subscrips a, b, etc. are used after the designations.
4. Unsaturated compounds, for which (n + p + q) = 2m, are distinguished by putting the digit 1
before (m − 1).
5. For inorganic refrigerants, numerical designations have been given according to then molecular
weight added to 700.
6. In the case of butane, C4H10 and higher hydrocarbons, arbitrary designation R 600 is used.

18.15  ❐  DESIRABLE PROPERTIES OF REFRIGERANTS


A good refrigerant should have the following properties:
1. Thermodynamic properties
 (a)  Low boiling point
(b)  Low freezing point
 (c)  High saturation temperature
(d)  High latent heat of vaporization
 (e)  High critical pressure and temperature.
2. Chemical properties
 (a) Non-toxic
(b) Non-flammable
 (c) Non-explosive
(d) Non-corrosive
 (e) Chemically stable
 (f) Odourless
3. Physical properties
 (a)  Low specific volume
(b)  High thermal conductivity
 (c)  High electrical insulation
(d)  Low specific heat
 (e)  Miscibility with lubricating oil.

M18_THERMAL ENGINEERING_SE_XXXX_CH18.indd 989 6/16/2018 11:06:12 AM


990 Chapter 18

4. Other properties
 (a)  Ease of leakage detection
(b)  Ease of handling
 (c) High COP
(d)  Ease of availability and of low cost.
 (e)  Low power consumption per ton of refrigeration.

18.16  ❐  APPLICATIONS OF REFRIGERANTS


The refrigerants and their applications are given in Table 18.1.
The primary-refrigerants are further classified into the following four groups
1. Halo-carbon refrigerants
2. A zoetrope refrigerants.
3. Inorganic refrigerants, and
4. Hydro-carbon refrigerants.
1. Halo-carbon Refrigerants: The commonly used halo-carbon refrigerants, their applications and
method of leak testing are given in Table 18.2.
2. Azeotrope Refrigerants: The term ‘azeotrope’ refers to a stable mixture of refrigerants whose
vapour and liquid phases retain identical compositions over a wide range of temperatures.
  Some of the azeotropic refrigerants, their applications and method of leak testing are given in
Table 18.3.

Table 18.1  Refrigerants and their applications

Refrigerant Applications
1. Inorganic:
Ammonia (NH3) Reciprocating and screw compressors in cold storages, ice plants, food, refrig-
Water (H2O) eration, etc.
Carbon dioxide (CO2) Steam-ejector system for air-conditioning, lithium bromide-water absorption
system
Dry ice (solid CO2) in transport refrigeration
2. Organic:
Freon-11 Centrifugal compressors in large capacity central air-conditioning plants
Freon-12 Reciprocating compressors in small units, especially domestic refrigerators,
water coolers, etc.
Refrigerant-22 Reciprocating compressors in large units such as package units and central
air-conditioning plants, cold storage, food freezing and storage etc.
Refrigerant-502 Large supermarket frozen food cabinets
Refrigerant-13 Low temperature refrigeration.
Refrigerant-113 Centrifugal compressors for air conditioning.
Refrigerant-114 Rotary compressors.
3. Secondary refrigerants:
Brines (CaCl2 Industrial ice plants, cold storages, skating rinks, etc. NaCl2 in water) Inhibited glycols Freez-
ing point depressants,(ethylene glycol, propylene glycol in water)

M18_THERMAL ENGINEERING_SE_XXXX_CH18.indd 990 6/16/2018 11:06:12 AM


Introduction to Refrigeration 991

Table 18.2   Commonly used halo carbon refrigerants


Refrigerant Chemical Chemical Applications Method of leak
number formula name testing
R-ll CC13F Trichloromo- Large centrifugal compres- Soap solution, halide
no-fluoromethane sors. Flushing agent for torch, electronic detector
cleaning compressor parts
R-12 CC12F2 Dichlorodifluoro Refrigerators freezers, water Soap solution, halide
methane coolers room and window torch, electronic leak
air conditioners, large cen- detector
trifugal compressor
R-13 CClF3 Monochloro- Cascade system, recipro-
tri-fluoromethane cating compressors
R-14 CF4 Carbontetra- Cascade system –
fluoride
R-21 CHC12F Dichloromo- Centrifugal compressors –
no-fluoromethane
R-22 CHC1F2 Monochloro- Household refrigerators, Soap, solution, halide
difluoromethane air-conditioners, recip- torch, electronic leak
rocating and centrifugal detector
compressors.
R-30 CH2Cl2 Methylene Theatre air- Halide torch
Chloride conditioners, auditoriums,
office buildings
R-40 CH3Cl Methyl Domestic units Soap solution, electronic
chloride leak detector
R-100 C2H6C1 Ethyl chloride Not used
R-118 CC12FCC1F2 Trichlorotri-fluo- Commercial and industrial –
or C2C13F3 roethane air-conditioning
R-114 CC1F2CC1F2 Dichlorotet- Household refrigerating –
or C2C12F4 ra-fluoroethane system, water coolers
R-115 CC1F2CF3 Monochloro-pen- – –
or C2C1F5 tafluoroethane

Table 18.3  Azeotrope refrigerants

Refrigerant Chemical Applications Leak testing method


number formula
R-500 CC12F2/ Industrial and commercial reciprocat- Soap solution, halide torch,
CH3CHF2 ing compressor systems coloured tracing agent
R-502 CHC1F2/ Frozen food lockers, frozen food pro- Soap solution, halide torch,
CC1F2CF3 cessing plants, display cases, storage electronic leak detector
units and ice cream
(Continued)

M18_THERMAL ENGINEERING_SE_XXXX_CH18.indd 991 6/16/2018 11:06:12 AM


992 Chapter 18

Table 18.3  (Continued )

Refrigerant Chemical Applications Leak testing method


number formula
R-503 CHF3/CClF3 Cascade systems Soap solution, halide torch,
electronic leak detector
R-504 CH2F2/ Industrial processes Soap solution, halide torch,
CC1F2CF3 electronic leak detector

3. Inorganic Refrigerants: The various inorganic refrigerants, their applications and leak testing
method are given in Table 18.4.
4. Hydro-Carbon Refrigerants: The various hydro-carbons are given in Table 18.5. They are
mostly used for industrial and commercial installations.

Table 18.4  Inorganic refrigerants

Refrigerant Chemical Chemical Applications Method of leak


number formula name testing
R-717 NH3 Ammonia Large and commercial reciprocating Burning sulphur can-
compression systems, absorption dle gives white fumes
systems, Cold storage, warehouse
plants, ice cream manufacture, ice
manufacture, beer manufacture,
food freezing plants
R-729 – Air Aircraft air-conditioning –
R-744 CO2 Carbon di oxide Industrial unit and aboard ships –
(dry ice)
R-764 SO2 Sulphur dioxide Ripener and preservative of foods Soap solution ammo-
nia swab white
giving smoke
R-118 H2O Water Absorption system

Table 18.5  Hydrocarbon refrigerants

Refrigerant Chemical Chemical


number formula name
R-170 C2H6 Ethane
R-290 C3H3 Propape
R-600 C4H10 Butane
R-600a C4H10 Isobutane
R-1120 C2H4CL3 Trichloroethylene
R-1130 C2H4CL2 Dichloroethylene
R-1150 C2H4 Ethylene
R-1270 C3H6 Propylene

M18_THERMAL ENGINEERING_SE_XXXX_CH18.indd 992 6/16/2018 11:06:12 AM


Introduction to Refrigeration 993

Table 18.6  Ozone Depletion Potential (ODP) of Refrigerants

Sub group Refrigerant Chemical formula Refrigerant name ODP


designation
R-ll CC13F Trichloro-fluoro methane 1.0

R-12 CC12F2 Dichlorodi-fluoro methane 1.0

CFCs R-113 CF2C1—CFC12 Trifluoro-trichloro ethane 0.8

R-114 CF2C1—CF2C1 Tetrafluoro- dichloro ethane 1.0


HCFCs R-22 CHClF2 Chlorodi-fluoro methane 0.05
Dichiororri-fluoro methane
0.02
R-123 CHCl2F3
HFC R-134a* CH2FCF3 Tetra fluoro ethane 0.0
Azeotropes R-500 R-12/R-IS2 – 0.74
R-717 NH3 Ammonia 0.0
*- R-134a is most commonly used refrigerant now-a-days.

5. Ozone Layer Depletion: Stratosphere is about 25-30 km away from the earth’s surface. It con-
tains ozone (O3). Ozone is formed in the stratosphere by the reaction that occurs when sunlight
interacts with oxygen. This ozone shelters the earth from ultraviolet (UV) radiation. This protec-
tion is essential for human health. This ozone is easily destroyed by the UV radiation. The sun’s
radiation will break down the O3 molecule into a standard O2 molecule and elemental free oxygen
(O) atom. More O3 is produced through photosynthesis and the bonding of O2 with free oxygen O.
Thus ozone is constantly being formed and destroyed in the stratosphere.
  In the stratosphere, UV radiation will break off a chlorine atom (Cl) from the CFC and HCFC
molecule. The chlorine atom destroys many O3 molecules The depleted O3 molecules in the strat-
osphere let more UV radiation reach the earth This causes an increase in skin cancers and fre-
quency of cataracts in humans; weakens human immune system and decreases plant and marine
life. Ozone depletion potential (ODP) has been used as an index to show the effect of a refrig-
erant on O3. Lower the ODP, it will be better. The ODP of some of the refrigerants is given in
Table 18.6.

18.17  ❐  ECO-FRIENDLY REFRIGERANTS


With the increasing awareness of environmental degradation, the production, use and disposal
of Chloro Fluoro Carbons (CFCs) and Hydro Chloro Fluoro Carbons (HCFCs) as refrigerants in
mechanical refrigeration systems has become a subject of great concern. However, such systems are
being developed using more eco-friendly refrigerants, such as
1. Hydrocarbons (HG-290, HC-600a).
2. Ammonia (NH3)

M18_THERMAL ENGINEERING_SE_XXXX_CH18.indd 993 6/16/2018 11:06:13 AM


994 Chapter 18

3. Carbon Dioxide (CO2)


4. Water (R-718)
5. Air.
Eco-friendly refrigerants are also known as natural refrigerants. The important features and applica-
tions of eco-friendlv refrigerants are given in Table 18.7.

Table 18.7  Important features and applications of eco-friendly refrigerants

Refrigerant Important features Applications


1. Hydrocarbons 1. Zero Global Warming Potential (GWP) 1. Domestic refrigerant (HC-270,
2. Zero Ozone Depletion Potential (OOP) HC-600a)
3. COP comparable to CFC-I2 2. Commercial refrigeration (HC-290,
4. Flammable HC-600a)
5. Compatible with mineral oils 3. Transport refrigeration (HC-290/
HC-600a or HC-290)
4. Cold storage and food processing (HC
290)
5. Industrial refrigeration (HC-290)
6. Air-conditioning (HC-290)
2. Ammonia 1. Excellent thermodynamic and thermo- 1. In small and large plants
physical properties
2. Zero ODP 2. Commercial and cold storage
3. Zero GWP 3. Super markets
4. High efficiency 4. Water chillers
5. Low cost 5. Centralized air-conditioning
6. Flammable 6. water
3. Water 1. Thermodynamically attractive 1. Water cooled chillers
2. Low volumetric refrigeration capacity 2. Eco-chiller
3. High COP (7-10)
4. Can be used both as primary and sec-
ondary refrigerant
5. Less corrosive
4.  Carbon dioxide 1. Zero OOP 1. Commercial air-conditioning system.
2. Low critical temperature 2. Mobile air-conditioning system.
3. High operating pressure
4. Non-flammable, non-toxic and cheap
5. Excellent performance
6. Low COP
5. Air 1. Simple and compact 1. Used in aircrafts.
2. Non-toxic and non-flammable 2. Used in railways and bus
3. Easy repair and maintenance air-conditioning
4. Low COP
5. Requires less space and lighter in
weight

M18_THERMAL ENGINEERING_SE_XXXX_CH18.indd 994 6/16/2018 11:06:13 AM


Introduction to Refrigeration 995

18.18 ❐ REFRIGERANT SELECTION


The choice of a refrigerant depends upon the following factors:
1. Refrigeration temperature required.
2. Refrigerating capacity of plant.
3. Thermodynamic requirements: Normal boiling point, condensing and evaporating pressure, crit-
ical temperature and pressure, freezing point, volume of suction vapour per ton, COP, power
consumption per ton etc
4. Chemical requirements: Flammability, toxicity, action with water and oil (mixibility), and action
with materials of construction
5. Physical requirements: Dielectric strength, thermal conductivity, viscosity, heat capacity, surface
tension, leak tendency and cost of refrigerant
6. Economies.
7. Equipment type and size.
8. Application.

Multiple-choice Questions

1. One tonne of refrigeration is equal to (c) reversed Carnot cycle


(a) 211 kJ/min (b) 220 kJ/min (d) Bell-Coleman cycle
(c) 420 kJ/min (d) 620 kJ/min 6. In air-conditioning of aeroplanes, using air as a
2. One tonne refrigerating machine means that refrigerant. the cycle used is
(a) one tonne is the total mass of the machine (a) reversed Carnot cycle
(b) one tonne of refrigerant is used (b) reversed -Joule cycle
(c) one tonne of water can be converted into ice (c) reversed Brayton cycle
(d) one tonne of ice when melts from and at 0°C (d) reversed Otto cycle
m 24 hours, the refrigeration effect produced 7. Co-efficient of performance of a Reversed Carnot
is equivalent to 211 kJ/min. cycle refrigerator working between higher tem-
3. The ratio of heat extracted in the refrigerator to perature T2 and lower temperature T1
the work done on the refrigerant is called (a) will increase with increase in T1 keeping T2
(a) coefficient of performance of refrigeration fixed
(b) coefficient of performance of heat pump (b) will decrease with increase in T1 keeping T2
(c) relative coefficient of performance fixed
(d) refrigerating efficiency (c) will first increase with increase in T1 and then
4. The relative coefficient of performance is equal to decrease with increase in T1 keeping T2 fixed
(d) None of the above.
Theoretical COP
(a) 8. A reversible refrigerator working between two
Actual COP
fixed temperatures
Actual COP (a) has the same COP whatever be the working
(b)
Theoretical COP substance
(b) has its COP increased for working substance
(c) Actual COP × Theoretical COP with high enthalpy of evaporation
5. Air refrigerator works on (c) has its COP increased for working substance
(a) Carnot cycle with higher specific heats
(b) Rankine cycle (d) none of the above.

M18_THERMAL ENGINEERING_SE_XXXX_CH18.indd 995 6/16/2018 11:06:14 AM


996 Chapter 18

9. Reversed Carnot cycle comprises 15. A Carnot refrigerator requires 70 kJ/min of work
(a) two isentropic processes and two adiabatie to produce one ton of refrigeration at -40°C. The
processes COP of this refrigerator is
(b) two isentropic processes and two isothermal (a) 4     (b)  3
processes (c) 5     (d)  not possible to find
(c) two isentropic processes and two constant 16. The coefficient of performance of refrigerator
pressure processes working on Reversed Carnot cycle with T1 being
(d) two isentropic processes and two constant lower temperature and T2 being higher tempera-
volume processes. ture is
10. In Reversed Carnot cycle working on perfect gas T −T T1
(a) 2 1     (b) 
(a) Isothermal work of compression is equal to T1 T2 − T1
isothermal work of expansion
(b) Isentropic work of compression is equal to T2 T −T
(c)     (d)  2 1
isentropic work of expansion T2 − T1 T2
(c) Net work of the cycle is zero 17. The coefficient of performance of Carnot refrig-
(d) Net heat transfer of the cycle is zero. erator can be expressed (r = Volume compression
11. In a Reversed Carnot-cycle working on vapour ratio for isentropic compression)
(a) Isentropic work of compression is equal to
1
isentropic work of expansion (a) 1 − rg −1 (b) g −1
(b) Isothermal heat absorption is equal to iso- 1− r
thermal heat rejection 1 1
(c) There is no work done during isothermal pro- (c) g −1
(d)
r −1 1+ rg −1
cesses
(d) There is no work done during isentropic pro- 18. The COP of Carnot Refrigerator is 3 and it pro-
cesses duces 1 ton of refrigeration. The work that will be
12. Reversed Carnot cycle assumes that all processes done is equal to
in the cycle are (a) 210 kJ/min (b) 70 kJ/min
(a) Non-flow only (c) 100 kJ/min (d) 200 kJ/min.
(b) Steady flow only 19. The ratio of high temperature to low temperature
(c) Non-flow or steady flow for Reversed Carnot refrigerator is 1.25. The COP
(d) Transient flow will be
13. Two Carnot refrigerators are employed, one for (a) 2 (b) 3 (c) 4 (d) 5
ice making and other for comfort cooling 20. A carnot refrigerator operates between 300.3K
(a) The COP of refrigerator for ice making is and 273K. The fraction of cooling effect required
higher than that for the other as work input is
(b) The COP of refrigerator for ice making is (a) 20%
lower than that for the other (b) 10%
(c) The COP of refrigerator for ice making is (c) 50%
same as that for the other (d) not possible to find with the data
(d) The COP of Carnot refrigerator will depend
on refrigerant used
21.
The relationship between COPheat pump
COPrefrigerator for the same range of temperature
14. A reverse Carnot cycle has a COP of 4. The
operation is
ratio of higher temperature to lower temperature
(a) COPheat pump = COPrefrigerator
will be
(a) 1.5 (b) 2 (c) 1.25 (d) 2.5 (b) COPheat pump − COPrefrigerator = 1

M18_THERMAL ENGINEERING_SE_XXXX_CH18.indd 996 6/20/2018 12:59:47 PM


Introduction to Refrigeration 997

(c) COPheat pump = 2COPrefrigerator the COP and temperature of heat rejections
(d) not possible to predict without knowing work- respectively are (0°C = 273K)
ing substance. (a) COP = 6.5, t = 27°C
(b) COP = 7.5. t = 27°C
22. Carnot refrigerator absorbs heat at −13°C and (c) COP = 6.5, t = 30°C
requires 1 kW for each 6.5 kW of heat absorbed, (d) COP = 7.5. t = 37°C

Review Questions
1. How is the effectiveness of a refrigeration system measured?
2. Explain the term “tonne of refrigeration”.
3. Discuss the advantages of the dense air refrigerating system over an open air refrigeration system.
4. What is the difference between a refrigerator and heat pump? Derive an expression for coefficient of perfor-
mance for both if they are running on reversed Carnot cycle.
5. Discuss the main differences between Reversed Carnot cycle operating on perfect gas and wet vapour.
6. What is refrigeration?
7. List few applications of refrigeration.
8. Define coefficient of performance.
9. What are the SI units of refrigeration?
10. Enumerate the various methods of refrigeration.
11. Name few refrigeration systems.
12. What is refrigeration effect?

Exercises

18.1 A Reversed Carnot Cycle operates on the following substances given below and the range of temperatures
is 300K and 250K.
(a) Working substance is water between the range of dry saturated liquid to dry saturated vapour at 300K.
(b) Working substance is water in the form of super heated steam with the range of pressure from 0.01 bar
to 0.05 bar.
(c) Working substance is CO2 in the super heated region with pressure range of 0.5 bar to 5 bar
(d) Working substance is air with pressure range of 1 bar to 10 bar
For the same heat rejected from the system of 60000 kJ/hr, determine the work input. Carnot coefficient
of performance as refrigerator and as heat pump and the mass flow rate.
18.2 Carnot heat engine draws heat from a reservoir at temperature of 600 K and rejects heat to another reservoir
at temperature T. This engine drives a Carnot reverse cycle refrigerator which absorbs heat from reservoir
at temperature 200 K and reject heat to reservoir at temperature T. Determine the temperature T such that
heat supplied to Carnot heat engine equal to heat absorbed by Carnot refrigerator.
18.3 A refrigerator using Carnot cycle requires 1.25 kW per tonne of refrigeration to maintain a temperature
of −30°C. Find: (a) COP of the Carnot refrigerator, (b) temperature at which heat is rejected; and (c) Heat
rejected per tonne of refrigeration. [Ans. 2.8; 55.4°C; 284. kJ/min]
18.4 A Carnot cycle machine operates between the temperature limits of 47°C and 30°C. Determine the COP
when it operates as (a) a refrigerator (b) a heat pump [Ans. 3.16; 4.16]
18.5 Ten tonnes of fish is frozen to − 30°C per day. The fish enters the freezing chamber at 30°C and freezing
occurs at −3°C. The frozen fish is cooled to − 30°C. The specific heats of fresh and frozen fish are 3.77 kJ/kg K
and 1.67 kJ/kg K respectively while latent heat of freezing is 251.2 kJ/kg K. Find the tonnage of the plant
which runs for 18 hours per day. The evaporator and condensor temperatures are −40°C and 45°C respec-
tively. It the COP of the plant is 1.8, determine the power consumption of the plant in kW.
[Ans. 18.6 TR; 36.1 kW]

M18_THERMAL ENGINEERING_SE_XXXX_CH18.indd 997 6/20/2018 12:59:47 PM


998 Chapter 18

 18.6   A Carnot refrigeration system has working temperature of −30°C and 40°C. what is the maximum COP
possible? If the actual COP is 75% of the maximum calculate the actual refrigerating effect produced per
kilowatt hour. [Ans. 3.47; 0 713 TR]
 18.7   A 5 tonne refrigerating machine operating on Bell Coleman cycle has an upper limit of pressure of 12 bar.
The pressure and temperature at the start of compressor are 1 bar and 17°C respectively. The compressed
air cooled at constant pressure to a temperature of 40°C enters the expansion cylinder. Assuming both the
expansion and compression processes to be isentropic with g = 1.4; Determine (a) COP; (b) the quantity
of air in circulation per minute; (c) piston displacement of compressor and expander (d) bore of compres-
sor and expansion cylinders. The unit runs at 250 r.p.m. and is double acting. Stroke length is 200 mm; and
(e) power required to drive the unit. Take cp = 1 kJ/kg K; cv = 0.71 kJ/kg K; R = 0.287 kJ/kg K.
[Ans. 0.952; 7.65 kg/min; 6.37 m3/min; 3.35 m3 /min; 284 mm; 18.4 kW]
 18.8   An air refrigerator used for food storage provides 50 TR. The temperature of air entering the compressor
is 7°C and the temperature before entering into the expander is 27°C. Assuming a 70% mechanical effi-
ciency find (a) actual COP and (b) The power required to run the compressor.
The quantity of air circulated in the system is 100 kg/min. The compression and expansion follow the law
pv1.3 = constant. Take g = 1.4 : cp = 1 kJ/kg K for air. [Ans. 1.13; 110.6 kW]
 18.9   A dense air refrigerating system operating between pressures of 17.5 bar and 3.5 bar is to produce 10 tonnes
of refrigeration. Air leaves the refrigerating coils at −7°C and it leaves the air cooler at 15.5°C. Neglecting
losses and clearance, calculate the net work done per minute and the coefficient of performance. For air
cp = 1.005 kJ/kg K and g = 1.4 [Ans. 1.237 kJ/min; 1.7]
18.10   A refrigerating plant is required to produce 2.5 tonnes of ice per day at −4°C from water at 20°C. If the
temperature range in the compressor is between 25°C and −6°C, calculate power required to drive the
compressor. Latent heat of ice = 335 kJ/kg and specific heat of ice = 2.1 kJ/kg K. [Ans. 1 437 kW]
18.11   A refrigerator storage is supplied with 30 tonnes of fish at a temperature of 27°C. The fish has to be cooled to
−9°C for preserving it for long period without determination. The cooling takes place in 10 hours. The specific
heat of fish is 2.93 kJ/kg K above freezing point of fish and 1.26 kJ/kg K below freezing point of fish which is
−3°C. The latent heat of freezing is 232 kJ/kg. What is the capacity of the plant in tonnes of refrigeration for
cooling the fish? What would be the ideal COP between this temperature range? If the actual COP is 40% of
the ideal, find the power required to run the cooling plant.[Ans. 78 TR; 7.33; 93.3 kW]
18.12   An air refrigeration plant working on the Carnot cycle between the temperature limits of 34°C and −1°C
has a capacity of 10 ton. Determine the power input to the plant.
18.13   An air refrigeration plant working on Carnot cycle produces refrigeration capacity equivalent to produc-
tion of 10 tons of ice in 24 hours at 0°C from water at 20°C. The compressor temperature limits are 30°C
and −15°C. Determine the power input to the plant. The latent heat of ice is 335 kJ/kg.
18.14  Find the minimum power input to perfect reversed heat engine that will produce 1 ton of ice per hour at
−5°C from water at 15°C. The temperature limits of the reversed heat engine are 15°C and −5°C. Assume
specific heat of ice as 2.1 kJ/kg. K and latent heat of ice as 335 kJ/kg.

ANSWERS TO MULTIPLE-CHOICE QUESTIONS


1. (a) 2. (d) 3. (a) 4. (b) 5. (d)
6. (c) 7. (a) 8. (a) 9. (b) 10. (b)
11. (c) 12. (c) 13. (b) 14. (c) 15. (b)
16. (b) 17. (c) 18. (c) 19. (c) 20. (b)
21. (b) 22. (a)

M18_THERMAL ENGINEERING_SE_XXXX_CH18.indd 998 6/16/2018 11:06:18 AM


Chapter 19

Vapour Compression and


Vapour Absorption Systems

19.1 ❐ INTRODUCTION
The vapour compression refrigeration system is based upon the fact that fluids absorb heat while changing
from a liquid phase to vapour phase and reject heat while changing from a vapour phase to a liquid phase.
The temperature during the phase change remains constant. However, the temperature varies with the pres-
sure and the fluid. In vapour compression system, the refrigerant condenses and evaporates at temperatures
and pressures close to the atmospheric conditions. The closed refrigeration cycle is used in which refrig-
erant does not leave the system. The refrigerant alternately condenses and evaporates. During evaporating
the refrigerant absorbs its latent heat from the brine (salt water), which is used for circulating it around
the cold chamber. While condensing, it gives out its latent heat to the circulating water of the cooler. The
vapour compression system is used in a small domestic refrigerator to big industrial refrigeration systems.

19.2 ❐ COMPARISON OF VAPOUR COMPRESSION


SYSTEM WITH AIR REFRIGERATION SYSTEM
Following are the advantages and disadvantages of the vapour compression refrigeration system over
air refrigeration system:

Advantages
1. It has smaller size for the given capacity of refrigeration.
2. It has less running cost.
3. It can be employed over a large range of temperatures.
4. The coefficient of performance is quite high.

Disadvantages
1. The initial cost is high.
2. The prevention of leakage of the refrigerant is the major problem in vapour compression system.

M19_THERMAL ENGINEERING_SE_XXXX_CH19.indd 999 6/16/2018 11:36:48 AM


1000 Chapter 19

19.3 ❐ SIMPLE VAPOUR COMPRESSION REFRIGERATION


SYSTEM
The schematic diagram of a simple vapour compression refrigeration system is shown in Fig. 19.1. It
consists of the following parts:
1. Compressor
2. Condenser
3. Throttle valve
4. Evaporators.
1. Compressor: The low pressure and temperature vapour refrigerant from the evaporator is drawn
into the compressor through the suction valve. It is compressed to a high pressure and temper-
atures in the compressor. The high pressure and temperature vapours are discharged into the
condenser through the delivery valve.
2. Condenser: The condenser or cooler consists of coils of pipes in which the high pressure and
temperature vapour refrigerant is cooled and condensed. The refrigerant, while passing through
the condenser coils gives up its latent heat to the condensing medium. The condensing medium is
normally air or water.
3. Throttle Valve: Its function is to allow the liquid refrigerant under high pressure and temperature
to pass at a controlled rate after reducing its pressure and temperature. Some of the liquid refrig-
erant evaporates. The enthalpy of the refrigerant remains unaltered in passing through the throttle
valve.
4. Evaporator: It consists of coils of pipe in which the liquid-vapour refrigerant at low pressure and
temperature is evaporated and converted into vapour refrigerant at low pressure and temperature.
During evaporation, the liquid-vapour refrigerant absorbs its latent heat of vaporisation from the
medium (air, water or brine), which is to be cooled.

Section
LPV 1 valve

Brine Evaporator 2 Discharge


solution valve
Compressor

Cooling water
leaving
LPV: Low pressure
vapour
Condenser HPV: High pressure
vapour
Cooling water LPL: Low pressure
Throttle entering liquid
3
valve HPL: High pressure
LPL + LPV HPL liquid
4

Figure 19.1  Schematic diagram of simple vapour compression refrigeration system

M19_THERMAL ENGINEERING_SE_XXXX_CH19.indd 1000 6/16/2018 11:36:49 AM



Vapour Compression and Vapour Absorption Systems 1001

19.4 ❐ VAPOUR COMPRESSION REFRIGERATION SYSTEM


A schematic diagram of a vapour compression refrigeration system is shown in Fig. 19.2. It consists
of a compressor, condenser, expansion device for throttling and an evaporator. In plants with a large
amount of refrigerant charge, a receiver and a drier is installed in the liquid line. The p-v, T-s, and p-h
diagrams for the system are shown in Fig. 19.3. The sequence of operations is as given below:
1. The process 1-2 is isentropic compression.
s2 = s1, Q = 0. Work done by the compressor per kg of refrigerant
 W1− 2 = − ∫ v dp = − ∫ dh = −( h2 − h1)

2. The process 2-3 is constant pressure condensation.


Heat rejected, Qr = h2 − h3
3. The process 3-4 is isenthalpic expansion.
h3 = h4 = h f 4 + x( h1 − h f 4 )

h3 − h f 4
or Dryness fraction, x =
h1 − h f 4
4. The process 4-1 is constant pressure evaporation. Refrigeration effect or Heat absorbed,
Qa = h1 − h4

Heat absorbed Q h −h
COP (cooling) = = a = 1 4 (19.1)
Work done W1− 2 h2 − h1
Heat rejected Q h −h
COP (heating) = = r = 2 3  (19.2)
Work done W1− 2 h2 − h1

Qr

Condenser

Expansion Wc
device 1

Compressor
Evapotator
4

Qa

Figure 19.2  Vapour compression system

M19_THERMAL ENGINEERING_SE_XXXX_CH19.indd 1001 6/16/2018 11:36:51 AM


1002 Chapter 19

p
p2 3 2′ 2

p1
4 1
v
(a)

T 2 p
2′
3
T2 2″ 3 2′ 2
p2
W
T1 0 4 1
4′ p1
4 1
Qa

s h
8 7 6 5 (h1 − h4) (h2 − h1)
(b) (c)

Figure 19.3  Vapour compression cycle: (a) p-v diagram, (b) T-s diagram,
(c) p-h diagram

(a) From Fig. 19.3(b), we have for vapours dry and saturated at the beginning of compression and
superheated at the end of compression.
Refrigerating effect, Qa = area 1–4−6−5−1
Heat rejected, Qr = area 2−2′−3−7−5−2
Work done,   W = Qr − Qa
   = area 1−2−2′−3−7−6−4−1
  Area 4−4′−7−6−4 = area 3−0−4′−3
(b) When vapours are dry and saturated at the end of compression, as shown in Fig. 19.4.
Work done = area 1−2−3−0−1 = Heat at 2 − Heat at 1 = h2 − h1
Refrigeration effect = area 1−4−6−5 = Heat at 1 − Heat at 4 = h1 − h4
   Area 3−0−4′−3 = area 4−4′−7−6−4
∴   Work done, W = area 1−2−3−7−6−4−1
Dryness fraction at suction,
sg 2 − s f 1
x1 =
sg1 − s f 1
h1 = h f 1 + x1 ( hg1 − h f 1 )
v1 = v f 1 + x1 ( v g1 − v f 1 )
Qa = h1 − h4 = h1 − h3
W = h2 − h1
Q h −h
COP = a = 1 3 (19.3)
W h2 − h1

M19_THERMAL ENGINEERING_SE_XXXX_CH19.indd 1002 6/16/2018 11:36:54 AM


Vapour Compression and Vapour Absorption Systems
 1003

T p

T2 3 2

W 3 T2
2
0 4 sg2
T1 1
4′ sf1 T1
Qa 4 1 sg1

s h
8 7 6 5 h3 h1 h2
(a) (b)

Figure 19.4  Vapour compression cycle: (a) T-s diagram, (b) p-h diagram

The COP of vapour compression cycle is lower than that of reversed Carnot cycle. Power per ton of
refrigeration,
3.5167
P = kW (19.4)
COP
TR
3.5167 3.5167
Refrigerant mass flow rate, m= = kg/s  (19.5)
Qa h1 − h4
h − h 
= m  2 1  kW 
Motor power    (19.6)
 1000 
 v 
Volume flow rate of refrigerant    = 3.5167 ×  1   (19.7)
 h1 − h4 
where v1 = specific volume of the refrigerant at the inlet of compressor

p N 
   =  D 2 L n × h v  (19.8)
4 60 
    D = dia of compressor piston or cylinder
  L = stroke length
N = rpm
n = number of cylinders in compressor
    h v = volumetric efficiency
1
 p n
 = 1+ c − c  2 
 p1 
 c = clearance ratio
Mass flow rate of cooling medium in the condenser:
me c p ∆T = m ( h2 − h3 ) (19.9)

M19_THERMAL ENGINEERING_SE_XXXX_CH19.indd 1003 6/16/2018 11:36:58 AM


1004 Chapter 19

19.5 ❐ USE OF T-s AND p-h CHARTS


The T-s (Temperature-entropy) diagram for the vapour compression cycle is shown in Fig. 19.5.

T Critical
point

Sub cooled
region
T2 2
3 p2 2′

−Compression
T3 Super heated

Thro
Condensation region
Temperature, (K)

work done on
273 K ttling
Compressor
x4 wc = 1 −2−3−5−1
T1 = T4 = T5 5 = h3 − h4 p1
10 4′ Evaporation

Sa
Net refrigerating
d
te

tu
va
effect
ra

ra
e
tu

lin

po

te
Sa

qa = 4 −1−7−6−4

d
id

ur
gu

lin
= h 1 − h4
Li

e
s
0 9 8 6 7
Specific entropy

Figure 19.5  T-s diagram for vapour compression cycle

Critical
T3 = T2′

point
T2

Sub cooled T2 Super heat


region 3 p2 2′ region
2
Condensation
Throttling
T4′ = T4 = T1
Pressure, p

n
sio

x4
es
pr
m

p1
Co

4′ 1
4 Evaporation

(h1 − h4) (h2 − h1)


T2′
= qa = wc T2
T1
0 Specific enthalpy, h

Figure 19.6  p-h diagram for vapour compression cycle

M19_THERMAL ENGINEERING_SE_XXXX_CH19.indd 1004 6/16/2018 11:36:59 AM



Vapour Compression and Vapour Absorption Systems 1005

Critical point

Critical pressure
Super heated
vapour region
Super cooled
Wet vapour
liquid region
region

e
Pressure, p

lin
Saturated

id
vapour line
iqu
dl
ate
tur
Sa

Specific enthalpy, h
Constant volume
Constant temperature
Constant entropy

Figure 19.7  Pressure-enthalpy (p-h) chart

Process 1-2: Isentropic compression. State 1 represents saturated vapour and state 2 is super
heated vapour.
Process 2-3: Condensation. State 2 represents saturated vapour and state 3 is saturated liquid.
Heat is rejected to the condensing medium.
Process 3-4: Throttling process. The enthalpy of saturated liquid at state 3 is equal to the
enthalpy at state 4 or 1.
Process 4-1: Evaporation. Heat is absorbed from the medium and refrigeration effect is produced.
The p-h (pressure-enthalpy) diagram is shown in Fig. 19.6. The various processes are also shown
in this figure. The p-h chart is shown in Fig. 19.7. It is the most convenient chart for studying the
behaviour of a refrigerant. The vertical ordinates represent pressure and horizontal ordinates represent
enthalpy. The saturated liquid line and the saturated vapour line merge into one another at the critical
point. The space to the left of the saturated liquid line is the sub-cooled liquid region. The space
between the liquid and vapour lines is called the wet region. The space to the right of the saturated
vapour line is the superheated vapour region.

Example 19.1
An R-12 vapour compression system is operating at a condenser temperature of 40°C and an evap-
orator temperature of −5°C. Calculate the Carnot COP and actual COP of the cycle.
Solution
From p-h diagram of R-12, we have
h1 = 185.4 kJ/kg, h2 = 208.0 kJ/kg, h3 = h4 = 74.6 kJ/kg

M19_THERMAL ENGINEERING_SE_XXXX_CH19.indd 1005 6/16/2018 11:37:00 AM


1006 Chapter 19

T1 273 − 5 268
Carnot COP = = = =6
T2 − T1 40 − ( −5) 45
h1 − h4 185.4 − 74.6 110.8
Actual COP = = = = 4.9
h2 − h1 208.0 − 185.4 22.6

Example 19.2
An ammonia refrigerating machine has working temperature of 35°C in the condenser and −15°C
in the evaporator. Calculate the COP for (a) dry Compression, and (b) wet compression. The data
for ammonia is given below:

Satura- vf vg hf hg sf sg Superheated vapour


tion 3
l/kg m /kg kJ/ kJ/ kJ/ kJ/ by 50°C by =100°C
Temp. C° kg kg kg.K kg.K h s h s
kJ/kg kJ/kg.K kJ/kg kJ/kg.K
−15 1.52 0.509 112.3 1426 0.457 5.549 1543 5.963 1656 6.301
35 1.7 0.096 347.5 1471 1.282 4.930 1616 5.368 1744 5.702

Solution

(a) By interpolation for entropy,


∆t 5.549 − 5.368 0.181
= =
50 5.702 − 5.368 0.334
∆t = 27.1°C
Degree of superheat at discharge = 50 + 27.1 = 77.1°C
Discharge temperature = 35 + 77.1 − 112.1°C
27.1
h2 = 1616 + (1744 − 1616) = 1685.4 kJ/kg
50
Refrigerating effect, Qa = h1 − h4 = 1426 − 347.5 = 1078.5 kJ/kg
Work done, W = h2 − h1 = 1685.4 − 1426 = 259.4 kJ/kg
Qa 1078.5
COP = = = 4.16
W 259.4
4.930 − 0.457
(b) x1 = = 0.88
5.549 − 0.457
h1 = 112.3 + 0.88 (1426 − 112.3) = 1268.4 kJ/kg
v1 = v f 1 + x1 ( v g1 − v f 1 )

= 0.00152 + 0.88(0.509 − 0.00152) = 0.448 m3 /kg


Qa = h1 − h4 = 1268.4 − 347.5 = 920.9 kJ/kg

M19_THERMAL ENGINEERING_SE_XXXX_CH19.indd 1006 6/16/2018 11:37:05 AM



Vapour Compression and Vapour Absorption Systems 1007

W = h2 − h1 = 1471 − 1268.4 = 202.6 kJ/kg


920.9
COP = = 4.55
202.6
Example 19.3
A refrigerating plant using CO2 as refrigerant works between 25°C and −5°C. The dryness fraction
of CO2 is 0.6 at the entry of compressor. Find the ice formed per day if the relative efficiency is
50%. Ice is formed at 0°C from water at 10° C. The quantity of CO2 circulated is 6 kg/min. For
water cp = 4.187 kJ/kg, L = 335 kJ/kg.
Properties of CO2

Temperature hf hfg sf
°C kJ/kg kJ/kg kJ/kg.K
25 81.25 121.5 0.2513
−5 −7.53 254.8 −0.04187

Solution
The T-s cycle is shown in Fig. 19.8.
T1 = 273 − 5 = 268 K,
T2 = 273 + 25 = 298 K
h1 = h f 1 + x1h fg1 = −7.53 + 0.6 × 245.8
= 140 kJ/kg
s1 = s2
x1h fg1 x2 h fg 2
sf1 + = sf 2 +
T1 T2
0.6 × 245.8 x × 121.5
− 0.04187 + = 0.2513 + 2
268 298
x2 = 0.63
    
h2 = h f 2 + x2 h fg 2 = 81.25 + 0.63 × 121.5 = 157 kJ/kg
Work done, w1− 2 = h2 − h1 = 157 − 140 = 17 kJ/kg

3 2
298 K

268 K
4 1

Figure 19.8  T-s diagram

M19_THERMAL ENGINEERING_SE_XXXX_CH19.indd 1007 6/16/2018 11:37:08 AM


1008 Chapter 19

Refrigerating effect,   qa = h1 − h4 = h1 − h3 = h1 − h f 3
qa 58.75
COP =
  = = 3.45
w1− 2 17
Actual   COP = 3.45 × 0.5 = 1.725
17 × 6
Work done per second = = 1.7 kJ/s
60
Actual cooling effect = 1.725 × 1.7 = 2.94 kJ/s
Heat carried to form 1 kg of ice
 = c p (t2 − t1 ) + L = 4.187 × 10 + 335 = 376.87 kJ
2.94 × 3600
Ice formed per hour = = 28 kg
376.87
Example 19.4
A refrigerator operating on a standard vapour compression cycle has a coefficient of performance
of 6.5 and is driven by a 50 kW compressor. The enthalpies of saturated liquid and saturated
vapour refrigerant at the operating condensing temperature of 35°C are 69.55 kJ/kg and 201.45 kJ/
kg respectively. The saturated refrigerant vapour leaving the evaporator has an enthalpy of 187.53
kJ/kg. Find the refrigerant temperature at the compressor discharge. The cp of refrigerant vapour
may be taken to be 0.6155 kJ/kg.K.
Solution
COP = 6.5, Compressor power = 50 kW
Refrigerating capacity = 50 × 6.5 = 325 kW
Heat extracted per kg of refrigerant = 187.53 − 69.55 = 117.98 kJ/kg
325
Refrigerant rate, m = = 2.7547 kg/s
117.98
50
Heat input = = 18.15 kJ/kg
2.7547
Enthalpy of vapour after compression = 187.53 + 18.15 = 205.68 kJ/kg
Superheat = 205.68 − 201.45 = 4.23 kJ/kg
4.23
Degree of superheat = = 6.87°C
0.6155
Temperature of refrigerant at compressor discharge = 35° + 6.87° = 41.87°C
Example 19.5
A refrigeration system of 15 tons capacity operates on standard simple vapour compression cycle
using Refrigerant -22 to an evaporator temperature of 5°C and condensing temperature 50°C. Draw
the p-h diagram for the cycle. Calculate (a) the refrigerant mass flow rate, and (b) the compressor
intake volume flow rate if the compressor volumetric efficiency is 0.72. Use the refrigerant prop-
erty data given in Table below.

Temperature Pressure Enthalpy, kJ/kg Specific volume


°C bar Sat. liquid Sat. vapour Sat. liquid l/kg Sat. vapour m3/kg
5 5.836 205.9 407.1 0.791 0.0404
50 19.423 263.3 417.7 0.922 0.0117

M19_THERMAL ENGINEERING_SE_XXXX_CH19.indd 1008 6/16/2018 11:37:11 AM


Vapour Compression and Vapour Absorption Systems
 1009

Condenser

p
3
2
p1 3 50°C
Expansion valve 2

4
p2 5°C
4 1
Evapoator 1
Compressor h
(a) (b)

Figure 19.9  (a) Simple vapour compression system, (b) p-h diagram
Solution
Refer to Fig. 19.9.
m ( h1 − h4 )
(a) Capacity of plant =
3.5167
h1 = 407.1 kJ/kg,
h4 = h3 = 263.3 kJ/kg
15 × 3.5167
m = = 0.367 kg/s
( 407.1 − 263.3)
 g
mv 0.367 × 0.0117
(b) Volume flow rate = = = 5.96 × 10 −3 m3 /s
hc 0.72

Example 19.6
A refrigeration cycle uses Freon −12 as the working fluid. The temperature of the refrigerant in
the evaporator is −10°C. The condensing temperature is 40°C. The cooling load is 150 W and the
volumetric efficiency of the compressor is 80%. The speed of compressor is 720 rpm. Calculate the
mass flow rate of the refrigerant and the displacement volume of the compressor.
Properties of Freon − 12
Tempera- Saturation Enthalpy, kJ/kg Specific volume m3/kg
ture pressure MPa Saturated liquid Saturated vapour Saturated vapour
°C
−10 0.22 26.8 183.0 0.08
40 0.96 74.5 203.1 0.02

Solution
Cooling load        Q = 150 W
Refrigeration effect = h1 − h4 (Fig.19.10)
= 183 − 74.5
= 108.5 kJ/kg
m( h1 − h4 ) = Q


M19_THERMAL ENGINEERING_SE_XXXX_CH19.indd 1009 6/16/2018 11:37:13 AM


1010 Chapter 19

3 40°C
2

−10°C
4 1

Figure 19.10  p-h diagram

Mass flow rate of refrigerant,


150
m = = 0.001382 kg/s
108.5 × 103
Displacement volume of compressor,
 1 0.001382 × 0.08
mv
V1 = = = 1.382 × 10 −4 m3 /s
hv 0.8

Example 19.7
In a simple vapour compression cycle, following are the properties of the refrigerant R-12 at var-
ious points.
Compressor inlet: h1 = 183.2 kJ/kg, v1 = 0.0767 m3/kg
Compressor discharge: h2 = 222.6 kJ/kg, v2 = 0.0164 m3/kg
Condenser exit: h3 = 84.9 kJ/kg, v3 = 0.0083 m3/kg
The piston displacement volume for the compressor is 1.5 litres per stroke and its volumetric
efficiency is 80%. The speed of the compressor is 1600 rpm.
Find (a) the power rating of the compressor (kW), (b) the refrigerating effect (kW).
Solution
Piston displacement volume (Fig. 19.11),
p
V1 = d 2 l = 1.5 × 10 −3 m3 /stroke
4

3
2

4 1

Figure 19.11  p-h diagram

M19_THERMAL ENGINEERING_SE_XXXX_CH19.indd 1010 6/16/2018 11:37:15 AM



Vapour Compression and Vapour Absorption Systems 1011

Compressor discharge,
V1N 1.5 × 10 −3 × 1600
q= =
hv 0.8
= 3 m3 /min
Mass flow rate of compressor,
q 3
m = =
v1 0.0767
= 39.12 kg/min
Power of compressor   = m ( h2 − h1)
39.12
 60
= ( 222.6 − 183.2)
= 25.68 kW
Refrigerating effect = m ( h1 − h4 ) = 39.12(183.2 − 84.9) = 3845.5 kJ/min or 64.1kW

Example 19.8
In a standard vapour compression refrigeration cycle operating between an evaporator temperature
of −10°C and condenser temperature of 40°C, the enthalpy of the refrigerant, Freon-12, at the end
of compression is 220 kJ/kg. Show the cycle diagram on T-s plane.
Calculate (a) the COP of the cycle, (b) the refrigerating capacity and the compressor power
assuming a refrigerant flow rate of 1 kg/min. You may use the extract of the Freon-12 property
table given below:

t(°C) p(MPa) hf  (kJ/kg) hg(kJ/kg)


-10 0.2191 26.85 183.1
40 0.9607 74.53 203.1

Solution
The T-s diagram is shown in Fig. 19.12
h1 − h4
COP =
h2 − h1
183.1 − 26.85
=
    = 4.23
220 − 183.1
Refrigerating capacity
= m (h1 − h4)
= 1(183.1 − 26.85)
= 156.25 kJ/min or 2.6 kW
Compressor power = m (h2 − h1)
= 1 (220 − 183.1)
= 36.9 kJ/min or 0.615 kW

M19_THERMAL ENGINEERING_SE_XXXX_CH19.indd 1011 6/16/2018 11:37:20 AM


1012 Chapter 19

T
2
3 40°C 2′

−10°C
4 1

Figure 19.12  T-s diagram

Example 19.9
An R-717 (ammonia) system operates on the basic vapour compression refrigeration cycle. The
evaporator and condenser pressures are 0.119 MPa and 1.389 MPa respectively. The mass flow rate
of refrigerant is 0.1 kg/s. The volumetric efficiency of the compressor is 84%. Determine the com-
pressor displacement rate. If the COP of the cycle is 2, determine the power input to the compressor.
Saturation Properties of R-717 (ammonia)

Temperature °C Pressure MPa Specific volume Enthalpy. kJ/kg


MPa of vapour m3/kg
Liquid Vapour
−30 0.119 0.9638 63.9 1423.6
36 1.389 0.0930 371.4 1488.6

Solution
Compressor displacement (Fig. 19.13)

mv 0.1 × 0.9638
= = = 0.1174 m3 / s
hv 0.84
Refrigerating effect = h1 − h4
    = 1423.6 − 371.4
    = 1052.2 kJ/kg
Refrigerating effect
         COP =
Work done
T

3 2
36°C

−30°C
4 1

Figure 19.13  T-s diagram

M19_THERMAL ENGINEERING_SE_XXXX_CH19.indd 1012 6/16/2018 11:37:21 AM


Vapour Compression and Vapour Absorption Systems
 1013

1052.2
   Work done = = 526.1 kJ/kg
2
Power input to compressor = 526.1 × 0.1 = 52.61 kW

Example 19.10
In a 5 kW cooling capacity refrigeration system operating on a simple vapour compression cycle,
the refrigerant enters the evaporator with an enthalpy of 75 kJ/kg and leaves with an enthalpy of
183 kJ/kg. The enthalpy of the refrigerant after compression is 210 kJ/kg. Show the cycle on T-s
and p-h diagrams. Calculate the following: (a) COP, (b) power input to compressor, and (c) rate of
heat transfer at the condenser.

Solution
Refer to Fig. 19.14.
h1 − h4 183 − 75 108
COP = = = =4
h2 − h1 210 − 183 27
Mass flow rate of refrigerant,
Cooling capacity
m =
Cooling effect
5
=
h1 − h4
5
= = 0.046 kg/s
108
Power input to compressor,
= m (h2 − h1)= 0.046 (210 − 183) = 1.24 kW
Rate of heat transfer to condenser,
Qr = m (h2 − h4)
h4 = h3
Qr = 0.046 (210 − 75) = 6.21 kW
T p
2
3 2′
3 2
2′

1
4 4 1

s h
(a) (b)

Figure 19.14  (a) T-s diagram, (b) p-h diagram

Example 19.11
An ice making plant using refrigerant R-12 is having an evaporator saturation temperature of
−25°C and the condenser saturation temperature of 35°C. The vapour is leaving the compressor at
65°C. Following table shows the properties of the refrigerant:

M19_THERMAL ENGINEERING_SE_XXXX_CH19.indd 1013 6/16/2018 11:37:24 AM


1014 Chapter 19

Temperature Pressure Saturation enthalpy, kJ/kg


°C kPa Liquid Vapour
−25 123.7 13.3 176.5
35 850.0 69.6 201.5

Enthalpy of superheated refrigerant at 850 kPa and 65°C = 225.5 kJ/kg


(a) Calculate the coefficient of performance (COP) of this system.
(b) If the capacity of the plant is 5 kW, calculate mass flow rate of refrigerant and power con-
sumption.

Solution
Refer to Fig. 19.15.
h1 − h4 176.5 − 69.6 106.9
(a) COP = = = = 2.18
h2 − h1 225.5 − 176.5 49.0
(b) m (h1 − h1) = 5
5
  m = = 0.0467 kg/s
106.9
Power consumption = m (h2 − h1)
   = 0.0467 × 49 = 2.288 kW

3
2

4 1

Figure 19.15  p-h diagram

19.6 ❐ EFFECT OF SUCTION PRESSURE


The suction or evaporator pressure decreases due to the frictional resistance to the flow of refrigerant.
Let the suction pressure ps decrease to ps′ as shown in p-h diagram of Fig. 19.16. The effect of decrease
in suction pressure are:
1. The refrigerating effect decreases from ( h1 − h4 ) to ( h1′ − h4′ )
2. The compression work increases from ( h2 − h1 ) to ( h2′ − h1′ )
refrigerating effect
Now COP =
compression work
Therefore, the COP decreases for the same amount of refrigerant flow. Hence, the refrigerating capac-
ity of the system decreases and the refrigeration cost increases.

M19_THERMAL ENGINEERING_SE_XXXX_CH19.indd 1014 6/16/2018 11:37:26 AM



Vapour Compression and Vapour Absorption Systems 1015

pd 3 2
2′

4 1
ps
ps′
4′ 1′

h
hf 3 = h4 = h4′ h1′ h1 h2 h2′

Figure 19.16  p-h diagram

19.7 ❐ EFFECT OF DISCHARGE PRESSURE


The discharge (or condenser) pressure pd increases to pd’ due to frictional resistance to flow or refrig-
erant. The effects of increase in discharge pressure are (Fig. 19.17):
1. The refrigerating effect decreases from ( h1 − h4 ) to ( h1 − h4′ ).
2. The compression work increases from ( h2 − h1 ) to ( h2′ − h1 ).

p′d 3′
2′
pd 3
2

ps
4 1
4′

h
hf3 = h4 hf 3′ = h4′ h1 h2 h2′

Figure 19.17  p-h diagram

M19_THERMAL ENGINEERING_SE_XXXX_CH19.indd 1015 6/16/2018 11:37:28 AM


1016 Chapter 19

Thus the COP decreases. The effect of increase in discharge pressure is similar to that of decrease in
suction pressure.

19.8 ❐ EFFECT OF SUPERHEATING OF REFRIGERANT


VAPOUR
The vapour compression refrigeration cycle with superheated vapour before compression on T-s and
p-h diagrams are shown in Fig. 19.18. The evaporation starts at state 4 and continues upto state 1′,
when it is dry and saturated. The vapours are now superheated from state 1′ to state 1, where they enter
the compressor. Its effect is to increase the COP.

h1 − h f 3 h1 − h4
COP = =
h2 − h1 h2 − h1

2
Condenser
3
2′

Compressor

Throttling

1
Evaporator
4 Superheating
1′

s
(a)

3 2′ 2

4 1
1′

(b)

Figure 19.18  Superheated vapour compression cycle: (a) T-s diagram, (b) p-h diagram

M19_THERMAL ENGINEERING_SE_XXXX_CH19.indd 1016 6/16/2018 11:37:29 AM


Vapour Compression and Vapour Absorption Systems
 1017

19.8.1 Superheat Horn


Consider the T-s diagram for a vapour compression cycle shown in Fig 19.19. The vapours of refrig-
erant are wet at state 1′ and dry saturated at state 2′. The process 1′−2′ represents wet compression.
The vapours at state 1 are dry saturated and at state 2 superheated. The process 1−2 represents dry
compression. The increased work of the cycle due to the replacement of wet compression by dry com-
pression appears as the area 2−2′−2′′, generally known as superheat horn.

T
2
p2

2′
T2 2′′ p1

T1 1
1′

Figure 19.19  Wet and dry compression processes

19.9 ❐ EFFECT OF SUBCOOLING (OR UNDERCOOLING)


OF REFRIGERANT VAPOUR
Consider the T-s and p-h diagrams for the vapour compression cycle shown in Fig. 19.20 in which
the refrigerant after condensation process 2′−3′, is cooled below the saturation temperature T3′ before
throttling process to temperature T3. Such a process is called undercooling or subcooling of the refrig-
erant. It is generally done along the saturated liquid line. The effect of undercooling is to increase the
COP.
Refrigerating effect, RE = h1 - h4 = h1 - hf3, where hf3 = hf3′-cpf(T3′ - T3)
Work done,     w = h2 - h1
R h1 − h f 3
COP = E =
w h2 − h1

M19_THERMAL ENGINEERING_SE_XXXX_CH19.indd 1017 6/16/2018 11:37:30 AM


1018 Chapter 19

T p
Subcooling
Saturated
liquid line
2
3′ 2′
3 2
Condensation
3′
2′

3
Subcooling
Compression

Exp.
1
4 4′
4 4′ Evaporation 1

s h
(a) (b)

Figure 19.20 
Vapour compression cycle with subcooling of refrigerant: (a) T-s diagram,
(b) p-h diagram

Example 19.12
A food storage locker requires a refrigeration system of 42 kW capacity at an evaporator tempera-
ture of −5°C and a condenser temperature of 40°C. The refrigerant, R-12, is sub-cooled 5°C before
entering the expansion valve, and the vapour is superheated 6°C before leaving the evaporator coil.
The compression of the refrigerant is reversible adiabatic. A two-cylinder vertical single acting
compressor with stroke equal to 1.5 times the bore is to be used operating at 960 rpm. Determine
(a) the refrigerating effect/kg, (b) the mass of refrigerant to be circulated per minute, (c) the theoret-
ical piston displacement per minute, (d) the theoretical power, (e) the co-efficient of performance,
(f) the heat removed through condenser/kg and (g) the theoretical bore and stroke of compressor.
Data for R-12 refrigerant is given below:

Satura- Absolute Specific Enthalpy Enthalpy Enthalpy Enthalpy Specific Specific


tion pressure volume of of liquid of vapour of liquid of vapour heat heat
temper- bar varpour kJ/kg kJ/kg kJ/kg.K kJ/kg.K liquid vapour
ature m3/kg kg/kg.K kJ/kg.K
°C
∼5°C 2.6032 0.06559 195.37 350.44 0.97805 1.5635 0.936 0.6455
+40°C 9.5909 0.01837 239.03 368.81 1.1315 1.5459 1.030 0.762

Solution
Given: Q = 42 kW, T1 = −5°C or 273−5= 268 K, T2′ = 40°C or 273 + 40 = 318K,
T3′ −T3 = 5°C, T1−T1′ = 6°C, n = 2, L = 1.5D, N = 960 rpm.
The T-s diagram is shown in Fig. 19.21.

M19_THERMAL ENGINEERING_SE_XXXX_CH19.indd 1018 6/16/2018 11:37:31 AM


Vapour Compression and Vapour Absorption Systems
 1019

326.38 2

Subcooling in
condenser Superheat
3′ 2′ remove in
313 condenser
Condensation
308

Compression
Isentropic
3
Throttling
Temperature, K

1
374
4′ 1′
268 Superheat
4 Evaporation
Evapoator

s4′ = 0.97805 s2′ = 1.5459 s1 = s2 = 1.5778


s3′ = 1.1315 s1′ = 1.56135
Specific Entropy, kJ/kg⋅K

Figure 19.21  T-s diagram


Process l−1: superheating
h1 = hg1′ + c pg1′ (T1 − T1′ ) = 350.44 + 0.6455 × 6
= 354.313 kJ/kg
T1 274
s1 = sg1′ + c pg1′ ln
= 1.5635 + 0.6455 ln
T1′ 268
= 1.5778 kJ/kg.K
Process 1−2: Isentropic compression
T
s1 = s2 = sg 2′ + c pg 2′ ln 2
T2′
T
1.5778 = 1.5459 + 0.6455 In 2
T2 = 326.38 K 313
h2 = hg 2′ + c pg 2′ (T2 − T2′ )
= 368.81 + 0.6455 (326.38 − 313)
= 379.00 kJ/kg
Process 3−3: Subcooling
h3 = h f 3′ − c pf 3′ (T3′ − T3 ) = 239.03 − 1.030 × 5 = 233.88 kJ/kg
Process 3−4: Throttling
h4 = h3 = 233.88
Specific volume at suction to compressor
v1 v1′
=
T1 T1′
274
v1 = × 0.06559 = 0.06706 m3 /kg
268

M19_THERMAL ENGINEERING_SE_XXXX_CH19.indd 1019 6/16/2018 11:37:35 AM


1020 Chapter 19

(a) Refrigeration effect/kg, q  4−1 = h1 − h4 = 354.313 −233.88 =120.433kJ/kg


Q × 60 42 × 60
(b) Mass of refrigerant required/min, m r = = = 20.924 kg/min
q4 −1 120.433
(c) Theoretical suction volume/min = m × v1 = 20.924 × 0.06706 = 1.403 m3/min
( h2 − h1 ) 20.924
(d) Power, P = m r × = (379.00 − 354.313) = 8.61kW
60 60
Q 42
(e) COP = = = 4.88
p 8.61
(f) Heat removed through the condenser/kg = h2 − h3 = 379.00 − 233.88 = 145.12 kJ/kg
1.403
(g) Theoretical suction volume per cylinder per minute = = 0.7015 m3 /min
2
p 2
D × 1.5 D × N = 0.7015
4
4 × 0.7015
D3 = = 0.0006198
p × 1.5 × 960
D = 0.085 m or 85 mm.
L = 1.5 D = 127.5 mm

19.10 ❐   VAPOUR ABSORPTION SYSTEM


The vapour absorption system uses heat energy instead of mechanical energy in order to change
the conditions of the refrigerant required for the operation of the refrigeration cycle. In the vapour

Qrc
High pressure
NH3 vapour
Qg Condenser

Liquid
ammonia
Generator Weak solution
Pump
Receiver
Pressure
reducing valve
Qp

Absorber Evaporator
Strong
solution Expansion
Qra Qe valve

Low pressure
NH3 vapour

Figure 19.22  Vapour absorption refrigeration system

M19_THERMAL ENGINEERING_SE_XXXX_CH19.indd 1020 6/16/2018 11:37:39 AM



Vapour Compression and Vapour Absorption Systems 1021

absorption system, the compressor is replaced by an absorber, a pump, a generator and a pressure
reducing valve. The vapour refrigerant from the evaporator is drawn into the absorber where it is
absorbed by the weak solution of the refrigerant forming a strong solution. This strong solution is
pumped to the generator where it is heated by some external source like steam or heating oil. During
the heating process, the vapour refrigerant is driven off by the solution and enters into the condenser
where it is liquefied. The liquid refrigerant then flows into the evaporator, thereby completing the
cycle. A line diagram of vapour absorption refrigeration system is shown in Fig. 19.22.

19.11  ❐  W
 ORKING PRINCIPLE OF VAPOUR ABSORPTION
REFRIGERATION SYSTEM
The schematic diagram of vapour absorption system is shown in Fig. 19.23. It consists of
1. an absorber,
2. a pump,
3. a generator and
4. a pressure reducing valve
The function of the pressure reducing valve is to replace the compressor of vapour compression
system. The other components of the system are
5. condenser,
6. receiver,
7. expansion valve and
8. evaporator as in the vapour compression system.

Cooling water
Heat
added High pressure ammonia vapour
Condenser
High pressure
Generator liquid ammonia

Weak solution Heat rejected


Receiver

Steam

Pressure
reducing valve Heat absorbed
Pump

Low pressure
Absorber Evaporator
Strong ammonia vapour
solution Expansion
valve
Cooling Heat
water rejected

Figure 19.23  Schematic diagram for vapour absorption system

M19_THERMAL ENGINEERING_SE_XXXX_CH19.indd 1021 6/16/2018 11:37:40 AM


1022 Chapter 19

19.11.1 Working
In the vapour absorption system, the low pressure ammonia vapour leaving the evaporator, enters
the absorber where it is absorbed by the cold water in the absorber. The solution of ammonia vapour
in water is called aqua-ammonia. The absorption of ammonia vapour in water lowers the pressure
in the absorber which in turn draws more ammonia vapour from the evaporator. As a result of this,
the temperature of solution rises. Some cooling arrangement (usually water cooling) is employed
in the absorber to remove the heat of solution evolved there. This increases the absorption capacity
of water. The strong solution thus formed in the absorber is pumped to the generator by the liquid
pump.
The strong ammonia solution in the generator is heated by some external source such as hot gas,
steam or heating oil. During the heating process, the ammonia vapour is driven off the solution at high
pressure leaving behind the hot weak solution of ammonia in the generator. This weak ammonia solu-
tion flows back to the absorber at low pressure after passing through the pressure reducing valve. The
high pressure ammonia vapour from the generator is condensed in the condenser to a high pressure
liquid ammonia. This liquid ammonia is passed to the expansion valve through the receiver and then
to the evaporator. This completes the vapour absorption cycle.

19.12  ❐  A
 DVANTAGES OF VAPOUR ABSORPTION SYSTEM
OVER VAPOUR COMPRESSION SYSTEM
The advantages of vapour absorption system over vapour compression system are listed in Table 19.1.

Table 19.1  Comparison of vapour absorption and vapour compression systems

Feature Vapour absorption system Vapour compression system


1.  System operation. Pump is the only moving part. Many moving parts. More wear,
Quiet operation and little wear. tear and noise.
2. Type of energy used to change Heat energy. Mechanical energy.
the condition of refrigerant
from the evaporator
3.  Source of energy Steam at high or low pressure. Electric power, which is quite
Exhaust steam from furnaces and expensive.
solar energy may be used. It is
quite cheap.
4. Evaporative pressure and Can operate on reduced evapora- Capacity decreases rapidly with
temperature. tor pressure and temperature by lowered evaporator pressure.
increasing the steam pressure to
generator, with little decrease in
capacity.
5. Effect of variations in load. No effect on performance as The performance is poor at
the load variations are met by partial loads.
controlling the quantity of aqua
circulated and quantity of steam
supplied to generator.
(Continued)

M19_THERMAL ENGINEERING_SE_XXXX_CH19.indd 1022 6/16/2018 11:37:40 AM


Vapour Compression and Vapour Absorption Systems
 1023

Table 19.1  (Continued )

Feature Vapour absorption system Vapour compression system


6. Effect of liquid refrigerant It has no bad effect on the system It is essential to superheat, the
leaving in evaporator. except that of reducing the refrig- vapour of refrigerant leaving the
erating effect. evaporator so that no liquid may
enter the compressor.
7. Space and automatic control More favourable Less favourable.
requirements.

19.13  ❐  C
 OEFFICIENT OF PERFORMANCE OF AN IDEAL
VAPOUR ABSORPTION SYSTEM
The ideal vapour absorption refrigeration system is shown in Fig. 19.22
Let Qg = heat given to the refrigerant in the generator.
Qrc = heat rejected from the condenses to the atmosphere.
Qra = heat rejected from the absorber to the atmosphere.
Qe = heat absorbed by the refrigerant in the evaporator.
Qp = heat added to the refrigerant due to pump work.
Qr = Qrc + Qra
= total heat rejected from the condenser and absorber to atmosphere
For heat balance, neglecting Qp according to the first law of thermodynamics, we have
Qr = Qrc + Qra = Qg + Qe
Let Tg = temperature at which heat is given to the generator.
Tr = temperature at which heat is rejected to atmosphere or cooling water from the condenser
and absorber.
Te = temperature at which heat is absorbed in the evaporator.
The vapour absorption system can be considered as a perfectly reversible system. Therefore, the
initial entropy of the system must be equal to the entropy of the system after the change in its
conditions.
Qg Qe Qrc + Qra Qg + Qe
∴ + = =
Tg Te Tr Tr

 1 1  1 1
or Qg  −  = Qe  − 
 Tg Tr   Tr Te 

 Tr − Tg   Te − Tr 
Qg   = Qe 
 Tg Tr   Tr Te 

(Te − Tr )Tg  T − T   Tg 
   Qg = Qe = Qe  e r   
(Tr − Tg )Te  Te   Tr − Tg 

M19_THERMAL ENGINEERING_SE_XXXX_CH19.indd 1023 6/16/2018 11:37:42 AM


1024 Chapter 19

Maximum coefficient of performance,


Heat absorbed in evaporator
 (COP)max =
Heat supplied to generator + Work done by pump
Neglecting pump work,
Qe  Te   Tr − Tg 
 (COP)max = =   (19.10a)
Qg  Te − Tr   Tg 

T   T 
=  r − 1 1 − r  (19.10b)
 Tg   Te 
If Ta = temperature at which heat is rejected from the absorber
Tc = temperature at which heat is rejected from the condenser.
 Te   Ta − Tg   Te   Tg − Ta 
Then (COP) max =   =   (19.11)
 Te − Tc   Tg   Tc − Tc   Tg 

Example 19.13
A geothermal well at 130°C supplies heat at a rate of 100,500 kJ/h to an absorption refrigeration
system. The environment is at 30°C and the refrigerated space is maintained at −22°C. Determine
the maximum possible heat removal from the refrigerated space.

Solution
Given: Tr = 273 + 35 = 308 K, Tg = 273 + 110 = 383 K,
Te = 273 − 5 = 268 K
 Tr 
 − 1  308 − 1
 Tg   383 
(COP) max =  = =1.315
 Tr   308 
1 −  1 − 268 
 Te 

Example 19.14
In an absorption type refrigerator, the heat is supplied to ammonia generator by condensing steam
at 2.5 bar and dryness fraction 0.9. The temperature in the refrigerator is to be maintained at −5°C.
Calculate the maximum COP possible.
If the refrigeration load is 20 tonnes and actual COP is 75% of the maximum COP, find the mass
of steam required per hour. Take ambient temperature as 30°C.
Solution
Given: p = 2.5 bar, x = 0.9, Te = 273 − 5 = 268 K, Q = 20TR, (COP)max = 0.75(COP)max,
Tr = 273 + 30 = 303 K, From steam tables hf = 535.34 kJ/kg. hg = 2716.9 kJ/kg.

Saturation temperature of steam at p = 2.5 bar, Ts = 273 + 127.43 = 400.43 K = Tg


T   Tr 
(COP) max =  r − 1 1 − 
 Tg   Te 

M19_THERMAL ENGINEERING_SE_XXXX_CH19.indd 1024 6/16/2018 11:37:44 AM


Vapour Compression and Vapour Absorption Systems
 1025

 303   303 
=  − 1 1 − = 1.863
 400. 43   268 
(COP)act = 0.75 × 1.863 = 1.397
211Q 211 × 20
Actual heat supplied = = = 3020.76 kJ/min
(COP)act 1.397
Latent heat of steam, hfg = hg − hf = 2716.9 − 535.35 = 2181.56 kJ/kg

Actual heat supplied


Mass of steam required per hour, ms =
x × h fg

3020.76 × 60
= = 92.3 kg/h
0.9 × 2181.56

19.14  ❐  A
 MMONIA-WATER (OR PRACTICAL) VAPOUR
ABSORPTION SYSTEM (nh3 – h2o)
In the NH3 − H2O vapour absorption system, ammonia is the refrigerant and water is the absorber. An
analyser, a rectifier and two heat exchangers are added to the simple vapour absorption system. The
complete schematic diagram of ammonia-water absorption system is shown in Fig. 19.24.
The functions of various accessories are:

1. Analyser: In the simple vapour absorption system, some water is also varporised along with
ammonia and flows into the condenser. These water vapours further move and enter into the
expansion valve, where they freeze and choke the pipeline. The analyser is used to remove these
water vapours. The analyser may be an integral part of the generator or as a separate piece of
equipment. It consists of a series of trays mounted above the generator. The strong solution
from the absorber and the aqua from the rectifier (or dehydrator) are introduced at the top of the
analyser and flows downward on the trays and into the generator. In this way, considerable liquid
surface area is exposed to the vapour rising from the generator. The vapours are cooled and most
of the water vapours condense, so that mainly ammonia vapours leave the top of the analyser. This
also reduces the external heat required in the generator since aqua is heated by the vapour.
2. Rectifier or Dehydrators: The function of the rectifier is to completely remove the water vapours
still left in the analyser. The rectifier is a closed type vapour cooler of the double pipe type, shell
and coil, or shell and tube type. Its function is to further cool the ammonia vapours leaving the
analyser so that the remaining water vapours are condensed. The condensate from the rectifier is
returned to the top of the analyser by a drip return pipe.
3. Heat exchangers: The heat exchanger-1 provided between the pump and the generator is used to
cool the weak hot solution returning from the generator to the absorber. The heat removed from
the weak solution raises the temperature of the strong solution leaving the pump and going to
analyser and generator. This operation reduces the heat supplied to the generator and the amount
of cooling required for the absorber.

M19_THERMAL ENGINEERING_SE_XXXX_CH19.indd 1025 6/16/2018 11:37:46 AM


1026 Chapter 19

Cooling Cooling
water Qrd water

Vapour Liquid NH3


Condenser
Drip
Rectifier or
Analyser Qg dehydrator Qrc

Strong

Receiver
Generator Heating
solution coils

Weak solution
Heat
exchanger-2
Heat
exchanger-1

Pressure
Pump
reducing valve

Qp Low temperature
NH3 vapour
Absorber
Evaporator
Expansion
Qra valve
Cooling Qe
water

Figure 19.24  Ammonia-water vapour absorption system

The heat exchanger-2 provided between the condenser and the evaporator may be called liquid
sub-cooler. The liquid refrigerant leaving the condenser is sub cooled by the low temperature
ammonia vapour from the evaporator. This sub-cooled liquid is now passed to the expansion valve
and then to the evaporator.
  Net refrigerating effect, RE = Heat absorbed in the evaporator
Total energy supplied to the system,
Wt = Work done by the pump + Heat supplied in generator
RE
COP = (19.12)
Wt

19.15  ❐  L
 ITHIUM BROMIDE-WATER VAPOUR
ABSORPTION SYSTEM (LiBr-H2O)
In the LiBr-water system, water is the refrigerant and LiBr the absorber. The lithium bromide solu-
tion has a strong affinity for water vapour because of its very low vapour pressure. The lithium bro-
mide-water vapour absorption system is shown in Fig. 19.25. The evaporator and absorber are placed
in one shell which operates at the same low pressure of the system. The generator and condenser are
placed in another shell which operates at the same high pressure of the system.

M19_THERMAL ENGINEERING_SE_XXXX_CH19.indd 1026 6/16/2018 11:37:47 AM



Vapour Compression and Vapour Absorption Systems 1027

Cooling water out


Condenser
Generator

Weak
solution } Heating
coils (Steam)

H2O – LiBr
Strong solution Generator-Condenser
LiBr
Condensate
solution Cooling water water Evaporator
Chilled water
Pressure
tubes
reducing
Heat valve Low
exchanger temperature
High
Cooled water temperature
Pump

Evaporator Absorber
Pump
-Absorber

Cooling
Pump water
pond

Figure 19.25  Lithium bromide-water vapour absorption system

19.15.1  Working Principle


The water for process requirements is chilled as it is pumped through the chilled-water tubes in the
evaporator by giving up heat to the refrigerant water sprayed over the tubes. Since the pressure inside
the evaporator is maintained very low, therefore, the refrigerant water evaporates. The water vapours
thus formed are absorbed by the strong lithium bromide solution which is sprayed in the absorber.
In absorbing the water vapour, the lithium bromide solution helps in maintaining very low pressure
needed in the evaporator, and the solution becomes weak. This weak solution is pumped by a pump to
the generator where it is heated up by using steam in the heating coils. A portion of water is evaporated
by the heat and the solution becomes more strong. This strong solution is passed through the heat
exchanger and then sprayed in the absorber. The weak solution of lithium bromide from the absorber
to the generator is also passed through the heat exchanger. This weak solution gets heat from the
strong solution in the heat exchanger, thus reducing the quantity of steam required to heat the weak
solution in the generator.

M19_THERMAL ENGINEERING_SE_XXXX_CH19.indd 1027 6/16/2018 11:37:48 AM


1028 Chapter 19

The refrigerant water vapours formed in the generated due to healing of solution are passed to the con-
denser where they are cooled and condensed by the cooling water flowing through the condenser water
tubes. The cooling water for condensing is pumped from the cooling water pond. This cooling water
first enters the absorber where it takes away the heat of condensation and dilution. The condensate
from the condenser is supplied to the evaporator to compensate the water vapor formed in the evapora-
tor. The pressure reducing valve reduces the pressure of condensate from the condenser pressure to the
evaporator pressure. The cooled water from the evaporator is pumped and sprayed in the evaporator in
order to t h e cool t h e water flowing through the chilled tubes. This completes the cycle.
19.15.2  Lithium Bromide-Water System Equipment
A line diagram of two-shell lithium bromide-water vapour absorption refrigeration system is shown
in Fig. 19.26. The various components are:

Condenser

Water

Generator
Steam
Solution with low water concentration

Solution with high water concentration

Throttle valve

Evaporator Water from Ref. Load

Water to Ref. Load

Absorber
Cooling
water

Heat
Exchanger
Reducing Pump
valve Purge Recirculating
Unit pump

Figure 19.26  Lithium bromide-water vapour absorption system

M19_THERMAL ENGINEERING_SE_XXXX_CH19.indd 1028 6/16/2018 11:37:48 AM


Vapour Compression and Vapour Absorption Systems
 1029

1. Generator: It consists of tube bundles submerged in the solution heated by steam or hot liquids.
2. Condenser: It. consists of tube bundles located in the vapour space over the generator shielded
from carry over of salt by eliminators. Cooling water to condenser first passes through the absorber.
3. Absorber: It consists of bundles over which the strong absorbent is sprayed. Refrigerant vapours
are condensed into the absorbent releasing heat to the cooling water passing through it.
4. Evaporator: Evaporators are tube bundles over which the refrigerant water is sprayed and evap-
orated. The liquid to be cooled passes inside the tubes.
5. Solution heat exchangers: Solution heat exchangers are all of steel shell and tube construction.
6. Purgers: All units include a purger which is used to remove non-condensable gases. Non-
condensable gases present in small quantities can raise the total pressure in the absorber
sufficiently to significantly change the evaporator pressure. Very small pressure increases cause
appreciable change in the refrigerant evaporating temperatures.
7. Expansion Device: Mechanical expansion valves are not used in absorption units. The flow or
refrigerant liquid to evaporator is controlled by an orifice or other fixed restriction between the
condenser and the evaporators.
8. Pump: Other details about pumps etc., are shown in Fig. 19.26.
Capacity control: All lithium bromide-water cycle absorption machines meet load variations and
maintained chilled water temperature control by varying the rate of re-concentration of the absorbent
solution.
At any given constant load, the chilled water temperature is maintained by a temperature dif-
ference between refrigerant and chilled water. The refrigerant temperature is maintained in turn by
absorber being supplied with a flow rate and concentration of solution, and by absorber cooling water
temperature.
Load changes are reflected by corresponding changes in chilled water temperature. A load reduc-
tion, for example, results in less temperature difference being required in the evaporator and a reduced
requirement for solution flow or concentration. The resultant chilled water temperature drop is met
basically by adjusting the rate of re-concentration to match the reduced requirements of the absorber.
All units sense the chilled water temperature changes resulting from a load change with a thermo-
stat in the leaving chilled water.

19.16  ❐  C
 OMPARISON OF AMMONIA-WATER AND
LITHIUM BROMIDE-WATER ABSORPTION
SYSTEMS
The comparison of ammonia-water and lithium bromide-water systems are given m Table 19.2

Table 19.2  Comparison of ammonia-water and lithium bromide-water systems

Features Ammonia-Water System Lithium bromide-water


systems
1. Refrigerant Ammonia solution (aqua) Water
2. Absorbent Water Lithium bromide
3. Difference in boiling points of 138°C. This system is not suitable >200°C satisfies the boiling point
refrigerant and absorber from boiling point view requirement
(Continued)

M19_THERMAL ENGINEERING_SE_XXXX_CH19.indd 1029 6/16/2018 11:37:48 AM


1030 Chapter 19

Table 19.2 (Continued )

Features Ammonia-Water System Lithium bromide-water


systems
4. Quality of vapours leaving the Contains some amount of water. A pure substance
generator
5. Solubility in water Satisfactory Satisfactory
6. Applications Refrigeration systems like ice Air-conditioning purposes only.
plants, ice cream plants, etc.
7. Freezing point of refrigerant. −78°C, does not choke the evapo- 0°C clokes the evaporator tubes
rator tubes at working
8. Corrosiveness of mixture Poisonous gas in nature and aqua Yes
solution is corrosive

Exercises

19.1. The evaporator and condenser temperatures in an NH3 refrigeration system are –10°C and 40°C respec-
tively. Determine per TR basis: (a) mass flow rate; (b) compressor work, (e) condenser heat rejection; (d)
C.O.P.; and (e) refrigerating efficiency. Use only the properties of NH3 given below:

Saturation Pres- Specific volume of Enthalpy, kJ/kg Entropy, kJ/kg.K


temperature (t) sure (p) vapour (vg) m3/kg Liquid Vapour Liquid Vapour
°C bar
−10 2.908 0.418 134.95 1431.41 0.5435 5.4712
40 15.55 0.09833 371.47 1472.02 1.3574 4.8728

For superheated NH3 at 15.55 bar, the following values may be taken

Superheat K Specific volume (v) m3/kg Enthalpy kJ/kg Entropy (s) kJ/kg.K
60 0.108 1647.9 5.3883
80 0.116 1700.3 5.4253

[Ans. 0.198 kg/min: 0.82 kW; −4. 32 kW; 4.27; 81.2%]


19.2. In a vapour compression refrigeration system using R-12, the evaporator pressure is 1.4 bar and the con-
denser pressure is 8 bar. The refrigerant leaves the condenser sub-cooled to 30°C. The vapour leaving the
evaporator is dry and saturated. The compression process is isentropic. The amount of heat rejected in
the condenser is 13.42 MJ/min. Determine: (a) refrigerating effect in kJ/kg; (b) refrigerating load in TR;
(c) compressor input in kW; and (d) COP. [Ans. 114 k//kg; 49 TR: 51.4 kW; 3.35]
19.3. A vapour compression refrigerator works between the temperature limits of −20°C and 25°C. The refriger-
ant leaves the compressor in dry saturated condition. If the liquid refrigerant is undercooled to 20°C before
entering the throttle valve determine:
(a) work required to drive the compressor ;
(b) refrigerating effect produced per kg of the refrigerant; and
(c) theoretical COP.
Assume specific heat of the refrigerant as 4.8. The properties of the refrigerant are

M19_THERMAL ENGINEERING_SE_XXXX_CH19.indd 1030 6/16/2018 11:37:48 AM


Vapour Compression and Vapour Absorption Systems
 1031

Temperature, °C Enthalpy, kJ/kg Entropy, kJ/kg.K


Liquid Vapour Liquid Vapour
–20 89.78 1420.02 0.3684 5.6244
25 298.90 1465.84 1.1242 5.0391

[Ans. 189.7 kJ/kg; 990.2 kJ/kg; 5.01]


19.4.  A food storage chamber requires a refrigeration system of 12 TR capacity with an evaporator temperature
of −8°C and condenser temperature of 30°C. The refrigerant R-12 is sub-cooled by 5°C before entering
the throttle valve, and the vapour is superheated by 6°C before entering the compressor. If the liquid and
vapour specific heats are 1.235 and 0.733 kJ/kg.K respectively, find: (a) refrigerating effect per kg; (b) mass
of refrigerant circulated per minute; and (c) coefficient of performance.
   The relevant properties of the refrigerant R-12 are given below:
Saturation temper- Enthalpy, kJ/kg Entropy, kJ/kg.K
ature, °C
Liquid Vapour Liquid Vapour
−8 28.70 184.06 0.1148 0.7007
30 64.59 199.62 0.2400 0.6853

 [Ans. 130.05 kJ/kg; 19.4 kg; 6.2]


19.5.  An ammonia refrigerator produces 30 tonnes of ice form and at 0°C in 24 hours. The temperature range
of the compressor is from 25°C to −15°C. The vapour is dry saturated at the end of compression and an
expansion valve is used. Assume a coefficient of performance to be 60% of the theoretical value. Calculate
the power required to drive the compressor. Latent heat of ice = 335 kJ/kg. Properties of ammonia are.

Temperature, °C Enthalpy, kJ/kg Entropy, kJ/kg.K


Liquid Vapour Liquid Vapour
25 298.9 1465.84 1.1242 5.0391
−15 112.34 1426.54 0.4572 5.5490

[Ans. 33.24 kW]


19.6.  A freezer of 20 TR capacity has evaporator and condenser temperature of −30°C and 25°C respectively.
The refrigerant R-12 is sub-cooled by 4°C before it enters the expansion valve and is superheated by 5°C
before leaving the evaporator. The compression is isentropic and the valve throttling and clearance are to
be neglected. If a six cylinder, single acting compressor with stroke equal to bore running at 1000 rpm
is used, determine (a) COP, of the refrigerating system, (b) mass of refrigerant to be circulated per min,
(c) theoretical piston displacement per minute, and (d) theoretical bore and stroke of the compressor. The
specific heat of liquid R-12 is 1.235kJ/kg K and of vapour R-12 is 0.733 kJ/kg.K. The properties of R-12
are given below:

Saturation Pressure Enthalpy, kJ/kg Entropy, kJ/kg.K Specific volume, m/kg


temp.,°C bar Liquid Vapour Liquid Vapour Liquid Vapour
−30 1.0044 8.86 174.20 0.0371 0.7171 0.006.73 0.1596
25 6.5184 59.7 197.73 0.2239 0.6868 0.007.64 0.0269

[Ans. 3.64; 34.12 kg/min; 5.56 m/min; 0.106 m]

M19_THERMAL ENGINEERING_SE_XXXX_CH19.indd 1031 6/16/2018 11:37:49 AM


1032 Chapter 19

19.7.  A refrigeration plant of 8 TR capacity has its evaporation temperature of − 8°C and condenser temperature
of 30°C. The refrigerant is sub-cooled by 5°C before entering into the expansion valve and vapour is super-
heated by 6°C before leaving the refrigerator. The suction pressure drop is 0.2 bar in the suction valve and
discharge pressure drop is 0.1 bar in the discharge valve.
  If the refrigerant used is R-12, find out the COP, of the plant and theoretical power required for the com-
pressor. Assume compression is isentropic. Use p-h chart for calculation.
19.8.  An ammonia refrigerator works between − 6.7°C and 26.7°C, the vapour being dry it the end of isentropic
compression. There is no under cooling of liquid ammonia and the liquid is expanded through a throttle
valve after leaving the condenser. Sketch the cycle on the T-s and p-h diagram and calculate the refriger-
ation effect per kg ammonia and the theoretical coefficient of performance of the unit with the help of the
properties given below:
Temperature, °C Enthalpy, kJ/kg Entropy, kJ/kg.K
Liquid Vapour Liquid Vapour
−6.7 152.18 1437.03 0.6016 5.4308
26.7 307.18 1467.03 1.1515 5.0203

 [Ans. 1028.3 kJ/kg, 7.2]


19.9.  An ammonia refrigerating machine fitted with an expansion valve works between the temperature limits
of −10°C and 30°C. The vapour is 95%, dry at the end of isentropic compression and the fluid leaving
the condenser is at 30°C If the actual coefficient of performance is 60% of the theoretical, find the ice
produced per kW hour at 0°C from water at 10°C. The latent heat of tee is 335 kJ/kg. The ammonia has
the following properties:

Temperature, °C Liquid heat, kJ/kg Latent heat kJ/kg Entropy, kJ/kg.K


Liquid Vapour
30 323.08 1145.79 1.2037 4.9842
−10 135.37 1297.68 0.5443 5.4770

 [Ans. 33.24 kg/kWh]


19.10.  A R-12 refrigerating machine works on vapour-compression cycle. The temperature of refrigerant in the
evaporator is − 20°C. The vapour is dry saturated when it enters the compressor and leaves it in a super-
heated condition. The condenser temperature is 30°C. Assuming specific heat at constant pressure for
R-12 in the superheated condition as 1,884 kJ/kg.K, determine:
  (a)  condition of vapour at the entrance to the condenser:
  (b)  condition of vapour at the entrance to the evaporator; and
  (c)  theoretical COP of the machine. The properties of R-12 are:

Temperature, °C Enthalpy, kJ/kg Entropy, kJ/kg.K


Liquid Vapour Liquid Vapour
− 20 17.82 178.73 0.0731 0.7087
30 64.59 199.62 0.2400 0.6843
 [Ans. 33.8°C; 29% dry; 4.07]
19.11.   A CO2 refrigerating plant fitted with an expansion valve, works between the pressure limits of 54.81
bar and 20.93 bar. The vapour is compressed isentropically and leaves the compressor cylinder at 32°C.

M19_THERMAL ENGINEERING_SE_XXXX_CH19.indd 1032 6/16/2018 11:37:49 AM


Vapour Compression and Vapour Absorption Systems
 1033

The condensation takes place at 18°C in the condenser and there is no undercooling of the liquid. Deter-
mine the theoretical coefficient of performance of the plant. The properties of CO2 are:

Pressure, bar Saturation tem- Enthalpy, kJ/kg Entropy, kJ/kg.K


perature, °C Liquid Vapour Liquid Vapour
54.81 + 18 137.48 302.55 0.5065 1.0738
20.93 − 18 43.27 323.06 0.1733 1.2692

 [Ans. 4.92]
19.12.   A single stage NH3 refrigeration system has cooling capacity of 500 kW. The evaporator and condenser
temperatures are −10°C and 30°C respectively. Assuming saturation cycle, determine: (a) mass flow rate
of refrigerant; (b) adiabatic discharge temperature, (c) compressor work in kW; (d) condenser heat rejec-
tion, (e) COP; and (f) compressor swept volume in m3/min, if volumetric efficiency is 70%.
  The following values may be taken:
   hg (−10°C) = 1431.6 kJ/kg; hf (30°C) = 322.8 k.J/kg;
   vg (−10°C) = 0.4185 m3/kg; sg = (−10°C) = 5.4717 kJ/kg.K
 The properties of superheated NH3 at condenser pressure of 1 1.66 bar (30°C) are as follows:
 At 85°C, h = 1621.8 kJ/kg; s = 5.5484 kJ/kg.K.
 At 90°C, h = 1634.5 kJ/kg;
          s = 5.4838 kJ/kg.K; [Ans. 0.45 kg/s; 88.3°C; 89.5 kW; 590 kW; 5.585; 16.2 m3/min]
19.13.   A 15 TR Freon 22 vapour compression system operates between a condenser temperature of 40°C and an
evaporator temperature of 5°C.
(a)  Determine the compressor discharge temperature:
(i)  Using the p-h diagram for Freon 22.
   (ii) Using saturation properties of Freon 22 and assuming the specific heat of its vapour as 0.8 kJ/kg.K.
(iii)  Using superheat tables for Freon 22.
(b) Calculate the theoretical piston displacement and power consumption of the compressor per ton of
refrigeration.
19.14.   A simple saturation ammonia compression system has a high pressure of 1.35 MN/m2 and a low pressure
of 0.19 MN/m2. Find per 400,000 kJ/h of refrigerating capacity, the power consumption of the compres-
sor and COP of the cycle.
19.15.  (a) A Freon 22 refrigerating machine operates between a condenser temperature of 40°C and an evapo-
rator temperature of 5°C. Calculate the increase (per cent) in the theoretical piston displacement and
the power consumption of the cycle:
      (i)  If the evaporator temperature is reduced to 0°C,
(ii)  If the condenser temperature is increased to 45°C.
(b) Why is the performance of a vapour compression machine more sensitive to change in evaporator
temperature than to an equal change in the condenser temperature?
19.16.   In a vapour compression cycle saturated liquid Refrigerant 22 leaving the condenser at 40°C is required
to expand to the evaporator temperature of 0°C in a cold storage plant.
(a) Determine the percentage saving in network of the cycle per kg of the refrigerantif an isentropic
expander could be used to expand the refrigerant in place of thethrottling device.
(b) Also determine the percentage increase in refrigerating effect per kg of refrigerant as a result of use
of the expander. Assume that compression is isentropic from saturated vapour state at 0°C to the
condenser pressure.

M19_THERMAL ENGINEERING_SE_XXXX_CH19.indd 1033 6/16/2018 11:37:49 AM


Chapter 20

Air-Conditioning and
Psychrometrics
20.1 ❐ INTRODUCTION
The air-conditioning is that branch of engineering which deals with the study of supplying and main-
taining desirable internal room atmospheric conditions of air for human comfort. The four important
factors for human comfort are: temperature, humidity, purity and motion of air.
The working substance for air-conditioning is moist air, which a mixture of two gases. One of these
is dry air, which itself is a mixture of a number of gases and the other is water vapour which may exist
in a saturated or superheated state. Moist air is not a pure substance in any process in which conden-
sation or evaporation of moisture occurs. Both dry air and water vapour can be considered as perfect
gases since both exist in the atmosphere at low pressures. Hence, perfect gas laws to both and Dalton’s
law of partial pressures for non-reactive mixture of gases can be applied to dry air part only.

20.2 ❐ PRINCIPLES OF PSYCHROMETRY


The psychrometry is that branch of engineering which deals with the study of moist air i.e. dry air
mixed with water vapour.
Dry air: The dry air consists of O2 = 0.2099, N2 = 0.7803, Ar = 0.0094, CO2 = 0.0003 and
H2 = 0.0001 parts by volume.
Molecular weight of air     = 28.966
Gas constant, Ra    = 0.287 kJ/kg K
Density of air       = 1.293 kg/m3 at 1.01325 bar and 0°C
Specific heat of air at constant pressure = 1.005 kJ/kg K
Molecular weight of water (vapour)    = 18.02
Gas constant for water vapour,    Rv = 0.461 kJ/kg K
Barometric pressure     = 1.01325 bar or 760 mm Hg
Dalton’s law of partial pressures: The Dalton’s law of partial pressures states that for a mixture of
ideal gases, the total pressure is equal to the sum of the partial pressures, which each constituent would
exert if it occupied the whole space alone. Thus,
Barometric pressure of mixture of dry air and water vapour is given by
pb = pa + pv  (20.1)

M20_THERMAL ENGINEERING_SE_XXXX_CH20.indd 1034 6/16/2018 11:27:13 AM


Air-Conditioning and Psychrometrics 1035

where pa = partial pressure of dry air


pv = partial pressure of water vapour
Moist air: It is a mixture of dry air and water vapour.
Saturated air: It is a mixture of dry air and water vapour when air has absorbed maximum amount of
water vapour. At saturation pv = ps, the saturation pressure.
Degree of saturation (l):
actual mass of water vapour in a unit mass of dry air
=
mass of water vapour in the same mass of dry air when it
is saaturated at the same temperature.
Specific humidity or humidity ratio w: It is the mass of water vapour present in one kg of dry air. It
is expressed in terms of kg per kg of dry air (kg/kg d.a)
Absolute humidity: It is the mass of water vapour present in 1m3 of dry air. It is expressed in terms
of kg per cubic metre of dry air (kg/m3 d.a.)
Relative humidity (RH or f): It is the ratio of actual mass of water vapour in a given volume of
moist air to the mass of water vapour in the same volume of saturated air at the same temperature and
pressure.
Dry Bulb Temperature (DBT, td): It is the temperature of air recorded by a thermometer, when it is
not affected by the moisture present in the air.
Wet Bulb Temperature (WBT, tw): It is the temperature of air recorded by a thermometer, when its
bulb is surrounded by a wet cloth exposed to the air.
The DBT and WBT are measured by a Sling Psychrometer, as shown in Fig. 20.1.
Wet Bulb Depression, WBD = DBT − WBT
Dew Point Temperature (DPT, tdp): It is the temperature of air recorded by a thermometer, when the
water vapour present in it begins to condense. Thus, DPT is the saturation temperature (tsat) corre-
sponding to the partial pressure of water vapour pv
Dew Point Depression, DPD = DBT − DPT

Handle

Dry bulb
Wet bulb
thermometer
thermometer

Bulb covered
with wetted wick

Figure 20.1  Sling psychrometer

M20_THERMAL ENGINEERING_SE_XXXX_CH20.indd 1035 6/16/2018 11:27:14 AM


1036 Chapter 20

20.3 ❐ PSYCHROMETRIC RELATIONS


1. Specific (or absolute) humidity, humidity ratio or moisture content, w: It is defined as the
ratio of the mass of water vapour to the mass of dry air in a given volume of the mixture.
Let p, V, T, m = pressure, volume, absolute temperature and mass respectively
R, M, v, R = gas constant, molecular mass, specific volume, and universal gas constant respectively
Subscripts a, v = air and vapour respectively.
V
m v v
Specific humidity, w= v = v = a  (20.2)
ma V vv
va
R
For air,   pa va = T 
Ma
R
And    paV = ma T
Ma
For water vapour,
R
pv vv = T 
Mv
R
and   pvV = mv T
Mv
Substituting for mv and ma from these equations in Eq. (20.2), we get
p M 18.02 pv p
w = v v = = 0.622 v (kg of water vapour per kg dry air) (20.3)
pa M a 28.996 pa pa
pb = pa + pv
Now actual total pressure,
0.622 pv
∴     w =
 (20.4)
pb − pv
   For ma = 1 kg, mv = w kg and total mass of moist air
  m = (1 + w) kg (20.5)
For saturated air,   pv = ps and maximum humidity ratio
0.622 ps
  ws = wmax = (20.6)
pb − ps
where ps = partial pressure of air corresponding to saturation temperature i.e. td.
2. Degree of saturation or percentage humidity, l: The ratio of actual specific humidity w to the
specific humidity ws of saturated air at the same temperature is termed as the degree of saturation.
 0.622 pv 
 
w  pb − pv   pv   pb − ps   pv   1 − ( ps / pb ) 
m = = =  =   (20.7)
ws  0.622 ps   ps   pb − pv   ps   1 − ( pv / pb ) 
 
 pb − ps 
The degree of saturation is a measure of the capacity of air to absorb moisture.

M20_THERMAL ENGINEERING_SE_XXXX_CH20.indd 1036 6/16/2018 11:27:19 AM


Air-Conditioning and Psychrometrics 1037

3. Relative Humidity (RH orf): It is defined as the ratio of mass of water vapour mv in a certain
volume of moist air at a given temperature to the mass of water vapour mvs in the same volume of
saturated air at the same temperature.

 pvV 
m  RT  p
f = v = = v (20.8)
mvs psV ps
RT
For a perfect gas   pv vv = ps vs
p v
∴    v = s

ps vv

pv vs
or    f = = (20.9)
ps vv

  ps     ps  
1 −   1−   
pv   
pb    pb  
Now    m = =f 
ps   pv     pv  
1 −    1 − f  p  
  pb    b 

m
   f = (20.10)
p 
1 − (1 − m )  s 
 pb 
4. Pressure of water vapour:
(a)  The partial pressure of water vapour, according to Carrier’s equation, is given by
( pb − pw )(td − tw )
pv = pw − (20.11)
1544 − 1.44 tw
where   pw = saturation pressure corresponding to WBT
       
pb = barometric pressure
td = DBT
tw = WBT
(b)  Modified Apjohn equation:
pb (td − tw )
pv = pw (20.12)
1500

(c)  Modified Ferrel equation:


 t 
pv = pw − 0.00066 pb (td − tw ) 1 + d  (20.13)
 872 .78 

M20_THERMAL ENGINEERING_SE_XXXX_CH20.indd 1037 6/16/2018 11:27:23 AM


1038 Chapter 20

5. Vapour density or Absolute humidity:


Let pv = density of water vapour corresponding to its partial pressure and DBT, td.
pa = density of dry air.
Then mv = Vvrv and ma = Vara
mv Vv rv
=
ma Va ra

Since Va = Vv, therefore,


mv rv
w= =
ma ra

Or rv = w ra

Now    pa Va= maRaTd

1
For ma = 1 kg, Va = va =
ra

1

pa × = RaTa
ra

pa
or ra =
RaTa
w ra w ( pb − pv )

rv = = (20.14)
RaTd RaTa

20.4 ❐ ENTHALPY OF MOIST AIR


The temperature-entropy diagram is shown in Fig. 20.2.
Enthalpy of moist air = Enthalpy of dry air + Enthalpy of water vapour associated with dry air
h = ha + whv per kg of dry air
where    ha = cpa td = 1.005 td kJ/kg
cpa = 1.005 kJ/kg. K
td = DBT of air in °C.
hv = c pw tdp + [( h fg )dp + c pv (td − tdp )] kJ/kg
= 4.1868 tdp + ( h fg )dp + 1.88 (td − tdp )

where   c pw = 4.1868 kJ/kg K is the specific heat of liquid water

( h fg )dp = latent heat of vaporization at DPT

c pv = 1.88 kJ/kg.K is the specific heat of superheated water vapour

M20_THERMAL ENGINEERING_SE_XXXX_CH20.indd 1038 6/16/2018 11:27:27 AM


Air-Conditioning and Psychrometrics 1039

t
ps
pv

td C
A
Temperature tdp
Saturated
water vapour Superheated
B
water vapour

s
Entropy

Figure 20.2  Temperature-entropy diagram

tdp = DPT, °C
w = specific humidity
h = 1.005 td + w [4.1868 tdp + (hfg)dp + 1.88 (td + tdp)]
= (1.005 + 1.88 w)td + w [(hfg)dp + 2.3068 tdp] kJ/kg d.a. (20.15)
= 1.005 td + w (2500 + 1.88 td) kJ/kg d.a. (20.16)

20.5 ❐ HUMID SPECIFIC HEAT


Eq. (20.16) can be written as,
h = (c pa + wc pv )td + w ( h fg )0°C

   = c p td + w ( h fg )0°C (20.17)

where c p = (c pa + wc pv ) = 1.005 + 1.88w kJ/(kg d.a.K) (20.18)

is termed as the humid specific heat. It is the specific heal of moist air (1 + w) kg per kg of dry air.
At low temperatures of air-conditioning the value of w is very small. An approximate value of cp
of 1.0216 kJ/(kg d.a.).K may be taken for nil practical purposes
h = 1.0216 td + w[( h fg )dp + 2.3068tdp ] (20.19)

Example 20.1
A mixture of dry air and water vapour is at a temperature of 20°C under a total pressure of 735 mm Hg.
The dew point temperature is 15°C. Calculate the following:
(a)  Partial pressure of water vapour,
(b)  Relative humidity,

M20_THERMAL ENGINEERING_SE_XXXX_CH20.indd 1039 6/16/2018 11:27:30 AM


1040 Chapter 20

(c)  Specific humidity,


(d)  Specific enthalpy of water vapour
(e)  Enthalpy of air per kg of dry air, and
(f)  Specific volume of air per kg of dry air.

Solution

(a) From steam tables, the partial pressure of water vapour at tdp = 15°C
pv =12.79 mm Hg

Now    760 mm Hg = 1.01325 × 105 N/m 2

1 mm Hg = 133.32 N/m2

∴                pv = 12.79 × 133.32 = 1705.16 N/m 2



(b) Saturation pressure of water vapour at 20°C DBT
ps = 17.54 mm Hg = 17.54 × 133.32 = 2338.4 N/m 2
pv 12.79
       Relative humidity, f = × 100 = × 100 = 72.92%
ps 17.54
0.622 pv 0.622 × 12.79
(c) Specific humidity, w = = = 0.011 kg water vapour/kg. d.a.
pb − pv 735 − 12.79
(d) Latent heat of vaporization of water at td = 20°C,(h fg )20°C = 2454.1 kJ/kg
Latent heat of vaporization of water at tdp = 15°C,(h fg )15°C = 2465.9 kJ/kg
Specific enthalpy of water vapour,
hv = [4.1868 tdp + ( h fg )dp + 1.88 (td − tdp )]

= [4.1868 × 15 + 2465.9 + 1.88 ( 20 − 15)]


 = 2538.1 kJ/kg w.v
(e) Enthalpy of air per kg d.a., h = 1.005 td + whv
 = 1.005 × 20 + 0.011 × 2538.1
 = 48.02 kJ/kg d.a.
RaT
(f) Specific volume of air per kg of d.a. v = va =
pa
0.287 × 103 × ( 273 + 20)
=
(735 − 12.79) × 133.32
= 0.873 m3/kg d.a

M20_THERMAL ENGINEERING_SE_XXXX_CH20.indd 1040 6/16/2018 11:27:35 AM


Air-Conditioning and Psychrometrics 1041

Example 20.2
A sling psychrometer gave the following readings:
DBT = 30°C, WBT = 20°C, Barometer reading = 740 mm of Hg.
Determine: (a) dew point temperature, (b) relative humidity, (c) specific humidity (d) degree of
saturation, (e) vapour density, and (f) enthalpy of mixture per kg of dry air
Solution
Given: td = 30°C, tw = 20°C, pb = 740 mm Hg
(a) Corresponding to tw = 20°C, from steam tables, saturation pressure, pw = 0.023385 bar
Barometric pressure, pb = 740 × 133.32 × 10−5 = 0.986568 bar
Partial pressure of water vapour
( p − pw ) (td − tw )
pv = pw − b (Carrier’s eqn.)
1544 − 1.44 tw

= 0.023385 −
(0.986568 − 0.023385) (30 − 20)
(1544 − 1.44 × 20)
= 0.01703 bar
Corresponding to pv = 0.01703 bar pressure, from steam tables, dew point temperature
tdp = 15°C
(b) Corresponding to td = 30°C, from steam tables, saturation pressure of water vapour,
ps = 0.04246 bar

pv 0.01703
RH, f = = = 0.401 or 40.1%
ps 0.04246
0.622 pv 0.622 × 0.01703
(c) Specific humidity,    w = =
pb − pv 0.986568 − 0.01703
= 0.010925 kg/kg d.a
0.622 ps
(d) Specific humidity of saturated air, ws =
pb − ps
0.622 × 0.04246
= = 0.02797 kg / kg d.a.
0.986568 − 0.04246
w 0.010925
 Degree of saturation,    m = = = 0.3905 or 39.05%
ws 0.02797
w ( pb − pv ) 0.010925 (0.986568 − 0.01703) × 105
(e) Vapour density, rv = =
RaTd 287 ( 273 + 30 )
= 0.01218 kg/m3 of d.a.
(f) From steam tables, (hfg)dp = 15°C = 2465.9 kJ/kg
 Enthalpy of mixture per kg d.a.,
  h = 1.0216 td + w[(hfg)dp + 2.3068 tdp]
  = 1.0216 × 30 + 0.010925 [2465.9 + 2.3068 × 15] = 57.966 kJ/kg d.a.

M20_THERMAL ENGINEERING_SE_XXXX_CH20.indd 1041 6/16/2018 11:27:38 AM


1042 Chapter 20

Example 20.3
A room 5m × 4m × 3m contains moist air at 40°C. The barometric pressure is 1 bar and relative
humidity is 70%. Determine: (a) humidity ratio, (b) dew point temperature, (c) mass of dry air, and
(d) mass of water vapour.
If the moist air is further cooled at constant pressure until the temperature is 15°C, find the
amount of water vapour condensed.

Solution
Given: V = 5 × 4 × 3 = 60 m3, td = 40°C, pb = 1 bar, f = 0.70.
(a) Saturation pressure of vapour corresponding to td = 40°C, from steam tables is,
ps = 7.384 kPa or 0.07384 bar
p
RH, f = v
ps
pv = 0.7 × 0.07384 = 0.05169 bar
0.622 pv 0.622 × 0.05169
Humidity ratio, w= =
pb − pv 1.0 − 0.05169
= 0.0339 kg/kg d.a or 33.9 g/kg d.a
(b) DPT, tdp = saturation temperature corresponding to pv = 0.05169 bar or 5.169 kPa
= 33.46°C from steam tables
(c) pa = pb − pv = 1.0 − 0.05169 = 0.94831 bar

paV 0.94831 × 10 2 × 60
Mass of dry air, ma = = = 63.34 kg
RaTd 0.287 × ( 273 + 40)
mv
(d) Now w =
ma
Mass of water vapour, mv = 0.0339 × 63.34 = 2.147 kg
Saturation pressure corresponding to 15°C, from steam tables is,
ps = pv = 1.707 kPa or 0.01707 bar
0.622 pv 0.622 × 0.01707
w= =
pb − pv 1.0 − 0.01707
= 0.0108 kg/kg d.a or 10.8 g/kg d.a.
  Now  pa = pb − pv = 1 − 0.01707 = 0.98293 bar or 98.293 kPa

paV 98.293 × 60
  Mass of dry air, ma = = = 71.35 kg
RaTa 0.287 × ( 273 + 15)
  Mass of water vapour, mv = wma = 0.0108 × 71.35 = 0.77 kg
 Amount of water vapour condensed = 2.147 – 0.77 = 1.377 kg

M20_THERMAL ENGINEERING_SE_XXXX_CH20.indd 1042 6/16/2018 11:27:41 AM


Air-Conditioning and Psychrometrics 1043

20.6 ❐ THERMODYNAMIC WET BULB TEMPERATURE OR


ADIABATIC SATURATION TEMPERATURE (AST)
The thermodynamic wet bulb temperature or AST is the temperature at which the air can be brought to
saturation state adiabatically by the evaporation of water in the flowing air. A schematic representation
of this process, called the adiabatic saturation process, is shown in Fig. 20.3. It consists of an adiabatic
enclosure containing adequate quantity of water. There is also an arrangement for feed water from the
top.
The unsaturated air enters the enclosure at section 1. As the air flows through the enclosure water
evaporates which is carried by the flowing stream of air. Thus the specific humidity of air increases.
The water level in the enclosure is maintained by the feed water. Both the air and water are cooled as
the evaporation takes place. The process continues until the energy transferred from the air to water
is equal to the energy required to vaporise water. When steady conditions are reached, the air flowing
at section 2 becomes saturated with water vapour. The temperature of the saturated air at section 2
is known as thermodynamic wet bulb temperature or adiabatic saturation temperature. The adiabatic
saturation process is represented on T-s diagram as shown by the curve 1-2 in Fig. 20.4. The adiabatic
saturation temperature is taken equal to the wet bulb temperature.
Let td1 = DBT of initial unsaturated air
td2 = DBT of saturated air
    = adiabatic saturation temperature, tw
   h1, h2 = enthalpy of unsaturated and saturated air respectively
    w1, w2 = specific humidity of air at sections 1 and 2 respectively
hfw = sensible heat of water at adiabatic saturation temperature
For energy balance of air at inlet and outlet, we have
h1 + ( w2 − w1 )h fw = h2 
or h1 + w1h fw = h2 + w1h fw 
Now h1 = ha1 + w1 hs1
and h2 = ha 2 + w2 hs 2
where ha1 = enthalpy of 1 kg dry air at DBT = td1 = cpa td1
hs1 = enthalpy of superheated vapour at td1 per kg of vapour
ha2 = enthalpy of 1 kg dry air at WBT = tw = cpa td2
hs2 = enthalpy of saturated vapour at WBT = tw per kg of vapour
Thus, we get
( ha1 + w1hs1 ) − w1h fw = ( ha 2 + w2 hs 2 ) − w2 h fw
w1 ( hs1 + h fw ) = w2 ( hs 2 + h fw ) + ( ha 2 − ha1 )
w2 ( hs 2 − h fw ) + ( ha 2 − ha1 )
w1 = (20.20)
( hs1 − h fw )
Also h1 = c pa td1 + w1 hs1
  h2 = c pa td 2 + w2 hs 2

M20_THERMAL ENGINEERING_SE_XXXX_CH20.indd 1043 6/16/2018 11:27:49 AM


1044 Chapter 20

Feed water
tw, hfw,(w2 − w1)
Adiabatic
enclosure

Unsaturated tw, h2, w2


1
Inlet air Water (tw) Outlet seturated
td1, h1, w1 2 air

Figure 20.3  Adiabatic saturator

Adiabatic saturation
T process

DBT 1
td1 (Unsaturated
Saturated air)
WBD

air
WBT 2
td2 = tw
p = c (cooling)
tdp DPT Dew point of
unsaturated
air at 1.

Figure 20.4  T-s diagram for adiabatic saturation process

where hs 2 = h f 2 + h fg 2

hs1 = hs 2 + c pv (td1 − td 2 ) = h f 2 + h fg 2 + c pv (td1 − td 2 )


From the energy balance, we get
c pa td1 + w1[h f 2 + h fg 2 + c pv (td1 − td 2 )] + ( w2 − w1 ) h f 2
= c pa td 2 + w2 ( h f 2 + h fg 2 )
or (c pa + w1 c pv )td1 − (c pa + w2 c pv )td 2 = h fg 2 ( w2 − w1 )

or c p1td1 − c p 2td 2 = h fg 2 ( w2 − w1 )(20.21)


where cp1 and cp2 are humid specific heats
If cp1 = cp2 = cp, then
c p (td1 − td 2 ) = h fg 2 ( w2 − w1 )
h fg 2
∴ td 2 = td1 − ( w2 − w1 )(20.22)
cp

M20_THERMAL ENGINEERING_SE_XXXX_CH20.indd 1044 6/16/2018 11:27:55 AM


Air-Conditioning and Psychrometrics 1045

Example 20.4
Atmospheric air at 1 bar enters the adiabatic saturator. The dry bulb temperature is 30°C and wet
bulb temperature 20°C during adiabatic saturation process. Calculate the following:
(a) Humidity ratio of entering air,
(b) Vapour pressure and relative humidity at 30°C, and
(c) Dew point temperature.
Solution
Given: pb = 1 bar, td = 30°C, tw = 20°C
(a)  Saturation pressure of vapour at 20°C, from steam tables is,
pv2 = 2.34 kPa or 0.0234 bar
Enthalpy of saturated vapour at 20°C, hs2 = hg2 = 2538.2 kJ/kg
Sensible heat of water at 20°C, h fw = 83.9 kJ/kg
Enthalpy of saturated vapour at 30°C, hs1 = hg1 = 2556.4 kJ/kg
Enthalpy of 1 kg unsaturated air at td = 30°C,
ha1 = mc pa td = 1 × 1.005 × 30 = 30.15 kJ/kg
Enthalpy of 1 kg saturated air at tw = 20°C,
ha 2 = mc pa tw = 1 × 1.005 × 20 = 20.10 kJ/kg
0.622 pv 2 0.622 × 0.0234
w2 = = = 0.0149 kg/kg d.a.
pb − pv 2 1.0 − 0.0234
w2 ( hs 2 − h fw ) + ( ha 2 − ha1 )
w1 =
( hs1 − h fw )
0.0149( 2538.2 − 83.9) + ( 20.10 − 30.15)
=
( 2556.4 − 83.9)
= 0.0107 kg/kg d.a.
0.622 pv1
(b) Now w1 =
pb − pv1
0.622 pv1
0.0107 =
1.0 − pv1
0.0107 – 0.0107 pv1 = 0.622 pv1
pv1 = 0.0169 bar
Saturation pressure corresponding to 30°C from steam tables is,
ps = 4.246 kPa or 0.04246 bar
p 0.0169
Relative humidity,   f = v1 = = 0.3980 or 39.80%
ps 0.04246
(c) Dew point temperature is the saturation temperature corresponding to the partial pressure of
water vapour, pv1 = 0.0169 bar or 1.69 kPa
From steam tables, we have
tdp = 14.85°C

M20_THERMAL ENGINEERING_SE_XXXX_CH20.indd 1045 6/16/2018 11:28:02 AM


1046 Chapter 20

20.7 ❐ PSYCHROMETRIC CHART


The psychrometric chart is a graphical representation of the complete thermodynamic properties of
moist air and psychrometric analysis of air-conditioning processes. The chart most commonly used is
the w-td chart, which specifies specific humidity (w) or water vapour pressure (pv) along the ordinate
and the dry bulb temperature (td) along the abscissa. The chart is normally constructed for a standard
atmospheric pressure of 760 mm Hg or 1.01325 bar corresponding to the pressure at the mean sea
level. A typical layout of this chart is shown in Fig. 20.5.
Some of the important lines on the psychrometic chart are explained below:
DBT lines: These lines are vertical, i.e. parallel to the ordinate and are uniformly, spaced. These lines
are drawn at an interval of 5°C and up to the saturation line.
Specific humidity or moisture content lines: These lines are horizontal i.e parallel to the abscissa
and are uniformly spaced.
DPT lines: These lines are horizontal, i.e. parallel to the abscissa and are non-uniformly spaced. At
any point on the saturation line, the DBT and DPT are equal.
WBT lines: These are inclined straight lines and are non-uniformly spaced. At any point on the satu-
ration line DBT and WBT are equal.
Enthalpy line: These are inclined straight lines and uniformly spaced. These lines are parallel
to the WBT lines. The enthalpy values are given above the saturation line. Enthalpy deviation
(ws − w)hw is read form the deviation lines which are curved non-equally spaced with +ve and −ve
correction.
Specific volume lines: These are obliquely inclined straight lines and are uniformly spaced.
Vapour pressure lines: These lines are horizontal and uniformly spaced. These lines are not drawn
on the chart. A scale showing vapour pressure in mm of Hg is given on the extreme left or right side
of the chart.
Relative humidity lines: These are curved lines and follow the saturated line. The saturation line
represents 100% RH.
The psychrometric chart is shown in Fig. 20.6.
Temperature
Wet bulb

Enthalpy
deviation
Temperature
Dew point

line
Water vapour pressure, pv
Sensible heat factor, SHF

Alignment
circle
DPT Moisture
Specific humidity, w

Content
Spe
W

e
tion lin
DBT

BT

Satura idity
e hum
cific

tiv
Rela
volu
me

tds
Dry bulb temperature, td

Figure 20.5  Layout of psychrometric chart

M20_THERMAL ENGINEERING_SE_XXXX_CH20.indd 1046 6/16/2018 11:28:02 AM


6/16/2018 11:28:04 AM
1047

12
5
12
0 0.33 0.35
11
Psychrometric chart 11
5
0
(At standard atmospheric pressure of 10 30 x 10−3
1.0132 bar or 1.033 ata or 760mm of Hg) 5
10 29 0.40
in Sl units with SHF 0 28
30 125

5
95
90
27

0.9
Twb – Wet-Bulb temperature, C

80
90 26
120

70
φ – Relative humidity, % 0.45
e

4
25
lin

60

0.9
85
on
– Reference circle (Tdb = 25° and φ =50%) 24

50
115
ati
80

Figure 20.6  Psychrometric chart


23
tur
0.50

40
v – Specific volume, m3/kg of dry air

2
22

Sa
75 110

0.9

35
w – Specific humidity, kJ/kg of dry air 21

30
e
70
Air-Conditioning and Psychrometrics

25 20 0.55
105
65 h

SHF
r 19
ai 0.60
y 60 18 100
dr
of

0
17

0.9
g 55 0.65
/k 16 95
kJ Twb Tw 0.70
py 50 20 b 15
al 0.75
th 4 14 90
En 5
h, 4 13 w 0.80

8
25

0.8
0 12 85
20
35 0.90
15 11
30 15 10 80 1.0

6
9

0.8

M20_THERMAL ENGINEERING_SE_XXXX_CH20.indd 1047


25

5
10 8 75

0.8
20

4
7

0.8
10 70
15 5 6

0.82
5
10 0 65
5 4

0.8
5 3
60
0 2
1
55
0
0 5 10 15 20 25 30 35 40 45 50
Tdb, Dry-bulb temperature C 50
0 5 10 15 20 25 30 35 40 45
h, enthalpy kJ/kg of dry air
1048 Chapter 20

20.8 ❐ PSYCHROMETRIC PROCESSES


The psychrometric processes by which the state of moist air can be altered are shown in Fig. 20.7.
These processes are:
1. Sensible heating-process OA
2. Sensible cooling-process OB
3. Humidifying-process OC
4. Dehumidifying-process OD
5. Heating and humidifying-process OE
6. Cooling and dehumidifying-process OF
7. Cooling and humidifying-process OG
8. Heating and dehumidifying-process OH

Specific humidity, w
G C
E

B A

F H
D

DBT, td

Figure 20.7  Basic psychrometric processes

20.8.1 Sensible Heating or Cooling Process


The heating/cooling of air without any change in its specific humidity is known as sensible heating/
cooling. The process of sensible heating (line 1-2) or cooling (line 2-1) is shown by a horizontal line
in Fig. 20.8. In this process, the specific humidity of air remains constant, i.e. w1 = w2. The heat has to
be transferred which goes to change the temperature of air. The sensible heat transfer rate is given by,
Q s = m a ( h2 − h1 )
= m a [c pa (t2 − t1 ) + wc pv (t2 − t1 )]

= m a [1.005 + 1.88w ](t2 − t1 ) (20.23)


= m a c p (t2 − t1 )

c p = c pa + wc pv = humid specific heat


where  
m a = mass flow rate of dry air, kg d.a./s

M20_THERMAL ENGINEERING_SE_XXXX_CH20.indd 1048 6/16/2018 11:28:06 AM


Air-Conditioning and Psychrometrics 1049

y h2
alp
th
En
h1

DPT w1 = w2
1 2

tdp td1 td2


DBT

Figure 20.8  Sensible heating process

Vr
= = Volume of air V in m3/min × density of air r in kg/m3
60
For the purpose of calculations, standard air is taken at 20°C and 50% RH. The density of standard
air is approximately 1.2 kg/m3 d.a. The value of humid specific heat is taken as 1.0216 kJ/(kg d.a.)(K)
V (1.2) (1.0216 ).V ∆
(1t.2) (1.0216). ∆t
∴ Q s = ∴ Q s = = 0.0204 V . ∆ 0204 V . ∆t kW (20.24)
= t0.kW
60 60
The sensible heating of moist air can be done to any desired temperature. But the sensible cooling can
be done only up to the dew point temperature tdp, as shown in Fig. 20.8.

20.8.2  Humidification or Dehumidification Process


Humidification is the addition of moisture to air without change in its DBT. Similarly dehumidifi-
cation is the removal of moisture without change in DBT. When the state of air is altered along the
constant DBT line, such as 1-2 in Fig. 20.9, moisture in the form of vapour has to be transferred to
change the humidity ratio of air. This transfer of moisture is given by,
G = m a ( w2 − w1 ) (20.25)
Because of this change in humidity ratio, there is also a change in the specific enthalpy of air, as shown
in Fig. 20.9. This change in enthalpy due in the change in humidity ratio is considered to cause a latent
heat transfer, given by
Q  = m ( h − h ) = m [(c t + h w ) − (c t + h w )]
L a 2 1 a p d2 fg 2 p d1 fg 1
Since td1 = td2
∴ Q L = m a h fg ( w2 − w1 ) = 2500 G
∴ Q = m a h fg ( w2 − w1 ) = 2500 G
              L (20.26)

M20_THERMAL ENGINEERING_SE_XXXX_CH20.indd 1049 6/16/2018 11:28:10 AM


1050 Chapter 20

h2

h1
2 w2

w1 w
1

td = td1 = td2
DBT

Figure 20.9  Humidification or dehumidification

where  hfg = latent heat of vaporisation at td1

V (1.2) ( 2500)
Q L = × ∆w kW
60
= 50 V . ∆w kW (20.27)

20.8.3  Heating and Humidification


This process involves both a change in temperature as well as in the humidity ratio as shown in
Fig. 20.10. The change in temperature causes a sensible heat load, given by,
QS = m a ( hA − h1 ) = m a c p (td 2 − td1 )
The change in humidity ratio causes a moisture transfer, given by
G = ma ( w2 − w A ) = ma ( w2 − w1 )

Latent heat load QL = ma ( h2 − hA ) = ma h fg ( w2 − w1 )

Total heat load QL = Qs + QL = ma [c p (td 2 − td1 ) + h fg ( w2 − w1 )] (20.28)

Vr
m a =
Now  
60
V (1.2) V (1.2)
∴ ∴ Q = .02V .∆h .∆h = 0.02V .∆h
.∆h =Q0=
60 60

= V [ 0.0204 ∆t + 50 ∆w ] kW
     (20.29)

M20_THERMAL ENGINEERING_SE_XXXX_CH20.indd 1050 6/16/2018 11:28:14 AM


Air-Conditioning and Psychrometrics 1051

h2
)
hA SHF
(1-

F
h1 SH

∆h 2 w2

1 A w1
w

∆t

td1 td2 = tdA


DBT

Figure 20.10  Heating and humidification

20.8.4 Sensible Heat Factor-SHF


The ratio of the sensible heat transfer to the total heat transfer is termed as the sensible heat factor.
Thus
Q Qs
 SHF = s =
Q Qs + QL
hA − h1 h −h
= = A 1
( hA − h1 ) + ( h2 − hA ) h2 − h1
0.0204 ∆t 0.0204 ∆t
= = (20.30)
0.0204 ∆t + 50 ∆w 0.02 ∆h
The process line 1-2 is called the sensible heat factor line (or process or condition line). The point A
divides the total enthalpy change ( h2 − h1 ) in the ratio of SHF and (1 − SHF). Qs is proportional to
SHF and QL to (1 − SHF).

20.8.5 Cooling and Dehumidification


In this process, the DBT as well as specific humidity of air decreases. The final relative humidity of the
air is generally higher than that of the entering air. This is achieved by passing the air over a cooling
coil or through a cold water spray. The effective surface temperature of the coil is known as apparatus
dew point (ADP). The cooling and dehumidification process is shown in Fig. 20.11.
Qs = hA − h2
QL = h1 − hA
Q = Qs + QL = h1 − h2

M20_THERMAL ENGINEERING_SE_XXXX_CH20.indd 1051 6/16/2018 11:28:17 AM


1052 Chapter 20

h1

hA

y
lp
h2

a
1

th
3
w1

En
h4
2
w2
A
ADP w4
4
w

BPF (1-BPF)

td4 = ADP td2 td3 = tdp1 td1


DBT

Figure 20.11  Cooling and dehumidification

QS h −h
SHF = = A 2 (20.31)
QS + QL h1 − h2

td 2 − td 4 td 2 − ADP
By pass factor BPF = = (20.32a)
td1 − td 4 td1 − ADP

w2 − w4 h2 − h4
Also BPF = =  (20.32b)
w1 − w4 h1 − h4

20.8.6 Air Washer


The schematic representation of an air washer is shown in Fig. 20.12. It involves the flow of air
through a spray of water. During the course of flow, the air may be cooled or heated, humidified or
dehumidified, or simply adiabatically saturated, depending on the mean surface temperature of water.
The water is, accordingly, externally cooled or heated or simply circulated by a pump. Make-up water
is added for any loss in the case of humidification of air. Eliminator plates are provided to minimise
the loss of water droplets.
The thermodynamic changes of state of air along path 1-2 in an air washer, depending on the mean
surface temperature of water droplets ts (equal to tu), is shown in Fig. 20.13. The following processes
are possible:
1. Process 1-2a: Heating and humidification (ts > td1). The water is externally heated.
2. Process 1-2b: Humidification (ts = td1). The enthalpy of air increases. The water is externally
heated.

M20_THERMAL ENGINEERING_SE_XXXX_CH20.indd 1052 6/16/2018 11:28:20 AM


Air-Conditioning and Psychrometrics 1053

3. Process 1-2c: Cooling and humidification (tw1 < ts < td1). The mean surface temperature of water
is greater than the wet bulb temperature of air, tw1. The enthalpy of air increases as a result of
humidification. The water has to be externally heated.
4. Process 1-2d: Adiabatic saturation (tw1 = ts). The water is simply recirculated without any exter-
nal heating or cooling.
5. Process 1-2e: Cooling and humidification (tdp < ts < tw1). The enthalpy of air decreases. The water
is required to be externally cooled.
6. Process 1-2f: Cooling (ts = tdp). Water is required to be cooled.
7. Process 1-2g: Cooling and humidification (ts < tdp). The condition line drawn from the initial
state 1 is tangential to the saturation line.
The mass balance of an air washer is shown in Fig. 20.14.

Eliminator
plates Spray nozzles

Air in Air out


Tank

Make-up
water

Pump S
Heating or
cooling coil

Figure 20.12  Air-Washer

s 2c 2b
2d 2a
s 2e
s 2f
w1
1
2g
w
s

tdp tw1 td2 td1


td

Figure 20.13  Range of psychrometric processes with an air washer

M20_THERMAL ENGINEERING_SE_XXXX_CH20.indd 1053 6/16/2018 11:28:21 AM


1054 Chapter 20

3
Make
. up water
mw tw3

1 2
Air
. in Air
. out
ma, w1 ma, w2

4
. .
[mw − ma(w2 − w1)], tw4

Figure 20.14  Mass balance of an air washer

Let m a = mass flow rate of dry air


m w = mass flow rate of water
Energy balance gives,
m a ( h2 − h1 ) = m w c pw tw 3 − [m w − m a ( w2 − w1 )] c pw tw 4

or     m a ( h2 − h1 ) = m w c pw (tw 3 − tw 4 )tw 3 + m a ( w2 − w1 ) c pw tw 4

Neglecting the effect of temperature of water in the last term, we have

m a ( h2 − h1 ) = m w c pw (tw 3 − tw 4 ) (20.33a)

or m a dh = m w c pw dtw  (20.33b)
For adiabatic saturation case, dh = 0
dtw = 0 and tw3 = tw4

h2 − h1 w2 − w1
Humidifying efficiency of air washer h H = = (20.34)
hs − h1 ws − w1

ws − w1 w − w1
By pass factor BPF = =1− 2 = 1 − h H (20.35)
ws − w1 ws − w1

20.8.7 Cooling with Adiabatic Humidification


When the air is passed through an insulated chamber, as shown in Fig. 20.15(a), having water sprays
maintained at a temperature (tf) higher than the DPT of entering air (tdp1), but lower than its DBT
(td1) of entering air or equal to the WBT of entering air (tw1), then the air, is said to be cooled and
humidified. Since the chamber is insulated and same water is circulated again and again, therefore, a
condition of a adiabatic saturation is reached. Ultimately tf  = tw1. This process is shown by line 1-3 in
Fig. 20.15(b). This line follows the path along constant WBT line or constant enthalpy line.

M20_THERMAL ENGINEERING_SE_XXXX_CH20.indd 1054 6/16/2018 11:28:26 AM


Air-Conditioning and Psychrometrics 1055

Insulated
chamber

Spray
Air in nozzles Air out

tf = tw1

Pump
Make-up water
(a)

tw1 3″

3 w3
3′ 2 2″
2′ w2
w1
1 w

td3′
td3 = tw1 td2 td1
Dry bulb temperature
(b)

Figure 20.15  Cooling with adiabatic humidification

For perfect humidification, the final condition of air will be at point 3, where td3 = tw1. In actual
practice the humidification is not perfect, therefore the final condition of air at outlet will be repre-
sented by point 2 on line 1-3.
Effectiveness or humidifying efficiency of spray chamber, for tf = tw1, is given by

actual drop in DBT actual drop in w


hH = =
ideal drop in DBT ideal drop in w

td1 − td 2 w2 − w1
= = (20.36a)
td1 − td 3 w3 − w1

M20_THERMAL ENGINEERING_SE_XXXX_CH20.indd 1055 6/16/2018 11:28:28 AM


1056 Chapter 20

When tf < tw1, then the process follows the path 1-2′-3′. In that case
t −t w − w1
h H = d1 d 2′ = 2′ (20.36b)
td1 − td 3′ w3′ − w1

When tf > tw1, then the process follows the line 1-2′′-3′′. In that case
td1 − td 2′′ w2′′ − w1
hH = = (20.36c)
td1 − td 3′′ w3′′ − w1

20.8.8 Cooling and Humidification by Water Injection


(Evaporative Cooling)
Let liquid water at temperature tf be injected and sprayed into air stream with the help at nozzles. The
condition of air will change depending on the amount of water that evaporates. The enthalpy of evap-
oration will come from the enthalpy of air. The process is shown in Fig. 20.16.
Let m a = mass flow state of air
m v = mass flow rate of water evaporated
= mass flow rate of water supplied.
hf = enthalpy of liquid water
Then mass and enthalpy balances given
m v
w2 = w1 + (20.37)
m a

m v
h2 = h1 + h = h + ( w2 − w1 ) h f (20.38)
m a f 1
The term ( w2 − w1 ) h f is extremely small as compared to h1 and h2.
Hence h1 = h2. Thus the water injection process is a constant enthalpy process, irrespective of the
temperature of injected water.

tw
=
co
ns
t.
tw1 = tw2
2
w2

w1 w

td2 td1
td

Figure 20.16  Cooling and humidification by water injection

M20_THERMAL ENGINEERING_SE_XXXX_CH20.indd 1056 6/16/2018 11:28:34 AM


Air-Conditioning and Psychrometrics 1057

h2

2
w2

h1

1 w1 w

td1 td2
td

Figure 20.17  Heating and humidification by steam injection

20.8.9 Heating and Humidification by Steam Injection


Steam is normally injected into fresh outdoor air which is then supplied for the conditioning of textile
mills where high humidity needs to be maintained. The process is shown in Fig. 20.17. If ms is the
mass of steam supplied with enthalpy hs and ma the mass of air, then from the mass balance,
m
w2 = w1 + s (20.39)
ma
and for the enthalpy balance,
m
h2 = h1 + s hs = h1 + ( w2 − w1 ) hs (20.40)
ma

20.8.10 Heating and Adiabatic Chemical Dehumidification


This process is mainly used in industrial air-conditioning. It can also be used for some comfort air-con-
ditioning requiring low relative humidity or low dew point temperature in the room. In this process,
the air is passed over chemicals which have infinity for moisture. As the air comes in contact with
these chemicals, the moisture gets condensed out of air and gives up its latent heat. Due to condensa-
tion, the specific humidity decreases and the heat of condensation supplies sensible heat for heating
of air. As a result of this the DBT increases. The process is shown in Fig. 20.18 by line 1-2. The path
followed during the process is along the constant WBT line or constant enthalpy line.
The chemicals used are hygroscopic solutions or brines of calcium chloride, lithium chloride,
lithium premie and ethyl glycol. These are called absorbents which undergo a chemical or physical or
both changes during taking moisture. The other type of chemicals are called adsorbents which are in
the solid state to take up moisture but do not undergo changes chemically or physically. These include
silica gel and activated alumina. The effectiveness or efficiency of the dehumidifier,
t d 3 − t d1
hH =  (20.41)
t d 2 − t d1

M20_THERMAL ENGINEERING_SE_XXXX_CH20.indd 1057 6/16/2018 11:28:37 AM


1058 Chapter 20

Actual

W
adsorption

BT
dehumidification

=
co
ns
t
1 w1

3 w3
Adiabatic 2′
w2 w
dehumidification 2

td1 td3 td2


td

Figure 20.18  Heating and adiabatic chemical dehumidification

20.9 ❐ ADIABATIC MIXING OF TWO AIR STREAMS


Consider two air streams 1 and 2 mixing adiabatically, as shown in Fig. 20.19(a)
Let ma1, h1, w1 = mass, enthalpy and specific humidity of entering air at 1.
ma2, h2, w2 = corresponding values of entering air at 2.
ma3, h3, w3 = corresponding values of leaving air at 3.
The mixing process on psychrometic chart is shown in Fig. 20.19(b).
For the mass balance,
ma1 + ma2 = ma3
For the energy balance,
ma1h1 + ma2 h2 = ma3 h3
ma1h1 + ma 2 h2
h3 = (20.42)
ma3
For the mass balance of water vapour,
ma1w1 + ma2 w2 = ma3 w3
ma1w1 + ma 2 w2
w3 = (20.43)
ma3
Eq. (20.42) can be written as,
ma1 m
c p td 3 + h fg w3 = (c t + h w ) + a1 (c t + h fg w2 )
ma3 p d1 fg 1 ma3 p d 2
Simplifying, we get
ma1td1 + ma 2td 2 h fg  ma1 m 
td 3 = +  w1 + a 2 w2 − w3 
ma3 c p  ma3 ma3 

M20_THERMAL ENGINEERING_SE_XXXX_CH20.indd 1058 6/16/2018 11:28:40 AM


Air-Conditioning and Psychrometrics 1059

h2

h3
ma1, h1, w1 1 w1
ma3, h3, w3 h2
1 w3
3

w2 w
2
3
2 , w2
2
, h2
ma td2 td3 td2
td
(a) (b)

Figure 20.19 
(a) Adiabatic mixing of air streams,
(b) Mixing process on the psychrometric chart

The second term in the above expression being negligible, we get


m t + ma 2td 2
  td 3 = a1 d1 (20.44)
ma3

Example 20.5
The atmospheric air at 760 mm of Hg, DBT = 20°C and WBT = 10°C enters a heating coil whose
temperature is 40°C. Assuming bypass factor of heating coil as 0.5, determine for the air leaving
the coil: (a) DBT, (b) WBT, (c) relative humidity, and (d) sensible heat added to the air per kg of
dry air.
Solution
Given: pb = 760 mm Hg, td1 = 20°C, tw1 = 10°C, td3 = 40°C, BPF = 0.5
(a) Let td2 = DBT of air leaving the coil
The psychrometric process is shown in Fig. 20.20.
t −t
BPF = d 3 d 2
t d 3 − t d1
40 − td 2
0.5 =
40 − 20
td2 = 30°C
(b) and (c) from the psychrometric chart we find that tw2 = 14.4°C
and RH, f2 = 15%
(d) From the psychrometric chart,
h1 = 30 kJ/kg. d.a.
and     h2 = 40 kJ/kg. d.a.

M20_THERMAL ENGINEERING_SE_XXXX_CH20.indd 1059 6/16/2018 11:28:41 AM


1060 Chapter 20

h2

h1
f2

w
2
1 3

t w2
t w1

=
=

14
10

.4
°C

°C
td1 = 20°C td2 td3 = 40°C
td

Figure 20.20

Sensible heat added to air = h2 − h1


= 40 − 30
= 10 kJ/kg.d.a.

Example 20.6
Moist air enters a refrigeration coil at the rate of 120 kg d.a/min at 40°C and 50% RH. The appa-
ratus dew point of coil is 10°C and by-pass factor is 0.20. Determine the outlet state of moist air
and capacity of coil in TR.

Solution
Given: ma =120 kg d.a/min, td1 = 40°C, f1 = 50%, ADP = 10°C, BPF = 0.20
(a) Let td2 and f2 be the DBT and RH of air leaving the cooling coil respectively
The psychrometric process is shown in Fig. 20.21.

h1

h2 f 1 = 50%

tdp1
f2 1

A
P 2
AD w
3

10°C td2 td1 = 40°C


td

Figure 20.21

M20_THERMAL ENGINEERING_SE_XXXX_CH20.indd 1060 6/16/2018 11:28:42 AM


Air-Conditioning and Psychrometrics 1061

Locate point 1 (td1 = 40°C, f1 = 50%) on the psychrometric chart. Join point 1 with ADP = 10°C
point on the saturation line.
t −t t − ADP
Now BPF = d 2 d 3 = d 2
td1 − td 3 td1 − ADP
t − 10
0.2 = d 2

40 − 10
  td2 = 16°C
(b) Draw a vertical line at td2 = 16°C to intersect line 1-3 at point 2. Then f2 = 93%
(c) From the psychrometric chart, h1 = 101 kJ/kg d.a, and h2 = 43 kJ/kg d.a.
Cooling capacity of coil, Q = ma (h1 – h2)
1
= 120(101 − 43) × = 32.986 TR
211

Example 20.7
The atmospheric air at 30°C DBT and 70% RH enters a cooling coil at the rate of 220 m3/min. The
coil DPT = 15°C and its BPF = 0.15. Determine
(a) the temperature of air leaving the cooling coil,
(b) the capacity of the cooling coil in TR,
(c) the amount of water vapour removed per minute, and
(d) sensitive heat factor for the process.
Solution
Given: td1 =30°C, f1 = 70%, V1 = 220 m3/min (cmm). ADP = td3 = 15°C, BPF = 0.15
(a) Let td2 = temperature of air leaving the cooling coil
The psychromatric process is shown in Fig. 20.22.
At td1 = 30°C and f1 = 70%, tdp1 = 24°C from psychrometric chart.

h1
70%
hA

h2 tdp1 1
w1

w2
2 A w
P
AD
3

td3 = 15°C td2 td1 = 30°C


td

Figure 20.22

M20_THERMAL ENGINEERING_SE_XXXX_CH20.indd 1061 6/16/2018 11:21:00 AM


1062 Chapter 20

td 2 − td 3
BPF =
t d1 − t d 3
td 2 − 15
0.15 =
30 − 15
td2 = 17.25°C
From psychrometric chart, we have
w1 = 18.8 g/kg.d.a.
w2 = 11.8 g/kg.d.a.
v1 = 0.885 m3/kg
h1 = 78.5 kJ/kg.d.a.
h2 = 47.8 kJ/kg.d.a.
hA = 61.8 kJ/kg.d.a.
V 220
Mass of air flowing through cooling coil, m a = 1 = = 248.6 kg/min
v1 0.885
(b) Capacity of cooling coil = m a (h1 – h2) = 248.6(78.5 – 47.8) = 7632 kJ/min
7632
= = 36.17 TR
211
7632
= = 127.2 kW
60
(c) Amount of water vapour removed per minute,
m v = m a (w1 – w2)
= 248.6 (18.8 – 11.8) × 10−3 = 1.74 kg/min
hA − h2 61.8 − 47.8
(d) Sensible heat factor, SHF = = = 0.45.
h1 − h2 78.8 − 47.8

Example 20.8
In a certain environment, the DBT of air is 25°C and RH is 40%. Determine the specific humidity,
dew point and wet bulb temperature of air. This air is cooled in an air washer using recirculated
spray water and having a humidifying efficiency of 0.85. Find the DBT and DPT of air leaving the
air washer.

Solution
Given: td1 = 25°C, f1 = 40%, hH = 0.85
The psychrometric process is shown in Fig. 20.23
At point 1 (td1 = 25°C, f1 = 40%), w1 = 7.6 g/kg d.a., tdp1 = 10.4°C
and tw1 = 16°C = td3
Let td2 = DBT of air leaving the air washer
t −t 25 − td 2
h H = d1 d 2 = = 0.85
td1 − td 3 25 − 16
td2 = 17.35°C
Then      tdp2 = 15.2°C

M20_THERMAL ENGINEERING_SE_XXXX_CH20.indd 1062 6/16/2018 11:21:03 AM


Air-Conditioning and Psychrometrics 1063

tw1 3
40%
tdp2 2
tdp1 w1 w
1

td3 td2 td1


td

Figure 20.23

Example 20.9
The atmospheric air at 25°C DBT and 15°C WBT is flowing at the rate of 120 cmm through a duct.
The dry saturated steam at 100°C is injected into the air stream at the rate of 75 kg/h. Calculate the
specific humidity, enthalpy, DBT, WBT, and RH of leaving air.
Solution
Given: td1=25°C, tw1 =15°C, V1 =120 cmm, ts = 100°C, ms = 75 kg/h or 1.25 kg/min.
The psychrometric process is shown in Fig. 20.24
At state 1 (td1 = 25°C, tw1 = 15°C), w1 = 6.6 g/kg d.a., v1 = 0.853 m3/kg, h1 = 42.2 kJ/kg d.a.
V 120
Mass flow rate of air, m a = 1 = = 140.7 kg/min
v1 0.853
m s 1.25
w2 = w1 + = 6.6 × 10 −3 + = 0.0155 kg/kg d.a.
m a 140.7

h2

tw2 f2
2
w2
h1

tw1
w
1 w1

v1
td1 td2
td

Figure 20.24

M20_THERMAL ENGINEERING_SE_XXXX_CH20.indd 1063 6/16/2018 11:21:05 AM


1064 Chapter 20

Enthalpy of saturated steam at 100°C from steam tables, hs = 2676 kJ/kg


m s 1.25
h2 = h1 + × h = 42.2 + × 2676 = 65.97 kg/kg d.a.
m a s 140.7
Locate point 2 (w2 = 0.0155 kg/kg d.a., h2 = 65.97 kJ/kg) on the psychrometric chart
Then td2 = 260°C, WBT, tw2 = 22.4°C, f2 = 73%.

Example 20.10
Saturated air leaving the cooling section of an air-conditioning system at 15°C at the rate of 60 m3/
min is mixed adiabatically with outside air at 30°C and 60% RH at the rate of 20 m3/min. Find the
specific humidity, relative humidity, DBT and the volume flow rate of the mixture.
Solution
Given: td1 = 30°C, f1 = 60% V1 =20 m3/min, td2=15°C, V2 =60 m3/min
The psychrometric process for mixing is shown in Fig. 20.25.
Locate point 1 (td1=30°C, f1 = 60%) and point 2 (td2 = 15°C on the saturation curve)
From the psychrometric chart, we have
h1 = 71.6 kJ/kg.d.a., h2 = 42.1 kJ/kg.d.a.
v1 = 0.88 m3/kg.d.a., v2 = 0.83 m3/kg.d.a.
V 20
       m a1 = 1 = = 22.73 kg/min
v1 0.88
V2 60
       m a2 = = = 72.29 kg/min
v2 0.83
m a1 h3 − h2
Now        =
m a 2 h1 − h3

h1

h3
f1
h2
f3
1
w1
2 3
v1
w

v2

td2 = 15°C td3 td1 = 30°C


td

Figure 20.25

M20_THERMAL ENGINEERING_SE_XXXX_CH20.indd 1064 6/16/2018 11:21:07 AM


Air-Conditioning and Psychrometrics 1065

22.73 h3 − 42.1
   =
72.29 71.6 − h3
  0.3144(71.6 – h3) = h3 − 42.1
h3 = 49.16kg/kg.d.a.
w3 = 0.012 kg/kg.d.a., f3 = 88% and td3 = 18.8°C,
v3 = 0.843 m3/kg.d.a.
V = (m + m )v = (22.73 + 72.29) × 0.843 = 80.1 m3/min
3 a1 a2 3

20.10  ❐  THERMAL ANALYSIS OF HUMAN BODY


The rate at which body produces heat is called the metabolic rate. The heat produced by a normal
healthy person while sleeping is called the basal metabolic rate, which is of the order of 60 W. The
maximum value may be 10 times as much as this for a person engaged in sustained hard work. Human
comfort is influenced by physiological factors determined by the rate of heat generation within the
body and the rate of heat dissipation to the environment.
The body loses heat to the environment mainly by convection (Qconv), radiation (Qrad) and evap-
oration of moisture (Qevap). In addition, there is heat loss by respiration having sensible component
(QSr) and latent component (QLr) the total heat loss from the body is thus,
Q = Qconv + Qrad + Qevap + (QSr + QLr)
= (Qconv + Qrad + QSr) + (Qevap + QLr)
= QST + QLT
where QST = total sensible heat loss
QLT = total latent heat loss.
The total sensible heat component depends on the temperature difference between the surface of
the body and the surroundings. The latent heat component depends in the difference in the water
vapour pressures.
In summer, the temperature difference available for sensible heat transfer is less. Thus, the convec-
tive and radiative heat losses are reduced. To maintain thermal equilibrium, the body starts perspiring
to increase the evaporative loss. On the other hand in winter the sensible heat transfer is increased so
that the evaporative losses tend towards zero.
The heat exchange between human body and environment can be expressed by the following
energy balance equation:
QM − W = Q ± QS(20.45)
 where QM = metabolic heat produced within the body
  W = useful work done by human being
    QM − W = heat to be dissipated to environment
  Q = Qconv + Qrad + Qevap+ (QSr + QLr)
 Qs = heat stored in the body
1. The metabolic heat produced depends upon the rate of food energy consumption in the body.
2. The heat loss by evaporation is always positive. It depends upon the vapour pressure difference
between the skin surface and the surrounding air.
Qevap = Cd A ( ps − pv ) h fg Cc (20.46)

M20_THERMAL ENGINEERING_SE_XXXX_CH20.indd 1065 6/16/2018 11:21:08 AM


1066 Chapter 20

where Cd = diffusion coefficient, kg of water evaporated per unit surface area and pressure
difference per hour.
A = skin surface area = 1.8 m2 for a normal human being
ps = saturation vapour pressure corresponding to skin temperature
pv = vapour pressure of surrounding air
hfg = latent heat of vaporisation = 2450 kJ/kg
Cc = factor which accounts for clothing worn.
3. The heat loss or gain by radiation from the body to the surroundings depends upon the mean radi-
ant temperature. It is the average surface temperature of the surrounding objects. Qrad is positive
when the mean radiant temperature is lower than the DBT of room air, i.e. the human body will
undergo a radiant heat loss. When Qrad is negative, the body will undergo a radiant heat gain.
4. The heat loss by convection from the body to the surroundings is given by
Qconv = UA (tb − ts)(20.47)
where U = body film coefficient of heat transfer
A = body surface area = 1.8 m2 for normal human being
tb = body temperature
ts = surroundings temperature.
 The heat will be gained by body when ts > tb and Qconv is negative
5. QS is negative when Q > (QM − W), i.e. body temperature falls down. QS is positive when
Q < (QM − W), i.e. a human being gets fever.
A human body feels comfortable when there is no change in the body temperature, i.e. when
QS = 0. Any variation in the body temperature acts as a stress on the brain which results in either
perspiration or shivering.

20.10.1 Factors Affecting Human Comfort


The following factors affect human comfort:
1. Effective temperature.
2. Heat production and regulation in human body.
3. Heat and moisture losses from the human body.
4. Moisture content of air.
5. Quality and quantity of air.
6. Air velocity.
7. Hot and cold surfaces.
8. Air stratification.

20.10.2 Physiological Hazards Resulting from Heat


The physiological hazards resulting from rise in body temperature are:
1. Heat exhaustion: It is due to the failure of normal blood circulation. The symptoms of heat
exhaustion include fatigue, headache, dizziness, vomiting and abnormal mental reactions such as
irritation. Severe heat exhaustion may cause fainting. It does not cause permanent injury to the
body and recovery is usually rapid when the person is shifted to a cool place.

M20_THERMAL ENGINEERING_SE_XXXX_CH20.indd 1066 6/16/2018 11:21:08 AM


Air-Conditioning and Psychrometrics 1067

2. Heat Cramp: It results from loss of salt due to an excessive rate of body perspiration. It causes
severe pain in high muscles. This may be avoided by using salt tablets.
3. Heat stroke: It occurs when a person is exposed to excessive heat and work. If the body tempera-
ture rises rapidly to 40.5°C or 105°F, sweating ceases and a person may enter a coma, with immi-
nent death. It may lead to permanent damage to brain. Heat stroke can be a avoided by drinking
sufficient water at frequent intervals.

20.11  ❐  EFFECTIVE TEMPERATURE


Effective temperature (ET) is defined as the temperature of saturated air (RH = 100%) at which the
human beings would experience the same feeling of comfort as experienced in the actual unsaturated
environment. It may also be defined as that index which correlates the combined effects of air temper-
ature, relative humidity and air velocity on the human body.
Correspondingly, another effective temperature (ET*) can also be defined as the temperature at
50% RH at which the human body would experience exactly same feeling of comfort as at ET at 100%
RH, and as experienced in the actual environment.
The effective temperature is defined to evaluate the combined effect of DBT, RH, and air velocity.
The numerical value of effective temperature is made equal to the temperature of still (i.e. 5 to 8 m/
min air velocity) saturated air, which produces the same sensation of warmth or coolness as produced
under the given conditions.

20.11.1 Comfort Chart


The comfort chart is defined as that index which correlates the combined effects of air temperature,
relative humidity and air velocity on the human body. The numerical value of effective temperature is
made equal to the temperature of still (i.e. 5 to 8 m/min air velocity) saturated air, which produces the
same sensation of warmth or coolness as produced under the given conditions.
The comfort chart, shown in Fig. 20.26, represents the concept of effective temperature in the
comfort chart, DBT is taken as the abscissa and WBT as ordinates. The RH lines are replotted from
the psychrometric chart. The statistically prepared graphs corresponding to summer and winter season
are also superimposed. These graphs have effective temperature scale as abscissa and percentage of
people feeling comfortable as ordinate.
There is a correlation between comfort level, temperature, humidity sex, length of exposure, etc.
According to ASHRAE (American Society for Heating, Refrigeration, and Air conditioning Engi-
neering) thermal sensation scale for exposure for a period of one hour, the value of index y for comfort
for men, women and both sexes separately are expressed as follows
Men: y = 0.22td + 0.233pv − 5.673 (20.48a)
Women: y = 0.272 td + 0.248 pv – 7.245 (20.48b)
Both y = 0.245 td + 0.248 pv − 6.475 (20.48c)
where      td = DBT, °C
    pv = vapour pressure, kPa

M20_THERMAL ENGINEERING_SE_XXXX_CH20.indd 1067 6/16/2018 11:21:08 AM


1068 Chapter 20

The values for feeling of comfort of y are as follows:

Feeling of Hot Warm Slightly Neutral Slightly Cool Cold


comfort warm cool
Value of y +3 +2 +1 0 −1 −2 −3

The comfort chart shown in Fig. 20.26 represent the range for both summer and winter conditions
within which a condition of comfort exists for most people. The general practice is to recommend the
following optimum inside design conditions for comfort:

32
100% 90%
30 80%
30 70%
on

60%
as

28
se

28
er

50%
m
m

26
Su

26
es 40%
Lin
24 24 H.
R.
0
10

22 30%
90

22
80

Ef
e
bl
fe
70
Wet bulb temperature, °C

rta ct
60

20 iv
o
20%
f e
20 m pl lin e t
50

co peo es em
40

p
30

g
in of
el
20

18
18 fe %
10

Comfort 10%
16 zone
16 26

14 24
14
10
op ble

12 22 20
pe forta

12
le

30
% com

20 40
10 50
f
g
o

60
l in

70
fee

18
8 80
90
16 0
10
6
14
n
4 so
ea
12 rs
nte
Wi
10 12 14 16 18 20 22 24 26 28 30 32 34 36 38
Dry bulb temperature, °C

Figure 20.26  Comfort chart for still air (air velocities from 5 to 8 m/min)

M20_THERMAL ENGINEERING_SE_XXXX_CH20.indd 1068 6/16/2018 11:21:09 AM


Air-Conditioning and Psychrometrics 1069

Summer air-conditioning: Winter air-conditioning:


  ET = 22°C   ET = 19°C

DBT = 25° ± 1°C DBT = 21°C
 RH = 50 ± 5%   RH = 50%
Room air velocity = 0.4 m/s        Room air velocity = 0.15 – 0.20 m/s
The comfort chart does not take into account the variations in comfort conditions when there are wide
variations in the mean radiant temperature. The comfort chart shown in Fig. 20.27 has become obsolete
due to its shortcomings of over exaggeration of humidity at lower temperature and under estimation of
humidity at heat tolerance level. The modified comfort chart shown in Fig. 20.27 is commonly used now.
.036
Enthalpy Vapour
.034 pressure
mm of Hg
.032
30 WB .028 .036
Tem
am

p
.030
gr

.026 30
er
rp

.034
%
ou

.028
90

.024
t-h
at
W

.026 .032
0%
y,

.022
alp

=8

25
th

25
RH
En

.024 .020
70

.030
35
.022 .018
%

20
60

.020 .016 .028


Sp humidty

20
7
% 32.
.018 50 .014
.026
.4 15
25 scale .012
.016 15 p. 0%
.7 em 4
26 ive t .024
t .010
ec
Eff
.014 %
30 .022 10
.008
21.1
%
.012 18.3 20 .006
.020
15.5
.004 5
.010 hum idity
R elative 10% .018
.002

15 20 25 30 35 .016
Dry bulb temperature, °C
.014
.010 .012

Figure 20.27  Modified comfort chart

M20_THERMAL ENGINEERING_SE_XXXX_CH20.indd 1069 6/16/2018 11:21:10 AM


1070 Chapter 20

20.11.2 Factors Affecting Optimum Effective Temperature


The factors affecting optimum effective temperature are:
1. Climate and seasonal differences: The ET changes with season. During summer it is 22°C and
during winter 19°C.
2. Clothing: Persons with light clothings need lower optimum ET than a person with heavy clothings.
3. Age and sex: Women require higher ET (about 0.5°C) than men. Children also need higher ET
than adults.
4. Duration of stay: For shorter stay, higher ET is required than that needed for longer stay.
5. Kind of activity: Low ET is required for people doing heavy activity, like factory workers, danc-
ers, etc.
6. Density of occupants: Lower ET is needed when density of persons in a place is high.

20.12  ❐  SELECTION OF INSIDE AND OUTSIDE DESIGN


CONDITIONS
20.12.1 Selection of Inside Design Conditions
The inside design conditions depend on the particular air-conditioning system. The important air-
conditioning systems are:
1. Cold storage, 2. Industrial air-conditioning, 3. Comfort air conditioning

Table 20.1  Storage conditions and properties of food products


Product Tempera- Relative Approximate Water Highest Sp. heat Sp. heat
ture humid- storage life content freezing above below
°C ity % % point °C freezing freezing
kJ/kg.K kJ/kg.K
 1. Apples -1 to 0 85–90 84.1 -1.5 3.643 1.884
 2. Bananas 14.5 95 For ripening in 74.8 -0.8 3.35 1.76
8-10 days
 3. Butter 0 to 4.4 80–85 2 months 15.5–16.5 - 1.382 -
-18 to -23 80–85 1 year 15.5–16.5 - - -
 4. Milk 0.5 - 7 days 87 -0.6 3.77 1.93
pasteurized
 5. Eggs -1.5 to -0.5 80–85 6–9 months 66 2.2 3.06 1.68
 6. Fish-fresh 0.5 to 1.5 90–96 5–15 days 62–85 -2.2 2.93–3.6 -
frozen -23.5 to -18 90–95 8–10 months 62–85 - - 1.59–1.884
 7. Grapes - 0.5 85–90 3–8 weeks 81.9 -1.3 3.6 1.842
(American)
- - 3–6 months 81.6 -2.2 3.6 1.842
(European)
(Continued)

M20_THERMAL ENGINEERING_SE_XXXX_CH20.indd 1070 6/16/2018 11:21:10 AM


Air-Conditioning and Psychrometrics 1071

(Continued )

Product Tempera- Relative Approximate Water Highest Sp. heat Sp. heat
ture humid- storage life content freezing above below
°C ity % % point °C freezing freezing
kJ/kg.K kJ/kg.K
 8. Beef-fresh 0 to 1 88–92 1–6 weeks 62–77 - 2.93–3.52 -
frozen -23.5 to 90–95 9–12 months 62–77 - - 1.59–1.8
-18
 9. Mangoes 10 85–90 2–3 weeks 81.4 -0.9 3.56 1.842
10. Potatoes, 3 to 4.5 85–90 5–8 months 77.8 -0.6 3.483 1.8
late crop
11. Tomatoes− -
green 14 to 21 85–90 2–4 weeks 94.7 -0.6 3.98 2.01
ripe 7 to 10 85–90 2–7 days 94.1 -0.5 3.98 2.01

1. Cold Storage: In a cold storage air-conditioning system, the room air is cooled to much
lower temperature over a cooling coil and supplied back to the storage space. The condi-
tion-maintained inside the storage space depend on the nature of the product stored. In cold
storages, strict control of both temperature and relative humidity is required. The required
storage conditions for a number of products are given in Table 20.1.
2. Industrial Air-conditioning: The inside design conditions for industrial air-conditioning
depend on the category of application. One category comprises those where constancy of
temperature is the prime consideration, such as metrology laboratories, precision machine
tools, computer centres, etc. In these applications a variation in relative humidity of 10 to 20
percent will not have much effect. The other category may comprise paper and textile mills
where the relative humidity is to be maintained constant at a high value of 70 to 75 percent.
The temperature requirements of such spaces are not severe. In another category of applica-
tions, strict control of both temperature and relative humidity is required, such as chemical
and biological process industries.
3. Comfort air-conditioning: This has been dealt with in art 20.11.

20.12.2 Selection of Outside Design Conditions


For the outside design conditions in summer, it is recommended to use the mean monthly maximum
dry bulb temperature and its corresponding wet bulb temperature.
During winter it has been observed that the fuel consumption for heating of buildings varies also
directly as the difference between the outside temperature and a reasonably comfortable inside tem-
perature of 18.5°C. Thus the power consumption would be practically nil if the outside temperature
is 18.5°C. On the other hand, the power consumption would double if the outside temperature drops
from 18.5°C to 8.5°C. A degree-day is obtained for every degree when the mean outside temperature
is below 18.5°C during the 24 hour period. If in a given locality the outside temperature average of
30 days is 10°C, then the degree days (D) for the period would be
D = (18.5 − 10) × 30 = 255

M20_THERMAL ENGINEERING_SE_XXXX_CH20.indd 1071 6/16/2018 11:21:10 AM


1072 Chapter 20

The outside design temperature may be calculated as follows:


Number of days in a month × (18.5 − tdo) = D (20.49)
where tdo = outside design temperature.

20.13  ❐  COOLING LOAD ESTIMATION


The components of a cooling load are:
1. Sensible heat gain: This is the direct addition of heat to the space to be conditioned. It may occur
due to any one or all of the following sources of heat transfer:
   (a) The heat flowing into the building by conduction through exterior walls, floors, ceilings, doors
and windows due to the temperature difference on their two sides.
   (b)  The heat received from solar radiation. It consists of the following:
       (i)  The heat transmitted directly through glass of windows, ventilators, and doors.
       (ii) The heat absorbed by walls and roofs exposed to solar radiation and later on transferred to
the space to be conditioned by conduction.
   (c)  The heat conducted from adjoining un-conditioned rooms.
   (d)  The heat generated by fans, lights, machinery, cooking operations, industrial processes, etc.
   (e) Occupancy load.
   (f )  Air infiltration load through doors, windows, and their frequent opening.
   (g)  Heat gain from walls of ducts, etc.
2. Latent heat gain: The latent heat load occurs due to the presence of water vapour in the air of
conditioned space. This load occurs due to the following sources:
   (a)  Moisture in the air entering by infiltration
   (b)  Condensation of moisture from occupants, food cooking process.
   (c)  Moisture entering directly into the conditioned space through permeable walls, partitions, etc.
 Total cooling load = Sensible heat load + Latent heat load.

20.13.1 Heat Transfer Through Walls and Roofs


The heat transferred through a plane wall by conduction is given by Fourier law. For a single wall
shown in Fig. 20.28(a), we have
kA .∆t
Qcond = c
∆x
where      k = thermal conductivity of wall, W/(m. °C)
   Ac = area of wall cross-section, m2
∆t = temperature difference on two sides of wall, °C
∆x = wall thickness, m
The heat transfer by convection from a plane wall is given by Newton’s law
Qconv = hAs .∆t
where     h = heat transfer coefficient, W/(m2 °C)
    As = surface area of wall, m2
 ∆t = temperature difference between wall and surroundings, °C
Heat gained through a wall by the combined effect of conduction and convention becomes,

M20_THERMAL ENGINEERING_SE_XXXX_CH20.indd 1072 6/16/2018 11:21:12 AM


Air-Conditioning and Psychrometrics 1073

Air film
Air

t1 to 1 2 3
to to hi
t2 ti ti ti
k1 k2 k3 k1 k2 k3
ho ho
ho k ha
hi hi

x x1 x2 x3 x1 x2 x3
(a) (b) (c)

Figure 20.28 Heat transfer by conduction through a plane wall: (a) Single wall,
(b) Composite wall, (c) Composite wall with air space

k
Q = h0 (to − t1 ) Ac + (t1 − t2 ) Ac + hi (t2 − ti ) Ac = UAc (t0 − ti )(20.50)
x
where subscripts o, i refer to outside and inside wall conditions respectively
and       U = overall coefficient of heat transfer of wall
1
= for a single wall (Fig. 20.28a)
1 x 1
+ +
ho k hi
1
= for a composite wall (Fig. 20.28b)
3
1 x 1
+∑ m+
ho m = 1 km hi
1
= for a composite wall with air gap (Fig. 20.228c)
3
1 1 x 1
+ +∑ m+
ho ka m = 1 km hi
where ka = thermal conductance of air space

20.13.2 Heat Gain from Solar Radiation


The heat from solar radiation is received by building surfaces by direct radiation and diffuse radiation.
The direct radiation is the impingement of the Sun’s rays upon the surface. The diffuse radiation is
received from moisture and dust particles in atmosphere which absorbs part of the energy of the Sun’s
rays. The diffuse radiation is received by surfaces which do not face the sun.
The heat gain through outside walls and roofs is given by,
Q = UA.∆te (20.51)
where ∆te = equivalent temperature differential.

20.13.3 Sol Air Temperature


It is a hypothetical temperature used to calculate the heat received by the outside surface of a building
wall by the combined effect of convection and radiation.

M20_THERMAL ENGINEERING_SE_XXXX_CH20.indd 1073 6/16/2018 11:21:14 AM


1074 Chapter 20

Total heat received by the outside surface of the wall per unit area.
qos = qconv + qrad = ho (to − tos ) + I a
 Ia 
= hoto − hotos + I a = ho  to + ht
 ho  o os
       = hote − hotos = ho (te − tos ) (20.52)
Ia
where     te = to + = sol air temperature.
ho
t o = temperature of outside air
tos = temperature of outside surface of wall
 ho = outside film coefficient
               I = total radiation intensity
a = absorptivity of the surface

20.13.4 Solar Heat Gain Through Glass Areas


Heat gain through glass area = absorbed solar radiation + convection and radiation heat exchanges
between glass and outdoor surfaces (20.53)

20.13.5   Heat Gain Due to Infiltration


There are two methods of estimating the infiltrated air
(i) Crack length method, and (ii) Air change method.
The crack length method is usually used where greater accuracy is required. In most cases the air
change method is used for calculating the quantity of infiltrated air. According to this method, the
quantity of infiltrated air through windows and walls is,
1  LWH × Ac  3
V =   m / min (20.54)
2 60 
where L, W, H = length, width and height of room respectively, m
  Ac = air changes per hour
1
Factor is used because infiltration takes place on the windward side of building only.
2

20.13.6 Heat Gain from Products


This heat gain is very important in case of cold storages.
1. Chilling load above freezing,
mc pm (T1 − T2 )
Qch = (20.55)
tch
where m = mass of product
   cpm = mean specific heat of product
   T1, T2 = initial and final temperature of product respectively
    tch = chilling time.

M20_THERMAL ENGINEERING_SE_XXXX_CH20.indd 1074 6/16/2018 11:21:17 AM


Air-Conditioning and Psychrometrics 1075

m × h fg
2. Freezing load Qf = (20.56)
tf
where h fg = latent heat of freezing
  t f = freezing time
3. Cooling load below freezing,
mc′ pm (T1′ − T2′ )
Qc = (20.57)
tc
where c′ pm = mean specific heat of freezing product
   T1′ T2′ = actual storage and freezing temperatures of product
 tc = cooling time
4. Product respiration heat, Qr = m × rate of heat evolution of food  (20.58)

20.13.7 Heat Gain from Lights


Heat gain from lights, Qt = total wattage of lights × use factor × allowance factor
where use factor =1.0 for residences and commercial stores  (20.59)
                = 0.5 for industrial workshops
Allowance factor = 1.25 for flourescent tubes.

20.13.8 Heat Gain from Power Equipments


kW rating
Heat gain from electric motor, Qem = × Load factor(20.60)
hem
where  hem = efficiency of electric motor.

20.13.9 Heat Gain Through Ducts


Heat gain through the duct, QD = UAD (ta – ts)(20.61)
where U = overall heat transfer coefficient
AD = surface area of duct
      ta, ts = temperatures of ambient and supply airs respectively

20.13.10 E
 mpirical Methods to Evaluate Heat Transfer
Through Walls and Roofs
There are two approaches to calculate empirically heat transfer through walls and roofs.
They are:
1. The decrement factor and time lag method.
2. The equivalent temperature differential method.

M20_THERMAL ENGINEERING_SE_XXXX_CH20.indd 1075 6/16/2018 11:21:19 AM


1076 Chapter 20

Both the methods use analytical-experimental results for their formulations. The equivalent tempera-
ture differential method is more commonly used by the air-conditioning engineers as it is also appli-
cable to sunlit walls and roofs.
1. Using Decrement Factor and Time Lag: If the thermal capacity of the wall is ignored, then the
instantaneous rate of heat transfer through the wall at any time t is given by
Qt = UA(te – ti)
where te = sol-air temperature.
and on an average basis, the mean heat flow is given by
Qm = UA(tem – ti)
where tem = mean sol-air temperature.
For the sake of simplicity, the dot above Q has been dropped to indicate the rate.
But most building materials have a finite thermal capacity which is expressed as

mc = rcV = rc ( A∆x ) 

where m = Mass of wall, V = volume of wall


    r, c = Density and specific heat of wall material
  A = Cross-sectional area of wall
      ∆x = Wall thickness.
It has been seen that there is a two-fold effect of thermal capacity on heat transfer
(a) There is a time lag between the heat transfer at the outside surface qo and the heat transfer
at the inside surface qi, defined by f.
(b) There is a decrement in the heat transfer due to the absorption of heat by the wall and sub-
sequent transfer of a part of this heat back to the outside air when its temperature is lower,
defined by l.
The rigorous analytical method to determine the time lag f and decrement factor l is quite com-
plicated. In the limiting case, when the wall thickness approaches zero l → 1 and f → 0
Then Qt = UA(te – ti)
i.e. the heat transfer through the wall is equal to its instantaneous value.
Considering the effect of thermal capacity, the actual heat transfer at any time t is,
Qt = UA (te − ti ) + UAl (te − te ) 
m t −f m

where tet − f = sol-air temperature at time t − f . i.e., f hours before the heat transfer is to be
calculated.
   In a locality where the daily range of variation of the outside air temperature is small it is
immaterial what thickness of wall is provided. But in a locality when the daily range of temper-
ature is large, it is desirable to have thick walls so as to cut the cooling load in summer and the
heating load in winter. Such walls also do not allow the inside temperature to rise very much
during the day and drop at night.
2. Equivalent temperature differential (ETD) method: The actual heat transfer can be written in
terms of an equivalent temperature differential ∆t E defined by the equation.
Q = UA∆t E 
 where ∆t E = (tem − ti ) + l (tet −f − tem )

M20_THERMAL ENGINEERING_SE_XXXX_CH20.indd 1076 6/16/2018 11:21:22 AM


Air-Conditioning and Psychrometrics 1077

∆t E depends on the following:


(a) l and f, which in turn depend on the thermo-physical properties of construction.
(b) Outside air temperature t0 and solar radiation intensity I.
(c) Room temperature ti.
Thus, the equivalent temperature differential approach takes care of the exposure of the wall or
roof to the sun.

20.14  ❐  HEATING LOAD ESTIMATION


An estimate of the heating load is made on the basis of maximum probable heat loss of the conditioned
heated space. The heating loads are:
1. Transmission heat loss: The transmission heat loss from walls roofs, etc., is given by,
   Q = UA(ti – to)(20.62)
  where ti, to = inside and outside design temperature respectively
2. Solar radiation: The solar radiation heat gain is neglected as peak heating load occurs in the
early hours of morning.
3. Internal heat gains: The internal heat gains from occupants, lights, appliances, etc., decrease
the heating load. These negative loads should be accounted for in applications, such as theatres,
assembly halls, stores, office buildings, etc.,

20.15  ❐  ROOM SENSIBLE HEAT FACTOR (RSHF)


It is defined as the ratio of room sensible heat to the total heat (Fig. 20.29).

RSH RSH
RSHF = = (20.63)
RTH RSH + RLH

where RSH, RLH, RTH = room sensible, latent and total heats respectively.

H
RT H
RL Room design
RSHF R conditions
H
RS line

S
Supply air w
ADP
condition

td1 td2
DBT

Figure 20.29  Room and supply air conditions and ADP

M20_THERMAL ENGINEERING_SE_XXXX_CH20.indd 1077 6/16/2018 11:21:24 AM


1078 Chapter 20

Calculated
RSHF

f = 50%
Alignment a
Baseline
circle
b

Sensible heat
factors scale
RSHF line R
R′

Room w
design
conditions
td = 26°C
DBT

Figure 20.30  Grand sensible heat factor

20.15.1 Estimation of Supply Air Conditions


When the supply air conditions are not known, then the RSHF line may be drawn from the RSHF
calculated value, as described below (Fig. 20.30).
1. Mark point ‘a’ on the sensible heat factor scale drawn on the right hand corner of psychrometric
chart. The point ‘a’ represents the calculated value of RSHF.
2. Joint point ‘a’ with the alignment circle or reference point b (td − 26°C, f = 50%). The line ab is
called the base line.
3. Mark point R on the psychrometric chart to represent the room design conditions.
4. Draw a line RR’ through point R parallel to the base line ab. This line is required RSHF line.

20.16  ❐  GRAND SENSIBLE HEAT FACTOR


It is defined as the ratio of total sensible heat to the grand total heat which the cooling coil or condi-
tioning apparatus is required to handle (Fig. 20.31).
TSH TSH RSH + OASH
GSHF = = = (20.64)
GTH TSH + TLH ( RSH + ORSH ) + ( RLH + OALH )
where  TSH = total sensible heat = RSH + OASH
TLH = total latent heat = RLH + OALH
GTH = grand total heat = TSH + TLH = (RSH + OASH) + (RLH + OALH)
Let V1 = volume rare of outside air m3/min
  td1, td2 = DBT of outside and room air respectively, °C
  w1, w2 = specific humidity of outside and room air respectively kg/kg.d.a
   h1, h2 = enthalpy of outside and room air respectively, kJ/kg d.a
Then outside air sensible heat, OASH = 0.0204 V1 (td1 − td2) kW (20.65)
Outside air latent heat, OALH = 50 V1 (w1 − w2)kW(20.66)
Outside air total heat,  OATH = OASH + OALH
   = 0.02 V1 (h1 − h2)kW(20.67)

M20_THERMAL ENGINEERING_SE_XXXX_CH20.indd 1078 6/16/2018 11:21:26 AM


Air-Conditioning and Psychrometrics 1079

Outdoor
1 condition
Mixture condition
line 3 entering
SF

Specific humidity
GH cooling coil
5
4 2
ADP
6 Room design
condition
Supply air
from cooling coil

DBT

Figure 20.31  RSHF line estimation

20.17  ❐  EFFECTIVE ROOM SENSIBLE HEAT FACTOR


It is defined as the ratio of the effective room sensible heat to the effective room total heat
ERSH ERSH
ERSHF = = (20.68)
ERTH ERSH + ERLH
where ERSH = Effective room sensible heat
= RSH + OASH × BPF
= RSH + 0.0204 V1 (td1 − td2) × BPT

Saturation
curve

Outdoor
PF)
condition
-B 1
(1
Room supply Mixture
air condition condition w
PF
SHF scale

B entering
3
cooling coil a
F line b
4 GSH
5 RSHF line
2 Alignment
ADP H F line
4′ ER S circle
6 Room
design
condition
BPF (1 - BPF)
BPF (1 - BPF)
Calculated
RSHF
DBT

Figure 20.32  Effective room sensible heat factor

M20_THERMAL ENGINEERING_SE_XXXX_CH20.indd 1079 6/16/2018 11:21:28 AM


1080 Chapter 20

  ERLH = Effective room latent heat


       = RLH + OALH × BPF
       = RLH + 50 V1 (w1 − w2) × BPF
  ERTH = Effective room total heat
       = ERSH + ERLH
  BPF = By-pass factor
The line joining the point 2 and point 6, i.e. ADP as shown in Fig. 20.32, gives the effective room
sensible heat factor line (ERSHF line). From point 4, draw line 4-4′ parallel to line 3-2. From similar
triangles 6-4-4′ and 6-3-2, we have

length 4 − 6 length 4′ − 6
BPF = = (20.69a)
length 3 − 6 length 2 − 6
td 4 − ADP td 4′ − ADP
Also BPF = = (20.69b)
td 3 − ADP td 2 − ADP

20.18  ❐  AIR CONDITIONING SYSTEMS


20.18.1 S
 ummer Air-conditioning System with Ventilation
Air and Zero By-pass Factor
The schematic arrangement of summer air-conditioning system with ventilation without by-pass fac-
tor is shown in Fig. 20.33. The outside air flows through the damper and mixes with the recirculated
air. The mixed air then passes through the filter to remove dirt and dust and then through cooling coil
whose temperature is kept much below the DBT of air in the conditioned space. The cooled air then
passes through a perforated membrane where it losses its moisture in the condensed form. The mois-
ture is collected in a sump. After that, the air is made to pass through a heating coil which heats the air
slightly as per the requirement of designed air DBT and RH. Now the conditioned air is supplied to the

.
mao Room Qs
td1, tw1
Q1
.
mai Recirculated
i air Perforated .
membrane s mas
Supply
air fan
.
mao Outside
air 1
o 2
Damper

Filter
Cooling Heating
Sump coil
coil

Figure 20.33  Schematic diagram of system with ventilation air without bypass factor

M20_THERMAL ENGINEERING_SE_XXXX_CH20.indd 1080 6/16/2018 11:21:29 AM


Air-Conditioning and Psychrometrics 1081

ho

Saturation 0
h1 wo
curve
hi

h2
F line w1
GSH 1
v1 w
ADP wi
s 2 RSHF line i w2
φ1

tdp2 tdi td1 tdo


DBT

Figure 20.34  Summer air conditioning process with ventilation air without BPF
room by a supply air fan. From the room a part of the used air is exhausted to the atmosphere by ven-
tilation. The remaining part of the used air (recirculated air) is mixed with fresh air for conditioning.
Figure 20.34 shows the air-conditioning processes.
    o, i = outside and inside air states respectively
   1 = state of air after mixing recirculated room air with ventilation air
m ai , m a0 = mass flow rate of recirculated room air and ventilation air respectively
 2 = supply air state for a minimum rate of supply air
line i-2 = ADP, tdp2
      = RSHF line
           line 1-2 = GSHF line
    m as = mass flow rate of mixture air
     =  ( − ) + ( 
Q mas hi h2 mai hi + m ao ho )
= ( m − m )h + m h − m h
as ao i ao o as 2

     = m as ( hi − h2 ) + m ao ( ho − hi ) (20.70)
      = RTH + OATH
   (a)  Ventilation load
 Let  (cmm)o = outside ventilation air volume flow rate = m as × vo
Then    OASH = 0.0204 (cmm)o (tdo − tdi)(20.71)
OALH            = 50 (cmm)o (wo − wi)(20.72)
and     OATH = OASH + OALH = 0.02 (cmm)o (ho – hi)(20.73)
   (b)  Room load
Let        (cmm)1 = mixture air volume flow rate
   m a1 = m ai + m ao = m as

M20_THERMAL ENGINEERING_SE_XXXX_CH20.indd 1081 6/16/2018 11:21:33 AM


1082 Chapter 20

(cmm)1
m a1 =
Now     (20.74)
v1
RTH = RSH + RLH
   (c) Air-conditioning equipment load
TSH = RSH + OASH
TLH = RLH + OALH
GTH = TSH + TLH
TSH TSH
Grand sensible heat factor,       GSHF = = (20.75)
TSH + TLH GTH

20.18.2 S
 ummer Air-conditioning System with Ventilation Air
and By-pass Factor
The summer air-conditioning processes with ventilation air and BPF is shown Fig. 20.35.

td 2 − (tdp )coil w2 − wS ’ h2 − hS ’
BPF = = =  (20.76)
td1 − (tdp )coil w1 − wS ’ h1 − hS ’

1 w1
F line
GSH
S s
w2
S′ 2 RSHF line i
w

BPF
(1-BPF)

(tdp)coil (tdp)room td2 td1


DBT

Figure 20.35  Summer air-conditioning processes with ventilation air and BPF

20.18.3 Winter Air-conditioning System


The schematic arrangement of winter air-conditioning system is shown in Fig. 20.36. The outside air
flows through a damper and mixes up with the recirculated air. The mixed air passes through a filter
to remove dirt, dust and other impurities. The air now passes through a preheating coil to prevent the

M20_THERMAL ENGINEERING_SE_XXXX_CH20.indd 1082 6/16/2018 11:21:36 AM


Air-Conditioning and Psychrometrics 1083

Recirculated air
Room

i
Preheater s
Supply
Damper air fan
2
Outside
air
o 1

Reheat
Filter
coil
Humidifier

Figure 20.36  Winter air-conditioning system

possible freezing of water and to control the evaporation of water in the humidifier. Then the air is
passed through a reheating coil to bring the air to the designed DBT. This conditioned air is supplied
to the room by means of a supply an fan. From the room, a part of the used air is exhausted to the
atmosphere and the remaining part is recirculated again.
The air-conditioning processes are shown in Fig. 20.37.
Process 1-2: Preheating
Process 2-3: Adiabatic saturation.
Process 1-3: Heating and humidifying air in air washer with pumped recirculation and external
heating of water.
Process 3-s: Reheating

Adiabatic
saturation
Reheating
3 s
Air washer process e
lin
with external
i RS HF
heating of water
ω
2
Saturation curve 1 Preheating

td

Figure 20.37  Winter air-conditioning processes

M20_THERMAL ENGINEERING_SE_XXXX_CH20.indd 1083 6/16/2018 11:21:39 AM


1084 Chapter 20

Example 20.11
Air at 10°C DBT and 90% RH is to be brought to 35°C DBT and 22.5°C WBT with the help of
winter air-conditioner. If the humidified air comes out of the humidifier at 90% RH, determine:
(a) the temperature to which the air should be preheated, and (b) the efficiency of the air washer.

Solution
Given: td1 =10°C, f1 = 90%, tds = 35°C, tws = 22.5°C
The psychrometric processes are shown in Fig. 20.38
Locate point l(tdl, f1) and point s(tds, tws).
Draw horizontal lines from points 1 and s. From point 3 draw WBT = const. line to locate point 2.
From psychrometric chart, the temperature to which air should be preheated
td2 = 31.2°C
td3 = 18.5°C and td3′ = 17.5°C
td 2 − td 3 31.2 − 18.5
Efficiency of air-washer = = = 0.927 or 92.7%
td 2 − td 3′ 31.2 − 17.5

W
BT
= f = 90%

22
3′ .5
°C
s
3
WB

w
T=
Co

1
ns
t

td1 = 10°C td3′ td3 td2 tds = 35°C


DBT

Figure 20.38

20.18.4 Comfort Air-conditioning System


In comfort air-conditioning, the air is brought to the required DBT and RH for the human health, com-
fort and efficiency. The comfort conditions have been discussed in detail in art. 20.11.1. Normally it
is assumed to be 21°C DBT and 50% RH. The SHF is generally kept between 0.7 to 0.9. The comfort
air-conditioning may be adopted for homes, offices, shops, restaurants, theatres, hospitals, schools, etc.

20.18.5 Industrial Air-conditioning System


In an industry the inside DBT and RH of air have to be kept constant for proper working of the
machines and manufacturing processes. Special type of air-conditioning systems have to be provided

M20_THERMAL ENGINEERING_SE_XXXX_CH20.indd 1084 6/16/2018 11:21:45 AM


Air-Conditioning and Psychrometrics 1085

for textile mills, paper mills, machine-parts manufacturing plants, tool rooms, CNC machining cen-
tres, photo-processing plants, etc.

Example 20.12
Atmospheric air at a pressure of 1-0132 bar has a dry bulb temperature of 30°C and Relative
Humidity of 70%. Calculate:
(a) partial pressure of Water Vapour and air in moist air,
(b) the specific humidity,
(c) the degree of saturation, and
(d) the Dew Point temperature.

Solution
Given: pb = 1.0132 bar, td = 30°C, f = 70%
(a) Corresponding to td = 30°C, saturation pressure of water vapour, from steam tables, is
ps = 0.04246 bar,
p
f = v , pv = 0.7 × 0.04246 = 0.02972 bar
ps
Now      pb = pa + pv
Partial pressure of air, pa = 1.0132 – 0.02972 = 0.98348 bar
0.622 pv 0.622 pv 0.622 × 0.02972
(b) Specific humidity, w = = = = 0.01879 kg/kgdd.a
pb − pv pa 0.98348
(c) Specific humidity of started air:
0.622 pv 0.622 × 0.04246
ws = = = 0.02721 kg/kgd.aa
pb − pv 1.0132 − 0.04246
w 0.01879
Degree of saturation, m = = = 0.6906 or 69.05%
ws 0.02721
(d) DPT = saturation temperature corresponding to pv = 0.02972 bar
 From steam tables, corresponding to pv = 0.02972 bar
0.02972 × 105
or = 22.92 mm Hg,
133.32
 24.46 − 23.72 
DPT,             tdp = 23.72 +  ( 22.29 − 22)
 23 − 22 
= 23.72 + 0.21 = 23.93°C

Example 20.13
In a laboratory test on a particular day, a psychrometer recorded the dry bulb and wet bulb temper-
atures for atmospheric air as 35°C and 25°C respectively. The atmospheric pressure is 1.013 bar 1.
Calculate:
(a) the relative humidity,
(b) the specific humidity,

M20_THERMAL ENGINEERING_SE_XXXX_CH20.indd 1085 6/16/2018 11:21:56 AM


1086 Chapter 20

(c) the dew point temperature, and


(d) the degree of saturation.
The partial pressure of water vapour (pv) can be calculated with the help of Carrier’s Equation,
given as;
 ( p − pwb ) (tdp − twb ) 
pv =  pwb − 
 1547 − 1.44 twb 
where pwb = Saturation pressure corresponding to wet bulb temperature,
p = Barometric pressure, and
tdb and twb = are the dry bulb and wet bulb temperatures of air (in 0°C).
Gas constant of air, Ra = 0.287 kJ/kg.K.

Solution
Given: td = 35°C, tw = 25°C, pb = 1.013 bar,
(a) Corresponding to td = 35°C, ps = 0.5622 bar, and for tw = 25°C, pw = 0.03166 bar
( p − pw ) (td − tw )
Partial pressure of water vapour, pv = pw − b
1547 − 1.44 tw

= 0.03166 −
(1.013 − 0.03166) (35 − 25)
1547 − 1.44 × 25
pv 0.2516
Relative humidity, f= = = 0.4476 or 44.76%
ps 0.05622
0.622 pv 0.622 × 0.02516
(b) Specific humidity, ws = = = 0.0158 kg/kgd.a
pb − pv 1.013 − 0.02516
(c) Corresponding to pv = 0.02516 bar,
 22 − 21 
DPT, tdp = 21 +  + ( 0.02516 − 0.02486 ) = 21 + 0.191°C = 21.191°C
 0.02643 − 0.02486 
0.622 ps
(d) Specific humidity of saturated, ws =
pb − ps
0.622 × 0.05622
= = 0.03655 kg/kgd.a
1.013 − 0.05622
w 0.01584
Degree of saturation, m= = = 0.4333 or 43.33%
ws 0.03655

Review Questions
1. Explain the following terms:
(a) Degree of saturation (b) Specific humidity
(c) Relative humidity (d) Dew point temperature
(e) Thermodynamic wet bulb temperature

M20_THERMAL ENGINEERING_SE_XXXX_CH20.indd 1086 6/16/2018 11:22:02 AM


Air-Conditioning and Psychrometrics 1087

2. Represent the following processes on psychrometric chart:


(a) Sensible heating and cooling
(b) Adiabatic heating and dehumidification
(c) Adiabatic cooling and humidification
(d) Cooling and dehumidification
3. What is the difference between WBT and thermodynamic WBT?
4. Define SHF and LHF.
5. What is by-pass factor?
6. What are the factors that determine human comfort?
7. What is effective temperature? List the factors that affect effective temperature.
8. Differentiate between heat stroke and heat camp.
9. What is the difference between summer and winter air conditioning?
10. What is alignment circle?

Exercises
20.1. An air conditioned space is to be supplied with 300 cmm of air at 20°C dry bulb and 40% relative humidity.
Unconditioned air from atmosphere at 32°C dry bulb and 28°C dew point is supplied to the dehumidifier
coils, from where the same passes on to a heating coil before entering the conditioned space. Determine the
cooling required. Solve the problem both from first principles and with Psychrometric chart.
20.2. Room air at 26°C dry bulb and 50% relative humidity is mixed with outdoor air at 43°C dry bulb and 40%
relative humidity in the ratio of 4 : 1. The mixture is passed through a cooling coil maintained at 5°C with a
by-pass factor of 0.15. The air from the cooling coil is mixed with room air in the ratio of 4 : 1. The mixture
is then reheated to 20°C dry bulb and supplied to the conditioned space.
(a) Show the flow diagram schematically.
(b) Show the different process on a Psychrometric chart.
(c) For 450 kg of supply air per minute determine the quantity of fresh air needed, the refrigeration load
and the heat supplied to the reheater coils.
20.3. Given: Summer Design = 43°C DBT, 24°C WBT
Inside Design = 25°C DBT, 50% RH
Room Sensible Heat = 300000 kJ/hr
Room Latent Heat = Nil
Ventilation = 30 cmm
Determine:
(a) effective sensible heat factor
(b) dehumidified air-quantity
(c) supply air temperature
Assume by-pass factor = 0.05.
20.4. In an Industrial Application:
Summer Design = 40°C DBT; 24°C WBT
Inside Room Design = 60% RH
Room Sensible Heat = 200000 kJ/hr (55.556 kW)
Room Sensible Heat Factor = 0.9.
Use all outdoor air and evaporative cooling process with washer efficiency 80%. Determine
(a) Room DBT maintained
(b) Supply quantity in cmm.

M20_THERMAL ENGINEERING_SE_XXXX_CH20.indd 1087 6/16/2018 11:22:02 AM


1088 Chapter 20

   20.5.  The following data refer to a laboratory


 Summer Design    = 40°C DBT, 24°C WBT
 Inside Design  = 25°C DBT, 50% RH
Room Sensible heat = 60000 kJ/kg
Room Latent Heat   = 12000 kJ/hr
 Ventilation     = 40 cmm.
  Also all out side fresh air to be supplied through coil, and no return air. Determine
(a)  effective sensible heat factor
(b) ADP
(c)  actual quantity of outdoor air to be supplied to take care of the loads
(d)  dehumidified air quantity supplied.
Assume by-pass factor = 0.05.
20.6.  Air 35°C dry bulb temperature and 25°C wet bulb temperature is passed through a cooling coil at the rate
of 280 m3/min. The air leaves the cooling coil at 26.5°C dry bulb temperature and 50% relative humidity.
Find
(a)  Capacity of the cooling coil in tonnes of refrigeration;
(b)  Wet bulb temperature of the leaving air;
(c)  Water vapour removed per minute: and
(d)  Sensible heat factor. [Ans. 31.98 TR; 19.2°C; 1.56 kg/kg of dry air:0.39]
20.7.  The outside air at 30°C dry bulb temperature and 18°C wet bulb temperature enters a cooling coil at the
rate of 40 m3/min. The effective surface temperature of the cooling coil is 4.5°C and its cooling capacity is
12.5 kW of refrigeration. Find:
(a)  dry bulb and wet bulb temperatures of the air leaving the coil,
(b)  enthalpy of air leaving the coil, and
(c)  by-pass factor of the coil. [Ans. 18.8°C; 12.7°C; 35.6 kJ/kg of dry air; 0.52]
20.8.  Air at 40°C dry bulb temperature and 15% relative humidity is passed through the adiabatic humidifier
at the rate 200 m3/min. The outlet conditions of air are 25°C dry bulb temperature and 20°C wet bulb
temperature. Find (a) dew point temperature, (b) relative humidity of exit air; and (c) amount of water
vapour added to the air per minute.  [Ans. 17.8°C; 65%; 1 26 kg/min]
20.9.  Air at 0°C and 95% relative humidity has to be heated and humidified to 25°C and 40% relative humidity
by the following three processes:
      (i)  preheating
     (ii)  adiabatic saturation in a recirculated water air washer and
  (iii)  reheating to final state.
Calculate:
(a)  The heating required in two heaters
(b)  The makeup water required in washer and temperature of washer
Assume effectiveness of washer as 80 per cent.
 [Ans. 27.3 kJ/kg of dry air; 9 kJ/kg of dry air; 4.1 g/kg of dry air 12.6°C]
20.10.  The following data relates to a cold storage for storing 450 tonnes of vegetables
Inside design conditions 19°C DBT 60% RH
Outside design conditions 36°C DBT 28°C WBT
Infiltrated air 180 m3/hr
Fresh air supply 4500 m3/hr
No. of persons working in cold store 20
Sensible heat gain through glass, walls, ceiling etc., 16.3 kW
Water content of vegetables 74%

M20_THERMAL ENGINEERING_SE_XXXX_CH20.indd 1088 6/16/2018 11:22:02 AM


Air-Conditioning and Psychrometrics 1089

Loss of water content 0.01% per hour


Heat from equipment and reaction heat of vegetables 3.1 kW
If the air conditioning is achieved by first cooling and dehumidifying and then heating and the tempera-
ture of air entering the room is not to exceed 16°C, determine
(a)  amount of recirculated air, if the recirculated air is mixed with fresh air before entering the cooling coil.
(b)  capacity of the cooling coil in tonnes of refrigeration and its by-pass factor if the dew point temper-
ature of the coil is 6°C and
(c)  capacity of heating coil [Ans. 3.384 kg/s; 46.75 TR; 34 82 kW]
20.11.  The following data relates to a conference room having a seating capacity of 80 persons.
Inside design conditions 22°C DBT 55% RH
Outside design conditions 38°C DBT 28°C WBT
Sensible and latent heat loads per person 75W and 45W respectively
Lights and fan loads 12000 W
Sensible heat gain through glass, walls, ceiling etc. 12000 W
Air infiltration 18 m3/min
Fresh air supply 80 m3/min
By pass factors of the coils 0.1
If two third of recirculated room air and one third of fresh air are mixed before entering the cooling coils
determine
(a)  apparatus Dew point
(b)  grand total heat load
(c)  effective room sensible heat factor.
20.12.  A sling psychrometer reads 40°C DBT and 28°C WBT. Calculate the following:
(a)  Specific humidity
(b)  Relative humidity
(c)  Dew point, temperature
(d)  vapour density
(e)  Enthalpy of mixture per kg of dry air.
Assume atmospheric pressure = 1.03 bar
[Ans. 0.0189 kg/kg d.a., 40.4%, 24°C, 0.0338 kg/m3, 88.8 k/kg d.a]
 20.13. The atmospheric conditions are 25°C DBT and specific humidity of 10 g/kg of d.a. Determine the partial
pressure of vapour and relative humidity.
Take atmospheric pressure = 1 bar. [Ans. 0.0158 bar 49%]
 20.14. Air enters an evaporative cooler at a pressure of 1 atm. DBT of 38°C and 25% RH. The air leaves the
cooler at 1 atm and 18°C DBT. Assuming adiabatic process, determine the relative humidity of cool air
and quality of water that must be supplied to cooler for each kg of dry air.
[Ans. 63%, 0.00808 kg/kg. d.a]

M20_THERMAL ENGINEERING_SE_XXXX_CH20.indd 1089 6/16/2018 11:22:02 AM


Thispageisintentionallyleftblank
Appendix A

Z01_THERMAL ENGINEERING_SE_XXXX_Z01.indd 1091


A.1 ❐ Steam Tables*

Table A.1.1  Saturates steam: Temperature table

Specific volume, m3/kg Internal energy, kJ/kg Enthalpy, kJ/kg Entropy, kJ/kg K
Temp. Pressure Sat. Sat. Sat. Sat. Sat. Sat. Sat. Sat.
°C kPa, MPa liquid vapour liquid Evap. vapour liquid Evap. vapour liquid Evap. vapour
T p vf vg uf ufg ug hf hfg hg sf sfg sg
0.01 0.6113 0.001000 206.132 0.00 2375.3 2375.3 0.00 2501.3 2501.3 0.0000 9.1562 9.1562
5 0.8721 0.001000 147.118 20.97 2361.3 2382.2 20.98 2489.6 2510.5 0.0761 8.9496 9.0257
10 1.2276 0.001000 106.377 41.99 2347.2 2389.2 41.99 2477.7 2519.7 0.1510 8.7498 8.9007
15 1.7051 0.001001 77.925 62.98 2333.1 2396.0 62.98 2465.9 2528.9 0.2245 8.5569 8.7813
20 2.3385 0.001002 57.790 83.94 2319.0 2402.9 83.94 2454.1 2538.1 0.2966 8.3706 8.6671
25 3.1691 0.001003 43.359 104.86 2304.9 2409.8 104.87 2442.3 2547.2 0.3673 8.1905 8.5579
30 4.2461 0.001004 32.893 125.77 2290.8 2416.6 125.77 2430.5 2556.2 0.4369 8.0164 8.4533
35 5.6280 0.001006 25.216 146.65 2276.7 2423.4 146.66 2418.6 2565.3 0.5052 7.8478 8.3530
40 7.3837 0.001008 19.523 167.53 2262.6 2430.1 167.54 2406.7 2574.3 0.5724 7.6845 8.2569
45 9.5934 0.001010 15.258 188.41 2248.4 2436.8 188.42 2394.8 2583.2 0.6386 7.5261 8.1647
(Continued)

6/16/2018 12:55:59 PM
Table A.1.1  (Continued ) 1092

Specific volume, m3/kg Internal energy, kJ/kg Enthalpy, kJ/kg Entropy, kJ/kg K
Temp. Pressure Sat. Sat. Sat. Sat. Sat. Sat. Sat. Sat.
°C kPa, MPa liquid vapour liquid Evap. vapour liquid Evap. vapour liquid Evap. vapour
T p vf vg uf ufg ug hf hfg hg sf sfg sg
50 12.350 0.001012 12.032 209.30 2234.2 2443.5 209.31 2382.7 2592.1 0.7037 7.3725 8.0762
55 15.758 0.001015 9.568 230.19 2219.9 2450.1 230.20 2370.7 2600.9 0.7679 7.2234 7.9912
60 19.941 0.001017 7.671 251.09 2205.5 2456.6 251.11 2358.5 2609.6 0.8311 7.0784 7.9095

Z01_THERMAL ENGINEERING_SE_XXXX_Z01.indd 1092


65 25.033 0.001020 6.197 272.00 2191.1 2463.1 272.03 2346.2 2618.2 0.8934 6.9375 7.8309
70 31.188 0.001023 5.042 292.93 2176.6 2469.5 292.96 2333.8 2626.8 0.9548 6.8004 7.7552
75 38.578 0.001026 4.131 313.87 2162.0 2475.9 313.91 2321.4 2635.3 1.0154 6.6670 7.6824
80 47.390 0.001029 3.407 334.84 2147.4 2482.2 334.88 2308.8 2643.7 1.0752 6.5369 7.6121
85 57.834 0.001032 2.828 355.82 2132.6 2488.4 355.88 2296.0 2651.9 1.1342 6.4102 7.5444
90 70.139 0.001036 2.361 376.82 2117.7 2494.5 376.90 2283.2 2660.1 1.1924 6.2866 7.4790
95 84.554 0.001040 1.982 397.86 2102.7 2500.6 397.94 2270.2 2668.1 1.2500 6.1659 7.4158
Appendix A

100 0.10135 0.001044 1.6729 418.91 2087.6 2506.5 419.02 2257.0 2676.0 1.3068 6.0480 7.3548
105 0.12082 0.001047 1.4194 440.00 2072.3 2512.3 440.13 2243.7 2683.8 1.3629 5.9328 7.2958
110 0.14328 0.001052 1.2102 461.12 2057.0 2518.1 461.27 2230.2 2691.5 1.4184 5.8202 7.2386
115 0.16906 0.001056 1.0366 482.28 2041.4 2523.7 482.46 2216.5 2699.0 1.4733 5.7100 7.1832
120 0.19853 0.001060 0.8919 503.48 2025.8 2529.2 503.69 2202.6 2706.3 1.5275 5.6020 7.1295
125 0.2321 0.001065 0.77059 524.72 2009.9 2534.6 524.96 2188.5 2713.5 1.5812 5.4962 7.0774
130 0.2701 0.001070 0.66850 546.00 1993.9 2539.9 546.29 2174.2 2720.5 1.6343 5.3925 7.0269
135 0.3130 0.001075 0.58217 567.34 1977.7 2545.0 567.67 2159.6 2727.3 1.6869 5.2907 6.9777
140 0.3613 0.001080 0.50885 588.72 1961.3 2550.0 589.11 2144.8 2733.9 1.7390 5.1908 6.9298
145 0.4154 0.001085 0.44632 610.16 1944.7 2554.9 610.61 2129.6 2740.3 1.7906 5.0926 6.8832
150 0.4759 0.001090 0.39278 631.66 1927.9 2559.5 632.18 2114.3 2746.4 1.8417 4.9960 6.8378
155 0.5431 0.001096 0.34676 653.23 1910.8 2564.0 653.82 2098.6 2752.4 1.8924 4.9010 6.7934
(Continued)

6/16/2018 12:55:59 PM
Table A.1.1  (Continued )

Specific volume, m3/kg Internal energy, kJ/kg Enthalpy, kJ/kg Entropy, kJ/kg K
Temp. Pressure Sat. Sat. Sat. Sat. Sat. Sat. Sat. Sat.
°C kPa, MPa liquid vapour liquid Evap. vapour liquid Evap. vapour liquid Evap. vapour
T p vf vg uf ufg ug hf hfg hg sf sfg sg
160 0.6178 0.001102 0.30706 674.85 1893.5 2568.4 675.53 2082.6 2758.1 1.9426 4.8075 6.7501
165 0.7005 0.001108 0.27269 696.55 1876.0 2572.5 697.32 2066.2 2763.5 1.9924 4.7153 6.7078
170 0.7917 0.001114 0.24283 718.31 1858.1 2576.5 719.20 2049.5 2768.7 2.0418 4.6244 6.6663

Z01_THERMAL ENGINEERING_SE_XXXX_Z01.indd 1093


175 0.8920 0.001121 0.21680 740.16 1840.0 2580.2 741.16 2032.4 2773.6 2.0909 4.5347 6.6256
180 1.0022 0.001127 0.19405 762.08 1821.6 2583.7 763.21 2015.0 2778.2 2.1395 4.4461 6.5857
185 1.1227 0.001134 0.17409 784.08 1802.9 2587.0 785.36 1997.1 2782.4 2.1878 4.3586 6.5464
190 1.2544 0.001141 0.15654 806.17 1783.8 2590.0 807.61 1978.8 2786.4 2.2358 4.2720 6.5078
195 1.3978 0.001149 0.14105 828.36 1764.4 2592.8 829.96 1960.0 2790.0 2.2835 4.1863 6.4697
200 1.5538 0.001156 0.12736 850.64 1744.7 2595.3 852.43 1940.7 2793.2 2.3308 4.1014 6.4322
205 1.7230 0.001164 0.11521 873.02 1724.5 2597.5 875.03 1921.0 2796.0 2.3779 4.0172 6.3951
210 1.9063 0.001173 0.10441 895.51 1703.9 2599.4 897.75 1900.7 2798.5 2.4247 3.9337 6.3584
Appendix A

215 2.1042 0.001181 0.09479 918.12 1682.9 2601.1 920.61 1879.9 2800.5 2.4713 3.8507 6.3221
220 2.3178 0.001190 0.08619 940.85 1661.5 2602.3 943.61 1858.5 2802.1 2.5177 3.7683 6.2860
225 2.5477 0.001199 0.7849 963.72 1639.6 2603.3 966.77 1836.5 2803.3 2.5639 3.6863 6.2502
230 2.7949 0.001209 0.07158 986.72 1617.2 2603.9 990.10 1813.8 2803.9 2.6099 3.6047 6.2146
235 3.0601 0.001219 0.06536 1009.88 1594.2 2604.1 1013.61 1790.5 2804.1 2.6557 3.5233 6.1791
240 3.3442 0.001229 0.05976 1033.19 1570.8 2603.9 1037.31 1766.5 2803.8 2.7015 3.4422 6.1436
245 3.6482 0.001240 0.05470 1056.69 1546.7 2603.4 1061.21 1741.7 2802.9 2.7471 3.3612 6.1083
250 3.9730 0.001251 0.05013 1080.37 1522.0 2602.4 1085.34 1716.2 2801.5 2.7927 3.2802 6.0729
255 4.3195 0.001263 0.04598 1104.26 1496.7 2600.9 1109.72 1689.8 2799.5 2.8382 3.1992 6.0374
260 4.6886 0.001276 0.04220 1128.37 1470.6 2599.0 1134.35 1662.5 2796.9 2.8837 3.1181 6.0018
265 5.0813 0.001289 0.03877 1152.72 1443.9 2596.6 1159.27 1634.3 2793.6 2.9293 3.0368 5.9661
1093

(Continued)

6/16/2018 12:56:00 PM
Table A.1.1  (Continued ) 1094

Specific volume, m3/kg Internal energy, kJ/kg Enthalpy, kJ/kg Entropy, kJ/kg K
Temp. Pressure Sat. Sat. Sat. Sat. Sat. Sat. Sat. Sat.
°C kPa, MPa liquid vapour liquid Evap. vapour liquid Evap. vapour liquid Evap. vapour
T p vf vg uf ufg ug hf hfg hg sf sfg sg
270 5.4987 0.001302 0.03564 1177.33 1416.3 2593.7 1184.49 1605.2 2789.7 2.9750 2.9551 5.9301
275 5.9418 0.001317 0.03279 1202.23 1387.9 2590.2 1210.05 1574.9 2785.0 3.0208 2.8730 5.8937
280 6.4117 0.001332 0.03017 1227.43 1358.7 2586.1 1235.97 1543.6 2779.5 3.0667 2.7903 5.8570

Z01_THERMAL ENGINEERING_SE_XXXX_Z01.indd 1094


285 6.9094 0.001348 0.02777 1252.98 1328.4 2581.4 1262.29 1511.0 2773.3 3.1129 2.7069 5.8198
290 7.4360 0.001366 0.02557 1278.89 1297.1 2576.0 1289.04 1477.1 2766.1 3.1593 2.6227 5.7821
295 7.9928 0.001384 0.02354 1305.21 1264.7 2569.9 1316.27 1441.8 2758.0 3.2061 2.5375 5.7436
300 8.5810 0.001404 0.02167 1331.97 1231.0 2563.0 1344.01 1404.9 2748.9 3.2533 2.4511 5.7044
305 9.2018 0.001425 0.01995 1359.22 1195.9 2555.2 1372.33 1366.4 2738.7 3.3009 2.3633 5.6642
310 9.8566 0.001447 0.01835 1387.03 1159.4 2546.4 1401.29 1326.0 2727.3 3.3492 2.2737 5.6229
315 10.547 0.001472 0.01687 1415.44 1121.1 2536.6 1430.97 1283.5 2714.4 3.3981 2.1821 5.5803
Appendix A

320 11.274 0.001499 0.01549 1444.55 1080.9 2525.5 1461.45 1238.6 2700.1 3.4479 2.0882 5.5361
330 12.845 0.001561 0.012996 1505.24 993.7 2498.9 1525.29 1140.6 2665.8 3.5506 1.8909 5.4416
340 14.586 0.001638 0.010797 1570.26 894.3 2464.5 1594.15 1027.9 2622.0 3.6593 1.6763 5.3356
350 16.514 0.001740 0.008813 1641.81 776.6 2418.4 1670.54 893.4 2563.9 3.7776 1.4336 5.2111
360 18.651 0.001892 0.006945 1725.19 626.3 2351.5 1760.48 720.5 2481.0 3.9146 1.1379 5.0525
370 21.028 0.002213 0.004926 1843.84 384.7 2228.5 1890.37 441.8 2332.1 4.1104 0.6868 4.7972
374.14 22.089 0.003155 0.003155 2029.58 0 2029.6 2099.26 0 2099.3 4.4297 0 4.4297
*Adapted from Joseph H. Keenan, Frederick G.Keyes, Philip G. Hill, and Joan G. Moore, Steam Tables, John Wiley and Sons, New York, 1969.

6/16/2018 12:56:00 PM
Table A.1.2  Saturated water: Pressure table

Specific volume, m3/kg Internal energy, kJ/kg Entropy, kJ/kg Entropy, kJ/kg
Pres- Temp. Sat. Sat. Sat. Sat. Sat. Sat. Sat. Sat.
sure °C liquid vapour liquid Evap. vapour liquid Evap. vapour liquid Evap. vapour
MPa T vf vg uf ufg ug hf hfg hg sf sfg sg
p
0.6113 0.01 0.001000 206.132 0 2375.3 2375.3 0.00 2501.3 2501.3 0 9.1562 9.1562
1.0 6.98 0.001000 129.208 29.29 2355.7 2385.0 29.29 2484.9 2514.2 0.1059 8.8697 8.9756
1.5 13.03 0.001001 87.980 54.70 2338.6 2393.3 54.70 2470.6 2525.3 0.1956 8.6322 8.8278

Z01_THERMAL ENGINEERING_SE_XXXX_Z01.indd 1095


2.0 17.50 0.001001 67.004 73.47 2326.0 2399.5 73.47 2460.0 2533.5 0.2607 8.4629 8.7236
2.5 21.08 0.001002 54.254 88.47 2315.9 2404.4 88.47 2451.6 2540.0 0.3120 8.3311 8.6431
3.0 24.08 0.001003 45.665 101.03 2307.5 2408.5 101.03 2444.5 2545.5 0.3545 8.2231 8.5775
4.0 28.96 0.001004 34.800 121.44 2293.7 2415.2 121.44 2432.9 2554.4 0.4226 8.0520 8.4746
5.0 32.88 0.001005 28.193 137.79 2282.7 2420.5 137.79 2423.7 2561.4 0.4763 7.9187 8.3950
7.5 40.29 0.001008 19.238 168.76 2261.7 2430.5 168.77 2406.0 2574.8 0.5763 7.6751 8.2514
10.0 45.81 0.001010 14.674 191.79 2246.1 2437.9 191.81 2392.8 2584.6 0.6492 7.5010 8.1501
15.0 53.97 0.001014 10.022 225.90 2222.8 2448.7 225.91 2373.1 2599.1 0.7548 7.2536 8.0084
20.0 60.06 0.001017 7.649 251.35 2205.4 2456.7 251.38 2358.3 2609.7 0.8319 7.0766 7.9085
25.0 64.97 0.001020 6.204 271.88 2191.2 2463.1 271.90 2346.3 2618.2 0.8930 6.9383 7.8313
30.0 69.10 0.001022 5.229 289.18 2179.2 2468.4 289.21 2336.1 2626.3 0.9439 6.8247 7.7686
40.0 75.87 0.001026 3.993 317.51 2159.5 2477.0 317.55 2319.2 2636.7 1.0258 6.6441 7.6700
50.0 81.33 0.001030 3.240 340.42 2143.4 2483.8 340.47 2305.4 2645.9 1.0910 6.5029 7.5939
75.0 91.77 0.001037 2.217 384.29 2112.4 2496.7 384.36 2278.6 2663.0 1.2129 6.2434 7.4563
    Appendix A 

MPa
0.100 99.62 0.001043 1.6940 417.33 2088.7 2506.1 417.44 2258.0 2675.5 1.3025 6.0568 7.3593
0.125 105.99 0.001048 1.3749 444.16 2069.3 2513.5 444.30 2241.1 2685.3 1.3739 5.9104 7.2843
0.150 111.37 0.001053 1.1593 466.92 2052.7 2519.6 467.08 2226.5 2693.5 1.4335 5.7897 7.2232
(Continued)
1095

6/16/2018 12:56:00 PM
Table A.1.2  (Continued ) 1096

Specific volume, m3/kg Internal energy, kJ/kg Entropy, kJ/kg Entropy, kJ/kg
Pres- Temp. Sat. Sat. Sat. Sat. Sat. Sat. Sat. Sat.
sure °C liquid vapour liquid Evap. vapour liquid Evap. vapour liquid Evap. vapour
MPa T vf vg uf ufg ug hf hfg hg sf sfg sg
p
0.175 116.06 0.001057 1.0036 486.78 2038.1 2524.9 486.97 2213.6 2700.5 1.4848 5.6868 7.1717
0.200 120.23 0.001061 0.8857 504.47 2025.0 2529.5 504.68 2202.0 2706.6 1.5300 5.5970 7.1271
0.225 124.00 0.001064 0.7933 520.45 2013.1 2533.6 520.69 2191.3 2712.0 1.5705 5.5173 7.0878

Z01_THERMAL ENGINEERING_SE_XXXX_Z01.indd 1096


0.250 127.43 0.001067 0.7187 535.08 2002.1 2537.2 535.34 2181.5 2716.9 1.6072 5.4455 7.0526
0.275 130.60 0.001070 0.6573 548.57 1992.0 2540.5 548.87 2172.4 2721.3 1.6407 5.3801 7.0208
0.300 133.55 0.001073 0.6058 561.13 1982.4 2543.6 561.45 2163.9 2725.3 1.6717 5.3201 6.9918
0.350 138.88 0.001079 0.5243 583.93 1965.0 2548.9 584.31 2148.1 2732.4 1.7274 5.2130 6.9404
0.375 141.32 0.001081 0.4914 594.38 1956.9 2255.13 594.79 2140.8 2735.6 1.7527 5.1647 6.9174
0.40 143.63 0.001084 0.4625 604.29 1949.3 2553.6 604.73 2133.8 2738.5 1.7766 5.1193 6.8958
Appendix A

0.45 147.93 0.001088 0.4140 622.75 1934.9 2557.6 623.24 2120.7 2743.9 1.8206 5.0359 6.8565
0.50 151.86 0.001093 0.3749 639.66 1921.6 2561.2 640.21 2108.5 2748.7 1.8606 4.9606 6.8212
0.55 155.48 0.001097 0.3427 655.30 1909.2 2564.5 655.91 2097.0 2752.9 1.8972 4.8920 6.7892
0.60 158.85 0.001101 0.3157 669.88 1897.5 2567.4 670.54 2086.3 2756.8 1.9311 4.8289 6.7600
0.65 162.01 0.001104 0.2927 683.55 1886.5 2570.1 684.26 2076.0 2760.3 1.9627 4.7704 6.7330
0.70 164.97 0.001108 0.2729 696.43 1876.1 2572.5 697.20 2066.3 2763.5 1.9922 4.7158 6.7080
0.75 167.77 0.001111 0.2556 708.62 1866.1 2574.7 709.45 2057.0 2766.4 2.0199 4.6647 6.6846
0.80 170.43 0.001115 0.2404 720.20 1856.6 2576.8 721.10 2048.0 2769.1 2.0461 4.6166 6.6627
0.85 172.96 0.001118 0.2270 731.25 1847.4 2578.7 732.20 2039.4 2771.6 2.0709 4.5711 6.6421
0.90 175.38 0.001121 0.2150 741.81 1838.7 2580.5 742.82 2031.1 2773.9 2.0946 4.5280 6.6225
0.95 177.69 0.001124 0.2042 751.94 1830.2 2582.1 753.00 2023.1 2776.1 2.1171 4.4869 6.6040
1.00 179.91 0.001127 0.19444 761.67 1822.0 2583.6 762.79 2015.3 2778.1 2.1386 4.4478 6.5864
1.10 184.09 0.001133 0.17753 780.08 1806.3 2586.4 781.32 2000.4 2781.7 2.1791 4.3744 6.5535
(Continued)

6/16/2018 12:56:00 PM
Table A.1.2  (Continued )

Specific volume, m3/kg Internal energy, kJ/kg Entropy, kJ/kg Entropy, kJ/kg
Pres- Temp. Sat. Sat. Sat. Sat. Sat. Sat. Sat. Sat.
sure °C liquid vapour liquid Evap. vapour liquid Evap. vapour liquid Evap. vapour
MPa T vf vg uf ufg ug hf hfg hg sf sfg sg
p
1.20 187.99 0.001139 0.16333 797.27 1791.6 2588.8 798.64 1986.2 2784.8 2.2165 4.3067 6.5233
1.30 191.64 0.001144 0.15125 813.42 1777.5 2590.9 814.91 1972.7 2787.6 2.2514 4.2438 6.4953
1.40 195.07 0.001149 0.14084 828.68 1764.1 2592.8 830.29 1959.7 2790.0 2.2842 4.1850 6.4692

Z01_THERMAL ENGINEERING_SE_XXXX_Z01.indd 1097


1.50 198.32 0.001154 0.13177 843.14 1751.3 2594.5 844.87 1947.3 2792.1 2.3150 4.2198 6.4448
1.75 205.76 0.001166 0.11349 876.44 1721.4 2597.8 878.48 1918.0 2796.4 2.3851 4.0044 6.3895
2.00 212.42 0.001177 0.09963 906.42 1693.8 2600.3 908.77 1890.7 2799.5 2.4473 3.8935 6.3408
2.25 218.45 0.001187 0.08875 933.81 1668.2 2602.0 936.48 1865.2 2801.7 2.5034 3.7938 6.2971
2.50 223.99 0.001197 0.07998 959.09 1644.0 2603.1 962.09 1841.0 2803.1 2.5546 3.7028 6.2574
2.75 229.12 0.001207 0.07275 982.65 1621.2 2603.8 985.97 1817.9 2803.9 2.6018 3.6190 6.2208
3.00 233.90 0.001216 0.06668 1004.76 1599.3 2604.1 1008.41 1795.7 2804.1 2.6456 3.5412 6.1869
3.25 238.38 0.001226 0.06152 1025.62 1578.4 2604.0 1029.60 1774.4 2804.0 2.6866 3.4685 6.1551
Appendix A

3.50 242.60 0.001235 0.05707 1045.41 1558.3 2603.7 1049.73 1753.7 2803.4 2.7252 3.4000 6.1252
4.0 250.40 0.001252 0.049778 1082.28 1520.0 2602.3 1087.29 1714.1 2801.4 2.7963 3.2737 6.0700
5.0 263.99 0.001286 0.039441 1147.78 1449.3 2597.1 1154.21 1640.1 2794.3 2.9201 3.0532 5.9733
6.0 275.64 0.001319 0.032440 1205.41 1384.3 2589.7 1213.32 1571.0 2784.3 3.0266 2.8625 5.8891
7.0 285.88 0.001351 0.027370 1257.51 1323.0 2580.5 1266.97 1505.1 2772.1 3.1210 2.6922 5.8132
8.0 295.06 0.001384 0.023518 1305.54 1264.3 2569.8 1316.61 1441.3 2757.9 3.2067 2.5365 5.7431
9.0 303.40 0.001418 0.020484 1350.47 1207.3 2557.8 1363.23 1378.9 2742.1 3.2857 2.3915 5.6771
10.0 311.06 0.001452 0.018026 1393.00 1151.4 2544.4 1407.53 1317.1 2724.7 3.3595 2.2545 5.6140
11.0 318.15 0.001489 0.015987 1433.68 1096.1 2529.7 1450.05 1255.5 2705.6 3.4294 2.1233 5.5527
12.0 324.75 0.001527 0.014263 1472.92 1040.8 2513.7 1491.24 1193.6 2684.8 3.4961 1.9962 5.4923
13.0 330.93 0.001567 0.012780 1511.09 985.0 2496.1 1531.46 1130.8 2662.2 3.5604 1.8718 5.4323
1097

(Continued)

6/16/2018 12:56:00 PM
Table A.1.2  (Continued ) 1098

Specific volume, m3/kg Internal energy, kJ/kg Entropy, kJ/kg Entropy, kJ/kg
Pres- Temp. Sat. Sat. Sat. Sat. Sat. Sat. Sat. Sat.
sure °C liquid vapour liquid Evap. vapour liquid Evap. vapour liquid Evap. vapour
MPa T vf vg uf ufg ug hf hfg hg sf sfg sg
p
14.0 336.75 0.001611 0.011485 1548.53 928.2 2476.8 1571.08 1066.5 2637.5 3.6231 1.7485 5.3716
15.0 342.24 0.001658 0.010338 1585.58 869.8 2455.4 1610.45 1000.0 2610.5 3.6847 1.6250 5.3097
16.0 347.43 0.001711 0.009306 1622.63 809.1 2431.7 1650.00 930.6 2580.6 3.4760 1.4995 5.2454

Z01_THERMAL ENGINEERING_SE_XXXX_Z01.indd 1098


17.0 352.37 0.001770 0.008365 1660.16 744.8 2405.0 1690.25 856.9 2547.2 3.8078 1.3698 5.1776
18.0 357.06 0.001840 0.007490 1698.86 675.4 2374.3 1731.97 777.1 2509.1 3.8713 1.2330 5.1044
19.0 361.54 0.001924 0.006657 1739.87 598.2 2338.1 1776.43 688.1 2464.5 3.9387 1.0841 5.0227
20.0 365.81 0.002035 0.005834 1785.47 507.6 2293.1 1826.18 583.6 2409.7 4.0137 0.9132 4.9269
21.0 369.89 0.002206 0.004953 1841.97 388.7 2230.7 1888.30 446.4 2334.7 4.1073 0.6942 4.8015
22.0 373.80 0.002808 0.003526 1973.16 108.2 2081.4 2034.92 124.0 2159.0 4.3307 0.1917 4.5224
22.09 374.14 0.003155 0.003155 2029.58 0 2029.6 2099.26 0 2099.3 4.4297 0 4.4297
Appendix A

Table A.1.3  Superheated vapour

p = 10 kPa (45.81) p = 50 kPa (81.33) p = 100 kPa (99.62)


T v u h s v u h s v u h s
Sat. 14.674 2437.9 2584.6 8.1501 3.240 2483.8 2645.9 7.5939 1.6940 2506.1 2675.5 7.3593
50 14.869 2443.9 2592.6 8.1749 – – – – – – – –
100 17.196 2515.5 2687.5 8.4479 3.418 2511.6 2682.5 7.6947 1.6958 2506.6 2676.2 7.3614
150 19.513 2587.9 2783.0 8.6881 3.889 2585.6 2780.1 7.9400 1.9364 2582.7 2776.4 7.6133
200 21.825 2661.3 2879.5 8.9037 4.356 2659.8 2877.6 8.1579 2.1723 2658.0 2875.3 7.8342
250 24.136 2736.0 2977.3 9.1002 4.821 2735.0 2976.0 8.3555 2.4060 2733.7 2974.3 8.0332
300 26.445 2812.1 3076.5 9.2812 5.284 2811.3 3075.5 8.5372 2.6388 2810.4 3074.3 8.2157
400 31.063 2968.9 3279.5 9.6076 6.209 2968.4 3278.9 8.8641 3.1026 2967.8 3278.1 8.5434
(Continued)

6/16/2018 12:56:01 PM
Table A.1.3  (Continued )

p = 10 kPa (45.81) p = 50 kPa (81.33) p = 100 kPa (99.62)


T v u h s v u h s v u h s
500 35.679 3132.3 3489.0 9.8977 7.134 3131.9 3488.6 9.1545 3.5655 3131.5 3488.1 8.8341
600 40.295 3302.5 3705.4 10.1608 8.058 3302.2 3705.1 9.4177 4.0278 3301.9 3704.7 9.0975
700 44.911 3479.6 3928.7 10.4028 8.981 3479.5 3928.5 9.6599 4.4899 3479.2 3928.2 9.3398
800 49.526 3663.8 4159.1 10.6281 9.904 3663.7 4158.9 9.8852 4.9517 3663.5 4158.7 9.5652
900 54.141 3855.0 4396.4 10.8395 10.828 3854.9 4396.3 10.0967 5.4135 3854.8 4396.1 9.7767

Z01_THERMAL ENGINEERING_SE_XXXX_Z01.indd 1099


1000 58.757 4053.0 4640.6 11.0392 11.751 4052.9 4640.5 10.2964 5.8753 4052.8 4640.3 9.9764
1100 63.372 4257.5 4891.2 11.2287 12.674 4257.4 4891.1 10.4858 6.3370 4257.3 4890.9 10.1658
1200 67.987 4467.9 5147.8 11.4090 13.597 4467.8 5147.7 10.6662 6.7986 4467.7 5147.6 10.3462
1300 72.603 4683.7 4409.7 11.5810 14.521 4683.6 5409.6 10.8382 7.2603 4683.5 5409.5 10.5182
p = 200 kPa (120.23) p = 300 kPa (133.55) p = 400 kPa (143.65)
Sat. 0.88573 2529.5 2706.6 7.1271 0.60582 2543.6 2725.3 6.9918 0.46246 2553.6 2738.5 6.8958
150 0.95964 2576.9 2768.8 7.2795 0.63388 2570.8 2761.0 7.0778 0.47084 2564.5 2752.8 6.9299
200 1.08034 2654.4 2870.5 7.5066 0.71629 2650.7 2865.5 7.3115 0.53422 2646.8 2860.5 7.1706
Appendix A

250 1.19880 2731.2 2971.0 7.7085 0.79636 2728.7 2967.6 7.5165 0.59512 2726.1 2964.2 7.3788
300 1.31616 2808.6 3071.8 7.8926 0.87529 2806.7 3069.3 7.7022 0.65484 2804.8 3066.7 7.5661
400 1.54930 2966.7 3276.5 8.2217 1.03151 2965.5 3275.0 8.0329 0.77262 2964.4 3273.4 7.8984
500 1.78139 3130.7 3487.0 8.5132 1.18669 3130.0 3486.0 8.3250 0.88934 3129.2 3484.9 8.1912
600 2.01297 3301.4 3704.0 8.7769 1.34136 3300.8 3703.2 8.5892 1.00555 3300.2 3702.4 8.4557
700 2.24426 3478.8 3927.7 9.0194 1.49573 3478.4 3927.1 8.8319 1.12147 3477.9 3926.5 8.6987
800 2.47539 3663.2 4158.3 9.2450 1.64994 3662.9 4157.8 9.0575 1.23722 3662.5 4157.4 8.9244
900 2.70643 3854.5 4395.8 9.4565 1.80406 3854.2 4395.4 9.2691 1.35288 3853.9 4395.1 9.1361
1000 2.93740 4052.5 4640.0 9.6563 1.95812 4052.3 4639.7 9.4689 1.46847 4052.0 4639.4 9.3360
1100 3.16834 4257.0 4890.7 9.8458 2.11214 4256.8 4890.4 9.6585 1.58404 4256.5 4890.1 9.5255
1200 3.39927 4467.5 5147.3 10.0262 2.26614 4467.2 5147.1 9.8389 1.69958 4467.0 5146.8 9.7059
1099

(Continued)

6/16/2018 12:56:01 PM
Table A.1.3  (Continued ) 1100

p = 200 kPa (120.23) p = 300 kPa (133.55) p = 400 kPa (143.63)


T v u h s v u h s v u h s
1300 3.63018 4683.2 5409.3 10.1982 2.42013 4683.0 5409.0 10.0109 1.81511 4682.8 5408.8 9.8780
p = 500 kPa (151.86) p = 600 kPa (158.85) p = 800 kPa (170.43)
Sat. 0.37489 2561.2 2748.7 6.8212 0.31567 2567.4 2756.8 6.7600 0.24043 2576.8 2769.1 6.6627
200 0.42492 2642.9 2855.4 7.0592 0.35202 2638.9 2850.1 6.9665 0.26080 2630.6 2839.2 6.8158
250 0.47436 2723.5 2960.7 7.2708 0.39383 2720.9 2957.2 7.1816 0.29314 2715.5 2950.0 7.0384

Z01_THERMAL ENGINEERING_SE_XXXX_Z01.indd 1100


300 0.52256 2802.9 3064.2 7.4598 0.43437 2801.0 3061.6 7.3723 0.32411 2797.1 3056.4 7.2372
350 0.57012 2882.6 3167.6 7.6328 0.47424 2881.1 3165.7 7.5463 0.35439 2878.2 3161.7 7.4088
400 0.61728 2963.2 3271.8 7.7937 0.51372 2962.0 3270.2 7.7078 0.38426 2959.7 3267.1 7.5715
500 0.71093 3128.4 3483.8 8.0872 0.59199 3127.6 3482.7 8.0020 0.44331 3125.9 3480.6 7.8672
600 0.80406 3299.6 3701.7 8.3521 0.66974 3299.1 3700.9 8.2673 0.50184 3297.9 3699.4 8.1332
700 0.89691 3477.5 3926.0 8.5952 0.74720 3477.1 3925.4 8.5107 0.56007 3476.2 3924.3 8.3770
800 0.98959 3662.2 4157.0 8.8211 0.82450 3661.8 4156.5 8.7367 0.61813 3661.1 4155.7 8.6033
Appendix A

900 1.08217 3853.6 4394.7 9.0329 0.90169 3853.3 4394.4 8.9485 0.67610 3852.8 4393.6 8.8153
1000 1.17469 4051.8 4639.1 9.2328 0.97883 4051.5 4638.8 9.1484 0.73401 4051.0 4638.2 9.0153
1100 1.26718 4256.3 4889.9 9.4224 1.05594 4256.1 4889.6 9.3381 0.79188 4255.6 4889.1 9.2049
1200 1.35964 4466.8 5146.6 9.6028 1.13302 4466.5 5146.3 9.5185 0.84974 4466.1 5145.8 9.3854
1300 1.45210 4682.5 5408.6 9.7749 1.21009 4682.3 5408.3 9.6906 0.90758 4681.8 5407.9 9.5575
p = 1.00 MPa (179.91) p = 1.20 MPa (187.99) p = 1.40 MPa (195.07)
Sat. 0.19444 2583.6 2778.1 6.5864 0.16333 2588.8 2784.8 6.5233 0.14084 2592.8 2790.0 6.4692
200 0.20596 2621.9 2827.9 6.6939 0.16930 2612.7 2815.9 6.5898 0.14302 2603.1 2803.3 6.4975
250 0.23268 2709.9 2942.6 6.9246 0.19235 2704.2 2935.0 6.8293 0.16350 2698.3 2927.2 6.7467
300 0.25794 2793.2 3051.2 7.1228 0.21382 2789.2 3045.8 7.0316 0.18228 2785.2 3040.4 6.9533
350 0.28247 2875.2 3157.7 7.3010 0.23452 2872.2 3153.6 7.2120 0.20026 2869.1 3149.5 7.1359
400 0.30659 2957.3 3263.9 7.4650 0.25480 2954.9 3260.7 7.3773 0.21780 2952.5 3257.4 7.3025
(Continued)

6/16/2018 12:56:01 PM
Table A.1.3  (Continued )

p = 1 MPa (179.91) p = 1.2 MPa (187.99) p = 1.4 MPa (195.07)


T v u h s v u h s v u h s
500 0.35411 3124.3 3478.4 7.7621 0.29463 3122.7 3476.3 7.6758 0.25215 3121.1 3474.1 7.6026
600 0.40109 3296.8 3697.9 8.0289 0.33393 3295.6 3696.3 7.9434 0.28596 3294.4 3694.8 7.8710
700 0.44779 3475.4 3923.1 8.2731 0.37294 3474.5 3922.0 8.1881 0.31947 3473.6 3920.9 8.1160
800 0.49432 3660.5 4154.8 8.4996 0.41177 3659.8 4153.9 8.4149 0.35281 3659.1 4153.0 8.3431
900 0.54075 3852.5 4392.9 8.7118 0.45051 3851.6 4392.2 8.6272 0.38606 3851.0 4391.5 8.5555

Z01_THERMAL ENGINEERING_SE_XXXX_Z01.indd 1101


1000 0.58712 4050.5 4637.6 8.9119 0.48919 4050.0 4637.0 8.8274 0.41924 4049.5 4636.4 8.7558
1100 0.63345 4255.1 4888.5 9.1016 0.52783 4254.6 4888.0 9.0171 0.45239 4254.1 4887.5 8.9456
1200 0.67977 4465.6 5145.4 9.2821 0.56646 4465.1 5144.9 9.1977 0.48552 4464.6 5144.4 9.1262
1300 0.72608 4681.3 5407.4 9.4542 0.60507 4680.9 5406.9 9.3698 0.51864 4680.4 5406.5 9.2983
p = 1.60 MPa (201.40) p = 1.80 MPa (207.15) p = 2.00 MPa (212.42)
Sat. 0.12380 2595.9 2794.0 6.4217 0.11042 2598.4 2797.1 6.3793 0.09963 2600.3 2799.5 6.3408
225 0.13287 2644.6 2857.2 6.5518 0.11673 2636.6 2846.7 6.4807 0.10377 2628.3 2835.8 6.4146
250 0.14184 2692.3 2919.2 6.6732 0.12497 2686.0 2911.0 6.6066 0.11144 2679.6 2902.5 6.5452
Appendix A

300 0.15862 2781.0 3034.8 6.8844 0.14021 2776.8 3029.2 6.8226 0.12547 2772.6 3023.5 6.7663
350 0.17456 2866.0 3145.4 7.0693 0.15457 2862.9 3141.2 7.0099 0.13857 2859.8 3137.0 6.9562
400 0.19005 2950.1 3254.2 7.2373 0.16847 2947.7 3250.9 7.1793 0.15120 2945.2 3247.6 7.1270
500 0.22029 3119.5 3471.9 7.5389 0.19550 3117.8 3469.7 7.4824 0.17568 3116.2 3467.6 7.4316
600 0.24998 3293.3 3693.2 7.8080 0.22199 3292.1 3691.7 7.7523 0.19960 3290.9 3690.1 7.7023
700 0.27937 3472.7 3919.7 8.0535 0.24818 3471.9 3918.6 7.9983 0.22323 3471.0 3917.5 7.9487
800 0.30859 3658.4 4152.1 8.2808 0.27420 3657.7 4151.3 8.2258 0.24668 3657.0 4150.4 8.1766
900 0.33772 3850.5 4390.8 8.4934 0.30012 3849.9 4390.1 8.4386 0.27004 3849.3 4389.4 8.3895
1000 0.36678 4049.0 4635.8 8.6938 0.32598 4048.4 4635.2 8.6390 0.29333 4047.9 4634.6 8.5900
1100 0.39581 4253.7 4887.0 8.8837 0.35180 4253.2 4886.4 8.8290 0.31659 4252.7 4885.9 8.7800
1200 0.42482 4464.2 5143.9 9.0642 0.37761 4663.7 5143.4 9.0096 0.33984 4463.2 5142.9 8.9606
1101

(Continued)

6/16/2018 12:56:01 PM
Table A.1.3  (Continued ) 1102

p = 1.60 MPa (201.40) p = 1.80 MPa (207.15) p = 2.00 MPa (212.42)


T v u h s v u h s v u h s
1300 0.45382 4679.9 5406.0 9.2364 0.40340 4679.4 5405.6 9.1817 0.36306 4679.0 5405.1 9.1382
p = 2.50 MPa (223.99) p = 3.00 MPa (233.90) p = 3.50 MPa (242.66)
Sat. 0.07998 2603.1 2803.1 6.2574 0.06668 2604.1 2804.1 6.1869 0.05707 2603.7 2803.4 6.1252
225 0.08027 2605.6 2806.3 6.2638 – – – – – – – –
250 0.08700 2662.5 2880.1 6.4084 0.07058 2644.0 2855.8 6.2871 0.05873 2623.7 2829.2 6.1748

Z01_THERMAL ENGINEERING_SE_XXXX_Z01.indd 1102


300 0.09890 2761.6 3008.8 6.6437 0.08114 2750.0 2993.5 6.5389 0.06842 2738.0 2977.5 6.4460
350 0.10976 2851.8 3126.2 6.8402 0.09053 2843.7 3115.3 6.7427 0.07678 2835.3 3104.0 6.6578
400 0.12010 2939.0 3239.3 7.0147 0.09936 2932.7 3230.8 6.9211 0.08453 2926.4 3222.2 6.8404
450 0.13014 3025.4 3350.8 7.1745 0.10787 3020.4 3344.0 7.0833 0.09196 3015.3 3337.2 7.0051
500 0.13998 3112.1 3462.0 7.3233 0.11619 3107.9 3456.5 7.2337 0.09918 3103.7 3450.9 7.1571
600 0.15930 3288.0 3686.2 7.5960 0.13243 3285.0 3682.3 7.5084 0.11324 3282.1 3678.4 7.4338
700 0.17832 3468.8 3914.6 7.8435 0.14838 3466.0 3911.7 7.7571 0.12699 3464.4 3908.8 7.6837
Appendix A

800 0.19716 3655.3 4148.2 8.0720 0.16414 3653.6 4146.0 7.9862 0.14056 3651.8 4143.8 7.9135
900 0.21590 3847.9 4387.6 8.2853 0.17980 3846.5 4385.9 8.1999 0.15402 3845.0 4384.1 8.1275
1000 0.23458 4046.7 4633.1 8.4860 0.19541 4045.4 4631.6 8.4009 0.16743 4044.1 4630.1 8.3288
1100 0.25322 4251.5 4884.6 8.6761 0.21098 4250.3 4883.3 8.5911 0.18080 4249.1 4881.9 8.5191
1200 0.27185 4462.1 5141.7 8.8569 0.22652 4460.9 5140.5 8.7719 0.19415 4459.8 5139.3 8.7000
1300 0.29046 4677.8 5404.0 9.0291 0.24206 4676.6 5402.8 8.9442 0.20749 4675.5 5401.7 8.8723
p = 4.00 MPa (250.40) p = 4.50 MPa (257.48) p = 5.00 MPa (263.99)
Sat. 0.04978 2602.3 2801.4 6.0700 0.04406 2600.0 2798.3 6.0198 0.03944 2597.1 2794.3 5.9733
275 0.05457 2667.9 2886.2 6.2284 0.04730 2650.3 2863.1 6.1401 0.04141 2631.2 2838.3 6.0543
300 0.05884 2725.3 2960.7 6.3614 0.05135 2712.0 2943.1 6.2827 0.04532 2697.9 2924.5 6.2083
350 0.06645 2826.6 3092.4 6.5820 0.05840 2817.8 3080.6 6.5130 0.05194 2808.7 3068.4 6.4492
400 0.07341 2919.9 3213.5 6.7689 0.06475 2913.3 3204.7 6.7046 0.05781 2906.6 3195.6 6.6458
450 0.08003 3010.1 3330.2 6.9362 0.07074 3004.9 3323.2 6.8745 0.06330 2999.6 3316.1 6.8185
(Continued)

6/16/2018 12:56:02 PM
Table A.1.3  (Continued )

p = 4.00 MPa (250.40) p = 4.50 MPa (257.48) p = 5.00 MPa (263.99)


T v u h s v u h s v u h s
500 0.08643 3099.5 3445.2 7.0900 0.07651 3095.2 3439.5 7.0300 0.06857 3090.9 3433.8 6.9758
600 0.09885 3279.1 3674.4 7.3688 0.08765 3276.0 3670.5 7.3109 0.07869 3273.0 3666.5 7.2588
700 0.11095 3462.1 3905.9 7.6198 0.09847 3459.9 3903.0 7.5631 0.08849 3457.7 3900.1 7.5122
800 0.12287 3650.1 4141.6 7.8502 0.10911 3648.4 4139.4 7.7942 0.09811 3646.6 4137.2 7.7440
900 0.13469 3843.6 4382.3 8.0647 0.11965 3842.1 4380.6 8.0091 0.10762 3840.7 4378.8 7.9593

Z01_THERMAL ENGINEERING_SE_XXXX_Z01.indd 1103


1000 0.14645 4042.9 4628.7 8.2661 0.13013 4041.6 4627.2 8.2108 0.11707 4040.3 4625.7 8.1612
1100 0.15817 4248.0 4880.6 8.4566 0.14056 4246.8 4879.3 8.4014 0.12648 4245.6 4878.0 8.3519
1200 0.16987 4458.6 5138.1 8.6376 0.15098 4457.4 5136.9 8.5824 0.13587 4456.3 5135.7 8.5330
1300 0.18156 4674.3 5400.5 8.8099 0.16139 4673.1 5399.4 8.7548 0.14526 4672.0 5398.2 8.7055
p = 6.00 MPa (275.64) p = 7.00 MPa (285.88) p = 8.00 MPa (295.06)
Sat. 0.03244 2589.7 2784.3 5.8891 0.02737 2580.5 2772.1 5.8132 0.02352 2569.8 2757.9 5.7431
300 0.03616 2667.2 2884.2 6.0673 0.02947 2632.1 2838.4 5.9304 0.02426 2590.9 2785.0 5.7905
350 0.04223 2789.6 3043.0 6.3334 0.03524 2769.3 3016.0 6.2282 0.02995 2747.7 2987.3 6.1300
Appendix A

400 0.04739 2892.8 3177.2 6.5407 0.03993 2878.6 3158.1 6.4477 0.03432 2863.8 3138.3 6.3633
450 0.05214 2988.9 3301.8 6.7192 0.04416 2977.9 3287.0 6.6326 0.03817 2966.7 3272.0 6.5550
500 0.05665 3082.2 3422.1 6.8802 0.04814 3073.3 3410.3 6.7974 0.04175 3064.3 3398.3 6.7239
550 0.06101 3174.6 3540.6 7.0287 0.05195 3167.2 3530.9 6.9486 0.04516 3159.8 3521.0 6.8778
600 0.06525 3266.9 3658.4 7.1676 0.05565 3260.7 3650.3 7.0894 0.04845 3254.4 3642.0 7.0205
700 0.07352 3453.2 3894.3 7.4234 0.06283 3448.6 3888.4 7.3476 0.05481 3444.0 3882.5 7.2812
800 0.08160 3643.1 4132.7 7.6566 0.06981 3639.6 4128.3 7.5822 0.06097 3636.1 4123.8 7.5173
900 0.08958 3837.8 4375.3 7.8727 0.07669 3835.0 4371.8 7.7991 0.06702 3832.1 4368.3 7.7350
1000 0.09749 4037.8 4622.7 8.0751 0.08350 4035.3 4619.8 8.0020 0.07301 4032.8 4616.9 7.9384
1100 0.10536 4243.3 4875.4 8.2661 0.09027 4240.9 4872.8 8.1933 0.07896 4238.6 4870.3 8.1299
1200 0.11321 4454.0 5133.3 8.4473 0.09703 4451.7 5130.9 8.3747 0.08489 4449.4 5128.5 8.3115
1103

(Continued)

6/16/2018 12:56:02 PM
Table A.1.3  (Continued ) 1104

p = 6.00 MPa (275.64) p = 7.00 MPa (285.88) p = 8.00 MPa (295.06)


T v u h s v u h s v u h s
1300 0.12106 4669.6 5396.0 8.6199 0.10377 4667.3 5393.7 8.5472 0.09080 4665.0 5391.5 8.4842
p = 9.00 MPa (303.40) p = 10.00 MPa (311.06) p = 12.50 MPa (327.89)
Sat. 0.02048 2557.8 2742.1 5.6771 0.01803 2544.4 2724.7 5.6140 0.01350 2505.1 2673.8 5.4623
325 0.02327 2646.5 2855.9 5.8711 0.01986 2610.4 2809.0 5.7568 – – – –
350 0.02580 2724.4 2956.5 6.0361 0.02242 2699.2 2923.4 5.9442 0.01613 2624.6 2826.2 5.7117

Z01_THERMAL ENGINEERING_SE_XXXX_Z01.indd 1104


400 0.02993 2848.4 3117.8 6.2853 0.02641 2832.4 3096.5 6.2119 0.02000 2789.3 3039.3 6.0416
450 0.03350 2955.1 3256.6 6.4843 0.02975 2943.3 3240.8 6.4189 0.02299 3912.4 3199.8 6.2718
500 0.03677 3055.1 3386.1 6.6575 0.03279 3045.8 3373.6 6.5965 0.02560 3021.7 3341.7 6.4617
550 0.03987 3152.2 3511.0 6.8141 0.03564 3144.5 3500.9 6.7561 0.02801 3124.9 3475.1 6.6289
600 0.04285 3248.1 3633.7 6.9588 0.03837 3241.7 3625.3 6.9028 0.03029 3225.4 3604.0 6.7810
650 0.04574 3343.7 3755.3 7.0943 0.04101 3338.2 3748.3 7.0397 0.03248 3324.4 3730.4 6.9218
700 0.04857 3439.4 3876.5 7.2221 0.04358 3434.7 3870.5 7.1687 0.03460 3422.9 3855.4 7.0536
Appendix A

800 0.05409 3632.5 4199.4 7.4597 0.04859 3629.0 4114.9 7.4077 0.03869 3620.0 4103.7 7.2965
900 0.5950 3829.2 4364.7 7.6782 0.05349 3826.3 4361.2 7.6272 0.04267 3819.1 4352.5 7.5181
1000 0.06485 4030.3 4613.9 7.8821 0.05832 4027.8 4611.0 7.8315 0.04658 4021.6 4603.8 7.7237
1100 0.07016 4236.3 4867.7 8.0739 0.06312 4234.0 4865.1 8.0236 0.05045 4228.2 4858.8 7.9165
1200 0.07544 4447.2 5126.2 8.2556 0.06789 4444.9 5123.8 8.2054 0.05430 4439.3 5118.0 8.0987
1300 0.08072 4662.7 5389.2 8.4283 0.07265 4660.4 5387.0 8.3783 0.05813 4654.8 5381.4 8.2717
p = 15 MPa (342.24) p = 17.5 MPa (354.75) p = 20 MPa (365.81)
Sat. .010338 2455.4 2610.5 5.3097 .0079204 2390.2 2528.8 5.1418 .0058342 2293.1 2409.7 4.9269
350 .011470 2520.4 2692.4 5.4420 .0017139 1632.0 1662.0 3.7612 .0016640 1612.3 1645.6 3.7280
400 .015649 2740.7 2975.4 5.8810 .0124477 2685.0 2902.8 5.7212 .0099423 2619.2 2818.1 5.5539
450 .018446 2879.5 3156.2 6.1403 .0151740 2844.2 3109.7 6.0182 .0126953 2806.2 3060.1 5.9016
500 .020800 2996.5 3308.5 6.3442 .0173585 2970.3 3274.0 6.2382 .0147683 2942.8 3238.2 6.1400
(Continued)

6/16/2018 12:56:02 PM
Table A.1.3  (Continued )

p = 15.00 MPa (342.24) p = 17.5 MPa (354.75) p = 20 MPa (365.81)


T v u h s v u h s v u h s
550 .022927 3104.7 3448.6 6.5198 .0192877 3083.8 3421.4 6.4229 .0165553 3062.3 3393.5 6.3347
600 .024911 3208.6 3582.3 6.6775 .0210640 3191.5 3560.1 6.5866 .0181781 3174.0 3537.6 6.5048
650 .026797 3310.4 3712.3 6.8223 .0227372 3296.0 3693.9 6.7356 .0196929 3281.5 3675.3 6.6582
700 .028612 3410.9 3840.1 6.9572 .0243365 3398.8 3824.7 6.8736 .0211311 3386.5 3809.1 6.7993
800 .032096 3611.0 4092.4 7.2040 .0273849 3601.9 4081.1 7.1245 .0238532 3592.7 4069.8 7.0544
900 .035457 3811.9 4343.8 7.4279 .0303071 3804.7 4335.1 7.3507 .0264463 3797.4 4326.4 7.2830

Z01_THERMAL ENGINEERING_SE_XXXX_Z01.indd 1105


1000 .038748 4015.4 4596.6 7.6347 .0331580 4009.3 4589.5 7.5588 .0289666 4003.1 4582.5 7.4925
1100 .042001 4222.6 4852.6 7.8282 .0359695 4216.9 4846.4 7.7530 .0314471 4211.3 4840.2 7.6874
1200 .045233 4433.8 5112.3 8.0108 .0387605 4428.3 5106.6 7.9359 .0339071 4422.8 5101.0 7.8706
1300 .048455 4649.1 5375.9 8.1839 .0415417 4643.5 5370.5 8.1093 .0363574 4638.0 5365.1 8.0441
p = 25 MPa p = 30 MPa p = 35 MPa
375 .001973 1798.6 1847.9 4.0319 .001789 1737.8 1791.4 3.9303 .001700 1702.9 1762.4 3.8721
400 .006004 2430.1 2580.2 5.1418 .002790 2067.3 2151.0 4.4728 .002100 1914.0 1987.5 4.2124
425 .007882 2609.2 2806.3 5.4722 .005304 2455.1 2614.2 5.1503 .003428 2253.4 2373.4 4.7747
450 .009162 2720.7 2949.7 5.6743 .006735 2619.3 2821.4 5.4423 .004962 2498.7 2672.4 5.1962
500 .011124 2884.3 3162.4 5.9592 .008679 2820.7 3081.0 5.7904 .006927 2751.9 2994.3 5.6281
Appendix A

550 .012724 3017.5 3335.6 6.1764 .010168 2970.3 3275.4 6.0342 .008345 2920.9 3213.0 5.9025
600 .014138 3137.9 3491.4 6.3602 .011446 3100.5 3443.9 6.2330 .009527 3062.0 3395.5 6.1178
650 .015433 3251.6 3637.5 6.5229 .012596 3221.0 3598.9 6.4057 .010575 3189.8 3559.9 6.3010
700 .016647 3361.4 3777.6 6.6707 .013661 3335.8 3745.7 6.5606 .011533 3309.9 3713.5 6.4631
800 .018913 3574.3 4047.1 6.9345 .015623 3555.6 4024.3 6.8332 .013278 3536.8 4001.5 6.7450
900 .021045 3783.0 4309.1 7.1679 .017448 3768.5 4291.9 7.0717 .014883 3754.0 4274.9 6.9886
1000 .023102 3990.9 4568.5 7.3801 .019196 3978.8 4554.7 7.2867 .016410 3966.7 4541.1 7.2063
1100 .025119 4200.2 4828.2 7.5765 .020903 4189.2 4816.3 7.4845 .017895 4178.3 4804.6 7.4056
1200 .027115 4412.0 5089.9 7.7604 .022589 4410.3 5079.0 7.6691 .019360 4390.7 5068.4 7.5910
1300 .029101 4626.9 5354.4 7.9342 .024266 4616.0 5344.0 7.8432 .020815 4605.1 5333.6 7.7652
(Continued)
1105

6/16/2018 12:56:02 PM
Table A.1.3  (Continued ) 1106

p = 40 MPa p = 50 MPa p = 60 MPa


T v u h s v u h s v u h s
375 .0016406 1677.1 1742.7 3.8289 .0015593 1638.6 1716.5 3.7638 .0015027 1609.3 1699.5 3.1740
400 .0019077 1854.5 1930.8 4.1134 .0017309 1788.0 1874.6 4.0030 .0016335 1745.3 1843.4 3.9317
425 .0025319 2096.8 2198.1 4.5028 .0020071 1959.6 2060.0 4.2733 .0018165 1892.7 2001.7 4.1625
450 .0036931 2365.1 2512.8 4.9459 .0024862 2159.6 2283.9 4.5883 .0020850 2053.9 2179.0 4.4119
500 .0056225 2678.4 2903.3 5.4699 .0038924 2525.5 2720.1 5.1725 .0029557 2390.5 2567.9 4.9320
600 .0080943 3022.6 3346.4 6.0113 .0061123 2942.0 3247.6 5.8177 .0048345 2861.1 3151.2 5.6451

Z01_THERMAL ENGINEERING_SE_XXXX_Z01.indd 1106


650 .0090636 3158.0 3520.6 6.2054 .0069657 3093.6 3441.8 6.0342 .0055953 3028.8 3364.6 5.8829
700 .0099415 3283.6 3681.3 6.3750 .0077274 3230.5 3616.9 6.2189 .0062719 3177.3 3553.6 6.0824
800 .0115228 3517.9 3978.8 6.6662 .0090761 3479.8 3933.6 6.5290 .0074588 3441.6 3889.1 6.4110
900 .0129626 3739.4 4257.9 6.9150 .0102831 3710.3 4224.4 6.7882 .0085083 3681.0 4191.5 6.6805
1000 .0143238 3954.6 4527.6 7.1356 .0114113 3930.5 4501.1 7.0146 .0094800 3906.4 4475.2 6.9126
1100 .0156426 4167.4 4793.1 7.3364 .0124966 4145.7 4770.6 7.2183 .0104091 4124.1 4748.6 7.1194
1200 .0169403 4380.1 5057.7 7.5224 .0135606 4359.1 5037.2 7.4058 .0113167 4338.2 5017.2 7.3082
1300 .0182292 4594.3 5323.5 7.6969 .0146159 4572.8 5303.6 7.5807 .0122155 4551.4 5284.3 7.4837
Appendix A

Table A.1.4  Compressed liquid water


p = 5.00 MPa (263.99) p = 10.00 MPa (311.06) p = 15.00 MPa (342.24)
T v u h s v u h s v u h s
Sat. .0012859 1147.78 1154.21 2.9201 .0014524 1393.00 1407.53 3.3595 .0016581 1585.58 1610.45 3.6847
0 .0009977 0.03 5.02 0.0001 .0009952 0.10 10.05 0.0003 .0009928 0.15 15.04 0.0004
20 .0009995 83.64 88.64 0.2955 .0009972 83.35 93.32 0.2945 .0009950 83.05 97.97 0.2934
40 .0010056 166.93 171.95 0.5706 .0010034 166.33 176.36 0.5685 .0010013 165.73 180.75 0.5665
60 .0010149 250.21 255.28 0.8284 .0010127 249.34 259.47 0.8258 .0010105 248.49 263.65 0.8231
80 .0010268 333.69 338.83 1.0719 .0010245 332.56 342.81 1.0687 .0010222 331.46 346.79 1.0655
(Continued)

6/16/2018 12:56:02 PM
Table A.1.4  (Continued )

p = 5.00 MPa (263.99) p = 10.00 MPa (311.06) p = 15.00 MPa (342.24)


T v u h s v u h s v u h s
100 .0010410 417.50 422.71 1.3030 .0010385 416.09 426.48 1.2992 .0010361 414.72 430.26 1.2954
120 .0010576 501.79 507.07 1.5232 .0010549 500.07 510.61 1.5188 .0010522 498.39 514.17 1.5144
140 .0010768 586.74 592.13 1.7342 .0010737 584.67 595.40 1.7291 .0010707 582.64 598.70 1.7241
160 .0010988 672.61 678.10 1.9374 .0010953 670.11 681.07 1.9316 .0010918 667.69 684.07 1.9259
180 .0011240 759.62 765.24 2.1341 .0011199 756.63 767.83 2.1274 .0011159 753.74 770.48 2.1209

Z01_THERMAL ENGINEERING_SE_XXXX_Z01.indd 1107


200 .0011530 848.08 853.85 2.3254 .0011480 844.49 855.97 2.3178 .0011433 841.04 858.18 2.3103
220 .0011866 938.43 944.36 2.5128 .0011805 934.07 945.88 2.5038 .0011748 929.89 947.52 2.4952
240 .0012264 1031.34 1037.47 2.6978 .0012187 1025.94 1038.13 2.6872 .0012114 1020.82 1038.99 2.6770
260 .0012748 1127.92 1134.30 2.8829 .0012645 1121.03 1133.68 2.8698 .0012550 1114.59 1133.41 2.8575
280 .0013216 1220.90 1234.11 3.0547 .0013084 1212.47 1232.09 3.0392
300 .0013972 1328.34 1342.31 3.2468 .0013770 1316.58 1337.23 3.2259
320 .0014724 1431.05 1453.13 3.4246
340 .0016311 1567.42 1591.88 3.6545
Appendix A

Sat .0020353 1785.47 1826.18 4.0137 – – – – – – – –


0 .0009904 0.02 20.00 0.0004 .0009856 0.25 29.82 0.0001 .0009766 0.20 49.03 – 0.0014
20 .0009928 82.75 102.61 0.2922 .0009886 82.16 111.82 0.2898 .0009804 80.98 130.00 0.2847
40 .0009992 165.15 185.14 0.5646 .0009951 164.01 193.87 0.5606 .0009872 161.84 211.20 0.5526
60 .0010084 247.66 267.82 0.8205 .0010042 246.03 276.16 0.8153 .0009962 242.96 292.77 0.8051
80 .0010199 330.38 350.78 1.0623 .0010156 328.28 358.75 1.0561 .0010073 324.32 374.68 1.0439
100 .0010337 413.37 434.04 1.2917 .0010290 410.76 441.63 1.2844 .0010201 405.86 456.87 1.2703
120 .0010496 496.75 517.74 1.5101 .0010445 493.58 524.91 1.5017 .0010348 487.63 539.37 1.4857
140 .0010678 580.67 602.03 1.7192 .0010621 576.86 608.27 1.7097 .0010515 569.76 622.33 1.6915
160 .0010885 665.34 687.11 1.9203 .0010821 660.81 693.27 1.9085 .0010703 652.39 705.91 1.8890
180 .0011120 750.94 773.18 2.1146 .0011047 745.57 778.71 2.1024 .0010912 735.68 790.24 2.0793
1107

(Continued)

6/16/2018 12:56:03 PM
1108

Table A.1.4  (Continued )

p = 5.00 MPa (263.99) p = 10.00 MPa (311.06) p = 15.00 MPa (342.24)

Z01_THERMAL ENGINEERING_SE_XXXX_Z01.indd 1108


T v u h s v u h s v u h s
200 .0011387 837.70 860.47 2.3031 .0011302 831.34 865.24 2.2892 .0011146 819.73 875.46 2.2634
220 .0011693 925.89 949.27 2.4869 .0011590 918.32 953.09 2.4710 .0011408 904.67 961.71 2.4419
240 .0012046 1015.94 1040.04 2.6673 .0011920 1006.84 1042.60 2.6489 .0011702 990.69 1049.20 2.6158
260 .0012462 1108.53 1133.45 2.8459 .0012303 1097.38 1134.29 2.8242 .0012034 1078.06 1138.23 2.7860
280 .0012965 1204.69 1230.62 3.0248 .0012755 1190.69 1228.96 2.9985 .0012415 1167.19 1229.26 2.9536
Appendix A

300 .0013596 1306.10 1333.29 3.2071 .0013304 1287.89 1327.80 3.1740 .0012860 1258.66 1322.95 3.1200
320 .0014437 1415.66 1444.53 3.3978 .0013997 1390.64 1432.63 3.3538 .0013388 1353.23 1420.17 3.2867
340 .0015683 1539.64 1571.01 3.6074 .0014919 1501.71 1456.47 3.5425 .0014032 1451.91 1522.07 3.4556
360 .0018226 1702.78 1739.23 3.8770 .0016265 1626.57 1675.36 3.7492 .0014838 1555.97 1630.16 3.6290
380 – – – – .0018691 1781.35 1837.43 4.0010 .0015883 1667.13 1746.54 3.8100

6/16/2018 12:56:03 PM
A.2 ❐ THERMODYNAMIC PROPERTIES OF REFRIGERANT-134A

(1,1,1,1–TETRAFLUOROETHANE)

Table A.2.1  Saturated R-134a

Abs Specific volume, m3/kg Enthalpy, kJ/kg Entropy, kJ/kg K


Temp. Press. Sat. Sat. Sat. Sat. Sat. Sat.
°C MPa liquid Evap. vapour liquid Evap. vapour liquid Evap. vapour
p vf vfg vg hf hfg hg sf sfg sg

Z01_THERMAL ENGINEERING_SE_XXXX_Z01.indd 1109


– 33 0.0737 0.000718 0.25574 0.25646 157.417 220.491 377.908 0.8346 0.9181 1.7528
– 30 0.0851 0.000722 0.22330 0.22402 161.118 218.683 379.802 0.8499 0.8994 1.7493
26.25 0.1013 0.000728 0.18947 0.19020 165.802 216.360 382.162 0.8690 0.8763 1.7453
– 25 0.0173 0.000730 0.17956 0.18029 167.381 215.569 382.950 0.8754 0.8687 1.7441
– 20 0.1337 0.000738 0.14575 0.14649 173.744 212.340 386.083 0.9007 0.8388 1.7395
– 15 0.1650 0.000746 0.11932 0.12007 180.193 209.004 389.197 0.9258 0.8096 1.7354
– 10 0.2017 0.000755 0.098454 0.099209 186.721 205.564 392.285 0.9507 0.7812 1.7319
–5 0.2445 0.000764 0.081812 0.082576 193.324 202.016 395.340 0.9755 0.7534 1.7288
Appendix A

0 0.2940 0.000773 0.068420 0.069193 200.000 198.356 398.356 1.0000 0.7262 1.7262
5 0.3509 0.000783 0.057551 0.058334 206.751 194.572 401.323 1.0243 0.6995 1.7239
10 0.4158 0.000794 0.048658 0.049451 213.580 190.652 404.233 1.0485 0.6733 1.7218
15 0.4895 0.000805 0.041326 0.042131 220.492 186.582 407.075 1.0725 0.6475 1.7200
20 0.5728 0.000817 0.035238 0.036055 227.493 182.345 409.838 1.0963 0.6220 1.7183
25 0.6663 0.000829 0.030148 0.030977 234.590 177.920 412.509 1.1201 0.5967 1.7168
30 0.7710 0.000843 0.025865 0.026707 241.790 173.285 415.075 1.1437 0.5716 1.7153
35 0.8876 0.000857 0.022237 0.023094 249.103 168.415 417.518 1.1673 0.5465 1.7139
40 1.0171 0.000873 0.019147 0.020020 256.539 163.282 419.821 1.1909 0.5214 1.7123
45 1.1602 0.000890 0.016499 0.017389 264.110 157.852 421.962 1.2145 0.4962 1.7106
1109

(Continued)

6/16/2018 12:56:03 PM
Table A.2.1  (Continued ) 1110

Abs Specific volume, m3/kg Enthalpy, kJ/kg Entropy, kJ/kg K


Temp. Press. Sat. Sat. Sat. Sat. Sat. Sat.
°C MPa liquid Evap. vapour liquid Evap. vapour liquid Evap. vapour
p vf vfg vg hf hfg hg sf sfg sg
50 1.3180 0.000908 0.014217 0.015124 271.830 152.085 423.915 1.2381 0.4706 1.7088
55 1.4915 0.000928 0.012237 0.013166 279.718 145.933 425.650 1.2619 0.4447 1.7066
60 1.6818 0.000951 0.010511 0.011462 287.794 139.336 427.130 1.2857 0.4182 1.7040

Z01_THERMAL ENGINEERING_SE_XXXX_Z01.indd 1110


65 1.8898 0.000976 0.008995 0.009970 296.088 132.216 428.305 1.3099 0.3910 1.7009
70 2.1169 0.001005 0.007653 0.008657 304.642 124.468 429.110 1.3343 0.3627 1.6970
75 2.3644 0.001038 0.006453 0.007491 313.513 115.939 429.451 1.3592 0.3330 1.6923
80 2.6337 0.001078 0.005368 0.006446 322.794 106.395 429.189 1.3849 0.3013 1.6862
85 2.9265 0.001128 0.004367 0.005495 332.644 95.440 428.084 1.4117 0.2665 1.6782
90 3.2448 0.001995 0.003412 0.004606 343.380 82.295 425.676 1.4404 0.2266 1.6670
95 3.5914 0.001297 0.002432 0.003729 355.834 64.984 420.818 1.4733 0.1765 1.6498
Appendix A

101.15 4.0640 0.001969 0 0.001969 390.977 0 390.977 1.5658 0 1.5658

Table A.2.2  Superheated R-134a

Temp. v h s v h s v h s
°C m3/kg kJ/kg kJ/kg K m3/kg kJ/kg kJ/kg K m3/kg kJ/kg kJ/kg K
0.10 MPa 0.15 MPa 0.20 MPa
– 25 0.19400 383.212 1.75058 – – – – – –
– 20 0.19860 387.215 1.76655 – – – – – –
– 10 0.20765 395.270 1.79775 0.13603 393.839 1.76058 0.10013 392.338 1.73276
0 0.21652 403.413 1.82813 0.14222 402.187 1.79171 0.10501 400.911 1.76474
(Continued)

6/16/2018 12:56:03 PM
Table A.2.2  (Continued )

Temp. v h s v h s v h s
°C m3/kg kJ/kg kJ/kg K m3/kg kJ/kg kJ/kg K m3/kg kJ/kg kJ/kg K
10 0.22527 411.668 1.85780 0.14828 410.602 1.82197 0.10974 409.500 1.79562
20 0.23393 420.048 1.88689 0.15424 419.111 1.85150 0.11436 418.145 1.82563
30 0.24250 428.564 1.91545 0.16011 427.730 1.88041 0.11889 426.875 1.85491
40 0.25102 437.223 1.94355 0.16592 436.473 1.90879 0.12335 435.708 1.88357
50 0.25948 446.029 1.97123 0.17168 445.350 1.93669 0.12776 444.658 1.91171

Z01_THERMAL ENGINEERING_SE_XXXX_Z01.indd 1111


60 0.26791 454.986 1.99853 0.17740 454.366 1.96416 0.13213 453.735 1.93937
70 0.27631 464.096 2.02547 0.18308 463.525 1.99125 0.13646 462.946 1.96661
80 0.28468 473.359 2.05208 0.18874 472.831 2.01798 0.14076 472.296 1.99346
90 0.29303 482.777 2.07837 0.19437 482.285 2.04438 0.14504 481.788 2.01997
100 0.30136 492.349 2.10437 0.19999 491.888 2.07046 0.14930 491.424 2.04614
0.25 MPa 0.30 MPa 0.40 MPa
0 0.082637 399.579 1.74284 – – – – – –
10 0.086584 408.357 1.77440 0.071110 407.171 1.75637 0.051681 404.651 1.72611
Appendix A

20 0.090408 417.151 1.80492 0.074415 416.124 1.78744 0.054362 413.965 1.75844


30 0.090139 425.997 1.83460 0.077620 425.096 1.81754 0.056926 423.216 1.78947
40 0.097798 434.925 1.86357 0.080748 434.124 1.84684 0.059402 432.465 1.81949
50 0.101401 443.953 1.89195 0.083816 443.234 1.87547 0.061812 441.751 1.84868
60 0.104958 453.094 1.91980 0.086838 452.442 1.90354 0.064169 451.104 1.87718
70 0.108480 462.359 1.94720 0.089821 461.763 1.93110 0.066484 460.545 1.90510
80 0.111972 471.754 1.97419 0.092774 471.206 1.95823 0.068767 470.088 1.93252
90 0.115440 481.285 2.00080 0.095702 480.777 1.98495 0.071022 479.745 1.95948
100 0.118888 490.955 2.02707 0.098609 490.482 2.01131 0.073254 489.523 1.98604
110 0.122318 500.766 2.05302 0.101498 500.324 2.03734 0.075468 499.428 2.01223
(Continued)
1111

6/16/2018 12:56:03 PM
Table A.2.2  (Continued ) 1112

Temp. v h s v h s v h s
°C m3/kg kJ/kg kJ/kg K m3/kg kJ/kg kJ/kg K m3/kg kJ/kg kJ/kg K
120 0.125734 510.720 2.07866 0.104371 510.304 2.06305 0.077665 509.464 2.03809
0.50 MPa 0.60 MPa 0.70 MPa
20 0.042256 411.645 1.73420 – – – – – –
30 0.044457 421.221 1.76632 0.036094 419.093 1.74610 0.030069 416.809 1.72770
40 0.046557 430.720 1.79715 0.037958 428.881 1.77786 0.031781 426.933 1.76056

Z01_THERMAL ENGINEERING_SE_XXXX_Z01.indd 1112


50 0.048581 440.205 1.82696 0.039735 438.589 1.80838 0.033392 436.895 1.79187
60 0.050547 449.718 1.85596 0.041447 448.279 1.83791 0.034929 446.782 1.82201
70 0.052467 459.290 1.88426 0.043108 457.994 1.86664 0.036410 456.655 1.85121
80 0.054351 468.942 1.91199 0.044730 467.764 1.89471 0.037848 466.554 1.87964
90 0.056205 478.690 1.93921 0.046319 477.611 1.92220 0.039251 476.507 1.90743
100 0.058035 488.546 1.96598 0.047883 487.550 1.94920 0.040627 486.535 1.93467
110 0.059845 498.518 1.99235 0.049426 497.594 1.97576 0.041980 496.654 1.96143
Appendix A

120 0.061639 508.613 2.01836 0.050951 507.750 2.00193 0.043314 506.875 1.98777
130 0.063418 518.835 2.04403 0.052461 518.026 2.02774 0.044633 517.207 2.01372
140 0.065184 529.187 2.06940 0.053958 528.425 2.05322 0.045938 527.656 2.03932
0.80 MPa 0.90 MPa 1.00 MPa
40 0.027113 424.860 1.74457 0.023446 422.642 1.72943 0.020473 420.249 1.71479
50 0.028611 435.114 1.77680 0.024868 433.235 1.76273 0.021849 431.243 1.74936
60 0.030024 445.223 1.80761 0.026192 443.595 1.79431 0.023110 441.890 1.78181
70 0.031375 455.270 1.83732 0.027447 453.835 1.82459 0.024293 452.345 1.81273
80 0.032678 465.308 1.86616 0.028649 464.025 1.85387 0.025417 462.703 1.84248
90 0.033944 475.375 1.89427 0.029810 474.216 1.88232 0.026497 473.027 1.87131
100 0.035180 485.499 1.92177 0.030940 484.441 1.91010 0.027543 483.361 1.89938
110 0.036392 495.698 1.94874 0.032043 494.726 1.93730 0.028561 493.736 1.92682
(Continued)

6/16/2018 12:56:03 PM
Table A.2.2  (Continued )

Temp. v h s v h s v h s
°C m3/kg kJ/kg kJ/kg K m3/kg kJ/kg kJ/kg K m3/kg kJ/kg kJ/kg K
120 0.037584 505.988 1.97525 0.033126 505.088 1.96399 0.029556 504.175 1.95371
130 0.038760 516.379 2.00135 0.034190 515.542 1.99025 0.030533 514.694 1.98013
140 0.039921 526.880 2.02708 0.035241 526.096 2.01611 0.031495 525.305 2.00613
150 0.041071 537.496 2.05247 0.036278 536.760 2.04161 0.032444 536.017 2.03175
1.20 MPa 1.40 MPa 1.60 MPa

Z01_THERMAL ENGINEERING_SE_XXXX_Z01.indd 1113


50 0.017243 426.485 1.72373 – – – – – –
60 0.018439 438.210 1.75837 0.015032 434.079 1.73597 0.012392 429.322 1.71349
70 0.019530 449.179 1.79081 0.016083 445.720 1.77040 0.013449 441.888 1.75066
80 0.020548 459.925 1.82168 0.017040 456.944 1.80265 0.014378 453.722 1.78466
90 0.021512 470.551 1.85135 0.017931 467.931 1.83333 0.015225 465.145 1.81656
100 0.022436 481.128 1.88009 0.018775 478.790 1.86282 0.016015 476.333 1.84695
110 0.023329 491.702 1.90805 0.019583 489.589 1.89139 0.016763 487.390 1.87619
120 0.024197 502.307 1.93537 0.020362 500.379 1.91918 0.017479 498.387 1.90452
Appendix A

130 0.025044 512.965 1.96214 0.021118 511.192 1.94634 0.018169 509.371 1.93211
140 0.025874 523.697 1.98844 0.021856 522.054 1.97296 0.018840 520.376 1.95908
150 0.026691 534.514 2.01431 0.022579 532.984 1.99910 0.019493 531.427 1.98551
160 0.027495 545.426 2.03980 0.023289 543.994 2.02481 0.020133 542.542 2.01147
170 0.028289 556.443 2.06494 0.023988 555.097 2.05015 0.020761 553.735 2.03702
1.80 MPa 2.0 MPa 2.50 MPa
70 0.011341 437.562 1.73085 0.009581 432.531 1.71011 – – –
80 0.012273 450.202 1.76717 0.010550 446.304 1.74968 0.007221 433.797 1.70180
90 0.013099 462.164 1.80057 0.011374 458.951 1.78500 0.008157 449.499 1.74567
100 0.013854 473.741 1.83202 0.012111 470.996 1.81772 0.008907 463.279 1.78311
110 0.014560 485.095 1.86205 0.012789 482.693 1.84866 0.009558 476.129 1.81709
1113

(Continued)

6/16/2018 12:56:04 PM
Table A.2.2  (Continued ) 1114

Temp. v h s v h s v h s
°C m3/kg kJ/kg kJ/kg K m3/kg kJ/kg kJ/kg K m3/kg kJ/kg kJ/kg K
120 0.015230 496.325 1.89098 0.013424 494.187 1.87827 0.010148 488.457 1.84886
130 0.015871 507.498 1.91905 0.014028 505.569 1.90686 0.010694 500.474 1.87904
140 0.016490 518.659 1.94639 0.014608 516.900 1.93463 0.011208 512.307 1.90804
150 0.017091 529.841 1.97314 0.015168 528.224 1.96171 0.011698 524.037 1.93609
160 0.017677 541.068 1.99936 0.015712 539.571 1.98821 0.012169 535.722 1.96338

Z01_THERMAL ENGINEERING_SE_XXXX_Z01.indd 1114


170 0.018251 552.357 2.02513 0.016242 550.963 2.01421 0.012624 547.399 1.99004
180 0.018814 563.724 2.05049 0.016762 562.418 2.03977 0.013066 559.098 2.01614
190 0.019369 575.177 2.07549 0.017272 573.950 2.06494 0.013498 570.841 2.04177
3.0 MPa 3.50 MPa 4.0 MPa
90 0.005755 436.193 1.69950 – – – – – –
100 0.006653 453.731 1.74717 0.004839 440.433 1.70386 – – –
110 0.007339 468.500 1.78623 0.005667 459.211 1.75355 0.004277 446.844 1.71840
Appendix A

120 0.007924 482.043 1.82113 0.006289 474.697 1.79346 0.005005 465.987 1.76415
130 0.008446 494.915 1.85347 0.006813 488.771 1.82881 0.005559 481.865 1.80404
140 0.008926 507.388 1.88403 0.007279 502.079 1.86142 0.006027 496.295 1.83940
150 0.009375 519.618 1.91328 0.007706 514.928 1.89216 0.006444 509.925 1.87200
160 0.009801 531.704 1.94151 0.008103 527.496 1.92151 0.006825 523.072 1.90271
170 0.010208 543.713 1.96892 0.008480 539.890 1.94980 0.007181 535.917 1.93203
180 0.010601 555.690 1.99565 0.008839 552.185 1.97724 0.007517 548.573 1.96028
190 0.010982 567.670 2.02180 0.009185 564.430 2.00397 0.007837 561.117 1.98766
200 0.011353 579.678 2.04745 0.009519 576.665 2.03010 0.008145 573.601 2.01432

6/16/2018 12:56:04 PM
A.3 ❐ THERMODYNAMIC PROPERTIES OF AMMONIA

Table A.3.1  Saturated ammonia table

T p Specific volume (m3/Kg) Enthalpy (KJ/kg) Entropy (KJ/kg K)


(°C ) (kPa) Sat. Sat. Sat. Sat. Sat. Sat.
liquid vapour, liquid, Evap. vapour, liquid, vapour
vt t vg ht htg hg st sg
– 50 40.88 0.001424 2.6254 – 44.3 1416.7 1372.4 – 0.1942 6.1561

Z01_THERMAL ENGINEERING_SE_XXXX_Z01.indd 1115


– 48 45.96 0.001429 2.3533 – 35.5 1411.3 1375.8 – 0.1547 6.1149
– 46 51.55 0.001434 2.1140 – 26.6 1405.8 1379.2 – 0.1156 6.0746
– 44 57.69 0.001439 1.9032 – 17.8 1400.3 1382.5 – 0.0768 6.0352
– 42 64.42 0.001444 1.7170 – 8.9 1394.7 1385.8 – 0.0382 5.9967
– 40 71.77 0.001449 1.5521 0.0 1389.0 1389.0 0.0000 5.9589
– 38 79.80 0.001454 1.4058 8.9 1383.3 1392.2 0.0380 5.9220
– 36 88.54 0.001460 1.2757 17.8 1377.6 1395.4 0.0757 5.8858
– 34 98.05 0.001465 1.1597 26.8 1371.8 1398.5 0.1132 5.8504
Appendix A

– 32 108.37 0.001470 1.0562 35.7 1365.9 1401.6 0.1504 5.8156


– 30 119.55 0.001476 0.9635 44.7 1360.0 1404.6 0.1873 5.7815
– 28 131.64 0.001481 0.8805 53.6 1354.0 1407.6 0.2240 5.7481
– 26 144.70 0.001487 0.8059 62.6 1347.9 1410.5 0.2605 5.7153
– 24 158.78 0.001492 0.7388 71.6 1341.8 1413.4 0.2967 5.6831
– 22 173.93 0.001498 0.6783 80.7 1335.6 1416.2 0.3327 5.6515
– 20 190.22 0.001504 0.6237 89.7 1329.3 1419.0 0.3684 5.6205
– 18 207.71 0.001510 0.5743 98.8 1322.9 1421.7 0.4040 5.5900
– 16 226.45 0.001515 0.5296 107.8 1316.5 1424.4 0.4393 5.5600
– 14 246.51 0.001521 0.4889 116.9 1310.0 1427.0 0.4744 5.5305
– 12 267.95 0.001528 0.4520 126.0 1303.5 1429.5 0.5093 5.5015
1115

(Continued)

6/16/2018 12:56:04 PM
Table A.3.1  (Continued ) 1116

T p Specific volume (m3/Kg) Enthalpy (KJ/kg) Entropy (KJ/kg K)


(°C ) (kPa) Sat. Sat. Sat. Sat. Sat. Sat.
liquid vapour, liquid, Evap. vapour, liquid, vapour
vt t vg ht htg hg st sg
 – 10 290.95 0.001534 0.4185 135.2 1296.8 1432.0 0.5440 5.4730
–8 315.25 0.001540 0.3878 144.3 1290.1 1434.4 0.5785 5.4449
–6 341.25 0.001546 0.3599 153.5 1283.3 1436.8 0.6128 5.4173

Z01_THERMAL ENGINEERING_SE_XXXX_Z01.indd 1116


–4 368.90 0.001553 0.3343 162.7 1276.4 1439.1 0.6469 5.3901
–2 398.27 0.001559 0.3109 171.9 1269.4 1441.3 0.6808 5.3633
 0 429.44 0.001566 0.2895 181.1 1262.4 1443.5 0.7145 5.3369
 2 462.49 0.001573 0.2698 190.4 1255.2 1445.6 0.7481 5.3108
 4 497.49 0.001580 0.2517 199.6 1243.0 1447.6 0.7815 5.2852
 6 531.51 0.001587 0.2351 208.9 1240.6 1449.6 0.8148 5.2599
 8 573.64 0.001594 0.2198 218.3 1233.2 1451.5 0.8479 5.2350
Appendix A

10 614.95 0.001601 0.2056 227.6 1225.7 1453.3 0.8808 5.2104


12 658.52 0.001608 0.1926 237.0 1218.1 1455.1 0.9136 5.1861
14 704.44 0.001616 0.1805 246.4 1210.4 1456.8 0.9463 5.1621
16 752.79 0.001623 0.1693 255.9 1202.6 1458.5 0.9788 5.1385
18 803.66 0.001631 0.1590 265.4 1194.7 1460.0 1.0112 5.1151
20 857.12 0.001639 0.1494 274.9 1186.7 1461.5 1.0434 5.0920
22 913.27 0.001647 0.1405 284.4 1178.5 1462.9 1.0755 5.0692
24 972.19 0.001655 0.1322 294.0 1170.3 1464.3 1.1075 5.0467
26 1033.97 0.001663 0.1245 303.6 1162.0 1465.6 1.1394 5.0244
28 1098.71 0.001671 0.1173 313.2 1153.6 1466.8 1.1711 5.0023
30 1166.49 0.001680 0.1106 322.9 1145.0 1467.9 1.2028 4.9805
32 1237.41 0.001689 0.1044 332.6 1136.4 1469.0 1.2343 4.9589
34 1311.55 0.001698 0.0986 342.3 1127.6 1469.9 1.2656 4.9374
(Continued)

6/16/2018 12:56:04 PM
Table A.3.1  (Continued )

T p Specific volume (m3/Kg) Enthalpy (KJ/kg) Entropy (KJ/kg K)


(°C ) (kPa) Sat. Sat. Sat. Sat. Sat. Sat.
liquid vapour, liquid, Evap. vapour, liquid, vapour
vt t vg ht htg hg st sg
36 1369.03 0.001707 0.0931 352.1 1118.7 1470.8 1.2969 4.9161
38 1469.92 0.001716 0.0880 361.9 1109.7 1471.5 1.3281 4.8950
40 1554.33 0.001726 0.0833 371.7 1100.5 1472.2 1.3591 4.8740

Z01_THERMAL ENGINEERING_SE_XXXX_Z01.indd 1117


42 1642.35 0.001735 0.0788 381.6 1091.2 1472.8 1.3901 4.8530
44 1734.09 0.001745 0.0746 391.5 1081.7 1473.2 1.4209 4.8322
46 1829.65 0.001756 0.0707 401.5 1072.0 1473.5 1.4518 4.8113
48 1929.13 0.001766 0.0669 411.5 1062.2 1473.7 1.4826 4.7905
50 2032.62 0.001777 0.0635 421.7 1052.0 1473.7 1.5135 4.7696

Table A.3.2  Superheated ammonia table

Abs, Press. (kPa) Temperature (°C)


Appendix A

(Sat, temp.,°C ) –20 –10 0 10 20 30 40 50 60 70 80 100


v 2.4474 2.5481 2.6482 2.7479 2.8479 2.9464 3.0453 3.1441 3.2427 3.3413 3.4397
50 h 1435.8 1457.0 1478.1 1499.2 1520.4 1541.7 1563.0 1584.5 1606.1 1627.8 1649.7
(– 46.54) s 6.3256 6.4077 6.4865 6.5625 6.6360 6.7073 6.7766 6.8441 6.9099 6.9743 7.0372
v 1.6233 1.6915 1.7591 1.8263 1.8932 1.9597 2.0261 2.0933 2.1584 2.2244 2.2903
75 h 1433.0 1454.7 1476.1 1497.5 1518.9 1540.3 1561.8 1583.4 1605.1 1626.9 1648.9
(– 39.18) s 6.1190 6.2028 6.2828 6.3597 6.4339 6.5058 6.5756 6.6434 6.7096 6.7742 6.8373
v 1.2110 1.2631 1.3145 1.3654 1.4160 1.4664 1.5165 1.5664 1.6163 1.6659 1.7155 1.8145
100 h 1430.1 1452.2 1474.1 1495.7 1517.3 1538.9 1560.5 1582.2 1604.1 1626.0 1648.0 1692.6
(– 33.01) s 5.9695 6.0552 6.1366 6.2144 6.2894 6.3618 6.4321 6.5003 0.5668 6.6316 6.6950 6.8177
v 0.9635 1.0059 1.0476 1.0889 1.1297 1.1703 1.2107 1.2509 1.2909 1.3309 1.3707 1.4501
(Continued)
1117

6/16/2018 12:56:04 PM
Table A.3.2  (Continued ) 1118

Abs, Press. (kPa) Temperature (°C)


(Sat, temp.,°C ) –20 –10 0 10 20 30 40 50 60 70 80 100
125 h 1427.2 1449.8 1472.0 1493.9 1515.7 1537.5 1559.3 1581.1 1603.0 1625.0 1647.2 1691.8
(– 29.08) s 5.8512 5.9389 6.0217 6.1006 6.1763 6.2494 6.3201 6.3887 6.4555 6.5206 6.5842 6.7072
v 0.7984 0.8344 0.8697 0.9045 6.9388 0.9729 1.0068 1.0405 1.0740 1.1074 1.1408 1.2072
150 h 1424.1 1447.3 1469.8 1492.1 1514.1 1536.1 1558.0 1580.0 1602.0 1624.1 1646.3 1691.1
(– 25.23) s 5.7526 5.8424 5.9266 6.0066 6.0831 6.1568 6.2280 6.2970 6.3641 6.4295 6.4933 6.6167
v 0.6199 0.6471 0.6738 0.7001 0.7261 0.7519 0.7774 0.8029 0.8282 0.8533 0.9035
200 h 1442.0 1465.5 1448.1 1510.9 1533.2 1555.5 1577.7 1599.9 1622.2 1644.6 1689.6

Z01_THERMAL ENGINEERING_SE_XXXX_Z01.indd 1118


(– 18.86) s 5.6863 5.7737 5.8559 5.9342 6.0091 6.0813 6.1512 6.2189 6.2849 6.3491 6.4732
v 0.4910 0.5135 0.5354 0.5568 0.5780 0.5989 0.6196 0.6401 0.6605 0.6809 0.7212
250 h 1436.6 1461.0 1484.5 1507.6 1530.3 1552.9 1575.4 1597.8 1620.3 1642.8 1688.2
(– 13.67) s 5.5609 5.6517 5.7465 5.8165 5.8928 5.9661 6.0368 6.1052 6.1717 6.2365 6.3613
v 0.4243 0.4430 0.4613 0.4792 0.4968 0.5113 0.5316 0.5488 0.5658 0.5997
300 h 1456.3 1480.6 1504.2 1527.4 1550.3 1573.0 1595.7 1618.4 1641.1 1686.7
(– 9.23) s 5.5193 5.6366 5.7186 5.7963 5.8707 5.9423 6.0114 6.0785 6.1437 6.2693
v 0.3605 0.3770 0.3929 0.4086 0.4239 0.4391 0.4541 0.4689 0.4837 0.5129
Appendix A

350 h 1451.5 1478.5 1590.7 1521.4 1547.6 1570.7 1593.6 1616.5 1639.3 1685.2
(– 5.35) s 5.4600 5.5502 5.6312 5.7135 5.7800 5.8615 5.9314 5.9990 6.0647 6.1910
v 0.3125 0.3274 0.3417 0.3556 0.3692 0.3826 0.3959 0.4090 0.4220 0.4478
400 h 1446.5 1472.4 1497.2 1521.3 1544.9 1568.3 1591.5 1614.5 1637.6 1683.7
(– 1.89) s 5.3803 5.4735 5.5597 5.6405 5.7173 5.7907 5.8613 5.9296 5.9957 6.1228
v 0.2752 0.2887 0.3017 0.3143 0.3266 0.3387 0.3506 0.3624 0.3740 0.3971
450 h 1441.3 1468.1 1493.6 1518.2 1542.2 1565.9 1589.3 1612.6 1635.8 1682.2
(1.26) s 5.3078 5.4042 5492.6 5.5752 5.6532 5.7275 5.7989 5.8678 5.9345 6.0623
20 30 40 50 60 70 80 100 120 140 160 180
v 0.2698 0.2813 0.2926 0.3036 0.3144 0.3251 0.3357 0.3565 0.3771 0.3975
500 h 1489.9 1515.0 1539.5 1563.4 1587.1 1610.6 1634.0 1680.7 1727.5 1774.7
(4.14) s 5.4314 5.5157 5.5950 5.6704 5.7425 5.8120 5.8793 6.0079 6.1301 6.2472
v 0.2217 0.2317 0.2414 0.2508 0.2600 0.2691 0.2781 0.2957 0.3130 0.3302
600 h 1482.4 1508.6 1533.8 1558.5 1582.7 1606.6 1630.4 1677.7 1724.9 1772.4
(Continued)

6/16/2018 12:56:05 PM

Table A.3.2  (Continued )

Abs, Press. (kPa) Temperature (°C)


(Sat, temp.,°C ) 20 30 40 50 60 70 80 100 120 140 160 180
(9.29) s 5.3222 5.4102 5.4923 5.5697 5.6436 5.7144 5.7826 5.9129 6.0363 6.1541
v 0.1874 0.1963 0.2048 0.2131 0.2212 0.2291 0.2369 0.2522 0.2672 0.2821

Z01_THERMAL ENGINEERING_SE_XXXX_Z01.indd 1119


700 h 1474.5 1501.9 1528.1 1553.4 1578.2 1602.6 1626.8 1674.6 1722.4 1770.2
(13.81) s 5.2259 5.3179 5.4029 5.4826 5.5582 5.6303 5.6997 5.8316 5.9562 6.0749
v 0.1615 0.1696 0.1773 0.1848 0.1920 0.1991 0.2060 0.2196 0.2329 0.2459 0.2589
800 h 1466.3 1495.0 1522.2 1548.3 1573.7 1598.6 1623.1 1671.6 1719.8 1768.0 1816.4
(17.86) s 5.1387 5.2351 5.3232 5.4053 5.4827 5.5562 5.6268 5.7603 5.8861 6.0057 6.1202
v 0.1488 0.1559 0.1627 0.1693 0.1757 0.1820 0.1942 0.2061 0.2178 0.2294
900 h 1488.0 1516.2 1543.0 1569.1 1594.4 1619.4 1668.5 1717.1 1765.7 1814.4
(21.54) s 5.1593 5.2508 5.3354 5.4147 5.4897 5.5614 5.6968 5.8237 5.9442 6.0594
v 0.1321 0.1388 0.1450 0.1511 0.1570 0.1627 0.1739 0.1847 0.1954 0.2058 0.2162
1000 h 1480.6 1510.0 1537.7 1564.4 1590.3 1615.6 1665.4 1714.5 1763.4 1812.4 1861.7
Appendix A

(24.91) s 5.0889 5.1840 5.2713 5.3525 5.4292 5.5021 5.6392 5.7674 5.8888 6.0047 6.1159
v 0.1129 0.1185 0.1238 0.1289 0.1338 0.1434 0.1526 0.1616 0.1705 0.1792
1200 h 1497.1 1526.6 1554.7 1581.7 1608.0 1659.2 1709.2 1758.9 1808.5 1858.2
(30.96) s 5.0629 5.1560 5.2416 5.3215 5.3970 5.5379 5.6687 5.7919 5.9091 6.0214
v 0.0944 0.0995 0.1042 0.1088 0.1132 0.1216 0.1297 0.1376 0.1452 0.1528
1400 h 1483.4 1515.1 1544.7 1573.0 1600.2 1652.8 1703.9 1754.3 1804.5 1854.7
(36.28) s 4.9534 5.0530 5.1434 5.2270 5.3053 5.4501 5.5836 5.7087 5.8273 5.9406
v 0.0851 0.0895 0.0937 0.0977 0.1053 0.1125 0.1195 0.1263 0.1330
1600 h 1502.9 1534.4 1564.0 1592.3 1646.4 1698.5 1749.7 1800.5 1851.2
(14.05) s 4.9584 5.0543 5.1419 5.2232 5.3722 5.5084 5.6355 5.7555 5.8699
1119

6/16/2018 12:56:05 PM
Thispageisintentionallyleftblank
Index

A Air cooling system, 584 Auxiliary part carburettor, 514


Absolute humidity, 1035, 1038 Air infiltration in condenser, 397 Auxiliary valve carburettor, 514
Absorbents, 1057 effects of, 397 Axial flow air compressors,
Absorber, 1029 sources of, 397 789–833
Acceleration pump system, 515 Air injection system, 531 applications of, 802
Accumulators Air motors, 705 constructional features, 789
constant pressure, 114 Air preheater, 110 losses in, 802
variable pressure, 113 Air pump, 398 working principle, 790
Actual velocity diagrams for cen- Air refrigeration cycles, 970 Azeotrope refrigerants, 990
trifugal compressor, 753 Air standard cycle, 423 Azeotropes, 988
Additives, 572, 678 Air stream jet engines, 922
Adiabatic compression Air swirl, 574 B
process, 748 Air washer, 1052 Babcock and Wilcox boiler, 93
Adiabatic efficiency, 699 Air-breathing engines, 922 Back pressure turbine, 187,
Adiabatic mixing of two air Air-conditioning, 1034 339, 341
streams, 1058 Air-extraction pump, 387 Back-suction control, 513
Adiabatic saturation process, Airless injection, 531 Baffle plates, 101
1043 Air-standard efficiency, 423 Balanced draught, 142
Adiabatic saturation tempera- Alcohol, 5, 604 Barometric condenser, 388
ture (AST), 1043 Ammonia, 607 Barrel calorimeter, 63
Adsorbents, 1057 Ammonia-water vapour Basal metabolic rate, 1065
Advanced cooling system, 591 absorption system, 1025 Base line, 1078
After burning, 553, 556 Angle of advance, 554 Battery ignition system, 541
After-burner, 667 Anthracite coal, 2 components of, 545
Aftercooler, 704 Anti-knock agents, 598 working of, 543–544
Air bleeding device, 512 Apparatus dew point (ADP), Bell-Coleman cycle, 977
Air conditioning systems, 1080 1051 with polytropic processes,
comfort, 1084 Aqua-ammonia, 1022 979
industrial, 1084 Artificial draught, 140 Benson boiler, 97
summer, 1080 Atkinson cycle, 428, 459 Benzene, 5
winter, 1089 Auto-ignition, 572 Binary vapour cycles, 184

Z03_THERMAL ENGINEERING_SE_XXXX_Index.indd 1121 5/8/2018 8:24:57 PM


1122 Index

Biofuels, 8–9 Bomb calorimeter, 17–19 height and diameter of, 137
Biogas, 605 Bomb, 17 Chloro fluoro carbons (CFCs),
Biomass, 8 Bore, 423 993
Biomass-generated gas, 605 Bottom dead centre (BDC), 424 Choke valve with spring-
Bituminous coal, 2 Bottoming cycle, 185 loaded bypass, 517
Blade cooling, 896 Boys gas calorimeter, 19 Choke, 516
Blade materials, 895 Brake mean effective pressure Choking, 800, 880
requirements of, 896 (BMEP), 622 Circulation, 99
Blade velocity coefficient, 323 Brayton cycle, 436, 845 Clearance control, 708
Blast furnace gas, 6 Brayton’s air cycle, 977 Clearance ratio, 424
Blow-down cock, 106 Briquetted coal, 3 Closed air refrigeration cycle,
Blowing air to waste, 708 Brown coal, 2 971
Blue water gas, 6 Bubbling fluidised bed boiler Closed cycle constant pressure
Boiler, 87 (BFBB), 102 gas turbine, 842
Babcock and Wilcox, 93 By-pass filtering system, 580 Coal
Benson, 97 By-pass governing, 336 anthracite, 2
bubbling fluidised bed, 102 bituminous, 2
circulation, 95 C briquetted, 3
Cochran, 91 Camshaft, 488 brown, 2
cornish, 91 Capacity control, 1029 gasification, 606
description of, 89 Carburetion, 505 grading of, 4
fire tube, 88, 89 Carburettor, 488, 505 liquefaction, 606
fluidised bed, 102 auxiliary part, 514 properties of, 3
high pressure, 94 auxiliary valve, 514 proximate analysis of, 9
lamont, 96 complete, 511 pulverised, 3
lancashire, 89 main metering system ranking of, 4
locomotive, 91 of, 511 Coal gas, 6
loeffler, 97 simple, 506 Cochran boiler, 91
mountings, 103 Carburetted water gas, 6 Cogeneration, 187, 341
once-through, 99 Carnot cycle, 425
Coke, 3
Schmidt-Hartmann, 98 Carnot refrigeration cycle, 966
Coke–oven gas, 6
selection, 89 Carnot vapour cycle, 161
Cold storage, 1071
stirling, 94 Carry over losses, 319
Cold water spray, 692
velox, 98 Catalytic converters, 668, 674
Central flow condenser, 392 Collection electrodes, 132
water level indicator, 104
Centrifugal air compressors, Combination method, 597
water tube, 88, 93
Boiler accessories, 109 744–780 Combined power and heating
Boiler circulation, 95 constructional features, 744 cycle-cogeneration, 187
forced, 95 working principle, 745 Combined separating and
natural, 95 Cetane number (CN), 601, 603 throttling calorimeter, 66
once-through forced, 95 Charge lubricating system, 578 Combustion chamber, 488
once-through with Chemical energy, 1 Combustion chamber configu-
recirculation, 95 Chimney ration, 667
Boiler feed pump, 387 condition for maximum Combustion chamber design,
Boiler thermal efficiency, 116 discharge through, 138 566
Boiler trial, 117 efficiency of, 139 principles, 565

Z03_THERMAL ENGINEERING_SE_XXXX_Index.indd 1122 5/8/2018 8:24:57 PM


1123
Index

Combustion chamber injection, lobe, 735 Continuity equation, 254


527 Lysholm, 737 Continuous combustion gas
Combustion induced swirl, 575 multi-stage, 692, 693 turbine, 842
Combustion period, 555 screw, 738 Continuous port injection, 527
Comfort air-conditioning single-acting, 691 Control of emission from
system, 1084 single-stage, 692 diesel engine, 673
Comfort chart, 1067 swept volume of the, 692 Convergent nozzle, 253
Common rail system, 532 two-stage, 700 Convergent–divergent nozzle,
Compensating jet, 506, 511 Condensate extraction pump, 253
device, 512 387 Cooling and humidification by
Complete carburettor, 511 Condenser, 386, 387, 962, 1029 water injection, 1056
Compounding, 320 air removal from the, 397 Cooling load estimation, 1072
pressure, 319 central flow, 392 Cooling of blades, 896
velocity, 319 efficiency, 399 Cooling of exhaust valve, 595
Compressed natural gas ejector, 390 Cooling process, 1048
(CNG), 8, 606 evaporative, 392 Cooling systems, 584
Compression, 690 functions of, 386 advanced, 591
isothermal, 692 high level jet, 388 air, 584
multi-stage, 699 jet, 388 evaporative, 589
single-stage, 693 low level counter-flow jet, types of, 584
types of, 692 388 water, 586
Compression ignition (CI) low level parallel flow jet, Cooling tower, 387
engine, 485 388 Cooling with adiabatic humidi-
combustion in, 567 mass of cooling water fication, 1054
emissions, 669 required in, 396 Cornish boiler, 91
fuel injection in, 531 regenerative, 392 Corona, 132
ignition delay period, 568 surface, 388, 390 Crank case, 487
performance of, 630 types of, 388 Crank case emission control, 669
stages of combustion in, water tube surface, 390 Crankcase emission, 666
567–568 Condenser cooling water Crankcase scavenged two-
supercharging of, 660 pump, 387 stroke SI engine, 491
Compression ratio, 424 Condition curve, 335 Crankshaft, 488
Compression stroke, 490, 491 Condition line, 1051 Critical pressure, 56
Compression swirl, 574 Connecting rod, 488 Critical pressure ratio, 256
Compressor coupled of engine Constant BMEP line, 632 Critical temperature, 56
shaft, 661 Constant pressure accumulator, Cross scavenging, 498
Compressor geared with 113 Crude oil, 4
engine and free turbine, Constant pressure combustion Cut-off of steam, 219
661 gas turbine, 842 Cut-out, 546
Compressors, 690, 962 Constant pressure process, 58 Cycle with intercooling and
axial flow air, 789 Constant speed line, 632 reheating, 867
control of, 707 Constant volume combustion Cycle with intercooling, regen-
displacement of the, 692 gas turbine, 843 eration and reheating,
double-acting, 692 Constant volume cycle, 429 869
indicated power of a, 705 Constant volume process, 57 Cyclone separators, 101

Z03_THERMAL ENGINEERING_SE_XXXX_Index.indd 1123 5/8/2018 8:24:57 PM


1124 Index

Cylinder, 487 Discharge, 132 Effect of pressure ratio, 891


Cylinder block, 487 Displacement of the compres- Effect of subcooling of refrig-
Cylinder volume, 424 sor, 692 erant vapour, 1017
Distilled artificial oils, 4 Effect of suction pressure, 1014
D Distributor system, 533 Effect of superheating of refrig-
Dalton’s law of partial pres- Divergent nozzle, 253 erant vapour, 1016
sures, 395, 1034 Dopes, 563, 572, 678 Effective room sensible heat
DBT lines, 1046 Double eye impellers, 746 factor, 1079
Dead weight safety valve, 107 Double-acting compressor, 692 Effective temperature (ET),
Decrement factor and time lag DPT lines, 1046 1067
method, 1076 Draught, 136 Effects of operating variables,
Deflection coefficient, 798 artificial, 140 890
Degree of reaction, 330, 794 balanced, 142 Ejector condenser, 390
Degree of saturation, 1035, 1036 classification of, 136 Ejector-compression system,
Degree of superheat, 52 forced, 141 963
Degree of supersaturation, 260 induced, 141 Electrical tachometers, 624
Degree of undercooling, 260 natural, 136 Electro-lux refrigeration, 964
Dehumidification process, 1049 steam jet, 143 Electronic control unit, 528
Dehydrators, 1025 Dry air, 1034 Electronic fuel injection
De-Laval turbine, 316 Dry bulb temperature (DBT), system, 528
Delay in SI engines, 556 1035 Electrostatic precipitator, 131
Delay period, 531 Dry saturated steam, 52 Emission control programme,
Dense air refrigeration cycle, Dry steam, 52 666
971 Dry sump lubricating system, Emitting electrodes, 132
Determination of 581 Empirical methods to evaluate
air supplied, 13–14 Dryer, 101 heat transfer through
excess air supplied for gase- Dryness fraction of a liquid– walls and roofs, 1075
ous fuel, 15–16 vapour mixture, 52 Emulsion tube, 511, 512
minimum quantity of air Dryness fraction of steam, 62 Engine radiators, 592
required for complete Dual circuit cooling, 591 Enriched water gas, 6
combustion of gaseous Dual cycle, 434 Enthalpy line, 1046
fuel, 15 Enthalpy of moist air, 1038
percentage of carbon in fuel E Enthalpy of saturated water, 52
burning to CO and CO2, Eco-friendly refrigerants, 993 Enthalpy of water, 52
14–15 Economiser system, 515 Enthalpy–entropy, 56
Detonation, 561 Economiser, 111 Enthalpy–entropy diagram, 279
Dew point depression (DPD), Edward’s airpump, 398 Entropy of superheated steam,
1035 Effect of blade friction on 54
Dew point temperature (DPT), velocity diagrams, 327 Entropy of water, 54
1035 Effect of boiler pressure, 169 Entropy of wet steam, 54
Diagram factor, 224 Effect of condenser pressure, Environmental problems cre-
Diesel, 5 169 ated by exhaust emission
Diesel cycle, 431 Effect of discharge pressure, from IC engines, 677
Diesel emissions, 674 1015 Equivalent temperature dif-
Diesel odour, 671 Effect of friction on expansion ferential (ETD) method,
Diffuser, 756 of steam, 258 1076

Z03_THERMAL ENGINEERING_SE_XXXX_Index.indd 1124 5/8/2018 8:24:57 PM



Index 1125

Ericsson cycle, 427 Flywheel, 488 constant volume combus-


Erosion of steam turbine Forced draught, 141 tion, 843
blades, 355 Forced pump system, 587 continuous combustion, 842
Evaporation emission control Four-stroke compression- fuels, 895
device, 669 ignition engine, 488, 490 limitations of, 840
Evaporative condenser, 392 Four-stroke diesel engine, 501 open cycle constant pres-
Evaporative cooling, 1056 Four-stroke spark-ignition sure, 842, 853
Evaporative cooling system, engine, 489 Gaseous fuel, 2, 6, 895
589 Free air delivered (FAD), 692 calorific value of, 6
Evaporative emission, 666 Frictional and turbulence loss, properties of, 7
Evaporator, 963 335 Gasoline, 4, 602
Exhaust emissions, 665 Fuel ignition, 540 Governing, 232, 336, 596
Exhaust gas oxidation, 667 Fuel injection in CI engines, by-pass, 336
Exhaust gas recirculation 531 hit and miss, 596
(EGR), 591 Fuel injection systems in SI nozzle control, 336
Exhaust gas treatment, 667 engines, 527 qualitative, 597
Exhaust manifold reactor, 668 Fuel nozzle, 535 quantitative, 597
Exhaust manifold, 488 Fuel(s), 1090 throttle, 336
Exhaust stroke, 490, 491 analysis of, 9 Governor, 232
Expansion device, 1029 calorific value of, 9–10 Grand sensible heat factor,
Expansion stroke, 490, 491 classification of, 1 1078
Expansion valve, 963 combustion of hydrocarbon, Gravity separation, 100
External combustion engine, 11 Gudgeon pin, 488
214, 485 combustion of, 10
External fins, 693 for CI engines, 603 H
Extraction turbine, 340 for spark-ignition engines, Halo-carbon compounds, 988
602 Halo-carbon refrigerants, 990
F gas turbine, 895 Head coefficient, 797
Factor of evaporation, 115 gaseous, 2, 6, 895 Heat balance sheet, 117, 626
Factors affecting optimum liquid, 1, 4, 895 Heat cramp, 1067
effective temperature, primary, 2 Heat engine, 485, 967
1070 secondary, 3 Heat exchangers, 1025
Fan draught, 140 solid, 1, 895 Heat gain due to infiltration,
Fanno line, 278 Fumigation, 673 1074
Feed check valve, 105 Fusible plug, 106 Heat gain from lights, 1075
Feed-water and drum level Heat gain from power equip-
control, 130 G ments, 1075
F-head combustion chamber, Gas power cycles, 423–441 Heat gain from products, 1074
567 Gas turbines, 840–912 Heat gain from solar radiation,
Film cooling, 896 applications of, 840 1073
Fire tube boiler, 89 classification of, 842 Heat gain through ducts, 1075
Flame propagation, 557 closed cycle constant Heat losses in a boiler plant,
Flow coefficient, 797 pressure, 842 116
Flue gas analysis, 16–17 constant pressure Heat of steam, 52
Fluidised bed boiler, 102 combustion, 842 Heat of superheat, 52

Z03_THERMAL ENGINEERING_SE_XXXX_Index.indd 1125 5/8/2018 8:24:57 PM


1126 Index

Heat pump, 967 multi-stage, 746 Isentropic process, 60, 748


Heat stroke, 1067 single eye, 746 Iso-octane, 597
Heat transfer through walls and types of, 745 Isothermal efficiency, 697
roofs, 1072 Impulse turbine with several Isothermal process, 59
Heating and adiabatic chemical blade rings, 328
dehumidification, 1057 Impulse turbine, 316 J
Heating and humidification, compounding of, 319 Jet condensers, 388
1050 Impulse-reaction turbine, 317 Jet efficiency, 932
by steam injection, 1057 Incomplete combustion, 116 Jet propulsion, 922–954
Heating load estimation, 1077 Indicator diagram, 706 principle of, 922
Heavy fuel oil, 5 Individual pump and nozzle systems, 922
High level jet condenser, 388 system, 534 Joule cycle, 436, 977
High pressure boilers, 94 Induced draught, 141
High pressure side, 963 Induction phenomenon, 550 K
High steam safety valve, 108 Induction swirl, 574 Kerosene, 4
Higher calorific value, 10 Induction system, 667 Knock reducing fuel injector,
Highest useful compression Induction; principle of, 542 572
ratio (HUCR), 563, 599 Industrial air-conditioning, Knocking, 561, 571
Hit and miss governing, 596 1071 controlling the, 571
Hot well, 387 Industrial air-conditioning effects of, 564
Humid specific heat, 1039 system, 1084 factors affecting; in CI
Humidification, 1049 Injectors, 528 engines, 571
Humidity ratio, 1035, 1036 Inlet and exhaust valves, 488 in CI engines, 570
Hydro chloro fluoro carbons Inlet manifold, 488 methods for suppressing, 564
(HCFCs), 993 Inner dead centre (IDC) posi- L
Hydro-carbon refrigerants, 992 tion, 424 Labyrinth packing, 338
Hydrocarbons, 988 Inorganic compounds, 988 Labyrinth seals, 338
Hyperbolic process, 59 Inorganic refrigerants, 992 LaMont boiler, 96
Intake manifold, 505 Lancashire boiler, 89
I Intercooler, 703 Latent heat gain, 1072
IC engine cooling, 584 Intercooling, 857 Latent heat of fusion of ice, 52
IC engine fuels, 601 Internal air cooling, 896 Latent heat of ice, 52
Ideal velocity diagrams, 750 Internal combustion (IC) Latent heat of steam, 52
Idling system, 515 engine systems, 485–608 Latent heat of vapourisation of
Ignition delay, 531 applications of, 499 steam, 52
Ignition delay period, 556, 568 classification of, 485 Lead emission, 666
Ignition lag, 553, 554, 556 combustion in, 552 Leakage loss, 335
Ignition systems, 541 governing of, 596 Leaving losses, 319
battery, 541 performance of, 622–678 Lever safety valve, 107
magneto, 549 performance parameters, L-head combustion chamber,
principle, 543 622 566
Ignition timing, 551, 667 p-v diagrams, 500 Lift, 790
I-head combustion chamber, supercharging of, 658 Lignite, 2
567 valve timing diagrams, 495 Limitations of single jet carbu-
Impellers, 745 working of, 489–492 rettor, 510
double eye, 746 Isentropic efficiency, 749 Limited pressure cycle, 434

Z03_THERMAL ENGINEERING_SE_XXXX_Index.indd 1126 5/8/2018 8:24:57 PM



Index 1127

Liquefied gases, 7–8 mist, 579 Multi-shaft system turbines in


Liquefied petroleum gas pressure feed, 578 series, 895
(LPG), 8 splash, 577 Multishaft systems, 894
Liquid fuels, 1, 4, 895 types, 577 Multi-stage compression, 699
calorific value of, 5 wet sump, 580 Multi-stage compressor, 692, 693
properties of, 5 Lucas mechanical petrol injec- Multi-stage impellers, 746
Liquid sub-cooler, 1026 tion system, 527
Lithium bromide-water system Lysholm compressor, 737 N
equipment, 1028 Natural draught, 136
Lithium bromide-water vapour M Natural gas, 6
absorption system, 1026 Magneto ignition system, 549 Natural refrigerants, 994
working principle, 1027 working, 550 Negative loop, 501
Lobe compressor, 735 Main metering system, 511 n-heptane, 597
Locomotive boiler, 91 Make up water pump, 387 Non-air breathing engines, 922
Loeffler boiler, 97 Mass flow rate of steam, 255 Non-edible vegetable oils, 607
Loop scavenging, 499 Mass rate of flow through an Non-edible wild oils, 607
Losses in a cetrifugal compres- isentropic nozzle, 273 NOx−emission control, 674
sor, 755 Mean effective pressure Nozzle, 253
Losses in axial flow compres- (MEP), 424, 431 Nozzle control governing, 336
sors, 802 Measurement of Nozzle efficiency, 259
Losses in steam turbines, 335 air consumption by air-box O
frictional and turbulence method, 636 Octane number, 602
loss, 335 air consumption, 625 Odour control, 674
leakage loss, 335 brake power, 638 Olefins, 8
mechanical friction loss, 335 exhaust smoke, 625 Once-through boiler, 99
radiation loss, 335 fuel consumption, 625 One-dimensional isentropic flow
residual velocity loss, 335 heat carried away by cooling through a nozzle, 269
wet steam loss, 335 water, 625 Open air refrigeration cycle, 971
Low level counter-flow jet heat carried away by Open cycle constant pressure
condenser, 388 exhaust gases, 625 gas turbine, 842
Low pressure side, 963 Mechanical draught, 140 Operating variables, 890
Lower calorific value, 10 Mechanical friction loss, 335 Orsat apparatus construction, 16
Lower compression ratio, 667 Mechanical injection, 531 Otto cycle, 429
LPG as SI engine fuel, 605 Mechanical tachometers, 624 Outer dead centre (ODC) posi-
Lubricating oil, 577 Metabolic rate, 1065 tion, 424
Lubrication of Metastable flow of steam, 260 Overall efficiency, 931
big end and small end of Mist lubrication system, 579 Overhead valve combustion
connecting rod, 583 Mixed pressure cycle, 434 chamber, 567
cylinder, 582 Modifications in the engine Oxides of carbon, 665
crank, 582 design, 667 Oxides of nitrogen, 665
gudgeon pin, 582 Modifying the fuel used, 667 Oxides of sulphur, 665
main bearings, 581 Moist air, 1035 Ozone layer depletion, 993
small end bearing of con- Moisture content, 1036
necting rod, 582 Moisture content lines, 1046 P
Lubrication systems, 577 Mollier diagram, 283 Paraffin oil, 4
for IC engines, 579 Mollier diagram of steam, 56 Parson’s reaction turbine, 318,
functions of, 577 Morse test, 627 331

Z03_THERMAL ENGINEERING_SE_XXXX_Index.indd 1127 5/8/2018 8:24:57 PM


1128 Index

Pass out turbine, 340 Psychrometric chart, 1046 properties of, 989
Peat, 2 Psychrometric processes, 1048 secondary, 988
Percentage humidity, 1036 Psychrometric relations, 1036 selection, 995
Performance curves, 246 Psychrometry, 1034 Refrigeration systems, 961
Performance maps, 631 principles, 1034 vapor compression, 962
Performance number (PN), 599 Pulse jet engine, 924 Refrigeration, 961
Petrol, 4 Pulverised coal, 3 effect, 965
Physiological hazards resulting Pumping loop, 501 methods of, 962
from heat, 1066 Pumping loss, 501 Refrigerator, 961, 967
Piped natural gas (PNG), 8 Pumping power, 501 Regenerative condenser, 392
Piston rings, 488 Purgers, 1029 Regenerative cycle, 174, 175
Piston, 488 p-v diagram, 434, 500 with closed heaters, 177
Piston-cylinder arrangement, 423 with open heaters, 176
Point of release, 222 Q Regenerative heating, 111, 174
Polytropic efficiency, 795, 796 Qualitative governing, 597 Reheat and regenerative cycle,
Polytropic process, 61 Quality of steam, 52 863
Ports, 216 Quantitative governing, 597 Reheat factor, 333
Positive crank case ventilation Reheating, 861
(PCV) system, 669 R Reheat-regenerative cycle, 182
Power input factor, 754 Radiation loss, 335 Relative humidity (RH), 1035,
Practical vapour absorption Radiator matrix, 592 1037
system, 1025 Ram air efficiency, 932 Relative humidity lines, 1046
Precision cooling, 591 Ram effect, 923 Residual velocity loss, 335
Pressure and velocity com- Ram pressure, 923 Reverse scavenging, 499
pounding, 319 Ramjet engine, 923 Reversed carnot cycle, 971
Pressure- and velocity- Rankine cycle, 165 limitations of, 977
compounded impulse Rateau turbine, 320 Reversible adiabatic, 60
turbine, 321 Rayleigh line, 278 Ribbon-cellular matrix, 592
Pressure coefficient, 754, 798 Reaction turbine, 317, 318 Ricardo turbulent L-head side
Pressure compounded impulse Receiver, 963 valve design, 567
turbine, 320 Reciprocating air compressors, Rim cooling, 897
Pressure compounding, 319 690–726 Ring lubrication, 583
Pressure feed lubricating working principle, 691 Rocket propulsion, 927
system, 578 Reciprocating compressor, 690 Room sensible heat factor
Pressure gauge in boiler, 104 Rectifier, 1025 (RSHF), 1077
Pressure reduction method, 513 Reduced valve overlap, 667 Roots blower, 735
Pre-whirl, 757 Refrigerant control valve, 963 Rope brake dynamometer, 638
Primary fuels, 2 Refrigerant(s), 961, 988 Rotary air compressors,
Primary refrigerant, 988 applications of, 990 735–741
Prime mover, 214 azeotrope, 990 working principle, 735
Process line, 1051 designation of, 989 Rotary gate meter fuel injec-
Producer gas, 6 eco-friendly, 993 tion system, 530
Prony brake, 638 halo-carbon, 990
Propulsive efficiency, 931 hydro-carbon, 990 S
Propulsive power, 930 inorganic, 992 Safety valves, 107
Proximate analysis of coal, 9 primary, 988 Saturated air, 1035

Z03_THERMAL ENGINEERING_SE_XXXX_Index.indd 1128 5/8/2018 8:24:57 PM



Index 1129

Saturation curve, 243 stages of combustion in, properties of, 51–67


Scavenging process, 498 553–556 separation of, 100
Schmidt-hartmann boiler, 98 supercharging of, 659 tables, 55
Scoop, 577 Spark plug, 488, 547 Steam-ejector system, 964
Screw compressor, 738 Specific fuel consumption Stirling boiler, 94
Screw propeller, 922 (SFC), 622 Stirling cycle, 426
Scrubber, 101 Specific humidity, 1035, 1036, Stroke, 423
Secondary refrigerant, 988 1046 Subcooling, 1017
Secondary, 3 Specific power output (SP), 622 Submerged jet, 512
Self-contained rocket engines, Specific volume lines, 1046 Suction pressure, 1014
922 Specific weight (SW), 622 Suction stroke, 490, 491
Sensible heat factor line, 1051 Splash lubricating system, 577 Summer air-conditioning sys-
Sensible heat factor-SHF, 1051 Spring-loaded safety valve, 107 tem with ventilation air
Sensible heat gain, 1072 Stagnation pressure, 748 and by-pass factor, 1082
Sensible heating, 1048 Stagnation temperature, 748 Summer air-conditioning sys-
Separating calorimeter, 64 Stalling, 800 tem with ventilation air
Separation of moisture in Static temperature, 747 and zero by-pass factor,
drum, 100 Steam condensers, 386–400 1080
Sewer gas, 6 Steam condensing plant, 387 Supercharger, 658, 660
Simple carburettor, 506 Steam engines, 214–246 configurations of, 661
theory of, 507 Steam generator control, 130 Supercharging, 658
Simple impulse turbine, 316 Steam generators, 87–150 effects of, 660
Simple theory of aerofoil blad- classification of, 87 objectives of, 661
ing, 790 Steam jet draught, 140, 143 of single cylinder engines,
Single eye impellers, 746 Steam nozzle, 253 662
Single jet carburettor, 510 Steam power cycles, 161–185 Supercooled flow, 260
Single-acting compressor, 691 Steam pressure control, 131 Superheat horn, 1017
Single-stage compression, 693 Steam stop valve, 105 Superheated steam, 52
Single-stage compressor, 692 Steam temperature control, 131 Superheater, 93, 90, 112
Slip, 754 Steam turbines, 316–369 Supersaturated flow of steam,
Slip factor, 754 classification of, 316 260
Smoke-suppressing additives, governing of, 336 Surface condensers, 388, 390
673 losses in, 335 Surging and choking, 760
Solar heat gain through glass principle of operation of, 316 Surging, 760, 799
areas, 1074 Steam, 51 Swept volume, 424
Solar refrigeration, 964 accumulators, 113 Swept volume of the compres-
Solid fuels, 2, 895 constant pressure formation sor, 692
calorific value of, 2–3 of, 51
Solid injection, 531, 532 drum, 100 T
Solution heat exchangers, 1029 dryness fraction of, 62 Tachometers, 624
Spark ignition (SI) engine, 485 entropy of superheated, 54 electrical, 624
combustion chambers for, 565 entropy of wet, 54 mechanical, 624
delay in, 556 generators, 87–150 Temperature–entropy diagram
emissions, 663 heat of, 52 for water and steam,
fuel injection systems in, 527 internal energy of, 54 55–56
ignition lag in, 556 latent heat of, 52 Temperature-entropy (T-s)
performance of, 627 processes for, 57 diagram, 1004

Z03_THERMAL ENGINEERING_SE_XXXX_Index.indd 1129 5/8/2018 8:24:57 PM


1130 Index

Tetra-ethyl-lead (TEL), 597 U Velocity diagrams for velocity


T-head combustion chamber, 566 Ultimate analysis, 9 compounded impulse
Thermal analysis of human Unburnt hydrocarbons (HC), 665 turbine, 325
body, 1065 Undercooling of refrigerant Velocity diagrams, 749, 792
Thermal efficiency, 931 vapour, 1017 Velocity of flow of steam
Thermodynamic cycle, 658 Undercooling, 1017 through nozzles, 254
Thermodynamic wet bulb Uniflow scavenging system, 498 Velox boiler, 98
temperature, 1043 Unsaturated organic com- Volumetric efficiency, 696
Thermo-electric refrigeration, pounds, 988 factors affecting, 697
964 Use of additives, 678 Vortex tube refrigeration, 965
Thermo-siphon system, 586 Use of unleaded petrol, 678
Thermostatic regulator, 587 W
Uses of compressed air in
Three-way catalytic converter, Water boxes, 391
industry, 691
676 Water cooling, 896
V Water cooling system, 586
Throat, 253
Vacuum efficiency, 399 Water gas, 6
Throttle control, 708
Vacuum gauge, 394 Water jacketing, 693
Throttle governing, 336
Vacuum measurement, 394 Water level indicator in boiler,
Throttle valve, 233, 963
Value of enthalpy, 54 104
Throttling calorimeter, 65
Valve combustion chamber, 566 Water requirements of radiator,
Throttling process, 62
Valve timing diagrams, 495 593
Thrust power, 930
Vaned diffuser, 756 Water safety valve, 108
Top dead centre (TDC), 424 Vanes type blower, 737 Water tube boiler, 93
Topping cycle, 185 Vapour absorption system, 1020 Water tube surface condenser,
Torque and mean effective ammonia-water, 1025 390
pressure (MEP), 628 lithium bromide-water, 1026 WBT lines, 1046
Total head pressure, 748 practical, 1025 Wet bulb temperature (WBT),
Total head temperature, 748 working principle of, 1021 1035
Town gas, 6 Vapour absorption system, 963 Wet compression, 973
T-s diagram, 434 Vapour compression refriger- Wet region, 56
Turbine efficiencies, 335 ation system, 962, 999, Wet steam loss, 335
Turbo-charger, 661 1001 Wet steam, 52
Turbo-jet engine, 925 simple, 1000 Wet sump lubricating system,
basic cycle for, 928 Vapour cycle, 161 580
Turbo-prop engine, 926 Vapour density, 1038 Willams line, 233
Two-stage compressor, 700 Vapour pressure lines, 1046 Willan’s line method, 626
Two-stroke CI engine, 489, 497 Variable pressure accumulator, Wilson line, 260
Two-stroke compression- 113 Winter air-conditioning
ignition engine, 491 Velocity compounding, 319 system, 1082
Two-stroke diesel engine, 504 Velocity diagrams for impulse Wood charcoal, 3
Two-stroke petrol engine, 503 steam turbine, 321 Wood, 2
Two-stroke SI engine, 489 Velocity diagrams for Work coefficient, 797
Two-stroke spark-ignition impulse-reaction turbine, Work factor, 754
engine, 491 330

Z03_THERMAL ENGINEERING_SE_XXXX_Index.indd 1130 5/8/2018 8:24:57 PM

You might also like